EXAMMASTER

Lakukan tugas rumah & ujian kamu dengan baik sekarang menggunakan Quizwiz!

Anuria Urine Output

<50 mL/day

CKD stage 5/kidney failure GFR

<15

A 62-year-old woman with past medical history of hyperlipidemia presents due to shortness of breath. She is a nonsmoker and drinks a pint of vodka daily. Chest X-ray reveals severe cardiomegaly. She is ultimately diagnosed with cardiomyopathy. What type of cardiomyopathy does the patient most likely have? A Dilated cardiomyopathy B Hypertrophic cardiomyopathy C Restrictive cardiomyopathy D Takotsubo cardiomyopathy E Postpartum cardiomyopathy

A

CKD stage 2 GFR

60-89

Your patent is a 1-day-old 'floppy baby'; he was born full term by normal vaginal delivery in the hospital. Birth weight was 2.4 kgs; Apgar scores were 6 (1st min), 8 (5 min), and 8 (10 min). His mother had a normal pregnancy, except for an episode of a monoucleois-like illness, but her heterophile antibodies were negative. On examination today, the patient is febrile, with a heart rate of 130/min, shallow breathing, and bilateral fine crepitations. You notice petechial purpura. He has hepatosplenomegaly and generalized lymphadenopathy (cervical, axillary and inguinal). Neurological examination reveals hypotonia and bulging anterior fontanelles; there are no meningeal signs. Ophthalmological examination reveals multiple foci of chorioretinitis on both eyes. He is polypneic and hypoxic; nasal oxygen therapy and wide-spectrum antibiotic therapy is introduced (ampicillin, gentamycin, and cefotaxime). His hemogram shows leukocytosis with anemia [Hb = 8.0 gm%, WBC = 28,200 cells/cu mm (51% polymorphs and 49% lymphocytes)]. Value Finding Normal for 12 - 24 hr old full term Hb (g/L) 95 184 WCC x 109/L 29.0 18.9 (9.4 - 34) Neutro x 109/L 16.0 11.5 (5 - 21) Lymph x 109/L 12.9 5.8 (2 - 11.5) Platelets103/mm3 120 150 - 350 Cerebrospinal fluid (CSF) Value Finding Normal for 12 - 24 hr old full term WCC (mm3) 10 5 (0 - 30) RCC (mm3) 10 9 (0 - 50) Protein (g/L) 8.0 0.6 (0.3 - 2.5) Glucose (millimol/L) 45 (blood sugar 48 mg/dl) Greater than 2/3 of the blood sugar level The rest of his laboratory values, including liver function tests are normal. Chest X-ray reveals interstitial pneumonia. CT shows intracranial diffuse calcifications. Question: Due to the mechanism of the disease, you ask if the mother did what? Answer Choices: A Ate undercooked meat during the pregnancy B Had contact with febrile children during the pregnancy C Had unprotected sex during the pregnancy D Consumed alcohol during the pregnancy E Received rubella vaccine prior to pregnancy

A

Tumor marker for colon cancer

Carcinoembryonic antigen (CEA)

Metabolic Alkalosis

High HCO3 and High pH

24 hour urine collection for protein, If 3-3.5 g protein=

Nephrotic syndrome! Send to nephrologist immediately!

Normal amount of Protein Excretion by the kidneys

no more than 150 mg/day

A 32 year-old obese woman presents to the clinic complaining of swelling, warmth, and redness of her left lower extremity. Doppler ultrasound confirms a deep venous thrombosis is the greater saphenous vein. Which of the following best describes the most likely pathophysiology underlying her thrombophilia? A Antithrombin deficiency B Increased plasma prothrombin C Protein C deficiency D Protein S deficiency E Resistance to activated protein C

resistance to activated protein c

Normal GFR

120

A 46-year-old man presents with severe insomnia and anxiety. While hospitalized, an overnight polysomnogram was performed over 2 consecutive nights. Sleep latency was 60 minutes; REM latency was 45 minutes. He reports feeling paresthesias deep within his legs while lying in bed, especially while falling asleep. He denies recent illness or illicit drug use. His physical exam and lab work were within normal limits. Question What is the most likely cause of the patient's symptoms? Answer Choices 1 Deep vein thrombosis 2 Peripheral neuropathy 3 Restless legs syndrome 4 Alcoholic peripheral neuropathy 5 Periodic limb movements in sleep

Restless leg syndrome Restless legs syndrome (RLS) is a neurological disorder with symptoms of an unpleasant sensations in the legs, such as insects crawling inside the legs, burning, tugging, or creeping. There is an uncontrollable urge to move the limb when at rest (lying down and trying to relax activates the symptoms). Most people with RLS have difficulty falling asleep and staying asleep. Women may be slightly more affected than men. Symptoms may begin at any stage of life, although the disorder is more common with increasing age. The severity of the disorder appears to increase with age. In some patients, symptoms will improve over a period of weeks or months. In most cases, the cause of RLS is idiopathic. A family history of the condition is seen in many cases, suggesting a genetic component. People with familial RLS tend to be younger when symptoms start, and they have a slower progression of the condition. Hypoglycemia can worsen the condition. The disorder is diagnosed clinically by evaluating the patient's history and symptoms. Needle electromyography and nerve conduction studies should be considered if polyneuropathy is suspected on clinical grounds, even if results of neurologic examination are apparently normal. Ropinirole is approved for the treatment of moderate-to-severe RLS. Benzodiazepines may be prescribed for patients who have mild or intermittent symptoms. These drugs help patients obtain a more restful sleep, but they do not fully alleviate the symptoms and can cause daytime drowsiness. For more severe symptoms, opioids may be prescribed since they can cause relaxation and decrease pain. Anticonvulsants, such as carbamazepine and gabapentin, are also useful for some patients; they can decrease the creeping and crawling sensations. There may be no symptoms associated with deep vein thrombosis (DVT), but the classical symptoms of DVT include pain, swelling, redness of the leg, and dilation of the surface veins. Peripheral neuropathy, of which alcoholic peripheral neuropathy is a type, causes tingling or burning pain in the feet. At times, it may be so severe that it interferes with walking, which is a result of injury to sensory fibers. As the condition worsens, the pain typically decreases and numbness increases. Periodic limb movements in sleep are associated with periodic episodes of highly repetitive limb movements during sleep. These repetitive episodes of muscle contractions are usually grouped into series.

A 65-year-old African-American man with a past medical history of hypertension, hyperlipidemia, and diabetes experiences sub-sternal chest pain while shoveling snow. The patient describes the pain starting after 10 minutes of shoveling wet snow, which eventually resolved after he sat down and rested. The patient described the pain as a "heaviness," which did not radiate to any other part of his body, was described as a 4 or 5/10 on a subjective pain scale, and in total he experienced this discomfort for approximately 1 - 2 minutes. The patient's current vital signs are 168/98 with a pulse of 92 and respirations of 16. What drug would be the best choice to rapidly reduce the patient's chest pain in a future similar situation? A Aspirin 81mg PO B Metoprolol 50mg PO C Nitroglycerin 0.4mg SL D Simvastatin 10mg PO E Lisinopril 5mg PO

C

A 36-year-old woman presents with vaginal discharge. She has a history of itching and white discharge. She is sexually active with her husband only. She takes oral contraceptive pills for contraception, and she has never been diagnosed with an STD (sexually transmitted disease). On examination, the vagina is hyperemic and covered with white cottage cheese-appearing discharge. Question: What is the treatment of choice? Answer Choices: A Metronidazole B Metronidazole and tinidazole C Acyclovir D Miconazole E Clobetasol

D

A 22 year-old male presents with a 2-week history of an upper respiratory infection that hasn't improved after taking amoxicillin for 6 days. He notes persistent sore throat, intermittent fever, and a worsening nonproductive cough. Physical examination reveals bilateral diffuse crackles. What is the most appropriate antibiotic to initiate after discontinuing the amoxicillin? A Amoxicilin and clavulanic Acid B Azithromycin C Cephalexin D Ciprofloxacin E Trimethoprim/Sulfamethoxazole

Azithromycin B The patient most likely has atypical pneumonia that responds best to macrolide antibiotics (B). Antibiotics that inhibit cell wall synthesis (A, C) are generally ineffective against these atypical organisms that are either intracellular or lack a cell wall.

A 62-year-old man collapses in the emergency department. The patient is connected to a cardiac monitor that shows the presence of an irregular waveform with wandering baseline and no identifiable P waves or QRS complexes. What intervention has the greatest impact on this patient's survival? A CPR B Defibrillation C Epinephrine D Vasopressin E Amiodarone

B

A 68-year-old man with a past medical history of diabetes mellitus type II, hypertension, and osteoarthritis presents due to a progressive brown-yellow discoloration to his right toenails over the past month. While he denies any pain to the area, he notes that the nails have become "crooked and appear to be cracking." He denies any history of trauma, fever, chills, swelling, erythema, or affliction of any other parts of his body. His physical exam is notable for a whitish to brownish-yellow opacification of all 5 right toenails with associated toenail dystrophy, brittleness, onycholysis, and subungual hyperkeratosis. Question: What is the preferred treatment for this patient? Answer Choices: A Oral griseofulvin B Oral terbinafine C Topical clotrimazole D Topical ciclopirox E Oral ketoconazole

B

A 68-year old man presents due to urinary hesitancy. He describes difficulty initiating his urinary stream as well as reduced force, intermittent and slow flow. These symptoms have become progressively worse over the past 12 months. The patient is otherwise in good health and has no significant medical history. On exam, the abdomen is soft with no organomegaly and the external genitalia are unremarkable. Digital rectal examination reveals an enlarged, smooth prostate. To reduce his symptoms, the patient is given a prescription for finasteride. Question What explanation describes the correct mechanism of action of this agent? Answer Choices 1 Blockade of the enzymatic conversion of testosterone to dihydrotestosterone 2 Relaxation of prostatic smooth muscle cells 3 Inhibition of androgen synthesis 4 Blockade of prostatic DHT receptors 5 Blockade of the enzymatic conversion of testosterone to estrogen

Blockade of the enzymatic conversion of testosterone to dihydrotestosterone Explanation Benign prostatic hypertrophy (BPH) is a condition that arises as a result of proliferation of stromal cells in the periurethral glands of the prostate. Consequently, whereas cancer most commonly arises in the outer layer of the prostate, BPH is a condition associated with the inner portion or transition zone. BPH is a condition that accompanies aging. By age 60, approximately 50% of men have microscopic evidence of BPH and by age 85, about 85% of men will have BPH. The exact cause of BPH remains unclear; however, it is most likely the result of changes in the local hormonal milieu of the prostate. Dihydrotestosterone (DHT) is produced in the prostate via conversion of testosterone by the 5-alpha reductase enzyme. DHT is known to promote stromal proliferation. It is hypothesized that elevated estrogen levels that accompany aging increase the density of DHT receptors in the prostate, leading to hyperplasia. In addition, the density of alpha-1 adrenergic receptors, which contract smooth muscle cells in the prostate, also appears to increase with age, contributing to the obstructive urinary symptoms that typically bring patients with BPH to clinical attention. Microscopic prostatic hyperplasia in and of itself is not dangerous; there is no known association between BPH and prostate cancer. However, prostatic enlargement around the urethral canal can cause urinary outflow obstruction. Clinically, patients report urinary hesitancy, decreased force of urination, interrupted urinary stream, and a feeling of incomplete bladder emptying. Recurrent urinary tract infections may also occur because of residual urine pooling. In severe cases, hydronephrosis may result. In patients with BPH, digital rectal examination reveals lateral and/or median lobe prostatic enlargement. However, prostate size correlates poorly with obstructive symptoms and urodynamic studies are the most useful means of quantifying the severity of BPH. Peak flow rates under 10 ml/sec and residual urine volumes of greater than 150 ml are indicators of significant obstruction. There are 2 main categories of pharmacological agents used to treat BPH: alpha-1 adrenergic antagonists and 5-alpha reductase inhibitors. Alpha-1 adrenergic antagonists (e.g., terazosin, doxazosin, tamsulosin) relax prostatic smooth muscle by blocking the action of alpha-1 adrenergic agonists, reducing obstructive symptoms. By contrast, 5-alpha reductase inhibitors (e.g., finasteride) reduce DHT levels by blocking enzymatic conversion of testosterone. It should be noted that finasteride also lowers prostate specific antigen (PSA) levels, which may complicate the use of PSA as a screening test for prostate cancer.

A 25-year-old man states that he feels sad all of the time, he feels lonely, and all of his relationships seem to fail. His last relationship ended just a few weeks ago and only lasted about 9 weeks. He says his relationships always start out great. He and his love interest will spend endless hours together discussing every detail of their lives. A few weeks into the relationship, things always change. His girlfriends always pull away from him, and they stop respecting his needs. He often tells his girlfriends that if they leave him, he will kill himself. These threats are frequently followed by intense displays of anger. He has been hospitalized 1 time for overdosing on medication and 2 times for superficial cuts to his arm. When asked about his parents, the man states that he rarely speaks with them. They do not seem to understand him or care about him as much as they do his siblings. Although the man went to college for a while, he never seemed able to settle on a major. He has a poor work history, and he frequently changes careers. He states that sometimes the jobs seem like more than he can handle, and he just wants something that does not require so much concentration. He further explains that he does not feel like getting up in the morning, and he has difficulty forcing himself to go to work. He frequently has difficulty sleeping and often spends hours tossing and turning in bed. He says he gets up in the morning feeling just as tired as he was when he went to bed. In accordance with the DSM-IV, what is the Axis II diagnosis for this patient? Answer Choices 1 Dysthymic disorder 2 Histrionic personality disorder 3 Dependent personality disorder 4 Major depressive disorder 5 Borderline personality disorder

Borderline personality disorder Axis II diagnoses consist of personality disorders and mental retardation. The symptoms in this case study suggest that the client may have dysthymic disorder, which is an Axis I diagnosis. Major depressive disorder is also an Axis I diagnosis. The most appropriate Axis II diagnosis for this client is borderline personality disorder. This client is experiencing unstable personal relationships, an unstable sense of self, unpredictable and unstable moods, and difficulty controlling his anger. He also expresses a sense of devaluation in his personal relationships. The case study shows that the client has made suicidal threats and attempts in order to manipulate others or make them feel guilty. All of these symptoms are characteristic of borderline personality disorder. Borderline personality disorder can be differentiated from dependent personality by the fact that those with borderline personality react to abandonment with an outward expression of anger; those with dependent personality disorder, however, react to abandonment with increased submissiveness. Borderline personality disorder can also be differentiated from histrionic personality disorder by the fact that those with borderline personality disorder are often self-destructive and display angry disruptions in close relationships; in contrast, those with histrionic personality disorder do not.

A 35-year-old man presents with right flank pain that has progressively worsened over the past 12 hours and is now radiating into his right groin and testicle. He currently rates the pain as an 8/10 and complains of some nausea but no vomiting. He denies ever having this type of pain previously. He states that he thinks he had some blood in his urine at his last void, but he did not notice any prior to that. The patient is unable to sit still during the interview and refuses the portion of the physical exam where CVA tenderness is assessed; he also refuses any palpation of the abdomen or testicle on the right side. Vital signs include a temperature of 99.2, and BP is 156/84 mm Hg; RR is 12, and oxygen sat 99% on room air. Question What is the best study to assess this patient's condition? Answer Choices 1 CT scan of abdomen and pelvis without contrast 2 MRI of abdomen and pelvis 3 Ultrasound of kidneys 4 Kidney, ureters, bladder X-ray 5 Intravenous urogram

CT scan of abdomen and pelvis without contrast Explanation This patient most likely suffers from urolithiasis. These patients typically present with unilateral renal colic that often radiates to the ipsilateral groin or testis and hematuria. These patients generally cannot sit still secondary to the pain and discomfort. They can also exhibit guarding, nausea, and vomiting in some cases. A stone protocol (non-contrast) CT scan has become the standard initial workup of patients with suspected stone. This study is especially useful in the emergency department, since it can quickly and effectively diagnose urolithiasis. A CT scan gives the most information (location and size of stone, hydronephrosis, any anatomical variations, etc.) to the urologist who will likely see the patient. Stones are more common in men than women (3:1), and initial presentation is typically in the 3rd of 4th decade of life. MRIs are used more often in assessing soft tissues; they are not typically useful in urolithiasis cases. Ultrasound of the kidneys can reveal hydronephrosis and cystic or solid lesions. Stones can sometimes be assessed as well, but small stones are occasionally not visible. A kidney, ureter, bladder X-ray will show approximately 90% of stones. Uric acid stones are not visible on plain film, and some stones are difficult to see because of size or location. A KUB is a viable option, but not the best study to order. Intraveneous urogram is most useful after a thorough bowel preparation, so a non-emergent study will typically yield more information than an emergent study. However, if high-grade obstruction is suspected, emergent intervention with intraveneous urogram may be warranted. These patients can have an elevated temperature and a CT scan can show high-grade obstruction, so that is still the best initial study.

A 45-year-old man presents for a certification of disability. The man works in a nursery. He gives history of occasional breathlessness for which no medical record is present. Examination shows tachypnea, and scattered rhonchi are heard all over the chest. In what way is spirometry helpful in this case? Answer Choices 1 Done alone 2 Done once and repeated again with bronchodilator therapy 3 Done only with bronchodilator therapy 4 Spirometry is not important 5 Spirometry conducted alone or with bronchodilator therapy, does not matter

Done once and repeated again with bronchodilator therapy Spirometry is the most useful assessment of impairment caused by airway obstruction. It is used to determine the impairment leading to disability. It should be done initially and then repeated with bronchodilator therapy, because it proves that the 1st result was due to blockage of the airway.

A 20-year-old woman presents with a 5-day history vaginal discharge and vulvar itching. She has had unprotected sexual intercourse with a new partner for the last month. On clinical examination, there is vulvar erythema and edema. The vaginal discharge is noted to have an unpleasant odor, a white-gray color, and a frothy appearance. The upper vagina and the cervix have a strawberry appearance with multiple erythematous epithelial papillae. The vaginal pH is 6.5. What test is likely to give a positive diagnostic? Answer Choices: A Culture on Sabouraud agar B Dark field microscopy C Rapid plasma reagins D Tzanck smear E Wet vaginal prep

E

A 23-year-old woman presents to clinic complaining of amenorrhea for 3 months. She also complains of increasing facial hair and weight gain. On exam, PMH: menarche age 13. Physical exam reveals a well-developed, slightly obese female with a BMI of 29. Her amenorrhea can likely be improved with which therapy? A Testosterone administration B Progesterone administration C Estrogen administration D Estrogen suppression E Testosterone suppression

Progesterone administration Progesterone administration slows GnRH pulses, thereby improving FSH secretion and follicular maturation.

A 44-year-old woman undergoes surgery for a follicular neoplasm of the thyroid gland. The tumor is successfully resected, and she is transferred to the post-operative ward. Her condition is noted to be stable. On post-operative day 2, she notices a tingling sensation around her lips that runs down her arms. She experiences a cramping sensation in the muscles of her hands and legs, with her fingers going into spasm. An intern on the floor notices the patient's distress and decides to investigate. Question What is the best initial diagnostic test? Answer Choices 1 Serum free T3 and T4 2 Serum TSH 3 Arterial blood gas analysis 4 Serum calcium levels 5 CT scan of the anterior neck

Serum Calcium Levels Explanation The patient has symptoms of hypocalcemia, a possible complication of thyroid surgery. The parathyroid glands are closely related to the thyroid gland and may be inadvertently removed during surgery. If damaged, the serum calcium levels usually drop within 48 hours of surgery. Symptoms of hypocalcemia include muscle pain, fatigue, and muscle cramps if severe. Chvostek's sign, which is the contraction of ipsilateral facial muscles with a light tap over the facial nerve, and spasm of the carpal muscles (Trosseau's sign) may be seen. The best screening test for hypocalcemia is the measurement of ionized calcium in the serum. Calcium exists in equilibrium between the fraction bound to albumin and the free or ionized fraction. The biological effects of calcium are dependent on the ionized fraction. Serum calcium level varies with level of serum albumin (calcium binding protein). Correction for calcium level when albumin is abnormal is as follows: Corrected calcium (mg/dL) = measured total Ca (mg/dL) + 0.8 (4.0 - serum albumin [g/dL]), where 4.0 represents the average albumin level. The risk factors for hypocalcemia after thyroid surgery include large goiters, hyperthyroidism, low pre-operative vitamin D levels, and planned extensive neck dissection. Hypocalcemia is prevented to some degree by the administration of large amounts of calcium, usually per oral, and 1,25 dihydroxy vitamin D. In the event of severe hypocalcemia, calcium gluconate may be used via IV. It is important to check magnesium levels and correct levels prior to discharge, as hypomagnesemia can compound hypocalcemia and make it refractory to correction. Serum TSH, T3, and T4 are incorrect, as thyroid function has no bearing on the patient's symptoms. They are expected to change after thyroid surgery. Typically, TSH levels will rise, and T3 and T4 levels will fall over the ensuing 3 - 6 weeks. An ABG is incorrect. It may demonstrate low Ca levels, but a direct measure of serum calcium in patients that demonstrates symptoms of hypocalcemia is more accurate and effective. A CT scan of the neck is incorrect, as it provides no additional information about the person's calcium metabolism.

A 28-year-old man presents for evaluation of a rash. His trunk is covered with a pink-red papulosquamous eruption with scattered discrete coppery papules on the palms of his hands and a few lesions on the soles of his feet. He is afebrile and has not experienced any illness in the past month that he can recall. Skin exam reveals soft, flat, moist papules scattered on his perineum. Question: Which of following laboratory tests will yield the most information about the patient's condition? Answer Choices: A KOH prep B HIV antibody C VDRL D Peripheral blood smear E Tzanck smear

C

A 32-year-old nulligravida woman presents with gradual onset of excessive hair growth on the face, chest, abdomen, back, and upper parts of her limbs, together with the irregular menstruation. She denies changes in weight, body shape, and voice. Her menarche started when she was 8. Her family history is non-contributing. Physical examination reveals short stature, excessive male-pattern hair growth, and slight baldness. Her BMI is 18.5. The rest of the examination, including pelvic ultrasound, is normal. Question What is the most likely diagnosis? Answer Choices 1 Adrenal tumor 2 Ovarian tumor 3 Congenital adrenal hyperplasia 4 Polycystic ovarian syndrome 5 Idiopathic

CAH Explanation This patient has male-pattern growth of body hair. It usually occurs in androgen-stimulated parts of the body (the face, chest, areolae). In women, androgens are produced in ovaries, adrenal glands, and the hair follicle. Precocious puberty, gradual onset of the hirsutism without virilization in second or third decade, and irregular menstruation in a woman with short stature will lead you to consider late onset congenital adrenal hyperplasia. Congenital adrenal hyperplasia (CAH) is one of the most common autosomal recessive disorders. It is proven by the presence of elevated first morning 17-hydrohyprogesterone because of the deficiency of the enzyme 21- hydroxylase. Absolute or partial deficiency of the enzyme can manifest early or late in life. There are early forms in female newborn with masculinization and ambiguous genitalia (the shunting of 17-hydroxyprogesterone to the production of androgens) or in some salt-wasting cases (the deficiency in production of cortisol and aldosterone). The combination is also possible. Late onset cases are most commonly mild. They usually manifest as some type of androgen excess later in life and without the aldosterone deficiency. An adrenal tumor will present with rapid onset of virilization, which is not the case in this patient. Elevated dehydroepiandrosten sulfate (DHEAS) produced by adrenal glands will confirm the diagnosis. Ovarian tumor will also present with rapid onset of virilization, probably with palpable adnexal mass. Elevated testosterone will confirm your diagnosis. Polycystic ovarian syndrome (PCOS) is the most common endocrine disorder of reproductive-age women. It typically presents with irregular vaginal bleeding, obesity, infertility, and hirsutism, and is characterized by ovarian dysfunction, disordered gonadotropin secretion, and hyperandrogenemia. Anovulatory cycles, infertility, amenorrhea, hirsutism, acne, and acanthosis nigrans are sometimes the result. Ultrasound may reveal hyperplastic endometrium because of the absence of ovulation and progesterone cyclic shedding (this may lead to the endometrial cancer) and characteristic bilateraly enlarged ovaries with thick capsules, multiple subcapsullar follicles, and hyperplasia of ovarial stroma. The levels of testosterone might be mildly elevated, but DHEAS and 17-hydrixyprogesterone are normal. Confirmation test consists of LH:FSH ratio and decreased level of sex-hormone binding protein. BMI that puts a patient in the underweight group and short stature are not parts of the clinical picture. In most cases, hirsutism is a benign, primarily cosmetic condition. When accompanied by masculinizing signs or symptoms, particularly when these arise after puberty, hirsutism may be a manifestation of a more serious underlying disorder such as an ovarian or adrenal neoplasm. Therefore, patients with clinical evidence of hyperandrogenemia, including hirsutism and acne, should have an evaluation of adrenal androgens. Before you declare the diagnosis of idiopathic hirsutism, you should exclude those disorders.

Chron's Disease appearance on colonoscopy

Granulomas, skip lesions, "string" sign, transmural involvement and cobblestoning

Tangentiality

Tangentiality is a disturbance in thought causing the person to start a train of thought, but never getting to the point.

A 16-year-old girl is brought to the emergency room for evaluation of continuous seizures. The patient was first diagnosed with idiopathic epilepsy at age 10. She was started on valproic acid. Phenobarbital and carbamazepine (Tegretol) have been tried without success, and the phenobarbital resulted in transient severe personality changes. 2 previous EEG's have been normal. This spell occurred abruptly at the family dinner table without apparent warning. Past medical history has been normal except for the usual childhood diseases without complications. 2 days prior to presentation, she saw her primary physician for nausea and vomiting; as he believed she had the flu, he started her on prochlorperazine (Compazine), which relieved her nausea. Physical exam reveals a well-developed, well-nourished young woman lying on a gurney; there is continuous but variable motor jerking of all 4 extremities. The jerking varies in intensity from side to side. The head intermittently turns from side to side. Eyes are closed tightly and cannot be passively opened. Old cutdown sites in the right antecubital fossa and left supraclavicular regions are noted. Deep tendon reflexes cannot be assessed. The response to plantar stimulation is withdrawal bilaterally. There are no signs of trauma to the head or elsewhere. Skin is warm and dry with normal color, and vitals are normal. The general medical exam is otherwise normal. Labs are done stat, and they include normal complete blood count and differential, normal chest X-ray, normal urinalysis on cathed specimen, normal chemistry profile, and normal arterial blood gases. Question Pseudoseizure (psychological rather than epileptic event) is suspected. What test would be of greatest value in this acute clinical setting in differentiating pseudoseizures from true epileptiform seizures? Answer Choices 1 Stat electroencephalogram (EEG) 2 Stat serum prolactin level 3 Stat creatine kinase (CK) 4 Stat referral to a comprehensive epilepsy center 5 Stat psychiatric referral

stat EEG If an electroencephalogram can be obtained on a stat basis, it can be of immeasurable help in differentiating pseudoseizures from true seizures. During a spell suggesting generalized tonic-clonic seizure, the electroencephalogram should show generalized spike and wave activity; after the spell, it should show generalized slowing and disorganization lasting 30-90 minutes. Even if the active phase of the spell were over before the electroencephalogram, if done in this 30-90 minutes post-event time frame, it could be compared with a later electroencephalogram, which would probably be normal in case of pseudoseizure (5). If an electroencephalogram is normal during or immediately after this event, it is major evidence against epilepsy and for pseudoseizures. In view of the above, and with a history of 2 previous normal electroencephalograms, obtaining an outpatient electroencephalogram or one during subsequent admission would most likely be a waste of time and money. If a video camera were available, video recording the patient during a spell, especially during electroencephalography, may provide excellent evidence or even lead to a definitive diagnosis. A serum prolactin level is a very good test in differentiating seizure from pseudoseizure; most generalized seizures, and many complex partial seizures, result in elevated levels during this initial 30-minute period following the event (1). It should be done but the results may not be back for several days. In a study by Holtkamp et al., serum creatine kinase (CK) in psychogenic status had a mean of 39 U/L (range 16-90), but in true epileptic status the mean was 699 with a range of 57-2,625 (1). In this study, creatine kinase began to rise after a delay of 3 hours and peaked after more than 36 hours. Therefore, it will not help in an emergency room setting, but it should be done and repeated (e.g., at 12, 24, and 36 hours after the event), as it can provide good evidence later on for true seizures (levels will rise) or against seizures, and for pseudoseizures if levels do not rise or rise only minimally. Through the use of 24-hour electroencephalography and video monitoring, comprehensive epilepsy centers can be extremely valuable and even definitive in differentiating pseudoseizures from true epileptic seizures. However, they are tertiary centers for evaluation of patients who have not received definitive diagnoses or have not responded to treatment for either. Also, these centers are not that common, and referral can be prolonged and complicated. As valuable as psychiatric referral could be if the patient is having pseudoseizures, a consult at this time would not be helpful. The patient's condition would preclude any meaningful interview. Also, pseudoseizure is a diagnosis of exclusion, and a full evaluation for other causes of her spells has not yet been done. Furthermore, premature referral could negatively impact cooperation from the patient and family. Psychiatric referral should be considered only after a firm diagnosis of pseudoseizures has been made, or where it becomes apparent that behavioral input is necessary to help in this differential. Summary of objective diagnostic testing that can be done to differentiate seizures from pseudoseizures: Electroencephalograms, if available, can be very helpful during the acute phase; with tonic and/or clonic movements, one would expect to see epileptiform abnormalities. Psychomotor seizures may show in the electroencephalogram, but not necessarily. A negative electroencephalogram is strong evidence of pseudoseizures, but not proof. An encephalogram done within 90 minutes after an epileptic seizure frequently shows transient postictal changes that will not be seen after a psychogenic seizure (5). Prolactin levels, done within 30 minutes of a seizure, may be helpful since most generalized seizures, and many complex partial seizures, result in elevated levels (1). Serum creatine kinase (CK) in psychologic status had a mean of 38 U/L (range 16-90), but in true epileptic status, the mean was 699 with a range of 57-2,625. In the study cited in (1), with true epileptic seizures, creatine kinase begins to rise after a delay of 3 hours, and it peaks after more than 36 hours. EEGs, if they can be obtained on a stat basis, can be most valuable in picking up the presence or absence of epileptiform abnormalities during the spells or the presence or absence of postictal changes in the 30-90 minute period following true seizures.Video recording of the patient, if a video camera is available, can be most helpful in later efforts at diagnosis. A combination of EEG and video recording could be of major importance. At a later time, continuous video and electroencephalography (vEEG) may be arranged if necessary, and it constitutes the gold standard for diagnosis.

A 55-year-old African American woman presents to you with fatigue, non-productive cough, and hair loss as well as admitting to a 40 pack-year smoking history. You take a thorough history and perform an appropriate physical examination and you suspect Human Immunodeficiency Virus (HIV) infection. Which of the following physical exam findings is most specific for HIV infection? Answer Choices: A Basilar rales on lung auscultation B Oral hairy leukoplakia on tongue C Vitiligo D Alopecia areata E Clubbing

B

A woman comes to your office after a 6-month sabbatical working in caves in the eastern part of the South America. Upon questioning, the patient reports fever, chills, productive cough, and joint stiffness that started 1 month before her return. Physical exam reveals 3 ulcerated lesions on her inner cheek. What is your treatment recommendation? Answer Choices: A Corticosteroid B Ketoconazole C Penicillin D Amphotericin B E Supportive treatment

B

A 13-year-old girl is 74 inches (1.88 m) tall. She has had no major medical illnesses, but on review of systems, she reports joint laxity and a history of dental crowding. Her family history is significant for her mother also being tall and having a history of retinal detachment. Her maternal uncle died at the age of 33 due to a ruptured thoracic aortic aneurysm. On physical examination, you note a narrow high-arched palate, pectus carinatum, high-pitched decrescendo diastolic murmur at the left sternal border, arachnodactyly, and an increased arm span to height ratio. What is the most likely diagnosis? A Ehlers Danlos syndrome B Marfan syndrome C Noonan syndrome D Klinefelter syndrome E Turner syndrome

B

Normal Urine Output

0.5- 1 mL/kg/hr

A 30-year-old HIV-positive man presents to your clinic for the first time with a history of pneumocystosis, and he is on trimethoprim/sulfamethoxazole for secondary prophylaxis. Primary prophylaxis of Pneumocystis pneumonia is recommended for patients at what CD4 cell count? Answer Choices: A 50 cells/mcL B 100 cells/mcL C 200 cells/mcL D 350 cells/mcL E 500 cells/mcL

C

A 35-year-old woman with acute lymphocytic leukemia and undergoing chemotherapy suddenly developed small vesicular eruptions across her left breast. The eruptions were asynchronous, painful, and lasted 6 days before clearing. She scratched the affected area, which only intensified her discomfort. The affected area developed small vesicles that progressed to ulcerated and crusted pustules, causing her clothing to stick to the open sores. She had a similar occurrence last year. Appearance of her current symptoms most likely occurred through which of the following pathways? Answer Choices: A Inhalation of contaminated aerosol droplets B Ingestion of contaminated foods C Transfusion of blood or blood products D Activation from latency E Sexual contact

D

Amantadine, used for the prophylaxis of influenza, functions by what mechanism? Answer Choices: A Inhibition of viral transcription B Inhibition of viral translation C Neutralization of extracellular viral particles D Inhibition of the release of viral particles E Inhibition of penetration of the cell membrane of the target cell

E

A 30-year-old man presents with a 1-week history of a painful tongue. He has also been having diarrhea, and he has been forgetting things more easily. He recently emigrated from India and lives alone on a maize-based diet. On examination, you note that the tip and margins of his tongue are a bright scarlet color. He also has a symmetrical erythematous rash on his forearms. Question What is the most likely diagnosis? Answer Choices 1 Vitamin A deficiency 2 Pyridoxine deficiency 3 Vitamin C deficiency 4 Niacin deficiency 5 Vitamin E deficiency

Niacin Niacin deficiency causes pellagra, which is characterized by:A symmetrical dermatitis, usually on parts of the body exposed to sunlight Scarlet glossitis and stomatitis Diarrhea Mental aberrations, such as memory impairment, depression, and dementia. These may appear alone or in combination. Causes include inadequate dietary intake, especially in patients with corn-based diets or alcoholism. Dietary sources include legumes, yeast, meat, and enriched cereal products. Vitamin E deficiency may cause a hemolytic anemia in premature infants. Laboratory investigations reveal low plasma tocopherol levels, a low hemoglobin level, reticulocytosis, hyperbilirubinemia, and creatinuria. Causes of vitamin E deficiency in premature infants include limited placental transfer of vitamin E and the resultant low levels at birth combined with its relative deficiency in the infant diet. Dietary sources for older children and adults include wheat germ, vegetable oils, egg yolk, and leafy vegetables. In cases of vitamin A deficiency, patients can present with inability to see well in dim light or night blindness. There may also be conjunctival and corneal xerosis, as well as pericorneal and corneal opacities, and Bitot's spots. Bitot's spots are a collection of keratin appearing as triangular foamy spots on the conjunctiva. The patient may also have xeroderma, hyperkeratotic skin lesions, and increased susceptibility to infections. Causes include inadequate dietary intake and malabsorption. Dietary sources of vitamin A include fish, liver, egg yolk, butter, cream, dark green leafy vegetables, as well as yellow fruits and vegetables. In cases of vitamin C (or ascorbic acid) deficiency, patients can present with bleeding tendencies (as a result of weakened capillaries) and impaired wound healing due to impaired formation of connective tissue. On examination, the gums may be swollen and friable; the teeth may be loose. There may also be multiple splinter hemorrhages on the nails and ecchymoses, especially over the lower limbs. Causes include inadequate dietary intake and certain conditions, such as pregnancy and lactation, which increase vitamin C requirements. Dietary sources of vitamin C include citrus fruits, such as oranges, lemons, and tangerines, as well as tomatoes and potatoes. In cases of pyridoxine or (vitamin B6 deficiency), patients can present with peripheral neuropathy, seborrheic dermatosis, glossitis, and cheilosis. Laboratory investigations reveal anemia with lymphopenia. Causes include malabsorption as well as medications, such as isoniazid and penicillamine. Dietary sources of vitamin B6 include liver, legumes, whole grain cereals, and meats.

What is the most common cause of hemodynamically significant bleeding from the lower gastrointestinal tract? Answer Choices 1 Colonic diverticula 2 Colonic cancer 3 Colonic polyps 4 Vascular ectasia of the colon 5 Inflammatory bowel disease

The most common cause of hemodynamically significant hemorrhage from the lower gastrointestinal tract is colonic diverticula, which accounts for about 25% of the cases. The bleeding is typically painless and associated with voluminous hematochezia. Colonic cancer and polyps together make up about 20% of the cases of hemodynamically significant bleeding from the lower gastrointestinal tract; inflammatory bowel disease and vascular ectasia are considerably less frequent causes.

Non Oliguria Urine Output

>400 ml/day

What exposure is linked to brucellosis

Brucellosis can be caused by exposure to hogs, cattle, or goats.

Tumor marker for pancreatic cancer

CA 19-9

A 25-year-old Asian man presents with a 1-month history of an inability to see well while walking home from work at night. He also reports that he has been getting sore throats more frequently. He is a recent immigrant from his home country; he lives alone and eats mainly a rice-based diet. On examination, you note pericorneal and corneal opacities. Question What is the most likely diagnosis? Answer Choices 1 Vitamin A deficiency 2 Pyridoxine deficiency 3 Vitamin C deficiency 4 Niacin deficiency 5 Vitamin E deficiency

In cases of vitamin A deficiency, patients can present with inability to see well in dim light or night blindness. There may also be conjunctival and corneal xerosis, as well as pericorneal and corneal opacities, and Bitot's spots. Bitot's spots are a collection of keratin appearing as triangular foamy spots on the conjunctiva. The patient may also have xeroderma, hyperkeratotic skin lesions, and increased susceptibility to infections. Causes include inadequate dietary intake and malabsorption. Dietary sources of vitamin A include fish, liver, egg yolk, butter, cream, dark green leafy vegetables, yellow fruits, and vegetables. Vitamin E deficiency may cause a hemolytic anemia in premature infants. Laboratory investigations reveal low plasma tocopherol levels, a low hemoglobin level, reticulocytosis, hyperbilirubinemia, and creatinuria. Causes of vitamin E deficiency in premature infants include limited placental transfer of vitamin E and the resultant low levels at birth combined with its relative deficiency in the infant diet. Dietary sources for older children and adults include wheat germ, vegetable oils, egg yolk, and leafy vegetables. Niacin deficiency causes pellagra, which is characterized by: A symmetrical dermatitis usually on parts of the body exposed to sunlight Scarlet glossitis and stomatitis Diarrhea Mental aberrations, such as memory impairment, depression, and dementia. These may appear alone or in combination. Causes include inadequate dietary intake, especially in patients with corn-based diets and alcoholism. Dietary sources include legumes, yeast, meat, and enriched cereal products. In cases of vitamin C (or ascorbic acid) deficiency, patients can present with bleeding tendencies (as a result of weakened capillaries) and impaired wound healing due to impaired formation of connective tissue. On examination, the gums may be swollen and friable; the teeth may be loose. There may also be multiple splinter hemorrhages on the nails and ecchymoses, especially over the lower limbs. Causes include inadequate dietary intake and certain conditions, such as pregnancy and lactation, which increase vitamin C requirements. Dietary sources of vitamin C include citrus fruits, such as oranges, lemons, and tangerines, as well as tomatoes and potatoes. In cases of pyridoxine (or vitamin B6 deficiency), patients can present with peripheral neuropathy, seborrheic dermatosis, glossitis, and cheilosis. Laboratory investigations reveal anemia with lymphopenia. Causes include malabsorption as well as medications, such as isoniazid and penicillamine. Dietary sources of vitamin B6 include liver, legumes, whole grain cereals, and meats.

CKD stage 4/Severe Kidney disease GFR

15-29

A 37-year-old woman, G3P2 at 30 weeks gestation, complains of lower extremity swelling and her weight is up 5 pounds this week. Her PMH is insignificant, and her other pregnancy was uncomplicated. Her BP baseline is now 142/92. On exam her BP is unchanged, her UA shows 2+ protein, and FHTs are 152. What is the cause of the protein in her urine? A Glomeruloendotheliosis B Glomerulonephritis C Renal vasospasm D Glomerular hemorrhage E Glomerular infarct

A Glomeruloendotheliosis A The classic histological change that occurs in the renal system (in preeclampsia) is swelling and inflammation of the endothelium and of the glomeruli, which leads to endothelial leaking.

A 22-year-old woman presents with an 8-month history of amenorrhea. The patient is also experiencing backaches, headaches, and acne. Physical examination reveals a female patient with a moon-shaped facies, multiple purple striae, and significant central obesity (body mass index of 36). Question What is the most likely diagnosis? Answer Choices 1 Diabetes type II 2 Hypothyroidism 3 Cushing syndrome 4 Hyperthyroidism 5 Diabetes insipidus

Cushings Syndrome Explanation This patient likely is suffering from Cushing syndrome, which is also known as hypercortisolism. Consequences of excessive levels of circulating cortisol, no matter the etiology, will lead to signs and symptoms such as central obesity, thin extremities, a moon face, a buffalo hump, supraclavicular fat pads, protuberant abdomen, oligomenorrhea, and amenorrhea; in men, erectile dysfunction is sometimes seen. Backaches, headaches, hypertension, acne, purple striae, and impaired wound healing may also be found in these patients. Diabetes type II is caused by insulin resistance in tissues to circulating endogenous insulin. Eventually, insulin is unable to be produced by the pancreatic islet cells, and an exogenous supply is required to prevent significant hyperglycemia and its related co-morbidities. Features of these patients include central obesity (specifically visceral obesity), with less fat noted on extremities. Patients will have a history or evidence of acanthosis nigricans, eruptive xanthomas on flexor surfaces, skin tags, chronic skin infections, frequent candidal vulvovaginitis in women, and erectile dysfunction or balanoposthitis in men. Both hypothyroidism and hyperthyroidism are related to dysfunction of the thyroid gland; hypothyroidism is due to failure of the thyroid gland itself or deficient supply of pituitary TSH (thyroid stimulating hormone). Symptoms characteristic of hypothyroidism include weight gain, fatigue, lethargy, depression, weakness, menses irregularities, athralgias, muscle cramps, cold intolerance, dry skin, and constipation; other signs may include thinning hair, brittle nails, bradycardia, or even peripheral edema. Hyperthyroidism is the disease state in which there is an excessive serum level of T3 and or T4 along with suppressed TSH levels. Signs and symptoms may include restlessness, nervousness, heat intolerance, increased sweating, weight loss, palpitations, atrial fibrillation, exophthalmos, or (depending on the severity) even pretibial myxedema. Diabetes insipidus (DI) is an antidiuretic hormone deficiency. Signs and symptoms of DI include an extreme, unquenchable thirst with a very specific craving for ice water. Patients also will have significant polyuria or hypernatremia.

Proteinuria can be falsely positive in:

Fever, exercise, prolonged standing or infection

Respiratory Alkalosis

High pH and Low CO2

Absolute indications for dialysis

Absolute indications for dialysis!!!! Uremic pericarditis Hyperkalemia, severe Diuretic unresponsive fluid overload (pulmonary edema) Intractable metabolic acidosis

A 36-year-old woman presents with a 24-hour history of sudden, severe diarrhea described as profuse, gray, cloudy, watery stools without blood or fecal odor. She was recently in Bangladesh for work and returned yesterday, when the diarrhea began. She complains of mildly elevated temperature with a very dry mouth, headache, and severe fatigue. Question: The most likely offending organism is: Answer Choices: A Enterotoxigenic E. coli B Vibrio cholerae C Norwalk virus D Shigella dysenteriae E Clostridium difficile

B

A 5-year-old girl presents with a 5-day history of diarrhea and abdominal discomfort. Stool cultures reveal Giardia lamblia, and the patient is to start treatment immediately. Question: What is the recommended treatment for her condition? Answer Choices: A Hydration and pain control only B Metronidazole for 7 days C Ciprofloxacin for 7 days D Ampicillin for 14 days E Loperamide for 14 days

B

An 18-year-old man presents with urethral discomfort and dysuria and urethral discharge. He notes no other symptoms. His temperature is normal, and physical exam is unremarkable. He is sexually active, heterosexual, and had sex with 3 different women in the last 2 weeks. He was not practicing safe sex. The urethral discharge was noted to be clear and slightly viscous in nature. The urethral discharge is Gram stained and examined. It is found to contain a few WBCs and no organisms. This presentation is most suggestive of what organism? Answer Choices: A Neisseria gonorrhoeae B Chlamydia trachomatis C Herpes simplex D Haemophilus ducreyi E Treponema pallidum

B

You are evaluating a 59-year-old male with history of heavy alcohol use for the past 5 years who appears acutely ill. Which of the following findings is most likely to be an indicator of increased mortality in this patient with severe alcoholic hepatitis? A fatty liver B INR of 7 C AST > ALT by a factor of 2:1, but less than 300 units/L D serum bilirubin 1.8mg/dl E folic acid deficiency

B INR of 7 The correct answer is choice (B). A significantly elevated PT/INR as well as bilirubin > 10 mg/dl are indicators of severe alcoholic hepatitis and increased mortality requiring hospitalization. Choices (A), (C), and (E) are all common findings in alcoholic hepatitis. Mild elevation of serum bilirubin, choice (D), is also common.

A 72-year-old man with known hypertension and high cholesterol presents with acute chest pain. He describes the pain as an excruciating tearing pain radiating to his back. His blood pressure in his right arm is 105/65 mm Hg and in his left arm 140/90 mm Hg. He also has a decreased pulse in his right arm. On auscultation of the chest, a III/VI early diastolic murmur is best heard at the left sternal border, third intercostal space. His EKG shows a sinus tachycardia with no acute changes. What is the most likely diagnosis? A Pericarditis B Musculoskeletal pain C Aortic dissection D Myocarditis E Acute myocardial infarction

C

A couple has recently had a newborn baby boy. He was born with a congenital cardiac defect known as a ventricular septal defect (VSD). He is now only a week old and is still in the hospital due to mild respiratory distress, persistent tachycardia, and poor weight gain. What first line treatment is appropriate for this patient? A Observation; no treatment necessary with degree of symptoms B Continuous IV fluids only C Diuretics, digoxin, and afterload reduction D Surgical closure of the VSD E Digoxin only

C

An 80-year-old man was treated for ventricular arrhythmias. He presents 1 month later with joint pain. He also has an unusual mask-like rash over his face and body. Discontinuation of drug therapy causes the symptoms to abate. What drug was most likely administered to this patient? A Tocainide B Quinidine C Procainamide D Phenytoin E Propranolol

C

An HIV-positive young woman gives birth to a female infant. The infant's HIV PCR is found to be positive after birth. Her Apgar score is 9 and first general exam is completely normal except for the presence of positive HIV nucleic acids. What vaccination is contraindicated for this infant? Answer Choices: A MMR (measles, mumps, and rubella) B HBV (Hepatitis B) C HiB (Haemophilus influenzae type b) D IPV (inactivated poliovirus) E Tdap (tetanus toxoid, reduced diphtheria toxoid, and acellular pertussis)

C

What is only oral regimen that is recommended for the treatment of vulvovaginal candidiasis? Answer Choices: A Clindamycin 300 mg PO BID x 7 days B Valacyclovir 500 mg PO BID x 7-10 days C Fluconazole 150 mg PO x1 dose D Metronidazole 500 mg PO BID x 7 days E Tinidazole 2 g PO x 1 dose

C

Your patient is a 42-year-old female diagnosed with depression. She has taken two different serotonin reuptake inhibitors (SSRIs) at the usual dosage, each for 3 weeks, and reports again that there have been no ill effects but that she doesn't feel that this medication has helped either. What is the most likely problem with her therapy? A She has a genetic resistance to the medication B The dosage was too low C The medication was discontinued too soon D The wrong diagnosis was made E The wrong medication was given

C The medication was discontinued too soon C Treatments for depression generally require from 2 to 6 weeks of therapy in before effects can be evaluated (C). Too low a dose (B) is another common problem, but an increase in dose should not be considered until the medication has had time to take effect. The effect of these agents can be highly variable from one individual to another, but no specific genetic resistance (A) is known. There is always a possibility of making the wrong diagnosis (D), but there is no indication of this from the information given, and SSRIs are the first line therapy for depression (E), best combined with counseling.

CAP therapy for peptic ulcer caused by H.Pylori includes what 3 drugs?

Clarithromycin Amoxicillin PPI

A 35-year-old female presents to discuss non-permanent methods of birth control. She is a G3P2, PMH negative. She is a smoker. Which method of contraception would have the lowest risk profile for her? A Combination low dose pill daily B Injectable progestin monthly C Transdermal combination patch D Levonorgestrel intrauterine device E Copper intrauterine device

Copper IUD E A women who is over 35 and smokes is at high risk for cardiovascular complications. All choices except the copper IUD contain hormones, which may increase the risk of complications. The copper IUD is long term but non-permanent.

A 12-month-old girl presents with her parents after a 3-day history of intermittent episodes of strange behavior. A neurologist is consulted because the parents are concerned she is having some form of seizure activity. Her health history includes 2 episodes of otitis media, but she is otherwise healthy. Her initial vital signs and physical exam by the emergency room staff are all normal. Question What description of the infant's strange behavior would lead the neurologist to suspect a diagnosis of simple partial seizures? Answer Choices 1 Facial and right arm twitching lasting about a minute, then loss of consciousness 2 Blank stare and impaired awareness for 10-20 seconds 3 Eye deviation with facial twitching that lasts 1-2 minutes 4 Loss of consciousness, followed by stiffening of the body and extremities 5 Very brief periods of whole body going limp, followed by a brief loss of consciousness

Correct Answer: Eye deviation with facial twitching that lasts 1-2 minutes Eye deviation with facial twitching that lasts 1-2 minutes' is the correct answer. Simple partial seizures can vary depending on what portion of the cortex has been effected. Patients may display isolated tonic or clonic activity or may have transient altered sensory perception. Clonic movements are jerks that may involve a specific body part or may spread to adjacent body parts, known as "Jacksonian march." Tonic activity is stiffening or sustained posturing, such as eye deviation in this case. Sensory seizures can involve illusions or hallucinations, but this would be difficult to diagnose in a young child. One important hallmark of a simple partial seizure is that the patient does NOT lose consciousness. 'Facial and right arm twitching lasting about a minute then loss of consciousness' is not the correct answer. While a patient having a simple partial seizure might experience facial and right arm twitching for that period of time, they are not going to have loss of consciousness. The type of seizure involved in this answer choice best describes a complex partial seizure. 'Blank stare and impaired awareness for 10-20 seconds' is not the correct answer. This description is best used to describe a patient who is having a nonconvulsive generalized seizure, otherwise known as an absence seizure. Patients having an absence seizure will rarely have any movement during the seizure. At most, they may have some minor motor activity, such as blinking. In an older child or adult, these episodes will leave the patient feeling as though they have short periods of time that they do not remember. 'Loss of consciousness followed by stiffening of the body and extremities' is not the correct answer. The fact that this description includes loss of consciousness is what excludes it from being a description of a patient having a simple partial seizure. This scenario better describes a person who is having a generalized tonic-clonic seizure. 'Very brief periods of whole body going limp followed by brief loss of consciousness' is not the correct answer. The fact that this description includes loss of consciousness is what excludes it from being a description of a patient having a simple partial seizure. This scenario better describes a person who is having an atonic seizure, which is characterized as an abrupt, brief loss of body tone, possibly followed by brief loss of consciousness.

A 50-year-old man is in persistent cardiac arrest despite 2 electrical shocks and adrenaline medication. While he and his wife played golf at a local course, he collapsed. When the paramedics arrived, he did have a pulse and blood pressure, but he arrested on the way to the hospital. The only medication he takes on a regular basis is an ACE-antagonist for high blood pressure. At the base of his neck you feel a hump, and you see some purple stretch scars on his abdomen; in addition to his obesity, he has a moon face. You draw some blood for electrolytes; the potassium is 1.9. What is most likely causing the patient's symptoms? Answer Choices 1 Cortisol producing tumor 2 Hashimoto thyroiditis 3 Myocardial infarction 4 Cardiomyopathy 5 Pheochromocytoma

Cortisol producing tumor Explanation Trunk obesity, moon face, purple stretch marks on the abdomen, and buffalo hump are typical symptoms of Cushing's syndrome. Cushing's syndrome can be caused by iatrogenic glucocorticoid therapy for M. Addison, skin diseases, inflammatory joint diseases, systemic diseases, hemolytic anemia, kidney diseases, leukemia, and pituitary hypersecretion of ACTH cortisol producing tumors: e.g. adrenal adenoma or small cell carcinoma of the lung. Because of the excessive secretion of corticosteroids, too much aldosterone is produced; this leads to an increased secretion of K+ and H+ ions through the kidney, and it ends in hypokalemia and metabolic alkalosis. The latter are the cause of the cardiac symptoms. Hashimoto thyroiditis, an autoimmune disease, is very often coexisting with M. Addison. The latter is hypocortisolism, the opposite of M. Addison. Symptoms include lethargy, weakness, and hunger for salt. There are no symptoms described, which would make the diagnosis of myocardial infarction or cardiomyopathy likely. Pheochromocytoma, a tumor of chromaffin cells, secretes excessive amounts of catecholamines and causes hypertension. They are mostly found in the adrenal medulla, in contrast to the glucocorticoid producing adrenal adenoma, which are located in the adrenal cortex; they are usually benign. Symptoms include tachycardia, hypotension, tachypnea, flushing, headaches, dyspnea, paresthesias, and constipation.

A 2-year-old boy presents with chronic respiratory infections, foul-smelling pale stools, weight loss, and wheezing. His mother reports using moist mist treatments repeatedly, and they offer some relief. A clinical exam shows delayed growth, an enlarged spleen upon palpation, and clubbing of fingers and toes. Auscultation of the lungs demonstrates pulmonary congestion. Lab results report a positive fecal fat test and sweat chloride test. What is the most likely diagnosis? Answer Choices 1 Cystic fibrosis 2 Pulmonary tuberculosis 3 Pulmonary histiocytosis 4 Pulmonary aspergilloma 5 Pulmonary nocardiosis

Cystic Fibrosis ystic fibrosis is an inherited disease that affects the respiratory and digestive systems. It affects the exocrine (mucus and sweat) glands of the body and is caused by a defective gene. It is the most common cause of chronic lung disease in children and young adults. Thick mucus is formed in the bronchial tree, which predisposes the person to chronic lung infections. Symptoms include no meconium stool in the first 24 hours of life, stools (pale or clay colored, foul-smelling, and floating), skin may taste salty (infants), persistent respiratory infections, coughing or wheezing, weight loss, clubbing of the fingers or toes, diarrhea, delayed growth, easy fatigue, and splenomegaly. Tests should include sweat chloride test, a fecal fat test (positive), bone X-ray, chemistry panel, upper GI, small bowel series, as well as immunoreactive trypsinogen. Treatments include antibiotics for respiratory infections and pancreatic enzymes to replace the missing enzymes. Mucomyst may be used on occasion to thin secretions. The use of ibuprofen has been shown to slow lung deterioration in some children. Other treatments include postural drainage, chest percussion, and other breathing treatments. Lung transplant may be considered in some cases. About 50% of patients with cystic fibrosis live beyond age 20. Few patients live beyond 35. Death occurs from pulmonary (lung) complications. Pulmonary tuberculosis is a contagious bacterial infection caused by Mycobacterium tuberculosis (TB). The lungs are primarily involved, but the infection can spread to other organs. The disease is characterized by the development of granulomas (granular tumors) in the infected tissues. Primary infection is usually asymptomatic. Symptoms may include initially but are not limited to minor cough and mild fever, fatigue, weight loss, coughing up blood, slight fever and night sweats, wheezing, rales, excessive sweating, joint pain, hearing loss, diarrhea, chest pain, breathing difficulty, positive Babinski's reflex, and clubbing of the fingers or toes. Pulmonary histiocytosis is characterized by inflammation of the small airways (bronchioles) and the small blood vessels in the lungs. This inflammation leads to stiffening (fibrosis) and destruction of the walls of the alveoli. Systemic involvement may cause rashes, pulmonary problems, enlargement of the spleen and liver, anemia, and death. In children, symptoms can include failure to thrive, weight loss, irritability, fever, seborrheic dermatitis of the scalp, generalized rash (petechiae or purpura), and chronically draining ears; bone pain may or may not be present. Pulmonary aspergilloma is a fungal mass that grows in pre-existing lung cavities (or can cause new lung cavities). The preexisting cavities may have been caused by a previous infection of histoplasmosis, tuberculosis, sarcoidosis, lung abscess, cystic fibrosis, or previous lung cancer. Symptoms include cough, coughing up blood, weight loss, and fever. Tests include bronchoscopy or bronchoscopy with lavage (BAL), chest X-ray, and sputum culture. The most effective therapy is a surgical resection (cutting out the fungus growth). If life-threatening bleeding occurs, emergency surgery may be the only choice of treatment. Pulmonary nocardiosis is an infection of the lung (pneumonia that is caused by a fungus-like bacterium). The causative organism is present throughout the world, and the infection is acquired through inhalation, causing a pneumonia-like illness. The disease can spread to any part of the body, but brain lesions and subcutaneous (under the skin) lesions are most common. Symptoms include cough with sputum production, progressive difficulty breathing, general discomfort, uneasiness or ill feeling (malaise), weight loss, fever (intermittent), night sweats, chest and joint pain, as well as liver and spleen enlargement (hepatosplenomegaly).

A 20-year-old woman on the orthopedic floor develops redness and inflammation of a knee that had ACL repair done. The knee is inflamed and tender to the touch. The surgical sites are significant for a white, pus-like drainage. There is no significant odor present, which would be indicative of a possible anaerobic infection. Drainage material was collected for Gram stain and culture. The Gram stain result report had many neutrophils present and many Gram-positive cocci. The culture grew colonies on blood agar that were beta-hemolytic (refer to the image) and catalase/coagulase-positive. This patient has what infection? Answer Choices: A Escherichia coli B Pseudomonas aeruginosa C Streptococcus pyogenes D Staphylococcus aureus E Streptococcus pneumoniae

D

A 21-year-old woman developed low-grade fever, sore throat, malaise, and fatigue that lasted several days. Physical examination revealed swollen lymph nodes and discomfort in the left upper quadrant of the abdomen. Examination of peripheral blood smear revealed 50% of atypical lymphocytes of the total white cells. Result for heterophile antibody test was positive. What is the most probable cause of infection? Answer Choices: A Cytomegalovirus B Respiratory syncytial virus C Influenza virus D Epstein-Barr virus E Varicella zoster virus

D

A 25-year-old man develops clinical signs of bacteremia in the hospital. Examination reveals erythema and tenderness, and a slight purulent discharge around the insertion site of a central venous catheter. Gram stain of discharge shows Gram-positive cocci in grape-like clusters. Culture sensitivity of the fluid showed methicillin-resistant staphylococcus epidermidis. What is the most appropriate antibiotic therapy? Answer Choices: A Cloxacillin B Cefazolin C Sulfamethoxazole-Trimethoprim D Vancomycin E Penicillin

D

A 33-year-old man presents with a 2-day history of severe diarrhea and vomiting. He had been on a business trip to Asia 3 days prior to presentation, and he reports eating food bought from street vendors. He describes his stools as watery and not bloodstained. He is allergic to seafood, and he takes antacids for peptic ulcers. On examination, he is moderately dehydrated; temperature is 37 C, PR is 100, and BP is 120/60. What pathogenic organism is most likely causing his symptoms? Answer Choices: A Staphylococcus aureus B Bacillus cereus C Escherichia coli, serotype O157:H7 D Vibrio cholera E Vibrio parahaemolyticus

D

A 48-year-old woman presents with persistent dry cough. She has a 28-year history of smoking. Her chest X-ray is unremarkable. The cough began shortly after she started a new medication. What medication do you suspect is the cause? A Verapamil B Digoxin C Lovastatin D Lisinopril E Acetaminophen

D

A 5-year-old boy presents with a 4-day history of bloody diarrhea. He has had fever up to 104 degrees F, abdominal pain, and painful defecation. His past medical history is unremarkable, and he has had no surgeries. He is on no medications and has no drug allergies. He attends a local daycare with 9 other children. On physical examination, his abdomen is tender with hyperactive bowel sounds. While in the emergency department, he has a 5-minute generalized seizure. What pathogen is the most likely cause of the patient's seizure? Answer Choices: A Campylobacter jejuni B Giardia lamblia C Rotavirus D Shigella sonnei E Salmonella typhimurium

D

A 56-year-old man presents with primary hypertension. His additional medical history includes only nephrolithiasis. Past analysis of his kidney stones has revealed a calcium oxalate composition. What class of antihypertensives might best be avoided due to his history of nephrolithiasis? A Angiotensin-converting enzyme (ACE) inhibitors B Beta blockers C Calcium channel blockers D Loop diuretics E Thiazide diuretics

D

A 62-year-old woman presents with extreme fatigue and shortness of breath. The symptoms began about 24 hours ago and have progressively worsened within the last 4 hours. Vital signs on arrival are as follows: HR 90 beats per minute; BP 165/72 mmHg; RR16/min; SpO2 98% on 4L/min supplemental oxygen by nasal cannula. 12-lead ECG demonstrates ST-segment elevation of 2 mm in leads V4-V6. In addition to an aspirin tablet, what medication would be most appropriate in the emergency management of this patient? A Dobutamine B Dopamine C Morphine D Nitroglycerin E Vasopressin

D

A 63-year-old woman presents with a 1-hour history of left shoulder pain and nausea. She has a past medical history of coronary artery disease and had a stent placed 5 years ago. An ECG shows large R waves and ST segment depression in leads V1, V2, and V3. These ECG findings are most consistent with what condition? A Acute ischemia without myocardial infarction B Acute lateral myocardial infarction C Acute inferior myocardial infarction D Acute posterior myocardial infarction E Acute anterior myocardial infarction

D

A 75-year-old woman was hospitalized with a localized subcutaneous abscess at the site of a cat bite. Pus drained from the abscess was cultured in the microbiology laboratory. Colonies of small Gram-negative coccobacilli grew on chocolate agar incubated at 37°C for 24 hours. The bacterium was non-motile, oxidase and catalase positive, urease negative, and showed fermentative activity on carbohydrates. The isolate is most likely to be what zoonotic pathogen? Answer Choices: A Bartonella henselae B Ehrlichia chaffeensis C Coxiella burnetii D Pasteurella multocida E Bordetella bronchiseptica

D

What organism is a common cause of various levels of infection, requiring treatment, but also is identified as common normal flora of the human body? Answer Choices: A Histoplasma capsulatum B Cryotococcus neoformans C Aspergillus fumigatus D Candida albicans E Pneumocystis jiroveci

D

A 47-year-old female presents to the office complaining of excessive menstrual flow for the past two months. She complains of mid-cycle bleeding, as well as periods that last for nine days. What is the mostly likely cause of her bleeding? A Chlamydia trachomatis infection B Adenocarcinoma of the uterus C Uterine leiomyoma D Anovulatory uterine bleeding E Pregnancy

D Anovulatory uterine bleeding D While all are causes for abnormal bleeding, anovulotory bleeding occurs in perimenopausal women due to the unopposed estrogen. STD and pregnancy would be more likely during reproductive years. Neoplasms accounts for approximately 20% of abnormal uterine bleeding in the post reproductive years.

A 70-year-old woman presents to the ED with nausea, anorexia, vomiting, fever, chills, and flank pain. Symptoms appeared abruptly after a 2-day history of mild dysuria. Her family reports that about 2 months ago, she started having urinary urgency and frequency and noted excessive urinary production of around 3-4 liters daily. She has a history of hypertension, poorly regulated type II diabetes, and mild cognitive impairment. Her long-term medications are aspirin and metformin, and 2 months ago, her family practitioner included furosemide, statin, and proposed insulin, but she was not willing to take any parenteral medication. Her fear of needles was the reason she gave for not agreeing to get recommended immunizations. On admission, she appears lethargic; her blood pressure is 80/40; pulse rate is 120; respiratory is 18; and temperature is 103.4. Question What is the primary risk factor responsible for her current condition? Answer Choices 1 Diabetes insipidus 2 Diabetes mellitus 3 Diuretic therapy 4 Cognitive impairment 5 Missed immunization

DM Explanation Your patient has a serious systemic infection, most probably pyelonephritis in the settings of poorly regulated diabetes mellitus. In diabetes, infections may precipitate metabolic disturbances, but metabolic disturbances may also facilitate the infection. Metabolic disturbances (hyperglycemia, acid-basic dysregulations) impair humoral immunity and the function of polymorphonuclear leukocytes and lymphocytes. Not only diabetes, but other chronic diseases, increase the risk of acquiring systemic infections. The strongest connection was found in chronic lung disease and chronic kidney disease. Long-standing diabetes may also lead to chronic kidney disease, which could be an additional risk factor in this patient. Although polyuria with urinary frequency and urgency may be caused by diabetes insipidus, it is much more frequently caused by uncontrolled diabetes mellitus. Another reason that your patient may have polyuria is diuretic therapy. Diuretic therapy may contribute to polyuria, but it does not contribute to the development of systemic infection. Patients with mild cognitive impairment can forget to take their medications from time to time, thus contributing to poor regulation of their diseases, but mild cognitive impairment is not a risk factor for systemic infection. Vaccines recommended in seniors (over 65) are: influenza, herpes zoster, diphtheria/tetanus, pertussis, and pneumococcal. Not getting immunized because of the fear of a needle will put your patient at risk for developing those diseases. However, she was not exposed to any of those diseases and has no signs and symptoms of any of them.

3 types of Dialysis

Dialysis started CKD stage 5 or symptoms of Uremia: Three options for dialysis- Hemodialysis: • MC type; Usually done in center, 3x week 4hrs at a time Peritoneal Dialysis: • MC complication is infection/peritonitis Renal transplantation: • Most satisfactory treatment for ESRD • Living related donor is best match

A 5-year-old girl presents with a 2-month history of a gradually increasing swelling in the neck. The child is otherwise asymptomatic. There is a similar complaint in most neighboring children. Cauliflower is the main crop in the region. She has no other developmental problems. On physical exam, her thyroid gland is non-tender, soft, and symmetrically enlarged without any palpable nodules or signs of compression. The thyroid function test showed slightly low thyroxine (T4) and normal TSH and triiodothyronine (T3) levels. Her urinary iodine level was also low. Question What is the most likely diagnosis? Answer Choices 1 Toxic nodular goiter 2 Graves' disease 3 Diffuse non-toxic goiter 4 De Quervain thyroiditis 5 Multinodular goiter

Diffuse non-toxic goiter Explanation The clinical presentation, exam, and lab findings are suggestive of diffuse non-toxic goiter. Diffuse non-toxic goiter, colloid goiter, or simple goiter is the name given to the diffuse enlargement of the thyroid gland unaccompanied by hyperthyroidism. Most cases are in the euthyroid stage, although they have passed a transient phase of hypothyroidism due to iodine deficiency. TSH levels are normal or slightly increased. Goiter occurs in 2 forms, either endemic or sporadic. The endemic form occurs due to the lack of dietary iodine or due to the intake of goitrogens like consumption of cassava root, cabbage, cauliflower, turnips, etc. Endemic goiter is most prevalent in high mountainous areas. Sporadic form is more common in the U.S. and in Europe, and the etiology is not known. This is said to be multifactorial in origin. Generally, most simple goiters are asymptomatic, manifesting as a painless symmetric enlargement of the thyroid gland. Spontaneous hemorrhage may cause sudden onset of pain and swelling. Examination of the gland reveals a nontender, soft gland with symmetric enlargement, without any palpable nodules. A low urinary iodine level (<10 μg/dL) is supportive of a diagnosis of iodine deficiency goiter. A thyroid function test generally shows a low total T4, with normal T3 and TSH. Treatment is with iodine or thyroid hormone replacement, which induces variable levels of goiter regression, depending on the duration of swelling and the degree of fibrosis. Multinodular goiter is an end stage of a long-standing simple goiter, and it is characterized by an extreme degree of thyroid enlargement with characteristic nodularity. Most cases are euthyroid, but about 10% cases develop thyrotoxicosis. Large goiters can cause compressive symptoms such as dysphagia, respiratory distress (due to tracheal compression), or plethora (venous congestion). On examination of the thyroid gland, the architecture is distorted with presence of multiple nodules of varying size. A thyroid function test is usually normal. Toxic nodular goiter is characterized by subclinical hyperthyroidism or mild thyrotoxicosis in addition to features of goiter. The clinical presentation of toxic nodular goiter includes tachycardia, nervousness, atrial fibrillation or palpitations, tremor, or weight loss. Plummer disease or toxic nodular goiter differs from Graves' disease by lack of ophthalmopathy and dermatopathy. The thyroid function test shows low TSH level with normal or minimally increased T4 and elevated T3, which is usually of a greater degree than T4. Heterogeneous uptake and multiple regions of elevated and diminished uptake is evident on thyroid scan. Graves' disease is characterized by hyperthyroidism, diffuse thyroid enlargement, and ophthalmopathy. It is an autoimmune disease. Thyroid function test shows low TSH level and elevated total and unbound thyroid hormone levels. De Quervain thyroiditis is characterized by a transient inflammation of the thyroid gland, which may be due to viral infections. Patients generally present with a painful, enlarged thyroid gland, and it is sometimes associated with fever. Thyroid function test evolves through 3 distinct phases: the thyrotoxic phase, the hypothyroid phase, and the recovery phase. The thyrotoxic phase shows elevated T4 and T3 levels and suppressed TSH levels. A high ESR and low radioiodine uptake helps in confirming the diagnosis.

Which of the following, is a classical hydatidiform mole? Answer Choices 1 Diploid karyotype, absence of an embryo, swelling of all villi 2 Triploid karyotype, trophoblastic hypoplasia, viable embryo 3 Trophoblastic hyperplasia, normal karyotype, twin embryos 4 Vascular malformations, intraplacental mass lesions, fetal hydrops 5 Decreased uteroplacental blood flow, multiple placental lobes, vesicular pattern

Diploid karyotype, absence of an embryo, swelling of all villi

A 42 year-old woman presents complaining of shortness of breath, three days of fever as high as 103F, and has a cough productive of green sputum. On physical examination, you hear crackles in her lungs. A chest radiograph reveals a consolidation in the left lower lobe. What do you expect to hear when you percuss this patient's left lower thorax? A Dull B Flat C Hyperresonant D Resonant E Tympanic

Dullness Consolidation leads to dullness (A) to percussion. Flat sounds (B) are noted with organ percussion (i.e., hepatic flatness), hyperresonant (C) sounds are noted in pneumothorax, normal lung parenchyma results in resonance (D), and percussion of the gastric air bubble leads to tympanic sounds.

A 3-year-old girl presents with vomiting and diarrhea that started while she was at day care. Her diarrhea was described as watery with increased frequency. She has just started toilet training. She threw up 2 times while at day care and 1 time in the car. Her mother was called to take her home. What condition requires exclusion from day care due to the potential for outbreaks? Answer Choices: A Pediculosis B Ringworm C Impetigo D Viral respiratory illness E Varicella-zoster

E

A 33-year-old woman presents due to a 15 pound weight gain over 2 months; there is also muscle weakness, menstrual irregularities, amenorrhea, infertility, skin bruising, memory loss, and periods of depression. She denies any medication use or dietary changes; she has tried to lose weight unsuccessfully through increased exercise. She denies any headache, vision changes, hearing changes, headache, chest pain, abdominal symptoms, polyuria, polydipsia, or breast discharge. Her physical exam reveals a blood pressure of 145/94 mm Hg. Her skin physical exam is remarkable for the findings in the image. Question What is an expected diagnostic finding in this patient? Answer Choices 1 Hyperkalemia 2 Metabolic acidosis 3 Elevated FSH and LH levels 4 Hypoglycemia 5 Elevated urinary free cortisol (UFC) levels

Elevated urinary free cortisol (UFC) levels Explanation This patient is presenting with signs and symptoms consistent with Cushing's syndrome; in this case, it is most likely from an adrenal adenoma source. Expected laboratory findings associated with the syndrome may include an elevated leukocyte count, hyperglycemia, and a hypokalemic metabolic alkalosis, which may occur in patients with markedly elevated cortisol due to cortisol activation of the renal mineralocorticoid receptor. The diagnosis of Cushing syndrome due to endogenous overproduction of cortisol requires the demonstration of inappropriately high serum cortisol or urine cortisol levels. Urinary free cortisol (UFC) determination has been widely used as an initial screening tool for Cushing syndrome because it provides measurement of cortisol over a 24-hour period. Menstrual irregularities, amenorrhea, infertility, and decreased libido may occur in women due to inhibition of pulsatile secretion of luteinizing hormone (LH) and follicle-stimulating hormone (FSH), which is likely due to interruption of luteinizing hormone-releasing hormone (LHRH) pulse generation.

A 6-year-old boy presents due to lethargy, polyuria, nocturnal enuresis, and polydipsia. His mother tells you that he complains of being tired and thirsty all of the time. You note that he has lost 5 pounds since his last visit 6 months ago. Question What is the most likely diagnosis? Answer Choices 1 Diabetes mellitus type I 2 Diabetes mellitus type II 3 Leukemia 4 Lymphoma 5 Diabetes insipidus

Explanation Diabetes mellitus type I can occur at any age, but it most commonly arises in children and young adults; peak incidences are before school age and during puberty. Patients often present with polyuria, polydipsia, weakness, fatigue, thirst, blurred vision, and/or nocturnal enuresis. Type II diabetes and diabetes Insipidus are much less common in children. Leukemia and lymphoma can both have symptoms of fatigue (leukemia more so than lymphoma), but patients do not experience polyuria and/or polydipsia.

A 2-year-old boy is brought to the pediatrician because of weakness and recurrent rectal bleeding. According to his mother, he has been passing 3-4 fluid, maroon and red stools per day, and has become progressively more apathetic and withdrawn, but never showed signs of abdominal pain, fever, vomiting, diarrhea, jaundice, or bleeding from other sources. He has reached all developmental landmarks for his age. Vital signs are within normal ranges, and physical examination, including an anorectal examination, is normal except for pallor and apathy. A complete blood count shows the following: Ht 32% Hb 10.0 g/dL MCV 78 fl MCHC: 26 g/dL WBC 8.600 /mm3. w/ normal differential. A small-bowel follow-through radiograph shows a diverticulum near the terminal ileum. Question What is the most appropriate next step? Answer Choices 1 Barium enema 2 Colonoscopy 3 99mTc scintigraphy 4 Upper endoscopy 5 Tagged erythrocyte scan

99mTc scintigraphy Lower gastrointestinal bleeding is a common complaint in infants. Diagnosis requires careful exclusion of extraintestinal bleeding sources (e.g. nasopharyngeal or maternal blood ingestion during partum) and of medications or foods that may impart a blood-like appearance to stool such as foods with red food coloring in them (e.g., red jello, etc). The table below lists the most common etiologies of lower gastrointestinal bleeding in each age group: Neonates Bacterial enteritis, milk-protein allergies, anal fissures, intussusception, lymphoid nodular hyperplasia; Infants Intussusception, esophagitis, gastritis, duodenitis, ulcers, colonic polyps, anorectal disorders, Meckel's diverticulum, infectious diarrhea, volvulus, bowel duplication; Children All the above conditions; duodenal ulcer, Mallory-Weiss tears Henoch-Schönlein purpura. In a chronically bleeding patient without signs of anorectal disease, evaluation should start with contrasted radiography (barium swallow, upper GI series, small-bowel follow-through, or barium enema). These radiographs may reveal foreign bodies, mucosal lesions such as ulcers, esophagitis, or inflammatory bowel disease; congenital anomalies such as volvulus, duplication, and malrotation. However, colonoscopy has higher sensitivity for detecting lower gastrointestinal bleeding and enjoys an overall success rate of 80%. The combination of a diverticulum in the distal ileum and rectal bleeding in an infant is strongly suggestive of Meckel's diverticulum. Another feature associated with this disease is passage of maroon or brick-red stools. A 99mTc scan (Meckel scan) is the examination of choice for locating this lesion. Meckel's diverticulum is a remnant of the omphalomesenteric duct. It's present in 1-2% of the general population, and is the most common congenital anomaly of the small bowel. Most diverticula are asymptomatic, but a small percentage ulcerates because of acid-producing ectopic gastric mucosa and causes rectal bleeding. Perforation and peritonitis may also occur. Treatment is surgical.

A 34-year-old ornithologist is seeking medical advice because he plans to travel to Africa. He is worried because of the endemic areas of yellow fever in Africa, and he wants to know if there is a vaccine. He once had an anaphylactic reaction to eggs; otherwise, he is healthy. You explain to him that he cannot be immunized. What vaccine is also contraindicated for this patient? Answer Choices: A Influenza B Hepatitis B C Clostridium tetani D Bordetella pertussis E Corynebacterium diphtheriae

A

A 42-year-old man has had systolic blood pressure in the 140's and diastolic blood pressure in the 80's on several occasions despite changing his diet and exercise regimen. His physician decides to start him on hydrochlorothiazide. What electrolyte abnormality may be associated with hydrochlorothiazide? A Hypokalemia B Hypoglycemia C Hypernatremia D Hypocalcemia E Hypermagnesemia

A

A 50-year-old man presents to your office for a follow up appointment of his hypertension. He has complaints of some non-specific chest discomfort, so you decide to perform an ECG. The ECG demonstrates peaked T waves in several leads without any other abnormality. Which of the following medications is most likely to cause this ECG finding? A Lisinopril B Furosemide C Atenolol D Hydrocholorothiazide E Prazosin

A

A 2.9 kg baby boy was born to a female in her late twenties. This is the couple's first child. The pregnancy and the delivery were uneventful. The baby is doing fine. However, at the routine neonatal baby check, you notice that the baby has small ears, down-sloping oval eyelid openings, and a flat occiput. He also has broad, short-fingered hands. You send blood for karyotyping on suspicion of a genetic disorder and inform the parents accordingly. Refer to the image. Which other features could have been associated with this disorder? A Micrognathia B Duodenal atresia C Polydactyly D Cleft lip E Rocker-bottom feet

A Micrognathia *B Duodenal atresia* C Polydactyly D Cleft lip E Rocker-bottom feet Down syndrome is the most common autosomal trisomy. Ninety-five percent of children with Down syndrome have 47 chromosomes with three number 21 chromosomes. Trisomy 21 occurs in 1 in 600 live births. The risk of having a child with extra chromosome 21 increases with advancing maternal age. This risk rises dramatically after age 35 years. Most children with trisomy 21 are born to females younger than 35 years of age, however, most females give birth prior to age 35 years. The mechanism for the increased incidence of trisomy 21 in fetuses of older mother is not understood. The extra chromosome comes from the father in a small percentage of cases. The recurrence risk of parents of children with trisomy 21 increases 1%- 2%, (unless age-related risk is greater). Four percent of children with Down syndrome have 46 chromosomes, with a translocation of the third number 21 chromosome to another chromosome- usually a D-group (number 13, 14, or 15) or a G-group (number 21 or 22). One percent of children with Down syndrome have chromosome mosaicism; some cells having 46 chromosomes, and two number 21 chromosomes, and some cells having 47 chromosomes, and three number 21 chromosomes. Clinical features include: round face, flat profile, flat occiput, epicanthus, 'mongoloid' slant to eyes, Brushfield spots on the iris, protuberant tongue, broad short-fingered hands, clinodactyly little finger, single palmar crease, wide-spaced big toe, short stature, hypotonia, and general learning difficulties. Associated problems may include congenital anomalies- heart disease, duodenal atresia, and Hirschsprung's disease; hypothyroidism, increased risk of leukemia and increased incidence of Alzheimer's disease. Associated minor problems may include dry skin, feeding difficulties, recurrent upper respiratory tract infections, glue ear, feeding difficulties, hypermetropia, and myopia. With increase life expectancy in Down syndrome, issues relating to employment, financial security, health care and living situation must be addressed by families and social service agencies. Supervised group home living has become a successful alternative for these and other mentally retarded adults. Micrognathia (small mandible) and rocker bottom feet are features of trisomy 18, which occurs in 1 in 8000 live births and has a bad prognosis with almost 90% of affected children dying by the end of 1 year. Cleft lip with or without cleft palate, and polydactyly is features of trisomy 13, which occurs from 1 in 4000 to 1 in 10,000 newborns. The prognosis is extremely poor.

In a patient who does not have thyroid disease, an elevated serum thyroid stimulating hormone (TSH) may be found under which of the following conditions? A acute corticosteroid administration B acute psychiatric admission C development of an hCG-secreting tumor D pregnancy E use of amphetamines

acute psych admission

Which of the following lab test is considered the "gold standard" for detecting/diagnosing an inherited thrombophilia? A anticardiolipen deficiency B factor V Leiden deficiency C hyperhomocysteinemia D protein C and S deficiency E There is no single lab test.

factor 5 leiden def

You are evaluating an 80-year-old female for the first time. She has a history of mild Alzheimer's disease, for which she takes Aricept. She states that she feels fine but her daughter feels she is depressed and has been complaining of not feeling well. Her daughter admits that the patient has a history of primary hyperparathyroidism. What laboratory results would be most consistent with her diagnosis of hyperparathyroidism? A high serum calcium B low intact PTH C low cortisol D low urine calcium E high cortisol

high serum calcium The correct answer is (A). The hallmark of primary hyperparathyroidism is a high serum calcium and high intact PTH. A low intact PTH is consistent with hypoparathyroidism. The urine serum calcium is usually high in primary hyperparathyroidism. Cortisol is related to endocrine conditions affecting the adrenal cortex.

During the examination of the infant, either for a well baby checkup or if ill, it is important to listen carefully to the infant's breathing and the quality of the cry. What might a shrill high-pitched cry indicate? Answer Choices 1 Cri-du-chat 2 Profound mental retardation 3 Congenital hypothyroidism 4 Tracheomalacia 5 Increased intracranial pressure

increased intracranial pressure ncreased intracranial pressure (ICP) may result from cerebral edema associated with head trauma, hypoxic-ischemic events, infection, metabolic disorders, hydrocephalus, and space-occupying lesions. Besides a high-pitched cry, infants may also present with widened sutures, a bulging fontanelle, persistent downward eye deviation (sunset eye sign), and increasing head circumference. If left unrecognized and untreated, increased ICP can result in significant morbidity and mortality by causing irreversible neurologic injury. Cri-du-chat syndrome is an autosomal deletion syndrome caused by a partial deletion of chromosome 5p. It is characterized by a distinctive mewing, catlike cry that occurs in infancy. This cry is due to structural abnormalities of the larynx and CNS dysfunction. In addition, affected individuals also display growth failure, microcephaly, facial abnormalities, and mental retardation. Profound mental retardation is suggested by the absence of any cry. Congenital hypothyroidism may result from absent or abnormal development, destruction, or failure of stimulation of the thyroid gland by the pituitary, and/or by defective or abnormal synthesis of thyroid hormones. Infants profoundly affected have a distinctive appearance that has been termed cretinism, which includes a puffy-appearing face, a dull look, a large protruding tongue, dry brittle hair, a low hairline, and jaundice. Feeding is poor, with possible choking; constipation is common. Cries are hoarse and infrequent, and the infants are sluggish and inactive. Muscle tone is decreased. If left untreated, it can lead to severe mental and growth retardation. If recognized and treated early, significant improvement can be seen. Tracheomalacia is the most common cause of expiratory stridor. It is caused by a defect of the cartilage, resulting in loss of rigidity or partial collapse of the tracheal cartilage with respiration. This causes an expiratory high-pitched, crowing noise. Tracheomalacia may also be found with lesions, such as vascular rings and slings, that compress the airway Most children outgrow this condition by the time they are 3 years-old.

The pregnant mother of a 2-year-old patient is concerned about exposure to environmental neurotoxins affecting her developing fetus. She asks you questions about whether the placenta would block any transfer of toxins to a growing fetus. Which environmental neurotoxin transfer to the fetal system is actually enhanced by the placenta? Answer Choices 1 Mercury 2 Cadmium 3 Polychlorinated biphenyls 4 Insecticides 5 Environmental tobacco smoke

mercury Explanation Methyl mercury is absorbed almost completely and crosses the placenta easily and then becomes stored in the fetus. Fetal blood concentrations exceed maternal concentrations by 50-100%. High doses can result in mental retardation, spastic paralysis, and death. Low doses are linked to deficits in neuromotor performance, cognition, memory, and language. The placenta effectively blocks the transfer of cadmium. Studies have shown cadmium concentrations in umbilical cord blood to be significantly lower than that in maternal blood and placental cadmium concentration to be highest. Polychlorinated biphenyls (PCP) have contaminated sport fish, particularly bottom-feeding species from water contaminated with PCPs. Also during the 1940s and 1950s the inside of concrete silos on many farms in the Midwest were coated with sealants containing PCPs that over time have peeled off and become mixed with silage to feed beef and dairy cattle. Incinerators and other PCP-disposal facilities or hazardous waste sites are other sources of PCP exposure. PCPs are synthetic hydrocarbons that are lipophilic, have a long half-life, and can cross the placenta easily. Fetuses and neonates are more sensitive to PCPs because the hepatic microsomal enzyme system that facilitates metabolism and excretion are not fully functional. Insecticides or pesticides appear to cross the placenta relatively easily being lipophilic and having a low molecular weight. Residue levels of DDT and its metabolites were detected in maternal blood, placenta, and umbilical cord blood of mother/child pair studies. A correlation exists between pesticide concentration and age, dietary habits, and area of residence of pregnant women. Environmental tobacco smoke (ETS) exposure during fetal development is one of the most ubiquitous and hazardous of environmental exposures. Placental vascular resistance is often increased when women smoke during pregnancy and there are subsequent alterations of protein metabolism and enzyme activity in cord blood. Maternal uterine blood flow is lowered with reduced flow of oxygen from the uterus to the placenta. There are increased levels of carboxyhemoglobin in both maternal and fetal blood when the mother smokes that can lead to fetal hypoxia with consequent chronic fetal hypoxic stress.

A 13-year-old girl presents with concerns about excessive vaginal discharge without itching or burning for the past 3 days. She denies ever being sexually active. Her last menstrual period was 10 days ago. On examination you find a thin, white, homogeneous discharge that has a distinct amine odor when potassium hydroxide is added. On saline wet mount, epithelial cells are covered with bacteria. What is the most appropriate management of this patient? Answer Choices 1 Metronidazole 2 Tricholoroacetic acid 3 Clotrimazole 4 Fluconazole 5 Nystatin

metronidazole Explanation This patient has the classic signs of bacterial vaginosis (BV), including the thin discharge with the fishy odor on KOH prep, and clue cells on wet prep. Standard treatment of BV is metronidazole 500 mg po bid for 7 days. Tricholoroacetic acid is used for the treatment of genital warts. Clotrimazole, fluconazole, and nystatin are all used in the treatment of candidiasis.

An 18-year-old woman comes in for evaluation of "losing weight without meaning to." She also feels weak and in "always in the bathroom." Her appetite is normal but she "can't get enough to drink." Examination shows that she has lost 17# since her last visit a year ago. She is 66" tall and now weighs 120#. She is mildly orthostatic, but no other abnormalities are noted. A random blood sugar done in the office is 260mg/dl. Which results are most likely on measurement of her lipoproteins at this time? A decreased high density lipoprotein level B extremely elevated triglycerides C markedly increased total cholesterol D mildly elevated low density lipoproteins E normal profile

mildly elevated low density lipoproteins In persons with diabetes mellitus, type 1, low density lipoproteins, trigycerides (B), and total cholesterol (C) are likely to be slightly elevatated. High density lipoproteins (A) remain about the same as the patient's baseline. Once the glucose level is controlled, the lipid levels (E) on the profile typically return to normal.

A 45-year-old woman status-post total abdominal hysterectomy and bilateral salpingo-oophorectomy secondary to Stage I ovarian cancer. She has completed a course of cisplatin and cyclophosphamide, and she presents for follow-up. Serial CA-125 has returned to normal. What is the most common site of metastasis for this tumor? Answer Choices 1 Omentum 2 Lung 3 Liver 4 Bone 5 Brain

omentum Explanation The most common route for spread of ovarian carcinomas is transcelomic, along with normal flow of peritoneal fluid, in which case common sites for metastasis include the pericolic gutters, the omentum, small and large bowel. Only rarely do ovarian tumors spread via hematogenous dissemination to the lung, liver or bone, making follow-up imaging of these areas unnecessary. The brain is not considered a site for ovarian metastasis.

Couvoisier Sign

presence of an enlarged gallbladder which is nontender and accompanied with mild jaundice, the cause is unlikely to be gallstones.

A 31-year-old woman is being evaluated for irregular, infrequent menstrual periods. On further questioning, she complains of headaches, fatigue, and breast discharge. She takes ibuprofen only occasionally. Which of the following labs would most likely be elevated in this patient? A BUN and creatinine B luteinizing hormone (LH) and follicle-stimulating hormone (FSH) C oxytocin D prolactin E TSH

prolactin This patient's symptoms are consistent with a pituitary adenoma. Prolactinomas account for about half of all functioning pituitary tumors and may secrete PRL, GH, and ACTH.

A 22-year-old couple presents for the evaluation of the primary infertility. The woman has no symptoms; her family and gynecological history are not relevant. Her spouse regularly takes pancreatic enzymes, albuterol, and vitamins; he also frequently takes antibiotics because of the presence of the recurrent respiratory tract infections, bronchiectasis, and bronchiolectasis; there is also exocrine pancreatic insufficiency and intestinal dysfunction. Question What is the next step in the evaluation of their inability to conceive? Answer Choices 1 Sweat test 2 Serial ovarian sonography 3 Semen analysis 4 Progesterone test 5 Hysterosalpingogram

semen analysis Explanation The correct response is semen analysis. Infertility is due to the female partner 1/3 of the time, the male partner 1/3 of the time, and both partners or the unknown problems in the remaining 1/3 of cases. In this case, the husband has signs and symptoms that could cause problems conceiving. The most common causes of the male infertility are structural abnormalities, sperm production disorders, ejaculatory disturbances, and immunologic disorders. Symptoms, signs and the therapy this patient is is receiving should raise the possibility that he suffers the cystic fibrosis (CF). It is estimated that more than 95% of CF male patients are infertile. The most common findings is azoospermia due to the epididymal obstruction and/or the failure of the vas deferens to develop properly. A small minority of male CF patients are still fertile, and for that reason sperm analysis should be routinely offered in all CF patients. Sample semen is evaluated for volume, sperm count, motility and morphology. Patients with poor semen quality or numbers may be prepared for intrauterine insemination by washing and concentrating the ejaculate. Symptoms, signs, and the therapy that the husband is receiving should raise the possibility that he suffers the cystic fibrosis (CF). The diagnosis of CF rests on the combination of clinical criteria and abnormal CFTR (cystic fibrosis transmembrane conductance regulator), as documented by elevated sweat chlorde values, nasal potential difference responses, and the administration of the various solutions and by CFTR mutation analysis. Even if it is estimated that more than 95% of male CF patients are infertile, a minority with certain mutations is still fertile. For that reason, in all CF patients, sperm analysis should be routinely offered along with reproductive health education. Serial ovarian sonographic evaluations may help in demonstrating the existence of the ovulation: the development of a mature antral follicle and its subsequent collapse during ovulation. It is time consuming and is not always accurate; however, it may be helpful in supporting the diagnosis of polycystic ovary syndrome (PCOS). There is no reason to perform this test in a female patient with a non-contributiong personal and family history when the most probable cause is a husband's disease. The progesterone test is a test used to confirm ovulation; blood is drawn about 7 - 10 days after ovulation at a time when progesterone levels peak, and the level of progesterone helps in the confirmation of ovulation. There is no reason to perform this test in a female patient with a non-contributing personal and family history. Hysterosalpingogram (HSG) is a test of the patency of the Fallopian tubes and exploration of the atrial cavity. Dye is injected through the cervix and up into the uterus and Fallopian tube; its movement is observed on a video screen. There is no reason to perform this test in a female patient with a non-contributing personal and family history.

Which one of the following is a characteristic finding on computed tomography (CT) of the abdomen in a patient with acute diverticulitis? A toxic megacolon B air-fluid levels C soft tissue inflammation of the pericolic fat D thinning of the colon wall E paucity of bowel gas in the colon

soft tissue inflammation of the pericolic fat

A patient presents with clinical features of thrombotic thrombocytopenic purpura. They also have an associated diarrhea positive hemolytic uremic syndrome. Which is the best treatment option for this patient? A Aspirin and dipyridamole B Glucocorticoids C Plasma exchange D Splenectomy and prostacyclin E Supportive therapy and dialysis

supportive therapy and dialysis This patient has a diarrhea-associated hemolytic uremic syndrome with clinical features of TTP. D+HUS is associated with infectious etiologies, usually through food borne illnesses. Treatment consists of supportive therapy, treating the underlying infection, and dialysis if renal function warrants. All other therapies listed are used in the treatment of TTP or diarrhea negative HUS

A 43-year-old obese man presents for a health maintenance visit. On physical exam, it is noted that his waist circumference is 106 cm and blood pressure is 148/92 mm Hg. Which of the following fasting laboratory levels would suggest a diagnosis of metabolic syndrome (syndrome X) in this patient? A HDL of 45 mg/dL B LDL of 180 mg/dL C triglyceride of 190 mg/dL D glucose of 100 mg/dL

triglyceride of 190 mg/dL Metabolic syndrome is found in approximately 25% of Americans. It is defined as three or more of the following findings: waist circumference of greater than 102 cm in men or greater than 88 cm in women; serum triglyceride level of at least 150 mg/dL, HDL level of less than 40 mg/dL in men or less than 50 mg/dL in women; blood pressure of at least 130/85 mm Hg; and serum glucose level of at least 110 mg/dL.

A 62-year-old woman presents to her OB/GYN clinic for an annual exam. She reports that she has not felt well for several months, but her family practice physician retired, and she kept hoping her symptoms would improve. She reports stretch marks and a weight gain of about 20 pounds, especially noticeable in her face. She feels fatigued and has no sex drive; she also feels more irritable. She notes increased dark hairs growing on her chin. The patient reports easy bruising, but denies blood in the stools. She denies fever, chills, foreign travel, a change of sex partner, and suicidal thoughts. Her past medical history is significant for asthma, with periodic hospitalizations for asthma attacks. She achieved menopause at the age of 51 years. She has not had any surgeries. She takes prednisone 40 mg daily, an inhaler with fluticasone propionate/salmeterol 2 times daily, and an albuterol inhaler as needed for her asthma. She has no allergies. She is retired and lives at home with her husband; she denies use of tobacco, alcohol and street drugs. BP is 162/92 mm Hg, and pulse is 98. Respiratory rate is 12. Weight is 170 lbs, and height is 63", making her BMI 30.11. On physical exam, the patient has a rounded, full face; she appears fatigued, but she is pleasant and cooperative. Striae are noted on her abdomen, and hirsutism is present on her chin and abdomen. There appears to be a large fat deposit on the upper back. Her abdomen is obese, with some muscle wasting on the patient's extremities. The remainder of her exam is normal. Question What prescription intervention is the most appropriate for this patient's condition? Answer Choices 1 Discontinue inhaled fluticasone propionate/salmeterol 2 Discontinue prednisone 3 Initiate beta-blocker (e.g., metoprolol) 4 Initiate combination hormone therapy (e.g., estradiol/norethindrone acetate) 5 Initiate stimulant for weight loss (e.g., phentermine hydrochloride)

Explanation This patient is presenting with Cushing syndrome, an excess of glucocorticoids; it is most likely iatrogenic, caused by her daily prednisone. The most appropriate prescription intervention for her condition is to discontinue the prednisone. This common steroid is often given for acute flares of chronic diseases. However, chronic daily use can lead to a glucocorticoid excess known as Cushing syndrome. Cushing syndrome is characterized by central obesity, muscle wasting, hirsutism, a 'buffalo hump' on the upper back, 'moon facies', weight gain, hypertension, decreased libido, irritability, and several other possible signs/symptoms. Unrecognized and untreated Cushing syndrome can lead to osteoporosis. Tapering off the prednisone (rather than immediate discontinuation) helps prevent sudden adrenocortical insufficiency. This patient's inhaled fluticasone propionate is also a steroid medication; it is given to reduce inflammation in the airways. (The salmeterol is a long-acting beta-agonist.) This patient has no current respiratory signs or symptoms, so we can assume her asthma is currently controlled. Typically, the inhaled and intranasal steroids are considered low enough strength that they will not lead to corticosteroid excess and Cushing syndrome. The systemic steroid, prednisone, is much more potent, so it is the likely culprit for this patient's symptoms. It would be inappropriate to discontinue her inhaled fluticasone propionate/salmeterol, unless an alternate asthma treatment is started; changing the inhaled asthma medication would not alter her Cushing syndrome symptoms. This patient's blood pressure is elevated, but not at urgently high levels. Cushing syndrome is associated with hypertension. It would be wise to discontinue the prednisone and monitor this patient's blood pressure. With resolution of her Cushing syndrome (and anticipated weight loss), the elevated blood pressure may normalize. If her blood pressure remains elevated, it would not be advisable to initiate a beta-blocker (e.g., metoprolol) due to the common side effect of fatigue; this patient already complains of fatigue. This patient's current condition cannot be attributed to normal menopausal changes. Normal post-menopausal changes include vasomotor symptoms (such as hot flashes and night sweats) and vulvovaginal changes (such as atrophy, vaginal dryness and dyspareunia). Some women may experience some mild weight gain and irritability, but this patient's other symptoms are not related to drops in sex hormones. It may be acceptable to initiate combination hormone therapy (e.g., estradiol/norethindrone acetate) for women with acute menopausal symptoms, but it would not be appropriate in this patient's case. This patient's concerns should not be attributed to a weight-gain only issue. Weight gain is a common side effect from exogenous corticosteroids and should reverse with discontinuation of the medication. A dietician referral could be offered in addition to treatment of this patient's Cushing syndrome. It would be inappropriate to initiate a stimulant for weight loss (e.g., phentermine hydrochloride), especially with elevated blood pressure/hypertension.

An 18-year-old woman presents for advice regarding emergency contraception. She had sexual intercourse the day prior to presentation; she did not use contraception, and she wants to know if it is possible that she is pregnant. Her last menstrual period (LMP) began 5 days prior to presentation. She usually gets her menses once in 28 days and the period usually lasts for 5 days. The physical exam is normal. Question What hormone is responsible for the stimulation of the ovaries at this point of the menstrual period? Answer Choices 1 Estrogen 2 Follicle-stimulating hormone (FSH) 3 Inhibins 4 Luteinizing hormone (LH) 5 Progesterone

FSH Explanation The 5th day of the menstrual period corresponds to the follicular phase of the cycle, which starts from the 1st day of bleeding to the day of the LH peak. In this phase, the FSH, which is produced by the pituitary, stimulates the development of follicles in the ovaries; only 1 follicle is dominant at the end. During the 2nd part of this phase, the granulosa cells of the follicle begin to produce estrogen. Estrogen is produced by the granulosa cells of the follicle; they are very important in the follicular phase. Their level is very low at the beginning of this phase and reaches its maximal point at the middle of the cycle, just before the the LH peak. The cells are the feedback for the LH production, and in part, for the FSH production. Inhibins are also hormones produced by granulosa cells of the follicles; they are part of the negative feedback for the FSH. Inhibin B levels rise during the luteal-follicular transition, are highest during the mid follicular phase, go up again during the LH peak, and finally decrease in the late follicular phase. Inhibins A levels decrease during the late luteal phase. Ovulation occurs approximately at the middle of the menstrual cycle (day 14). At the end of the follicular phase a rise in the level of estrogens takes place, preceding the LH peak. Ovulation will occur an average of 30 hours after this peak. The 2nd 1/2 of the menstrual cycle corresponds to the luteal phase of the menstrual cycle which starts from the day of ovulation to the 1st day of menstruation. In this phase, the ovarian corpus luteum begins to produce progesterone. If there is fertilization of the ovum, the production of progesterone continues for 4 to 5 weeks until the placenta can produce a sufficient quantity of the hormone to maintain the pregnancy. If there is no fertilization, progesterone levels decrease and menstruation occurs.

A 29-year-old woman, G0P0, and her husband present to your office after 1 year of infertility. Their histories elicit irregular menses in the woman and that the man is an avid cyclist. Apart from semen analysis in the man, what laboratory and/or diagnostic studies should be pursued in the woman to further evaluate this couple's infertility? Answer Choices 1 Fasting prolactin, TSH 2 TSH, LH, T3, T4 3 Fasting prolactin, FSH 4 FSH, T3, T4 5 Fasting prolactin, TSH, FSH, LH, hysterosalpingogram

Fasting prolactin, TSH, FSH, LH, hysterosalpingogram Explanation Fasting prolactin, TSH, FSH, and LH should all be obtained in the female patient as her irregular menses more or less indicate a high probability of oligomenorrhea. It is also important to evaluate by hysterosalpingogram to look for evidence of uterine and/or tubal defects. It is also important to evaluate the male patient's semen to investigate male infertility related to oligospermia or hypomotility (1).

A 36-year-old woman has a past medical history of diabetes mellitus; she presents with weight gain and skin changes. Her review of systems is positive for menstrual irregularities with extended periods of amenorrhea, infertility, depression, cognitive dysfunction, and emotional lability. Lately, her fasting glucose levels have been above normal. Her physical exam notes increased adipose tissue in the face, upper back, and above the clavicles. Her skin reveals ecchymosis, telangiectasias, and purpura along her back and lower extremities; there is also facial acne and cutaneous atrophy. Her abdominal exam reveals the following image. Question What manifestation is expected in this patient? Answer Choices 1 A waist-to-hip ratio of less than 0.8 2 Increased proximal muscle strength 3 Hyperreflexia 4 Hypertension 5 Increased libido

HTN This patient is demonstrating signs and symptoms consistent with Cushing's syndrome. Hypertension (and possibly edema) may be present due to cortisol activation of the mineralocorticoid receptor, leading to sodium and water retention. Patients may have increased adipose tissue in the face (moon facies), upper back at the base of neck (buffalo hump), and above the clavicles (supraclavicular fat pads); there may be central obesity with increased adipose tissue in the mediastinum and peritoneum as well as an increased waist-to-hip ratio greater than 1 in men and 0.8 in women. Upon CT scan of the abdomen, increased visceral fat is evident. Proximal muscle weakness may be evident. Because of progressive proximal muscle weakness, patients may have difficulty climbing stairs, getting out of a low chair, and raising their arms. Signs of hypothyroidism, such as slow reflex relaxation, may occur from an anterior pituitary tumor whose size interferes with proper thyroid-releasing hormone (TRH) and thyroid-stimulating hormone (TSH) function. Similarly, other pituitary function may be interrupted. Menstrual irregularities, amenorrhea, infertility, and decreased libido may occur in women due to inhibition of pulsatile secretion of luteinizing hormone (LH) and follicle-stimulating hormone (FSH), which is likely due to interruption of luteinizing hormone-releasing hormone (LHRH) pulse generation.

A 53 year-old man with a past medical history of coronary artery disease, hypertension, diabetes, and allergic rhinitis is admitted to the hospital secondary to a non-ST elevation MI. On his second hospital day he develops severe thrombocytopenia with a platelet count of 17,000. Which of the following medications most likely caused his thrombocytopenia? A Atenolol B Erythromycin C Heparin D Loratadine E Metformin

Heparin C Heparin (C) directly causes the production of heparin/PF4 antibodies that affect platelets, which leads to decreased platelet numbers, but an increased thrombosis risk.

A 34-year-old male presents to the primary care office with a complaint of heartburn that has been present for three months. He has symptoms two to three times a week, which occurs about 30 minutes after eating. He has tried over-the-counter antacids and they were helping to relieve his symptoms for a few months, but they are not working well now. He denies dysphagia, odynophagia, or weight loss. You decide to treat him with a proton pump inhibitor at this visit, and he achieves good symptomatic relief with this therapy. What length of therapy is appropriate in this patient? A Two to four weeks B Eight to twelve weeks C Four to six months D One year E Continue indefinitely

If a patient achieves good symptomatic relief with a course of an empiric, once-daily proton pump inhibitor, therapy may be discontinued after eight to twelve weeks.

A 9-year-old boy presents with headache and dark cola colored urine that appeared 2 days after a respiratory tract infection, for which he was given amoxicillin. He has microscopic hematuria and proteinuria. The rest of his past medical history and family history are non-contributing. Physical examination reveals elevated blood pressure (120/80), and the rest of the examination is normal. Urinalysis shows hematuria (the presence of erythrocytes and erythrocytes casts) and protein loss of 2.8 grams. Question What is the most probable diagnosis? Answer Choices 1 IgA nephropathy 2 Alport Syndrome 3 Amoxicillin side effect 4 Acute poststreptococcal glomerulonephritis 5 Hemolytic-uremic syndrome (HUS)

IgA nephropathy Both gross and microscopic hematuria a couple of days after nonspecific upper respiratory tract infection and hypertension in male patients are highly suggestive on IgA nephropathy (Bergers disease). IgA nephropathy is the most common chronic glomerular disease worldwide. It may also be associated with gastroenteritis, acute or chronic renal failure, or may be asymptomatic when erythrocytes (RBCs), RBC casts, and proteinuria are discovered on urinalysis. Some patients also have hypertension. Pathophysiological mechanisms are subendothelial deposits of amorphous material that lead to vascular occlusions, mechanical RBC, and platelet damage, resulting in prothrombotic state. Alport syndrome is hereditary X-linked dominant hereditary nephritis that will also present with hematuria (asymptomatic or gross) 1-2 days after upper respiratory infection. This progressive hereditary nephritis will, however, be accompanied with bilateral sensorineural deafness and visual problems (patognomonical extrusion of central part of lenses into anterior ocular chamber). Amoxicillin side effects are not probable. Amoxicillin side effects include nausea, vomiting, rashes, antibiotic-associated colitis, and diarrhea, in addition to more rare side effects such as mental changes, lightheadedness, insomnia, confusion, anxiety, sensitivity to lights and sounds, and unclear thinking. Even allergy to amoxicillin presents with a change in mental state initially, followed by itching skin rash, fever, nausea, and vomiting any time during the treatment up to a week after treatment has stopped. Acute overdose of amoxicillin may manifest with renal dysfunction, lethargy, and vomiting, but this usually happens in very young children. Acute poststreptococcal glomerulonephritis can present with the same clinical picture: sudden hematuria, edema, and hypertension, usually together with non-specific constitutional symptoms. However, there is always a latent period between the streptococcal infection and the onset of signs and symptoms of acute glomerulonephritis. Latent period is 1-2 weeks after a throat infection and 3-6 weeks after a skin infection. Hemolytic-uremic syndrome (HUS) is acute renal failure associated with non-immune (Coombs-negative) microangiopathic hemolytic anemia and thrombocytopenia. It is the most common cause of acute renal failure in children (though it may occur in adults as well). In HUS, there is usually a prodromal gastroenteritis, fever, or bloody diarrhea for 2-7 days before the onset of renal failure, sometimes with central nervous system signs (irritability, lethargy, even seizures). Acute renal failure with anuria follows. Physical findings may reveal hypertension, edema, fluid overload, and severe pallor.

A patient is admitted to the hospital for an acute adrenal crisis. He has a history of chronic adrenal insufficiency and was admitted with severe weakness, nausea and vomiting while fighting a pulmonary infection. Which of the following suggested plans might this patient have forgotten or been unable to do? A Restrict fluid intake during times of metabolic stress B Increase the daily dose of hydrocortisone during times of metabolic stress C Hold the daily dose of hydrocortisone during times of metabolic stress D Add levothyroxine to the daily dose of hydrocortisone during times of metabolic stress E Increase the daily ingestion of proteins during times of metabolic stress

Increase the daily dose of hydrocortisone during times of metabolic stress The correct choice is B, increase the daily dose of hydrocortisone during times of metabolic stress. The cortisol dose should be increased to between 60 and 80 mg/day, to mimic the normal physiologic response of the body. Increased mineralocorticoid therapy is generally not required. Choice A, restrict fluid, would aggravate the potential for the development of dehydration in this patient. Choice C would lead to further cortisol depletion. Choices D and E would not benefit this patient.

A 46-year-old woman presents for a follow-up evaluation for her total thyroidectomy; it was performed 1 year ago for Stage II differentiated mixed papillary and follicular carcinoma. Laboratory tests were performed. What change in levels may suggest the recurrence of thyroid malignancy in this patient? Answer Choices 1 Increasing serum thyroglobulin levels 2 Decreasing serum thyroglobulin levels 3 Increasing T4 levels 4 Increasing TSH levels 5 Decreasing TSH levels

Increasing serum thyroglobulin levels Explanation The correct answer is increasing thyroglobulin levels. Detectable thyroglobulin levels may be found in patients with incomplete thyroidectomies and those thyroid remnant ablations with radioactive iodine. While these levels do not necessarily indicate metastatic or residual thyroid cancer, increasing levels over time in the absence of serum anti-thyroglobulin antibodies have been associated with recurrent thyroid malignancy. The presence of anti-thyroglobulin antibodies may invalidate the test results by interfering with the assay. Baseline and stimulated thyroglobulin levels that are greater than or equal to 2 ng/mL need follow up with neck ultrasound and possibly other scanning. Rising thyroglobulin levels are at high risk for occult thyroid cancer metastases. Other scanning may be used to detect this possibility. The other responses: decreasing thyroglobulin levels, increasing, or decreasing T4 and TSH levels are not necessarily associated with recurrent thyroid malignancy.

A 25-year-old G1P1 Caucasian woman who recently delivered a healthy male infant presents with breast pain and tenderness on the right; associated symptoms include malaise and fever. The patient has been compliant with all of her prenatal and post-natal obstetrical appointments. No medical complications were incurred during or after the pregnancy. The patient is currently breast feeding. Physical inspection of the right breast shows focal breast erythema and tenderness. The patient has no known co-morbidities, and she denies significant family medical problems. The patient denies palpitations, shortness of breath, or syncope. Pulse is 78 bpm, and respirations are 16 per minute; blood pressure is 102/68 mm Hg, and temperature is 101.2°F. What is the most likely diagnosis? Answer Choices 1 Inflammatory breast carcinoma 2 Congestive heart failure 3 Venous thromboplebitis 4 Infection with Staphylococcus aureus 5 Simple breast engorgement

Infection with Staphylococcus aureus Mastitis is a condition that reflects cellulitis of the periglandular tissue in breast-feeding mothers, typically due to infection with Staphylococcus aureus. Clinical symptoms of mastitis include breast pain, skin redness, fever, and flu-like symptoms. Physical findings include focal breast erythema, swelling, and tenderness, while fluctuance suggests a breast abscess. Diagnosis rests on breast milk cultures and complete blood count analysis. If fluctuance is present on physical examination, then a breast ultrasound can be used to exclude abscess formation. Treatment for mastitis involves continued breast feeding and antibiotics such as dicloxacillin or erythromycin. The treatment of a breast abscess is incision and drainage. Inflammatory breast carcinoma can clinically resemble unilateral mastitis, but it would not resolve with antibiotic therapy. Additionally, inflammatory breast carcinoma would be extremely unusual in a 25-year-old woman. If the patient's breast erythema, swelling, and tenderness continues despite antibiotic therapy, then a mammogram with an associated punch skin biopsy should be considered to rule out inflammatory breast carcinoma. Congestive heart failure can lead to bilateral breast swelling with increased breast interstitial markings and trabeculations on mammography, but it would be unusual in this patient without a past medical history of congestive heart failure or complaints of palpitations, shortness of breath, or syncope. Thrombophlebitis of the superficial veins of the breast and the anterior chest wall (Mondor Disease), a rare condition, presents as a tender, cordlike structure that may be demonstrated best by tensing the skin and elevating the arm. The clinical symptoms described in this patient are more consistent with mastitis than Mondor's disease. Simple breast engorgement can present in patients as a swollen, firm, and diffusely tender breast, but usually produces a low-grade fever rather than a high fever, as in the patient above.

A 59-year-old woman presents with a firm, non-tender mass with a "rock-hard" consistency in her right breast when palpated by her gynecologist. On further examination, the right breast exhibits redness and dimpling of the skin and her right nipple appears slightly retracted. In addition, she reports a single episode of a bloody nipple discharge. A diagnostic mammogram reveals dense fibroglandular tissue with possible microcalcifications in the area of the palpable lump. A sonographic exam reveals an immobile 10 mm hypoechoic mass. Question What is the most likely diagnosis? Answer Choices 1 Benign cyst 2 Fibroadenoma 3 Galactorrhea 4 Gynomastia 5 Infiltrating duct carcinoma

Infiltrating duct carcinoma: Explanation Infiltrating (invasive) duct carcinoma is the most common type of breast cancer. It typically does not respect anatomic borders and invades the normal breast at random. Invasion of the fat tissue can be seen on gross examination as stellate white tissue extending into the yellow fat tissue. Invasive ductal carcinomas cause a dense fibroblastic response in the host tissue that gives these tumors a rock-hard consistency. The desmoplastic reaction caused by infiltrating ductal carcinoma resembles an irregular scar which deforms and causes a dimpling of the skin and retraction of the nipple (peau d'orange appearance). This patient has many classic signs of infiltrating ductal carcinoma:firm breast mass, dimpling of the skin and retraction of the nipple (peau d'orange) and a bloody discharge. A biopsy should be performed to confirm the imaging diagnosis. Benign cyst is incorrect. Benign cystic changes are typical of non-proliferative fibrocystic changes. These changes are found in some breasts of women older than 45 years of age and are probably related to normal aging of the female breast. The breast mass described in this case does not have the characteristics of a benign cyst. Fibroadenoma is incorrect. Fibroadenoma is the most common benign breast tumor. It is composed of elongated ducts surrounded by loose fibrous stroma. These well circumscribed tumors typically present as easily movable spherical masses in breasts of 20-35 year old young women which does not the describe the patient or tumor in this case. Galactorrhea is incorrect. Galactorrhea is a milky breast discharge in non-nursing women due to a prolactinoma, a pituitary adenoma, that causes an abnormal proliferation of lactotrophs and the secretion of excess amounts of prolactin. Prolactin stimulates milk synthesis in lactating women. Galactorrhea does not occur in men because men do not have enough breast tissue. Gynecomastia is incorrect. Gynecomastia is the excessive growth of male mammary glands. Physiologic causes include: in the newborn, due to elevated maternal estrogens during pregnancy; during puberty, due to an increased estrogen to androgen ratio; and in old age, due to combined effect of decreasing testosterone and increased estrogen due to peripheral aromatization of androgen to estrogens in adipose tissue.

A 40-year-old woman presents with a 6-month history of a painless swelling on her neck. On examination, you find a nontender anterior neck swelling that rises with deglutition. Laboratory evaluation reveals normal thyroid function tests. Question What is the most likely diagnosis? Answer Choices 1 Iodine deficiency 2 Zinc deficiency 3 Copper deficiency 4 Folic acid deficiency 5 Cobalamin deficiency

Iodine deficiency Explanation In cases of iodine deficiency, patients can present with an anterior neck swelling that rises with deglutition. This is usually a colloid goiter, as the thyroid hypertrophies as it tries to concentrate iodide in itself. Though most of these patients are euthyroid, some may develop hypothyroidism. Cretinism can develop in infants as a result of iodine deficiency; impaired brain development and fetal growth can also develop. Causes include inadequate dietary intake. Dietary sources of iodine include iodized table salt, seafood, eggs, and dairy products. In cases of zinc deficiency, patients can present with hypogeusia or decreased taste sensation, anorexia, delayed sexual maturation, night blindness, and hair loss. On examination, they have alopecia, growth retardation, delayed sexual maturation, and hypogonadism. Laboratory investigations may reveal hypospermia. Causes include malabsorption states, alcoholism, and prolonged parenteral nutrition. Dietary sources of zinc include beef, liver, eggs, and oysters. Menkes syndrome is an inherited copper deficiency caused by mutations in an X-linked gene. It occurs in male infants and is characterized by intellectual disabilites, kinky hair, hypopigmentation, and vascular aneurysms. Laboratory investigations reveal hypocupremia and decreased circulating ceruloplasmin. Other causes of copper deficiency include severe malabsorption, infants with persistent diarrhea fed on milk diets, copper-free total parenteral nutrition, and excess intake of a zinc salt dietary supplements, which can interfere with copper absorption. Dietary sources of copper include organ meats, oysters, nuts, dried legumes, and whole grain cereals. Patients with folic acid deficiency present with symptoms of anemia such as fatigue, weakness, and syncope. On examination, they are pale. Laboratory investigations reveal macrocytic red blood cells and hemoglobin less than 12 g/dl. Causes include malabsorption and inadequate intake by chronic alcoholics and malnourished individuals, as well as increased demand in pregnancy and chronic hemolytic anemias. Dietary sources of folic acid include green leafy vegetables, liver, and yeast. Patients with a vitamin B12 (cobalamin deficiency) can present with symptoms and signs of anemia, such as fatigue, light-headedness, syncope, and pallor; there are also neurological signs and symptoms, such as ataxia, paraesthesias, and diminished proprioceptive and vibratory sensations in the lower limbs. Laboratory investigations reveal macrocytic red blood cells. Causes include pernicious anemia, blind loop syndrome, fish tapeworm infestation, and vegetarian diets. Dietary sources of vitamin B12 include liver, beef, eggs, and milk.

A 62-year-old man passed a foul-smelling, tarry stool 1 week ago. 3 weeks ago, he began taking diclofenac because of worsening osteoarthritis. His past medical history includes primary hypertension, coronary heart disease, and 2 episodes of venous thrombosis in the calf. Besides diclofenac, his medications include warfarin, enalapril, low-dose aspirin, propranolol, and sublingual nitroglycerin. A screening colonoscopy performed 2 weeks ago was entirely normal. Vital signs are stable, and the physical examination is unremarkable. Upper endoscopy shows only pangastritis and erosions. What is the most effective way of reducing his risk of rebleeding? Answer Choices 1 Misoprostol 2 Omeprazole 3 Famotidine 4 Switch to valdecoxib 5 Take the diclofenac tablets with at least 250 ml of water

Misoprostol The correct response is misoprostol. Besides NSAID use, this patient has several risk factors for gastric bleeding (a previous episode, advanced age, use of anticoagulants and low-dose aspirin). Therefore, prophylaxis against bleeding must be seriously considered. Prostaglandin analogues are the most effective drugs for reducing the risk of rebleeding. One study described reductions of up to fivefold in the incidence of gastric ulcers on endoscopy. However, this drug requires 2 to 4 daily doses and has several side effects. The most common are diarrhea (up to 30% of patients) and abdominal discomfort. It is also a potent abortifacient. Starting the drug at a low dose and then progressively raising it might reduce the incidence of these side effects and improve tolerance. Proton-pump inhibitors such as omeprazole also reduce the incidence of gastrointestinal complications in chronic users of NSAIDs. They are slightly less effective than prostaglandin analogues, but their once-daily dosage is more convenient and the incidence of side effects is lower. The use of low-dose aspirin abrogates the protective effects of COX-2-selective anti-inflammatory drugs (i.e., celecoxib, rofecoxib, or valdecoxib). Therefore, switching to COX-2 inhibitors would not be advantageous for this patient. H2-blockers such as famotidine have only marginal efficacy in reducing the incidence of NSAID-associated gastric ulcers and should not be prescribed in this case. The medications that should be taken with at least 250 ml of water are the bisphosphonates. This precaution reduces the risk of pill esophagitis. This drug should also be taken on an empty stomach and with the patient in an upright position. If these precautions are not followed, bisphosphonates can cause severe esophagitis, which can even lead to strictures or bleeding.

"Emotional Freezing," according to the DSM-IV, is characterized by solitary activities, constricted affect, and deficits in communication; it can easily be erroneously diagnosed as what personality disorder? Answer Choices 1 Paranoid 2 Schizoid 3 Schizotypical 4 Delusional 5 Antisocial

Schizoid Different cultural backgrounds, defensive behaviors, and interpersonal styles are often erroneously misdiagnosed and labeled schizoid. The DSM-IV criteria for Schizoid Personality Disorder is marked by a failure to enjoy or desire close relationships, a preference for solitary activities, a lack of close friends, very little emotion, and generally flat affect.

A 28-year-old woman presents with nervousness and palpitations associated with heat intolerance. On examination, there is no evidence of thyromegaly, but there is a palpable nodule that is "hot" on a thyroid scan. The TSH was low and T3 and T4 were both elevated. Which of the following is the recommended treatment for this patient? A Propylthiouracil (PTU) B Thyroid lobectomy C Total thyroidectomy D Radioiodine ablation

Thyroid Lobectomy In Graves' disease, the thyroid is diffusely enlarged in contrast to a toxic adenoma in which the thyroid is normal sized but with a palpable nodule. Surgery is the treatment of choice for a toxic adenoma. Surgical treatment of a toxic adenoma is a thyroid lobectomy and isthmusectomy. A subtotal or total thyroidectomy is indicated for toxic multinodular goiters or Plummer disease. Thionamides and radioiodine ablation are not effective therapies for toxic adenomas.

A 56-year-old patient presents to the office with a complaint of a discoloration of her fingertips over the past few months. The symptoms have been recurrent and worsening. Upon clinical exam, the tips of all fingers present with splinter hemorrhages. What is the most likely diagnosis for this finding? A Vasculitis B Wilson's disease C Argyria D CHF E Nutritional disorder

A

You have completed comprehensive testing on an 80-year-old woman, and the diagnosis is dementia of the Alzheimer's type. You believe she is in the very early stages of the disease and want to try a medication to possibly slow disease progression. What drug may be an effective treatment? Answer Choices 1 Cognex (Tacrine HCL) 2 Vitamin E 3 Risperdal (Risperidone) 4 Donepezil (Aricept) 5 Haldol (Haloperidol)

Donepezil (Aricept) Alzheimer's dementia accounts for over 60% of dementia in older adults. It develops with multiple cognitive deficits, including memory impairment and 1 or more of the following: aphasia (language disturbance), apraxia (difficulty in carrying out motor activities), agnosia (the inability to recognize familiar objects or people), and/or disturbance in organizing and planning. These symptoms usually develop over a gradual period of time and represent a distinct decline from previous functioning. Another mental illness or serious medical problem does not account for the symptoms. To make an accurate differential diagnosis, the physician must first complete a history and physical exam in order to rule out physical causes of the dementia. A complete mental status exam must then be done, along with initial lab testing. All other additional testing, such as neurological or psychological testing, should be done at a later stage after all other physical causes have been ruled out. Also, it is very important to refer the patient and family to a medical social worker at some point in order to assist the family in community resources and emotional support, especially if the diagnosis becomes dementia of the Alzheimer's type. There are no current cures for Alzheimer's disease and no one really knows what causes it. There are 2 major drugs being used to treat some of the symptoms: tacrine (Cognex) and donepezil (Aricept). Donepezil has become the treatment of choice because it has once-a-day dosing; there is also no need for close monitoring, and it causes fewer side effects. Tacrine has multiple side effects and requires close liver monitoring. Vitamin E is being used to reduce the effects of drugs, but it is not a definitive treatment of choice. Risperidone (Risperdal) and haloperidol (Haldol) are being used as later choice drugs to treat later-stage patients who have developed psychotic or agitated reactions. Both have side effects, have to be more closely monitored, and are usually prescribed by a psychiatrist/liaison. Normal pressure hydrocephalus is characterized by gradual onset of dementia, gait disturbance, and usually urinary incontinence. Pick's disease is very rare and presents itself with changes in personality, deterioration in social skills, emotional flatness, and significant language problems. Vascular dementia usually has a more sudden onset, and there is evidence of cerebrovascular disease with focal neurological signs and symptoms and usually a history of a previous stroke. Pseudo dementia is the term used to describe false dementia or major depression in the elderly; it usually consists of depressed mood, crying episodes, hopelessness, helplessness, and fatigue. Patients may also present with insomnia or hypersomnia.

A 67 year-old with chronic kidney disease presents to the clinic with a hemoglobin of 8.6 mg/dl and hematocrit of 24%. Which of the following agents, if administered, can further increase this patient's risk of thrombotic vascular events? A Cyanocobalamin B Erythropoietin C Ferrous sulfate D Folic acid E Niacin

Erythropoietin Erythropoietin (B) increases red blood cell mass and toxicity may lead to increased blood viscosity and potential thrombotic events. Cyanocobalamin (A), ferrous sulfate (C), folic acid (D), and niacin (E) don't increase thrombotic risk.

A 24-year-old G1P1 presents to the office complaining of a red, tender area of her right breast. She is four weeks postpartum and is nursing her infant with good success. She complains of no other symptoms. On physical exam, her vitals are normal. Lungs CTA, CV RRR, left breast is normal, right breast has a 3-cm area that is warm with erythema, and no mass or area of fluctuance is noted. She has a MRSA mastitis. How did she most likely contract the infection? A Community acquired B Hospital acquired C From her infant D Self inoculated

From her infant Infants usually contract MRSA due to poor hand washing technique from the hospital staff, but it is then spread to the mother via the infant.

A 22-year-old engineering student presents after a major motor seizure. The patient was well until 3 days prior to presentation, when he developed generalized headache and a fever of 101.2 F (by mouth), for which he took aspirin. He later complained that he could not concentrate on his studies, and he added that all his teachers wanted to flunk him anyway. On more than 1 occasion, he accused his roommate of stealing his food from their shared refrigerator. He complained that he could not smell his food. Several times, he was noted to be having staring spells and lip smacking; during these episodes, he did not respond to his roommate. The spells lasted less than 1 minute. On the day of presentation, the roommate returned to the dorm to find the patient standing naked in the hallway outside their room; he was screaming "We're going to settle this now!" He then fell to the floor and sustained a major motor seizure. The patient's mother denies a history of major illness, but reveals he had measles and chicken pox as a child. The roommate states that the patient drinks a lot of beer at weekend parties and smokes marijuana occasionally, but he denies knowledge of other drug use. There is no history of seizures. Exam shows a well-developed, lean, muscular young man. He is lying on a gurney and fighting his restraints. He is alert but mute, and he is unresponsive to the spoken word. Temperature is 101.2 F (taken rectally). Heart rate is 110 and regular; respirations are 22 and unlabored. Blood pressure is 140/85 mm Hg. Neurological exam is limited by patient agitation and lack of cooperation, but it reveals normal pupils and eye movements, full symmetric facial movements, a mild right hemiparesis, and right extensor plantar response. Labs include complete blood count with a white blood cell count of 8,500; there is a normal differential. The chemistry profile, chest X-ray, and electrocardiogram are normal. Stat computed tomography of the head with contrast reveals left inferomedial temporal and frontal confluent areas of hypointensity with surrounding edema as well as scattered areas of hemorrhage in the same regions. These areas enhance densely. Cerebrospinal fluid exam reveals moderately elevated pressure, mildly elevated protein, and 267 white blood cells (80% lymphs, 20% polymorphs). Glucose is normal. Question What is the most likely cause of this patient's illness? Answer Choices 1 Herpes simplex encephalitis 2 Acute hemorrhagic leukoencephalitis 3 Subdural empyema 4 Cerebral abscess 5 Cerebral venous thrombosis 6 Septic embolism 7 Ruptured saccular aneurysm

HSv encephalitis This is a rather classic presentation for focal encephalitic form of herpes simplex encephalitis. A prodrome of fever and headache is common. Anosmia, apparent temporal lobe seizures, and personality changes point to the inferomedial temporal and frontal regions. A convulsion is frequently the event that precipitates admission. The computed tomogram is classic. The cerebrospinal fluid is typical, but it can be normal early in the clinical course. This is the most common and serious of the acute encephalitides. This is a very grave illness, and it must be recognized and treated with antiviral agents (acyclovir) as soon as possible. Acute hemorrhagic leukoencephalitis is seen mostly in children, but it can also be seen in young adults. However, it is almost always preceded by symptoms of respiratory infection of 1 - 14 days duration. These patients may also develop headache and confusion, but neck pain and signs of meningeal irritation are more common and pronounced. Anosmia and temporal lobe seizures would be less common or pronounced. In this condition, white blood cell counts as high as 20,000 - 30.000 can be an important point of differential. Cerebrospinal fluid may be identical, but pleocytosis of up to 3,000 can help to differentiate it from herpes simplex encephalitis. The most helpful information at this point in differentiating this condition is the computed tomogram and magnetic resonance imaging scan, which would be expected to show diffuse asymmetric, large, and confluent edematous lesions in the white matter, with multiple small hemorrhages in both white and gray matter. Subdural empyema is a suppurative condition between the inner surface of the dura and the outer surface of the arachnoid. Since it occurs in a preformed space, it is properly termed an empyema, although it is frequently and erroneously called subdural abscess. It most commonly follows a worsening of chronic sinusitis, in which cases it most commonly involves adolescent and young men. There is usually a history of worsening of brow or mastoid pain and nose or outer ear canal purulent discharge. Fever, malaise, and headache appear. The headache pain worsens, and then becomes generalized; vomiting is frequent. Focal neurological signs may appear, and stupor and coma may ensue. Fever, leukocytosis, and stiff neck are almost universal. Cerebrospinal fluid is rather similar to that seen in herpes simplex encephalopathy. However, computed tomography scan of the head shows ear or sinus lesions and possible bone erosion. Enhanced meninges around the pus may be seen. Magnetic resonance imaging is more likely to visualize the empyema. About 40% of cerebral abscesses are related to diseases of the paranasal sinuses, middle ear, and mastoid air cells. Other conditions include bronchiectasis, lung abscess, acute bacterial endocarditis, congenital heart disease, pulmonary arterial venous malformation (AVM), abscessed teeth, osteomyelitis, or other sites of bacterial infection. The clinical course may be as seen in this patient, but onset and progression are much more variable depending on where the abscess is and the stage of development it has reached. The main concern here would be in differentiating herpes simplex encephalopathy from temporal or inferior frontal abscess. Spinal fluid is usually quite similar, and while finding bacteria might resolve the issue, this is rarely the case. Here, contrast studies are most helpful. Computed tomography would show a central hypodensity, a surrounding ring of enhancement, and hypodensity beyond the enhancing ring. MRI T1-weighted images show capsular enhancement with central hypodensity. T2-weighted images show capsular hypodensity, and surrounding edema is obvious. A septic embolism usually produces sudden neurological deficit, with the clinical picture depending on the site of ischemia. It might be mild and brief, its history being obtained only later when the patient presents with a clinical picture of expanding intracranial lesion (abscess), or it may present as a major acute typical appearing ischemic cerebrovascular accident. If it formed a temporal or frontal abscess, magnetic resonance imaging and computed tomography would probably be definitive. They may also form mycotic aneurysms of the more distal cerebral arteries; in turn, they may rupture, producing a picture of acute subarachnoid hemorrhage with or without intracerebral hemorrhage. Elevated sedimentation rate, leukocytosis, fever, malaise, fatigue, and known heart disease should prompt a thorough search for a septic embolic source. Ruptured saccular aneurysms are not usually a problem. There should be no prodrome (preceding headache, fever, stiff neck, malaise, abnormal behavior, etc.). The patient would be expected to have sudden severe headache, nausea, vomiting, and possibly loss of consciousness. There would be no fever or leukocytosis (except for a possible stress pattern). Focal neurological deficits are the exception, at least initially. Optic disc hemorrhages may be seen. Computed tomography generally shows blood (90%) in the subarachnoid spaces, which can be dramatic, or show only a subtle shadow along the tentorium or in the Sylvian or adjacent fissures. Rarely, the bleed may extend into brain tissue, producing an intracerebral hematoma. In the 5 - 10% of cases where no blood is seen on computed tomography or magnetic resonance imaging, a lumbar puncture generally reveals red blood cells in the thousands to a million, frequently with xanthochromia after several hours.

A 55-year-old woman with a 15-year history of type II diabetes presents for follow-up of her diabetes. Her spot albumin/creatinine ratio was 100 mg/g 4 months ago and was confirmed at 100 mg/g yesterday. Her urinary analysis shows no cells, casts, or blood. Her creatinine is 0.7mg/dl, and her estimated glomerular filtration rate is 95 ml/min/1.73m2. Question What medication(s) should you prescribe to help prevent her progression from micro to macroalbuminuria and to help prevent progressive decline in glomerular filtration rate? Answer Choices 1 Potassium chloride 2 Calcium carbonate 3 Lisinopril 4 Sodium bicarbonate 5 Calcium citrate

Lisinopril Explanation Lisinopril, an ACE Inhibitor, should help decrease albuminuria, prevent progression of diabetic kidney disease from micro to macroalbuminuria, and prevent a decline in glomerular filtration rate. This class of medications has been studied extensively for these purposes. Angiotensin II receptor blockers (e.g., irbesartan) may also reduce urinary albumin to normal levels. Monotherapy with either of these classes of medications should be attempted first in patients with microalbuminuria. This will test tolerance, effectiveness, and adverse reaction such as hyperkalemia. For patients with greater degrees of albuminuria (e.g., 1 g/day), poor response to monotherapy and blood pressure control, and no hyperkalemia associated with therapy, combination therapy should be considered. Combination therapy with both ACE inhibitors (e.g., lisinopril) and angiotensin II receptor blockers (e.g., irbesartan) is used to treat both diabetic and non-diabetic kidney disease. These medications act on different parts of the renin angiotensin system. In combination, irbesartan could block the effect of angiotensin produced by non-ACE pathways and lisinopril could block the production of angiotensin stimulated by irbesartan in a negative feedback system; however, combination therapy is usually preceded by monotherapy. Although combination therapy is currently being used in both diabetic and non-diabetic kidney disease, this therapy still being researched. It is unknown whether monotherapy alone is sufficient. It does not appear to be sufficient in all patients, particularly those with persistent micro and macroalbuminuria despite monotherapy. Sodium bicarbonate is used to treat the metabolic acidosis that commonly occurs secondary to chronic kidney disease. Diseased nephrons eliminate acids poorly, allowing them to build up in the blood stream. Sodium bicarbonate buffers these acids. Sodium bicarbonate is not known to significantly alter progression of proteinuria or decline in glomerular filtration rate. Potassium chloride is a mineral used in the management of hypokalemia. We have no knowledge of hypokalemia in this patient and no indication to use potassium supplements. Potassium is not known to alter the progression of diabetic nephropathy. In fact, this woman is more likely to be hyperkalemic due to her diabetes, which can cause hyporeninism, hypoaldosteronism, and hyperkalemia. Giving potassium to patients already at risk for hyperkalemia is not advised, as hyperkalemia may provoke cardiac arrhythmias. Calcium carbonate and calcium citrate are used to treat the hyperphosphatemia found commonly in patients with chronic kidney disease. They will not alter the progression of diabetic proteinuria.

Tx of Metabolic Acidosis

Metabolic Acidosis: Low serum bicarb and low pH • Tx is alkali therapy 0.5 meq/kg per day to maintain normal serum bicarb; give them sodium bicarb or sodium citrate o Be aware of volume overload because these contain sodium

A 64-year-old woman presents for evaluation of myxedema. She is experiencing constipation, headaches, weakness, fatigue, lethargy, somnolence, cold intolerance, decreased sweating, paresthesias, arthralgias, and muscle cramps. She has dry, coarse skin. Her face and extremities are swollen, and her voice is coarse. Question What gait abnormality would you most likely expect to see in this patient? Answer Choices 1 Astasia-abasia 2 Ataxic 3 Myopathic 4 Steppage 5 Limping 6 Magnetic

Myopathic Myxedema is the accumulation of hydrophilic mucopolysaccharides in subcutaneous tissues in severe hypothyroidism. Neuromuscular symptoms are present in 30-80% of patients with hypothyroidism, and they usually improve or disappear with correction of the hypothyroidism. The most common complaints are of muscle cramping, proximal symmetrical muscle weakness, muscle stiffness, and exercise intolerance. These manifestations can occur at any time during the presentation of hypothyroidism. Proximal muscle weakness presents as dystrophic or circumduction gait. These patients usually complain of difficulty climbing stairs because they are actually using the handrails or their own hands to pull themselves up with their arms. Thyroid myopathy is also characterized by slowness of both muscle relaxation and muscle contraction. This is due to the decreased amount of myosin ATPase, leading to the slowing of release and reaccumulation of calcium in the endoplasmic reticulum. In thyroid disease, segmental demyelination of peripheral nerves with decreased nerve conduction velocities may also be observed; there may also be a decrease in vibration, joint-position, and touch-pressure sensations. Astasia-abasia is seen most commonly in the very late stages of normotensive hydrocephalus. Patients may demonstrate the inability to walk, inability to stand, sit, rise from a chair, or turn over in bed; this advanced stage is called "hydrocephalic astasia-abasia." Ataxic gait is a wide-based gait with difficulty or inability to tandem walk due to the midline cerebellar diseases. Patients sway from side to side as they walk. Typically, patients who have midline cerebellar degeneration have difficulty with tandem gait, but they have normal coordination in their arms. Unilateral cerebellar lesions often cause incoordination of the arm and leg on the same side as the lesion. A cerebellar abnormality is implicated in ataxia only if weakness, spasticity, and position-sense abnormality are absent. A steppage gait occurs when the patient has either a foot drop from weakness of the foot dorsiflexors or poor position sense in the feet. With a foot drop, the patient flexes the hip so that the foot will clear the floor. Patients with poor position sense have a wide-based gait; their feet slap the floor; and they require visual assistance to walk. Limp is a gait abnormality usually due to pain ("antalgic gait") or skeletal deformities, e.g., hip deformity with or without associated muscular weakness ("Trendelenburg gait"), with the body shifted over the affect hip. In magnetic gait (when feet seem attached to floor as if by a magnet), each step is initiated in a "wresting" motion carrying feet upward and forward. Magnetic gait and gait disturbances are typically the initial and most prominent symptom of the triad of normotensive hydrocephalus (dementia, incontinentia, and gait disturbance) and may be progressive due to expansion of the ventricular system with a progressive traction of motor fibers in pyramidal tract. The gait disturbance in normotensive hydrocephalus can be classified as mild (unsteadiness, impaired balance, especially when encountering stairs and curbs, usually described as cautious gait), marked (evident difficulty walking or considerable unstable gait when the patient needs mobility aids to provide some stability), or severe (unaided gait not possible, as there is a constant tendency to fall backwards; gait is broad based in order to increase the base of support and provide some stability; and steps height is decreased, as well as foot-floor clearance and the speed of walk). This style may remind one of Parkinsonian gait, except there is no rigidity or tremor.

You are evaluating a 69-year-old female who complains of an intermittent sensation of hot flashes, flushing of her face/chest, and pruritus after starting a new medication for her cholesterol. Which of the following medications is the most likely cause of her symptoms? A niacin B lovastatin C gemfibrozil D ezetimibe E fenofibrate

Niacin The correct answer is (A). Niacin has a characteristic side effect of hot flashes, flushing, and pruritus. These symptoms can be reduced by addition of ASA or a nonsteroidal anti-inflammatory drug (NSAID) if there are no contraindications. The other choices are unlikely to cause this combination of symptoms.

A 37-year-old man presents with diarrhea, nausea, vomiting, and a rash. He also complains of memory problems and depression. He states that he and his family live on a farm that produces corn. Due to recent financial problems, his diet has become more limited. He eats mainly corn products and only small amounts of fish and vegetables. He takes no vitamin supplementation, and he states that his symptoms began about 1 month ago. He began having trouble with his memory about 1 week ago. He has no significant past medical history, but he does admit to daily alcohol consumption. His physical examination is significant for stomatitis, glossitis, and a rash over his arms and legs. His mini mental status exam score is 24. Question What deficiency does this patient most likely have? Answer Choices 1 Niacin 2 Zinc 3 Folate 4 Vitamin B12 5 Vitamin B1

Niacin This patient probably has niacin deficiency. Niacin (nicotinamide, nicotinic acid) deficiency is uncommon in the United States. It is often found in people who live on a diet that consists mainly of corn (maize). This is due to the fact that the niacin in corn cannot be absorbed unless it is chemically treated with alkali first. If a person consumes a diet rich in tryptophan, but low in niacin, they are able to compensate since tryptophan can be converted into niacin. Deficiency may also result from alcoholism, cirrhosis, or diarrhea. Men and women are affected equally. Symptoms of niacin deficiency include nausea, vomiting, diarrhea, rash, glossitis, stomatitis, depression, and psychosis. Niacin deficiency, also known as pellagra, manifests as the '3 Ds': diarrhea, dermatitis, and dementia. Treatment consists of niacin supplementation. The causes of zinc deficiency include malnutrition, chronic debilitating diseases, chronic renal disease, alcoholism, drugs such as penicillamine and diuretic, and genetic disorders, such as sickle cell disease. Clinical manifestations in severe cases include alopecia, diarrhea, weight loss, infections, dermatitis, hypogonadism in men, and intercurrent infections. Supplementation with zinc is the treatment of choice. Folate deficiency causes megaloblastic macrocytic anemia, as folate plays a key role in nucleic acid synthesis. The early manifestation of folate deficiency, especially in its suboptimal state, predisposes to occlusive vascular disease and thrombosis. These manifestations are linked to increased homocysteine levels found in folate deficiency. Neurological disturbances, such as mood disturbance and spinal cord syndromes, are also seen. It is also associated with predisposition to neoplasia and interferes with immunologic status. Folate replacement is the option to prevent and to treat the deficiency. Vitamin B1 (or thiamine deficiency) causes beri beri, occurring mostly in the malnourished and alcoholics. The deficiency manifests with acute heart failure, neurologic deficits, and epilepsy. Empiric use of thiamine and prophylactic use in high-risk population is strongly recommended even before blood reports are obtained, as the treatment is inexpensive and prevents major catastrophes. Vitamin B12 (or cobalamin deficiency) manifests as megaloblastic macrocytic anemia, pancytopenia, and a spectrum of neuropsychiatric disorders such as peripheral neuropathy, parasthesias, and demyelination of corticospinal tract. Nutritional deficiency, alcoholism, and malabsorption syndromes are some causes of B12 deficiency. It is also associated with homocysteinemia and atherosclerosis. Diagnosis is by serum estimation of B12, and oral supplementation is safe and effective. Intramuscular injections may also be used.

CKD stage 1 GFR

Normal GFR of 120 to a GFR of 90

A woman who is pregnant suffered a spontaneous abortion at 12 weeks gestation. She is now a G2P1Ab1 and is Rh negative. When should she receive her next Rhogam (Rho D immune globulin) shot? A Now B In one month C At conception of her next pregnancy D 28 weeks gestation of next pregnancy E After delivery of her next viable infant

Now A Placental implantation occurred and separated with the spontaneous miscarriage. Therefore, there is a slight chance of isoimmunization, so Rhogam should be given now so that the mother does not develop antigens that can cross the placenta during the first half of the next pregnancy.

A phobia is an excessive fear of an object or place that leads to or can be preceded by: A panic attack B depression C hallucinations D delusions E confabulations

Panic Attack A Patients who have a phobia realize it is an irrational fear and try to avoid whatever they have the fear of. In attempts to avoid the "problem," patients can develop anxiety or panic attacks

A 22-year-old woman presents with a 2-month history of weight loss; it is occurring despite the woman having a good appetite. She also reports of having frequent bouts of diarrhea. On detailed questioning, she reveals a feeling of heat intolerance and menstrual irregularity. The right lobe of the thyroid is palpably enlarged, and further investigations confirm the diagnosis of hyperthyroidism. What symptom is characteristic of hyperthyroidism? Answer Choices 1 Fatigue and lethargy 2 Palpitations and tremors 3 Slowed speech and movement 4 Thickening and dryness of skin 5 Impaired memory and sleepiness

Palpitations and tremors Explanation The common symptoms of hyperthyroidism include nervousness, palpitation, tremors, weight loss with preserved appetite, heat intolerance, diarrhea, increased perspiration, and decreased menstrual flow. Common signs include tachycardia, systolic hypertension, hyperactivity, atrial fibrillation, lid lag, eyelid retraction, tremors, and hyperreflexia. The causes of hyperthyroidism include Graves' disease (most common cause), toxic adenoma, toxic multinodular goiter, sub-acute thyroiditis, lymphocytic thyroiditis, factitious hyperthyroidism, and struma ovarii. Diagnosis is by complete history and examination in correlation with complete blood count and thyroid profile. Other tests include radioactive iodine uptake, fine needle aspiration cytology (FNAC), and biopsy. Treatment depends on the cause and severity of the disease, the patient's age, comorbid conditions, goiter size, and treatment requirements. The goal of therapy is the correction of the hypermetabolic state with the fewest side effects and the lowest incidence of hypothyroidism. Antithyroid drugs, radioactive iodine, beta-blockers, and surgery are the main treatment options. Fatigue and lethargy, slowed speech and movement, thickening and dryness of skin, impaired memory, and sleepiness are features associated with hypothyroidism.

A 21-year-old female returns to clinic for treatment of her abnormal pap smear. She had been sexually active since age 18. She uses oral contraceptives for birth control. Her pap smear showed mild cervical intraepithelial neoplasm, with positive high-risk HPV. What is the most appropriate next step for management of her pap smear? A HPV testing in 6 months B Pap smear in 6 months C Colposcopy with directed biopsy D LEEP procedure E STD testing

Pap smear in 6 months B According to new ACOG guidelines, cancer screening should begin at age 21. Routine follow up is based on normal pap smear and no history of high-risk HPV. The presence of high-risk HPV necessitates the exclusion of a higher grade cervical lesion.

Your patient is a 26-year-old mother of two young children whose second child was just delivered two weeks ago and she is breast-feeding. She complains of insomnia and depression. She denies the use of any drugs or alcohol. You would like to hold off on pharmacologic therapy unless her symptoms progress. Which of the following will be your first line of treatment? A Encourage more time with the baby B Encourage her to begin planning to get back to work C Promote adequate sleep D Suggest she get out of the house more often E Suggest her husband help more around the house

Promote adequate sleep C The correct answer is to promote adequate sleep (C). Postpartum "blues" are very common. Depression can occur in some cases, but most resolve without therapy. When required, SSRIs may be used even when a woman is breast feeding, though no studies have been done.

An adult patient was recently diagnosed with type 2 diabetes mellitus. She met with her diabetic educator to discuss suggested changes to her diet and exercise regimens. What percent of her total daily calories should be in the form of carbohydrates? A 5% to 15% B 20% to 25% C 35% to 40% D 45% to 65% E 70% to 80%

The correct choice is D, 45% to 65%. The American Diabetes Association recommends a diet with 45% to 65% carbohydrates, 25% to 35% fats, and 10% to 35% protein.

A premature infant presents with jaundice and a hemoglobin level of 8 g/dL. The peripheral blood film reveals reticulocytosis; a vitamin deficiency is suspected. Question What is the most likely diagnosis? Answer Choices 1 Vitamin A deficiency 2 Pyridoxine deficiency 3 Vitamin C deficiency 4 Niacin deficiency 5 Vitamin E deficiency

Vit E deficiency is the correct answer. Vitamin E deficiency may cause a hemolytic anemia in premature infants. Laboratory investigations reveal low plasma tocopherol levels, a low hemoglobin level, reticulocytosis, hyperbilirubinemia, and creatinuria. Causes of vitamin E deficiency in premature infants include limited placental transfer of vitamin E and the resultant low levels at birth; this is combined with its relative deficiency in the infant diet. Dietary sources for older children and adults include wheat germ, vegetable oils, egg yolk, and leafy vegetables. In cases of vitamin A deficiency, patients can present with night blindness or an inability to see well in dim light. There may also be conjunctival and corneal xerosis, as well as pericorneal and corneal opacities and Bitot's spots. Bitot's spots are a collection of keratin appearing as triangular, foamy spots on the conjunctiva. The patient may also have xeroderma, hyperkeratotic skin lesions, and increased susceptibility to infections. Causes include inadequate dietary intake and malabsorption. Dietary sources of vitamin A include fish, liver, egg yolk, butter, cream, dark green leafy vegetables, and yellow fruits and vegetables. In cases of pyridoxine (vitamin B6 deficiency), patients can present with peripheral neuropathy, seborrheic dermatosis, glossitis, and cheilosis. Laboratory analysis reveals anemia with lymphopenia. Causes include malabsorption, as well as medications (e.g., isoniazid and penicillamine). Dietary sources of vitamin B6 include liver, legumes, whole grain cereals, and meats. In cases of vitamin C (ascorbic acid) deficiency, patients can present with bleeding tendencies (as a result of weakened capillaries) and impaired wound healing due to the impaired formation of connective tissue. On examination, the gums may be swollen and friable; the teeth may be loose. There may also be multiple splinter hemorrhages on the nails and ecchymoses, especially over the lower limbs. Causes include inadequate dietary intake and certain conditions (e.g., pregnancy and lactation) which increase vitamin C requirements. Dietary sources of vitamin C include citrus fruits, (e.g., oranges, lemons, and tangerines), as well as tomatoes and potatoes. Niacin deficiency causes pellagra, which is characterized by: A symmetrical dermatitis, usually on parts of the body exposed to sunlight Scarlet glossitis and stomatitis Diarrhea Mental aberrations (e.g., memory impairment, depression, and dementia) which may appear alone or in combination Causes of niacin deficiency include inadequate dietary intake (especially in patients with corn-based diets or alcoholism). Dietary sources include legumes, yeast, meat, and enriched cereal products.

A 31-year-old man presents with burning substernal chest pain following meals. He is not experiencing nausea or vomiting. A stool sample is negative for occult blood. An upper endoscopy reveals some erythema of the lower esophagus. Esophageal biopsies from this area demonstrate neutrophils infiltrating a hyperplastic squamous epithelium. This biopsy finding by itself is most consistent with what? Answer Choices 1 Mild reflux esophagitis 2 Barrett's esophagus 3 Scleroderma 4 Herpes simplex infection 5 Congenital Atresia

mild reflux esophagitis

A 56-year-old man presents with fatigue following evening walks. The patient has been treated with atorvastatin for hyperlipidemia for about 6 months. Other medications that the patient has been receiving include diltiazem, ciprofloxacin, alpha-tocopherol, aspirin, and pioglitazone. What pair of medications can cause this adverse reaction? A Atorvastatin and diltiazem B Ciprofloxacin and alpha-tocopherol C Atorvastatin and actos D Ciprofloxacin and diltiazem E Atorvastatin and ciprofloxacin

A

A male infant presents for his 2-week physical exam. On exam, you notice white plaques on the buccal mucosa and palate. When you try to remove the plaques, there are small punctate areas of bleeding. What is the most likely diagnosis? Answer Choices: A Thrush B Epstein pearls C Bohn nodules D Milk residue E Aphthous ulcers

A

A couple presents to the office seeking genetic counseling advice regarding their child's potential risk of being born with Cystic Fibrosis. Both parents are identified as carriers for the cystic fibrosis gene mutation. Which of the following genetic inheritance patterns should be used to predict the probability of their child being affected by Cystic Fibrosis? A Autosomal dominant B Autosomal recessive C Mitochondrial inherited D X-linked dominant E X-linked recessive

Autosomal recessive Cystic fibrosis has an autosomal recessive (B) genetic inheritance pattern. In this case their children have a 25% chance of being affected, 50% chance of being a carrier, and 25% chance of lacking inheritance of the genetic trait.

A 45-year-old woman develops a high fever, severe headache, cough, and muscle aches. She is quite upset about missing work because she owns her own pet store and has no employees to cover for her in her absence. She tries to work for a day despite her illness, but she is unable to do so. The next day, she seeks medical attention. On physical examination, she is found to be febrile. Based on her occupational history and her physical examination, you suspect psittacosis. From what animal did she probably acquire the psittacosis? Answer Choices 1 Squirrel 2 Bird 3 Dog 4 Cat 5 Rabbit

Bird

A 40-year-old female is status post a dilatation and curettage for hydatidiform mole. On week 3 post surgery, her follow-up quantitative hCG level has elevated slightly. What is the most likely diagnosis? A Adenocarcinoma of the ovary B Adenocarcinoma of the uterus C Retained hydatidiform mole D Choriocarcinoma E Corpus luteum cys

Choriocarcinoma 5% of hydatidiform mole progress to choriocarcinoma; the longer the mole in intrauterine the higher the risk. Pre-surgical evaluation for mole removal includes a chest x-ray to rule out distant metastasis. hCg that either plateuas or elevates is choriocarcinoma until proven otherwise, and requires prompt evaluation.

A 73-year-old male presents to clinic with a history of blood in his urine for the past month. He has mild irritation with voiding but denies any other symptoms except an unintentional weight loss of 20 pounds in the past 6 months. His past medical history includes hyperlipidemia, seasonal allergies, and fibromyalgia. He denies alcohol use and has a 22 pack-year history of tobacco use. Which risk factor supports your diagnosis? A Fibromyalgia B Hyperlipidemia C Seasonal allergies D Cigarette smoking E Age

Cigarette smoking D Risk factors for bladder cancer include cigarette smoking (D) and exposure to industrial dyes and solvents. Age (E) is not a risk factor but the mean age of diagnosis is 73 years old and is more common in men than women (3.1:1). Fibromyalgia (A), hyperlipidemia (B), and seasonal allergies (C) have not shown any correlation to predisposing a person to bladder carcinoma.

A 23-year-old woman with epilepsy discovers she is pregnant. The epilepsy has been well controlled with phenytoin for several years. What is the best course of action? Answer Choices 1 Discontinue all anticonvulsants 2 Switch to phenobarbital 3 Switch to valproic acid 4 Continue phenytoin at the same dose 5 Continue phenytoin and monitor monthly levels

Continue phenytoin and monitor monthly levels Control of epilepsy is especially important during pregnancy. Generalized seizures or status epilepticus in a pregnant woman poses a significant risk to the fetus. Ultimately, the best course of action is prevention; women of childbearing potential who have epilepsy should be under optimal control, preferably with a single agent. Either phenobarbital or phenytoin is ideal, although both are teratogenic. Standard prenatal supplements and/or vitamins should be taken based upon the individual patient. During pregnancy, changing volume of distribution and protein binding require frequent monitoring of drug levels; dosage adjustments should be made as necessary. Although it used to be standard practice to give vitamin K to pregnant women on antiepileptics, it is no longer a universal practice.

A 27-year-old man with Marfan syndrome presents due to exercise intolerance and heart palpitations. On exam, you note a mid-systolic click and late systolic murmur heard at the apex of the heart. The click and murmur are noted later in systole with squatting and earlier in systole with sudden standing. What is the most likely diagnosis? A Aortic stenosis B Aortic regurgitation C Mitral stenosis D Mitral valve prolapse E Aortic dissection

D

Classification of shock is typically categorized into 4 areas of origin. The cause of shock can drastically affect the mode of treatment. If the etiology of a patient's shock is myopathic, mechanical, or arrhythmic, then this would be considered what type of shock? A Hypovolemic B Distributive C Obstructive D Cardiogenic E Septic

D

A 28-year-old man presents with increasing polyuria and nocturia without significant changes in his weight. He has noted increased thirst subsequent to increased urination. Lab results are as follows: Test Result Normal range Glucose 98 70 - 100 mg/dL Sodium 145 135 - 145 mEq/dL Serum osmolality 298 285 - 295 mOsm/kg Urine specific gravity <1.005 Urine osmolality is low, and the urine is poorly concentrated on the water restriction test. Question What treatment would best address this patient's symptoms and condition? Answer Choices 1 Bromocriptine 2 Desmopressin 3 Growth hormone 4 Hydrocortisone 5 Spironolactone

Desmopressin Explanation This patient has subjective and objective findings consistent with diabetes insipidus. Diabetes insipidus is characterized by the decreased production or action of antidiuretic hormone (ADH), which is produced by the posterior pituitary. The body is then unable to concentrate the urine, resulting in polyuria, nocturia, elevated serum osmolality, and decreased urine specific gravity. The treatment of choice for this patient's diabetes insipidus is desmopressin, which is a potent antidiuretic. Bromocriptine is an FDA-approved dopamine-agonist used in the treatment of hyperprolactinemia. Other uses include the treatment of acromegaly and Parkinson's disease. The mechanism of action for bromocriptine includes stimulation of dopamine receptors and inhibition of anterior pituitary prolactin secretion. It has no direct action on the posterior pituitary's secretion of ADH, nor on the kidney's urine-concentrating ability. Because autoimmune destruction of the pituitary is theorized as a cause in idiopathic central diabetes insipidus, there are often deficiencies in growth hormone (GH), thyroid stimulating hormone (TSH), and adrenocorticotropic hormone (ACTH), as well as the deficiency in antidiuretic hormone (ADH). Growth hormone (GH) would be helpful in treating disorders affecting the anterior pituitary's secretion of GH; however, treating this patient with growth hormone will not directly address the polyuria and nocturia caused by his diabetes insipidus. The presentation of a GH deficiency would include short stature, growth failure, hypoglycemia in infancy, and central distribution of body fat. This patient did not have a presentation consistent with GH deficiency. In cases of panhypopituitarism, secretion decreases for all pituitary hormones including ADH and ACTH. In this case, the low ACTH levels would lead to low cortisol levels and necessitate treatment with hydrocortisone. Hydrocortisone would not appropriately address this patient's polyuria and nocturia from his diabetes insipidus. Spironolactone, a potassium-sparing diuretic, inhibits the effects of aldosterone on the renal tubules. Using a diuretic on this patient would only exacerbate the fluid loss and polyuria.

Which of the following treatments is first-line therapy for sputum culture-positive Legionnaire's pneumonia in an immunocompetent patient? A ampicillin/sulbactam B erythromycin C ceftriaxone D vancomycin E clindamycin

Erythromycin First-line therapy for legionella pneumonia (mild to moderate) in the immunocompetent host is erythromycin 500 mg to 1 g IV qid or 500 mg po qid for 14 to 21 days. Another option for first-line therapy is doxycycline 200 mg IV or po once daily for 14 to 21 days. Alternatives include levofloxacin 500 mg IV or po q day for 7 to 10 days or azithromycin 500 mg IV or po q day for 3 days. Severe infection or treatment in the immunocompromised patient is levofloxacin or azithromycin.

A 76 year-old woman with steroid dependent chronic obstructive pulmonary disease is hospitalized with fever, chills, and a productive cough. The sputum gram stain shows many WBCs and small, pleomorphic gram-negative rods. Which of the following is the most likely causative agent? A Chlamydia pneumoniae B Haemophilus influenzae C Mycoplasma pneumoniae D Staph aureus E Strep pneumoniae

H influenzae Haemophilus influenzae (B) is a gram-negative pleomorphic coccobacillus. Strep pneumonia (E) and Staph aureus (D) are gram positive organisms. Mycoplasma pneumonia (C) and Chlamydia pneumoniae (A) aren't visible on gram stain.

A 68-year-old man presents for a routine physical exam. A soft venous hum over the epigastric and umbilical area is noted on auscultation. The venous hum is due to increased collateral circulation resulting from what type of condition? Answer Choices 1 Hepatic cirrhosis 2 Chlamydial perihepatitis 3 Gonococcal perihepatitis 4 Liver tumor 5 Recent liver biopsy

Hepatic cirrhosis A venous hum is due to an increased collateral circulation between the systemic and portal venous system resulting from hepatic cirrhosis. Chlamydial and gonococcal perihepatitis, liver tumor, and a recent liver biopsy cause grating sounds called "friction rubs" along the hepatic and splenic borders of the abdomen.

Which of the following is the causative agent in patients with Fifth disease? A herpesvirus 6 (HHV-6) B human parvovirus B19 C paramyxovirus D varicella-zoster virus (VZV) E none of the above

Human parvovirus B19 The disease is caused by parvovirus B19. It appears sporadically, but often in epidemics in communities. Children are infectious during the prodromal stage, which is unapparent or mild and usually indistinguishable from an upper respiratory infection. The rash is an immune-mediated phenomenon that occurs after the infection, so children with the rash are not infectious and should not be restricted from school or other activities. Aka slapped cheeks disease

A 40-year-old man presents with drooping of eyelids and difficulty getting up from a sitting position. He also gives a history of impotence and frequent dryness of his mouth. On examination, he is found to have ptosis, depressed reflexes, and loss of power in the proximal muscle groups of the lower extremity. On electrodiagnostic testing, he is found to have incremental response on repetitive nerve stimulation. What is the likely diagnosis in this patient? Answer Choices 1 Neurasthenia 2 Botulism 3 Lambert Eaton syndrome 4 Progressive external ophthalmoplegia 5 Myasthenia gravis

Lambert eaton syndrome The above history and findings are suggestive of Lambert-Eaton syndrome. Lambert-Eaton myasthenic syndrome (LEMS) is a presynaptic neuromuscular disorder that causes weakness of the muscles. LEMS is caused by antibodies directed against the calcium channel on the motor nerve terminal, leading to the decreased release of acetylcholine. 1/2 of the patients have an autoimmune form, the other half have a tumor-associated form (usually small-cell lung cancer) of the disease. Hence a thorough investigation has to be done to detect occult malignancy. Clinical features include: Weakness of proximal muscle groups initially (especially the lower limb) and later other muscles. Ptosis and diplopia. Depressed or absent reflexes. Autonomic changes such as dry mouth, impotence, etc. Worsening of symptoms in the morning and improvement with exercise, as opposed to myasthenia gravis Diagnostic data includes incremental response of the muscle groups on repetitive nerve stimulation, as opposed to decreased response in case of myasthenia gravis. Treatment includes plasmapheresis and immunosuppression. A significant number of patients have clinically benefited from 3, 4-diaminopyridine (DAP). Immunotherapy should be considered if severe weakness persists despite DAP. Plasma exchange and high-dose immunoglobulin induce a transient improvement in many patients, but function rarely becomes normal. Botulism is a paralytic disease involving the cranial nerves. It progressively involves the extremities in a rostrocaudal manner. It is caused by neurotoxin produced by the bacteria Clostridium botulinum (anaerobic gram positive organism) and is primarily associated with home canned foods. Cranial nerve involvement marks the onset of the disease with diplopia, dysarthria, dysphagia, and muscular weakness/descending paralysis. Ptosis is frequently seen. Diagnostic features include absence of fever, ptosis, depressed pupillary reflexes, suppressed gag reflex, and normal or depressed deep tendon reflexes. Paralytic ileus, severe constipation, and urinary retention may also be seen. Myasthenia gravis is a neuromuscular disorder characterized by weakness and fatigability of the skeletal muscles. The pathophysiological cause is the decrease in the number of acetylcholine receptors in the postsynaptic membrane. Due to the decreased receptors, the amount of acetylcholine bound to the postsynaptic receptors are decreased, leading to inadequate depolarization and ultimately resulting in weakness of the muscles. Clinical features include muscular weakness and generalized fatigue. The weakness increases on repetitive use of a group of muscles, and it reduces on rest. Neurasthenia usually presents as tiredness and weakness, and it mimics neuromuscular junction disorder. Muscle testing in these patients reveals a jerky release. The complaint of muscular weakness in these patients is due more to tiredness than any real decrease in the muscular power. Progressive external ophthalmoplegia is a rare condition usually involving the extraocular muscles and proximal muscle groups. This is usually caused by some mitochondrial disorder, and it is usually diagnosed by muscle biopsy.

Which one of the following organisms is the cause of epididymitis in young men under 35 years Answer Choices 1 N. gonorrhea 2 P. aeruginosa 3 C. acuminata 4 T. pallidum 5 H. ducreyi

N gonorrhea Explanation The cause of epididymitis in young men under 35 years is usually sexually transmitted organisms such as N. gonorrhea and C. trachomatis. T. pallidum causes syphilis and H. ducreyi causes chancroid.

A 19-year-old man presents for evaluation of excessive daytime sleepiness. Despite getting a full night's sleep, his coworkers have repeatedly noticed him suddenly dozing off at his desk and during meetings; sometimes, he falls asleep mid-conversation. Upon further questioning, the patient admits that he occasionally experiences nighttime episodes during which he has the sensation of being unable to speak or move. The remainder of the history and physical exam are unremarkable. Aside from the occasional ibuprofen for shoulder pain, he does not take any medications. Question The patient's symptoms are most consistent with what disorder? Answer Choices 1 Central sleep apnea 2 Insomnia 3 Medication side effect 4 Narcolepsy 5 Obstructive sleep apnea

Narcolepsy Based on the symptoms described, the patient is most likely to have narcolepsy. Narcolepsy is marked by 4 major symptoms: 1) excessive daytime sleepiness; 2) cataplexy; 3) sleep-related hallucinations;and 4) sleep paralysis. It is a central sleep disorder that is attributed to abnormal neurotransmission in the brain's sleep centers. Daytime sleepiness can be so severe that it results in involuntary somnolence during activities such as driving or talking. Cataplexy describes the brief loss of muscle tone that can occur sporadically during waking hours. Patients also may suffer from visual, auditory, or tactile hallucinations at the onset of or upon awakening from sleep. In addition, they may experience inability to move on falling to or emerging from sleep (sleep paralysis). Obstructive sleep apnea is defined as partial or complete upper airway obstruction caused by anatomic collapse of pharyngeal structures. Patency of the upper airway is partially (hypopnea) or completely (apnea) lost during sleep. This results in the development of progressively increasing inspiratory effort against an obstructed glottic opening, leading to the build-up of increasingly negative intrathoracic pressure. At some point, brain stem reflexes result in an altered sleep state that prompts reopening of the airway. This may awaken the patient or be observed as a "gasping" respiratory effort by an onlooker. Snoring is indicative of obstructed airflow, and daytime somnolence is a function of disrupted sleep. Central sleep apnea is an uncommon neurological condition that involves impairment of a patient's normal spontaneous cue to breathe based on changes in blood oxygen and carbon dioxide levels. The symptoms are similar to those of obstructive sleep apnea. Insomnia is a generic condition marked by difficulty falling asleep or inability to remain sleeping for extended periods of time. It is a symptom, not a diagnosis in and of itself. Sleep derangement is a common medication side effect; it is especially observed with psychotropic medications. There is no reason to suspect medication-related sleep impairment in this patient because he reports no medication use.

A 36-year-old presents to the office for evaluation of painful breasts, which is worse before her period. She complains of them feeling fuller and lumpier before onset of her menses. She has tried acetaminophen and ibuprofen with minimal relief. Her symptoms resolve at the end of her menses. What is the most likely cause of the symptoms? A Normal cyclic hormone fluctuation B Abnormal cyclic hormone fluctuation C No cyclic hormone elevation D Hormone secreting tumor E Hypothalamic dysfunction

Normal cyclic hormon fluctuation A Age, cyclic history, and resolution are essential in the diagnosis of benign fibrocystic breast disease.

A 65-year-old man presents with dry mouth and difficulties rising from a chair, climbing stairs, and walking. His symptoms started about 1 year ago, are worse in the morning, and improve with exercise. He is a heavy smoker and was recently diagnosed with lung cancer. Question What is the most likely underlying mechanism of his weakness? Answer Choices 1 Infiltration of the nerves 2 Spinal cord compression 3 Spinal cord metastasis 4 Paraneoplastic 5 Nicotine poisoning

Paraneoplastic Paraneoplastic syndrome (Lambert-Eaton syndrome), resembling myasthenia gravis, occurs in some people with small cell carcinoma of the lungs. It usually manifests as progressive weakness in the large muscles. Lambert-Eaton syndrome is caused by the inhibition of voltage-gated calcium channels on the presynaptic membrane of the neuromuscular junction; this prevents the release of acetylcholine. As the muscles continue to contract, acetylcholine can build up in sufficient quantities for the strength to get better; weakness improves after repetitive muscle contraction. Although the underlying mechanism is autoimmune, Lambert-Eaton syndrome in this patient is regarded as paraneoplastic because it is a consequence of a cancer, not due to a local presence of cancer cells. Tumor infiltration is a local manifestation of a tumor associated with the production of various types of extracellular matrix-degrading enzymes. Local infiltration of the nerves excludes variegated symptoms of autonomic nervous system disturbance and fluctuating weakness in several muscle groups. Secondary spinal cord tumors usually follow hematogenous spread to the vertebral bodies, epidural expansion, and/or intramedullary metastasis. Subsequent symptoms of compression (i.e., pain, radicular, or medullar symptoms) gradually worsen, not improve with exercise. Nicotine poisoning is not likely; it is impossible to overdose on nicotine through smoking alone. Smoking causes vascular disease, cancer, lung disease, peptic ulcer, and reproductive disturbances (e.g., premature birth). Nicotine may contribute to tobacco-related disease, but direct causation has not been determined because nicotine is consumed simultaneously with a multitude of other potentially harmful substances that occur in tobacco smoke. The effects of nicotine on nerves and muscles are generally dose-dependent; they also occur in nicotine-tolerant individuals. Initially, nicotine has a short-lived stimulatory phase followed by a longer inhibitory phase which leads to a neuromuscular blockade. Neuromuscular symptoms include hypotonia, decreased deep tendon reflexes, weakness, fasciculations, and paralysis of muscles (including respiratory muscles). Cholinergic effects on the autonomic nervous system, often observed initially, include diaphoresis, salivation, lacrimation, increased bronchial secretions, miosis, and later mydriasis. Nicotine acts on the sympathetic ganglia, chemoreceptors of the aorta, and carotid bodies; it affects the adrenal medulla, releasing catecholamines.

A 60-year-old woman presents for evaluation of loss of recent and remote memory and has psychomotor slowing with difficulty in complex tasks. On examination, the patient also shows some tremors in the hands and face. She also presents with a shuffling gait. Family history reveals 2 of her brothers had a movement disorder; they are now deceased. What would be your provisional diagnosis? Answer Choices 1 Delirium 2 Dementia of the Alzheimer's type 3 Vascular dementia 4 Parkinson's disease 5 Substance induced dementia

Parkinson's Dementia due to medical conditions include loss of memory and other cognitive deficits due to diseases such as Huntington's disease, Parkinson's, Pick's disease, multiple sclerosis, HIV disease, vitamin B-12 deficiencies, and hypothyroidism. In addition, head traumas and tumors can cause dementia, and all physical causes have to be ruled out first. Vascular dementia presents with the same signs and symptoms as other dementias, but the patient usually has a strong history of cerebrovascular disease, multiple infarcts, or history of arteriosclerotic disease. Vascular dementia usually has a sudden acute onset after the infarcts. Vascular dementia may also include behavioral disturbances in the patient. In patients with a delirium, there are usually rapid fluctuations in cognitive functioning, brief durations of normalcy, and disturbances in the sleep-wake cycle. In addition, deliriums come on acutely and rapidly. It is somewhat common in older patients who have had heart surgery and in those who have experienced a change of environments, such as a hospital intensive unit. In addition, falls have been known to bring on a delirium. When an older adult has a fall, it is very important to perform a CT scan of the head to rule out concussions, head trauma, hematomas, and other injuries. Clients with substance-induced dementia have cognitive deficits related to a drug or alcohol use or abuse and/or toxicity of prescription drugs; the toxicity can be accidental or intentional. This appears to be more common in the elderly than once thought. Many elderly take prescription drugs incorrectly or mix them with contraindicated over-the-counter drugs. Also, patients may suffer side effects and not report them to their physicians. Dementia of the Alzheimer's type involves gradual loss of memory and other cognitive functioning, which results in the inability to accomplish activities of daily living. Before a diagnosis is made, all other physiological causes should be ruled out. This type of dementia may also present itself as uncomplicated, with depressed mood, with delirium, or with behavioral disturbance.

A 32-year-old Caucasian woman with a past medical history of Hashimoto's thyroiditis, type I diabetes mellitus, and pernicious anemia presents to her family medicine office with a 2-year history of insidious and intermittent fatigue, anorexia, involuntary weight loss, nausea, abdominal pain, vomiting, and dizziness associated with position changes. Her physical exam is noteworthy for postural hypotension with a maximum systolic blood pressure of 104 in the supine position. Additionally, she has a low-grade fever and a generalized pigment change to her skin, as indicated in the image. Question What is the next most important intervention in this patient? Answer Choices 1 Refer the patient for an MRI of the brain 2 Recommend a low salt diet and begin potassium replacement 3 Begin a broad spectrum antibiotic 4 Start metyrapone (metopirone) 5 Perform a rapid ACTH stimulation test

Perform a rapid ACTH stimulation test Explanation This patient's most likely diagnosis is primary adrenal insufficiency, known as Addison's disease. The diagnosis of adrenocortical insufficiency rests on the assessment of the functional capacity of the adrenal cortex to synthesize cortisol. This is accomplished primarily by use of the rapid ACTH stimulation test (cortrosyn, cosyntropin, or synacthen). Imaging of the brain is unnecessary for this patient at this time since Addison's disease is adrenocortical insufficiency due to the destruction or dysfunction of the entire adrenal cortex. The onset of disease usually occurs when 90% or more of both adrenal cortices are dysfunctional or destroyed. This patient's hyperpigmentation signifies adrenal insufficiency, since hyperpigmentation only occurs with primary adrenal insufficiency (not secondary corticosteroid insufficiency, i.e., Hypothalamic-Pituitary-Adrenal Axis (HPA) Dysfunction). It is caused by the stimulant effect of excess adrenocorticotrophic hormone (ACTH) on the melanocytes to produce melanin. The hyperpigmentation is caused by high levels of circulating ACTH that bind to the melanocortin 1 receptor on the surface of dermal melanocytes. A low salt diet and increased potassium intake is inappropriate for this patient since most patients with Addison's disease have hyponatremia and hyperkalemia. Antibiotics are not indicated at this time, as this patient does not have an infectious pathology. Metyrapone (metopirone) is an inhibitor of endogenous adrenal corticosteroid synthesis. It inhibits the 11-B-hydroxylation reaction in the adrenal cortex and is thus utilized as an agent that inhibits steroidogenesis in Cushing syndrome.

A 30-year-old woman presents to the office complaining of an inability to achieve pregnancy after over a year of trying. What is the couple's least likely cause for infertility? A Azoospermia B Ovarian failure C Endometrial disease D Tubal disease E Pituitary disease

Pituitary disease E In the vast majority of women who are infertile or subfertile, the issue is due to either ovarian dysfunction or structural abnormalities, with hormonal abnormalities accounting for only 10%.

A 68-year-old female has been having increasing low back pain that radiates into her right hip and down her right leg. She has failed physical therapy and anti-inflammatories. She feels like the bones "are rubbing on each other." Radiographic imaging of the spinal cord, hip, and leg reveal several lytic lesions. The most helpful finding to confirm the diagnosis would include which of the following?

Plasmocytosis in the marrow Plasmacytosis in the marrow is the most prominent and diagnostically important finding. Reed-Sternberg cells are seen in Hodgkin's lymphoma. Granulocytosis is an increased white cell count seen in leukemias. Macrocytosis is enlargement of red cells, and can be seen in vitamin B12 deficiency. Monocytosis is an increase in monocytes, and can be seen in infectious mononucleosis or some leukemias.

What primitive, or immature, defense mechanism is demonstrated by a patient who attributes their own, unacknowledged, feelings onto others while they search for perceived wrongdoings, no matter how small? A acting out B isolation C projection D splitting

Projection C These patients are sensitive to any criticism and are constantly searching for any insult or mistreatments, no matter how small or unintentional they may be. Confrontation is to be avoided as it is only counterproductive and will reinforce their beliefs. This is commonly seen in paranoid personality disorders.

What is the most sensitive test available for the screening and detection of early thyroid dysfunction? A Radioactive iodine uptake B Serum T3 resin uptake C Serum total T4 level D Serum TSH level E Thyroid scan

Serum TSH level The correct choice is D, serum TSH level. Very small changes in serum TSH level can provide clues that there are changes in the functioning of the hypothalamic-anterior pituitary-thyroid axis. The test is easier and less expensive than any thyroid imaging tests. All of the other choices can be used in the work up of patients for thyroid dysfunction, but they are less sensitive, and many are more expensive and more invasive.

A 23-year-old Asian woman presents for follow-up after being seen in the emergency department. She was seen 2 months ago in the emergency department for a 3.2 cm simple ovarian cyst, which had resolved on repeat ultrasound in 6 weeks. She reports that she had severe pain with the cyst, but all of her symptoms resolved after cyst resolution. However, she had the same experience about 18 months prior to that due to another simple ovarian cyst of 3.5 cm, and her severe pain resolved after cyst resolution. She is not sexually active, and she does not use contraception. She has no chronic medical conditions, and she takes no medications. Medical records confirm her history. The physical exam is unremarkable, showing a patient of normal body habitus, without acne and hirsutism. She is very interested in preventing future recurrences of these painful ovarian cysts. Question What should be done for this patient who wishes to prevent future ovarian cysts? Answer Choices 1 Insert a levonorgestrel intrauterine system (Mirena) 2 Perform monthly ovarian ultrasound 3 Start her on a folic acid supplement 4 Start her on a moderate-dose combined oral contraceptive pill 5 Test her CA-125 levels

Start her on a moderate-dose combined oral contraceptive pill Explanation This patient's described ovarian cysts are simple and related to follicular development. Moderate-dose combined oral contraceptives have been shown to reduce ovarian cysts in population studies. Although a mechanism of oral contraceptives is suppression of ovulation, birth control pills do not treatexisting ovarian cysts. Furthermore, the lower-dose regimens on the market currently do not appear to lower the overall incidence of ovarian cysts and achieve prevention of cysts. Starting this patient on a moderate-dose combined oral contraceptive pill is reasonable, especially if indicated for prevention of pregnancy or other reasons. Performing a monthly ultrasound may identify early cysts, but doing so will not prevent them. Ultrasounds often show incidental functional cysts in the ovaries of asymptomatic women. Ultrasound would be indicated in a symptomatic patient or to follow a concerning cyst to resolution. Once resolved, this imaging is neither necessary nor cost-efficient. Insertion of a levonorgestrel intrauterine system (Mirena) is indicated for prevention of pregnancy and menorrhagia. The primary effect of the progestin, levonorgestrel, is intrauterine; therefore, ovarian function is generally unaffected. A levonorgestrel intrauterine system would not reduce ovarian cyst recurrence. Folic acid supplementation has long been recommended for women with cervical dysplasia, but few studies have actually confirmed that supplementation reverses dysplasia. Furthermore, there are neither recommendations nor evidence for folic acid supplementation preventing ovarian cysts. Testing for CA-125 should be done in women with current ovarian cancer or an ovarian mass suggestive of an ovarian cancer. This patient denies current symptoms; she is young and described simple ovarian cysts, which resolved.

A woman is diagnosed with primary tuberculosis. She is 5 months pregnant. What drug should be avoided in this patient? Answer Choices 1 Isoniazid 2 Rifampin 3 Ethambutol 4 Streptomycin 5 Pyrazinamide

Streptomycin Tuberculosis in pregnancy is treated with an initial regimen of isoniazid, rifampin, and ethambutol. Because the teratogenicity of pyrazinamide is not determined, pyrazinamide is added only if a resistance to the other drugs is documented/suspected, and susceptibility to pyrazinamide is likely. Streptomycin is contraindicated in pregnancy because it may cause congenital defects. Isoniazid, rifampin, and ethambutol are considered safe to use during pregnancy.

A 51-year-old man presents with what he describes as "fast, shooting pains" in his legs; he is also experiencing a slight tremor when he writes or eats. In the past, he has experienced an occasional URI, but he appears to be immunocompetent and has not required treatment for any infections subsequent to these viral syndromes. His past medical history does not include any significant neurological events, but the examination today reveals a loss of reflexes in his leg and bilateral Argyll Robertson pupils, pointing to a diagnosis of general paresis. How long after infection with Treponema pallidum would these symptoms manifest themselves? Answer Choices: A 1 to 2 years B 10 to 20 years C 3 to 5 years D 3 to 6 months E 5 to 10 years

B

You transfuse 3 units of whole blood to a gun shot victim. Unknown to you, there was a shortage of blood, and the units you gave to your patient were from the bottom of the freezer and about to expire. Within 2 days, the transfused cells are breaking down, and your patient's ECG is beginning to show high T waves and an increased PR interval. You recognize that he is becoming hyperkalemic and that the excess potassium is affecting the electrical conduction system of the heart. What best describes the resting membrane potential of a sinus nodal fiber under normal conditions? A -124 mV B -91 mV C -85 mV D -55 mV E -25 mV

D

A 23 year-old woman seeks guidance on foods, medications, and supplements that can help increase the amount of iron supplement. Which of the following is the best advice to provide this patient? A Co-administer with a calcium supplement B Co-administer with proton pump inhibitors C Co-administer with vitamin C D Taking with a basic beverage E Taking with food

Co-administration with Vitamin C An acidic achieved through presence of additional acids (C) (e.g., ascorbic acid) increases absorption of iron. Food (E), other heavy metals (A), and basic stomach environments (B, D) negatively affect the absorption of iron.

Glomerular damage inflicted in a patient with Goodpasture's disease is best described by which of the following? Answer Choices 1 Auto-antibodies are produced against glomerular basement membrane 2 Antibodies are produced against planted antigen 3 Auto-antibodies against Heymann antigen 4 Deposition of circulating immune complex in the glomeruli 5 C3Ne deposition in the glomeruli

1 Auto-antibodies are produced against glomerular basement membrane In Goodpasture's disease, autoantibodies (IgG) are produced against basement membrane antigen (type IV collagen). Simultaneous pulmonary hemorrhage and glomerulonephritis due to autoantibody deposition in pulmonary and glomerular basement membrane is known as Goodpasture's syndrome.

A 55-year-old man patient presents with tachycardia and heart palpitations. Physical exam shows a multinodular goiter. He does not have obstructive symptoms. He has suppressed TSH and elevated T 3 and T 4 , and a thyroid scan shows multiple functioning nodules. What is the treatment of choice for this patient?

131 I ablation the treatment of choice for multinodular goiter is 131 I ablation. In patients with very large thyroid glands with obstructive symptoms, surgical resection may be the best option.

Patients diagnosed with type 2 diabetes are encouraged to maintain their plasma glucose, to prevent or slow the initiation of chronic complications. What is the maximum level that is within ADA guidelines for a one to two hour postprandial plasma glucose in these patients? A 70 mg/dL B 100 mg/dL C 120 mg/dL D 180 mg/dL E 220 mg/dL

180 mg/dL The correct choice is D, 180 mg/dL. Patients are encouraged to adhere to lifestyle and medication treatment plans in order to help achieve this goal. Other ADA guidelines include maintaining the hemoglobin A1c < 7% or as close to normal (<6%) as possible, blood pressure < 130/80, and preprandial glucose between 90 and 130 mg/dL.

A women is being evaluated at her 36-week obstetrical appointment. She is not obese, her bladder is empty, and she does not have any complications. The fetus is in a cephalic position by Leopold maneuver. You measure her fundal height. What should it measure? A 32 to 34 cm B 32 to 36 cm C 34 to 38 cm D 35 to 39 cm E 32 to 39 cm

34 to 38 cm Fundal height in an uncomplicated, normal weight pregnancy should be within 1 to 2 cm per week of gestation in pregnancies above 20 weeks.

A 44-year-old man starts to notice that his eyelids are drooping. Some time afterwards, his jaw becomes weak. He has difficulty swallowing and also experiences weakness in his limbs. He is quite embarrassed when he eats because he must use his hand to help support his jaw. His weakness gets progressively worse. Finally, he seeks medical attention. His physical examination demonstrates the weakness in his limbs; however, no sensory defects are present. A Tensilon test is done and is positive. His doctor is concerned about an associated malignancy. What is the underlying pathology of this disease? Answer Choices 1 Inhibition of acetylcholine release 2 Blockage of the sodium channels 3 Demyelination 4 Subacute combined degeneration of the spinal cord 5 Antibodies to the acetylcholine receptor

5 Antibodies to the acetylcholine receptor Antibodies directed towards the acetylcholine receptor at the neuromuscular junction are seen with myasthenia gravis. This man has myasthenia gravis. Ocular muscle weakness, ptosis, dysphagia, and limb weakness can all be seen with myasthenia gravis. When the initial symptom is ocular weakness, Eaton Lambert Syndrome is extremely unlikely. Eaton Lambert Syndrome tends to not involve the extra-ocular muscles or the muscles involving chewing, swallowing, or speech. The Tensilon test is used in the diagnosis of myasthenia gravis. The Tensilon test consists of the administration of edrophonium. Edrophonium is a quick acting anticholinesterase. Thymic tumors are associated with myasthenia gravis. Thymic tumors are also referred to as thymomas. Approximately 10 - 15% of patients with myasthenia gravis have an associated thymoma. The majority of patients with myasthenia gravis have hyperplasia of their thymus. Botulinum toxin inhibits acetylcholine release. The site of action is at the neuromuscular junction. Botulinum toxin is an enterotoxin produced by Clostridium botulinum. Botulism can result from incorrectly canned foods. Tetrodotoxin is a toxin produced by puffer fish. The sodium channels are blocked by tetrodotoxin. The blockage of the sodium channels interferes with the inflow of sodium. As a result, the propagation of nerve and muscle action potentials is affected. Demyelination refers to the loss of myelin around the axon. Several disorders result in demyelination. An example of a demyelinating disease is multiple sclerosis. Subacute combined degeneration of the spinal cord is also called combined systems disease. Subacute combined degeneration of the spinal cord is a neuropathy secondary to B12 deficiency. It is seen in patients with pernicious anemia, especially pernicious anemia that has been present for quite some time. Symptoms include paresthesias and a loss of proprioception.

A 12-year-old girl presents with a 1-week history of a rash. The rash is annular in appearance and is slightly erythematous. There is no history of allergies and no illness in family members. Vaccinations are up to date; the family recently returned from a vacation in the northeast. Physical examination is unremarkable except for the rash. There has been consistent lethargy and fatigue; headache, fever, and chills have been intermittent. What is the treatment of choice in this patient? Answer Choices: A Tetracyclines B Sulfadiazine C Erythromycin D Cephalexin E Steroids

A

What orally effective antiviral agent, used to treat type-A influenza, may produce enhanced nervousness, confusion, or even psychoses, especially when used in the presence of antihistamines, anticholinergic drugs, Levodopa, or caffeine? See the attached image. Answer Choices: A Amantadine (Symmetrel) B Vidarabine (Vira-A) C Acyclovir (Zovirax) D Zidovudine (Retrovir) E Idoxuridine (Herplex)

A

A 37-year-old woman, G3P2 at 30 weeks gestation, complains of lower extremity swelling and her weight is up 5 pounds this week. Her PMH is insignificant, and her other pregnancy was uncomplicated. Her BP baseline is now 142/92. On exam her BP is unchanged, her UA shows 2+ protein, and FHTs are 152. What is the cause of the protein in her urine? A Glomeruloendotheliosis B Glomerulonephritis C Renal vasospasm D Glomerular hemorrhage E Glomerular infarct

A Glomeruloendotheliosis The classic histological change that occurs in the renal system (in preeclampsia) is swelling and inflammation of the endothelium and of the glomeruli, which leads to endothelial leaking.

Your patient is a 16-year-old female who weighs 300lbs and is 5 ft. 2 inches tall. She has severe facial acne vulgaris, hirsutism, and amenorrhea. Her pediatrician has sent her to your 24-hour endocrinology clinical research station, because she thinks that the patient may have polycystic ovary syndrome. Since ovulation occurs after a sequence of steps involving changing blood hormonal levels, you could follow these levels in the patient during an entire ovarian cycle by an indwelling monitoring catheter. What would you look for to rule out the disease? Answer Choices 1 A very large rise in estrogen level during secretory phase 2 A very large drop in progesterone just before ovulation 3 A very large rise in luteinizing hormone (LH) just before ovulation 4 A very large rise in LH just before ovulation with the FSH level dropping during proliferative phase 5 A very large drop in estrogen level just before ovulation

A very large rise in luteinizing hormone (LH) just before ovulation Explanation There is a very large rise in LH level just before ovulation. FSH produces a maturing effect on the follicle and ova. In addition, the follicle produces a protein, inhibin that inhibits FSH production from the gonadotrope. This protein will become important in the events leading up to ovulation. When the follicle and ova are fully matured, LH binds to LH receptors and LH sponsors ovulation. But just prior to ovulation, the LH spike occurs by the following mechanism: at normal levels of the estradiol, the gonadotrope is inhibited, but if the levels of 17b-estradiol are very high, they produce a stimulatory effect on the gonadotrope. Just prior to ovulation, the levels of estradiol rise to extremely high levels. This stimulates the gonadotrope to produce both FSH and LH, but since FSH is inhibited by inhibin, it has a small peak, whereas LH levels rise dramatically. This is called the "LH spike." This dramatic rise in LH triggers ovulation. After ovulation, the follicle changes to become the corpus luteum. Under the primary control of LH, the corpus luteum synthesizes progesterone. Stimulation of the endometrial cells through these receptors leads to vascularization and thickening of the endometrium. These are the preliminary stages of readying the endometrial lining for trophoblast implantation. If fertilization of the ovum does not occur, then a series of events leads to the involution of the corpus luteum. Since the corpus luteum is dependent on LH, it eventually involutes under decreasing levels of LH. Involution of the corpus luteum leads to the decline of estradiol and progesterone levels. This, in turn, leads to the programmed cell death of the endometrial cells lining the uterine cavity and to the process of menses. The inhibition on the gonadotrope cell is released, and the cycle repeats itself.

A 72-year-old man presents with low-grade fever, nausea, confusion, and lethargy. His past medical history is significant for hypertension, hypercholesterolemia, and diabetes. He had sinusitis approximately 1 week prior to presentation; otherwise, he has been healthy. Laboratory workup shows a CSF with elevated opening pressure and blood glucose levels. They also show a low CSF glucose. A Gram stain on the CSF shows Gram-positive coccus. CBC shows an elevation of PMNs, but it is otherwise normal. What is the most appropriate treatment in this case? Answer Choices 1 Antibiotic therapy 2 Steroids 3 Supportive care 4 Transfusion 5 Antiviral treatment

ABX therapy This patient is suffering from meningitis, which is an infection of the meninges that causes inflammation. It may be acute or chronic, and many different agents can cause it. Acute meningitis develops in a few hours to a few days, and chronic meningitis may develop weeks to months after exposure. Symptoms may be different in children or in older adults with underlying health conditions, such as diabetes or other immunocompromised states, IV drug users, those on a shunt for hydrocephalus, and extremes of age. Instead of presenting with headache and nuchal rigidity, they may present with lethargy and confusion. Both infectious and non-infectious agents can cause this disease. There are many infectious agents, including bacteria, viruses, fungi, and parasites. The Gram-positive cocci in the patient's CSF points to Streptococcus pneumoniae as the organism responsible. Bacterial meningitis is treated mainly with antibiotic therapy. The treatment usually consists of ampicillin plus ceftriaxone or cefotaxime plus vancomycin when meningitis is suspected and no other data is available. Vancomycin is added to cover penicillin-resistant S. pneumoniae, which may be present. If a Gram stain has been done and shows Gram-positive cocci, the treatment of choice is penicillin G or ceftriaxone. Meropenem is the drug of choice for cephalosporin-resistant isolates. When deciding on an antibiotic regimen, it is important to keep in mind the person's age and health status. Steroids are not considered first-line treatment. However, they can be used as an adjuvant therapy. They are thought to help lessen the severity of symptoms. Dexamethasone may be considered to reduce cerebral edema. Supportive care is important, especially if the patient's disease is advanced. However, antibiotic therapy should be initiated as soon as possible to avoid complications or death. On rare occasions, anticonvulsants may be required. Transfusion is not considered the treatment of choice for bacterial meningitis. If the patient has an underlying anemia or lacks certain clotting factors, transfusions would be supplied as part of his care. However, there is no indication of this in this case. Since he has a Gram stain showing Gram-positive cocci in his CSF, antiviral therapy is not indicated in this patient. Antibiotic therapy should be initiated immediately, and supportive care should be given as needed.

Your patient comes to the office for a follow up of her atrial fibrillation and hypertension. She has noted that she have been more tired than usual. Laboratory findings include a thyroid-stimulating hormone (TSH) < 0.05 mU/L. What medication is the most likely cause of her laboratory findings? A cardizem B amiodarone C warfarin D dotalol E labetalol

Amiodarone The correct answer is (B). Amiodarone is an antiarrhythmic medication containing iodine that is commonly used in treatment of atrial fibrillation. The use of amiodarone can cause thyrotoxicosis by several mechanisms and may also cause hypothyroidism. In this case the patients suppressed TSH would suggest the presence of amiodarone induced thyrotoxicosis. A high T 3 and FT 4 would support your diagnosis. All the other choices used in the treatment of atrial fibrillation would not cause thyroid dysfunction.

A 30-year-old woman delivers a viable 7 pound 6 ounce female infant by normal spontaneous vaginal delivery. The infant was delivered and held below the introitus. The infant was dried, stimulated, and apgars were assigned. The cord was then clamped and the infant was placed on the maternal abdomen. Which of the following is a complication that can arise from this? A Hypovolemia B Hyperbilirubinemia C Hypoglycemia D Hyperglycemia E Hypoxia

B Hyperbilirubinemia B In a normal delivery, after the infant is delivered through the introitus it should not be held below it; excessive fluids can be passed to the infant, resulting in increased hematocrit and hemoglobin, which will hemolyze and cause hyperbilirubinemia.

A 52-year-old male presents to the office with a complaint of intermittent dysphagia. His symptoms occur mostly when he eats steak, and have been ongoing for about six months. The symptoms have not worsened and he denies weight loss. You are concerned that he may have an esophageal web or ring. The diagnostic test that best visualizes an esophageal web or ring is which of the following? A Endoscopy B Esophageal manometry C Barium esophagram D Esophageal pH monitoring E CT scan of the abdomen

Barium Esophagram

A 15-year-old male adolescent presents with a 1-week history of malaise, a low-grade fever, and a sore throat. On exam, you note pharyngeal erythema, scant yellow exudates, and enlarged posterior cervical lymph nodes. There are no current signs of airway compromise. There is no skin eruption noted, and the rapid strep screen is negative. What other physical examination finding is usually present with this condition? Answer Choices: A Sclera icterus B Pancreatitis C Splenomegaly D Hepatomegaly E Cullen sign

C

How do you best describe dilated cardiomyopathy? A Left ventricular volume is either normal or reduced, and diastolic dysfunction is almost always present. B Non-dilated ventricles with impaired ventricular filling. C Dilatation and impaired contraction of one or both ventricles. D Transient systolic dysfunction of the apical and/or mid segments of the left ventricle that is often provoked by stress. E Diffuse thickening of the left ventricular endocardium secondary to proliferation of fibrous and elastic tissue.

C

A 61-year old female patient who is a smoker is undergoing a routine physical examination in your family practice. She is otherwise asymptomatic but her urinalysis reveals microscopic hematuria. Your next definitive step would include which of the following? A Refer her for a spiral CT scan of the kidneys to ascertain where her renal calculi may be and to rule out hydronephrosis. B Repeat her urinalysis in the morning after asking the patient to hydrate with at least eight glasses of water in the interim. C Send her urine for cytology and refer her to a urologist to rule out bladder cancer. D Send her urine for a C&S and depending upon the result, start her on antibiotics E Start a course of levofloxacin to resolve her occult urinary tract infection.

C Send her urine for cytology and refer her to a urologist to rule out bladder cancer. C Painless hematuria must always include bladder cancer in the differential diagnosis. Without another reasonable explanation—something that is not the case in this patient—bladder cancer must be ruled out, beginning with cytology and subsequent referral to an urologist.

What is the most common hematologic finding in a patient with pulmonary hypertension? A Anemia B Thrombocytopenia C Polycythemia D Leukocytosis E Elevated mean corpuscular volume (MCV)

C. Polycythemia Polycythemia is the most common finding. Hematocrits that are >60% usually require phlebotomy, to reduce the numbers and prevent a hypercoagulable state.

A young pregnant woman comes to her physician for a routine check-up. She tells her physician that she would like to test for any fetal abnormalities. After you discuss with her the tests available, she agrees on the procedure that allows for the earliest prenatal diagnosis in pregnancy. Which of the following test is the most likely? Answer Choices 1 Amniocentesis 2 Fetal Biopsy 3 Chorionic villus samplings 4 Umbilical blood sampling 5 Ultrasound

CVS Explanation Chorionic villus sampling can be performed as early as the eighth week of pregnancy under the guidance of an ultrasound. Chromosomal and biochemical abnormalities can be detected. This procedure has potential complications such as bleeding from the biopsy site, compromise of fetal membranes, and infection. Amniocentesis may be performed from 12-16 weeks of gestation. Penetration of the placenta is an unwanted complication. Amniocentesis should be considered if the woman is over 35-years-old, or if there is a previous child with a chromosomal anomaly.

A 67-year-old female with a history of oxygen dependent emphysema presents with a 4-hour history of increasing shortness of breath and pleuritic chest pain on the right side. Her resting oxygen saturation rate is 90%, and she is having pain on inspiration. On examination, the patient has decreased lung sounds with wheeze on the left and absent sounds on the right. There is also tympany to percussion on the right. Based on these findings, what is the best therapy for this patient? A Needle insertion to right chest wall B Supportive care C Increased oxygen delivery D Chest tube insertion E Nebulized albuterol

Chest Tube Insertion The treatment for this patient, who has a pneumothorax, is chest tube insertion and reinflation of the lung. Once the air leak has been eliminated and the lung appears reinflated on serial chest x-rays, the chest tube may be removed.

Circumstantiality

Circumstantiality is seen in someone who eventually gets to the point after a delay in the thought process.

Complications of Dialysis

Complications in hemodialysis: ◼ Hypotension most common acute complication Estimation of "dry weight" Effects of dialysate composition Effects of medications Usually feel dizzy Manage by re-calculating dry weight or hold off on BP medication and keep well hydrated ◼ Cramps: muscle cramps in 20% Improved by reducing ultrafiltration rate ◼ Arrhythmias and angina SVT, PVC's Effects of K removal ◼ Hypoxia: supplemental oxygen should be given if angina occurs ◼ Hemorrhage: Heparin is used to prevent clotting in extracorporeal circuit. If too much heparin is given can cause hemorrhage Light pressure over hemorrhage and give Protamine to reverse heparin overload ◼ Small amounts of blood remain in artificial kidney and tubing after each treatment

Defective red cell membrane protein (spectrin) deficiency is seen in which of the following hemolytic anemias? Answer Choices 1 Congenital spherocytosis 2 Thalassemia 3 Sickle cell anemia 4 Paroxysmal nocturnal hemoglobinuria 5 Hemoglobin C disease

Congenital Spherocytosis Hereditary spherocytosis is an autosomal dominant disorder characterized by deficiency of a red cell membrane protein known as spectrin. Patients with hereditary spherocytosis suffer from anemia and recurrent hemolysis. Bone marrow shows compensatory hyperplasia. Sequestration of spherocyte by the spleen results in splenomegaly.

A 42-year-old woman presents complaining of shortness of breath, three days of fever as high as 103F, and a cough productive of green sputum. On physical examination, you hear crackles in her lungs. A chest radiograph reveals a consolidation in the left lower lobe. What tactile fremitus findings do you expect to observe in the left lower lobe region? A Absent B Decreased C Hyperresonant D Increased E Normal

D increased Tactile fremitus is increased (D) by processes that cause consolidation within the lung parenchyma and decreased (B) by processes that insulate the lung parenchyma from transmitting vibration (e.g., pneumothorax, hemothorax) or airway obstruction.

A 73-year-old man presents to the local A+E with a 1-hour history of chest pain and shortness of breath. An ECG is obtained and found to have T wave inversion and wide Q waves in leads V1, V2, and V3. What are these ECG findings are most consistent with? A Acute ischemia without myocardial infarction B Acute lateral myocardial infarction C Acute inferior myocardial infarction D Acute posterior myocardial infarction E Acute anterior myocardial infarction

E

A 12-year-old boy presents with a 3-hour history of extreme, severe pain in the right testis. It started suddenly, is 8/10 in intensity, and does not radiate. It is associated with nausea and scrotal swelling. He never had such pain in his lifetime, and he denies any problem in urination. He has never been operated on, and he denies any history of trauma. He is allergic to penicillin. On physical exam, the child is in visible distress. Temperature is 37.0°C, heart rate is 95, blood pressure is 120/70 mm Hg, and respiratory rate is 20 per minute. Genital examination reveals enlargement and edema of the entire scrotum. The right testicle is erythematous and tender to palpation; it appears to sit higher and lies horizontally in the scrotal sac relative to the left side. The cremasteric reflex is absent ipsilaterally, and there is no relief of pain upon elevation of the scrotum (Prehn's sign). Abdomen is non-tender and tympanic to percussion in all 4 quadrants. Bowel sounds are audible. Chest auscultation shows normal vesicular breathing with mild crepitations over the lower lung fields. Cardiac exam reveals normal S1 and S2, without rubs, murmurs, or gallop. His initial labs show a hemoglobin of 14.5 g/dL, WBC of 13,000/mm³, platelets of 210,000/mm3, sodium of 140 mmol/dL, potassium of 3.8 mmol/dL, chloride of 95 mmol/dL, urea of 25 mg/dL, and creatinine of 0.9 mg/dL. Question What sign or symptom is the most sensitive for the diagnosis of this condition? Answer Choices 1 Tenderness 2 Edema 3 Horizontal lie 4 Prehn sign 5 Loss of cremasteric reflex

Explanation Testicular torsion is a true urologic emergency and needs to be differentiated from other causes of testicular pain (e.g., trauma, epididymitis/orchitis, incarcerated hernia, varicocele, idiopathic scrotal edema, and torsion of the appendix testis). The finding of an ipsilateral absent cremasteric reflex is the most accurate and sensitive sign of testicular torsion. This reflex is elicited by stroking or pinching the medial thigh, causing contraction of the cremaster muscle which elevates the testis. The cremasteric reflex is considered positive if the testicle moves at least 0.5 cm.

An otherwise healthy patient presents with a small lesion on the lips that, on biopsy, proves to be a mucosal neuroma. The patient's mother had medullary carcinoma of the thyroid. Question In addition to medullary carcinoma of the thyroid, to what condition would this patient be particularly vulnerable? Answer Choices 1 Gastrinoma 2 Insulinoma 3 Parathyroid adenoma 4 Pheochromocytoma 5 Pituitary adenoma

Explanation This is a probable case of multiple endocrine neoplasia, specifically MEN III. Features of this autosomal dominant condition include medullary carcinoma of the thyroid, pheochromocytoma, and oral and intestinal ganglioneuromatosis (including mucosal neuromas). Gastrinomas and insulinomas are found in MEN I. Parathyroid adenomas are found in MEN I and II. Pituitary adenomas are found in MEN I.

A 15-year-old girl presents due to primary amenorrhea. She was always healthy, does not take any medications, tobacco, or drugs, and denies intercourse. She has never experienced cyclic abdominal and/or pelvic pain. Her family history is negative for gynecologic or fertility problems, autoimmune diseases, and endocrinopathies; her mother's and female relatives' menarche presented in the ages from 12-14 years. Her breasts are absent. The rest of your examination is normal. The presence of her uterus is demonstrated by ultrasound. Question What should you obtain as your next step in diagnosis of this patient? Answer Choices 1 Estrogen levels 2 Folliculostimulating hormone (FSH) levels 3 Testosterone levels 4 Pregnancy test 5 Progesterone challenge test

FSH Explanation Primary amenorrhea is the absence of menstruation by the age of 14 if the patient has no secondary sexual characteristics, or by the age of 16 if the patient has secondary sexual characteristics present. The absence of breasts indicates inadequate estrogen production. The initial workup includes a pregnancy test and serum luteinizing hormone, follicle-stimulating hormone, prolactin, and thyroid-stimulating hormone levels. If history or examination suggests a hyperandrogenic state, serum free and total testosterone and dehydroepiandrosterone sulfate concentrations are useful. FSH levels will show where the lesion is. If there are no functional ovaries, FSH will be high; and if there is no FSH, ovaries will not be stimulated. The next step should be obtaining the karyotype. The lack of X chromosome, which is necessary for developing ovarial follicles, will point to the gonadal dysgenesis, and low FSH will point to hypothalamic dysfunction (normal ovaries are not stimulated to produce estrogen). Autoimmune oophoritis with anti-ovarian antibodies should be considered when previous tests are normal. Patients with autoimmune oophoritis are at risk for the development of adrenal insufficiency and other autoimmune endocrinopathies (thyroid and parathyroid, diabetes mellitus, myasthenia gravis, pernicious anemia, etc). There is no need to obtain estrogen levels; having no breasts, your patient most probably has no estrogen. The main question is why she does not have estrogen. FSH and karyotyping will help in the diagnosis.Testosterone levels should be obtained in the patient who has breasts but has no uterus in order to decide where the estrogen originates from; if testosterone levels are that of a normal female and ovaries are present, then the patient might have Mullerian agenesis (she also will have normal pubic and axillary hair and normal karyotype). If testosterone levels are that of a normal male, the estrogen source is probably the testes, as in androgen insensitivity syndrome, when a psychologically and physically female patient (with karyotype 46, XY) presents with primary amenorrhea with the lack of pubic and axillary hair and absent uterus. A pregnancy test is indicated in secondary amenorrhea. In the case of primary amenorrhea strongly suggestive for anatomical lesion or hormonal dysfunction, pregnancy test is not indicated as a first diagnostic step. Progesterone challenge test is usually performed in the case of secondary amenorrhea when other causes are excluded (pregnancy, hypothyroidism, prolactinoma, medications). Positive test is when a single dose of progesterone or 7 days of oral medroxyprogesterone causes withdrawal bleeding, as in anovulatory cycles. Negative test will demand further evaluation with estrogen-progesterone challenge (21 days of estrogen followed by 7 days of progesterone). Negative test will prompt endometrial problems (scars, adhesions) or outflow obstruction from other cause. Withdrawal bleeding will reveal inadequate estrogen levels, and your next step will be to find out why. This can be done by obtaining FSH levels.

A 22-year-old female present complaining of a lump in her left breast. She noticed it two days ago while taking a shower. She is a non-smoker and has three to four drinks per week. PMH is negative and FMH is negative. On physical exam, vitals are normal, and a breast exam reveals a 1-cm discrete, soft, and rubbery lesion in the upper outer quadrant—it is non-tender, and the remainder of the breast exam is normal. What is the most likely diagnosis? A Abscess B Fibroadenoma C Phylloides tumor D Cyst E Carcinoma

Fibroadenoma B The most common mass in premenopausal women by far is a fibroadenoma. While any lesion needs to be followed, soft, mobile, non-tender, and small lesions in young women without family history are very characteristic of fibroadenoma.

A 56-year-old man presents with signs and symptoms of hyperthyroidism in the presence of a palpable goiter. Laboratory studies are borderline suggestive, but they do not confirm thyrotoxicosis. Further testing includes a Radioactive I 131 Uptake (RAIU) and scan of the thyroid, which reveals a high uptake in a diffuse, uniform pattern throughout the gland. Question What is the most likely diagnosis? Answer Choices 1 Thyroiditis 2 Graves' disease 3 Toxic multinodular goiter (TMNG) 4 Toxic Thyroid Adenoma 5 Thyroid cancer

Graves' Disease Explanation The correct answer is Graves' disease, which is identified on Radioactive I 131 Uptake (RAIU) with an increased amount of iodine uptake consistent with hyperthyroidism and a scan that reveals a diffuse, uniform pattern throughout the gland. This is due to the autoimmune process of Graves' disease, in which circulating Thyroid Stimulating Immunoglobulins affect the entire gland. On physical exam, the gland will be firm and diffusely enlarged, which explains the diffuse uptake of iodine throughout the gland in the typical uniform pattern on RAIU and scan. A toxic thyroid adenoma and toxic multinodular goiter (TMNG) are also etiologies of thyrotoxicosis, and they can present with signs, symptoms, and laboratory values consistent with hyperthyroidism. On thyroid exam, a toxic adenoma is often a palpable thyroid nodule; however, a multinodular goiter will present, just as the name implies, with many fibrotic areas in an enlarged gland. Likewise, a toxic adenoma and TMNG may have an increased uptake on RAIU consistent with thyrotoxicosis, but they are distinguished from Graves' disease on the subsequent scan. A toxic adenoma reveals 1 area of increased uptake, which is localized to the adenoma and is often described as a "hot nodule," indicating the adenoma is hyperfunctioning. TMNG will reveal a heterogeneous pattern of radioactive iodine uptake localized to multiple hyperfunctioning nodules of various sizes throughout the gland. Patients with thyroid cancer most often present asymptomatically, with or without a palpable thyroid nodule and normal thyroid function studies. There is typically a normal RAIU uptake, but the scan reveals an area of decreased iodine uptake localized to the site of the thyroid cancer, which is often referred to as a "cold nodule." Thyroiditis usually occurs during or after a respiratory illness, and it presents acutely with symptomatic pain in the gland; the pain is often associated with fever, elevated WBCs, and sedimentation rate. Changes in serum thyroid hormone levels will depend upon the phase of the disease. Early in the disease, thyroid function is normal; then, hyperthyroidism develops as thyroid hormone is released from the gland after the initial insult. The pituitary gland responds to the excess circulating thyroid hormone by decreasing TSH production. Subsequently, the patient will progress from hyperthyroidism to euthyroidism to hypothyroidism. As thyroid hormone levels drop, the pituitary will respond by increasing TSH production, and the patient will enter the recovery phase, resuming normal thyroid function. RAIU will reveal a normal or low iodine uptake because there is no increased thyroid tissue activity and there is often no pattern visible on scan due to the limited uptake.

A 72-year-old male presents with bony pain, Bence Jones protein in his urine, an elevated creatinine level of 2.0, hypercalcemia, and lytic lesions to the long bones of his legs. The best initial treatment to correct this patient's renal insufficiency is which of the following?

Hydration and Calcitonin Management of renal impairment in patients with multiple myeloma is primarily supportive care. Hydration and use of calcitonin is the mainstay. If the hypercalcemia needs rapid correction, a slow infusion of bisphosphonate may be used. Dialysis would be useful if the patient has renal failure but has difficulty removing light chains from the blood, which are causing the problem. Plasmaphoresis doesn't play a role in this setting.

A 37-year-old G3P2 female at 39 weeks gestation presents to the labor and delivery unit complaining of abdominal pain. Laboratory evaluation reveals anemia coagulopathy that is felt to be consumptive. What is the most likely diagnosis to have caused this? A Placental abruption B Placenta previa C Preeclampsia D Labor E Pre-existing anemia

Placental abruption Reproductive placental abruption is the most common cause of coagulopathy in pregnancy; the hemorrhage may be concealed and is not always evident. In the presence of pain, anemia, and coagulopathy, abruption should be assumed until proved otherwise.

What is the most common blood gas abnormality in patients with a pulmonary embolism? A Respiratory alkalosis B Metabolic acidosis C Metabolic alkalosis D Respiratory acidosis E Compensated respiratory acidosis

Respiratory Alkalosis Respiratory alkalosis occurs as a result of the hyperventilation.

A 50-year-old man presents with a several month-history of increasing shortness of breath, fever, weight loss, and night sweats. The patient is a non-smoker who works in cotton fields. A chest X-ray reveals lymphadenopathy and reticulonodular pattern in all lung fields. A transbronchial biopsy is performed and microscopically demonstrates non-caseating granuloma. What is the most likely diagnosis? Answer Choices 1 Adenocarcinoma 2 Sarcoidosis 3 Histoplasmosis 4 Unusual interstitial pneumonitis 5 Berylliosis

Sarcoidosis Sarcoidosis is a multisystem granulomatous disease. Histologically, it is characterized by the presence of non-caseating granulomas, hence the diagnosis. The mediastinal and superficial lymph nodes, lungs, liver, spleen, skin, eyes, parotid glands, and phalangeal bones are frequently involved; however, all tissues may be involved. Sarcoidosis is not fatal unless it affects the vital organs such as the heart or the central nervous system. Calcium metabolism may be disturbed, causing hypercalcemia and (rarely) nephrocalcinosis and renal failure. 30% of the patients of sarcoidosis are asymptomatic. The disease is commonly detected by an abnormal chest radiograph, which reveals bilateral hilar lymphadenopathy in an asymptomatic patient. However, with extensive lung involvement, there may be exertional dyspnea or cough. The diagnosis can be made confidently from the clinical and radiological features. It can also be confirmed histologically from the biopsy of a superficial lymph node or skin lesion. Kveim test is a useful diagnostic procedure for sarcoidosis. The antigen (0.1 ml) obtained from human sarcoid tissue is injected intradermally and a small nodule develops 4 weeks later. If the test is positive, biopsy of the nodule reveals typical sarcoid follicle. The development of positive Kveim test is suppressed by corticosteroid therapy. Asymptomatic cases resolve spontaneously, but patients with persistent erythema nodosum, pyrexia, and arthralgias require oral corticosteroid therapy for a short duration. Symptomatic pulmonary sarcoidosis and sarcoidosis involving the eye or other vital organs usually needs to be treated with corticosteroids for several years. Adenocarcinoma will yield malignant cells, not granulomas, on biopsy. Histoplasmosis is caused by dimorphic fungi, Histoplasma capsulatum, or a variant, Histoplasma duboisii, which is found in the soil. It is caused by inhalation of the infected dust, which produces lesions similar to tuberculosis and have granulomas with central caseation. Interstitial pneumonitis is the end result of diffuse fibrosing alveolitis. Many cases are idiopathic. The end stage is a honeycomb appearance and no granulomas are seen. Berylliosis produces sarcoid-like lesions, but there would have to be a history of exposure to beryllium. Exposure to beryllium is now extremely uncommon.

A 70-year-old woman is brought to your attention by her family because of the slowly progressive gait disorder, the impairment of mental function, and urinary incontinence. About 1 year ago, she started having weakness and tiredness in her legs, followed by unsteadiness; her steps became shorter and shorter, and she also experienced unexplained backward falls. She is becoming emotionally indifferent, inattentive, and her actions and thinking have became "dull". Over the past month, she has started having urinary urgency and involuntary leaking of urine. Besides multivitamins and local application of the Timolol for glaucoma, she takes no other medications; there are no other symptoms. Question What is most likely the best method of treating the patient's urinary problems? Answer Choices 1 Antimuscarinic drug (Tolterodine) 2 Antibiotic (Sulfamethoxazole/trimethoprim) 3 Acetylcholinesterase inhibitor (Donepezil) 4 Ventriculoperitoneal shunt 5 Kegel exercises

Ventriculoperitoneal shunt Clinical triad of slowly progressive gait disorder, followed by impairment of mental function and then sphincteric incontinence strongly suggests the presence of normal-pressure hydrocephalus. Ventricular expansion is the cause of symptoms, and surgical CSF shunting is the main treatment modality. The potential benefit from surgery is usually evaluated by testing gait, cognition, and micturition before and after CSF drainage. Antimuscarinic Tolterodine is an antispasmodic that is used for symptomatic treatment of urinary incontinence in patients with an overactive bladder (urge incontinence). Antimuscarinic drugs are contraindicated in patients with glaucoma. A urinary tract infection will probably manifest with a strong, persistent urge to urinate, burning sensation when urinating, passing frequent, small amounts of urine that has unusual smell and the appearance. Your patient has no such signs and symptoms; therefore, in this case, antibiotics are not indicated. Donepezil is used to treat dementia, but in the case of normal-pressure hydrocephalus, the problem is anatomic (the distortion of the periventricular limbic system and frontal lobes), and the best treatment is probably surgical. Kegel exercises can prevent or control urinary incontinence and other pelvic floor problems in cases of pelvic sphincter weakness. However, pelvic sphincter weakness will probably manifest as stress incontinence.

A 23-year-old complains of chronic pelvic pain. It is worse with intercourse, several days before her period, and throughout her period. She is a GOPO, LMP two weeks ago. She is married and has had one partner for the last three years. Her symptoms have been increasing over the last year. What would the most appropriate initial management of her symptoms include? A Expectant management B Combined oral contraceptives C Surgical intervention D Acetaminophen E Androgen therapy

combined oral contraceptives B Endometriosis treatment is aimed at reducing pain and preserving fertility. Surgical interventional is not first line therapy until medication has been tried. NSAIDS are used, as opposed to acetaminophen, to lower the prostaglandin levels. Combined oral contraceptives suppress ovulation, decrease menstrual flow, and decidualize implants.

A 13-year-old girl presents with a very itchy foot. Laboratory tests show a microcytic anemia. A fecal sample for ova and parasite shows that there are the microscopic hookworm eggs in her feces. Where does the causative agent reside? Answer Choices: A Small intestine B Colon C Lung D Liver E Blood

A

A 52-year-old Caucasian man presents with easy fatigability, poor concentration, and "stronger than usual" effort-related palpitations. He denies any constipation or weight loss. His past medical history includes type II diabetes, which was diagnosed 10 years ago; it is well controlled with diet and metformin, and there are no complications. Both his parents are in good health; his sister had a malignant breast nodule removed 2 years ago, but she showed no signs of metastatic disease. His physical examination shows mild obesity (BMI 32 kg/m2), but it is otherwise normal. A laboratory workup shows glucose 91 mg/dL, Na+ 138 mEq/L, K+ 4.9 mEq/L, Cl- 100 mEq/L, hemoglobin 11 g/dL, hematocrit 32%, mean corpuscular volume(MCV) 82 fl, mean corpuscular hemoglobin concentration (MCHC) 24 pg/mL, leukocytes 7,400 /mm3 w/normal differential. Platelets 310,000 /mm3, ferritin 29 ng/L (25-300 ng/mL), and a total iron binding capacity of 450 ng/dL. Question What is the most appropriate next step in diagnosis? Answer Choices 1 Fecal occult blood testing (FOBT) 2 Full colonoscopy 3 Bone marrow biopsy 4 Double-contrast barium enema 5 Capsule endoscopy

The correct response is fecal occult blood testing. The pattern of low mean corpuscular volume and hemoglobin concentration, low ferritin, and elevated total iron-binding capacity (TIBC) is diagnostic of iron deficiency. Lack of iron inhibits hemoglobin synthesis, thereby reducing the volume and hemoglobin concentration of erythrocytes. Some patients have thrombocytosis. The mechanism is unclear, but hypotheses include stimulation by high circulating erythropoietin concentrations or circulating cytokines. Besides asthenia and pallor, other symptoms include pagophagia (craving ice or cold foods), pica (clay eating), and leg cramps; physical examination may reveal esophageal webbing, koilonychia (spoon-shaped nails), glossitis, angular stomatitis, and gastric atrophy. New-onset anemia with hypochromia and microcytosis in an adult should be considered secondary to gastrointestinal bleeding until proved otherwise. The most common etiologies are diverticular disease, angiodysplasia, colitis (including from NSAIDs), and colon cancer, particularly left-sided colon cancer. Testing for occult blood in a stool specimen is the initial diagnostic method of choice. If the fecal occult blood test is positive, the patient should undergo upper endoscopy and colonoscopy examinations. Colonoscopy is a useful exam for identifying sources of occult gastrointestinal bleeding. It is more sensitive than barium enema or CT scans for early lesions and permits biopsy when indicated (e.g., lesions suspicious for malignancy or colitis). However, none of these are cost-effective screening examinations. Bone marrow biopsy can evaluate bone marrow iron stores and reveal erythroid hyperplasia. However, it has been practically abandoned in the workup of iron-deficiency anemia, as there is considerable interobserver variability and assays for iron, ferritin, and TIBC permit evaluation of the body's iron stores less invasively. Capsule endoscopy is the most sensitive method for diagnosing small-bowel bleeding. However, small-bowel bleeding is much less common than upper (proximal to the angle of Treitz) or colonic bleeding. Therefore, it should only be undertaken after colonoscopy and upper endoscopy are negative.

Volume Overload tx

Volume overload treatment: • Restrict sodium intake < 2g day • Daily loop diuretic (furosemide) if edema o Thiazide diuretics ineffective if GFR <30 ml/min o If diuresis is not achieved with a loop diuretic, then try Metolazone

A 31-year-old female is admitted to the hospital with a microangiopathic hemolytic anemia, thrombocytopenia, and elevated reticulocyte count and serum lactate dehydrogenase. Haptoglobin is decreased and a direct Coombs test is negative. There is a marked increase in schistocytes on blood smear. PT and aPTT are normal. What is the best first line of treatment? A Aspirin B Glucocorticoids C Plasma Exchange D Platelet transfusion E Vincristine

plasma exchange

What would be tactile remits be in a pleural effusion

Decreased

Subacute sclerosing panencephalitis is a late neurological sequelae of infection from what virus? Answer Choices: A Orthoreovirus B Measles virus C Hepatitis B virus D Papillomavirus E Parvovirus B19

B

A 35-year-old man presents with knee pain that makes him unable to move his knee. The pain began 3 days ago and has markedly increased. The knee pain was preceded by a flu-like illness 1 week ago. The illness was accompanied by a rash on his thigh that had red margins and a pale center. It started 2 weeks after he came back from a hiking trip last month. He denies any urinary or GIT symptoms. He has no other significant past history. What could be a complication of the patient's illness? Answer Choices: A Stroke B Heart failure C Bell's palsy D Chronic constipation E Renal failure

C

A 72-year-old woman is admitted to the hospital following a syncopal episode. She began having dizziness and palpitations approximately 2 days ago. She admits to episodes of intermittent palpitations and dizziness over the last 3 months. EKG reveals atrial fibrillation. She is started on warfarin. Cardiology is consulted and recommends electrical cardioversion. When is the most appropriate time for electrical cardioversion to be performed? A Immediately B 2 weeks C 4 weeks D 6 months E 1 year

C

What hormone facilitates conversion of protein to glycogen? A Glucagon B Insulin C Thyronine D Somatotropin E Cortisol

A Glucagon B Insulin C Thyronine D Somatotropin *E Cortisol* Glucocorticoids have a rapid effect by causing peripheral protein breakdown and providing amino acid substrates for gluconeogenesis. Their most important metabolic effect is the facilitation of conversion from protein to glycogen: enhance the release of amino acids from protein in the skeletal muscle, lead to negative nitrogen balance, and inhibit de novo synthesis of protein. In other words, they show catabolic and anti-anabolic effects. Glucocorticoids are critical in survival during fasting. Otherwise hypoglycemia can cause death without gluconeogenesis from protein.

What is the typical incubation period for Salmonella gastroenteritis? Answer Choices: A 4 - 6 hours B 1 - 2 days C 6 - 18 days D 15 - 50 days E 2 - 6 months

B

You are currently on an inpatient pediatric hospitalist team; you see a pre-term infant who has signs of failure to thrive. Other signs and symptoms found during the history and physical examination include tachypnea, bounding peripheral pulses, and a rough, machine-like murmur. Considering the most likely diagnosis at this time, what clinical intervention should be initiated? A Ampicillin B Indomethacin C Surgical repair D Dopamine E Digitalis

B

What childhood psychiatric disorder corresponds with the clinical picture of cyclic expansiveness, aggressiveness, and impulsivity? Answer Choices 1 Major depression 2 Schizophrenia 3 Bipolar illness 4 Substance abuse 5 Enuresis

bipolar Patients with bipolar disorder have at least 1 manic episode, which is characterized by abnormally elevated, expansive, or irritable mood that lasts at least 1 week or requires hospitalization. Patients may or may not experience depression. Hypomania is similar to mania, but the episodes are less severe.

A 34-year-old man presents with a 2-week history of severe fatigue, increased swelling in both feet, and slight pain in the right abdomen. He gives a history of shortness of breath on severe exertion such as trekking and intense exercises. Exam reveals an afebrile patient with a prominent right atrium on the right border of the sternum on palpation. There is pedal edema and tender hepatomegaly. There are prominent 'a' waves on the jugular venous pulsations (JVP). Auscultation reveals a tricuspid opening snap. A diastolic murmur is also heard in the tricuspid area, which increases on inspiration. There is also a widely split S1 heard. Diagnostic tests reveal a normal hemogram. Right atrial enlargement is seen on the chest X-ray, and the echocardiogram shows a decreased tricuspid valvular diameter. What is the cause of tender hepatomegaly in this patient? A Metastasis of tumor seedlings from the right atrial myxoma B Primary hepatic tumor C Obstructed venous flow to the right ventricle leading to backflow pressure and hepatomegaly D Backflow pressure due to Inferior vena cava obstruction E Acute infection of the liver by Hepatitis B virus

C

A 72-year-old man with a history of poorly-controlled HTN and previous myocardial infarction presents with nocturnal cough, bilateral ankle swelling, and dyspnea on exertion. He denies any fever, chills, URI symptoms, chest pain, headache, N/V, diaphoresis, or syncope. He further denies smoking, alcohol, or drug use. Physical exam reveals bipedal edema and bibasilar crackles. Chest x-ray is remarkable for enlargement of the cardiac silhouette and interstitial infiltrates, while EKG analysis indicates deep S waves in lead V1 and tall R waves in lead V5. What is the most appropriate next step in the evaluation and management of this patient? A Biopsy of the endocardium B Chest CT scan with IV contrast C Transthoracic echocardiography D Bronchoalveolar lavage E Coronary angiography

C

A 75-year-old African American man presents with a 5-month history of gradually-progressive dyspnea that is especially pronounced when climbing stairs. He also has been noticing that his ankles and lower legs have "gotten larger" over roughly the same time period, which does not allow him to fit into his sneakers any longer. He denies fever, chills, chest pain, palpitations, cough, pleurisy, calf pain, abdominal complaints, sick contacts, or travel. His psychosocial history is noteworthy for chronic alcohol use. His physical exam reveals tachycardia, bibasilar rales, JVD of 5 cm, an S3 gallop, and 2+ pitting edema to the level of the mid-calves, bilaterally. What is the expected echocardiogram finding in this patient? A Asymmetric septal hypertrophy B Diffuse increase of ventricular thickness without ventricular chamber dilation C 4-chamber dilatation with thin left ventricular walls D Presence of valvular vegetations E Abnormal segmental wall motion during systole or diastole

C

A 25-year-old man presents with a painless, ulcerated lesion on the glans of his penis. He admits he had unprotected sexual intercourse with a prostitute 3 days ago. Since that time, he has not had sexual intercourse. Other than his wife, he denies having had other sex partners in the last 12 months. Past medical history and family history are unremarkable. The patient has normal vital signs. Physical examination reveals an indurated, painless, non-secreting chancre at the tip of his penis. There is no urethral discharge. Enlarged, painless lymph nodes are palpated on both inguinal regions. The remainder of the examination is within normal limits. What is the best next step in management? Answer Choices: A Intravenous penicillin G for 14 days B Single IM dose of penicillin G benzathine C Single dose azithromycin D Biopsy of the lesion E Perform lumbar puncture

B

To which of the following areas does follicular thyroid cancer most commonly first spread?

Bone and Lung via bloodstream The correct choice is D, bone and lung via bloodstream. Follicular thyroid cancer can spread to regional lymph nodes and distant sites via the blood stream. Choice A, intraglandular metastasis, is seen more commonly in papillary thyroid carcinoma, and choice B, local spread into regional vocal cords, occurs in anaplastic thyroid carcinoma. Choice C, distant lymph nodes, and choice E, local extension into the muscle and trachea, are more commonly seen in patients with medullary thyroid cancer.

A 20-year-old male presents following a syncopal episode during football practice. The patient is conscious and is not in acute distress. He denies abusing alcohol or "street drugs." The patient says he did not have fatigue, dizziness, or palpitations. On auscultation, you identify a systolic murmur in late systole. The murmur increases when you try Valsalva's maneuver. Otherwise, the review of systems is unremarkable. The patient's BP is 125/70 mmHg and HR is 94 bpm. The ECG you obtained shows right bundle branch block with QT interval of 495 ms. You decide to proceed with the study of cardiac anatomy and function with cardiac MRI. Refer to the image. Out of the following, what is the most probable complication that can result from this patient's condition? A Atrial fibrillation B Thromboembolism C Sudden death D Progressive LV failure E Pulmonary hypertension

C

A 54-year-old man presents with chest pain. He has a past medical history of hypertension and diabetes mellitus. The pain is located in the middle of his chest and radiates to his jaw. The pain began about 20 minutes ago, and he rates the pain as a 10 on a 0 - 10 point scale, with 10 being the worst pain he has ever felt. He has had 3 similar episodes, but they have always resolved after 5 minutes or so of rest. He has smoked 1 pack of cigarettes a day for the past 36 years. He drinks 2 or 3 beers on Friday nights. Review of systems (ROS) is positive for diaphoresis, acute dyspnea, and impending doom. ROS negative for fever, chills, and malaise. Physical exam shows an obese, middle-aged man in moderate distress. BP 126/80, pulse 100, respirations 26. Heart and lung exams are normal, except for tachycardia and tachypnea. He has no pedal edema. What aspect of the patient's history is the largest risk factor for an acute myocardial infarction? A Alcohol use B Cigarette smoking C Diabetes mellitus D Hypertension E Obesity

C

A 55-year-old man presents with a 2-day history of confusion, increased respiratory rate, rapid pulse, notable malaise, thirst, and less-than-normal urination. In the morning, his wife gave him a multivitamin pill hoping that it would help; she tells you that he has long-standing hypertension and 1 week ago injured his neck in a car accident. On examination, you find BP 92/41, HR 150, RR 35, rapid and weak pulse, cutis marmorata, cold edematous extremities, and bluish discoloration of the tongue and nails. What kind of shock does this patient probably have? A Neurogenic B Hypovolemic C Cardiogenic D Septic E Anaphylactic

C

A 39-year-old woman presents for evaluation of a 6-month history of menstrual irregularities; she has a history of regular menses. She had a bilateral tubal ligation (BTL), and she reports multiple negative home pregnancy tests. Despite regular exercise, she has been gaining weight. She reports easy bruising without a history of trauma and the new development of "stretch marks" on her torso; she is also experiencing decreased libido and mood swings. She denies any health changes, medications, or stressors in relation to these changes. She denies hot flashes, night sweats, palpitations, chest pain, and depression. The patient is frustrated that her health fair labs (which include complete blood count, complete metabolic panel, lipid panel and thyroid stimulating hormone) were all normal and do not explain her symptoms. Her past medical history reveals no chronic conditions; she does not use any medications, and she has no drug allergies. Her only surgery was the BTL. She lives with her husband and 3 children. She works as a retail clerk, walks for exercise and denies use of tobacco, alcohol, and drugs. Her blood pressure is 168/98 mm Hg. Chart review demonstrates weight gain of 15 pounds over 6 months; blood pressure has been normal in the past. On physical exam, you observe an overweight woman with an especially rounded, full face. She also has a fatty fullness to her neck region and some central obesity; however, her arms show muscle wasting. She has purple striae on her torso; some bruising and hirsutism are observed. The remainder of her exam is unremarkable; the cardiovascular exam is normal. What test should be performed next to evaluate this patient's elevated blood pressure? A 12-lead electrocardiogram (ECG) B Beta-natriuretic peptide (BNP) C Creatine kinase (CK), CK-MB, and troponin D Dexamethasone suppression test E Echocardiogram F King of Hearts or similar cardiac event monitor G Renal duplex ultrasound

D

A 68-year-old man with a history of hypertension, hyperlipidemia, and myocardial infarction presents to the emergency room with a 3-day history of shortness of breath at rest. He has found it difficult to walk short distances due to shortness of breath. Additionally, he complains of orthopnea, nocturnal dyspnea, and generalized abdominal discomfort. He denies cough, fever, chills, diaphoresis, anxiety, chest pain, pleurisy, cough, nausea, vomiting, diarrhea, rashes, lightheadedness, and syncope. Upon physical examination, the patient is acutely dyspnic. He is afebrile, but tachypnic and diaphoretic. There is a diminished first heart sound, S3 gallop, laterally displaced PMI. The abdominal exam reveals distension with hepatomegaly in the right upper quadrant. There is 2+ pitting edema of the lower extremities to the level of the mid calf. A bedside chest x-ray revealed the following image. What additional finding is expected to occur in this patient? A Bibasilar rales B Hyperresonance to percussion C Warm and dry skin D Depressed jugular venous pressure E Bradycardia

A

A 70-year-old woman presents with a 3-day history of shortness of breath at rest. She has been finding it difficult to walk short distances due to shortness of breath. Additionally she is experiencing orthopnea and nocturnal dyspnea. Her past medical history is significant for hypertension, hyperlipidemia, and myocardial infarction. The patient denies cough, fever, chills, diaphoresis, anxiety, chest pain, pleurisy, cough, nausea, abdominal pain, vomiting, diarrhea, rashes, lightheadedness, and syncope. Upon physical examination, the patient is short of breath; she requires numerous pauses during conversation. She is afebrile but tachycardic, diaphoretic, and her extremities are cool. The exam reveals a diminished first heart sound, S3 gallop, laterally displaced PMI, bibasilar rales, dullness to percussion, and expiratory wheezing. There is no JVD noted; however, 2+ pitting edema of the lower extremities to the level of the mid-calf is evident. What is the most likely diagnosis? A Congestive heart failure B Pulmonary embolism C Myocardial infarction D Chronic obstructive pulmonary disease E Bacterial pneumonia

A

A 49-year-old woman presents to her GP with a history of gradual onset of reduced exercise tolerance while working out at the gym. She is afebrile and otherwise feeling well. Cardiac examination reveals a grade III/IV rumbling mid diastolic murmur located at the apex, heard best in the left lateral position. What is the most likely diagnosis? A Tricuspid stenosis B Mitral stenosis C Mitral valve prolapse D Pulmonic stenosis E Aortic regurgitation

B

A 54-year-old man presents with a 30-minute history of oppressive retrosternal chest pain that radiates to the left arm. The physical examination reveals a BP= 170/100mmHg, P= 90/min, R= 24/min with normal cardiac and lung auscultation. The initial EKG shows an elevation of ST segment in leads V3 and V4. What artery is most likely compromised in this patient? A Septal branch of left anterior descending artery B Diagonal branch of left anterior descending artery C Circumflex branch of left coronary artery D Posterior descending branch of right coronary artery E Proximal branches of right coronary artery

B

A 55-year-old man presents with a 30-minute history of retrosternal pain that radiates to the jaw. He describes it as constricting in nature and worsening in intensity. He denies any trauma to the chest. On examination, he is diaphoretic and dyspneic. What is the most likely cause of this patient's chest pain? A Tietze's syndrome B Myocardial infarction C Panic disorder D Mallory-Weiss Syndrome E Tuberculous pleuritis

B

A 62-year-old man with a 15-year history of hypertension presents with severe tearing chest pain radiating through to the back. Blood pressure is 180/110 mmHg, heart rate 120 bpm, and respiratory rate 34/min. Physical examination findings include neck negative for bruits/JVD, lungs clear to auscultation, heart regular rhythm, normal S1/S2 with an S4 present, and grade III/IV diastolic rumbling murmur noted with patient leaning forward. Radial pulses are 1+ on right and 3+ on left. EKG reveals a sinus tachycardia and evidence of left ventricular hypertrophy. You decide that the patient most likely has a thoracic aortic dissection. You obtain a STAT chest x-ray. Which of the following findings is most consistent with your presumptive diagnosis? A Prominent pulmonary hila B Widening of the mediastinum C Kerley B lines D Right sided pulmonary effusion E Prominent right cardiac shadowing

B

A 26-year-old HIV-positive man presents to after a series of strange dreams and lack of concentration. He started on antiretrovirals 1 week ago. For his HIV infection, he takes zidovudine, lamivudine, and efavirenz. For pneumocystosis prophylaxis, he takes trimethoprim/sulfamethoxazole. He also takes a multivitamin. What medication is probably causing the patient's symptoms? Answer Choices: A Zidovudine B Lamivudine C Efavirenz D Multivitamin E Trimethoprim/sulfamethoxazole

C

A 39-year-old Caucasian man has a history of hypertriglyceridemia. He is well known to your practice, and he presents today for evaluation of his recent cholesterol lab values. He has a significant family history of cardiovascular disease: his mother had a heart attack at the age of 57; his father had open heart surgery at the age of 60. He has a long history of high total cholesterol and triglycerides levels; low HDL levels have been found in the past few cholesterol screening tests. At the last office visit, lifestyle modifications were implemented by the patient. He has been extremely conscientious about his diet; he has been engaging in physical activity 6 days a week for the last 6 months. Current lab values for the patient are as follows: total cholesterol of 235 mg/dL; triglycerides of 350 mg/dL; HDL of 35; and a LDL of 275 mg/dL. At this current appointment, the patient refuses to initiate pharmacologic treatment and wishes to continue lifestyle modifications to treat his cholesterol levels. He inquires to you what else he could do to make his actions be more effective. Question What advice should you give the patient? Answer Choices 1 Increase consumption of alcohol 2 Increase consumption of saturated fats 3 Increase consumption of fish oils 4 Decrease his aerobic activity 5 Increase consumption of refined carbohydrates

Explanation The patient should increase consumption of fish oils. Most patients with high cholesterol levels present with no specific symptoms or signs, although this is a significant contributor to the number one killer of adults in the United States, which is cardiovascular disease. There are a multitude of modifiable as well as non-modifiable risk factors present that contribute to overall cholesterol levels in each patient. If a patient has any abnormal cholesterol levels, the first component of any treatment is appropriate patient education regarding lifestyle modifications. Modifiable risk factors a patient should be instructed to initiate include the concept of moderation of energy intake and specifically limiting the energy intake from animal fats, which are also known as saturated fats. Body weight control is critical and regular aerobic activity should be encouraged in these patients. Patients should be instructed to avoid high-carbohydrate and refined carbohydrates. Increasing consumption of unsaturated fats is recommended; this has been shown to specifically lower triglycerides. Fish oils have also been found to contain the essential fatty acids docosahexaenoic acid (DHA) and eicosapentaenoic acid (EPA), both with known effectiveness at also lowering triglyceride levels. Limiting consumption of alcohol as well as smoking cessation are also critical components of lifestyle modification initiation. If an earnest attempt at lifestyle modification is taken and the efforts fail to bring cholesterol levels to normal ranges, then pharmaceutical intervention should be initiated.

A 40-year-old Caucasian woman presents with excessive menstrual bleeding, as well as bleeding between her menses; these symptoms have been occurring over her last 6 cycles. She also admits to associated "pressure and fullness" in her pelvis, dysmenorrhea, urinary frequency, and generalized fatigue. She denies being pregnant and has never had children. She denies any weight loss, fever, chills, chest pain, shortness of breath, abdominal pain, early satiety, nausea, vomiting, diarrhea, changes to her urine color or odor, flank pain, hematuria, or dysuria. Her physical exam is remarkable for a "lumpy-bumpy" cobblestone sensation above the symphysis pubis with an enlarged, mobile, and irregular contour upon bimanual palpation; no adnexal masses, cervical motion tenderness, or vaginal discharge is noted. Question What medication would be most beneficial in the initial management of this patient? Answer Choices 1 Leuprorelin (Lupron) 2 Raloxifene (Evista) 3 Clomiphene (Clomid) 4 Danocrine (Danazol) 5 Estradiol (Estraderm)

Explanation This patient's history and physical exam are most consistent with a diagnosis of uterine leiomyoma (fibroid). Pharmacologic inhibition of the growth of fibroids can be achieved by suppression of the hypothalamic-pituitary-ovarian axis through the use of gonadotropin-releasing hormone agonists (GnRH analogs) that decrease estrogen production, thereby reducing fibroid size by as much as 40-60%. It is also commonly used prior to a planned hysterectomy to reduce blood loss. Raloxifene (Evista) may help reduce fibroid growth. However, whether efficacy in reducing symptoms is comparable to that of other drugs is unclear. Clomiphene (Clomid) is indicated in the treatment of infertility, as it stimulates ovulation. Danocrine (Danazol) is an androgenic agonist and can suppress fibroid growth. However, it is not as successful as GnRH analogs and is associated with a high rate of adverse, androgenic effects such as weight gain, acne, hirsutism, edema, hair loss, deepening of the voice, flushing, sweating, and vaginal dryness; it is thus often less acceptable to patients. Estrogen replacement agents are not suitable as treatments for uterine fibroids since uterine leiomyomas are hormonally responsive benign tumors of the uterus that proliferate in response to estrogen.

A 60-year-old woman is referred to you for depressed mood, crying episodes, and an inability to sleep. She says that she has had these episodes before and was treated with antidepressant therapy. Her daughter also says that there is a family history of this behavior. She has no significant medical problems and has had this current mood for 6 weeks. What would this be suggestive of? Answer Choices 1 Bipolar disorder 2 Major depression, recurrent 3 Major depression 4 Depressed mood 5 Adjustment reaction

major depression, recurrent There are several indicators of a major depression in older adults. A major depressive disorder must consist of several symptoms for at least 2 weeks. These are: depressed mood, decreased interest in normal activities, significant weight loss or gain, insomnia or hyper-sleeping, psychomotor agitation or retardation, feelings of worthlessness or guilt, and/or poor concentration and possible passive or direct suicidal threats. Depressed mood is an indicator, but not a diagnosis. Treatment of major depression in older adults is most successful with a combination of antidepressant medications plus interpersonal supportive psychotherapy. Behavioral, cognitive, and family therapies have shown less promising results. Also, brief or short therapy has shown to be as effective or more effective as long-term psychotherapy. Also, most insurance programs do not fund long-term therapies. ECT (or electroconvulsive) therapy is a treatment of choice in the elderly if they do not respond to antidepressants or have a vegetative type depression. Major depression, recurrent is a term used to indicate that the older adult has had a previous episode and was treated. In diagnosing major depression, it is very important to do a complete medical examination and take a history first; doing so will rule out possible physical causes before drugs are prescribed. In treating depression, older adults usually do not want to see a psychiatrist unless it is indicated, and most older adults see their primary physician first. Psychiatrists are usually brought in to consult or act as a liaison to the primary physician. Bipolar disorder is characterized by 1 or more maleic episodes or mixed with maleic and depressive episodes. There is usually grandiosity, a decreased need for sleep, psychomotor agitation, and excessive involvement with pleasurable activities such as sex, spending, or traveling. Older adults with adjustment reactions usually have much less severe symptoms, and these usually occur after a major stress such as a death, change in residence, or other psychosocial stressors.

CKD stage 3/ Moderate kidney disease GFR

30-59

You are caring for a 29-year-old G1P0 at who is pregnant with twins. She has received routine obstetrical care and her pregnancy has been uneventful to date. What is the average gestation age for twins at delivery? A 33 to 34 B 39 to 40 C 35 to 36 D 36 to 37 E 37 to 38

36 to 37 D The average length of gestation for a single fetus is 40 weeks; the average age of gestation decreases with increasing number of fetuses.

A 44-year-old woman has symptoms of alternating constipation and diarrhea for 18 months. These symptoms are followed by rectal pain for the next 6 months. She seeks treatment, but she is prescribed only topical medicine and pain killers. She ends up with metastatic rectal cancer and dies within 2 months. Her 41-year-old sister is furious with what she conceives as poor medical care for her sibling and decides that she will not go through the same. What advice should you give the sister regarding screening for colon cancer? Answer Choices 1 Fecal occult blood testing annually only 2 Fecal occult blood testing annually until age 50 then screening colonoscopy every 5 years 3 Fecal occult blood testing annually until age 50 then sigmoidoscopy every 3 years 4 Fecal occult blood testing and double contrast barium enema right away then repeat every 5 years 5 Screening colonoscopy right away and repeat every 3-5 years if normal

5 Screening colonoscopy right away and repeat every 3-5 years if normal The recommendations for screening someone who has a first degree relative who has had colorectal cancer diagnosed at less than 60 years of age include screening colonoscopy starting at age 40 years, or 10 years younger than the earliest diagnosis in their family, whichever comes first. It should be repeated every 5 years. The same is advised for anyone with 2 first degree relatives with colorectal cancer at any age. Fecal occult testing annually alone is not a sensitive screening test for colon cancer. It should be used with sigmoidoscopy every 3-5 years, or with colonoscopy every 10 years for screening in the general population, starting at age 50.

Oliguria Urine Output

50-400 mL/day

A G4P4 woman delivers a viable infant at 38 weeks gestation by normal spontaneous vaginal delivery. The infant has apgars of 7 and 8 (at 1 and 5 minutes respectively). What is the most crucial time for maternal and fetal physiologic changes to occur? A 30 minutes prior to birth B 60 minutes prior to birth C Birth D 30 minutes after birth E 60 minutes after birth

60 minutes after birth E The hour after birth is the most critical time for physiologic changes to occur, including maternal fluid shifts, hemorrhage, retained placenta, and fetal lung cardiovascular transitions.

A 29-year-old G2 P1 who is term requests induction. Her pregnancy has been uncomplicated. She has been bothered by significant, poor quality contractions, which have caused her pain and interfered with her sleep. An elective induction is considered safe when the Bishops score is greater than what number? A 6 B 7 C 8 D 9 E 10

8 C A Bishop score greater than 9 is considered a positive predictor for safe delivery in a term pregnancy.

What body fluid is considered potentially infectious for HIV as well as hepatitis B and C? Answer Choices: A CSF (cerebral spinal fluid), non-bloody B Saliva (non-bloody) C Vomitus (non-bloody) D Sputum (non-bloody)

A

A 32-year-old female is brought into the emergency department by her partner. His report indicates that she had been in her usual state of good health until a couple of days ago. At that time she started to complain of feeling fatigued. She now appears jaundiced and lethargic, and is complaining of chest pain. On exam, her spleen is palpable. Hemoglobin is 6 g/dl and she is Coombs positive. What is the most likely diagnosis? A Autoimmune hemolytic anemia B Glucose-6-phosphate dehydrogenase deficiency C Hereditary spherocytosis D Pyruvate kinase deficiency E Thalassemia A

A autoimmune hemolytic anemia The onset of autoimmune hemolytic anemia (AHA) is often abrupt and dramatic. Anemia can develop in days, along with jaundice and splenic enlargement. When this triad is present, the suspicion for AHA must be high. The diagnostic test for AHA is the Coombs test. If positive, it confirms the presence of the antibody on the red cells. All other diseases listed are Coombs negative hemolytic anemias.

Which serological test can be performed for Dx Chron's Disease?

ASCA

Metformin Cr Cl level

CrCl must be < 1.3

Question: A 52-year-old man presents with a 4-hour history of a grossly soil-contaminated wound on his arm. He tried to clean it with some disinfectants, but it still looks dirty; on exam, there is dead tissue and soil visible. The patient's immunization status is uncertain. After surgical debridement of the wound, the next step for tetanus prophylaxis involves administering what? Answer Choices: A Tdap vaccine (toxoid-reduced diphtheria toxoid-acellular pertussis) B Tetanus immunoglobulin (TIG) C Penicillin D Tetanus immunoglobulin (TIG) and Tdap vaccine E Tetanus immunoglobulin (TIG) and penicillin

D

You are prophylactically treating an 84-year-old man with amantadine and vaccination for avoidance of an upcoming influenza outbreak. The patient is to receive 100 mg b.i.d. for 8 days. What is the antiviral effect of amantadine? Answer Choices: A Amantadine is phosphorylated by viral thymidine kinase and used as a defective guanosine analog in viral DNA synthesis B Amantadine binds to the hydrophobic sites on the influenza virus capsule. This destabilizes the virus, not allowing it to attach to the cell C Amantadine binds to ICAM-1, preventing viral attachment D Amantadine prevents viral uncoating and inhibits production of viral mRNA E Amantadine serves as a "cap" to viral mRNA and prevents binding of reverse transcriptase

D

What is associated with syphilis? Answer Choices: A Calymmatobacterium granulomatis B Chlamydia trachomatis C Hemophilus ducreyi D Scarcoptes scabiei E Treponema pallidum

E

A 57-year-old man, who is well known to your practice, presents with a 6-month history of a daily productive cough. The patient is a nonsmoker, and he has worked in a local coal mine for the past 39 years. He very rarely comes in to see a healthcare provider. When asked if he is up to date with his vaccinations, he does not recall the last ones he received; he also does not recall when these may have been given. Question Given the most likely diagnosis at this time, what vaccinations would be recommended to help decrease any significant morbidity and mortality in this patient? Answer Choices 1 Influenzae and pneumococci 2 Influenzae and zoster 3 Varicella and pneumococci 4 Zoster and varicella 5 Influenzae

Influenzae and pneumococci It is very likely that the patient described above has an occupation pulmonary disease process, most likely coal worker's pneumoconiosis, or black lung disease. Most patients that develop this disease tend to have worked around coal dust for several years, if not decades. For this reason it is commonly diagnosed in patients who are over the age of 50. Smoking does not increase the risk of developing pneumoconiosis. The chronic ingestion of inhaled coal dust by macrophages in the alveoli leads to the formation of coal macules, usually 2 - 5 mm in diameter. These areas then appear on a chest radiograph as opacities, especially in the upper lung. Simple coal work's pneumoconiosis commonly is asymptomatic; at times a cough or even varying degrees of shortness of breath may be present. The patient will typically not have any significant abnormal pulmonary function results. More severe cases of this pathology are usually referred to as progressive massive fibrosis. There is no cure for coal worker's pneumoconiosis; therefore, prevention of excessive and long-term exposure is necessary. Once a patient has developed symptoms of this disease, such as the patient above, the healthcare provider must ensure the patient is up to date with all potential vaccinations to avoid any pulmonary co-morbidities. This should be viewed as a critical component of health maintenance. The influenzae and the Streptococcivaccines would be crucial for this patient to receive; consequences of these infections could be extremely dangerous. A zoster vaccination may be considered, but would not necessary be the best answer. This patient receiving a varicella vaccine would not be recommended at this time.

Respiratory Acidosis

Low pH and High CO2

Hyperkalemia in CKD

Hyperkalemia Tx: • Usually seen in stage 3 in association with ACEI • Low potassium diet, avoid additional drugs that can raise K+ (like NSAIDs) • Stop ACEI or ARB if K is consistently high >5 mEq/L

Metabolic Acidosis

Low HCO3 and Low pH

A 16-year-old boy presents with acute onset of stiff neck, fever, headache, and vomiting. On exam, he appears lethargic, has limited range of motion of his neck, and a petechial rash is noted. Fever is 103 °F. The patient's medical history is non-contributory. What is the most likely etiology of the suspected diagnosis? Answer Choices 1 Enterovirus 2 Varicella virus 3 Syphilis 4 Pneumococcus 5 Meningococcus

Meningococcus Meningitis is an inflammation of the membranes covering the brain and spinal cord. The most common causes of meningitis are bacterial infections that start in other parts of the body and spread to the brain or spinal cord via the bloodstream. Meningitis can be caused by bacteria, viruses, chemical irritation, or tumors. Symptoms include fever, severe headache, nausea and vomiting, stiff neck (nuchal rigidity), photophobia, and mental status changes. Complications include hearing loss, brain damage, loss of vision, and possible deafness. Meningococcal meningitis is a life-threatening infection caused by the Neisseria meningitidis bacterium. Most cases occur in children and young adults. The onset is often abrupt, and early treatment is essential to achieving reduced morbidity and mortality. In addition to the usual symptoms, a petechial rash (non-blanching, red-brown spots on all parts of the body) is characteristic and caused by capillary leak and platelet dysfunction. Routine vaccination against meningococcus is recommended for adolescents in the United States; the first dose at age 11 or 12, and a booster dose at age 16. If this patient has had not routine medical care in many years, then he will not have been vaccinated against this disease. Enterovirus is a term given to cases of meningitis caused by viruses. Enteroviruses, such as the Coxsackie and echovirus, account for the majority of cases of viral meningitis. Other viruses that can cause meningitis include mumps, varicella, and the human immunodeficiency virus. Viral meningitis is usually milder than that caused by bacteria; symptom onset is more gradual, and patients usually recover with conservative management. Syphilitic meningitis is a progressive, life-threatening complication of untreated syphilis infection. In addition to the usual symptoms of meningitis, patients may present with prominent mental status changes and focal neurologic deficits. Serum VDRL or RPR along with CSF (cerebrospinal fluid) examination is diagnostic. Pneumococcal meningitis is caused by the bacteria Streptococcus pneumoniae. Meningitis occurs as a consequence of bloodstream infection with subsequent invasion of the central nervous system. Patients often present with a history of recent pneumonia, upper respiratory infection, infective endocarditis, or ear infection.

What is the laboratory test that has the best predictability for diagnosing pneumonia caused by adenovirus? A Nasal washings B Viral DNA test C Complete blood count with differential D Sputum gram stain E Polymerase chain reaction test (PCR)

PCR PCR testing is the best method for determining adenovirus pneumonia. It has a sensitivity of 90 to 100% and the specificity is >95%.

Role of Kidneys

Role of the kidneys: • Homeostasis o Maintenance of body fluids by ADH o Regulation of Na, K, Ph concentrations o Excretion of metabolic waste products- urea, creatinine, uric acid o Acid-base balance o Drug and toxin metabolism and elimination • Hormonal o EPO production- hormone that stimulates RBC production o Vit D and calcium metabolism o RAAS (renin angiotensin system)

A 45-year-old man presents with fatigue and hair loss that began about 2 weeks ago. His past medical history is significant for alcoholism and type II diabetes mellitus. He states that over the past 2 weeks, he has begun feeling more and more tired, and he has noticed that he was losing much more hair than usual. He denies pain, nausea, vomiting, fever, and chills; he states that he had been in his usual state of health until 2 weeks ago. He also denies recent illness and travel. He states that he now gets tired after very minimal activity and has poor wound healing. He admits that his diet is poor and that he does not control his blood glucose very well. Physical examination reveals a thin, frail man in no acute distress. He has an ulcerous lesion on his left heel that is draining but does not appear to be infected. Blood work including urinalysis, CBC, and WBC were within normal limits. Question What deficiency does this patient most likely have? Answer Choices 1 Niacin 2 Zinc 3 Folate 4 Vitamin B-12 5 Vitamin B-1

Zinc This patient most probably has a zinc deficiency. The causes of zinc deficiency include malnutrition, chronic debilitating diseases, chronic renal disease, alcoholism, drugs such as penicillamine and diuretics, and genetic disorders, such as sickle cell disease. Clinical manifestations in severe cases include alopecia, diarrhea, weight loss, infections, dermatitis, hypogonadism in men, and intercurrent infections. Supplementation with zinc is the treatment of choice. Niacin (nicotinamide, nicotinic acid) deficiency is uncommon in the United States. It is often found in people who live on a diet that consists mainly of corn. Deficiency may also result from alcoholism, cirrhosis, or diarrhea. Men and women are affected equally. Symptoms of niacin deficiency include nausea, vomiting, diarrhea, rash, glossitis, stomatitis, depression, and psychosis. Niacin deficiency, also known as pellagra, manifests as the '3 Ds': diarrhea, dermatitis, and dementia. If this diagnosis is suspected, supplemental niacin should be given immediately. Some patients deficient in niacin may also be deficient in other vitamins, such as B vitamins, so they should be given that as well. Folate deficiency causes megaloblastic macrocytic anemia, as folate plays a key role in nucleic acid synthesis. The early manifestation of folate deficiency, especially in its suboptimal state, predisposes to occlusive vascular disease and thrombosis. These manifestations are linked to increased homocysteine levels found in folate deficiency. Neurological and immunological disturbances may also be seen. Folate replacement is the option to prevent and to treat the deficiency. Vitamin B1 (or thiamine deficiency) causes beri beri, occurring mostly in the malnourished and alcoholics. The deficiency manifests with acute heart failure, neurologic deficits, and epilepsy. Empiric use of thiamine and prophylactic use in high-risk population is strongly recommended even before blood reports are obtained, as the treatment is inexpensive and prevents major catastrophes. Vitamin B12 or cobalamin deficiency manifests as megaloblastic macrocytic anemia, pancytopenia, and a spectrum of neurological disorders such as peripheral neuropathy, parasthesias, and demyelination of corticospinal tract. Nutritional deficiency, alcoholism, and malabsorption syndromes are some causes of B12 deficiency. It also is associated with homocystenemia and atherosclerosis. Diagnosis is by serum estimation of B12, and oral supplementation is safe and effective, but intramuscular injections may also be used.

In a patient who does not have thyroid disease, an elevated serum thyroid stimulating hormone (TSH) may be found under which of the following conditions? A acute corticosteroid administration B acute psychiatric admission C development of an hCG-secreting tumor D pregnancy E use of amphetamines

acute psychiatric admission About 15% of patients who are admitted for an acute psychiatric illness will have an elevated TSH in the absence of thyroid disease. Acute corticosteroid administration (A), hCG-secreting tumors (C), pregnancy (D), and use of amphetamines (E) are all associated with low TSH levels.

A 28-year-old G1P0 at 37.5 weeks gestation complains of a thin, watery discharge for the last 5 hours. She has soaked 3 pads. She has no pain and the fetus continues to be active. An external fetal monitor reveals heart tones in the 140s, with variability and no contractions. The pH of the vaginal fluids is 8. What does this indicate? A Normal vaginal secretions B Amniotic fluid C Yeast vaginitis D Bacterial vaginitis E Urine

amniotic fluid B Premature rupture of membranes is the rupture of membranes before the onset of labor (within 2 hours); preterm rupture is the rupture prior to 37 weeks gestation. The absence of contraction on the monitor, in addition to no complaints indicates no labor. Urinary incontinence is common, but the ph of 8 indicates amniotic fluid.

A 22-year-old female presents to her obstetrical appointment at 39 weeks gestation. Her pregnancy to date has been uncomplicated. She is concerned that her infant may be larger than average, as her fundal height measures 41. On physical exam, her fetus is in a cephalic presentation, her cervix is soft and 1-cm dilated, and the fetus is at a -3 station. Her membranes are intact, she is not contracting, and her vitals are normal. She requests to be induced. For decreased risk of complication and optimal fetal outcome, when should she expect to be induced? A Now B At 40.5 weeks C 41.5 weeks D 42.5 weeks E Not expected

at 41.5 weeks In the absence of complication, the recommendation from ACOG is to wait for labor to occur. Large for gestational age is not an indication for induction in the absence of diabetes. Gestations greater than 42 weeks increase risk of fetal stillbirth.

A 59-year-old female with history of non-insulin-dependent diabetes mellitus (NIDDM), hypertension, and chronic kidney disease (CKD) returns for follow-up of her labs. You note that she her GFR has decreased from 40 to 36 ml/min/1.73m 2 . What stage of chronic kidney disease is she currently? A 1 B 2 C 3 D 4 E 5

c 3 There are five stages of CKD. Stage 3 Chronic kidney disease is referred to as a moderately decreased GRF between 30-59 ml/min/1.73m 2 . All other choices reflect different ranges of GRF above or below stage 3.

Viral encephalitis MCC in the US

herpes simplex is the most common cause of sporadic encephalitis in the U.S.

A 52-year-old Caucasian woman presents with a 2-week history of rectal bleeding. Over the last 3 months, she has become progressively more tired, and could no longer play tennis on weekends. She denies diarrhea, abdominal pain or tenesmus. Her past medical history includes peptic ulcer disease 5 years ago, treated with an anti-H.pylori scheme and which has not recurred. Her vital signs are stable, and the physical examination shows no abnormalities. A CBC shows the following: Ht: 28% Hb: 8.8 mg/dl MCV: 82 fl Leukocytes: 8.100/mm3 w/ normal differential An upper endoscopy is normal except for a duodenal ulcer scar without signs of active ulceration, and a repeat colonoscopy is normal. Question What is most likely to identify the source of bleeding? Answer Choices 1 Celiac angiography 2 Computed tomography of the abdomen 3 Wireless capsule endoscopy 4 Push enteroscopy 5 Biopsy of the healed ulcer bed 6 99mTc scan 7 Small bowel follow-through99mTc scan

wireless capsule endoscopy Wireless capsule endoscopy is performed by ingestion of a battery-powered capsule containing a camera and a transmitter. The capsule transmits images to sensors outside the patient during its transit in the gastrointestinal tract. Indications for capsule endoscopy are still evolving, but several series have shown that capsule endoscopy is more sensitive than small bowel radiography or push enteroscopy for locating the source of small-bowel bleeding. The most common etiologies of bleeding in this region are angioectasia and Crohn's disease. Several other small bowel lesions, such as tumors (leiomyoma, lymphomas, carcinoids, or carcinomas), varices, and polyps, have also been described. An important concern is the possibility of capsule impaction and small bowel obstruction. Some centers are reluctant to perform capsule endoscopy in patients with partial or intermittent obstruction of the small bowel. However, capsule impaction may reveal the site of a lesion and allow its surgical removal, obviating the need for intraoperative enteroscopy. Other relative contraindications are incapacity of cooperating with the examination (e.g. dementia), swallowing disorders, esophageal stricture, gastroparesis, and poor surgical risk. In patients with obstruction or gastroparesis, the examiner may place the capsule endoscopically. This patient has no signs of ulcer activity. Therefore, biopsy of the duodenal scar is not indicated. A 99mTc scan is the test of choice for Meckel's diverticulum, a remnant of the embryonic omphalomesenteric duct found in 1.5% of the general population. Its most common location is the antimesenteric border of the mid-to-distal small bowel. Some diverticula contain ectopic, acid-producing gastric mucosa. Therefore, they can ulcerate and bleed. Rectal bleeding caused by Meckel's diverticulum is often maroon or brick-red in color, and its incidence decreases with age. Therefore, this diagnosis is unlikely in this case. Angiography can be used if other less invasive procedures (i.e. small bowel radiography, push enteroscopy, or small bowel radiography) are non-diagnostic. Computed tomography is not indicated in the absence of other symptoms indicating a specific etiology (e.g. a palpable mass or suspicion of pancreatic disease). Of note, colonoscopy after rapid oral purging has emerged as the procedure of choice for the evaluation of acute lower GI bleeding, and it also provides a means for therapy. Scintigraphy and/or angiography also play important roles in diagnosis and embolization when colonoscopy reveals negative findings or when it is not feasible.

A 12-year-old girl presents with a 3-day history of progressive dysarthria, dysphagia, and weakness. The patient was well until 3 days ago, when she developed the onset and subsequent gradual worsening of dysarthria. She attributed this to a sore throat, which she had about 2 weeks before. Three days prior to presentation, she had the onset of mild dysphagia, which occurred primarily with liquids. 24 hours prior to presentation, she developed weakness in both upper extremities; the weakness worsened and began to involve the lower extremities. This limb weakness was not worsened by activity, but it was not improved by rest. She developed tingling in her toes 24 hours prior to presentation. When she became unable to walk without assistance (on the day of presentation), she decided to seek medical care. Past medical history is significant for measles and mumps. Because of family religious beliefs, she has not had any immunizations. Physical examination reveals a well-developed, well-nourished girl. She is awake, alert, cooperative, and in no acute distress. Temperature is 98.7 F by mouth, blood pressure is 140/80 mm Hg, heart rate is 84/min and regular, and respirations are 22/min and unlabored. Signs of trauma include multiple scratches and abrasions over most of her extremities; they are in varying stages of healing. Speech is moderately dysarthric. She experiences some mild choking when she tries to drink a glass of water. She can smile weakly, but cannot raise her eyebrows against resistance. She shows mild bilateral weakness of eye adduction. Pupillary responses are normal. There is mild to moderate upper extremity and mild lower extremity weakness, greater distally than proximally. Her motor strength is sustained over at least 30 seconds without fatigue. Reflexes are hypoactive to absent, and the response to plantar stimulation is downgoing bilaterally. Sensation is intact, except for mildly impaired position and vibratory sensation in both feet. A complete blood count, chemistry profile, chest X-ray, and EKG are all found to be normal. Computed tomography of the brain, with and without contrast, is negative. Question What is the most likely diagnosis? Answer Choices 1 Tetanus 2 Diphtheria 3 Guillain-Barre Syndrome 4 Botulism 5 Multiple sclerosis 6 Myasthenia Gravis

GBS Guillain-Barre is the most common cause of acute and subacute generalized paralysis. In 60% of cases, it is preceded by mild respiratory or gastrointestinal infection 1-3 weeks before the onset of paralysis (1). Although it more typically begins distally and spreads proximally, the pattern seen here is a well-known variant. While classically described as a motor neuropathy, varying degrees of sensory neuropathy can be seen, as evidenced here by her paresthesias and decreased position and vibration sensation. Reduced or absent deep tendon reflexes are usual findings. Oculomotor nerves are paralyzed in only the most severe of cases; even the pupils may be unreactive. Diphtheria of the faucial-pharyngeal type produces an inflammatory exudate in the throat and trachea. Neurological sequelae usually begin after about 5-12 days, however, and usually with soft palate paralysis, which may produce dysphagia but also regurgitation and nasal (dysphonic) speech (1). Soon, trigeminal, facial, vagus, and hypoglossal nerves become involved. Loss of accommodation and blurring of vision (with preserved light reaction) usually appears by weeks 2-3. External ocular muscles are rarely involved. While cranial nerve findings may resolve around this time, the patient may go on to develop a mixed sensorimotor peripheral neuropathy at 5-8 weeks into the clinical course. Although this peripheral neuropathy may resemble Guillain-Barre, the other symptoms mentioned above and this long time delay, are not seen in this patient. Botulism is a rare form of blood borne illness caused by the exotoxin of C. botulinum (1). This exotoxin acts at the presynaptic endings of neuromuscular junctions, interfering with acetylcholine release. Outbreaks of botulism are usually due to ingestion of home preserved foods, especially vegetables and cured ham (1). There is no indication of an outbreak in this case. Rarely, botulism can be due to contaminated wounds (she played in horse fields and had multiple skin lesions). Symptoms usually come on within 12-36 hours (not unlike this patient), but they usually consist of nausea, vomiting, and anorexia. Generally, the first neurologic symptoms are blurring of vision and diplopia. These symptoms are rapidly followed by other cranial nerve findings, weakness of neck, trunk, upper then lower extremity weakness, and respiratory insufficiency. While the clinical course of botulism may be confused with Guillain-Barre, sensory loss would not be expected in botulism. Multiple Sclerosis of an acute and fulminant nature could be considered. The symptoms did come on rather suddenly, but their clear and rapid progression, with all symptoms being bilateral and symmetric, would be most unusual. While a negative computed tomographic scan does not fully rule out MS, in a patient with such dramatic and extensive bulbar findings, some brainstem or bilateral cerebral computed tomogram lesions would be expected. For the same reason, normal pupils would be most unusual. Also, with such widespread findings, Babinski signs would be expected. The deep tendon reflexes, if changed, would be increased. Tetanus might come to mind in a patient with skin injuries associated with playing in fields contaminated by horse feces (a common source of Clostridium tetani), who has had no tetanus immunizations, and who has the onset of bulbar symptoms first, with subsequent descending motor symptoms. However, while all the other listed diseases produce weakness, tetanus produces a progressively severe motor irritability that usually begins in the jaw with trismus. In some cases, it begins with a feeling of stiffness in the jaw and neck, slight fever, aches, and pains. This progresses to spasm as it spreads to other cranial nerve muscles, the neck, trunk, and extremities. This proceeds to tonic rigidity and tonic spasms that can be spontaneous or induced by the slightest stimulation. Certainly, sensory symptoms or signs would not be expected. Deep tendon reflexes would be increased. Myasthenia Gravis can be considered since the patient appears to a have a motor weakness that began in bulbar musculature then spread distally (distended). This pattern of weakness can be seen in myasthenia gravis, but it is also seen in a relatively common variant of Guillain-Barre. However, in myasthenia gravis, the weakness is more proximal than distal, which is the opposite of the more distal severity expected in neuropathies like Guillain-Barre. Myasthenic weakness tends to improve after rest and worsen with exercise, a pattern not seen in this patient. Again, sensory findings would not be expected in myasthenia; also, they would not be expected in a patient of this age.

Treatment for alcohol withdrawl in hospital

Librium and ativan

Herpes virus can be detected by the Tzanck smear of the skin lesion. This test stains for the presence of which one of the following? Answer Choices: A Multinucleated giant cells B Inclusion bodies C Dead cells D Herpes DNA E Herpes cell envelope

A

Compared to cow's milk, mature human milk contains less of which of the following? A Calories B Protein C Fat D Carbohydrates E Iron

A Calories B Protein C Fat D Carbohydrates E Iron Human milk contains about one third the amount of protein as cow's. Cow's milk contains only trace amounts of iron.

What is true regarding toxoplasmosis? Answer Choices: A Pregnant women should be instructed to stay away from their cats during the first trimester B The inhalation of cat dander can result in this condition C Although once thought to be of infectious etiology, this condition is now considered to immunologic in origin D Active infection is usually accompanied by high fever and a rash E Outbreaks of this condition have been associated with the ingestion of undercooked meat

E

What noninvasive pathogen is involved in acute diarrheal illness? Answer Choices: A Vibrio parahaemolyticus B Salmonella C Shigella D Yersinia enterocolitica E Vibrio cholerae

E

A 5-year-old girl was hit by a slow moving car when she ran into the street while playing tag. She hit her head and had a brief period of loss of consciousness along with a few superficial scrapes and bruises. She was placed in cervical spine precautions and brought to the ED where cervical spine radiographs were read as negative. What transient symptoms immediately after injury suggest the presence of spinal cord injury without radiographic abnormality? Answer Choices 1 Burning hands syndrome 2 A see-saw respiratory pattern 3 Weakness in the arms greater than that in the legs 4 Lightning sensation down the spine with neck movement 5 Complete paralysis with loss of pain sensation

4 Lightning sensation down the spine with neck movement Spinal cord injury without radiographic abnormality (SCIWORA) occurs almost exclusively among children younger than 8 years of age. Increased elasticity of the pediatric spine allows injury to the cord without obvious bony disruption. Up to 27% of these children may experience delayed onset of neurological signs. This delayed period may range from 30 minutes to 4 days, with a mean of 1.4 days. Clues to this condition may be the presence of certain transient symptoms immediately after the injury. These include, in addition to a lightning sensation down the spine associated with neck movement, paresthesia in the hands and legs or a subjective feeling of generalized weakness. The chief measures to improve outcome include injury prevention, prompt recognition through examination, search for such transient symptoms, use of MRI with electrophysiological verification, and timely bracing of SCIWORA patients. A burning sensation in the hands, fingertips, and occasionally feet after an injury, also known as burning hands syndrome, is probably due to hyperextension of the cervical cord with contusion of the spinothalamic tract. It is more commonly seen in football players. A see-saw respiratory pattern where the chest retracts and the abdomen distends may indicate paralyzed intercostal muscles due to cervical spine injury. Incomplete injuries include the central cord syndrome in which there is weakness in the arms that is greater than that in the legs, as well as variable bladder involvement. Another incomplete injury is the anterior cord syndrome that leads to complete paralysis below the level of injury, as the voluntary motor impulses carried by the corticospinal tracts are anteriorly located. There is also loss of pain sensation and /or temperature sensation. Preservation of proprioception through light touch remains intact.

A 10-year old boy presents with blurred vision, difficulty of swallowing, and weakness. The symptoms evolved rapidly, and his speech became slurred. Pupils are dilated. There is ptosis and loss of accommodation. The muscle action potential is low, and there is an incremental response on rapid repetitive stimulation. What is the most likely causal organism? Answer Choices: A Clostridium botulinum B Campylobacter jejuni C Epstein-Barr virus D Poliovirus E Coxsackievirus

A

A 32-year-old man presents with bilateral blurred vision and a large number of "floaters". The patient is HIV-positive and has experienced failure of long-term highly active anti-retroviral therapy (HAART). His CD4+ T cell count is 50 cells per mm2. Fundoscopy shows white exudates around the retina with peripheral hemorrhages. What treatment would be most appropriate for this patient? Answer Choices: A Metronidazole B Gentamicin C Ganciclovir D Zidovudine E Interferon

C

The triad of "dermatitis, diarrhea, and dementia" (pellagra) results from a severe deficiency of which of the following vitamins? A thiamine B vitamin K C riboflavin D niacin E pyridoxine

D Niacin Niacin deficiency is known as pellagra. It is rare in the United States and is most often a complication of alcoholism or malabsorption syndrome. Clinical signs of pellagra are known as the 3 Ds—dermatitis, diarrhea, and dementia.

Derailment

Derailment is when a patient skips to another subject. This mainly occurs if a topic is brought up that the patient does not wish to discuss.

You are examining the wet prep of a young woman who presented complaining of vaginal discharge. You see 15 to 20 WBCs, 2+ bacteria, and clues cells. The KOH prep is whiff positive. What type of vaginitis does this indicate? A Yeast vaginitis B Viral vaginitis C Gardnerella vaginitis D Trichomonas vaginitis E Atrophic vaginitis

Gardnerella vaginitis Gardnerella produces a positive amine test when the preparation is mixed with KOH. Viral Vaginitis, yeast, and Trichomonas do not produce a positive amine smell when mixed with KOH.

Hematuria other causes include:

Hematuria other causes include: • Stones, UTI, kidney or bladder cancer

What is the most common cause of pneumothorax in a healthy patient? A Traumatic B Infectious C Ruptured bleb D Surfactant abnormality E Malignancy

Ruptured bleb

Consideration should be given to screening patients with type 1 diabetes mellitus should also be screened for which of the following: A sarcoidosis B Sheehan's syndrome C Sjögren's Syndrome D thyroid disease

Sjögren's Syndrome Type 1 diabetes mellitus (T1DM) is an autoimmune disease. As such, patients have a significantly higher risk of other autoimmune diseases, including celiac and thyroid disease. Most recommendations include screening for both diseases in patients with T1DM.

A 40-year-old man presents to the emergency department with severe pain in his left eye, decreased vision, nausea, and abdominal pain. On examination, the patient's left pupil is moderately dilated and nonreactive. The cornea is "steamy" in appearance and generally the eye is red. What do you suspect is the cause? Answer Choices 1 Conjunctivitis 2 Acute uveitis 3 Acute angle-closure glaucoma 4 Corneal ulcer 5 Corneal infection

acute angle closure glaucoma This patient has acute angle-closure glaucoma. The typical characteristics of this condition are all exhibited by this patient (i.e., steamy cornea, severe pain, blurred vision, dilated, and nonreactive pupil). A physical exam finding in a patient with acute uveitis would be a small pupil. In addition to these symptoms, patients may also present with abdominal pain. A corneal infection and corneal ulcer would cause circumcorneal injection and watery or purulent discharge.

A 65-year-old woman presents with a complaint of blood in her urine, intermittently for the last month. The patient denies any fever, chills, flank pain, or dysuria. Social history is positive for tobacco use (45 pack years), but patient reports stopping her tobacco use last year. What is the most likely cause of her hematuria? A urinary tract infection (UTI) B bladder cancer C renal calculi D pyelonephritis

bladder cancer B Hematuria in women older than 60 years is consistent with a bladder malignancy. Bladder cancer causes episodic, gross hematuria that is usually painless. Cigarette smoking is a risk factor that also increases the incidence of bladder cancer. Painful hematuria associated with suprapubic discomfort or dysuria (or both) is more indicative of cystitis or calculi. Pyelonephritis is associated with chills, fever, and flank pain.

A 45-year-old woman presents as a new patient. She was recently seen in the emergency department for right flank pain, and a CT scan revealed a right-sided ureteral stone. The stone was 4mm, a passable size, and she was sent home with analgesics and advised to hydrate well and strain her urine. She was straining her urine and noticed a small, dark fleck. She brought the sediment to the urologist's office to undergo a stone analysis, as this is her first episode of a renal or ureteral stone. Question What is the most likely composition of this stone? Answer Choices 1 Uric acid 2 Struvite 3 Cystine 4 Magnesium 5 Calcium

calcium The correct answer is calcium. Calcium stones are the most common type of renal calculi and can be further characterized as either calcium oxalate or calcium phosphate stones. Calcium oxalate stones make up about 60% and calcium phosphate stones make up about 20%, which results in calcium stones being about 80% of stones overall. Geography, fluid intake, and diet can all influence stone formation, but metabolism and genetics can also play a role. Absorptive hypercalciuria, renal hypercalciuria, and resorptive hypercalciuria can all result in calcium stone formation. Hyperuricosuria, gout, hyperoxaluria, and hypocitraturia are all other causes of calcium calculi. Uric acid is the not the correct answer, as this is not the most common type of renal calculus. Uric acid stones make up about 10% of stones in the United States. Hyperuricosuira and/or a urinary pH less than 5.5 are the 2 most common causes of uric acid stones. Gout, increased turnover of nucleic acids (such as in polycythemia or psoriasis), increased purine intake, and alcohol consumption are all causes of hyperuricosuria and can therefore put a patient at risk for uric acid stones. Struvite is not the correct answer, as this is not the most common type of renal calculus. Struvite stones make up about 7% of total renal calculi. Urinary tract infections secondary to the presence of urea splitting organisms can result in the formation of struvite stones. Klebsiella, Proteus, Staphylococcus, and Pseudomonoas produce urease. Urease breaks down urea and aids in the formation of ammonia. The ammonia then undergoes hydrolysis, which results in alkaline urine and reduced solubility of struvite, as well as urine that is supersaturated in struvite. Cystine is not the correct answer, as this is not the most common form of renal calculi. Cystine stones make up about 3% of total renal calculi. Cystine stones form due to an autosomal recessive disorder in the metabolism of cystine, which leads to cystinuria. Once the urinary saturation of cystine is more than 250 mg/L, cystine stones can start to form. Magnesium is not the correct answer, as this is not a type of renal calculi. In fact, magnesium has been known to be preventative of stone formation and is, therefore, a component in a lot of renal calculi prevention measures. Hypomagnesuria, usually dietary in nature, is a known risk factor for renal calculi formation. Maintaining urine magnesium about 50 mg/day is preventative.

Your patient states she has been gaining weight for no apparent reason over the past year and is concerned that she might have hypothyroidism. What other historical information would support a diagnosis of hypothyroidism? A anxiety B diarrhea C depression D palpitations E heat intolerance

depression The correct answer is (C). Depression is a common presenting symptom of hypothyroidism. Weight gain can occur with hypothyroidism. Other symptoms may be weakness, fatigue, and menorrhagia. Hoarseness may also be a presenting symptom. The other choices are common symptoms of hyperthyroidism.

Which of the following agents is a significant cause of pill-induced esophagitis? A fluoxetine B omeprazole C ibuprofen D Vitamin D E ciprofloxacin

ibuprofen

The risk of extrapyramidal side effects (pseudoparkinsonism) and tardive dyskinesia is associated with which class of medications? A amphetamines B benzodiazepines C monoamine oxidase inhibitors (MAOIs) D tricyclic antidepressants (TCAs) E typical (first-generation) antipsychotics

typical (1st generation) anti-psychotics E Typical antipsychotics (eg, haloperidol, chlorpromazine, fluphenazine) can produce pseudoparkinsonism via blockade of dopamine (D 2 ) receptors in the nigrostriatum. Symptoms can include akinesia, bradykinesia, mask-like facial expression, tremor, cogwheel rigidity, and postural abnormalities. Tardive dyskinesia may also occur, as the reported incidence with first generation antipsychotics ranges from 0.5% to 62%.

A 32-year-old woman is 2-hours status post cesarean delivery of a twin gestation at 36 weeks. Her pregnancy was uncomplicated. She presented in early labor, which became prolonged despite oxytocin infusion. A cesarean section was performed when her labor became non-progressive. In the recovery area, she begins to complain of nausea and lightheadedness. On exam, her heart rate is 133 beats per minute and blood pressure is 76/42. Significant vaginal bleeding is noted, and abdominal palpation reveals a soft uterus. Question What is the most likely diagnosis? Answer Choices 1 Cervical laceration 2 Placenta accreta 3 Prolapsed uterus 4 Uterine atony 5 Uterine rupture

uterine atony Explanation The most likely cause of bleeding in this patient is uterine atony. Uterine atony is when the uterine myometrium fails to contract following delivery. Contractions of the uterine wall after delivery normally tamponade bleeding by tightening around uterine arterioles. Absence of this response causes continued bleeding, which usually becomes evident early after delivery. Bimanual uterine palpation revealing a soft, "boggy" uterus is what typically confirms the diagnosis. Risk factors include uterine distension caused by multiple gestations or polyhydramnios, as well as prolonged oxytocin use prior to delivery, placental previa, and chorioamnionitis. Treatment involves uterine massage and the use of medications that promote uterine contractions, including oxytocin, ergot alkaloids, and prostaglandins (e.g., Hemabate). Uterine atony can also occur as a result of retained products of conception; these products inhibit uterine contraction. In this case, manual or surgical extraction is necessary. Lower genitourinary tract or rectal tears may cause bleeding after delivery, the site of which is often visible on pelvic exam and amenable to suture ligation. Neither of these conditions impairs uterine contractions. Placenta accreta describes a condition in which the placenta implants within the uterine myometrium. Although it can be a cause of postpartum hemorrhage, it is relatively less common than uterine atony and would have been recognized at the time of cesarean delivery. Uterine prolapse (also called uterine inversion) describes the collapse of the uterine fundus through the cervical os. The uterus may be seen protruding from the vagina and abdominal exam reveals absence of the uterine body. It is a rare but serious event requiring rapid correction to restore hemodynamic stability. Uterine ruptureis also a rare complication involving a complete tear of the uterine layers with extrusion of uterine content into the abdominal cavity. It most commonly occurs at the site of a uterine scar from prior cesarean section; and it often happens before delivery of the fetus, requiring emergent laparotomy.

A 46-year-old man presents with intermittent, painless rectal bleeding. Occasionally, he also experiences blood-tinged stools. He denies any difficulty passing stools, tenesmus, weight loss, or fatigue. His only past medical history is primary hypertension, which is well controlled with metoprolol, and there are no signs of end-organ damage. His family history is unremarkable. Vital signs are: BP 110 x 70 mmHg, HR 62 bpm, RR 16 bpm, and temperature 36°C (96.8°F). Physical examination, including a rectal examination, shows no abnormalities, except for a painless, purplish lesion in the anal verge. The lesion protrudes when the patient is asked to strain, but it is manually reducible. Basic chemistries and a complete blood count are normal. What is the most appropriate next step? Answer Choices 1 Sclerose the hemorrhoid 2 Perform sigmoidoscopy 3 Reassurance 4 Refer for hemorrhoidectomy 5 Perform a barium enema 6 Topical anesthetics

2 Perform sigmoidoscopy A sigmoidoscopy should be performed. Hemorrhoids consist of an accumulation of hemorrhoidal tissue in the perianal region. Their pathophysiology is related to constant straining and repeated trauma to the anal canal due to prolonged straining, hardened stools, lack of pelvic floor support, and increased intra-abdominal pressure. The high prevalence of these factors in Western societies makes hemorrhoids an extremely common disease, with 1 million cases per year in western civilization. Hemorrhoids can be internal (above the dentate line) or external hemorrhoids (below the line). Internal hemorrhoids have no pain innervation and can be destroyed in an office setting without local anesthesia. Some modalities are sclerosis, banding, cryotherapy, and laser ablation. The following classification is used for hemorrhoids: Grade: Characteristics- I: Enlargement; lesion remains above the dentate line II:Protrusion with straining and spontaneous reduction III: Protrusion with straining and manually reducible IV: Irreducible protrusion The most common symptoms are bleeding, usually described as painless, live bleeding, often seen on wiping; pain (usually dull and aching, caused by vascular engorgement), protrusion, and difficulties with anal hygiene. Thrombosis and strangling of a protruding hemorrhoid by the anal sphincter are very painful and require urgent attention. Some patients bleed persistently and develop iron-deficiency anemia. Hemorrhoids are the most common cause of anal pathology; however, other causes of anal disease, such as rectal prolapse, pelvic floor dysfunction, perianal fistulas, inflammatory bowel disease, psychogenic symptoms, and (in particular) colon cancer, must be actively sought in patients presenting with anal complaints. Painless rectal bleeding should raise suspicion of rectal cancer and screening is mandatory. Endoscopic methods such as sigmoidoscopy and colonoscopy are preferred because they permit biopsy of the lesion to confirm the diagnosis. In younger patients at low risk for colorectal cancer, it is reasonable to first treat the hemorrhoids and performing a further workup if the bleeding persists.

A 42-year-old man presents with a 3-week history of itching and dermatitis. This was followed by a bout of fever and chills, diarrhea, and hepatosplenomegaly. Blood investigations reveal nothing abnormal except eosinophilia. Stool examination reveals eggs of S. mansoni, confirming the suspicion that the person was suffering from schistosomiasis. What is the drug of choice for this patient? Answer Choices: A Praziquantel B Mebendazole C Diethylcarbamazine D Albendazole E Suramin

A

You are treating a 63-year-old man for diabetes (non-insulin dependent), high cholesterol, and hypertension. He is single and works as a clerk in a doctor's office. The patient tells you that, this winter, he has been experiencing episodes of "burning and pressure-type" chest pain 2 or 3 times per week; episodes last for about 3 to 10 minutes, and they are usually relieved by resting. The episodes generally occur after lunch when he takes a "cigarette break with a friend." He had attributed these episodes to cold weather or tension, but he then became really concerned last week when pain occurred while he was climbing stairs. He then became breathless, nauseated, and started sweating. His father died in his forties because of a "heart condition." You conclude that the patient has all risk factors for myocardial ischemia (cigarette smoking, diabetes, high cholesterol, hypertension, sedentary lifestyle, and family history of premature heart disease) and classic provocative factors for angina pectoris (pain after a meal, cold weather, anxiety). You decide to treat him with nitroglycerin, an antianginal drug. This treatment causes vasodilatation by producing what signal molecule? A Nitric oxide B Nitrous oxide C Nitrogen dioxide D Epinephrine E Verapamil

A

You examine the ECG trace on a 47-year-old woman that you suspect may have a myocardial infarction. The P wave looks normal, and the PR interval is shortened. She is experiencing runs of 3 or more PVCs. You understand that the SA node is the pacemaker of the heart, although the A-V nodal fibers, bundles of His, and Purkinje fibers also have an intrinsic rhythmicity. What best describes why the SA node is the heart's natural pacemaker? A It depolarizes and repolarizes at regular intervals unlike those of the A-V node, bundles of His, or the Purkinje fibers. B It sends stronger action potentials, overriding those of the A-V node, bundles or His, and the Purkinje fibers. C The A-V node, bundles of His, and Purkinje fibers only begin to depolarize if the sinus node fails. D The A-V node, bundles of His, and Purkinje fibers depolarize weakly and are not strong enough to initiate contraction of cardiac muscle. E It repolarizes more slowly than the A-V node, bundles of His, or Purkinje fibers.

A

A 15-year-old African-American boy with osteosarcoma is admitted to the Pediatric Hematology-Oncology unit for 12 hours of intravenous fluids prior to starting potentially nephrotoxic chemotherapy. The patient has done well with the therapeutic regimen in the past. He has been resting at home since his last chemotherapy dose and has not required any hospitalizations for fever, vomiting or other illnesses. Although he drinks fluids well, he does not consume much solid food. He supplements his diet with 4 cans of nutritional protein shakes a day. His weight is 22 kg. What hourly IV fluid rate will provide this patient with the maintenance fluids he requires prior to starting his chemotherapy? A 70 cc/hr B 62 cc/hr C 50 cc/hr D 72 cc/hr E 100 cc/hr

A 70 cc/hr *B 62 cc/hr* C 50 cc/hr D 72 cc/hr E 100 cc/hr In order to minimize the potential toxicity of certain chemotherapeutic agents, oncology patients often require hydration prior to beginning therapy. Maintenance fluid rates can be easily calculated using the "4-2-1" rule. Infants weighing 3-10 kg have a maintenance fluid requirement of 100 ml/kg/day. For children weighing >20 kg, the maintenance fluids are 1500 ml/day plus 20 ml/day for each kg of weight above 20 kg. If this daily requirement of 100-50-20- is divided by 24 hours to obtain an hourly rate of fluid requirements, then a 4-2-1 estimate results. In this case, the daily fluid requirement for a 22-kg child would be 1540 ml or an hourly flow rate of (10 kg x 4) + (10 kg x 2) + (2 kg x 1) =62 cc/hr. This rule is applicable for children up to 80 kg.

During her active phase of labor, a provider has placed an external fetal monitor on a patient. The fetal heart rate (FHR) is noted to have a base line rate of 109, there are no late or variable decelerations, and the baseline variability is 0 beats per minute. What is this monitor tracing indicative of? A A normal FHR pattern B An indeterminate FHR pattern C An unreadable FHR pattern D Poor contact with monitor E A distress FHR pattern

A distress FHR pattern According to the Three-Tier Fetal Heart Interpretation System, recommended by the 2008 NICHD workshop on electronic fetal monitoring, the definition of fetal distress includes absence of baseline variability, and either bradycardia (FHR<110) or recurrent variable or late decelerations.

You are monitoring a 30-year-old G2P1 at 40 weeks gestation, who is in an active stage of labor and is 6-cm dilated. The fetal heart tracing has a baseline heart rate of 140, with 7 to 10 beats of variability. With the last five contractions you have noted late decelerations. What would be the next most appropriate course of action? A Close observation of FHR tracing B Assessment of dilatation C Augment contractions with oxytocin D Intravenous analgesic E Surgical intervention

Assessment of dilatation B The presence of recurrent late decelerations should raise the suspicion for fetal distress. Vaginal evaluation for change in dilatation or cord prolapse, and to assess the fetal response to stimulation, are the first steps in evaluating the need for intervention.

A 22-year-old woman presents due to palpitations. She denies chest pain and shortness of breath; she has not had any recent infections. Aside from a tonsillectomy as a child, she has no significant past medical history. Examination reveals a thin woman in no acute distress. Lungs are clear to auscultation bilaterally. Cardiac exam reveals a mid-systolic click. What is the most likely diagnosis? A Mitral valve prolapse B Mitral stenosis C Mitral regurgitation D Aortic stenosis E Aortic regurgitation

A

A 23-year-old man presents with a 7-day history of diarrhea that started 4 days after he came back from a camping trip. It started abruptly, and it was initially watery. The stools are foul-smelling and fatty, and he has associated bloating and abdominal cramps. On physical examination, his blood pressure is 110/75 mmHg and pulse is 78 bpm. He has no signs of dehydration. His abdomen is tender on palpation with increased bowel movement sounds. Stool microscopy shows the presence of flagellated, binucleate trophozoites. What is the most widely used therapeutic agent for this infection? Answer Choices: A Metronidazole B Nitazoxanide C Paromomycin D Tetracycline E Trimethoprim-sulfamethoxazole

A

A 23-year-old woman gives birth to a healthy male infant in the hospital. On postnatal day 6, she begins to experience some abdominal discomfort as well as cramping lower pelvic pain. Suddenly, she begins to bleed profusely from her vagina. A rapid clinical assessment reveals a boggy sub involuted uterus. Her temperature is 37 degrees centigrade; systolic BP is 70. Heart rate is 130, and she appears pale. Her heart and lungs seem normal. With secondary post partum hemorrhage as her working diagnosis, what is the next best step in management? A Resuscitation with fluids B Examination for undiagnosed laceration to birth canal C Administration of uterotonic drugs D Bimanual compression of uterus E Ultrasonography for retained placenta

A

A 36-year-old male soybean farmer presents with the inability to walk. The patient reports having received a puncture wound on his left foot several days earlier; it became infected and bloated. Examination of the foot reveals a deep, highly purulent wound that has necrotized and produces a foul odor. The wound is drained and irrigated, and a wet mount of exudate is examined microscopically. Numerous motile rods containing terminal endospores with a 'tennis racket' appearance are seen. What organism is most likely responsible for the infection? Answer Choices: A Clostridium tetani B Clostridium perfringens C Clostridium difficile D Clostridium botulinum E Bacteroides fragilis

A

A 58-year-old African-American man with type II diabetes mellitus presents due to an elevated blood pressure reading; it was taken 4 weeks ago. The previous reading was 138/88 mm Hg. He was given antihypertensive lifestyle recommendations, and he was advised to follow-up at today's appointment. Today, he denies any symptoms; however, his blood pressure is currently 134/86 mm Hg. What is the most appropriate intervention at this time? A Continue to reinforce lifestyle recommendations B Begin hydrochlorothiazide C Begin enalapril D Start amlodipine E Initiate losartan and hydrochlorothiazide

A

A 58-year-old man with a history of chronic renal failure presents with a 2-week history of night sweats, weight loss, a productive cough, and shortness of breath. The patient had a positive PPD skin test 6 months ago, but he was never treated. You order a chest X-ray and find apical infiltrates. You collect a sputum sample and order an acid-fast stain and culture. The acid-fast stain comes back as acid-fast bacilli seen on smear. Assuming that your patient is HIV-negative, what is the initial phase of therapy? Answer Choices: A Daily isoniazid, rifampin, pyrazinamide, and ethambutol until sensitivities are known B Daily isoniazid, rifampin, and ethambutol C Twice daily isoniazid and rifampin for 9 months D Daily isoniazid, rifampin, pyrazinamide, ethambutol, and supplement riboflavin E Daily rifampin and streptomycin for 3 months

A

A 72-year-old man presents with double vision, weakness, fatigue, nausea, and abdominal discomfort. He describes seeing everything greenish. He suffers from congestive heart failure and is treated for that with digoxin, furosemide, and potassium. He has been taking acetylsalicylic acid since a TIA 2 years ago. He also takes amoxicillin for a urinary tract infection. What medication can cause his symptoms? A Digoxin B Furosemide C Potassium D Acetylsalicylic acid E Amoxicillin

A

A 38-year-old man presents to the emergency department experiencing a severe headache and heart palpitations. He appears to be anxious and perspiring heavily. On exam, he is found to be tachycardic and his blood pressure is 158/102 mm Hg. His urine catecholamines are increased. If imaging were performed, what is the most likely location where a lesion would be found? A pituitary gland B liver C adrenal gland D testicle E kidney

Adrenal gland Pheochromocytomas produce, store, and secrete catecholamines. They are usually derived from the adrenal medulla, although they may be found in other locations.

A 56 year-old woman develops a fever of 102.1 F on her first post-operative surgery day following cholecystectomy. Which of the following is the most likely diagnosis? A Atelectasis B Catheter-related phlebitis C Drug reaction D Urinary tract infection E Wound infection

Atelectasis Post-operative fever that occurs in the first 24 hours is most likely caused by atelectasis (A). Catheter-related phlebitis (B), drug reactions (C) and urinary tract infection (D) typically present between post-operative days 2 and 4, while wound infection (E) typically presents on or after post-operative day 5.

A 24-year-old intravenous drug abuser who is HIV positive presents with a 4-week history of fever, dry cough, tachypnea, and dyspnea. There is a diffuse interstitial pattern on his X-ray (see image). What is the most likely cause of the patient's symptoms? Answer Choices: A Pulmonary emboli B Pneumocystis jiroveci C Kaposi's sarcoma D Legionnaire's disease E Lyme disease

B

A 25-year-old woman presents with intermittent palpitations that are associated with lightheadedness; she admits to a past medical history of having a self-described "hole in her heart". These seem to occur upon significant exertion and when she is "stressed out." She denies chest pain, shortness of breath, wheezing, hemoptysis, cough, syncope, abdominal pain, rashes, peripheral edema, diaphoresis, and vomiting. Her physical exam is remarkable for a mid-to-late systolic click; it is followed by a high-pitched, 'whooping' late systolic crescendo-decrescendo murmur, and it is heard best at the apex. The click occurs earlier with standing and upon Valsalva strain, and it also occurs later in the cardiac cycle with squatting and sustained handgrip. What is the most likely diagnosis? A Aortic stenosis B Mitral valve prolapse C Ventricular septal defect D Aortic regurgitation E Mitral stenosis

B

A 55-year-old man presents with severe central chest pain. Pain started suddenly and it radiates to the back and neck. He is unable to lie flat. He feels sick but has not vomited. He has no major illnesses and knows of none that run in his family. He does not use alcohol, tobacco, or illicit substances. He is allergic to sulfa drugs. On physical exam he appears in extreme pain and is lying on his side. Temperature - 37.0°C, heart rate 110, blood pressure 180/105 mmHg, respiratory rate 20. Cardiac exam reveals normal S1 and S2 without rubs or gallop. The top of his internal jugular venous column is present at 2 to 3 cm above the sterna notch. Chest auscultation shows normal vesicular breathing. He has normal active bowel sounds tympanic to percussion. Extremity exam is normal and the lower motor and sensory function is intact. ECG shows left ventricular hypertrophy. Chest X-ray shows widened mediastinum. What treatment should be given immediately to this patient? A Nitroglycerin spray B Intravenous labetalol C Streptokinase D Angioplasty E Surgery

B

A 58-year-old woman presents with a 3-month history of postprandial abdominal pain. This crampy pain occurs 30 minutes after eating, every time. Due to these symptoms, the patient has lost 30 pounds and is afraid to eat. Her past medical history includes hypertension treated with enalapril, coronary artery disease for which she has undergone a right coronary artery stent, and she underwent a carotid endarterectomy for symptomatic carotid stenosis. She has smoked 2 packs of cigarettes a day for 30 years. What is the best initial test for this patient? A Mesenteric angiogram B Mesenteric duplex ultrasound C Computerized tomography (CT) D Magnetic resonance angiography (MRA) E Computerized tomography angiography (CTA)

B

A 60-year-old man with a history of recurrent sinus infections presents with hemoptysis and hematuria. Physical examination shows a temperature of 101 degrees Fahrenheit, a blood pressure of 145/85 mmHg, decreased breath sounds on his right lower lobe, and palpable purpura on his bilateral lower legs. No warm or swollen joints were noted. ANCA Positive Anti-ds DNA Negative Anti-Glomerular Basement Antibody Negative sed rate 50mm/hour Hgb 11 g/dL Serum creatinine 1.5 mg/dl Serum K:5 meq/l Serum Cl:21 meq/l Urinalysis 1.020, pH 6, numerous red cells, 500 mg albumin/g creatinine, red blood cell casts CXR:Right lower lobe effusion What is the most likely diagnosis? A Systemic lupus erythematosus B Wegener's Granulomatosis C Rheumatoid arthritis D Goodpasture's syndrome E Henoch-Schönlein Purpura (HSP)

B

A 66-year-old man presents with history of recurrent episodes of chest tightness and shortness of breath, especially with exertion. The pain is retrosternal, 6/10 in intensity, lasts for 10 minutes, and radiates to the neck, jaw, and shoulders. On examination, pulse is 65/min and low in volume and BP is 100/80mmHg. On auscultation, a systolic, crescendo-decrescendo murmur is heard at the 2nd right intercostal space and radiates to the carotid arteries bilaterally. What is the most likely diagnosis? A Stable angina B Aortic stenosis C Pericarditis D Acute myocardial infarction (MI) E Aortic dissection

B

An 8-year-old girl presents with a 3-day history of fever, generalized muscle weakness, bilateral knee pain, and chest pain. What illness, contracted about 1 month ago, would support the diagnosis of Acute Rheumatic Fever (ARF)? A Proteus mirabilis infection of the urinary tract B Group A Streptococcus infection of the upper respiratory tract C Helicobacter pylori infection of the gastrointestinal tract D Haemophilus influenza infection of the upper respiratory tract E E. coli infection of the urinary tract

B

In the United States since 2000, outbreaks of measles have most commonly been associated with Answer Choices: A Waxing immunity among elderly who were vaccinated in childhood B Unvaccinated persons who have refused vaccination for personal reasons C Individuals who received a single-dose vs. a two-dose vaccination regimen D Individuals who were vaccination at too young an age for seroconversion E Individuals who were nonresponders to routine vaccination

B

A 17-year-old boy presents to your office with a 2-week history of a red, itchy rash. The rash began on his feet, and over the last 2 weeks it appears to be migrating up his legs. The rash is intensely pruritic. There is no fever, vomiting, diarrhea, cough, weakness, or weight loss. The patient has no significant past medical history. He has just returned from a summer job in Georgia where he was working as a landscaper. There is no travel outside the United States. He denies the use of alcohol and intravenous drug use, but has smoked marijuana occasionally over the summer. On physical exam, there are serpiginous, raised, erythematous lesions streaking up both lower extremities. The rest of his exam is normal. What complications may occur as a result of this boy's infection? Answer Choices: A Meningitis B Peritonitis C Pneumonitis D Seizures E Decreased visual acuity

C

A 52-year-old man is hospitalized for a left lower lobe pneumonia. The patient has a past medical history of HIV and is known to be neutropenic. He also has type 2 diabetes mellitus and diabetic nephropathy. He is started on ciprofloxacin, ceftriaxone, and clindamycin. However, his fever does not resolve, and his blood cultures have been negative. The physician notes a white coating of the tonsils and oropharynx. The patient is able to eat normally. The physician obtains scrapings of the white coating. The sample is observed microscopically and confirms the presence of fungal hyphae and budding. The physician suspects a candidal infection of the oropharynx. What is the next step in patient management? Answer Choices: A Begin intravenous fluconazole immediately B Begin oral amphotericin B immediately C Begin oral fluconazole immediately D Disregard the results of the microscopic examination E Wait for the results of the culture and sensitivity and then treat appropriately

C

A 52-year-old man steps on a piece of glass, and he is seen in your office 1 day later. On exam, his wound appears clean, and it is not infected. He has never had the primary series of tetanus immunization. The patient asks if he needs tetanus immunization. What do you recommend? Answer Choices: A No tetanus immunization is needed since the wound looks clean B A tetanus booster should be given C A complete series of tetanus immunizations (3 doses) plus tetanus immuno globulin (TIG) should be given D Recheck wound in another 24 hours to determine if tetanus immunization is needed E Adult tetanus and diphtheria toxoid should be given

C

A 55-year-old man presents with a headache. On examination, his blood pressure is 200/150 mm Hg. The patient has a history of hypertension and is non-compliant with his medications. Uncontrolled hypertension causes damage to various end organs. What organ(s) is not typically at risk for damage due to hypertension? A Brain B Heart C Liver D Kidney E Brain and liver

C

A 10-year-old boy presents after being bitten by his cat on the dorsum of his left hand several hours ago. He was playing with the cat, and she snapped and bit him on the hand; the bite drew blood. His mother immediately washed the bite out with soap and water, but it has gotten increasingly red and swollen over the past several hours. He is currently afebrile. The dorsum of his left hand reveals a 4 cm oval area of redness that is tender to the touch; there are 4 puncture wounds at the center without pus. You suspect the wound is infected, so a culture is taken. Question: What is the most likely isolate in this case? Answer Choices: A Pseudomonas sp. B Staphylococcus sp. C Streptococcus sp. D Pasteurella sp. E Bacterioides sp.

D

A 38-year-old woman presents for a 3-month follow-up for her HIV infection. She states that she is doing well and adheres to her HIV medication regimen with minimal side effects. She is currently on zidovudine, lamivudine, and efavirenz; she has been on them for the past 5 years, and she has a consistent viral load of <50 copies/mcL and a current CD4 cell count of 306 cells/mcL. She is also on trimethoprim/sulfamethoxazole (Bactrim) for pneumocystis prophylaxis, and she takes insulin for diabetes. She states that she has not been taking her birth control pills, and she took a urine pregnancy test at home yesterday that was positive. A serum pregnancy test confirms her pregnancy. Question: What medication mentioned has the most reproductive risk potential? Answer Choices: A Insulin B Zidovudine C Lamivudine D Efavirenz E Sulfamethoxazole / trimethoprim (Bactrim)

D

A 42-year-old homosexual male presents with rectal pain and mucopurulent discharge. The diagnosis of gonorrhea is made. In this patient diagnosed with rectal gonorrhea, which of the following treatment regimens is recommended? A Ciprofloxacin only B Ceftriaxone only C Ciprofloxacin and azithromycin D Ceftriaxone and azithromycin E Azithromycin and doxycycline

D

A 58-year-old man who is a long standing patient of yours is experiencing worsening symptoms and signs of his dilated cardiomyopathy; symptoms include fatigue, dyspnea with mild exertion, paroxysmal nocturnal dyspnea, severe lower extremity edema, clubbing, an S3 gallop, and jugulovenous distention. Medications that this patient is taking on a daily basis include a β-blocker, adult dose aspirin, and an ACE inhibitor. What intervention would be the most logical next step in helping resolve his current symptoms? A Initiation of a calcium channel blocker B Initiation of an aldosterone antagonist C Initiation of a statin D Initiation of a diuretic E Initiation of an angiotensin II antagonist

D

A 67-year-old man with a prior history of endocarditis and a prosthetic mitral valve presents requesting antibiotic prophylaxis for a dental procedure that entails the extraction of one of his molars. He is able to speak and swallow normally and is presently asymptomatic. His physical exam is remarkable only for a "loudly-clicking S1" in the mitral position. He has a known allergy to penicillin, which results in urticaria and angioedema upon exposure. What is the best intervention concerning this patient? A Amoxicillin 2 grams PO within 1 hour before the procedure B No antibiotic prophylaxis for the dental procedure is necessary C Cephalexin 2 grams PO 1 hour before the procedure D Clarithromycin 500 mg PO 1 hour before the procedure E Clindamycin 600 mg IV or IM 1 hour before the procedure

D

A first year medical student is given an ECG tracing for the first time. He tries to determine a point on the tracing where the ventricles are completely depolarized. Where on the ECG tracing does this take place? A Q B P-R C R D S-T E T

D

A woman presents with her 5th episode of vulvovaginal candidiasis this year. She asks for suppression therapy. What regimen do you recommend? Answer Choices: A Clindamycin 900 mg PO BID x 2 - 4 months B Valacyclovir 500 mg PO daily x 3 months C Metronidazole 250 mg PO daily x 2 - 4 months D Fluconazole 100 mg PO weekly x 6 months E Cephalexin 500 mg 1 tab PO QID x 6 months

D

The newborn nursery calls you to see a 1-day-old male infant. He has developed a worsening mucocutaneous rash over his whole body, especially his palms and soles, which has begun to desquamate. He also has a runny nose. Worrying about possible syphilis, you inquire about the maternal history. What maternal risk factor is particularly associated with congenital syphilis? Answer Choices: A Young maternal age B Domestic violence C Alcohol abuse D Lack of prenatal care E Homelessness

D

A 45-year-old man with a history of neck irradiation for Hodgkin lymphoma at the age of 15 is found to have a 1.5-cm, nontender, firm thyroid nodule. Upon laboratory evaluation, the patient is found to be euthyroid, and fine needle biopsy reveals malignancy. What histologic type is most likely? A anaplastic B follicular C medullary D papillary

D Papillary Thyroid carcinoma often presents as an asymptomatic thyroid nodule. The most common histologic form is papillary carcinoma, representing more than 80% of cases.

A 2-week-old female infant presents for her scheduled newborn visit. The mother notes that the infant has been feeding poorly and seems to have difficulty catching her breath when crying. On examination, a continuous machine-like murmur is heard at the left first intercostal space. What is the most likely diagnosis? A Atrial septal defect B Ventricular septal defect C Coarctation of the aorta D Pulmonary stenosis E Patent ductus arteriosus

E

A 23-year-old man presents with a 2-day history of watery nasal discharge, malaise, sneezing, and nasal congestion. On examination, you notice inflammation of the nasal mucosa; pulse is 80/min; BP is 130/ 80mmHg; and temperature is 98.8 °F. Question: What investigations will you order to establish the diagnosis? Answer Choices: A Nasal swab for polymerase chain reaction (PCR) B Complete blood count C Viral serology D Plain X-ray of sinuses E No specific diagnostic workup needed

E

A 3-year-old boy is admitted to the emergency room in acute respiratory distress. The patient has a body temperature of 40° C, a respiratory rate of 70/min, and a pulse of 130/min. Auscultations of the lungs are unremarkable. An examination of the throat reveals an exudate in the posterior pharynx that is yellowish and membranous. Bleeding occurrs when it is scraped and removed. The parents of the child reveal that the child has no prior immunizations. A throat culture was ordered and worked up specifically for an organism that selectively grows on cystine tellurite agar. What organism is causing this child's disease? Answer Choices: A Respiratory syncytial virus B Bordetella pertussis C Haemophilus influenzae Type b D Streptococcus pyogenes E Corynebacterium diphtheriae

E

A 49-year-old man presents with chest pain. He has had this type of pain in the past, but it typically occurred with significant exertion, such as shoveling snow. Over the past 2 weeks, however, the pain has come on with progressively less activity, and today the pain persists despite rest. Although he has a history of hypertension, he admits to rarely taking his antihypertensive medication. On examination, his HR is 95 beats per minute and his blood pressure is 212/100. An electrocardiogram is performed and reveals ST elevation in the anterolateral leads. What medication would be the most appropriate treatment? A Adenosine B Furosemide C Methyldopa D Nifedipine E Nitroprusside

E

A new test for HIV has been developed. The local community clinic was selected to compare the new test with the Gold standard testing procedure. They found that of the 200 patients tested, 120 were confirmed cases of HIV. The new test detected 120 cases of HIV among the 200 patients as well. Of these, 30 were found to be HIV negative by the gold standard. What is the specificity of the new test? Answer Choices: A 15% B 25% C 75% D 35% E 62.5%

E

A 52-year-old man presents with heartburn, sometimes associated with reflux of sour-tasting material into the mouth. Some of these episodes are accompanied by increased salivation and coughing. He denies any bleeding or abnormalities in his stools. He is slightly overweight and has mild hypertension, which is well controlled with enalapril. Vital signs are within normal ranges and the physical examination is unremarkable. An upper endoscopy reveals coalescing linear erosions throughout the esophageal circumference and a 5.5-cm-long, circumferential cherry-red patch above the gastroesophageal junction. Biopsy of the patch reveals columnar metaplasia, but no dysplasia. What can improve the patient's symptoms most effiiciently? Answer Choices 1 Laparoscopic fundoplication 2 Start omeprazole 3 Esophagectomy 4 Radiofrequency thermal coagulation of the lower esophageal wall 5 Laser ablation of the Barrett's mucosa

Laparoscopic fundoplication Laparoscopic fundoplication leads to long-term relief for about 90% patients with gastroesophageal reflux disease (GERD). The most common technique is the Nissen procedure (i.e., suturing the gastric fundus to the lower esophagus throughout the full esophageal circumference). Side effects are uncommon and mostly minor: abdominal bloating, inability to belch, paraesophageal herniation, dysphagia, and diarrhea. Barrett's esophagus sometimes disappears after fundoplication, but this procedure does not eliminate the risk of dysplasia and carcinoma in all patients. Therefore, endoscopic screening after the procedure is mandatory. The other major therapeutic option for patients with GERD is long-term proton pump inhibitors (PPIs). Patients treated with these drugs have esophageal healing and symptom relief rates comparable to those of operated patients. However, about 30-40% of patients only improve after dose escalation. Therefore, PPI users take longer on average to achieve symptom control. Esophagectomy would be an option if there were an esophageal carcinoma or high-grade dysplasia, particularly in younger patients without significant comorbidities. The decision to operate must take into account the patient's general health and comorbidities, which are common given that Barrett's esophagus often occurs in elderly patients. However, advanced age by itself does not contraindicate esophagectomy. Several endoscopic techniques (e.g. mucosectomy, laser ablation, and photodynamic therapy) have had some success in the treatment of Barrett's esophagus. However, complication such as bleeding after extensive mucosectomies, laser perforations, and skin burns from photoporphyrin-induced photosensibility are common. Incomplete mucosectomies, after which small areas of high-grade dysplasia or even cancers remained in the esophageal wall, have also been documented. Radiofrequency thermal coagulation of the esophageal wall (Stretta procedure) consists of using a special catheter to induce controlled heating of the esophageal wall at the level of the gastroesophageal junction. This induces fibrosis and contraction of the gastroesophageal junction, which increases sphincter pressure and improves reflux. However, recurrence is high after this procedure (about 50%), and it cannot be performed when there is a hiatal hernia, as in this case.

In myasthenia gravis, weakness is a result of insufficient acetylcholine transmission at the neuromuscular junction; however, weakness can also occur with overdosing of the cholinergic medications used to treat myasthenia. What symptom helps differentiate a myasthenic crisis from a cholinergic crisis? Answer Choices 1 Respiratory failure 2 Bilateral ptosis 3 Muscle fasciculations 4 Diplopia 5 Normal muscle stretch reflexes

Muscle Fasiculations Signs of cholinergic overdosage include muscle fasciculation, rhinorrhea, lacrimation, salivation, increased bronchial secretions, nausea, or diarrhea. The presence of any of these suggests that the patient's weakness may be due to cholinergic crisis. The other signs are due to weakness and can occur in either condition.

A 25-year-old gravida 1 woman who is HIV positive arrives at the hospital in early labor. Membranes are intact and the cervix is 50% effaced and 3 to 4 cm dilated. Fetal heart rate is 150 beats/min. Which of the following procedures is contraindicated during labor? A amniotomy B augmentation of labor with oxytocin C external monitoring D operative delivery E use of fetal scalp electrodes

Use of fetal scalp electrodes E Use of fetal scalp electrodes and scalp sampling is contraindicated in the HIV-positive woman because it increases the risk of vertical transmission of the human immunodeficiency virus to the infant. While ruptured membranes for more than 4 hours is associated with an increased risk of vertical transmission, amniotomy per se is not contraindicated. Augmentation of labor, external monitoring, and operative delivery are not contraindicated and, in fact, may be indicated in specific instances for the well-being of the infant and/or mother.

A 23-year-old patient with type 1 diabetes mellitus (DM) has been having difficulty sleeping at night. Usually around 3 am the patient will wake up feeling sweaty, nauseated, and tachycardic. He has recorded the following blood glucose levels: 10 PM- 90 mg/dL 3 AM- 40 mg/dL 7 AM- 200 mg/dL What advise is the best for this patient? A stop eating a bedtime snack B increase the evening regular dosage C decrease the evening Lente dosage D exercise before going to bed at night

decrease the evening Lente dosage

A 42-year-old woman has experienced recent weight gain, heavy periods, fatigue, cold intolerance, and constipation. She has a rough voice, and her rate of speech is slow. Physical exam is significant for an enlarged thyroid, slow reflexes, and the presence of brittle and coarse hair. She denies any history of bipolar disease or treatment with lithium. Laboratory tests show an elevated TSH and low free T 4 . What is the most appropriate treatment for this patient? A propylthiouracil (PTU) B levothyroxine C surgical resection D radioiodide ablation

levothyroxine This patient's signs and symptoms are consistent with hypothyroidism. Treatment of choice is levothyroxine, which is partially converted in the body to T 3 . Significant increases are seen within 1 to 2 weeks, with maximum levels reached in 3 to 4 weeks.

You are providing care for a 21-year-old G1P1 who delivered a 3990 gram infant by normal spontaneous vaginal delivery. After delivery, she continues to bleed vaginally, more than is expected for routine delivery. What is the most likely cause of her hemorrhage? A cervical laceration B lateral vaginal wall laceration C retained placenta D uterine atony E placenta acreata

uterine atony D Uterine atony continues to be the most common cause of postpartum hemorrhage, even though all of the answer choices may cause it.

Which of the following abnormalities corresponds most with smooth endoplasmic reticulum? A Classic congenital adrenal hyperplasia (21-hydroxylase deficiency) B Insufficient mucosal defense in duodenal peptic ulcer disease C Ketone overproduction in untreated diabetes D Corynebacterium diphtheriae infection E N-myc amplification in neuroblastoma

*A Classic congenital adrenal hyperplasia (21-hydroxylase deficiency)* B Insufficient mucosal defense in duodenal peptic ulcer disease C Ketone overproduction in untreated diabetes D Corynebacterium diphtheriae infection E N-myc amplification in neuroblastoma The most common ("classic") form of congenital adrenal hyperplasia is deficiency of the cytochrome P-450 21 hydroxylase. The smooth endoplasmic reticulum is the site of glycogen synthesis, drug detoxification, and steroid synthesis (partially). Cisternae are tubular and not flat and there are no associated ribosomes. Particularly, the mixed-function oxygenase system is located on the smooth endoplasmic reticulum (microsomes). The enzymes include cytochrome P-450 reductase and cytochrome P-450. The oxidation reactions include carbon oxidation (e.g., phenytoin), deamination (e.g., amphetamine), desulfuration (e.g., thialbarbital). This system also metabolizes steroids and fatty acids. Through new protein synthesis, the mixed function oxygenase system can be induced by numerous chemicals, including barbiturates, phenytoin, nicotine, and ethanol. On the other hand, organophosphate cholinesterase inhibitors like parathion and carbon monoxide also inhibit this system. In the pediatric population, ranitidine is preferred instead of cimetidine in the treatment of peptic ulcer disease because it does not depress the hepatic P450 system.

In what childhood disease do ketone bodies reach dangerous levels in untreated cases? A Diabetes mellitus B Medium chain acyl-CoA dehydrogenase deficiency C Von Gierke's disease D Ornithine Transcarbamylase deficiency (OTCD) E Phenylketonuria

*A Diabetes mellitus* B Medium chain acyl-CoA dehydrogenase deficiency C Von Gierke's disease D Ornithine Transcarbamylase deficiency (OTCD) E Phenylketonuria Ketone bodies (acetone, acetoacetate, and beta-hydroxybutyrate) are synthesized in the liver mitochondria and serve as metabolic fuel for other tissues (except liver). Ketoacid synthesis is limited except in fasting and starvation. Ketonuria and ketonemia together are called ketosis. Excessive ketonemia can overload the buffer system and result in ketoacidosis. In untreated diabetes, ketoacidosis can be life threatening as the amount of ketone bodies becomes excessively high.

The deficiency of which of the following intestinal brush border functions will cause milk intolerance? A Lactase B GLUT transport system C Cl-/HCO3- exchanger D Receptor-mediated endocytosis E Neutral brush border (NBB) system

*A Lactase* B GLUT transport system C Cl-/HCO3- exchanger D Receptor-mediated endocytosis E Neutral brush border (NBB) system The major brush border oligosaccharides include lactase (lactoses into glucose and galactose), sucrase (sucrose into fructose and galactose), alpha-dextrinase (cleaves the alpha 1,6 linkages), and glycoamylase (maltooligosaccharides into glucose units). Milk intolerance, a common disorder, is caused be the deficiency of lactase in the brush border. The symptoms are borborygmi, intestinal distension, and diarrhea.

Signs of rickets are evident on the X-ray of a child. Which of the following possible diseases can be confirmed or ruled out by detailed history, serum calcium, and phosphorus? A Nutritional rickets B Vitamin D-dependent rickets C Chronic renal disease D X-linked hypophosphatemia E Chronic anticonvulsant therapy

*A Nutritional rickets* B Vitamin D-dependent rickets C Chronic renal disease D X-linked hypophosphatemia E Chronic anticonvulsant therapy The diagnosis of nutritional rickets heavily depends on the detailed history which can explore many predisposing factors (e.g., decreased exposure to sun, food faddish parents, winter season, overcrowded housing). Both calcium and phosphorus levels are low in the serum.

You transfuse 3 units of whole blood to a gun shot victim. Unknown to you, there was a shortage of blood, and the units you gave to your patient were from the bottom of the freezer and about to expire. Within 2 days, the transfused cells are breaking down, and your patient's ECG is beginning to show high T waves and an increased PR interval. You recognize that he is becoming hyperkalemic and that the excess potassium is affecting the electrical conduction system of the heart. What best describes the resting membrane potential of a sinus nodal fiber under normal conditions? Answer Choices 1 -124 mV 2 -91 mV 3 -85 mV 4 -55 mV 5 -25 mV

-55 mV Normal resting potential for most excitable cells is -70 mV. In order for cells of the SA node to be even more negative (-124 mV), they would either have a greater intracellular concentration of potassium or equally less potassium outside the cell. Increasing the intracellular concentration by that amount would result in severe osmolarity problems. -91 mV is the equilibrium potential for potassium. This value excludes the influence of potassium and other ions on resting membrane potential. To achieve a resting membrane potential of -85 mV, the cells would either have a greater intracellular concentration of potassium or equally less potassium outside the cell. Increasing the intracellular concentration by that amount would result in severe osmolarity problems. This significant decrease in the negativity of the membrane potential is caused by a decrease in permeability of the membrane of these cells to potassium. Therefore, sodium ions have a greater influence on resting membrane potential, bringing it closer to sodium's equilibrium potential (61 mV). If resting membrane potential of the SA nodes reaches -25 mV, the cells would depolarize, but not repolarize. Threshold for these cells is -40 mV. The membrane must return below threshold in order to be able to be depolarized again.

A 23-year-old Caucasian woman presents with weakness, fatigue, and occasional palpitations starting 6 months ago. She took a multivitamin supplement with transient improvement. In the last 3 months, blisters appeared on her elbows, which were associated with intense itching. Her only medication is an implantable contraceptive inserted 3 years ago. There is no relevant past medical history, and her family history is also unremarkable. Vital signs are BP 120 / 80 mmHg, HR 94 bpm, RR 18 rpm temperature 36.3° C (97.2° F). On physical examination, she is alert and oriented, with no dyspnea or cyanosis. Examination of the skin reveals bullous lesions in her elbows. The remainder of the examination shows no abnormalities. A complete blood count shows Hemoglobin 12 g/dL, Hematocrit 31%, Mean Corpuscular Volume (MCV) 78 fl, Mean Corpuscular Hemoglobin Concentration (MCHC) 29 pg/L, Leukocytes 7.400 /mm3 w/normal differential, Platelets 218,000 /mm3. A skin biopsy reveals a neutrophilic infiltrate, with fibrin and microblisters, most dense at the tips of the dermal papillae and containing IgA on immunofluorescence staining. Question What finding is most probable on subsequent workup? Answer Choices 1 Positive transglutaminase antibodies 2 Positive ASCA antibodies 3 A warm spot on scintigraphy with 99mTc-labelled red cells 4 Positive stool guaiac test 5 Reduced serum erythropoietin

1 Positive transglutaminase antibodies Dermatitis herpetiformis produces erythematous, pruriginous bullous lesions on the extensor surfaces of the elbows, knees, scalp, neck, and buttocks. These lesions, sometimes called skin celiac disease, are found in 10% of patients with celiac disease, and are highly specific for it; nearly all patients with dermatitis herpetiformis have histologic evidence of celiac disease on small-bowel biopsy. However, in many cases the enteropathy is subclinical. Histopathologic examination of the bullae reveals deposits of anti-gliadin IgA antibodies in the basal membrane of the epidermis. In most cases, the skin lesions involute after the patient is put on a gluten-free diet. However, this can take several months. In patients with severe pruritus, the mainstay of treatment is dapsone, which is highly effective in relieving this symptom. Second-line options include sulfasalazine and sulfapyridine. Celiac disease can present with isolated nutrient deficiencies (e.g. iron, folate, B-complex vitamins), and therefore could explain this patient's iron deficiency anemia. As transglutaminase antibodies are found in 70% of cases of celiac disease, there is a high probability that this assay will be positive in this patient, whether or not the anemia is caused by celiac disease. The more florid presentation of celiac disease - with overt steatorrhea, weight loss, and malnutrition - is becoming less common in the United States. Therefore, the physician must be aware of the possibility of celiac disease in patients with milder symptoms, such as occasional diarrhea with bulky stools, bloating, or more rarely, isolated micronutrient deficiencies. Anti-Saccharomyces (ASCA) antibodies are associated with Crohn's disease (60-70% of patients are positive). Lesions in these patients include oral aphthous ulcers, pyoderma gangrenosum, and rheumatoid nodules, as well as signs of non-cutaneous disease (e.g. enterocutaneous fistulas, malnutrition, micronutrient deficiencies). A 99mTc-red cell scintigraphy could be positive because occult gastrointestinal bleeding is still a possibility. Causes of a negative iron balance include: 1) Deficient intake, which is most prevalent in poor countries. 2) Pregnancy. 3) Chronic bleeding (gynecologic, gastrointestinal, hematuria). 4) Iron deposition in tissues (e.g. cardiac hemosiderosis) The deficiency in hemoglobin synthesis in the bone marrow tends to cause a compensatory rise in erythropoietin levels, which produces erythroid hyperplasia in the bone marrow. Other findings include absence of sideroblasts and stainable iron in the bone marrow's reticular network. However, bone marrow biopsy has been largely abandoned for diagnosis of iron-deficiency anemia because of interobserver variability and the availability of less invasive tests of iron status, particularly serum ferritin and total iron binding capacity (TIBC).

A women presents to the labor department complaining of contraction every 3 to 4 minutes for the last 3 hours. She is a G1P0 at 40 weeks gestation. Her pregnancy is uncomplicated; her group B strep culture is negative. Physical exam vitals are normal, the baby is cephalic in a +2 station, and the bag of water is intact. The fetal heart monitor reveals fetal heart tones in the 140s with contractions every 3 minutes lasting 45 seconds. Her cervix is 4-cm dilated and 50% effaced. What is the expected rate of cervical dilation? A .5 cm per hour B 1 cm per hour C 1.2 cm per hour D 1.5 cm per hour E 1.7 cm per hour

1.2 cm per hour C This patient is in stage one active labor and is a primigravida. She should expect 1.2 cm of dilatation per hour; multigravida women can expect a faster rate of dilatation.

Mrs. Jones was referred for screening colonoscopy at the age of 50. She has no personal or family history of colorectal cancer. No polyps or lesions were found during the exam. She should be advised that colonoscopy should be repeated in how many years? A 1 year B 2 years C 3 years D 5 years E 10 years

10 yrs E In average-risk individuals aged 50 or greater than 50, screening colonoscopy should be repeated every 10 years following an initial normal exam. If the individual has a first-degree relative with a history of adenomas or colorectal cancer, screening should begin earlier, generally at age 40 or 10 years younger than the age at diagnosis of the youngest affected relative.

BP goal in CKD patients

140/90 • Use a ACEI/ARB if HTN in- 1. DM pts 2. Non-DM with protein/cr ratio of 0.2 mg/mg or 200 mg/g AceI and ARBs decrease proteinuria (watch for hyperkalemia though!)

A 25-year-old woman brings in her menstrual calendar as part of a preconceptional counseling visit. Her cycles are regular, occurring every 30 days and lasting 3-4 days. She has mild cramping on days 1 and 2 that is easily relieved by ibuprofen or acetaminophen. On what day of her cycle is she most likely ovulating? A 12 B 14 C 16 D 18 E 20

16 C Day 1 of menses is the start of a new menstrual cycle. In normally menstruating women, the luteal phase is stable at 14 days, i.e., ovulation ordinarily occurs 14 days before the onset of the next menses. Therefore, in a woman with a very regular 30-day cycle it is most likely to occur on day 16. She may, however, become pregnant if she times intercourse for two days before (B) or after (D) the day of ovulation, due to other factors such as the duration of activity of sperm. Days 12 (A) and 20 (E) are at the margins of the period of fertility, but neither is the day of likely ovulation.

A 25-year-old woman brings in her menstrual calendar as part of a preconceptional counseling visit. Her cycles are regular, occurring every 30 days and lasting 3-4 days. She has mild cramping on days 1 and 2 that is easily relieved by ibuprofen or acetaminophen. On what day of her cycle is she most likely ovulating? A 12 B 14 C 16 D 18 E 20

16 Day 1 of menses is the start of a new menstrual cycle. In normally menstruating women, the luteal phase is stable at 14 days, i.e., ovulation ordinarily occurs 14 days before the onset of the next menses. Therefore, in a woman with a very regular 30-day cycle it is most likely to occur on day 16. She may, however, become pregnant if she times intercourse for two days before (B) or after (D) the day of ovulation, due to other factors such as the duration of activity of sperm. Days 12 (A) and 20 (E) are at the margins of the period of fertility, but neither is the day of likely ovulation.

A newborn boy is evaluated in the delivery room at 1 minute of life after an emergency C-section due to late decelerations. He is limp, pale, and unresponsive; he has a heart rate of 78 and a slow, irregular respiratory rate. What is his Apgar score? Answer Choices 1 0 2 10 3 6 4 2 5 5

2 The correct response is 2. The Apgar score is a system of assessing newborns immediately after birth to help identify those requiring resuscitation, and to predict survival in the newborn period. The 1-minute score may signal the need for immediate resuscitation, and the 5-minute, 10-minute, and subsequent scores may indicate the probability of success. The score is based on the observation of 5 signs including heart rate, respiratory effort, muscle tone, reflex response, and color. Each receives an individual score between 0 and 2. The highest possible score is 10. This patient would receive 1 point each for heart rate and respiratory effort, and no points for muscle tone, reflex response, or color. Immediate resuscitation efforts would be initiated due to the low score.

A 55-year-old man presents with a 12-hour history of severe epigastric abdominal pain that radiates into his back, nausea, vomiting, and chills. His pertinent physical examination findings include oral temperature of 102°F and epigastric tenderness upon palpation. He is admitted to the hospital for management of his condition. Question What criteria would increase the likelihood of a severe attack associated with a high incidence of short and long term morbidity and mortality? Answer Choices 1 White blood cell (WBC) count of 10,000cells/mm3 upon admission 2 A 14% decrease of his hematocrit at 48 hours after admission 3 Serum glucose of 130 mg/dL upon admission 4 Aspartate transaminase of 240 U/dL upon admission 5 Serum calcium of 10 mg/dL 48 hours after admission

2 A 14% decrease of his hematocrit at 48 hours after admissionA patient who is admitted to the hospital for treatment of acute pancreatitis is assessed using the Ranson criteria to determine the severity of their disease, which in turn helps determine prognosis. About 70 - 80% of cases of acute pancreatitis are considered mild and result in virtually no morbidity or mortality. The remainder are severe attacks and have a 10 - 30% mortality rate. Fast identification of the severe cases is helpful to reduce the morbidity and mortality for each patient. There are 11 Ranson criteria, 5 of which are determined upon admission, and 6 at 48 hours after admission. Patients who have 2 or fewer of the criteria have minimal mortality. Patients with 3 - 5 of the criteria have about a 10 - 20% chance of mortality. Patients with 5 or more of the criteria have at least a 50% mortality rate. A patient with at least a 10% decrease in their hematocrit at 48 hours after admission meets one of the Ranson criteria, so the correct answer is a 14% decrease in hematocrit 48 hours after admission. WBC count of 10,000 cells/mm3 is not one of the Ranson criteria. A WBC count of >16,000 cells/mm3 would meet one of the Ranson criteria. Serum glucose of 130 mg/dL upon admission is also not one of the Ranson criteria. The patient's serum glucose would have to be at least 200 mg/dL to count as one of the Ranson criteria and increase the severity of their disease. Aspartate transaminase of 240 U/dL upon admission is not one of the Ranson criteria either. The patient's aspartate transaminase would have to be at least 250 U/dL to count as a Ranson criteria and increase the severity of their disease. A serum calcium of 10 mg/dL 48 hours after admission is not one of the Ranson criteria. If the patient's serum calcium is under 8 mg/dL 58 hours after admission, this would count as one of the Ranson criteria.

Parents of a 6-month-old infant are very worried about a severe reaction that occurred in the infant after the first dose of DTaP. The infant had a high fever (up to 105ºF), was irritable, and was crying incessantly for a few hours following the injection. Parents do not want to give any more doses of DTaP to their child. Question What adverse reaction is a contraindication for giving the next dose of DTaP in this infant? Answer Choices 1 Persistent inconsolable crying for 3 or more hours 2 Encephalopathy (coma or longer or multiple seizures) within 7 days of dose 3 Fever of more than 105º F within 48 hours of dose 4 Seizure within 3 days after dose 5 Collapse or shock like state within 48 hours of dose

2 Encephalopathy (coma or longer or multiple seizures) within 7 days of dose If the infant develops signs of encephalopathy (coma or diminished level of consciousness, prolonged or multiple seizures not attributable to another identifiable cause, within 7 days of giving a dose of DTaP or DTP), it is a contraindication for subsequent doses of DTaP. Acute neurological illness after receiving DPT can have chronic neurological sequelae. The National Childhood Encephalopathy Study (NCES) data also are consistent with a possibility that some children with underlying brain or metabolic abnormality can have acute neurologic illness triggered by DPT, which might go to chronic nervous system dysfunction, which further affects the physical, social, behavioral, and educational outcome of children. Therefore, encephalopathy is a contraindication for subsequent doses of DTaP. If the infant has progressive or unstable neurologic disorder, including infantile spasms, uncontrolled seizures, or progressive encephalopathy, defer vaccination with DTaP or TdaP until the condition is stabilized. If the infant develops a fever of 105º or above following a dose of DTaP, it is not a contraindication for subsequent dose of DTaP. Risk and benefits of the vaccine should be carefully considered. Consider giving acetaminophen before and every 4 hours thereafter for 24 hours after giving the vaccine. Collapse or shock-like state, i.e., hypotonic hyporesponsive episode, within 48 hours after receiving a previous dose of DTP or DTaP is not a contraindication, but it is important to consider the benefits and the risks. If the benefits are believed to outweigh the risk (e.g., during an outbreak or foreign travel), give the vaccine under careful observation in a facility where anaphylaxis can be recognized and managed. Seizure within 3 days of a dose of DTaP is not a contraindication, but needs precaution and a consideration of the risks and the benefits. If the benefits outweigh the risk (e.g., during an outbreak or foreign travel), the vaccine should be given under observation. Persistent inconsolable crying lasting for 3 or more hours occurring within 48 hours of dose is not a contraindication, but it is important consider the benefits and the risks of this vaccine under the circumstances. If the benefits outweigh the risk (e.g., during an outbreak or foreign travel), give the vaccine.

A 45-year-old woman presents with worsening symptoms. She was diagnosed with idiopathic pulmonary hypertension about 2 years ago, and she is on home oxygen therapy. She has longstanding fatigue and dyspnea; she is now experiencing worsening dyspnea with exertion, some swelling in her ankles, and the inability to breathe well when lying down. Although she has always been thin, she notes her weight has increased by 10 pounds in the last month. She denies fever and chills. The patient's vitals are shown in the table. Weight- 152 lbs Height- 66" Pulse- 95 Blood pressure- 132/86 Temperature- 98.4 °F/36.9°C Pulse oximetry- 91% On physical exam, the patient is in no apparent distress; she is using her portable oxygen by nasal cannula. Her ankles have visible edema, and her neck veins are mildly distended. Lung sounds are diminished bilaterally, but no rales or crackles are heard. A gallop is auscultated on her cardiac exam. Question What is the next most appropriate intervention for this patient? Answer Choices 1 Call an ambulance for immediate transport to emergency department 2 Initiate diuretics and refer to a pulmonologist this week 3 Initiate a macrolide antibiotic and cough syrup; follow-up in 2 days 4 Prescribe an albuterol inhaler for use as needed when shortness of breath occurs 5 Send the patient to the hospital for immediate admission

2 Initiate diuretics and refer to a pulmonologist this week This patient with pulmonary hypertension (which may be progressing into cor pulmonale) needs specialist care. It would be most appropriate to initiate diuretics and refer her to a pulmonologist this week. Treatment medications for pulmonary hypertension are complex; they can include endothelin receptor antagonists, phosphodiesterase inhibitors, intravenous prostacyclins, and others. This patient has presented to the outpatient clinic and is stable. If left untreated, her condition may deteriorate, but her current state is not an acute emergency requiring an ambulance for immediate transport to the emergency department. If the patient had pneumonia (you would expect a history of cough, fever, chills, and physical exam findings suggesting infection), it would be appropriate to initiate a macrolide antibiotic and cough syrup as well as follow-up in 2 days. However, this patient's pulmonary condition is not secondary to infection. If the patient's dyspnea was caused by asthma, it would be reasonable to prescribe an albuterol inhaler for use as needed when shortness of breath occurs. However, albuterol (a short-acting beta-agonist) will not address her pulmonary hypertension or the cor pulmonale. As with the discussion above regarding emergency transport, this patient does not present with an urgent or emergent condition that requires inpatient management. If there were indicators that the patient would not be compliant with outpatient care, inpatient admission could be considered. However, this patient describes prior specialist care, and it can be presumed she has been compliant with appointments, testing, and her home oxygen.

A 69-year-old man presents with a 2-hour history of right upper extremity hemiparesis; the hemiparesis resolved upon presentation to the hospital. A cerebral MRI demonstrates an isolated intracranial lesion, which is consistent with metastatic disease. A chest X-ray demonstrates a right hilar mass with right lower lung atelectasis. A CT is shown below that demonstrates right lower and middle lobe atelectasis secondary to an endobronchial cancer without lymphadenopathy. PET scan demonstrates uptake in the right lung and brain alone. Bronchoscopy demonstrates complete occlusion of the right lower lobe secondary to adenocarcinoma. The tumor encroaches upon the right middle lobe but does not occlude the orifice. The patient undergoes resection of the cranial metastasis uneventfully. Question What should the next treatment include? Answer Choices 1 Radiation therapy alone 2 Lung resection including the middle and lower lobe 3 Lung resection including a pneumonectomy 4 Lung resection of the lower lobe only 5 Photodynamic therapy alone

2 Lung resection including the middle and lower lobe Occasionally, patients present with resectable localized lung cancer and evidence of solitary metastasis. These patients should be considered for combined approach in which the primary tumor and solitary metastasis are resected. Locations that are amenable to a combined approach include adrenal, brain, lung, and bone. Most brain metastases are histologically adenocarcinoma in this case. Cranial resection is done first and thoracotomy thereafter, unless symptoms from the primary tumor site dominate the clinical picture. The best surgical treatment for this particular lung cancer is lower and middle lobectomy. Lower lobectomy alone would leave a cancerous middle lobe and offer no survival advantage. Pneumonectomy would be an unnecessary extensive resection in view of the bronchoscope findings. Also, a pneumonectomy would have a higher mortality rate without the benefit of a higher cure rate. For patients who are unable or unwilling to undergo surgery, radiation can be used with curative intent. The long-term survival rate for patients with early stage resectable disease is from 17%-33%. Results of curative photodynamic therapy are usually for microinvasive carcinoma, defined as lesions less than 10 mm. In 1997, a report documented 9 of 21 patients had no evidence of disease at a mean of 68 months. Currently, there is a limited role for photodynamic therapy with primary respectable lung cancer due to higher survival rates with surgical removal.

A 70-year-old man presents with difficulty breathing with a persistent cough and hemoptysis. He had a left radical nephrectomy 8 years ago for clear cell carcinoma of the kidney. See the attached image. The chest X-ray shows several large peripheral and hilar masses along with an interstitial infiltrate, which is characteristic of bronchoalveolar spread of cancer. A noninvasive chest CAT scan is useful to delineate the actual presence and extent of the tumor in the chest and mediastinum. If it appears that there is proximal bronchial involvement by cancer, then bronchoscopy and biopsy would be helpful in confirming the diagnosis. If the bronchoscopy is non-diagnostic, what is the next diagnostic test? Answer Choices 1 Thoracotomy 2 Transthoracic CAT scan guided needle biopsy 3 None. Treat the patient based upon the X-ray findings 4 Positron emission tomography 5 Mediastinoscopy

2 Transthoracic CAT scan guided needle biopsy Generally, in patients with clinical suspicion of recurrent or metastatic cancer, histologic confirmation is obtained in the least invasive manner possible. CAT scan-guided needle biopsy is a common method of obtaining tissue diagnoses without a surgical procedure.

A 38-year-old man presents with sudden onset of acute upper abdominal pain since the previous night associated with nausea, several episodes of vomiting, and weakness. The pain is mostly in the epigastric region with constant, severe, and steady radiation to the back. He also has a low grade fever since this morning without any chills. He denies diarrhea or dysuria. His past history is significant for hypertension, for which he takes amlodipine 10 mg daily and enalapril 5 mg daily. He has smoked half a pack of cigarettes daily for the past 12 years. He initially denies drinking alcohol except on the weekends occasionally, but on further and repeated questioning he says he drinks 3-4 beers daily and had been drinking continuously for the last 2 days with his friends while watching sports on TV. Family history is unremarkable. On examination he has a temperature of 100.6° F, pulse 106/minute, BP 150/92 mm Hg, and respiratory rate is 20/minute. There is no pallor, icterus, cyanosis, or lymphadenopathy. Mucus membranes are dry, and skin is somewhat clammy. Lungs are clear, and heart sounds normal except for sinus tachycardia. Abdominal exam reveals diffuse tenderness in the epigastric and right as well as left upper quadrants. There is some distension and mild guarding in the upper abdomen. Bowel sounds are hypoactive, but there is no ascites or hepatosplenomegaly. Rectal exam is normal. Labs reveal Hb 15g%, WBC 14,500/uL, platelets 400,000/uL, AST 42 U/L, ALT 36 U/L, AP 26 U/L, amylase 3600 U/L, lipase 546 U/L, BUN 25 mg/dL, creatinine, 1.5 mg/dL, bilirubin 1.2 mg/dL, and random blood sugar 110 mg/dL. Question Which of the following is one of the predictors of acute pancreatic necrosis if present at diagnosis along with 2 other factors? Answer Choices 1 Age over 50 years 2 WBC count more than 16,000/uL 3 Blood glucose over 180mg/dL 4 Serum LDH over 300 U/L 5 AST more than 200 U/L

2 WBC count more than 16,000/uL Explanation This patient is suffering from acute alcoholic pancreatitis. The 2 most common causes of acute pancreatitis are gallstones and alcohol. Other causes include hypertriglyceridemia, hypercalcemia, abdominal trauma, ERCP, and drugs like valproic acid, azathioprine, mercaptopurine, didanosine, thiazides, tetracyclines etc. Assessment of severity is done by either Ranson's criteria on admission and at 48 hours or by the Acute Physiology and Chronic Health (APACHE) II scoring system. Ranson's criteria include: (1) Age more that 55 years (2) WBC count more than 16,000/Ul (3) Blood glucose more than 200mg/dL (4) Serum LDH over 350 U/L (5) AST over 250 U/L. 3 or more criteria on admission predict a complicated course with possibility of pancreatic necrosis. At 48 hours development of any of the following indicates a worsening prognosis: (1) Hematocrit drop of more than 10% (2) BUN rise greater than 5 mg/dL (3) Arterial PO2 less than 60 mm hg (4) Serum calcium less than 8 mg/dL (5) Base deficit over 4 meq/L (6) Estimated fluid sequestration of more than 6 L. An elevated C reactive protein at 48 hours suggests the development of pancreatic necrosis. A high amylase and lipase are suggestive of acute pancreatitis. Leukocytosis is usually present. Other than the labs ordered above, imaging may also be done. Plain abdominal X-ray may show radio-opaque gallstones, the sentinel loop sign (localized ileus of a small segment of small intestine, usually in the left upper quadrant), and the colon cut-off sign (lack of air in the colon in the area of the inflamed pancreas immediately preceded by a gas filled segment of transverse colon). There may be a reactive pleural effusion with atelectasis in the lower lobes of the lungs. Ultrasound is non specific and may show gallstones. CT scan of the abdomen will show the inflamed pancreas and detect complications like necrosis or pseudocyst formation. Since Ranson's criteria and APACHE scoring systems are cumbersome and time consuming, a CT Severity Index (CTSI) has recently become popular. It uses a grading system based on unenhanced CT of the pancreas and a necrosis score based on contrast enhanced CT of the pancreas. CTSI is the unenhanced score plus the necrosis score, the maximum of which can be 10 and more than 6 indicates severe disease. MRI and MRCP are being increasingly used where available for diagnosis and management of pancreatitis. Treatment involves bowel rest, aggressive hydration, pain control, and bed rest in a hospital. Electrolyte imbalances should be corrected, especially calcium since saponification may lower the level. Broad spectrum antibiotics are not routinely recommended, but in patients with impending necrotizing pancreatitis, antibiotics have shown to decrease mortality. Nutritional support is of utmost importance. For mild to moderate cases, IV hydration and gentle advancement of oral feeding is recommended, where as in severe cases total parenteral nutrition may be needed. Alcohol abstinence will be needed to prevent further episodes. Complications include prerenal azotemia, acute tubular necrosis, shock, pancreatic necrosis, pseudocyst formation, ARDS, and pancreatic abscess. Intra-abdominal hemorrhage may cause ecchymoses around the umbilicus (Cullen's sign) or in the flanks (Grey-Turner sign), though these are rare and not specific for pancreatitis. Prognosis is good for mild cases who abstain from drinking. Recurrences are common in alcoholics. Prognosis is poor for severe necrotizing pancreatitis, especially with multi-organ involvement.

When is surgery necessary for gastric polyps? Answer Choices 1 For all polyps larger than 2 cm 2 When the question of cancer cannot be completely answered by endoscopic biopsy 3 When the polyp is pedunculated and greater than 2 cm in size 4 When there are multiple polyps 5 For ulcerated bleeding polyps only

2 When the question of cancer cannot be completely answered by endoscopic biopsy Explanation Gastric polyps are often found by barium x-ray studies or at the time of upper endoscopy. Polyps which are small can be easily biopsied, and pedunculated polyps, even those larger than 2 cm in diameter can be excised by use of a snare. Lesions which are sessile do not lend themselves to snare excision endoscopically, and large sessile polyps may harbor cancer not found in the forceps biopsy specimen. When patients have multiple large polyps or have polyposis, it is difficult to excise all the polyps to ensure that none of them are harboring cancer. Bleeding polyps are not an absolute indication for surgical excision. When there is a question as to the presence of cancer, surgery is indicated for resection and possible cure of early gastric carcinoma.

A 6-week-old male with sickle cell disease presents to the pediatric office for his well-child visit. When should this child begin taking daily prophylactic penicillin? A Now B 2 months C 6 months D 12 months E 5 years

2 months Patients with sickle cell disease develop functional asplenia as early as 3 months of age and should begin treatment with prophylactic penicillin at 2 months of age to prevent infection by encapsulated organisms (i.e., pneumococcus). Discontinuation of therapy can be considered beginning about age 5 (E).

A 19-year-old female patient presents to her family practice office for her annual Pap test and her first dose of the quadravalent human papillomavirus (HPV) vaccine. As her physician assistant, when would you schedule her to come in for her second dose of the HPV vaccine? A two weeks B two months C three months D four months E six months

2 months People are frequently confused by differences between the quadravalent and bivalent versions of this vaccine. But dosing schedules are not one of the confusing issues. The Centers for Disease Control and Prevention (CDC), Advisory Committee on Immunization Practices (ACIP), and the manufacturers of both the HPV4 and the HPV2 vaccines all agree: The dosing and administration schedules are the same for HPV4 and HPV2. Each dose is 0.5 mL, administered intramuscularly, preferably in a deltoid muscle. The vaccines are administered in a three-dose schedule. The second dose is administered one to two months after the first dose, and the third dose is administered six months after the first dose.

A patient was recently diagnosed with acromegaly. He was reading on the Internet that acromegaly is associated with diabetes mellitus. He is especially concerned, since his father was recently diagnosed with diabetes as well. How would you reply to the patient's following question: "What percentage of people with acromegaly develop diabetes mellitus?" A 10% B 25% C 50% D 65% E 80%

25% The correct choice is B, 25%. Growth hormone is a counter-regulatory hormone of insulin, and therefore acts against insulin. This can lead to hyperglycemia, glucose intolerance, and diabetes mellitus in 25% of patients with acromegaly.

A 23-year-old woman was diagnosed with myasthenia gravis a few years ago, and she now presents for a routine check-up. She is currently being treated with 15 mg of neostigmine taken orally every 4 hours. In myasthenia, there is autoantibody production against the acetylcholine nicotinic post-synaptic receptors. This results in the reduction of the number of acetylcholine receptors. The nicotinic acetylcholine receptors are composed of a, b, g, and d subunits in the molar ratio of what sequence? Answer Choices 2 : 1 : 1 : 1 3 : 1 : 1 : 1 4 : 1 : 1 : 1 5 : 1 : 1 : 1 6 : 1 : 1 : 1

2:1:1:1 Nerve cells and their target cells abut but do not touch each other. An electrical signal traveling along the nerve cell is transmitted across the gap that is called the synapse. The cell that sends the signal across the synapse is called the presynaptic cell; the cell receiving the signal that traveled across the synapse is called the postsynaptic cell. Receptors on the membrane of the presynaptic cell are called presynaptic receptors; receptors on the postsynaptic cell are called postsynaptic receptors. The electrical signal that traveled along the presynaptic cell is passed across the synapse to the postsynaptic cell as a chemical signal. The chemicals that travel across the synaptic gap are called neurotransmitters. Typically, neurotransmitters are released from vesicles in the presynaptic cells. The vesicles fuse with the presynaptic membrane and release their contents into the cleft. The vesicles are called presynaptic vesicles. In vertebrates, acetylcholine is an excitatory neurotransmitter. The neurotransmitter has a postsynaptic receptor that has been visualized by electron microscopy. This shows that the receptor is composed of 4 different polypeptides subunits labeled a, b, g, and d. These subunits occur in a molar ratio of 2 : 1 : 1 : 1, and they have a total mass of 255,000 daltons. Each of the subunits spans the membrane forming a cylinder with a water-filled channel. In addition, each of the subunits exhibits sequence similarity implying that each arose as a gene duplication and diverged from a single gene.

A 24-year-old man undergoes a routine medical check-up to become a volunteer in the ER. PPD skin test shows a diameter of 9 mm. Sputum and chest X-ray were negative for tuberculosis. He is otherwise healthy and shows no systemic effect of Mycobacterium tuberculosis infection. What is the most appropriate explanation for the doubtful tuberculin test in this patient? Answer Choices 1 Sputum and chest X-ray report could be wrong 2 Adequate sputum samples were not supplied 3 He was given BCG vaccination earlier in life 4 He may suffer from chronic cavitary lesion in the lung 5 He has an active tubercular infection

3 He was given BCG vaccination earlier in life The Mantoux tuberculin skin test (TST) is performed to determine whether the person is infected with Mycobacterium tuberculosis. The TST is performed by injecting 0.1 ml of tuberculin purified protein derivative (PPD) into the inner surface of the forearm and when placed correctly it should produce a wheal of 6 - 10 mm in diameter. The skin test results should be read 48 - 72 hours after administration of the PPD. Generally any palpable induration measuring 10 mm or more is considered a positive reaction. In the case of tuberculosis suspects, or close contacts of individuals with tuberculosis, an induration of 5 mm or more should be interpreted as a positive reaction. Induration measuring 5-9 mm is considered a doubtful reaction. In case of doubtful reaction, the possibility of skin sensitivity due to previous immunization or atypical mycobacterium should be considered. Though prior BCG vaccination increases the risk of a reactive PPD, this effect is known to be inconsistent. Studies have shown that reactions >10 mm should not be attributed to prior BCG vaccine. Positive tuberculin test indicates exposure of the immune system to tuberculous protein, either in the form of BCG vaccineor an active tuberculous infection or a chronic tuberculous infection. Proper vaccination history could have unmasked the cause of doubtful reaction to tuberculin test in this patient. Other causes, such as a wrong sputum and chest X-ray report, inadequate sputum samples, chronic cavitary lesion, or active tubercular infection are not the cause of doubtful reaction in this patient.

A 46-year-old man recently recovered from a bout of influenza, and he now presents due to an 8-hour history of right-sided facial paralysis. He is having trouble closing his right eye, cannot raise his right eyebrow, and cannot smile with the right side of his mouth. Question What medication(s) should this patient be started on? Answer Choices 1 Ibuprofen 800mg PO q 6 hours plus acyclovir 400mg PO 5 times daily 2 Prednisone 60mg PO daily alone 3 Prednisone 60 mg PO daily plus acyclovir 400mg PO 5 times daily 4 Acyclovir 400mg PO 5 times daily alone 5 Prednisone 60mg PO daily plus augmentin 500mg PO 2 times daily

3 Prednisone 60 mg PO daily plus acyclovir 400mg PO 5 times daily The correct answer is prednisone 60mg PO daily plus acyclovir 400mg PO 5 times daily. Either medication alone can be used with fairly good results, but the most recent research has shown that patients treated within 3 days of onset with both a glucocorticoid and an antiviral have a better outcome than patients treated with either prednisone or acyclovir alone. Ibuprofen 800mg PO q 6 hours plus acyclovir 400mg PO 5 times daily is not the correct answer. Acyclovir is an appropriate choice for treatment, as it is an antiviral, and herpes simplex virus is the likely etiologic agent in the pathogenesis of Bell's palsy. However, a nonsteroidal anti-inflammatory is not the correct type of anti-inflammatory to treat Bell's palsy, so ibuprofen is an incorrect choice. Prednisone 60mg PO daily alone is not the correct answer. Prednisone is an appropriate treatment choice for Bell's palsy. It is typically given 1mg/kg for 5-7 days and then tapered over the next 5-7 days. This appears to shorten the duration of symptoms and improve the functional outcome. It should be started as early as possible, but not at all if it has been 7 days or more since the onset of symptoms. However, outcomes are even better if this treatment is used in conjunction with an antiviral, such as acyclovir. Acyclovir 400mg PO 5 times daily alone is not the correct answer. Acyclovir 400mg PO 5 times per day for 10 days is a good treatment option for Bell's palsy. It is best if started within 72 hours of symptoms, but can be started up to 7 days after the onset. However, outcomes are even better if this treatment is used in conjunction with a glucocorticoid, such as prednisone. Prednisone 60mg PO daily plus augmentin 500mg PO 2 times daily is not the correct answer. Prednisone is an appropriate treatment choice for Bell's palsy. It is typically given 1mg/kg for 5-7 days and then tapered over the next 5-7 days. This appears to shorten the duration of symptoms and improve the functional outcome. However, because it is an antibiotic used to treat bacterial infections, augmentin would not be used to treat Bell's palsy. Bell's palsy is caused by a virus, not a bacterium; therefore, an antibiotic would not help the outcome.

A 55-year-old man presents with a 4-hour history of severe left-sided chest pain. His wife tells you that they went out to dinner earlier in the evening, but the patient felt ill shortly after returning home. He had several episodes of vomiting, followed by severe pain. On exam he appears ill; he is cool and diaphoretic. Blood pressure is 90/60, heart rate is 126, respiratory rate is 32, and temperature is 39.8o C. There is no JVD. Cardiac exam is tachycardic, but there is no murmur. Lung sounds are decreased in the left base, with dullness to percussion. The abdominal exam reveals tenderness, but there is no guarding or rebound. Laboratory studies reveal a white blood cell count of 14,000, a hemoglobin of 14 gm/dL, and a hematocrit of 44 gm/dL. EKG reveals a sinus tachycardia without any Q waves. Chest X-ray reveals a left pleural effusion. The described case scenario represents a patient with Boerhaave's syndrome, which is a spontaneous esophageal rupture after forceful vomiting. Once the diagnosis is confirmed, what is the optimal definitive treatment? Answer Choices 1 Intravenous antibiotics 2 Tube thoracostomy 3 Surgical exploration 4 Observation 5 Pulmonary function tests

3 Surgical exploration A gastrografin swallow will usually confirm the diagnosis and delineate the site of rupture. If this is unsuccessful, a barium swallow should follow. It is important to delineate the site of rupture since this will impact the surgical approach. Chest X-ray findings that are more suggestive of the diagnosis include hydropneumothorax, pneumomediastinum, and mediastinal widening. The most definitive treatment is surgical exploration, identification, and repair of the perforation. Since the time frame is relatively short in this case, suture repair with an omental or pleural flap should suffice. Wide chest tube drainage is necessary to prevent postoperative sequelae. Antibiotic therapy should be also initiated, but will never be the definitive treatment alone. The mortality rate from this condition is directly related to the time delay. In patients where the perforation is repaired within 24 hours, the mortality rate is 10 - 15%; the mortality rate is over 50% in patients where the therapy is delayed beyond 24 hours.

What is the therapeutic window for using recombinant tissue plasminogen activator in acute ischemic stroke? Answer Choices 1 1 hour 2 2 hours 3 3 hours 4 4 hours 5 6 hours

3 hours Tissue plasminogen activator (TPA) catalyzes the conversion of plasminogen to plasmin, which promotes fibrinolysis. Ischemic stroke is caused by sudden occlusion of a cerebral artery by thrombus or embolus, producing a focal neurologic deficit. Timely reperfusion of ischemic brain tissue has been shown to limit irreversible neuronal injury in both humans and animals. The benefit of TPA in acute ischemic stroke was demonstrated in a large, multicentered national study. Patients whose symptoms began within 3 hours of treatment with TPA had significantly less disability at the end of 3 months than those given placebo. Although treated patients had higher incidence of secondary brain hemorrhage (6.4% vs. 0.8 %), there was no difference in mortality between the 2 groups. TPA must be given no later than 3 hours of symptom onset in patients with acute ischemic stroke. Prior to treatment, it is essential to exclude cerebral hemorrhage on a head CT scan. The presence of a bleeding diathesis, uncontrolled hypertension, recent prior stroke, and past history of cerebral hemorrhage are important contraindications for using TPA in acute ischemic stroke. TPA given later than 6 hours of stroke onset is associated with unacceptable risk of cerebral hemorrhage; it is not known whether fibrinolytic therapy can be given between 3 and 6 hours of stroke onset.

A 52-year-old Caucasian woman presents with a 2-week history of rectal bleeding. Over the last 3 months, she has become progressively more tired, and could no longer play tennis on weekends. She denies diarrhea, abdominal pain or tenesmus. Her past medical history includes peptic ulcer disease 5 years ago, treated with an anti-H.pylori scheme and which has not recurred. Her vital signs are stable, and the physical examination shows no abnormalities. A CBC shows the following: Ht: 28% Hb: 8.8 mg/dl MCV: 82 fl Leukocytes: 8.100/mm3 w/ normal differential An upper endoscopy is normal except for a duodenal ulcer scar without signs of active ulceration, and a repeat colonoscopy is normal. Question What is most likely to identify the source of bleeding? Answer Choices 1 Celiac angiography 2 Computed tomography of the abdomen 3 Wireless capsule endoscopy 4 Push enteroscopy 5 Biopsy of the healed ulcer bed

3. Wireless capsule endoscopy Wireless capsule endoscopy is performed by ingestion of a battery-powered capsule containing a camera and a transmitter. The capsule transmits images to sensors outside the patient during its transit in the gastrointestinal tract. Indications for capsule endoscopy are still evolving, but several series have shown that capsule endoscopy is more sensitive than small bowel radiography or push enteroscopy for locating the source of small-bowel bleeding. The most common etiologies of bleeding in this region are angioectasia and Crohn's disease. Several other small bowel lesions, such as tumors (leiomyoma, lymphomas, carcinoids, or carcinomas), varices, and polyps, have also been described. An important concern is the possibility of capsule impaction and small bowel obstruction. Some centers are reluctant to perform capsule endoscopy in patients with partial or intermittent obstruction of the small bowel. However, capsule impaction may reveal the site of a lesion and allow its surgical removal, obviating the need for intraoperative enteroscopy. Other relative contraindications are incapacity of cooperating with the examination (e.g. dementia), swallowing disorders, esophageal stricture, gastroparesis, and poor surgical risk. In patients with obstruction or gastroparesis, the examiner may place the capsule endoscopically. This patient has no signs of ulcer activity. Therefore, biopsy of the duodenal scar is not indicated. A 99mTc scan is the test of choice for Meckel's diverticulum, a remnant of the embryonic omphalomesenteric duct found in 1.5% of the general population. Its most common location is the antimesenteric border of the mid-to-distal small bowel. Some diverticula contain ectopic, acid-producing gastric mucosa. Therefore, they can ulcerate and bleed. Rectal bleeding caused by Meckel's diverticulum is often maroon or brick-red in color, and its incidence decreases with age. Therefore, this diagnosis is unlikely in this case. Angiography can be used if other less invasive procedures (i.e. small bowel radiography, push enteroscopy, or small bowel radiography) are non-diagnostic. Computed tomography is not indicated in the absence of other symptoms indicating a specific etiology (e.g. a palpable mass or suspicion of pancreatic disease). Of note, colonoscopy after rapid oral purging has emerged as the procedure of choice for the evaluation of acute lower GI bleeding, and it also provides a means for therapy. Scintigraphy and/or angiography also play important roles in diagnosis and embolization when colonoscopy reveals negative findings or when it is not feasible.

A 32-year-old man presents with a severe headache; he has had 2 similar headaches within the past week. He describes a burning, 'hot poker'-type of pain located primarily behind his right eye. He notes that his eye waters profusely with the headache; in addition, his nose is initially congested, then it starts running. Only his right side is affected. The headache is so severe that he cannot work or sleep through it, and he is unable to concentrate on anything else. The headaches have been unresponsive to over-the-counter pain medications. The episodes seem to last about 1 hour. He denies any other symptoms. This patient has no chronic medical conditions, and he takes no regular medications. Question What is the most likely underlying pathophysiology of this patient's condition? Answer Choices 1 Antigen binds to IgE, triggering the release of histamines and other inflammatory substances 2 Growth of an intracranial tumor near the base of skull, leading to pressure on the cerebellum 3 Increased emotional stress, leading to contractions of head and neck musculature 4 Increased trigeminal nerve and parasympathetic activity, leading to vasodilation 5 Reactivation of varicella zoster virus, leading to inflammation in a ganglion

4 Increased trigeminal nerve and parasympathetic activity, leading to vasodilation This patient presents with a history indicating cluster headaches. While the exact mechanism is unclear, it is known that there is increased trigeminal nerve and parasympathetic activity, leading to vasodilation in the intracranial vasculature. Key features of cluster headaches include unilateral pain, involvement of the eye and nose (autonomic/parasympathetic system), an episodic pattern of attacks, and excruciating pain. Antigen binding to IgE, leading to release of histamines, leukotrienes, and prostaglandins is the typical type I hypersensitivity reaction of allergies. Seasonal allergies could cause a headache, along with nasal and ophthalmic symptoms; however, this patient's severity of headaches exceeds a usual allergic sinus headache. If this patient's symptoms had been caused by an allergic response he would have presented with a history of allergies, chronic lower-grade headaches, and sinus symptoms; he would not have acute, episodic attacks of high intensity. While the growth of an intracranial tumor could present with headaches, this patient's history is not suggestive of cerebellar dysfunction. If there were a tumor in this patient's cerebellar region, some motor, and possibly language, dysfunction would be expected. Headaches with lacrimation and rhinitis are not typically seen from a tumor. Increased emotional stress and tight neck musculature is associated with tension-type headaches, which are the most common form of headaches. Typically, these headaches are more chronic; they are associated with psychosocial stressors and respond to analgesics. Nasal and ophthalmic symptoms are not seen with tension-type headaches. The reactivation of the varicella zoster virus (shingles) can produce severe throbbing, stinging, or other pain symptoms along the affected dermatome. Shingles would be a disorder to consider on this patient's differential, but the patient's young age, lack of skin lesions, and the episodic nature of the attacks fit much better with a diagnosis of cluster headaches.

A 15-year-old female adolescent is brought into the emergency department by her mother with a history of ingesting an unknown number of acetaminophen tablets within the past hour. Neither the mother nor the daughter had any idea how many pills had previously been used or were ingested. The daughter stated she took a lot of pills. The mother stated her daughter had no other medical problems. Both the mother and daughter said that no other medications were being taken. What is the most appropriate time to evaluate an acetaminophen level after the reported ingestion in this patient to determine if therapy is indicated? Answer Choices 1 8 hours 2 4 hours 3 12 hours 4 24 hours 5 2 hours

4 hrs In an acute ingestion, the peak concentration may not be achieved until 4 hoursafter ingestion. Absorption may be affected by coingestants that affect gastric motility. Concretions of multiple tablets ingested at the same time may form bezoars in the stomach and alter absorption, or provide a continuing source of supply. A single acetaminophen level drawn at least 4 hours after ingestion is sufficient to determine patient management. If time of ingestion is unknown, an immediate level and at least 2 additional levels drawn 4 hours apart may be useful. Clinical toxicity may not be evident soon after overdose, and the risk of morbidity increases when the initiation of therapy is delayed. At therapeutic doses, peak concentration is generally achieved after 1 hour, with half-life of 2 to 4 hours. The toxic dose of acetaminophen is approximately 150 mg/kg or 7 grams in adults. Children may tolerate doses up to 200 mg/kg. Acetaminophen is rapidly absorbed from the stomach and small intestine and metabolized by conjugation in the liver to nontoxic agents. These water-soluble conjugates are then eliminated in the urine. Ninety percent are metabolized to either metabolites of sulfate (primarily in children) or glucuronide (primarily in adults). Approximately 4% is metabolized by the cytochrome p450 system to an active metabolite, NAPQI (n-acetyl-p-benzoquinoneimine). Normally, NAPQI is conjugated with glutathione resulting in detoxification and excreted as mercapturic acid and cysteine conjugates. In acute ingestion, glutathione is depleted, so NAPQI covalently binds to vital proteins and the lipid bilayer of hepatocyte membranes. The result is hepatocellular death and centrilobular liver necrosis. Other organs may be affected. The acetaminophen level should be plotted on the Rumack-Matthew nomogram, a semi-logarithmic plot of the acetaminophen level over time (4-24 hours post ingestion) based on adult data. Approximately 60% of patients with levels above the line will develop serum transaminases >1000 IU/L. A second line 25% below the first line was added. Levels between the first and second line are considered possibly toxic and take into account such factors as lab error and error in time of ingestion. There is no clinical evidence suggesting that patients in this range need to be treated. Levels below the second line are low risk. The nomogram is not valid for ingestions less than 4 hours, chronic ingestions, or for patients who have ingested toxic doses of extended release preparations. Because the absorption of extended release preparations is delayed, a measurement at 4 hours will not accurately reflect absorption. The nomogram is also not valid for patients with chronic alcohol consumption.

You are moonlighting in a clinic that provides family planning services for low-income women. The following choices represent the only pertinent historical findings for individual patients. Who would be an appropriate candidate for oral contraceptives? Answer Choices 1 23-year-old Laotian immigrant taking rifampin for tuberculosis 2 25-year-old African-American woman with familial dyslipidemia (HDL 35, LDL 190) 3 35-year-old Caucasian woman with a history of classic migraines with aura 4 36-year-old Caucasian woman who smokes 1 pack of cigarettes per day 5 43-year-old African-American woman taking valproic acid for seizure control

43-year-old African-American woman taking valproic acid for seizure control The correct response is a 43-year-old African-American woman taking valproic acid for seizure control. Some anticonvulsants (phenytoin and carbamazepine for example) decrease steroid levels in women taking birth control pills by inducing hepatic enzymes. It is not clear that this results in ovulation, but caution is advised. This effect is not seen with valproic acid.

A 14-year-old girl presents because she has not yet had her 1st menstrual period; she has no other health problems. Her mother states that her daughter suffers from what seems to be mild mental retardation and attends special education classes. On examination, the girl is 4 feet 5 inches tall, her neck skin shows web-like folds on the side, her breasts are not fully developed, and the nipples are widely separated. No heart murmurs are detected; the lower limb pulse is weaker than that of the upper extremities. What is the most likely karyotype of this girl? Answer Choices 1 45,X 2 47,XYY 3 47,XXY 4 46,XY 5 47,XXX

45, X Explanation Turner syndrome is a leading cause of gonadal (ovarian) dysgenesis. It is caused by sex chromosomal abnormalities resulting in a girl/woman having only 1 X chromosome. The typical karyotype of Turner syndrome is 45,X, but it may also present with mosaic pattern 46,XX, 45,X, and sometimes 46,XX where the 2nd X chromosome is nonfunctional due to an abnormality. Turner syndrome presents in 1 of 2500 live female births. It affects many systems in the body and can be diagnosed prenatally by amniocentesis. It causes short stature, webbed neck, widely separated nipples, and intellectual disabilities. Gonadal dysgenesis leads to amenorrhea; if a mosaic pattern is the case, premature ovarian failure occurs. The gonads are always streak gonads with a small number of ovarian follicles. The pattern 47,XYY is also called super male; it is characterized by intellectual disabilities and large testicles in boys/men. The pattern 47,XXY is Klinefelter syndrome, resulting in a tall boy/man with small gonads and mild intellectual disabilities. The pattern 46,XY is the normal male karyotype. The pattern 47,XXX, is also called super female; it results in tall, relatively normal girls/women who have a lower IQ than their siblings.

A 35-year-old man presents with unusual sensations in his legs. He states that his symptoms began in college and were mild, but they have been steadily worsening over the last 4 years. His symptoms begin whenever he is lying down attempting to go to sleep. He describes the sensation as an uncomfortable crawly sensation deep within his legs. If he tries to ignore them, the feeling worsens. He has brief relief with movement, but the feelings return as soon as he is still again. He was given a trial of a benzodiazepine to take at bedtime, but it did not really help. In addition, he complains of being sleepy during the day as a result of the medication. He also tried leg massages and exercise without much relief. His past medical history is significant for depression and anxiety. The patient denies snoring, vivid dreams, or cataplexy. He states that he has a depressed mood and reduced energy and concentration, but he denies suicidal ideation. His physical exam is normal. Complete blood cell count and chemistry panels are normal. Question What is true regarding the patient's condition? Answer Choices 1 Men are affected more than women 2 Opioids are the drug of choice for this condition 3 The condition is caused by a deficiency in calcium 4 Symptoms usually resolve when underlying depression is successfully treated 5 An abnormal electrical impulse conduction study is frequently seen

5 An abnormal electrical impulse conduction study is frequently seen An abnormal electrical impulse conduction study is frequently seen in restless legs syndrome (RLS), which is a neurological disorder characterized by unpleasant sensations in the legs and an uncontrollable urge to move when at rest in an effort to relieve these feelings. RLS sensations are often described by people as burning, creeping, tugging, or like insects crawling inside the legs. Often called paresthesias or dysesthesias, the sensations range in severity from uncomfortable to painful. Needle electromyography and nerve conduction studies should be considered if polyneuropathy is suspected on clinical grounds, even if results of the neurologic examination are apparently normal. The most distinctive aspect of the condition is that lying down and trying to relax activates the symptoms. Many patients with RLS have difficulty falling asleep and staying asleep. The disorder is diagnosed clinically by evaluating the patient's history and symptoms. Women may be slightly more affected than men. Calcium deficiency causes mainly muscle spasm and twitching, and is usually associated with other symptoms of arm and leg numbness, numbness around the mouth, nervousness, heart palpitations, and an inability to sleep, which are all not found in this case. In most cases, the cause of RLS is not known; in some cases, it may be caused by iron deficiency, folic acid deficiency, vitamin B12 deficiency, or arthritis. Sometimes, RLS can be associated with the use of antidepressant medications, necessitating the use of an alternative treatment.

A 12-year-old girl presents with a 3-day history of progressive dysarthria, dysphagia, and weakness. The patient was well until 3 days prior to admission to the hospital; at that time, she developed the onset and subsequent gradual worsening of dysarthria. She attributed the dysarthria to a sore throat that she had had about 2 weeks earlier. 3 days prior to admission, she also had the onset of mild dysphagia; it mostly occurred with liquids. 24 hours prior to admission, she developed weakness in both upper extremities, which increased and began to involve the lower extremities. This limb weakness was neither worsened by activity nor improved by rest. She also developed tingling in her toes 24 hours prior to presentation. When she became unable to walk without assistance on the day of admission, she decided to seek medical attention and was admitted to the hospital. Past medical history is significant for measles and mumps. Because of family religious beliefs, she has not had any immunizations. She is very athletic, and frequently plays soccer with friends and siblings in the fields on her grandfather's horse farm. Physical examination reveals a well-developed, well-nourished girl. She is awake, alert, cooperative, and in no acute distress. Temperature is 98.7 F by mouth, blood pressure of 140/80 mm Hg, heart rate is 84/min and regular, and respirations are 22/min and unlabored. There are multiple scratches and abrasions in varying stages of healing over most of her extremities. Her speech is moderately dysarthric. She experiences some mild choking when she tries to drink a glass of water. She can smile weakly, but she cannot raise her eyebrows against resistance. She shows mild bilateral weakness of eye adduction. Pupillary responses are normal. There is mild to moderate upper extremity and mild lower extremity weakness, greater distally than proximally. Her motor strength is sustained over at least 30 seconds without fatigue. Her gait is ataxic, and she cannot walk without assistance. Reflexes are hypoactive to absent, and the response to plantar stimulation is downgoing bilaterally. Sensation is intact, except for mildly impaired position and vibratory sensation in both feet. A complete blood count, chemistry profile, chest X-ray, and EKG are all normal. Computed tomography of the brain, with and without contrast, is negative. A nerve conduction study reveals a moderate degree of mostly motor demyelinating peripheral neuropathy, highly suggestive of Guillain-Barre. Question What statement best describes the patient's prognosis? Answer Choices 1 With a predominantly demyelinating rather than axonal neuropathy, her prognosis is especially bad 2 Mortality is 25-30% 3 Full functional recovery is expected in 45% in several months to a year 4 Whether recovery is full or partial, relapses do not occur during the recovery phase 5 Her rapidly evolving clinical course indicates a poor prognosis

5 Her rapidly evolving clinical course indicates a poor prognosis This patient must be watched very closely for the very real possibility of respiratory failure and the need for ventilatory support (2). Mortality is expected to be less than 5% with good medical support (1). With a demyelinating pattern on EMG, her prognosis is better. A consistent indicator of residual muscle weakness is an EMG pattern of axonal damage, with the more severe degrees of damage suggesting the worse prognosis (1). About 85% of patients with GBS have a full functional recovery within a year; however, some may be left with minor residuals such as areflexia on exam (2). Between 5-10% of patients with Guillain-Barre have 1 or more relapses; these cases are referred to as chronic inflammatory demyelinating peripheral neuropathy (CIDP) (1).

A 31-year-old HIV-positive woman presents for ongoing care. She was diagnosed with HIV 2 years ago, and she began antiretroviral therapy. Her CD4 T cell count is 400 cells/mL, and she has a history of oral candidiasis. As part of evaluation, her physician decides to perform a tuberculin skin test (TST) using 5 TU of purified protein derivative (PPD). The test site is examined 48 hours later and the skin reaction is measured. What is the minimum diameter of induration at which this test result should be considered positive in this patient? Answer Choices 1 2 mm 2 5 mm 3 10 mm 4 15 mm 5 20 mm

5 mm Persons with HIV should be tested yearly for tuberculosis using the purified protein derivative (PPD) skin test, also referred to as the Mantoux test. In those with HIV, and in certain other cases (refer to the table), an induration of more than, or equal to 5mm, is considered positive. Preventive therapy should be prescribed for all patients having a positive PPD. Those with a positive skin test or high-risk exposure should undergo prophylaxis. A common regimen consists of isoniazid (INH) and pyridoxine daily, usually for at least 1 year. CDC recommends INH (300mg/day) for a period of 12 months to any HIV-infected persons with positive TST (> 5 mm), along with supplemental pyridoxine (25 - 50mg/day), to prevent peripheral neuropathy. Both isoniazid-resistant and multidrug-resistant strains of Mycobacterium tuberculosis are becoming more prevalent. Classifying positive TST reactions: Interpretation of the Tuberculin Skin Test* Induration (dia.) Positive in Persons With > 5mm - HIV infection - People with chest X-ray findings consistent with prior TB - Close contacts of a person with infectious TB - Patients with organ transplant and other immunosuppressed persons > 10mm - Medical risk factors such as chronic renal disease, diabetes, gastrectomy, and silicosis - Residents/employees of high-risk congregate settings (jails, nursing homes, hospitals and other long-term facilities for elderly) - IV drug users - Mycobacteriology laboratory personnel > 15mm - Healthy persons without known risk factors * 5 TU PPD

In taking the family history of your 25-year-old male patient, you discover that he has numerous relatives with breast and ovarian cancers. In the past, his mother received genetic counseling and testing for the BRCA1 and BRCA2 gene mutations and was found to be positive for a mutant allele. What is his risk for developing this genetic cancer? A His chance is 0%, because this is not transmitted to men. B 25% C 50% D 75% E 100%

50%

You are caring for a pregnant woman who is Rh-negative. The father of the child is Rh-positive and heterozygous. What percent chance will the fetus have of being Rh-positive? A 0% B 25% C 50% D 75% E 100%

50% C The mother is negative and has no genetically positive material to pass to the child; the father is heterozygous, so only half of his genetics contain the positive antigens.

A 62-year-old woman has an unexplained weight loss. She has a vague, non-descriptive pain in her stomach accompanied by diarrhea. When her son sees that she has become yellow, he insists that she seek medical attention. Numerous tests are done, including endoscopic retrograde pancreatography, and she is told that she has pancreatic cancer. The tests reveal that the pancreatic cancer is located at the head of the pancreas. Question What percentage of pancreatic cancers are located at the head of the pancreas? Answer Choices 1 1-10 2 20-30 3 45-55 4 70-80 5 90-99

70-80 About 70-80% of pancreatic cancers are located at the head of the pancreas. Despite the fact that pancreatic tumors that arise in the head of the pancreas may present with obstructive symptoms, the tumor still is usually advanced by the time of diagnosis. Other pancreatic cancers are located in the body and tail of the pancreas. Tumors that arise in the tail of the pancreas are even more insidious.

At a yearly history and physical examination of a male patient, digital rectal exam reveals an enlarged prostate. You would more likely suspect benign prostatic hypertrophy, rather than prostate cancer, if the patient's history also included what presentation? Answer Choices 1 68-year-old with a free prostate specific antigen of 5% 2 48-year-old African American who is asymptomatic 3 52-year-old with a prostate specific antigen of 10 ng/mL 4 62-year-old with a brother treated for prostate cancer 5 72-year-old Caucasian who complains of a poor urinary stream

72-year-old Caucasian who complains of a poor urinary stream Explanation The male patient most likely to have benign prostatic hypertrophy (BPH) is the 72-year-old Caucasian who complains of a poor urinary stream. BPH occurs in 90% of men >70 years old. It occurs in the periurethral zone of the prostate and usually presents with lower urinary symptoms (LUTS) that suggest obstruction (i.e. hesitancy, weak stream, intermittent stream, straining, incomplete emptying, post-void leaking) or irritation (i.e. nocturia, frequency, urgency). Risk factors for prostate cancer include men >50 years old or African American men >45 years old or a first-degree relative with prostate cancer. Prostate cancer most often develops in the peripheral zone of the prostate. Early prostate cancer is usually asymptomatic, but locally advanced prostate cancer may encroach on the central transition zone of the prostate and present with irritative urinary symptoms (i.e. nocturia, frequency, urgency). Laboratory findings suggestive of prostate cancer/BPH include: An elevated prostate specific antigen (PSA) >4 ng/mL: sensitivity of this value for prostate cancer is 57-79% In BPH, level of PSA is generally below 10 ng/dl, A rise in PSA >0.75 ng/mL per year would suggest prostate cancer

What is the most appropriate age to administer the pneumococcal vaccine in healthy individuals with no other comorbid diseases? A Under age 16 B After age 16-34 C > Age 35 D > Age 50 E > Age 65

> Age 65 E The Advisory Committee on Immunization Practices of the Centers for Disease Control and Prevention recommend that the most appropriate age to administer the pneumococcal vaccine in healthy individuals with no other comorbid diseases is over the age of 65.

A 13-year-old boy goes camping with his Boy Scout troop over Memorial Day weekend. Unbeknownst to him, a tick attaches itself to his scalp. A few days after returning home, he suddenly develops a bad headache, nausea, and muscle aches accompanied by a high fever. 3 days later, a patchy rash develops on his wrists, hands, ankles, and feet. He also complains of joint pain. 3 days later, the rash spreads to his trunk. What is the classification of the agent that caused his disease? Answer Choices: A Rickettsia B Spirochete C Gram-negative rod D Chlamydia E Mycobacteria

A

A 16-year-old girl was admitted to the hospital 2 days ago with the diagnosis of fever, the cause of which is not known. She has a 1-week history of fever, chills, headache, anorexia, nausea, vomiting, and myalgia. She developed a rash yesterday. She denies abdominal pain or diarrhea. She has had no previous medical problems, hospitalizations, or surgeries. The week prior to the onset of symptoms, the patient went camping in Texas with her church youth group. No one else from the group is ill. Upon admission to the hospital, her initial laboratory results revealed leukopenia, anemia, and elevated liver enzymes. While performing a physical exam, you unexpectedly discover a tick embedded in the girl's scalp. After sending the appropriate diagnostic studies to the laboratory, what medication should be started empirically? Answer Choices: A Doxycycline B Chloramphenicol C Gentamicin D Clindamycin E Penicillin

A

A 17-year-old boy presents with pain in his wrists, elbows, and knees bilaterally. He has felt fatigued, and he has been unable to work his summer job as a cashier and bagger in his family's community grocery store for the past 2 weeks. He also reports intermittent fevers and a large rash on his back in the area of his right shoulder; the rash is depicted in the image. All of these symptoms have emerged in the last 4 weeks, after a weeklong backpacking trip in upper state New York. He has no significant past medical history. His only medication is acetaminophen daily for joint pain. He does not smoke tobacco; he does not drink alcohol or use illicit drugs. He has no known allergies. Physical examination reveals a thin male adolescent in no acute distress. Temp 99.1°F, P 100 bpm, RR 14, BP 120/70 mm Hg. Small, nontender, mobile lymph nodes are palpable in the neck and axilla bilaterally. There is a large, warm, erythematous patch with central clearing at the patient's posterior right shoulder region; it extends across the arm and axilla and measures approximately 25 cm in diameter. There is limited range of motion in his right wrist and left elbow. There were no gross focal neurologic deficits. Question: What is the most likely cause of these symptoms? picture of a bullseye rash Answer Choices: A Lyme disease B Pityriasis rosea C Pityriasis versicolor D Rocky Mountain spotted fever (RMSF) E Secondary syphilis

A

A 17-year-old girl presents with her boyfriend due to vaginal discharge. She has not had any previous episodes and she states that she has been in a monogamous relationship with her boyfriend. She takes the oral contraceptive pill. The couple does not regularly use condoms. Her last menstrual period was 2 weeks ago. When reviewing her charts, you notice that she has not showed for several of her appointments in the past. On physical exam, she is comfortable. Her temperature is 98 degrees Fahrenheit (36 Celsius), blood pressure of 120/80 mmHg, pulse 72/minute and respirations of 16/minute. You do a speculum examination that confirms a mucopurulent discharge. You obtain a cervical swab. The manual exam does not demonstrate cervical motion tenderness or adnexal tenderness. The cervical swab is sent for a polymerase chain reaction analysis, which returns positive for chlamydia trachomatis. At this stage, what is appropriate therapy for this patient? Answer Choices: A Azithromycin 1 gram in a single dose B Doxycycline 100mg oral twice daily for 7 days C Erythromycin 500mg 4 times daily for 7 days D Metronidazole vaginal gel, twice daily for 5 days E Treatment of her partner and counseling about condom use

A

A 18-year-old previously healthy woman presents with low-grade fever, chest pain, dry cough, wheezing, and intermittent abdominal pain; symptoms started 1 week ago during her visit to the relatives in the rural Alabama. Aspirin has not helped. Today, her vitals are a blood pressure of 110/65mmHg, a pulse of 80 beats/min, a respiratory rate of 15 breaths/min, and a temperature of 37.3 C. Auscultation reveals bilateral diffuse crackles, and the rest of the examination is normal. Her CBC is below: Findings Normal Leucocytes 10, 500 cells/mcL 3,500 to 10,500 cells/mcL Neutrophils 50% 40 - 80% Eosinophils 15% 1 - 6% Basophils 0.4% < 1 - 2% Lymphocytes 27% 20 - 40% Monocytes 5.3% 2 - 10% IgE 230 <88 IU/mL Metabolic panel and liver function tests are within normal limits. The chest X-ray shows bilateral patchy shadows in her upper lung fields. Immunological studies, including ANA, ANCA, and rheumatoid factors are all negative; stool cultures for eggs, cysts and parasites are negative. Serology for viruses (CMV, EBV, HSV, influenza, adenovirus, echovirus, and Coxsackie virus) is also negative. Question: What causative treatment of her condition should you consider? Answer Choices: A Benzimidazole B Corticosteroids C Macrolides D Monoclonal anti-IgE antibodies E Histamine H2-receptor antagonist

A

A 19-year-old female college student starts to feel weak and tired. She becomes ill with a fever that is accompanied by a sore throat. After a week, she is seen by the student health services. She denies runny nose or nasal congestion. She denies sputum production or cough. On physical examination, she is noted to be febrile and have cervical lymphadenopathy. Her oral cavity and pharynx are slightly edematous. Her spleen is palpable. Blood is drawn, and her laboratory results are as follows: TEST RESULTS REFERENCE RANGE Hematocrit 42 40 - 50 (male) 36 - 44 (female) Hemoglobin 14 gm/dL 13.8 - 17.2 gm/dL (male) 12.1 - 15.1 gm/dL (female) RBC 4.8 x 106/µl 4.5 - 5.7 x 106/µl (male) 3.9 - 5.0 x 106/µl (female) Platelets 250,000/mm3 130,000 - 400,000/mm3 MCV 92 cu µ 80-100 cu µ WBC 15.4 x 103/mm3 4.0 - 11 x 103/mm3 Neutrophils 41 40 - 75% Lymphocytes 56 15 - 45% Monocytes 2 1 - 10% Eosinophils 1 1 - 6% Basophils 0 0 - 2% Comments: Numerous atypical lymphocytes are seen. Question: What is the most likely diagnosis? Answer Choices: A Infectious mononucleosis B Common cold C Strep throat D Thrush E Hand-foot-mouth disease

A

A 20-year-old college student spent his summer working in the forest on eastern shore of Maryland. A month after returning home to Virginia, he experienced a sudden onset of fever, nausea, muscle aches, and headaches. Blood and stool cultures were negative for obvious bacterial and parasitic pathogens. His symptoms subsided and he assumed he had recovered. Two weeks later, the symptoms reappeared, now accompanied by marked splenomegaly. He also noticed a single, spreading rash with a pale halo surrounding a brighter red rash (Figure K1.3) on the back of his right shoulder which increased in size and sensitivity. This time a blood smear showed the presence of spirochetes. The etiologic agent responsible for the infection was MOST likely acquired by which of the following methods? Answer Choices: A a tick bite B drinking contaminated water C sexual contact D consumption of contaminated foods E inhalation (Image of bulls eye rash)

A

A 20-year-old man presents with a 2-week history of fever. He states the fever has been as high as 104.5° and it goes down with acetaminophen (Tylenol). He has no other symptoms. His past medical history is noncontributory; family history and social history are noncontributory. He states that he has been with his friends (1 of whom has strep throat), but he does not have throat pain, headache, or stiff neck. He went hot tubbing with friends 7 days ago. His labs are all within normal range. What is the most likely diagnosis? Answer Choices: A FUO (fever of unknown origin) B Sinusitis C Measles D Infectious mononucleosis E Tuberculosis

A

A 22-year-old male off-shore medical student presents with a 2-day history of dysuria accompanied by a painful genital lesion that has formed into an ulcer. He also reports fever with a recorded oral temperature of 101°F, generalized malaise, myalgias, and headache. He denies lesion blistering, and there is no previous history of this condition. He has otherwise been in good general health. He is heterosexual and admits to multiple sexual partners in the recent past with inconsistent use of condoms. T=100.9°F; BP=130/80; P=70; RR=12. Pertinent findings on the physical examination are relegated to the genital exam. Exam reveals a non-circumcised penis with a non-indurated ulcer of the penile frenulum and foreskin, sharply defined borders, undermined edges, and purulent yellow-gray exudative discharge. There is bilaterally tender inguinal adenopathy. Testicles are descended bilaterally with no lesions or masses. Question: What is the most likely diagnosis? Answer Choices: A Chancroid B Granuloma inguinale C Herpes simplex infection D Lymphogranuloma venereum E Primary syphilis

A

A 23-year-old man presents with a 2-day history of burning urine. He also reports a slight purulent urethral discharge. He denies any fever, malaise, or chills. He smokes 1 pack of cigarettes daily and drinks socially; he has multiple sexual partners. On exam, his vitals are normal, and lungs are clear; abdominal exam is unremarkable, without any renal angle or suprapubic tenderness, and external genitals reveal only slight urethral discharge. Labs show WBC of 6500/uL, and urinalysis has 5 - 10 WBC and 0 RBC. Gram stain of the urethral discharge shows neutrophils and intracellular Gram-negative diplococci. Question: What is the best treatment regimen for this patient? Answer Choices: A Ceftriaxone 125 mg IM and azithromycin 1gm PO B Ofloxacin 400 mg BID for 7 days C Trimethoprim-sulfamethoxazole for 3 days D Doxycycline 100 mg daily for 7 days E Ceftriaxone 125 mg IM single dose

A

A 23-year-old man presents with a 2-day history of watery nasal discharge, cough, sore throat, and hoarseness of voice. On examination, pulse is 80/min; BP is 130/ 24mmHg; and temperature is 98.8 °F. Direct laryngoscopy shows laryngeal edema and erythema. The rest of the examination is normal. Question: How will you treat this patient? Answer Choices: A Humidification and voice rest B Antibiotics + non-steroidal anti inflammatory drugs (NSAIDs) C Antiviral drugs + non-steroidal anti inflammatory drugs (NSAIDs) D Antibiotics alone E Antiviral drugs alone

A

A 24-year-old man presents with an 8-month history of a chronic, ongoing, mildly productive cough. When further questioned, the patient notes having an intermittent fever during this time period. A 5-pound unintentional weight loss is also discovered. He also states he has noted an increased amount of breathlessness with just simple activities; the breathlessness was never bothersome before the cough began. The patient denies smoking, and both his HIV and TB tests are negative. Question: What pharmacotherapy should be initiated in this patient? Answer Choices: A Isoniazid B Azithromycin C Pyrazinamide D Erythromycin E Ampicillin

A

A 24-year-old woman presents for follow-up 7 days after medical abortion with mifepristone and misoprostol. She complains of fever, chills, and abdominal pain. Examination shows a temperature of 38° C, tachycardia, slight vaginal bleeding, and sanguinopurulent cervical discharge. Blood cultures are negative, and preexisting Chlamydia trachomatis is suspected. What is the best course of action for this patient? Answer Choices: A Azithromycin with ceftriaxone B Curettage C High-dose parenteral penicillin D 15-methyl prostaglandin F2α E Prostaglandin suppositories

A

A 24-year-old woman presents with dry mouth and blurred vision associated with nausea, malaise, and the recent onset of diplopia. You find out that 2 days ago she had been eating home-canned foods at her brother's house. Previously, she had been healthy and had no medical problems. On physical exam, you confirm slight ptosis and a slight disconjugate gaze. The latest lab work is as follows: Sodium 139 mmol/L Potassium 5.0 mmol/L Chloride 99 mmol/L Bicarbonate 24 mmol/L Blood Urea Nitrogen 7 mg/dL Creatinine 0.7 mg/dL Calcium 7.7 mg/dL What test would be appropriate in this case? A Detection of toxin of Clostridium botulism in the patient's serum B Doxycycline challenge for tick paralysis C Lumbar puncture to rule out a cerebrovascular event D MRI to rule-out Guillain-Barré syndrome E Psychiatric consult to evaluate hysteria

A

A 24-year-old woman with a past medical history of mild scoliosis presents with palpitations, occasional chest pain, and dizziness upon standing from a supine position. Her symptoms have been ongoing for a while, but the patient's parents finally convinced her to come be evaluated. On cardiac auscultation, a mobile mid-to-late systolic click and a late systolic murmur heard best at the apex is noted; no other abnormalities are found. Blood pressure was 112/68 mm/Hg. What is the most likely diagnosis? A Mitral valve prolapse B Aortic stenosis C Dehydration D Atrial myxoma E Atrial fibrillation

A

A 25-year-old African-American man presents with blood pressure issues. He states that he attended a work-sponsored health fair; he had his blood pressure taken, and the health fair representative strongly urged him to make an appointment to be seen by his primary care provider. The fair occurred 3 months ago. He has randomly checked his blood pressure at different times since the health fair in local grocery stores, and although he does not remember the numbers, he knows that "they were above normal". The patient also believes that both his mother and father are taking blood pressure medication, but he is not 100% sure. Vitals at this time reveal a BMI of 30 kg/m2 and a waist circumference of 41 inches; blood pressure in the left arm is 175/95 mm Hg, and in the right arm it is 172/99 mm Hg. Based upon the most likely diagnosis at this time, what clinical pharmaceutical intervention should be initiated along with the appropriate lifestyle modifications? A Hydrochlorothiazide B Metoprolol C Enalapril D Losartan E Aliskiren

A

A 25-year-old man, who claims to have never been acutely ill in his life, presents after developing an abrupt onset of fever, headache, and painful body aches. The patient is now developing a cough and sore throat. The cough is non-productive. It is the middle of January and his travel history is unremarkable, except for a ski trip within the state. There is no history of any insect or arthropod bite. The physical examination is notable for small, tender cervical lymph nodes, an elevated body temperature of 39° C, and red, watery eyes. Transient scattered rhonchi or localized areas of rales are noted. A direct fluorescent antibody test is ordered stat and comes back positive for the suspected etiological agent. Based on the abrupt onset, clinical findings, and laboratory result, what is the diagnosis? Answer Choices: A Influenza virus B Mycoplasma pneumoniae C Respiratory syncytial virus D Streptococcus pneumoniae E Staphylococcus aureus

A

A 25-year-old sexually active man presents with a 2-week history of a painless sore on his penis. On questioning, he denies any discharge or pain with urination. On physical examination, the sore is still present. He is afebrile and is found to have palpable inguinal lymph nodes. The lymph nodes are enlarged, but painless. Dark-field examination results are positive. Question: What is the most likely diagnosis? Answer Choices: A Syphilis B Condyloma acuminatum C Herpes genitalis D Chancroid E Gonorrhea

A

A 25-year-old woman had an episode of angina. On physical exam you notice a systolic heart murmur that is best heard at the 2nd right intercostal space. Valsalva, standing, and handgrip maneuvers decrease the intensity of the murmur. A transesophageal echocardiogram reveals a bicuspid aortic valve among other findings. What adaptive response is most likely to be present in the left ventricular wall muscle of this patient's heart? A Hypertrophy B Hyperplasia C Fibrosis D Metaplasia E Atrophy

A

A 25-year-old woman with a history of a childhood murmur has chronic exertional dyspnea associated with intermittent chest pain, hemoptysis, and lightheadedness. She denies smoking, a history of travel, medication usage, fever, chills, cough, palpitations, abdominal pain, and peripheral edema. Her physical exam reveals central cyanosis, a right ventricular parasternal heave, JVD of 4 cm, and an accentuated P2 heart sound. A loud, harsh holosystolic murmur in the left third and fourth interspaces was present along the sternum, with a systolic thrill. An electrocardiogram noted right atrial enlargement, RVH, and right axis deviation. A bedside echocardiogram confirmed RVH, right atrial enlargement, and tricuspid regurgitation with a bidirectional ventricular shunt. What health maintenance recommendations should be made to this patient? A The patient should avoid using oral contraceptives. B This patient should avoid foods containing iron. C Daily use of aspirin is recommended. D Regular, strenuous aerobic exercise should be strongly encouraged. E Routine CBC, EKG, and echocardiograms should not be made available.

A

A 26-year-old man, residing in Thailand, presents with a 7-day history of high-grade fever, dull, frontal headache, malaise, anorexia, and vague abdominal discomfort. He has mild diarrhea, dry cough, and myalgia. On examination, temperature is 39°C and pulse is 65 per minute. He has a coated tongue, tender abdomen, and a soft, palpable spleen. Clinical diagnosis is enteric fever. Appropriate clinical samples are sent for culture and serology. What laboratory data would be helpful in making a definitive diagnosis? Answer Choices: A Blood culture positive for S. typhi B Stool culture positive for S. typhi C Urine culture positive for S. typhi D Serum titer of S. typhi O agglutinins 1:80 E Serum titer of S. typhi H agglutinins 1:160

A

A 26-year-old woman presents for her 2nd obstetric visit in the 1st trimester. Routine screening tests (blood typing, testing for syphilis, hepatitis, rubella immunity and HIV) are performed, and the test returns positive for HIV. She is counseled to start antiretroviral therapy and to have a cesarean delivery. To reduce the risk of mother-to-newborn transmission, the best drug treatment is a drug that prevents what mechanism of the infection? Answer Choices: A Virus replication B Viral assembly C Fusion of virus with the host cell D Integration of HIV genetic material into the host chromosome E Binding of the HIV virion to the surface of the cells

A

A 27-year-old man presents with a 4-day history of fever, chills, and a non-productive cough. He says he feels like he has the "flu" and complains of muscle and joint aches. Several of his friends have similar symptoms, and 1 of them is hospitalized. The group of friends took a road trip together 3 weeks ago through a variety of regions and terrains. What exposure would lead you to suspect infection with Histoplasmosis? A Bats B Fleas C Mosquitos D Parrots E Rabbits F Ticks

A

A 28-year-old African-American man presents with dyspnea; it is associated with mild substernal chest pain and dizziness. Symptoms are provoked by sporting activities, and they are relieved with rest. His physical exam reveals a harsh murmur best heard at the left lower sternal border; it decreases in intensity upon squatting. There is also an S4 gallop. A bedside electrocardiogram was remarkable for left ventricular hypertrophy and septal Q waves in the anterolateral leads. An echocardiogram noted asymmetric LVH and a septum that was 2 times the thickness of the posterior wall. What would be the next most appropriate step in the management of this patient? A Begin this patient on a -adrenergic blocker B Recommend increased aerobic activity to improve conditioning C Refer this patient for immediate septal myectomy D Implantation of a dual-chamber pacemaker E Initiate oral digitalis, diuretic and nitrate therapy

A

A 28-year-old HIV-positive man presents with night sweats and chronic cough with very little sputum. His sputum for acid-fast rods is positive. His PPD skin test is negative. His CD4 count is 120/mm3. His sputum specimen is sent for culture, and his treatment is started. What organism is likely to be isolated from this patient's sputum? Answer Choices: A Mycobacterium avium-intracellulare complex B Mycobacterium kansasii C Mycobacterium marinum D Mycobacterium scrofulaceum E Mycobacterium fortuitum-chelonei complex

A

A 28-year-old woman presented to Accident and Emergency complaining of a 3-hour history of nausea, abdominal cramping, vomiting, and watery diarrhoea. She recounts her lunch approximately 8 hours ago, which consisted of a prawn salad and a diet fizzy drink. She denies any neurological or other symptoms. She also denies any similar problems prior to this episode and was feeling well just before this incident. She denies fever or recent travel. What is the most likely organism responsible for her symptoms? A Vibrio parahaemolyticus B Giardia Lamblia C Clostridium difficile D Clostridium perfringens E Campylobacter jejuni

A

A 28-year-old woman recently completed inpatient chemotherapy for acute myelogenous leukemia; she presents with mouth pain. Examination of the mouth reveals white patches; when rubbed off, they reveal a red base. Question: What is the most likely diagnosis? Answer Choices: A Candidiasis B Leukoplakia C Erythroplakia D Oral lichen planus E Oral cancer

A

A 29-year-old woman presents with a previous history of mitral valve prolapse with murmur of regurgitation confirmed on echocardiogram with prosthetic valve replacement one year ago. Based on the recommendations by the American Heart Association, antibiotic prophylaxis would be recommended before which of the following procedures? A Prosthetic valve replacement B Postoperative suture removal C Orthodontic appliance adjustment D Intracanal endodontic treatment E Taking of oral impression

A

A 3-month-old male infant presents for a routine evaluation. His mother states that the child is gaining weight and feeding appropriately and has been without fever, chills, dyspnea, or other abnormal objective signs. Upon physical examination, the examiner noticed a loud, harsh holosystolic murmur in the left third and fourth interspaces along the sternum that was associated with a systolic thrill. There were no other abnormalities. What is correct regarding this patient's diagnosis? A The smaller the defect is, the louder the associated murmur. B This is a common finding in the adult population. C A right-to-left shunt is expected in this patient. D This patient is expected to have a lower life expectancy. E Small shunts require immediate surgical repair.

A

A 31-year-old man presents with a tick bite. He describes locating a tick in the left axillary area while showering in the evening, and he denies that the tick was present the previous evening. He removed the tick with forceps and cleaned the wound with alcohol. Examination of the bite shows local erythema and mild induration of 3 - 5 mm diameter. The tick is identified as an adult female Ixodes scapularis. What is the recommended course of management for this patient? Answer Choices: A No treatment is needed B The tick should be tested for Borrelia burgdorferi C Prophylaxis with doxycycline D Lyme disease serology E Urine antigen test

A

A 32-year-old farmer presents with a deep wound on his right thigh. The wound was caused by a soil-contaminated sharp object. He remembers getting his most recent tetanus vaccine when he was 17 years old. What is the best approach in terms of treating this wound? Answer Choices: A Tetanus vaccine, TIG (tetanus immune globulin), as well as cleaning and debriding the wound B TIG and cleaning and debriding the wound C Cleaning and debriding the wound D Tetanus vaccine and cleaning the wound E TIG and tetanus vaccine

A

A 32-year-old woman returns from a 2-week camping trip along the Appalachian Trail in New York; she presents with a low-grade fever and a non-pruritic rash on her back and buttocks. Physical exam reveals 2 large 5 cm erythematous lesions, with central clearing on her low back and buttocks. There is no lymphadenopathy. CBC with differential and ESR are normal. What is the most likely diagnosis? Answer Choices: A Lyme disease B Rhus dermatitis C Pityriasis rubra pilaris D Erythema nodosum E Erythema multiforma

A

A 34-year-old ornithologist is seeking medical advice because he plans to travel to Africa. He is worried because of the endemic areas of yellow fever in Africa, and he wants to know if there is a vaccine. He once had an anaphylactic reaction to eggs; otherwise, he is healthy. You explain to him that he cannot be immunized. What vaccine is also contraindicated for this patient? Answer Choices: A Mumps B Hepatitis B C Clostridium tetani D Bordetella pertussis E Corynebacterium diphtheriae

A

A 35-year-old Caucasian woman presents with consistent Stage I hypertension on 3 separate visits and has already attempted lifestyle changes to decrease blood pressure with little success. You decide to start her on hydrochlorothiazide 25 mg daily and have her return in 1 week for a blood pressure check. Which of the following statements is true about thiazide diuretics? A They inhibit sodium reabsorption in the distal tubule B They inhibit sodium reabsorption in the proximal convoluted tubule C They act as aldosterone receptor antagonists in the distal tubule D They inhibit sodium-potassium-chloride transport in the ascending Loop of Henle. E They inhibit the transport of bicarbonate out of the proximal convoluted tubule

A

A 35-year-old Costa Rican woman emigrated to the United States 5 years ago. She presents to the office with complaints of chronic, progressive dyspnea. She reports no chest pain, fever, or cough and is a non-smoker. Her daily activities are becoming increasingly limited due to her shortness of breath. On auscultation she has an accentuated S1, a loud opening snap, and a diastolic murmur heard best at the apex in the left lateral recumbent position. She has 1+ ankle edema bilaterally. What is the most likely cause of this patient's symptoms? A Mitral stenosis B Mitral valve prolapse C Tricuspid stenosis D Aortic stenosis E Aortic regurgitation

A

A 35-year-old man has just returned to the United States from Thailand. He presents with a 3-day history of chills, fever, headache, myalgia, weakness, and loss of appetite. For the past 12 years he lived in Thailand, where he had 3 episodes of malaria. He gives a history of exploratory laparotomy and splenectomy following an auto accident in his 20's. Based on the above information, what is the most important diagnosis to consider? A Malaria B Schistosomiasis C Influenzal syndrome D Trypanosomiasis E Smallpox

A

A 35-year-old man presents with a non-productive cough and progressive dyspnea on exertion; he's been experiencing both symptoms for "a little over a week". He denies any past medical history. He has not been to a doctor since he had a pre-employment physical examination 10 years prior. However, he has been using acetaminophen for headache for about 10 days. He denies any fever. He thinks he has lost some weight and has night sweats, but there is no history of contact with persons with chronic cough with associated weight loss. Systemic review shows anorexia with some difficulty swallowing solids. There is no dysuria or change in bowel habits. He has smoked a pack of cigarettes daily for the last 20 years; he drinks alcohol socially and he tried heroin a few times in the past. On examination, he has no pallor but has some whitish plaques in the mouth; his chest was clear on auscultation, with no abdominal signs. Laboratory studies are shown in the table. WBC 2800/μL Neutrophils 89% Lymphocytes 8% Hemoglobin 8g/dL Platelets 135,000/mm3 Serum glucose 96 mg/dl Sodium 140mmol/L Chloride 102mmol/L Potassium 4.2mmol/L Bicarbonate 22mmol/L BUN 9mmol/L Creatinine 0.7mmol/L LDH 546U/L Serum albumin 1.8g/dL Serum protein 7.9g/dL What is the next step in the management of this patient? Answer Choices: A IV sulfamethoxazole/trimethoprim (Bactrim) B IV macrolide C Anti-TB medications D IV diuretics E HIV medications

A

A 38-year-old patient with HIV develops diabetes; he takes stavudine. What diabetes medication is most likely to exacerbate potential acid base disorders of his antiviral therapy? Answer Choices: A Metformin B Glyburide C Glipizide D Exenatide E Insulin

A

A 4-year-old boy accompanied by his mother presents with fever, sore throat, muffled voice, and breathing and swallowing difficulty. The child is leaning forward with his head and nose tilted upward and forward. He is irritable with moderate respiratory distress and inspiratory stridor. Vitals are as follows: pulse is 94/min; BP is 110/70 mm Hg; and temperature is 101˚F. Direct fiberoptic laryngoscopy is performed and shows an edematous larynx. Cultures are taken, and an endotracheal tube is placed. The epiglottis cultures reveal Haemophilus influenzae, and the diagnosis of acute Haemophilus influenzae epiglottitis is made. The mother is worried about her second 1-year-old child living in the same house who is not vaccinated for H. influenzae. Question: What prophylactic measure would you recommend at this time? Answer Choices: A All family members, including the patient, should receive prophylactic rifampicin. B All family members, excluding the patient, should receive prophylactic rifampicin. C Only the 1-year-old child should receive prophylactic rifampicin. D The 1-year-old-child should receive the Hib polysaccharide vaccine. E Only the infected child need be treated.

A

A 4-year-old boy presents with poor weight gain, small size for his age, and dyspnea upon feeding. His mother notes that the child suffers from frequent upper respiratory tract infections. On physical exam, the child is underweight for his age. You note a precordial bulge, a prominent right ventricular cardiac impulse, and palpable pulmonary artery pulsations. You also find a widely split and fixed second heart sound as well as a mid-diastolic rumble at the left sternal border. What pharmacologic agent would be most appropriate in the medical management of this patient at this time? A Lasix (Furosemide) B Coumadin (Warfarin) C Procardia (Nifedipine) D Inderal (Propranolol) E Indocin (Indomethacin)

A

A 4-year-old boy, who is currently hospitalized with pneumonia, develops vomiting and stiff neck. The new symptoms are followed by a seizure. On physical examination, he is febrile and has signs of meningeal irritation. A lumbar puncture is done to determine if he has meningitis. The results are as follows: TEST RESULTS REFERENCE RANGE CSF pressure 238 mm H2O 70-80 mm H2O CSF leukocytes 8,200 cu mm 0-10 cu mm CSF glucose 26 mg/dL 45-80 mg/dL CSF protein 253 mg/dL 15-45 mg/dL CSF gram stain Pending What organism is probably causing his meningitis? Answer Choices: A Neisseria meningitidis B Staphylococcus epidermidis C Staphylococcus aureus D Streptococcus agalactiae E Listeria monocytogenes

A

A 42-year-old man presents with stumbling, memory loss, and confusion, which his girlfriend claims has been occurring for the past month. Additionally, he reportedly had a tonic-clonic seizure 1 week ago, for which he refused treatment. The patient's past medical history is significant for HIV infection/AIDS and he has been non-compliant with his HIV medication, with no medication use at all for at least 4 months. His CD4 count was 123 cells/mcL 6 months ago, and his HIV viral load was 926 copies/mL. A CT of his head shows multiple peripheral ring-enhancing lesions. Question: What is the most likely diagnosis? Answer Choices: A Toxoplasmosis B HIV dementia C Progressive multifocal leukoencephalopathy (PML) D CNS lymphoma E Cryptococcal meningitis

A

A 43-year-old man visits the internal medicine clinic of a university hospital presenting with a 3-week history of shortness of breath, fever, and chills. Examination shows a temperature of 38.0° C, and laboratory results suggest hypoxia. Previous history shows the patient has been HIV-1 positive for 4 years and presently has a CD4+ T-cell count of 50/mm3. A presumptive diagnosis of Pneumocystis carinii pneumonia (PCP) is made, which is confirmed by bronchoalveolar lavage. What physical finding in PCP would be expected from chest auscultation in this patient? Answer Choices: A No findings B Unilateral consolidation C Rales D Egophony E Infiltrates F Crepitation

A

A 43-year-old patient presents with an acute onset of palpitations and dyspnea for the past 3 hours. There is no other significant medical history. Chest X-ray and echocardiography are normal. EKG performed in the ED reveals absence of P wave with variable R-R interval with atrial rate of 300/min and ventricular rate of 120/min. Vitals: patient is afebrile, radial pulse- 90/min, BP- 110/70 mm Hg and RR- 18/min. A diagnosis of newly detected atrial fibrillation is made. What is the next course of action? A Start diltiazem therapy B Start propafenone C Observation only D AV (atrioventricular) nodal ablation E Start amiodarone therapy

A

A 45-year-old man presents to the ER complaining of tearing chest pain that radiates to his back. He has known coronary artery disease and a previous history of myocardial infarction. Vital signs reveal a blood pressure of 150/90 mm Hg and a heart rate of 120/min. After a titration of esmolol was started his blood pressure was 110/70 mm hg and his heart rate was 70/min. An electrocardiogram revealed new ischemic changes. A CT scan reveals a proximal aortic dissection. The patient does not demonstrate any signs of limb malperfusion or renal insufficiency. What is the next step? A Obtain a cardiac catheterization B Proceed immediately to the operating room for repair of the dissection C Add a nitroprusside infusion D Continue the esmolol infusion and monitor patient closely E Obtain an echocardiogram to evaluate for the presence of pericardial effusion

A

A 49-year-old homeless man presents with worsening belly pains. He is a recovering heroin addict and is afflicted with Acquired Immune Deficiency Syndrome (AIDS). He states that he has been attending 12-step Narcotics Anonymous meetings 3 times each day with his NA sponsor, and that he has not been using heroin, morphine, meperidine (Demerol), or hydromorphone (Dilaudid). He is taking his methadone as usual. What medication could account for his new medical issue? Answer Choices: A Sustiva (Efavirenz) B Fluoxetine (Prozac) C Lanoxin (Digoxin) D Acetaminophen (Tylenol) E Buffered normal saline nasal drops (Ocean Nasal Mist)

A

A 5-week-old male infant presents with a 2-week history of prominent cough, nasal congestion, and wheezing. His symptoms have been getting progressively worse. Yesterday, the patient's mother took her son to her primary care doctor. The doctor started albuterol nebulizers every 4 hours and told her that the child had a viral infection and would get better. His mother is now particularly concerned that her child has had dry diapers for over 15 hours. The child has been irritable during this time. He has not had any fevers, vomiting, or diarrhea. The child is not tolerating breast-feeding or bottle-feeding well. The physical exam shows that the child is acyanotic and alert. The temperature is 97.8 Fahrenheit (36.5 Celsius), respirations are 40/minute, and the pulse is 119/minute. There are no lymph nodes observed, and his tympanic membranes appear normal. There are rales noted diffusely on auscultation. The mucous membranes are moist and the skin has good turgor. You also detect conjunctivitis. Blood work is obtained; including a set of blood cultures, and a urine culture. The following lab values return: Hemoglobin 12.6mg/dL Hematocrit 37.1% Platelet count 204 x 109 Eosinophilia count 6% The chest film demonstrates interstitial infiltrates and hyperinflation. What is the organism causing the infant's symptoms? A Chlamydia trachomatis B Human Parvovirus C Parainfluenza virus D Respiratory Syncytial Virus E Staphylococcal species

A

A 5-year-old boy presents with 6-day history of fever, fatigue, and rash. He has no significant past medical history. He is current with his vaccinations, except for varicella, which his parents have refused in the past. On exam, his temperature is 101.3º F; heart rate is 110, and blood pressure is 94 He has bilateral conjunctival injection, an erythematous pharynx without exudate, cracked red lips, and an erythematous right tympanic membrane. He has shotty enlarged anterior cervical lymph nodes bilaterally, the largest nodes measuring 1.6 cm on the right side and 1.5 cm on the left side. His lungs are clear, and his heart has a regular rhythm. His abdomen is soft. He is in no acute distress, and he has a generalized maculopapular rash. What are some other possible findings associated with his probable diagnosis? A Sterile pyuria, meningismus B Peripheral neuropathy, thrombocytopenia C Epistaxes, hematuria D Colonic polyps, gastritis E Splenomegaly, pleuritis

A

A 5-year-old girl presents with a rash of clear and grayish vesicles on a reddened base. Her hands and feet are affected, including her palms and soles. She complains of sores in her mouth. There is no desquamation to the rash. Her heart and lungs are clear. She is febrile, and there is an accompanying tachycardia. Her blood pressure is normal. Her oral cavity shows ulcerations. What is the most likely diagnosis? Answer Choices: A Hand-foot-mouth disease B Measles C Kawasaki disease D Toxic shock syndrome E Rocky Mountain spotted fever

A

A 50-year-old man presents with multisystem failure secondary to bilateral pneumonia. 4 days prior, he presented to his primary care physician with fever, cough, and malaise; he was given a broad-spectrum antibiotic, and he progressively became worse over the course of the antibiotic treatment. His body temperature is 40° C, respiration is 35/min, and pulse is 100/min. Laboratory examination is significant for impaired liver and renal function. Patient history is significant for the use of corticosteroids. A sputum culture is performed and is significant for the presence of WBC in the Gram stain, but there are no significant organisms present. The culture result is negative. A urinary antigen test is ordered for Legionella pneumophila and the result is positive. What is the drug of choice in treating this patient? Answer Choices: A Erythromycin B Tetracycline C Penicillin D Vancomycin E Amphotericin B

A

A 55-year-old male firefighter suddenly develops fever, palpitations, and shortness of breath seven days after skin transplant for severe facial burns. He also has fever resistant to intravenous antibiotics therapy introduced at the onset of fever. Because of oropharyngeal lesions, he is on parenteral nutrition. On examination, transplant shows neither signs of infections nor signs of rejection. Patient appears lethargic, with a blood pressure of 80/40, pulse rate of 120, respiratory rate of 18, and temperature of 103.4. Stat CBC shows neutropenia and eosinophilia. Question: For what reason would you order a blood culture for systemic fungal infection? Answer Choices: A Neutropenia B Intravenous antibiotics C Parenteral nutrition D Facial surgery E Age

A

A 56-year-old man with a past medical history of diabetes mellitus type II, hypertension, myocardial infarction 1 year ago, and a 50-pack/year smoking history presents with unilateral leg edema and leg pain for the last 6 hours. His physical exam is remarkable for an elevated systolic blood pressure, obesity, and localized left lower extremity edema, erythema, and tenderness to the left lateral calf. What additional physical exam finding would be most consistent with a diagnosis of superficial thrombophlebitis in this patient? A Linear induration along the course of a vein B Peripheral cyanosis C A positive Homan's sign D Reduced amplitudes of the dorsalis pedis and tibialis anterior pulses E A shallow, well-circumscribed ulcer above the medial malleolus

A

A 6-year-old child presents with a 2-day history of fever, cough, coryza, and rash. The rash started behind the ear and spread downwards towards the trunk. On examination, you notice erythematous lesions, which are primarily located on the trunk. The palm and soles are spared. On oral examination you notice 1-2mm bluish lesions surrounded by an erythematous halo. Question: Which virus is most likely responsible for this patient's condition? Answer Choices: A Paramyxovirus B Togavirus C Human parvovirus B19 D Human herpesvirus 6 E Varicella-Zoster Virus

A

A 65-year-old male presents with complaints of palpitations for 2 days, and a 3 hour history of severe, unrelenting, sharp mid-line abdominal pain. His past medical history includes hypertension, diabetes, and peripheral vascular disease. He underwent a femoral artery stent placement 3 years ago. He has also smoked 2 packs of cigarettes a day for 30 years. His physical exam consists of a blood pressure of 80/60 mmHg, pulse rate of 120 beats/ minute, and respiratory rate of 24 breaths/minute. He has minimal tenderness in the midline of the abdomen and hypoactive bowel sounds. His WBC count is 23/uL and serum lactate of 25/uL. His plain films of the abdomen are non-specific, and a CT demonstrates small bowel thickening. An aortogram and mesenteric angiogram demonstrate an abrupt cut-off of the superior mesenteric artery, just beyond its first arterial branch, with absent flow beyond this occlusion. The next treatment of this patient should include: A Operative exploration B Thrombolysis C Antibiotics D Bowel transplantation E Appendectomy

A

A 70-year-old African American man presents to his primary care provider with a complaint of gradual but progressive dyspnea and fatigue on exertion. He notes difficulty in climbing stairs with associated lightheadedness, increased abdominal girth, and swollen bilateral lower extremities. He admits to chronic alcohol use, but denies any heart disease, chest pain or pressure, diaphroesis, palpitations, a history of diabetes, cigarette smoking, or claudication. His physical exam reveals sinus tachycardia, bibasilar rales, a laterally-displaced PMI, an elevated JVP, an S3 gallop, a mitral regurgitation murmur, peripheral edema, and abdominal ascites. What is the next most appropriate step in the management of this patient? A Obtain an echocardiogram B Refer the patient for cardiac catheterization C Perform a cardiac biopsy D Order coxsackie and adenovirus serological titers E Order cardiac troponin levels

A

A 74-year-old Caucasian woman with a history of hypertension, hyperlipidemia, and myocardial infarction presents with shortness of breath upon exertion, lightheadedness, increased fatigue, and palpitations for 3 days. She has found it difficult to walk short distances due to shortness of breath. She denies any orthopnea, nocturnal dyspnea, abdominal pain cough, fever, chills, diaphoresis, anxiety, chest pain, pleurisy, cough, nausea, vomiting, diarrhea, rashes, or syncope. She is well nourished and afebrile, but tachypnic. Her cardiac exam reveals an irregularly irregular pulse, hypotension, diminished first heart sound, S3 gallop, and laterally displaced PMI. Her lung, peripheral vascular, and abdominal exams are normal. An echocardiogram is remarkable for a left ventricular ejection fraction of 25% while an electrocardiogram displayed the included results. What agent would be most appropriate in the management of this patient at this time? A Digoxin B Furosemide C Isosorbide dinitrate and hydralazine D Nifedipine E Milrinone

A

A 76-year-old man presents with progressive exertional dyspnea; it is associated with substernal chest pain, easy fatigability, and dizziness. These symptoms are exacerbated with walking short distances, and they are relieved with rest. He denies fever, chills, cough, wheezing, pleurisy, calf pain, abdominal complaints, peripheral edema, cigarette, drug use, alcohol use, sick contacts, or travel. His physical exam reveals a normal blood pressure and a rough, harsh, low-pitched crescendo-decrescendo systolic murmur beginning after the first heart sound; it is best heard at the second intercostal space in the right upper sternal border. Its intensity is increased toward midsystole; the murmur radiates to both carotid arteries and is accentuated upon squatting, and it is reduced during Valsalva strain. His lungs are without adventitious sounds. What medication is most appropriate in the medical management of patient? A Lopressor (Metoprolol) B Vasotec (Enalapril) C Nitropress (Nitroprusside) D Lasix (Furosemide) E Digoxin (Lanoxin)

A

A 79-year-old man presents with severe chest pain and dyspnea. He has a past medical history of diabetes mellitus, hypertension, and hyperlipidemia. He appears pale, apprehensive, and diaphoretic. He is in a confused state and agitated. His pulse is weak and tachycardic, with a systolic blood pressure of 60 mmHg. He has a narrow pulse pressure, tachypnea, a weak apical impulse, and significant jugular venous distention. His lungs are free of crackles. Bedside electrocardiogram reveals ST-segment elevations in the anterior and septal leads. What concerning this patient's diagnosis is correct? A The leading contributory cause of this patient's illness is myocardial ischemia B A profound reduction in intravascular volume is responsible for hypotension C Elevated serum lactate levels suggest an improved prognostic rate D Coronary artery angiography and revascularization is contraindicated E A high cardiac index and low pulmonary capillary wedge pressure is expected

A

A female neonate born at 29 weeks gestation is admitted to the neonatal intensive care unit. She has a continuous machine-like murmur heard at the left first intercostal space. Which of the following medications is most beneficial in the treatment of the patient's condition? A Indomethacin B Azithromycin C Cyclosporine D Labetalol E Furosemide

A

A healthy couple presents to your outpatient clinic for evaluation prior to travel to the Dominican Republic. They leave in 1 month and will stay there 4 weeks. You see that the CDC web site lists the Dominican Republic as a malaria-endemic area but is not considered to have a resistant strain of Plasmodium falciparum. Question: What is the best advice for this couple? Answer Choices: A They should both take chloroquine weekly, starting 1 week before travel. B If they get any mosquito bites, they need to return to the U.S. immediately for malaria treatment. C They should be aware that malaria typically causes a mild 3-4 day, self-limiting diarrheal illness. D If they choose to take prophylactic medications for malaria, they can discontinue the medications upon arrival if they have no symptoms. E They should both pack prescriptions of high-dose amoxicillin to take if malarial symptoms begin.

A

A local company arranged an outdoor end-of-the-summer luncheon for its coworkers. The food was prepared by a local caterer and included sandwiches, hot dogs, hamburgers, and a selection of cold salads. It was a beautiful, hot summer day and the party seemed to be going very well. However, about 30 minutes after lunch was served, several of the guests developed nausea and severe vomiting. The illness affected so many workers that the company was forced to shut down for the rest of the day. A food-borne illness was suspected, and the public health authorities were called to investigate. Based on the description of the event, what pathogen most likely caused this outbreak? Answer Choices: A Staphylococcus aureus B Bacillus cereus C Clostridium perfringens D Giardia lamblia E Entamoeba histolytica

A

After successful cardioversion for atrial fibrillation (AF), a 65-year-old woman with rheumatic mitral valve disease patient feels better, with the exception of slight weakness in her left arm as a consequence of previous ischemic stroke. Her blood pressure is 120/80; her EKG is normal and shows sinus rhythm with rate 75. What chronic medical therapy is recommended? A Warfarin B Aspirin C Ticlopidine D Clopidogrel E Propafenone

A

An 8-year-old boy develops fever and an upper respiratory infection in late December. On physical examination, the pediatrician notes blue-gray spots in his mouth near his second molars. A day later the boy develops a rash on his face. The rash consists of reddened macules and papules. Over time, the rash becomes confluent and moves downward over his whole body. Desquamation of the rash is not present. What is the diagnosis? Answer Choices: A Measles B Hand-foot-mouth disease C Kawasaki disease D Toxic shock syndrome E Rocky Mountain spotted fever

A

An 86-year-old woman presents with recent onset of intractable headaches, jaw claudication, and visual field changes, including diplopia. Her past medical history is significant for Polymyalgia Rheumatica. Based on the most likely diagnosis, what is the primary reason for prompt diagnosis and treatment of this condition? A Prevent blindness B Prevent stroke C Prevent facial palsy D Prevent myocardial infarction E Prevent hemorrhage

A

An autopsy is performed on a deceased 57-year-old man. There is pathology in the aorta consisting of a strange wrinkling to the intima that looks like "tree barking." The pathologist pulls an old chest X-ray and notes that there is eggshell calcification of the ascending aorta. Based on the autopsy findings, what disease did this person have? Answer Choices: A Syphilis B Temporal arteritis C Atherosclerosis D Mönckeberg's sclerosis E Polyarteritis nodosa

A

A 55-year-old man presents with a 12-hour history of severe epigastric abdominal pain that radiates into his back, nausea, vomiting, and chills. His pertinent physical examination findings include oral temperature of 102°F and epigastric tenderness upon palpation. He is admitted to the hospital for management of his condition. Question What criteria would increase the likelihood of a severe attack associated with a high incidence of short and long term morbidity and mortality? Answer Choices 1 White blood cell (WBC) count of 10,000cells/mm3 upon admission 2 A 14% decrease of his hematocrit at 48 hours after admission 3 Serum glucose of 130 mg/dL upon admission 4 Aspartate transaminase of 240 U/dL upon admission 5 Serum calcium of 10 mg/dL 48 hours after admission

A 14% decrease of his hematocrit at 48 hours after admission is the correct response. A patient who is admitted to the hospital for treatment of acute pancreatitis is assessed using the Ranson criteria to determine the severity of their disease, which in turn helps determine prognosis. About 70 - 80% of cases of acute pancreatitis are considered mild and result in virtually no morbidity or mortality. The remainder are severe attacks and have a 10 - 30% mortality rate. Fast identification of the severe cases is helpful to reduce the morbidity and mortality for each patient. There are 11 Ranson criteria, 5 of which are determined upon admission, and 6 at 48 hours after admission. Patients who have 2 or fewer of the criteria have minimal mortality. Patients with 3 - 5 of the criteria have about a 10 - 20% chance of mortality. Patients with 5 or more of the criteria have at least a 50% mortality rate. A patient with at least a 10% decrease in their hematocrit at 48 hours after admission meets one of the Ranson criteria, so the correct answer is a 14% decrease in hematocrit 48 hours after admission. WBC count of 10,000 cells/mm3 is not one of the Ranson criteria. A WBC count of >16,000 cells/mm3 would meet one of the Ranson criteria. Serum glucose of 130 mg/dL upon admission is also not one of the Ranson criteria. The patient's serum glucose would have to be at least 200 mg/dL to count as one of the Ranson criteria and increase the severity of their disease. Aspartate transaminase of 240 U/dL upon admission is not one of the Ranson criteria either. The patient's aspartate transaminase would have to be at least 250 U/dL to count as a Ranson criteria and increase the severity of their disease. A serum calcium of 10 mg/dL 48 hours after admission is not one of the Ranson criteria. If the patient's serum calcium is under 8 mg/dL 58 hours after admission, this would count as one of the Ranson criteria.

Which of the following medication classes has been shown to improve the short-term rate of cognitive preservation in mild to moderate Alzheimer's dementia patients? A Acetylcholinesterase inhibitors B Decarboxylase inhibitors C Dopamine receptor agonists D Monoamine oxidase inhibitor E N-methyl-D-aspartate receptor antagonist

A Acetylcholinesterase inhibitors Alzheimer's dementia is a chronic, progressive, neurodegenerative disorder. Acetylcholinesterase inhibitors have been associated with a modest decrease of cognitive decline and increased functioning. These medications increase the levels of acetylcholine, a neurotransmitter, and increase cholinergic activity within the affected brain regions. They have been approved for use in mild to moderate Alzheimer's disease. Memantine, an N-methyl-D-aspartate receptor antagonist, is indicated for use in moderate to severe disease, has been associated with decreased destruction of cholinergic neurons, and may slow cognitive decline. It is often added to acetylcholinesterase inhibitor therapy for progressed disease, but may be first line in a patient unable to tolerate the acetylcholinesterase inhibitor medications. Decarboxylase inhibitor medications inhibit the metabolism of dopamine; one example, carbidopa, is used in conjunction with L-dopa in the treatment of Parkinson's disease. Dopamine receptor agonists activate dopamine receptors in the absence of dopamine, and are useful in conditions with low dopamine levels, such as Parkinson's, prolactinomas, and restless leg syndrome. Selegiline, a monoamine oxidase inhibitor, is selective for monoamine oxidase-B and inhibits catabolic dopamine breakdown, with a potential neuroprotective effect. Use in moderately advanced Alzheimer's disease may slow progression, but remains controversial.

A patient presents with hyperleukocytosis (WBC 100,000). The bone marrow is consistent with T-cell ALL. The uric acid is less than 7.0 mg/dl. Prior to starting chemotherapy, what is the best therapy to initiate on this patient? A Allopurinol and IV fluids B Aluminum hydroxide and calcium carbonate C Glucocorticoids and vincristine D Leukapheresis and cranial irradiation E Sevelamer and mercaptopurine

A Allopurinol and IV fluids Hyperuricemia is often a finding in patients with hyperleukocytosis. The optimal treatment is to start IV fluids, due to numbers of circulating white cells and allopurinol, to treat hyperuricemia. The use of aluminum hydroxide is appropriate if they have hyperphosphatemia, and calcium carbonate if they have a low serum calcium concentration. Glucocorticoids and vincristine are used with hyperleukocytosis of >400,000. Leukapheresis and cranial irradiation are used for patients with extreme leukocytosis of >400,000. Sevelamer is used to treat hyperphosphatemia, and mercaptopurine is a byproduct produced during production of leukemic cells.

A 51-year-old male patient presents to your family practice office complaining of genital discomfort with dysuria. His digital rectal exam reveals an enlarged, tender prostate. His prostate-specific antigen (PSA) returns elevated with a value of 11.1 mg/mL, which you fractionate, and this reveals approximately 75% free PSA. His urinalysis reveals moderate white cells and trace blood. What would be your next step in treating this patient? A Begin him on 6 weeks of doxycycline to treat his prostatitis and when resolved, repeat his PSA level. B Immediately refer him to a urologist for prostate biopsy to rule out prostate cancer. C Immediately refer him to a urologist for cystoscopy to rule out bladder cancer and perform a computed tomography (CT) scan of the abdomen and pelvis in the interim. D Order a stat testicular sonogram to rule out torsion. E Order a CT scan of the abdomen and pelvis.

A Begin him on 6 weeks of doxycycline to treat his prostatitis and when resolved, repeat his PSA level. A This patient has signs and symptoms consistent with prostatitis. Additionally, while his PSA is elevated, this is common in prostatits as well as prostate cancer, and his free PSA is of a percentage that prostate cancer is unlikely. However, it would be prudent to recheck his PSA after treatment and resolution of his symptoms to confirm that an underlying cancer is not smoldering.

Which of the following etiologies is the most common cause of nontraumatic subarachnoid hemorrhage (SAH)? A Cerebral aneurysm B Poorly controlled hypertension C Anticoagulant use D Arteriovenous malformation

A Cerebral aneurysm Congenital cerebral aneurysms or Berry aneurysms account for 75% to 80% of nontraumatic subarachnoid hemorrhages (SAHs). Poorly controlled hypertension and anticoagulant use are more commonly associated with intracerebral hemorrhages (ICH). AVMs can cause either SAH or ICH

A 43 year-old woman is brought to the emergency department in critical care secondary to a traumatic brain injury. Which of the following best describes her Cheyne-Stokes respiratory pattern? A Alternating periods of shallow and deep breathing B Difficult or labored breathing C Difficult of labored breathing while supine D Periods of absent breathing E Sudden awakening due to shortness of breath

A Cheyne-Stokes respirations are characterized by shallow breaths that increase in rate and depth followed by periods of apnea (A). Dyspnea is difficult or labored breathing (B) and if it occurs while supine (C) is termed orthopnea. Apnea is noted periods of absent breathing (D). Sudden awakening due to shortness of breath (E) is paroxysmal nocturnal dyspnea.

Which of the following abnormalities corresponds most with the Golgi apparatus? A Classic congenital adrenal hyperplasia (21-hydroxylase deficiency) B Insufficient mucosal defense in duodenal peptic ulcer disease C Ketone overproduction in untreated diabetes D Corynebacterium diphtheriae infection E N-myc amplification in neuroblastoma

A Classic congenital adrenal hyperplasia (21-hydroxylase deficiency) *B Insufficient mucosal defense in duodenal peptic ulcer disease* C Ketone overproduction in untreated diabetes D Corynebacterium diphtheriae infection E N-myc amplification in neuroblastoma Golgi apparatus is a series of stacked , flat saccules or cisternae which are slightly curved. Golgi apparatus plays a critical role in processing and modifying secretion products. Various carbohydrate moieties are added to the protein components of mucus. Lipoproteins are produced in part, by enzymes of the Golgi apparatus. The Golgi cisternae and cisternae of the rough endoplasmic reticulum communicate and proteins are passed. The exclusive Brunner glands of the duodenum and pyloric part of the stomach secrete mucus and have prominent apical Golgi apparatus. Imbalance between acid production and mucosal defense plays a significant role in the development of duodenal ulcer. Duodenal peptic ulcer disease is seen most frequently in boys older than 6 years of age.

A 24-year-old male has an eight-month history of loose thought associations, social withdrawal, auditory hallucinations, and deterioration in his personal appearance and hygiene. Upon examination, he is noted to have a flat affect, perceptual distortions, and behaves like he is detached from his own actions. If chosen for treatment, which of the following medications would require weekly white blood cell count monitoring for the first six months? A Clozapine (Clozaril) B Haloperidol (Haldol) C Olanzapine (Zyprexa) D Risperidone (Risperdal) E Quetiapine (Seroquel)

A Clozapine Clozapine has a risk of agranulocytosis. While the risk is only 1%, weekly monitoring of the white blood cell count for the first six months, followed by monitoring of the white blood cell count every other week thereafter, is required. The other medications listed do not have the risk of agranulocytosis.

In what childhood disease does free fatty acids reach dangerous levels in untreated cases? A Diabetes mellitus B Medium chain acyl-CoA dehydrogenase deficiency C Von Gierke's disease D Ornithine transcarbamylase deficiency (OTCD) E Phenylketonuria

A Diabetes mellitus *B Medium chain acyl-CoA dehydrogenase deficiency* C Von Gierke's disease D Ornithine transcarbamylase deficiency (OTCD) E Phenylketonuria Fatty acid oxidation defects, inborn metabolic errors, cause acute encephalopathy when fasting exhausts hepatic glycogen stores, and ketone bodies do not become available for the brain. The most common defect is medium chain acyl-CoA dehydrogenase deficiency. Lipolysis mobilizes fat stores and triglycerols are hydrolyzed to glycerol and free fatty acid, nonesterified products of lipolysis. Free fatty acids serve as metabolic fuel for skeletal, cardiac muscle, and kidney. Beta-oxidation of fatty acids is the principal mechanism for the catabolism of fatty acids. The process takes place in the matrix of mitochondria. The first step of beta-oxidation is catalyzed the acyl CoA dehydrogenase. Free fatty acids supply more than 50% of the energy needs of resting and contracting skeletal muscle. During fasting, they play a critical role in the energy supply of the brain.

In what childhood disease does phenylalanine reach dangerous levels in untreated cases? A Diabetes mellitus B Medium chain acyl-CoA dehydrogenase deficiency C Von Gierke's disease D Ornithine transcarbamylase deficiency (OTCD) E Phenylketonuria

A Diabetes mellitus B Medium chain acyl-CoA dehydrogenase deficiency C Von Gierke's disease D Ornithine transcarbamylase deficiency (OTCD) *E Phenylketonuria* Phenylketonuria (PKU) does not produce acute encephalopathy but intellectual development will be seriously impaired over time, if the disorder remains untreated. Phenylalanine is an essential amino acid with aromatic side chain. It is transformed into tyrosine, a non-essential amino acid. Phenylalanine hydroxylase is the critical enzyme for the transformation of phenylalanine to tyrosine. Affected infants are normal at birth but the level of phenylalanine soon reaches extremely high levels. The elevated phenylalanine and its metabolites are neurotoxic. The resulting lack of tyrosine causes hypopigmentation of the skin. A lifelong diet restricting phenylalanine intake is necessary to prevent intellectual handicap. Newborn screening programs can reach the goal of treating PKU within the first weeks of life. In some cases of PKU, the blood level does not reach the critical threshold of neurotoxicity.

Which of the following are predictive or diagnostic of anorexia nervosa? A having a weight that is 85% less than predicted normal weight B homosexual orientation in females C promiscuity D menorrhagia

A Diagnostic criteria for anorexia nervosa include a weight loss to 85% of the required body weight. Homosexual orientation in men, not women, is considered a predisposing factor. Anorexia is associated with amenorrhea and decreased interest in sex.

A 36-year-old female presents to your family practice office concerned about a breast lump she discovered in the shower last night. You obtain a detailed history and these are some of your findings: • G2P2 • LMP: 2 weeks ago, normal • Sister diagnosed with breast cancer at age 42. • Mother is currently being treated for ovarian cancer. • Father is of Ashkenazi Jewish ancestry. Which of the following is an appropriate recommendation for this patient? A Enhanced surveillance, including mammography alternating with MRI, every six months. B Genetic counseling and testing for BRCA1 and BRCA2 status. C Referral to a breast surgeon for mastectomy D Chemoprophylaxis with an aromatase inhibitor weekly.

A Enhanced surveillance, including mammography alternating with MRI, every six months. A Because of her ethnicity (Ashkenazi Jewish descent) and family history (two first degree relatives with breast and ovarian cancers), this patient is at a higher than normal risk for breast and ovarian cancers. With a physical finding of a palpable breast mass, this is even more ominous. She should be referred for an immediate mammogram but not necessarily MRI - and have continued enhanced surveillance annually and not every 6 months regardless of the result - as well as a referral to a breast surgeon for immediate consideration of a biopsy. A mastectomy is a possible consideration but not mandatory. She is certainly an appropriate candidate for consideration of referral to a genetic counselor and genetic testing to ascertain her BRCA1 and BRCA2 status. She is also an appropriate candidate for chemoprophylaxis.

What hormone stimulates hypertrophy and hyperplasia with an increase in organ function? A Glucagon B Insulin C Thyronine D Somatotropin E Cortisol

A Glucagon B Insulin C Thyronine *D Somatotropin* E Cortisol Somatotropin (growth hormone) and insulin, together, stimulate the production of somatomedins and accretion of lean body mass. *Various visceral organs, endocrine glands, skeletal muscle, heart, skin, and connective tissue, show hypertrophy (increase in cell size) and hyperplasia (increase in the number of cells) in response to growth hormone.* The stimulated development also implies an increase in organ function. Somatotropin diverts amino acids from oxidation to protein synthesis, causing nitrogen retention, hypoaminoacidemia, and decreased urea production.

A 19-year-old G1 P0 presents to the emergency department complaining of abdominal pain. Her LMP was 2 months ago, and she has been spotting for the last two days. Her HCG is positive, and transvaginal ultrasound reveals no intrauterine gestational sac. The patient has an ectopic pregnancy, which has likely implanted in the fallopian tube. Why is the fallopian tube the most common non-uterine implantation site? A Lack of sub-mucosal layer in the fallopian tube B Inflammatory response in the endometrium C Proliferative endometrium D Extra myometrial tissue (fibroids) E Excessive trophoblastic proliferation

A Lack of sub-mucosal layer in the fallopian tube A The fallopian tube is the most common site of ectopic pregnancy, accounting for over 95% of ectopic pregnancies. The lack of a submucosal layer allows for easy wall access and implantation of the fertilized ovum. The increasing rate of Chlamydia infections in the U.S. also impacts the physical anatomy of the fallopian tube, and impacts where the ovum implants. Excessive trophoblastic activity does not impact where the ovum implants, and proliferative myometrium enhances uterine implantation if the fertilized ovum makes it to the uterine cavity.

The deficiency of which of the following intestinal brush border functions will cause glucose-galactose malabsorption syndrome? A Lactase B GLUT transport system C Cl-/HCO3- exchanger D Receptor-mediated endocytosis E Neutral brush border (NBB) system

A Lactase * B GLUT transport system* C Cl-/HCO3- exchanger D Receptor-mediated endocytosis E Neutral brush border (NBB) system Glucose and galactose are actively taken up by the brush border epithelial cells. Glucose and galactose compete for entry but others sugars are less competitive. The entry is facilitated by Na+. Glucose is poorly soluble and, therefore, several transport mechanisms exist in prokaryotes (GLUT - glucose facilitated transporters). GLUT4 is the insulin-responsive transporter. GLUT2 and GLUT5 are intestinal systems. In glucose-galactose malabsorption, ingestion of starch leads to flatulence and diarrhea but fructose is well-tolerated.

The deficiency of which of the following intestinal brush border functions will cause congenital chloride diarrhea? A Lactase B GLUT transport system C Cl-/HCO3- exchanger D Receptor-mediated endocytosis E Neutral brush border (NBB) system

A Lactase B GLUT transport system *C Cl-/HCO3- exchanger* D Receptor-mediated endocytosis E Neutral brush border (NBB) system This defect of the Cl-/HCO3- exchanger in the ileum and colon leads to diarrhea with stools high in chloride. Normally, Cl- is reabsorbed and HCO3- is secreted in the ileum and colon. In this disease, the HCO3- retention and loss of H+ lead to metabolic alkalosis. This is also related to the fact that the Na+/H+ exchange pump continues to operate but H+ is not neutralized by HCO3-. The duodenum and jejunum normally lacks Cl-/HCO3- exchanger.

The deficiency of which of the following intestinal brush border functions will cause idiopathic hemochromatosis? A Lactase B GLUT transport system C Cl-/HCO3- exchanger D Receptor-mediated endocytosis E Neutral brush border (NBB) system

A Lactase B GLUT transport system C Cl-/HCO3- exchanger *D Receptor-mediated endocytosis* E Neutral brush border (NBB) system The epithelial cells of duodenum and jejunum release *transferrin*, a protein similar but not identical to the transferrin in the blood. The transferrin-iron complex binds to a brush border receptor which triggers endocytosis. Subsequently, iron is released, crosses the basolateral membrane and binds to ferritin in the plasma. The receptor is recycled. In cases of idiopathic hemochromatosis an excessive amount of iron is absorbed from a diet which contains normal iron content.

Compared with mature milk, colostrum contains which of the following? A Less minerals B More sugar and fat C Colostrum corpuscles D No detectable immunoglobulins E Less protein

A Less minerals B More sugar and fat *C Colostrum corpuscles* D No detectable immunoglobulins E Less protein Colostrum corpuscles are thought to be either epithelial cells that have undergone fatty degeneration or mononuclear phagocytes. They contain large fat globules. Immunoglobulin A offers protection for the newborn against enteric pathogens.

In the management of hypoxia, oxygen must be administered in the smallest concentration possible (in the mid-50 range). The reason for that is an adverse effect on the regulation of respiration by the: A Medulla oblongata B Carotid body - n. glossopharyngeus - nucleus solitarius C Ventral surface of medulla oblongata D Nucleus retroambiguous E Hering-Breuer reflex

A Medulla oblongata *B Carotid body - n. glossopharyngeus - nucleus solitarius* C Ventral surface of medulla oblongata D Nucleus retroambiguous E Hering-Breuer reflex Regulation of respiration relies on a variety of receptors and neural centers. The medulla oblongata generates rhythmic impulses for transmission to the respiratory muscles. Central chemoreceptors collect information about H+ and CO2 concentrations (e.g., ventral surface of the medulla oblongata). CO2 stimulus provides about 85% of the respiratory drive. The carotid body - n. glossopharyngeus - nucleus solitarius pathway transfers the only information about O2 concentration. In addition, carotid and aortic bodies, the peripheral chemoreceptors, provide information about H+ and CO2 concentrations. Stretch receptors of small airways terminate respiration (Hering-Breuer reflex). J receptors collect information about distention of pulmonary capillaries and cause tachypnea in cases of emboli and pulmonary edema. Elimination of hypoxic drive is a major concern in oxygen therapy.

As compared to mature human milk, cow's milk contains: A More vitamin A B Less lactose C More fat D More vitamin C E Less protein

A More vitamin A *B Less lactose* C More fat D More vitamin C E Less protein Mature human milk has 6.8 gram of lactose per 100 ml while cow's milk has 4.9 grams of lactose per 100 ml. *Components (per 100 ml)* Human Milk Cow's Milk Vitamin A (μg) 53 34 Lactose (gm) 6.8 4.9 Fat (gm) 4.5 3.7

A 34-year-old woman presents for her obstetrical checkup. She is a G2P1 and her pregnancy is uncomplicated to date. During her visit, what is the most important predictor of fetal well-being? A Normal maternal vitals and fetal activity B Maternal weight gain and fetal activity C Absence of contractions and fetal activity D Normal maternal vitals and absence of contractions E Maternal weight gain and normal maternal vitals

A Normal maternal vitals and fetal activity A Maternal vitals are a sign of maternal well-being; combined with normal fetal activity, this gives the best predictive value for fetal well-being without direct fetal surveillance.

Signs of rickets are evident on the X-ray of a child. Which of the following possible diseases can be confirmed or ruled out by history, BUN, creatinine clearance, and urine tests? A Nutritional rickets B Vitamin D-dependent rickets C Chronic renal disease D X-linked hypophosphatemia E Chronic anticonvulsant therapy

A Nutritional rickets B Vitamin D-dependent rickets *C Chronic renal disease* D X-linked hypophosphatemia E Chronic anticonvulsant therapy Chronic renal disease can lead to renal osteodystrophy (osteopenia, osteitis fibrosa and rickets). The destruction of the renal parenchyma leads 1alpha-hydroxylase deficiency and low levels of ,25 hydroxyvitamin D3.

Which of the following agents can cause a deficiency of Vitamin B6 (pyridoxine) after a period of prolonged application? A Phenobarbital B Isoniazid C Warfarin D Total parenteral nutrition E Ethanol

A Phenobarbital *B Isoniazid* C Warfarin D Total parenteral nutrition E Ethanol Vitamin B6 (pyridoxine) deficiency manifests as generalized seizures in infants and polyneuritis and sideroblastic anemia in older children and adults. Sideroblastic anemias, caused by pyridoxine deficiency or lead poisoning, are characterized by hypochromic microcytic indices and increased iron stores. Isoniazid, a biochemical competitor of niacin, interferes with the synthesis of mycolic acid, a constituent of the mycobacterial cell wall. One of its adverse effects is increased excretion of pyridoxine and, therefore, concurrent vitamin B6 administration is recommended for prevention.

Which of the following agents can cause a deficiency of Vitamin B1 after a period of prolonged application? A Phenobarbital B Isoniazid C Warfarin D Total parenteral nutrition E Ethanol

A Phenobarbital B Isoniazid C Warfarin D Total parenteral nutrition *E Ethanol* Vitamin B1 (thiamine) acts as a co-enzyme in the oxidative decarboxylation of pyruvate and alpha-ketoglutarate. Clinical deficiency can be seen in children in underdeveloped countries and alcoholics in developed countries. The two forms of deficiency disease are: 1) Wet beriberi (heart disease with peripheral vasodilation, edema with sodium and water retention, and biventricular myocardial failure) and 2) Dry beriberi or Wernicke-Korsakoff syndrome (peripheral neuropathy, nystagmus, palsies, ophthalmoplegia, progressive mental deterioration, retrograde amnesia, inability to learn, confabulation).

A neonatology resident examines a new-born male baby, born to an alcoholic mother for birth defects. The infant was delivered 10 minutes ago, vaginally at full-term. The resident notices that the infant has facial abnormalities such as smooth philtrum, thin vermillion border, and small palpebral fissures. The resident also documents the height, weight, and head circumference. The infant is small for gestational age with the weight and length being below the 10th percentile. He suspects fetal alcohol syndrome (FAS) in the infant. The criteria for the diagnosis of FAS require the documentation of facial dysmorphia, growth deficits. What other abnormalities may be found? A Respiratory tract abnormalities B Renal abnormalities C Gastrointestinal tract abnormalities D Cardiovascular abnormalities E Central nervous system abnormalities

A Respiratory tract abnormalities B Renal abnormalities C Gastrointestinal tract abnormalities D Cardiovascular abnormalities *E Central nervous system abnormalities* Consumption of alcohol during pregnancy results in adverse neonatal and pediatric effects. The most serious outcome is fetal alcohol syndrome (FAS), the diagnosis of which is based on the presence of 3 criteria. The criteria for the diagnosis of FAS require the presence of all 3 findings: • Evidence of all 3 facial abnormalities-smooth philtrum, thin vermillion border, and small palpebral fissures; • Documentation of growth deficits- small for gestational age (weight and length < 10th percentile); and • Documentation of CNS abnormalities-structural, functional, or neurological, or a combination of all The CNS abnormalities include: • Structural abnormalities such as: o Microcephaly, where the head circumference is at or below the 10th percentile adjusted for age and sex o Brain imaging studies documenting clinically significant abnormalities, often involving various brain regions including the corpus callosum, basal ganglia, cerebellum, and hippocampus. • Functional deficits involve: o Cognitive or developmental deficits or discrepancies o Motor functioning delays o Executive functioning deficits o Social skills impairment o Problem with attention or hyperactivity o Others such as sensory problems, pragmatic language problems, and memory deficits. • Neurological deficits: neurological problems not arising due to fever or a postnatal insult The criteria for the diagnosis of FAS include facial and CNS abnormalities along with growth deficits. Respiratory, renal, gastrointestinal tract, or cardiovascular abnormalities are not the components involved in the criteria for diagnosis of FAS.

A 77-year-old female has been treated for the last two weeks for a community-acquired pneumonia. While on oral azithromycin, the patient continues to develop fevers, some as high as 103 0 F. Her oral intake has decreased, and her effort to breathe continues to be labored. On examination, the patient continues to have rhonchi and some mild rales that are best appreciated in the anterior right lung region. A follow-up chest x-ray reveals a consolidated infiltrate of the right middle lobe. A CT of the chest reveals a loculated, fluid-filled area of the right middle lobe with no evidence of a foreign body. Based on these new findings, what is the most likely pathogen causing this ongoing infection? A Staphylococcal aureus B Pseudomonas aeurginosa C Hemophilus influenza D Klebsiella pneumoniae E Chlamydia pneumoniae

A Staph Aureus This patient is having a history and physical exam that is consistent with an empyema. This loculated collection of fluid will harbor bacteria, the most common pathogen being Staphylococcus aureus. Intravenous antibiotics as well as surgical drainage are warranted.

A 17-year-old female presents to your office with intermittent menstrual pain. She is sexually active with a single male partner, consistently utilizing condoms. She reports that she has had this pain before, most commonly two weeks before her period, and that it has been increasing in severity over the past few months. You perform a pelvic examination and she has no lesions, discharge, or discomfort on bimanual and speculum exam. She is urinary chorionic gonadotropin (UCG) negative. Of the following, what is the most appropriate treatment for this adolescent? A Tell her she has mittelschmerz and prescribe an anti-inflammatory p.r.n. B Tell her she has primary dysmenorrhea and send her home on pain medications. C Recommend that she see a gynecologist to have an exploratory laparoscopic surgery to rule out endometriosis. D Refer her for a vaginal ultrasound to rule out uterine fibroids. E Draw a serum β-hCG.

A Tell her she has mittelschmerz and prescribe an anti-inflammatory p.r.n. Midcycle pain (mittelschmerz) is common in women with regular menstrual periods who are not taking birth control pills. These patients may commonly have midcycle spotting caused by an estrogen surge. There is no fever and no other abnormal bleeding such as that resulting from trauma to the cervix (e.g., coitus, douching). Pain usually occurs over several cycles. There is no history of intermittent lower abdominal pain. Examination at the time of mittelschmerz may reveal some lower quadrant tenderness with or without rebound. Bimanual examination may show localized tenderness. A palpable ovary may be present, but a history of regular menses, lack of fever, and negative pregnancy tests confirm the diagnosis. Mild analgesics, especially nonsteroidal anti-inflammatory drugs (NSAIDs), and reassurance are usually adequate for these patients.

The inadequate availability of which vitamin, listed below, can cause diarrhea and anemia of an infant who is breast fed by a strict vegetarian mother? A Vitamin A B Vitamin B12 C Vitamin C D Vitamin D E Vitamin K

A Vitamin A B Vitamin B12 C Vitamin C D Vitamin D E Vitamin K Inadequate dietary intake of vitamin B12 is seen in strict vegetarians, and this can lead to vitamin deficiency of the breast-fed infant. The abnormalities include anemia, diarrhea, weight loss, and subacute degeneration of the spinal cord. The diagnosis can be confirmed by the demonstration of subnormal level of vitamin B12. In older children, the usual cause of vitamin B12 deficiency is absorptive problem (lack of gastric intrinsic factor, small bowel diseases like Crohn's disease, fistulas, blind-loop syndrome).

Which of the following sex chromosome linked diseases has the clinical feature of short stature, streak gonads, and degradation of oocytes by 2 years of age? A Wiskott-Aldrich syndrome B Turner syndrome C Klinefelter syndrome D Duchenne-Becker muscular dystrophy E Hemophilia A

A Wiskott-Aldrich syndrome *B Turner syndrome* C Klinefelter syndrome D Duchenne-Becker muscular dystrophy E Hemophilia A Turner syndrome is characterized by an X0 genotype. In most cases, the missing X chromosome is paternal. More than 90% percent of embryos with this abnormality are aborted. Phenotypic females have infantile external genitalia, short stature (less than 5 ft/152cm), and webbed neck with low posterior hairline. The uterus is small and the characteristic "streak gonads" show fibrous tissue without germ cells. Primary amenorrhea is the rule with infertility. Unlike in normal aging, oocytes are degraded by 2 years of age. Cardiovascular abnormalities are common (coarctation of aorta, essential hypertension, dissecting aneurysm). Overall, patients treated with growth hormone and estrogen can have a normal life expectancy.

A 53-year-old woman presents to clinic complaining of amenorrhea, irritability, and hot flashes for 6 months. She would like to try hormone replacement therapy. You are counseling her about possible risks versus benefits. Which of the following is a benefit of combined estrogen progestin therapy? A A decreased risk of breast cancer B A decreased risk of myocardial infarction C A decreased risk of stroke D A decrease in somatic symptoms E A decreased rate of cognitive decline

A decrease in somatic symptoms D The WHI study showed increase chance of cardiovascular risks and breast cancer, and showed no improvement in prevention of cognitive decline.

A 21-year-old man is brought to the emergency room by his friends. One hour earlier he started having excessive retching and he vomited violently several times. He has been drinking for the last 3 hours. The vomitus was mainly bright red blood (about 2 quarts) and dizziness followed. What is the likely causer of this clinical picture? Answer Choices 1 Acute gastritis 2 Carcinoma of stomach 3 Acute esophagitis 4 A gastroesophageal mucosal tear 5 Rupture of the lower esophagus

A gastroesophageal mucosal tear This is a classic presentation of Mallory-Weiss syndrome. Vomiting and retching may cause a tear that involves only the mucosa and is not transmural. The tear usually involves the gastric mucosa near the squamous to columnar mucosal junction, but it may also involve the esophageal mucosa. A wide variety of causes such as gastroenteritis, alcohol binging, ulcers, and hiatus hernia have been cited. In fact, any condition which causes retching and vomiting can cause the tear. Patients present with upper gastrointestinal bleeding that may be severe. Most patients recover with only conservative management, but those with severe arterial bleeding require surgery.

You are performing an annual physical examination on a 14-year-old girl. In the last couple years, she started having constipation "every other day or so," followed by loose stools; she still feels depressed because of her parents' recent divorce. She has no problems at school; her grades are good; and she participates in sport activities. Her mother is worried because she is so slim (despite excellent appetite), has not grown enough, and still has not gotten her period (her mother had her first period when she was 13-year-old). The rest of personal and family history is not contributing. Your patient's height is 5 ft, weight 79 lbs (BMI 15,46; percentile 3%); she is in Tanner stage 2 (the same as the last year according to her records); and her bone age is 12.5 years. The rest of physical examination is normal. Laboratory shows Hct of 31% and MCV of 73, low insulin-like growth factor (IGF), low FSH, positive anti-tissue transglutaminase antibodies, and the rest of laboratory results are within normal limits, including TSH and prolactin. Question What will restore normal puberty, growth, and weight in your patient? Answer Choices 1 A gluten-free diet 2 Ferrous sulfate 3 Fluoxetine 4 Estrogen 5 Growth hormone

A gluten free diet In a patient with gastrointestinal symptoms, anemia, pubertal and delayed bone maturation, weight gain and linear growth, with positive anti-tissue transglutaminase antibodies, you should consider celiac disease. A gluten-free diet will restore both her physical and sexual development. Sideropenic anemia is a consequence of celiac disease. Iron supplements will not restore her pubertal, growth, and height development. She may feel depressed, but psychogenic delay is a diagnosis of exclusion, whereas here you have a probable organic cause of the delay that shall be addressed. Sex steroid replacement is probably not needed because her hypothalamic-pituitary-gonadal axis and LH&FSH secretion will resume after her nutritional status improves. IGF levels may be low in cases of nutritional deficiencies and are not diagnostic of growth hormone deficiency. Growth hormone therapy is not needed because her normal growth will probably resume after her nutritional status is corrected.

A patient who resides in northern Arizona presents with signs and symptoms that are consistent with pneumonia. He is afebrile and appears non-toxic. His physical exam is unremarkable, and blood work is within normal limits. A chest x-ray is ordered and reveals bilateral upper lobe infiltrates. Based on this information, what is the best medication treatment for this patient? A Itraconazole B Erythromycin C Oseltamivir D Doxycycline E Amoxicillin

A itraconazole Patient is presenting with clinical signs and symptoms of fungal pneumonia (coccidiomycosis). The presentation of being afebrile, along with upper lobe infiltrates bilaterally in a region of the country that consistently has more fungal pneumonia's would lead the clinician with the reasoning that a fungal process is present. The azole's are the best treatment for fungal pathogens.

A 23-year-old woman presents with complaints of pelvic discomfort and a vaginal discharge for the past 3 days. She finished her period last week. She is taking oral contraceptives as directed. Her medical history is significant for a therapeutic abortion with no other hospitalizations or pregnancies. She has had three sexual partners in the past 6 weeks and does not use condoms. Her most recent partner reported that he was treated recently for gonorrhea. On examination, she has a mucopurulent discharge with "strawberry" cervix on speculum examination. After collecting the appropriate specimens, the best therapeutic option for this patient is A ofloxacin 400 mg i po × 1 dose plus azithromycin 250 mg iiii po × 1 dose B fluconazole 150 mg i po × 1 dose C metronidazole 500 mg iiii po × 1 dose D ceftriaxone 250 mg IM × 1 dose

A ofloxacin 400 mg i po × 1 dose plus azithromycin 250 mg iiii po × 1 dose A Clinical presentation is consistent with cervicitis in a young woman with risk factors for sexually transmitted infection. She has likely been exposed specifically to Neisseria gonorrhea. Coinfection with Chlamydia trachomatis is common. While test results are pending, the Centers for Disease Control and Prevention in its 2006 Guidelines for STD treatment recommend treating for both with single doses (improved compliance) of ofloxacin and azithromycin first-line. In areas with quinolone resistance, intramuscular ceftriaxone is an option but coverage for Chlamydia is still necessary. Metronidazole is the appropriate therapy for trichomoniasis and fluconazole for vaginal candidiasis.

A 65-year-old woman presents with a 2-month history of rectal bleeding; initially, it was thought to be hemorrhoidal in nature. She has a history of hypertension, but she is otherwise healthy. Examination is notable for a large circumferential tumor 2 cm from the anal verge. The hemoglobin is 7 gm/dl. A biopsy performed several days later shows anal melanoma. What statement regarding anal melanoma is true? Answer Choices 1 Anal melanoma is the most common of all anal cancers 2 A positive S100 stain makes the diagnosis 3 It is a slow growing tumor 4 This tumor metastasizes infrequently 5 The prognosis is good following abdominoperineal resection

A positive S100 stain makes the diagnosis Anal melanoma is a rare cancer and accounts for 2% of all anal cancers. Anal melanoma is an aggressive malignancy easily confused with benign hemorrhoids. The presentation is usually that of intermittent rectal bleeding. Sometimes these lesions are not pigmented and may be difficult to identify grossly. Immunohistochemical study of positive S100 stain makes the diagnosis. Unfortunately, melanoma of the anal canal is an aggressive disease. Melanomas commonly metastasize through the lymphatic and hematogenous routes, typically to the liver, lung, bone, and brain. If metastases occur in the GIT, the most common sites include the small bowel, colon, and anorectum. Commonly, the patients have distant disease with metastasis to the lungs and liver. Wide local excision is recommended as primary therapy for primary anorectal melanoma. Comparative studies of the survival of patients who underwent APR with those who underwent wide local excision revealed no statistically significant advantage at all disease stages; therefore, APR should only be performed when local excision is not possible or for palliative purposes. The prognosis for anorectal melanoma is poor, irrespective of surgical treatment performed. Squamous cell carcinomas are the most common type of anal cancer.

A 22-year-old sexually active woman presents for her annual gynecologic evaluation. She reports one partner for the past 6 months and takes oral contraceptive pills as directed. Her periods have been regular. Her examination is unremarkable and her Pap smear returns with atypical squamous cells of undetermined significance and positive for human papillomavirus-16. The next most appropriate step for this patient is to A proceed with colposcopy B repeat Pap smear in 12 months C repeat Pap smear in 24 months D schedule her for a loop electrosurgical excision procedure (LEEP)

A proceed with colposcopy Human papillomavirus subtypes 6, 11, 16, and 18 increase risk for the development of cervical cancer. In a young woman over 21 years old with atypical squamous cells of undetermined significance and positive HPV 16 subtype, the next step in evaluation is the colposcopic evaluation. Alternatively, she could be followed with Pap smears at 6 and 12 months. The LEEP procedure is indicated for those with recurrent histologic finding of cervical intraepithelial neoplasm grade 2 or 3.

A 16-year-old boy with no significant past medical history presents to learn the results of a lipid panel that was performed. The lipid panel was ordered due to periorbital and extensor tendon xanthomas on the patient's body. The patient consumes a low fat and low calorie diet, and he exercises daily. He denies any bothersome symptoms, and his physical exam is otherwise unremarkable. Question What is the most likely mechanism for this patient's metabolic disorder? Answer Choices 1 There is increased production of high density lipoprotein 2 This patient has undiagnosed diabetes mellitus 3 Hypersecretion of triglyceride-carrying chylomicrons from the small intestine 4 A reduction in the amount of LDL receptors on hepatocytes 5 Increased VLDL synthesis by the liver

A reduction in the amount of LDL receptors on hepatocytes LDL (low density lipoprotein) is removed from the circulation by either LDL receptors on hepatocytes or by scavenger cells such as monocytes or macrophages. When there is a decrease in LDL receptors on hepatocytes, or when LDL levels exceed receptor availability, the amount of LDL that is removed by scavenger cells is greatly increased. The LDL receptors are deficient or defective in the genetic disorder known as familial hypercholesterolemia (type 2A). Patients with this primary hypercholesterolemia disorder may be heterozygotes or homozygotes, with homozygotes developing cutaneous xanthomas and myocardial infarctions in childhood. HDL (high density lipoprotein) is synthesized in the liver and transports cholesterol from the peripheral tissues back to the liver; an inverse relationship exists between HDL levels and the development of atherosclerosis. Chylomicrons are involved in the transport of dietary (exogenous pathway) triglycerides and cholesterol that have been absorbed from the gastrointestinal tract, which are then transported to the skeletal muscle and adipose. This patient denies a high triglyceride diet. The production of VLDL (very low density lipoprotein) is increased by the liver as a result of a high calorie diet, obesity, and diabetes mellitus. VLDL represents the primary pathway for transport of endogenous triglycerides produced in the liver; these are then carried to muscle and fat cells. Elevated VLDL levels are not consistent with familial hypercholesterolemia (type 2A). Diabetes mellitus is a cause of secondary hypercholesterolemia, not primary hypercholesterolemia.

A 23-year-old primiparous woman visits her primary care provider for follow up with her pregnancy. Today her gestational age is 25 weeks. Her pregnancy has been uneventful so far. She has no personal or family history of diabetes mellitus. Her height is 5 feet 9 inches and her weight is 140 lbs. She has no chief complaints. The physician would like to screen for gestational diabetes and opts to perform a 50-g oral glucose challenge test. Regarding performance and interpretation of this test, what is true? Answer Choices 1 A standard fasting 50-g glucose challenge test with a glucose level of ≥100 mg/dL after 1 hour is abnormal and warrants further testing 2 A standard nonfasting 50-g glucose challenge test with a glucose level of ≥135 mg/dL after 1 hour is abnormal and warrants further testing 3 A standard fasting 50-g glucose challenge test with a glucose level of ≥110 mg/dL after 1 hour is abnormal and warrants further testing 4 A standard nonfasting 50-g glucose challenge test with a glucose level of ≥110 mg/dL after 1 hour is abnormal and warrants further testing 5 A standard fasting 50-g glucose challenge test with a glucose level of ≥95 mg/dL after 1 hour is abnormal and warrants further testing

A standard nonfasting 50-g glucose challenge test with a glucose level of ≥135 mg/dL after 1 hour is abnormal and warrants further testing Explanation Gestational diabetes affects approximately 5 - 6% of pregnancies in the United States. Most obstetricians adopt a policy of universal screening of pregnant women between 24 and 28 weeks gestational age using the oral 50 g glucose challenge test. The test is performed by administering a 50-g oral glucose load without fasting before the test. The test has a low specificity with handheld monitors; therefore venous blood should be used to obtain blood samples for glucose measurement. Most clinicians use a value above 135-140 mg/dL to prompt the need for follow-up testing (usually with 3-hour 100-g oral glucose testing, given after an overnight fast).

The effect of steroid therapy is evaluated in an 8 year-old African-American boy being treated for fatigue and generalized edema following a "bad cold". His vitals are T 37 C, pulse 90/min, RR 20/min and BP 110/70. Physical exam reveals presence of mild periorbital edema and marked peripheral edema in hands and feet with the remainder of exam within normal limits. Lab values include dipstick urine protein 3+; urine protein 50 mg/m2/hr (<40 mg/m2/hr) Specific gravity 1030 (1008-1020) Urine protein/creatinine ratio 2.0/gm creatinine (< 0.2/gm creatinine) Serum albumin 3.9 (5.9-8.0 gm/dl) Cholesterol 250 (112-247 mg/dl) Remainder of laboratory values including BUN and plasma creatinine are within normal limits. Question What additional treatment should be initiated in this patient to decrease the risk of chronic kidney disease? Answer Choices 1 Diuretics 2 Spironolactone 3 ACE inhibitor 4 Beta blocker 5 Mixed alpha + beta antagonist

ACEI Explanation The pediatric patient described is apparently suffering from nephrotic syndrome. Glomerular disease induced proteinuria is the most common cause of nephrotic syndrome in children due to damage to the glomerular filtration barrier resulting in leakage of plasma proteins into the glomerular ultrafiltrate. Signs and symptoms in children include edema, urine protein: creatinine ratio > 0.2/gm creatinine; heavy proteinuria (urine protein >40 mg/m2/hr), hypoalbuminemia, and hyperlipidemia. The nephrotic range of proteinuria in children is higher than in adults (> 40 mg/m2/hr). Angiotensin-converting enzyme inhibitors and angiotensin receptor blockers not only lower blood pressure but have that additional benefit of slowing the progression of kidney disease even in patients with normal blood pressure. Although this child is normotensive and is already receiving steroid treatment, the next best additional treatment, therefore, is an ACE inhibitor or an angiotensin receptor blocker (ARB) to decrease the proteinuria and GFR decline in order to reduce the risk of chronic kidney disease. Diuretic is incorrect. Although a diuretic would provide symptomatic relief for the edema, it would not prevent the development of chronic kidney disease in this patient. Spironolactone is incorrect. Spironolactone is an inhibitor of renal aldosterone effects such as sodium and water retention and would help combat the edema in this patient but would not decrease the risk of chronic kidney disease. Beta blocker is incorrect. Beta blockers can be useful in the treatment of hypertension but this patient has not yet developed hypertension. Mixed alpha and beta antagonist is incorrect. The use of a mixed alpha and beta antagonist drug is not indicated in this normotensive patient and would likely not reduce the risk of chronic kidney disease.

An obese 45-year-old woman is requesting medical weight loss therapy. She reports excessive weight gain (37 kgs) over the past 3 years. A detailed history also reveals easy bruising, oligomenorrhea, and increased hair growth in various parts of her body. A thorough examination by the attending doctor shows hypertension (B.P 180/110), truncal obesity with a buffalo hump, and moon face. He also notes hirsutism and pigmentation, with purple abdominal and lower leg striae. Lab reports confirm the diagnosis of Cushing's Syndrome. What substance's high serum levels cause hyperpigmentation in Cushing's Syndrome? Answer Choices 1 Adrenocorticotropic hormone 2 Androgens 3 Cortisol 4 Luteinizing hormone 5 Follicle stimulating hormone

ACTH Explanation Hyperpigmentation in Cushing's Syndrome is due to high serum levels of adrenocorticotropic hormone. Skin pigmentation is a special feature of ectopic ACTH Syndrome. It is a result of alpha melanocyte stimulating hormone (MSH) sequence, which is part of the ACTH molecule. In addition, the rest of the ACTH precursor molecule contains two other MSH sequences. Unlike pituitary tumors secreting ACTH, ectopic tumors have no residual feedback thus there is marked pigmentation. Hyperpigmentation is seen in 4-16% of patients with Cushing syndrome. Additionally, ectopic ACTH is the second most common cause of Cushing syndrome. In patients with pituitary dependent Cushing's disease, treatment with bilateral adrenalectomy may lead to the development of a locally invasive pituitary tumor with high levels of ACTH and hyperpigmentation. Hyperpigmentation is not due to high levels of androgens, cortisol, luteinizing hormone, or follicle stimulating hormone. Some clinical features are more common in ectopic ACTH Syndrome.

A 2-year-old boy presents with a firm, painless mass in his right testicle. It is determined that he has an endodermal sinus tumor. What tumor marker is most likely to be elevated? Answer Choices 1 Alpha-fetoprotein 2 5 hydroxytryptamine 3 Gastrin 4 Vasoactive intestinal peptide (VIP) 5 Acid phosphatase

AFP Explanation An endodermal sinus tumor is also called a yolk sac tumor, infantile embryonal carcinoma, embryonal adenocarcinoma of the prepubertal testis, or orchioblastoma. The presence of alpha-fetoprotein is very typical. Alpha-fetoprotein is seen with a variety of tumors, such as hepatocellular carcinoma, pancreatic carcinoma, testicular tumors, and others. 5-hydroxytryptamine is serotonin. Serotonin is an indolamine. The precursor for serotonin is tryptophan. Serotonin is a neurotransmitter. Serotonin is also the precursor to melatonin. As a tumor marker, 5-hydroxytryptamine (serotonin) is the major product seen with carcinoid tumors. Gastrin is a hormone that is ordinarily secreted by cells within the stomach. Specifically, gastrin is secreted by G cells, which are in the stomach antrum. Gastrin stimulates acid secretion of the stomach. Pathologically, gastrin can be produced by pancreatic islet cells tumors. Vasoactive intestinal peptide is sometimes abbreviated as VIP. Vasoactive intestinal peptide can be seen with islet cell tumors and pheochromocytoma. The prostate has the enzyme acid phosphatase. Acid phosphatase is actually a group of enzymes, which can be found in a few other tissues as well. With the development of prostate cancer, elevated serum acid phosphatase can be seen.

A 56-year-old man presents with weakness of his grip. On exam, the problem is bilateral. During the next few office visits, you note the development of hyperactivity of his deep tendon reflexes (DTRs), extensor plantar reflexes, and dysarthria. The patient's sensory system remains normal, and he denies any urinary symptoms. Question What is the most likely diagnosis? Answer Choices 1 Multiple sclerosis 2 Alzheimer's dementia 3 Huntington's disease 4 Amyotrophic lateral sclerosis 5 Myasthenia gravis

ALS The clinical picture is suggestive of amyotrophic lateral sclerosis (ALS). ALS is a progressive bilateral muscle disease that causes fasciculations and hyperreflexia, plantar reflexes, and dysarthria. Sensation and bladder function remain normal. The common clinical presentation seen in multiple sclerosis is weakness, numbness, tingling, or unsteadiness in a limb; spastic paraparesis; retrobulbar neuritis; diplopia; disequilibrium; or a sphincter disturbance such as urinary urgency or hesitancy, which is not present in this patient. Alzheimer's patients present with memory impairment and at least 1 of the following: language impairment, apraxia, agnosia, and impaired executive function. Rigidity and bradykinesia are primary signs; tremors are rare. These symptoms are not present in this patient. Huntington's disease is characterized by chorea and dementia. Clinical onset is usually between 30 and 50 years of age. Patients present with writhing choreiform movements of the body, which are not present in this patient. In myasthenia gravis, small motor units (such as ocular muscles) are most often affected, producing the nystagmus. Weakness that improves after a period of rest and fluctuating fatigue are the major clinical features of this disease, and these symptoms improve after administering acetylcholinesterase inhibitors. These symptoms are not present in this patient.

A 15-year-old boy presents with abdominal pain and rectal bleeding. His family history is significant for the premature deaths of his mother and maternal grandmother from metastatic colon cancer. Both died before the age of 35 years; in both, autopsy findings included hundreds of colon polyps along with multiple primary colon cancers. Endoscopy of the boy also demonstrates extensive colonic polyp disease. Question A mutation in what gene is causing this patient's symptoms? Answer Choices 1 BRCA1 2 BRCA2 3 MSH2 4 APC 5 RET

APC The presentation of hundreds of colonic polyps often 'carpeting' the large bowel, in multiple family members, is typical for familial adenomatosis polyposis syndrome. The typical course is for polyps to develop in late childhood and the early teenage years, with transformation of 1 or more polyps to colon cancer in the 3rd or 4th decade. Mutations in the APC gene, a known tumor suppressor gene, cause this syndrome. Mutations in BRCA1 and BRCA2 lead to breast cancer and ovarian cancer; the highest risk is seen in women, although some men with BRCA2 mutations may develop breast cancer. BRCA1 and BRCA2 are not associated with colonic polyposis. MSH2 mutations are a cause of hereditary non-polyposis colon cancer syndrome (HNPCC); this leads to colon cancer without extensive polyposis, although a few polyps may be present in some cases. Mutations in the RET gene are a cause of multiple endocrine neoplasia syndrome type II (MEN-II) which is typically associated with medullary thyroid cancer, parathyroid adenomas, and pheochromocytomas. Gastrointestinal ganglioneuromas are sometimes also found.

Your patient is taking atorvastatin 40 mg daily for her history of hyperlipidemia. Based on the potential side effects of this medication what labs are indicated for periodic monitoring? A HGB/HCT B AST/ALT C WBC count D TSH/T4 E B12

AST/ALT The correct answer is (B). Atorvastatin is an HMG-CoA reductase inhibitor, which has a potential to cause liver injury. It is recommended that liver enzymes are monitored regularly due to the potential for liver injury. The remaining choices are incorrect because statins are not known to cause changes in these laboratory values and monitoring based on potential adverse reactions to atorvastatin is not recommended.

A 55-year-old woman presents with poor appetite and nausea. She has vomited 2 times over the past week and lost 4 pounds in the past month. Past medical history is significant for 20 years of alcoholism, hypertension, and diabetes for the past 5 years. She takes no medications; she is not involved in any therapy for her alcoholism. She has been drinking 2 pints of beer almost every day for the past 20 years, and consumes greater quantities on weekends. Her vitals include a heart rate of 102 BPM, blood pressure of 140/100 mm Hg, respiratory rate of 20/min, and a temperature of 99.8°F. Physical exam reveals hepatomegaly. A liver biopsy reveals ballooning degeneration, spotty necrosis, and polymorphonuclear infiltration. Question What lab finding would be characteristic of this patient's condition? Answer Choices 1 AST/ALT >2 2 Reduced γ-Glutamyl transpeptidase 3 Reduced alkaline phosphatase 4 Reduced prothrombin time 5 Microcytic hypochromic anemia

AST/ALT>2 The patient's history of alcoholism, physical findings, and liver biopsy results point towards a diagnosis of alcoholic liver disease (ALD), where typically the AST/ALT >2 (the ratio of serum aspartate aminotransferase to serum alanine aminotransferase). Patients with increased alcohol consumption may not have a ratio >2, and it may be more of an indication of advanced alcoholic liver disease. ALD is a result of the consumption of large quantities of alcohol over a long duration of time. The threshold for men is 60 - 80 g/d for > 10 years. Initially there is a fatty infiltration of the liver, which gradually progresses to alcoholic hepatitis. Alcoholic hepatitis is characterized by hepatocyte injury, as revealed by the biopsy findings in this patient. Lab findings include a moderately elevated AST, ALT, γ-Glutamyl transpeptidase, and alkaline phosphatase. Both AST and ALT are elevated by 2 - 7 fold, and AST is greater than ALT level (AST/ALT >2). Bilirubin is also elevated. Prothrombin time is >5 seconds (normal is 11 - 15 seconds), and macrocytic anemia may also be observed.

A 63-year-old woman presents with acute onset of abdominal pain. She describes it as a steady deep discomfort in the left lower quadrant. Initially she was constipated, but is now experiencing diarrhea. On physical examination, she has a temperature of 38°C. The abdomen is tender in the LLQ with guarding and rebound tenderness. She has positive fecal occult blood. What is the best test to determine her most likely diagnosis? Answer Choices 1 Abdominal ultrasound 2 Abdominal CT 3 Barium enema 4 Upper GI series 5 Colonoscopy

Abdominal CT Explanation Diverticulitis is a common cause of left lower quadrant abdominal pain that is associated with fecal occult blood. It is best diagnosed with abdominal CT. Findings of sigmoid diverticula, a thickened colonic wall and inflammation within the pericolonic fat suggest the diagnosis. While an ultrasound is less expensive and non-invasive, it is less specific and operator dependent. Barium enema and colonoscopy will both demonstrate diverticula, but there is risk of perforation with these modalities. An upper GI is not indicated in this patient with lower abdominal pain.

A 60-year-old man presents with severe abdominal pain that started 10 hours ago. It is increasing in severity and is colicky in nature. The patient has not had a bowel movement for 3 days. In the last 2 days, he vomited up what he ate 4 times. He looks tired and dehydrated. On examination, increased bowel sounds were noticed. There is also a mass in the right inguinal area. The patient said he the mass has been present for 10 years but disappears when he lies on his back. Question What is the investigation of choice to confirm the diagnosis? Answer Choices 1 Abdominal X-Ray Erect film 2 Abdominal Ultrasound 3 CBC 4 Colonoscopy 5 Upper GIT endoscopy

Abdominal X-ray Erect film. Intestinal obstruction occurs when bowel movement is encountering an obstacle in the passage through the bowel. Actually this can be caused by any mass, adhesion, or any other means of mechanical obstruction. The symptoms of obstruction vary according to the site of obstruction, but generally there is increased bowel movement proximal to obstruction with the reflected increase in bowel sounds. There may be vomiting. There may be no bowel movement, from absolute constipation to feces and flatus. Here the patient has inguinal hernia which is recently irreducible and obstructed. The best diagnostic procedure is erect X-ray film which shows multiple fluid levels and gives confirmation of the diagnosis. The treatment is exploratory surgery, and there is no place for any conservative treatment without surgery. Paralytic ileus is a type of intestinal obstruction that usually follows general anesthesia and is characterized by silent abdomen with no bowel sounds, usually treated by bowel rest and proper hydration. Colonoscopy here is contraindicated. Upper GIT endoscopy is generally used to detect pathology in upper GIT, not in the case of intestinal obstruction. Abdominal ultrasound is of value in diagnosis of gall bladder disease and also urinary bladder pathology. CBC is generally used to detect leukocytosis in acute appendicitis or inflammatory reactions.

A 41-year-old man presents with right upper quadrant pain. His pain began gradually following a meal, but it has now become constant. He notes that he has had previous episodes of similar pain, but it has never been quite this severe. The pain radiates to his right shoulder and is worsened with inspiration. He has experienced nausea and vomiting, and he notes feeling chilled. Examination reveals an overweight man in moderate distress. He develops rigors during the physical exam. He has scleral icterus, and his skin has a yellow hue. Heart and lungs are clear. His abdomen is soft and non-distended with positive Murphy's sign. Vital signs reveal BP 109/62, pulse 112, respirations 18, and temperature 102.3 degrees F. Laboratory studies include: Component Value: WBC-15.66 k/uL Hemoglobin-15.5 g/dL Hematocrit-43.8 % Platelets-128 k/uL Albumin-3.4 g/dL Total Bilirubin-6.3 mg/dL Conjugated Bilirubin-4.5 mg/dL Alkaline Phosphatase-114 U/L AST-97 U/L ALT-132 U/L Total Protein-5.6 g/dL Glucose-104 mg/dL BUN-11 mg/dL Creatinine-1.05 mg/dL Sodium-139 mmol/L Potassium-4.3 mmol/L Chloride-102 mmol/L CO2-27 mmol/L Calcium-8.5 mg/dL Amylase- 36 U/L Lipase- 23 U/L Question What is the most appropriate initial diagnostic study for this patient? Answer Choices 1 Abdominal X-ray 2 Abdominal MRI 3 Abdominal ultrasound 4 HIDA scan 5 ERCP

Abdominal u/s Abdominal ultrasound is correct. The above patient is suffering from right upper quadrant pain, jaundice, and fever, which are all part of the Charcot triad associated with cholangitis. Cholangitis is defined as infection of the bile ducts. It typically occurs when the bile ducts are obstructed, leading to infection. Given his history of postprandial right upper quadrant pain, the patient likely has cholelithiasis. His laboratory data is suggestive of biliary obstruction. The best initial testing for gallstones is abdominal ultrasound. Abdominal X-ray is incorrect. Plain abdominal X-ray may demonstrate radiopaque gallstones, but it is not the best test to demonstrate cholelithiasis because it may miss radiolucent stones. Abdominal MRI is incorrect. While abdominal MRI may show cholelithiasis, it is not the preferred initial study for suspected cholangitis. HIDA scan is incorrect. Hepatobiliary iminodiacetic acid (HIDA) scan may be used to demonstrate ductal obstruction, but it is typically done following abdominal ultrasound. ERCP is incorrect. If the patient in this case has obstructing stones, he may require endoscopic retrograde cholangiopancreatography (ERCP) for stone removal. However, ERCP would not be the initial study performed.

A 9-year-old boy presents after an episode that his mother describes as "zoning out". He was eating dinner when he looked up and began to stare at the wall. He would not respond to her verbal questioning. Almost immediately, he lost control of his bladder. She stated that he then looked at her and realized he had wet himself; he then began to cry. Question What is the most likely diagnosis? Answer Choices 1 Grand mal seizure 2 Simple partial seizure 3 Complex partial seizure 4 Absence seizure 5 Atypical absence seizure

Absence Seizure The correct answer is absence seizure; they are characterized by an impairment of consciousness, occasionally with mild tonic, clonic, atonic, or autonomic symptoms. The patient is typically unaware of the attacks. They almost always begin in childhood, and they end by age 20. Atypical absence seizures are those characterized by more striking changes in tone; seizures may be more gradual in onset and completion than typical absence seizures. Patients with simple partial seizures do not experience a loss of consciousness. Complex partial seizures typically have impaired consciousness with focal motor symptoms or somatosensory symptoms, and they do not have the 3 phases. Grand mal seizures are characterized by a tonic phase, which includes a sudden loss of consciousness and rigidity that typically lasts less than a minute. A clonic phase follows; it is characterized by jerking motions, and they usually last for a few minutes. The clonic phase is followed by a flaccid coma state.

A mother brings her 10-year-old daughter to the pediatrician's office. Over the past few weeks, the girl's teacher has noticed the child staring off into the distance and pausing in the middle of a sentence. Yesterday, the child started twitching at her desk, but it only lasted a few seconds. The child does not seem to remember any of these claims. The mother states that her pregnancy with the child was without complications, and you note that the child has achieved all growth milestones uneventfully. The child is not currently on any medications. The mother has not noticed any of these behaviors at home. She adds that she and her husband are separating. What is the most likely cause of this child's behavior? Answer Choices 1 Simple partial seizure 2 Absence seizure 3 Emotional stress 4 Attention deficit disorder 5 complex partial seizure

Absence Seizures Absence seizure is correct because this child seems to be exhibiting episodic impairment in her consciousness, as evidenced by staring off into the distance and breaks in communication. She also had some twitching, which could be mild clonus. Absence seizures also typically present in children. Simple partial seizure is incorrect because she would not be exhibiting impaired consciousness during the episodes. The clonic activity would also be more apparent. Emotional stress is incorrect because this patient is exhibiting characteristic activity of an absence seizure. Attention deficit is incorrect because this patient is exhibiting characteristic activity of an absence seizure. Complex partial seizure is incorrect. Even though her consciousness is impaired, her clonic activity is minimal.

Ethosuximide is a primary treatment for what type of epilepsy? Answer Choices 1 Febrile seizures 2 Absence seizures 3 Generalized tonic-clonic seizures 4 Status epilepticus 5 Simple partial seizures

Absence seizures The term epilepsy refers to a group of central nervous system disorders. These disorders are characterized by repeated occurrence of sudden and transitory seizures, which may be of motor, sensory, autonomic, or psychic origin. Seizures are almost always accompanied by abnormal and excessive discharges in the brain. The many types of seizures are classified into 10 different categories. Partial seizures, also known as focal or local seizures, are broken down into 3 groups: simple partial seizures, complex partial seizures, and partial seizures secondarily generalized. The generalized seizures (convulsive and nonconvulsive) are grouped into absence seizures, atypical absence seizures, myoclonic seizures, clonic seizures, tonic seizures, tonic-clonic seizures, and atonic seizures. Ethosuximide and valproic acid are the primary agents for the treatment of absence seizures. These drugs are generally ineffective against generalized tonic-clonic seizures and simple partial seizures, which are best treated with carbamazepine or phenytoin. Febrile seizures, if treated at all, are best treated with phenobarbital. Status epilepticus is a life-threatening disorder that must be treated promptly. Intravenous diazepam and/or phenytoin is indicated for control of this disorder.

A 31-year-old man presents with gastric pain and heartburn. A gastroscopy was performed and a gastric ulcer diagnosed. What medication should he take for the pain? Answer Choices 1 Aspirin 2 Diclofenac 3 Ibuprofen 4 Naproxen 5 Acetaminophen

Acetaminophen Acetaminophen is a mild analgesic and antipyretic without anti-inflammatory properties. The mechanism of action is unknown; it is believed to block generation of pain impulses by inhibiting the synthesis or action of substances that sensitize pain receptors. The antipyretic effect is thought to be by central action in the hypothalamic heat-regulating center. Long-term use can result in liver damage, especially in combination of alcohol abuse. It can lead to neutropenia, leukopenia, pancytopenia, and hemolytic anemia. The drug may cause false positive test results for urinary 5-hydroxyindoleacetic acid and interfere with home glucose testing. Aspirin, acetylsalicylic acid, is an analgesic, antipyretic and anti-inflammatory drug, which inhibits the prostaglandin synthesis. Since prostaglandin also stimulates mucus production and decreases acid production in the stomach blockage, it is reversing those processes. Therefore, aspirin is contraindicated in patients with gastrointestinal ulcers. Emanated from acetylsalicylic acid, many other anti-inflammatories, analgesic, and antipyretic drugs have been developed and comprehended as acid-antiphlogistics. They have the same mode of action and similar side effects. diclofenac, ibuprofen, and naproxen are in this group and therefore contraindicated in patients with gastrointestinal ulcers.

A 64-year-old man presents with a history of progressive dysphagia to solids; recently, he has been having trouble with liquids. He also experiences occasional regurgitation of undigested food. His physical exam is unremarkable. A barium esophagram reveals a distinctive bird's beak appearance of the distal esophagus. Question What is the most likely diagnosis? Answer Choices 1 Gastroparesis 2 Gastroesophageal reflux disease 3 Mallory-Weiss syndrome 4 Achalasia 5 Candidal esophagitis

Achalasia The correct answer is achalasia; it often presents with progressive dysphagia to liquids and solids as well as complaints of regurgitation of undigested food. Classic radiographic findings include a bird's beak appearance in the distal esophagus, demonstrating the narrowing of lumen. Gastroparesis is delayed gastric emptying, not an esophageal disorder that would cause the progressive dysphagia noted by the patient. The vagus nerve is not functioning properly in these cases. Gastroesophageal reflux disease can cause dysphagia, but it is not typical; this condition causes regurgitation of undigested food and does not have the classic appearance on esophagram. Mallory-Weiss syndrome presents typically with hematemesis, typically due to forceful vomiting or retching, which causes tears in the esophagus. It is associated with alcoholism. Candidal esophagitis is a fungal infection of the esophagus often seen in immunocompromised patients, such as those who are HIV infected. It presents with progressive dysphagia, but oral thrush is often present in these patients, and an esophagram will not show the classic appearance of achalasia.

A 35-year-old man presents with a 4-week history of difficulty swallowing food. He has developed chest pain over the last few days, and he has also noticed regurgitation of food. On detailed history, he says that the regurgitation contains undigested fragmented food material; it is unassociated with a foul smell. He has lost 3 kg over the past month. On examination, his pulse is 94 bpm and blood pressure is 130/86 mm Hg. Mild pallor is noted, but there is no icterus. Epigastric tenderness is present. CBC shows a hemoglobin of 9.5 g/dL (total count 7400 cells/mm3 and ESR 12 mm/hr). An X-ray of the abdomen shows an absence of a fundic air shadow. The patient undergoes barium studies which show a dilated esophagus; the lower end appears beak-shaped. Question What is the most likely diagnosis? Answer Choices 1 Achalasia cardia 2 Barrett's esophagus 3 Hiatal hernia 4 Plummer-Vinson syndrome 5 Zenker's diverticulum

Achalasia Cardia Gradual onset of difficulty in swallowing food, the regurgitation of undigested food, the absence of fundic air on a plain X-ray of the abdomen, and a dilated esophagus with a beak-shaped lower end are diagnostic of achalasia cardia. It occurs due to the degeneration of inhibitory neurons in the lower end of esophagus. The most common cause of achalasia is primary/idiopathic. There is a deficiency of inhibitory neurons containing vasoactive intestinal peptide and nitric oxide synthase in this condition. Barrett's esophagus is the metaplasia of the lower esophageal junction; it is usually a consequence of gastroesophageal reflux disorder. Hiatal hernia is usually a sliding type and is not associated with the esophageal narrowing. Plummer-Vinson syndrome is associated with web formation in the upper esophagus; it is associated with iron deficiency anemia in menopausal women. Zenker's diverticulum is a posterior pharyngeal wall abnormality due to dehiscence between the 2 layers of inferior constrictors. Undigested food usually comes back with a foul smell a long time after the ingestion.

A 38-year-old woman is going through a divorce and simultaneously filing bankruptcy. She is very stressed about her financial situation and failed marriage. One day, after a particularly long crying spell, she notices a sudden onset of extreme difficulty swallowing at dinnertime. She has difficulty swallowing both solids and liquids. She feels that the food is sticking in her throat. She ignores it. She has numerous bouts of these episodes of difficulty with swallowing. She notices that when she lies down, undigested food comes up. Finally, she sees her doctor. On questioning by her doctor, she insists she has no heartburn. She has not seen any blood when she regurgitates nor has she vomited blood. Manometry is performed which shows an absence of normal peristalsis and an elevated LES pressure. What is the most likely diagnosis? Answer Choices 1 Achalasia 2 Scleroderma 3 Esophageal varices 4 Gastroesophageal reflux disease 5 Mallory-Weiss syndrome

Achalasia is the right answer it is characterized by dysphagia. There is poor peristalsis in achalasia. This patient has a characteristic history for achalasia. The onset is during a period of stress. The symptoms consist of dysphagia, complaints of food sticking in the throat, and regurgitation. Scleroderma can present with dysphagia. However, manometry would show a decreased LES pressure, not an elevated LES pressure as the case here. In addition, scleroderma can be associated with heartburn. Esophageal varices usually present with hematemesis. Gastroesophageal reflux disease would present with heartburn. If anything, the LES pressure would be lower than normal, rather than elevated. Mallory-Weiss syndrome is an esophageal tear. This would present with hematemesis.

A 30-year-old man presents with excessive thirst for the past few days. He consumes 3 to 4 liters of water per day but is still thirsty. He has also been passing urine very frequently. He gives a history of a road traffic accident 1 month ago, wherein he hit his head against the dashboard. Urine specific gravity and osmolality are 1.002 and 180 mOsm/kg, respectively. There is an increase in urine osmolality with exogenous antidiuretic hormone (ADH) administration. Question What is the probable diagnosis? Answer Choices 1 Idiopathic central diabetes insipidus 2 Familial central diabetes insipidus 3 Acquired central diabetes insipidus 4 Hereditary nephrogenic diabetes insipidus 5 Acquired nephrogenic diabetes insipidus

Acquired Central DI xplanation The correct answer is acquired central diabetes insipidus. Diabetes insipidus (DI) is a condition of excessive thirst and urination. It may be neurogenic (central) or nephrogenic. Central DI is characterized by decreased ADH secretion, resulting in the person's decreased ability to concentrate urine. Causes include tumors of the pituitary, head trauma, and neurosurgery. It may also be idiopathic or familial. Idiopathic central DI is believed to be an autoimmune condition with destruction of cells in the hypothalamus. Familial central DI is extremely rare. The history of head trauma should raise suspicion of acquired central DI. ADH administration causes an increase in urine osmolality in central DI but not in nephrogenic DI. Nephrogenic diabetes insipidus is caused by a resistance to the antidiuretic hormone (ADH) in the kidney. It may be hereditary, which is rare, or acquired. Acquired nephrogenic DI is caused by lithium toxicity, chronic renal disease, hypercalcemia, etc. The patient has polyuria and polydypsia.

Because of the lack of epiphyseal cartilage, adult bone cannot increase in length when stimulated by an excess of growth hormone; however, they do increase in width by periosteal growth. In adults, what does an increase in growth hormone cause? Answer Choices 1 Gigantism 2 Osteomalacia 3 Acromegaly 4 Rickets 5 Osteoporosis

Acromegaly Explanation In adults, an increase in growth hormone causes acromegaly, a disease in which the bones, mainly the long ones, become very thick. An excess of hormone during growing years causes gigantism. Osteomalacia is caused by calcium deficiency in adults; rickets is caused by calcium deficiency in children. Osteoporosis is the decrease in bone mass caused by decreased bone formation, increased bone resorption, or both.

A 30-year-old female presents to your office for a routine physical exam. She has not seen a health care provider in many years. Upon talking with the patient, you find out that she had been diagnosed with hypertension several years ago, but was unable to afford the antihypertensive medications that were prescribed to her. She has no complaints at this time. Upon exam of the head and neck, you note widened spaces between her lower incisor teeth and a large, fleshy nose. Her skin is oily and she demonstrates mild proximal muscle weakness. Her EKG reveals a left axis deviation and widened QRS. What is the most likely rationale for her clinical presentation? A Diabetes mellitus B Cushing's syndrome C Hypothyroidism D Acromegaly E Clinical depression

Acromegaly The correct choice is D, acromegaly. Patients with acromegaly have an abundance of growth hormone secretion. This leads to excessive growth of many areas of the body including soft tissue. Patients with acromegaly also have an increased incidence of hypertension and left ventricular hypertrophy. None of the other choices will cause this patient's constellation of symptoms. Patients with many endocrine disorders may develop weaknesses as seen in this patient, but the large nose and widely spaced teeth are characteristic of acromegaly.

A 27-year-old female is 8 weeks postpartum with her first child and has been exclusively nursing since discharge at the hospital. She has a 5-day history of engorgement in her right breast, which is red, tender, and feels warm to the touch. She states she is feverish but has not taken her temperature. On physical examination you see the breast as shown below. Which of the following is the most likely diagnosis? Source: McPhee SJ, Papadakis, MA: Current Medical Diagnosis and Treatment 2011, 50 th Edition: http://www.accessmedicine.com A Galactorrhea B Fibroadenoma C Acute mastitis D Lobular breast cancer

Acute Mastitis C Postpartum mastitis occurs sporadically in mothers that are nursing and usually develops after hospital discharge. The inflammation is usually unilateral and begins with an engorged breast or sore nipple. Frequently, mastitis begins within 3 months postpartum and affects first time nursing mothers more often. Cellulitis is usually seen in the affected breast and redness, tenderness, local warmth, and fevers and chills are common complaints. Fibroadenoma (B) and galactorrhea (A) do not look like this clinical picture. Gynecomastia (E) affects males. Inflammatory carcinoma can in rare cases be mistaken for mastitis. Lobular carcinoma (D) would not present this way.

A 44-year-old pre-menopausal Caucasian woman with a BMI of 36 presents with persistent upper right quadrant abdominal pain that radiates to the back. It has gotten so bad that she has difficulty eating any food and needs to force herself to eat. She has nausea with some episodes of vomiting. She denies bulimia but admits to anorexia. On exam, patient has a positive Murphy's sign and tenderness to palpation on the epigastric and upper right quadrant area. Patient has a slightly elevated temperature. The physician assistant is awaiting labs and imaging. Question What is the most likely diagnosis? Answer Choices 1 Acute cholecystitis 2 Chronic cholecystitis 3 Appendicitis 4 Cholangitis 5 Gastritis

Acute cholecystitis Patients with acute cholecystitis present with abdominal pain accompanied by nausea and vomiting. Usually the patient consumes a large fatty meal 1 hour before the onset of symptoms; however, the pain is persistent. This usually occurs due to cholelithasis blocking the cystic duct. Patients commonly affected by cholelithasis are those who meet the four Fs: fat, female, forty, and fertile, which describes the patient above. Chronic cholecystitis is usually an intermittent type of pain and a mild inflammation of the gallbladder. This can turn into an acute cholecystitis. Appendicitis is usually right lower quadrant pain that presents as a diffuse abdominal pain that localizes to the right lower quadrant. The patient would not have a positive Murphy sign, but would have McBurney's point, obturator sign, and psoas sign. The patient would have a fever as well. Cholangitis is defined as a jaundice, fever, and right upper quadrant abdominal pain known as Charcot's triad. Gastritis is inflammation of the stomach, which is localized in the epigastric area. This can be caused by numerous reasons but is most commonly secondary to infectious or autoimmune causes.

Acute dystonia

Acute dystonias from neuroleptic medications consist of bizarre muscle spasms of the head, neck, and tongue.

A 32-year-old man comes to the doctor's room complaining of nausea, vomiting, dark urine, and yellow eyes for the last 3 - 4 days. He lives alone, and he admits to having more than 1 sexual partner but denies any intravenous drug use. He usually buys his food from different fast food places. He doesn't remember any contact with anybody who had hepatitis. On examination, he is slightly dehydrated, he has jaundice, tender hepatomegaly, splenomegaly, and the rest of the examination is normal. The doctor suspects viral hepatitis, so he sends bloods to the laboratory to check his hepatitis markers. The results are as follows: Anti-HAV- of IgM type is positive Anti-HAV- of Ig G type is negative HBsAg is positive HBsAb is negative HBcAb of IgG type is positive HBeAg is negative HBeAb is positive Question What is the most likely diagnosis? Answer Choices 1 Resolved hepatitis B and acute hepatitis A infection 2 Acute hepatitis A and B infections 3 Acute hepatitis B and past hepatitis A infection 4 Past hepatitis A and resolved hepatitis B infection 5 Acute hepatitis A infection and inactive carrier hepatitis B state

Acute hepatitis A infection and inactive carrier hepatitis B state

A 28-year-old man with a history of crack cocaine abuse was rushed in to the emergency room. His mother found him in his apartment. He was blue and severely short of breath. No other history is available. His examination revealed a young man in severe respiratory distress with temperature 99.2° F, pulse 102/min, respiration 40/min, BP165/95 mm Hg, and a pulse oximetry of 66%. He was intubated at the site by the EMS personnel because of lack of improvement on 100% non-rebreathing mask. His chest X-ray shows bilateral alveolar and interstitial infiltrates. The arterial blood gas on the mechanical ventilation is set at a rate of 12 cycles/min, tidal volume 500 mL, and PAO2/Fi02 ratio = 100 mmHg; PH 7.52, PCO2 30, PO2 55, and O2 saturation 88%. What is the most likely diagnosis? Answer Choices 1 Acute respiratory distress syndrome 2 Bacterial pneumonia 3 Myocardial infarction 4 Pulmonary embolism 5 Viral pneumonia

Acute respiratory distress syndrome Acute respiratory distress syndrome (ARDS) is an acute lung injury in which the ratio of PaO2/FIO2 is 200 or less and there is bilateral interstitial infiltrates with no evidence of raised left atrial pressure. Common etiologies are aspiration, sepsis, pancreatitis, toxic inhalation burns, multiple blood transfusions, drugs, and trauma. It is characterized by dyspnea, tachypnea, rales, bilateral infiltrates, marked hypoxemia that is not corrected by O2, and leukocytosis. Multi-organ failure may also be present. The patient in question has all of these features, and his PaO2/FIO2 ratio is 100. Treatment of ARDS includes identifying and treating the underlying condition as well as cardiopulmonary support. No pharmacological agents have been shown to be beneficial. Adding positive end-expiratory pressuremay improve oxygenation by opening the fluid-filled alveolar units (alveolar recruitment). Other modalities that may improve oxygenation include prone positioning and inverse ratio ventilation. Bacterial and viral pneumonia presents as cough with or without sputum production, fever, and leukocytosis. It does not present with this degree of shunt hypoxemia. It also does not lead to such rapid deterioration. Myocardial infarction may be precipitated by cocaine use. For it to have caused such infiltrates, however, there would have also been signs of right heart failure and hypotension; it would not have caused so much shunt hypoxemia. Pulmonary embolism can present acutely. It has to be massive to present this way, in which case the patient would be hypotensive, and CXR would not show bilateral infiltrate. Increasing the tidal volume or respiratory rate will not improve his oxygenation since he has very high minute ventilation. An emergent thoracentesis will not improve the patient's status.

A 45-year-old male comes into your family practice office for his second follow-up appointment since being diagnosed with type 2 diabetes. He does not complain of any symptoms. He is currently taking metformin 1000 mg BID and his HgA 1c at this visit 7.6%. You ask him about his diet and if he is regularly monitoring his blood glucose. He says that he does not really watch what he eats, but he does check his glucose levels daily. He tells you that on average his fasting plasma glucose (FPG) is usually around 88 mg/dl and his postprandial glucose (PPG) is around 180 mg/dl. What would be the BEST next appropriate step in management for this patient? A Nothing, his levels are within normal limits. B Consider switching to basal insulin therapy to control his FPG. C Add a dipeptidyl peptidase 4 (DPP-4) inhibitor to control his PPG. D Add a sulfonylurea to control his PPG. E Increase his metformin to 1500 mg twice daily.

Add a dipeptidyl peptidase 4 (DPP-4) inhibitor to control his PPG. DPP-4 inhibitors modulated glucagon-like peptide-1 (GLP-1). Their mechanism of action is thought to result from increased incretin levels, especially GLP-1. GLP-1 inhibits glucagon release which, in turn, results in increased insulin secretion, delays gastric emptying, and decreases serum glucose levels. The class is particularly appropriate to utilize in patients who have near-normal HbA 1C and elevated postprandial serum glucose because they work only when food enters the gut and have little to any chance of hypoglycemia.

A 44-year-old woman presents with near-syncope. She states that, over the past month, she has noticed profound fatigue, nausea, and occasional vomiting. She has also experienced a 15-pound weight loss. On physical exam, she appears profoundly weak. Vital signs are: BP 95/55; pulse 110; respiration 14; temp 37.4° C (99.3° F). Orthostatic evaluation shows a positive tilt. HEENT: NCAT, mucous membranes are dry, and areas of pigmentation are noted. Lung exam is clear. Cardiac exam reveals distant heart sounds and tachycardia. Abdominal exam shows mild, diffuse tenderness, without guarding or rebound. The patient's skin is dry and appears very tanned. No bruising or other rashes are noted. Laboratory studies show Chem 7: glucose 80; Na 130; K 5.2; Cl 100; HCO3 21 BUN 55, creat 2.2. With what is the patient most likely afflicted? Answer Choices 1 Bartter syndrome 2 Liddle's syndrome 3 Cushing's syndrome 4 Addison's disease 5 Functional adrenal insufficiency

Addison's Disease Explanation The overall clinical picture of a patient with primary adrenal insufficiency, or Addison's disease, is that of one who is weak and lethargic with loss of vigor and fatigue on exertion and an inability to withstand even minor stresses without shock. The clinical presentation of Addison's disease can be explained on the basis of a deficiency of cortisol and aldosterone and a lack of feedback suppression of ACTH and MSH. Bartter syndrome is hypertrophy and hyperplasia of the cells of the renal juxtaglomerular apparatus, resulting in a saline-resistant metabolic alkalosis. Liddle's syndrome, or pseudohyperaldosteronism, has a clinical picture of hyperaldosteronism; however, there is normal aldosterone secretion, resulting in a saline-resistant metabolic alkalosis. Cushing's syndrome is a constellation of findings due to an excess of cortisol. The most common etiology is iatrogenic from steroid use for the treatment of other diseases. The syndrome is characterized by weight gain, especially on the abdomen, face (moon face), neck and upper back (buffalo hump), thinning and weakness in the muscles of the upper arms and upper legs, in addition to thinning of the skin with easy bruising striae on the abdomen, thighs, breasts, and shoulders. Laboratory abnormalities include hyperglycemia and mild hypokalemia. Functional adrenal insufficiency exists when administration of exogenous steroids results in depression of ACTH secretion.

A 57-year-old woman with a 40 pack-year history of smoking and a long history of Type 2 Diabetes presents with fatigue, middle abdominal pain, and loss of appetite. She indicates that these bouts have been coming and going for the last few months and she finally decided to get checked out. She indicates that she has also lost considerable weight. On physical examination, she appears mildly jaundiced. You perform a CT scan that reveals a mass in the pancreas. Question What is the most likely diagnosis? Answer Choices 1 Gastrinoma 2 Adenocarcinoma 3 Teratoma 4 Insulinoma 5 Pancreatic sarcoma

Adenocarcinoma The clinical picture is suggestive of a chronic pancreatitis that has developed into pancreatic cancer. 90% of all pancreatic cancers are adenocarcinomas. The rest are adenosquamous, anaplastic, and acinar cell carcinomas. Virtually all pancreatic carcinomas (99%) originate in duct cells. A gastrinoma is a tumor that secretes the hormone gastrin. It is commonly found in the duodenum and is associated with Zollinger-Ellison syndrome. Symptoms include diarrhea, epigastric pain, hematemesis, and difficulty eating. Teratoma is a neoplasm consisting of all 3 germ layers. Insulinoma is a tumor of the pancreas associated with the beta-cells. Sarcomas are neoplasms of connective tissue. Pancreatic cancers are of epithelial cell origin.

A 50-year-old man is sent for a screening colonoscopy for colon cancer by his primary care physician. The test shows no evidence of cancer but reveals 1 polyp in the right colon. It is removed and sent for tissue biopsy by the gastroenterologist, who asks him to return to his office in 2 weeks for the result and further advice. What biopsy result indicates an increased risk for cancer? Answer Choices 1 Hyperplastic polyp 1 cm in size 2 Adenomatous polyp 2 cm in size 3 Pedunculated mucosal polyp 4 Inflammatory pseudopolyp 5 Tubular polyp 0.5 mm in size

Adenomatous polyp 2 cm in size Adenomatous polyps are precursors of adenocarcinoma, and the risk is more if the size is more than 1 cm, if it is sessile rather than pedunculated, and if it is villous rather than tubular. Adenomatous polyps with high-grade dysplasia and invasive cancer are at a high risk of developing colon cancer and should be treated aggressively. Removal by colonoscopy is advocated. The relative risk decreases to 2.3 after removal, as compared to those who do not get it removed, for whom the risk is 8. Hyperplastic polyp is the most common type of colon polyp in the general population and rarely premalignant. Mucosal polyps are benign, small, and of unknown significance. Whether pedunculated or sessile, they are not premalignant. Inflammatory pseudopolyps occur in inflammatory bowel disease (IBD) and are themselves not premalignant. They are caused by boggy mucosa between ulcerations and are not dysplastic. IBD, however, is a premalignant condition by itself. Small tubular adenomatous polyps do not increase colon cancer risk significantly.

A 58-year-old female two days status-post coronary artery bypass graft (CABG) surgery is being treated with heparin to prevent thrombosis. Her routine CBC reveals a hemoglobin of 11.2 mg/dL, hematocrit of 35%, WBC count of 5.6, and platelet count of 22,000. In addition to discontinuing heparin, which of the following is the most appropriate intervention? A Administer prednisone B Administer warfarin C Bone marrow aspirate D CT of the abdomen E Observation

Administer Warfarin The patient most likely has developed heparin-induced thrombocytopenia (HIT), which is associated with qualitative platelet function changes that result in increased risk of thrombosis. She should begin warfarin (B) and be evaluated for thrombosis (e.g., lower extremity Dopplers). Prednisone (A) is not indicated for the treatment of HIT, and a bone marrow aspirate (C) would not aid in establishing the diagnosis. CT of the abdomen (D) would be indicated if she had symptoms consistent with thrombosis in that region, and observation (E) fails to address her increased thrombotic risk.

A 17-year-old G1, P0 girl presents after being found in a crack house by local police. She was initially cooperative, but she is now experiencing severe abdominal pain, and she has developed vaginal bleeding. She says that she is pregnant, but she has not received any prenatal care. Examination reveals a blood pressure of 90/50 mm Hg, pulse of 120/min, and respiratory rate of 25/min. She is diaphoretic and clammy. Her uterus measures 25 cm from the pubic symphysis, but no fetal heart tones can be appreciated. There is a large amount of dark blood around the vagina. Ultrasound reveals an intrauterine fetal demise and a hyperechoic retroplacental hematoma. Labs are pending. Question What is the best course of action at this time? Answer Choices 1 Emergent labor by artificial rupture of membranes 2 Emergent labor induction with Pitocin 3 Administer fibrinogen 4 Emergency cesarean section 5 Administer crystalloids and obtain type and crossmatch for packed RBCs

Administer crystalloids and obtain type and crossmatch for packed RBCs Explanation This patient probably has a grade 3 placental abruption, characterized by external uterine bleeding and fetal demise, as well as maternal hemodynamic instability. Patients with placental abruption should have adequate intravenous access established. Crystalloid fluids are initially used to resuscitate the patient. Blood for transfusion should be typed and crossed (usually at least 4 units). Coagulation studies should be drawn to assess for signs of coagulopathy. Fibrinogen is usually given if the fibrinogen level is <100 mg/dL. Platelets may be administered if the platelet count is <50,000 /uL. Placental abruption (abruptio placenta) is defined as the premature separation from the uterus of a normally implanted placenta. As the placenta separates, there is a large amount of bleeding which irritates the uterus and causes uterine contractions, as well as fetal distress because fetal perfusion is compromised. The incidence of placental abruption ranges from 1 in 75 to 1 in 225 births. Placental abruption is classified by degree of separation. Grade 1 abruption presents with slight vaginal bleeding, mild uterine irritability, and a normal fetal heart rate tracing. Maternal blood pressure is normal, as is the maternal serum fibrinogen level. A grade 2 abruption is characterized by mild-to-moderate vaginal bleeding and pronounced uterine irritability, which may include tetanic uterine contractions. The patient may exhibit orthostatic blood pressure changes and often has an elevated pulse. The fetal heart rate usually shows some evidence of distress. Maternal fibrinogen levels are usually reduced to 150 to 250 mg/dL. A patient with a grade 3 abruption will have moderate-to-severe vaginal bleeding (unless concealed in the uterus), painful, tetanic uterine contractions, and hypotension. The fetus will be dead and the patient is likely to exhibit a coagulopathy with fibrinogen levels less than 150 mg/dL, thrombocytopenia, and reduced levels of clotting factors. Abruptio placenta is associated with several risk factors. Maternal hypertension is one of the most commonly associated risk factors identified in patients who develop placental abruption, especially those with grade 3 abruption (where 40 - 50% of patients will have hypertensive disease of pregnancy). A history of prior abruption, tobacco abuse, cocaine abuse, poor nutrition, and chorioamnionitis are also associated with an increased risk for abruption. Uterine trauma is also associated with a risk for placental abruption, causing 1 - 2% of grade 3 abruptions. Patients with trauma in pregnancy may present with minimal physical evidence of trauma, but they will still have a significant abruption that can progress from grade 1 to grade 3 within 24 hours. Multiple gestations and polyhydramnios can also cause placental abruption if the uterus decompresses rapidly during labor. Placental abruption will classically present with painful vaginal bleeding in the 3rd trimester of pregnancy (80% of patients present with bleeding), although the amount of bleeding may be concealed within the uterus (20%). Ultrasound is used to rule out placenta previa (the other common and dangerous cause of third trimester bleeding) and may identify placental abruption. Unfortunately, the ultrasound appearance of an abruption may lag behind the clinical degree of bleeding, so this imaging modality cannot rule out abruption. The prognosis and management of abruptio placenta depends greatly on the gestational age of the fetus, as well as the grade of the abruption. A grade 1 abruption with a term fetus can often be managed with a controlled induced delivery, and since blood is a strong uterine irritant, many of these patients will deliver relatively quickly. However, the management of preterm pregnancies is less clear and depends on the degree of abruption and fetal distress, with the risks of premature delivery measured against the risks of progression of the abruption.

A 24-year-old woman at 32 weeks of gestation presents with a fever, chills, generalized malaise, and vomiting. T=100.9oF, BP=110/70 mm Hg, P=100/bpm, RR 20/min On pelvic examination, she has acute fundal tenderness, the cervix is 2 to 3 cm dilated, she is 40% effaced, and vertex is at -1 station. Contractions are palpated and recorded every 5 to 10 minutes. Urinalysis shows no evidence of bacteria. On vaginal examination, membranes are ruptured. Question In addition to the administration of steroids, what is the most appropriate next step? Answer Choices 1 Administration of intravenous antibiotics and induction of labor 2 Assess fetal well-being witha non-stress test and biophysical profile 3 Fetal fibronectin testing 4 Performance of a cervical length ultrasound 5 Treatment with magnesium sulfate for tocolysis

Administration of intravenous antibiotics and induction of labor Explanation This patient has preterm premature rupture of membranes (PPROM) and she is in active labor. PPROM is defined as rupture of fetal membranes prior to 37 weeks' gestation. Given her high temperature and fundal tenderness, her clinical presentation is most consistent with chorioamnionitis. In the setting of ruptured membranes, the diagnosis is more likely. Management: Immediate antibiotics for chorioamnionitis therapy should be started, and the commencement of labor induction or augmentation should be considered once the patient is stabilized. Chorioamnionitis, labor, or non-reassuring fetal heart rate testing mandates delivery at any gestational age. In the absence of labor, chorioamnionitis, or non-reassuring fetal heart rate testing, patients with PPROM can be expectantly managed until 34 to 35 weeks' gestation with corticosteroids and broad-spectrum antibiotics: Corticosteroids: A complete course is indicated from 24 to 34 weeks' gestation. Because the degree of lung development is critical for premature neonates, betamethasone or dexamethasone are given to stimulate fetal lung development before delivery. This reduces the risk of respiratory distress syndrome (RDS) and intracranial hemorrhage (ICH). A recent review of several studies found significant benefits and no increase in risk of infection for mothers or infants given steroids. Current recommendations are that all patients with PPROM before 32 weeks receive steroids. Broad-spectrum antibiotics: A 7-day course is given. Antibiotics prolong the latency period and improve perinatal outcomes in patients with PPROM. Fetal well-being is assessed daily with a non-stress test, and a follow-up biophysical profile as needed is used for ongoing monitoring following pharmacologic intervention. Fetal fibronectin is a glycoprotein that plays a role in fetal membrane adhesion. It can be detected in the cervicovaginal fluid in the late 2nd and early 3rd trimester, and it has been associated with preterm birth. Fetal fibronectin testing may be useful in women with symptoms and negative tests because the negative predictive value is greater than 95%. This may avoid unnecessary treatment. However, the poor positive predictive value creates clinical ambiguity in patients who test positive. It is a diagnostic tool and would not be useful in this case. Tocolysis with magnesium sulfate is only useful for the immediate 48 hours following membrane rupture while achieving steroid therapy. Treatment beyond this period has not found to be effective; in fact, it may be harmful.

A 45-year-old woman presents with recent, unexplained weight loss, and lethargy. She has had several episodes of nausea and vomiting. She admits to craving salty foods. On physical examination, there is a hyperpigmentation to her skin and mucus membranes. Her blood pressure is 85/60 mm Hg. Her laboratory results show hyponatremia and hyperkalemia. Her serum ACTH level is elevated. Question What is the most likely site of the pathology? Answer Choices 1 Pituitary 2 Adrenal medulla 3 Pancreas 4 Adrenal cortex 5 Sympathetic ganglia

Adrenal Cortex Explanation This patient has signs and symptoms consistent with Addison's disease. Addison's disease is primary adrenal insufficiency. The symptoms include weight loss, lethargy, nausea, vomiting, and salt craving. Due to the lack of mineralocorticoids, there will be hyponatremia and hyperkalemia. Addison's disease is primary adrenal insufficiency; therefore, the pathology is located in the adrenal cortex. The primary problem is not in the pituitary. There would be an ACTH deficiency, rather than an elevated ACTH, if the primary pathology were located in the pituitary. In this scenario, the pituitary is responding appropriately by increasing ACTH secretion, thus the elevated ACTH level. Addison's disease is also not due to pathology in the adrenal medulla, pancreas, or sympathetic ganglia.

When is RhoGAM (D immunoglobulin) given to Rh-negative patients? Answer Choices 1 At the time pregnancy is diagnosed 2 At the beginning of the 2nd trimester 3 If the pregnancy continues beyond the estimated due date 4 After delivery if the infant is found to be Rh negative 5 After delivery if the infant is found to be Rh positive

After delivery if the infant is found to be Rh positive Explanation D immunoglobulin is administered to Rh negative mothers to prevent isoimmunization to the D antigen from an Rh positive fetus. It is recommended that all pregnant patients have a determination of blood type and antibody screening as part of their initial prenatal workup. A negative antibody screen indicates a non-isoimmunized mother who should have a repeat screen at 28 weeks and then have D immunoglobulin administered if her repeat screen remains negative. Once the D antibody is present, the patient is D-sensitized and there is no longer any benefit to the use of D immunoglobulin. Additional times in a pregnancy when the administration of D immunoglobulin to an Rh negative mother would be appropriate include: after spontaneous or induced abortion, after ectopic pregnancy, after any procedure which invades the uterine cavity (amniocentesis, chorionic villus sampling, fetal surgery), and after external cephalic version of a breech infant. Finally, D immunoglobulin should be administered post-partum to a patient whose infant is confirmed to be Rh positive. There is no indication to administer RhoGAM post-partum if the fetus is Rh-negative.

A 45-year-old male comes to your primary care office complaining of recurrent panic attacks that have led to a disabling fear of being in places from which escape might be difficult, such as a bus or a train. He now finds he can barely leave his home. You diagnose him with panic disorder and which of the following complications? A Agoraphobia B Generalized anxiety disorder C Obsessive-compulsive disorder D Posttraumatic stress disorder E Social phobia

Agoraphobia A Agoraphobia (A) is a complication of panic disorder in which the attacks are associated with being in a crowd or around others. Generalized anxiety disorder (B) is a more overarching term, but does not necessarily include panic attacks. Obsessive-compulsive disorder (C) involves recurrent intrusive thoughts and rituals. Social phobia (E) is more focused on performance in normal social situations and post-traumatic stress (D) requires a traumatic trigger event.

A 55-year-old woman is broughtin by her daughter, who claims that her mother has lost 4 pounds over the last month, has poor appetite, and complains of nausea. She has vomited twice over the past 1 week. History is significant for alcoholism for the past 15 years and hypertension and diabetes for the past 5 years, for which she takes no treatment. She has been drinking 2 pints of beer almost every day for the past 20 years and greater quantities on weekends. Vitals: BP 140/100 mm Hg, HR 102/min, RR 20/min, and Temp 99.8 degrees F. Physical exam reveals hepatomegaly. A liver biopsy reveals ballooning degeneration, spotty necrosis, and polymorphonuclear infiltration. Question What is the most likely diagnosis? Answer Choices 1 Alcoholic fatty liver 2 Alcoholic hepatitis 3 Alcoholic cirrhosis 4 Alcoholic siderosis 5 Hepatocellular carcinoma

Alcoholic Hepatitis This patient most likely has alcoholic hepatitis, as her biopsy reveals ballooning degeneration, spotty necrosis, and polymorphonuclear infiltration. Other findings that may be seen are fibrosis of the perivenular and perisinusoidal space of Disse. Mallory bodies may be present. Alcoholic Liver Disease (ALD) begins with a fatty infiltration of the liver (fatty liver), which gradually progresses to alcoholic hepatitis. Alcoholic hepatitis is characterized by hepatocyte injury, as revealed by the biopsy findings in this patient. Lab findings include a moderately elevated AST, ALT, γ-Glutamyl transpeptidase, and alkaline phosphatase. Both AST and ALT are elevated by 2 - 7 fold, and AST is typically greater than ALT level (AST/ALT >1). Bilirubin is also elevated. Prothrombin time is >5 seconds, and macrocytic anemia may also be observed. Alcoholic cirrhosis refers to the irreversible injury caused to the liver parenchyma by excessive alcohol intake. Extensive fibrosis and formation of regenerative nodules is seen. Alcoholic siderosis is characterized by iron deposits in Kupffer cells in alcoholics. Alcoholic cirrhosis can progress to hepatocellular carcinoma, which is the primary malignant tumor of the liver.

A 66-year-old English teacher is in the hospital. Her neurologist gives her a magazine to read, and she is unable to do so. What is this phenomenon called? Answer Choices 1 Paraphasia 2 Echolalia 3 Alexia 4 Apraxia 5 Agraphia

Alexia When a patient is able to hear things and repeat them, it is called echolalia. With echolalia, the patient does not understand what he has heard. This is also referred to as echophrasia. Paraphasia is a type of aphasia. The substitution of a similar sounding word for another word is called paraphasia. With paraphasia, the words can also be jumbled. Alexia is a type of aphasia. An aphasia where there is a problem with reading is called alexia. Alexia is word blindness or text blindness. Alexia is also called optical aphasia or visual aphasia. Apraxia refers to the condition where a patient has difficulty performing motor acts, despite having the muscular capacity and coordination to do so. A patient with apraxia cannot execute the intended movement. A writing disturbance is called agraphia. There are various forms of agraphia. With absolute agraphia, even simple letters cannot be written. This is also referred to as literal agraphia.

Which of the following statements regarding diabetic medications is/are most correct? A Incretin-mimetics like exenatide commonly result in weight loss. B Thiazolidinediones (TZDs) should be held prior to and for 48 hours after administration of ionidated contrast material. C Sulfonylureas have the highest risk of hypoglycemia of all oral diabetic agents. D Sulfonylureas help preserve beta-cell function. E All of the above.

All of the above

A 72-year-old man presents due to worsening shortness of breath, orthopnea, and chest pain; symptoms have been occurring for the past few weeks. The patient admits to some chronic heart problems as well as fatigue, dyspnea, and a non-productive cough; however, he feels like symptoms have worsened recently. He denies fever, chills, and a productive cough. On physical exam, the man has mildly increased respiratory effort, but he does not appear in distress. He is barrel-chested. His breath sounds are diminished bilaterally, with dullness to percussion over the right and left lower lungs. No pleural friction rub is noted. On cardiovascular exam, an S3 gallop and mild tachycardia (110 bpm) are noted. Clubbing of the fingers, dependent edema in the lower extremities, and jugular venous distention are also noted. His cardiac enzymes and electrocardiogram demonstrate no acute cardiac pathology. Pleural fluid and cardiomegaly are found on the chest X-ray, and a thoracentesis is performed. The pleural fluid is generally clear in color, testing negative for chylomicrons and triglycerides. It has low levels of red blood cells, white blood cells, protein, and lactate dehydrogenase (LDH). Question What is the likely underlying mechanism for the pleural effusion in this patient? Answer Choices 1 Chylothorax from disruption of the thoracic duct 2 Empyema from infection in the pleural space 3 Exudates from local inflammation in capillary beds 4 Hemothorax 5 Transudates from increased hydrostatic pressure or decreased oncotic pressure

All of the listed choices are possible mechanisms for pleural effusions. According to this patient's history and physical exam, as well as the characteristics of the pleural fluid, it should be apparent that his pleural effusion stems from transudates from increased hydrostatic pressure or decreased oncotic pressure. He presents with typical chronic congestive heart failure symptoms. The pleural effusion occurs not from diseased pleura, but from the underlying illness. Transudative pleural effusions are the most common type, and heart failure is the most common underlying cause of pleural effusions. Chylothorax from disruption of the thoracic duct is relatively uncommon. The primary distinguishing characteristics are seen in the milky appearance of pleural fluid, with a large amount of chylomicrons and triglycerides. Empyema from infection in the pleural space might have been a cause if this patient had pneumonia or other severe pulmonary infection. This patient's clinical history is not suggestive of infection. The pleural fluid of empyema may be grossly purulent or milky white. If the milky white fluid is centrifuged, showing clear supernatant with a pellet of white cells below, empyema should be considered. Exudates from local inflammation in capillary beds is a mechanism for pleural effusions secondary to pulmonary embolism, malignancy, as well as some infections, such as pneumonia. Exudates can be distinguished from transudates by a combination of history, appearance, and characteristics of the pleural fluid (using Light's criteria). Simplified, the pleural fluid with exudates may be cloudy in appearance, with relatively high levels of protein and LDH. Hemothorax is blood in the pleural space. It can occur with trauma to the chest wall or other structures of the thorax. Typically, there would be a history of trauma. Hemothorax is unlikely to be bilateral, except in cases of significant trauma. Additionally, the pleural fluid may appear grossly bloody and demonstrate high numbers of red blood cells on analysis.

A 29-year-old G3P2 presents to the office for her obstetrical visit. She is currently 16 weeks gestation by ultrasound. Her pregnancy has had no complications to date. She is a non-smoker and takes her prenatal vitamin. What routine obstetric lab should be offered at this visit? A Amniocentesis B Alpha fetal protein screen C 3D ultrasound D D. Glucose challenge E E. HIV testing

Alpha fetal protein screen AFP testing is only available between 15 and 20 weeks gestation. HIV is done on initial visit and amniocentesis is offered for risk factors, advanced age, or abnormal AFP. 3D ultrasound is not routine standard of care, and diabetic screening is done between 24 and 32 weeks gestation.

What category of pharmaceutical agents is commonly used to treat benign prostatic hypertrophy (BPH)? Answer Choices 1 Alpha-1 adrenergic antagonists 2 Alpha-1 adrenergic agonists 3 Alpha-2 adrenergic antagonists 4 Alpha-2 adrenergic agonists 5 5-alpha reductase agonists

Alpha-1 adrenergic antagonists Explanation The correct response is alpha-1 adrenergic antagonists. Benign prostatic hypertrophy (BPH) is a condition that arises as a result of proliferation of stromal cells in the periurethral glands of the prostate. Consequently, whereas cancer most commonly arises in the outer layer of the prostate, BPH is a condition associated with the inner portion or transition zone. BPH is a condition that accompanies aging. By age 60, approximately 50% of men have microscopic evidence of BPH; by age 85, about 85% of men will have BPH. The exact cause of BPH remains unclear; however, it is most likely the result of changes in the local hormonal milieu of the prostate. Dihydrotestosterone (DHT) is produced in the prostate via conversion of testosterone by the 5-á reductase enzyme. DHT is known to promote stromal proliferation. It is hypothesized that elevated estrogen levels that accompany aging increase the density of DHT receptors in the prostate, leading to hyperplasia. In addition, the density of alpha-1 adrenergic receptors, which contract smooth muscle cells in the prostate, also appears to increase with age, contributing to the obstructive urinary symptoms that typically bring patients with BPH to clinical attention. Microscopic prostatic hyperplasia in and of itself is not dangerous; there is no known association between BPH and prostate cancer. However, prostatic enlargement around the urethral canal can cause urinary outflow obstruction. Clinically, patients report urinary hesitancy, decreased force of urination, interrupted urinary stream, and a feeling of incomplete bladder emptying. Recurrent urinary tract infections may also occur because of residual urine pooling. In severe cases, hydronephrosis may result. In patients with BPH, digital rectal examination reveals lateral and/or median lobe prostatic enlargement. However, prostate size correlates poorly with obstructive symptoms and urodynamic studies are the most useful means of quantifying the severity of BPH. Peak flow rates under 10 ml/sec and residual urine volumes of greater than 150 ml are indicators of significant obstruction. There are 2 main categories of pharmacological agents used to treat BPH: alpha-1 adrenergic antagonists and 5-alpha reductase inhibitors. Alpha-1 adrenergic antagonists (e.g., terazosin, doxazosin, tamsulosin) relax prostatic smooth muscle by blocking the action of alpha-1 adrenergic agonists, reducing obstructive symptoms. By contrast, 5-alpha reductase inhibitors (e.g., finasteride) reduce DHT levels by blocking enzymatic conversion of testosterone. It should be noted that finasteride also lowers prostate specific antigen (PSA) levels, which may complicate the use of PSA as a screening test for prostate cancer.

A 62-year-old woman is being treated for chronic congestive heart failure. She has been put on hydrochlorothiazide therapy. Her serum electrolyte levels are being monitored and show a persistent hypokalemia. The addition of what to her therapeutic regimen would be most appropriate? Answer Choices 1 Acetazolamide 2 Amiloride 3 Furosemide 4 Indapamide 5 Mannitol

Amiloride Explanation Hydrochlorothiazide is a thiazide diuretic often used in congestive heart failure. It causes increased excretion of sodium and chloride and loss of potassium. Thus, hypokalemia is an important side effect. Amiloride is a potassium-sparing diuretic. Its diuretic effect is not very potent, and hence it is good to use in combination with other diuretics. Acetazolamide is a carbonic anhydrase inhibitor. It causes a mild diuresis, a marked elevation of urinary pH, and a significant loss of potassium. Furosemide is a loop diuretic. It has a rapid onset of action and is a potent diuretic. However, it also causes potassium depletion and would only worsen the hypokalemia. Indapamide is a thiazide analog with a long duration of action. Mannitol is an osmotic diuretic and would not be recommended in this patient.

A 48-year-old woman presents for follow up of her diarrhea-predominant irritable bowel syndrome. She has been having worsening of her cramping and bloating symptoms, and she has been having 2 to 3 bowel movements a day intermittently over the past few weeks. She has been feeling more stressed lately due to a promotion at work and additional business travel obligations. She is having problems sleeping as well, which she attributes to anxiety about her increased responsibilities at work. Past medical history is positive for mild premenstrual dysphoric disorder for which she has tried sertraline (Zoloft) and fluoxetine (Prozac); she discontinued these agents due to headaches and worsening diarrhea. Abdominal exam is benign. She asks for something to help her symptoms. What medication should she try? Answer Choices 1 Amitriptyline at bedtime 2 Acetaminophen with codeine as needed 3 Amitriptyline as needed 4 Paroxetine at bedtime 5 Lorazepam as needed

Amitriptyline at bed time Irritable bowel syndrome (IBS) is a disorder of gastrointestinal motility; it causes abdominal pain, constipation, diarrhea, and/or bloating. The cause of irritable bowel syndrome is unknown; however, many things may aggravate the symptoms. Some patients have mood disorders, but the gastrointestinal and psychological symptoms are not necessarily synchronous in these patients. In others, emotional factors, dietary, drug, or hormonal changes may be associated with changes in gastrointestinal motility. There is an increased incidence of irritable bowel syndrome in individuals who were victims of physical or sexual abuse, either as children or adults; therefore, when evaluating patients with IBS, particularly those with refractory complaints, an investigation for a history of prior abuse is important. 2 major clinical types of IBS exist: constipation-predominant and diarrhea-predominant varieties. In constipation-predominant IBS, episodes of constipation are common, and many patients complain of abdominal pain, which may be intermittent and colicky or a dull constant ache. The pain may be relieved by a bowel movement. In diarrhea-predominant IBS, diarrhea often occurs after eating or upon waking. Pain and bloating are common. Treatment of IBS is largely supportive. The chronic nature of IBS should be emphasized, and education about the disorder is important. Increasing dietary fiber can help many patients' constipation. Regular exercise is also helpful, and it can help with psychological stressors that may be exacerbating their bowel symptoms. Tricyclic antidepressants help many patients with IBS, helping with pain, bloating, and diarrhea. They must be taken chronically, however, to be of use. Thus, amitriptyline at bedtime is the only correct choice of the options given. Selective serotonin reuptake inhibitor (SSRI) agents have also been useful in the management of IBS, but given this patient's failure of 2 such agents, a trial of amitriptyline is a better choice then trying paroxetine, which is another SSRI. Anticholinergic agents may also be useful, as may diphenoxylate or loperamide for those patients with diarrhea. Narcotics and sedatives are not good choices for therapy because they have the potential for dependency, and IBS is a chronic condition.

A 37-year-old female presents to the labor and delivery department complaining of intermittent pain and contractions. Upon arrival, she also complains of vaginal bleeding. She is a G3P2 at 39 weeks gestation; no other prenatal complications are noted. She is a non-smoker. A physical exam reveals the following: P 90, BP 130/80, T 98.7°F, abdomen gravid, positive bowel sounds, and left lower quadrant tenderness noted. A sterile speculum exam reveals the cervix to be dilated 8, fetus is cephalic, and membranes are intact. The fetal monitor reveals heart tones in the 140s with mild, decreased variability and good quality contractions noted. Delivery is felt to be imminent, and vaginal delivery has been determined to be the best course of action. What will likely decrease bleeding and shorten time to delivery? A Increased activity level B Amniotomy C Oxytocin therapy D Epidural placement E IV sedation

Amniotomy If the fetus is mature and vaginal delivery (versus c-section) has been determined to be the best course of action, then amniotomy may diminished amnionic fluid volume. This might also allow for better spiral artery compression, and serve to both decrease bleeding from the implantation site and reduce entry of thromboplastin into the maternal circulation.

What risk factor(s)/lipid class combination most closely identifies the need to institute pharmacotherapy? Answer Choices 1 An LDL cholesterol of > 110; no history of Coronary Heart Disease (CHD) 2 A Total Cholesterol of <200; Hypertension (BP> 140/90) 3 An HDL cholesterol of >60; cigarette smoking 4 An HDL of <40; Age (men >45yrs, women >55yrs) 5 VLDL of <100; Diabetes

An HDL of <40; Age (men >45yrs, women >55yrs) Explanation The NCEP/ATEP has set forth explicit Risk Factors as well as specific lipid goals/classifications that clearly address what parameters dictate the initiation or cessation of pharmacotherapy. This question illustrates that, in patients with a major risk factor (CHD/Diabetes), the goal of drug therapy is to keep the level of LDL lipids <100. Likewise, drug therapy should be started when the LDL is >130. An HDL of <40 is undesirable, and so is the age factor (>45yrs for men and >55 years for women). This case illustrates the rationale of pharmacotherapy in the presence of 2 or more risk factors. The specific LDL lipid level should be identified to guide the initiation and goal of pharmacotherapy. Drug therapy for patients with 2+ risk factors should be started when the LDL is >130. The LDL goal is <130. A patient with a TC of <200 is already in the desirable range and would not need drug therapy based on lipid classification alone. It should be noted that the TC is only a general marker; therefore, the individual lipid molecules need to be identified via a lipid panel/profile. The presence of 0-1 risk factors (HBP) mandates that drug therapy should be started when the LDL is >190; the goal of therapy is an LDL <160. Once the LDL level is identified, then the decision whether or not to initiate drug therapy can be discussed. An HDL of >60 is very desirable. There is a direct correlation between the level of HDL and the absence of coronary plaque. It should be noted that the LDL level should still be identified in order to ascertain the true lipid picture. Cigarette smoking is only 1 risk factor; therefore, drug therapy should be initiated when the LDL is >190. The goal of therapy is an LDL level of <160. Once the LDL lipid level is identified, the option of drug therapy can be discussed. The VLDL lipid molecule can be a separate risk factor and is usually elevated along with TG. The VLDL level becomes important (VLDL > 200 - 300) when it is accompanied by elevated TG (200-499). As in the above explanations, the LDL lipid level should be identified due to the direct correlation between LDL and CHD. A major risk factor, such as diabetes, mandates that drug therapy be initiated when the LDL is >130. The LDL goal is <100. Once the LDL level is lowered to an acceptable range, the elevated VLDL and TG (especially if TG is >500) should be addressed by initiating either a combination of diet and exercise (will lower VLDL) and/or drug therapy. The Statins have a modest effect on VLDL/TG, whereas niacin and gemfibrozil have been shown to dramatically reduce these lipid levels.

Your patient is a 22-year-old male who has experienced significant dysfunction for the past eight months. He exhibits only one symptom of schizophrenia, but that one symptom has been present for a significant part of each day. Which of the following would qualify the patient to be diagnosed with schizophrenia according to the DSM-IV TR? A A firm belief that someone is following him B An auditory hallucination in which two or more voices are conversing C A sensation that bugs are crawling on his skin D Refusal to go out without a hat, certain he will become ill without one E Speaking entirely in rhymes that are related but mostly nonsensical

An auditory hallucination in which two or more voices are conversing Generally, two or more characteristic symptoms of schizophrenia are required in order to make a diagnosis unless the one symptom present is a bizarre delusion or an auditory hallucination (B) that consists of either a running commentary or two voices conversing. Positive symptoms of schizophrenia also include disorganized or catatonic behavior and disorganized speech as well as delusions and hallucinations. Negative symptoms include affective flattening, alogia and avolition. (A) and (D) are both potentially delusions, but neither is particularly bizarre. A sensation of bugs on the skin (C, fomication) is a type of hallucination and speaking in rhyme (E) qualifies as disorganized speech. Each of these might count as one of the two required symptoms but would not be sufficient without a second symptom.

A 36-year-old woman presents with fatigue and diplopia; symptoms started in the morning and worsened during her time at work. On examination, the patient has ptosis bilaterally and decreased ocular muscle power. A CT scan of the chest shows a thymoma. What is the pathophysiology of this patient's neuromuscular disorder? Answer Choices 1 A genetic defect resulting in the production of atypical, or faulty, plasma cholinesterase 2 An autoimmune response that initially blocks, then destroys, nicotinic receptors at the neuromuscular junction 3 An increase in the ratio of muscle fibers to motor nerve fibers, resulting in production of a motor unit (MU) that provides less control over muscle actions 4 Production of IgA antibodies against acetylcholine receptors of the motor end plates of skeletal muscle 5 Genetic defect resulting in the deficiency of dystrophin

An autoimmune response that initially blocks, then destroys, nicotinic receptors at the neuromuscular junction Myasthenia gravis is a neuromuscular disorder caused by immune mediated loss of acetylcholine receptors. It is associated with thymic hyperplasia in 65% of the cases and with thymoma in 15% of the cases. Myasthenia gravis (MG) is an autoimmune disorder affecting the neurochemical transmission of information between motor nerve fibers and skeletal muscle fibers. In this disorder, nerve impulses are transmitted in a normal fashion to the axon terminals, and skeletal muscle is capable of contracting normally. The dysfunction occurs at the interface between the 2 components of the neuromuscular junction (NMJ). In cases of MG, an autoimmune response causes antibodies of the IgG (not IgA) class to be produced against the nicotinic receptors of the NMJ. The antibodies fix onto the nicotinic receptors, block acetylcholine (ACh) from complexing with the receptors, and ultimately bring about destruction of the receptors. The patient may present with ophthalmic or non-ophthalmic manifestations. The ocular disturbances mainly include ptosis and diplopia; the non-ophthalmic symptoms include difficulty in talking, swallowing, or chewing and upper limb and trunk weakness. Anti-acetylcholine receptor antibodies assay is positive in 90% of the patients with generalized MG, whereas it is positive in 50-70% in ocular MG. Other tests include the Tensilon test, imaging studies such as CT or MRI scanning, repetitive nerve stimulation, and single fiber electromyography. Medical treatment includes neostigmine therapy, plasmapheresis, or intravenous immunoglobulin therapy. Thymectomy may be considered in cases of thymoma. The reduction in receptor population decreases neurotransmission, and the result is that muscle depolarization is not produced. The quantity and quality of cholinesterase enzymes is unaffected by the disorder, and so is any alteration in the composition of motor units at the neuromuscular junctions. A genetic disorder associated with deficiency of the structural protein dystrophin in muscles occurs in Duchenne muscular dystrophy, and it is not seen in myasthenia gravis.

A 4-day-old infant is seen for a routine newborn check; he has yellow skin from his face down to his chest. He is a full-term baby, had a normal vaginal delivery, weighed 3.180 kg at birth, and 3.100 kg at discharge. He is breastfeeding well, and stooling and voiding regularly. He is well-hydrated and active. Laboratory results reveal an indirect bilirubin of 12.2 mg/dL, and a direct bilirubin of 1.0 mg/dL. Question What is the physiologic mechanism behind this jaundice? Answer Choices 1 An elevation of direct bilirubin due to decreased bilirubin production 2 The increased production of red blood cells 3 The increased conjugation of bilirubin by the liver 4 An elevation of indirect bilirubin due to increased bilirubin production 5 The decreased production of red blood cells

An elevation of indirect bilirubin due to increased bilirubin production The correct response is an elevation of indirect bilirubin due to increased bilirubin production. Unconjugated indirect bilirubin enters the liver and is excreted from the hepatic cells in a water-soluble form (conjugated and direct) into the biliary system. The direct bilirubin leaves the body in the form of stool. The most common cause of indirect hyperbilirubinemia is physiologic jaundice, which peaks at 2 - 4 days of life. Physiologic jaundice is due to increased bilirubin production after the breakdown of red blood cells, in conjunction with limited conjugation of bilirubin by the immature liver. Physiologic jaundice can be worse in babies that are breastfed, due to a higher level of dehydration as the mother waits for her milk to come in.

A patient with early onset of hypertension, hematuria, and palpable kidneys was diagnosed to have adult polycystic kidney disease, and he is being evaluated for associated berry aneurysms. What investigation is considered the "gold standard" for the detection of aneurysms of the cerebral blood vessels? Answer Choices 1 MRI 2 Angiogram 3 PET scan 4 CAT scan 5 Electroencephalogram

Angiogram Adult polycystic kidney disease is associated with cysts of the liver in 30 to 40% of patients. Other associations include berry aneurysms of the cerebral blood vessels, colonic diverticula, abdominal wall hernias, and mitral and aortic regurgitation. APKD is an autosomally dominant disorder associated with the PKD 1 gene on chromosome 16. Angiography involves taking several X-rays after the injection of a radio opaque dye in the blood vessel thus making the outline of the vessel visible. From this outline, it would be discernible as to whether there is or is not an aneurysm present. Computer-assisted tomography (CAT), like conventional radiography, produces X-ray images of various structures. The CAT scan, however, is able to reveal subtle differences that can distinguish gray and white matter, the ventricles, the thalamus, the basal ganglia, and others. It can produce an image of a single plane or section of the brain. (The term tomography comes from the Greek word meaning, "cut".) Positron emission tomography (PET) provides live, real time images of the functioning of the brain. PET scanning measures the differential rates of glucose metabolism in the active brain. Both CAT and PET images are obtained by a detector that rotates around the patient's head. However, CAT images reflect the absorption of radiation, whereas PET images are produced by the emission of radiation. Electroencephalography measures the electrical potentials of the surface layers of the cerebral cortex by means of electrodes placed on the skull.

A 25-year-old man presents due to heartburn. It only occurs 1 or 2 times a month, but it is extremely noticeable when these episodes do occur. He states that eating large meals, eating late at night right before going to sleep, and excessive alcohol consumption seem to exacerbate and even initiate episodes. He has noticed that limiting the aforementioned behaviors has decreased the severity of the symptoms in recent months, but it is still occurring despite this modification. He denies any difficulty swallowing, weight loss, night sweats, chest pain, use of tobacco, or coughing up blood. Physical examination is unremarkable for any abnormalities. Question Since this patient's symptoms would be classified as mild, intermittent symptoms and modifications have been attempted and failed, what would be the next best option for this patient? Answer Choices 1 Antacids 2 H2-receptor antagonists 3 Proton-pump inhibitor 4 Antibiotics 5 Prokinetic agents

Antacids Mild or intermittent symptoms of gastroesophageal reflux disease (GERD) are not typically seen as adversely affecting patients' quality of life. Initially, action is taken in terms of modification of behaviors, such as eating smaller meals as well as eliminating acidic foods and foods known to precipitate the reflux (fatty foods, alcohol, chocolate, or peppermint). Weight loss has also been shown to help decrease issues, and so has advising patients to avoid lying down at least 3 hours after eating meals. Elevation of the head of the bed is also suggested to initiate relief. From a pharmacological standpoint, antacids are considered the mainstay for rapid relief of occasional heartburn. Antacids work by neutralizing stomach acid only; alone, they will not heal any inflammation that may be caused by stomach acid. Patients should be told that the duration of action of these medications is typically less than 2 hours. Antacids are also contraindicated in any patients with renal failure. Many types of antacids are available over the counter. The next step up in terms of treatment would be considered to be H2-receptor antagonists. These medications work by reducing stomach acid production. Again, these agents are available over the counter as 50% the prescription dose. Examples include cimetidine 200 mg, ranitidine 75 mg, and famotidine 10 mg. When these agents are taken for active reflux symptoms, they have a delay of onset of at least 30 minutes; once they do take effect, relief is supplied for up to 8 hours per dose. Proton pump inhibitors are initial pharmacological treatment reserved for patients with moderate to severe symptoms of reflux as well as known complications from GERD. Proton pump inhibitors work by actually blocking the production of acid. Examples of these include omeprazole 20 mg or lansoprazole 30 mg. There are proton pump inhibitors available over the counter, but some are only available with a prescription. Proton pump inhibitors help patients achieve adequate control of their heartburn symptoms, and they can potentially offer resolution of these symptoms; they even aid in healing of erosive esophagitis if it is present. Antibiotics in relation to reflux or heartburn would not be indicated directly for the symptoms of this pathology; antibiotics could potentially be used as a component of the treatment of a H. pylori infection. Prokinetic agents are not indicated as treatment of mild, intermittent reflux symptoms.

A 6-year-old boy presents to the office with a fever, malaise, and dark urine. His mother states he missed school earlier this month with a sore throat. On examination blood pressure is 120/88 mm Hg, pulse is 82/min, temperature 100.6°F, and respirations 16/minute. On physical assessment, the patient appears ill with only mild costovertebral angle tenderness noted. Urinalysis reveals the following: Urinalysis Result Specific gravity 1.00 pH 5.2 protein + 1 blood + 2 glucose negative ketones negative bilirubin negative urobilinogen negative nitrates negative leukocyte esterase + 1 Microscopic examination reveals: RBCs, renal tubular epithelial cells, RBC casts, and granular casts. Question What test will determine the most likely etiology? Answer Choices 1 Urine for culture and sensitivity 2 Urine cytology 3 Erythrocyte sedimentation rate 4 Anti DNAase B serology 5 Urine protein electrophoresis

Anti DNAase B serology Explanation The correct answer is anti-DNAase B serology to identify post-streptococcal glomerulonephritis. Group A β-hemolytic streptococci pharyngitis may result in the delayed complication of post-streptococcal glomerulonephritis 10-14 days after the infection. Patient presentations may range from subclinical symptoms to acute nephritic syndrome as streptococci may produce streptolysin, DNAase, and hyaluronidase that lead to tissue destruction and disseminate infection. Serology testing to identify antibodies to these exoenzymes can aid in the diagnosis by demonstrating indirect evidence of infection. Confirmation may require serial antibody draws that reveal a rise in titer levels above baseline. Urine culture and sensitivity would be appropriate if the clinical picture only entailed fever and flank tenderness in the presence of pyuria and hematuria suggesting a urinary tract infection. That diagnosis does not explain the presence of proteinuria with renal tubular epithelial cells and casts. These indicate intrinsic kidney damage, which is not seen in urinary tract infections. Further serology testing is indicated in the post pharyngitis period. Urine cytology is ordered in the presence of gross or microscopic hematuria, which is often painless, to identify malignant cells in the urinary tract. This patient does not fit the epidemiologic profile or clinical presentation of malignancy to warrant cytology testing. Erythrocyte sedimentation rate can aid in detecting an inflammatory response, such as nephritis, but it lacks specificity to identify the infectious process. Urine protein electrophoresis is indicated to identify abnormal levels of free monoclonal light chains (Bence Jones protein) from immunoglobins in cases of myeloma.

A 42-year-old woman IV drug user presents with vague symptoms of fatigue, aches, pains, and nausea. Recently, she appeared jaundiced for a few days; however, she no longer does. Preliminary blood tests indicate the patient does not have hepatitis B. However, the tests could be wrong, as she may be in the window period. If she were in the window period, what would be the only evidence of hepatitis B infection? Answer Choices 1 Anti-HBc 2 HBsAg 3 HBcAg 4 HBeAg 5 Anti-HBs

Anti-HBc Hepatitis B is transmitted by exposure to blood or blood products. If she shared needles as an IV drug user, she has put herself at risk of infection with hepatitis B. The window is the period in which HBsAg (HBV surface antigen) is not detectable and anti-HBs (antibody to HBV surface antigen) has yet to appear. During the window period of a hepatitis B infection, the only marker present would be IgM anti-HBc (antibody to HBV core antigen). Because of this, the window period is also referred to as the core window. This IgM antibody is present in the early convalescence period (2 - 16 weeks after infection). It is an expensive test and is ordered only when there is very high suspicion. Positivity of this antibody, along with HbsAg, indicates acute infection. PCR test for genome identification is the other test useful during the window period. HBsAg (HBV surface antigen) is the 1st serological marker to appear in hepatitis B infection. After HBsAg is cleared from the serum, the patient is in the window period. HBeAg (HBV 'e' antigen) can be seen with acute hepatitis. It is a marker for infectivity. HBcAg is the HBV core antigen. Anti-HBs (antibody to HBV surface antigen) is seen at the end of an infection. The appearance of anti-HBs marks the end of the window period. The presence of anti-HBs gives a patient immunity.

A 32-year-old IV drug user presents with fatigue, joint pain, and nausea. She initially thinks she has the flu until she becomes visibly jaundiced. She sees her doctor and blood work is done. She is diagnosed with hepatitis B infection. She recovers without complications. On a follow up visit, she asks her doctor if she can get hepatitis B again or if she is immune. Question What would confer immunity from further infections of hepatitis B? Answer Choices 1 Anti-HBs 2 Anti-HBc 3 HBsAg 4 HBcAg 5 HBeAg

Anti-Hbs Hepatitis B is transmitted by exposure to blood or blood products. If this patient shared needles when using drugs, she put herself at risk of infection with hepatitis B. Anti-HBs (antibody to HBV surface antigen) is seen at the end of an infection. The appearance of anti-HBs marks the end of the window period. The presence of anti-HBs gives a patient immunity. HBcAg is the HBV core antigen. HBsAg (HBV surface antigen) is the first serological marker to appear in hepatitis B infection. After HBsAg is cleared from the serum, the patient is in the window period. The window is the period in which HBsAg (HBV surface antigen) is not detectable and anti-HBs (antibody to HBV surface antigen) is yet to appear. During the window period of a hepatitis B infection, the only marker present would be anti-HBc (antibody to HBV core antigen). Because of this, the window period is also referred to as the core window. HBeAg (HBV "e" antigen) can be seen with acute hepatitis. It is a marker for infectivity.

A 63 year-old woman complains of fatigue, loss of appetite, a sore-red tongue, paresthesias of her feet and hands, and unsteadiness of her gait. Which of the following tests is be used to confirm the patient's suspected diagnosis? A Anti-intrinsic factor antibodies B Antiparietal cell antibodies C Coomb's test D Schilling Test E Serum folate levels

Anti-intrinsic factor antibodies A diagnosis of pernicious anemia (PA) can be supported through the presence of anti-intrinsic factor antibodies. Antiparietal (B) cell antibodies aren't associated with the development of PA. The Coomb's test (C) is used in the evaluation of hemolytic anemias. Serum folate (D) levels are beneficial in evaluating macrocytic anemias, but will not establish a diagnosis of PA. The Schilling test (D) was once commonly used to diagnosis PA, but is no longer available due to lack of available radiolabeled human intrinsic factor.

A 31-year-old woman presents with a purpural rash covering her arms, legs, and abdomen. She also has fever, chills, nausea, abdominal tenderness, tachycardia, and generalized myalgias. Prior to the development of the rash, the patient noted that she had a headache, cough, and sore throat. Laboratory studies were positive for Gram-negative diplococci in the blood, along with thrombocytopenia and an elevation in PMNs. Urinalysis showed blood, protein, and casts. Vital signs are as follows: PB 92/66, P 96, RR 14, T 39. The patient denies any foreign travel and does not have any sick contacts. However, she does work part time as a nurse in a local hospital. Question The patient is diagnosed with Meningococcemia; she is admitted to the hospital and placed in respiratory isolation. What major course of therapy should this patient receive? Answer Choices 1 Steroids 2 Supportive care 3 Antibiotics 4 Transfusion 5 Bactericidal/permeability-increasing protein

Antibiotics Antibiotics are the treatment of choice for meningococcemia. The preferred drug for active infection is penicillin G. For those allergic to penicillin, chloramphenicol and cephalosporins (ie, cefotaxime, cefuroxime) may be used as alternatives. Patients will also receive supportive care, but antibiotic therapy must be initiated quickly if the patient is to survive. Intensive care placement may be necessary if organ failure is imminent. Ventilatory support, inotropic support, and IV fluids are necessary in some. If adrenal insufficiency occurs, corticosteroid replacement may be considered. A central venous line helps to provide large amounts of volume expanders and inotropic medications for adequate tissue perfusion. Steroids have not been shown to play a major role in the treatment of meningococcemia. However, they have been used in addition to antibiotic therapy. In the case of adrenal insufficiency, for example, steroid replacement has been shown to be beneficial. Transfusion does not generally play a major role in treatment. If the patient suffers from a devastating coagulopathy, blood or blood products may be replaced as necessary. Bactericidal/permeability-increasing protein is a protein stored in the granules of neutrophils. It binds to endotoxin in vitro and neutralizes it. This technique is experimental, and it is not used in everyday treatment of meningococcemia.

Your patient is a 55-year-old man presenting with a strong, sudden need to urinate. He feels his bladder spasms and sometimes has involuntary loss of urine. The problem started several months ago and seems to be worsening. He is very upset because it sometimes happens during his business meetings. His urinalyses were always normal. He takes no medications. Careful examination shows that he has urge incontinence. Bladder training and behavioral techniques were unsuccessful, and you decided to introduce medication. Before suggesting a medication for this condition, you will tell him that this drug may have some side effects: dry mouth, difficulty in urination, constipation, blurred vision, tachycardia, drowsiness, and dizziness. Question This may happen because you will be prescribing what type of drug? Answer Choices 1 Cholinergic 2 Anticholinergic 3 Epinephrine 4 Antibiotic 5 Botulinum toxin

Anticholinergic Urge incontinence is defined as involuntary loss of urine occurring for no apparent reason together with a feeling of urinary urgency (a sudden need or urge to urinate) that represents a hygienic or social problem to the individual. The most common cause of urge incontinence is involuntary and inappropriate detrusor muscle contractions. The drug you will suggest is anticholinergic (like Oxybutynin). It will relieve urinary and bladder difficulties, including frequent urination and urge incontinence by decreasing muscle spasms of the bladder, increasing the capacity of the bladder, and delaying the initial urge to void. It is a competitive antagonist of M1, M2, and M3 muscarinic acetylcholine receptors and in higher doses can act as spasmolytic on bladder smooth muscle. Anticholinergic side effects are dry mouth, difficulty in urination, constipation, blurred vision, tachycardia, drowsiness, and dizziness. Cholinergic drug will cause slowing of the heartbeat and increases in normal secretions. For this reason, patients who already have a problem with incontinence should not be advised to use these drugs. Epinephrine is not indicated in this patient. Epinephrine stimulates the ends of the sympathetic or inhibitory nerves of the bladder, with the effect of relaxation of the bladder muscles and the increase in tone and rate of contraction of the ureter. The secretion of urine is increased synchronously with the rise in arterial pressure. It will also cause overacting heart, palpitation, and vomiting. There is no need for antibiotics in a patient with normal urine analysis for a problem that lasts several months. Botulinum toxin is given as intradetrusor injection in patients who have failed pharmacological therapy. It has been shown to decrease episodes of urinary leakage by preventing the release of acetyl choline from presynaptic membrane. It is also indicated for urinary incontinence in patients with neurologic conditions (e.g., spinal cord injury, multiple sclerosis). It sometimes can cause urinary retention given as intradetrusor injections and occasionally headache, light-headedness, fever, abdominal pain, and diarrhea (not necessarily a direct result of Botox).

A 19-year-old male is brought by his mother to your primary care office. She is at her wits' end as he has just gotten out of juvenile detention but continues to engage in the behaviors that resulted in his incarceration. He lies, cheats, steals, and seems to disregard the rights and needs of others to the point of endangering their safety. This pattern has been present since early childhood, since at least age four. He does not feel that there is a problem, but his family and others around him are disturbed by his attitudes and behaviors. What is the most likely diagnosis? A Antisocial personality disorder B Borderline personality disorder C Histrionic personality disorder D Narcissistic personality disorder E Schizoid personality disorder

Antisocial personality disorder A All personality disorders involve a persistent pattern of behavior in which there is a disturbance in impulse control, interpersonal behavior, interpretation of people and events, and/or emotional response that begins no later than adolescence. Antisocial PD involves a reckless disregard for others, usually manifesting in lawlessness, lying, cheating, and a lack of remorse. Borderline PD (B) involves interpersonal reactivity and impulsivity often focused on perceived abandonment. Histrionic PD (C) involves a need to be the center of attention, often manifesting in behavior that is theatrical or seductive. Narcissistic PD (D) involves a lack of empathy and grandiosity. Schizoid PD (E) involves symptoms similar to schizophrenia.

A 62-year-old woman with a long-standing history of hypertension presents with severe headache; it started this morning and is rapidly worsening. During the interview, she suddenly collapses. Your brief examination shows that she responds with extensor posturing on external stimuli. Her deep tendon reflexes are 3, and you elicit Babinski bilaterally. You also notice that her breathing has a peculiar pattern: deep inspiration with a pause at full inspiration, followed by a brief insufficient release and the end-inspiration pause. Question How do you best describe her respiratory pattern? Answer Choices 1 Cheyne-Stokes 2 Apneusis 3 Ataxic 4 Cluster 5 Central neurogenic hyperventilation

Apneusis In the hypertensive patient with sudden loss of consciousness and decerebrate response (extensor posturing), you should consider brain stem hemorrhage. Abnormal breathing patterns can be observed in both pontine and medullary lesions; they sometimes can be prognostic. Her breathing pattern is apneustic. Apneustic breathing pattern characterizes deep, gasping inspiration with a pause at full inspiration followed by a brief, insufficient release and the end-inspiration pause before expiration. Lesion in the pons or upper medulla causes the removal of input from the vagus nerve and the pneumotaxic center. Normally, apneustic center of the lower pons promotes inspiration by stimulation of the dorsal respiratory center in the medulla to delay the 'switch off' signal of the inspiratory ramp provided by the pneumotaxic center of pons. Therefore, athe pneustic center controls the intensity of breathing. Apneusis is an ominous sign, with a generally poor prognosis. Cheyne-Stokes respiration is an abnormal pattern of breathing characterized by repetitive progressively deeper and sometimes faster breathing, followed by a gradual decrease in breathing that results in temporary apnea. Cycles usually take 30 seconds to 2 minutes. Increased CO2 during the period of apnea causes compensatory hyperventilation. Hyperventilation in turn causes the decrease in CO2, which causes apnea and the cycle to restart. Causes include CNS dysfunction, cardiac failure with low cardiac output, sleep, hypoxia, or profound hypocapnia. Ataxic breathing (Biot's respiration) is an abnormal pattern of breathing characterized by groups of quick, shallow inspirations followed by regular or irregular periods of apnea. It is generally a poor prognostic sign. Biot's respiration is caused by the medullary lesion due to strokes or trauma or by pressure on the medulla due to uncal or tentorial herniation. Kussmaul breathing is a deep, labored, and gasping breathing pattern seen often in severe metabolic acidosis (diabetic ketoacidosis, renal failure). In metabolic acidosis, breathing is first rapid and shallow, but later on, as acidosis worsens, breathing gradually becomes Kussmaul breathing. Central neurogenic hyperventilation is a very deep and rapid pattern of breathing, usually seen in the lesions of the midbrain and upper pons. Respirations are generally regular, and the PACO2 decrease due to the hyperventilation. Cluster breathing is irregular breathing with periods of apnea that occurs at irregular intervals. It is generally seen in lesions in the low pons or upper medulla. It differs from Cheyne-Stokes pattern because there is no increasing and decreasing depth of respirations.

A 60-year-old man presents with a 1-year history of increasing dyspnea. The patient used to work as a construction worker, and he is a chronic smoker. A chest radiograph shows diaphragmatic pleural plaques and interstitial lung disease. What is the likely diagnosis? Answer Choices 1 Silicosis 2 Siderosis 3 Byssinosis 4 Berylliosis 5 Asbestosis

Asbestosis The radiological changes in asbestosis are usually confined to the lower 2/3 of the lung-fields. Pleural plaques are a frequent finding in a patient with asbestosis. The classical clinical feature in asbestosis is increasing exertional breathlessness due to pulmonary fibrosis. Digital clubbing is usually present, and inspiratory crepitations are audible over the lower zones of both lungs. Silicosis produces a pattern of silicotic nodules in the lung. Silica is a fibrogenic dust and causes the development of hard nodules, which coalesce as the disease progresses. The radiological changes of silicosis are more marked in the upper zones of the lung. Siderosis is a disorder due to increased iron. Increased iron in the lung is an uncommon pneumoconiosis. It is more likely to occur with hemochromatosis with iron overload. It occurs in occupations like arc welding. Byssinosis produces asthma-like symptoms. Berylliosis produces sarcoid-like granulomatous disease (chronically) and an acute pneumonitis (acutely). Patient will have history of exposure to beryllium.

During a routine X-ray examination for employment insurance purposes, the radiologist notices a lesion on the right upper lobe of the pulmonary X-ray of a middle-aged man. The patient was treated for pulmonary cavitary tuberculosis (TB) 2 years ago; he has completed treatment, and he has not had any problems since. Refer to the image. What late complication of TB is seen in this patient? Answer Choices 1 Fibrothorax 2 Aspergilloma 3 Broncholithiasis 4 Reactivation of TB 5 Bronchiectasis

Aspergilloma Aspergilloma is a solid mycelial mass caused by the growth of Aspergillus species within a cavity, usually within the lung. This condition is most commonly a result of a previous tuberculous infection. Although the patient may be asymptomatic, cough is frequent, and a severe life-threatening hemoptysis may occur. Patients present with hemoptysis, or it can be an incidental diagnosis with a routine X-ray. Previous TB history and anti-TB treatment are helpful for the diagnosis. The diagnosis is confirmed by the demonstration of a mobile intracavitary mass on the radiograph and air-crescent sign around the mass. Although cultures are often negative, patients frequently have high titers of antibody to Aspergillus. If the patient is asymptomatic, the treatment is conservative. Surgical removal is the only reliable curative method indicated for uncontrollable hemoptysis. Spontaneous expectoration of the fungal mass may occur. Oral itraconazole results in partial or complete resolution of aspergillomas in 60% of patients. In this question, the radiologic picture has a typical Easter egg shaped aspergilloma in the right upper lobe old TB cavity. Lung destruction can be seen after recurrent TB, usually due to noncompliance. Patient's asymptomatic status and chest X-ray is suggestive of the late TB complication aspergilloma, not broncholithiasis, fibrothorax, reactivation of TB, or bronchiectasis.

A 40-year-old man who is a chronic alcoholic presents with cough productive of large amounts of purulent sputum. He developed the cough about 2 weeks ago, and it has gradually worsened over time. Now his sputum is foul-smelling. He has had high intermittent pyrexia for 4 days. The vitals are as follows: temperature of 38.9°C, pulse of 108BPM, respirations of 24/min, blood pressure of 140/80 mm Hg. On auscultation, there is pleural rub and diminished air entry on the right side. The chest radiograph shows a large dense opacity on the right side with a fluid level. The patient is diagnosed with lung abscess. Question What statement is true regarding lung abscess? Answer Choices 1 Only anaerobic organisms are responsible for this condition 2 Aspiration is the most common cause 3 Sputum culture is the best investigation 4 Computed Tomography (CT) has no role as an investigation in this condition 5 Treatment is with antibiotics for 3 days

Aspiration is the most common cause Explanation Lung abscesses are the result of microbial infection causing cavities containing necrotic pulmonary tissue to develop in the lung. Aspiration of oropharyngeal material is the most common cause. It is always secondary, due to infection reaching the lungs from elsewhere. About 65% of cases are caused by anaerobic bacteria and the rest by a mixture of both aerobic and anaerobic organisms. Symptoms include cough, foul-smelling sputum, fever, and sometimes hemoptysis. A chest X-ray can demonstrate a cavity with fluid in it. But other associated conditions, such as empyema or pulmonary infarction, are best demonstrated on CT scan. Knowledge of the underlying cause of the abscess can assist in deciding on the right antibiotic course. A sputum culture, although routinely done, is usually a contaminated specimen (by the organisms in the mouth) and hence not very useful. Although improvement is expected in 3-5 days, therapy is generally continued for 4-6 weeks, depending on further radiograph findings.

A 73-year-old, male with a 30 pack-year smoking history presents to the clinic with complaints of headache and dizziness. Labs reveal a hemoglobin of 21.3 g/dl, hematocrit of 63%, and platelet count of 498,000. He tests positive for the JAK2 mutation. Which of the following should be recommended to this patient to prevent secondary complications from his diagnosis? A Aspirin 81 mg daily B Ferrous Sulfate 325 mg twice daily C Eat a diet rich in vitamin B12 D Eat a diet rich in folate E Maintain stable consumption of foods that contain vitamin K

Aspirin 81 mg daily A The patient most likely has polycythemia vera and is at risk for thrombotic events that can be reduced through the use of daily aspirin. In addition, he should be counseled on smoking cessation if he is still currently smoking. The patient should not take iron supplements (B) as they compete with phlebotomy therapy that seeks to reduce iron stores. Patient's with PCV similarly do not require vitamin B 12 (C) or folate (D) supplementation, and are not impacted by fluctuations in vitamin K consumption (E) unless they are being treated with warfarin for a current/recent thrombotic event.

A 5-year-old boy presents with an erythematous skin rash. It is associated with intense itching. The boy's mother has noticed that her son's rash has been recurrent, with 3 to 4 episodes per year. The itching and rash increases after consumption of certain foods. On examination, erythematous raised papules are seen on the cheek, trunks, and upper arms; an example is shown. Question What is most likely associated with the boy's condition? Answer Choices 1 Gluten enteropathy 2 Asthma 3 Vitamin A deficiency 4 Psoriasis 5 Raised Dengue titers

Asthma Asthma is the correct answer. The vignette describes a child with atopic dermatitis, one of the most common childhood dermatological conditions. It usually begins before 2 years of age and displays a chronic relapsing course. Typical sites of involvements are the cheeks, flexural aspect of the elbows, natal cleft, and the hands. Clinical examination usually reveals xerosis, lichenification, and eczematous lesions. Atopic dermatitis is commonly associated with other atopic conditions, such as hay fever, extrinsic asthma, allergic conjunctivitis, and allergic rhinitis. A high IgE level and eosinophilia are common to all of these. Gluten enteropathy is incorrect. Gluten enteropathy or celiac disease may be associated with skin manifestations, namely dermatitis herpetiformis. The typical patient is middle aged and develops crops of fluid filled blisters resembling herpetic lesions (hence the name herpetiformis). The face, hairline, and shoulder are common sites. IgE levels are usually normal. The association with celiac disease is strong; associated symptoms of abdominal pain, nausea and bloating may be contributory findings. Vitamin A deficiency is incorrect. Vitamin A deficiency may result in dry, thickened 'toad skin'; it may also be associated with frequent skin infections. Eye manifestations, such as night blindness, corneal ulcers, and xerophthalmia, are more common. Psoriasis is incorrect. Psoriasis usually causes discrete scaly lesions and is more common in older age groups, except for Guttate psoriasis. It shows a predilection for extensor surfaces and the scalp, as opposed to atopic dermatitis, which favors the flexural aspects. Raised Dengue titers is incorrect. Dengue fever is an arboviral infection that may cause a fine maculopapular rash throughout the body. Dengue hemorrhagic fever may be associated with a low platelet count and petechial rashes. They are never associated with chronic discrete rashes.

A 6-year-old male child presents to the clinic for a cough that occurs only after he has been running, according to his mother. She says she first noticed this about 6 months ago, after he had had one of his usual winter colds, and his cough persisted for about a week. On the basis of this history, what is the most likely diagnosis? A airway foreign body B asthma C cystic fibrosis D laryngomalacia

Asthma Asthma, in this case exercise-induced, is the most likely cause of this problem. The symptoms commonly associated with acute exacerbations of asthma include wheezing, cough, dyspnea, and chest pain. Some symptoms that might be suggestive of asthma include exercise-induced cough, nighttime cough, cough after cold air exposure, and cough after laughing. Airway foreign bodies, though not common, are an acute problem that may present as sudden cough, choking, and wheezing. Cystic fibrosis (CF) is the most common, lethal, genetic disease affecting the Caucasian population. Up to 50% of patients with CF are diagnosed in infancy, but others may not be diagnosed until adolescence or adulthood. Chronic or recurrent cough should be an indicator for consideration of CF as a differential diagnosis. Laryngomalacia is the most common cause of stridor in infants. It is the incomplete development of the cartilaginous support of the laryngoglottic structures. This congenital condition is usually self-limiting and occurs most commonly in infants at or just after birth. The inspiratory collapse of the epiglottis or arytenoid cartilages is heard as stridor.

Following emergent appendectomy, a 58-year-old obese male develops a temperature of 102.4˚F, 18 hours postoperatively. His respiratory rate is 26 and his pulse is 116bpm. A physical exam reveals scattered fine rales. What is the most likely diagnosis? A Atelectasis B Aspiration pneumonitis C Pleural effusion D Pneumonia E Pulmonary embolus

Atelectasis A Pulmonary alveoli collapse, also known as atelectasis, occurs during operative procedures for a variety of reasons, including decreased clearance of secretions and decreased intra-alveolar pressure. Postoperatively, often due to pain, patients may not mobilize secretions appropriately, also contributing to atelectasis. Atelectasis is the most common postoperative pulmonary complication, and is often associated with emergent and prolonged surgeries, especially those of the thorax and abdomen. Atelectasis is associated with fever, an increased respiratory rate, an increased pulse, and lung exam findings ranging from normal to rales and decreased breath sounds. Symptoms usually present within the first 48 hours postoperatively. Pulmonary aspiration pneumonitis, although possible, is less likely due to appropriate preoperative and intraoperative measures being utilized to decrease risk. Pneumonia is also a common postoperative complication, due to the same contributing factors as atelectasis. Physical exam findings may also be similar. However, postoperative pneumonia is likely to become evident between 24 and 96 hours postoperatively. A postoperative pleural effusion may form, due to free peritoneal fluid as well as a complication of atelectasis, but has a lower incidence of occurrence than atelectasis alone. Patient symptoms will be based upon the size of the effusion, associated inflammation, and whether or not the effusion is infectious. Consideration must be given to pulmonary emboli for any post-surgical patient with tachypnea, tachycardia, and dyspnea. Pulmonary emboli may occur at any point postoperatively, but atelectasis remains a more common cause of postoperative fever and respiratory changes.

A 60-year-old man is 3 years post-coronary artery bypass grafting. His last lipid profile showed: Lipid Result Triglycerides 165 mg/dL HDL 34 mg/dL LDL 135 mg/dL He is normotensive and almost chest-pain-free on isosorbide mononitrate and metoprolol. Given his lipid profile, how can this patient most appropriately be managed? Answer Choices 1 Fenofibrate, 134 mg daily 2 Simvastatin, 20 mg daily 3 Atorvastatin, 80 mg daily 4 Pravastatin, 20 mg daily 5 Lovastatin, 10 mg daily

Atorvastatin, 80 mg daily Explanation Hyperlipidemia contributes to the accelerated atherogenesis in vein grafts. Thus, aggressive control of hyperlipidemia is vitally important for the patency of the grafts. The target intervention is to obtain the biggest reduction in LDL levels. According to the National Cholesterol Education Program, this patient's Serum triglycerides level falls under borderline high category, LDL under borderline high category, and his HDL cholesterol values are borderline low. Since this patient has deranged values of LDL, HDL, and triglycerides, Atrovastatin 80 mg would be the drug of choice. Atorvastatin in 10 mg doses reduces LDL up to 39% and increases HDL by 6 - 8%; in 80 mg doses, it brings about a 60% reduction in the LDL levels and up to 37% reduction in triglyceride levels. Simvastatin at a dose of 20 mg daily is able to produce a 38% decrease in LDL cholesterol combined with an 8% increase in HDL cholesterol. Fenofibrate would be a better choice if the patient had increased triglycerides because it is effective in treating hypertriglyceridemia. Pravastatin offers only 2% increase in HDL and a 32% decrease in LDL. Lovastatin in a 10 mg dose brings about a 21% reduction in the LDL level and approximately 2 - 9% increase in the HDL level.

A 28-year-old woman presents for a well woman examination. She complains about a foul smelling vaginal discharge. She thinks it may be greenish in color. She does have a history of sexually transmitted disease, including chlamydia. She has been treated for it twice. A Pap smear a year ago showed evidence of HPV infection, for which she had colposcopy and cryotherapy. Her last menstrual period was 10 days ago, and it was normal. Past medical history is otherwise significant only for obesity and intermittent low back pain. She takes ibuprofen on a regular basis. She is not allergic to any medications. Family history is significant for breast cancer (grandmother and aunt), diabetes, hypertension, and stroke. Social history reveals that she smokes 1 pack per day and has done so for the past 10 years. She admits to alcohol use and experimentation with marijuana. She is unemployed, but she is attending night school. She lives with her mother and 3 children. On review of systems, she does not have any abdominal pain, dysuria, hematuria, diarrhea, constipation, or fever. She has not noticed any concerning lumps during breast self-examination. She has used oral contraceptives in the past, but wishes to try a different method of birth control because she frequently forgets to take the pill. On physical examination, blood pressure is 140/90 mm Hg, and weight is 245lbs. Pulse is 70 and regular, and respirations are 12. HEENT is normal. Heart and lung examinations are normal. Abdominal examination is difficult due to her body habitus, no abdominal tenderness, and normoactive bowel sounds. Breast examination is normal. External genitalia are normal. On speculum examination, the cervix is parous, and there are no suspicious lesions. A Pap smear and cultures are obtained. Bimanual examination shows there to be no cervical motion tenderness; there is no adnexal mass or tenderness. Question What Pap smear result is potentially the most serious and requires more aggressive investigation and treatment? Answer Choices 1 Normal squamous cells but lack of endocervical cells 2 Colonization with Gardnerella vaginalis and Candida 3 Atypical glandular cells, not otherwise specified 4 Atypical squamous cells of undetermined significance 5 Substantial colonization with Trichomonas vaginalis

Atypical glandular cells, not otherwise specified Explanation Atypical glandular cells, not otherwise specified ( AGC-NOS) indicate a much higher likelihood of more serious underlying disease than atypical squamous cells of undetermined significance (ASCUS). AGC-NOS is found on less than 1% of all Pap smears, but on further study a significant percentage (20 - 50%) of these patients will be found to have high-grade pre-invasive disease, carcinoma in situ, or invasive adenocarcinoma. Colposcopy is recommended for these patients, and depending on the findings, conization biopsy should be strongly considered. ASCUS Paps can simply be repeated once any underlying infection is treated, and does not immediately require colposcopy or more invasive investigation. A lack of endocervical cells is indicative of an inadequate specimen. The Pap can simply be repeated at an interval best determined by underlying risk factors, but there is no need for an immediate colposcopy. Gardnerella and Candida are frequently part of the normal vaginal flora, and they do not need to be treated unless symptomatic. Trichomonas is a sexually transmitted disease and is readily treated with metronidazole. While it is important to treat this infection, the more serious finding would be abnormal cytology. Having this infection does not increase risk of future cervical dysplasia; however, certain strains (16 and 18) of human papillomavirus do increase risk.

Which of the following is the most common cause of primary adrenal insufficiency in the United States? A Tuberculosis B Adrenal hemorrhage C Lymphoma D Autoimmune destruction E Metastatic carcinoma

Autoimmune destruction The correct choice is D, autoimmune destruction. This is responsible for 80% of cases of primary adrenal insufficiency in the United States. All of the other choices can cause adrenal insufficiency, but they are less common. Tuberculosis, choice A, is a common cause of adrenal insufficiency in other areas of the world, where the infection is more common. Bilateral adrenal hemorrhage, choice B, can occur as a complication of sepsis, heparin use, anti-phospholipid syndrome, and after major trauma or surgery. Lymphoma, choice C, and metastatic carcinoma, choice E, are rare causes of adrenal insufficiency.

A 12-year-old girl becomes comatose and is rushed to the hospital by her parents. 2 days before the admission, she went to school feeling ill. She vomited that evening. Her vomiting persisted with only an 8-hour pause during sleep. She is breathing deeply and rapidly; her breath has a fruity odor. Her parents mention that her appetite has increased. She has also been drinking lots of fluids; subsequently, she has been urinating more than normal. Urinalysis reveals 3+ glucose levels and 2+ ketone bodies. Question Based on the above information, what is the etiological cause of this patient's symptoms? Answer Choices 1 Insulin resistance 2 Increase in counterregulatory hormones 3 Autoimmune destruction of B-cells of pancreas 4 Post Epstein-Barr virus infection 5 Autoimmune destruction of pancreatic acini cells

Autoimmune destruction of B-cells of pancreas The etiology of Diabetes Mellitus Type I is autoimmune destruction of pancreatic B-cells. Some common signs and symptoms of Diabetes Type I include hyperglycemia, polyuria, increased thirst, weight loss, increased appetite, and nausea/vomiting. Blood glucose is elevated and urine may show presence of glucose. Diabetic ketoacidosis (as in this patient) may present with ketone bodies in urine and Kussmaul breathing along with other symptoms. Insulin resistance is the etiology of Diabetes mellitus Type II. A counterregulatory hormone associated with insulin is glucagon. Glucagon counters the effects of insulin, but it does not destroy the B-cells of the pancreas. Epstein-Barr virus does not affect the pancreas. The autoantibodies do not destroy the acini cells.

A 35-year-old woman presents with 5-hour history of progressive shortness of breath, cough, and wheezing. This morning she felt that she was "catching a cold" because of sore throat and thin purulent rhinorrhea, for which she took aspirin. Her past medical history is significant for persistent rhinitis resistant to therapy. Question What should your patient do to prevent future asthma attacks? Answer Choices 1 Avoid aspirin 2 Take antihistamine 3 Inhale cromolyn 4 Take albuterol 5 Get influenza vaccine

Avoid aspirin Your patient most probably has aspirin-exacerbated respiratory disease. It is chronic sinusitis characterized by nasal polyposis, nonallergic induced asthma, and aspirin sensitivity. The condition is sometimes called aspirin triad. It commonly starts in the third decade. Clinical symptoms of aspirin-sensitive patients are characterized by mucosal inflammation and rhinitis, severe asthma precipitated by aspirin ingestion, and aggressive bilateral nasal polyposis. The rhinitis is persistent and difficult to manage, and the rhinorrhea is thin and nonpurulent (nonallergic rhinitis with eosinophilia syndrome). Asthma usually appears an average of 2 years after rhinitis, followed by the intolerance to aspirin and the co-occurrence of nasal polyps. Severe acute asthma attack can occur within a few minutes and up to 3 hours after ingestion of aspirin. Aspirin challenge can be used to confirm a diagnosis of aspirin sensitivity in these patients. Antihistamines are commonly used for the relief of allergies caused by intolerance of proteins. They will not help your patient. Inhaled cromolyn or nedocromil, inhaled corticosteroids, long-acting bronchodilators, and other drugs are used to control asthma, but they will not to prevent asthmatic attack in this patient. Albuterol and other quick relief medications used to relieve acute asthma exacerbations, to prevent exercise-induced asthma symptoms, and to speed recovery from acute exacerbations, but albuterol will not prevent aspirin-exacerbated respiratory disease. Influenza vaccine is useful to prevent influenza and the eventual need for the use of nonsteroidal anti-inflammatory drugs. It will not, however, prevent aspirin-exacerbated respiratory disease.

A 1-month-old infant is being evaluated for rapid breathing, feeding difficulty, lethargy, and poor weight gain. The physical exam is notable tachypnea, tachycardia, a cardiac gallop, and a medium-pitched systolic murmur, which is best heard posteriorly in the interscapular area with radiation to the left axilla, apex, and anterior precordium. A prominent anterior chest heave is also observed. The lower extremities demonstrate a 12 mmHg pressure difference as compared to the upper extremities. Additionally, there are delayed femoral pulsations; his upper extremity pulsations are normal. Which of the following is correct regarding the long-term management of this patient? A Continued monitoring of blood pressure is unnecessary. B Evaluation by a cardiovascular surgeon is essential for definitive treatment. C Cardiac catheterizations are required in the nonsurgical approach to this patient. D Medical management of this patient is not expected to be of any benefit.

B

A 1-year-old boy presents with a rash that followed a 2-day fever of 102 degrees Fahrenheit (38° Celsius). His mother states, "I saw a pink, raised rash on his trunk when the fever ended. The rash looks like pink or red spots that turn white when I touch them." On physical exam, the child has a temperature of 98 degrees Fahrenheit (36.6° Celsius), and there is a diffuse, maculopapular rash on the chest, abdomen, and thighs. The rash spares the face, palms, and soles. The child has no symptoms of upper respiratory infection, and the physical examination is otherwise normal. What is the most likely diagnosis? Answer Choices: A Erythema infectiosum B Roseola C Rubella D Rubeola E Scarlet fever

B

A 10-month-old infant presents with a fever and painful, swollen left elbow. The swelling of the elbow restricts movement of the arm. The patient had been in good health 2 days prior. He has a body temperature of 40.8°C, pulse of 175, respiration of 40, and is in the 95th percentile (length and weight) for his age. Blood counts are significant for a WBC count of 25,000 with a left shift. A radiograph of the elbow is significant for the presence of soft tissue swelling and a widening of the joint space. The elbow joint is aspirated; a Gram stain result is significant for the presence of many WBCs and Gram-negative bacilli. 24 hours later, the culture is positive for 4+ growth on the chocolate agar, but not on the MacConkey or blood agar. The colonies are gray in appearance and have a 'musty' or 'mousy' smell. The organism is found to require the presence of hemin (X factor) and nicotinamide adenine dinucleotide (V factor) for growth. What is the cause of this joint infection? Answer Choices: A Kingella kingae B Haemophilus influenzae C Streptobacillus moniliformis D Neisseria gonorrhoeae E Streptococcus pyogenes F Staphylococcus aureus

B

A 10-year-old boy presents with chest pain and joint swelling. His mother says about 3 weeks ago he had a sore throat with fevers and cervical lymphadenopathy. He was prescribed penicillin, but stopped taking it after a day because of nausea. His symptoms soon resolved. About a week ago, he complained of knee pain and swelling and later had elbow pain and swelling. He also experienced chest pain and recurrence of his fever. His mother is concerned. His vital signs are blood pressure 110/70 mm Hg, heart rate 120/min, temperature 100.1° F. On physical examination, normal S1/S2 and a II/VI holosystolic murmur is heard, and mild knee tenderness without swelling is present. Elbow still has swelling and pain. His EKG shows a sinus tachycardia with a prolonged PR interval. His chest X-ray is normal. Question: What is the most likely diagnosis? Answer Choices: A Infective endocarditis B Acute Rheumatic fever C Pericarditis D Kawasaki's Disease E Lyme Disease

B

A 13-month-old boy presents with a rash. The mother tells you that the child has had high fevers over the past 4 days, although he has not had a fever for the past 24 hours. The rash began 6 hours ago; it started on his chest and back, and it spread to his neck, face, and arms. The child doesn't appear to be itchy, and he has been acting normally since the fever subsided. The mother denies cough, runny nose, vomiting, and diarrhea. The only medication that the child has taken is acetaminophen. On examination, the child is happy and playful. The only physical finding is the rash that is shown in the image. What is the most likely diagnosis? Answer Choices: A Rubella B Roseola C Measles D Drug hypersensitivity E Varicella

B

A 13-year-old girl presents to the emergency department with complaints of febrile episodes (Tmax 102°F), joint aches in her knees and wrists, chest pain and a raised, red rash. She denies sexual activity or intravenous drug use. Vital signs are BP 90/60mmHg, HR 115/min, T 101°F, RR 25/min. Her physical exam is remarkable for diffuse, scattered ring shaped macules on her extremities that is consistent with erythema marginatum, a III/VI systolic ejection murmur, and guarded passive range of motion in wrists and knees bilaterally with no apparent swelling. Laboratory findings are as follows; WBC 16,000mcL, Hematocrit 35%, Platelets 350,000mcL, ESR greater than 15mm/h and a positive antistreptolysin O titer. Question: What should you suspect as this patient's diagnosis? Answer Choices: A Still's Disease B Acute rheumatic fever C Septic arthritis D Systemic lupus erythematosus E Lyme Disease

B

A 14-year-old boy presents with a 1-week history of acute watery diarrhea with abdominal discomfort and vomiting. Now he has developed facial and periorbital edema and myalgias. He is also experiencing pain and swelling of the calf muscles. The patient gives history of eating some delicacies prepared from pork and game meat in a restaurant about 3-4 weeks back. Blood examination shows eosinophilia. Muscle biopsy shows that the muscle fibers are edematous and have basophilic degeneration. Muscle fibers contain a cyst containing a coiled worm. The surrounding areas show lymphocytic and eosinophilic infiltration. Question: What is the treatment of choice for this patient? Answer Choices: A Metronidazole B Mebendazole C Thiabendazole D Nitazoxanide E Ivermectin

B

A 14-year-old boy presents with a 1-week history of acute watery diarrhea with vague abdominal discomfort and vomiting. Now he has developed fever, malaise, facial and periobital edema, and myalgias. He is experiencing pain and swelling of the calf muscles. The patient gives history of consuming some delicacies prepared from pork and game meat in a restaurant about 3 - 4 weeks back. Blood examination shows moderate eosinophilia. Question: What is the most likely etiological agent responsible for the patient's illness? Answer Choices: A Ascaris lumbricoides B Trichinella Spiralis C Taena solium D Necator Americanus E Giardia lamblia

B

A 14-year-old boy presents with painful swelling of his right index finger. He bites his nails regularly, and has had the same problem 3 times in the past. There are no other known comorbidities. The ER attending requests an orthopedic consultation in order to make a management decision. The clinical picture noted by the on-call orthopedic resident is shown. Question: What is the best next step in management? pic = pus-filled fingertip Answer Choices: A Oral Amoxycillin-clavulanate B Surgical drainage C Circumferential debridement D IV antibiotics E Topical antifungal

B

A 15-year-old girl presents with a 1-hour history of rapid heartbeat, faintness, sweating, and nervousness. She is also experiencing shortness of breath and chest pain. The patient has no significant past medical history. There is no history of similar episodes. The patient is on no medications, and she denies illicit drug use. On exam, her vital signs are BP70/60 mmHg; pulse 200 bpm; RR 22/min, temperature afebrile. She looks pale, and her palms are slightly sweaty. She is not comfortable sitting up, so she prefers lying down. She looks slightly apprehensive. Her heart and lung exam are negative except for the tachycardia; except for cool sweaty hands, a brief abdominal and extremity exam are non-revealing. The physician quickly places the paddles on the patient's chest to record the rhythm; this shows a narrow-complex regular tachycardia at 210 bpm. He requests oxygen, IV line, and continuous monitoring. An EKG is in the process of being completed. At this point, what should be done? A Carotid sinus massage B Synchronized cardioversion C Adenosine 6 mg IV push D Diltiazem 10 mg IV push E Verapamil 5 mg IV push

B

A 15-year-old girl with no record of any significant illness in the last 2 years presents after a 5-day history of fever. She has no cough, her lungs are clear, but she has a fever of 38.2°C. There is the presence of a significantly large circular rash on her abdomen. Her history is notable for having recently returned from spending the early summer at her grandparent's cottage on the shoreline of Connecticut. She spent most of her time hiking through the meadows and collecting photographs. She submits a jar containing an insect that she found feeding on her body. You analyze the insect microscopically (refer to the image). A CBC is ordered and the results are unremarkable. Based on the clinical history and presentation, you should order tests for what infection? Answer Choices: A Francisella tularensis B Borrelia burgdorferi C Rickettsia rickettsii D Ehrlichia chaffeensis E Babesia microti

B

A 2-year-old girl presents with a fever and an inflamed throat. Her mother states the child does not want to eat. You decide to treat her with 7 days of penicillin. After 4 days, the mother calls your office to report that the child has not significantly improved. You suspect the child's infection may be caused by penicillin-resistant bacteria. What mechanism is involved in this case? Answer Choices: A Mutations in the enzyme glycopeptide transferase B Acquisition of the enzyme β-lactamase C Alteration in bacterial cell wall composition D Inactivation of the enzyme β-lactamase E Acquisition of the enzyme phosphotransferase

B

A 21-year-old man presents with an acute onset of nausea and vomiting. He is a college student. The symptoms began in the middle of the night. He denies alcohol or other drug ingestion. When he returned from his evening class, he reheated some fried rice from yesterday's takeout, did a little reading, and went to bed. On physical exam, he appears pale and tired. The abdomen is mildly tender, but there is no guarding. He is afebrile, pulse is 103, and blood pressure is 100/70. Food-borne illness is suspected, and the patient is advised to drink fluids and return to the clinic the following day. As anticipated, at his next visit, the patient's symptoms have resolved completely. Based on the clinical scenario, what agent is most likely to have caused this patient's illness? Answer Choices: A Staphylococcus aureus B Bacillus cereus C Clostridium perfringens D Giardia lamblia E Entamoeba histolytica

B

A 24-year-old man presents with a 2-day history of skin rash on his back. He also notes pain in his joints, tiredness, and feeling uneasy all the time. He has just returned from a camping trip. Vital signs are normal. Examination reveals an erythematous, expanding lesion with concentric circles. The rash is shown in the image. If left untreated, what is a long-term complication of this patient's condition? Answer Choices: A Endocarditis B Myocarditis C Restrictive cardiomyopathy D Uveitis E Endarteritis obliterans

B

A 24-year-old man recently returned to the United States from a South American vacation; he presents with an itchy, red, palpable, serpiginous lesion on the pretibial skin. The skin lesion has slowly lengthened during the past week. What is most consistent with this presentation? Answer Choices: A Bubonic plague B Cutaneous larva migrans C Lyme disease D Viral exanthem E Acute rheumatic fever

B

A 24-year-old woman started her new job as an elementary school teacher. She graduated from a college in Colorado and looked forward to her first full-time job. During the winter months, many of her students became ill with fevers, runny noses, and headaches. The teacher also experienced the same symptoms, which lasted several days then disappeared. A week or so later, the illness reappeared with similar symptoms, sometimes with a sore throat. Some students received antibiotics. The symptoms described above are characteristics of what condition? Answer Choices: A Meningitis B Common cold C Flu D Encephalitis E Conjunctivitis

B

A 25-year-old landscaper presents with a 2-week history of generalized malaise and an 'unusual rash' on his right thigh. The patient reports that this rash has been widening, but denies any pruritus or pain in association with his other symptoms. In the past week, he has also noticed a constant headache and mild fever. His past medical history is unremarkable. The physical exam reveals vital signs within the normal limits, enlarged, non-tender diffuse lymph nodes in cervical and inguinal areas, as well as an erythematous rash with central clearing and few satellite lesions. You immediately assume that the patient has Lyme disease. What would you suggest as the initial line of therapy? Answer Choices: A Fluoroquinolones B Tetracyclines C Aminoglycosides D Corticosteroids E Fluconazole

B

A 25-year-old man with no prior history of any significant illness comes to your office presenting with a wound infection to the hand; he acquired the wound 3 weeks ago, and it has been worsening. Physical examination of the hand is remarkable for the presence of an initial wound site that is ulcerated and surrounded with papules that are blue-purple in color. Nodules are present (of lymphatic origin) ascending from the wound site on the forearm. The patient acquired the initial wound while at work at a pet store, where he accidentally cut his hand while cleaning out an aquarium. Material for bacterial, fungal and mycobacterial culture is obtained from the wound site. Special instructions on incubation requirements are given for the mycobacterial culture. Bacterial and fungal cultures prove to be negative; however, at day 7 there is 4+-mycobacterial growth detected that produces the following characteristics: GROWTH RATE NIACIN NITRATE REDUCTION OPTIMUM TEMPERATURE TWEEN HYDROLYSIS UREASE 7 DAYS NEG NEG 32° C POS POS Based on the clinical and isolate characteristics, what is the infection in the hand of this patient? A Mycobacterium kansasii B Mycobacterium marinum C Mycobacterium gordonae D Mycobacterium avium-intracellulare E Mycobacterium tuberculosis F Mycobacterium bovis

B

A 25-year-old sexually active man presents with what he describes as an infection of his penis. His history is significant for having had sex with a prostitute in the past 2 months. He has no history of fever, chills, headache, joint pain, malaise, or anorexia. There is no history of drug use. Physically, he appears in very good shape and all vital signs are normal. Physical examination of the penis is significant for the presence of a chancre that is located on the glans penis. The base appears smooth with raised borders. Although painless, the chancre is sensitive to touch. There appears to be little to no exudate at the chancre. The patient also has the presence of regional lymphadenopathy, which is non-tender to the touch. The chancre is scraped for material to be examined. The scraped material is placed on a slide with sterile isotonic saline and coverslipped. It is observed under dark field microscopy and is reported by the laboratory as positive. What is the most likely diagnosis? Answer Choices: A Chancroid B Primary stage syphilis C Primary genital herpes D Localized candidiasis E Anogenital warts F Bacterial vaginosis

B

A 26-year-old man was diagnosed HIV-positive 4 months ago, and he now wants to discuss potential treatment. Following a history, physical examination, and laboratory testing, it is determined that the patient has no signs of HIV disease. His CD4+ T-cell count is 400/mm3 and plasma viral load is 8,000 copies/mL. As part of the care for this patient, a series of vaccinations are recommended. What is an acceptable vaccine battery in this case? Answer Choices: A Hepatitis A, DPT, oral typhoid B Pneumococcus, DPT, hepatitis A C Influenza, DPT, BCG D Oral typhoid, MMR, meningococcus E Hepatitis B, hepatitis A, yellow fever

B

A 27-year-old man presents with burning pain during urination and a 5-day history of urethral discharge. He sees his family physician because of his complaints. Several laboratory tests are done, including a Gram stain and culture on a sample of the discharge. The results are negative, and gonorrhea is ruled out. What is the most common cause of his condition? Answer Choices: A Chlamydia psittaci B Chlamydia trachomatis C Mycoplasma genitalium D Trichomonas vaginalis E Mycoplasma hominis

B

A 28-year-old woman presents with a 7-year history of AIDS. 1 month ago, her CD4 cell count was 48 cells/mm3 (stable over the past 3 months) and her viral load was detectable at 6,500 copies/mL. She feels well, and she is taking antiviral medication for her HIV infection. To date, she has not had any opportunistic infections. She is not on any medications for prophylaxis of opportunistic infections. Question: Based on her current CD4 cell count, you would expect her to be on what medication(s) for prophylaxis (in addition to her other medications)? Answer Choices: A Trimethoprim/Sulfamethoxazole only B Trimethoprim/Sulfamethoxazole and azithromycin C Trimethoprim/Sulfamethoxazole, oral gancyclovir, and azithromycin D Trimethoprim/Sulfamethoxazole and oral gancyclovir E Trimethoprim/Sulfamethoxazole, pyrimethamine, and leukovorin

B

A 3-year-old boy presents with a 7-day history of fever. On physical examination, you note bilateral non-exudative conjunctivitis, strawberry tongue, and cervical lymphadenopathy. The boy also has a raised, deep plaque-like rash on his trunk and proximal extremities. You order a CBC; it shows moderate leukocytosis. Question: What condition does this child have? Answer Choices: A Scarlet fever B Kawasaki disease C Rheumatic fever D Streptococcal pharyngitis E Henoch-Schönlein purpura

B

A 30-year-old man presents to his primary care physician's office with a 3-day history of having an itchy scalp. He was recently on vacation and slept on the floor of a house with pets. Otherwise, nothing has changed recently in his daily living and he denies all other symptoms. His overall health is good. Which physical examination description is mostly closely associated with tinea capitis? A Round patches of hair loss with normal appearance of scalp B Erythematous, scaly patches associated with patchy hair loss C Group of 3-5 inflamed, tender nodules that have purulent drainage D Thick patchy areas with silvery scales attached to hair shafts E Erythematous, greasy skin covered in yellow scales

B

A 30-year-old woman presents with lower abdominal pain; she is thought to have pelvic inflammatory disease (PID). She admits to prostitution and illicit drug use. Labs on admission reveal blood glucose of 260 mg/dL, a positive HIV screen, and a non-reactive RPR. Aside from the abdominal/pelvic pain, the admitting physical is also notable for moderate obesity, the absence of pronounced lymphadenopathy, and an erythematous macerated rash in the intertriginous distribution. The patient reports her rash is "really itchy and wet all the time", and it began within the last 3 months; it is now at its worst. She also reports intense itching of the vulva over the last few weeks. The intertriginous rash is most likely a manifestation of what condition? Answer Choices: A Atopic dermatitis B Candidiasis C Lichen planus D Psoriasis E Secondary syphilis

B

A 35-year-old woman with acute lymphocytic leukemia and undergoing chemotherapy suddenly developed small vesicular eruptions across her left breast. The eruptions were asynchronous, painful, and lasted 6 days before clearing. She scratched the affected area, which only intensified her discomfort. The affected area developed small vesicles which progressed to ulcerated and crusted pustules, causing her clothing to stick to the open sores. She had a similar occurrence last year. The appearance of localized cutaneous lesions over a specific dermatome is most characteristic of what agent? Answer Choices: A Chickenpox B Shingles C Generalized herpes simplex D Measles E Rickettsialpox

B

A 36-year-old woman presents with fatigue, breathlessness, and abdominal discomfort and swelling. The onset of symptoms was gradual. She is experiencing prominent pulsations in the neck. She also gives a history of pharyngitis caused by Streptococcus during her childhood. Physical examination reveals raised jugular venous pulsations and prominent a waves. Extremities show presence of 2+ edema, and abdominal examination reveals a positive fluid thrill. Cardiac examination shows a prominent right atrium on palpation to the right of the sternum. Auscultation reveals a tricuspid opening snap with a diastolic murmur heard along the left sternal border, which increases with inspiration. The first heart sound is split widely. Diagnostic studies reveal an enlarged right atrium on chest X-ray. There is right atrial enlargement and valvular structural deformity in the tricuspid area. What is the pathophysiological cause of the fluid thrill seen in the abdomen in this patient? A Seeding of the peritoneum by rheumatic vegetations from the tricuspid valve, leading to inflammation of the peritoneum and effusion B Increase in the right atrial pressure causes venous congestion and forms ascites C Myxoma of the right atrium leading to back-flow pressure and ascites D Congenital tricuspid stenosis leading to obstructed venous flow, back flow pressure, and ascites E Tuberculosis of the peritoneum and pericardium leading to cardiologic manifestation and ascites

B

A 36-year-old woman returning from a conference in Indonesia presents with profuse watery diarrhea. She reports having consumed fried rice, iced tea, and fruit at a street market several hours before her flight to the United States. The patient has 4 - 6 bowel movements daily, and they are preceded by cramps and nausea. She also has overwhelming thirst. Examination reveals dehydration, diaphoresis, orthostatic hypotension, and abnormal laboratory values for potassium (2.8 mmol/L). No leukocytes are seen in stool specimens, and cultures yield an oxidase-positive curved Gram-negative rod on blood and TCBS agar. What is the most probable etiology of the patient's illness? Answer Choices: A Clostridium perfringens B Vibrio cholerae C Clostridium difficile D Campylobacter jejuni E Shigella sonnei

B

A 40-year-old male inmate, who was born in the United States, has been incarcerated for the past 20 years; he gets a required annual PPD. 2 days later, the nurse, who is a new graduate, reads his PPD at 11 mm; she verbally informs you of this result. However, all of his previous readings have been 0 mm, and he has not transferred to any other institution in the past 5 years. There are no known cases of active TB in your facility. Question: What should be your next step? Answer Choices: A Obtain a chest X-ray B Recheck his PPD C HIV screen D Obtain a Hepatic Panel/CBC E Obtain a Quantiferon-TB Gold test

B

A 40-year-old man presents for an evaluation of a scratch sustained after he entered a room with a bat. Since he fell after being startled, he is unsure if this scratch was from contact with the animal. The bat subsequently flew out the window. On exam, he has a tiny scratch; there are no puncture marks. What is the most appropriate course of action? Answer Choices: A Tell the patient to monitor symptoms B Initiate immune globulin, rabies vaccination, and tetanus immunization C Monitor symptoms and give a tetanus vaccination D Administer only immune globulin E Start oral antibiotics to prevent infection

B

A 40-year-old man presents with irregular heartbeats lasting several days. His past medical history is significant for the presence of mitral valve stenosis and atrial fibrillation (AF). He takes beta blockers regularly. His ECG shows atrial fibrillation with an irregular heart rate around 80. To prevent further complications, you decide to restore his sinus rhythm by cardioversion and prescribe what treatment? A Benzodiazepine B Warfarin C Digoxin D Amiodarone E Heparin

B

A 42-year-old hypertensive, diabetic Native American woman is scheduled for a follow-up visit today. She says that her father recently passed away after having a heart attack. She is extremely worried and wants to know what she can do to reduce her risk of cardiovascular disease (CAD). You explain to her that that the risk factors for CAD are classified as either non-modifiable or modifiable. What is the strongest non-modifiable risk factor for CAD in this woman? A Her age B Her ethnicity C Hypertension D Diabetes E Her gender

B

A 42-year-old man with a history of HIV presents with blurred vision. The patient also notes occasional small spots in his vision. The patient is non-compliant with his medications. Physical examination reveals vital signs of temperature 100.8°F, BP 114/80 mm Hg, RR 12/min, and pulse 76 BPM.. Decreased visual acuity as well as loss of peripheral vision is demonstrated. Fundoscopic examination is shown below (hemorrhages and cotton wool spots). Blood work is notable for a CD4 count of 30 cells/mm3. What organism is most likely responsible for this patient's symptoms? Answer Choices: A Mycobacterium-avium intracellulare B Cytomegalovirus C Toxoplasma gondii D Epstein-Barr Virus E Herpes Simplex

B

A 43-year-old woman is admitted for nausea, emesis, ataxia, and dizziness. Previous medical history reveals HIV positive status for 4 years, with the most recent CD4 T-cell count of 100/mm3. Vital signs and neurologic exam are normal. An MRI shows 2 ring-enhancing lesions of the basal ganglia, each approximately 1 cm in diameter. The patient is treated with sulfadiazine plus pyrimethamine, and she is discharged following rehydration. Within 1 week, the patient's symptoms improve; a 2nd MRI 4 weeks later shows a significant reduction in the size of the lesions. What is the most likely diagnosis? Answer Choices: A Lymphoma B Toxoplasmosis C Mycobacterial abscess D CMV encephalitis E Fungal abscess F HIV encephalitis

B

A 45-year-old man is admitted to the hospital with complaints of fever, weakness, weight loss, muscle and testicular pain, and a rash on his legs. He states that his symptoms began about 1 week ago. The testicular pain began about 1 day ago and has increased significantly over the last 24 hours. He denies recent illness or injury and states that has been in good health for as long as he can remember. On physical exam, the patient was well-developed, well-nourished, and in mild physical distress. His blood pressure was elevated at 152/94mmHg, and a chest radiograph was negative. Laboratory analysis revealed an elevated sedimentation rate and C-reactive protein, elevated BUN, and creatinine. His red blood cell count was decreased, and his ANCA was negative. An arteriogram showed diffuse arterial saccular aneurysms and narrowing of the arteries. What is the most likely cause of this patient's symptoms? A Systemic lupus erythematosus B Polyarteritis nodosa C Rheumatoid arthritis D Sjögren syndrome E Kawasaki disease

B

A 45-year-old woman develops a high fever, severe headache, cough, and muscle aches. She is quite upset about missing work because she owns her own pet store and has no employees to cover for her in her absence. She tries to work for a day despite her illness, but she is unable to do so. The next day, she seeks medical attention. On physical examination, she is found to be febrile. Based on her occupational history and her physical examination, you suspect psittacosis. From what animal did she probably acquire the psittacosis? Answer Choices: A Squirrel B Bird C Dog D Cat E Rabbit

B

A 45-year-old woman reports for follow-up of a hypertension diagnosis. In her last 2 visits, her BP was 145/90mmHg and 150/85mmHg. Today her BP is 146/92mmHg. Physical examination and review of systems is unremarkable. You decide to initiate therapy with candesartan cilexetil (ATACAND®) 16mg/daily. Provided the patient does not have an urgent reason to visit you, in what time frame should the follow-up visit be scheduled in order to evaluate the efficacy of antihypertensive therapy? A After 1 week B After 4 weeks C After 8 weeks D After 10 weeks E After 12 weeks

B

A 45-year-old woman with diabetes and an estimated glomerular filtration rate of 58 mL//min/1.73 m2 and a spot urine albumin to creatinine ratio of 30 mg/g presents for evaluation. She has had blood pressure readings ranging from 150/80 mm Hg to 160/90 mm Hg. She is otherwise healthy and asymptomatic. Her family history is notable for renal disease. What blood pressure is considered adequately controlled? A 150/90 mm Hg B 140/80 mm Hg C 130/98 mm Hg D 130/80 mm Hg E 90/60 mm Hg

B

A 50-year-old man presents for the evaluation of a 1-year history of progressive cognitive, motor, and behavioral problems. He complains of inattention, reduced concentration, slowing of processing, and difficulty changing mental sets. What started as slow movements now is clumsiness and problems with coordination. His friend states that a patient is "not himself anymore" and has become apathetic, non-communicative, and "down." He is HIV positive and was diagnosed with AIDS 2 years ago because of the presence of Pneumocystis carinii with CD4 of 100. However, he had an excellent response to antiretroviral therapy, and his last CD4+ lymphocyte counts were normal and viral load undetectable. On examination, you find an apathetic male in mild distress. Neurological exam shows loss of coordination, unsteadiness, generalized weakness (more pronounced in legs), ataxia, and tremor. Question: What should be the next diagnostic step in this patient? Answer Choices: A Neuropsychological testing B Neuroimaging methods C Electroencephalography D Cerebrospinal fluid examination E CD4+ lymphocyte counts

B

A 56-year-old man presents with a 1-week history of palpitations and shortness of breath. He has a long-standing history of poorly controlled hypertension. Physical examination reveals an elevated blood pressure of 190/98 mm Hg, elevated jugular venous pressure (JVP), mild hepatomegaly, bilateral pedal edema, and rales at the lung bases. Diagnostic studies reveal concentric left ventricular hypertrophy without significant valvular abnormalities on echocardiogram. What drug is beneficial in the treatment of the patient's condition by virtue of both afterload and preload reduction? A Loop diuretics (e.g., furosemide) B Angiotensin-converting enzyme inhibitor (e.g., enalapril) C Positive inotropic agents (e.g., digoxin) D Sodium channel blocker (e.g., procainamide) E Arterial vasodilators (e.g., hydralazine)

B

A 58-year-old man presents with a 6-month history of constipation that alternates with diarrhea. 3 years ago, he was treated for intestinal amoebiasis that he acquired after a trip to Mexico. His family history of appearance of colorectal carcinoma is positive; both of his parents had colorectal cancer at the ages of 59 and 61 years. His general physical findings are unremarkable. Routine laboratory analyses reveal only increased erythrocyte sedimentation rate (53-mm/1st hour), and no trophozoites or cysts of Entamoeba histolytica can be discovered in his stool. A striking narrowing of colonic lumen is observed in the cecum after barium enema, but the patient is reluctant to undergo colonoscopy. What else should be done to properly manage this patient? Answer Choices: A Undertake surgical treatment of involved cecal lesion B Perform indirect hemagglutination test for amoebiasis C Search for presence of E. histolytica in patient's stool again D Perform a CT examination of abdomen E Determine a blood level of carcinoembryonic antigen (CEA)

B

A 6-month-old infant is brought in for routine vaccination. Measles still continues to be a major health problem worldwide. Sporadic endemics continue in the United States in spite of efforts to eradicate the disease. The vaccine used for measles is a live-attenuated vaccine, and the age at which it is given is very important. What is the current vaccination guideline for measles? Answer Choices: A 6 - 8 months, 10 years B 12 - 15 months and 4-6 years C 1 months, 2 months and 6 months D 11 - 12 months and 12 - 14 years E 2 months, 4 months, 6 months

B

A 68-year-old man with a past medical history of diabetes mellitus type II, hyperlipidemia, myocardial infarction 1 year ago, and congestive heart failure with left ventricular ejection fraction of 35% is rushed to his local emergency room by his wife after he collapsed and became unresponsive at their residence. He admitted to her that he had been experiencing severe chest pain and pressure, fatigue, palpitations, diaphoresis, and lightheadedness for several minutes prior to his collapse. His present medications include aspirin, atorvastatin, lisinopril, glipizide, and carvedilol. Upon physical exam, he is found to have a blood pressure of 60/palpable, is pulseless, and has gasping respirations. His troponin T level was found to be elevated at 0.2 ng/ml, and troponin I level elevated and measured to be 0.25 ng/ml. The admission ECG revealed bizarre, irregular, random waveform, no clearly identifiable QRS complexes or P waves, and a wandering baseline. Following appropriate stabilization, what is best next step for this patient? A Prophylactic lidocaine B Implanatable cardioverter-defibrillator (ICD) implantation C Long-term metoprolol use D Percutaneous coronary intervention (PCI) E Discontinue aspirin, atorvastatin, and lisinopril

B

A 68-year-old man with a past medical history of diabetes mellitus type II, hypothyroidism, and hyperlipidemia presents with a constant moderate to severe "squeezing, pressure, and tight" left-sided chest pain for 1 hour. Additionally, he admits to shortness of breath, nausea, productive cough with a frothy sputum, and profound diaphoresis. He notes that he has had a 1-week history of similar, recurrent chest pain of about 10 minutes duration that has been occurring following exposure to the cold weather and following consumption of a meal. He denies chills, abdominal pain, diarrhea, cough, and pleurisy. His physical exam reveals tachycardia, hypertension, cyanosis, cool and moist skin, diaphoresis, an S3 gallop, and evidence of painful respiratory distress. His lung exam is noteworthy for diffuse crackles. Bedside electrocardiogram demonstrates sinus tachycardia, ST-segment elevations, and occasional premature ventricular contractions. What is the most appropriate treatment of the pain, hypertension, and suspected pulmonary edema in this patient? A Clopidogrel B Nitroglycerin C Metoprolol D Lisinopril E Nifedipine

B

A 7-year-old boy presents with a 3-day history of fever, abdominal pain, and severe diarrhea. The diarrhea is described as watery and bloody. Before falling ill, the boy was at a friend's birthday party and ate some chicken. His stool demonstrates many WBCs and S-shaped bacteria that are motile. Gram stain results indicated Gram-negative rods. What antimicrobial agent should be used to treat this patient? Answer Choices: A Metronidazole B Erythromycin C Ampicillin D Ciprofloxacin E Doxycycline

B

A biochemistry professor cuts short his trip to South East Asia after developing malaria. When he gets home, he reviews his prescription with his family physician and is curious to know the mechanism of action of the drug he is taking. He is told that the drug inhibits synthesis of nucleic acids within the organism as well as inhibits the parasites' ability to metabolize and utilize erythrocyte hemoglobin. What drug is he taking? Answer Choices: A Aspirin B Chloroquine C Metronidazole D Suramin E Melarsoprol

B

A woman comes to your office after a 6-month sabbatical during which she worked in caves in the eastern part of South America. The patient presents with fever, chills, productive cough, and joint stiffness that started 1 month before her return. Physical exam reveals 3 ulcerated lesions on her inner cheek. What is the diagnosis? Answer Choices: A Acute pulmonary eosinophilia B Histoplasmosis C Pulmonary actinomycosis D Acute coccidioidomycosis E Pulmonary asbestosis

B

An 18-year-old boy seeks help from you because of bumps he has noticed on and near his penis. They have been present for weeks. He states he has no pain, but sometimes they itch. He has done nothing to treat them. He denies dysuria, urinary frequency, or urethral discharge. He has been sexually active over the past year with 3 or 4 girls from his high school. On examination, you find condylomata accuminata. Based on the history and examination, what is a true statement regarding this diagnosis? Answer Choices: A Condoms are effective prevention B Imiquimod may be therapeutically effective C Spirochetes are the causative organism D The lesions are generally pre-malignant E Visible lesions must be present for contagiousness

B

An 18-year-old man presents to a university health clinic with a very sore throat and headache. He has had severe pharyngitis for about 12 days. He has felt exhausted and anorexic for 3 to 4 weeks. On physical exam, he has a temperature of 40 degrees C and his BP is 118/75. On palpation, his spleen is slightly tender and moderately enlarged. Blood work shows relative and absolute lymphocytosis; it also shows some atypical cells. Most of the lymphocytes are identified as T cells. Based on these findings, what is the diagnosis? Answer Choices: A Streptococcus pyogenes pharyngitis B Infectious mononucleosis C Influenza D Mycoplasmal pneumonia E Herpes simplex virus pharyngitis

B

Case #336560: A 73-year-old man with no significant past medical history presents with a 1-month history of light-headedness, dizziness, and near syncope; it has been occurring in response to sitting up and standing from a supine position. He denies chest pain, palpitations, shortness of breath, cough, loss of consciousness, vision or speech changes, nausea or vomiting, numbness, tingling, paresthesias, and focal weakness. His physical exam is noteworthy for a drop of systolic blood pressure of 24 mm Hg from a supine to standing position. What clinical intervention should be recommended to this patient? A Begin clonidine or a diuretic B Fludrocortisone and compression stockings C Large, carbohydrate meal consumption D Sodium and water restrictions E Rise rapidly from seated and supine positions

B

Following an uncomplicated pregnancy, a 34-year-old woman gives birth to a healthy boy. 2 days after birth, the woman develops a low-grade fever and vesicular lesions on her face and abdomen. History reveals an outbreak of chickenpox in her 2-year-old daughter's day care facility. The mother is diagnosed with chickenpox. What is the best management for the newborn? Answer Choices: A Varicella vaccine B Varicella-zoster immune globulin C Refrain from breast feeding D Prophylactic acyclovir E Monitor for signs of chickenpox

B

The family of viruses known as herpes viruses includes Epstein-Barr virus, varicella-zoster, and herpes simplex. A herpes simplex virus infection usually occurs during infancy or early childhood, and it causes a mild, acute illness with stomatitis and slight fever. The virus travels up the axons of sensory nerves to the trigeminal ganglion supplying the mouth and related areas. It is challenging to treat a herpes simplex infection because it exhibits what? Answer Choices: A Antigenic shift B Latency C Antigenic drift D Molecular mimicry E Gene conversion

B

The mechanism of the anticoagulant effect of low molecular weight heparin is A Degradation of activated coagulation factors B Forming a complex with antithrombin III C Inhibition of a vital enzyme in the coagulation factor activation D Inhibition of coagulation factors synthesis E Stimulating the production of anticoagulant proteins

B

What clinical condition is associated with the human papilloma virus? Answer Choices: A Hepatocellular carcinoma B Cervical cancer C Herpangina D Bone break fever E Gingivostomatitis

B

You are the first responder to a call for help broadcast over the intercom at a local supermarket. On arrival, you find an elderly woman who is unresponsive. Paramedics are on the way. You perform 2 rescue breaths upon confirming absence of effective respiratory effort. Unable to palpate a carotid pulse, you decide to initiate single-rescuer cardiopulmonary resuscitation (CPR). Based on the American Heart Association's Guidelines, what is the recommended ratio of chest compressions to ventilations for CPR? A 30:1 B 30:2 C 15:2 D 15:1 E 10:1

B

Your 25-year-old patient is 31 weeks pregnant. Her 3rd-trimester chlamydia screening comes back positive. Question: How do you proceed to counsel the patient? Answer Choices: A Take Doxycycline 100 mg BID X 7 days B Treat with Azithromycin 1 gm PO x 1D, then a test-of-cure will be performed when she comes for her Group Beta strep screening C Abstain from sexual intercourse until the last day of treatment completion D Take Erythromycin base 50 mg/kg/day orally divided into 4 doses daily for 14 days E Research has shown that treatment of sexual partners has no effect on the recurrence of chlamydia

B

A 34-year-old male presents to the primary care office with a complaint of heartburn that has been present for three months. He has symptoms two to three times a week, which occurs about 30 minutes after eating. He has tried over-the-counter antacids and they were helping to relieve his symptoms for a few months, but they are not working well now. He denies dysphagia, odynophagia, or weight loss. You decide to treat him with a proton pump inhibitor at this visit, and he achieves good symptomatic relief with this therapy. What length of therapy is appropriate in this patient? A Two to four weeks B Eight to twelve weeks C Four to six months D One year E Continue indefinitely

B 8-12 weeks If a patient achieves good symptomatic relief with a course of an empiric, once-daily proton pump inhibitor, therapy may be discontinued after eight to twelve weeks.

A 47-year-old female patient is diagnosed with a duodenal ulcer. She was determined to have H. pylori infection. Which of the following medications used to treat peptic ulcer disease is specifically used to eradicate H. pylori? A Omeprazole B Amoxicillin C Ranitidine D Lansoprazole E Misoprostol

B Amoxicillin Amoxicillin is an antibiotic that is specifically used to eradicate the H. pylori bacteria. Omeprazole and lansoprazole are proton pump inhibitors, and are used in peptic ulcer disease to suppress acid production in the stomach. Ranitidine is an H2 receptor antagonist, and its role is also to reduce acid production in the stomach. Misoprostol is a prostaglandin analog that stimulates gastric and duodenal mucus and bicarbonate secretion.

Your patient is a 22-year-old male who has experienced significant dysfunction for the past eight months. He exhibits only one symptom of schizophrenia, but that one symptom has been present for a significant part of each day. Which of the following would qualify the patient to be diagnosed with schizophrenia according to the DSM-IV TR? A A firm belief that someone is following him B An auditory hallucination in which two or more voices are conversing C A sensation that bugs are crawling on his skin D Refusal to go out without a hat, certain he will become ill without one E Speaking entirely in rhymes that are related but mostly nonsensical

B An auditory hallucination in which two or more voices are conversing B Generally, two or more characteristic symptoms of schizophrenia are required in order to make a diagnosis unless the one symptom present is a bizarre delusion or an auditory hallucination (B) that consists of either a running commentary or two voices conversing. Positive symptoms of schizophrenia also include disorganized or catatonic behavior and disorganized speech as well as delusions and hallucinations. Negative symptoms include affective flattening, alogia and avolition. (A) and (D) are both potentially delusions, but neither is particularly bizarre. A sensation of bugs on the skin (C, fomication) is a type of hallucination and speaking in rhyme (E) qualifies as disorganized speech. Each of these might count as one of the two required symptoms but would not be sufficient without a second symptom.

You are taking care of a 32-year-old G2P1 at 39 weeks gestation in active labor. Her pregnancy is complicated by gestation diabetes. The fetal head delivered, but the anterior shoulder did not deliver with gentle downward traction. What would be the next most appropriate action? A More forceful traction and fundal pressure B Call for assistance and McRoberts maneuver C Call for assistance and more forceful traction D Call for help and fundal pressure

B Call for assistance and McRoberts maneuver Shoulder dystocia is an obstetrical emergency, and help should always be summoned. The McRoberts maneuver increased the AP diameter, thus accommodating a large head; subrapubic pressure can help dislodge the anterior shoulder, but simple fundal pressure continues to impact it against the pelvic bone.

A 24-year-old female, with a history of type 2 diabetes, presents with the inability to conceive after 14 months of unprotected sexual intercourse with her husband. Her vital signs are unremarkable and you calculate a BMI of 31. Physical examination reveals acne vulgaris and hirsutism. Which of the following treatment options for her infertility would be the most effective considering your suspected diagnosis? A Medroxyprogesterone acetate B Clomiphene citrate C Metformin D Spironolactone E Mini-pill (progestin only)

B Clomiphene citrate Clomiphene citrate is highly effective as the first line treatment for infertility in PCOS. It can be accompanied with metformin, weight loss, exercise, and exogenous gonadotropins when clomiphene fails. PCOS in over half of patients is accompanied with obesity, abnormalities in insulin control, metabolic syndrome, and infertility. Medroxyprogesterone acetate (A) and the mini-pill (E) are used for endometrial protection and with oral contraceptive pills. Metformin (C) will help with her diabetes. Spironolactone (D) is a diuretic, which acts as a weak androgen receptor antagonist.

A 50-year-old woman presents with a history of polyuria, polydipsia, muscle aches, bone pain, nausea, and constipation for the past few months. Her past medical history is significant for a pituitary adenoma, peptic ulcer disease, and kidney stones. You are considering a diagnosis of primary hyperparathyroidism. Which of the following lab values would you expect in this patient? A Low serum calcium and elevated serum PTH B Elevated serum calcium and PTH levels C Elevated serum calcium and low serum PTH levels D Low serum phosphate and PTH levels E Elevated serum phosphate and calcium levels

B Elevated serum calcium and PTH levels The correct choice is B, elevated serum calcium and PTH levels. In primary hyperparathyroidism, excess PTH is secreted and stimulates a rise in serum calcium by increasing calcium release from bone, thereby reducing renal clearance of calcium and increasing calcium absorption through the intestine. In choice A, low serum calcium is not seen in hyperparathyroidism. In choice C and D, low serum PTH levels aren't seen in hyperparathyroidism. In choice E, elevated serum phosphate levels aren't seen in hyperparathyroidism. It typically presents with low serum phosphate levels.

A 24-year-old delivers twins by cesarean section. The twins are monozygotic. The placenta was fused. What does this implies? A Single ovum and single chorion B Single ovum and double chorion C Double ovum and single chorion D Double ovum and double chorion E Triple chorion

B Identical twins indicate single ovum. If the placenta is fused or double it means there are two chorions, and that the trophoblast differentiation occurred before day 3.

A 45-year-old female presents with a sudden onset of vertigo, nausea, and vomiting. Upon physical exam, you note that she is holding on to the rails of the bed, and her pain gets worse when you attempt any movement of her head. Neurologic exam is grossly normal. Which combination of the following medications is indicated to treat the patient's symptoms? A Hydrochlorothiazide, lorezapam, and gentamycin B Lorezapam, prochlorperazine, and diphenhydramine C Scopolamine, aspirin, and cisplatin D Metoclopromide, hydrochlorothiazide, and cyclobenzaprine E Diazepam, hydrocodone, and hydrochlorothiazide

B Lorezapam, prochlorperazine, and diphenhydramine All other combinations include an ototoxic medication: furosemide, gentamycin, aspirin, and cisplatin. Treatment of acute vertigo is more effective using a combination of vestibular suppressants (benzodiazepines), anti-emetics (prochlorperazine), and anticholinergics (diphenhydramine or scopolamine).

A 29-year-old woman presents in July to your office with symptoms of palpitations, sore neck, and excessive sweating, despite using her air conditioner all the time. No surgical or trauma history is noted. She is currently not taking any medications. Vitals include the following: BP = 124/68, pulse = 110 beats per minute, respirations = 18 per minute, and temperature = 101 o F orally. Upon exam, her thyroid is mildly enlarged without nodules, and severely tender. No local erythema or heat is noted. Which of the following lab results would you expect in this patient? A Serum total T4 level = 5.0 ug/dL B Serum TSH level = 0.25 uIU/mL C Sedimentation rate = 15 mm/hr D Free thyroxine index = 8.0 E Positive thyroid stimulating antibodies

B Serum TSH level = 0.25 uIU/mL The correct choice is B, Serum TSH level=0.25 uIU/mL. The reference range for TSH is 0.34 to 4.25 uIU/mL, and therefore the level in this patient is low. This patient is presenting with signs and symptoms of hyperthyroidism, most likely due to subacute thyroiditis. The leaking of thyroid hormone into the circulation causes anterior pituitary suppression and reduced TSH secretion. Choice A is seen in patients with hypothyroidism. Choice C is within the reference range for woman. Since subacute thyroiditis is an acute inflammatory disorder, patients with this disorder will commonly present with an elevated sedimentation rate. Choice D corresponds to a euthyroid situation. It is an estimate of the free thyroid hormone level in the plasma. This result is within the reference range. Choice E is not seen in subacute thyroiditis. They are commonly found in patients with Graves' disease.

A 24-year-old delivers twins by cesarean section. The twins are monozygotic. The placenta was fused. What does this implies? A Single ovum and single chorion B Single ovum and double chorion C Double ovum and single chorion D Double ovum and double chorion E Triple chorion

B Single ovum and double chorion B Identical twins indicate single ovum. If the placenta is fused or double it means there are two chorions, and that the trophoblast differentiation occurred before day 3.

Which bone is the most susceptible and most often fractured at birth? A Calcaneus B Clavicle C Femur D Humerus E Patella

B The clavicle is the most common bone broken during childbirth. It often is associated with shoulder dystocia, but clavicular fractures can occur in uncomplicated pregnancies. They are usually of the greenstick variety and heal without complications. Calcaneal and patellar fractures are highly unlikely to occur since they are not long bones which are much more vulnerable to fracture. Fractures of the humerus and femur are possible during childbirth, but generally only in traumatic births. Humerus and femur fractures are much less common than clavicular fractures

A 24-year-old male has an eight-month history of loose thought associations, social withdrawal, auditory hallucinations, and deterioration in his personal appearance and hygiene. Upon examination, he is noted to have a flat affect and perceptual distortions, and he behaves like he is detached from his own actions. He is started on a neuroleptic medication, and a few weeks later he is noted to pace frequently and seems to be unable to sit or stand still. What is the extrapyramidal symptom this patient is exhibiting called? A Acute dystonia B Akathisia C Drug-induced parkinsonism D Tardive dyskinesia E Verbigeration

B akathisia The symptom this patient is exhibiting is akathisia, the most common extrapyramidal symptom of the neuroleptic medications. Acute dystonias from neuroleptic medications consist of bizarre muscle spasms of the head, neck, and tongue. Drug-induced parkinsonism consists of the same symptoms as idiopathic parkinsonism, including signs of reduced facial and arm movements, festinating gait, rigidity, and pill-rolling tremor. Tardive dyskinesia usually appears months or years after starting neuroleptic medication, and consists of involuntary stereotyped movements of the face, mouth, tongue, trunk, and limbs. Verbigeration is a symptom of schizophrenia and other psychotic disorders that consists of repetition of senseless words or phrases, but is not a side effect of neuroleptic medication.

A 30-year-old man presents to the office for follow-up on an endoscopically diagnosed gastric ulcer. At endoscopy, he was found to have a Helicobacter pylori infection and now he has completed appropriate therapy. He has another refill available on the proton-pump inhibitor. He is currently asymptomatic. What is the most appropriate follow-up on the infection? A Because he is asymptomatic, no further testing is required. B Check urea breath test or fecal antigen today. C Repeat endoscopy with histologic testing for H. pylori. D Check H. pylori serology today. E Collect stool specimen for culture.

B check urea breath test or fecal antigen today Helicobacter pylori is a spiral, Gram-negative rod that resides in the gastric mucosa, where it causes PUD. It may be diagnosed by rapid urease test or by histology when endoscopy is performed. Noninvasive H pylori testing options include the urease breath test, fecal antigen testing, and serology. Serological and fecal antigen tests are the most cost-effective methods. All three noninvasive tests have sensitivities and specificities greater than 90%. Proton-pump inhibitor therapy should be discontinued 1 to 2 weeks prior to the fecal antigen or breath tests because PPIs may increase the number of false negatives. In this case, serology is the least invasive, most cost-effective, and least likely to be invalidated by the proton-pump inhibitor therapy.

A 24-year-old man presenting to the clinic 1 week ago was diagnosed with depression and subsequently prescribed 10 mg/day of fluoxetine. He unexpectedly shows up today and states that he is not experiencing any improvement since starting the medication. What is the best treatment option at this time? A double the dose of fluoxetine to 20 mg/day B maintain the current dose of fluoxetine and comfort the patient that the medication may still take at least 1 to 2 more weeks to work C discontinue the fluoxetine and start sertraline D discontinue the fluoxetine and start amitriptyline E maintain the current dose of fluoxetine and add phenelzine to the medication regimen

B maintain the current dose of fluoxetine and comfort the patient that the medication may still take at least 1 to 2 more weeks to work B Alleviation of symptoms associated with depression is typically slow in onset following initiation with SSRIs. Fluoxetine, for instance, can take anywhere between 2 to 6 weeks to achieve substantial benefit when used for depression. After just 1 week of therapy, there is little justification to increase the current dose or switch to another SSRI such as sertraline. Switching the patient to a TCA such as amitriptyline at this point would further delay symptom relief, as TCAs can take several weeks to produce improvement. Compared to SSRIs, TCAs are also more likely to create unwanted side effects such as weight gain, orthostatic hypotension, and constipation. Combining an SSRI with a monoamine oxidase inhibitor (MAOI) such as phenelzine can cause serotonin syndrome that can be lethal. In order to avoid interaction between SSRIs and MAOIs, it is recommended that at least 4 to 5 weeks pass after discontinuing one and starting the other.

A 16-year-old girl presents to the clinic complaining of strong desires to sleep at inappropriate times. She is very concerned because she "felt paralyzed" while falling asleep on the couch last night. Which of the following is the best diagnostic test to confirm this patient's diagnosis? A CT of the head B multiple sleep latency test C Tensilon test D thyroid stimulating hormone E polysomnography

B multiple sleep latency test Narcolepsy is characterized by hypersomnolence, loss of muscle tone prior to sleep, hallucinations upon initiating or arising from sleep, and episodes of sleep paralysis. The diagnostic test that is used in conjunction with clinical history to establish the diagnosis is the multiple sleep latency test. The Tensilon test is utilized to assess for the presence of myasthenia gravis. Polysomnography can be useful in excluding other sleep disorders, but it does not assess sleep latency time necessary to support the diagnosis of narcolepsy.

The patient is a 68-year-old male retired steel worker and a past smoker with a 36-pack-year history. He has been under the care of a urologist for about 5 years, ever since an episode of gross hematuria led to the diagnosis of bladder cancer. He is s/p transurethral resection of bladder tumor (TURBT) on 2 occasions now for noninvasive carcinoma in situ. He still follows with surveillance cystoscopy every 3 months, and a recent appointment revealed another bladder tumor. Another TURBT is scheduled, but he will also need adjunctive treatment. Question What medication can be administered intravesically to treat the bladder cancer in this patient? Answer Choices 1 Bacillus Calmette-Guérin 2 Ciprofloxacin 3 Potassium chloride 4 Sulfamethoxazole-trimethoprim 5 Pentosan polysulfate sodium

Bacillus Calmette-Guérin Explanation The correct answer is bacillus Calmette-Guérin (BCG), as this is a common intravesical treatment for patients who have either recurrent bladder cancer or multiple tumors at one time. BCG is administered to patients with bladder cancer on an outpatient basis, typically weekly for 6 weeks, but can also be given as maintenance monthly or every 6 months after initial treatment at the urologist's discretion. After treatment, bladder surveillance is extremely important, as bladder cancer recurrence rates can be as high as 50% over 5 years. Bladder cancer surveillance consists of outpatient cystoscopy and urinary cytology every 3 months for 18-24 months, every 6 months for 2 years, and then annually. Ciprofloxacin is not the correct answer, as this is an antibiotic that is typically used to treat urinary tract, upper respiratory, and skin infections. Ciprofloxacin is available in oral and intravenous forms, but not intravesical. Potassium chloride (KCL) is also not the correct answer. While KCL could, in fact, be administered intravesically, it is not used to treat bladder cancer. It is sometimes administered intravesically as a "potassium sensitivity test" to help with the diagnosis of interstitial cystitis. 40ml KCL solution is administered through a catheter, and then the patient's pain and urgency are rated compared to administration of plain water. Sulfamethoxazole-trimethoprim is also not the correct answer, as this is an antibiotic that is typically used to treat urinary tract, upper respiratory, and skin infections. Sulfamethoxazole-trimethoprim is available in oral and intravenous forms, but not intravesical. Pentosan polysulfate sodium is also not the correct answer. This medication is available in an oral form and can also be administered intravesically as a bladder instillation. It is used in both forms for the treatment of interstitial cystitis, not bladder cancer.

4 hours after eating fried rice at a restaurant, a 30-year-old woman, her husband, and his sister developed nausea, vomiting, and diarrhea. What organism is likely to be involved in the patients' condition? Answer Choices 1 Salmonella enteritidis 2 Bacillus cereus 3 Clostridium perfringens 4 Staphylococcus aureus 5 Vibrio parahaemolyticus

Bacillus cereus Bacillus cereus cause 2 syndromes: (1) one involves a short incubation period with nausea and vomiting and is similar to staphylococcal food poisoning; (2) the other involves a long incubation period (18 hours) with watery, non-bloody diarrhea and resembles clostridial gastroenteritis. Clostridium perfringens cause food poisoning with an 8- to 16-hour incubation period. It is characterized by watery diarrhea with cramps and little vomiting. It resolves in 24 hours. Staphylococcus aureus cause food poisoning characterized by vomiting being more prominent than diarrhea. The disease is due to ingestion of enterotoxin, which is performed in foods and hence has a short incubation period (1-8 hours). Vibrio parahaemolyticus are a marine organism transmitted by contaminated seafood, especially when raw fish is eaten. The clinical picture varies from mild to quite severe watery diarrhea, nausea and vomiting, abdominal cramps, and fever. The illness is self-limited, lasting about 3 days. Salmonella enteritidis cause enterocolitis after an incubation period of 6-48 hours. The disease begins with nausea and vomiting and then progresses to abdominal pain and diarrhea, which can vary from mild to severe and present with or without blood. Usually, the disease lasts a few days and is self-limited.

A 52-year-old woman with a 5-year history of heartburn presents with difficulty in swallowing food. She is able to drink fluids without any problem, but solids go down with difficulty. She denies any weight loss or weight gain. She has had dyspepsia-like symptoms for a long time, with belching and abdominal fullness after meals. Her medications include the intermittent use of over-the-counter ranitidine and acetaminophen for mild aches and pains. Her vitals are stable, and the physical examination is unremarkable. What is the next step in the evaluation of this patient? Answer Choices 1 Empiric treatment with a course of proton pump inhibitor 2 Endoscopy 3 Barium swallow 4 Swallow evaluation and video fluoroscopy 5 Esophageal manometry

Barium Swallow This patient has had a long standing gastrointestinal reflux disease (GERD), which is obvious based on her symptoms. She has not used H2 blockers consistently as she should have. They would have likely reduced or eliminated her symptoms. Now, due to repeated acid-induced injury to the esophageal lining, she has developed an esophageal stricture; this caused dysphagia, which usually starts with solids and progresses to both solids and liquids. A barium swallow is indicated because it is the least invasive and will give information regarding the location, length, number of strictures, and size of the lumen. It is has a sensitivity of 100% for luminal diameter less than 9 mm and above 90% for luminal diameter greater than 10 mm. It will also impact the next step in the treatment, which includes endoscopic dilatation of the stricture, in the technique of dilatation and number of sessions needed; in addition, it impacts counseling the patient on the risks of the procedure. Dysphagia due to other pathologies, like esophageal diverticulum, extra-luminal pathology, malignancy, and perforation, can be ruled out too. Empiric treatment with proton pump inhibitors is indicated in simple GERD without symptoms of stricture. Once the patient has dysphagia, it is important to assess them for complications, like stricture, especially if they have had long standing GERD. In the majority of patients, endoscopy is the most useful test for the work-up of dysphagia. Endoscopy can help identify structural abnormalities such as esophageal webs and strictures, as well as identify tumors and esophagitis. During endoscopy, biopsies may be performed to help confirm diagnosis and direct treatment. Therapeutic interventions may also be undertaken, such as esophageal dilatations for the treatment of strictures. In certain patients, endoscopy is not the preferred first-line test in the management of dysphagia. These include patients with a history of esophageal or laryngeal cancer and those who have been exposed to a caustic ingestion or radiation therapy. In these patients, blind insertion of the endoscope may cause esophageal damage. Therefore, barium swallow is the preferred diagnostic exam. Swallow evaluation and video fluoroscopy permit evaluation of the cervical esophagus only. The rest of the esophagus cannot be evaluated; therefore, this option is inadequate. These tests are more important for pathology suspected in the pharynx, larynx, and upper esophageal sphincter. Esophageal manometry is indicated in motility disorders of the esophagus. Motility disorders present with dysphagia to both liquids and solids, as in achalasia, esophageal spasms, and scleroderma. Manometry is indicated when a barium swallow has ruled out the more common causes of dysphagia.

A 3-week-old male infant is brought in by his mother due to his vomiting. The mother notes that a few days ago her son started vomiting after feeding, and it has become projectile in nature. The vomitus is non-bilious and contains no blood. The child seems hungry and nurses regularly, but the vomiting has become more frequent and is occurring with every feeding now. On physical examination, an oval mass is palpated in the right upper quadrant. What imaging study is the best initial test to obtain in this patient? A Abdominal plain film B Barium enema C Barium upper GI series D CT scan of the abdomen E Upper GI endoscopy

Barium Upper Gi series The suspected diagnosis is pyloric stenosis. An upper GI series with barium is the best test to obtain in this instance. A barium enema is a good choice in suspected intussusception cases. An upper GI endoscopy is best used when a gastric or duodenal ulcer is suspected. An abdominal plain film or CT could be ordered, but aren't the best initial tests to obtain in this patient's presentation.

A 65-year-old man presents because this morning a morsel of meat he had eaten 3 days ago reappeared on his pillow. About a year ago, he noticed difficulty swallowing, particularly solid foods, which seems to be worsening. His wife complains about his bad breath, and he noticed that people are avoiding being close to him. He does not drink, does not smoke, and was in a good health before. His physical examination is within normal range for his age, except that you notice that he is repeatedly clearing his throat, as if he is embarrassed. Question What is the next best step in the management of this patient? Answer Choices 1 Barium study 2 Upper endoscopy and tissue biopsy 3 Endoscopic ultrasound 4 24 hours pH monitor 5 Manometry

Barium study The recollection of slow progressing difficulties in initiating swallowing and regurgitation in an older patient makes you suspect Zenker diverticulum. Your clinical diagnosis is supported by bad breath and throat clearing. Definitive diagnosis is made by barium studies. Endoscopy is actually contraindicated in this patient because the lesion is proximal and there is a risk of perforation of the pharynx. Endoscopic ultrasound is performed in patients with esophageal carcinoma for staging. Your patient has no typical signs of esophageal carcinoma (progressive dysphagia for solid foods first, followed by liquids, with weight loss in a smoker and/or alcoholic). That diagnosis is not excluded; however, the probability of Zenker diverticulum will prevent you from choosing endoscopy as the first step in management. 24 hours pH monitoring should be considered in a patient with symptoms of gastroesophageal reflux (GERD) when the diagnosis is not clear. GERD will present with epigastric or substernal pain, sore throat, metallic taste in mouth, hoarseness, cough, and wheezing. It is not likely that your patient has GERD. Manometry should be considered if you suspect a disorder of esophageal peristalsis, as is the case in achalasia, esophageal spasm, nutcracker esophagus, or lower esophageal sphincter diseases. These disorders can present with dysphagia for both solid foods and liquids, sometimes accompanied by pain and weight loss. Manometry includes the placement of a nasogastric tube, and the procedure will increase the risk of pharyngeal perforation. Manometry should be performed only after barium studies exclude Zenker diverticulum.

A 60-year-old man presents with a 6-month history of dysphagia to solids, regurgitation of undigested food, and halitosis. He denies a decrease in appetite, abdominal pain, weight loss, or change in bowel habits. His past medical history is significant for a total hip replacement. His lab work is as follows: Hemoglobin 14.0 g/dL Hematocrit 43% White cell count 6.0 x 109/L Platelets 300 x 109/L Sodium 135 mEq/L Potassium 4.0 mEq/L Urea 6 mg/dL Creatinine 0.6 mg/dL Glucose 98 mg/dL Chloride 100 mEq/L HCO3- 25 mEq/L His liver panel is normal. Question The diagnosis can be determined initially by what technique? Answer Choices 1 Barium swallow 2 Esophageal manometry 3 MRI of the head and neck 4 Radionuclide scanning 5 Upper endoscopy

Barium swallow The patient may have a Zenker's diverticulum, which could predispose the patient to an aspiration pneumonia. Zenker's diverticulum is an outpouching of the posterior pharyngeal wall immediately above the upper esophageal sphincter. Collection of food particles in the pouch results in halitosis and the symptom of regurgitation of undigested food particles. The diagnosis is likely to be Zenker's diverticulum because the patient has dysphagia to solids, regurgitation of undigested food particles, and halitosis. Barium swallow is the diagnostic procedure of choice because it is the least invasive. Esophageal manometry would not be useful in this case; it is normally used in evaluating patients with noncardiac chest pain, motor disorders of the esophagus, achalasia, or gastroesophageal reflux disease. MRI of the head and neck is not routinely used for the diagnosis of Zenker's diverticulum. Radionuclide scanning would be used in evaluating the patient for a thyroid tumor, which is low on the differential list for this patient based on his symptoms (e.g., lack of weight loss or appetite loss). As part of the work up before surgery (to reduce the size of the diverticulum), an upper endoscopy is used to rule out an esophageal cancer.

A 27-year-old female presents to the office complaining of a lump on one side of her vagina. It has been present for several weeks, and causes only slight discomfort with intercourse. She has no new sexual partners and no other vaginal or systemic symptoms. On physical exam you note a 1.5-cm area of swelling in the left posterior labia majora. It is firm, well circumscribed, and minimally tender, and no changes are noted in the skin. No vaginal discharge or other lesions are noted. What is the most likely diagnosis? A Vulvar abscess B Bartholin gland duct cyst C Vulvar lipoma D Acrochordon E Lichen planus

Bartholin gland duct cyst B Bartholin obstruction leads to an asymmetrical posterior labia majora or vestibule, which is not typically painful unless it abscesses. Lipomas are less likely, and lichen planus causes pain and itching without a discrete mass.

An 8-year-old male presents to your office with his parents for an evaluation of his bedwetting. He is being teased about his bedwetting on overnight outings and his parents are concerned about his self-esteem. He has never had a prolonged period of dryness at night. He has no wet accidents during the day and has not had difficulties with stool incontinence. His parents report that his mother wet the bed until age twelve. They state that he appears to be a very sound sleeper. He is on no medications and his past medical history is unremarkable. The physical examination is within normal limits. Question Which of the following is considered first line intervention due to proven evidenced success rates? Answer Choices 1 Bed-wetting alarm system 2 Imipramine administration 3 Desmopressin acetate administration 4 Behavioral therapy-repeated wakening at night 5 Fluid restriction

Bed-wetting alarm system Explanation Bedwetting, or enuresis, is a common problem in the pediatric age group. Prevalence at the age of 5 years is 7% in males and 3% in females. At the age of 10 years the prevalence is 3% for males and 2% in females. Enuresis is divided into primary enuresis, where the child has never been dry at night, and secondary enuresis, where the child who has been continent for at least 6 months starts to wet the bed again. Primary nocturnal enuresis is associated with a smaller bladder capacity, abnormal arousal patterns during sleep, and inappropriate or inadequate toilet training. Secondary enuresis is generally precipitated by situations of psychological stress, such as a recent move, marital conflict, or a new sibling in the household. Only rarely does enuresis have an organic component and usually urological procedures are not warranted. General treatment guidelines that patients and especially the parents should be educated on include voiding before retiring, limiting fluids prior to bedtime, and expecting older children to clean their own bedding. More often than not the first line intervention is considered bed-wetting alarms. These have been found in multiple studies to be extremely effective in producing a long-term cure. The bed-wetting alarms are moisture-sensing devices that are placed near the child's genitals, and are activated to trigger an alarm when the child voids in bed. These alarms go off at the initial first few drops of voiding; thus awakening the child to get out of bed and finish voiding in the toilet or hold urine until later. This evokes a conditioned response of waking and inhibiting urination. They are generally recommended in children older than seven years. Therapy is recommend for at least 3 months and used every night. Parents must get up with the child to ensure the child does not just turn off the alarm and go back to sleep. If used appropriately, bed-wetting alarms are successful in 2/3 of the patients that utilize them. Alarm therapy requires a cooperative, motivated child and family. Parental involvement plays an essential role when using alarm devices due to the consistency that is necessary. Treatment must involve education and avoidance of being judgmental and even shaming the child due to most children feeling ashamed; the goal of treatment is to help the child establish their continence and at the same time maintain or gain self-esteem. It is recommended that children use these alarm devices until they experience three weeks of complete dryness. For complete resolution of nocturnal enuresis, the bed-wetting alarm may be needed to be used for up to 15 weeks. Relapse rates are higher when the alarm system is discontinued after shorter dry periods. Studies have shown that compared with other skill-based or pharmacologic treatments, the bed-wetting alarm has a higher success rate (75%) and a lower relapse rate. Desmopressin and imipramine are the primary drugs used in the treatment of nocturnal enuresis but should not be considered first line interventions. Imipramine administration is only slightly less effective than the alarm systems in bringing about dryness, usually within 2 weeks. Long-term results are less promising as the enuresis tends to return while off the medication. Desmopressin acetate typically has an excellent response over the short-term, but again, the enuresis tends to recur when the child is taken off the medication. Fluid restriction and wearing diapers at night may resolve the consequences of the issue but will not create the behavior which will lead to the actual bedwetting from stopping; behavioral modification/training is a main key to treatment.

A 30-year-old female presents with a five day history of a sore throat. She denies cough or nasal congestion. She also denies vomiting or diarrhea. On physical exam, her temperature is 101˚F, the pharynx is red with tonsillar exudates, and she has tender anterior cervical lymphadenopathy. What is your next step? A Perform culture and sensitivity B Perform rapid strep C Treat symptomatically with antipyretics D Begin oral penicillin E Begin oral ciprofloxacin

Begin Oral PCN The centor criteria include fever, tonsillar exudates, tender anterior lymphadenopathy and lack of cough. These signs and symptoms highly suggest group A beta hemolytic strep. Treatment with penicillin would be the most appropriate step, especially if cost is a concern to the patient. Penicillin v potassium, 250 mg three times per day, or 500 mg twice daily for 10 days, is highly effective. Some studies show a five-day regime to be as effective.

A 51-year-old male patient presents to your family practice office complaining of genital discomfort with dysuria. His digital rectal exam reveals an enlarged, tender prostate. His prostate-specific antigen (PSA) returns elevated with a value of 11.1 mg/mL, which you fractionate, and this reveals approximately 75% free PSA. His urinalysis reveals moderate white cells and trace blood. What would be your next step in treating this patient? A Begin him on 6 weeks of doxycycline to treat his prostatitis and when resolved, repeat his PSA level. B Immediately refer him to a urologist for prostate biopsy to rule out prostate cancer. C Immediately refer him to a urologist for cystoscopy to rule out bladder cancer and perform a computed tomography (CT) scan of the abdomen and pelvis in the interim. D Order a stat testicular sonogram to rule out torsion. E Order a CT scan of the abdomen and pelvis.

Begin him on 6 weeks of doxycycline to treat his prostatitis and when resolved, repeat his PSA level. This patient has signs and symptoms consistent with prostatitis. Additionally, while his PSA is elevated, this is common in prostatits as well as prostate cancer, and his free PSA is of a percentage that prostate cancer is unlikely. However, it would be prudent to recheck his PSA after treatment and resolution of his symptoms to confirm that an underlying cancer is not smoldering.

A 61-year-old man with known cirrhosis presents with a 1-week history of "puffy" ankles and increased shortness of breath. A week prior to symptom onset, he had traveled on vacation, where he engaged in walking, sightseeing, and eating out. He admits more dyspnea with lying down and with increased exertion. His shoes feel snug, and he notes a definite line from wearing socks. The patient denies chest pain, leg pain, fevers, claudication, nausea, headache, lethargy, and hemoptysis. His past medical history is remarkable for cirrhosis and a history of alcoholism. He is awaiting a liver transplant. He had a liver biopsy, but no other surgeries. He takes no medications, has no allergies, and has been abstinent of alcohol for 9 months. He lives with his wife, works as an electrician, and smokes cigarettes at 1 pack-per-day. Vitals are normal, including oxygen saturation. On physical exam, the patient appears in no acute distress and with normal mental status. His physical exam is remarkable for mild jugular venous distention, 2-3+ edema in lower extremities, and mild dullness to lung percussion. No hepatomegaly or ascites is noted. This patient's laboratory results are shown in the chart. Complete blood count (CBC)- Normal Chest x-ray- Mild bilateral effusions Urinalysis- Normal Beta-natriuretic peptide (BNP)- Normal Complete Metabolic Panel (CMP)- Abnormal as indicated below: Test Result Normal range Units Sodium 126 134-144 mmol/L Potassium 3.4 3.4-4.9 mmol/L Chloride 100 100-109 mmol/L HCO3 26 20-31 mmol/L Glucose 98 70-99 mg/dL Bun 18 7-18 mg/dL Creatinine 1.1 0.6-1.2 mg/dL Calcium 8.8 8.8-10.5 mg/dL Albumin 2.6 3.5-5.0 g/dL Total Protein 5.9 6.4-8.2 g/dL AST (SGOT) 112 15-37 U/L ALT (SGPT) 108 5-43 U/L Alk Phosphatase 158 50-136 U/L Total Bilirubin 1.9 0.1-1.2 mg/dL eGFR >60 >60 mL/min/1.73m2 Osmolality 265 285-293 Mmol/kg H2O Question What is the most appropriate intervention for this patient's fluid and sodium status? Answer Choices 1 Administer intravenous hypertonic saline solution 2 Advise the patient to contact his gastroenterologist for an appointment 3 Begin intravenous or oral diuresis 4 Evaluate for possible pulmonary embolus 5 Initiate hemodialysis

Begin intravenous or oral diuresis This patient is in a hypervolemic hypotonic hyponatremic state. The hypervolemia can be identified primarily by history (edema and dyspnea) and physical exam (jugular venous distention, edema, and pulmonary effusion). The hypotonia (low serum osmolality) and hyponatremia most likely are due to a "dilution" effect, in which this patient's overall fluid status is high, but the sodium levels are normal or even elevated. This patient has hypoalbuminemia (common with chronic liver disease) and increased extracellular fluid, which is causing his mild pulmonary effusions and dyspnea. The most appropriate treatment for this patient is to begin intravenous or oral diuresis. Diuretics should be used with caution in patients with cirrhosis, but his pulmonary edema must be addressed. It would be potentially harmful to this patient to administer intravenous hypertonic saline solution, especially as his overall sodium level is likely normal to high. Hypertonic saline should only be used in acutely ill patients, with careful monitoring. Without prior records, it is difficult to assess any potential worsening in this patient's chronic cirrhosis. While it would be acceptable to advise the patient to contact his gastroenterologist for an appointment in addition to other interventions, it would not address his current hypervolemic hyponatremia or his pulmonary edema. With a complaint of shortness of breath, especially in a smoker, it could be rationalized to evaluate for a possible pulmonary embolus. However, he does not meet the Wells criteria for a PE (no suspected DVT, no tachycardia, no immobilization, no history of DVT/PE or malignancy, no hemoptysis, and alternative diagnoses are more likely than PE). Further evaluation for PE is not indicated. With severe electrolyte abnormalities, or those that do not respond to diuresis, it would be reasonable to initiate hemodialysis. This patient's abnormalities are not severe, and he is mildly symptomatic. A trial of diuresis is indicated before considering hemodialysis.

A 58-year-old man with a past medical history of diabetes mellitus type II and hypothyroidism presents for a follow up of blood work results that were performed 1 week prior. His low density lipoprotein level was found to be 150 mg/dL; total cholesterol level was 230 mg/dL; high-density lipoprotein was 36 mg/dL; and triglyceride levels were 260 mg/dL. He denies any complaints at this time and is compliant to the only medications he takes, which are Levothyroxine 137 mcg/day and Metformin 500mg twice a day. His labwork demonstrated a euthyroid state and a hemoglobin A1c of 6.1%. Question What is an appropriate step in the management of this patient? Answer Choices 1 Recommend a 300 mg/day cholesterol diet 2 Prescribe aspirin 325 mg every day 3 Begin rosuvastatin 10 mg every day 4 Restrict fiber and fruit intake 5 Prescribe cholestyramine resin daily

Begin rosuvastatin 10 mg every day Explanation This patient should be prescribed a statin. The statin (HMG-CoA reductase inhibitor) class of drugs has revolutionized the treatment of hypercholesterolemia, as they are highly efficacious and very well tolerated. The LDL-C goal for patients with coronary heart disease (CHD) equivalent risk, including patients with diabetes mellitus, should also be less than 100 mg/dL. In patients considered to be very high risk, a goal of less than 70 mg/dL is an acceptable option. Patients with CHD or CHD equivalent are prescribed drug therapy simultaneously with therapeutic lifestyle changes if their LDL-C concentration is greater than or equal to 130 mg/dL. Drug therapy is optional for patients whose LDL-C value is 100-129mg/dL. For rosuvastatin at 5-40mg/d, the LDL-C level is lowered by 45 - 63%. Dietary cholesterol should be limited to less than 200mg/day. Other dietary changes, such as increased soluble fiber found in oat bran or psyllium, may reduce LDL cholesterol by 5-10%. Diets rich in antioxidant vitamins found in fruits and vegetables may be helpful in the reduction of oxidation of LDL, thereby reducing atherogenesis. All patients whose risk from CHD is considered high enough to warrant pharmacologic therapy of an elevated LDL cholesterol should be given aspirin prophylaxis at a dose of 81mg/day unless contraindicated. Cholestyramine is a bile acid sequestrants used primarily as additional therapy in patients with familial hypercholesterolemia who experience inadequate LDL-C lowering with statins.

A 78-year-old Caucasian woman is admitted with a provisional diagnosis of diverticulitis, with complaints of acute abdominal pain and lack of eating or drinking well for several days. Her past medical history is significant for diabetes mellitus, diagnosed 12 years ago, and hypertension (though she was hypotensive at admission). Both conditions were reported to have been under good control. Her medications include regular and long-acting insulin and hydrochlorothiazide/lisinopril 25/20 mg QD. Her son had recently been giving her 800mg ibuprofen BID-TID for her abdominal pain during the last week. She has no known allergies. While in the hospital, the patient's laboratory results are followed daily. 2 days after the CT with contrast, it is noted that her serum creatinine has risen to a level of 3.5 mg/dL. Records from 1 month ago at her family physician showed her labs to include a hemoglobin A1C of 6.8%, creatinine of 1.8 mg/dL, and her blood pressure was 107/68 mm Hg. Question What measure would have been the best prevention for this patient's sudden decline in renal function? Answer Choices 1 Begin volume expansion prior to the contrast study 2 Change her insulin to metformin prior to the contrast study 3 Initiate a loop diuretic prior to the contrast study 4 Start IV mannitol immediately after the contrast study and continue for 72 hours 5 Use high-osmolar iodinated radiocontrast material during the contrast study

Begin volume expansion prior to the contrast study Explanation Mild volume expansion prior to the CT with IV contrast is recommended to prevent contrast nephropathy in higher-risk patients.1 Hydration should begin an hour or more before the procedure and continue several hours after the CT. There is evidence showing better results with NaHCO3 over regular saline (NaCl).2 This patient had several indicators of volume depletion - she was taking a diuretic and had decreased oral intake, along with potentially some fluid/blood loss with the diverticulitis, and her BP was hypotensive. A change to metformin is not recommended for this patient. Metformin, while not nephrotoxic directly, can increase the likelihood of lactic acidosis.3 She likely has some pre-existing renal disease (diabetic nephropathy) and metformin is relatively contraindicated in this patient. Some experts recommend cessation of metformin when patients undergo contrast studies in order to decrease likelihood of contrast nephropathy. Loop diuretics are also not recommended. These drugs can increase risk of contrast nephropathy and should be discontinued, preferably a few days prior to any contrast studies.4 Though mannitol has a role in treatment and prevention of oliguria, it is an osmotic diuretic and can increase risk of renal damage after exposure to contrast.3,5 Traditional high-osmolar iodinated contrast material carries a higher risk of contrast nephropathy than the newer low osmolar contrast materials. Risk is also dose-dependent, so a single study with a shorter exposure to a lower dose has less chance of inducing contrast nephropathy compared to repeat/multiple studies, with longer administration of high-dose.2,3,4

What is a commonly shared feature of bipolar disorder and ADHD in pediatric patients? A disruptive B obsessed with ideas C behavior problems D impaired concentration E insomnia

Behavior problems C Behavior problems are a commonly shared feature of pediatric ADHD and bipolar disease. Disruptive and impulsive behaviors are a shared feature of conduct disorder. Disruptive behavior and being annoying to others can be found with oppositional defiant disorder; impaired attention and concentration can be found in major depression. Bipolar patients are obsessed with ideas while ADHD and conduct disorder patients are not.

A 37-year-old woman presents to her GP surgery with a history of right-sided facial weakness and peri-auricular discomfort since she awoke this morning. She is afebrile. Question What is the most likely diagnosis? Answer Choices 1 Trigeminal neuralgia 2 Bell's Palsy 3 Multiple sclerosis 4 Myasthenia gravis 5 Primary lateral sclerosis

Bell's Palsy Explanation The correct answer is Bell's palsy, which is a condition typically with sudden onset that affects the facial nerve, causing unilateral facial weakness. Trigeminal neuralgia presents with sharp pain on one side of the mouth that radiates to the ipsilateral ear, eye, or nostril. Multiple sclerosis is a demyelinating disorder, causing a multitude of symptoms that typically include diplopia or blurred vision early on, then an insidious onset of progressive weakness, numbness, and/or tingling in the extremities. Myasthenia gravis commonly presents with ptosis and diplopia, as well as difficulty swallowing, fatigue, and muscle weakness. Primary lateral sclerosis is an upper motor neuron disease that causes limb weakness, stiffness, and fasciculations.

A 37-year-old woman presents with a history of right-sided facial weakness and periauricular discomfort since she awoke this morning. She is afebrile. Question What is the most likely diagnosis? Answer Choices 1 Trigeminal neuralgia 2 Bell's Palsy 3 Multiple sclerosis 4 Myasthenia gravis 5 Primary lateral sclerosis

Bell's Palsy The correct answer is Bell's palsy, which is a condition with typically sudden onset that affects the facial nerve, causing unilateral facial weakness. Trigeminal neuralgia presents with sharp pain on 1 side of the mouth that radiates to the ipsilateral ear, eye, or nostril. Multiple sclerosis is a demyelinating disorder; it causes a multitude of symptoms that typically includes diplopia or blurred vision early on, followed by an insidious onset of progressive weakness, numbness, and/or tingling in the extremities. Myasthenia gravis commonly presents with ptosis and diplopia, as well as difficulty swallowing, fatigue, and muscle weakness. Primary lateral sclerosis is an upper motor neuron disease that causes limb weakness, stiffness, and fasciculations.

A 58-year-old male comes to your primary care office complaining of sadness, insomnia, loss of appetite, weight loss, and feelings of guilt or hopelessness for the past month. On further questioning you find that his wife of 30 years died of breast cancer shortly before the onset of symptoms. Which of the following is the most likely diagnosis? A Bereavement B Dysthymia C Depressive episode D Depressed mood E Major depression

Bereavement A Bereavement (A) is the normal grief response experienced after a significant loss and includes all the symptoms of depression and, by definition, lasts no longer than two months, although many will suffer from some symptoms for longer. Dysthymia (B) is a longstanding depressed mood for at least two years but not meeting the full criteria for a depressive episode (C). Major depression (E) requires at least one depressive episode, which requires at least five of the symptoms, one of which is depressed mood (D).

A 4-day-old male infant is seen for a routine newborn check; he has yellow skin from his face down to his chest. He is a full-term infant; vaginal delivery was normal. He weighed 3.180 kg at birth, and 3.100 kg at discharge. He is breastfeeding well; he is also stooling and voiding regularly. He is well-hydrated and active. Laboratory results reveal a direct bilirubin of 3.4 mg/dL, and an indirect bilirubin of 12.3 mg/dL. Question What is a possible pathologic cause of the jaundice? Answer Choices 1 Breast milk 2 Birth trauma 3 Breastfeeding 4 TPN cholestasis 5 Biliary atresia

Biliary atresia The correct response is biliary atresia. Breastfeeding contributing to dehydration and intensified physiologic jaundice, breast milk jaundice, and birth trauma causing cephalohematoma with increased hemolysis are all possible causes of indirect hyperbilirubinemia. Since the infant is 4 days old, his indirect bilirubin level is not so high as to require treatment or concern at this time. Any direct bilirubin >2.0 mg/dL is abnormal. If it is persistent or increasing it is considered pathologic; it must be evaluated immediately. TPN cholestasis and biliary atresia can both contribute to direct hyperbilirubinemia. TPN cholestasis can occur when an infant has received hyperalimentation for an extended period of time, so it would not occur in a 4-day-old infant. Cholestasis is impaired excretion of the conjugated bilirubin in bile. The most common hepatic cause of jaundice of direct hyperbilirubinemia in infancy is biliary atresia. Ultrasound should be performed to identify blockages or anatomical anomalies.

What is the most common visual field defect occurring in patients with pituitary macroadenoma? Answer Choices 1 Heteronymous hemianopia 2 Bitemporal hemianopia 3 Quadrantanopia 4 Homonymous hemianopia 5 Homonymous hemianopia with macular sparing

Bitemporal Hemianopia Explanation Pituitary macroadenoma compresses the optic chiasma to produce visual symptoms, the most common being bitemporal hemianopia. Local complications of Pituitary tumors: Upward extension: Chiasmal compression and visual field defect Compression of pituitary stalk Lateral extension: Dysfunction of cranial nerves 3,4,6 (diplopia) Stretch of dura mater: Headache Acute infarction / hemorrhage: Pituitary apoplexy Investigations reveal: Increased levels of growth hormone Increased levels of prolactin Hyperglycemia Paradoxical rise of GH levels during an oral glucose tolerance test. Pituitary macroadenomas that are best visualized by MRI or CT scan Management options: Trans-sphenoidal surgery (especially for microadenoma) External radiotherapy to stop growth and decrease GH Octreotide (will lower GH levels but will not stop tumor growth)

A 62-year-old woman presents with bright red gross hematuria for the past 2 months. She states it is painless, but persistent. She denies other symptoms. She has no chronic medical problems. Upon further questioning, she admits to a 50-pack year smoking history, and she states she is currently retired from her job in a rubber factory. Vital signs are within normal limits, and physical exam is normal. Urine dipstick only shows too numerous to count RBCs, and urine cultures are negative. Question What is the most likely diagnosis? Answer Choices 1 Acute cystitis 2 Bladder cancer 3 Renal cell carcinoma 4 Urethritis 5 Ureteral calculi

Bladder cancer Explanation Hematuria is the most common presenting sign of urinary tract cancer, and bright red gross hematuria is usually of lower urinary tract origin. Silent or painless hematuria suggests tumor or renal parenchymal disease; therefore, the clinical picture points to cancer, with bladder cancer as the most likely diagnosis. Smoking and exposure to industrial dyes or solvents (like in a rubber plant) are risk factors for bladder cancer. Bladder cancer is the 2nd most common urologic cancer, and the mean age at diagnosis is 65 years. It is more common in men than women (2.7:1), and 98% of primary bladder cancers are epithelial malignancies (majority urothelial cell carcinomas). Ordering cytology of the urine sample is often helpful with higher grade and stage bladder cancers. Patients can become anemic with chronic blood loss, so a CBC is justified. Diagnosis is made by cystoscopy with biopsy. Acute cystitis typically presents with irritative voiding symptoms (frequency, urgency, dysuria) and suprapubic discomfort in addition to possible hematuria. Urinalysis will show pyuria, bacteriuria, and varying degrees of hematuria. Urine cultures will show specific organisms. Renal cell carcinoma can present with flank pain, hematuria, persistent back pain, and an abdominal mass; also, it can be found incidentally on CT scan. RCC is more common in men than women (2:1), and it has a peak incidence in the 6th decade of life. This could be a possible option for diagnosis, but the significant history of smoking and previous work history points more toward bladder cancer. Urethritis is inflammation of the urethra that presents with urethral discharge, dysuria, and itching. Urethritis is most often caused by a STD. This patient does not have a history of unprotected sexual intercourse or any other symptoms that would indicate this diagnosis. Ureteral calculi can present with hematuria, but it also typically presents with flank or abdominal pain as well. If the stone is in the ureter, it often causes some hydroureter with or without hydronephrosis, both of which cause some pain or discomfort. The patient has no past history of forming stones.

Which of the following is true regarding Mallory-Weiss syndrome? Answer Choices 1 Operative repair usually is necessary 2 Is the most common cause of gastrointestinal bleeding during pregnancy 3 Endoscopy is contraindicated 4 Bleeding stops spontaneously in the majority of cases 5 Accounts for 1/3 of deaths from upper gastrointestinal bleeding

Bleeding stops spontaneously in the majority of cases Explanation Mallory-Weiss syndrome involves a tear of the mucosa and submucosa of the lower esophagus or gastric cardia and accounts for up to 9% of cases of significant upper gastrointestinal bleeding. This syndrome results from forceful events that increase intra-abdominal pressure, including trauma, seizures, coughing, and, most commonly, vomiting, with retching preceding hematemesis in approximately 50% of patients. Mallory-Weiss syndrome is associated with alcoholism, hiatal hernias, esophagitis, or gastritis. The majority of episodes involve only mild to moderate bleeding that stops spontaneously, and surgery is rarely required. Only 3% of deaths from upper GI bleeding are due to Mallory-Weiss tears. Endoscopy is the diagnostic procedure of choice if performed within the first 12 to 24 hours. Esophagitis secondary to reflux and repeated vomiting is the most common cause of upper GI bleeding during the pregnancy.

A 38-year-old man presents with fatigue, dry mouth, and passing large amounts of urine. He describes his urine as light in color and non-odorous. He says he never had this before. He has always been healthy, and he has never been hospitalized before. During a routine pre-employment screening, his serum electrolytes were investigated; his sodium level was slightly above normal. At that time, he was reassured that it was due to slight dehydration. He was advised to repeat the test after 3 months. When he repeated it about a week ago, it showed more of an increase. He takes vitamin supplements and sometimes uses energy drinks. To avoid going to the bathroom too often, he tries to restrict fluid intake to a minimum, but he says it has never helped. Question What is the most appropriate next step in the management of this patient? Answer Choices 1 Serum osmolarity 2 Brain MRI 3 Urine osmolarity 4 DDAVP (desmopressin test) 5 Blood glucose level

Blood Glucose Level Explanation Diabetes mellitus (DM) should be excluded first in all patients with polyuria. Therefore, checking blood glucose level is the next best step. Should DM be ruled out, you should then determine the correct etiology of the patient's symptoms. Diabetes insipidus (DI) is also on the differential diagnosis. DI is caused by either a deficiency of arginine vasopressin (central DI) or an insensitivity to its effects at the level of the kidney (nephrogenic DI). Symptoms include polyuria, polydipsia, and decreased urine specific gravity. Causes: 1) Central; due to tumor, anoxia, or hemorrhage. 2) Nephrogenic; due to the effect of central drugs, of which lithium is a well-known cause. Differential diagnoses: 1) Diabetes mellitus. 2) Psychogenic polydipsia Workup: 1) First, exclude DM by random blood sugar; then, psychogenic polydipsia should be excluded by water deprivation test (in this case scenario, the patient did it himself). 2) Second, do a desmopressin test to differentiate nephrogenic from central DI. 3) If it is positive, then perform MRI to exclude brain tumors. Treatment: 1) In mild cases, adequate hydration is enough. 2) For central DI, give desmopressin. 3) In cases of psychogenic polydipsia, psychiatric consultation is needed. If drug induced, stop offending drug. The DDAVP test is performed to differentiate central from nephrogenic DI. Brain MRI can be used to detect pituitary tumors that may cause diabetes insipidus. Urine osmolarity is significantly reduced in diabetes insipidus and may be helpful in determining a diagnosis. Although all of these has a role in diagnosis, they are not the best answer to the question.

A 34-year-old African-American woman presents with recurrent vaginal yeast infections. Over the past 2 years, she has had repeated episodes of similar infections that have been only partially responsive to over-the-counter treatments. She has not seen a physician in the 5 years since her last pregnancy, and she denies a history of any major medical illness. She has been moderately obese for most of her adult life; her maximal weight was 240 lb. at a height of 5'1"; however, she has recently had a 15 lb. unintentional loss in weight. She also reports nocturia for the past several months. Physical examination is remarkable for a blood pressure of 155/95 mm Hg, obesity, and findings consistent with vaginal candidiasis. Question What is the most useful test for underlying disease diagnosing in this patient? Answer Choices 1 A 50 g oral glucose tolerance test 2 A 100 g oral glucose tolerance test 3 Blood glucose after an overnight fast (FBG) 4 Point-of-care (POC) Hemoglobin A1c (HbA1c) level 5 Immediate measurement of blood glucose concentration

Blood glucose after an overnight fast (FBG) Explanation This woman's nocturia and her recurrent vaginal candidiasis, which is a common correlate of hyperglycemia in women, are symptoms of diabetes mellitus. Women who are diabetic are more prone to vaginal yeast infections due to vaginal secretions containing greater amount of glucose than normal. Yeast cells are nourished by this excess glucose. Additionally, acute hyperglycemia can affect all major components of innate immunity, thereby impairing the ability of the host to combat infection. The standard means of diagnosing diabetes is measurement of FBG; the measurement should be repeated if the value is greater than 110 mg/dl. The current criteria considered diagnostic of diabetes is the finding of an FBG level of 126 mg/dl on more than one occasion. An FBG of 110 to 126 mg/dl is indicative of abnormal glucose metabolism, with higher values indicating an increased likelihood of progression to diabetes. Immediate measurement of blood glucose concentration is an incorrect response. A determination of random blood glucose level is diagnostic if that level is greater than 200 mg/dl in a symptomatic patient. Random blood glucose values vary considerably more than do FBG values and thus are not as reliable for establishing a diagnosis. Point-of-Care (POC) hemoglobin A1c (HbA1c) level is an incorrect response. Although in 2009, an international expert committee recommended the A1c test as one of the tests available to help diagnose type 2 diabetes and prediabetes, point-of-care (POC) A1c assays are not sufficiently accurate at this time to use for diagnostic purposes. HbA1c as a diagnostic test should be performed using a method that is certified by the National Glycohemoglobin Standardization Program (NGSP) and standardized. Oral glucose tolerance test is an incorrect response. OGTT are quite sensitive in the detection of diabetes, and the 50 g and 100 g tests have a clear place in the diagnosis of gestational diabetes; however, these tests are more expensive and inconvenient for patients than measurement of FBG, and their results are less reproducible. Because abnormalities in fasting and postprandial glycemia tend to progress in tandem, FBG measurement has supplanted the oral glucose tolerance test for the diagnosis of diabetes (except during pregnancy).

A 20-year-old woman presents to the medical office to start birth control pills. She is engaged to be married soon. She has never been sexually active. She states she feels healthy, and she denies any current complaints. A summary of her past medical history includes: Medications: None Allergies: Sulfa medications Surgical history: None Medical history: No known conditions. OB/GYN history: Menarche age 13. Regular monthly menses, with no menstrual complaints. Family history: Paternal grandfather has diabetes and hypertension. Maternal grandmother had a stroke. Maternal grandfather had prostate cancer. Father has hypertension and history of a heart attack. Social history: Patient works as a retail clerk part-time and is attending college. She currently lives with her parents. She denies the use of tobacco, alcohol, and recreational drugs. Question What component of the physical exam is the most important before prescribing birth control to this patient? Answer Choices 1 Bimanual pelvic exam 2 Blood pressure measurement 3 Breast exam 4 Cardiovascular exam 5 Pap smear with vaginal speculum exam

Blood pressure measurement Explanation It is most useful and important to obtain a blood pressure measurement prior to initiation of combined hormonal contraceptives. Routine screening, including physical exam and tests, is generally not recommended prior to initiating contraceptives. If the patient is found to have severe hypertension, combination hormonal contraceptives should be avoided. Likewise, if blood pressure measurements rise dramatically after using these contraceptives, they should be discontinued and alternate methods initiated. A bimanual pelvic exam has been somewhat controversial, with no compelling evidence to continue the practice in asymptomatic women. In fact, exams viewed as invasive or embarrassing (such as pelvic and breast exams) can be seen as a barrier for preventing unintended pregnancies; some women may avoid encounters to obtain contraceptives in order to avoid the exams. Breast exam is another routinely performed exam; it has limitations in the asymptomatic patient population for screening. It becomes much more useful in screening for breast cancer in older women, who are higher risk for breast cancer. A breast exam is not routinely recommended prior to initiating birth control. A cardiovascular exam is not recommended prior to initiating birth control. Even with some cardiovascular disease in this patient's family history, the cardiovascular exam is likely be normal and would not be useful in initiating contraceptives. The Pap smear (short for Papanicolaou) with vaginal speculum exam is primarily a test for cervical cancer and pre-cancerous dysplasia. Many years ago, it was thought that birth control increased risk of cervical cancer. That myth has been disproved. There is no compelling reason to obtain a Pap smear in order to initiate birth control. Routine cervical cancer screening guidelines should be followed, which would indicate that this patient should wait until the age of 21 years before beginning Pap testing. If the patient noted vaginal complaints or desired testing for sexually transmitted diseases, a speculum exam would be useful.

A 66-year-old man presents with tingling sensations in his hands and feet; his movements are clumsy, and he has difficulty with walking. He notes that he feels generally weak, and that he sometimes has 'the feeling that he can not locate his limbs'. He often feels depressed or irritable. His vitals are normal, but neurological examination reveals decreased reflexes. Question What test should be performed next? Answer Choices 1 Blood tests to measure level of cobalamin 2 Blood tests to measure level of thiamine 3 Blood tests to measure the level of glucose 4 Thyroid-stimulating hormone (TSH) test 5 Neuroimaging studies

Blood tests to measure level of cobalamin Blood tests should be performed to measure the level of cobalamin. Based on the symptoms, the patient likely suffers from subacute combined degeneration of spinal cord due to degeneration of the posterior and lateral columns of the spinal cord. This is a result of a vitamin B12 (cobalamin) deficiency; a blood test to determine the level of vitamin B12 should be performed to confirm the diagnosis. Performing blood tests to measure the level of thiamine is an incorrect response. A deficit in vitamin B1 (thiamine) causes symptoms that are characteristic of patients with Wernicke-Korsakoff syndrome. These include confusion and movement disability; however, the patients with Wernicke-Korsakoff syndrome do not generally have tingling or numbness sensations. Performing blood tests to measure the level of glucose is an incorrect response. Symptoms such as pain, tingling, and numbness in the hands and feet, as well as depression, can be signs of developing diabetic neuropathy. This patient lacks other symptoms associated with diabetes, making this diagnosis unlikely; therefore, testing for glucose levels is not a necessary next step. A thyroid-stimulating hormone (TSH) test is an incorrect response. Muscle weakness, numbness, pain, as well as depression, are associated with the late stages of chronic hypothyroidism; however, these symptoms are not sufficient to suspect hypothyroidism. Hypothyroidism is characterized by weight gain, loss of concentration, and hearing loss in the elderly, which are not conditions that affect this patient; therefore, a TSH test would not be useful in this case. Neuroimaging studies are performed to provide information about brain tumors, skull fractures, and other diseases of the brain and blood vessels in the brain. Based on the symptoms seen in this patient, neuroimaging would not help in establishing a diagnosis.

The patient is a 26-year-old woman who presents to her gynecologist's office with a 4-month history of the chief complaint of amenorrhea. She has had some breast tenderness, but denies nausea, vomiting, fatigue, and abdominal pain. She was sexually active until about 6 weeks ago when she broke up with her boyfriend, but states that they used condoms. Question Which of the following choices would best describe the appearance of her cervix during an internal examination if the patient is 10 weeks pregnant? Answer Choices 1 purulent discharge from cervical os 2 strawberry-like appearance to cervix 3 bluish appearance to cervix 4 cervical os slightly open 5 blood from cervical os

Bluish appearance to the cervix Explanation Bluish appearance to the cervix is the correct answer. While some pregnant women can have a normal appearance to their cervix, most will have a bluish appearance. In addition, bimanual palpation will reveal the cervix to be soft and flexible. The uterus also increases in size with gestational age and will fill the pelvis around 12 weeks. Strawberry-like appearance to the cervix is not the correct answer, as this appearance is more likely to be seen in patients who have some form of cervicitis. In particular, patients with cervicitis caused by trichomonas usually have purulent vaginal discharge and a friable cervix with punctate hemorrhages that give it a "strawberry-like" appearance. Cervical os slightly open is not the correct answer, as it is not the most likely appearance of the cervix, but could actually be seen in the cervix of a pregnant female. However, the os is usually only slightly open if the patient is experiencing a spontaneous abortion, has an incompetent cervix, or is in the early stages of labor. Purulent discharge from the cervical os is not the correct answer. If purulent discharge is seen coming from the cervical os, the likely diagnosis has something to do with an infection. The patient may have cervicitis or pelvic inflammatory disease. Blood from cervical os is an incorrect answer, as it is not typically indicative of a pregnant patient. Blood coming from the cervical os is more likely to be seen in a patient currently having their menses, a patient experiencing a spontaneous abortion, cervical malignancy, and during some stages of labor. If a female is known to be pregnant and has blood coming from the os, there are likely to be other symptoms associated with various disorders such as placental abnormalities.

A 23-year-old male presents to the clinic complaining of left anterior neck pain that developed over the past week following recovery from an acute upper respiratory infection. On physical exam a tender mass is felt anterior to the left sternocleidomastoid muscle from the mandible inferiorly to the level of the cricoid cartilage. Which of the following is the most likely diagnosis? A Branchial cleft cyst B Dermoid cyst C Peritonsillar abcess D Salivary gland tumor E Thryoglossal duct cyst

Branchial cleft cyst The development of a neck mass in a young adult following URI is consistent with branchial cleft cyst (A) and thyroglossal duct cyst (E). The location of this mass away from the midline and anterior to the SCM is most consistent with branchial cleft cyst (A). The location of the mass and history are inconsistent with dermoid cysts (B), which are typically midline, peritonsillar abcesses (C), which would be located in the retropharyngeal space, and salivary gland tumors (D), which would be located in the parotid, submandibular, or submental salivary regions.

An 87-year-old woman presents with progressive shortness of breath. She has been in a wheelchair for 15 years due to paralysis of her lower extremities from unknown causes. At this time, she is unable to transfer from the chair to her wheelchair without having dyspnea. She is extremely tired, but denies chest pain, palpitations, cough, hemoptysis, dysphagia, hoarseness, or sick exposures. She has never smoked. Her past medical history is positive for hypertension (treated with enalapril), heart failure, chronic kidney disease, hepatitis C, breast cancer, s/p lumpectomy, and radiation treatment 10 years ago. You order a chest X-ray. Question What in her medical history would lead you to suspect an exudative pleural effusion? Answer Choices 1 Breast cancer 2 Chronic kidney disease 3 Cirrhosis 4 Heart failure 5 Hypertension

Breast Cancer A pleural effusion results from an abnormal accumulation of fluid in the pleural space. Transudative pleural effusions suggest the absence of localized pleural disease and are caused by increased hydrostatic pressure, as seen in heart failure, or decreased oncotic pressure, as seen in cirrhosis. Because there is no underlying pleural disease process, transudative pleural effusions are often bilateral. Exudative pleural effusions are indicative of an underlying pleural disease process, such as pneumonia or a malignancy. A diagnostic thoracentesis can help distinguish transudative from exudative pleural effusions. The fluid should be sent for a cell count, chemistry, and, if indicated, cytology or culture. A pleural effusion is diagnosed as exudative based on the chemistry when the pleural fluid meets 1 or more of the following criteria: "1) ratio of pleural fluid protein to serum protein greater than 0.5; 2) ratio of pleural fluid LDH to serum LDH greater than 0.6; and 3) pleural fluid LDH greater than 2/3 the upper limit of normal serum LDH." The patient presented in this case was treated for left-sided breast cancer with a lumpectomy and radiation therapy 10 years ago. She underwent a thoracoscopy and pleurodesis. During the procedure, 1500 cc of pleural fluid was removed. Cytology was positive for breast cancer. A patient with chronic kidney disease, heart failure, or cirrhosis is more likely to have bilateral transudative pleural effusions. Hypertension by itself is not a risk factor for a pleural effusion.

A 60-year-old man with a past medical history of COPD, hypertension, peptic ulcer disease, and hyperlipidemia is being evaluated for a 1-hour history of severe pain in the mid-abdominal region. His history includes abdominal surgery 6 months ago for a small intestinal obstruction, and his pain emanates from his incisional site. He also states that he has shaking chills, nausea, and vomiting, but denies chest pain, shortness of breath, wheezing, or abnormal bowel habits. Upon physical exam, he is found to be hypotensive, tachycardic, diaphoretic, and in acute painful distress. The abdomen is obese; bowel sounds are hypoactive; and rebound tenderness is noted. There is an exquisitely tender 2.0 cm ventral hernia located inferior to the umbilicus that is indurated, tender to palpation, and is not reducible. Auscultation of the mass reveals the presence of bowel sounds. Question What is the most appropriate therapeutic intervention for this patient at this time? Answer Choices 1 Broad spectrum antibiotics 2 Maintain a solid diet 3 Apply hot packs to the hernia 4 Oral motrin for pain 5 Attempt hernia reduction

Broad spectrum antibiotics This patient's presentation is significant of a strangulated hernia. Manifestations of hernia strangulation, a medical emergency, include an exquisitely tender mass associated with systemic signs and symptoms, such as intestinal obstruction, toxic appearance, peritonitis, or meeting sepsis criteria. General surgery should be consulted immediately.Additional interventions include the administering broad-spectrum IV antibiotics, such as cefoxitin, providing fluid resuscitation and adequate narcotic analgesia, and obtaining preoperative laboratory studies. If there is any concern for strangulation, do not attempt hernia reduction. The reintroduction of ischemic, necrotic bowel back into the peritoneal cavity can result in subsequent perforation and sepsis. Bedside ED US, using a linear high frequency probe with color or power Doppler of the hernia sac can be useful in these borderline cases to establish the presence or absence of blood flow.

A 40-year-old man presents with a 5-day history of cough and purulent sputum. The patient has had recurrent attacks of cough with sputum production since his childhood. There are no other systemic complaints. The patient is febrile and has grade III finger clubbing. Rales are present all over the chest on auscultation. The chest X-ray shows a characteristic honeycomb appearance. What is the most likely diagnosis? Answer Choices 1 COPD 2 Tuberculosis 3 Bronchiectasis 4 Pneumonia 5 Cystic fibrosis

Bronchiectasis The clinical picture along with clubbing and the honeycomb appearance on chest X-ray is diagnostic of bronchiectasis. Bronchiectasis is a disease caused by irreversible dilatation of the bronchial tree. Pathogenesis can be obstruction, congenital disorder, and infection. It can develop as a result of severe bacterial infections in childhood, often as a complication of whooping cough or measles. It can be cylindrical, varicose, or cystic, and it corresponds to the severity of degree of bronchiectasis. Typically, the patient has recurrent respiratory infections, usually cough with lots of sputum production. There will be clubbing of fingers and cyanosis (depending on the severity), and it presents at a later age compared to cystic fibrosis. Chest X-ray shows a typical honeycomb appearance. Diagnosis can be confirmed by CT scan or bronchography. Treatment is by antibiotics and assisting postural drainage. Cystic fibrosis presents with associated symptoms of malabsorption and infertility. COPD is associated with a history of smoking. It will usually have a normal X-ray, as in bronchitis, or be seen with associated features of emphysema (e.g., bullae and hyper-translucent lung field with loss of peripheral vascular markings). Pneumonia will have an acute history, and recurring infections are not characteristic. X-ray will be diagnostic. Tuberculosis will also be diagnosed on the X-ray. Bronchiectasis may develop secondary to a tuberculous hilar lymph node obstructing a major bronchus.

Your patient is a 26-year-old male in whom you've diagnosed major depression. You wish to start him on pharmacologic therapy, but he expresses significant concern regarding sexual side effects. Which of the following depression medications is an option that will reduce the risk of sexual dysfunction? A Bupropion B Citalopram C Fluoxetine D Paroxetine E Venlafaxine

Bupropion A The correct answer is bupropion (A). All the SSRIs, including citalopram (B), fluoxetine (C), and paroxetine (D), as well as venlafaxine (E), which is a combination serotonin and norepinephrine reuptake inhibitor (SNRI), have high rates of sexual side effects for men and women. Bupropion is a norepinephrine and dopamine reuptake inhibitor and can be helpful in averting or reducing both sexual side effects and weight gain. It is also indicated for smoking cessation.

Which nonnicotine related agent has been approved by the FDA as a first-line medication in the treatment of smoking cessation? A nortriptyline B clonidine C bupropion D fluoxetine

Bupropion C Bupropion SR has been approved by the U.S. Food and Drug Administration (FDA) for smoking cessation. The drug has been successful in doubling cessation rates. Side effects include dry mouth, agitation, insomnia, and headache.

Your patient is a 26-year-old male in whom you've diagnosed major depression. You wish to start him on pharmacologic therapy, but he expresses significant concern regarding sexual side effects. Which of the following depression medications is an option that will reduce the risk of sexual dysfunction? A Bupropion B Citalopram C Fluoxetine D Paroxetine E Venlafaxine

Bupropion The correct answer is bupropion (A). All the SSRIs, including citalopram (B), fluoxetine (C), and paroxetine (D), as well as venlafaxine (E), which is a combination serotonin and norepinephrine reuptake inhibitor (SNRI), have high rates of sexual side effects for men and women. Bupropion is a norepinephrine and dopamine reuptake inhibitor and can be helpful in averting or reducing both sexual side effects and weight gain. It is also indicated for smoking cessation.

A 38-year-old thin, HIV+ male is found to have a distended abdomen and is complaining of a feeling of fullness. A CT scan of the abdomen revealed a large abdominal mass with ascites in the retroperitoneal region. Laboratories found an elevated EBV of 5400 copies (normal=0), LDH 554 (100 to 250), and Ig-positive MYC translocation. What is the most likely diagnosis? A Burkitt lymphoma B Cat scratch disease C Hodgkin's lymphoma D Infectious mononucleosis E Sarcoidosis

Burkitt Lymphoma In immunosuppressed individuals, including those who are HIV+ and in the presence of highly reactive EBV, the development of an abdominal mass with elevated LDH is most likely Burkitt's lymphoma. Of the masses that are found, 30 to 40% will be positive for EBV, and it appears that this virus drives the conversion of cells to a cancerous state. Infectious mononucleosis would be positive for EBV, but does not develop large abdominal masses.

A 22-year-old woman presents with increased vaginal discharge. She is sexually active with 2 male partners, and she uses birth control pill for contraception. Her last menstrual period was 12 days ago, and she has noticed an increased whitish vaginal discharge for the past week. Physical exam reveals a soft, non-tender abdomen. On pelvic exam, she has a light-yellow cervical discharge with erythema of the cervical os. There is no cervical motion tenderness and no adnexal masses or tenderness. Wet mount of the vaginal discharge reveals epithelial cells and WBCs, and no yeast or protozoa. Gram stain of the vaginal discharge reveals many leukocytes. A urine pregnancy test is negative. What is the best treatment for this patient? Answer Choices 1 Azithromycin 1 g once PO, and doxycycline 100 mg twice daily, for 14 days 2 Ciprofloxin 500 mg PO, and ceftriaxone 125 mg IM once 3 By mouth, azithromycin 1 g once, and Ciprofloxin 500 mg once 4 By mouth, azithromycin 1 g PO, and ceftriaxone 250 mg IM once 5 By mouth, metronidazole 2 g once

By mouth, azithromycin 1 g PO, and ceftriaxone 250 mg IM once Explanation This patient has a mucopurulent cervicitis on physical exam, which is usually caused by Neisseria gonorrhoeae or Chlamydia trachomatis. In all practicality, a patient should be treated with antimicrobial agents that will cover both pathogens because there is a high risk of co-infection with both pathogens (at least 50%). Patients should be tested for infection with both chlamydia and gonorrhea before treatment, usually with an immunoassay. In addition, a Gram stain of the cervical discharge can help elucidate the responsible pathogen. In 60% of women with gonorrhea, Gram-negative intracellular diplococci may be identified. In chlamydial infections, the Gram stain will reveal many leukocytes (>10/high powered field) but no gonococci. These patients should also be screened for co-infection with syphilis and possibly HIV. This patient does not have signs of salpingitis or tubo-ovarian abscess (pelvic inflammatory disease). Specifically, she has no cervical motion tenderness or peritoneal signs, and no adnexal tenderness or masses. She will qualify for outpatient treatment of cervicitis and can avoid the more aggressive treatment used for salpingitis. In either condition, both of the patient's sexual partners will require medical evaluation and treatment. Additionally, if either Neisseria gonorrhoeae or Chlamydia trachomatis is identified with culture or immunoassays, infection with these pathogens must be reported to the local health department. Acceptable regimens for the treatment of gonorrheal cervicitis include one-time treatment with ceftriaxone 250 mg IM or cefixime 400 mg PO. Alternative regimens for gonorrheal cervicitis include ceftizoxime 500 mg IM; cefoxitin 2 g IM with oral probenecid 1 g; or cefotaxime 500 mg IM. Spectinomycin 2 mg IM is the recommended treatment for patients with penicillin or cephalosporin allergy; however, spectinomycin is not currently available in the United States. Previously, quinolones were an option for gonorrheal cervicitis, but these regimens are no longer recommended by the CDC due to increasing resistance to the quinolones in Asia and many metropolitan areas of the U.S. Additionally, all patients with cervicitis should receive a regimen to treat chlamydia. These regimens include azithromycin 1 g PO once or doxycycline 100 mg PO twice a day for 7 days. Metronidazole 2 g PO is the regimen used to treat vaginitis due to Trichomonas, not mucopurulent cervicitis.

6 weeks after returning from summer camp, a 17-year-old boy presents with fever, flu-like symptoms, and a sore throat. He is clearly tired, and for the past week, he has been experiencing a lack of appetite, difficulty in swallowing, and sleepiness. On further examination, he has upper lid edema, swollen lymph nodes, and pharyngitis. Blood tests reveal increased level of lymphocytes. Question: What is the most likely infecting agent? Answer Choices: A Herpes simplex virus-1 B Herpes simplex virus-2 C Epstein-Barr virus D Varicella-zoster virus E Cytomegalovirus

C

A 1-month-old full term male infant has been diagnosed with Tetralogy of Fallot. His disease is being classified as moderate, and he has been admitted to the neonatal intensive care unit for monitoring. He is now stable and is doing well. During a consultation between the newborn's parents and the pediatric cardiologist, treatment options are being discussed. What is the pediatric cardiologist likely to recommend as definitive treatment? A Chronic oral β-blocking agents B No treatment, as this disease is self-limiting C Closure of ventricular septal defect and pulmonary valvulotomy D Closure of atrial septal defect and aortic valvulotomy E Closure of atrial septal defect and pulmonary valvulotomy

C

A 10-year-old boy presents with a 1-week history of progressing joint pain. The pain started in his ankles, and then progressed to his knees; his hips are now starting to hurt. His ankles feel slightly better. He had contact with someone who had strep throat within the past couple of weeks. The patient's heart rate is 130. On exam, there is erythema and edema over the knees and hips as well as minimal edema over the ankles. A high-pitched holosystolic murmur is noted over the apex and radiates to the axilla with a noted friction rub. Question: Based on the above history and physical exam, what is the most likely diagnosis? Answer Choices: A Juvenile Idiopathic Arthritis B Systemic Lupus Erythematosus C Acute Rheumatic Fever D Kawasaki Disease E Septic Arthritis

C

A 17-year-old girl presents with sore throat and weakness. She has a fever of 100° F. There is cervical lymphadenopathy in physical exam, and the test of Monospot is positive. After 8 days, the patient develops acute abdominal pain. During the abdominal examination, she suddenly becomes pale, sweaty, and cold. What is the proper regimen in this case? Answer Choices: A Antibiotics, intravenous fluids, and observation B Urgent gastroscopy C Laparotomy D Colonoscopy E Double-contrast barium enema

C

A 2-year-old boy is from a poor, rural family. He is experiencing a childhood exanthematous disease that involves a maculopapular rash and a fever. It started 7 days ago. He is now suffering from corneal ulcers and pneumonia. He was not given the childhood immunizations properly. Question: Refer to the image (same image as above) and case. What best describes the distribution of the rash? Answer Choices: A Faint rash begins on the cheeks and spreads to the trunk and extremities B Pruritic rash starts from trunk and spreads peripherally C Maculopapular rash starts behind the ears and spreads to the face and down to the trunk and the extremities D Diffuse erythema on mucous membranes, palms, and soles with polymorphous rash E Diffuse erythematous rash, more prominent on abdomen and cutaneous folds with circumoral pallor

C

A 2-year-old boy is from a poor, rural family. He is experiencing a childhood exanthematous disease that involves a maculopapular rash and a fever. It started 7 days ago. He is now suffering from corneal ulcers and pneumonia. He was not given the childhood immunizations properly. Question: Refer to the image and case. Which of the followings are the all-prodromal signs of this disease? Answer Choices: A Koplik's spots, fever, cough, conjunctivitis, and maculopapular rash B Koplik's spots, coryza, fever, and cough C Koplik's spots, coryza, fever, cough, and conjunctivitis D Coryza, fever, and conjunctivitis E Coryza, fever, and maculopapular rash

C

A 21-year-old male college student presents in student health office with a maculopapular rash that began 1 day ago. He explains that the rash started on his forehead and has spread down to his trunk and extremities over the past 24 hours. Upon further questioning, you learn that he also experienced a 2-3 day history of mild fever, aches, and pink eye that has since resolved. For religious reasons, his parents refused immunizations for him. Physical exam reveals a maculopapular rash scattered about the face and trunk, with no oropharyngeal lesions noted. Mildly enlarged, slightly tender anterior cervical adenopathy is noted. The remainder of the physical exam was unremarkable. Question: The most likely diagnosis is Answer Choices: A Rubeola B Varicella C Rubella D Infectious mononucleosis E Fifth disease

C

A 21-year-old man presents with acute onset of pleuritic chest pain accompanied by 2-3 days of fever, chills, arthralgias, and myalgias. Upon further questioning the patient notes that 4 weeks ago he had a severe sore throat and fever but was not evaluated for these symptoms. Physical examination reveals a febrile patient in mild distress. A diastolic murmur is noted in the left 4th/5th intercostal space that radiates to the left axilla. A friction rub is also appreciated on exam. Laboratory results reveal an elevated erythrocyte sedimentation rate (ESR) and antistreptolysin antibodies. Question: What is the most likely diagnosis for this patient's presentation? Answer Choices: A Pericarditis B Pleurisy C Acute Rheumatic Fever D Endocarditis E Influenza

C

A 23-year-old Hispanic man presents with a 2-week history of upper abdominal pain, occasional right lower chest pain, and low grade fever with malaise and appetite loss. He has been in USA for 2 years, and he recently went to Mexico about 3 months ago to visit his family. He denies nausea or vomiting. He does remember having a short, self-limiting period of diarrhea right after returning from Mexico. He has no past medical history and takes no medications. He does not smoke or drink. On examination, his temperature is 101.2 F, pulse 90/minute, BP 120/76 mm Hg, and respiratory rate 16/minute. There is no icterus, cyanosis, or pallor. Chest exam reveals decreased breath sounds in the base of the right lung and normal heart sounds. Abdominal examination shows tenderness in the right upper quadrant and an enlarged liver about 2 inches below the subcostal margin. Bowel sounds are normal, there is no ascites or splenomegaly, and rectal exam is unremarkable. Labs show WBC of 18,000/uL, Hb 12g%, platelets 320,000/uL, ALT 76 U/L, AST 62 U/L, and AP 85 U/L. Gamma GT is normal. CXR reveals small right-sided pleural effusion, and EKG is normal. Stool culture and blood cultures are pending. Ultrasound of the liver shows a single abscess 3 inches by 2 inches in the right lobe of the liver. Question: What regimen would be best for this patient? Answer Choices: A Amoxicillin B Levofloxacin C Metronidazole D Percutaneous needle aspiration and drainage of the abscess E Combination therapy with metronidazole and paromomycin

C

A 23-year-old man presents with a 2-day history of a swollen, painful, red right knee. The pain and swelling have been progressing steadily since they began. The last few hours, he has also been feeling feverish. He admits to having unprotected sexual intercourse with multiple women over the past few months. On examination, his temperature is 100.4 degrees Fahrenheit (38 degrees Celsius), and BP is 110/70. Cardiovascular, respiratory, and abdominal exam are normal. His right knee demonstrates clinical signs of an effusion, with severe tenderness and surrounding muscle spasm. An aspirate of the joint fluid yields the following findings: • WBC count- 60,000/cc, neutrophils 95% • No crystals • Gram stain is negative • Synovial fluid culture - Gram-negative diplococci on Thayer-Martin media Question: Considering the probable diagnosis, what is the most appropriate pharmacotherapy? Answer Choices: A Clindamycin B Fluconazole C Ceftriaxone D Ampicillin E Vancomycin

C

A 23-year-old man presents with a 2-day history of watery nasal discharge, malaise, sneezing, and nasal congestion. On examination you notice inflammation of the nasal mucosa; pulse is 80/min; BP is 130/ 84mmHg; and temperature is 98.8°F. The rest of the examination is normal. Question: How will you treat this patient? Answer Choices: A Azithromycin + nasal decongestants + Ibuprofen B Ibuprofen only C Nasal decongestants + non-pharmacologic remedies like steam inhalation D Anti-viral agents + nasal decongestants + Ibuprofen E Antiviral agents only

C

A 25-year-old Caucasian male landscaper presents with a 2-week history of generalized malaise and an 'unusual rash' on his right thigh. The patient reports that this rash has been widening, but denies any pruritus or pain in association with his complaints. In the past week, he has also noticed a constant headache and mild fever. The past medical history is unremarkable. The physical exam reveals vital signs within the normal limits, enlarged, non-tender diffuse lymph nodes in cervical and inguinal areas, as well as an erythematous rash with central clearing and few satellite lesions. If your diagnostic impression of Lyme disease were to be confirmed, how would this patient's disease stage be defined? Answer Choices: A Early-onset B Late-onset C 1st stage D 2nd stage E 3rd stage

C

A 27-year-old woman presents because she noticed some cobblestone changes in her genital area 1 month prior. She has no other symptoms. Her current sex partner has similar changes on the skin at the base of his penis. Examination of the genitalia reveals multiple dry small skin-colored, papular keratotic lesions in the skin of the perineal area. The rest of the examination is normal. You suspect the diagnosis of condyloma acuminata. Question: Which of the following is an appropriate next step in therapy? Answer Choices: A No further treatment is indicated B Topical fluorouracil C Podofilox 0.5% gel D Silver nitrate E Gardasil

C

A 27-year-old woman presents because she noticed some cobblestone changes in her genital area 1 month prior. She has no other symptoms. Her current sex partner has similar changes on the skin at the base of his penis. Her last Pap test, which was done about 2 years ago, was normal. She has been sexually active with both sexes since age 18, and she has had a total of 8 sex partners over her lifetime. For the last 2 years, she has been sexually active with 1 partner, and she uses oral contraceptives for birth control. She has no history of STDs. Examination of the genitalia reveals multiple dry small skin-colored, papular keratotic lesions in the skin of the perineal area. The rest of the examination is normal. Question: What should be your next step in order to make a diagnosis? Answer Choices: A VDRL testing B DNA polymerase C Pap smear D Biopsy of the lesion E HIV viral load

C

A 28-year-old man presents with severe headaches. He states that they began a few weeks ago, and he has been taking over-the-counter medications with some relief. He admits that he has AIDS. You ask what the patient was doing prior to when he experienced these headaches. He says that he was taking care of the neighbor's cat while they went on vacation. Concerned, you order a CT scan; it demonstrates multiple ringed-enhanced lesions. What organism do you suspect? Answer Choices: A Histoplasma capsulatum B Cryptococcus neoformans C Toxoplasma gondii D Cytomegalovirus (CMV) E Herpes simplex virus

C

A 3-month-old infant is brought in by her parents because they are concerned about her health. They report that the infant has grown lethargic and weak. When they try to feed her, she is unresponsive to the presence of the baby bottle or breast-feeding. Physical examination is remarkable for hypotonia of the extremities and a flaccid neck. She has a poor gag and suck reflex. Her body temperature is normal, with a slightly decreased respiration rate. Further questioning of the parents reveals that the infant had been previously induced to bottle feed by coating the nipple with honey. Blood is drawn for toxin studies, and they come back positive. The honey is also examined for the presence of toxin, and it is also found to be positive. What organism is causing the infant's disease? Answer Choices: A Campylobacter jejuni B Bacillus cereus C Clostridium botulinum D Vibrio parahaemolyticus E Listeria monocytogenes

C

A 3-year-old boy is evaluated for a 24-hour history of diarrhea. His mother reports that he had 5 episodes of foul-smelling, watery diarrhea associated with decreased appetite. A few other children at the same day care center have also presented with the same problem. On physical examination, the child is well hydrated and his abdomen is tender. Stool microscopy shows the presence of motile trophozoites of the etiological agent. What is causing the patient's symptoms? Answer Choices: A Cryptosporidiosis B Cyclosporiasis C Giardiasis D Isosporiasis E Microsporidiosis

C

A 30-year-old Asian man develops fever, cough and expectoration. 2 days later, he presents with red tender nodules on his shins. He reveals that he returned 10 days ago from a vacation in California. While there, he went exploring the desert with his friends. He has no known allergies. An X-ray film of the chest shows a left pleural effusion. A dimorphic fungal infection endemic in the state of California is suspected as the cause of the patient's illness. What is the diagnosis? Answer Choices: A Aspergillosis B Blastomycosis C Coccidioidomycosis D Histoplasmosis E Paracoccidiomycosis

C

A 30-year-old woman with no significant past medical history presents with a history of recurrent palpitations. These episodes occur primarily upon exertion. She recalls periodic bouts of anxiety, panic attacks, and lightheadedness. She denies fever, chills, changes in weight, chest pain, shortness of breath, rashes, diaphoresis, abdominal pain, nausea, and vomiting. She denies any history of cigarette smoking, drug, or alcohol use. Her physical exam revealed normal vital signs. The cardiac exam revealed a high-pitch late systolic click at the apex. The valsalva maneuver and a standing position result in prolongation of the murmur and a movement of the click to earlier in the cardiac cycle. The remainder of her examination is normal. What is the most likely diagnosis? A Patent ductus arteriosus B Mitral regurgitation C Mitral valve prolapse D Mitral stenosis E Hypertrophic cardiomyopathy

C

A 34-year-old man closely adhered to a personal policy of annual physical checkups and routinely received a clean bill of health. 3 weeks after returning from a business trip to California, he noticed a small circular lesion on tip of his penis. The area became more sensitive and began to ooze fluid. Medical examination revealed lymphocytes and plasma cells and no unexpected Gram-stainable bacteria. A serum sample showed a positive VDRL test with a titer of > 1:64. If the patient does not receive medical treatment, and the lesion clears, there remains the possibility of what condition? Answer Choices: A Nongonococcal urethritis B Primary syphilis C Secondary syphilis D Tertiary syphilis E Latent syphilis

C

A 35-year-old Caucasian woman presents with a 5-day history of nervousness and palpitations. She has had diarrhea for most of the previous month and has had a 3 kg weight loss despite an increased appetite. She also reports increased fatigue and sweating. Her vital signs are an irregular pulse of 114 BPM, blood pressure of 125/75 mm Hg, respirations of 18/min, and a temperature of 37.8°C. Physical exam reveals exophthalmos, a mass in the midline of her neck that moves with deglutition, fine resting tremor, and hyperactive reflexes. Her thyroid stimulating hormone (TSH) level is low. An ECG is performed, revealing atrial fibrillation with an atrial rate of 420/min, and a ventricular response ranging between 110 and 130/min. What would be the most appropriate initial intervention regarding her arrhythmia? A Heparin B Electrical cardioversion C Esmolol D Ibutilide E Amiodarone

C

A 35-year-old man is admitted to the hospital with progressive shortness of breath, fever, and worsening cough. The patient had been in good health until 2 months ago, when he began losing weight. This was associated with anorexia, intermittent diarrhea, night sweats, and then a nonproductive cough. He had lost more than 20 pounds by the time he was admitted to the hospital. His past medical history is unremarkable. He has been divorced for 5 years, and he has 1 child. He is employed as a medical equipment salesman, traveling extensively in the Midwest. He admits to drinking alcohol in large amounts on weekends, but he denies tobacco and intravenous drug use. He gives history of a previous homosexual encounter. Physical examination shows that the chest was normal to percussion and clear by auscultation, except for a few scattered ronchi. The heart is normal except for tachycardia. The abdomen is soft with normal bowel sounds. Genitalia are normal; however, there is a painful 2 cm ulceration at the anal verge. The neurologic exam is unremarkable. Chest radiological findings show diffuse bilateral interstitial infiltrates. Arterial blood gases on room air show pO2- 57mmHg, pCO2 31 mmHg, and pH 7.45. His alveolar-arterial O2 gradient is 55mmHg. Bronchoalveolar lavage fluid with lung biopsy shows the presence of cysts. Sputum cytology is negative for acid-fast bacilli. Question: What is the most likely diagnosis? Answer Choices: A AIDS-related tuberculosis B Secondary syphilis C Pneumocystis pneumonia D Lung cancer E Legionella pneumonia

C

A 35-year-old patient is admitted to the ER presenting with a severe headache. The headache is now 1 week old and has progressively worsened to the point where the patient finds it difficult to tolerate. He has been experiencing photophobia, malaise, and fevers that have gone up to 40° C. His present temperature is 38.3° C. The patient appears confused and disoriented, and his response to questioning is delayed. He mentions a 3-year history of IV drug abuse. His vital signs are all within normal limits. A chest examination and radiograph is normal; a CT scan is normal. The patient's mental status continues to deteriorate, and he is admitted. A lumbar puncture is done, and the results can be found in the table. An India ink prep prepared by the microbiology laboratory produces the following (refer to the image), and a specific antigen test is positive. The organism is recovered from the CSF 48 hours later. What is the most likely organism causing the patient's illness? glucose - 22 Protein - 89 WBC - 32 diff - lymphocytes 89%, monocytes 11% RBC - 0 Answer Choices: A Neisseria meningitidis B Streptococcus pneumoniae C Cryptococcus neoformans D Haemophilus influenza E Listeria monocytogenes

C

A 35-year-old woman presents with fatigue and yellowish coloration of her eyes and skin that started several weeks after noneventful implantation of the prosthetic mechanical heart valve. Physical examination reveals the presence of regurgitant murmur and subicterus. Laboratory results are: hemoglobin 7.0 g/dL; reticulocytes 21%; white blood cells 11,500/µL; platelets 80,000/µL; and undetected levels of haptoglobin. In lactate dehydrogenase, the direct and indirect bilirubin levels are all elevated (3,100 U/L, 2.1 and 1.2 mg/dL, respectively). Peripheral blood smear shows burr and helmet cells (schistocytes) and polychromasia. Both direct and indirect Coombs' tests are negative. You suspect microangiopathic hemolytic anemia. What is the next step in management? A Hemoglobin electrophoresis B Glucose-6-phosphate dehydrogenase deficiency C Echocardiography D Hepatitis B panel E Direct agglutination test

C

A 38-year-old man presents with a 2-week history of headache and a 1-day history of mild right-sided weakness. On exam, he is alert, mildly dysarthric, and has a mild to moderate right hemiparesis. A contrasted head CT shows 3 ring enhancing lesions in the left hemisphere. What is the proper initial management of this patient? Answer Choices: A A biopsy of the lesions B Empiric treatment with amphotericin B and roxithromycin C Empiric treatment with sulfadiazine and pyrimethamine D Lumbar puncture E Empiric treatment with amphotericin B and flucytosine

C

A 39-year-old man presents with a 1-week history of severe chest pain. He states that the pain seems to worsen when he lies down. He describes the pain as radiating to the back, and it also worsens when he takes a deep breath. His vital signs are as follows: blood pressure 124/ 84 mm Hg, respiratory rate 18/ min, temperature 101°F, and pulse 74 beats per minute. On auscultation of the chest, you cannot distinguish a S1 or S2 but hear a scratching or grating sound. What is the first step in treatment of this patient? A Pericardiocentesis B Beta blockers C Rest and NSAIDs D Corticosteroids E No treatment necessary

C

A 42-year-old woman was diagnosed with deep vein thrombosis of the left leg 3 weeks ago, and therapy was initiated with heparin. History includes smoking and birth control pills for endometriosis several years ago. On exam lungs show a decrease in air entry bilaterally. Which of the following is most likely? A Lupus anticoagulant B Oral contraceptive induced thrombosis C Trousseau's syndrome D Heparin induced thrombosis E Paroxysmal nocturnal hemoglobinuria (PNH)

C

A 43-year-old woman is interested in getting pregnant; she is being evaluated for a follow-up. She has a history of poorly controlled hypertension, but she is compliant to 3-drug antihypertensive therapy. Her physical exam reveals hypertension, a cardiac gallop, and a medium-pitched systolic murmur that is best heard posteriorly in the interscapular area; there is radiation to the left axilla, apex, and anterior precordium. The lower extremities demonstrate a 16 mm Hg pressure difference as compared to the upper extremities. Additionally, there are delayed femoral pulsations; her upper extremity pulsations are normal. What health care recommendation would be considered appropriate regarding this patient? A A high sodium, low potassium diet and restricted physical activity should be encouraged B Cardiovascular surgical intervention is contraindicated due to her increased age C Following procedural correction, blood pressure should be continuously monitored D Conceiving a child can be safely encouraged without risk to a developing fetus E Surveillance echocardiograms are not necessary in this patient

C

A 45-year-old man presents with a 3-week history of fever, malaise, and cough. The illness began with low-grade fever, headache, sore throat, and non-productive cough. He tried symptomatic treatment with over-the-counter medicines and did not take any antibiotics. His condition worsened, and he developed expectoration of non-blood stained sputum, shortness of breath, and chest soreness. There was no history of exposure to birds or any new environment. Chest auscultation detected scattered rales and expiratory wheezes. Chest X-ray showed bilateral diffuse infiltrates and consolidation of right lower lobe. Gram-stain of sputum showed presence of leukocytes and normal flora. Routine culture for bacterial pathogens grew only normal flora. The microbiology laboratory was not equipped for isolation of fastidious organisms. Considering the presumptive clinical diagnosis of atypical pneumonia, a supportive non-specific serological test was done to detect cross-reacting antibodies to human erythrocytes. Test result showed significant titer of the antibodies. What is likely to be the causative agent in the above patient? A Chlamydia pneumoniae B Legionella pneumoniae C Mycoplasma pneumoniae D Coxiella burnetii E Chlamydia psittaci

C

A 45-year-old woman with a no significant past medical history presents with a 4-month history of dull, aching heaviness sensation in her proximal right leg. She notes that this sensation is provoked by extended periods of standing and walking, and is relieved when she lies in a recumbent position. Her past medical history is remarkable for pregnancy 4 times, the most recent being approximately 2 years ago. She denies a history of smoking, trauma, injuries, fever, chills, chest pain, shortness of breath, hemoptysis, cough, skin changes and coolness, and peripheral edema. Her physical exam reveals several dilated, tortuous, elongated veins along the medial right thigh, which are especially pronounced upon standing. The remainder of the physical exam is normal. What will be the most appropriate therapeutic approach for this patient at this time? A Warfarin (Coumadin) B Clopidogrel (Plavix) C Sclerotherapy D Furosemide (Lasix) E Cilostazol (Pletal)

C

A 46-year-old man presents for evaluation of his diabetes. He has been erratic with taking his medication. His BMI is 28 and BP is 155/100 mmHg; he exercises regularly. He currently smokes 1 pack of cigarettes per day. His lab values are as follows: Blood Urea: 32 mg/dl Creatinine: 1.4 mg/dl Fasting Blood Sugar: 148 mg/dl Post Prandial Blood Sugar: 213 mg/dl HbA1c: 9.6% HDL: 39 mg/dl LDL: 131 mg/dl Triglyderides: 280 mg/dl What is the best step in the management of his cholesterol levels? A Recommend diet alone B Repeat the lipid profile after 6wks and reassess C Start him on a statin drug at this time D Recommend exercise alone E Start niacin for high triglycerides and vitamin deficiency

C

A 49-year-old man presents with chronic fatigue, headache, fevers, and muscle and joint pain. He describes transient "reddish spots", which can be quite large, that have appeared on his skin and then faded away. His symptoms developed about 4 months ago with no apparent cause; they have gradually gotten worse with the recent addition of the musculoskeletal pains. He is married and monogamous, and lives in a small rural community; nevertheless, he is concerned that he may have somehow contracted a communicable STD. What condition is causing the patient's symptoms? Answer Choices: A Acute rheumatic fever B Alcoholic cirrhosis C Lyme disease D Malaria E Polymyalgia rheumatica

C

A 52-year-old man is hospitalized due to an acute myocardial infarction. Cardiac enzymes are as follows: Myoglobin Normal CK-MB Elevated Troponin T Elevated Troponin I Elevated Given the above information, when did the patient's myocardial infarction most likely occur? A Within 2 hours of when the cardiac enzymes were drawn B Within 12 hours of when the cardiac enzymes were drawn C Within 2 days of when the cardiac enzymes were drawn D Within 5 days of when the cardiac enzymes were drawn E Within 10 days of when the cardiac enzymes were drawn

C

A 52-year-old patient, with a known case of renovascular hypertension, presents in the with poorly-controlled hypertension. He has been treated with both enalapril and nifedipine. He had been diagnosed with unilateral left renal artery stenosis, but recent tests have demonstrated mild changes in the right renal artery also. What should be the next step in management? A Diuretics B Reduction of NaCl consumption C Percutaneous transluminal angioplasty D Add atorvastatin and observe E Left nephrectomy

C

A 52-year-old woman presents for a routine checkup. She has 2 children, and she attained menopause 1 year prior to presentation. Pap smears, mammogram, and DEXA bone scan are normal. She is a non-smoker. Her previous biennial checkups were always normal. Her BP is 142/96; pulse is 72 bpm. Her lab values are below. • Fasting Blood sugar: 112 • Post Prandial Blood sugar: 138 • Total cholesterol: 190 • LDL: 102 • TSH: Normal levels What is the next best step in the management of this patient? A Reassurance B Thiazide C Diet and exercise D Statin E Metformin

C

A 58-year-old man with recently diagnosed Type II diabetes on metformin has developed Stage I hypertension over the past 3 months. When deciding what anti-hypertensive medication to begin for this patient, the primary reason for using an ACE inhibitor is to: A Aid in glycemic control and decrease insulin resistance B Decrease plasma volume and cardiac output C Delay the progression to end-stage renal disease D Decrease peripheral vascular resistance E Improve serum lipid profile

C

A 60-year-old man, following up after a recent myocardial infarction, presents with sharp inspiratory chest pain. Other than his recent myocardial infarction, his past medical history is significant for peptic ulcer and renal insufficiency. You suspect Dressler's syndrome after seeing diffuse ST elevations on an electrocardiogram. What is the most appropriate treatment for this patient? A Indomethacin B Ibuprofen C A corticosteroid taper D Hydrocodone E Diclofenac sodium

C

A 60-year-old woman presents with pyrexia, malaise, and dry cough. 15 days ago, the patient had an upper respiratory tract infection with pyrexia. The chest shows an interstitial pattern of lymphocytic inflammatory infiltrate. What pulmonary infection is causing the patient's symptoms? Answer Choices: A Streptococcus pneumoniae B Candida albicans C Influenza A D Actinomyces Israeli E Mycobacterium kansasii

C

A 62-year-old man presents at the coronary care unit following a massive anterolateral myocardial infarction. After coronary stenting and intensive care, he is fit for discharge, which occurs 10 days after initial admission. He is a known hypertensive and diabetic, and his conditions were well controlled prior to the attack. At the time of discharge, his BP is 120/70; pulse is 70/minute. Physical examination reveals no significant findings. An intern suggests placing the patient on spironolactone (Aldactone), quoting a recent study that he had read. What pathophysiologic mechanism of this disease does the intern aim to combat? A Potassium retention by aldosterone B Potassium depletion by renin C Cardiac remodelling by aldosterone D Fluid retention by ADH E Hypertension by Aldosterone

C

A 62-year-old man with a past medical history significant for a 15-year history of hypertension presents with severe tearing chest pain radiating through to the back. His blood pressure is 180/110 mm Hg, heart rate is 120 BPM, and respiratory rate is 34/min. Physical exam findings include neck negative for bruits/jugular venous distension (JVD), lungs clear to auscultation, heart regular rhythm despite tachycardia, normal S1/S2 with an S4 present, and a grade III/IV diastolic rumbling murmur noted with patient leaning forward. Radial pulses are 1+ on right and 3+ on left. EKG reveals a sinus tachycardia and evidence of left ventricular hypertrophy. What is the most likely etiology of this patient's symptoms A Spontaneous pneumothorax B Pulmonary saddle embolus C Thoracic aortic dissection D Coarctation of the aorta E Acute viral pericarditis

C

A 66-year-old man with a past medical history of diabetes mellitus type II, reflux esophagitis, hypothyroidism, and hyperlipidemia presents with constant, severe "squeezing, crushing chest pressure and tightness" that is experienced in left chest, left shoulder pain and epigastrum. He states that he had just eaten breakfast when the pain began and that his pain has persisted over the last 45 minutes. He denies any additional provoking or relieving factors, including an over-the-counter antacid. He admits to associated weakness, lightheadedness, nausea, and 2 episodes of vomiting. He denies fever, chills, cough, shortness of breath, hemoptysis, or pleurisy. Upon physical exam, he is found to be anxious and restless while his skin was cool and diaphoretic. His blood pressure is 140/91, and he has an obese BMI. His chest is free of deformity or tenderness. His heart rate was normal with a regular rhythm and free of murmurs, gallops, or rubs. His cardiac, musculoskeletal, and pulmonary exams were normal. A bedside EKG revealed ST-segment elevations of 2 mm in 3 anatomically contiguous leads. What is the most likely diagnosis? A Peptic ulcer disease B Bacterial pneumonia C Myocardial infarction D Acute pericarditis E Pulmonary embolism

C

A 69-year-old man was brought to the Emergency Department by ambulance with sudden onset of severe, described as tearing, chest pain that radiates down the back for the past 25 min. He admits to a long-standing history of hypertension that has not been treated effectively for the past 25 years. He states that he feels short of breath but without dyspnea, is beginning to feel dizzy, his legs are becoming weak, and paralysis is beginning to occur. His vitals are as follows: respirations are 24/min, pulse is 110/min, BP is 208/152 mm Hg in right arm and 212/156 mm Hg in the left arm. He is afebrile. At this time, based on the above information, the most likely diagnosis is A Myocardial infarction B Acute pancreatitis C Dissecting aortic aneurysm D Pulmonary embolism E Inferior vena cava syndrome

C

A 7-year-old boy from a family of Caribbean immigrants presents for a regular medical check-up. He is in good health. His immunization status is not known. What will be your recommendation regarding pertussis protection? Answer Choices: A Provide herd immunity B Influenza vaccine C Tetanus and diphtheria toxoids and acellular pertussis vaccine (Tdap) D DTP vaccine containing the whole cell pertussis component, diphtheria, and tetanus toxoid E Preventive antibiotics

C

A 70-year-old African-American man presents with gradual but progressive dyspnea and fatigue on exertion. He notes difficulty in climbing stairs; there is associated lightheadedness, increased abdominal girth, and swollen bilateral lower extremities. He admits to chronic alcohol use, but he denies any heart disease, chest pain or pressure, diaphroesis, palpitations, a history of diabetes, cigarette smoking, or claudication. His physical exam reveals sinus tachycardia, bibasilar rales, a laterally-displaced PMI, an elevated JVP, an S3 gallop, a mitral regurgitation murmur, peripheral edema, and abdominal ascites. A bedside echocardiogram noted four-chamber dilatation, increased left ventricle end-diastolic diameter and volume, thinning left ventricle walls, left atrial enlargement, and limited mitral and aortic valve opening, What is correct regarding this patient? A In temperate zones, Trypanosoma cruzi is most common infectious etiology B There is no genetic association of this illness C Atrophy and hypertrophy of myocardial fibers occurs histologically D Prolonged bradycardia is a known precipitant of acute episodes E Endomyocardial biopsy is an essential element in the diagnosis

C

A 70-year-old woman with a history of hypertension, hyperlipidemia, and myocardial infarction presents with a 3-day history of shortness of breath at rest. She has found it difficult to walk short distances due to shortness of breath and is experiencing orthopnea and nocturnal dyspnea. She denies cough, fever, chills, nausea, abdominal pain, vomiting, diarrhea, rashes, or edema. Upon physical examination, the patient is short of breath, requiring numerous pauses during conversation. She is tachycardic, diaphoretic, and extremities are cool. There is a diminished first heart sound, S3 gallop, laterally displaced PMI, bibasilar rales and dullness to percussion, and expiratory wheezing noted. There is no JVD noted, and 2+ pitting edema of the lower extremities to the level of the mid calf. What intervention will provide the greatest symptomatic relief to this patient? A Pericardiocentsis B Intravenous beta-blocker C Intravenous diuretic D Oral calcium channel blocker E Subcutaneous lovenox

C

A 72-year-old man presents to the office with a 1-week history of a tingling and burning sensation on the left side of his forehead. He also reports reddening and blisters in the same area; they have been present for the past 3 days. There is accompanying intense pain. He reports a mild prodrome of headache. On exam, vesicles are present about the temple region in a dermatomal distribution that does not cross midline. What is the most likely diagnosis? Answer Choices: A Herpes simplex virus 1 B Herpes simplex virus 2 C Herpes zoster D Impetigo E Folliculitis

C

A 9-month-old female infant presents with an acute onset of a rash on her trunk. She has a 4-day history of fever up to 104oF, but the mother states her daughter has no fever today. She has had some diarrheal stools, but no vomiting. No coughing or nasal congestion has been noted. The child has previously been well. Her past medical history is unremarkable, and she is up-to-date on her immunizations. She attends day care, and her mother notes that some children have been ill with non-specific febrile illnesses over the past 2 weeks. Her only medication has been ibuprofen for the fever. Physical exam shows a temperature of 98.8 F, pulse of 124 BPM, and respiratory rate of 28/min. She is alert and shows no other abnormalities. Her exam shows normal tympanic membranes in the ear and normal conjunctivae in the eye. The neck shows some shotty anterior cervical adenopathy; the throat shows slight erythema of the posterior pharynx, and the skin appears with an erythematous maculopapular rash most pronounced on the trunk. Her chest is clear to auscultation, heart rhythm is regular without murmurs, abdomen is soft and non-tender, and her neurological exam is normal. Question: What is the most appropriate intervention for this patient? Answer Choices: A Draw measles titers of acute and convalescent sera B Oral acyclovir C Observe the child and reassure the parent D Draw a complete blood count and blood cultures E Treat presumptively with oral antibiotics

C

A mother has brought her 4-year-old daughter to you because the child has been suffering from a 1-week history of intense itching on her scalp since she returned from a summer camp with a group of other children. Physical examination shows a number of nits attached to the hair shafts and a few full grown lice on the scalp. Due to scratching, there are raw excoriated areas on the scalp with a few areas of pyoderma. Occipital group of lymph nodes are enlarged. Question: What is the treatment of choice for this child? Answer Choices: A Permethrin 5% topical B Permethrin 1% topical C Malathion 0.5% in isopropanol topical D Ivermectin topical E Benzyl benzoate 25% topical

C

A nurse in your office may have been exposed to blood from a patient with AIDS. She was administering an antibiotic injection intramuscularly to a HIV-positive patient and accidentally sustained a needle prick injury. As part of her post-exposure prophylaxis therapy, you instruct her to begin daily tenofovir. What is the mode of action of this medication? Answer Choices: A Cannot be phosphorylated by host cell enzymes B Is an extremely effective inhibitor of the host cell RNA polymerase C Blocks the viral enzyme in reverse transcriptase D Is incorporated into viral DNA as a purine analog causing chain termination E Prevents the virus from entering the cells

C

An 8-month-old boy presents for a routine evaluation. His mother states that the child is gaining weight and feeding appropriately and has been without fever, chills, dyspnea, or other abnormal objective signs. Upon physical examination, the examiner noticed a loud, harsh holosystolic murmur in the left third and fourth interspaces along the sternum that was associated with a systolic thrill. The vital signs were normal, with normal length and weight and a physiologic splitting of S2. There was no cyanosis, edema, or hepatomegaly. What is the most appropriate course of action for this patient? A Cardiac catheterization B Prescribe furosemide and captopril C Observation D Refer for cardiac MRI E Immediate surgical repair

C

An otherwise asymptomatic, thin 18-year-old woman presents with a history of scoliosis and the presence of a mid-systolic click that is auscultated during physical examination; the patient most likely has what valvular disease? A Mitral stenosis B Aortic stenosis C Mitral valve prolapse D Pulmonic stenosis E Aortic valve prolapse

C

Patients who suffer from malaria caused by either P.vivax or P.ovale can have relapses because of the late hepatic tissue phase in the life cycle of these 2 organisms. In order to overcome this, the therapeutic regimen should be designed to ensure a radical cure. What drug in this regimen is most likely to produce hemolysis in glucose-phosphate dehydrogenase deficient patients? Answer Choices: A Pentamidine B Amphotericin B C Primaquine D Metronidazole E Quinine

C

What blood vessel of the prenatal circulatory system forms the medial umbilical ligaments? A Ductus arteriosus B Ductus venosus C Umbilical artery D Umbilical vein E Inferior vena cava

C

What is the most accurate statement regarding histoplasmosis? Answer Choices: A Its most common clinical manifestation is cavitary pulmonary infection B The principal route of acquisition is skin contact with contaminated soil C Itraconazole, amphotericin B, ketaconazole, and fluconazole are all reasonable choices for therapy D The causative fungus exists in the yeast phase in soil and in the mycelial phase in the body E It is rarely seen as an opportunistic infection in immunocompromised hosts

C

What trophozoite demonstrates "falling leaf" motility in the wet mount? Answer Choices: A Entamoeba histolytica B Endolimax nana C Giardia lamblia D Dientamoeba fragilis E Trichomonas vaginalis

C

What vaccination should be used in order to prevent measles? Answer Choices: A HBsAG B Allantoic fluids of infected embryonated hens eggs C Immune serum globulin (ISG) D Serogroup C vaccine E BCG

C

While driving cross-country in a rural part of Southern Ohio, a traveling salesman's car breaks down. His mobile phone is not working. He finds a local farmer, and the farmer lets the salesman spend the night in his farmhouse. His car is repaired the next day, and the salesman promptly returns home. The next morning, he experiences an abrupt onset of a fever, nonproductive cough, shortness of breath, and mild chest pain. Believing he has just caught a cold, the salesman thinks nothing of it. 2 days later, he is much better, but he presents to your office for an opinion. The examination and the chest X-ray are normal. Question: What is the most likely cause of his symptoms? Answer Choices: A Spontaneous pneumothorax B Goodpasture syndrome C Blastomycosis D Community-acquired pneumonia E Mycobacterium infection

C

While making rounds in the NICU, you are asked to evaluate an 8-hour-old male neonate; he is in respiratory distress. On evaluation, you note the neonate to be in mild (but progressing) respiratory distress. He has minimal perioral cyanosis. He is tachycardic and tachypneic. Blood pressure is stable, but you note a blood pressure discrepancy between the upper and lower extremities. He also has absent femoral pulses. You place the appropriate consolations; in the meantime, what is the drug of choice to order at this time? A A beta-blocker B A calcium-channel blocker C A prostaglandin D An ACE-I E A vasodilator

C

A 25-year-old woman and her husband have been using condoms and spermicidal foam for the 8 months since the birth of their baby. She plans to wean the baby from the breast sometime between a year and 18 months of age, but would like to begin a "less messy" method of contraception. Prior to her pregnancy she took combination oral contraceptives for several years without any difficulties. Which of the following is an appropriate recommendation for this woman? A "Breast feeding alone will prevent pregnancy until the baby is weaned." B "Condoms and spermicide are your only option until you wean the baby." C "Progestin-only pills are recommended for women who are breast feeding." D "A vaginal ring will not affect your milk production." E "You can start back on the same oral contraceptive you took before."

C "Progestin-only pills are recommended for women who are breast feeding." C The progestin-only pill is ideal for breast feeding mothers because this pill does not interfere with lactation the way combination pills do. Breast feeding (A) is reasonably effective in preventing pregnancy only as long as breast milk is the infant's only source of nutrition. Condoms and spermicide (B) are options, but not the only ones for breastfeeding couples. A vaginal ring (D) does decrease the amount of milk production somewhat, but may be an effective option if lactation is well established. Combination oral contraceptives (E) are not recommended for breastfeeding women.

A 68-year-old female presents to the emergency department with signs and symptoms of an acute ischemic stroke. The initial CT scan is normal. Her blood pressure is 164/105. What is the most appropriate treatment for the blood pressure of this patient? A Atenolol PO B Clonidine PO C Close monitoring D Labetolol IV E Nicardipine IV

C Close monitoring Aggressively lowering blood pressure may decrease blood flow to the ischemic tissue, thus decreasing the chances of recovery or increasing the risk of further infarction. In the setting of an acute ischemic stroke, blood pressure elevation should be monitored closely, with some elevation expected. This elevation is expected to decline without medication in the first few hours to days, but if elevation continues to a systolic blood pressure greater than 220mmHg, or mean arterial pressure greater than 120mmHg, medication is advised. Medications may include intravenous labetolol or nicardipine, with close monitoring of the patient. After the acute phase following a stroke, appropriate oral medications may be considered for outpatient hypertension management.

Which disorder is characterized by episodes of hypomania and depression for greater than 2 years? A dysthymia B major depressive disorder C cyclothymia D bipolar E mood disorder

C Cyclothymia Cyclothymia is characterized by symptoms of depression and hypomania for at least 2 years. Symptoms are milder than a regular depressive or manic episode. Occasionally, patients will have regular depressive or manic symptoms at which time they need to be reclassified as bipolar.

You are providing care to a woman who is at 33 weeks gestation. Her pregnancy is complicated by gestational diabetes. She is being provided education by the dietician, and has weekly obstetrical appointments. What fasting blood sugar (FBS) readings should necessitate switching from diet control to insulin therapy? A FBS > 70 mg/dL B FBS > 90 mg/dL C FBS > 95 mg/dL D FBS > 110 mg/dL E FBS > 126 mg/dL

C FBS > 95 mg/dL Gestational diabetes has different risks associated with it, in particular for the fetus. Stricter glycemic controls are recommended for pregnant versus non pregnant women by the ACOG and the ADA.

A 26-year-old female presents to clinic complaining of increasing headaches for one year, irritability, bloating and fluid retention, and abdominal discomfort with loose stools during her menstrual cycle. The symptoms begin a day or two before her menses, and last until the middle of her cycle. She has tried acetaminophen and ibuprofen without improvement. On physical exam she is a well-developed, well-nourished female in no acute distress. Vitals are normal, CV and lungs are normal, pelvis exam is normal, pap smear is normal, and GC and Chlamydia testing are negative. What would be the most appropriate next step? A Pelvic ultrasound B FSH, LH levels C Fluoxetine on cycle day 21-7 D Paroxetine daily E Serum HCG

C Fluoxetine on cycle day 21-7 (Prozac) C Hx and Px are key to diagnosing premenstrual syndrome. Laboratory and radiologic procedures are not useful, particularly in light of a normal exam. With failure of NSAIDS, treatment is aimed at reducing symptoms. For mild to moderate symptoms, SSRI therapy prior to and through the menstrual cycle has become a primary therapy.

What type of pharmacological agent would be used as a first-line medication to treat obsessive compulsive disorder? A risperidone B lonazepam C fluvoxamine D trazadone E venlafaxine

C Fluvoxamine, paroxetine, and sertraline are all approved for the treatment of obsessive compulsive disorder. Use of an SSRI in combination with behavioral therapy is recommended.

A 27-year-old woman and her male partner come to the emergency department for assistance with emergency contraception. They experienced condom failure during intercourse an hour ago and neither desires pregnancy. Her last menstrual period was approximately two weeks ago and her cycles occur every 28-30 days. Her medical history includes a deep venous thrombosis during labor and delivery 5 years ago. What is the most appropriate course of action at this time? A Administration of an ethinyl estradiol and levonorgestrel combination now and in 12 hours B Dilation and curettage C Insertion of a copper-containing intrauterine device D Serial beta hCG determinations E Testing for factor V Leiden

C Insertion of a copper-containing intrauterine device C Insertion of a copper-containing IUD is an effective means of preventing an unintended pregnancy in this case. The woman's history of clotting is a contraindication to use of combination oral contraceptives (A). Dilation and curettage (B) is neither appropriate nor effective for emergency contraception. Performing beta hCG determinations (D) would merely detect pregnancy if it were to occur. Given her history of clotting, testing for factor V Leiden (E) may be appropriate but will not affect management at this time.

You are evaluating a 67-year-old male with known cirrhosis of the liver secondary to alcoholic liver disease, although he has been sober for the past year. He is brought in to the emergency department by his daughter, who notes that for the past few days he has seemed to be more confused. On examination you note the patient to be mildly confused but alert to person and place. He has noticeable asterixis. He is not currently taking any medications and his blood alcohol level is undetectable. What is the treatment of choice in this case based on your physical examination findings? A amoxicillin B prednisone C lactulose D folic acid E thiamine

C Lactulose The correct answer is (C). This patient most likely has hepatic encephalopathy due to end-stage liver disease. Asterixis indicates an increase in serum ammonia. The treatment of choice is lactulose. Both folic acid and thiamine are used in the treatment of alcoholic liver disease, but do not treat elevated ammonia levels. Antibiotics may be used secondarily in patients nonresponsive to lactulose, but amoxicillin is not preferred. Prednisone is not a treatment for hepatic encephalopathy.

A 36-year-old male with a history of acute lymphoblastic leukemia (ALL) is treated with high dose methotrexate and glucocorticoids, vincristine, and L-asparaginase at age 10. He has been in remission since induction therapy was completed. He presents with pain to his right hip/back area, which radiates down his leg. He is also having some back pain that keeps him up at night. What is the most likely diagnosis? A Bladder cancer with referred pain B Lymphoma with metastasis to the bone C Osteonecrosis of the right hip D Osteoporosis E Relapsed ALL

C Osteonecrosis of the right hip Common treatment sequelae that can present years out from therapy include osteonecrosis, especially following the use of high dose methotrexate and glucocorticoids. Lymphoma can arise as a post transplant complication, but is not typically seen following anti-leukemic therapy. Osteoporosis is also a complication of anti-leukemic therapy, but is more associated with the use of mercaptopurine. Relapsed ALL is typically seen in the first two years following therapy, but can arise more than 10 years out. Typically, this is seen with fever, pancytopenia, and bony pain. The pain is not typically isolated to one side of the body.

A 63-year-old white male is seen in the ambulatory outpatient clinic with complaints of midepigastric pain, weight loss, and jaundice. On examination, he is jaundiced and his sclerae are icteric. On palpation of the abdomen, you find a distended nontender gallbladder. Which of the following is the most likely diagnosis? A Gastric carcinoma B Chronic pancreatitis C Pancreatic carcinoma D Choledocholithiasis

C Pancreatic Cancer Pancreatic carcinoma presents with weight loss, jaundice, and midepigastric pain. A palpable, nontender gallbladder (Courvoisier sign) is more often associated with a pancreatic malignancy than cholelithiasis, especially if the tumor is in the head of the pancreas. In acute cholecystitis, the obstruction in the cystic duct is associated with inflammation, resulting in a tender gallbladder on palpation of the right upper quadrant (Murphy sign); obstruction of the common bile duct in choledocholithiasis will result in jaundice but not weight loss. Gastric carcinoma will present with midepigastric pain and weight loss but not jaundice or a palpable gallbladder. Midepigastric pain is the most common symptom seen in chronic pancreatitis, and weight loss may be seen in association with malabsorption secondary to exocrine insufficiency.

A 30-year-old woman and her husband have been trying unsuccessfully to become pregnant for the past year. Over-the-counter ovulation tests have indicated that she is ovulating. As part of her evaluation she undergoes a hysterosalpingogram that reveals tubal scarring. Which of the following is the most likely explanation for this finding? A Congenital anomaly of the tubes B Diethylstilbestrol exposure C Past asymptomatic chlamydial infection D Previous abdominal surgery E Scarring from prior uterine instrumentation

C Past asymptomatic chlamydial infection C Chlamydial infection can cause "silent" pelvic inflammatory disease, leading to scarring with subsequent tubal obstruction that can cause infertility or ectopic pregnancy. Congenital anomalies of the tubes (A) may also contribute to infertility, but is not associated with scarring. DES exposure (B) in utero may lead to reproductive-system anomalies in offspring. Past abdominal surgery (D) may lead to adhesions that inhibit fertility. Uterine instrumentation (E) may lead to intrauterine synechiae.

A patient presents to your family practice office with classic renal colic with his pain being a 6 on a 1 to 10 scale, with 10 being the worst pain imaginable. You refer your patient for a stat computed tomography (CT) scan of the kidneys. Your radiologist calls and advises that the CT reveals that your patient has a 4 mm stone at the ureterovesical junction (UVJ) on the left side without evidence of hydronephrosis. What should you advise your patient? A Go directly to the emergency department for admission. B Go directly to the emergency department for hydration and pain management. C Return to the office, administer ketorolac IM, give a prescription for an opioid, increase fluids, strain their urine, go to the emergency department if the pain worsens, and return in 48 hours for a reevaluation. D Go home, take four Motrin IB tablets every eight hours for pain, and go to the emergency department if the pain worsens. E Refer the patient for immediate lithotripsy.

C Return to the office, administer ketorolac IM, give a prescription for an opioid, increase fluids, strain their urine, go to the emergency department if the pain worsens, and return in 48 hours for a reevaluation. C Nonsteroidal anti-inflammatory drugs (NSAIDs) in general, and ketoralac specifically more so than other, causes constriction of the renal afferent arteriole, reducing pressure on a kidney stone and providing significant pain relief. Generally speaking a stone of less than 5 mm, especially at the UVJ, will pass relatively rapidly (within 48 hours) and the patient only needs to strain their urine to collect the stone and receive analgesia.

A 30-year-old female G2 P2, who delivered via normal spontaneous vaginal delivery, presents complaining of increasing vaginal pressure, low back pain, and stress incontinence. What is the mostly likely cause of her condition? A Damage to the levator muscles B Increased intra abdominal pressure C Widening of the levator gap D Widening of the AP pelvis diameter E Endopelvic fascia remodeling and cervical elongation

C Widening of the levator gap Damage to the levator and increased abdominal pressure are known risk factors, but the widening of the gap with the associated risk factors is what allows the defect to occur. Fascia remodeling can occur instead of a prolapse defect.

A 27-year-old woman presents with 3 days of fever, chills, headache, and a deep dry cough. She has been working at a pet store for the past month and thinks that one of the parakeets that came in 10 days ago may be sick. On examination, she has dullness to percussion of the right lung base and right-sided coarse crackles. The most likely diagnosis is A sarcoidosis B tularemia C psittacosis D brucellosis E listeriosis

C psittacosis The key piece of history in this question is the new exposure to parakeets. The symptoms and signs, including atypical pneumonia, are consistent with psittacosis but are not pathognomonic. Sarcoidosis is an illness of unknown cause. Listeriosis has been linked to exposures to contaminated food, particularly dairy products and hot dogs. Brucellosis can be caused by exposure to hogs, cattle, or goats. Tularemia is associated with contact with rabbits, other rodents, and biting arthropods.

A 43-year-old obese man presents for a health maintenance visit. On physical exam, it is noted that his waist circumference is 106 cm and blood pressure is 148/92 mm Hg. Which of the following fasting laboratory levels would suggest a diagnosis of metabolic syndrome (syndrome X) in this patient? A HDL of 45 mg/dL B LDL of 180 mg/dL C triglyceride of 190 mg/dL D glucose of 100 mg/dL

C trig of 190 mg/dL Metabolic syndrome is found in approximately 25% of Americans. It is defined as three or more of the following findings: waist circumference of greater than 102 cm in men or greater than 88 cm in women; serum triglyceride level of at least 150 mg/dL, HDL level of less than 40 mg/dL in men or less than 50 mg/dL in women; blood pressure of at least 130/85 mm Hg; and serum glucose level of at least 110 mg/dL.

A 54-year-old woman has suffered a stroke that has resulted in dramatic changes to her personality, left leg and foot weakness, loss of sensation in the left leg, and apathy. Which of the following arteries was most likely affected by the stroke? A left anterior cerebral B left middle cerebral C right anterior cerebral D right middle cerebral E right posterior cerebral

C. the RAC The anterior cerebral arteries supply the frontal lobes as well as the medial aspects of the parietal and occipital lobes rostral to the parietooccipital sulcus. The prefrontal cortex of the frontal lobe is concerned with a person's personality, depth of feeling, and initiative. Hence, occlusion of an anterior cerebral artery can cause neuronal injury to this area, leading to feelings of apathy and personality changes. The paracentral lobule represents the medial aspects of the precentral gyrus (frontal lobe) and postcentral gyrus (parietal lobe), which are responsible for motor control and somatosensory perception, respectively, of the leg and foot. Hence, occlusion of an anterior cerebral artery can produce contralateral hemiparesis and hemisensory loss involving the leg and foot. With the 54-year-old patient, the symptoms were occurring on the left side, which points to a right anterior cerebral artery occlusion.

Radiculopathy due to nerve root compression occurs most commonly at which nerve root within the brachial plexus? A C5 B C6 C C7 D C8 E T1

C7 the C7 nerve root is affected the most often (approximately 45-60%). This radiculopathy can result from foraminal encroachment of the spinal nerve, cervical disk herniation, tumor, and multiple sclerosis. C7 radiculopathy can present with weakness in the triceps, which cause elbow extension, and finger flexion and extension. C6 is another common site of radiculopathy. C6 radiculopathy can present with weakness in the biceps, brachioradialis, and wrist extensor muscles. Cervical radiculopathy at the C5, C8, and T1 are less common, but still possible. C5 radiculopathy can present with deltoid and biceps muscle weakness. C8 radiculopathy can present with finger flexor weakness and T1 radiculopathy with finger abduction weakness.

A 28-year-old man presents following a road traffic accident 20 minutes prior. He states that it was just a "minor bump from behind," but feels he might have whiplash. His neck is stiff and sore, and he has developed numbness and tingling on the lateral surface of his right arm extending into his right fourth and fifth digits. On physical examination, his bicep strength is +5/5 on the left and +5/5 on the right. His biceps tendon reflex is 2+ on the left and 2+ on the right. In addition, his triceps tendon reflex is 2+ on the left and 2+ on the right. His grip strength on the right is diminished as compared to the left. The remainder of the physical examination is normal. Question Based on the above presentation, what is the cervical nerve root most likely affected? Answer Choices 1 C4 2 C5 3 C6 4 C7 5 C8

C8 Impingement of C8 may cause numbness and tingling primarily on the medial surface of the arm and into the lateral hand into the fourth and fifth digits. It may also cause dysfunction of the hand as it innervates the small hand muscles. Impingement of the C4 nerve root may cause neck and upper shoulder numbness and pain. Impingement of C5 nerve root may cause deltoid and shoulder numbness and pain, and biceps tendon reflex may be diminished. Impingement of the C6 nerve root can cause numbness and tingling down the arm into the thumb, with weakness in the bicep muscle and diminished brachioradialis tendon reflex in the affected extremity. Impingement of C7 also causes numbness and pain down the affected arm but into the middle finger, and the triceps reflex may be diminished on examination.

A 28-year-old man presents following a motor vehicle accident 20 minutes prior. He states that it was just a "fender bender," but he feels he might have whiplash. His neck is stiff and sore, and he has developed numbness and tingling on the medial surface of his right arm and into his right fourth and fifth digits. On physical examination, his bicep strength is +5/5 on the left and +5/5 on the right. His biceps tendon reflex is 2+ on the left and 2+ on the right. In addition, his triceps tendon reflex is 2+ on the left and 2+ on the right. His grip strength on the right is diminished as compared to the left. The remainder of the physical exam is normal. Question Based on the above presentation, what is the cervical nerve root most likely affected? Answer Choices 1 C4 2 C5 3 C6 4 C7 5 C8

C8 The most likely nerve affected in this case is the C8 nerve root. Impingement of C8 may cause numbness and tingling primarily on the medial surface of the arm and into the lateral hand into the fourth and fifth digits. Impingement may also cause dysfunction of the hand, as it innervates the small hand muscles. Impingement of the C4 nerve root may cause neck and upper shoulder numbness and pain. Impingement of C5 nerve root may cause deltoid and shoulder numbness and pain, and biceps tendon reflex may be diminished. Impingement of the C6 nerve root can cause numbness and tingling down the arm into the thumb, with weakness in the bicep muscle and diminished brachioradialis tendon reflex in the affected extremity. Impingement of C7 also causes numbness and pain down the affected arm but into the middle finger, and the triceps reflex may be diminished on exam.

A 29-year-old female complains of a two-month history of easy bruising. She describes that the bruising is located primarily on her shins, but has noted them on other areas as well. She also describes red freckles on her lower extremities. On exam, you note non-blanching and non-palpable purpura to both legs and petechiae. She denies recent illnesses and states that she has essentially been feeling fine. The PE is normal other than the skin findings. Which diagnostic study would be most helpful as a first choice? A Antiplatelet antibody test B Complete blood count C Direct antiglobulin test D Reticulocyte count E Thyroid function

CBC count This patient is exhibiting signs of Idiopathic (Immune) thrombocytopenic purpura. The diagnosis is based on history, physical examination, blood count, and blood film. The American Society of Hematology guidelines recommend no further diagnostic studies. The other studies may be useful if you believe another underlying disease is causing the symptoms and thrombocytopenia, but a complete blood count should be sufficient.

An 8-month-old is admitted to the hospital for the evaluation of possible middle ear infection and chronic diarrhea. Since birth, he has episodes of frequent loose stools. He has had 2 episodes of lower respiratory tract infection with bronchospasm, treated with intravenous antibiotics and bronchodilators. His parents are not related, and they report that their firstborn died of pneumonia at the age of 8 months. On examination, the patient appears irritable, screams, shakes his head, and tries to rub the right ear. Signs of dehydration are noted. His weight is < 5th percentile; his height is 5th percentile, and his head circumference is 50th percentile. His temperature is 38 C. Mucocutaneous changes suggestive of a fungal infection are noted. Right tympanic membrane is erythematous and bulging, with poor mobility on pneumatic otoscopy. Left tympanic membrane appears clear with good mobility. Throat is erythematous. Thyroid, heart, lungs, genitalia, and nervous system are within normal limits; his abdomen is tender. You are not able to palpate lymph nodes, and there are no signs of hepatosplenomegaly. Question What is the next step in the diagnosis of the primary disorder in this child? Answer Choices 1 KoH prep of skin changes 2 CBC with differential 3 Genetic testing 4 Test stool for ova and parasites 5 Tympanocentesis

CBC with differential Your patient experiences recurrent respiratory infections and diarrhea; he also has skin changes and a family history highly suggestive of an immunodeficiency; most likely, it is severe combined immunodeficiency (SCID). SCID is a group of congenital diseases caused by different genetic mutations, resulting in the severe deficiency of both T- and B-lymphocytes. X-linked is the most common type. The clinical picture is, however, similar in all of them: recurrent infections caused by bacteria, viruses, fungi, and opportunistic infections. The initial step in the evaluation of immune status should be complete blood count with the differential. You should evaluate blood cell counts and cell morphology for the presence of lymphopenia, which is the classic hallmark of SCID. KoH prep may help you to differentiate dermatophytes and Candida albicans symptoms from other skin disorders, but it will not be useful in the diagnosis of the primary condition. You may consider genetic testing to differentiate between various forms of SCID as well as the other combined immune deficiencies, but testing should only be done after you prove the existence of immune deficiency (lymphocytes, immune globulins). Testing stool for ova and parasites may be considered in cases of diarrhea, but the testing will not help you to establish the diagnosis of the primary condition. The patient probably has otitis media, and he may have the mastoiditis (peak incidence for the mastoiditis is at age 6 - 13 months; it is much more common in immunocompromised patients). In the case of mastoiditis, fluid extracted from the middle ear through either perforated drums or by tympanocentesis may be sent for Gram staining, culture, and acid-fast stain; however, this procedure will not contribute to the evaluation of immune deficiency.

An 8-year-old boy complained of abdominal pain in the afternoon and vomited twice. No fever is noted, and he has not had any diarrhea. On examination the next morning, he has diffuse lower abdominal pain with rebound tenderness on the lower right side. On rectal examination, there is no localization. Question What is your next recommendation? Answer Choices 1 Complete blood count (CBC) and urine analysis (UA) only 2 Immediate surgery for appendicitis 3 Observation without further treatment 4 CBC, UA, and abdominal ultrasound 5 Barium enema to diagnose and treat a presumed intussusception

CBC, UA, and abdominal U/S This child's history and physical examination are not specific enough to go directly to surgery for presumed appendicitis; however, the child needs more of a work-up than just observation. While not all children with appendicitis have an elevated white count and a left shift, other findings on the CBC such as a hemolytic anemia with thrombocytopenia may lead one to consider Henoch-Schönlein purpura. The same is true if there is marked hematuria and proteinuria. A periappendiceal abscess against the bladder can produce some hematuria and pyuria. A barium enema is sometimes used to assist in the diagnosis of appendicitis. If the appendix fills with barium, it is not swollen from inflammation. However, in the situation outlined, a CBC, UA, and ultrasound to look for a swollen appendix or evidence of a periappendiceal abscess would be the least invasive evaluative protocol with the lowest risk of side effects.

A 35-year-old man presents with headaches. He describes them as only occurring on one side of his head; there is a "sharp stabbing pain just above his left eye". Each headache lasts for about 2 hours at a time; they occur daily for spans of 2 -3 weeks, and they have had the same pattern every 3 - 4 months for the past couple years. During these periods, the patient's nose feels congested and his left eye appears red and waters excessively. The patient has deduced that aggravating factors include alcohol and stressful situations. He denies any known family history of headaches similar to these; there is also no family history of migraines. The physical examination is within normal limits. Question Based on the given history and physical examination, what is an effective preventative medication that might prevent these headaches in the future? Answer Choices 1 Triptans 2 Calcium channel blockers 3 Local anesthetics 4 Oxygen 5 NSAIDs

CCB (like Verapamil) The patient in the above scenario is experiencing a cluster headache. Cluster headaches are headaches that occur more commonly in middle-aged men. Patients will complain of severe unilateral periorbital pain that occurs daily for several weeks and potentially are associated with 1 or more of the following symptoms: ipsilateral nasal congestion, rhinorrhea, lacrimation, or redness of the eye. Patients are restless and agitated during the acute phase of these headaches. Episodes vary in length from around 15 minutes up to 3 hours; at the same time, these occurrences may last for several weeks and then resolve for many weeks at a time. Any time a healthcare provider or a patient approaches a headache of any type, treatments for the acute signs/symptoms as well as preventative treatments should be considered. Fast acting treatments to give the patient immediate relief from the signs and symptoms of a cluster headache include oxygen, triptans, octreotide, local anesthetics, and dihydroergotamine (intravenous only). Various NSAIDs may also be attempted for symptoms, but not for prevention of cluster headaches. Preventative therapies are used at the onset of the cluster episode with the overall goal of suppressing the severity of the headaches. Various options may be attempted as preventative therapy; corticosteroids, lithium carbonate, nerve blocks, ergots (sublingual), calcium channel blockers, and even melatonin usage have been effective. At times, the anti-seizure medications divalproex and topiramate may also be considered. The calcium channel blocker verapamil is very often the first choice for preventing cluster headaches.

Which of the following tests should an HIV-positive pregnant woman undergo in each trimester of pregnancy? A CD4+ lymphocyte count B cytomegalovirus serology C postpartum depression with controls D shielded chest radiography E venereal disease research laboratory (VDRL)

CD4+ count lymphocytic count A HIV-positive pregnant women should undergo CD4+ serology each trimester. Early in the pregnancy, they should undergo shielded chest radiography, CMV baseline testing, and tuberculosis testing with controls. Syphilis testing should be completed initially and as usually recommended later in pregnancy.

A 25-year-old woman presents with pelvic pain for the last 3 - 4 menstrual cycles, but the most recent episode with increasing symptoms for approximately 2 weeks. She has tried over-the-counter pain relievers without relief. She admits deep dyspareunia and has been with her current sexual partner for about 6 months. She has no chronic medical conditions and no allergies to any medications. She denies urinary issues. She admits to nausea, but no vomiting or diarrhea. She reports a negative home pregnancy test this morning. Her last pelvic exam was over 1 year ago. LMP was 1 week ago. Question What physical exam findings would best narrow your diagnosis? Answer Choices 1 Abdominal guarding 2 Adnexa approximately 2.5 cm in size 3 Anteverted uterus 4 Cervical motion tenderness 5 Discomfort with speculum insertion

CMT Explanation Cervical motion tenderness, along with a history suspicious for pelvic inflammatory disease (PID), is a red flag for the diagnosis. While testing for sexually transmitted infections (especially Chlamydia trachomatis and Neisseria gonorrhea) are indicated, the diagnosis is typically based upon history and physical exam. The Centers for Disease Control Sexually Transmitted Treatment Guidelines recommend empiric treatment for PID if one or more of the three following minimum criteria are present on exam: cervical motion tenderness, uterine tenderness, and/or adnexal tenderness. A high index of suspicion is needed, and treatment should be initiated while awaiting laboratory results. The examiner should further elicit this woman's sexual partner history and potential vaginal/cervical discharge. Other conditions, such as endometritis and endometriosis, could cause cervical motion tenderness. The CDC guidelines are helpful in selecting antibiotic therapy and whether to treat as inpatient or outpatient. Most women will find that pelvic exam is uncomfortable, but tenderness with movement of the cervix is not a normal finding and indicates inflammation in the uterus.The remainder of the physical exam findings listed are less helpful than a finding of cervical motion tenderness. Abdominal guarding is a common physical exam finding with many gastrointestinal, urinary, and pelvic disorders. Guarding refers to voluntary or involuntary contraction of the abdominal musculature in response to pain felt during palpation. It is not a specific finding for pelvic inflammatory disease and would not help narrow a diagnosis as much as cervical motion tenderness. A physical exam finding of an adnexa approximately 2.5 cm in size is a normal physical exam finding. When the patient is thin enough for a good estimate of ovarian size, an examiner may notice enlargement in the case of ovarian cysts, ectopic pregnancy, tumors, and other ovarian disorders. A finding of normal ovarian size does not narrow the differential for this patient. An anteverted uterus on physical exam simply indicates the angle of the fundus points toward the ventral side of the patient. This is also a normal finding and does not assist in arrival at a definitive diagnosis. Discomfort with speculum insertion can be related to many factors, including examiner skill, patient anxiety, vaginal inflammation, and many gynecologic disorders. This finding is nonspecific and also does not help with diagnosis.

A 62 year-old woman with a 40 pack year smoking history presents with a history of progressive dyspnea of several months duration. She denies hemoptysis, cough, fever, or orthopnea. Sputum production is positive. On physical exam, she shows evidence of accessory muscle use when breathing, sits in a slightly bent forward position, uses pursed lips, and you notice she is very thin. Auscultation reveals decreased vesicular breath sounds with prolonged expiration. Which of the following is the most likely diagnosis? A CHF B COPD C Lung cancer D Pulmonary emboli E Pulmonary fibrosis

COPD The patient scenario is consistent with COPD (B) based on the history of progressive dyspnea and physical exam findings of obstructive lung disease (i.e., pursed lip breathing and decreased breath sounds with prolonger expiratory phase. The patient lacks physical exam findings of pulmonary edema due to CHF (A). She is at increased risk of lung cancer (C), but lacks red flag symptoms at this time. Pulmonary embolism (D) typically occurs in patients with venous stasis, hyper coagulable state, and vascular wall injury. Pulmonary fibrosis (E) would present with sign of restrictive lung disease (e.g., shallow breaths with rapid expiration)

A 4-month-old febrile infant presents with loss of appetite, irritability, seizures, focal sensory and motor deficits, and an acute petechial rash. On physical examination, a bulging fontanelle is noted; rectal temperature is 102.8°F. What study would be most important in this child's evaluation? Answer Choices 1 CBC 2 Urinalysis 3 Chemistry panel 4 Serum glucose 5 CSF analysis

CSF analysis The clinical picture is suggestive of meningitis, which is diagnosed after obtaining and evaluating the cerebrospinal fluid (CSF). Although a bulging fontanelle is an indication of increased intracranial pressure (ICP), it is not an automatic contraindication for a lumbar puncture (LP) unless the patient displays additional signs of increased ICP. Tonic seizures would be an example of an indication of increased ICP. Even though this child has had seizures, an LP to run a CSF evaluation is still the best answer. This will be the most important study and diagnostic tool for this child. It is likely that a CT or MRI would be done to look at the level of swelling in the brain, and possibly determine the level of ICP. It would be likely for either of these tests to be done prior to a lumbar puncture. Normally, meningitis causes fever, inactivity, and mental status changes; however, these symptoms are often hard to detect in young children. In infants, signs and symptoms may include appearing to be slow or inactive (lack of alertness), irritable, vomiting, or feeding poorly. When CSF infections are suspected, blood should be obtained for a CBC, general chemistry panel, and culture. However, obtaining CSF analysis is the most important.

A 66-year-old woman is brought to the emergency room by her daughter; the patient is becoming incoherent and confused. She was in good health until early that morning when she reported having a headache. She is not a known hypertensive or diabetic, and she is not on any medications. On examination, she is well-nourished, has a fever, and is disorientated to time, space, and person. Brudzinski's sign is positive. What would you expect to find on examination of her cerebrospinal fluid (CSF)? Answer Choices 1 CSF pressure 250 mm H2O, cells 2000/uL with predominant neutrophils 2 CSF pressure 150 mm H2O, cells 500/uL with predominant lymphocytes 3 CSF pressure 150 mm H2O, cells 250/uL with predominant lymphocytes 4 CSF pressure 150 mm H2O, cells 150/uL with predominant lymphocytes 5 CSF pressure 250 mm H2O, predominantly red blood cells

CSF pressure 250 mm H2O, cells 2000/uL with predominant neutrophils The correct response is CSF pressure 250 mm H2O, cells 200/µL with predominant neutrophils. The clinical picture is suggestive of acute bacterial meningitis; this is an inflammatory disease involving the arachnoid layer of the meninges and the cerebrospinal fluid (CSF). It is a medical emergency and antibiotic treatment should be started immediately. Patients with acute bacterial meningitis usually have an increased CSF pressure, cells 500 - 5000/uL, and neutrophils are the predominant cells. The CSF glucose is decreased and the protein level is usually over 100 mg/dL. A Gram stain should be done on the CSF smear. Serologic latex agglutination tests for meningococcus, H. influenzae type B, and pneumococcal antigens in the CSF can provide a diagnosis rapidly. The CSF should be cultured. Other relevant cultures (e.g., blood, nasopharyngeal and respiratory secretions, urine, and skin lesions) should also be done. A lumbar puncture should be done to obtain CSF samples after a CT scan has excluded a mass lesion. Normal values of CSF parameters are as follows: A. CSF pressure 100 - 200 mm H2O B. Cells 0 - 3/uL C. Predominant cell type - lymphocyte D. Glucose 50 - 100 mg/dL E. Protein 20 - 45 mg/dL The following are the common bacterial causes of acute meningitis in the various age groups: Escherichia coli and Streptococci agalactiae are found in neonates and infants. Haemophilus influenzae type B, Streptococcus pneumoniae, and Meisseria meningitidis are found in early childhood. Neisseria meningitidis is found in adolescents and young adults. Meningococci exist in the nasopharynx of about 5% of the population and can spread by respiratory droplets through kissing, sneezing, or coughing. They tend to cause epidemics among closed populations (e.g., military barracks and boarding schools). Streptococcus pneumoniae is the most common cause of acute meningitis in people over 25 years of age. Especially at risk are alcoholics and persons with chronic otitis, sinusitis, pneumococcal pneumonia, sickle cell disease, and asplenia. Staphylococcus aureus is a common cause of acute meningitis in immunocompromised patients after penetrating head wounds or neurosurgical procedures. Gram-negative bacteria, such as Escherichia coli, Klebsiella, and Enterobacter, are common causes of nosocomial meningitis and meningitis after neurosurgical procedures, trauma, and genitourinary manipulation in the elderly. Listeria monocytogenes meningitis is common in patients with immunosuppression, carcinomas, and those receiving corticosteroids or cytotoxics. Patients usually present with headache, fever, chills, photophobia, vomiting, seizures, confusion, drowsiness, stupor, and coma (especially in the elderly). There may be a history of a prodromal upper respiratory tract infection, especially in patients with pneumococcal meningitis. Dehydration and vascular collapse due to adrenal insufficiency may lead to shock in Waterhouse-Friderichsen syndrome in meningococcal septicemia. Complications include cranial nerve palsies, cerebral infarction, convulsions, and mental retardation. On examination, patients may be febrile and have a stiff neck. A petechial or purpuric rash suggests meningococcal meningitis. Patients with increased intracranial pressure may have papilledema. The following signs may be positive: Brudzinski's sign ( flexion of the neck in a supine patient results in involuntary flexion of the hips and knees), and Kernig's sign (attempts to extend the knee from the flexed thigh position are met with strong passive resistance). There also may be evidence of extracranial infections (e.g., sinusitis, otitis, and mastoiditis). The differential diagnoses include viral meningitis, Lyme disease, tuberculous meningitis, and cerebral hemorrhages; all have different CSF findings. In tuberculous meningitis, the presentation is subacute or chronic and the CSF protein and pressure are increased. Lymphocytes are the predominant cell types with cell counts of 100 - 700/uL. In Lyme disease, the CSF protein is normal or increased; lymphocytes are the predominant cell types with cell counts of 0 - 500/uL. In viral meningitis, CSF protein and pressure may be normal or increased; lymphocytes are the predominant cell types with cell counts of 100 - 200/uL. In cerebral hemorrhages, the CSF is bloody, the pressure is increased, and red blood cells are the predominant cell types. The CSF glucose is within the normal range. Penicillin G is used for Streptococcus pneumoniae, and Neisseria meningitidis is sensitive to the drug. Ceftriaxone and chloramphenicol are used for Haemophilus influenzae and Gram-negative bacilli. Oxacillin is used for Staphylococcus aureus. Ampicillin is used for Listeria monocytogenes. Early use of corticosteroids, such as dexamethasone, may avert the development of major neurologic sequelae; they do so by inhibiting the release of pro-inflammatory cytokines triggered by bacterial fragments (which result from antibiotic activity).

A 22-year-old man presents after being hit on the right temple by a baseball 1 hour ago. He lost consciousness for 2 minutes after the impact, but he did not seek immediate medical attention. He has no other significant medical problems and family history is unremarkable. On presentation, he is comatose, with no response to painful stimuli. There is anisocoria with fixed dilatation of the right pupil. Vital signs are as follows: BP 100 / 66 mm Hg, RR 14 rpm, HR 62 bpm, temperature 36.6 °C (97.9 F). What is the most appropriate next step to confirm the suspected diagnosis? Answer Choices 1 MRI of the brain 2 Perform a lumbar puncture 3 Burr hole craniotomy 4 Computed tomography of the brain 5 Observation only

CT The signs and symptoms described in this clinical scenario are consistent with an acute epidural hematoma. This lesion's pathophysiology is a dural tear with laceration of an artery or, less commonly, a vein. The most commonly injured vessel is the middle meningeal artery (66% of cases), but lesions of the anterior ethmoidal artery and the dural sinuses may also occur. About 85-95% of these cases are associated with a fracture of the skull. However, there is little injury to the underlying intracranial components; because of this, the prognosis is good. Other findings, such as intracranial air or vascular malformations, may also be present. Due to the progressiveness of this pathology if not treated, immediate diagnosis and surgical intervention is required. The most commonly used method to make this diagnosis to date is a non-contrast CT of the head. Not only will this help locate the hematoma, this will also help visualize any other abnormalities that may be present, such as skull fractures. A CT scan would also enable the patient to be observed during the entirety of the procedure to ensure his stability and safety. MRI of the brain may be considered appropriate, however should not be used in any patients who are unstable. Due to the patient being comatosed with no response to painful stimuli, this would not be an acceptable choice. Burr hole craniotomy is used primarily in the treatment approach in a rapidly deteriorating patients with signs of imminent herniation. Although the patient is in a comatose state, his vitals are acceptable. Knowing if there is indeed an epidural hematoma as well as where it is located will both be accomplished via a CT of the head. A lumbar puncture would not be an acceptable choice in this patient scenario and in fact may result in detrimental results. Observation only would also most likely have devastating circumstances

A 58-year-old woman with a past medical history of hypertension, hyperlipidemia, and breast cancer is being evaluated in the emergency room after presenting with a 2-hour history of acute-onset severe left-sided pleuritic chest pain. The pain is associated with feelings of anxiety, hemoptysis, shortness of breath, and nausea. She "feels warm" but denies other complaints. She endorses a 30 pack-year smoking history. Upon physical exam, she is found to be febrile, normotensive, tachypneic, diaphoretic, and in acute painful distress. There is perioral cyanosis and a pleural friction rub to the left lung fields; the remainder of the exam is normal. A bedside electrocardiogram revealed the following image. Question What is the next correct step in the management of this patient? Answer Choices 1 Order a CT angiography of the chest 2 Refer the patient for an echocardiogram 3 Obtain magnetic resonance imaging of the chest 4 Prescribe an oral proton-pump inhibitor 5 Begin recombinant tPA infusion

CT angio This patient's presentation suggests a pulmonary embolism. EKG findings include an S1Q3T3 pattern, sinus tachycardia, T wave inversion in leads V1 - V3, Right Bundle Branch Block, and low amplitude deflections. Computed tomography angiography (CTA) is the initial imaging modality of choice for stable patients with suspected pulmonary embolism.The American College of Radiology (ACR) considers chest CTA to be the current standard of care for the detection of pulmonary embolism. The echocardiogram generally has limited accuracy in the diagnosis of pulmonary embolism. Few investigators have reported the feasibility of MRI in the evaluation of pulmonary embolism. However, the role of MRI is mostly limited to the evaluation of patients who have impaired renal function or other contraindications for the use of iodinated contrast material. Prescribing a proton pump inhibitor is not appropriate at this time, as the clinical presentation suggests pulmonary embolism. Thrombolytic therapy is not recommended for most patients. The role of thrombolytic therapy in the management of acute pulmonary embolism is controversial and has not demonstrated benefits in terms of reduced mortality rates or earlier resolution of symptoms when currently compared with heparin. The currently accepted indications for thrombolytic therapy include hemodynamic instability, such as systolic blood pressure less than 90 mmHg or systolic blood pressure drop of greater than 40 mmHg from baseline for at least 15 minutes, or right ventricular dysfunction demonstrated on echocardiogram. Thrombolytic therapy should not be used in patients with major contraindications due to bleeding risks.

A 15-month-old toddler presents with sudden onset of generalized tonic and clonic convulsions for the last 30 minutes. There is no history of trauma, fever, vomiting, or irritability prior to the onset of convulsions. This is the first episode of seizure, and there is no history of convulsions in the family. Birth history, neonatal period, and developmental milestones are normal. Question After initial stabilization by securing the airway, oxygenation, and controlling the acute convulsion, what is the investigation of choice for the above child? Answer Choices 1 Cerebrospinal fluid examination 2 CT scan of the head 3 Electroencephalogram 4 Cranial ultrasonography 5 MRI scan of the head

CT scan of head CT scan of the head is the best diagnostic imaging study in a previously asymptomatic child who presents with status epilepticus, particularly if there is suspicion of intracranial hemorrhage, a space-occupying lesion, or midline shift. It is a non-invasive and rapid procedure and is widely available. It has a short imaging time and requires less sedation in young children. Though CT imaging is largely replaced by MRI, it still remains the most sensitive modality to detect intracranial hemorrhage and calcifications during the first 24 hours. Other indications are head injury, hypoxic ischemic encephalopathy, inflammatory disorders like neurocysticercosis, CNS tuberculosis, basal exudates, pyogenic abscess, and malformations like hydrocephalus and porencephaly. Myelination, posterior fossa, and brain stem structures are not well visualized by CT scan. As the child presented with his 1st episode of convulsion with status epilepticus without any prior symptoms, a CT scan of the head would be the first choice of investigation. Cranial ultrasonography (USG) is mainly used in the diagnosis and follow-up of newborns with intraventricular hemorrhage and for assessment of ventricular size. It can be performed through the open anterior fontanel, and it can also be performed at the bedside of the patient. It can visualize the ventricles, periventricular tissue, and a part of cerebral cortex. As the above child is 15 months old and the anterior fontanel is either closed or is very small at this age, it cannot be performed in this patient. Electroencephalogram (EEG) is commonly used to confirm a clinically doubtful convulsion, to characterize the type of convulsion, to locate an epileptic focus, or to distinguish a simple febrile convulsion from epilepsy. It can help to distinguish between a seizure and a non-seizure state, such as a fainting spell, hypoxic episode, or a breath-holding spell. It is especially helpful in the diagnosis of absence attacks, herpes encephalitis, and myoclonic seizures. It is not the investigation of first choice in the above patient. Lumbar puncture (LP) and examination of cerebrospinal fluid (CSF) is usually indicated for the diagnosis of conditions like meningitis, encephalitis, subarachnoid hemorrhage, demyelinating illnesses, and slow virus infections of the CNS. As the above child was asymptomatic before the episode of convulsion, lumbar puncture is not the investigation of first choice in this child. MRI scan of the head can demonstrate myelination abnormalities, edema, and infarcts more clearly as compared to CT. MRI is the study of choice for evaluating the spine, spinal cord, and spinal canal in children. It is more sensitive than CT scan for most brain tumors involving the brainstem, posterior fossa, and spinal column. It is not the investigation of first choice for acute intracranial hemorrhage during the first 24 hours. Also, it is not widely available for use during emergency.

A 28-year-old man presents with a history of headache, vomiting, and seizure. On examination he is febrile; pulse is 110/min; and blood pressure is 110/70mmHg. On central nervous system (CNS) examination, he appears alert; he is oriented to time, place, and person. He has aphasia and weakness of the right hand and leg. You suspect brain abscess. Question What initial test could confirm your diagnosis? Answer Choices 1 Magnetic Resonance Imaging (MRI) of brain with diffusion weighting 2 Computed Tomography (CT) scan of brain with contrast 3 CT scan of the brain without contrast 4 Stereotactic aspiration and examination of abscess fluid 5 Ultrasonography

CT with Contrast!! The correct answer is CT scan of brain with contrast. It is an excellent tool for establishing diagnosis and can demonstrate evidence of a ring-enhancing lesion in a well-defined abscess, as well as features of cerebral edema in the stage of cerebritis1. CT aids in determining the location of the abscess, its size, number of abscesses, mass effect, shifts, and the presence of intraventricular rupture. It provides information regarding the cause, and the paranasal sinuses and mastoids are also imaged concomitantly1. CT scan of the brain without contrast does not form a ring around the lesion, making the diagnosis difficult. Hence, it is not the right choice. Although magnetic resonance imaging (MRI) obtained with diffusion weighting may be more sensitive in the differentiation of an abscess from other cystic brain lesions, as well as in detection of the cerebritis stage, it may not be useful in an acutely ill patient, and routine MRI is not recommend to diagnose patients with a suspected brain abscess1. Stereotactic aspiration and examination of abscess fluid is used to make a definitive microbiological diagnosis by collecting pus from the abscess for testing. It is not used as an initial test to establish the diagnosis. Ultrasonography is not used for diagnosing a brain abscess in adults because the skull causes bone artifact. In children with an open anterior fontanelle, a sonogram can be used to diagnose an abscess1.

A 30-year-old woman has a family history of MEN type 2. Her doctor becomes quite concerned when he notices a nodule on her thyroid. Further work up is done, and she is found to have medullary carcinoma of the thyroid. What tumor marker is most likely to be found in this patient? Answer Choices 1 ADH 2 HCG 3 CA 125 4 Alpha feto-protein 5 Calcitonin

Calcitonin Explanation Calcitonin is normally secreted by the parafollicular cells (C cells) of the thyroid gland. It lowers serum calcium. Calcitonin can be seen with medullary cancer of the thyroid. ADH is antidiuretic hormone. It is normally secreted by the posterior pituitary. It is involved in water homeostasis. Ectopic ADH production can sometimes be seen with lung cancer, intracranial neoplasms, and a few other tumor types. This is the syndrome of inappropriate secretion of ADH (SIADH). HCG is human chorionic gonadotropin. It is normally produced during pregnancy. It can be seen in gestational trophoblastic disease, choriocarcinoma, and germ cell tumors. CA 125 can be found with some ovarian cancer, uterine cancer, pancreatic cancer, lung cancer, breast cancer, and colon cancer. CA 125 can also be seen in some normal conditions, such as menstruation, pregnancy, and other medical conditions. Alpha-fetoprotein is a tumor marker that can be seen in hepatocellular carcinoma, some germ cell tumors, and some other malignancies and conditions.

A 26-year-old woman at 32 weeks gestation is brought via ambulance to the ER with a 30-minute history of convulsions; they occurred while she was at work. Her vital signs include a blood pressure of 160/95 mm Hg, heart rate of 84 beats/min, and respiratory rate of 22 breaths/min. On physical examination, she is noted to have lower extremity edema and is hyperreflexic. She was treated with intravenous magnesium sulfate at a rate of 2 g/hour for 12 hours, and she is now is hyporeflexic; her respirations are decreased. What would you prescribe this patient? Answer Choices 1 Diazepam 2 Oxygen 3 Continue magnesium 4 Calcium 5 Potassium

Calcium Explanation The clinical picture is suggestive of eclampsia. In this instance, she became hyporeflexic and her respirations decreased, indicating possible magnesium toxicity. This can be reversed by giving calcium gluconate. Diazepam does not reverse the effects of magnesium toxicity. Oxygen will also not reverse the effects of magnesium toxicity. Continuing the magnesium would worsen the condition. Potassium does not reverse the effects of magnesium toxicity.

A 45-year-old woman presents as a new patient. She was recently seen in the emergency department for right flank pain, and a CT scan revealed a right-sided ureteral stone. The stone was 4mm, a passable size, and she was sent home with analgesics and advised to hydrate well and strain her urine. She was straining her urine and noticed a small, dark fleck. She brought the sediment to the urologist's office to undergo a stone analysis, as this is her first episode of a renal or ureteral stone. Question What is the most likely composition of this stone? Answer Choices 1 Uric acid 2 Struvite 3 Cystine 4 Magnesium 5 Calcium

Calcium Explanation The correct answer is calcium. Calcium stones are the most common type of renal calculi and can be further characterized as either calcium oxalate or calcium phosphate stones. Calcium oxalate stones make up about 60% and calcium phosphate stones make up about 20%, which results in calcium stones being about 80% of stones overall. Geography, fluid intake, and diet can all influence stone formation, but metabolism and genetics can also play a role. Absorptive hypercalciuria, renal hypercalciuria, and resorptive hypercalciuria can all result in calcium stone formation. Hyperuricosuria, gout, hyperoxaluria, and hypocitraturia are all other causes of calcium calculi. Uric acid is the not the correct answer, as this is not the most common type of renal calculus. Uric acid stones make up about 10% of stones in the United States. Hyperuricosuira and/or a urinary pH less than 5.5 are the 2 most common causes of uric acid stones. Gout, increased turnover of nucleic acids (such as in polycythemia or psoriasis), increased purine intake, and alcohol consumption are all causes of hyperuricosuria and can therefore put a patient at risk for uric acid stones. Struvite is not the correct answer, as this is not the most common type of renal calculus. Struvite stones make up about 7% of total renal calculi. Urinary tract infections secondary to the presence of urea splitting organisms can result in the formation of struvite stones. Klebsiella, Proteus, Staphylococcus, and Pseudomonoas produce urease. Urease breaks down urea and aids in the formation of ammonia. The ammonia then undergoes hydrolysis, which results in alkaline urine and reduced solubility of struvite, as well as urine that is supersaturated in struvite. Cystine is not the correct answer, as this is not the most common form of renal calculi. Cystine stones make up about 3% of total renal calculi. Cystine stones form due to an autosomal recessive disorder in the metabolism of cystine, which leads to cystinuria. Once the urinary saturation of cystine is more than 250 mg/L, cystine stones can start to form. Magnesium is not the correct answer, as this is not a type of renal calculi. In fact, magnesium has been known to be preventative of stone formation and is, therefore, a component in a lot of renal calculi prevention measures. Hypomagnesuria, usually dietary in nature, is a known risk factor for renal calculi formation. Maintaining urine magnesium about 50 mg/day is preventative.

Which of the following is appropriate in the pharmacologic management of patients with hypoparathyroid tetany? A Aggressive IV hydration B Cinacalcet hydrochloride PO C Calcium gluconate IV D Pamidronate IV E Calcitonin IV

Calcium Gluconate IV The correct choice is C, calcium gluconate IV. In severe hypocalcemia, replacement calcium must be started promptly, as well as airway maintenance and magnesium and vitamin D replacement, as necessary. Choices A, aggressive hydration, B, cinacalcet hydrochloride, D, pamidromate, and E, calcitonin, are all possible treatment options for patients with hypercalcemia.

Hypocitraturia is associated with: Answer Choices 1 Cystine nephrolithiasis 2 Uric acid nephrolithiasis 3 Struvite nephrolithiasis 4 Calcium oxalate nephrolithiasis

Calcium oxalate nephrolithiasis Explanation Various substances have been proposed to inhibit renal stone formation, and in this regard, citrate is well studied. Hypocitraturia (<470mg/24 hours) is present in up to 60% of patients with nephrolithiasis. Hypocitraturia may have secondary causes, including chronic diarrheal states, renal tubular acidosis, hypomagnesemia, urinary tract infection, and thiazide-induced hypokalemia. Hypocitraturia is associated with idiopathic calcium oxalate stone formation, and after its correction, there is a notable reduction in stone forming events.

A 55-year-old patient presents with symptoms of polyuria and is worried that he may have diabetes mellitus. His past medical history includes hypertension and lung cancer. His family history is negative for diabetes mellitus. He works as an accountant and has no history of head trauma. He has an average diet and fluid intake. His fasting plasma glucose is 110 mg/dL and his hemoglobin A1c is 5%. What is the most likely cause of his polyuria? A Cancer related diabetes insipidus B Type 2 diabetes mellitus C Nephrogenic diabetes insipidus D Familial hypothalamic diabetes insipidus E Primary polydipsia

Cancer related Diabetes Insipidus

A 52-year-old woman presents with vaginal discharge that is white curd-like in appearance but is not malodorous. She has a 19-year history of obesity and poorly controlled type 2 diabetes mellitus. Microscopic examination of the discharge with 10% potassium hydroxide demonstrates filaments and spores. Which of the following is the most likely etiologic agent? A Candida B Gardnerella C Lactobacillus D Staphylococcus epidermidis E Trichomonas vaginalis

Candida A This case has several clues pointing to a Candida infection, including the fact that diabetes mellitus can predispose patients to Candida infections and the presence of the white curd-like discharge that is not malodorous. In Trichomonas vaginalis, the discharge is malodorous and yellow-green in color. With Gardnerella, there is also a malodorous discharge. Lactobacillus is the predominant, normal microorganism of the vagina and keeps it slightly acidic to help reduce the growth of potentially harmful organisms. Staphylococcus epidermidis is also part of the natural flora of the vagina

While doing routine newborn exams in the nursery on full term infants, you note a 4 - 5 cm soft, raised, round, nontender, bruised swelling overriding the left occipitoparietal suture on a 30-hour-old male infant. He was born by spontaneous vaginal delivery with rupture of membranes at home. He has been feeding well. On the rest of his exam, his red reflex is present bilaterally, lungs, heart and abdominal exams seem normal. Rest of his skin is pink, warm and dry, without cyanosis. What is the most likely diagnosis? Answer Choices 1 Cephalohematoma 2 Hydrocephalus 3 Caput succedaneum 4 Plagiocephaly 5 Craniosynostosis

Caput succedaneum Explanation Caput succedaneum represents an accumulation of serosanguineous subcutaneous fluid. This fluid is thus not bound by suture lines, but may have poorly defined margins and extend across midline and suture lines. It is caused by mechanical trauma of the presenting portion of scalp pushing through a narrowed cervix. It is more likely to be seen after a prolonged or difficult delivery especially after prolonged rupture of membranes. This fluid collection will resorb within a few days, thus no treatment is necessary. Jaundice might result with very large caputs. Cephalohematoma is due to subperiosteal hemorrhage involving the outer table of a cranial bone. Unlike the caput, the swelling does not extend across a suture line. This may result from an instrumented delivery causing rupture of blood vessels between the skull and periosteum. Healing by resorption can take several weeks to months with some peripheral calcification. Large cephalohematomas can contribute to hyperbilirubinemia. Hydrocephalus refers to the presence of excess cerebrospinal fluid (CSF) within the ventricles and is caused by either excessive production of CSF or blockage of CSF circulation. Clinically, this presents with an increasing head circumference, full fontanelles, irritability, poor feeding and vomiting. There may be sunset eye sign with the eyes deviated downward revealing the upper sclera. Plagiocephaly is an asymmetry of the cranial vault caused by constant pressure from positioning on the dependent side. It disappears as the baby becomes more active and spends less time in one position. Craniosynostosis is the premature closure of one or more sutures altering the shape of the head. This can be differentiated into primary craniosynostosis, which has no underlying brain abnormality or metabolic defect, but may be related to intrauterine constraints and secondary craniosynostosis that may be due to a structural cerebral abnormality or related to other conditions such as hyperthyroidism, severe anemia, or a metabolic condition. Some genetic abnormalities can also cause craniosynostosis.

A 65-year-old man presents with abdominal pain, vomiting, and weakness. On examination, he is cachectic, pale, and jaundiced. A walnut-sized mass is palpable in the right hypochondrium. What is the most likely diagnosis? Answer Choices 1 Carcinoma of head of pancreas 2 Acute or chronic cholecystitis 3 Fibrolipoma 4 Hydatid cyst of the liver 5 Primary biliary cirrhosis

Carcinoma of the head of the pancreas Carcinoma of the head of pancreas is a common 'duodenal C-loop cancer'. Seen mostly in men, it is usually a ductal adenocarcinoma. It causes obstructive jaundice, weight loss, itching, and upper abdominal pain. There may be diabetes, bleeding problems, malabsorption syndromes, and LFT showing obstructive jaundice. This carcinoma has a better prognosis than cancer of the body and tail of the pancreas when resection is possible. Courvoisier's law states that when the gall bladder is palpable (but not tender) in the presence of jaundice, the cause is usually cancer. Cholecystitis is a diagnosis of 'female, fat, fertile, forty, and flatulent'. It does occur in men. Jaundice may or may not be present. Vomiting usually brings relief to the upper abdominal pain. Muscle guarding and rebound tenderness is present. A fibrolipoma over the right hypochondrium may be a coincidental finding, but would not be the cause of the overall clinical features of a malignancy. A hydatid cyst results from an E. Granulosus infection, and cysts may form in the liver, lung and other organs. It may become palpable in the right hypochondrium and cause pressure symptoms. Migrant workers and immigrants from endemic areas may carry this infection. Primary biliary cirrhosis is an autoimmune disease that destroys the liver structure and system; it is mostly seen in women over the age of 40. There is itching, xanthomata, and hepatosplenomegaly.

A 45-year-old man presents with multiple symptoms, including a 2-year history of chronic fatigue, headaches, and joint pain. He was finally prompted to seek care when he noticed an increase in both his hat and shoe size. His past medical history is unremarkable, with no known medical conditions; there is no history of surgery, and he does not take any medications. He lives with his girlfriend, and he works as a building contractor. On physical exam, his facial features appear "coarse"; he has a wide nose, and macroglossia is noted. The remainder of his physical exam is normal, and no observable abnormalities are noted on the patient's head or feet. Several tests are performed and significant findings include: Growth hormone: Elevated Oral glucose tolerance test (OGTT): Elevated glucose levels Insulin-like growth factor-1 (IGF-1): Elevated Question After treatment of this patient's condition is initially completed, for what complication of this condition should he be monitored? Answer Choices 1 Cardiovascular disease 2 Hypogammaglobulinemia 3 Multiple myeloma 4 Osteoporosis 5 Parkinson disease

Cardiovascular Disease Explanation This patient is presenting with acromegaly, a rare disorder of excess growth hormone (GH), most often caused by a GH-secreting pituitary adenoma. Because the changes are insidious, patients may take years to present with symptoms and be diagnosed. The patient and close family members may not notice the acral skeletal growth enlargement, unless it affects the fit of shoes, rings and/or hats. Other possible signs and symptoms of acromegaly include fatigue, headaches, visual field deficits, hypertension, coarse facial features, carpal tunnel syndrome, and joint pain. Treatment may include surgical resection of the adenoma, medications or radiation. Patients with acromegaly are at risk for early death; the primary cause of death is cardiovascular disease. Patients with acromegaly may develop hypertension, cardiomyopathy, and hypertrophy. Hypogammaglobulinemia is an immune disorder in which the affected patient has reduced levels of immunoglobulins and is at risk for infection. Hypogammaglobulinemia, and other immune system disorders, are not identified as acromegaly complications. Multiple myeloma is a plasma cell cancer that affects bone marrow. While acromegaly does affect bone growth, it has not been linked with bone malignancy. Osteoporosis is the demineralization of bone structure, leading to bone weakness and fracture. It is not considered a complication of acromegaly. Parkinson disease is a chronic, progressive neurologic disorder associated with tremor, rigidity, abnormal gait, and mental changes, such as dementia and depression. Patients with acromegaly are not known to be at a higher risk of Parkinson disease.

A 24-year-old man presents with gradual onset of scrotal pain. The pain is constant, and it lessens with elevation of the testicles. The patient developed a fever this morning. He has been voiding small amounts frequently, and he is also experiencing dysuria. The patient has a negative past medical history and past surgical history. He does not smoke, drink alcohol, or abuse illicit drugs. He is currently sexually active with a new partner. Examination reveals edema of the left testicle; there is tenderness to palpation. Cremasteric reflex is positive. Question What is the most appropriate intervention at this time? Answer Choices 1 Biopsy of the testicle 2 Testicular ultrasound 3 Insertion of foley catheter 4 Ciprofloxacin and doxycycline 5 Ceftriaxone and doxycycline

Ceftriaxone and doxycycline The patient is most likely suffering from acute epididymitis, so he should be treated with a single dose of IM ceftriaxone along with a 10-day course of oral doxycycline. Epididymitis is inflammation of the epididymis, the tube that connects the testicle to the vas deferens. Epididymitis is typically the result of infection. In younger, sexually active man, it is usually caused by Chlamydia trachomatis or Neisseria gonorrhoeae. In older man, epididymitis is associated with Gram-negative rods. Given the patient's age and sexual history, he should be treated with ceftriaxone, which will cover gonorrhea, as well as doxycycline, which is active against chlamydia. Biopsy of the testicle is an incorrect response. The patient has no identifiable mass on examination, only testicular edema. A testicular ultrasound is not indicated at this time. Insertion of a foley catheter is an incorrect response. Ciprofloxacin and doxycycline is an incorrect response. If the patient had been older and did not have sexual risk factors for sexually transmitted epididymitis, then ciprofloxacin might have been required to cover for Gram-negative rods. However, in this case, ceftriaxone and doxycycline will better cover the likely causative agents.

A 40-year-old woman presents with a 1-month history of having at least 3 pasty, malodorous bowel movements each day; the bowel movements are hard to flush. The patient is not taking any medications. She states that her father died of a similar illness. On examination, she is of short stature, has mild pallor, and pedal pitting edema. Her BP is 110/60 mm Hg, and she has mild bowing of her lower limbs. Laboratory analysis reveals elevated alkaline phosphatase. Question What is the most likely diagnosis? Answer Choices 1 Celiac disease 2 Whipple's disease 3 Short bowel syndrome 4 Tropical sprue 5 Lactose intolerance

Celiac disease Celiac disease is a hereditary disorder caused by gluten intolerance. Gluten is found in wheat, rye, barley, and oats. Patients may be asymptomatic or may present with diarrhea, abdominal discomfort, distention, and steatorrhea (stools are pale, malodorous, and difficult to flush because they float on the toilet water). Patients may have anemia from iron and folate deficiency, osteomalacia and bone pains from calcium deficiency, and edema from hypoproteinemia. Diagnosis is confirmed by a small intestine mucosal biopsy which shows a flat mucosa due to villous atrophy, and by subsequent improvement on a gluten-free diet. Specific treatment includes a gluten-free diet. Whipple's disease mainly affects men; it is caused by the bacterium Tropheryma whippelii. It is a multisystemic disease that affects the small intestines, joints, brain, heart, and eyes. Patients can present with diarrhea, steatorrhea, abdominal pain, weight loss, and joint pains. On examination, patients may be pale with lymphadenopathy. Histological examination of a small bowel mucosal biopsy reveals PAS-positive foamy macrophages. Treatment is with trimethoprim-sulfamethoxazole or chloramphenicol. Short bowel syndrome is usually the result of surgical resection of the intestines or a jejunoileal bypass. The malabsorption is a result of inadequate absorptive surface. Malabsorption of vitamin B12 results in paraesthesias; malabsorption of calcium results in bone pain and carpopedal spasms. Tropical sprue is an acquired disease that affects both visitors and natives of tropical areas (e.g., the Caribbean and South India). Its etiology is unknown. Patients usually present with diarrhea and weight loss; they report that their stools are soft and bulky (steatorrhea). They may also develop deficiencies of folate and cobalamin. Stool microscopy should be done to look for cysts and trophozoites. Histological examination of a small bowel mucosal biopsy aids in making the diagnosis. Treatment is with tetracycline or oxytetracycline. Lactose intolerance results from a deficiency of lactose which is a disaccharidase in the mucosal cells of the small intestine; it splits the disaccharide lactose into glucose and galactose. Patients complain of borborygmi, flatulence, nausea, abdominal cramps, pain, and diarrhea after ingesting lactose-containing food (e.g., milk). Treatment includes following a lactose-free diet.

A 75-year-old man is involved in a motor vehicle accident and strikes his forehead on the windshield. He complains of neck pain and severe burning in his shoulders and arms. His physical examination reveals weakness of his upper extremities. What type of spinal cord injury does this patient have? A anterior cord syndrome B central cord syndrome C Brown-Séquard syndrome D complete cord transection E cauda equina syndrome

Central Cord Syndrome the central cord syndrome involves loss of motor function that is more severe in the upper extremities than in the lower extremities, and is more severe in the hands. There is typically hyperesthesia over the shoulders and arms. Anterior cord syndrome presents with paraplegia or quadriplegia, loss of lateral spinothalamic function with preservation of posterior column function. Brown-Séquard syndrome consists of weakness and loss of posterior column function on one side of the body distal to the lesion with contralateral loss of lateral spinothalamic function one to two levels below the lesion. Complete cord transection would affect motor and sensory function distal to the lesion. Cauda equina syndrome typically presents as low back pain with radiculopathy.

A 38-year-old man presents with fatigue, dry mouth, and passing large amounts of urine. He describes his urine as light in color and non-odorous. He says he never had this before. He has always been healthy, and he has never been hospitalized before. During a routine pre-employment screening, his serum electrolytes were investigated; his sodium level was slightly above normal. At that time, he was reassured that it was due to slight dehydration. He was advised to repeat the test after 3 months. When he repeated it about a week ago, it showed more of an increase. He takes vitamin supplements and sometimes uses energy drinks. His blood glucose is normal. To avoid going to the bathroom too often, he tries to restrict fluid intake to a minimum, but he says it has never helped. Question DDAVP (desmopressin) injection led to decrease in urine volume and increase in urine specific gravity. What is the most likely diagnosis? Answer Choices 1 Central diabetes insipidus 2 Nephrogenic diabetes insipidus 3 Diabetes mellitus 4 Psychogenic polydipsia 5 Drug-induced polyuria

Central diabetes insipidus Explanation Diabetes insipidus is a disease in which ADH (antidiuretic hormone) from the posterior pituitary is either deficient or the kidneys become insensitive to it. In this patient, since there was a decrease in urine output after desmopressin injection (i.e., kidneys are sensitive to ADH), the diagnosis is central diabetes insipidus. ADH is secreted from posterior pituitary and works on distal renal tubules on specific receptors to reabsorb water, and this occurs in response to osmoreceptors to help keep the body in balance. Symptoms include polyuria, polydipsia, and decreased urine specific gravity. Causes: Central; due to tumor, anoxia, or hemorrhage. Nephrogenic; due to the effect of drugs of which lithium is a well known reason. Differential diagnoses: Diabetes mellitus Psychogenic polydipsia Workup: First, exclude diabetes mellitus by random blood sugar and then exclude psychogenic polydipsia by water deprivation test (in this case scenario, the patient did it himself). Second, perform a desmopressin test to differentiate nephrogenic from central DI. If it is positive, do MRI to exclude brain tumors. Treatment: In mild cases, adequate hydration is enough. In central, give desmopressin. In cases of psychogenic polydipsia, psychiatric consultation is required. In drug-induced cases, stop the offending drug.

A patient presents to the office. His gait is wide-based, staggering, and unsteady. He appears to have difficulty with turns and hangs on to his wife as he crosses your waiting room. What type of gait does he have? Answer Choices 1 Scissors gait 2 Steppage gait 3 Sensory ataxia 4 Cerebellar ataxia 5 Myopathic gait

Cerebellar Ataxia: Cerebellar Ataxia= The gait characterized by cerebellar ataxia is associated with a disease of the cerebellum or its affiliated tracts. The gait is staggering, unsteady, and wide-based. The patient has difficulty with turns, and cannot stand steady when feet are together and eyes closed (Romberg's sign). Scissors gait is apparent when each leg is advanced slowly and the thighs tend to cross each other. The gait is stiff and the steps are short. This is associated with bilateral spastic paresis of the legs. Steppage gait is associated with a foot drop, often secondary to lower neuron disease. The patient drags his foot, or lifts the knee high, and slaps the foot against the ground. This gait may be unilateral or bilateral. Sensory ataxia gait is unsteady and wide-based. The feet are thrown forward, out, then down. There is a distinct 2-slap sound, secondary to the heel striking first then the forefoot. This is associated with polyneuropathy or posterior column damage. A myopathic gait is where there is weakness on one side. This displays a a drop in the pelvic area on the contralateral side of the pelvis while walking (Trendelenburg sign). When there is a bilateral weakness, it will display as a dropping of the pelvic girdle on both sides of the body while walking that results in a waddle. This is seen in patients whom are afflicted with myopathies, such as muscular dystrophy.

A 52-year-old woman present to her gynecologist's office with a 6-month history of hot flashes, night sweats, mood swings, and vaginal dryness that interferes with intercourse. The symptoms seem to be worsening and are now interfering with her productivity at work and with her relationships with family and friends. Her LNMP was 8 months ago; the patient notes: "after that period, they just stopped." She denies any history of tobacco use, and she drinks one glass of red wine daily. She has a history of coronary heart disease (CHD), with stent placement 2 years ago. She recently read an article about hormone replacement and would like you to prescribe this for her. Question What is true regarding hormone replacement for this patient? Answer Choices 1 She is not a candidate for hormone replacement because of her coronary artery disease. 2 Estrogen replacement alone would be recommended due to the increased risk of endometrial cancer with progesterone administration. 3 Hormone replacement will only control her hot flashes and night sweats, but will not affect her vaginal dryness. 4 Hormone replacement is contraindicated for her because of her chronic alcohol use. 5 Certain antidepressant could be an alternate therapy for treatment of her menopause related mood swings.

Certain antidepressant could be an alternate therapy for treatment of her menopause related mood swings. Explanation This patient is presenting with classic symptoms of natural menopause and could be a candidate for hormone replacement therapy based on the history information that is provided. CHD is not a contraindication for HRT. Some studies have shown a decrease in CHD in postmenopausal women on HRT. Estrogen therapy alone is contraindicated in this patient because her uterus is intact and estrogen that is unopposed by progesterone increases the risk of endometrial cancer. Hormone replacement will in fact diminish vasomotor symptoms, as well as vaginal dryness. Modest alcohol use is not a contraindication for hormone replacement. SSRIs would be an alternative therapy option for the treatment of this patient's mood swings. However, SSRIs would not result in relief of vasomotor symptoms or vaginal dryness.

You are caring for a 29-year-old G3P2 at 39 weeks gestation, who has been laboring for 6 hours. She is a diet-controlled diabetic. Her last child was 9 pounds 8 ounces. She has been completely dilated for 2 hours, and the fetal head is at a plus 2 station, which is unchanged. What is the next most appropriate course of action? A Begin oxytocin B Vacuum extraction C High forcep extraction D High forcep rotation E Cesarean section

Cesarian Section The patient is a diabetic with a history of a macrosomic infant; the likelihood of macrosomia in this infant is significant. Instrument delivery is not recommended if macrosomia is suspected. By definition, she has had an arrest of descent of the fetal head and one should be highly suspicious for macrosomia, in which case a c-section is the preferred method of delivery.

A 25-year-old primigravida presents at 8 weeks gestational age for her first prenatal visit. She denies any abnormal vaginal discharge or pelvic pain. On pelvic exam, you note cyanosis of the proximal vagina and cervix. Question What is this finding consistent with? Answer Choices 1 Acute cervicitis 2 Chronic cervicitis 3 Hegar's sign 4 Chadwick's sign 5 Cervical ectopic pregnancy

Chadwick's sign

A woman is leaving with her 1-year-old son to go grocery shopping. She lives in a condominium with a small covered garage. As she starts her car, her telephone rings. She runs into the house to answer the phone, leaving her son safely buckled into his car seat and the car engine still running. When she returns to her car, she finds her son still buckled into his car seat. He is unconscious. She calls the paramedics. They tell her that the boy was exposed to noxious fumes. Question What will the child's mother notice about his skin? Answer Choices 1 Cherry-red (hyperemic) 2 White as a ghost (anemic) 3 Slate gray (pigmented) 4 Yellow (jaundiced) 5 Blue (cyanotic)

Cherry-red (hyperemic) The child inhaled carbon monoxide from the exhaust fumes of the car. A car running in a small covered garage will produce dangerous levels of carbon monoxide in a short period of time. Carbon monoxide has a much higher affinity for hemoglobin than oxygen (200-300 times). This leads to cellular hypoxia. The cherry red color is secondary to the carboxyhemoglobin. Paleness, a slate gray skin color, jaundice, and cyanosis are not seen with carbon monoxide poisoning.

A 60-year-old man presents with a 2-year history of shortness of breath. He has been a heavy smoker for 40 years. He has a chronic cough; it produces at least ½ a cup of mucoid sputum a day. He was hospitalized 3 times last year for infective respiratory disease. He is currently on asthmatic medication; he is taking 5mg salbutamol, 250-mcg ipratropium 4 hourly by nebulizer, slow release theophylline 300-mg b.i.d., and prednisolone 10 mg daily. At present, he presents with history of worsening of cough and shortness of breath. On examination, the patient is cyanotic, T 100, P 100/m, and RR 25. What should the initial investigation be? Answer Choices 1 Pulmonary function test 2 Chest X-ray 3 Sputum culture 4 CBC 5 Blood culture

Chest Xray A chest X-ray can determine the extent of pulmonary consolidation. It will also exclude pneumothorax or carcinoma. Therefore, it is the best choice. A pulmonary function test is not done during the acute illness, and an estimation of blood gases is unlikely to influence the immediate management. CBC is not important in immediate management. Blood culture and sputum should be done afterward to determine further antibiotic treatment, but it would not influence immediate management.

A 25-year-old sexually active man notices that he has burning and pain while urinating. He also notices some urethral discharge. He sees you in your office for a consultation, and you order several laboratory tests. One of the tests that you order is a Gram stain and culture on a sample of the discharge. The results are negative, and gonorrhea is ruled out. After reviewing all the lab results, you tell your patient that he has nongonococcal urethritis (NGU). What is the most likely cause of this patient's nongonococcal urethritis? Answer Choices 1 Chlamydia psittaci 2 Chlamydia trachomatis 3 Ureaplasma urealyticum 4 Trichomonas vaginalis 5 Pneumocystis jiroveci

Chlamydia trachomatis Chlamydia are bacteria. Chlamydia was once thought to be protozoa as well as viruses, but it is neither; they are bacteria in the order Chlamydiales. Chlamydia is obligate intracellular organisms. Characteristic of chlamydia infections is the development of inclusion bodies. Chlamydia trachomatis cause approximately 50% of all cases of nongonococcal urethritis in males. Chlamydia trachomatis causes more nongonococcal urethritis in boys/men than Trichomonas vaginalis does. Urethritis is an inflammation of the urethra. It is classified as either gonococcal urethritis (caused by Neisseria gonorrhoeae) or nongonococcal urethritis (caused by something other than Neisseria gonorrhoeae). Common causes of nongonococcal urethritis are Chlamydia trachomatis, Trichomonas vaginalis, and Ureaplasma urealyticum. Chlamydophila psittaci does not cause nongonococcal urethritis. Chlamydophila psittaci causes a systemic illness called psittacosis, parrot fever, or ornithosis. Psittacosis is acquired from birds. Trichomonas vaginalis is a protozoan. Trichomonas vaginalis can cause nongonococcal urethritisin males.It also causes trichomoniasis vaginitis, sometimes called trichomoniasis. Ureaplasma urealyticum is considered a mycoplasma. Ureaplasma urealyticum is in the family Mycoplasmataceae and the genus Ureaplasma. Bacteria in the genus Ureaplasma require urea; therefore, Ureaplasma urealyticum is found primarily in genitourinary tract. Ureaplasma urealyticum is a common cause of nongonococcal urethritis. Pneumocystis jiroveci (formerly called Pneumocystis carinii) is a fungus, not a protozoan. In an immunosuppressed host, Pneumocystis jiroveci can cause pneumonia. Pneumocystis jiroveci does not cause nongonococcal urethritis.

A 48-year-old man presents with right upper quadrant pain. His pain developed after eating fried chicken at a cookout, and it is now continuous. He has experienced previous episodes of less severe right upper quadrant pain; they resolved spontaneously. The pain radiates to his right shoulder; it is accompanied by nausea, vomiting, and chills. On physical examination, the patient is noted to have scleral icterus and jaundice. Rigors are present. The abdomen is slightly distended, with positive Murphy's sign. Vital signs include a BP of 120/85 mm Hg, pulse 117, respirations 18, and temperature 102.5 degrees F. CBC reveals mild leukocytosis, elevations of serum aminotransferases and bilirubin, as well as normal lipase. Question What is the likely diagnosis in this patient? Answer Choices 1 Cholelithiasis 2 Cholangitis 3 Pancreatitis 4 Cirrhosis 5 Hepatocellular carcinoma

Cholangitis Cholangitis is the correct response. The patient has right upper quadrant pain, jaundice and fever, which are all part of the Charcot triad associated with cholangitis. Cholangitis is an infection of the common bile duct that typically occurs when the duct becomes obstructed. The patient reports previous episodes of right upper quadrant pain; they were likely caused by gallstones (cholelithiasis). The patient probably has a stone obstructing the bile duct, causing cholangitis. Cholelithiasis is an incorrect response. The patient likely does have gallstones. However, gallstones do not always lead to cholangitis. It is only when a stone becomes obstructed that cholangitis may develop. Therefore, while cholelithiasis is likely contributing to the patient's diagnosis, cholangitis is a more appropriate diagnosis. Acute pancreatitis is an incorrect response. The pain associated with pancreatitis is usually epigastric, with radiation into the back. Pancreatitis may be associated with jaundice if the cause is gallstone pancreatitis. However, if the patient had pancreatitis, his lipase would be elevated. Cirrhosis and hepatocellular carcinoma are both incorrect responses. While these conditions may be associated with jaundice, the presence of Charcot's triad strongly suggests cholangitis.

A 45-year-old man presents with a 1-week history of high-grade fever, abdominal pain, occasional rigors, loss of appetite, malaise, nausea, and vomiting. He was all right about a week ago when he had low-grade fever for the first 2 days. Fever was high grade during the last 3 - 4 days, with occasional rigors accompanied by abdominal pain predominantly in right upper abdomen. Pain is moderate in intensity, continuous without any radiation, and is not relieved by acetaminophen (Tylenol), which he is taking on his own. Examination reveals tender hepatomegaly. No other organs are palpable. Abdomen is slightly distended, but bowel sounds are normal. Vital parameters: temp 40° C, pulse 108 per minute, blood pressure 136/90mmHg, and respiration rate 20 per minute. Patient appears sick and toxic. The rest of the physical examination is as following: Neck: Supple. No stridor. Lymphatics: No lymph node groups were palpable. Lungs: Decreased breath sounds at the base of the right lower lung. Cardiac: Normal S1 and S2. No extra heart sounds. No heave. Back: Negative Groin: Unremarkable Genitalia: Unremarkable Neurological: Normal cranial nerves. Normal motor and sensory examination. Ultrasonography reveals multiple liver abscesses in both lobes of the liver, which is confirmed by similar findings in CT scan. Question What is the most common cause of the patient's pyogenic liver abscess? Answer Choices 1 Pylephlebitis 2 Cholangitis 3 Endocarditis 4 Sub-phrenic abscess 5 Pyelonephritis

Cholangitis Cholangitis is the most common cause of pyogenic liver abscess. Biliary pathologies are the most common predisposing cause of pyogenic liver abscess and accounts for 60% of the cases. The etiological classification of bacterial abscess of the liver (pyogenic liver abscess) is made on the basis of the route by which infection reaches the liver. There are 5 mechanisms: Pylephlebitis is the inflammation of the portal vein or any of its branches. Portal bacteremia [via portal vein] from an intra-abdominal site, such as diverticulitis, suppurative appendicitis, and infected carcinoma of the colon, especially after resection, ulcerative colitis, inflamed hemorrhoids, typhoid, and paratyphoid fever. Along the bile ducts, causing ascending cholangitis in a biliary tract partially or completely obstructed by stone, tumor, or stricture. Systemic bacteremia originating in a distant location, with organisms reaching the liver via the hepatic artery; causes enumerated can be septicemia and pyemia, endocarditis, pyelonephritis, and infection of the hydatid cyst. Direct extension from an adjacent infection outside the biliary tract, such as from a sub-diaphragmatic abscess, from an empyema thoracis, and from trauma, either penetrating (with direct implantation of bacteria into the liver) or blunt, causing a hematoma that becomes secondarily infected. From the umbilicus along the umbilical vein of the newborn and along the paraumbilical veins. A cause is usually obvious, but sometimes the abscess is unexplained. Although most abscesses are single, multiple (usually microscopic) abscesses are common with systemic bacteremia or complete biliary tract obstruction. Prompt antibiotic treatment, with agents such as meropenem, imipenem, and cefuroxime, followed by percutaneous drainage, form the essential components of treatment. Biliary tract diseases account for 60% of the pyogenic liver abscess cases. Systemic bacteremia accounts for 15% of the cases. 24% of the pyogenic liver abscesses can be attributed to pylephlebitis. Direct extension from an adjacent infection like sub-phrenic abscess accounts for 4% of the cases.

our patient, a 48-year-old woman, presents with vaginal bleeding and states that she is "alarmed" because she is quite sure she is 2 months pregnant. History includes unremarkable live birth of a male child 7 years ago and a molar pregnancy a year ago. Examination reveals a uterus that is inappropriately large for gestational length and hCG levels are higher than expected. Fetal parts and heart sounds are not present. Your diagnosis is that of carcinoma, but you are able to reassure your patient that this neoplasm is of the type that is most sensitive to chemotherapy. What is your diagnosis? Answer Choices 1 Endometrioid carcinoma 2 Ovarian dysgerminoma 3 Choriocarcinoma 4 Serous cystadenocarcinoma 5 Ovarian teratoma

Choriocarcinoma Explanation Gestational Trophoblastic Diseases are a group of related diseases forming a spectrum from Benign Hydatidiform Mole, to Invasive Mole, to Placental-Site Trophoblastoma, and finally to Choriocarcinoma. Treatment of women who have nonmetastatic gestational trophoblastic disease is almost 100% successful, and allows reproductive function to be preserved. The cure rate for metastatic disease is approximately 90 percent. Serous cystadenocarcinoma occurs in the ovary and is a cystic or semi-cystic neoplasm. It usually occurs bilaterally An ovarian dysgerminoma is a malignant ovarian neoplasm, hypothesized to be derived from primordial germ cells of the sexually undifferentiated embryonic gonad. Endometrioid carcinoma resembles the typical carcinoma of the endometrium, as its name suggests, but is an ovarian carcinoma. This neoplasm may be seen with endometrial carcinoma. Ovarian teratoma is composed of tissues that are derived from three germinal layers, the endoderm, mesoderm, and ectoderm. Many teratomas will contain hair or teeth. These true neoplasms will usually present with lower quadrant pain.

A 40-year-old female is status post a dilatation and curettage for hydatidiform mole. On week 3 post surgery, her follow-up quantitative hCG level has elevated slightly. What is the most likely diagnosis? A Adenocarcinoma of the ovary B Adenocarcinoma of the uterus C Retained hydatidiform mole D Choriocarcinoma E Corpus luteum cyst

Choriocarcinoma 5% of hydatidiform mole progress to choriocarcinoma; the longer the mole in intrauterine the higher the risk. Pre-surgical evaluation for mole removal includes a chest x-ray to rule out distant metastasis. hCg that either plateuas or elevates is choriocarcinoma until proven otherwise, and requires prompt evaluation.

A 22-year-old man presents with a 3-month history of worsening diarrhea that comes and goes. While performing a comprehensive oral exam, you note 2 lesions on the buccal mucosa on the right side of the oral cavity. You document these lesions as 2 round lesions that measure approximately 2 mm in diameter and have a white-yellow center, which is surrounded by a red halo. Pertinent positives also include a reduced appetite, abdominal pain, and cramping. Question Based on these current history, physical exam findings, and possible disease pathologies, what diagnosis is very high on your differential? Answer Choices 1 Ulcerative colitis 2 Cholecystitis 3 Crohn's disease 4 GERD 5 Colon cancer

Chron's disease The patient above has an aphthous ulcer, also sometimes referred to as a canker sore or aphthous stomatitis. These painful, open sores are found in the oral cavity and are the most common form of mouth ulcer. Many times these lesions are described as having a white or yellow center and are surrounded by a bright red area. They are benign, noncancerous, noninfectious, and many times, the cause is unknown; however, aphthous lesions are common extra intestinal manifestations found in patients suffering from Crohn's disease. Patients who have the other form of inflammatory bowel disease, ulcerative colitis, will not typically develop such signs as aphthous ulcers. More common signs of cholecystitis include right-sided abdominal pain, vomiting, fever, a positive Murphy's sign, or even palpable gallbladder. GERD symptoms generally surround the complaint of heartburn, although it can cause issues such as asthma, chronic cough, chronic laryngitis, sore throat, or even non-cardiac chest pain. Colon cancer can present with anemic qualities on serology tests, abdominal pain, changes in bowel habits, as well as hemoccult-positive stool.

A 53-year-old woman presents with a 2-year history of chronic cough. The cough produces large volumes of grossly purulent sputum. She has a history of recurrent respiratory infections; they resulted in 5 hospitalizations in the past year. She also had similar complaints during the previous year.. Shortness of breath limits her daily activity considerably. Upon pulmonary examination, bilateral breath sounds are audible, with inspiratory and expiratory crackles at the lung bases. Chest X-rays reveal increased lung volumes, flattened diaphragm, and tram track lines. What is the initial diagnosis? Answer Choices 1 Asthma 2 Emphysema 3 Pulmonary edema 4 Chronic bronchitis 5 Pulmonary fibrosis

Chronic Bronchitis The clinical picture is suggestive of chronic bronchitis. Chronic bronchitis is defined by a clinical history of productive cough for 3 months of the year for 2 consecutive years. Smoking is the leading cause. The principle pathologic feature is airway injury and narrowing, hypertrophy of the airway mucous glands, infiltrate of inflammatory cells, and loss of ciliated epithelium. The cough produces thick, often purulent sputum because of the ongoing local inflammation and the high likelihood of bacterial colonization and infection. The increased mucous production and defective mucociliary escalator function leads to inspiratory and expiratory crackles. On imaging, common findings are hyperinflation of lung volumes, depressed diaphragm, and parallel linear densities known as tram track lines. Asthma produces wheezing lung sounds. They are not heard in this patient. A productive cough is not a clinical manifestation of emphysema. Pulmonary edema may present with mild exertional dyspnea or a nonproductive cough, although a frothy or blood-tinged sputum may be seen. Pulmonary fibrosis is a restrictive lung disease with the clinical features of progressive dyspnea. It is typically accompanied by a dry, persistent hacking cough.

A 56-year-old man presents with abdominal pain, indigestion, weight loss, nausea, and vomiting, and gray colored stools for the past month. His past medical history is positive for alcoholism. Lab results demonstrate an elevated serum amylase and lipase, decreased trypsinogen, and a positive fecal fat test. What is the most likely diagnosis? Answer Choices 1 Acute pancreatitis 2 Chronic pancreatitis 3 Pancreatic carcinoma 4 Pacreatic abscess 5 Insulinoma

Chronic Pancreatitis Pancreatitisis an inflammation or infection of the pancreas. Chronic pancreatitis, which is the correct response, is caused by alcohol abuse, hemochromatosis (a condition of excess iron in the blood), and other unknown factors. Inflammation and fibrosis cause the destruction of functioning glandular tissue in the pancreas. This results in an inability to properly digest fat caused due to a lack of pancreatic enzymes. The production of insulin is also affected. Symptoms include abdominal pain (mainly in the upper abdomen), nausea, vomiting, weight loss, and fatty stools. Additional symptoms may include swelling (overall), stools (clay colored), and abdominal indigestion. Tests should include serum lipase (may be elevated), serum amylase (may be elevated), serum trypsinogen (may be low), and fecal fat test (shows fatty stools). Abdominal ultrasound and CT may show an enlarged pancreas. Treatment of chronic pancreatitis includes reducing pancreas stimulation, alleviating fat indigestion, reducing pain, and treating diabetes. A reduced-fat diet, vitamin supplementation, no alcohol or caffeine, and regulation of blood sugar levels are indicated in the treatment. The chief causes of acute pancreatitis in adults are gallstones, other biliary disease, and alcohol use. Viral infection (mumps, Coxsackie B, mycoplasma pneumonia, and Campylobacter), injury, pancreatic or common bile duct surgical procedures, and certain medications (especially estrogens, corticosteroids, thiazide diuretics, acetaminophen, tetracycline) are other causes. After the triggering event, the process continues with autodigestion that causes swelling, hemorrhage, and damage to the blood vessels. An attack may last for 48 hours. Symptoms include abdominal pain (mainly located in the upper abdomen) nausea, vomiting, weakness, sweating, anxiety, fever, clammy skin, and mild jaundice. General examination may show a low blood pressure and a heart rate above 90. Most cases resolve within a week with supportive measures, such as analgesics and fluid replacement. However, some cases can be life threatening. Pancreatic abscess occurs in 5 to 10% of people with acute pancreatitis. An abscess may be caused by inadequate drainage of a pancreatic pseudocyst, which is a complication associated with pancreatitis. Symptoms include fever, chills, abdominal pain, and abdominal mass. Physical exam will show signs of pancreatitis, and tests should include an abdominal CT and ultrasound. Treatment will include laparotomy with drainage and possible resection of dead tissue. Pancreatic cancer is the 4th most common cancer causing death in the U.S. The disease is more common in men, especially those between 60 and 70 years. The cause is unknown; however the incidence is greater in smokers. A high fat diet and chemical exposures may increase the risk. Symptoms include weight loss, abdominal pain, loss of appetite, jaundice, nausea, weakness, fatigue, vomiting, diarrhea, indigestion, back pain, stools (clay colored), pallor, and depression. Tests should include a pancreatic biopsy, an abdominal CT scan, and abdominal ultrasound. Only 20% of the tumors are operable at the time of diagnosis. Palliation is generally the treatment, along with chemotherapy and radiation. Insulinomas are generally benign tumors of the insulin-secreting cells of the pancreas, which secrete excess amounts of insulin. Risk factors include a prior history of multiple endocrine neoplasia Type I (MEN I). Symptoms include sweating, tremor, rapid heart rate, anxiety, hunger, dizziness, headache, clouding of vision, confusion, behavioral changes, convulsions, and loss of consciousness. Surgery is the treatment of choice to remove the tumor. If the tumor is not found during surgery, diazoxide may be given. A diuretic is always given with this medication to avoid retaining too much salt.

A 62-year-old male presents with complaints of numbness in his hands and feet, with occasional foot drop, memory disturbance, fatigue, paleness, anorexia, nausea, and weight loss. He has a known history of diabetes and hypertension. Which of the following conditions is most likely responsible for these symptoms?

Chronic Renal Failure Explanation: Chronic renal disease is associated with functional disturbances in all organ systems, including the central nervous system. Renal disease promotes CNS complications including neuropathies and neuromuscular irritability, along with systemic symptoms. The symptoms are typically progressive if the underlying renal disease is not addressed. Although other conditions promote similar neuropathies, such as diabetes, they are differentiated by the level of involvement, progression, and associated symptoms. With Guillain Barre, an acute polyradiculoneuropathy would be expected to progress, and have associated weakness. Cerebrovascular accidents are not typically accompanied by generalized systemic symptoms, and a middle cerebral artery occlusion would be expected to have contralateral hemiparesis and hemisensory deficit.

A 67-year-old man presents with a subacute onset of lower urinary tract symptoms. He is unable to discuss his past medical history or current medications. An initial genitourinary workup is started, and a microscopic urinalysis reveals granular and waxy casts. Question With what disease process are his results closely associated? Answer Choices 1 Chronic renal disease 2 High urinary protein nephrotic syndrome 3 Glomerulonephritis 4 Pyelonephritis 5 Acute tubular necrosis

Chronic renal disease Explanation The correct response is chronic renal disease. Casts are cylindrical structures, consisting of clumps or clusters of cells or material that can form in the renal distal and collecting tubules of the kidney. Casts form when the pH of the urine is acidic and when the urine is very concentrated. Casts dislodge from the kidney and can be seen in the urine. In order to see casts, urine must be visualized under low power on a microscope. There are various types of casts that can be characterized into acellular versus cellular casts; each category can be further characterized, and the various casts can be associated with various disease processes. Granular casts are the 2nd most common type of cast and result from the breakdown of cellular material. They are most often indicative of chronic renal disease, but can also be seen if a patient has just vigorously exercised. Waxy casts are also indicative of advanced renal disease, specifically indicating a more chronic issue. Fatty casts are the result of the breakdown of lipid-rich epithelial cells; they are pathognomonic for high urinary protein nephrotic syndrome. High urinary protein nephrotic syndrome does not lead to the formation of granular casts. Nephritic syndromes, urinary tract injury, glomerulonephritis, and vasculitis can all result in red blood cell casts. Whenever there are red blood cells within a cast, there is a strong indication for glomerular damage from a number of different disease processes. Glomerulonephritis does not lead to the formation of granular casts. If white blood cells are seen within a cast, this is an indication that there is an inflammation or infection of the kidney known as pyelonephritis. Various other inflammatory states can also result in white blood cell casts. Pyelonephritis does not lead to the formation of granular casts. Acute tubular necrosis, cytomegalovirus, hepatitis, and toxic ingestion can all result in epithelial cell casts. These casts are the result of desquamation of the renal tubule cells into the collecting system. Acute tubular necrosis does not lead to the formation of granular casts.

A man presents with a chronic respiratory infection; he is seeking the advice of an ear, nose, and throat specialist. A biopsy of his respiratory epithelium reveals an alteration in certain epithelial structures. What is most likely to be abnormal? Answer Choices 1 Microvilli 2 Desmosomes 3 Cilia 4 Hemidesmosomes 5 Stereocilia

Cilia In some individuals, a structural abnormality involving the absence of dynein arms in cilia (Kartagener syndrome, a hereditary disease) is associated with chronic respiratory difficulty (e.g., bronchitis and sinusitis). Ciliary motility is severely impaired or absent in these individuals. The epithelium in the respiratory passages does not possess microvilli. Desmosomes (maculae adherens) are cell-to-cell junctions attached to the lateral sides of 2 cells; it is the only attachment device present in epidermal cells. In other epithelia, especially those with cuboidal or columnar cells, desmosomes are found in conjunction with a zonula adherens. They are thought to play a role in dissipating physical forces throughout the cell from the attachment site. Hemidesmosomes, present on the basal surface of the cells of the immune system, occur where epithelia require stronger adhesion to the connective tissue; they are believed to confer a 'social identity' onto cells. Stereocilia (long microvilli) are limited to the epididymis of the male reproductive system. Stereocilia of the sensory epithelium of the ear are uniform in diameter and possess an internal filamentous structure. They serve as a receptor device rather than an absorptive structure.

A patient presents to the clinic with a family member. Upon obtaining history from the patient, he responds with excessive details of his symptoms and the reason for his visit. He is unable to answer a question directly without signification elaboration. What problem does this patient have? A circumstantiality B derailment C incoherence D tangentiality

Circumstantiality

You are asked to see a patient who was admitted to the hospital. Upon attempts to obtain a history, you notice the patient states words that sound similar, but do not have the same meaning. He also does some rhyming of his words. What type of thought process would this be? A flight of ideas B circumstantiality C looseness of association D word salad E clanging

Clanging E Clanging is a disturbance in thought in which the person selects words that are similar by sound, but do not mean the same. Sometimes the person will rhyme the words. Flight of ideas is rapid transitioning between subjects, but tends to be connected. Looseness of association is when a person changes subjects, but there is no connection between the subjects. Circumstantiality is where the person has a point and eventually gets to that point, but with delay in the thought process. Word salad is a mixture of words that have no sense.

A 20-year-old college student describes his headaches as unilateral, a 5 out of 10 for pain, and with throbbing, associated with photophobia and often accompanied by nausea and vomiting. He states that the headaches occur two to three times a month and are associated with decreased sleep. He has taken OTC medication with relief and is trying to stay on a sleep schedule. He denies any other neurologic symptoms. Which of the following is the most appropriate next step for managing this patient? A Brain MRI B Hydrocodone-acetaminophen tablets C Physical therapy D Topiramate E Sumatriptan nasal spray

Classic migraines often present with episodic, unilateral, and throbbing headache pain, associated with photophobia and phonophobia. Nausea and vomiting may also be involved. Management of migraines includes nonpharmacologic therapies, such as healthy eating, sleep pattern stabilization, caffeine avoidance, and stress relief. If nonpharmacologic therapies are not sufficient, or if symptoms impact activities of daily living, pharmacologic management is indicated. Medications are available for migraine sufferers and are typically catergorized as abortive or preventive. Abortive therapy for this patient may include nonsteroidal anti-inflammatory medications. Additionally, studies have shown that stimulation of the 5-HT receptors can successfully stop a migraine, and the 5-HT1 receptor agonists, with selective agents (the triptans) often used successfully for headache improvement. Nasal spray formulations are useful for nausea and vomiting. Topiramate, an anticonvulsant, has received FDA approval for migraine prevention therapy in patients with increasing migraine frequency or poor response to abortive therapies. Narcotics are typically avoided for migraine management, and MRI is not warranted without additional clinical signs and symptoms, and other differential diagnoses.

A 24-year-old female, with a history of type 2 diabetes, presents with the inability to conceive after 14 months of unprotected sexual intercourse with her husband. Her vital signs are unremarkable and you calculate a BMI of 31. Physical examination reveals acne vulgaris and hirsutism. Which of the following treatment options for her infertility would be the most effective considering your suspected diagnosis? A Medroxyprogesterone acetate B Clomiphene citrate C Metformin D Spironolactone E Mini-pill (progestin only)

Clomiphene citrate B Clomiphene citrate is highly effective as the first line treatment for infertility in PCOS. It can be accompanied with metformin, weight loss, exercise, and exogenous gonadotropins when clomiphene fails. PCOS in over half of patients is accompanied with obesity, abnormalities in insulin control, metabolic syndrome, and infertility. Medroxyprogesterone acetate (A) and the mini-pill (E) are used for endometrial protection and with oral contraceptive pills. Metformin (C) will help with her diabetes. Spironolactone (D) is a diuretic, which acts as a weak androgen receptor antagonist.

A 29-year-old woman comes in for evaluation of an increased vaginal discharge for the past week. She describes it as "sort of whitish gray with a disgusting odor." She has no other symptoms. She has had no new sexual partners, has taken no antibiotics, and has not used any new hygiene products. Examination reveals no vulvar erythema. She does have an adherent whitish discharge in the vaginal vault, but no vaginal erythema. Whiff test of the secretions is positive. Microscopic examination of saline and potassium hydroxide preparations is most likely to reveal which of the following? A Clue cells B Increased polymorphonucleocytes C Motile flagellates D Small, rounded parabasal epithelial cells E Spores and filaments

Clue cells A Examination of vaginal secretions in a woman with bacterial vaginosis demonstrates the presence of clue cells, which are epithelial cells that appear granulated due to G vaginalis cells adhering to them. Increased PMNs (B) may be present in a number of vaginal conditions, but not typically in bacterial vaginosis. Motile flagellates (C) are present in vaginal trichomoniasis, while small parabasal epithelial cells (D) are present in atrophic vaginitis. Spores and filaments (E) (pseudohyphae) are typical of candidiasis.

A 53-year-old man presents with a 2-week history of severe headaches. The headaches occur primarily at night. The patient is pacing while he is talking. The pain surrounds 1 eye and lasts for 30 to 90 minutes. He also states that there is ipsilateral lacrimation, conjunctival injection, and nasal congestion during the attacks. The patient states that he's had these headaches 2 or more times in a day over a period of several weeks. He cannot point to any one thing that causes them. On examination, the patient has features of partial Horner's syndrome. His vital signs are temperature: 97.0 °F, heart rate: 80/min, respiration: 16/min, and blood pressure: 126/80mmHg. Question What type of headache is the patient experiencing? Answer Choices 1 Common migraine 2 Classic migraine 3 Cluster headache 4 Stress headache 5 Chronic daily headache

Cluster HA Cluster headaches come in groups and attacks may be separated by months or years. There are 2 types: episodic and chronic. The patient is often a middle-aged man. Cluster headaches are characterized by excruciating, unilateral pain (stabbing sensation) often occurring in the ocular, frontal, or temporal areas. These headaches generally last for a period of 15 minutes to 3 hours, recurring at the same time of day. They typically occur during the night. Pain often radiates to the upper teeth, jaw, and neck. The pain is normally accompanied by one or more of the following: ipsilateral conjunctival injection or lacrimation, ipsilateral nasal congestion or rhinorrhea, ipsilateral eyelid edema, ipsilateral forehead and facial sweating, ipsilateral miosis or ptosis, and a sense of restlessness or agitation. Restlessness is one of the characteristic features of cluster headaches noted in patients; behaviors such as pacing or rocking the head and trunk with head in hands are common. Other symptoms of cluster headache include facial flushing or pallor, tenderness on palpation of the ipsilateral carotid artery, bradycardia, and tenderness of the scalp and facial areas. The absence of aura, nausea, or vomiting helps in distinguishing cluster headaches from migraines. The treatment of choice for acute cluster headaches includes the administration of oxygen (7 L per minute for 15 minutes), sumatriptan, or a combination of both. Other therapeutic measures include intranasal dihydroergotamine, intranasal capsaicin, or intranasal lidocaine. Prednisone and verapamil are often used in prophylaxis. Common migraine is an intermittent syndrome characterized by hemi-cranial pulsatile cephalic discomfort, including nausea, vomiting, photophobia, anorexia, and phonophobia. Classic migraine typically presents with an aura. It also includes features of nausea, vomiting, photophobia, anorexia, phonophobia, and a sense of just wanting to hide in a quiet dark room. Stress/tension headaches are a generalized type of headache precipitated by stress. They are not one-sided. They do not involve photophobia, nausea, vomiting, aura, or phonophobia. Chronic daily headaches occur on a daily basis and are often disabling. This type of headache may be challenging to diagnose and treat. Headaches caused by medication overuse or drug rebound are the most treatable forms of refractory daily headache.

A 45-year-old man goes to a party and enjoys several glasses of an alcoholic cocktail. His past medical history is significant for headaches. The drinks trigger a unilateral right-sided headache. The headache is behind his right eye and spreads to his forehead. He also notices that his right nostril has a watery discharge and his right eye is tearing. He describes the pain as if he were "being stabbed in my eye". Question What kind of headache does he have? Answer Choices 1 Tension headache 2 Hangover headache 3 Common migraine 4 Classic migraine 5 Cluster headache

Cluster HA This patient has a cluster headache. Typically, cluster headaches occur in the spring and fall, as the case here. During a cluster period, the headaches can be triggered by alcohol. During an attack, there can be discharge from the nose and eye on the same side of the face that the headache is on. The pain of a cluster headache begins rapidly. The pain is described as stabbing or knifelike, as the case here. The headaches last up to 2 hours. Tension headaches are associated with stress. This kind of headache would not be associated with water discharge from the nose and eye. A hangover headache would not occur while the man is still drinking. This kind of headache would not be associated with water discharge from the nose and eye. Common migraine and classic migraine would not be associated with water discharge from the nose and eye.

A 40-year-old man presents with a 3-week history of a daily headache that awakens him from sleep. The headaches occur around the left eye. He notes that the left eye waters and his left nostril gets stuffy with each headache. What is the most likely diagnosis? Answer Choices 1 Sinusitis 2 Tension headache 3 Cluster headache 4 Intracranial mass 5 Migraine

Cluster HA The clinical picture is suggestive of a cluster headache. Cluster headaches are typically seen in middle-aged men. Unilateral periorbital pain can occur daily for several weeks. They are often accompanied by ipsilateral nasal congestion, rhinorrhea, lacrimation, and redness of the eyes. Episodes often occur at night, and they last between 15 minutes and 3 hours. Sinusitis commonly presents with unilateral face fullness, pressure, and tenderness over the cheeks. Tension headaches are caused by muscle contractions that usually elicit a 'bandlike' tightness rather than throbbing pain. Often brought on by stress, they are usually not associated with nausea or vomiting, and they more commonly cause bilateral rather than unilateral pain. Intracranial masses cause an increase in intracranial pressure, which can cause headaches with vomiting. Headaches related to intracranial masses are typically worst in the morning, as recumbency increases intracranial pressure. Headaches due to intracranial pathology are likely to worsen over a short period and be progressive in nature, rather than present as a cyclical course. Migraine headache pain is usually described as pulsatile in nature and is commonly unilateral. Other common manifestations include nausea, vomiting, and photophobia. They are more common in women than in men.

A 16-year-old girl with a 2-year history of ulcerative colitis presents with signs of an acute exacerbation (i.e., abdominal pain and passing large, frequent quantities of blood and mucus from the rectum). It is treated with sulfasalazine, glucocorticoids, and intravenous alimentation. The diarrhea decreases markedly, but her status continues to deteriorate. Tachycardia, volume depletion, and electrolyte imbalance develops; her current temperature is 101.8° F. Physical examination finds abdominal tenderness, but no mass. Plain radiography shows extensive ulceration and the transverse colon is dilated up to 7 cm. What is the most appropriate next step in the management? Answer Choices 1 Proctocolectomy, distal rectal mucosectomy, and ileal pouch-anal anastomosis 2 Colectomy 3 Barium enema 4 Colonoscopy 5 Tapering glucocorticoids

Colectomy The correct response is colectomy. This patient's symptoms and signs are characteristic of toxic megacolon, a dangerous complication of ulcerative colitis. It carries the life-threatening risk of perforation and septicemic shock. A barium enema and colonoscopy have to be avoided due to the danger of further precipitating toxic megacolon. The failure of medical treatment also indicates the need for surgery. Proctocolectomy, distal rectal mucosectomy, and ileal pouch-anal anastomosis is an increasingly popular surgical treatment of colitis ulcerosa; however, it cannot be performed in the midst of an acute exacerbation. Emergency colectomy remains the only viable option. Tapering of glucocorticoids is appropriate when healing progresses, but it does not represent an effective treatment of toxic megacolon.

A 54-year-old man presents because he has very large urine output and is constantly thirsty. His urine output is many liters per day. He wakes up at night to urinate. His past medical history is remarkable for a 25-year history of a bipolar disorder; it is treated effectively with lithium. His lab results are as follows: TEST RESULTS REFERENCE RANGE BUN 17 mg/dL 10-20 mg/dL Calcium 9.9 mg/dL 8.5-10.5 mg/dL Potassium 4.2 mEq/L 3.5-5.0 mEq/L Sodium 149 mEq/L 135-145 mEq/L Glucose (fasting) 109 mg/dL 65-110 mg/dL The results of his urinalysis are as follows: TEST RESULTS REFERENCE RANGE Urine dipstick blood Negative Negative Urine dipstick glucose Negative Negative Urine dipstick ketones Negative Negative Urine dipstick protein Negative Negative 24 hour urine protein 124 mg/24 hr <150 mg/24 hr Urine osmolality 40 mOsm/kg 50-1400mOsm/kg Urine specific gravity 1.001 1.001-1.035 Urine pH 6.2 4.5-8.5 Question What structure is most involved in the pathophysiology of this condition? Answer Choices 1 Neurohypophysis 2 Adenohypophysis 3 Glomerulus 4 Loop of Henle 5 Collecting duct

Collecting ducts Explanation Collecting dust is the correct response. A known side effect of lithium is nephrogenic diabetes insipidus. This side effect is experienced by a significant minority of the patients taking lithium. Nephrogenic diabetes insipidus results when the collecting duct of the kidney does not respond to the antidiuretic hormone (ADH). The posterior lobe of the pituitary is the neurohypophysis. The neurohypophysis secretes antidiuretic hormone (ADH) and oxytocin. The production of the posterior pituitary hormones actually occurs in the hypothalamus. The anterior lobe of the pituitary is the adenohypophysis. The anterior lobe of the pituitary secretes follicle-stimulating hormone (FSH), luteinizing hormone (LH), prolactin, growth hormone (GH), thyroid-stimulating hormone (TSH), and adrenocorticotropic hormone (ACTH) (and the corticotropin related peptides). The collecting duct is a portion of the nephron. It is also called the collecting tubules. Urine enters the collecting duct from the distal convoluted tubule. The collecting duct is responsive to ADH. The glomerulus is a specialized capillary tuft. The endothelium of the glomerulus is fenestrated. The glomerulus is surrounded by Bowman's capsule. A renal corpuscle consists of the glomerulus and Bowman's capsule. A renal corpuscle is sometimes called a Malpighian corpuscle. Renal corpuscles are seen in the kidney cortex. The loop of Henle is part of the nephron. It is shaped like a hairpin. The proximal convoluted tubule leads into the descending limb of the loop of Henle. The ascending limb of the loop of Henle goes to the distal convoluted tubule.

A new patient is seen in your internal medicine office today. She is coming in to request the removal of several skin tags. She is a 55-year-old woman with a history of untreated acromegaly. A health maintenance plan is set up with the patient, and includes a colonoscopy. This patient is at increased risk for which of the following findings on colonoscopy? A Anal fissures B Ulcerative colitis C Colon polyps D Pseudomembranous colitis E Colonic fistulas

Colon polyps The correct choice is C, colon polyps. Approximately 30% of patients with acromegaly have been found to have colon polyps. These patients also have an increased risk of colon cancer. Patients with acromegaly have not been found to be at increased risk for the other response choices listed here.

A 35-year-old man has a routine physical examination with no abnormal findings. His family history, however, is positive for familial adenomatous polyposis. What screening test would be best for him to obtain? Answer Choices 1 No screening necessary before the age of 50 2 Colonoscopy 3 Fecal occult blood test 4 Fecal immunochemical test 5 Stool DNA test (sDNA) 6 AFP tumor marker

Colonoscopy The best test to recommend is a colonoscopy since it detects both polyps and cancer. Colorectal cancer is the 4th most common form of cancer in the United States; it arises from adenomatous polyps in the colon. Familial adenomatous polyposis (FAP) has different inheritance patterns and different genetic causes. In this patient, the pattern is probably autosomal dominant which puts him at a risk of nearly 100% for developing colorectal cancer by the age of 40. That means that screening tests for colorectal cancer must be performed earlier than suggested for the rest of the population (i.e., before the age of 50). A fecal occult blood test, fecal immunochemical test, and stool DNA test (sDNA) find cancer, not polyps. The AFP (alpha-fetoprotein) tumor marker is a useful marker for hepatocellular carcinoma and germ cell tumors, but not for colorectal cancer.

A 62-year-old man presents with vision problems and difficulty swallowing. Over the past week, he has had a constellation of symptoms, beginning with numbness and tingling in his feet that progressed to weakness that now affects both lower and upper extremities. Within the past day, he has started to notice difficulty swallowing and double vision. He also feels it is difficult for him to take a big breath. His past medical history is noncontributory, and he takes no medications. Exam reveals bilateral absence of patellar and ulnar reflexes. Question What is the most likely prognosis for this patient? Answer Choices 1 Complete resolution of symptoms 2 Gradual resolution of paralysis and residual paresthesias 3 Gradual resolution of paresthesias and residual lower extremity weakness 4 Progressive paralysis and premature death most likely due to respiratory failure 5 Waxing and waning polyarticular weakness and paresthesias

Complete Resolution The vast majority of patients (>80%) with Guillain-Barré syndrome (GBS) experience a full recovery with supportive care. In addition to supportive care, plasmapheresis and/or intravenous immune globulin are often used to reduce the duration and severity of symptoms once the diagnosis is recognized. Symmetrical ascending paralysis as described in this patient is indicative of Guillain-Barré syndrome (GBS). The cause of GBS is unknown, but it is generally thought to be an inflammatory autoimmune process. More than 50% of patients with GBS report an antecedent illness. The antibodies produced in response to antigens present in the infectious agent are thought to cross-react with components of human neurons, prompting an acute post-infectious demyelinating process. Lumbar puncture characteristically reveals elevated CSF protein content. Other results are normal, although the white blood cell count may be somewhat elevated. Decreased CSF glucose and increased polymorphonuclear cell counts are seen in acute bacterial meningitis. Decreased CSF glucose and elevated CSF lymphocyte counts are commonly seen with meningitis caused by fungi. Viral meningitis usually presents with elevated lymphocyte counts, normal CSF glucose, and normal or slightly elevated CSF protein levels.

A 36-year-old gravida 5, para 3013 at 34 weeks gestational age presents to the Triage Unit of Labor and Delivery with a 2-hour history of painless bright red bleeding per vagina. She states that after urinating, she found the toilet "filled with blood" when she stood up. There is normal fetal movement. She had 1 Caesarean section for uncertain fetal status 2 years ago. Her other 2 term pregnancies resulted in normal vaginal deliveries. She denies recent intercourse, prior history of vaginal bleeding during this pregnancy, syncope, dizziness, and headache. Vital signs: BP is 100/72 mm Hg, and maternal pulse is 110 bpm; respirations are 18/min. The patient is afebrile. Fetal heart rate is160 bpm, absent variability, absent accelerations, and occasional variable decelerations; therefore it is a category II fetal heart rate tracing. Occasional irregular contractions are seen on tocodynamometer. Question What management step should be taken immediately? Answer Choices 1 Caesarean section 2 Complete blood count, type and hold, administration of IV fluids 3 Biophysical profile 4 Amniocentesis for lecithin and sphingomyelin 5 Induction of labor; anticipate normal vaginal delivery

Complete blood count, type and hold, administration of IV fluids Explanation The most likely diagnosis for this patient with 3rd trimester bleeding is placenta praevia. Obstetrical hemorrhage is still one of the most common etiologies of maternal death in the United States. In a patient with significant blood loss, attention must first be paid to determination of the degree of blood loss, volume expansion with isotonic IV fluids, and access to blood replacement if necessary. Caesarean section is not indicated at this time, as maternal complete blood count is not yet available, and as at this point the fetal heart rate tracing is Category 2. A biophysical profile may be of benefit in determining fetal status, but is not the most critical next step. Sonography to confirm placenta praevia is more critical than the biophysical profile at this time. Amniocentesis for lecithin and sphingomyelin may be of benefit in determining whether the fetus has achieved lung maturity, but it is not the next best step in managing this emergent situation. Induction of labor and normal vaginal delivery are contraindicated in placenta praevia.

A 20-year-old woman who was 3 months pregnant aborted a mass which resembled a bunch of grapes. The specimen that she passed is shown in Figure G3.5. Karyotype analysis of the specimen revealed that it was 46, XX. The MOST likely diagnosis is which of the following? Answer Choices 1 Incomplete abortion, normal placenta 2 Choriocarcinoma 3 Ectopic pregnancy 4 Partial (incomplete) hydatidiform mole 5 Complete hydatidiform mole

Complete hydatidiform mole Explanation A hydatidiform mole occurs once in about 2000 pregnancies. Its incidence is higher among women at the two extremes of maternal age than in other women. The serum and urinary chorionic gonadotropin levels are usually abnormally high. In complete mole, the cause is fertilization of an ovum which has lost its chromosomes. Almost all have a 46XX diploid pattern from the sperm by androgenesis. There are no fetal parts. Almost every chorionic villus is cystic and avascular. There is diffuse trophoblastic proliferation. About 2 percent develop choriocarcinoma. Partial mole is due to fertilization of an ovum by 2 sperms. It has a triploid 69XXY chromosomal pattern. Only some villi are cystic, and there is slight trophoblastic proliferation. Fetal parts may be present. Choriocarcinoma rarely develops from a partial mole. The first manifestation of the presence of a mole is often the excessively rapid enlargement of the uterus due to growth of the placental mass. This is usually followed, at about the third to fifth month of pregnancy, by uterine bleeding or abortion. Placental tissue passed at this time show that the villi are cystic and grape-like. Microscopically, the mole shows cystic avascular villi together with irregular clumps of large syncytial and cytotrophoblastic cells. Hydatidiform mole may be complete or partial. It is characterized by cystic chorionic villi. Choriocarcinoma may develop in a mole, especially the complete type. About 50 percent of gestational choriocarcinomas arise in moles. Many of these have been treated successfully with chemotherapy. Choriocarcinoma is a rare neoplasm. In about half the cases, it arises in women with a hydatid mole. The rest of the cases are about equally divided between abortions and full-term pregnancy. A few develop as a teratoma unrelated to a pregnancy. Grossly, the uterus containing a choriocarcinoma shows shaggy, fleshy hemorrhagic masses filling the uterine cavity and extending into the myometrium. Microscopically, the neoplasm is composed of clusters of large, bizarre, often multinucleated giant trophoblastic cells, extending into the myometrium and often filling the vascular sinuses. Necrosis and hemorrhage are seen, both in the tumor masses and in the myometrium. Very rarely both the primary neoplasm and the metastases have disappeared spontaneously. Numerous cures have been achieved by methotrexate and actinomycin D in choriocarcinomas related to a mole or pregnancy. Typically the tumor spreads locally to the vagina and parametrium, invades the veins, and metastasizes to the lungs, liver, and brain. Usually the neoplastic cells produce chorionic gonadotropin in both the primary growth and the metastases, sometimes with extremely high titers. Most ectopic pregnancies occur in a uterine tube. Any pathologic change which causes narrowing or blockage of the tubal lumen predisposes to tubal pregnancy. The narrowing may be intrinsic due to inflammation or extrinsic due to formation of adhesions around the tube. Tubal ligation is designed to prevent fertilization of ovum by sperm and thus any pregnancy. Tubal pregnancy is the most common form of ectopic pregnancy. It occurs about once in 200 pregnancies. The burrowing trophoblast usually erodes and ruptures the tube early in pregnancy. Sometimes tubal abortion occurs, the fetus being extruded from the lateral end of the tube into the abdominal cavity. Interstitial pregnancy is implantation in a cornu of the uterus in the interstitial portion of the uterine tube. 'Abdominal' pregnancy (peritoneal pregnancy) may result either from direct implantation of the fertilized ovum upon the peritoneum or from extrusion of the fetus of a tubal pregnancy into the abdominal cavity. Ovarian pregnancy is extremely rare and presumably results from fertilization of the ovum at the time of ovulation.

A 45-year-old secretary presents with a 1-month history of paresthesias on the medial aspect of her right hand. She has no neck pain or shoulder pain, but she has had some difficulty typing with the right hand. Neurological exam reveals diminished sensation to pain and light touch on the medial aspect of the palmar and dorsal surfaces of the hand to the wrist, the 5th finger, and the medial aspect of the ring finger. Mild weakness of the right abductor minimi digiti is present, but the other intrinsic hand muscles are of normal strength. The remainder of the neurological exam is normal. Question What condition does patient have? Answer Choices 1 Compressive ulnar neuropathy at the elbow 2 Compressive median neuropathy at the wrist 3 Compressive radial neuropathy at the mid-humerus 4 Compressive C8-radiculopathy 5 Compression of axillary nerve

Compressive ulnar neuropathy at the elbow This patient has a right ulnar neuropathy, which is most likely due to nerve compression at the elbow. The abnormalities on neurologic exam are limited to weakness of the right abductor digiti minimi, which is innervated by the ulnar nerve, and a sensory disturbance in the distribution of the ulnar nerve, which includes splitting of the 4th finger. The ulnar nerve exits the brachial plexus as the terminal branch of the medial cord. The nerve then descends through the axilla and upper arm, after which it traverses the groove ("ulnar groove") between the medial epicondyle of the lower humerus and the olecranon of the ulnar bone. The nerve runs through the forearm deep to the flexor carpi ulnaris muscle; it exits the wrist through an anatomic space distinct from the carpal tunnel, which is where the median nerve is located. Ulnar nerve compression can occur in the axilla, elbow, or wrist. Occupational injury is most often located in the elbow because of the nerve's proximity to the bony surfaces; therefore, it is susceptible to external compression (e.g., in persons who lean their elbow against a hard surface). In addition, work-related repetitive flexion and extension of the elbow causes narrowing of the ulnar groove, and it results in ulnar nerve entrapment. The diagnosis can be made by radiographs of the elbow, neck, chest, and wrist. Definitive diagnosis is made by electromyography and nerve conduction tests. Conservative treatment can be tried; it should consist of elbow rest and padding, non-steroidal anti-inflammatory drugs (NSAIDs), and vitamin B-6 supplements. If the paresthesias worsen, surgical decompression coupled with transposition can be performed, depending on the site of entrapment. Median neuropathy (carpal tunnel syndrome) characteristically produces a sensory disturbance of the palmar aspect of the hand, thumb, 2nd and 3rd fingers, and the lateral aspect of the 4th finger. Radial neuropathy produces weakness of wrist extension. A C8-radiculopathy produces weakness of all intrinsic hand muscles and does not cause splitting sensory loss of the 4th finger. Radial neuropathy does not cause weakness of the abductor digiti minimi. When there is axillary nerve entrapment, the patient complains of weakness and numbness of the shoulder, with signs of deltoid atrophy.

A 52-year-old woman with a history of untreated bronchogenic carcinoma presents with a 1-week history of dyspnea, facial edema, and marked neck vein distention; the distention has progressed over the past 36 hours. A chest X-ray confirms a right hilar mass with a small pleural effusion in the right hemithorax. Which of the following steps is most appropriate? Answer Choices 1 Consult a cardiologist for pericardiocentesis 2 Consult an oncologist for radiation therapy 3Consult a cardiothoracic surgeon for a mediastinoscopy 4 Consult a cardiovascular surgeon for exploration thoracotomy 5 Prepare the patient for a pleurocentesis in the office

Consult an oncologist for radiation therapy The patient is presenting with signs and symptoms of Superior Vena Cava (SVC) syndrome, caused by obstruction of blood flow through the SVC due to compression, thrombosis, or infiltration from a right upper hilar malignancy. Obstruction of venous drainage at the upper thorax causes venous distention. Morbidity may result from irreversible thrombosis, central nervous system damage, or pulmonary complications. The most common cause of obstruction is malignant disease, with bronchogenic carcinoma at the top of the list. Signs and symptoms of SVC syndrome include swelling and cyanosis in the face and upper trunk, dyspnea, dysphagia, neck vein distension, and headache. Cerebral edema is a potential complication. In patients who present with serious airway or central nervous system systems, the preferred management strategies are SVC stenting and/or radiation therapy. Diuretics, steroids, and thrombolytics, are sometimes used as temporizing measures, but their effectiveness is questionable. The diagnosis of pericardial effusion is not supported by the history, physical exam, and chest film; therefore, pericardiocentesis is not indicated. Direct surgical interventions are ineffective in this case. The small pleural effusion present on chest x-ray is unlikely to account for the patient's symptoms, and an in-office pleurocentesis would not be advisable.

A 27-year-old G3P2 at 39 weeks gestation has been in labor for 6 hours. Her membranes ruptured 3 hours ago and revealed a large amount of clear fluid. She has previously delivered a 7 pound 2 ounce infant. She is having good quality contractions, and has been completely dilated and pushing for 55 minutes. The fetal head is +4, FHT 128. What would be the most appropriate management at this time? A Continue pushing B Instrument delivery C Emergency caesarean section D Non emergent caesarean section E Epidural pain control

Continue pushing A This is a multigravida patient with an adequate pelvis. She is making steady progress and there are no signs of maternal or fetal distress, so continued labor management is expected.

A 48-year-old woman presents with new-onset headache that she describes as nonspecific, worse on awakening, intermittent throughout the day but can worsen with bending over or coughing. Her husband reports that she has not been herself since the headaches started about 4 to 6 weeks ago. Which of the following tests would be best for determining the etiology of her presenting symptoms? A noncontrast head CT scan B lumbar puncture C contrast-enhanced brain MRI D noncontrast brain MRI E cerebral angiography

Contrast MRI this patient has an intracranial mass until proven otherwise. Headaches starting later in life and accompanied by other neurologic or cognitive problems should raise a high suspicion of a tumor. Obtaining a contrast-enhanced MRI of the brain will demonstrate an intracranial mass lesion. The contrast will follow blood flow distribution and help in determining possible tumor type. Some lesions are difficult to see without contrast enhancement. MRI scans have a much higher resolution for soft tissue over CT scans and are preferred for looking at brain parenchyma. Prior to the advent of CT and MRI, cerebral angiography was used to look for intracranial masses. Vascular tumors have characteristic blush patterns, and if a mass effect is present, it will distort the position of the blood vessels.

A 69-year-old man presents with dyspnea on exertion that has been slowly progressive over the course of the last year. He notes impairment in climbing stairs and walking short distances. His review of systems is positive for fatigue, palpitations, intermittent retrosternal chest pain, swelling of his lower extremities, dizziness, and "feeling faint;" his associated symptoms are also known to occur upon exertion. He denies any fever, chills, weight changes, cough, abdominal pain, early satiety, nausea, vomiting, diarrhea, changes to his urine color or odor, flank pain, hematuria, or dysuria. He denies any cigarette, alcohol, or drug use. His cardiac exam is remarkable for an increased pulmonic component of the second heart sound (P2), wide, inspiratory splitting of S2 over the cardiac apex, right-sided S3 and S4 gallops, a left parasternal lift, a loud diastolic murmur that increases with inspiration and diminishes with the Valsalva maneuver, prominent "A" waves in jugular venous pulsations, and increased JVD. He has an enlarged liver with hepatojugular reflux, peripheral edema, and ascites. A bedside EKG analysis revealed peaked P waves, rightward axis deviation, and prominent R waves in the early V leads. Question What is the most likely diagnosis? Answer Choices 1 Myocardial infarction 2 Cor pulmonale 3 Primary biliary cirrhosis 4 Left ventricular heart failure 5 Pulmonary embolism

Cor Pulmonale Myocardial infarctions occur at rest and most commonly in the early morning. The pain is similar to angina in location and radiation, but it may be more severe and builds up rapidly or in waves to maximum intensity over a few minutes or longer. Associated symptoms include diaphoresis, weakness, apprehension, and a feeling of impending doom; patients may move about, seeking a position of comfort, preferring not to lie quietly. Light-headedness, syncope, dyspnea, orthopnea, cough, wheezing, nausea and vomiting, or abdominal bloating may occur. Physical exam findings may include fever, anxiousness, diaphoresis, bradycardia or tachycardia, low cardiac output, or arrhythmia. There may be hypertension (in hypertensive patients) or low in patients with shock. Respiratory distress, jugular venous distention a Kussmaul sign, soft heart sounds, and atrial gallops (S4) or ventricular gallops (S3) usually indicate heart failure. Primary biliary cirrhosis is most common in middle-aged females and is characterized by fatigue, pruritus, hepatosplenomegaly, xanthomatous lesions on the skin, eyelids, and tendons, jaundice, and steatorrhea. Signs of portal hypertension are late findings. Other findings include orthostatic hypotension and cognitive dysfunction. Left-sided or forward failure may account for many of the clinical manifestations of heart failure, such as mental confusion from decreased cerebral perfusion, fatigue and weakness from decreased skeletal muscle perfusion, and sodium and water retention with secondary venous congestion from decreased renal perfusion. Isolated left-side heart failure is associated with dyspnea, fatigue, weakness, cough, paroxysmal nocturnal dyspnea, and orthopnea in the absence of peripheral edema, jugular venous distention (JVD), or hepatojugular reflux. Chest pain, dyspnea, and tachypnea are the most frequent signs and symptoms of pulmonary embolism. Other manifestations may include tachycardia, pleurisy, low-grade fever, apprehension, and productive cough with blood-tinged sputum. Massive PE may manifest as sudden collapse, crushing substernal chest pain, shock, diaphoresis, hypotension, distended neck veins, and loss of consciousness.

A 19-year-old man presents with a sudden onset of vomiting, headache, vision changes, numbness and tingling of extremities, difficulty speaking, difficulty writing, loss of coordination, and loss of balance. His past medical history is significant for sickle cell anemia. Physical exam findings are consistent with acute cerebrovascular insufficiency. Early detection and prevention of stroke in sickle cell anemia is now possible by using screening tests. What imaging modality is recommended as a screening test for cerebrovascular disease/stroke in sickle cell disease (SCD) patients? Answer Choices 1 Magnetic resonance imaging (MRI) 2 Computed tomography (CT) skull 3 X-ray skull 4 Neurocognitive testing 5 Transcranial Doppler ultrasonography (TCD)

Correct Transcranial Doppler ultrasonography (TCD) Stroke or cerebrovascular insufficiency occurs in about 11% of patients under 20 years of age with sickle cell anemia. The major symptom includes sudden hemiparesis with or without aphasia. Most of these strokes occur in the distribution of the intracranial internal carotid artery or the middle cerebral artery. The Stroke Prevention Trial in Sickle Cell Anemia (STOP Trial) confirmed that TCD can identify children with SCD at high risk for first-time stroke. As the greatest risk of stroke occurs in early childhood, it is recommended that children aged 2 - 16 years receive TCD screening. Stroke in SCD is a medical emergency and requires mandatory exchange transfusion followed by maintenance hypertransfusion. Studies have shown that about 90% of strokes could be prevented in children with stenotic cranial-artery lesions as demonstrated on transcranial Doppler ultrasonography if they are maintained on a regular program of chronic transfusion. The chronic transfusion should be such that it suppresses erythropoiesis so that no more than 30% of the circulating red cells are their own; this is done in order to maintain the Hb S level below 30%. Computed tomography (CT) and magnetic resonance imaging (MRI) are used for diagnosing strokes, but they are not used as a screening modality. Patients with poor educational performance should undergo neurocognitive testing, as they may have early ischemia of the central nervous system. It is recommended that children with sickle cell disease be screened for cerebrovascular disease with TCD and not MRI, CT, X-ray skull, or neurocognitive testing.

A 15-year-old boy presents with bloody diarrhea and abdominal cramping. A double contrast barium enema shows fine serrations and narrowing of the rectum and sigmoid. Stool contains mucus, blood, and white blood cells, but no parasites or bacterial pathogens. Endoscopy shows inflamed mucosa and pseudopolyps. A biopsy finds an extensive inflammatory process in the mucosa and submucosa. The glands are filled with eosinophilic secretions; there is also mild involvement of the terminal ileum. Sulfasalazine treatment is attempted, but fails to bring about improvement. What is the most appropriate next step in the management? Answer Choices 1 Corticosteroids 2 Metronidazole 3 6-mercaptopurine 4 Diphenoxylate 5 Loperamide

Corticosteroids Fine serrations (ulcers) and pseudopolyps suggest colitis ulcerosa (ulcerative colitis). It is usually limited to the colon but occasionally, as in this case, the terminal ileum is also affected (backwash ileitis). In addition to sulfasalazine or 5-amino-salicylacid, more severe ulcerative colitis cases require corticosteroids. Corticosteroids are also helpful in Crohn's disease involving the small bowel. Metronidazole is helpful in treating the characteristic fistulas of Crohn's disease. Immunosuppressive agents (e.g., 6-mercaptopurine) are used in severe Crohn's disease cases. Diphenoxylate and loperamide provide only symptomatic relief. While surgical removal of the colon can give relief in ulcerative colitis, Crohn's disease tends to recur in approximately 1/2 of the cases involving removal of the diseased bowel.

A 38-year-old male complains of increasing fatigue, weakness, weight loss, and intermittent nausea, vomiting, and diarrhea over the past few months. He has noted some agitation at times. When this first started he thought that this was related to a virus but the GI symptoms have reoccurred on multiple occasions. Labs show a complete blood count (CBC) within the normal reference range. He is noted to have hyponatremia. On examination you note the multiple areas of hyperpigmentation as seen below. His blood pressure in the office is 100/50, P = 66, T = 97.1˚F. What test would you order to confirm your suspected diagnosis? A dexamethasone suppression test B vasopressin challenge test C radioactive iodine uptake scan D cosyntropin stimulation test E follicular stimulation test

Cosyntropin stimulation test

A 34-year-old woman presents due to a 6-month history of progressive fatigue. She has had to stop running due to lack of energy and notes that she is napping frequently. She also has a poor appetite, dry skin, and notes darkening of her skin over her elbows and knees. Physical examination is significant for orthostatic hypotension and hyperpigmentation of the elbows and knees. Basic metabolic panel reveals the following: Sodium: 125 (135-146 mmol/L) Potassium: 4.8 (3.5-5 mmol/L) Chloride: 78 (98-107 mmol/L) Bicarbonate: 23 (23-32 mmol/L) Glucose: 95 (65-100 mg/dL) BUN:17 (8-25 mg/dL) Creatinine: 0.7 (0.8-1.30 mg/dL) Question What testing is recommended to confirm her diagnosis? Answer Choices 1 Dexamethasone suppression test 2 Plasma fractionated metanephrine 3 Cosyntropin stimulation test 4 Serum CA 19-9 5 Radioactive iodine uptake test

Cosyntropin stimulation test The patient's history, presence of hyperpigmentation, and presence of hyperkalemia are suggestive of Addison's disease, or primary adrenal insufficiency. The cosyntropin stimulation test (also called the ACTH test) is used to assess the function of adrenal gland response. A small amount of cosyntropin (synthetic ACTH) is injected, and the serum cortisol is obtained 45 minutes following administration. Normally, the cortisol level should respond to cosyntropin injection. Patients with Addison's disease lack the normal response to ACTH, so the cortisol level will fail to rise. Dexamethasone suppression test is incorrect. The dexamethasone suppression test is used in the diagnosis of Cushing syndrome, which is caused by hypercortisolism. Plasma fractionated metanephrine is incorrect. Plasma fractionated metanephrines are used in the diagnosis of pheochromocytoma. Serum CA 19-9 is incorrect. Serum CA 19-9 is a tumor marker for pancreatic cancer. Radioactive iodine uptake test is incorrect. The radioactive iodine uptake test is used in the work-up of hyperthyroidism.

Creatinine

Creatinine is a by product of msl breakdown and it is a good indicator of kidney function • If kidney function is declining Cr becomes elevated because the kidney is not filtering creatinine

A 3-year-old boy presents with difficulty in breathing and a cough that sounds like a seal. On examination, the child has fever, a harsh barking cough, a respiratory rate of 38/minute, and minimal stridor on agitation. On lung auscultation, there are no rales or wheezing. On cardiac auscultation, there is tachycardia. Radiological examination reveals the so-called 'steeple sign'. What is the most likely diagnosis? Answer Choices 1 Bronchiolitis 2 Croup 3 Epiglottitis 4 Foreign body aspiration 5 Pneumonia

Croup Croup, which is caused by the parainfluenza virus, is an upper respiratory infection with a characteristic brassy cough, hoarseness, and respiratory stridor. The typical clinical picture has a variable onset, from a progressive fever, sore throat, and cough for a couple of days to a sudden midnight onset. The severity varies from stridor only (with mild agitation) to stridor at rest to airway obstruction. Treatment involves humidified air, inhaled or oral corticosteroids, and nebulized racemic epinephrine. Bronchiolitis, caused by the respiratory syncytial virus (RSV) and other respiratory viruses (influenza, parainfluenza, rhinovirus), is a severe infection in infants that can progress to respiratory failure. Premature infants and infants with heart and lung disease have severe forms of bronchiolitis. The patients have wheezing, rales, prolonged expiratory phase, rhinorrhea, nasal congestion, fever, tachypnea, and respiratory distress. Antigen testing for RSV from nasal secretions provides a rapid diagnosis. Ribavirin administered through aerosol may shorten the clinical course. Polyclonal and monoclonal antibodies are used for prophylaxis in wintertime. Epiglottitis, which is caused by Haemophilus influenzae, is characterized by inflammation and rapidly progressive edema of epiglottis and contiguous tissue. Children ages 2 to 7 years who missed some immunizations are prone to this infection during winter. Drooling, hoarseness, high fever, sore throat, the characteristic 'sniffing dog' position, and the rapidly progressive respiratory obstruction make the diagnosis a clinical one. Immediate intubation and intravenous ceftriaxone or cefuroxime are recommended for treatment. Foreign body aspiration presents with complete or partial airway obstruction localized at a different location in the respiratory tract. The acute presentation after a choking episode provides the diagnosis; however, without a witnessed episode of choking, clinical presentation may take a couple of days after the aspiration. Tracheal obstructions present with acute asphyxia in total obstruction and with stridor in partial obstruction. Localized signs, such as wheezing, rhonchi, and decreased breath sounds, are found in lower respiratory tract obstruction. The treatment of choice is extraction with rigid bronchoscope. Pneumonia in children may be a difficult clinical diagnosis. The immune status, the age, and risk factors may help in suspicion of the etiologic organism. Profuse crackles and rales after an upper respiratory infection may suggest a viral pneumonia. A localized consolidation syndrome is more characteristic for a bacterial origin of pneumonia. The characteristic findings on chest radiography also suggest the etiology of the pneumonia.

Ulcerative Colitis appearance on colonoscopy

Crypt abscesses and if toxic megacolon present loss of haustra and blood vessels

A 15-year-old girl presents with a 1-year history of intermittent abdominal pain with nausea and occasional bloody diarrhea. She denies fever or weight loss, as well as any travel history. Past medical history is significant only for migraines. She takes a multivitamin. Her vital signs are within normal limits. She has mild diffuse abdominal tenderness to palpation and guaiac-positive stool. Her exam is otherwise normal. Her hemoglobin is 9.7, hematocrit is 28%, and her WBC is 12,000/uL. Question What finding would indicate a possible cure if a colectomy is performed? Answer Choices 1 Skip lesions, transmural involvement on colonoscopy 2 Crypt abscesses, superficial mucosal involvement on colonoscopy 3 Ileal disease 4 Fistulas, fissures 5 Mouth ulcers

Crypt abscesses, superficial mucosal involvement on colonoscopy Crypt abscesses and superficial mucosal involvement are colonoscopy findings associated with ulcerative colitis. A colectomy is performed for severe and intractable disease. A colectomy can be a relative cure since the disease is limited to the colon. Skip lesions, transmural involvement, fistulas, and fissures are all associated with colonoscopy findings in Crohn's disease. Crohn's disease can also affect any structure of the alimentary tract from the mouth to anus, causing ileal disease and mouth ulcers.

A 50-year-old woman with Hodgkin's disease has been treated with cytotoxic drugs and steroids. She is admitted to the hospital with a 5-week history of headache, low-grade fever, visual disturbances, and confusion. The patient also has a 2-month history of progressive memory loss. Physical examination reveals a stiff neck. After lumbar puncture, initial studies reveal a WBC of 100 cells/mm3, glucose of 20 mg/dL, and protein of 100 mg/dL. CSF testing with India ink is positive. What is the cause of the CNS disease? Answer Choices 1 Bacterial meningitis 2 Aseptic meningitis 3 Herpes simplex virus encephalitis 4 Tuberculous meningitis 5 Cryptococcal meningitis

Cryptococcal Meningitis

A 55-year-old man with a history of chronic renal failure, 6 months status post renal transplant, presents with chest pain, productive cough, and low-grade fever. He reports generalized malaise as well. Current medications include only those related to the transplant. He has no known allergies. Examination reveals a temperature of 102°F, unremarkable HEENT (head, ears, eyes, nose, throat), and few crackles anteriorly in the upper right lung field. Chest X-ray reveals a solitary nodule in the right upper lobe. The most likely etiology for his symptoms is A Streptococcus pneumoniae B Pneumocystis jiroveci C cryptococcosis D Candida E influenza A

Cryptococcosis Cryptococcal species are opportunistic organisms responsible for infections in immune-compromised hosts. With the rise of HIV infections in the past few decades in the United States, cryptococcosis is becoming increasingly prevalent. It is also a common infection in those who have undergone solid organ transplantation. The two most common areas for infection are the lungs and the central nervous system. Pulmonary involvement includes fever, productive cough, chest discomfort, and weight loss. Pleural effusions, lymphadenopathy, and solitary or multiple nodules can all be seen on chest x-ray. Central nervous system manifestations include meningitis and meningoencephalitis. Diagnosis is confirmed with India ink prep of cerebrospinal fluid showing yeast or histologic stains of tissue from the involved organs. Treatment is with oral or parenteral antifungal agents. (

A 57-year-old man presents with a sudden onset of left lower quadrant pain, fever, and guarding. The patient appears septic and ill. His hemoglobin is 12.2 g/dL, and hematocrit is 37%; white blood cell count (WBC) is 15 x109/L, and platelets are 150 x109/L. You suspect diverticulitis, with a probable perforation. What study would best confirm the diagnosis? Answer Choices 1 Barium enema 2 Colonoscopy 3 CT scan of the abdomen 4 Flexible sigmoidoscopy 5 Sonogram

Ct scan of the abdomen The correct response is a CT scan of the abdomen. Patients with acute diverticulitis can be diagnosed based on the clinical presentation. Since other acute surgical conditions can present in a similar clinical pattern, confirmation of the diagnosis is important. Contrast radiography (barium enema) or colonoscopy should be undertaken with caution in patients with suspected acute diverticulitis. The increased luminal pressure from the injected contrast material, or from insufflation of air, may lead to free rupture of a previously well-localized peridiverticular abscess or phlegmon. Although studies have suggested that both contrast enemas and colonoscopies (when performed by experienced personnel) can be done safely with a high diagnostic yield, the introduction of computerized tomography has substantially reduced their use. CT scanning has the advantages of being non-invasive, avoiding increased luminal pressure, and being able to detect extraluminal disease; therefore, it is the test of choice to confirm the diagnosis of diverticulitis. Fiber-optic endoscopy offers little information in the evaluation of suspected diverticulitis and carries a significant risk of increasing the pericolic contamination. A sonogram is not as sensitive in picking up diverticulitis as a CT scan, and it would probably end up requiring a CT scan to confirm.

A 30-year-old woman presents with depression. She denies any significant past medical history or surgeries. On physical examination, she is found to be obese, with her weight localized to her trunk; her arms and legs are relatively normal. Her abdomen has prominent purple striae. Her face is very round and covered with acne. Her blood pressure is found to be 164/102 mm Hg. Her laboratory results are as follows: TEST RESULTS REFERENCE RANGE BUN 14 mg/dL 7 - 18 mg/dL Calcium 9.6 mg/dL 8.4 - 10.2 mg/dL Potassium 4.1 mEq/L 3.5 - 5.0 mEq/L Sodium 139 mEq/L 135 - 145 mEq/L A further workup is done and she is found to have a pituitary adenoma. What condition is most likely? Answer Choices 1 Addison's disease 2 Conn's syndrome 3 Cushing's syndrome 4 Cushing's disease 5 Nelson's syndrome

Cushing Disease Explanation This patient has a pituitary adenoma that is secreting ACTH. Cushing's disease refers to excessive secretion of ACTH by the pituitary. A pituitary adenoma is the most common cause of pituitary hypersecretion of ACTH. The increase in ACTH secretion results in hyperplasia of the adrenal cortex and a subsequent increase in glucocorticoid release. Characteristics of glucocorticoid excess include depression, truncal obesity, purple striae, round face (moon facies), and hypertension. Addison's disease is primary adrenal insufficiency. The most common cause of Addison's disease is autoimmunity; tuberculosis is another cause. The symptoms seen with Addison's disease include weight loss, lethargy, hyperpigmentation, nausea, vomiting, and hypotension. Hyponatremia and hyperkalemia can be seen because of the lack of mineralocorticoids. When there is decreased stimulation of the adrenal cortex due to a decrease in ACTH, it is called secondary adrenal insufficiency. Conn's syndrome is due to excessive mineralocorticoids, usually due to an adenoma that is secreting aldosterone. Conn's syndrome results in hypernatremia and hypokalemia; there is an associated hydrogen loss, as well. There will be hypertension. A distinction exists between Cushing's disease and Cushing's syndrome. Cushing's disease refers specifically to excessive secretion of ACTH by the pituitary. In Cushing's disease, the excessive ACTH secretion by the pituitary results in an increase in glucocorticoid release by the adrenal glands. Cushing's syndrome refers to the effects of excessive glucocorticoids (other than those due to a pituitary cause); therefore, Cushing's syndrome is due to excessive glucocorticoids. There will be hypertension with both Cushing's disease and Cushing's syndrome. Nelson's syndrome is seen in patients who have had a bilateral adrenalectomy for Cushing's disease; there is the subsequent development, or progression, of an ACTH-secreting pituitary tumor.

A 1 day-old boy develops progressing abdominal distension, bilious vomiting and failure to pass a meconium stool. Abdominal radiographs show dilated loops of small bowel. Which of the following is the most likely diagnosis in this patient? A Cystic Fibrosis B Hypothyroidism C Imperforate anus D Intussusception E Pyloric Stenosis

Cystic Fibrosis The patient presents with a meconium ileus consistent with a diagnosis of cystic fibrosis (A). Pyloric stenosis (E) typically presents between 3 and 6 months of age, while intussusception (D) presents later (6 to 24 months). Imperforate anus (C) presents at birth, but infants typically lack acute abdominal distention and bilious vomit. Other common causes of intestinal obstruction in a newborn include meconium plug syndrome, Hirschsprung Disease, Intestinal Atresia, and Midgut Volvulus.

A 1-year-old child presents with acute watery diarrhea that was preceded by a low-grade fever and vomiting. What test could confirm the most likely cause of this disease? Answer Choices: A Serum antibody test B Stool culture C Abdominal x-ray D ELISA testing of stool E Throat culture

D

A 10-year-old boy presents with swelling on his face; it has been progressively increasing in size. He is an immigrant from East Africa. On examination, he has mild pallor and large swelling involving his right maxilla. A biopsy taken reveals a starry sky pattern of lymphocytes. What organism is associated with his condition? Answer Choices: A Hepatitis B virus B Human papilloma virus C Schistosoma hematobium D Epstein Barr virus (EBV) E Human T-cell lymphotrophic virus type 1

D

A 12-year-old boy eats hamburgers, hot-dogs, deviled eggs, macaroni salad, and potato salad at an Independence Day picnic. Approximately 3 hours after eating, he suddenly develops nausea, vomiting, and non-bloody diarrhea. His mother takes his temperature and is relieved to find that he does not have a fever. What caused the boy's food poisoning? Answer Choices: A Entamoeba histolytica B Shigella species C Campylobacter jejuni D Staphylococcus aureus E Yersinia enterocolitica

D

A 12-year-old girl presents with hypertension, which is not being treated at this time. Although she does not note any symptoms, she is once again found to be hypertensive; there is a cardiac gallop and a medium-pitched systolic murmur, which is best heard posteriorly in the interscapular area, with radiation to the left axilla, apex, and anterior precordium. A prominent anterior chest heave is also observed. The lower extremities demonstrate a 16 mm Hg pressure difference as compared to the upper extremities. Additionally, there are delayed femoral pulsations; her upper extremity pulsations are normal. The remainder of the physical exam is normal. A recent chest X-ray is notable for the following images. What pharmacotherapeutic agent would be most beneficial to this patient at this time? A Prostaglandin E1 B Dopamine C Enalapril (Vasotec) D Lopressor (Metoprolol) E Furosemide (Lasix)

D

A 15-year-old girl presents with a 2-day history of excessive vaginal discharge. She had unprotected sex with a boyfriend 4 days ago, and he later informed her that he has gonorrhea. On speculum exam, the girl is found to have a moderate amount of off-white and frothy vaginal secretions. The appearance of the cervix is normal. A cervical swab reveals copious Gram-negative intracellular diplococci. What should be the 1st step in the management of this patient? Answer Choices: A Obtain written or oral parental consent to treat B Ask the patient to return with parent or adult guardian C Request permission to treat from Department of Health D Counsel patient about safer sex and treat E Ask patient to provide a list of sexual contacts

D

A 17-year-old boy presents for an annual physical exam prior to the start of college. For the last 3 weeks, he has been camping in Los Alamos. Although he and his friends brought bottled water on the trip, he says the rivers were so clean that they all ended up just drinking from them directly. On further questioning, the boy explains that while he has been in generally good health, over the past few weeks he has been suffering from loose bowel movements and excessive flatulence. In addition, he intermittently has mild nausea and chronic abdominal pain that is interfering with his appetite. He thinks he has lost a little weight. He reports no history of fever, vomiting, or other non-gastrointestinal complaints. What is the probable cause of the boy's infection? Answer Choices: A Staphylococcus aureus B Bacillus cereus C Clostridium perfringens D Giardia lamblia E Entamoeba histolytica

D

A 19-year-old man presents 1 hour after stepping on a sharp nail. The nail penetrated deep into his foot; his last tetanus immunization was at the age of 6. What treatment should the patient receive? Answer Choices: A Tetanus immunoglobulin only B Tetanus toxoid only C Tetanus immunoglobulin plus toxoid D Td vaccine plus antibiotics E Only antibiotics

D

A 19-year-old man presents with a 1-week history of malaise, sore throat, fever, and myalgia. The throat examination is remarkable for enlarged tonsils. There is a presence of bilateral tender anterior and posterior cervical lymphadenopathy. Abdominal palpation is significant for splenomegaly. Liver function tests are significant for elevated levels of ALT, AST, and alkaline phosphatase. Throat culture is normal and electrolytes are within normal limits. Heterophile antibodies are ordered and come back positive. A CBC is ordered and shows a WBC of 8,000/uL with 35% neutrophils, 45% lymphocytes, and 20% monocytes. Refer to the peripheral smear (see image). What type of cells are present in the smear? Answer Choices: A Hypersegmented neutrophil B Myeloblast C Eosinophil D Atypical lymphocyte E Neutrophil with toxic granulation F LE cell

D

A 20-year-old male college student presents with fever, chills, malaise, headache, photophobia and confusion with numerous petechiae on his extremities and trunk. On examination, he has positive Brudzinski and Kernig signs. A CT scan of the head is within normal limits. A lumbar puncture reveals increased leukocytes particularly polymorphonuclear neutrophils, increased protein and decreased glucose levels. Gram staining of the CSF reveals Gram-negative cocci in pairs. Question: What is the most likely causative agent for the infection in this patient? Answer Choices: A Bordetella pertussis B Francisella tularensis C Haemophilus influenzae D Neisseria meningitidis E Yersinia pestis

D

A 20-year-old man presents after experiencing an attack of syncope while playing soccer. It is the 1st time he has experienced such an attack, although he has felt a little bit dizzy while exercising before. He had a cousin who died suddenly at the age of 20. Except for a small cut-wound on his forehead, the patient looks healthy. He has normal vitals and a pansystolic murmur heard at the apex that propagates to the axilla. In addition, a short systolic murmur is heard inside the apex. ECG shows left ventricular hypertrophy. Echocardiography shows asymmetrical septal hypertrophy. Systolic anterior motion of the mitral valve SAM, moderate mitral regurge, and left ventricular outflow tract obstruction gradient=88mmhg. What is the best management of this patient? A Digitalis B Diuretics C Nitrate D Beta-blocker E Angiotensin converting enzyme inhibitor

D

A 20-year-old man spent his summer working in the forest on eastern shore of Maryland. A month after returning home to Virginia, he experienced a sudden onset of fever, nausea, muscle aches, and headaches. Blood and stool cultures were negative for obvious bacterial and parasitic pathogens. His symptoms subsided and he assumed he had recovered. 2 weeks later, the symptoms reappeared, now accompanied by marked splenomegaly. He also noticed a single, spreading rash with a pale halo surrounding a brighter red rash on the back of his right shoulder. It has been increasing in size and sensitivity. This time, a blood smear showed the presence of spirochetes. What is the MOST probable etiologic agent for the above illness? Answer Choices: A Spirocheta B Rickettsia C Treponema D Borrelia E Leptospira picture of a bullseye rash

D

A 20-year-old primigravida with 12 weeks of gestation presents with a 2-day history of low-grade fever and swelling in her neck. She does not have any significant medical history and denies previous blood transfusion. On questioning, she states that she is not sexually promiscuous and that she is living with her husband and their pet cat. She is a homemaker and spends her free time gardening. She has received all immunizations, and they are up-to-date. She also states that she has abstained from sex since learning of her pregnancy 2 months ago. On examination, her vitals are temp 99°F, PR 88/min, BP 110/70mmHg, and RR- 20/min. She also has painless, prominent cervical lymph nodes. Abdominal examination reveals a just-palpable uterus. Question: The fetus is at increased susceptibility to what infection? Answer Choices: A Infection by Treponema pallidum B Infection by Cytomegalovirus C Infection by Rubella D Infection by Toxoplasma gondii E Infection by Herpes simplex virus

D

A 21-year-old man is a recent immigrant from Somalia; he presents with back pain. The patient reports a dull aching pain localized to the T9 - T10 vertebral region. He describes intermittent pain for 6 months, and an episode earlier in the week of severe pain following heavy lifting. Physical examination shows tenderness from T9 - T12 with point tenderness at T10. The patient has normal vital signs, and laboratory measures are within normal limits. He denies fever, chills, and parenteral drug use. A myelogram shows a nearly complete block at T9 - T10, and an MRI reveals erosion of T9 and T10 vertebral bodies; there is a large surrounding paraspinal abscess. A tissue specimen obtained by biopsy reveals granuloma with necrosis. Gram stains of abscess drainage are uninformative. Auramine O staining and culture results confirm the diagnosis. What pathogen is most likely responsible for the patient's symptoms? Answer Choices: A Brucella melitensis B Actinomyces israelii C Mycobacterium leprae D Mycobacterium tuberculosis E Calymmatobacterium granulomatis

D

A 21-year-old man presents with acute onset of pleuritic chest pain accompanied by 2-3 days of fever, chills, arthralgias, and myalgias. Upon further questioning, the patient notes that 4 weeks ago he had a severe sore throat and fever but was not evaluated for these symptoms. Physical examination reveals a febrile patient in mild distress. A systolic murmur is noted in the left 4th/5th intercostal space that radiates to the left axilla. A friction rub is also appreciated on exam. Laboratory results reveal an elevated erythrocyte sedimentation rate (ESR) and antistreptolysin antibodies. Question: What is the most appropriate initial management of this patient? Answer Choices: A Tetracycline B Ciprofloxacin C Amantadine D Aspirin E Prednisone

D

A 27-year-old man presents with generalized seizures. You are informed that the patient was with his wife when he had a seizure in the kitchen of his home; he then had 2 more within the span of 30 minutes. On arrival, the paramedics administered 10mg of diazepam intravenously. En route to the emergency room, the patient suffered an additional seizure in the ambulance. His wife revealed a past medical history of tuberculosis; he is being treated with isoniazid. His physical exam demonstrates a postictal patient who is responsive to painful stimuli. His vitals are 150/110 mmHg, pulse 110/minute, respirations 24/minute, and temperature of 98.2 degrees Fahrenheit (36.6 degrees Celsius). His pupils are reactive to light and there is no nystagmus. His lung and cardiac examinations are normal. His abdomen is soft and nontender. There is no evidence of cyanosis or needle tracks, and his neurological exam is grossly intact. His pulses are good. Lab work includes: Hemoglobin 16 mg/dL Hematocrit 48% White cell count 18 x109/L Sodium 140 mEq/L Potassium 4.9 mEq/L Chloride 100 mEq/L Bicarbonate 7 mEq/L Blood Urea Nitrogen 20 mEq/L Creatinine 2.0 mEq/L ABGs 6.99 / pO2= 112 / pCO2= 45. His electrocardiogram demonstrates a sinus tachycardia without ectopy or ischemia. The chest X-ray is normal. The toxicology screen is pending. Sodium bicarbonate, along with benzodiazepine and phenytoin, is given to correct the acidosis. The seizure activity does not stop, and the patient is eventually intubated. Question: What should be administered to the patient? Answer Choices: A Administer calcium gluconate intravenously B Administer ethanol infusion intravenously C Administer glucagon therapy intravenously D Administer pyridoxine (vitamin B6) intravenously E Administer sodium thiosulphate intravenously

D

A 27-year-old man presents with the "flu." He says that he has felt feverish, tired, and mildly nauseated for the past few weeks. He mentions a headache and non-pruritic rash. He denies any past medical history or medication use. On exam, he is afebrile. Examination reveals diffuse mild lymphadenopathy with mild hepatosplenomegaly; his soft palate has a few scattered shallow ulcerations. The palmar and plantar surfaces have a scattered papular rash that is copper colored, with a few papules on the flexor surfaces of the arms, legs, and trunk. Laboratory evaluation reveals hemoglobin of 12.8 g/dL, hematocrit of 38%, and white blood cell count of 11.1 x 103/ìL. What is the most likely diagnosis? Answer Choices: A Influenza B Rocky Mountain spotted fever C Hand-foot-and-mouth disease D Secondary syphilis E Streptococcal pharyngitis

D

A 27-year-old woman who is 18 weeks pregnant states that her 3-year-old daughter was just diagnosed with erythema infectiosum (Fifth disease). An antibody test is ordered, and the woman is found to be seronegative. She is healthy and has no signs or symptoms of parvoviral infection. 2 weeks later, the test is repeated and the patient is IgM positive. With regard to the patient's fetus, what is the significance of this test result? Answer Choices: A The patient has developed protective immunity that will also protect the fetus. B There is a 75% chance that the fetus has or will become infected with the virus during the pregnancy. C The patient should undergo amniocentesis in order to determine whether the virus has crossed the placenta. D The patient should undergo serial fetal ultrasounds to monitor for the appearance of signs of hydrops fetalis. E The virus in incapable of crossing the placenta; therefore, the fetus is not at risk of becoming infected.

D

A 28-year-old woman with AIDS presents with malaise. Her CD4 count is unchanged at 350 cells/field, and her viral count is undetectable. She is afebrile and has a normal exam. She takes zidovudine, indinavir, potassium, hydrochlorothiazide, and glyburide. Work-up shows: Serum bicarbonate 20 meq/l Serum sodium 140 meq/l Serum chloride 100 meq/l Serum lactate 6 mmol/l Serum potassium 4.2 meq/l Whole blood glucose 85 mg/dl HemoglobinA1C 6.2% Hemoglobin 13.5g/dl( unchanged) Question: Which of her medications is likely causing her elevated lactate? Answer Choices: A Hydrochlorothiazide B Glyburide C Indinavir D Zidovudine E Potassium

D

A 3-year-old child presents with history of severe bouts of coughing for the last 10 days. During coughing his face becomes red. Cough started with a common cold but has become very severe for the last 10 days. The episode of cough often ends with a "whooping sound" during breathing and is often followed by vomiting. Child also has low grade fever off and on for the last 10 days. Immunization records of the child are not available. Blood count shows leukocytosis with lymphocytosis. Question: What is the antibiotic of choice for this child for treatment and prevention of spread of infection? Answer Choices: A Cotrimoxazole B Erythromycin C Rifampicin D Azithromycin E Ampicillin

D

A 3-year-old girl tells her parents that she has itching in what seems to be the perianal area. The itching is something that wakes her at night, and this is when she has complained most to her parents. Her pediatrician performs an anal swab, and microscopic examination reveals ova. Question: What is the most likely recommended treatment for this patient? Answer Choices: A Cefdinir (Omnicef) B Miconazole (Oravig) C Doxycycline (Vibramycin) D Mebendazole (Vermox) E Metronidazole (Flagyl)

D

A 30-year-old white man complains of a 3-day history of fever with chills and severe weakness. There are no other complaints. The patient has had multiple sex partners in the past. He also gives history of travel to South America and consumption of street food while working there 1-month back. He admits to intravenous drug abuse and cocaine abuse (snorting) in his early 20s. He often ventures out into the woods and has been bitten by several insects in the recent past. Abdominal exam reveals mild hepatomegaly. You send for routine lab investigations including CBC, comprehensive panel, and serology of HIV and Hepatitis B and C. Liver enzymes are elevated, and anti-HCV comes back positive. Anti-HIV and HBV are negative. Question: Which of the following is the most likely mode by which this man acquired the hepatitis C infection? Answer Choices: A Sex with multiple partners B Consumption of street food C Cocaine abuse D Intravenous drug abuse E Tick bite

D

A 32-year-old Hispanic woman presents with a 3-day history of vaginal pruritus. She is worried she has another infection. She was treated with penicillin 2 weeks ago for Group A streptococcus pharyngitis. The patient reports occasional sinus and urinary tract infections and always struggles with vaginal symptoms afterward. She has episodes of vaginal pruritus and thick white discharge approximately 3 times per year. She usually returns to her urgent care clinic for evaluation and treatment, then her symptoms resolve. She has no chronic medical conditions. She is non-obese and is not pregnant. On exam, the vaginal mucosa is inflamed and coated with thick, clumpy white discharge. On a wet mount slide treated with potassium hydroxide, you see pseudohyphae. The patient would like treatment for her current vaginal symptoms and advice in preventing her symptoms in the future. Question: What is the best intervention for prevention of the patient's current condition? Answer Choices: A Advise the patient to drink cranberry juice B Advise the patient to eat yogurt C Avoid prescribing antibiotics for this patient in the future D Offer fluconazole, if antibiotics are indicated in the future E Prescribe nitrofurantoin for use after intercourse

D

A 32-year-old man with no significant past medical history presented with dyspnea, palpitations, feelings of anxiety, and dizziness, all of which occurred earlier in the morning following a brisk walk. He denied any prior episodes, illicit drug use, alcohol or cigarette use, skipping meals, or caffeine intake. He further denied fever, chills, chest pain, history of murmurs, cough, edema, rashes, syncope, headache, psychiatric, or focal neurological complaints. The physical examination demonstrated a fast, regular pulse with a constant-intensity first heart sound, but was otherwise normal. An EKG was performed, which revealed a short PR interval plus a slurred upstroke at the beginning of the QRS complex. What is the most likely mechanism responsible for this patient's presentation? A Conduction delay in the proximal part of the right or left branches B Pre-excitation occurring via an atrio-His bundle C Spontaneous ectopy from muscular sleeves of pulmonary veins D Early excitation due to accessory pathways between the atria and ventricles E Inappropriately enhanced automaticity of sinus node pacemaker cells

D

A 32-year-old man with no significant past medical history presents to his primary care provider with a 2-month history of increased dyspnea upon exertion, which becomes apparent following walking 10 city blocks. He denies any other associated symptoms such as fever, chills, changes in weight, chest pain, abdominal pain, nausea, or vomiting. He further denies any history of cigarette smoking, occupational risk factors, sick contacts, or recent travel. His physical exam revealed normal vital signs and no distension of his jugular vein. However, there was a prominent right ventricular impulse along the lower-left sternal border associated with a palpable pulmonary artery and a midsystolic ejection murmur at the upper left sternal border that does not vary in intensity with respiration. There is a fixed split second heart sound. The remainder of his examination is normal. Following diagnostic testing, this patient was referred for surgical repair. What is the major long-term complication that requires monitoring following surgical repair? A Hypertension B Myocardial infarction C Mitral valve prolapse D Supraventricular arrhythmia E Transient ischemic attack

D

A 32-year-old woman presents to you for initial evaluation of her Human Immunodeficiency Virus (HIV) infection. Her first and only confirmed positive blood test for HIV was taken 2 months previously. She also had a CD4+ cell count and HIV viral load (VL) test performed at the time she was given her positive results. According to the Department of Health and Human Services, under which of the following situations is it recommended to defer initial treatment with antiretrovirals? Answer Choices: A 190 CD4+ cells/mcL, VL 29,000 copies/mcL; asymptomatic patient B 330 CD4+ cells/mcL, VL 180,000 copies/mcL; symptomatic patient C 670 CD4+ cells/mcL, VL 680,000 copies/mcL; symptomatic patient D 410 CD4+ cells/mcL, VL 14,000 copies/mcL; asymptomatic patient E 25 CD4+ cells/mcL, VL 150,000 copies/mcL; asymptomatic patient

D

A 33-year-old man presents for an initial visit at a new primary care office. He has not seen a health care provider at all in the past 5 years. His past medical history reveals a coarctation of the aorta repair at age 13, after which he saw a cardiologist yearly until age 18. Since then, he has not had insurance and has only sought care for urgent problems in acute care clinics. What is a common complication of coarctation for which primary care should regularly monitor this patient? A Anemia B Palpatations C Orthostatic hypotension D Hypertension E Hypertriglyceridemia

D

A 34-year-old man presents with a 2-day history of right ankle pain and swelling. He reports experiencing discomfort with weight bearing, ambulation, and when driving an automobile. On further questioning, he denies experiencing a recent trauma, although he does recall spraining his ankle approximately 1 year ago. On physical examination, the patient's temperature is 99.9° F. His right ankle shows swelling, is warm to palpation, and reveals an effusion. With passive range of motion of the right ankle, significant pain is elicited. Question: What is the most appropriate next step in the management of this patient? Answer Choices: A Check the serum uric acid level; if elevated, initiate therapy with indomethacin and colchicine. B Obtain a plain radiograph of the right ankle to assess for chondrocalcinosis. C Obtain a plain radiograph of the right ankle to assess for structural damage. D Perform arthrocentesis of the right ankle with analysis of the synovial fluid. E Treat with ibuprofen and have the patient return to your office in 1 week for follow-up.

D

A 37-year-old Caucasian man has presented with shortness of breath (SOB). Further history reveals that the patient has been extremely fatigued in the last few weeks, experiencing excessive night sweats, possesses a worsening cough, chest pain, as well as general aches and pains. The patient currently is not taking any prescribed medications and is allergic only to penicillin. He has a history of on-and-off intravenous drug use and admits to last using around 1 month ago. Along with an urgent inpatient admission, you are planning to initiate orders to have the patient undergo an echocardiogram and obtain blood cultures, among other actions. Based on the most likely diagnosis for this patient, which of the following pharmaceutical interventions is most appropriate? A Intravenous corticosteroids B Intravenous diuretics C Intravenous nafcillin D Intravenous cefazolin E Intravenous oxacillin

D

A 38-year-old HIV-positive man presents with fever, headache, right hemiparesis, and expressive aphasia. Recent CD4 count is 45, but he is not on any prophylactic medications. Computed tomography scan (CT) reveals multiple ring-enhancing lesions in the parietal and frontal lobes. What is the most likely cause? Answer Choices: A Cytomegalovirus (CMV) B HIV encephalopathy C Tuberculosis D Toxoplasmosis E Cryptococcosis

D

A 38-year-old male HIV positive patient with a history of IV drug abuse is brought to the emergency room complaining of severe headache and neck stiffness. The headache has been present for the last 3 days and has progressively worsened. The patient appears slightly disoriented and answers his questions slowly. A CBC is ordered, and the results come back unremarkable. Because of the patient's history and CNS symptoms, a lumbar puncture is performed. CSF is sent to the laboratory for examination and testing. While waiting for the results a quick test is done in the laboratory by the attending physician that confirms a diagnosis (refer to the image same one as above). What is the reagent used in this quick diagnostic test? Answer Choices: A Lactophenol Cotton Blue B Lugol's Iodine C Kinyoun Acid Fast Stain D India Ink E Gomori Methenamine Silver Stain (GMI)

D

A 38-year-old man presents with nausea, vomiting, diarrhea, and abdominal cramping for the past 2 hours. The patient reveals that he had consumed meat that had been left uncovered for a night. Laboratory examination revealed that the meat was contaminated with circular, clustered organisms, which were found on Gram staining to be Gram-positive. Fairly large, yellow colored colonies were formed when the organism was grown on mannitol agar media in a temperature range of 15 to 45°C. The organism showed positive results for coagulase enzyme activity and showed resistance to penicillin and methicillin antibiotics. Question: What organism was the causative agent of the food poisoning? Answer Choices: A Clostridium perfringens B Enterococcus faecalis C Escherichia coli D Staphylococcus aureus E Salmonella species

D

A 42-year-old man with a prior history of rheumatic fever as a child presents with the sudden onset of acute aphasia and a right hemiparesis. On auscultation of the heart, a diastolic murmur is heard over the mitral area. What is the most likely cause of the patient's neurological findings? A Endocarditis with septic embolization B Carotid dissection C Mural thrombus from an MI D Mitral stenosis with an atrial thrombus E Aortic dissection

D

A 42-year-old obese woman who is 5 days status-post laparoscopic cholecystectomy presents with right calf pain. Social history is positive for cigarette smoking and oral birth control. Vital signs: BP 130/80, HR 72, respirations 16, temp. 36C, EKG and chest x-rays are unremarkable. Venous duplex ultrasound is negative for deep venous thrombosis. V/Q scan shows a large perfusion defect in the right lung. In this postoperative period, what factors could possibly contribute to thrombosis? A Birth control pills, stasis, age B Obesity, stasis, intimal injury C Stasis, intimal injury, birth control pills D Intimal injury, stasis, hypercoagulability E Smoking, birth control pills, hypercoagulability

D

A 42-year-old white female delivers her third child while you are on call at a local hospital on the Atlantic coast. She thanks you for your help and mentions that she is grateful that her baby was on time. She states that the family is planning on taking their newest with them to Colorado in 2 weeks. You advise her not to attempt the trip, as the baby will have difficulty adjusting to the high altitudes. Which of the following factors is a major component in closure of the ductus arteriosus? A Increase in pulmonary arterial pressure associated with increase in blood flow to the lungs B Reduction of aortic arterial pressure due to reduction in systemic resistance C Decrease in carbon dioxide levels in the blood D Increase in oxygen tension within the blood E Presence of prostaglandin E2 in the wall of the ductus arteriosus

D

A 47-year-old man presents with dyspnea on exertion, palpitations, and a persistent cough. He had been generally healthy up until about a week ago, when he began complaining of shortness of breath and weakness. Physical examination of the chest reveals crackles at the bilateral bases and tachycardia. An echocardiogram shows a small mass within the right atrium. Vital signs are as follows: blood pressure 166/110 mm Hg, pulse 91 BPM, respirations 22/min, temperature 98.7°F. Biopsy reveals a malignant tumor. What is the most likely diagnosis? A Carcinoma B Leukemia C Myxoma D Sarcoma E Lymphoma

D

A 48-year-old man with hypertension and coronary artery disease is evaluated for protracted fever, fatigue, anorexia, weight loss, night sweats, and nonspecific, nonradiating joint pains, which began following a routine dental cleaning. His physical exam is remarkable for a fever of 101.3 ° F, oral mucosa, conjunctival petechiae, palpable purpuric skin rashes, reduced bilateral peripheral pulsations, linear subungual hemorrhages, small, flat, irregular erythematous spots on the palms and soles, and tender, erythematous nodules occurring in the fingers. His funduscopic examination was remarkable for cytoid bodies and hemorrhages while his cardiac exam demonstrated a soft, medium-pitched holosystolic murmur located at the apex with radiation to the axilla. A comparison to the patient's last physical exam reveals no abnormal physical exam findings. What is the next most appropriate step in the management of this patient? A Begin empiric treatment with IV ceftriaxone and gentamicin B Refer the patient for a cardiac MRI C Begin unfractionated intravenous heparin D Obtain specimen for blood cultures E Obtain a throat culture and anti-streptolysin O antibody levels

D

A 56-year-old male is hospitalized with sudden onset of symptoms of chest pain, sweating, palpitation and shortness of breath. ECG showed ST elevation of 3mm above isoelectric ECG line, and Troponin I of 6?g/L. His BP is 130/75mmHg and HR is 65bpm. The next morning, Doppler and Transesophageal echocardiography were performed following new onset of chest pain, shortness of breath and systemic hypotension, which established mitral regurgitation with papillary muscle rupture. What would be the most appropriate therapeutic intervention at this point? A Increase the dose of nitrates B Initiate treatment with a beta-blocker C Perform surgery to attach a ventricular assist device D Valvuloplasty and myocardial revascularization procedure E Postpone the intervention until acute phase is over

D

A 66-year-old man has a past medical history of amyloidosis, hyperlipidemia, and hypereosinophilic syndrome; he presents with a 1-year history of progressive dyspnea upon exertion. Currently, he is only able to walk 3 blocks before having to stop to catch his breath. Additionally, he has noticed increased lower extremity swelling, abdominal "bloating", and a loss of appetite. Despite not eating as much, he has noticed a 15-pound weight gain over the past 6 weeks. His physical exam reveals bipedal pedal edema, increased jugular venous pressure (with Kussmaul's sign noted), abdominal distension with shifting dullness, and hepatosplenomegaly. The precordium is without any heaves, lifts, or thrills. An EKG demonstrates a normal sinus rhythm without abnormalities. An echocardiogram is performed; it reveals the attached image. What pharmacotherapeutic agent would be most beneficial to this patient at this time? A Enalapril (Vasotec) B Digoxin (Lanoxin) C Cordarone (Amiodarone) D Furosemide (Lasix) E Apresoline (Hydralazine)

D

A 68-year-old man with a past medical history of diabetes mellitus type II, hypothyroidism, and hypderlipidemia presents with intermittent bouts of moderate to severe "squeezing, pressure, and tight" left-sided chest pain, which have been occurring over the last 3 months. Additionally, he admits to associated shortness of breath and nausea during these episodes, which are provoked upon exposure to the cold weather, during times of emotional stress, and following the consumption of a meal. Chest pain lasts approximately 10-15 minutes and resolves spontaneously. He denies diaphoresis, fever, chills, abdominal pain, reflux, regurgitation, diarrhea, cough, and pleurisy. What pharmacologic agent should be avoided in the management of this patient? A Sublingual nitroglycerin B Enteric coated aspirin C Atorvastatin (Lipitor) D Immediate release nifedipine (short-acting) E Metoprolol (Lopressor)

D

A 70-year-old woman presents with shortness of breath at rest over the past 3 days. She has found it difficult to walk short distances due to shortness of breath. Additionally, she is experiencing confusion, orthopnea, nocturnal dyspnea, and lightheadedness. She denies cough, fever, chills, diaphoresis, anxiety, chest pain, pleurisy, cough, nausea, abdominal pain, vomiting, diarrhea, rashes, and syncope. On physical examination, the patient is short of breath, requiring numerous pauses during conversation. She is afebrile; however, she is tachycardic, diaphoretic, and her extremities are cool. There is a diminished first heart sound, S3 gallop, laterally displaced PMI, bibasilar rales, and dullness to percussion and expiratory wheezing noted. An elevated JVD and 2+ pitting edema of the lower extremities is evident. What statement regarding this patient's condition is correct? A The most common etiology of this condition is infiltrative diseases B Increased caloric and sodium intake improves patient outcomes C Confusion is the most common presenting complaint in older patients D Activation of the renin-angiotensin-aldosterone system occurs E The electrocardiogram is the most useful diagnostic test

D

A 76-year-old man presents with progressive exertional dyspnea associated with substernal chest pain, easy fatigability, and dizziness. Symptoms are exacerbated with walking short distances, and they are relieved with rest. He denies fever, chills, cough, wheezing, pleurisy, calf pain, abdominal complaints, peripheral edema, cigarette, drug, or alcohol use, sick contacts, and travel. His physical exam reveals hypertension and a rough, harsh, low-pitched crescendo-decrescendo systolic murmur beginning after the first heart sound; it is best heard at the second intercostal space in the right upper sternal border. Its intensity is increased toward midsystole; the murmur radiates to both carotid arteries. It is accentuated upon squatting and reduced during Valsalva strain. What regarding the management of this patient is correct? A β-adrenergic blocker therapy is contraindicated in this patient B An increase in aerobic activity to improve conditioning is recommended C Maximum reduction of preload and afterload with ACE inhibitors is necessary D Surgical intervention provides the only definitive treatment E Bacterial endocarditis prophylaxis is required in patients with this diagnosis

D

A mother has brought her 6-month-old infant to your office for a 1-day history of poor feeding, lethargy, and weak cry. In the beginning, the infant had difficulty sucking and swallowing and was not opening his eyes. This was followed by loss of head control, weakness of the arms, trunk, and then legs. Infant is constipated and had drooling of saliva since yesterday. The infant was healthy before the onset of the above symptoms. There is no history of fever, vomiting, cough, seizures, or difficulty in breathing. The infant has weak gag and corneal reflexes, generalized hypotonia, loss of head control, ptosis, and diminished deep tendon reflexes. Blood counts and CSF examination are within normal limits. Question: What is the most likely clinical diagnosis? Answer Choices: A Myasthenia gravis B Spinal muscular atrophy C Guillain-Barre syndrome D Botulism E Tick paralysis

D

A young couple will be traveling through South America next month. They have asked the advice of a travel medicine clinic as to what kind of malaria prophylaxis they should use. The woman is 1-month pregnant. What statement about malaria is true? Answer Choices: A Since chloroquine is contraindicated in pregnancy, mefloquine should be used for prophylaxis B Chloroquine resistance to Plasmodium vivax is common C There are 3 species of organisms causing malaria D Malaria can be transmitted by blood transfusion E While malaria can develop up to several months after exposure, true relapse does not occur

D

What is a likely cause of chronic meningitis in a patient with AIDS? Answer Choices: A Candida B Mycobacterium avium C Cryptosporidium D Cryptococcus neoformans E Actinomyces

D

Which of the following diagnostic studies is indicated for a patient with amaurosis fugax? A CT of the head B Intraocular pressure C Temporal artery biopsy D Carotid ultrasound E Ocular fluorescein angiogram

D Carotid Ultrasound Amaurosis fugax is a monocular vision loss that appears like a curtain passing over the eye, and comes from carotid artery disease. A CT of the head is indicated for lateralizing stroke symptoms. Intraocular pressure is taken for evaluation of chronic or acute glaucoma. A temporal artery biopsy is taken if giant cell arteritis is suspected. An ocular fluorescein angiogram is done to evaluate retinal disorders.

A 24-year-old female, with a history of type 2 diabetes, presents with the inability to conceive after 14 months of unprotected sexual intercourse with her husband. Vital signs are as follows: Temperature 37˚ C, BP 115/85, P 65, RR 20. Her BMI is 31. Which physical examination finding would you expect to observe with your suspected diagnosis? A Undeveloped breasts B Palpable uterus on abdominal exam C Acromegaly D Coarse dark hair on the face, chest, and back E Egophony on lung exam

D Coarse dark hair on the face, chest, and back Polycystic ovarian syndrome (PCOS) affects 5-10% of reproductive age women. It is associated with hirsutism (50% of cases) (D), obesity (80% of cases), and virilization (20%), and those affected have an increased risk of diabetes mellitus, cardiovascular disease, and metabolic syndrome. PCOS patients are often infertile. A palpable ovary may be noted on thin individuals with enlarged ovaries, not the uterus (B). Undeveloped breasts are not a diagnostic indicator for PCOS (A) and acromegaly (C) is seen with Marfan's Syndrome. Egophony (E) is when there is consolidation of the lung on exam.

An 18-year-old G1P0 presents for evaluation of her amenorrhea. Her LMP was 2 months ago. Her HCG is positive, and she has been having vaginal spotting x 2 weeks. Her ultrasound reveals an intrauterine heterogeneous echogenic mass, without fetus or placenta. What is the most likely diagnosis? A Threatened abortion B Missed abortion C Ectopic pregnancy D Hydatidiform mole E Early intrauterine pregnancy

D Hydatidiform mole D The classic signs for a mole are a heterogenous mass without the placenta or fetus, and vaginal spotting is present all of the time. The presence of the mass intrauterine without fetus or placenta rules out ectopic and IUP, and therefore threatened abortion. Missed abortion is not echogenic.

Which of the following statements about anemia associated with CKD is TRUE? A Iron and folic acid by mouth are the most effective treatments. B Transfusion of packed red blood cells monthly is the most effective treatment. C IM erythropoietin given monthly is the most effective treatment. D It is due to the inability of the kidney to transform erythropoietin into its physiologically active form. E It occurs early in the course of CKD.

D It is due to the inability of the kidney to transform erythropoietin into its physiologically active form. Anemia associated with CKD is the result of inadequate erythropoietin synthesis by the kidneys. This hormone signals the bone marrow to synthesize red blood cells. A deficiency will result in anemia. In the absence of erythropoietin, iron would not be of use since red blood cell synthesis is inadequate. Folic acid would also not be of use and does not play a role in the etiology of this type of anemia. Transfusion is a tempering measure only, used to increase oxygen-carrying capacity in the case of symptomatic ischemia. Anemia due to erythropoietin deficiency generally does not occur until the GFR decreases to <60 mL/min, or approximately 50% of normal. Intramuscular administration of erythropoietin is the only effective treatment to induce red blood cell production. Depending on the formulation used, this can be given once a week or once every 2 weeks. Oral iron supplementation is needed to produce adequate hemoglobin for the increased de novo red cell production.

A 27-year-old G 0 P 0 woman has been trying to become pregnant for the past 10 months. Menarche occurred at age 13 and her cycles have "always been irregular" except during the 10-year period when she took oral contraceptives. Since stopping contraception two years ago, she and her partner of six years have used condoms consistently until they desired pregnancy. Neither partner has a history of sexually transmitted infection, and both have been mutually monogamous. On physical examination she is 64" tall and weighs 189 pounds. She has increased dark hair on her upper lip and chin and on her lower abdomen and says she has had this hair distribution as long as she can remember. Of the following, what is the most likely cause of her apparent infertility? A Androgen-secreting ovarian neoplasm B Hypothyroidism C Pituitary adenoma D Polycystic ovarian syndrome E Premature ovarian failure

D Polycystic ovarian syndrome D Patients with polycystic ovarian syndrome typically are obese, infertile, hirsute, and have menstrual disorders, making this the most likely of the diagnoses. An androgen-secreting ovarian neoplasm (A) would cause infertility, but the associated hirsutism is rapidly progressing. Hypothyroidism (B) may cause amenorrhea and weight gain and contribute to infertility, but the overall picture is more suggestive of PCOS. The hyperprolactinemia associated with pituitary adenoma (C) might also produce mild hirsutism, but other symptoms such as galactorrhea would also likely be present. Premature ovarian failure (E) is the spontaneous cessation of menses prior to age 40, and does not apply to this woman.

A woman presents for a routine post-partum checkup four weeks after delivery of her child. She is currently breast feeding without difficulty. She tells you that she has been feeling anxious and very warm, despite the change of season into winter. She is happy as a mother and has not had problems caring for her baby. What is the next best step in the investigation of her symptoms? A Radioactive iodine uptake test B Serum total T4 level C Thyroid fine needle aspiration D Serum TSH level E MRI of the anterior pituitary

D Serum TSH level The correct choice is D, serum TSH level. This patient is presenting with post-partum thyroiditis. Thyroid dysfunction occurs in 2 to 5% of women after giving birth. It can recur with subsequent pregnancies and develop into long-term thyroid disease as well. Typically, women with this disorder first develop signs and symptoms of hyperthyroidism, which then later changes to hypothyroidism. Most symptoms resolve spontaneously within a few months. In the hyperthyroid stage of this disorder, blood tests will reveal a suppressed TSH level with elevated serum thyroid hormone levels, as is common in all forms of primary hyperthyroidism. Choice A, a radioactive iodine uptake test, would reveal little or no uptake, but this test should not be ordered in a woman who is breastfeeding. For choice B, a serum total T 4 level can be elevated, but is not as sensitive or specific as TSH as a screening test. Any protein status changes in the woman can cause an elevated total T 4 without any thyroid dysfunction. Choice C, a thyroid fine needle aspiration, can be performed, but is not likely necessary and would not be the best next step for this patient. Choice E, an MRI of the anterior pituitary, would only be suggested if there is a suspicion of a pituitary tumor. This is a rare cause of hyperthyroidism and would not be the best next step.

Your patient is a 4-year-old boy whose father brings him to your primary care office concerned about behavioral problems. He cannot seem to sit still in school. He seems to have a great deal of difficulty focusing for any length of time. Which of the following is required in order to make the diagnosis of attention deficit/hyperactivity disorder (ADHD) according to the DSM-IV-TR? A He exhibit more symptoms of inattention than hyperactivity B He must be at least 10-years-old C He behaves aggressively towards others on at least some occasions D Some impairment from the symptoms is present in two or more settings E Symptoms of inattention be present on a daily basis for 3 months

D Some impairment from the symptoms is present in two or more settings D The diagnosis of ADHD requires symptoms of both hyperactivity/impulsivity and of inattention (D). It does not require more or more prominent symptoms (A) of one or the other. Aggression (C) is seldom involved, although the impulsivity may manifest as lack of concern for others. The symptoms must be present before the age of seven (B) and must have been present for at least 6 months (E).

A mother expresses concern for her teen son after feeling a lump in his neck. He has no history of trauma to his neck. Surgical history is negative, and the patient does not take any medications. The mother tells you that thyroid problems run in the family. The patient has not been ill recently. Upon exam you feel a nontender, firm nodule on the right side of his thyroid with associated cervical lymphadenopathy. His serum TSH level is within the reference range. Radionuclide thyroid scanning demonstrates a "cold" nodule in the right side of the thyroid. What is the most appropriate next step in the work up of this patient? A MRI of the anterior pituitary B CT of the thyroid C MRI of the thyroid D Thyroid nodule fine needle aspiration E Emergent thyroidectomy

D Thyroid nodule fine needle aspiration The correct choice is D, thyroid nodule fine needle aspiration. With the advent of fine needle aspiration, it has become much easier, safer, and more reliable to obtain a specimen for biopsy. This patient has several characteristics that increase his risk of malignancy including his gender, young age, firmness of the nodule, and related lymphadenopathy. These, along with the ease of biopsy, suggest this path for diagnostic work up. Not enough information is known to warrant an emergent thyroidectomy, choice E. Choice A, MRI of the anterior pituitary, would be warranted if there was a suspicion of a pituitary cause of the thyroid nodules. Since the TSH is normal and the patient is not presenting with headaches or other pituitary related symptoms, this is not suggested. Choice B, CT of the thyroid, and choice C, MRI of the thyroid, would not provide any additional information after the thyroid scan. They may be helpful prior to any surgery if needed.

Which of the following findings is consistent with a lower motor neuron deficit? A aphasia B dysdiadochokinesia C sensory loss D weakness E hyperreflexia

D Weakness Weakness is one potential finding of a lower motor neuron process. Hyporeflexia is also common. Aphasia results from injury to the speech pathways within the brain. Sensory loss arises from many causes, but it is not a motor issue. Dysdiadochokinesia is consistent with cerebellar pathology. Hyperreflexia is typically a signal of upper motor neuron disease.

A progestin-only contraceptive, or "minipill," would be most appropriate for which of the following patients? A a 25-year-old woman in excellent overall health B a 28-year-old woman with a history of epilepsy C a 32-year-old woman with a history of pelvic inflammatory disease D a 37-year-old woman who smokes 2 packs per day and has a history of hypertension E a 38-year-old woman with a history of asthma and bronchitis

D a 37-year-old woman who smokes 2 packs per day and has a history of hypertension D In the majority of cases, a combined hormonal contraceptive (ie, one that contains both an estrogen and progestin) is the preferred method of oral contraception because of its efficacy when used perfectly (>99%). However, for women older than 35 years of age who are smokers or are obese, or who have a history of hypertension or vascular disease, progesterone-only contraceptives are recommended. Ethinyl estradiol (EE), the most common estrogen found in combined hormonal contraceptives, has been associated with an increased risk of myocardial infarction in women older than 35 years of age who are smokers. Additionally, EE has also been shown to cause increases in blood pressure in both normotensive and mildly hypertensive women. Progestin-only contraceptives, however, tend to be less effective than the combined hormonal contraceptives

A patient with no history of treatment for primary syphilis presents with symptoms and signs consistent with secondary syphilis. The most common sign of secondary syphilis is A generalized lymphadenopathy B aseptic meningitis C alopecia D generalized maculopapular rash E superficial painless gummas

D generalized maculopapular rash Secondary syphilis generally manifests itself a month or two after appearance of the primary chancre. Patients will complain of headache, fever, sore throat, and malaise and will exhibit generalized lymphadenopathy along with a maculopapular rash that begins at the sides of the trunk and later spreads over the rest of the body. The skin lesions may coalesce in warm moist areas, such as the perineum, and form large, flat-topped, pale papules termed condyloma lata. Skin and mucosal lesions are the most common signs of secondary syphilis. Aseptic meningitis and alopecia may also occur in secondary syphilis. Formation of granulomatous nodules (gummas) is not a feature of secondary disease, but rather is the hallmark of tertiary syphilis.

A young man presents with difficulty breathing at times. Upon exam you note evidence of a firm, fixed thyroid nodule with extension toward the trachea and surrounding muscles. The patient has a family history of thyroid cancer. You are concerned that the patient may have medullary thyroid cancer. After you confirm your suspicion, which of the following genetic tests would you suggest to his family members? A BRCA 1 B APC C CFTR D ret proto-oncogene E CAG triplet expansion

D ret proto-oncogene The correct choice is D, ret proto-oncogene. Genetic testing to identify this oncogene is available for patients whom you suspect have medullary thyroid cancer and/or pheochromocytoma, and for their asymptomatic families who may be at risk for the same disorders. A mutation in this gene leads to uncontrolled growth of the C cells of the thyroid. Choice A, BRCA 1, is an oncogene that is associated with risk for breast, ovarian, colon, and prostate cancers. Choice B, APC, is an oncogene noted in the familial colon cancer syndrome known as familial adenomatous polyposis. DNA analysis for CFTR mutations, choice C, can identify the gene mutations in patients with cystic fibrosis in the vast majority of instances. Choice E, CAG triplet expansion, is the result of a gene mutation in Huntington's disease.

A 37-year-old woman under your care is diagnosed with bipolar I disorder. As part of her drug regimen, you prescribe lithium carbonate as long-term maintenance therapy. Which of the following would be most appropriate to perform or order prior to the initiation of lithium carbonate? A electrocardiogram B fasting plasma glucose C liver function tests D serum creatinine E urine culture

D serum creatinine Patients on chronic lithium carbonate therapy have an approximate 10% to 20% risk of developing renal problems such as glomerulosclerosis, tubular atrophy, or interstitial nephritis. Each of these conditions can lead to filtration problems and a subsequent rise in serum creatinine. Hence, it is advised to obtain a baseline serum creatinine prior to administering lithium carbonate to follow any changes that may occur in renal function during therapy. It is also advised that lithium carbonate be avoided in patients with pre-existing renal disease.

A 35-year-old man presents with the chief complaint of "peeing a lot." The patient states for the last week, he has urinated at a rate of 4-5 times an hour; he even awakens at night to urinate and states this has severely interfered with his sleeping habits. He further describes significantly intense thirst and a powerful craving for only ice water to quench this thirst, although he never feels that this is accomplished. Pertinent past medical history includes a diagnosis of bipolar disorder at the age of 22; he takes lithium daily for this diagnosis. He is not taking any other medications regularly at this time. Question Given the medical history provided, the patient is most likely experiencing which of the following? Answer Choices 1 Cushing's disease 2 Hypothyroidism 3 Diabetes mellitus 4 Diabetes insipidus 5 Hypoparathyroidism

DI Explanation This patient is suffering from diabetes insipidus (DI). Characteristics of this disease process include an increase in thirst and the passage of large quantities of urine. The underlying pathologic feature of DI is a deficiency of vasopressin or a resistance to available vasopressin. There are a multitude of disease states that lead to DI and the patient above is most likely suffering from an acquired form of vasopressin-resistant diabetes insipidus. Disease states that lead to this form of DI can include pyelonephritis, renal amyloidosis, potassium depletion, Sj?gren's syndrome, use of some medications, and, in the patient's case, the use of lithium. Chronic use of lithium can lead to lithium toxicity and manifests as described in the patient scenario. This is one of the more common causes of DI in adult patients. Hypothyroidism is much lower on the differential due to the signs and symptoms being inconsistent. Findings such as weakness, fatigue, cold intolerance, constipation, weight changes, depression, or menstrual abnormalities would be some of the multiple features evident in patients with hypothyroidism. Hypoparathyroidism is also not as likely. Signs and symptoms of the acute phase of this disease include tetany, muscle cramps, irritability, altered mental status, and convulsions; a tingling sensation around the circumoral area, hands, and feet is almost always present. Cushing's disease is due to manifestations of hypercortisolism secondary to adrenocorticotropic hormone (ACTH) hypersecretion by the pituitary gland. Some features include but are not limited to: central obesity, apparent moon face or buffalo hump, purple striae, muscle wasting, hirsutism, psychological changes, hypertension, weakness, headache, backache, or poor wound healing. None of the signs or symptoms listed matches the patient's history above. Diabetes mellitus (DM) may also be considered in this patient scenario given the symptoms of polydipsia and polyuria described, as well as the strong family history evident. The main cause of this type of diabetes is that patients develop increased tissue insulin insensitivity, also termed insulin resistance. Some key components of the patient's history are not evident in diabetes mellitus. Nocturnal enuresis is not usually a significant chief complaint in patients with DM, although patients with DM may have complaints of polyuria, polydipsia, weakness, fatigue, blurred vision, vulvovaginitis, peripheral neuropathy, or even potentially be asymptomatic.

For what childhood endocrine and metabolic disorder is the appropriate pharmacologic test (stimulation or suppression) water deprivation? Answer Choices 1 Hypothyroidism 2 GH deficiency 3 Cushing's disease 4 Diabetes insipidus 5 Thyroid medullary carcinoma

DI The water deprivation test can establish the abnormally low osmolality when fluid intake is restricted, and it can document an increase of osmolality in response to long-acting analogue of ADH. These findings confirm the diagnosis of ADH-deficient diabetes insipidus.

A 45-year-old woman comes to the office for evaluation of "yeast infection again." Review of the chart shows that she has had six episodes of vaginal candidiasis in the past 18 months. She and her husband have been mutually monogamous during their 23-year marriage; he had a vasectomy following the birth of their second child. Her two children weighed 10# 2 oz. and 10# 4 oz. at delivery. Her last menstrual period occurred one week ago and was slightly heavier than normal, but her preceding two periods were scanty and "off schedule." Her medications include a thiazide diuretic for mild hypertension and a senior multivitamin. She is 62" tall and weighs 198#. Assuming her examination demonstrates candidiasis, this woman needs most urgently to be screened for which of the following? A diabetes mellitus B hyperlipidemia C hypothyroidism D perimenopause E polycystic ovarian syndrome

DM Diabetes causes recurrent vaginal candidiasis in women with uncontrolled glucose. Type 2 diabetes is further suggested by her obesity and history of macrosomia in offspring. Furthermore, thiazide diuretics may increase glucose levels. She needs also to be screened for hyperlipidemia (B), given her weight and probable diabetes, but that is not as urgent. She may be hypothyroid (C), although the clinical picture more strongly suggests diabetes; hypothyroidism is also more likely to cause menorrhagia than scanty periods. She may also be perimenopausal (D), given irregular menses. Polycystic ovarian syndrome (E) is unlikely given no history of infertility.

A 47-year-old Caucasian woman presents for evaluation of acute abdominal pain. She was brought in by her son, who reports the patient had not been eating or drinking well for several days. Further history, exam, and imaging studies were performed. The patient was pre-hydrated with sodium bicarbonate, had an abdominal CT with IV contrast, and was later admitted to the medical floor with a provisional diagnosis of diverticulitis. Her past medical history is significant for diabetes mellitus diagnosed 12 years ago and hypertension. Both conditions were reported to have been under good control. Her medications include regular and long-acting insulin and hydrochlorothiazide/lisinopril 25/20 mg QD. She has recently been taking 800mg ibuprofen BID-TID for her abdominal pain during the last week. She has no known allergies. While in the hospital, the patient's laboratory results are followed daily. 2 days after the CT with contrast, it is noted that her serum creatinine has risen to a level of 3.5 mg/dL. Records from 1 month ago at her family physician showed her labs to include a hemoglobin A1C of 6.8%, creatinine of 1.2 mg/dL, GFR of > 60 mL/min/1.73 m2, and blood pressure of 127/78. Question Which of the following is a major risk factor for this patient's sudden decline in renal function? Answer Choices 1 Baseline GFR > 60 mL/min/m2 2 Caucasian race 3 Diabetes mellitus 4 Pre-hydration with sodium bicarbonate 5 Younger age

DM Explanation Diabetes mellitus is a major risk factor for renal impairment after administration of IV contrast. In one study, DM had an odds ratio of 5.47 for development of contrast nephropathy. A baseline GFR > 60 mL/min/m2 estimates nearly normal creatinine clearance. Individuals with significantly reduced GFRs are at high risk for contrast nephropathy. Because creatinine clearance is inversely related to the creatinine level, this patient's lower creatinine level also supports less risk for the contrast nephropathy. However, it was not enough to offset the risk from her diabetes. Caucasian race is not considered a major risk factor in developing contrast nephropathy. Race is not a major factor in the development of contrast nephropathy. However, if any race is implicated for higher risk, it would be African Americans. Pre-hydration with sodium bicarbonate is a means of reducing risk, rather than a major risk factor, for contrast nephropathy. Pre-hydration seems to confer a protective effect, preventing hypotension and decreased renal blood flow. The sodium bicarbonate has been suggested in many studies to be more effective than traditional saline (sodium chloride). Younger age is not a risk factor for contrast nephropathy. Elderly age is a risk factor, with particular concern arising in patients 75 years of age and older.

A 45-year-old woman presents with vaginal itching and irritation that did not resolve with over-the-counter medications. She has not previously sought medical attention for the symptoms. During review of systems, she admits to polyuria, polydipsia, and occasional dysuria, which she attributes to the vaginal irritation. She states she has not had lab work done in over a year, but the last time she did was at a health fair; she was told her fasting glucose was 119 mg/dl. On exam, you note that the patient is 62 inches tall and weighs 243 pounds. Her vital signs are within normal limits except for her blood pressure, which is 138/84 mm Hg. You find that she has white discharge in the vaginal canal; there is beefy-red surrounding skin and similar irritation under her breasts. Question Based on this patient's presentation, what do you suspect is the patient's most serious condition? Answer Choices 1 Obesity 2 Hypertension 3 Sexually transmitted infection 4 Diabetes mellitus 5 Mucocutaneous candidiasis

DM Explanation One of the most common initial manifestations of diabetes mellitus type II in women is candidal infections. Polyuria and polydipsia are also common initial complaints. The patients often do not have ketonuria and weight loss at diagnosis. Many patients are totally asymptomatic. Most people are diagnosed by routine lab work rather than presenting with symptoms. People who are over the age of 40 as well as obese are more likely to develop this disease. Hypertension, dyslipidemia, and atherosclerosis are often associated as well.

A 62-year-old obese woman presents with progressive numbness and tingling in her feet for the past 3 months. On physical examination, the patient is found to have decreased sensation to pinprick and vibration, absence of ankle reflexes, and difficulty with tandem walking. Which is the most common etiology of her symptoms? A diabetes mellitus B alcoholism C vitamin B12 deficiency D spinal cord tumor E rheumatoid arthritis

DM Peripheral neuropathy is a syndrome that is manifested by muscle weakness, paresthesias, decreased deep tendon reflexes, and autonomic disturbances most commonly in the hands and feet, such as coldness and sweating. There are many causes of peripheral neuropathy ranging from metabolic conditions to malignant neoplasm, rheumatoid arthritis, and drug and alcohol use. The increase in non-insulin-dependent diabetes mellitus due to obesity in the American population has increased the incidence of associated disease states

A 54-year-old woman presents to her primary care physician due to persistent fatigue. She notes feeling more thirsty than usual, and she has been getting up several times throughout the night to urinate. Her physician obtains the following labs: Fasting blood glucose: 170 mg/dL 2 hour oral glucose tolerance test: 230 mg/dL Hemoglobin A1C: 9% Question What is the most likely diagnosis in this patient? Answer Choices 1 Type I diabetes mellitus 2 Type II diabetes mellitus 3 Impaired glucose tolerance 4 Diabetes insipidus 5 Diabetic ketoacidosis

DM t2 Explanation The above patient is most likely suffering from type II diabetes mellitus. The diagnostic criteria for diabetes includes a fasting blood glucose greater than or equal to 126 mg/dL, 2 hour oral glucose tolerance test with blood glucose value greater than 200, and a hemoglobin A1C level greater than or equal to 6.5%. The patient's polydipsia and polyuria are also suggestive of diabetes mellitus. Type I diabetes mellitus could also present with the above history and laboratory findings. However, type I diabetes mellitus typically presents in young adulthood, not in middle age. A patient is considered to have impaired glucose tolerance if their fasting blood glucose is 100-125 mg/dL, their 2-hour oral glucose tolerance test is 140 - 199 mg/dL, and their hemoglobin A1C is 5.7-6.4%. The above patient's laboratory values exceed this range. Though a patient with diabetes insipidus would complain of polyuria and polydipsia, they would not have the glucose and hemoglobin A1C findings seen in this patient. While a patient with diabetic ketoacidosis might present with history of polyuria and polydipsia, they would have a higher blood glucose level. The patient might also have associated nausea, vomiting, mental status changes, and a fruity breath odor.

A 14-year-old boy presents with worsening shortness of breath; it most often occurs when he plays soccer. He often awakens in the middle of the night due to 'attacks': he starts to feel anxious because he feels like he cannot breathe, and he experiences chest tightness. He suffers from a dry cough, especially after playing sports. His mother has put a humidifier in his room and has him use his sister's inhaler, which seems to help temporarily; the boy has been using it 5 - 6 times daily. The family history is significant for asthma in his sister, father, and 3 other paternal relatives. The patient and his mother are not aware of any allergies. He denies fever, chills, and chest pain. In between 'attacks', he feels well and normal. The patient's past medical history is noncontributory. There are no known medical conditions; he has no drug allergies, and he has not had any surgeries. Other than the aforementioned inhaler, he does not take any medications. Question In addition to his own albuterol inhaler, what medication should be prescribed for this patient? Answer Choices 1 Burst of oral prednisone 2 Daily inhaled salmeterol 3 Daily low-dose inhaled budesonide 4 Daily oral zileuton 5 Subcutaneous injections of omalizumab

Daily low-dose inhaled budesonide This patient is exhibiting persistent asthma symptoms. Using the step-wise approach to treatment, the 2nd step (after using a short-acting beta-agonist, such as albuterol) is to add a low-dose inhaled corticosteroid. Alternatives include cromolyn, a leukotriene receptor antagonist, nedocromil, and theophylline. For this patient, daily low-dose inhaled budesonide would be most appropriate. He should then be monitored for response; if necessary, the medication should be adjusted. The patient and his mother should receive patient education on asthma, the use of peak flow meters, and the proper use of medications. A burst of oral prednisone can be useful in management of an acute exacerbation of asthma. However, due to the side effects of systemic steroids, its use is discouraged. This patient is not in acute distress and should start with inhaled steroids, which pose fewer side effects than oral/systemic steroids. Daily-inhaled salmeterol, a long-acting beta-agonist (or LABA), should be added to an inhaled corticosteroid if low-medium doses of the inhaled steroid alone are unable to control symptoms. The salmeterol is not recommended as a stand-alone therapy, and it should only be used with other asthma control medications. Daily oral zileuton, a 5-lipoxygenase inhibitor, is extremely expensive and is dosed 4 times daily. It should not be recommended initially when the preferred agent (low-dose inhaled corticosteroid) has not yet been tried. Additionally, compliance in teens can be difficult, so a 4-times-daily medication is not ideal. Subcutaneous injections of omalizumab, which is an immunomodulator, are an option in the step 5 and 6 patients, who have failed to achieve symptom control with multiple and high-dose medications. Omalizumab is not appropriate in this patient's case.

A 34-year-old trauma patient in the ICU is in a lot of pain, so you order 100mg of meperidine hydrochloride every 4 hours. What will the patient have to be closely monitored for? Answer Choices 1 Bowel function 2 Increase in blood pressure 3 Increase in heart rate 4 Increase in respiration 5 Decrease in respiration

Decrease in respiration Meperidine hydrochloride (Demerol) is a CNS depressant and could therefore decrease respiration and heart rate. It binds with opiate receptors, altering perception of and emotional response to pain through an unknown mechanism. If there were an effect on blood pressure, it would be hypotension, especially after rapid i.v. injection, and not an increase in blood pressure. As with all narcotic analgesics, there is the danger of constipation when administered over a longer period of time, but there is no need to monitor bowel function closely. Other side effects of meperidine are shown in the following table. Central nervous system: Sedation, euphoria, dizziness, tremor, headache, hallucination, seizures, light-headedness Gastrointestinal: Nausea, vomiting, dry mouth, constipation, ileus, biliary tract spasms Cardiovascular: Bradycardia, tachycardia, hypotension, cardiac arrest, shock Respiratory: Respiratory depression, respiratory arrest Urogenital: Urine retention Musculoskeletal: Twitching Skin : Pruritus, urticaria, increased perspiration Other: Physical dependence, pain at injection site, induration, increased pancreatic enzyme levels Meperidine is contraindicated in patients with known hypersensitivity to the drug or who have been treated with MAO-inhibitors within the past 2 weeks.

A 28-year-old woman presents for a routine follow up. She is a type I diabetic and is on a multiple daily insulin injections regimen. At her last visit, she mentioned having difficulties with her morning glucose levels. She has been tracking her glucose levels daily. Her pre-breakfast glucose averages 285 mg/dL. At dinnertime, it averages 95 mg/dL, and it averages 68 mg/dL at 3 AM. Question What is the best change to make in her treatment regimen? Answer Choices 1 Increase the evening dose of long-acting insulin 2 Decrease the evening dose of long-acting insulin 3 Decrease the evening dose of short-acting insulin 4 Increase the evening dose of short-acting insulin 5 Increased activity before bedtime

Decrease the evening dose of long-acting insulin Explanation Since her 3 AM level was low, that indicates the patient's early morning hyperglycemia is due to the Somogyi effect. The evening dose of long-acting insulin should be decreased, or she should eat bedtime snacks. 3 mechanisms can account for early morning hyperglycemia in patients with diabetes on insulin: An inadequate dose of long acting insulin at night can be a cause. The Somogyi effect, or rebound hyperglycemia, is a condition often seen in patients with type I diabetes and nocturnal hypoglycemia. The resulting hypoglycemia stimulates the release of counter regulatory hormones leading to increase of glucose levels in the blood. Diagnosis can be made by several plasma glucose analyses (done at 2 AM, 3 AM, and 7 AM). The dawn phenomenon, where the hyperglycemia is secondary to an increased need for insulin in the early morning due to normal early morning surge of growth hormone, which antagonizes insulin action. Checking the blood glucose level at 3 AM can help differentiate between the Somogyi effect and dawn phenomenon: the level will be low in the Somogyi phenomenon. The treatment of the dawn phenomenon involves increasing the evening dose of long-acting insulin. There is no nocturnal hypoglycemia if the night dose of insulin is inadequate.

What would the tactile fremitis be in a Pneumothorax/hemothorax?

Decreased

Your patient is a 45-year-old woman concerned about the accelerated aging of her skin. 5 years ago, she underwent a total hysterectomy with oophorectomy. Since then, she gained about 20 pounds. She also has diabetes mellitus type II. On examination, you find a slightly overweight female (body mass index 26) with skin that has lost its elasticity and has reduced water-holding capacity, increased pigmentation, and decreased vascularity. Question What are her symptoms signs of? Answer Choices 1 Elevated testosterone 2 Decreased testosterone 3 Elevated estrogen 4 Decreased estrogen 5 Insulin resistance

Decreased estrogen Explanation Lower levels of estrogen are associated with skin aging, most probably because of telomeres shortening. The effects of reduced estrogen levels cause loss of elasticity, reduced water holding capacity, thickness, increased pigmentation, decreased vascularity in the skin, and facial hair. Some of those signs are found in your patient. Your patient has no ovaria, and testosterone is primarily secreted in the ovaries (small amounts are also secreted by the adrenal glands). Too much testosterone will cause excessive oiliness and acne rather than accelerated aging of the skin. Testosterone levels in this woman are decreased after the removal of the uterus and ovaries. Symptoms related to her skin are not characteristics of decreased testosterone levels. More often in decreased testosterone there will be a loss of muscle strength and mass, accumulation of fat, especially around the abdomen, osteoporosis, vaginal dryness, etc. High levels of estrogen can be found in women who are pregnant, extremely overweight, have diabetes, or have liver disease. They are sometimes associated with acne, red flushed appearance, spider nevi, etc. Skin changes in this patient cannot be attributed to high estrogen. Insulin resistance is a condition when insulin becomes less effective at lowering blood sugars. It can lead to weight gain and estrogen hyperproduction, resulting in skin changes attributable to high estrogen levels (acne, red flushed appearance, spider nevi, etc). Skin changes in this patient cannot be attributed to the high estrogen.

A 48-year-old female presents with a gradual progression of fatigue, pallor, and dyspnea on exertion over the last few months. Initial CBC results show a hemoglobin of 10.2 mg/dL, hematocrit of 31%, an MCV of 74 fL, and a reticulocyte count of 0.1%. Which of the following is the most likely pathophysiologic mechanism responsible for her anemia? A Acute blood loss B Defective bone marrow/stem cell function C Defective DNA production D Defective hemoglobin production E Increased destruction of red blood cells

Defective hemoglobin production The patient most likely has an underlying iron deficiency anemia (IDA), which requires confirmation and then evaluation for sources of chronic blood loss. IDA results from defective hemoglobin production. Acute blood loss (A) would present more acutely and with normal hemoglobin, hematocrit, and MCV until hemodilution occurs and lowers the hemoglobin concentration and hematocrit. Defective bone marrow/stem cell function (B) tends to produce normocytic red blood cells. Defective DNA production (C) results in failure of RBC maturation and macrocytosis (elevated MCV). Hemolysis (E) is less likely than IDA, but can be confirmed or ruled out through the assessment of iron studies, the reticulocyte count and further hematologic labs as necessary.

A 70-year-old woman presents with a 1-year history of short-term memory loss. She did not seek medical attention earlier because she believed the memory loss to be age-related. Her husband has also noticed that she is unable to perform her regular daily activities and is often misplacing things. Recently, she was unable to get back home after her evening walk, and neighbors brought her home. The husband also feels she has been acting strange lately, and she is suspicious of her own son. Her past medical history is significant for hypertension and she has been on medication for the past 3 years. On clinical examination, she appears conscious and alert. Her vitals are PR- 70/min, BP- 120/74 mmHg. She is afebrile. Question What is your provisional diagnosis? Answer Choices 1 Dementia of the Alzheimer's type 2 Dementia due to a medical condition 3 Vascular dementia 4 Major depression 5 Substance induced dementia

Dementia of the Alzheimer's Dementia of the Alzheimer's type involves multiple cognitive deficits including 1 or more of the following: aphasia or language disturbance; apraxia, or the inability to carry out motor activities despite intact motor function; agnosia, or the inability to recognize familiar objects; and the disturbance of executive activities, such as planning, organizing, sequencing, or abstracting. There can be changes in personality and behavior, in addition to the inability to learn and recall new information. Long-term memory is affected. The disease also has a gradual onset and continuing decline. Focal neurological signs and symptoms are usually absent. If the disease occurs before age 65, this is early-onset. If it occurs after age 65, the disease is late-onset. Vascular dementia has the same general symptoms and signs as Alzheimer's disease, except that the onset is usually more sudden and acute. In addition, it is picked up easily by clinical or laboratory evidence of a vascular cause, a history of cerebrovascular disease, or multiple infarctions. Dementia due to medical conditions may present with similar or the same symptoms as other dementias, but are due to other medical or neurological diseases such as head trauma, Parkinson's disease, Pick's disease, Huntington's disease, HIV, multiple sclerosis, vitamin deficiencies, and numerous other medical conditions. Many elderly suffer from major depression. Symptoms include short-term or recent memory loss, fatigue, insomnia or hypersomnia, low appetite, weight loss, overeating, and an inability of handling activities of daily living. In major depression, deficits in various cognitive domains may be present. Temporal lobe magnetic resonance imaging (MRI) is helpful in differentiating dementia of the Alzheimer's type from depression and other disorders that may cause diagnostic difficulties in clinical practice. Substance-induced dementia includes cognitive defects caused by inhalants, sedative drugs, hypnotic drugs, prescription side effects, overdoses of prescription drugs, or other substances. This is common in the elderly.

A 33-year-old man presents with severe thirst and frequent urination; symptoms have been developing over the past month. He claims to consume more than 8 liters of fluids per day. Despite that, he is always thirsty. Fasting blood glucose is 90 mg/dl. On brain MRI, the hyperintensity of the posterior pituitary is absent. What drug is likely used to treat this patient? Answer Choices 1 Triamterene 2 Desmopressin 3 Digoxin 4 Verapamil 5 Propranolol

Desmopressin Desmopressin is a synthetic analog of arginine vasopressin (DDAVP). It acts on the kidney to decrease urine output and reduce the subjective feeling of thirst in patients with diabetes insipidus. DDAVP is usually administered via the mucus membrane of the nose. The dose required to keep the patient in water balance must be determined by measuring plasma sodium and plasma osmolality. Triamterene is a potassium sparing diuretic that produces a mild diuretic effect. Digoxin is used in the treatment of congestive heart failure. Its principal effect is to enhance cardiac output and efficiency and not as a diuretic on the kidney. Verapamil is an antiarrhythmic, antihypertensive drug classified as a calcium channel blocker. The beta-receptor blocking drug propranolol is used as an anti-hypertensive; however, it is not classified as a diuretic.

A patient presents with polydipsia and polyuria. Diabetes mellitus is ruled out with a normal plasma glucose and hemoglobin A1c. You are concerned that he may have hypothalamic diabetes insipidus. If you are correct, what treatment would you recommend to this patient? A Lithium B Indomethacin C Metformin D Desmopressin E Fluid restriction

Desmopressin The correct choice is D, desmopression. Patients with hypothalamic diabetes insipidus can't secrete vasopression (or antidiuretic hormone), since the vasopressin producing neurons are dead. Vasopressin analog desmopressin is available in tablets, nasal solution, and parenteral solution for patients with this disorder. Choice A, lithium, can cause nephrotoxicity, and can be a cause of acquired nephrogenic diabetes insipidus. Choice B, indomethacin, is an anti-inflammatory medication used to treat conditions such as gout. Choice C, metformin, is an oral diabetic medication that acts by suppressing hepatic glucose production. Patients must drink sufficient fluids to maintain serum sodium levels since without ADH, they can't conserve water. Therefore, choice E, fluid restriction, is not correct.

A 75-year-old woman presents with frequent leakage of her urine over the past several months; she is also experiencing frequent urinary infections. Leakage is sometimes "continuous", occurring both day and night. She also complains of tingling in her feet and hands, nausea, and a feeling of fullness after eating only a small amount of food. Her past medical history is significant for a 15-year history of diabetes mellitus type II, mild hypertension, and renal insufficiency stage 1. She takes metformin and ACE inhibitors. On examination, you find distended bladder and dysesthesia of stocking and glove pattern; the rest of the examination is within normal limits for her age. Question What is the most likely cause of her symptoms? Answer Choices 1 Pelvic floor muscle weakness 2 Detrusor underactivity 3 Kidney failure 4 Detrusor overactivity 5 Urinary tract infection

Detrusor Underactivity Explanation Having symptoms and signs of sensory disturbance and gastroparesis, your patient most probably has diabetic polyneuropathy that also involves autonomic nervous system. Bladder dysfunction usually appears 10 or more years after the onset of diabetes mellitus. The mechanism is most probably segmental demyelination and impaired nerve conduction. Your patient does not realize that her bladder has reached the maximal storage capacity, and the incontinence is a result of a chronically over-filled bladder. Emptying the bladder is often also not effective because of poor contractibility of a detrusor muscle. Pelvic floor muscles weakness typically presents with leaking a small amount of urine when coughing, sneezing, laughing, or running, but not with continuous leaking. Detrusor overactivity, as seen in spastic bladder, is associated with symptoms of urge incontinence; it is usually associated with frequency and nocturia. The symptoms she has cannot be explained by kidney failure. In addition, patients with chronic kidney disease stage 1 are usually asymptomatic. Retention of the urine contributes to the development of urinary tract infections (UTI). UTI presentation is primarily dysuria, with accompanying urinary urgency and frequency. Your patient has no such symptoms.

A 40-year-old female presents to your office with symptoms of weight gain, hirsuitism, and easy bruising. Past medical and surgical history is noncontributory. She drinks one glass of wine on weekends and does not smoke cigarettes. She takes one multivitamin daily. Upon physical exam, you note facial fullness, central obesity, and thin skin. Which of the following is a valuable biochemical screening test for this patient that will aide in the diagnosis? A Dexamethasone suppression test B Radioactive iodine uptake C Glucose tolerance test D Cosyntropin stimulating test E Plasma fractionated free metanephrines

Dexamethasone suppression test The correct choice is A, dexamethasone suppression test. This patient is presenting with classic signs and symptoms of Cushing's syndrome. The dexamethasone suppression test is a simple test of the hypothalamic-pituitary-adrenal axis, and requires ingestion of oral dexamethasone at nighttime and a blood test in the morning hour, to measure the amount of plasma cortisol. Most patients with Cushing's syndrome demonstrate a lack of normal axis suppression and present with a morning plasma cortisol level >5 mcg/dL. Choice B, radioactive iodine uptake, is used in patients with suspected thyroid disorders. Choice C, glucose tolerance test, is used in patients with suspected diabetes mellitus and in prenatal testing, to investigate gestational diabetes. Choice D, cosyntropin stimulating test, is used to investigate possible adrenal insufficiency. Choice E, plasma fractionated free metanephrines, is used in the diagnostic workup of pheochromocytoma.

A 38-year-old woman presents because she feels that she is "peeing a lot". For the last week, she has urinated at a rate of 4 - 5 times an hour; she even wakes up at night to urinate. She states that this has severely interfered with her sleeping habits. She further describes a significantly intense thirst and a powerful craving for only ice water to quench this thirst, although she never feels that her thirst has been quenched. Pertinent past medical history includes a diagnosis of bipolar disorder when the patient was 20. She takes lithium daily for this diagnosis, and she takes ibuprofen as needed for headaches. She is not taking any other medications regularly at this time. Significant family history includes her mother and father being diagnosed with diabetes mellitus as well as an older sister who was diagnosed with hypothyroidism. Question Given the medical history provided, what is the patient most likely experiencing? Answer Choices 1 Cushing's disease 2 Hypothyroidism 3 Diabetes mellitus 4 Diabetes insipidus 5 Hypoparathyroidism

Diabetes Insipidus Explanation This patient is suffering from diabetes insipidus (DI). Characteristics of this disease process include an increase in thirst and the passage of large quantities of urine. The underlying pathologic feature of DI is a deficiency of vasopressin or a resistance to available vasopressin. There are a multitude of disease states that lead to DI, and the patient is most likely suffering from an acquired form of vasopressin-resistant diabetes insipidus. Disease states that lead to this form of DI can include pyelonephritis, renal amyloidosis, potassium depletion, Sj?gren's syndrome, and the use of certain medications; in the patient's case, the use of lithium resulted in her DI. Chronic use of lithium can lead to lithium toxicity, and it manifests as described in the patient scenario. This is a common cause of DI in adult patients. Hypothyroidism is much lower on the differential due to the signs and symptoms being inconsistent. Findings such as weakness, fatigue, cold intolerance, constipation, weight changes, depression, and/or menstrual abnormalities would be some of the multiple features evident in patients with hypothyroidism. Hypoparathyroidism is also not as likely. Signs and symptoms of the acute phase of this disease include tetany, muscle cramps, irritability, altered mental status, and convulsions; a tingling sensation around the circumoral area, hands, and feet is almost always present. Cushing's disease is due to manifestations of hypercortisolism secondary to adrenocorticotropic hormone (ACTH) hypersecretion by the pituitary gland. Some features include (but are not limited to): central obesity, apparent moon face or buffalo hump, purple striae, muscle wasting, hirsutism, psychological changes, hypertension, weakness, headache, backache, and poor wound healing. None of the signs or symptoms listed matches the patient's history. Diabetes mellitus (DM) may also be considered in this patient scenario given the symptoms of polydipsia and polyuria described as well as her family history. The main cause of this type of diabetes is that patients develop increased tissue insulin insensitivity, which is also termed insulin resistance. Some key components of the patient's history are not evident in diabetes mellitus. Nocturnal enuresis is not usually a significant symptom in patients with DM, although patients with DM may have complaints of polyuria, polydipsia, weakness, fatigue, blurred vision, vulvovaginitis, and peripheral neuropathy; they may even be asymptomatic.

A 13-year-old girl is drowsy and unable to answer questions; her mother says that she has been extremely thirsty lately and urinates frequently. Her father notes that the patient has also been fatigued. There is a fruity odor to the patient's breath. Blood gases are drawn and reveal the following: pH 7.3 CO2 32 HCO3 17 Question What is the most likely cause of the patient's condition? Answer Choices 1 Frequent vomiting 2 Diabetes mellitus 3 Myasthenia gravis 4 Ethylene glycol ingestion 5 Salicylate ingestion

Diabetes mellitus Explanation The patient's blood gas results indicate metabolic acidosis. Given her presentation of polydipsia, polyuria, fatigue, and fruity odor to her breath, she is likely suffering from diabetic ketoacidosis from undiagnosed diabetes mellitus. Frequent vomiting would lead to metabolic alkalosis from hydrogen loss. Myasthenia gravis would be associated with respiratory acidosis due to the associated decrease in pulmonary function and therefore decreased clearance of CO2. Ethylene glycol ingestion is associated with metabolic acidosis; however, the patient's presentation is classic for diabetic ketoacidosis. Salicylate ingestion is associated with respiratory alkalosis.

Which of the following is an acceptable method for diagnosing endometriosis in a patient who describes a history of increasing pelvic pain which is cyclic in nature and increases in intensity just before menses? Answer Choices 1 Ultrasound 2 Endometrial biopsy 3 Magnetic resonance image (MRI) 4 Serum carcinoembryonic antigen (CA 125) 5 Diagnostic laparoscopy

Diagnostic laparoscopy Explanation Endometriosis refers to the presence of endometrial glands and stroma implanted in a location outside of the intrauterine cavity. There are multiple theories regarding the mechanism by which this occurs, they include: retrograde menstruation, metaplasia of coelomic epithelium, and embryonic rests or remnants. It is a progressive disease and often results in significant management problems. Interestingly, the extent of the disease often does not correlate well with the perceived degree of discomfort. Although various imaging techniques have been described as being helpful in following the progression of known pelvic disease, such as endometriosis, none are considered acceptable for the actual diagnosis of the condition. CA 125 is a cell surface antigen found in tissues derived from coelomic epithelium, including gastrointestinal and gynecologic tissues. Although it can be use to monitor the progress of a diagnosed inflammatory or malignant process, it is not diagnostic and should not be used as a screening test. The current standard of care regarding the definitive diagnosis of endometriosis is laparoscopic visualization, preferably with a confirmatory biopsy of a lesion.

A 27-year-old female is 8 weeks postpartum with her first child and has been exclusively nursing since discharge at the hospital. She has a 5-day history of engorgement in her right breast, which is red, tender, and feels warm to the touch. She states she is feverish but has not taken her temperature. She reports no known drug allergies. On physical examination you see the breast as shown below. Which of the following is the most appropriate therapy in the management of this patient? A Cephalexin 500 mg every 6 hours for 3 days; discontinue breastfeeding B Cephalexin 500 mg every 6 hours for 3 days; continue breastfeeding C Discontinue breastfeeding and see if symptoms resolve D Dicloxacillin 500 mg orally every 6 hours for 10 days; continue breastfeeding E Dicloxacillin 500 mg orally every 6 hours for 10 days; discontinue breastfeeding

Dicloxacillin 500 mg orally every 6 hours for 10 days; continue breastfeeding Staphylococcus aureus is usually the causing agent in puerperal mastitis. Treatment includes antibiotics that are effective against penicillin-resistant staphylococci and nursing of the infant with the affected breast is safe. Alternatively, a cephalosporin can be given orally for 10-14 days. Cephalexin 500 mg every 6 hours (A and B) can be used but should be continued for longer than 3 days. Failure of the patient to respond to antibiotics after three days becomes concerning for an abscess or resistant organism. Further evaluation would be needed including surgical consultation, ultrasound, and hospital admission for IV antibiotics. Dicloxacillin is commonly used in uncomplicated cases without any known allergies. To preserve milk supply, it is encouraged to continue breastfeeding with mastitis; do not recommend discontinuing breastfeeding (A, C, and E).

A 26-year-old woman is in the 24th week of her first pregnancy. She is in fairly good shape, and the pregnancy is progressing well. However, a fasting blood glucose done in the office shows it to be 146 mg/dl. It is repeated the next day, and the value is 142 mg/dl. An oral glucose tolerance test is ordered, which comes back as abnormal. She is diagnosed with gestational diabetes and advised to meet with her obstetrician. What should be the recommended regimen for this patient? Answer Choices 1 Diet and exercise until delivery 2 Diet and metformin if blood glucose remains high despite diet control 3 Diet and sulfonylurea if blood glucose remains high despite diet control 4 Diet, exercise, and insulin therapy if blood glucose remains high despite diet control 5 Diet and acarbose if blood glucose remains high despite diet control

Diet, exercise, and insulin therapy if blood glucose remains high despite diet control Explanation Gestational diabetes mellitus (GDM) should be treated early and optimally to reduce risks to the fetus. In pregnancy there is an increased resistance to insulin which causes high levels of glucose, lipids, and insulin. Placental lactogen and high levels of estrogen and progesterone cause this scenario. Risk factors for GDM include family history of diabetes, prior delivery of large baby, age >25 years, obesity, steroid use, glycosuria in first antenatal visit, ethnicity like Hispanic, African, Native American, Asian or Pacific Islander, polycystic ovarian disease (PCOS), and prior fetal loss of unknown cause. Hyperglycemia is harmful to the mother and the fetus and can cause increased perinatal mortality. Preeclampsia and polyhydramnios have a higher incidence in GDM. Trauma during birth and operative delivery are some other adverse outcomes. Macrosomia, or larger than normal fetus, is common. Metabolic abnormalities in the baby post delivery like hypoglycemia and hyperbilirubinemia, childhood obesity, attention deficit disorder (ADHD), and early diabetes may occur in the child. Universal screening of all pregnant women is recommended (though very low risk females may be excluded), usually at 24-28 weeks of pregnancy except if risk factors are evident like obesity, prior history of GDM etc, in which case screening should be done as early as possible. A random blood sugar of >200 mg/dL or fasting glucose of >126 mg/dL on 2 occasions is diagnostic of diabetes and does not require screening. Screening is done with a 50 gm oral glucose challenge test. A value of >140 mg/dL is positive, and a 3-hour glucose tolerance test is then done for definitive diagnosis. GDM is confirmed if 2 or more of the following are present: fasting glucose> 95 mg/dL, 1-hour glucose >180 mg/dL, 2-hour glucose >155 mg/dL, and 3-hour glucose >140 mg/dL. Referral to a nutritionist for strict diet control is a must to maintain euglycemia, prevent ketosis, and monitor adequate weight gain. Blood glucose should be measured before breakfast and 1 hour after each meal by the patient and moderate exercise is encouraged. Fasting blood glucose should be <90 mg/dL and post-prandial should be <120 mg/dL. If blood glucose remains high in spite of these measures, then insulin is the treatment of choice. Oral hypoglycemic agents are not approved for use in pregnancy in USA due to transplacental passage and fetal morbidity. If fasting blood glucose is high, then NPH insulin is used at bedtime in the dose of 0.2 units/kg body weight. If post-prandial glucose is high, then regular insulin is used before meals. If both are high, then a regimen of 4 injections a day needs to be started, including NPH before breakfast and at bedtime and regular insulin before each meal.

A 42-year-old man presents with fatigue, polyuria, and polydipsia for the past 2 - 3 months. He has a history of hypertension and obesity. He is on 10 mg of amlodipine daily and 75 mg of aspirin daily. His family history is significant for diabetes mellitus in the patient's mother and 2 siblings. On exam, the patient is afebrile; he has a blood pressure of 130/80 mm Hg; he has a weight of 220 lbs, and his height is 66 inches. He has no pallor, icterus, or lymphadenopathy. Lungs are clear to auscultation, and heart sounds are regular. The abdomen is normal, and there is minimal pitting pedal edema. Initial fundus exam is normal. Tests are done, including fasting blood glucose on 2 different days, a basic metabolic panel, and lipid profile. Fasting blood glucose was 136 mg/dL on the 1st day and 140 mg/dL on the 2nd. He is diagnosed with type II diabetes mellitus. Question What is the best strategy for treatment of this patient? Answer Choices 1 Diet control, weight loss, and exercise for 6 - 12 months; then, follow up tests should be performed for further treatment 2 Diet, weight loss, and exercise indefinitely if symptoms resolve 3 Oral hypoglycemic therapy with a sulfonylurea; tests should be repeated every 3 months, and a 2nd agent should be added if fasting blood sugar still above 140 mg/dL 4 Diet, weight loss, exercise, and monotherapy with metformin for 6 months, followed by repeat tests and adjustment of medication dose or addition of sulfonylurea 5 Diet, weight loss, and exercise for 4 - 6 weeks; then, if fasting blood sugar level remains above 126 mg/dL, add metformin

Diet, weight loss, and exercise for 4 - 6 weeks; then, if fasting blood sugar level remains above 126 mg/dL, add metformin Explanation A newly diagnosed patient with type II diabetes mellitus should be given a trial of therapeutic lifestyle changes with strict diet control, weight loss, and a moderately strenuous exercise regimen for 4 - 6 weeks. Afterwards, if the fasting blood sugar level is above 126 mg/dL, metformin should be added. Dietary modifications include low-calorie and low-fat diet with high complex carbohydrates. Some patients with mild diabetes may remain well controlled for a long period of time, but may eventually need medical treatment. Most patients need treatment sooner. Metformin acts by decreasing hepatic gluconeogenesis and increasing peripheral utilization of glucose. It is the drug of choice in type II diabetics who are overweight, since it causes mild to moderate decrease in weight. It is also quite effective in non-obese patients. The common side effects include abdominal pain, diarrhea, a metallic taste in the mouth, nausea, and anorexia. Lactic acidosis is a serious side effect. The contraindications for use of metformin include prior history of lactic acidosis, renal insufficiency with creatinine of 1.5 mg/dL, alcoholism, hepatic insufficiency, sepsis and other severe infections, heart failure, hypoxia and respiratory depression, and concurrent or anticipated use of radiographic material. In normal weight patients, sulfonylureas or biguanides are used. The dose may be increased every 4 - 8 weeks until blood sugar is controlled and hemoglobin A1C has decreased, with a goal of 7.0. Lifestyle changes should be continued for life.

A family friend calls you for medical advice. She just gave birth to her 3rd baby 1 week ago. She is worried after a nurse from the hospital called her. The nurse said the baby tested high on a newborn screening for phenylalanine and that she would set up an appointment at a specialty center for the family. Your friend could not remember the name of the disorder, but she was told that it could prevent her baby from normal neurological development if left untreated. The friend reports that her pregnancy was uncomplicated, and the baby appears healthy so far. No one in the infant's immediate family has any health problems. Question What will most likely be recommended treatment for the newborn's condition? Answer Choices 1 Administration of growth hormone 2 Cognitive behavioral therapy 3 Dietary control 4 Oxygen therapy 5 Surgery

Dietary control Your friend's newborn most likely has phenylketonuria (PKU), which is an autosomal recessive metabolic disorder that causes abnormal metabolism of phenylalanine and results in elevated serum phenylalanine. Dietary control, primarily by restriction of phenylalanine, is the mainstay of treatment. PKU is rare and diagnosed by routine newborn screening. A strict diet low in phenylalanine can prevent complications of the disease, and afflicted patients can lead a relatively normal life. Neurologic deficits, such as mental retardation and seizures, can occur if untreated. If an afflicted patient does not adhere to the special diet, he or she may begin having cognitive deficits and/or seizures. Administration of growth hormone may be given in cases of congenital adrenal hypoplasia. It has no role in PKU. Cognitive behavioral therapy is an excellent treatment for several psychiatric disorders. However, it has no role in newborns and infants, and it generally cannot prevent the cognitive decline in PKU. Oxygen therapy and surgery are not appropriate treatments for PKU.

A full-term neonate with an Apgar score of 9 at birth developed abdominal distension with watery diarrhea and flatulence following the initiation of milk feeding. The infant became normal when the milk feeding is terminated. What is the most likely diagnosis? Answer Choices 1 Celiac sprue 2 Tropical sprue 3 Whipple disease 4 Disaccharidase deficiency 5 Amebic dysentery

Disaccharidase deficiency The disaccharidases, of which the most important is lactase, are located in the apical cell membrane of the villous absorptive epithelial cells, the deficiency of which leads to the incomplete breakdown of the disaccharide lactose into its monosaccharides, glucose, and galactose, leading to diarrhea from the unabsorbed lactose. Bacterial fermentation of the unabsorbed sugars leads to increased hydrogen production, which is readily measured in the exhaled air by gas chromatography. There are both hereditary and acquired forms. Histologically in both, there is no abnormality of the mucosal cells of the bowel. Malabsorption is promptly corrected when exposure to milk and milk products is terminated. Celiac sprue is a rare chronic disease in which there is a characteristic mucosal lesion of the small intestine in the form of blunting of the villi and an overall increase in plasma cells, lymphocytes, macrophages, and eosinophils in the lamina propria. Also known as the gluten-sensitive enteropathy, non-tropical sprue, and celiac disease, it is due to sensitivity to gluten, which is an alcohol soluble, water insoluble protein component called gliadin of wheat and other closely related grains. Detection of circulating anti-gliadin or anti-endomysial antibodies strongly favors the diagnosis. Both the symptoms and the mucosal histology improve on withdrawal of gluten from the diet. Tropical sprue is a celiac-like disease occurring in people living or visiting the tropics. Bacterial overgrowth by an enterotoxigenic organism is said to be the cause for this disease. Intestinal changes are similar to celiac sprue, but injury is seen at all levels of the small intestine. This is in contrast to celiac sprue, where it is concentrated in the proximal small intestine. It is mainly treated by broad-spectrum antibiotics. Whipple disease is a rare condition that may involve any organ of the body, but principally affects the intestine, central nervous system, and joints. It is caused by Gram-positive actinomycetes called Tropheryma whippelii. Histologically, the small intestinal mucosa is laden with distended macrophages in the lamina propria. These macrophages contain periodic acid-Schiff positive granules and rod-shaped bacilli by electron microscopy. Patients usually present with a form of malabsorption, including diarrhea and weight loss, and they generally respond to antibiotic therapy. Amebic dysentery is caused by Entamoeba histolytica, a protozoan, and generally causes diarrhea with blood and mucus. A stool examination will show the cysts and trophozoites of Entamoeba histolytica, which is diagnostic.

A 76-year-old Caucasian man with a 90 pack per year smoking history presents with progressive fatigue, tachypnea, exertional dyspnea, cough, and lower extremity edema. Inspection of his chest and abdomen reveals an increased chest diameter, labored respiratory efforts with retractions and cyanosis, left parasternal and subxiphoid heaves, hepatojugular reflux, and a pulsatile liver. Additionally, there is scattered wheezes and crackles in his lungs and bilateral lower extremity edema. Question What physical exam findings would be most consistent with the underlying diagnosis? Answer Choices 1 Distended neck veins with prominent a or v waves 2 Dullness to percussion and increased tactile fremitus of the lungs 3 Pericardial friction rub 4 Laterally displaced and enlarged point of maximal impulse 5 A systolic ejection murmur located at the aortic valve area

Distended neck veins with prominent a or v waves This patient's diagnosis is chronic cor pulmonale. Cor pulmonale is estimated to account for 6-7% of all types of adult heart disease in the United States, with chronic obstructive pulmonary disease (COPD) due to chronic bronchitis or emphysema the most common cause (the causative factor in more than 50% of cases). Cor pulmonale usually presents chronically; however, 2 main conditions can cause acute cor pulmonale: pulmonary embolism (more common) and acute respiratory distress syndrome (ARDS). In chronic cor pulmonale, right ventricular (RV) hypertrophy (RVH) generally predominates. Common symptoms are fatigue, tachypnea, exertional dyspnea, and cough. Other symptoms include anginal chest pain (due to right ventricular ischemia), hemoptysis, and, rarely, hoarseness due to compression of the left recurrent laryngeal nerve by a dilated pulmonary artery. Anorexia, right upper quadrant abdominal discomfort, and jaundice may occur due to passive hepatic congestion. Physical exam findings reflect the underlying lung disease or pulmonary hypertension, right ventricular hypertrophy (RVH), and RV failure. Expected signs include an increase in chest diameter, labored respiratory efforts with retractions of the chest wall, distended neck veins with prominent a or v waves, and cyanosis. Wheezes and crackles may be audible upon auscultation due to underlying lung disease. S2 heart sound splitting with an accentuated pulmonic component may be found early, while a systolic ejection murmur with sharp ejection click over the region of the pulmonary artery may be heard in advanced disease, along with a diastolic pulmonary regurgitation murmur. Other cardiac findings include third and fourth sounds and the systolic murmur of tricuspid regurgitation. RVH is characterized by a left parasternal or subxiphoid heave. Hepatojugular reflux and pulsatile liver are signs of RV failure with systemic venous congestion. On percussion, hyperresonance of the lungs may be a sign of underlying COPD; ascites can be seen in severe disease. Examination of the lower extremities reveals evidence of pitting edema. Dullness to percussion and increased tactile fremitus of the lungs suggests a lung consolidation, as in pneumonia, pulmonary edema, or pulmonary hemorrhage. Pericardial friction rubs suggest inflammation of the pericardial sac, while a laterally displaced and enlarged point of maximal impulse occurs with left ventricular hypertrophy, congestive heart failure, cardiomyopathy, and ischemic heart disease. The finding of a systolic ejection murmur located at the aortic valve area most likely suggests a diagnosis of aortic stenosis.

A 55-year-old man presents with a COPD exacerbation that is being managed with a ventilator. The patient's blood pressure drops and the ventilator alarm goes off. The only medication administered is amlodipine via nasogastric tube. This patient is afebrile, even though the hospital has been having problems with pseudomonas infection in ventilated patients. On examination, there is a middle-aged orally-intubated man with temperature 99.4° F, pulse 145/min, and BP 62/34 mm Hg; he breathes above the ventilator at a rate of 36 cycles/min even. His breath is shallow and he has diminished breath sounds in his right hemithorax. What is the most appropriate next step? Answer Choices 1 Add positive end-expiratory pressure 2 Do thoracotomy/chest tube placement 3 Start antibiotics 4 Give IV boluses 5 Start pressor agents

Do thoracotomy/chest tube placement The patient's diagnosis is right tension pneumothorax in a COPD patient on a ventilator. Features supportive of tension pneumothorax are hypotension, tachycardia, tachypnea, shallow breathing, and decreased breath sounds in the right hemithorax. Management of this condition is emergent thoracentesis through insertion of a large bore needle into the second intercostal space. Chest tube placement is indicated for definitive treatment. Positive end-expiratory pressure is the maintenance of positive pressure in the alveolar at the end of expiration. Its application will cause an increase in the mean and peak airway pressure and worsen the pneumothorax by forcing more air into the lungs. Antibiotics will be used if there is an infective process going on, but this is not evident. Since the hypotension is related to decreased venous return because of increased intrathoracic pressure leading to decreased cardiac output, IV fluids or pressors are unlikely to increase the blood pressure.

A 34-year-old woman presents with worsening headaches. She says that the headaches are present throughout the day and that she has been feeling nauseous. Lately she has also noticed difficulty in seeing vehicles overtake her on the freeway. She has had several close calls due to this handicap. Her periods, which were previously regular are now irregular, with heavy bleeding every 3 - 4 months. She has also noticed a milky discharge from both nipples. Her pregnancy test is negative. An MRI of the brain shows a mass compressing the hypothalamic pituitary axis. Question What is the best course of treatment? Answer Choices 1 Irradiation of the tumor 2 Surgical excision 3 Catheter ablation 4 Dopamine agonists 5 External beam laser coagulation

Dopamine Agonist The most likely diagnosis is a pituitary adenoma compressing the HP axis. The standard of care at the present time is the use of dopamine agonist drugs, such as bromocriptine and cabergoline. These drugs reduce prolactin levels to normal in 70 - 100% of patients. The presence of the pituitary mass blocks the action of dopamine on the prolactin-secreting cells and increases the secretion of prolactin. Cabergoline is an alternative to bromocriptine with fewer side effects. However, it is a much more expensive drug, and it is usually used when bromocriptine is not tolerated. Pergolide has been taken off the market due to associated heart valve damage with its use. Followup may be done by measurement of the fasting serum prolactin level. Surgery is incorrect, as it is usually indicated only in failure of medical therapy, continued visual field loss despite medication, or failure to tolerate medical therapy. Radiation therapy is incorrect. Although effective, it is seldom used due to the risk of permanent hypopitutarism after treatment. External beam coagulation and ablation are both incorrect. They are distractors that have no role in therapy.

A 15-year-old male presents complaining of a sore throat, headache, and mild cough that started 8 days ago and has progressed to include a worsening cough and increasing fatigue. His chest x-ray reveals bilateral hilar infiltrates, CBC is normal and a nasal secretions test positive for mycoplasma pneumoniae by PCR. What is the most appropriate therapy? A Amoxicillin B Cefuroxime C Clindamycin D Ciprofloxacin E Doxycycline

Doxycycline Mycoplasma pneumoniae is commonly treated with macrolides, doxycycline (E), or respiratory fluoroquinolones. Mycoplasma pneumonia doesn't respond to beta-lactam antibiotics (A, B) or non-respiratory fluoroquinolones (D).

You are a general practitioner in private practice and have been treating a 12-year-old boy with headache, fever and meningismus. You suspect meningitis and send him off to the nearest hospital; an analysis of cerebral spinal fluid indicates meningococcusmeningitis. All the members of your team who have come in close contact with the boy need to take rifampicin p.o. for 2 to 4 days prophylactically. One nurse is worried because she might be 1 week pregnant. What do you do? Answer Choices 1 Give her a urine pregnancy test 2 Draw blood for quantitative ßHCG 3 Give her a one-shot rifampicin 4 Give her the same rifampicin dosage as the others 5 Substitute rifampicin with tetracycline

Draw blood for quantitative ßHCG Explanation The correct response is to draw blood for quantitative βHCG. If the nurse is only 1 week pregnant, you could prescribe the same rifampicin p.o. dosage as for the others. At that time, there is no direct blood flow between the conceptus and maternal circulation; you have to draw blood for quantitative ßHCG level to make sure that, if she is pregnant, she is not further along. A urine pregnancy test usually shows a positive result about 2 weeks after the time the next menstruation is expected. Since rifampicin is not available in injectable form, there is no possibility for a one-shot. Tetracycline is not approved for prevention of meningococcus meningitis; furthermore, it is contraindicated during pregnancy.

A 24-year-old woman has recently been diagnosed with insulin-dependent diabetes. The disease is being managed on a split dose of 60/40 insulin suspension, which she injects herself at 8:00 a.m. and 5:00 p.m. She was told to call in if she experiences any strange symptoms, which she does this afternoon. At 2:45 p.m., she is not feeling well and notices that her skin is cool and damp. Her hands are shaking and she is very anxious. What do you tell her to do right away, before having somebody take her to your office? Answer Choices 1 Inject 4 IU of her insulin 2 Drink a can of diet soda 3 Drink four ounces of fruit juice 4 Eat a large candy bar 5 Eat a cube of sugar

Drink 4 ounces of fruit juice Explanation The above described symptoms are classical for hypoglycemia. An inexperienced patient needs to be monitored by medical personnel until the blood glucose level is stabilized again. The first priority, though, is to prevent the blood sugar to drop any further and send the patient into a coma. A four-ounce drink of fruit juice should be sufficient. Since diet soda contains only sugar substitutes, it is not going to influence the blood glucose level. A large candy bar could be eaten too, but it may raise the glucose level higher than required. Therefore, the fruit juice is a better choice. 1 cube of sugar is an insufficient amount to raise blood sugar; it will take several cubes. According to the American Diabetes Association, 15-20grams of carbohydrates are recommended. Since the symptoms suggest hypoglycemia, injecting insulin would worsen the situation. If unsure whether the patient is hypo- or hyperglycemic, and there is no possibility for a test, always give sugar first and see if the patient improves. An increase of a blood glucose level of, for example, 350 is not going to hurt the patient; however, lowering a level of 45 is going to send the diabetic into a coma.

A 64-year-old man with hypertension, coronary artery disease, and poorly-controlled left ventricular congestive heart failure presents with a 3-day history of insidious chest pain. Pain is made worse when he takes a deep breath in and when he coughs. He denies any relation of pain to position, activity, or food intake. He denies fever, chills, palpitations, sputum production, wheezing, abdominal pain, nausea, vomiting, diarrhea, or peripheral edema. His physical exam reveals a widespread friction rub upon inspiration, absent lung fremitus, and reduced lung sounds over the thoracic cavity. Question What additional physical exam finding would be most likely expected in this patient? Answer Choices 1 Vesicular breath sounds 2 Dullness to percussion 3 Tracheal shift to the affected side 4 Chest wall tenderness 5 Increased anteroposterior diameter

Dullness to percussion This patient's presentation is most consistent with a pleural effusion. This patient is demonstrating pleuritis, which is caused by damage and inflammation to the pleura of the lung. The most common cause of pleural effusion is left-ventricular failure. It usually causes a sharp, knife-like pain that is aggravated by inspiration or coughing. Vesicular breath sounds are the normal breath sounds, which occur in most of the lung fields. The trachea may deviate toward the side opposite a large pleural effusion. Chest wall tenderness is an expected finding in chest wall pain syndromes, such as costochondritis. An increased anteroposterior diameter, or barrel chest, may be observed in chronic obstructive pulmonary disease.

Which of the following symptoms is most frequently reported in patients that have peptic ulcer disease? A Dyspepsia B Sharp abdominal pain C Relief of pain with food D Pain that awakens the patient at night E Significant vomiting

Dyspepsia (epigastric pain) is the hallmark of peptic ulcer disease and is present in 80 to 90% of patients. The abdominal pain is typically described as dull, gnawing, aching or "hunger like," but isn't typically described as sharp. Relief of pain with food is reported in about 50% of patients. Furthermoe, 66% of patients with duodenal ulcers and 33% of patients with gastric ulcers report nocturnal pain that awakens them at night. Significant vomiting is not typical of uncomplicated peptic ulcer disease, and is more suggestive of gastric outlet obstruction or gastric malignancy.

A 10-year-old girl had a group A (β-hemolytic) streptococcal pharyngitis from which she recovered on penicillin therapy. A month later, she presents with fever, joint swelling, involuntary movements, and a skin rash. What valve is MOST commonly altered in her disease? Answer Choices: A Venous B Tricuspid C Aortic D Pulmonary E Mitral

E

A 12-year-old girl presents because her joints are hurting. Her knees and ankles have been painful, warm, and swollen for several weeks now. It started in her left knee, then her left ankle became involved, and then her right knee was affected. Her left knee is 'almost back to normal now,' but the other involved joints are becoming more painful. She is a middle school student and has been unable to go to school or church for the past 3 days because of the pain. She is taking acetaminophen arthritis formula, but the pain continues to get worse. Her mother did not want to give her non-steroidal antiinflammatory medications because of her age. Review of systems (ROS) is positive for fever, chills, malaise, mild chest pain when lying down, moderate headaches, and weight loss. ROS is negative for sexually transmitted diseases, trauma to affected joints, tick exposure, and recent travel. What component of the physical exam would lead you to suspect acute rheumatic fever as the cause of her joint pain? A Erythema infectiosum B Erythema migrans C Fever above 100 degrees F D Jaundice E Mitral regurgitation

E

A 17-year-old boy presents with tiredness, weakness, and a marked sore throat. There is no history of contact with others having the same symptoms. On review of symptoms, he admits to chills and excessive sweating, but denies coryza, cough, nausea, vomiting, or diarrhea. On physical examination, there is pharyngitis, marked cervical lymphadenopathy, a fever of 38 degrees C, and the spleen is palpable 3 cm below the left costal margin. There is neither hepatomegaly nor jaundice. Initial laboratory data reveals hemoglobin of 13%, a white blood count of 13,000, and a platelet count of 240,000. The differential white count includes 35% neutrophils, 5% bands, 50% lymphocytes, and 10% monocytes. Many of the lymphocytes have irregular or dark blue cytoplasm. What is the best short-term management of this patient? Answer Choices: A Digitalis B High-dose steroids C Penicillin D Chlorambucil E Restriction of sport activities

E

A 17-year-old football player is being evaluated for fatigue and a sore throat. He denies coughing or fevers. He does not smoke or drink. In addition, he denies blood in his stool or urine and any dysuric symptoms. He practices safe sexual intercourse with his girlfriend. Physical exam demonstrates a blood pressure of 130/65 mmHg, a pulse of 72/minute, respirations of 16/minute, and a temperature of 98 degrees Fahrenheit (36.6 Celsius). The lung and heart sounds are normal. There is no cervical adenopathy. When you look at the pharynx, there is tonsillar enlargement with evidence of exudates. The abdominal exam demonstrates a spleen tip that is easily palpable 1.5 centimeters below the left costal margin. Results of a Monospot test are positive. What management scheme is best for this patient? Answer Choices: A Empiric antibiotic treatment B Intravenous acyclovir therapy C Oral acyclovir therapy D Oral steroids for 5 days E Symptomatic treatment only

E

A 2-month-old infant presents with a 48-hour history of poor feeding, crying, limpness, and constipation. The doctor in charge asks the mother how she feeds the infant. The infant is given formula regularly, but the mother sometimes adds honey to make the formula sweeter. She keeps the honey in the kitchen cupboard. When the doctor examines her, she notices a very drowsy infant with flaccid paralysis. What is causing this infant's symptoms? Answer Choices: A Sepsis B Hypothyroidism C Congenital muscular dystrophy D Lactose intolerance E Botulism

E

A 21-year-old woman visits her gynecologist complaining of a purulent vaginal discharge and difficulty urinating. The patient is examined. Based on laboratory results, presumptive treatment is started for gonorrhea and Chlamydia. History reveals that the woman has a steady relationship with a male partner. She reports that she is sexually active, but she always uses condoms. When the gynecologist reinforces the importance of consistent condom use, the patient says that she believes she was raped at a party 2 weeks earlier. She provides an account of events consistent with the use of flunitrazepam and sexual assault, and she explains that she was too uncertain and too ashamed to reveal the attack. Additional presumptive antimicrobials are given and the patient asks about HIV risk. Which of the following statements is most accurate? Answer Choices: A The patient is not a candidate for post-exposure prophylaxis because she is presenting ≥72 hours after possible exposure. B The patient is a candidate for post-exposure prophylaxis; standard therapy with a single antiretroviral for 28 days is indicated. C The patient is not a candidate for post-exposure prophylaxis because she is presenting with signs and symptoms of an associated sexually transmitted disease. D The patient should be tested for HIV, and post-exposure prophylaxis should only be started if the test is positive. E The patient is a candidate for post-exposure prophylaxis; standard therapy with 2-3 antiretrovirals for 28 days is indicated.

E

A 23-year-old woman visits a community clinic presenting with vaginal pain, itching, and odor. The patient reports that she was treated for gonorrhea 3 weeks earlier. Examination shows a white vaginal discharge. Cervical cultures are submitted for Neisseria gonorrhoeae and Chlamydia trachomatis. Gram stain reveals predominant, slender, Gram-positive rods with rare neutrophils. Wet mount of a cervical smear shows flagellate protozoa. What is the most likely cause of the presenting problem? Answer Choices: A Gardnerella vaginalis B Neisseria gonorrhoeae C Lactobacilli D Herpes simplex virus E Trichomonas vaginalis

E

A 24-year-old man is brought to the emergency room in shock. A quick history from his girlfriend reveals that he has had a "blister on his left foot" for the past couple of days. According to the girlfriend, the patient developed a fever earlier in the day; he felt weak and eventually collapsed. He last passed urine the evening prior to presentation. On examination, his BP is 70/40 and pulse is 130/minute; his hands feel cold and clammy. Except for a swollen left foot, systemic examination is normal. A picture of his limb is shown. His lab values are as follows; Total WBC count- 21,000/cubic mm Neutrophils - 55% Lymphocytes -25% Hemoglobin - 11.0gm/dl CRP- 165mg/l Serum Creatinine - 2.3 mg/dl Urine Myoglobin- positive Serum K- 5.9 Meq/dl Serum Na- 133 Meq/dl ECG- Sinus tachycardia Question: What is most essential step in management of this patient? Answer Choices: A Administration of IV Calcium gluconate B Low dose Dopamine infusion C Broad spectrum antibiotics D Tetanus prophylaxis with immunoglobulin administration E Extensive early surgical debridement

E

A 24-year-old man presents with severe headache, fever and chills, fatigue, and pain in his joints. His wife states that he has "the flu." During the physical exam, his doctor discovers a rash on the patient's thighs and arms. The patient says he has had it for "a while." The rash consists of large red indurated lesions with bright red outer borders and pale centers. The patient believed it to be poison ivy or sumac, because last month he and his wife spent the July 4th weekend with his brother camping and hiking in the Catoctin Mountains of Northern Maryland. What microorganism is probably causing this illness? Answer Choices: A Staphylococcus aureus B Streptococcus pyogenes C Pseudomonas aeruginosa D Coccidioides immitis E Borrelia burgdorferi

E

A 24-year-old woman with no significant past medical history presents for her yearly medical examination. She denies any medical problems. Her cardiac exam is notable for a mid-systolic click. This finding precedes a high-pitched, late systolic crescendo-decrescendo murmur, described as "whooping" or "honking," and was heard best at the apex and is without radiation. The provider noticed that these sounds occurred earlier in the cardiac cycle when the patient was standing and during the Valsalva maneuver, and later when she squatted or during handgrip. What is the most likely diagnosis? A Aortic regurgitation B Aortic stenosis C Mitral stenosis D Mitral regurgitation E Mitral valve prolapse

E

A 27-year-old para 2 develops a fever on post natal day 5. A sustained elevated temperature over 24 hours prompts a thorough examination by her physician. Her breasts are engorged, non-tender with no erythematous. Her uterus feels firm and well contracted, but exquisitely tender to palpation. Speculum examination reveals a normal appearing cervical with no discharge. A presumptive diagnosis of puerperal sepsis is made and broad spectrum antibiotics (Ampicillin/Gentamycin/Metronidazole) are initiated. 48 hours later, there is no improvement. Regular temperature spikes are noted, and the abdominal pain has increased in severity. Pelvic sonogram shows no retained products of conception or pelvic fluid collection suggestive of an abscess. What is the next best step in management? A Plain CT abdomen B Laparotomy C IV Ceftriaxone administration D Addition of anti-fungal to regimen E IV Heparin administration

E

A 29-year-old woman is brought to the emergency room by her roommate. The patient's illness began 48 hours earlier with vomiting and diarrhea. The patient reports no travel history. Otherwise, she is healthy, with the exception of vaginitis for 1 week before becoming ill. Examination reveals a fever of 38.9° C, hypotension, generalized erythematous rash with desquamation, conjunctivitis, and hyperemic vaginitis. Laboratory results indicate renal failure and thrombocytopenia. Liver enzymes are elevated, but she is not icteric. Blood cultures are negative. What is the most likely diagnosis? Answer Choices: A Neisseria meningitidis septicemia B Leptospirosis C Measles D Malaria E Toxic shock syndrome

E

A 3-month-old infant develops rapid breathing and staccato cough, but otherwise appears well and is afebrile. Physical examination detects fine rales over the lungs as well as red and thickened tympanic membrane. A chest X-ray shows bilateral patchy infiltrates. Laboratory studies indicate eosinophilia and elevated serum immunoglobulin concentrations. What is the likely causative organism? Answer Choices: A Haemophilus influenzae B Mycoplasma pneumoniae C Streptococcus pneumoniae D Mycobacterium tuberculosis E Chlamydia trachomatis

E

A 3-year-old child presents because her mother found thin worms in her underpants. An infection of Enterobius vermicularis is suspected, so she is put on therapy with a drug that interferes with the synthesis and function of microtubules; this leads to inhibition of protein secretion and glucose transport in all species of intestinal nematodes. What drug is she being treated with? Answer Choices: A Praziquantel B Metronidazole C Niclosamide D Acyclovir E Mebendazole

E

A 30-year-old man receives a bone marrow transplant for treatment of acute myeloid leukemia. He develops increasing respiratory failure 3 weeks later. A bronchoalveolar lavage yields cells that are enlarged and have prominent intranuclear inclusions. What is the most likely diagnosis? Answer Choices: A Toxoplasmosis B Candidiasis C Tuberculosis D Pneumocystis carinii pneumonia E Cytomegalovirus infection

E

A 30-year-old woman who is 6-months pregnant comes in because her 4-year-old child has fifth disease. An antibody test is ordered, and the woman is found to be IgG positive. What is the best course of action? Answer Choices: A No further action is recommended B PCR detection of the virus C Administration of immunoglobulin D Administration of parvovirus vaccine E Ultrasound examination of the fetus

E

A 40-year-old obese Caucasian man says that his father recently passed away after having a heart attack. He is worried and wants to know whether he is at risk for cardiovascular disease. He is a hypertensive; however, he does not take any medications. His BP in the office today is 140/96 mmHg, and his BMI is 31. You explain to him that the risk factors for cardiovascular disease may be non-modifiable or modifiable. What is the non-modifiable risk factor for CAD in this man? A His age B His ethnicity C Hypertension D Obesity E His gender

E

A 42-year-old man with a past medical history of hypertension presents with intermittent fever of 6 weeks duration. He has an associated cough, dyspnea, anorexia, arthralgias, abdominal pain, diarrhea, a widespread rash throughout his body, and back pain. He has come to see you because he has experienced acute left upper and lower extremity weakness and painless hematuria since this morning. He denies chills, a history of travel, sick or confined contacts, exposure to animals, bites, stings, cigarette smoking, otalgia, sore throat, swollen glands, drug use, dysuria, preceding GI or GU infections, previous surgeries, or sexual contact in the past year. His physical exam is remarkable for fever, a generalized petechial rash and petechiae of the mucous membranes, dark red linear lesions of the nailbeds, tender subcutaneous nodules of the digital pads, and nontender maculae on the palms and soles. His heart is notable for a new harsh, medium pitched pansystolic murmur at the apex with radiation to axilla, reduced strengths to the left upper and lower extremities, and splenomegaly. What is the most likely diagnosis? A Reactive arthritis B Syphilis C Systemic lupus erythematosus D Lyme disease E Infective endocarditis

E

A 45-year-old man presents with a 6-hour history of retrosternal chest pain. He describes it as a sharp and episodic, which is relieved by sitting upright and worsened by lying down. He denies any trauma to the chest. What is the most likely cause of his chest pain? A Musculoskeletal pain B Herpes zoster C Spontaneous pneumothorax D Angina E Pericarditis

E

A 48-year-old man presents for an annual physical exam. He has a past medical history of obesity and a 5-year history of hypertension that is currently not well controlled; he was recently diagnosed with type II diabetes mellitus (DM). He is a 20-pack-year smoker and drinks 2-3 beers per night. On exam, his BMI is 41, and BP is 145/92 mmHg. The remainder of his exam is unremarkable. What is the most important matter to address with this patient concerning the prevention of morbidity and mortality? A Obesity B Alcohol use C Lack of exercise D Not wearing seat belts E Cigarette smoking

E

A 48-year-old man presents with a log book of home-recorded blood pressure readings between 125-185/75-100 mmHg. His past medical history is significant for hypertension and obesity (BMI 31 kg/m2). He is a non-smoker and does not drink alcohol. He has been adhering to a sodium-restricted diet and tries to walk 2 miles at least twice a week. For the past 2 years, he has been taking hydrochlorothiazide 12.5 mg daily with no apparent side effects. Physical exam is unremarkable, and vital signs reveal a heart rate of 85 beats per minute and blood pressure of 150/85. Based on this information, what is the most appropriate next step in the management of the patient's hypertension? A Add a calcium channel blocker to his current therapy B Change hydrochlorothiazide 12.5 mg daily to chlorothiazide 125 mg daily C Continue current management and schedule a follow-up visit in 3 months D Discontinue hydrochlorothiazide and begin an angiotensin-converting enzyme (ACE) inhibitor E Increase hydrochlorothiazide dose to 25 mg daily

E

A 49-year-old man presents with chest pain. He has had this type of pain in the past, but it typically occurred with significant exertion, such as shoveling snow. Over the past 2 weeks, however, the pain has come on with progressively less activity, and today the pain persists despite rest. Although he has a history of hypertension, he admits to rarely taking his antihypertensive medication. On examination, his HR is 95 beats per minute and his blood pressure is 212/100. An electrocardiogram is performed and reveals ST elevation in the anterolateral leads. What medication would be the most appropriate treatment? A Adenosine B Furosemide C Methyldopa D Nifedipine E Nitroprusside

E

A 50-year-old man presented with diffuse pain, stiffness and swelling of his right knee. His symptoms were of 3 weeks duration. He was living near a wooded area, and had history of tick-bite 7 months earlier. He had developed a local skin lesion at the site of the tick-bite, followed by flu-like symptoms, and had recovered without any specific treatment. Radiography of the knee joint showed evidence of effusion. Arthrocentesis yielded inflammatory fluid. What test is recommended for confirming the diagnosis? Answer Choices: A Microscopy of the synovial fluid B Culture of the synovial fluid C PCR analysis of the synovial fluid D Blood culture E Testing for serum antibodi

E

A 54-year-old man presents with chest pain. He has a past medical history of hypertension and diabetes mellitus. The pain is located in the middle of his chest and radiates to his jaw. The pain began about 20 minutes ago, and he rates the pain as a 10 on a 0-10 point scale, with 10 being the worst pain he has ever felt. He has had 3 similar episodes, but they have always resolved after 5 minutes or so of rest. He has smoked 1 pack of cigarettes a day for the past 36 years. He drinks 2 or 3 beers on Friday nights. Review of systems (ROS) is positive for diaphoresis, acute dyspnea, and impending doom. ROS is negative for fever, chills, and malaise. Physical exam shows an obese, middle-aged man in moderate distress. BP is 148/80; pulse is 100; and respirations are 26. Heart and lung exams are normal, except for tachycardia and tachypnea. He has no pedal edema. Electrocardiogram (ECG) shows ST elevation in leads II, III, and AVF; this is a new finding when compared to ECG from 3 months ago. What is the diagnosis? A Pericarditis B Stable angina C Pulmonary embolism D Chostochondritis E Acute myocardial infarction

E

A 56-year-old man is diagnosed with primary hypertension. His additional medical history includes only nephrolithiasis. Past analysis of his kidney stones has revealed a calcium oxalate composition. In addition to controlling his blood pressure, what class of antihypertensive agents might be helpful in managing his kidney stones? A Angiotensin-converting enzyme (ACE) inhibitors B Beta blockers C Calcium channel blockers D Loop diuretics E Thiazide diuretics

E

A 58-year-old man presents for a routine physical exam. The only abnormal finding is a blood pressure of 150/96 mm Hg. 20 years ago, he quit smoking cigarettes after smoking at a rate of 1 pack/day for 10 years. Family history is significant for diabetes and hypertension in 2 elder siblings. Past medical history is insignificant, except for an appendectomy at the age of 28 and fractured right radius 15 years ago. Repeat BP recordings show a BP of 148/98 mm Hg and the of 150/98 mm Hg. Fasting blood glucose is 122 mg/dL. A renal profile, lipid profile, and fundoscopy are ordered. Based on this patient's profile, what would be the best drug to start him on? A Nifedipine B Prazosin C Atenolol D Hydrochlorthiazide E Enalapril

E

A 62-year-old male with a past medical history significant for a 15-year history of hypertension presents with a chief complaint of severe tearing chest pain radiating through to the back. Blood pressure is 180/110 mmHg, heart rate 120 bpm, and respiratory rate 34/min. Physical examination findings include neck negative for bruits/JVD, lungs clear to auscultation, heart regular rhythm, normal S1/S2 with an S4 present, and grade III/IV diastolic rumbling murmur noted with patient leaning forward. Radial pulses are 1+ on right and 3+ on left. EKG reveals a sinus tachycardia and evidence of left ventricular hypertrophy. You decide that the patient most likely has a thoracic aortic dissection. A STAT chest x-ray shows a widening of the mediastinum. Which of the following medication classes delivered intravenously is now needed to stabilize the patient? A Sympathomimetic drug B Positive inotropic drug C Alpha blocker D ACE inhibitor E Beta blocker

E

A 63-year-old man with a past medical history of diabetes mellitus type II, hypothyroidism, and hyperlipidemia presents to his primary care provider with a 3-month history of intermittent bouts of mild to moderate "squeezing, pressure, and tight" left-sided chest and left shoulder pain, each episode lasting approximately 10 to 15 minutes. Additionally, he admits to associated shortness of breath and nausea during these episodes, which are provoked upon walking, exposure to the cold weather, during times of emotional stress, and following consumption of a meal, but he is not aware of any specific foods that provoke these symptoms. He denies diaphoresis, fever, chills, palpitations, peripheral edema, hemoptysis, abdominal pain, reflux, regurgitation, diarrhea, cough, and pleurisy. He also denies travel, sick contacts, cigarette, and drug or alcohol use. Upon physical exam, he is found to have a blood pressure of 146/92 and has an obese BMI. His chest is free of deformity or tenderness. His heart rate was normal with a regular rhythm and free of murmurs, gallops, or rubs. His pulmonary exam revealed normal fremitus, breath sounds, and percussion. His left shoulder exhibited no palpable tenderness or deformity and had normal ranges of motion. What is the most likely diagnosis in this patient? A Myocardial infarction B Peptic ulcer disease C Pneumonia D Shoulder arthritis E Angina pectoris

E

A 66-year-old man with a past medical history of myocardial infarction 2 years ago, angina pectoris, aortic regurgitation, congestive heart failure, atrial fibrillation, and chronic obstructive pulmonary disease is presently being monitored in the hospital status-post admission for chest pain 1 day ago. Myocardial infarction has been ruled out. An EKG performed upon admission revealed a prolonged Qt interval and significant Q waves in the anterior leads. A diagnostic echocardiogram confirms moderate aortic and mitral valve regurgitation and a left-ventricular ejection fraction of 30%. He denies any complaints upon bedside evaluation. His physical exam reveals a blood pressure of 105/70 and tachycardia. Continuous bedside ECG monitoring notes wide, monomorphic QRS complexes with a heart rate of 160 beats per minute that spontaneously resolve within 20 seconds, reverting to the pattern identified upon admission. What antiarrhythmic agent is considered to be the pharmacologic treatment of choice in the management of this patient? A Lidocaine B Procainamide C Verapamil D Sotalol E Amiodarone

E

A 68-year-old man with a past medical history of hypertension, hyperlipidemia, cluster headaches, polymyalgia rheumatica, and diabetes mellitus type II presents due to a 2-day history of left-sided, throbbing, and constant headache of moderate severity. He admits to associated symptoms such as pain across his mandible when he eats, fever, fatigue, and muscle aches. Most alarming to the patient was a single episode of complete left eye blindness that lasted for 30 minutes, but has since resolved. He denies extremity numbness, tingling, muscle weakness, incontinence, and changes in mental status. He further denies rhinorrhea, ocular discharge, nausea, and vomiting. The physical exam was remarkable only for a tender left scalp with a noticeable pulsation underlying the tender area. What is the most likely diagnosis? A Bell's palsy B Herpes Zoster ophthalmicus C Cerebrovascular accident D Cluster headache E Temporal arteritis

E

A 70-year-old Caucasian man presents for a 3rd visit within 3 months for a reevaluation of elevated blood pressure. On the initial visit, his blood pressure was 154/86; a follow-up reading 1 month later was 150/88. He was initially recommended to follow dietary and lifestyle approaches following the 1st reading, and those recommendations were reemphasized during the 2nd visit. At this visit, his blood pressure is 150/90 in an upright, seated position. His height is 5'9" and his weight is 230 pounds, which calculates as a 34.0 BMI. What is correct regarding this patient's condition? A The diagnosis of hypertension could have been made following his 1st visit B Cerebrovascular accidents are the leading cause of death in patients with hypertension C No established link exists between this patient's obesity and hypertension development D Diastolic hypertension and a low pulse pressure is the most common finding in older patients E Vascular volume, sympathetic activation, and the RAAS contribute to essential hypertension

E

A 70-year-old man with a history of hyperlipidemia, hypothyroidism, osteoarthritis, and hypertension presents for a routine evaluation. He denies any complaints today and otherwise has no significant past medical history. His physical examination is remarkable for a pulse rate of 44 beats per minute and a blood pressure of 150/94. An EKG assessment reveals sinus bradycardia and a type II Mobitz heart block. What antihypertensive should be avoided in the management of this patient? A Fosinopril B Hydrochlorothiazide C Olmesartan D Aliskiren E Metoprolol

E

A 74-year-old Caucasian man presents with progressive exertional shortness of breath, lower extremity edema, and lightheadedness over the previous 7 months. He has a past medical history of HIV, hepatic cirrhosis secondary to chronic alcohol abuse and hemochromatosis, obesity, and thiamine deficiency. His symptoms are improved with rest. The physical examination reveals rales, an elevated JVP, cardiomegaly, S3 gallop rhythm, a high-pitched, blowing holosystolic murmur at the apex, peripheral edema, and abdominal distension suggestive of ascites. A bedside EKG noted sinus tachycardia with nonspecific ST-T wave changes and Q waves. An echocardiogram of the patient is shown in the image. What is correct regarding the clinical intervention for this patient? A Calcium channel blockers and digoxin are considered first-line agents B This patient meets cardiac transplantation indications C The target systolic blood pressure in this patient is 140 mmHg D Oral anticoagulation is essential at this time E Beta-blockers have demonstrated reductions in cardiovascular mortality

E

A 74-year-old man with a past medical history of diabetes mellitus, hypertension, and hyperlipidemia presents with severe chest pain and dyspnea. On examination, he is confused, agitated, pale, apprehensive, and diaphoretic. His pulse is weak and tachycardic, with a systolic blood pressure of 80 mmHg. He has a narrow pulse pressure, tachypnea, a weak apical impulse, significant jugular venous distention, and pulmonary crackles. Bedside electrocardiogram reveals ST-segment elevations in the anterior and septal leads, while a portable chest X-ray notes diffuse pulmonary congestion. What is the most appropriate step in the management of this patient? A Crystalloid infusion B Initiate intravenous β-Blocker therapy C Begin phenylephrine D Nitrates and morphine E Emergent percutaneous coronary intervention

E

A 79-year-old man with a past medical history of diabetes mellitus, hypertension, and hyperlipidemia presents with severe chest pain and dyspnea. He appears pale, apprehensive, and diaphoretic. He is in a confused state and agitated. His pulse is weak and tachycardic, with a systolic blood pressure of 60 mmHg. He has a narrow pulse pressure, tachypnea, a weak apical impulse, and significant jugular venous distention. His lungs are free of crackles. Bedside electrocardiogram reveals ST-segment elevations in the anterior and septal leads. What is the preferred initial pharmacologic agent of choice for this patient? A Phenylephrine B Norepinephrine C Metoprolol D Dobutamine E Dopamine

E

A travel clinic frequently administers various vaccinations to people who will be traveling overseas. What is a contraindication to some vaccinations? Answer Choices: A Penicillin allergy B Iodinated dye allergy C Low-grade fever of short duration (less than 2 days) D Rhinorrhea E AIDS (Acquired Immune Deficiency Syndrome)

E

After a serious argument with his wife about his diet and habits (he is overweight and a heavy smoker), your 55-year-old neighbor went to a pub and got drunk. On the way back to home, he felt sudden chest pain radiating to the left shoulder, shortness of breath, sweating, and anxiety. His ECG shows pathological Q wave and ST elevation. Laboratory results show that troponin levels are elevated. What caused the rise in troponin levels in this patient? A Fatty changes in myocardial cells B Increased synthesis of adhesion molecules C Coexisting chronic viral infection D Alcohol E Injury to the myocardial cell membrane

E

An 18-year-old woman presents with a 1-week history of vaginal discharge that has become progressively worse. She describes it as watery with a lightly cloudy appearance. She indicates that it is mildly painful. You inquire about her sexual practices, and she confirms that she does not practice safe sex. A Gram stain identifies several WBCs, but no organisms are identified. A KOH wet prep reveals no hyphae. Question: What is the most appropriate treatment? Answer Choices: A Methenamine B Penicillin C Metronidazole D Ceftriaxone E Azithromycin

E

An 84-year-old man with poorly controlled hypertension is evaluated for severe chest pain that came on suddenly; the pain is accompanied by an episode of syncope. The chest pain is described as "ripping", and it radiates to the abdomen and back. There is no known history of trauma, injuries or accidents; he denies any fever, chills, cough, dyspnea, palpitations, nausea, vomiting or diarrhea. The physical exam is notable for tachycardia, a blood pressure of 188/120, and a high-pitched blowing decrescendo murmur at the right 2nd interspace. What agent is the preferred initial treatment in the management of this patient? A Nitroprusside B Diltiazem C Morphine sulfate D Enalapril E Labetalol

E

An audiologist is asked to evaluate a mentally disabled 1-year-old who does not seem to be responding appropriately to sound. The child has known congenital heart disease. What disorder might this child have? Answer Choices: A Congenital syphilis B Fetal alcohol syndrome C Cretinism D Lead poisoning E Congenital rubella

E

An elderly patient is admitted with a acutely severe myocardial infarction and quickly develops significant signs and symptoms of cardiogenic shock. These include hypotension, altered mental status, cold clammy skin, as well as metabolic acidosis seen on laboratory tests. What is the most appropriate initial pharmaceutical choice for a patient suffering from this type of shock? A Aspirin B Clopidogrel C Lidocaine D Beta blockers E Vasopressors

E

An illness characterized by fever, lymphadenopathy, and headache occurred among patrons of a riding stable; 29 people were ill. Most cases occurred in mid-October. Most of the 29 had serologic evidence of acute toxoplasmosis; an additional 5 persons at the stable had serologic evidence thereof, but they remained asymptomatic. Of the 34 persons, 30 were identified as women, and 24 were between 16 and 30 years of age. 20 people at another stable were interviewed and tested for toxoplasmosis antibodies; they were all negative. Cats from the stable associated with the outbreak were bled, and serologic tests revealed that 2 of the 3 cats had elevated toxoplasmosis titers. Based on this information, what is the most logical deduction? Answer Choices: A Women are much more likely to acquire toxoplasmosis than men B Symptoms of toxoplasmosis are diagnostic C Toxoplasmosis is mostly transmitted from horses D Cats are non-transmitters of toxoplasmosis E The patrons acquired the illness from the cats

E

A 57-year-old woman underwent a hemicolectomy for adenocarcinoma of the colon. Which of the following recommendations is part of postoperative monitoring for a potential recurrence? A Annual fecal occult blood testing B Annual chest radiograph C Annual CA 19-9 testing D Annual flexible sigmoidoscopy E Annual colonoscopy

E Annual Colonoscopy Routine follow-up after surgical resection of a colon cancer includes annual colonoscopy not sigmoidoscopy, which only assesses the distal colon. The tumor marker for colon cancer is carcinoembryonic antigen (CEA) not carbohydrate antigen 19-9 (CA 19-9), which is used for pancreatic cancer. There is no role for annual chest films or fecal occult blood testing to monitor for a recurrence.

A 37-year-old female presents to the office complaining of increasing heavy vaginal bleeding with her menses. CBC reveals an iron deficient anemia. Her pelvic exam is notable for a moderately enlarged uterus. A transvaginal ultrasound reveals a 3-cm leiomyoma and a 15-cm leiomyoma. The patient wishes to preserve her fertility. Appropriate management would include which of the following? A Expectant management aimed at pain reduction B Bilateral uterine artery embolization C Reduction of blood loss with combined oral contraceptives D Immediate hysterectomy E Gonadotropin-releasing hormone analog followed by myomectomy

E Gonadotropin-releasing hormone analog followed by myomectomy E GRH causes a reversible hypogonadism, which reduces tumor size, makes surgical intervention safer, and reduces bleeding. The patient wishes to preserve her fertility, making embolization and hysterectomy non-viable options. COC do not significantly reduce bleeding, and do not regress the tumor for optimal surgical removal.

A 29-year-old woman presents in July to your office with symptoms of palpitations, sore neck, and excessive sweating, despite using her air conditioner all the time. No surgical or trauma history is noted. She is currently not taking any medications. Vitals include the following: BP = 124/68, pulse = 110 beats per minute, respirations = 18 per minute, and temperature = 101 o F orally. Upon exam, her thyroid is mildly enlarged without nodules, and severely tender. No local erythema or heat is noted. What is the most likely diagnosis in this patient? A Thyroid abscess B Papillary thyroid cancer C Hashimoto's thyroiditis D Graves' disease E Subacute thyroiditis

E Subacute Thyroiditis This patient is presenting with signs and symptoms of hyperthyroidism, most likely caused by subacute thyroiditis. The enlarged and tender thyroid, along with the fever, are classic findings in this condition. Choice A, thyroid abscess, is a localized infection of the thyroid. It would present with heat, swelling, and erythema in the area of tenderness. Choice B, papillary thyroid cancer, commonly presents with either a local non-tender thyroid nodule or as lymph node metastasis. Choice C, Hashimoto's thyroiditis, is a cause of chronic thyroiditis and is associated with a clinical presentation of hypothyroidism. Choice D, Graves' disease, is the most common cause of hyperthyroidism, but does not present with thyroid tenderness and fever.

During a pre-surgical workup, a patient reports that she had received Desmopressin following the birth of her son. Her CBC is within normal range, as well as her PT and aPTT. What is the most likely cause of her needing this medication? A Allo immunization following vaginal delivery B Hemolytic anemia C Iron deficiency anemia D Thalassemia E Von Willebrand's disease

E Von Willebrand's Disease Desmopressin is the mainstay of therapy for people with von Willebrand's disease. Hemolytic anemia, iron deficiency anemia, and thalassemia would all have abnormal findings on the CBC. Allo immunization does not require desmopressin therapy.

Your patient is a 66-year-old female who has been dropping her coffee cup and concurrently slurring her speech. The episodes last for approximately 15 minutes. Her blood work, carotid dopplers, and MRI of the brain are normal and you suspect recurrent transient ischemic attacks (TIAs). Which of the following is NOT approved or recommended for the prevention of stroke in this patient? A aspirin B extended-release dipyridamole plus aspirin C clopidogrel D prasugrel E warfarin

E Warfarin Aspirin, aspirin plus extended-release dipyridamole, and clopidogrel are all antiplatelet agents and approved for use to reduce recurrent TIAs and ischemic cerebrovascular accidents (CVAs). Prasugrel is not FDA-approved for this indication. Warfarin is an anticoagulant and has no role in prevention of either recurrent TIA or ischemic CVA.

A 69-year-old man presents with a 1-day history of diarrhea that was initially watery; it is now bloodstained. He lives on a farm and reports that the drinking water had recently been contaminated by cow manure. A stool examination reveals polymorphonuclear leukocytes, but no trophozoites. Question What is the causative organism? Answer Choices 1 Staphylococcus aureus 2 Clostridium perfringens 3 Escherichia coli serotype 4 Giardia lamblia 5 Salmonella typhi

E coli In Escherichia coli serotype O157:H7 - also termed enterohemorrhagic E. coli food poisoning - patients present with severe crampy abdominal pain and watery diarrhea, which can become bloodstained within 24 hours. Sources of infection include drinking unpasteurized milk and water contaminated by cow manure, as well as ingesting raw or undercooked beef. A stool examination reveals polymorphonuclear leukocytes. The E. coli can be isolated from stool cultures. Complications include the hemolytic-uremic syndrome. Management includes supportive care with rehydration. Giardia lamblia can be acquired by drinking contaminated water. Patients present with watery diarrhea. The incubation period is around 1 week. There are trophozoites in the stool but no leukocytes. Patients with Salmonella typhi food poisoning usually present with bloody diarrhea. Sources of infection include beef, poultry, eggs, and dairy products. Examination of the stool reveals mononuclear leukocytes. Patients with Staphylococcus aureus food poisoning may present with vomiting and watery diarrhea 1 to 6 hours after ingesting contaminated food like processed meat. There are no fecal leukocytes or trophozoites. In Clostridium perfringens food poisoning, patients present with watery diarrhea 8 to 24 hours after ingesting contaminated food like inadequately cooked meat or legumes. No fecal leukocytes or trophozoites are present on stool examination.

A 64-year-old, nonsmoking male presents to the clinic with complaints of headache and dizziness. Initial labs reveal hemoglobin of 20.1 g/dl, hematocrit of 60%, and platelet count of 567,000. Which of the following laboratory finding confirms the suspected diagnosis? A Bone marrow hypocellularity B Elevated ferritin C Elevated serum erythropoietin D Presence of Philadelphia chromosome E Presence of JAK2 mutation

E presence of JAK2 mutation The clinical presentation is most consistent with polycythemia vera (PCV), which, in the presence of a markedly elevated red blood cell mass, is confirmed by the presence of the JAK2 mutation. The patient's erythropoietin (C) and ferritin (B) are most likely decreased as a result of bone marrow overproduction of RBCs and use of iron stores to produce these RBCs. The bone marrow of patients with PCV is typically hypercellular (A) across all cell lines.

A 48-year-old female presents with a gradual progression of fatigue, pallor, and dyspnea on exertion over the last few months. Initial CBC results show a hemoglobin of 10.2 mg/dL, hematocrit of 31%, an MCV of 74 fL, and a reticulocyte count of 0.1%. After treatment for her anemia begins, which of the following diagnostic studies can be ordered first (earliest) to demonstrate a response to therapy? A Bone marrow aspirate B Ferritin C Hemoglobin/hematocrit D Iron binding capacity E Reticulocyte count

E reticulocyte count The patient most likely has an underlying iron deficiency anemia (IDA) which requires confirmation and then evaluation for sources of chronic blood loss. IDA is treated with iron replacement (e.g., ferrous sulfate) and response to therapy can be assessed in 7-10 days via a reticulocyte count.

A 69-year-old man presents with a 1-day history of diarrhea that was initially watery; it is now bloodstained. He lives on a farm and reports that the drinking water had recently been contaminated by cow manure. A stool examination reveals polymorphonuclear leukocytes, but no trophozoites. Question What is the causative organism? Answer Choices 1 Staphylococcus aureus 2 Clostridium perfringens 3 Escherichia coli serotype 4 Giardia lamblia 5 Salmonella typhi

E. Coli In Escherichia coli serotype O157:H7 - also termed enterohemorrhagic E. coli food poisoning - patients present with severe crampy abdominal pain and watery diarrhea, which can become bloodstained within 24 hours. Sources of infection include drinking unpasteurized milk and water contaminated by cow manure, as well as ingesting raw or undercooked beef. A stool examination reveals polymorphonuclear leukocytes. The E. coli can be isolated from stool cultures. Complications include the hemolytic-uremic syndrome. Management includes supportive care with rehydration. Giardia lamblia can be acquired by drinking contaminated water. Patients present with watery diarrhea. The incubation period is around 1 week. There are trophozoites in the stool but no leukocytes. Patients with Salmonella typhi food poisoning usually present with bloody diarrhea. Sources of infection include beef, poultry, eggs, and dairy products. Examination of the stool reveals mononuclear leukocytes. Patients with Staphylococcus aureus food poisoning may present with vomiting and watery diarrhea 1 to 6 hours after ingesting contaminated food like processed meat. There are no fecal leukocytes or trophozoites. In Clostridium perfringens food poisoning, patients present with watery diarrhea 8 to 24 hours after ingesting contaminated food like inadequately cooked meat or legumes. No fecal leukocytes or trophozoites are present on stool examination.

A patient with severe hemophilia A is brought into the emergency department following an automobile accident. Concern is for hemorrhage. What is the best intervention to raise factor VIII levels to hemostatic levels? A Fresh frozen plasma B Cryoprecipitate C Recombinate D DDAVP (desmopressin) E EACA (aminocaproic acid)

EACA While fresh frozen plasma and cryoprecipitate have been used in the past for hemophilia A hemostatis, volumes may be too large or unable to reach levels to achieve hemostasis in severe hemophilic A patients. Recombinate is a commercial lyophilized factor VIII concentrate that can case factor VIII levels to reach hemostasis in smaller volumes, and do not have the disadvantages of plasma or cryopreciptate. DDAVP is not used in severe hemophilics. EACA is used to enhance hemostasis, but is not able to achieve initial hemostasis seen with recombinate therapy.

A 32-year-old woman presents several hours after a grand mal seizure. Her husband states that a few days before the seizure she experiences headache, nausea, vomiting, fever, and "was not herself". She is on glucocorticoid therapy due to a kidney transplant 1 year ago. On examination, you find a lethargic, febrile (39 C), dysphasic patient; there is right-sided hemiparesis, and meningeal signs are present, but detailed examination cannot be performed because of lack of cooperation. Cerebrospinal analysis (CSF) shows normal opening pressure, xanthochromia, mononuclear pleocytosis (100 white blood cells), elevated protein (100 mg/d), and a normal serum to glucose ratio; EEG shows focal abnormalities over temporal lobes, and her CT is normal. While waiting for the results of polymerase chain reaction, you start supportive care, anticonvulsants, diuretics, and acyclovir. Question What factor guides you to the choice of acyclovir? Answer Choices 1 EEG changes 2 CSF findings 3 Glucocorticoid therapy 4 Focal neurological deficit 5 Psychiatric symptoms

EEG In an immunocompromised patient with this clinical picture, you should think about herpes encephalitis. Its presentation is often atypical: fever, headache, and alteration of consciousness, focal neurological deficits, seizures, and other signs of acute encephalitis. When you suspect encephalitis, the workup must be initiated rapidly, but the workup should not delay treatment. There are no pathognomonic clinical findings in herpes simplex encephalitis, but electroencephalography (EEG), with a specificity of 32% and sensitivity of 84%, should guide your decision about the therapy. Usual abnormal EEG patterns are focal abnormalities (periodic complexes and periodic lateralizing epileptiform discharges, spike and slow- or periodic sharp-wave patterns) over temporal lobes or diffuse slowing. The diagnosis will be confirmed only by polymerase chain reaction or brain biopsy, but those results will not be available immediately, and they should not delay therapy. In this case, the CT excluded a space-occupying lesion so that a lumbar puncture could be performed. CSF findings are not specific; they will only raise the suspicion of viral cause of the disease. Patients on glucocorticoid therapy and consequent immune suppression are prone to different infections, not only herpes simplex encephalitis. A focal neurological deficit is a non-specific finding. Focal neurologic deficits, cerebrospinal fluid pleocytosis, and abnormalities on computed tomography may initially be absent in herpes simplex encephalitis. Psychiatric symptoms are not specific for herpes simplex encephalitis; they are part of clinical picture of a variety of CNS disorders.

A 22-year-old African-American woman comes to the emergency department because of low-grade fever, abdominal pain, and severe, watery, sometimes blood-tinged mucous diarrhea, with 10-15 stools per day for the last 48 hours. She says she has been drinking lots of liquids since the symptoms started, but remained diarrheic. One week ago, she had rhinorrhea with a purulent discharge, for which she received a prescription of amoxicillin. Her family history is unremarkable. Vital signs on admission are BP 110/70 mmHg, HR 86 bpm, RR 16/min, and temperature 38.0°C (100.4°F). Her abdomen is soft, diffusely tender to palpation, with mild rebound tenderness. The remainder of the physical examination shows no abnormalities. A complete blood count shows Hct 46%, Hb 13.1 mg/dL MCV 82 fl, MCHC 30 g/dL, WBC 16.200 /mm3. Na+ 138, K+ 4.8, Cl- 101. A plain abdominal radiograph is normal. Antibiotic-associated colitis is suspected, and a rapid diagnosis is sought. Question What is the most appropriate next step? Answer Choices 1 Sigmoidoscopy 2 Culture for Clostridium difficile 3 ELISA assay for Clostridium difficile toxin 4 Common antigen testing 5 Test for fecal leukocytes

ELISA for C diff Antibiotic-associated colitis is an important complication of antibiotic use, and is estimated to occur in 1% of hospitalized patients. The etiologic agent is Clostridium difficile, a Gram-positive bacillus that colonizes the bowel of 2-3% of healthy adults. Antibiotic use, particularly clindamycin, penicillins, and cephalosporins, can lead to proliferation of C. difficile. This organism produces 2 toxins, termed A (enterotoxic) and B (cytotoxic), which cause colitis. The most common symptoms are watery diarrhea and abdominal pain. Patients with severe disease can have intense diarrhea, with up to 30 stools per day, low-grade fever, and abdominal pain. A minority of cases progress to severe colitis and toxic megacolon. An ELISA assay for C. difficile toxin is one of the most popular tests for diagnosing C. difficle colitis because it yields rapid results (usually within 24 hours). It is highly specific (99%), but obtaining maximum sensitivity may require repeat testing. The estimated sensitivity of ELISA testing is 60-95%. An alternative to ELISA is PCR analysis for toxic. Compared to ELISA, PCR has a relatively higher sensitivity. Common antigen testing tests for GDH, an enzyme that is specifically produced by the C. difficle organism. This test is sometimes used to screen samples with follow-up toxin assays performed on positive samples. It is not a routine first-line test for the diagnosis of C. difficle colitis. Anaerobic culture for C. difficile take a long time, is costly, and is primarily reserved for epidemiological purposes. Fecal leukocytes have low sensitivity for antibiotic-associated colitis and are found in only 50% of cases. Sigmoidoscopy or colonoscopy may be used as an adjunct to laboratory diagnosis in certain settings, such as when there is a high clinical suspicion of C. difficle colitis despite a negative toxin assay. It should be emphasized that endoscopic bowel examinations are more dangerous in patients with megacolon and should be used carefully if this complication is suspected.

During the physical exam of a patient with a suspected pleural effusion, you ask the patient to make the sound "eee." You note on auscultation that the transmission is auscultated as "ay," suggestive of resonance through fluid. What is the name of this exam technique? A Bronchophony B Diaphragmatic excursion C Egophony D Tactile fremitus E Whispered pectoriloquy

Egophony Normal lungs transmit spoken sounds faintly and with indistinct syllables, except over main bronchi. An area of fluid, such as a pleural effusion, consolidation or atelectasis, and areas of fibrosis will increase sound transmission and alter the distinction of the sound. This occurs for both whispered and spoken sounds. The utilization of the spoken sound "eee," auscultating for a change to "ay" due to fluid within the lung fields, is termed egophony. The use of whispered sounds and generally spoken sounds to determine lung changes are termed whispered pectoriloquy and bronchophony respectively. Diaphragmatic excursion is performed to determine the thoracic diaphragmatic movement during respiration. Tactile fremitus assesses chest vibration during vocalization, with changes being noted in the presence of consolidation (increases fremitus) and pleural effusion (decreased or absent fremitus).

A 45-year-old woman presents with recent, unexplained weight loss and lethargy. She has had several episodes of nausea and vomiting. She admits to craving salty foods. On physical examination, there is a hyperpigmentation to her skin and mucus membranes. Her blood pressure is 85/60 mmHg. Her laboratory results show hyponatremia and hyperkalemia. Her serum ACTH level is elevated. Question What is causing her hyperpigmentation? Answer Choices 1 Elevated ACTH 2 Hyponatremia 3 Hyperkalemia 4 Elevated mineralocorticoids 5 Elevated glucocorticoids

Elevated ACTH Explanation This patient has signs and symptoms consistent with Addison's disease. Addison's disease is primary adrenal insufficiency. The symptoms include weight loss, lethargy, nausea, vomiting, and salt craving. Because of the low level of adrenocortical hormones, there is an increase in ACTH. Hyperpigmentation is secondary to the elevated ACTH. A portion of ACTH contains melanin-stimulating hormone, thus explaining the increase in pigmentation. Hyperpigmentation would not be associated with any of the other choices.

A 45-year-old woman presents with recent, unexplained weight loss and lethargy. She has had several episodes of nausea and vomiting. She admits to craving salty foods. On physical examination, there is a hyperpigmentation to her skin and mucus membranes. Her blood pressure is 85/60 mmHg. Her laboratory results show hyponatremia and hyperkalemia. Her serum ACTH level is elevated. Question What is causing her hyperpigmentation? Answer Choices 1 Elevated ACTH 2 Hyponatremia 3 Hyperkalemia 4 Elevated mineralocorticoids 5 Elevated glucocorticoids

Elevated ACTH This patient has signs and symptoms consistent with Addison's disease. Addison's disease is primary adrenal insufficiency. The symptoms include weight loss, lethargy, nausea, vomiting, and salt craving. Because of the low level of adrenocortical hormones, there is an increase in ACTH. Hyperpigmentation is secondary to the elevated ACTH. A portion of ACTH contains melanin-stimulating hormone, thus explaining the increase in pigmentation. Hyperpigmentation would not be associated with any of the other choices.

A 62-year-old man presents with vision problems and difficulty swallowing. Over the last week, he has had a constellation of symptoms; they began with numbness and tingling in his feet and progressed to weakness that now affects both lower and upper extremities. Within the last day, he has started to notice difficulty swallowing and double vision. He also feels it is difficult for him to take a big breath. His past medical history is noncontributory, and he takes no medications. Exam reveals bilateral absence of patellar and ulnar reflexes. A lumbar puncture is performed to confirm the diagnosis. Question What cerebrospinal fluid (CSF) finding is most likely? Answer Choices 1 Decreased CSF glucose content 2 Decreased CSF protein content 3 Elevated CSF polymorphonuclear cell count 4 Elevated CSF protein content 5 Elevated CSF lymphocyte count

Elevated CSF proteins Correct response is elevated CSF protein content. Symmetrical ascending paralysis as described in this patient is indicative of Guillain-Barré syndrome (GBS). The cause of GBS is unknown, but it is generally thought to be an inflammatory autoimmune process. More than half of patients with GBS report an antecedent illness. The antibodies produced in response to antigens present in the infectious agent are thought to cross-react with components of human neurons, prompting an acute postinfectious demyelinating process. Lumbar puncture characteristically reveals elevated CSF protein content. Other results are normal, although the white blood cell count may be somewhat elevated. Decreased CSF glucose and increased polymorphonuclear cell counts are seen in acute bacterial meningitis. Decreased CSF glucose and elevated CSF lymphocyte counts are commonly seen with meningitis caused by fungi. Viral meningitides usually presents with elevated lymphocyte counts, normal CSF glucose, and normal or slightly elevated CSF protein levels.

A 3-year-old girl presents with terrible head pain, increasing fever, and irritability. On examination, you find the child febrile, 103o F, and she cries when you lift her head from the examining table. The neck appears to be quite stiff when manipulated. You perform a lumbar puncture. What findings would you expect if this child had bacterial meningitis? Answer Choices 1 Elevated protein 2 Elevated glucose 3 Decreased number of WBC 4 Decreased opening pressure 5 Negative Gram stain

Elevated Protein Bacterial meningitis is usually an acute infection of the subarachnoid space and meninges caused by a variety of organisms. In the neonatal period, infection with group B. streptococcus, E. coli, Listeria monocytogenes, and other Gram-negative enteric bacteria are the most likely. In children, the most important pathogens are Haemophilus influenzae (most common), Neisseria meningitidis (meningococcus), and Streptococcus pneumoniae (pneumococcus). In adults, pneumococcus and meningococcus are the most important causes of meningitis. Older individuals are at increased risk for infection with Listeria and Gram-negative enteric bacteria. The classic triad of symptomsof meningitis include fever, headache and stiff neck. Respiratory distress, increased response to auditory stimulus, and increased Babinski reflex are not characteristic of presenting symptoms of this disease. Pallor is not characteristic, either. Once the diagnosis of meningitis is suspected, immediate examination of the spinal fluid is indicated with lumbar puncture. Spinal fluid abnormalities usually include elevated protein, low glucose (hypoglycorrhachia), elevated opening pressure, and elevated white cells in the 100 - 10,000 cells/cubic mm range with a predominance of polymorphonuclear cells. The Gram stain is usually positive in more than 90% of patients.

Elevated Serum Phosphate

Elevated Serum Phosphate: *Phosphorus goes up and Calcium goes down • Elevated serum Ph →Increases PTH! →Decreases Serum Ca→ Decreases Vit D • The role of PTH is to increase serum calcium and stimulates osteoclasts to breakdown bone to release more calcium in the serum (decalcification), osteosclerosis, osteoporosis, growth retardation in children • Secondary hyperparathyroidism causes Renal osteodystrophy • Tx- reduce phosphate by Rx phosphate binders; Selevamir Hydrochlroide (Renagel) • If calcidiol 25 (OH) D level is <30 ng/mL add supplemental Vit D

A 38-year-old woman presents with fatigue, amenorrhea, and easy bruising. Examination reveals multiple healing bruises, central obesity, a round face, supraclavicular fat pads, abdominal striae, and hypertension. Question What is an expected finding? Answer Choices 1 Hypoglycemia 2 Hyperkalemia 3 Elevated serum cortisol 4 Decreased urine cortisol 5 Positive cosyntropin stimulation test

Elevated serum cortisol Explanation The correct answer is elevated serum cortisol. The symptoms of fatigue, amenorrhea, and easy bruising in the setting of the above physical exam findings is consistent with a diagnosis of Cushing's syndrome. The patient will have an elevated serum cortisol level. Hypoglycemia is an incorrect response. Cushing's syndrome is associated with hyperglycemia. Hyperkalemia is an incorrect response. Cushing's syndrome is associated with hypokalemia. Decreased urine cortisol is an incorrect response. Cushing's syndrome is associated with elevated urine cortisol. Positive cosyntropin stimulation test is an incorrect response. A positive cosyntropin stimulation test occurs with Addison's disease.

A 45-year-old woman presents with a weight gain of 50 pounds; she attributes the weight gain to her hard-working lifestyle as a corporate attorney. She usually eats out and rarely exercises. She has a fasting blood glucose of 220 mg/dL and has a urinary albumin/g creatinine of 40 mg. These are new findings since her visit 5 years ago. She has been following a high-protein diet, primarily consisting of animal proteins, for the past couple of days. What dietary and lifestyle interventions should you recommend due to her blood glucose and urinary albumin findings? Answer Choices 1 Continue high-protein diet for weight loss and reducing proteinuria, begin exercising 2 Continue present high-protein diet for weight loss and glycemic control, begin exercising 3 Continue present high-protein diet for weight loss, glycemic control, and proteinuria reduction 4 Encourage a low-calorie diet, modest protein and carbohydrate restriction with exercise 5 Discontinue present diet and recommend a low-fat and extremely low-protein diet

Encourage a low-calorie diet, modest protein and carbohydrate restriction with exercise 1 popular diet therapy is the high-protein diet; patients consume >25% of calories from protein sources. Government guidelines typically suggest protein intake to be in the <20% range. Although these diets may assist in weight loss in the short term (≤6 months) by increasing satiety and decreasing postprandial sugar fluxes and insulin surges, their long-term benefit is less clear. Diets suggested for diabetics typically include a combination of food groups including carbohydrates, proteins, and fats. Weight loss typically assists with glycemic control and reduces insulin resistance. Some diabetic patients (1 diagnostic criterion is a random sugar >200 mg/dL) can control their diabetes with diet alone. This patient has obesity, hyperglycemia, and proteinuria, a marker of kidney disease, and she should follow modest reductions in protein intake. The kidney disease outcome initiative (National Kidney Foundation) suggests modest reductions in protein intake. Severe protein restriction is not suggested for diabetics or patients with kidney disease because it may not prevent declines in kidney function; it may contribute to protein, vitamin, and nutrient malnutrition. Although short-term protein loads will increase glomerular filtration rate, long-term, high-protein diets may cause a decline in glomerular filtration rate and kidney function, particularly in patients with diabetes and existing kidney disease. Dietary sources of protein (e.g., chicken, lamb, and beef) may include advanced glycosylation end products (AGE); these are the amino portions of proteins that have reacted through glycation and oxidation to cause the formation of sugar proteins. Consumed AGEs and AGEs created when excess proteins link with excess sugar may lead to cross-linking and scarring of the matrix and glomerular wall portions of the kidney, leading to scarring. Proteins may also promote tissue damage by generating free radicals and causing oxidative stresses. Vegetable-based proteins appear to have lesser effects on renal function than do animal-based proteins.

A 51-year-old female presents to the office complaining of intermittent vaginal spotting for three months. She has a history of well-controlled hypertension. She is a nonsmoker. Her LMP was two years ago. Her family history is significant for colon cancer. Physical exam and pelvic exam were performed and unremarkable. What is the best diagnostic step in evaluating her vaginal bleeding? A Transvaginal ultrasound B Endometrial sampling C Pap smear D Pregnancy test E STD testing

Endometrial sampling B In postmenopausal women with a family history of colon cancer, there is a 30% risk of endometrial cancer. Pregnancy and STD are less likely, and do not exclude endometrial cancer. Pap smear and ultrasound are useful but can be negative. Endometrial sampling is required to confirm or rule out cancer.

A patient with long-standing, untreated acromegaly is seen in your office with symptoms of severe headaches. After completing a thorough history and physical exam, you order a set of x-rays including a skull series. Which of the following findings would you expect in this patient? A Punched out lesions B Basilar skull fracture C Metastatic bone lesions D Enlarged sella tursica E Thinning of the skull

Enlarged Sella tursica The correct choice is D, enlarged sella tursica. This finding is seen in 90% of patients with acromegaly. Other findings on skull radiographs include thickened calvarium (upper portion of the skull), enlarged mandible, and sinuses. Bony growth is a hallmark of the disease. The pituitary adenoma, which typically causes the disease, can be found in the sella tursica. This disorder doesn't typically metastasize, and is not associated with metastatic bone cancer, as noted in choice C. Choice A, punched out lesions, are commonly associated with Paget's disease. There is no history of head trauma, as would be the case in choice B, basilar skull fracture. As noted earlier, the skull may be thickened. Therefore, choice E, thinning of the skull, would not fit this patient's presentation.

A 37-year-old man fell from a ladder as he finished hanging the Christmas lights on his house. The right side of his head hit the alley cement, and he lost consciousness for about 1 minute; he woke up with a headache, but he had no other complaints. A few hours later, the patient is brought to the emergency room by his neighbor because of an intense headache, confusion, and left hand hemiparesis. On examination, the patient has a bruise located over the right temporal region, mydriasis, and right deviation of the right eye, papilledema, and left extensor plantar response. An emergency CT scan of the head without contrast reveals a lens-shaped hyper-density under the right temporal bone with mass effect and edema. What is the most likely diagnosis? Answer Choices 1 Epidural hematoma 2 Subdural hematoma 3 Subarachnoid hemorrhage 4 Intracerebral parenchymal hemorrhage 5 Acute meningitis

Epidural Hematoma Epidural hematoma most often results from a traumatic tear of the middle meningeal artery. Although a lucid interval ranging from minutes to hours followed by altered mental status and focal deficits is typical for epidural hematoma, this clinical picture is only encountered in up to 1/3 of the patients. The collection of blood between the skull and dura mater causes an evident mass effect with ophthalmic nerve palsy and the contralateral hemiparesis. Surgical evacuation of the clot via burr holes is the treatment of choice. Subdural hematoma results from a traumatic rupture of the bridging veins that connect the cerebrum to the venous sinuses within the dura. This venous hemorrhage will result in a gradual increase of the hematoma, with a progressive clinical picture over days or weeks. The CT scan will show a concave, crescent-shaped hyper-density compared to the convex, lens-shaped hyper-density in epidural hematoma. Subarachnoid hemorrhage is the result of an aneurysm rupture; the most common is the congenital berry aneurysm. The clinical picture is of a sudden, severe headache with meningeal irritation. A CT scan will show blood in the subarachnoid space, and a lumbar puncture will reveal xanthochromia CSF. Intracerebral parenchymal hemorrhage is most likely caused by hypertension complicated with Charcot-Bouchard aneurysms. The blood accumulates into the brain substance and most commonly involves the basal ganglia. Acute meningitis is not associated with trauma. Fever and signs of meningeal irritation dominate the clinical picture. Lumbar puncture, indicated if there are no focal neurological signs on clinical examination, will be the diagnostic procedure. The CT scan of the patient presented in this case is characteristic for epidural hematoma, and there is no indication for a lumbar puncture.

A 46 year-old female has just been diagnosed with ovarian cancer. Where would you expect the ovarian neoplasm to most likely arise from? A Germ cell B Stromal cell C Epithelial cell D A Metastatic tumor E Endometrial cell

Epithelial cell C The most common and most lethal of the ovarian neoplasms arise from the ovarian epithelium found both on the surface of the ovary and in subsurface locations. The ovarian epithelium generally is in good health and appears as a simple epithelium. With neoplastic transformation there undergoes metaplastic changes into what is termed Müllerian epithelium. The Müllerian epithelium has a variety of subtypes, each providing a specific phenotype of the tumor and may have a different clinical presentation. Epithelial tumors are the most common ovarian neoplasm and may be: benign (50%); malignant (33%); or borderline malignancy (16%). Tumors may also metastasize (D) to the ovary from the breast, gastric, pancreatic, and colon primary cancers, but is not as common. Germ cell tumors (A) are more similar to testicular tumors in males. Stromal tumors (B) arise from steroid hormone producing cells. Endometrial (E) is usually specific to the uterus.

A 46 year-old female has just been diagnosed with ovarian cancer. Where would you expect the ovarian neoplasm to most likely arise from? A Germ cell B Stromal cell C Epithelial cell D A Metastatic tumor E Endometrial cell

Epithelial cells The most common and most lethal of the ovarian neoplasms arise from the ovarian epithelium found both on the surface of the ovary and in subsurface locations. The ovarian epithelium generally is in good health and appears as a simple epithelium. With neoplastic transformation there undergoes metaplastic changes into what is termed Müllerian epithelium. The Müllerian epithelium has a variety of subtypes, each providing a specific phenotype of the tumor and may have a different clinical presentation. Epithelial tumors are the most common ovarian neoplasm and may be: benign (50%); malignant (33%); or borderline malignancy (16%). Tumors may also metastasize (D) to the ovary from the breast, gastric, pancreatic, and colon primary cancers, but is not as common. Germ cell tumors (A) are more similar to testicular tumors in males. Stromal tumors (B) arise from steroid hormone producing cells. Endometrial (E) is usually specific to the uterus.

A 35-year-old man with a 10-year history of type I diabetes presents with early fullness, abdominal pain, stomach spasms, heartburn, nausea, vomiting, bloating, and lack of appetite. Symptoms are getting progressively worse despite the patient's recommended lifestyle and dietary changes. Subsequent isotopic gastric emptying study confirms gastroparesis. Question What initial treatment should be suggested to this patient? Answer Choices 1 Erythromycin 2 Botulinum toxin 3 Misoprostol 4 Amoxicillin 5 A jejunostomy tube

Erythromycin The patient has presented with diabetic gastroparesis, which is a delayed gastric emptying of the stomach. This results in food remaining in the stomach for a longer period of time than normal. Erythromycin is useful in treating gastroparesis due to this pro-motility effect. Botulinum toxin injection into the pyloric sphincter is sometimes used in improving symptoms of nausea and vomiting in patients with refractory diabetic gastroparesis. However, this invasive method can be considered only after all other methods with fewer side effects are exhausted. Misoprostol is used for the prevention of non-steroidal anti-inflammatory drugs-induced gastric ulcers. Your patient has no signs and symptoms of peptic ulcer disease and does not need misoprostol. Amoxicillin in combination with metronidazole and proton pump inhibitor is therapy suggested for uncomplicated cases of ulcers associated with Helicobacter pylori. Your patient has no signs and symptoms of ulcer caused by Helicobacter pylori. A jejunostomy tube is the invasive method that might be considered after all other methods with fewer side effects are exhausted.

A 45-year-old chronic alcoholic presents with history of massive hematemesis. This hematemesis followed a bout of prolonged vomiting. Patient has been a known alcoholic for 20 years. On examination, he has a pulse rate of 100/min and a BP of 90/70 mm Hg with cold extremities. These findings are typical for what condition? Answer Choices 1 Hiatus hernia 2 Esophageal pulsion diverticulum 3 Barrett's esophagus 4 Esophageal squamous cell carcinoma 5 Esophageal laceration

Esophageal Laceration Esophageal laceration extends to the submucosal veins that bleed profusely, also known as Mallory-Weiss syndrome. Hiatus hernia may cause a reflux along with inflammation and possible ulceration, but bleeding is not typically massive. Diverticula of the esophagus usually do not bleed. Barrett's esophagus (metaplasia with gastric mucosa) is associated with reflux and inflammation and possible ulceration, but bleeding is not usually massive. Esophageal squamous cell carcinoma will cause ulceration, but massive bleeding is not common.

A 45-year-old chronic alcoholic presents with history of massive hematemesis. This hematemesis followed a bout of prolonged vomiting. Patient has been a known alcoholic for 20 years. On examination, he has a pulse rate of 100/min and a BP of 90/70 mm Hg with cold extremities. These findings are typical for what condition? Answer Choices 1 Hiatus hernia 2 Esophageal pulsion diverticulum 3 Barrett's esophagus 4 Esophageal squamous cell carcinoma 5 Esophageal laceration

Esophageal laceration Esophageal laceration extends to the submucosal veins that bleed profusely, also known as Mallory-Weiss syndrome. Hiatus hernia may cause a reflux along with inflammation and possible ulceration, but bleeding is not typically massive. Diverticula of the esophagus usually do not bleed. Barrett's esophagus (metaplasia with gastric mucosa) is associated with reflux and inflammation and possible ulceration, but bleeding is not usually massive. Esophageal squamous cell carcinoma will cause ulceration, but massive bleeding is not common.

A 43-year-old man presents with fever, productive cough, and wheezing, leading to an intial diagnosis of bilateral patchy pneumonia and admission to the hospital. The patient's history is positive for increasing dysphagia for both solids and liquids over the past 2-3 years, and an approximate weight loss of 12 lbs recently. He occasionally gets severe chest pain for which he uses OTC meds but he has not consulted a physician. At night he experiences bouts of coughing. 8 months ago he had bronchitis and 4 months ago he had pneumonia. Both were treated on an outpatient basis. A barium swallow shows a dilated flaccid esophagus filled partly with fluid. Endoscopy reveals erythematous esophageal mucosa with resistance to passage of the endoscope at the lower esophageal sphincter. No structural lesions are found. What is the next step in the definitive diagnosis of this condition? Answer Choices 1 Biopsy of the mucosa of the esophagus 2 Swallow evaluation and video fluoroscopy 3 CT scan of the chest 4 24 hour pH monitoring 5 Esophageal manometry

Esophageal manometry Explanation This patient is suffering from achalasia of the cardia. This is a motility disorder of the esophagus that occurs due to a neurological deficit from fibrosis, scarring in the Auerbach's plexus, and degeneration of vagal fibers in the esophagus. This causes decreased peristalsis in the body of the esophagus and incomplete relaxation of the lower esophageal sphincter. Dysphagia to solids and liquids, weight loss, chest pain, nocturnal cough due aspiration of esophageal contents, and recurrent bronchitis and pneumonia are classic symptoms. Radiography and endoscopy are performed to rule out anatomic pathology. Manometry is needed for the definitive diagnosis, which shows elevated lower esophageal sphincter pressure, absence of normal peristalsis in the entire esophagus, and incomplete relaxation of the lower sphincter. Treatment is with pneumatic dilatation, endoscopic injection of botulinum toxin, or Heller myotomy. Biopsy of the mucosa may rule out malignancy at the esophagogastric junction (which can cause secondary achalasia or pseudoachalasia) will not assist in the definitive diagnosis of achalasia. Swallow evaluation and video fluoroscopy permit evaluation of the cervical esophagus only, and therefore are inadequate. CT scan of the chest is indicated in malignancy related and obstructive esophageal conditions. 24 hour pH monitoring is a tool used for diagnosis of reflux esophagitis.

A 41-year-old man presents with acute hematemesis. Abdominal exam reveals distension, no rebound or guarding, hepatosplenomegaly, and dull fluid wave. What is the most likely source of the upper gastrointestinal bleeding (UGI)? Answer Choices 1 Mallory-Weiss tear 2 Perforated duodenal ulcer 3 Chronic gastritis 4 Arteriovenous malformation (AVM) 5 Esophageal varices

Esophageal varices Bleeding from esophageal varices secondary to portal hypertension and obstruction of splenic blood flow is a common source of UGI bleeding presented to emergency departments. Mallory-Weiss tears and perforated duodenal ulcers may result in hematemesis, but they are associated with significant abdominal pain. Chronic gastritis is a common source of abdominal pain and may result in occult fecal blood. AVMs in the small intestine should be suspected in cases of hematochezia.

A 23-year-old woman presents with 'heartburn'; she mentions the reflux of her gastric juice. What pathology is most likely to be seen in this patient's esophagus? Answer Choices 1 Esophageal diverticula 2 Esophageal web 3 Esophageal laceration 4 Esophagitis 5 Mallory-Weiss tear

Esophagitis

A 23-year-old woman presents with 'heartburn'; she mentions the reflux of her gastric juice. What pathology is most likely to be seen in this patient's esophagus? Answer Choices 1 Esophageal diverticula 2 Esophageal web 3 Esophageal laceration 4 Esophagitis 5 Mallory-Weiss tear

Esophagitis Esophagitis is an inflammation of the esophagus. A variety of conditions, such as gastric reflux, are associated with esophagitis. Esophageal diverticula are pockets in the esophagus. Esophageal diverticula may not present with any symptoms, but a patient with esophageal diverticula may complain of dysphagia or regurgitation. Esophageal webs are mucosal folds; they can be single or multiple. Esophageal webs are typically observed in Plummer-Vinson syndrome and are located in the upper esophagus; however, they can be found anywhere in the esophagus. They often present with dysphagia. An esophageal laceration is also referred to as a Mallory-Weiss tear. An esophageal laceration would present with hematemesis. Esophageal lacerations are usually associated with vomiting. Alcoholics are more prone to esophageal lacerations.

A 42-year-old woman presents with a tremor that has become more bothersome lately. She thinks it originally began as a mild tremor twelve years ago. It affects her hands more than her head. It usually occurs when she moves her extremities. She drinks 1 or 2 alcoholic beverages every day because alcohol seems to lessen the tremor. Her mother suffered from a similar disorder. Her neurological exam is essentially normal other than a tremor of the hands and head. What movement disorder does this patient have? Answer Choices 1 Alcoholic withdrawal tremor 2 Cerebellar tremor 3 Essential tremor 4 Parkinsonian tremor 5 Psychogenic tremor

Essential tremor The patient has essential tremor, which is the most common movement disorder. This postural tremor may have its onset anywhere between the second and the sixth decades of life. Its prevalence increases with age. It is slowly progressive over a period of years. It may be familial with autosomal dominant penetrance. The frequency of the tremor is typically 5 Hertz. In addition to the hands, it also affects the head, voice, tongue, and legs. It may be alleviated by alcohol. Alcoholic withdrawal tremor affects only the hands, unlike the multiple sites of involvement of essential tremor. Cerebellar tremor is an intention tremor or goal-directed. It increases in severity as the extremity approaches a target. Parkinsonian tremor occurs at rest. Psychogenic tremor includes an abrupt onset, a static course, spontaneous remission, and unclassified tremors. They often increase in frequency and amplitude with attention, and they decrease with distraction.

A 52-year-old woman presents with a 1-year history of bilateral shaking in her hands. The shaking tends to worsen when she is using her hands, and her symptoms improve when she drinks a glass of wine on the weekends. Her 82-year-old mother also has a similar shaking in her hands which developed when she was in her 50s. Question What is the most likely diagnosis? Answer Choices 1 Alcohol withdrawal 2 Parkinson's disease 3 Essential tremor 4 Huntington's disease 5 Multiple sclerosis

Essential tremor (or benign tremor) typically involves the hands or head, but it spares the legs. The cause is unknown, but it may be inherited. Patients note that the tremor develops when moving the hands. The symptoms may worsen in times of stress, and they may be alleviated by alcohol intake. Alcohol withdrawal may be associated with a tremor, but is unlikely given the duration of the tremor. The tremor of Parkinson's disease is typically a resting tremor; this patient is not experiencing other symptoms of Parkinson's disease (e.g., rigidity, bradykinesia, and postural instability). Huntington's disease may be associated with abnormal movements and is inherited in a familial matter; however, Huntington's disease is typically fatal within 20 years of onset. This patient notes that her mother has been suffering from her tremor for close to 30 years. Multiple sclerosis is not typically associated with tremor.

An immigrant worker presents with a chronic cough, blood-stained sputum, and night sweats. Physical examination reveals a wasted middle-aged man with bronchial breathings on the right upper lobe. His sputum examination under microscope shows acid-fast staining rods, and his PPD is 15 mm. Based on clinical findings, sputum examination, and PPD result, pulmonary tuberculosis is diagnosed. Treatment is started after sending the sputum for culture. The patient comes back 4 weeks later for a check up, and he mentions not being able to see clearly and not being able to distinguish the color blue from green. What drug is most likely causing this side effect? Answer Choices 1 Isoniazid 2 Pyrazinamide 3 Ethambutol 4 Streptomycin 5 Rifampicin

Ethambutol The correct response is ethambutol. Important side effects of TB drugs are: Ethambutol: optic neuritis and difficulty distinguishing blue from green Isoniazid: hepatitis, and peripheral neuropathy (pyridoxine is the treatment for peripheral neuropathy) Rifampicin: hepatotoxic, hypersensitivity reactions, and red coloring of the urine and other secretions (saliva, tears, stool) Pyrazinamide: hepatotoxic (rare), and hyperuricemia Streptomycin: ototoxic, and nephrotoxic.

A 41-year-old man presents for a physical. He has not had a routine physical in a couple of years. Upon taking a review of systems, you become suspicious that the patient is not present for routine health maintenance alone. The patient admits to various abnormal symptoms that result in the patient receiving an order slip for a 24-hour urine study looking for vanillylmandelic acid (VMA), catecholamines, and metanephrines. Question What triad of symptoms indicated a need for this particular 24-hour urine study? Answer Choices 1 Dry skin, fatigue, and shortness of breath 2 Fatigue, flushing, and constipation 3 Headache, dry skin, and nausea 4 Flushing, constipation, and nausea 5 Headache, palpitations, and diaphoresis

Explanation 'Headache, palpitations, and diaphoresis' is the correct answer. These 3 symptoms together make up the "classic triad" in patients with a pheochromocytoma. A pheochromocytoma is a catecholamine-producing tumor that is most commonly found in the adrenal gland superior to the kidney. It is commonly diagnosed due to hypertension, but it causes less than 1% of the cases of hypertension. The symptom triad listed above has been found to have a specificity of almost 94% and a sensitivity of almost 91% for patients with pheochromocytomas. Patients may also experience other symptoms, including constipation, weight loss, flushing, and dizziness. Since the tumor produces catecholamines, excess production is usually measured by determining the levels of excreted metabolites during a 24-hour period by a 24-hour urine. Abdominal MRI can then be used to confirm the presence of the adrenal tumor. 'Dry skin, fatigue, and shortness of breath' is not the correct answer. Fatigue is not a common symptom in patients with a pheochromocytoma, but it could certainly occur secondary to the pain, anxiety, or shortness of breath. Dry skin is actually the opposite of the common symptom, diaphoresis, which tends to occur in patients with a pheochromocytoma. Shortness of breath is a symptom that patients with pheochromocytoma can experience, but it is not part of the classic triad of symptoms. 'Fatigue, flushing, and constipation' is not the correct answer. Fatigue is not a common symptom in patients with a pheochromocytoma, but it could certainly occur secondary to the pain, anxiety, or shortness of breath. Flushing and constipation can both occur in patients with a pheochromocytoma, but these symptoms are not part of the classic triad, nor do they have the high sensitivity and specificity of the classic triad of palpitations, diaphoresis, and headache. 'Headache, dry skin, and nausea' is not the correct answer. Headache is 1 of the symptoms associated with the classic triad of symptoms that patients with a pheochromocytoma can experience, but dry skin is certainly not a symptom of a pheochromocytoma. The opposite is true, actually, as patients tend to experience diaphoresis. Nausea can occur, but it is not common and is not a part of the classic triad of symptoms. 'Flushing, constipation, and nausea' is not the correct answer. All 3 of these symptoms can occur in a patient with a pheochromocytoma, but none of them are a part of the classic triad of diaphoresis, headache, and palpitations that has a high specificity and sensitivity for pheochromocytoma.

A 68-year-old man presents with scrotal swelling; he has had the swelling for the past few months. It is not bothering him, but his wife wants him examined. His history is not significant for any other GU history or symptoms. A genitourinary examination reveals a right hydrocele. Question What would the examination findings describe? Answer Choices 1 Fluid collection in the scrotum that transilluminates 2 Solid lesion in the scrotum that does not transilluminate 3 Fluid collection in the scrotum that does not transilluminate 4 Dilated veins in the scrotum that transilluminate 5 Cystic mass in the scrotum that transilluminates

Explanation A hydrocele is a collection of fluid between layers of the tunica vaginalis that surrounds the testicle. The typical presentation of a hydrocele is a painless scrotal swelling that can worsen throughout the day. When a flashlight or penlight is held behind the scrotum in a dark room, the light will transmit through the fluid; this is known as transillumination. A hydrocele is not usually bothersome, and patients will usually only complain if it is large enough to get in the way. Hydroceles can be treated surgically, but this is usually reserved for the most bothersome cases. A solid lesion in the scrotum that does not transilluminate describes how a testicular mass may present on physical examination. A hydrocele is not solid and does transilluminate. While a hydrocele is a fluid collection in the scrotum, it usually does transilluminate. There are no genitourinary findings that would involve a fluid collection in the scrotum that would not transilluminate. Dilated veins in the scrotum that transilluminate typically represent a varicocele rather than a hydrocele. The dilated veins are often described as feeling like a 'sack of worms' on physical examination. A cystic mass in the scrotum that transilluminates typically represents a spermatocele. A spermatocele is usually freely movable and located both superior and posterior to the testicle.

A 45-year-old woman presents with a 3-day history of persistent nipple discharge. She is essentially healthy with an insignificant past medical history. She quit her job to stay home with her young children in the past year; she has noted a weight gain of 15 lbs, which she attributes to being at home more, leading to increased snacking as well as less activity. She also says her periods have spaced out, occurring anytime from 28 - 53 days. They still last about 2 - 3 days, as they previously had. She denies hot flashes and the possibility of pregnancy; her husband had a vasectomy. Her review of systems is otherwise negative. She takes Vitamin E every day, but she takes no other medications or herbs; she denies the use of illicit drugs. On physical exam, you note a healthy-looking woman. Her breast exam is negative for lumps, dimpling, and nipple retraction. You are able to express some clear fluid from the left nipple. It seems serous. Her axillary exam is negative for any enlarged lymph nodes; the rest of her exam is normal. Question What is the next step in the management of this patient? Answer Choices 1 Check an FSH 2 Do microscopy on the fluid 3 Check a prolactin level 4 Order a mammogram 5 Check a TSH

Explanation A mammogram should be ordered. The patient is a 45-year-old woman with spontaneous serous discharge. Nipple discharge is a common breast complaint and is usually associated with benign etiology. Because most nipple discharge is a result of benign processes, less-invasive, non-surgical diagnostic modalities have been explored to alleviate the need for surgical intervention. Most of the patients with nipple discharge have benign disease, such as intraductal papilloma, papillomatosis, duct ectasia, or fibrocystic condition. Studies have shown that approximately 11% of patients will have an underlying carcinoma. To be significant, nipple discharge should be spontaneous, persistent, and non-lactational. Persistent clear or bloody nipple discharge merits, at the very least, diagnostic breast imaging. All nipple discharges associated with a breast lump must be surgically evaluated. The incidence of associated cancer is much higher when the discharge is serosanguineous or sanguineous. Mammography has been utilized for evaluation of obvious abnormality to guide surgical intervention. The sensitivity of mammography for detecting malignant ductal pathology is 57% and the positive predictive value is 16.7%. It should be the initial test in patients with clinically significant nipple discharge and in women above the age of 40 years. Cytological analysis of the discharge fluid has traditionally shown high false-negative rates, and it is not revealing in localizing the lesion. Hemoccult testing has been shown to be useful for determining the presence of blood, but its use in predicting histology is questionable. For the patient in this case, all the tests noted (A through E) could be indicated. An FSH can be used to check for menopausal status, and a prolactin level could be done to check for abnormal levels. A TSH to check for hypothyroidism could also be done; decreased thyroid hormones are associated with galactorrhea. Hypothyroidism would also explain the menstrual irregularity. Mammography should be conducted initially; later, hormonal assay and a cytological examination can be carried out. In summary, given that this patient presented with spontaneous persistent discharge, the most appropriate initial approach would be to perform a mammography.

A 27-year-old woman presents with a 2-month history of worsening irritability, fatigue, and weight loss. Her past medical history includes myasthenia gravis, but she is otherwise healthy. Question During her history taking, what positive review of systems symptoms would put hyperthyroidism on your differential list? Answer Choices 1 Excessive sweating, heat intolerance, heart palpitations 2 Hair loss, decreased energy, dry skin 3 Cramping, numbness around mouth, tingling in distal extremities 4 Bone pain, flank pain, anxiety 5 Dizziness, salt craving, chronic diarrhea

Explanation A patient with hyperthyroidism could experience excessive sweating, heat intolerance, heart palpitations, and a myriad of other possible symptoms. Excessive secretion of the circulating free thyroid hormones affects multiple organ systems, resulting in this myriad of potential symptoms. The facts that the patient is female, in her third decade of life, and has another autoimmune disorder are also risk factors for hyperthyroidism. Hair loss, decreased energy, and dry skin are symptoms of hypothyroidism rather than hyperthyroidism. Hypothyroidism is typically an autoimmune disease or a result of prior radiation therapy or thyroid surgery. While some of the symptoms can overlap with those of hyperthyroidism, that is not typically the case with those listed. In fact, a patient with hyperthyroidism is likely to have hyperactivity and irritability rather than decreased energy. They are also more likely to have excessive sweating as opposed to dry skin. Cramping, numbness around mouth, and tingling in distal extremities are symptoms that could occur in a patient with hypoparathyroidism. This disorder is a result of a deficiency of parathyroid hormone from congenital absence, injury, surgery, or other diseases. Most of the symptoms associated with hypoparathyroidism are a result of hypocalcemia. Bone pain, flank pain, and anxiety could occur in a patient who has hyperparathyroidism. This disorder is characterized as a dysfunction in the body's regulatory system for parathyroid hormone. While up to 75% of patients can be asymptomatic, the hypercalcemia associated with hyperparathyroidism can result in the classic complaints of painful bones, renal stones (causing flank pain), abdominal groans, and psychic moans. Dizziness, salt craving, and chronic diarrhea are symptoms that could occur in a patient with Addison's disease. Adrenal gland insufficiency can occur as a result of autoimmune inflammation and multiple other causes, and it is more common in women. Other symptoms that can commonly occur are weakness, fatigue, anorexia, nausea, vomiting, depression, and myalgias.

A 39-year-old male was out on a ranch with his friends for the weekend. He indulged in horseback riding daily, stretched out over several hours in the afternoon. On his return home, he experienced high fever with chills and malaise, myalgias, dysuria, perineal pain, and cloudy urine. Examination in the ER revealed a temperature of 101.5°F, pulse 110/min, BP 120/80 mmHg, and respiratory rate of 16/min. There was no pallor, jaundice, or lymphadenopathy. Lungs were clear and no murmurs appreciated. Abdominal exam showed no tenderness, masses, ascites, or hepatosplenomegaly. Bowel sounds were active, and rectal exam showed exquisite tenderness. Significant labs included WBC 13,400/uL and urinalysis with 15 WBC and 4 RBC. Question What would be a provisional diagnosis with blood and urine cultures pending? Answer Choices 1 Acute pyelonephritis 2 Acute urethritis 3 Rectal abscess 4 Anal fissure 5 Acute prostatitis

Explanation Acute prostatitis is defined as an inflammation of the prostate gland that develops suddenly and is common in men, likely due to reflux of infected urine into intraprostatic ducts. This can happen after instrumentation, catheterization, or trauma, like horseback riding, biking, etc., and worsened by dehydration, as in this patient. The National Institutes of Health classification of inflammatory conditions of the prostate is as follows: I Acute prostatitis II Chronic bacterial prostatitis III A Chronic prostatitis/pelvic pain syndrome, inflammatory III B Chronic prostatitis/pelvic pain syndrome, noninflammatory IV Asymptomatic inflammatory prostatitis Gram negative organisms are the main culprit, including E.coli, proteus, klebsiella, enterobacter, and pseudomonas. Symptoms of dysuria, fever, perineal pain, and tender prostate are typical. Treatment is with trimethoprim-sulfamethoxazole or quinolones for 4 weeks. In sicker patients, hospitalization may be needed, in which case IV antibiotics with aminoglycoside and ampicillin should be given until the patient is afebrile for 24-48 hours, then oral antibiotics continued for total of 4-6 weeks to avoid complications such as abscess formation or chronic prostatitis. Acute pyelonephritis presents with fever, flank pain, tender renal angle, and normal rectal exam. Treatment includes oral fluoroquinolone or trimethoprim-sulfamethoxazole for mild to moderate disease and IV ceftriaxone or a fluoroquinolone for hospitalized patients, to be substituted with oral antibiotics after improvement in symptoms. Total duration of antibiotics should be 10-14 days. Acute urethritis is associated with dysuria and urethral discharge with pruritus at urethral meatus. Fever, chills, frequency, urgency, and hematuria are uncommon. It may be gonococcal, which is the most common cause of urethritis in men or nongonococcal urethritis (NGU). Although most cases of NGU are due to chlamydia trachomatis, other etiologies include T. vaginalis, Mycoplasma genitalium, and Ureaplasma urealyticum. Gram stain and culture or the urethral discharge should be done. Treatment is with ceftriaxone 125mg IM, cefixime 400mg PO, ciprofloxacin 500mg PO, or ofloxacin 400mg PO, all in a single dose in gonococcal urethritis and azithromycin 1gm PO or doxycycline 100mg BID for 7 days or ofloxacin 400mg PO BID for 7 days for NGU. Rectal abscess is a distant possibility in this patient. It presents with constant pain in the rectal area and perhaps fever and malaise but no dysuria or cloudy urine. Rectal exam will be tender and reveal a fluctuant mass. UA, however, will not be abnormal. Treatment is with incision, drainage, and perhaps antibiotics for anaerobic coverage. Anal fissure presents with excruciating pain with the passage of bowel movements and is associated with constipation. The passage of stool may be accompanied by bright rectal bleeding usually limited to a small amount on the toilet paper but sometimes more profuse bleeding. Treatment aims at relaxing the sphincter, keeping bowel movements soft and smooth, and pain control.

A 22-year-old woman is started on carbimazole for the treatment of her hyperthyroidism. She has been diagnosed with Graves' disease and has been symptomatic for the last 6 months. She is otherwise healthy and does not take any other medications. You receive a call 2 weeks later from the patient; she tells you that she has had a mild sore throat for the past 2 days and has been feeling like she is coming down with the flu. She states that the symptoms of her hyperthyroidism are a little bit better. Question What should you do? Answer Choices 1 Increase dose of carbimazole 2 Prescribe a small dose of thyroxine 3 Tell her to present ASAP for total and differential blood count 4 Prescribe antibiotics 5 Prescribed saltwater gargles and acetaminophen

Explanation Asking her to come to clinic and have her total and differential blood count done is the correct answer. One uncommon, but well recognized, side effect of antithyroid medications is the potential to cause rapid agranulocytosis, characterized by a very low circulating neutrophil count. Patients usually develop agranulocytosis during the first 3 months of therapy. The drug most commonly associated with it is propylthiouracil. Symptoms most commonly encountered include a fever, sore throat, myalgia, and rigors. Any patient with these symptoms, especially within the first few weeks of initiating therapy, must have their white count checked. The incidence of such side effects is well below half a percent, thus making it economically unviable to routinely monitor counts. The suggested method of treatment once agranulocytosis is proven is the administration of broad spectrum antibiotics with intravenous granulocyte colony stimulating factor. This results in rapid resolution of neutropenia. Increasing her dose of carbimazole is incorrect. That would only worsen the problem. Self administration of thyroxine is incorrect. The patient has hyperthyroidism to begin with and is not exhibiting any signs of hypothyroidism. Prescribing antibiotics is incorrect. While it may be done, the neutrophil count must first be determined in order to rule out neutropenia. If the white count is normal, a mild pharyngitis may not need antibiotics in any case. Prescribing salt water gargles and acetaminophen is incorrect. The neutrophil count must first be determined.

What component of a patient's health history would be considered protective against breast cancer? Answer Choices 1 Onset of menses at age 11 years 2 Breast fed all 3 of her children 3 Recovering alcoholic for the last 6 months 4 Mother had menopause at the 4th decade of life

Explanation Breast-feeding is protective for breast cancer. Early menarche and excessive alcohol use are factors that increase susceptibility to breast cancer. Mother's age of menopause does not affect breast cancer risk one way or another.

A 33-year-old woman presents to discuss management of her premenstrual dysphoric disorder. The patient experiences significant depression and irritability for the 2nd half of her menstrual cycle nearly every month, but she has refused a trial of a selective serotonin reuptake inhibitor (SSRI) because she doesn't want to be on an antidepressant. She has previously been on oral contraceptives, but she stopped taking them due to worsening of her migraine headaches. She asks you about herbal remedies. What is true regarding herbal agents that contribute towards improvement in premenstrual dysphoric disorder? Answer Choices 1 Chaste tree berry decreases secretion of luteinizing hormone and has been proven to reduce PMS symptoms in many women 2 Gingko biloba has been shown to worsen breast pain associated with PMS 3 Chaste tree berry increases secretion of luteinizing hormone and has been proven to improve PMS symptoms 4 Natural progesterone has been proven to improve PMS symptoms in numerous well-controlled trials 5 St. John's Wort does not significantly improve the dysthymic symptoms of PMS

Explanation Chaste tree berry increases secretion of luteinizing hormone and has been proven to improve PMS symptoms. Premenstrual syndrome (PMS) is defined as symptoms of mood changes, such as depression, nervousness, increased irritability and lability, and anxiety, as well as physical symptoms, such as breast pain, headaches, and edema (weight gain). These symptoms usually begin 7 - 10 days before the onset of menses and abate quickly as soon as menses begin; however, some individuals with more severe premenstrual dysphoric disorder symptoms can experience symptoms beginning shortly after ovulation, and thus approximately half of each month is affected by these complaints. Nearly 80% of women experience some emotional or physical symptoms prior to their menses, although only about 1/3 of these women have significant problems due to these symptoms. Only about 2.5 - 5.0% describe their premenstrual symptoms as having a negative effect on their lives to the point where occupational or interpersonal relationships are affected. The type and intensity of symptoms are different between different patients, and they can vary from cycle to cycle in a given woman. PMS is most likely due to fluctuations in levels of progesterone and estrogen. Both estrogen and progesterone can cause fluid retention, which seems to explain some of the physical complaints associated with PMS. Some studies have suggested that patients with PMS symptoms metabolize progesterone differently, causing less production of allopregnanolone, a GABAergic substance that reduces anxiety. Selective serotonin reuptake inhibitors (SSRIs) have been proven to improve the symptoms of premenstrual dysphoric disorder and are the most effective agents in the treatment of the symptoms of PMS. Other useful agents include oral contraceptives to reduce the cyclic variations in estrogen and progesterone. Supplementing the diet with pyridoxine has also been shown to improve some of the symptoms of PMS. Diuretics had been used in the past to help with symptoms of water retention, but their use is losing favor in recent years. For patients who would prefer a naturopathic approach to the management of PMS, some "natural" agents have been shown to be effective in the treatment of PMS. Chaste tree berry is probably one of the most effective botanical agents for the treatment of PMS. It is thought that chaste tree berry increases secretion of luteinizing hormones and may affect progesterone. Several studies have found chaste tree berry to be more effective than placebo in treating both the emotional (mood changes, irritability, and anger) and some of the physical symptoms (headache and breast tenderness, although not bloating) of PMS. However, these improvements have not been replicated in all studies of this agent, with other studies showing no improvement in symptoms when compared to placebo. St. John's Wort has been shown to be useful in treating the emotional, dysphoric symptoms of PMS. Patients in one small trial showed improvement in depression, confusion, anxiety, nervousness, and insomnia associated with PMS when treated with St. John's Wort. Ginkgo biloba has been shown to improve the breast pain and tenderness associated with PMS, but has not been useful in treating other premenstrual complaints. Natural progesterone has been touted as an effective treatment for PMS, although there is not much data to support this claim; in fact, several studies have shown no improvement in the symptoms of PMS when compared to placebo, especially for the vaginal and rectal preparations of progesterone. An exception to this is the oral micronized formulation of progesterone, which has been shown to improve anxiety, depression, fluid retention, and hot flushes in PMS. Additionally, many naturopathic practitioners advocate the use of natural progesterone cream rubbed into the skin, although this is not supported by scientific study.

A 35-year-old woman presents with a 24-hour history of fever, right flank pain, vomiting, dysuria, and hematuria. A urinalysis reveals large amounts of red and white blood cells, as well as leukocyte esterase, and a subsequent urine culture was positive for >100,000 Escherichia coli. The patient denies having a history of renal calculi, and a bedside renal ultrasound does not show any stones or hydronephrosis bilaterally. She is diagnosed with a right-sided pyelonephritis and admitted for pain control, hydration, and IV antibiotic therapy. Her symptoms, including her fever, abate quickly, and she is discharged after 48 hours of being afebrile. Question Which prescription would she most likely receive upon discharge? Answer Choices 1 Motrin 800 mg PO q 8 hours prn pain 2 Nitrofurantoin 100 mg PO BID x 7 days 3 Ciprofloxacin 500 mg PO BID x 14 days 4 Levofloxacin 500 mg PO daily x 7 days 5 Amoxicillin 500 mg PO BID x 14 days

Explanation Ciprofloxacin 500 mg PO BID x 14 days is the correct answer. Patients with pyelonephritis who are sick enough to be treated as an inpatient receive IV antibiotics until they have been afebrile for 24 - 48 hours. They also need to be able to tolerate oral hydration and oral medications before being discharged. Upon discharge, they will be given a prescription for antibiotics that will complete at least 2 weeks of antibiotic treatment. Ciprofloxacin has good coverage for E. coli urinary tract infections, and given at 500 mg PO BID x 14 days would be an appropriate choice as long as sensitivities from her culture showed Ciprofloxacin to have sensitivity. Motrin 800 mg PO q 8 hours prn pain is not the correct answer. While motrin is a good choice for patients who may still have some discomfort related to the pyelonephritis, it is not the most likely prescription for this patient to receive. She had become asymptomatic prior to discharge, so there would not be any reason to prescribe anything to help with pain or discomfort at that time. In addition, she is more likely to receive treatment for her infection than for pain. Nitrofurantoin 100 mg PO BID x 7 days is not the correct answer for several reasons. While nitrofurantoin is often an appropriate antibiotic for E. coli urinary tract infections, it does not achieve tissue levels reliable enough for pyelonephritis treatment. In addition, only 7 days of treatment does not add up to at least 2 weeks total of antibiotics. Levofloxacin 500 mg PO daily x 7 days is not the correct answer. While levofloxacin is an appropriate antibiotic to treat E. coli pyelonephritis, 7 days of treatment is not long enough in this situation. Amoxicillin 500 mg PO BID x 14 days is not the correct answer. While 14 days of antibiotics is a good length of time, amoxicillin does not have good coverage for E. coli, so it would not be a good choice to treat an E. coli related pyelonephritis.

A 60-year-old man you have been treating for congestive heart failure symptoms has been diagnosed with diabetes mellitus. The endocrinologist who made the diagnosis decided to treat the patient with glipizide 5 mg daily since his blood glucose readings are 280 mg/dl. You manage the patient's congestive heart failure with enalapril 10 mg daily, and he takes a furosemide 20 mg daily if he needs it. What kind of effect on the patient's hyperglycemia may be expected in these settings? Answer Choices 1 Decrease of hypoglycemic effect 2 Increase of hypoglycemic effect 3 No hypoglycemic effect 4 Mild hyperglycemic effect 5 Euglycemia interchangeable with hyperglycemia

Explanation Co-administration of ACE inhibitors (captopril, enalapril) and oral sulfonylurea hypoglycemics results in an increase of the hypoglycemic effect, for which more careful monitoring of blood glucose is advised to avoid pronounced hypoglycemia. Co-administration of thiazide, but not loop diuretics, results in a decrease of hypoglycemic effect. There are no documented interactions of furosemide and sulfonylureas.

A 53-year-old man was recently diagnosed with a pituitary adenoma. He has been experiencing excessive thirst and large volume polyuria, as well as headaches and an increase in nocturia from once nightly to 3 to 4 times nightly. Question What medication would be the first-line treatment option given to him? Answer Choices 1 Hydrochlorothiazide 2 Indomethacin 3 Amiloride 4 Desmopressin acetate 5 Glucophage

Explanation Desmopressin acetate (DDAVP) is the correct answer, as this medication is used as the first-line treatment in patients with central or primary diabetes insipidus (DI). Arginine vasopressin (AVP), or antidiuretic hormone, is produced by the pituitary gland and works on the renal tubules within the kidney to concentrate the urine and reduce water loss. DI occurs when there is a deficiency in AVP, and these patients produce large amounts of dilute urine. As a result, patients can experience excessive thirst, polyuria, nocturia, headache, dehydration, and disturbances in vision. Central or primary DI can be diagnosed with 24-hour urine collection or vasopressin challenge test. Once the diagnosis is established, the first line treatment is usually desmopressin acetate, which is a synthetic form of AVP. It has a longer antidiuretic action than AVP and does not constrict smooth muscle. DDAVP is available in oral, nasal, and injectable preparations. While being treated, it is important to control fluid balance in order to prevent dehydration, monitor the patient's weight daily, and monitor electrolytes. Hydrochlorothiazide is not the correct answer, as this medication can be used in cases of nephrogenic DI rather than central DI and as a treatment for hypertension. Nephrogenic DI occurs in the presence of normal AVP production by the pituitary gland with insensitivity to AVP in the kidney. There is a disorder in the renal tubular function that results in the inability to respond to the presence of AVP. In these cases, hydrochlorothiazide can be given in a 25 mg dose either once or twice daily. Indomethacin is not the correct answer. Indomethacin is a non-steroidal anti-inflammatory that is typically used in the treatment of osteoarthritis, rheumatoid arthritis, or ankylosing spondylitis. However, it can also be used in the treatment of nephrogenic DI because indomethacin inhibits prostaglandins, which reduces urine output by decreasing the renal blood flow. This would not be effective in the treatment of central DI. Amiloride is not the correct answer. Amiloride is often added to HCTZ in order to increase the diuretic effects. However, this is a treatment course that is used in nephrogenic DI and not central DI. Glucophage is not the correct answer, as this medication is a biguanide used in the treatment of diabetes mellitus, not diabetes insipidus.

An 8-year-old boy is evaluated for persistent bed wetting. He has never been continent, averaging 2 - 3 episodes of bedwetting per week. His urological evaluation revealed a normal bladder and urethra, with no neurological problems. Lately, his problem has been a source of much embarrassment; he is unable to attend camp or sleepovers due to fear of wetting his bed. He has tried multiple interventions, including lifestyle changes, alarm systems, and reward systems. His physical exam shows no abnormalities. His parents are keen on a rapid resolution to his problems, and they insist treatment be initiated. Question What is the best therapy? Answer Choices 1 Continued lifestyle changes 2 Desmopressin 3 Imipramine 4 Continue alarm system 5 Corticosteroid

Explanation Desmopressin is the correct answer. Desmopressin is an analogue of anti-diuretic hormone, which reduces the production of urine at night. It is effective and has a rapid onset of action. The intra nasal form has been removed from the market in favor of the oral tablet. Desmopressin is indicated in enuresis not responding to lifestyle changes or alarms, when patients/parents want rapid improvement, or in children in whom alarm systems are ineffective. Continued lifestyle changes is incorrect. Lifestyle changes include fluid restriction after 5 pm, restriction of caffeinated drinks, and making the child urinate before bed. In this child, enuresis is pathological; it is persistent after the age of 5, and medical intervention is indicated due to the failure of lifestyle measures. Imipramine is incorrect. Imipramine is an effective drug, but is avoided in children due to its numerous side effects, such as dry mouth, constipation, and drowsiness. Alarm system is incorrect. Alarm systems are effective; however, they have already been tried in this child. Therefore, a different form of therapy is indicated. Coricosteroids is incorrect. They are not used in the management of enuresis.

A 44-year-old obese woman presents with increased nighttime urination. She has never had issues with having an increased urge to urinate nocturnally before, and it is extremely bothersome to her. She states that she has been waking up at least 3 times a night despite lifestyle modifications designed to help reduce this number. The patient admits to increased fatigue, worsening blurry vision, and 2 vaginal yeast infections in a span of 3 months. Question What is the most likely diagnosis? Answer Choices 1 Hyperthyroidism 2 Hypothyroidism 3 Diabetes type I 4 Diabetes type II 5 Cushing's syndrome

Explanation Diabetes type II is the correct answer for the clinical picture presented. This pathologic state consists of beta cell loss, but it also has other key characteristics that lead to its development; patients develop increased tissue insensitivity, also termed insulin resistance. This insensitivity leads to hyperactivity of the beta cells, producing excessive amounts of insulin, and over time the developed hyperplasia of the beta cells leads to impaired or even impeded function. Signs and symptoms that result from diabetes type II include but are not limited to: polyuria, polydipsia, chronic skin infections, or other indications of insulin resistance. Patients may be asymptomatic at the time of diagnosis. Diabetes should be high on one's differential diagnosis when a patient presents with frequent/chronic vulvovaginitis, which is part of the patient's complaint described above. Diabetes type I is the most common type of diabetes mellitus in patients under the age of 20. Presenting symptoms can include polydipsia, polyuria, significant weight loss, and even a dangerous state of hypergylcemia that is referred to as diabetic ketoacidosis. Over 95% of type I diabetics develop this through autoimmune activity on the pancreatic islet cells; the remaining ~5% is idiopathic. Hypothyroidism may from failure of the thyroid gland itself in secreting efficient levels of the main thyroid hormone free thyroxine, or FT4. Hypothyroidism may also occur because of deficiency of the secretion of thyroid stimulating hormone (TSH), which is secreted by the pituitary gland. Signs and symptoms highly suggestive of this disease state are weight gain, fatigue, lethargy, depression, weakness, cold intolerance, muscle cramps, and constipation. Simply put, hyperthyroidism is a patient suffering from excessive serum levels of T4 and T3, while TSH levels are abnormally low. Signs and symptoms of the disease state include excessive sweating, weight loss, anxiety, palpitation, heat intolerance, palpitations, fatigue, and weakness. Cushing's syndrome, which is also termed hypercortisolism, refers to the manifestations of excessive corticosteroids on the patient. Many of these cases are due to excessive ACTH secretion by a benign pituitary adenoma; however, other causes of excessive corticosteroids may lead to this condition, such as other non-pituitary secreting ACTH neoplasms as well as excessive secretion of t cortisol from the adrenal glands. In up to 30% of cases, the main cause may not be found. There are a multitude of symptoms and signs seen with Cushing's syndrome; they can include a moon facies, buffalo hump, abdominal purple striae, central obesity, menstrual irregularities, and even impaired wound healing.

A 53-year-old woman is seen by her gynecologist. She had 3 children, and she is tubectomized. She attained menopause at 48. Over the last few weeks, she has noted some vaginal bleeding; it occurs unpredictably. She has had regular pap smears that have always been normal. The last one was 11 months prior to presentation. Speculum examination and bi-manual palpation of the genitalia reveal no abnormalities. Question What is the next best step in management of the patient? Answer Choices 1 Repeat pap smear 2 Chlamydia serology 3 Laparoscopy 4 Endometrial biopsy 5 Hormonal profile

Explanation Endometrial biopsy is the correct answer. Vaginal bleeding in a post-menopausal woman mandates endometrial sampling, with a view to ruling out endometrial cancer. Histopathological evaluation of material is essential. The most common causes of post-menopausal bleeding are vaginal atrophy (60 - 80%), exogenous estrogens (15 - 25%), endometrial cancer (10%), endometrial hypertrophy (5 - 10%), as well as cervical and endometrial polyps (2 - 12%). Repeat pap smear is incorrect. The patient had one done less than a year ago, which was normal. In light of her previously normal pap smears, the next one is not due for another 3 - 5 years. Pap smears may show the occasional dysplastic cell of endometrial origin, but it is by no means an accurate means of diagnosing endometrial pathology. Chlamydia serology is incorrect. It is done for the diagnosis of pelvic discharge in sexually active women; it is not used in cases of post-menopausal bleeding Laparoscopy is incorrect. It is not useful in the workup of post menopausal bleeding. Hormonal profile is incorrect. While exogenous estrogen is a possible cause of post menopausal bleeding, it is a diagnosis made from taking a careful history.

A 28-year-old pregnant woman at 18-weeks gestation presents because she has been exposed to fifth disease. The patient is currently asymptomatic. What can you tell her about human parvovirus B19 and pregnancy? Answer Choices 1 Less than 25% of adults are immune to this virus, and she should be treated with immunoglobulin 2 Parvovirus B19 has not been associated with any fetal effects 3 If she develops an infection, she will need to be followed with serial fetal ultrasounds 4 The virus is spread by fecal-oral transmission, and she should wash her hands very carefully 5 The risk of fetal loss in an infected mother is between 45-55%

Explanation Fifth disease is caused by human parvovirus B19, which is a DNA virus. Fifth disease, also called erythema infectiosum, is usually a mild exanthem of childhood, but infection of a pregnant woman can have severe fetal complications Fetal (transplacental) infection with parvovirus B19 can result in a variety of fetal complications, including fetal loss, especially if the infection is between gestational weeks 10 and 20. A common complication includes fetal hydrops; it is caused by damage to fetal hematopoietic tissue, and it causes severe anemia and a resultant congestive heart failure. The virus can also cause a fetal viral myocarditis, which further worsens cardiac function and fetal hydrops. More rarely, first trimester infections with parvovirus B19 can cause teratogenic effects, including multiorgan abnormalities. Between 30-60% of adults are immune to parvovirus B19, as evidenced by the presence of IgG to B19 in their serum; therefore, most pregnant patients are probably immune to this virus. However, if a pregnant patient develops a rash or aplastic crisis that may be consistent with fifth disease, then igG and IgM serologies for parvovirus should be drawn to evaluate for acute infection. If parvovirus B19 infection is diagnosed in a pregnant patient, then her physician may choose to follow her with serial fetal ultrasounds to evaluate for the development of fetal hydrops. In some cases, fetal umbilical cordocentesis has been used to detect fetal infection. In children, fifth disease is characterized by a classic "slapped-cheek" facial erythema; it is associated with fever and often GI or other systemic symptoms. Adult patients will demonstrate a rash. The rash may be reticular, morbilliform, or even purpuric. Adult patients often have fever, lymphadenopathy, and/or arthritis. Parvovirus B19 is also associated with an acute transient aplastic crisis. Unfortunately, fifth disease is infectious for days before the onset of the rash; thus, many obstetrical patients may be exposed, particularly if they work closely with children. The virus is spread by aerosolized respiratory droplets, and it has an incubation period of 4 to 14 days.

A 60-year-old man presents with difficulty climbing stairs, dyspnea, and fatigue. He has gained 30 lbs over the past 1 year. On examination, he is found to have edema, pigmentation of the skin, palmar creases, and proximal muscle weakness. Chest X-ray shows an irregular mass in the right upper lobe. Lab values show an increase in evening cortisol levels and an increase in ACTH. Question What should be the next best step in diagnosis? Answer Choices 1 MRI of the brain 2 Adrenal venous sampling 3 Serum ACTH levels 4 High dose dexamethasone suppression test 5 ACTH stimulation test

Explanation High-dose dexamethasone suppression test is the correct answer. Lab values are indicative of raised cortisol and an increased ACTH level. The next step in diagnosis should be to differentiate between a pituitary and peripheral cause of high ACTH. A high-dose dexamethasone suppression test is used to differentiate between Cushing's syndrome caused by pituitary adenoma versus ectopic ACTH production by a lung mass. ACTH of pituitary origin is suppressed by high doses of dexamethasone by negative feedback; ACTH of peripheral origin is not. Adrenal venous sampling is incorrect; however, it is of great utility in patients with primary hyperaldosteronism. MRI of the brain is incorrect. An MRI is appropriate after an endocrine diagnosis is made. If done prior to this, an unacceptably high false positive rate ensues. Serum ACTH level is incorrect. It is already given in the question stem. ACTH stimulation test is incorrect. It is used in adrenal insufficiency, not hyperadrenalism.

A 14-year-old boy presents due to embarrassment after an incident in school: while undressing before the class in physical education, the other boys laughed at him because of his "underdevelopment". His personal and family history is non-contributing. Question If puberty has started, what would you expect to notice? Answer Choices 1 Pubertal growth spurt 2 Increase in size of testes 3 Increase in body mass 4 Increase in bone accretion 5 Elongation of penis

Explanation In boys, age of 13 and a half years is the upper limit of normal onset of puberty (to simplify, 14 years is often used as the upper limit). The initial sign is usually an increase in the size of the testes to more than 2.5 cm in their longest diameter, excluding the epididymis. Testicular size reaches adult size about 6 years after the onset of puberty. Increase in the speed of growth (pubertal growth spurt) is the initial sign of puberty in girls (although breast development is usually the first sign of puberty reported). In boys, a growth spurt usually follows the increase in size of testes. Changes in body composition (lean body mass, skeletal mass and body fat) usually follow changes in testicular size. They happen several years earlier in girls, who also have an earlier growth spurt and weight gain. Critical periods of bone accretion occur during infancy and puberty in both sexes. In puberty, peak of bone mineralization is registered after the peak height velocity. After testicles have enlarged and developed for about 1 year, the length of the penis increases; this is followed by an increase of the breadth of the shaft of the penis and the enlargement of the glans and corpora cavernosa.

A patient presents for routine analysis of cholesterol levels. The results show plasma cholesterol levels of 300-mg/100 ml. You prescribe the drug simvastatin (Zocor). She is reluctant to take drugs to treat her hypercholesterolemia. After further discussion, she agrees to take a vitamin to treat the elevated cholesterol. She also has questions concerning familial hypercholesterolemia. Question Familial hypercholesterolemia is due to what defect? Answer Choices 1 Cholesterol biosynthesis 2 Low density lipoprotein receptors 3 Cholesterol degradation 4 Lysosomal function 5 Cholesterol esterification

Explanation In familial hypercholesterolemia, which is a genetic disorder, the plasma levels of cholesterol and LDL are elevated. In this disease, cholesterol is deposited in many tissues because of the concentrations of cholesterol-LDL in the plasma. The molecular defect results in the absence or deficiency of functional LDL receptors. Therefore, the entry of LDL into the liver and other cells is impaired and all of the deleterious effects of the disease can be attributed to the elevated LDL-cholesterol levels. High levels of cholesterol have 3 major metabolic effects. Cholesterol inhibits the activity of the enzyme β-Hydroxy-β-methyl-glutaryl-CoA (HMG-CoA) reductase, which catalyzes the rate-limiting step of cholesterol biosynthesis. Cholesterol also inhibits the production of low-density lipoprotein (LDL) receptors by suppressing the transcription of the LDL receptor mRNA. The third point of regulation involves the activation of the enzyme acyl-CoA: cholesterol acyltransferase (ACAT), which esterifies cholesterol for storage. The LDL receptors are located in specialized regions of the membrane called coated pits. These coated pits contain a protein called clathrin. Clathrin participates in the endocytosis of LDL receptors, as well as other proteins such as transferrin and sialoglycoproteins. Clathrin forms a closed polyhedral lattice around the coated pit forming a coated vesicle. The coated vesicle then loses its clathrin and fuses with an endosome. In the endosome, the ligand is dissociated from the receptor, and sorting of the different components can occur.

A 26-year-old woman presents with an abnormal lump in her right breast. Physical examination demonstrates a well-circumscribed, palpable mass in the upper outer quadrant of the right breast. The patient denies a family history of breast carcinoma or any additional medical problems. Question What would be the best test to order for further evaluation of this palpable abnormality? Answer Choices 1 Bilateral mammography 2 Right breast MRI 3 Right galactogram 4 Stereotactic biopsy 5 Focused ultrasound

Explanation In women under 30 years of age with a palpable lump in their breast, the initial imaging modality should be an ultrasound. Ultrasound is an excellent modality in this scenario because it can define the borders of the mass and determine whether the lesion is solid or cystic. If the lesion is solid, then the characteristics of the mass can be better defined before either following the lesion or proceeding to biopsy. Bilateral mammography is not indicated in this patient (who is under the age of 30) at this point. If the patient were > 30 years of age with a palpable breast lump, then a unilateral mammogram and focused ultrasound would be the best tests to order for further evaluation. However, in patients < 30 years of age, ultrasound is a better choice for the initial test because most of these 'lesions' are fibroadenomas. Additionally, it is desirable to limit radiation exposure in patients younger than 30 years of age, thus supporting ultrasound as the best choice in this case. Breast MRI exams can be used to evaluate for lobular cancer; occult breast carcinoma; to define the extent of disease in a patient with a positive surgical margin; to evaluate for multifocal, multicentric, or bilateral cancers; and to evaluate for postoperative scar versus tumor recurrence. Breast MRI is not the initial test that should be ordered to evaluate a 26-year-old patient with a palpable breast mass. Galactograms are types of mammograms whereby a breast duct is accessed using a blunt needle to evaluate for unilateral or bilateral bloody nipple discharge, which is most often caused by an intra-ductal papilloma, but could be caused by intra-ductal papillary carcinoma or invasive ductal carcinoma. Galactograms would not be the initial test in the workup of a palpable mass in this young patient. A stereotactic biopsy is performed to obtain a tissue sample after a suspicious mass orcluster of microcalcifications is found on either a diagnostic or screening mammogram. A stereotactic biopsy would not be performed before an ultrasound is ordered in this patient. If a suspicious mass is seen in ultrasound, then an ultrasound-guided biopsy would be the next best course of action in order to exclude a breast carcinoma.

A 63-year-old diabetic woman is seen by her primary care physician. Despite having been an insulin dependent diabetic for over 20 years, her blood sugars remain poorly controlled. No amount of counselling seems to have made a difference to her attitude. At this visit, she complains of left-sided ear pain. She noticed it the previous night; she was unable to sleep with her head resting on the affected side. Since then, the pain has rapidly increased, and she feels feverish. The photograph shows the affected area at the time of the visit. Question What is the best next step in treatment of her condition? Answer Choices 1 Aggressive surgical debridement 2 High-dose crystalline penicillin 3 Intravenous clindamycin 4 Intravenous amikacin 5 Intravenous ceftazidime

Explanation Intravenous ceftazidime is the correct answer. The most common causative organism of malignant otitis externa is the Pseudomonas species. It has a rapid clinical course in diabetics that can be fatal if left untreated. Therapy involves the use of high doses of anti-pseudomonal drugs, such as Ceftazidime. Anti-pseudomonal penicillins, such as Piperacillin and Ticarcillin, are other very effective options. In earlier years, quinolones were used widely, but their routine use for upper respiratory infections has increased rates of Pseudomonal resistance. Surgical debridement is incorrect. With malignant otitis externa, infection spreads rapidly via fascial and vascular planes, and it requires radical debridement of the ear cartilage and temporal bone to prevent its growth; therefore, surgery is restricted to localized slough removal and the removal of bony sequestrum if needed. High-dose crystalline penicillin is incorrect. Pseudomonas is almost completely resistant to penicillin now. Intravenous clindamycin is incorrect. Clindamycin is ineffective against Pseudomonas. It is commonly used in anaerobic infections, diabetic ulcers, MRSA infections, and cases of acne. Intravenous amikacin is incorrect. As an aminoglycoside, it is used in Gram-negative sepsis.

A 23-year-old woman presents with a history of chronic pelvic pain. She attained menarche at 13, and she has had regular periods. She has been experiencing severe pain during menses; the pain has been increasing in frequency. She has never had inter-menstrual bleeding, and she regularly uses barrier contraception. However, she has refrained from intercourse for over 6 months due to the pain it causes her. Her vital signs and general physical examination appear to be normal. A vaginal exam reveals cervical motion tenderness, with reduced mobility and nodularity of the utero-sacral ligaments. Question For a definitive diagnosis, what is the diagnostic test of choice? Answer Choices 1 HSG 2 FSH and LH levels 3 Ultrasonography 4 Laparoscopy 5 Pap smear with endocervical curettage

Explanation Laparoscopy is the correct answer. Laparoscopy is considered the gold standard in diagnosis of endometriosis. Laparoscopy allows visualization of the islands of abnormally implanted endometrial tissue. However, visual inspection alone has a high false positive rate. Confirmation requires biopsy of the abnormal areas, with histopathology demonstrating endometrial epithelium, glands, stroma, or hemosiderin-laden macrophages. Hysterosalpingography is incorrect. A visualization of the uterine cavity and tubes by this technique is useful in delineating anatomic defects, such as a bicornuate uterus, as well as mechanical obstruction by adhesion or stenosis as seen post pelvic inflammatory disease. It is not of any use in endometriosis. FSH and LH levels is an incorrect response. They may be used in the workup of infertility, but they provide no information on endometriosis. Ultrasonography is incorrect. In general, imaging techniques are not particularly helpful in diagnosis. It is useful in ruling out other pelvic pathology. Pap smear with endocervical curettage is incorrect. It is mostly a screening procedure for cervical and endocervical pathology. It can help in the identification of endometrial dysplasia and malignancy, but not endometriosis.

A 2-month-old infant is being seen for a routine examination by his pediatrician. However, his mother admits to not following recommendations and has not had him seen by the pediatrician since hospital discharge. During the genitourinary examination, the pediatrician cannot palpate the testis on either side of the scrotum. The pediatrician is concerned that the infant has bilateral cryptorchidism (or undescended testes), but needs to make sure testes are present somewhere above the scrotum. Question What laboratory testing and imaging study combination would the pediatrician order? Answer Choices 1 Luteinizing hormone, follicle stimulating hormone, and testosterone levels followed by CT scan 2 Luteinizing hormone, follicle stimulating hormone, and testosterone levels followed by ultrasonography 3 Human chorionic gonadotropin stimulation test and testosterone levels followed by ultrasonography 4 Human chorionic gonadotropin stimulation test and testosterone levels followed by X-ray 5 Human chorionic gonadotropin stimulation test and testosterone levels followed by CT scan

Explanation Luteinizing hormone (LH), follicle stimulating hormone (FSH), and testosterone levels followed by ultrasonography is the correct answer. In male infants under the age of 3 months, LH, FSH, and testosterone levels are helpful in determining whether there are testes present. Ultrasonography has a sensitivity of 76%, a specificity of 100%, and an overall accuracy of 84% when diagnosing undescended testes that are nonpalpable on examination. MRI would have also been a good imaging choice, as it has a sensitivity of 86%, a specificity of 79%, and an overall accuracy of 85%. However, ultrasonography is both easier and a lower risk to perform on infants and children, making it the more popular choice in many cases. Luteinizing hormone, follicle stimulating hormone, and testosterone levels followed by CT scan is not the correct answer. CT scan findings in children when used in the diagnosis of nonpalpable undescended testes are historically not reliable. Therefore, the imaging study of CT scan makes this answer incorrect. Human chorionic gonadotropin (HCG) stimulation test and testosterone levels followed by ultrasonography is not the correct answer. The HCG stimulation test is done by administering 2000IU of HCG daily for 3 days and checking testosterone levels pre- and post-stimulation. This helps to determine the presence or absence of testicular tissue. However, this test is reserved for infants over 3 months of age. Ultrasonography is the correct imaging study, but the answer is incorrect due to the laboratory test being incorrect. Human chorionic gonadotropin stimulation test and testosterone levels followed by x-ray is not the correct answer. The HCG stimulation test is done by administering 2000IU of HCG daily for 3 days and checking testosterone levels pre- and post-stimulation. This helps to determine the presence or absence of testicular tissue. However, this test is reserved for infants over 3 months of age. X-ray is not used at all in the diagnosis of cryptorchidism. Both facets of this answer choice are incorrect. Human chorionic gonadotropin stimulation test and testosterone levels followed by CT scan is not the correct answer. The HCG stimulation test is done by administering 2000IU of HCG daily for 3 days and checking testosterone levels pre- and post-stimulation. This helps to determine the presence or absence of testicular tissue. However, this test is reserved for infants over 3 months of age. CT scan findings in children when used in the diagnosis of nonpalpable undescended testes are historically not reliable. Both facets of this answer choice are incorrect.

A 45-year-old man presents with severe pain in the left flank that began in the morning; it comes and goes, lasts 20 minutes, and has an intensity of 10/10. He denies fever or urinary symptoms. He has a cousin with pancreatic cancer. The physical exam (PE) is negative. Laboratory analysis shows UA with uncountable erythrocytes, and high plasma levels of PTH. Question What is the most probable diagnosis? Answer Choices 1 Li-Fraumeni syndrome 2 Von Hippel-Lindau disease 3 Turcot syndrome 4 Cowden syndrome 5 MEN1

Explanation Multiple endocrine neoplasia type 1 (MEN1) is an autosomal dominant tumor predisposition. The diagnosis is often made in patients between 40 - 50 years of age; the disease is characterized by tumors of the parathyroid (83 - 97%), the pancreas (38 - 84%), and the pituitary (18 - 65%). Hyperparathyroidism is the most common manifestation. MEN1 is caused by mutations in the MEN1 gene on chromosome 11q13. The diagnosis is important because the treatment of tumors in patients with this mutation can be different than customary treatment. Li-Fraumeni is an autosomal dominant tumor predisposition syndrome. Mutations in p53 have been identified in 40% of LFL. The tumors appear before 45 years of age. Usually, they are soft tissue sarcomas or osteosarcomas, but they can also manifest as brain cancer, breast cancer, leukemia, and other cancers. There are 3 criteria that are needed to be able to make this diagnosis: the patient had a sarcoma before the age of 45 a first-degree relative had a diagnosis of any cancer before the age of 45 another first-degree or second-degree family member had a tumor before the age of 45 or a sarcoma at any age Von Hippel-Lindau disease is an autosomal dominant tumor predisposition syndrome. The main criterion for the diagnosis is the presence of hemagioblastomas. Patients also can have multiple benign and malignant tumors of the central nervous system, kidneys, pancreas, adrenal glands, and paraganglia. The most malignant tumor that these patients can present with is renal cell carcinoma; it is seen in more than 50% of patients. Another tumor is the pheochromocytoma which is seen in 7 - 20% of patients. This tumor can produce a sudden release of catecholamines and result in death, especially pre or post-surgery. Turcot syndrome is an inherited tumor predisposition. It is characterized by the presentation of colon cancer and brain cancer in the same patient. There are at least 2 types: familial adenomatouspolyposis (FAP) and medulloblastoma caused by APC germline mutations hereditary nonpolyposis colon cancer (HNPCC) and glioblastoma caused by germline mutations in 1 of the DNA mismatch repair genes (e.g., hPSM2 or hMLH1) Cowden syndrome is an autosomal dominant tumor predisposition characterized by the development of multiple mucocutaneous lesions (hamartomas) and benign tumors. Patients also have a predisposition to a variety of neoplasias, particularly breast and thyroid cancers. This is related to mutations in the PTEN tumor suppressor gene.

A 38-year-old man presents with fatigue, dry mouth, and passing large amount of urine. He describes his urine as light in color and non-odorous. He says he has never had this before. He has always been healthy, and has never been hospitalized. He had serum electrolytes investigated during a routine pre-employment screening, and his sodium was slightly above normal; however, at that time, he was reassured that it was due to slight dehydration. He was advised to repeat the test after 3 months. When he did about a week ago, it showed more of an increase, and he was also worried about that. He takes vitamin supplements and sometimes uses energy drinks. To avoid going to the bathroom too often, he tries to restrict fluid intake to minimum, but it has never helped. Blood glucose level was measured, and it was normal. DDAVP (desmopressin) injection led to decrease in urine volume and increase in urine specific gravity. Question What is the best treatment? Answer Choices 1 Adequate fluid intake 2 Fluid restriction 3 Diet and exercise 4 Nasal desmopressin 5 Hydrochlorothiazide

Explanation Nasal desmopressin is used to treat central diabetes insipidus. Diabetes insipidus is a disease in which ADH (antidiuretic hormone) from the posterior pituitary is either deficient or not working. ADH is secreted from posterior pituitary and works on distal renal tubules on specific receptors to reabsorb water and this occurs in response to osmoreceptors to help keep the body in balance. Symptoms include polyuria, polydipsia, and decreased urine specific gravity. Causes: Central due to tumor anoxia or hemorrhage. Nephrogenic due to the effect of central drugs of which lithium is well known reason. Differential diagnosis: Diabetes mellitus Psychogenic polydipsia Workup: First exclude diabetes mellitus by random blood sugar then exclude psychogenic polydipsia by water deprivation test (here the patient did it himself!). Then do desmopressin test to differentiate Nephrogenic from central DI. If it is positive do MRI to exclude brain tumors. Treatment: In mild cases, adequate hydration is enough. For Central DI, give desmopressin. In psychogenic polydipsia, psychiatric consultation is needed. If drug induced, stop offending drug. Hydrochlorothiazide combined with indomethacin can be used in Nephrogenic DI as treatment.

A 32-year-old healthy woman is asking for advice about contraception. She wants contraception that is both effective and convenient in terms of application. She thinks that oral contraceptive pills will work the best, and she is interested in additional benefits that the pill can provide (particularly cancer protection). Question You inform her that, in her case, oral contraceptives may reduce the risk of death from what condition? Answer Choices 1 Breast cancer 2 Ovarian cancer 3 Colon cancer 4 Cervical cancer 5 Hepatic cancer

Explanation Ovarian cancers are often diagnosed in the advanced stages; this is the reason why ovarian cancer accounts for more deaths than any other gynecological cancer. The risk of ovarian cancer is reduced by 30% with pill use for <4 years, by 60% with use for 5 - 11 years, and by 80% after 12 or more years of use. Therefore, protection against ovarian cancer reduces the risk of gynecological cancer death. The relationship between breast cancer and OCP is controversial. Some research finds no increased risk of breast cancer in current or former users of OCPs aged 35 - 64, in women with a family history of breast cancer, and in women who started oral contraceptive use at a young age. Other research has found a very low risk of breast cancer in women of childbearing age (relative risk of 1.1 - 1.2, depending on other variables). This small increase is not substantially affected by duration of use, dose, type of component, age at 1st use, or parity. Even 10 years after the discontinuation of oral contraceptive use, no difference in breast cancer incidence between past users and those who have never used oral contraceptives has been found. Breast cancers diagnosed in women who have used oral contraceptives are more likely to be localized to the breast; therefore, overall, there is no significant difference in the cumulative risk of breast cancer between those who have used oral contraceptives and those who have never used them. There also is no evidence that OCP use offers protection. There is no evidence that oral contraceptive pills have any effect on colon cancer. Combined OCP use does not decrease the risk of cervical cancer; it increases it by about 2-fold, but only in long-term users (>5 years) with persistent human papilloma virus infection. Combined OCP use does not decrease the risk of hepatic cancer. Rarely, it has been associated with the development of benign or malignant hepatic tumors (adenoma and hepatocellular carcinoma). The risk increases with higher dosage, longer duration of use, and older age. Current estimates indicate there is about 2 times the risk of liver cancer after 4 - 8 years of use; therefore, protection by OCPs against ovarian cancer reduces the risk of gynecological cancer death.

An overweight woman presents with a fullness in her neck. TSH, T 3, and T 4 are normal, but the sonogram shows multiple masses on both sides of her thyroid. A fine needle biopsy is done, and it reveals a malignant tumor. What is the most common thyroid malignancy? Answer Choices 1 Follicular adenoma 2 Colloid goiter 3 Polyendocrine adenomatosis 4 Hashimoto's disease 5 Papillary carcinoma 6 Follicular carcinoma 7 Sipple's syndrome 8 Zollinger-Ellison syndrome

Explanation Papillary carcinoma is the most common thyroid gland malignancy. The incidence is greatest among adults 40-years of age or less. Females are more likely to develop this type of thyroid malignancy. It is the least malignant cancer. Polyendocrine adenomatosis is a rare syndrome where adenomas of the endocrine tissue are found in multiple sites. The most common are anterior pituitary gland, islets of Langerhans, and the parathyroid. This can be called the Zollinger-Ellison syndrome. Follicular adenomas are adenomas of the thyroid where the cells are arranged in the form of follicles. This benign epithelial tumor is diagnosed with a biopsy. The colloid goiter is a large soft goiter in which follicles of the gland are distended. Sipple's syndrome is the bilateral medullary carcinoma of the thyroid that is part of an autosomal dominant problem seen with Marfan's syndrome. This is also associated with pheochromocytoma and there is a tendency towards hyperparathyroidism due to adenomas or hyperplasia. Hashimoto's disease, named after a Japanese surgeon, is thyroiditis.

A 30-year-old woman presents with a 2-day history of fever, cough with sputum, and chest pain; there is also a 1-day history of frank blood in sputum (75 cc in the last 24 hours). She is a non-smoker and does not give a history of any recent inhalation exposure or illicit drug use. There is no past history of malignancy or autoimmune disease. On examination, pulse is 92/min; BP is 106/70mmHg; and temperature is 101 F. Oxygen saturation is 97%. On auscultation, S1, S2 normal, decreased breath sounds, and increased tactile vocal fremitus are noted on the right middle lobe. There is no pallor, icterus, cyanosis, edema, or lymphadenopathy. Question What is the next best step in evaluation of her hemoptysis? Answer Choices 1 Computed Tomography (CT) scan of chest 2 Sputum for Gram stain 3 Plain X-ray of chest 4 Bronchoscopy 5 Complete blood count (CBC)

Explanation Plain X-ray of chest is the correct answer. It is the next step in the evaluation of hemoptysis to identify the source of bleeding1. If the chest X-ray does not help identify the source of bleeding, then a CT scan of the chest should be done1. CT scan helps to identify the source of bleeding, as well as gives information on mediastinal lymhadenopathy, which may support the diagnosis of thoracic malignancy1. Both Gram stain and CBC should be obtained for complete evaluation1. Gram stain helps evaluate for the presence of infection, and CBC is used to assess platelet count and hematocrit1. These investigations should be done following chest X-ray. If all these procedures are unrevealing, then bronchoscopy should be considered1.

A 25-year old-woman is admitted to the hospital due to a 3-week history of episodic headaches; they are accompanied by palpitations and sweating. The episodes occur several times daily; each episode lasts about 15 minutes. On examination, the patient is diaphoretic and has elevated blood pressure, which prompted her admission. Her blood pressure is currently 220/100 mm Hg. Past medical history is significant for exercise-induced asthma, for which she takes albuterol. Question What test would be useful in the diagnosis of this patient? Answer Choices 1 Cosyntropin stimulation test 2 Dexamethasone suppression test 3 Plasma fractionated metanephrine 4 Electrocardiogram 5 Echocardiogram

Explanation Plasma fractionated metanephrine is the correct response. The patient presented with episodic headache, palpitations, and diaphoresis that was accompanied by hypertension; these symptoms are suggestive of pheochromocytoma. Pheochromocytomas are tumors that originate from the adrenal glands and secrete norepinephrine. This secretion causes hypertension and symptoms, such as headaches, palpitations and diaphoresis. Patients with pheochromocytoma have elevated plasma metanephrine. Cosyntropin stimulation test is an incorrect response. The cosyntropin stimulation test is used in the diagnosis of Addison's disease. Dexamethasone suppression test is an incorrect response. The dexamethasone suppression test is used in the diagnosis of Cushing syndrome. While patients with Cushing syndrome may have hypertension, it typically does not occur along with episodic headaches, palpitations and diaphoresis. Electrocardiogram is an incorrect response. EKG findings are not diagnostic for pheochromocytoma. Echocardiogram is an incorrect response. Echocardiogram is not used in the diagnosis of pheochromocytoma.

A 58-year-old man presents with severe headache, gradual vision loss, weakness, and enlargement of his hands. He has noticed that his wedding ring no longer fits on his ring finger and that his feet feel "too tight" in his current sneakers. He said his voice seems to be deeper, and his skin feels doughy and sweaty. Laboratory testing reveals an elevated insulin-like growth factor (IGF)-I, and there is a failure of growth hormone (GH) suppression following an oral glucose load. His physical exam reveales soft-tissue swelling resulting in bilateral hand and foot enlargement, macroglossia, bitemporal hemianopsia, hyperhidrosis, oily skin, kyphosis, and proximal muscle weakness. His facial appearance is shown in the image. A laboratory work-up demonstrates an elevated insulin-like growth factor (IGF)-I and failure of growth hormone suppression following an oral glucose load. Question What health maintenance recommendation should be provided to this patient? Answer Choices 1 Following diagnosis, performing GH and IGF-I levels is unnecessary 2 Noncompliance to therapy has no effect on survival 3 Routine colonoscopies should be scheduled for this patient 4 A high salt and carbohydrate diet improves overall prognosis 5 Periodic bone density scans are required to identify demineralized bone

Explanation Routine colonoscopies should be scheduled for this patient. This patient's most likely diagnosis is acromegaly. Acromegaly is nearly always caused by a pituitary adenoma. Patients should be advised that, if untreated, one's life span is decreased by an average of 10 years. Patients must receive lifelong follow-up, with regular monitoring of serum GH and IGF-I levels. Serum GH levels over 5 ng/mL and rising IGF-I levels usually indicate a recurrent tumor. Both IGF-I and GH levels correlate with mortality; survival improves greatly if GH and IGF-I can be normalized. Patients must receive lifelong follow-up, with regular monitoring of serum GH and IGF-I levels. Both an initial colonoscopy and echocardiogram are recommended. Colon polyps and mortality from colonic malignancy are more likely to develop in patients with acromegaly. As such, increased surveillance for colorectal cancer is recommended. Based on a cohort study of patients hospitalized for acromegaly (Denmark 1977-1993; Sweden 1965-1993) linked to tumor registry data for up to 28 years of follow-up, individuals with acromegaly have higher rates of small intestine, colon, rectal, kidney, and bone cancer. Patients with acromegaly have increased morbidity and mortality from cardiovascular disorders and progressive acromegalic symptoms. Complications of acromegaly include hypopituitarism, hypertension, glucose intolerance or frank diabetes mellitus, cardiac enlargement, and cardiac failure. Strict control of comorbidities, including diabetes and hypertension, are essential. This patient is not at risk for demineralized bone; a bone density (DEXA) scan is not recommended in patients with acromegaly.

A 32-year-old woman presents for a routine gynecological exam. She has been married for 5 years and plans to start a family with her husband in the near future. Her first menstrual period was at the age of 11. She is in a 28/4 cycle with no irregularities or discomfort. At age 18, she had an induced abortion and has used oral contraceptives since then. In the recto-vaginal exam, you palpate a mass on the left side. An ultrasound confirms a complex cystic tumor 6cm in diameter on the left ovary. There is no free fluid in the pelvis. There is no family history of any malignant tumors. What is your therapeutic approach? Answer Choices 1 Laparoscopy and cystectomy 2 Ultrasound examination after the next period 3 Hormonal treatment with progesterone 4 Pelvic exam in 6 months 5 Ovariectomy

Explanation Since this patient is pre-menopausal, with no family history and the tumor is cystic, unilateral, asymptomatic, and there is no ascites, the possibility of it being malignant is very small. Most of these cysts regress within 2 menstruation cycles; therefore, ultrasonic control after the next period is a legitimate approach. If, after 2 cycles, the cyst persists or increases in size, laparoscopy and cystectomy would be indicated. Since the patient expressed the wish for children, it is very important to try leaving the ovary intact. Hormonal treatment with progesterone has not shown any advantage in retrogression of ovarian cysts. Even with no indication of a malignancy, a control period of 6 months is too long and should always include a sonogram. An ovariectomy should only be performed if laparoscopy shows a highly suspicious result of the tumor being malignant.

Growth of the epiphyseal cartilage during endochondral bone formation is stimulated by a hormone; if over-secreted, it may result in an individual in which there is an abnormal increase in the length of bones and their density. What is the hormone? Answer Choices 1 Calcitonin 2 Parathormone 3 Corticotrophin 4 Testosterone 5 Somatotrophin

Explanation Somatotropin, or growth hormone (GH), is secreted from the pituitary gland; it stimulates growth in general, but it especially stimulates growth of the epiphyseal cartilage during childhood. Over-secretion of GH during childhood will result in an abnormal increase in the length and density of long bones, causing a clinical condition referred to as gigantism. Calcitonin and parathormone also have major effects on bones; they do so by either depressing or stimulating the deposition of calcium in the matrix of bone, respectively.

A 10-year-old boy is brought to the clinic by his mother. She noted that his face is swollen, and he told her that his urine was cloudy and reddish. He has a history of falling and abrading the skin of his right thigh 2 weeks ago. The next day, the skin became red, hot, and tender; the infection was treated with a topical antibiotic ointment. The cellulitis gradually healed. You suspect acute glomerulonephritis. What organism is the most likely cause of the disease? Answer Choices 1 Streptococcus pyogenes (group A beta-hemolytic) 2 Streptococcus agalactiae (group B) 3 Enterococcus faecalis 4 Peptostreptococcus 5 Streptococcus pneumoniae

Explanation Streptococcus pyogenes (group A beta-hemolytic) cause 3 types of diseases: 1) pyogenic diseases, such as pharyngitis and cellulitis, 2) toxigenic diseases, such as scarlet fever and toxic shock syndrome, and 3) immunologic diseases, such as rheumatic fever and acute glomerulonephritis. Glomerulonephritis occurs especially following skin infections. Streptococcus pneumoniae are Gram-positive lancet-shaped cocci arranged in pairs (diplococci) or short chains. On blood agar, they produce alpha-hemolysis. Virulence factors of Pneumococci are polysaccharide capsules. Pneumococci cause pneumonia, bacteremia, meningitis, and infections of the upper respiratory tract, such as otitis and sinusitis. Mortality rate is high in elderly, immunocompromised (especially splenectomized), and/or debilitated patients. They should be immunized with the polyvalent polysaccharide vaccine. Peptostreptococci grow under anaerobic or microaerophilic conditions and produce variable hemolysis. Peptostreptococci are members of the normal flora of the gut and female genital tract and participate in mixed anaerobic infections of the abdomen, pelvis, lungs, and brain. Streptococcus agalactiae (group B streptococcus) colonize the genital tract of some women and can cause neonatal meningitis and sepsis. They are usually bacitracin-resistant. Enterococcus faecalis (group D streptococcus), formerly known as Streptococcus faecalis, are part of the normal flora in the gut. They can cause urinary, biliary, and cardiovascular infections.

A 40-year-old woman presents with swelling in her neck, which has gradually increased in size in the past year. On physical examination, her thyroid gland is symmetrically and diffusely enlarged and non-tender. Her thyroid function tests shows: Serum T4 3.5 micrograms dl Serum T3 30 ng/dl Serum TSH 12 mU/L Radioiodine uptake is decreased. The patient was subjected to thyroidectomy. On microscopic examination, it shows extensive infiltration of the parenchyma by a mononuclear infiltrate containing small lymphocytes, plasma cells, and well-developed germinal centers. The thyroid follicles are small; in many areas, they are lined by Hurthle cells. What is the most likely diagnosis? Answer Choices 1 Hashimoto's thyroiditis 2 Juvenile lymphocytic thyroiditis 3 Atrophic thyroiditis 4 Painless thyroiditis 5 Graves' disease

Explanation Subacute lymphocytic thyroiditis, or painless thyroiditis, is an uncommon cause of hyperthyroidism. It is considered a variant of Hashimoto's thyroiditis, as this disorder is also characterized by the presence of autoantibodies to thyroid globulin and thyroid peroxidase. The symptoms develop over a period of 1 to 2 weeks and last from 2 - 8 weeks before subsiding. Infiltrative ophthalmopathy and other features of Graves' disease are absent. In addition, even though the serum T3 and T4 levels are increased and TSH levels are decreased during the episode of hyperthyroidism, the radioiodine uptake is decreased. Graves' disease, also known as diffuse toxic goiter or Basedow's disease, is characterized by hyperthyroidism, diffuse thyroid enlargement, and infiltrative ophthalmopathy with resultant exophthalmos and infiltrative dermopathy. It is an autoimmune disease and shows elevated levels of T3 and T4 and depressed levels of TSH. However, the radioiodine uptake is increased, and the thyroid scan shows a diffuse uptake of iodine. Refer to the table for additional information. Hashimoto's thyroiditis (also known as autoimmune thyroiditis, diffuse lymphocytic thyroiditis, struma lymphomatosa, or goitrous autoimmune thyroiditis) is characterized by 3 main features: Diffuse goitrous enlargement of the thyroid Lymphocytic infiltration of the thyroid gland Occurrence of thyroid autoantibodies Juvenile lymphocytic thyroiditis, a variant of Hashimoto's thyroiditis, is also known as lymphocytic thyroiditis of childhood and adolescence. It occurs mainly in children and young females and is characterized by a mild enlargement of the thyroid and a lower level of autoantibodies compared to Hashimoto's thyroiditis. The patient is mildly hypothyroid or euthyroid and is symptomless. Atrophic thyroiditis is also a variant of Hashimoto's thyroiditis, and it is also known as idiopathic myxedema or spontaneous hypothyroidism. Instead of enlargement, there is diminution in size of the thyroid, and it is characterized by the presence of autoantibodies.

A 56-year-old African-American man presents with urinary hesitancy, frequency, and nocturia. He has to get up and urinate 3 to 4 times per night, and he is not sure if he empties his bladder completely. He states that his symptoms have been getting worse over the past 2 years. His urinary stream is weaker than it was a 1 year ago. He denies blood in his urine, and there is no history of urinary tract infections, dysuria, or pain. He is otherwise in a good state of health, and he has no significant past medical or surgical history. Currently, he takes no medications, and he has no known drug allergies. On review of systems, you discover that his father and brother died of prostate cancer in their 50's. The remainder of the history and ROS is non-contributory. Physical Examination: Vital signs are stable and he is afebrile. General physical exam is unremarkable. Genital exam reveals a circumcised penis with no lesions or discharge. There is no inguinal adenopathy. Testicles are descended bilaterally with no lesions, masses, or hernias. Rectal exam reveals a smooth prostate with no nodules or tenderness. Diagnostic investigations: Urinalysis is normal and Prostate-Specific Antigen (PSA) test is within normal range for age. (For men aged 50 to 59 years, the normal reference range is 0 to 3.5 ng/mL) After emptying 250 mL of urine, the post-void residual urine volume is 50 mL. Question What is the most appropriate intervention? Answer Choices 1 Finasteride 2 Surgical therapy 3 Terazosin 4 Transrectal ultrasound with prostate biopsy 5 Urine culture and sensitivity

Explanation Terazosin is the correct response. Given the history, physical exam, and negative PSA, you have enough information to make the diagnosis of symptomatic benign prostatic hyperplasia (BPH). No further diagnostic studies are necessary. Benign prostatic hyperplasia (BPH), also known as benign prostatic hypertrophy, is a histologic diagnosis characterized by proliferation of the cellular elements of the prostate. A classic history is usually sufficient to make the diagnosis. Clinical manifestations include urinary hesitancy, urinary frequency, urgency, nocturia (awakening at night to urinate), decreased or intermittent force of stream, and/or a sensation of incomplete bladder emptying. Treatment: Depending on the patient's preferences, the next step is to begin treatment; in most cases, medical therapy is initiated first. If the symptoms do not significantly interfere with the patient's life, he may choose to wait and refuse treatment once he is reassured that he does not have a life-threatening illness. This decision would be medically acceptable in this case. If he selects treatment, management begins with a selective α1-receptor blocker, such as doxazosin or terazosin. A medication specific for α 1A-receptor subtype, such as tamsulosin (Flomax®), may be used in patients who cannot tolerate traditional α1-receptor blockers. If medical therapy fails or if a patient has severe BPH with ongoing obstruction, retention of large volumes of urine, or recurrent urinary tract infections, surgical therapy should then be considered. The most commonly performed surgery is transurethral resection of the prostate. Because of his family history, the patient in this case has an increased risk of prostate cancer; however, transrectal ultrasound with prostate biopsy is not indicated. This diagnostic procedure should be reserved for suspicion of prostate cancer. Based on this patient's family history and because he is African-American (African-Americans have a 50% higher incidence of and mortality from prostate cancer in comparison with Caucasians) a healthy index of suspicion is astute nonetheless. Given this patient's classic BPH presentation and the normal PSA, prostate cancer is low probability at this time. That being said, caution must be exercise when using PSA as a diagnostic tool to rule in or rule out prostate cancer. The USPSTF recommends against PSA-based screening for prostate cancer. In addition, the negative prostate exam on rectal probing, while classically taught to be important, adds no additional information in most cases; currently, it is not recommended by the U.S. Preventive Services Task Force. No evidence in the case points towards the need for urine culture and sensitivity. Finasteride is a 5 α-reductase inhibitor. If the patient does not receive sufficient relief from maximum doses of a α1-receptor blocker, it may be added. However, it may take up to 6 months for a 5 α-reductase inhibitor to result in a noticeable difference in symptoms. Thus, finasteride is not a first-line treatment. The full therapeutic benefit of a α1-receptor blocker, on the other hand, will be apparent within 4-6 weeks. Post-void residual is a diagnostic tool used to determine if a patient with BPH will benefit from scheduled bladder catheterizations. A post-void residual >200 mL is associated with an increased risk of urinary tract infections. Scheduled catheterizations are usually reserved for cases in which medical or surgical interventions do not correct the problem; they are also used when medical and surgical interventions are contraindicated.

A 17-year-old boy experienced an increasing sensation of left scrotal fullness and heaviness after a snowboarding injury 2 weeks prior to presentation. Over the past 2 days, he has experienced worsening pain without dysuria, penile discharge, back or flank pain, fever, or chills. He denies any sexual activity. On exam, heart and lungs are normal; there is no gynecomastia. Genitourinary exam reveals Tanner stage 5 and circumcision. There are no skin lesions or lymphadenopathy. Right scrotum and testicle appear normal and without any tenderness, swelling, or erythema. The epididymis and spermatic cord are easily palpable. The left testicle is also palpable and normal in size and shape without any swelling, but it is tender to palpation over the inferior pole, and does not transilluminate. Pain does not decrease with elevation of the testis. Urinalysis is unremarkable. Question Based on these findings, what is the most likely diagnosis? Answer Choices 1 Testicular torsion 2 Testicular cancer 3 Varicocele 4 Epididymitis 5 Trauma

Explanation Testicular cancer is the most common cancer of young men between 15 and 34 years of age and is 4.5 times more common among Caucasian men then African Americans. 12% will have a history of cryptorchidism. Tumor presents commonly as a circumscribed, nontender area of induration within the testes that does not transilluminate. However, sometimes pain may be associated with torsion of the tumor, infarction, or bleeding into the tumor. The most common complaint is a feeling of fullness or heaviness in the scrotum. In most cases the epididymis and spermatic cord feel normal. Advanced tumors are diffusely enlarged and can be rock hard. Ultrasound is a sensitive and specific test that can discriminate between testicular neoplasm and other nonmalignant disorders. Additional tests for tumor markers and staging are indicated. All patients undergo orchiectomy with close observation for early stage tumors and chemotherapy and radiation for others. Either way, the prognosis is generally excellent. Testicular torsion will usually present with abrupt onset of acute testicular pain, swelling, and erythema. Torsion occurs because the attachments between the epididymis and the tunica vaginalis are loose, allowing the testis to twist on its vascular pedicle compromising blood flow. This is also known as the bell-clapper deformity. The cremasteric reflex is absent and lack of pain relief with gentle elevation of the affected testis is suggestive. Treatment consists of rapid surgical detorsion and fixation of the testis to the scrotal wall (orchiopexy) before irreversible testicular necrosis occurs. Varicocele is a dilatation of the venous plexus of the spermatic cord and affects about 20% of adolescents and male young adults. A palpable mass, which feels like a 'bag of worms', can be felt above the testis. Varicoceles rarely cause pain. It is most evident in a standing position; it collapses while supine. If there is progressive discomfort or a discrepancy in testicular volume with possible effects on fertility, a urological referral for surgical repair is indicated. Epididymitis is usually associated with a urinary tract anomaly; however, it can be caused by a virus or bacteria, it is commonly caused by Chlamydia trachomatis. Acute epididymitis is characterized by severe pain, tenderness, and swelling of the testes. Urinalysis and culture will typically be abnormal. Trauma may cause an overlying hematocele, swelling of the scrotum, and poor transillumination; in general, it will cause ecchymosis. Although this patient experienced precipitating trauma, there is no evidence of swelling or ecchymosis to indicate traumatic hematocele alone. Testicular trauma is usually an event that is indelibly inscribed in the memory of the victim, and it can be recited with much precision. Traumatic hematoceles are best managed with surgical exploration, evacuation, and repair of the testis.

A 33-year-old man presents for his biannual checkup. He was diagnosed with Type I diabetes at the age of 4 and has been on insulin since then. 2 years ago, he was switched to a continuous subcutaneous insulin infusion system. He exercises regularly, controls his diet, and has had his sugar well controlled for the past few months, checking regularly with home monitoring. His BP is 120/75, pulse 70/min, no JVD, and cardiovascular and respiratory exam are normal. In addition to eye and foot exams, the physician wants to assess the patient's renal function. Question What is the best screening test? Answer Choices 1 Serum creatinine 2 Blood urea level 3 Urine specific gravity 4 24-hour urine protein 5 Urine dipstick for protein

Explanation Testing the urine with a dipstick for microalbumin is the most appropriate screening test for diabetic nephropathy. Results of 2 or 3 tests for microalbuminuria should be more than 30 mg per day or 6-month period to diagnose a patient with diabetic nephropathy. An early morning sample is preferred, although not strictly required. The dipstick measures a spot albumin to creatinine ration in the urine, which is a reliable indicator of albuminuria. Screening for diabetes is recommended at the time of diagnosis with Type II diabetes and 5 years after diagnosis of Type I diabetes. Some authorities recommend earlier screening in individuals with Type I diabetes with poor metabolic control. Without any intervention, over 80% of patients with microalbuminura will develop overt nephropathy within 10 - 15 years. Of this population, roughly 20% will go on to develop renal failure. Diabetes has become the most common single cause of end-stage renal disease (ESRD) in the U.S. and Europe. This is attributed to the following: 1) the increasing prevalence of diabetes; 2) increased acceptance of diabetic patients to dialysis programs that were earlier excluded; and 3) increase longevity of diabetic patients. As per the findings of the Diabetes Control and Complications Trial Research Group, tight blood sugar and blood pressure control are the most important variables to control in order to prevent nephropathy. An HbA1c of <7% and BP of <135/80 mmHg are the standard of care. ACE inhibitors are the drug of choice for hypertension, with non-dihydropiridine calcium channel blockers as the alternative if ACE is contraindicated. At an early stage, the use of ACE inhibitors has been shown unequivocally to delay the progression of nephropathy. Creatinine, blood urea, and urine specific gravity are incorrect. While they are indicators of renal function, none of them are as sensitive or cost effective as measuring a spot urine albumin to creatinine ratio; however, they are measured in order to monitor renal function and assess the side effects of medications, particularly ACE inhibitors and oral hypoglycemic drugs. A 24-hour urine protein is incorrect. It is an excellent test with high sensitivity. However, it requires urine to be collected over 24 hours, which is cumbersome; therefore, it is inappropriate as an initial screening test. It is sometimes used in patients who are dipstick positive in order to quantify the excreted protein.

There are three major carrier proteins for T3 and T4. More than 99% of these transport globulins are responsible for binding and transporting T3 and T4. What transport binding is responsible for the most transportation of T3 and T4? Answer Choices 1 Immunoglobulin A 2 Immunoglobulin B 3 Albumin 4 Thyroxine-binding prealbumin (TBPA) 5 Thyroxine binding globulin (TBG)

Explanation The 3 major binding globulins are albumin, thyroxine-binding prealbumin (TBPA) and thyroxine binding globulin (TBG). Albumin binds approximately 15 to 20% of T4. TBPA binds approximately 10% and is responsible for immediate delivery of T4 and T3 to the cell. TBG is the binding globulin that binds most of T3 and T4 (approximately 70%).

Classic symptoms of polyuria, polydipsia, and weight loss in a pediatric patient is likely due to autoimmune damage of the insulin-producing β-cells of the pancreatic islets. Symptoms usually appear gradually, and they occur when at least 80% of the islets have been damaged. Question What is the most likely diagnosis of this pathological process? Answer Choices 1 Acute pancreatitis 2 Hypothyroidism 3 Diabetes type I 4 Diabetes type II 5 Cushing's syndrome

Explanation The above summary is depicting diabetes type I, which is the most common type of diabetes mellitus in patients under the age of 20. Presenting symptoms can include polydipsia, polyuria, significant weight loss, or even a dangerous state of hyperglycemia known as diabetic ketoacidosis. Over 95% of type I diabetics develop diabetic ketoacidosis through autoimmune activity on the pancreatic islet cells; in the remaining ~5% of cases, it is idiopathic. Acute pancreatitis (acute inflammation of the pancreas) does not usually lead to permanent destruction of the pancreatic islet cells. Patients who develop acute pancreatitis are adults with significant gallbladder disease and/or a history of heavy alcohol use. Hypothyroidism may be the result of the thyroid gland's failure to secret efficient levels of the main thyroid hormone free thyroxine, or FT4. Hypothyroidism may also occur because of a deficiency of the secretion of thyroid stimulating hormone (TSH), secreted by the pituitary gland. Although hypothyroidism can virtually affect almost every body system in terms of signs and symptoms, it does not lead to autoimmune damage of the pancreatic islets. Diabetes type II does consist of beta cell loss, but there are also other key characteristics that lead to this disease state. Patients develop increased tissue insensitivity, which is also termed insulin resistance. This insensitivity lead to hyperactivity of the beta cells, producing excessive amounts of insulin; over time, the developed hyperplasia of the beta cells leads to impaired or even impeded function. The majority of the time, this is not due to autoimmune issues. Cushing's syndrome (or hypercortisolism) refers to the manifestations of excessive corticosteroids on the patient. Many of these cases are due to excessive ACTH secretion by a benign pituitary adenoma; however, other causes of excessive corticosteroids may lead to this condition, such as other non-pituitary secreting ACTH neoplasms and excessive secretion of t cortisol from the adrenal glands; in up to 30% of these cases, the main cause may not be found. There are a multitude of symptoms and signs seen with Cushing's syndrome, and glucose tolerance impairment secondary to insulin resistance is one of them.

A man with suspected acute renal failure provides a urine specimen that demonstrates high urine osmolality, low sodium urine, and high BUN-to-creatinine ratio. The urine microscopic examination reveals few red blood cells and white blood cells with no casts. What is the most likely explanation? Answer Choices 1 Postrenal cause 2 Prerenal cause 3 Glomerulonephritis 4 Acute tubular necrosis 5 Interstitial nephritis

Explanation The clinical picture is suggestive of a postrenal cause. Common etiology includes obstruction of the urinary tract. Initial urine findings include high urine osmolality, low sodium urine, and a high BUN-to-creatinine level. The microscopic examination may be normal, but it also may contain red blood cells, white blood cells, or crystals. A prerenal cause would have hyaline casts present. Glomerulonephritis would have dysmorphic red blood cells and red blood cell casts on microscopic examination. Acute tubular necrosis would have pigmented granular casts and renal tubular epithelial cells on microscopic examination. Interstitial nephritis would have pyuria (including eosinophiluria), white blood cell casts, and white blood cells on microscopic examination. Patients may also present with fever and a transient maculopapular rash.

A 38-year-old man presents with a 15-lbs weight loss over the past couple of months; the weight loss is associated with poor appetite, nausea, fatigue, and irritability. The patient has been training for a triathlon, but he did not experience such pronounced weight loss during previous training regimens. Physical exam reveals a pulse of 60 beats/min, and a BP of 76/58 mm Hg. Abdomen and rectal exams reveal no abnormal findings. A skin exam reveals a dark tan over this body, including his genital region. Question What is the most likely diagnosis? Answer Choices 1 Melanoma 2 Excessive physical training 3 Cushing syndrome 4 Addison's disease 5 Growth hormone deficiency

Explanation The correct answer is Addison's disease. Addison's disease is typically characterized by hypotension, hyperpigmentation, weight loss, anorexia, fatigue, nausea, arthralgias, myalgias, and irritability. It is due to an insufficiency of adrenocortical hormones. Melanoma is a skin cancer that is characterized by a hyperpigmented, irregularly colored and bordered lesion, which is not mentioned in the above patient. Cushing syndrome is due to an excess of endogenous corticosteroid production; it is more common in women, and it includes signs of central obesity, muscle wasting, and thin skin. Excessive physical training is not likely, as it would not cause significant hypotension or tan skin in areas not typically exposed to the sun such as the genital region. Growth hormone deficiency can cause muscle wasting, increased fat mass, fatigue, memory problems, and even depression, but it is not associated with hypotension or hyperpigmented skin.

A 47-year-old man presents for his annual physical exam. His past medical history is not significant, and he is not currently on any medications. He consumes 2 beers weekly and does not smoke. His blood pressure is normal at this visit. His primary care physician orders a fasting lipid panel with the following results: Component Value Triglyceride 135 HDL 50 LDL 220 Question In addition to diet and exercise, what medication should be started on this patient? Answer Choices 1 Cholestyramine 2 Fenofibrate 3 Nicotinic acid 4 Simvastatin 5 Ezetimibe

Explanation The correct answer is Simvastatin. This patient has elevated LDL, decreased HDL, and a normal triglyceride level. Simvastatin, a HMG CoA reductase inhibitor, will decrease LDL and increase HDL levels. The patient has no history of liver disease, which is the main contraindication to using HMG CoA reductase inhibitors. Cholestyramine isa bile acid sequestrant that may be used in combination with HMG CoA reductase inhibitors; however, is not recommended for monotherapy in patients with elevated LDL levels. Fenofibrate, a fibric acid, is primarily used for the treatment of hypertriglyceridemia. The above patient has a normal triglyceride level. Nicotinic acid (niacin) may be used in combination with HMG CoA reductase inhibitors, but it is not typically used in monotherapy for elevated LDL levels. Ezetimibe acts by blocking cholesterol absorption in the intestines. It is currently recommended for those who fail to respond to HMG CoA reductase inhibitor therapy. This patient has newly diagnosed hyperlipidemia; therefore, ezetimibe would not be an appropriate first line treatment.

A 38-year-old woman presents for a consult regarding her thyroid disease. She was recently diagnosed with Graves' disease, and she has not started any treatment yet. Question What finding might you see that is considered severe and is present in about 5-10% of patients? Answer Choices 1 Exophthalmos 2 Conjunctivitis 3 Chemosis 4 Periorbital edema 5 Corneal abrasion

Explanation The correct answer is exophthalmos, which is a bulging of the eye anteriorly out of the orbit. This is considered a severe form of ophthalmopathy and can be present in about 5-10% of patients who have Graves' disease. When patients are treated with high-dose, tapered prednisone, it is possible to see reversal of this finding. However, the treatment works best in patients who are non-smokers. Conjunctivitis is not the correct answer; conjunctivitis is a form of mild ophthalmopathy, and it can be present in 20-40% of patients with Graves' disease. Conjunctivitis is an inflammation of the conjunctiva secondary to a bacterial or viral infection. Chemosis is not the correct answer; chemosis is a form of mild ophthalmopathy that can be present in 20-40% of patients with Graves' disease. It is a term that refers to edema of the outer surface of the eye, and it can be related to allergies or an infection, such as conjunctivitis. Periorbital edema is not the correct answer; it is an ophthalmologic finding that is more commonly found in hypothyroidism, not hyperthyroidism. Corneal abrasion is not the correct answer. It is not associated with thyroid disease. It is more closely associated with some form of trauma or foreign body in the eye.

A 59-year-old woman presents for a routine annual physical. She is a housewife who does not smoke or drink, and her only significant past medical history is for an appendectomy and radiation treatment for acne as a child. She is post-menopausal, but she uses natural supplements rather than hormone replacement. She is not on any prescription medications. She completed some routine lab work prior to the office visit. The results are entirely within normal limits, including cholesterol panel, glucose, TSH, and T4. Her only complaints are recent dysphagia and sensation of fullness in her neck. Physical examination reveals a large nodule right lobe of patient's thyroid. The patient is referred to an endocrinologist and undergoes an ultrasound of the thyroid, which is followed by a thyroid scan. The results confirm a 1.7cm solid nodule on ultrasound that is deemed 'cold' on thyroid scan. Question Prior to scheduling a fine needle aspiration and based on the information you have, what is the the most likely diagnosis? Answer Choices 1 Hashimoto's thyroiditis 2 Medullary thyroid carcinoma 3 Parathyroid cyst 4 Graves' disease 5 Papillary thyroid carcinoma

Explanation The correct answer is papillary thyroid carcinoma. The patient has 3 risk factors for thyroid cancer, including female gender, age over 40, and history of radiation to face/neck region for acne. Thyroid function tests are typically normal in patients with thyroid cancers. The diagnostic findings of a solid, cold nodule are also suspicious for a thyroid cancer. However, those findings can be seen in benign nodules as well. Given the fact that papillary thyroid carcinoma is the most common type of thyroid cancer (about 60% of all thyroid cancers) and the patient has a history of radiation to face/neck region, the most likely type of thyroid cancer in this case is papillary. Hashimoto's thyroiditis is not the correct answer. While thyroid nodules can be present in cases of Hashimoto's thyroiditis, they are usually "multinodular" rather than a solitary thyroid nodule. Patients would also usually have more complaints upon presentation. In addition, the physical examination is likely to be normal or reveal a goiter with or without multiple nodules. The thyroid function testing would not be normal. Most commonly, the TSH would be elevated in the face of decreased T4. Medullary thyroid carcinoma is not the correct answer, as this is not the 'most likely' diagnosis in this case. The patient is at risk for thyroid cancer, but her history of radiation to face/neck region and the fact that papillary carcinoma is much more common (60% as opposed to 5% of all thyroid cancers) make her type of cancer more likely to be papillary rather than medullary. Parathyroid cyst is not the correct answer. A parathyroid cyst falls under the category of a benign cyst of the thyroid. Thyroid cysts make up only 15 - 25% of thyroid nodules, and benign cysts make up 85% of that group. A parathyroid cyst is 1 of the many types of benign thyroid cysts, and it would not be solid on an ultrasound. Graves' disease is not the correct answer. Patients will typically have more complaints and positive symptoms when they have Graves' disease. In addition, the examination of their thyroid is not likely to have a solitary thyroid nodule. The thyroid function tests would not be normal and are most likely to show a decreased TSH and elevated T4.

A 28-year-old woman in her 2nd trimester of pregnancy presents with a 3.5 cm painless mass on her left breast. She has no other associated symptoms. She has a positive family history of breast cancer. She is afebrile; pulse is 80/min, and BP is 110/72 mmHg. On examination of the breasts, you notice engorgement and hypertrophy. The mass appears solid on ultrasonography (USG). Question What is the next step in evaluation of this mass? Answer Choices 1 Magnetic Resonance Imaging (MRI) 2 Percutaneous biopsy under local anesthesia 3 Fine needle aspiration cytology (FNAC) 4 Sentinel lymph node sampling using blue dye 5 Avoid any intervention at this time

Explanation The correct answer is percutaneous biopsy under local anesthesia. This is used to obtain a histological diagnosis, which can be done safely during pregnancy with a sensitivity of around 90%1. During pregnancy, non-ionizing examinations are preferred to those needing ionizing agents1. Breast ultrasonography is the first diagnostic instrument used by clinicians when a breast mass and the axillary area need to be assessed in a pregnant woman; it is non-ionizing and has high sensitivity and specificity1. MRI is incorrect; it should only be used when it will alter clinical decision making and when ultrasonography is inadequate1. No well-designed studies of the efficacy and safety of MRI of the breast during pregnancy have been reported, and results of some studies have shown that gadolinium-based MRI contrast agents pass through the placental barrier and enter fetal circulation1. Fine needle aspiration cytology (FNAC) should be done only in case of a cystic mass1. Gestational and puerperal hormones induce physiological hyperproliferative changes of the breast, which could lead to a false positive or false negative result with FNAC1. Therefore, this procedure is not recommended during pregnancy1. Staging with sentinel lymph node biopsy is done once the diagnosis of breast cancer is established. Clinicians can safely use sentinel lymph node staging during pregnancy using technetium 991. However, blue dye is associated with a risk of an anaphylactic maternal reaction, and it would probably distress the fetus; therefore, the use of blue dye should be avoided during pregnancy. Although sensitivity and specificity of sentinel lymph node biopsies during pregnancy have not been established, researchers have successfully used technetium-based identification in pregnant women1. Staging examinations and tumor biology assessment affect management decisions in pregnancy and should not be avoided.

A 35-year-old man with type I diabetes presents for an acute visit with uncontrolled blood sugars with high and low readings throughout the day and night. When he called to make this appointment, he was advised to bring in his blood sugar log and check a few 3:00 am blood sugars. He is currently on 40 units of NPH and 20 units of regular insulin before breakfast, and 20 units of NPH as well as 10 units of regular insulin before dinner. Time 7am 11 am 5 pm 11 pm 3 am Average blood glucose mg/dL 393 210 175 140 50 Question What is the most appropriate insulin change based on his average blood glucose readings? Answer Choices 1 Add 4 units of regular insulin before breakfast 2 Add 2 units of NPH insulin before breakfast 3 Add 2 units of NPH insulin before dinner 4 Eliminate 4 units of NPH insulin before dinner 5 Eliminate 2 units of regular insulin before dinner

Explanation The correct answer is to eliminate 4 units of NPH insulin before dinner. The first step in managing insulin therapy is to eliminate low blood sugars. This patient is experiencing nocturnal hypoglycemia, which stimulates the release of counter-regulatory hormones to increase the blood sugar, thereby resulting in rebound hyperglycemia. This would account for his elevated fasting blood sugar readings. The NPH insulin has an onset of action of 1 - 2 hours and a peak effect in 6 - 12 hours, so decreasing the dinner dose 10 - 20% will decrease the likelihood of 3:00 am hypoglycemia and the subsequent morning rebound hyperglycemia. Eliminating 2 units of regular insulin before dinner may have some effect on the patient's 3:00 am hypoglycemia, but at the expense of higher 11:00 pm blood sugar readings. The regular insulin has an onset of action in 30-60 minutes, with a peak effect at 2-4 hours. The current dinner dose of 10 units of regular insulin is providing rapid-acting coverage for that meal with an acceptable average blood sugar of 140 mg/dL at 11:00 pm. Eliminating 2 units before dinner will result in higher blood sugars at 11:00 pm and suboptimal control. Adding 4 units of regular insulin before breakfast will serve to lower the elevated fasting blood sugars, but will do nothing to correct the nocturnal hypoglycemia, which is the main problem. Adding this extra rapid-acting insulin is a reactive response rather than a proactive step to control blood sugars. Adding 2 units of NPH insulin before breakfast will have minimal effect to correct the elevated morning blood sugars or the nocturnal hypoglycemia. Adding 2 units of NPH insulin before dinner will make the nocturnal hypoglycemia worse.

A 36-year-old man with a past medical history of diabetes mellitus presents with weight gain and skin changes. His review of systems is positive for a diminished libido, impotence, depression, cognitive dysfunction, and emotional lability. Lately, his fasting glucose levels have been above normal. His physical exam notes increased adipose tissue in the face, upper back, and above the clavicles. His skin reveals ecchymoses, telangiectasias, and purpura along his back and lower extremities; there is also facial acne and cutaneous atrophy. His abdominal exam reveals the findings in the attached image. Question What is the most appropriate next step in the management of this patient? Answer Choices 1 Obtain a KOH preparation of the skin lesion 2 Begin therapy with ketoconazole 3 Schedule an MRI of the pituitary gland 4 Begin oral prednisone replacement therapy 5 Order a 24-hour urinary free cortisol level

Explanation The correct response is a 24-hour urinary free cortisol level. This patient is demonstrating signs and symptoms consistent with Cushing's syndrome. The diagnosis of Cushing's syndrome due to endogenous overproduction of cortisol requires the demonstration of inappropriately high serum cortisol or urine cortisol levels. Currently, 4 methods are accepted for the diagnosis of Cushing's syndrome: urinary free cortisol level, low-dose dexamethasone suppression test, evening serum and salivary cortisol level, and dexamethasone-corticotropin-releasing hormone test. Urinary free cortisol determination has been widely used as an initial screening tool for Cushing's syndrome because it provides measurement of cortisol over a 24-hour period. A valid result depends on adequate collection of the specimen. Urinary creatinine excretion can be used to assess the reliability of the collection. Urine free cortisol values higher than 3 - 4 times the upper limit of normal are highly suggestive of Cushing's syndrome. A potassium hydroxide (KOH) preparation is a diagnostic test used to assess for the presence of a fungal disorder. Agents that inhibit steroidogenesis (e.g. mitotane, ketoconazole, metyrapone, aminoglutethimide, trilostane, and etomidate) have been used to cause medical adrenalectomy. These medications are used rarely, and they are often toxic at the doses required to reduce cortisol secretion. For instance, ketoconazole's prescribing information was revised to include a black box warning regarding hepatotoxicity, including fatalities and liver transplantation. Therefore, medical treatment should be initiated cautiously; ideally, it should be done in conjunction with a specialist. Ketoconazole has been the most popular and effective of these agents for long-term use and usually has been the agent of choice. However, the FDA has issued a warning that states clinicians should no longer prescribe ketoconazole, except to treat some life-threatening fungal infections; this is due to increased risk for severe liver injury, adrenal insufficiency, and adverse drug interactions. Imaging studies for Cushing's syndrome should be performed after the biochemical evaluation has been performed. The rationale for this is that unguided imaging of the pituitary or adrenal glands may yield a 10% incidence of incidental nonfunctioning pituitary or adrenal adenomas, which may mislead one from proper therapy and surgery. Ideally, the biochemical abnormalities should reconcile with the anatomic abnormalities before definitive therapy is offered. An abdominal CT scan is recommended if a primary adrenal problem is suspected. The presence of an adrenal mass larger than 4 - 6 cm raises the possibility that the mass is an adrenal carcinoma. Corticosteroid therapy (e.g. prednisone) is not appropriate treatment for a patient experiencing hypercortisolism.

An obese woman pregnant with twins is concerned about gaining too much weight. How do you counsel her regarding the Institute of Medicine's (IOM) recommended weight gain in women carrying twins? Answer Choices 1 A 25 to 42 pound weight gain is recommended. 2 A 37 to 54 pound weight gain is recommended. 3 An 11 to 20 pound weight gain is recommended. 4 She should maintain her current pre-pregnancy weight as much as possible. 5 A 25 to 35 pound weight gain is recommended.

Explanation The correct response is a 25 to 42 pound weight gain is recommended. The IOM did not distinguish different weight recommendations for women carrying twins from those for women pregnant with a singleton until recently. According to IOM, women pregnant with twins should attempt to gain between 37 - 54 pounds. A woman with a normal BMI (18.5 to 24.9) carrying only 1 baby should attempt to gain 25 to 35 pounds in pregnancy. IOM recommends that pregnant women with a low BMI (< 18.5) should attempt to gain 28 to 40 pounds if carrying 1 infant; overweight women with a high BMI (25 to 29.9) should attempt gaining 15 to 25 pounds, or 31 - 50 pounds if carrying twins; and obese women (BMI > 29.9) should gain 25 - 42 pounds if carrying twins, and only 11 - 20 pounds if pregnant with a singleton.

A 17-year-old girl presents with a 1-month history of vaginal discharge. The discharge is described as thin, greenish-yellow, and malodorous. She also admits to vaginal irritation and postcoital bleeding. She currently has 1 sexual partner and denies the use of barrier contraceptive methods. Speculum exam reveals erythematous vaginal mucosa, purulent, thin, frothy discharge, and a 'strawberry cervix'. Question Based on the patient history and physical examination findings, what is your most likely diagnosis? Answer Choices 1 Chronic cervicitis caused by Neisseria gonorrhea 2 Chronic cervicitis caused by Mobiluncus curtisii 3 Acute cervicitis caused by Trichomoniasis vaginalis 4 Acute cervicitis caused by Gardnerella vaginalis 5 Foreign body cervicitis

Explanation The correct response is acute cervicitis caused by Trichomoniasis vaginalis. This patient has the classic findings associated with an acute trichomoniasis infection, which are purulent, malodorous, thin, frothy discharge with vaginal irritation, and punctate hemorrhages on the cervix (strawberry cervix). The case does not support the other diagnoses. Chronic cervicitis is typically noninfectious, and cervicitis caused by Gardnerella vaginalis is not associated with purulent discharge or strawberry cervix; no foreign body was identified on speculum exam, which would support a diagnosis of foreign body cervicitis.

A 26-year-old woman wants to conceive over the next few months. She is married, has 1 child, and she and her husband have been using condoms and foam for contraception. Her past medical history is significant for hypothyroidism which is treated with levothyroxine (Synthroid) 0.125 mg PO per day. Her last thyroid-stimulating hormone (TSH) level, performed 3 months ago, was normal. Today, she presents with a 6-week history of amenorrhea. A urine pregnancy test is positive. You estimate that she is 6 weeks pregnant. What will she likely need? Answer Choices 1 A decrease in her levothyroxine (Synthroid) dose 2 Checking free T4 levels 3 Adding triiodothyronine 4 An increase in her levothyroxine (Synthroid) dose 5 Checking free T3 levels

Explanation The correct response is an increase in her levothyroxine (Synthroid) dose. This patient is pregnant and hypothyroid.The management of this patient requires careful titration of her levothyroxine (Synthroid) levels to maintain euthyroidism during pregnancy. Thyroid hormone is highly protein-bound, and any medical condition that changes the level of serum-binding proteins will change the level of free thyroid hormone available. For example, conditions that increase the level of serum-binding proteins (e.g., pregnancy and oral contraceptives) will decrease the level of free thyroid hormone. In such cases, the dose of replacement thyroid must be increased. Conditions that necessitate a decrease in replacement doses include aging, chronic liver disease, and protein-losing conditions (e.g., nephropathy and enteropathies). While free T4 levels estimate the level of thyroid hormone available, the variation in levels does not reflect the metabolic needs of patients. TSH assessment provides a better reflection of decreasing levels of available thyroid hormone; however, TSH levels lag behind changes in free T4 levels. TSH should be repeated no earlier than 4 weeks after any changes are made in thyroid doses; it is important to note that it may take 8 weeks for the TSH to stabilize after the change is made. Some patients may note an improvement in mood, memory, and cold tolerance with the addition of triiodothyronine to their regular thyroid replacement. In the patient presented, however, there was no problem with her thyroid replacement before her pregnancy; therefore, it is unlikely that she will need T3 added to her regimen. Free T3 levels are unnecessary in the management of this patient. The increased T4 dose requirement is typically seen by the 1st trimester of pregnancy. Monitoring the TSH regularly throughout pregnancy will allow adjustments to be made in the levothyroxine (Synthroid) dose. The goal of thyroxine replacement is to maintain serum TSH in the normal range. It must be remembered that hypothyroidism during pregnancy is associated with hypertension and premature labor. Dose adjustments upwards are usually in the range of 25 - 50% of the normal daily thyroid replacement dose.

A 25-year-old woman presents because she feels weak and tired all the time. She also reports that she has been amenorrheic for the past 3 months. On examination, she is not pale but she appears fatigued. Her BP is 100/50 mm Hg and a pregnancy test is negative. What other symptoms would you expect from this patient? Answer Choices 1 Hyperhidrosis 2 Insomnia 3 Heat intolerance 4 Diarrhea 5 Decreased axillary hair

Explanation The correct response is decreased axillary hair. The clinical picture is suggestive of hypopituitarism. Hypopituitarism results in decreased levels of growth hormone (GH), gonadotropins (FSH, LH), thyroid stimulating hormone (TSH), adrenocorticotrophic hormone (ACTH), and prolactin. The onset is usually insidious. The clinical features of hypopituitarism depend on the underlying cause and the specific hormones that are deficient. Thyroid stimulating hormone deficiency results in hypothyroidism. Patients present with feeling weak, drowsy, and fatigued. Some may complain of rough skin and decreased sweating. These symptoms may be accompanied constipation and cold intolerance. Adrenocorticotrophic hormone deficiency results in adrenal insufficiency, which is characterized by fatigue, hypotension, and the loss of axillary as well as pubic hair. Gonadotropin deficiency results in amenorrhea and genital atrophy in females, as well as impotence and testicular atrophy in males. Growth hormone deficiency results in growth failure in children, but has no clinically significant effects in adults. Prolactin deficiency results in failure of lactation in the postpartum period. When panhypopituitarism is suspected, all target organ hormone function should be evaluated. Initial evaluation should be aimed at detecting TSH and ACTH deficiencies, because their deficiencies can cause life-threatening conditions: myxedema coma and adrenal crisis, respectively. Serum thyroxine (T4), triiodothyronine (T3), TSH levels, serum cortisol, and ACTH levels are low in hypopituitarism. Gonadotropin levels are low in panhypopituitarism, and GH measurements are generally helpful only in children. Thyrotropin-releasing hormone, insulin, and gonadotropin-releasing hormone can be administered intravenously together; then, TSH, GH, LH, FSH, ACTH, cortisol, prolactin, and glucose levels can be measured. Diagnostic imaging includes skull X-rays of the sella turcica, which may provide evidence of the presence of pituitary tumors. High-resolution CT scans and MRI with contrast media may visualize smaller pathologies. The differential diagnoses for panhypopituitarism include anorexia nervosa, chronic liver disease, and polyglandular autoimmune disease. In anorexia nervosa, the secondary sexual characteristics are maintained even though the patients may be wasted and amenorrheic. Men with chronic alcoholic liver disease may develop testicular atrophy, but they have other features of chronic liver disease, and the serum levels of pituitary hormones are usually normal. Treatment includes replacing the hormones of the hypofunctioning target glands. Treatment of GH deficiency in adults is unnecessary. When hypopituitarism is due to a pituitary tumor, specific treatment must also be directed to the tumor as well as to hormone replacement therapy. The management of such neoplasms may include surgical resection, bromocriptine, or irradiation. Hyperhidrosis, insomnia, heat intolerance and diarrhea are symptoms of hyperthyroidism.

A 29-year-old pregnant woman presents for a prenatal check up. She is 6 months pregnant. She comments that she is constantly tired and is worried about her thyroid function because she has a friend who just had thyroid surgery. On physical examination, her pregnancy is progressing normally. Her thyroid is nonpalpable. No bruit over her thyroid is heard. Her skin is unremarkable. The doctor orders a series of blood tests. When the results come back, her doctor tells her that her abnormal lab results are probably due to a change in the level of thyroxine-binding globulin (TBG). Her lab results are on the chart. TEST RESULTS REFERENCE RANGE Serum total T4 15 µg/dL 5 - 11 µg/dL Free T4 2.5 ng/dL 1.5 - 3.5 ng/dL Question What condition does her blood test suggest? Answer Choices 1 Euthyroidism 2 Thyrotoxicosis 3 Hyperthyroidism 4 Hypothyroidism 5 Myxedema

Explanation The correct response is euthyroidism. Thyroid hormone exists in the circulation, either bound or free. It is the unbound component, which has the metabolic effect. Thyroxine-binding globulin (TBG) binds roughly 70 - 75% of the thyroid hormones. Many conditions, including pregnancy, can affect the level of thyroxine-binding globulin in the serum, but not affect the level of unbound hormone. Since her unbound level of thyroxine is normal and it is the unbound portion that exerts the metabolic effect, this patient probably is euthyroid. Thyrotoxicosis and hyperthyroidism are due to an excess of active thyroid hormone. Hypothyroidism and myxedema are due to a deficiency of thyroid hormone.

A 40-year-old Caucasian woman, with a past medical history of Hashimoto's thyroiditis, type I diabetes mellitus, and pernicious anemia, presents with a 2-year history of insidious and intermittent fatigue, anorexia, involuntary weight loss, nausea, abdominal pain, vomiting, and dizziness associated with positional changes. Her physical exam is noteworthy for postural hypotension, with a maximum systolic blood pressure of 104 in the supine position. A fingerstick determination reveals a blood glucose of 100 mg/dL, and she has a normal urinalysis. She has a low-grade fever and a generalized pigment change to her skin, as noted in the attached image. The comprehensive metabolic panel demonstrates hyponatremia, hyperkalemia, a mild non-anion gap metabolic acidosis, and elevated BUN and creatinine levels. Question What is the most appropriate treatment for this patient? Answer Choices 1 Bromocriptine 2 Hydrocortisone 3 Insulin aspart 4 Metyrapone 5 Octreotide

Explanation The correct response is hydrocortisone. Given this patient's strong autoimmune history, the most likely diagnosis is Addison's disease. The most common cause of Addison's disease is idiopathic autoimmune adrenocortical insufficiency resulting from autoimmune atrophy, fibrosis, and lymphocytic infiltration of the adrenal cortex. Manifestations include insidious hyperpigmentation of the skin (most often prominent on the sun-exposed areas of the skin, extensor surfaces, knuckles, elbows, knees, and scars formed after the onset of disease), progressive weakness, fatigue, poor appetite, and weight loss. Prominent gastrointestinal symptoms may include nausea, vomiting, and occasional diarrhea; dizziness with orthostasis due to hypotension occasionally may lead to syncope. The comprehensive metabolic panel reveals hyponatremia, hyperkalemia, a mild non-anion gap metabolic acidosis, and elevated BUN and creatinine levels. Treatment involves replacement of corticosteroids and mineralocorticoids. The normal adrenal gland output of cortisol is approximately 250 - 300 mg in 24 hours; this amount of hydrocortisone in soluble form (hydrocortisone sodium succinate or phosphate) should be given, preferably by continuous infusion. In patients in acute adrenal crisis, IV access should be established urgently, and an infusion of isotonic sodium chloride solution should be started to restore volume deficit and correct hypotension. Some patients may require glucose supplementation. The drugs that are effective in the treatment of hyperprolactinemia are dopamine agonists; they are the primary physiologic inhibitors of prolactin secretion. Bromocriptine is generally considered to be the agent of choice in the treatment of prolactinoma. Insulin aspart is not appropriate, as the patient is euglycemic; her urinalysis does not reveal any evidence of ketoacidosis. Metyrapone, along with ketoconazole, mitotane, cabergoline, and pasireotide, are medications useful in reducing ACTH or cortisol levels in patients with Cushing's syndrome. Somatostatin and dopamine analogues and growth hormone (GH) receptor antagonists are the mainstays of medical treatment for GH excess syndromes (e.g., acromegaly and gigantism). They are generally used when primary surgery fails to induce complete remission. The most extensively studied and used somatostatin analogue, octreotide, binds to the somatostatin receptor; this inhibits GH secretion.

A 29-year-old woman inquires when she should have her first mammogram. Her family history is significant for the death of her mother and grandmother due to breast cancer at the age of 32 years. Presently, the woman is asymptomatic. What is the recommendation for mammography for this patient? Answer Choices 1 Immediately 2 When she is 30 years old 3 When she is 32 years old 4 When she is 34 years old 5 When she is 36 years old

Explanation The correct response is immediately. Breast cancer is recognized as both an increasingly common disorder and a potentially hereditary disorder. Furthermore, early detection will provide a woman with her best chance at a cure of breast cancer. Given the patient's genetic burden of having both her mother and her grandmother die of metastatic breast cancer at the relatively young age of 32 years, it is reasonable to provide her with immediate and periodic mammography combined with instruction in breast self-examination. Women at high risk require a more aggressive schedule, frequent examinations, and other procedures such as ultrasound and MRI. The American Cancer Society (ACS) recommends annual mammography beginning at age 40 in average risk women in the absence of any positive family history. The American College of Obstetricians and Gynecologists (ACOG) and the American Medical Association (AMA) recommend mammography every 1 - 2 years in women age 40 to 49 years, and annually after the age of 50. Screening mammography annually or biennially significantly reduces the mortality from breast cancer. A prudent provider will recognize and explain that in average risk individuals, the risk of a false positive mammogram increases as the age of the patient decreases.

A 19-year-old man has known type I diabetes on insulin; he is brought to the ER by his family with a history of clouding of consciousness, vomiting, shortness of breath, and weakness. Symptoms have present since the morning. He also has a 2-day history of cough with productive sputum and fever. He is known to be non-compliant with insulin and diet and has had multiple ER visits. At home, his mother checked his blood sugar on his glucometer, which reported a "high level", signifying more than 500 mg/dl. On examination he is found to have a temperature of 101.5° F, pulse 110/min, BP 100/58 mm Hg, and respirations of 24/ minute. Skin and mucosa are very dry. Auscultation reveals crackles at the base of the right lung, tachycardia, and normal bowel sounds. He is drowsy but arousable and oriented. Labs are ordered, and supportive measures are started with fluids and insulin. Question The labs for the above patient show a blood sugar of 950 mg/dl, sodium 129 meq/L, chloride 91 meq/L, potassium 3.2 meq/L, BUN 35 mg/dL, creatinine 1.5 mg/dL, bicarbonate of 18 meq/L, anion gap 20, WBC 13,000/uL, serum ketones positive, and serum osmolality 300 mosm/kg. CXR shows right lower lobe consolidation and EKG shows tachycardia only. The best combination for the correct management of this patient includes fluids and what else? Answer Choices 1 SC insulin every 4 hours, potassium, bicarbonate, and antibiotics 2 Insulin drip, potassium, and antibiotics 3 Bolus insulin IV then SC every 4 hours, potassium, and antibiotics 4 IV insulin every 4 hours, potassium, and antibiotics 5 SC insulin every 2 hours, potassium, and antibiotics

Explanation The correct response is insulin drip, potassium and antibiotics. Therapy for diabetic ketoacidosis includes 4 components: fluids, insulin, electrolyte replacement, and treatment of precipitating factor. The average fluid deficit in such patients is 3 - 6 liters. Replacement is done aggressively, initially with normal saline, then with half normal saline after a few hours. However, too rapid hydration may cause cerebral edema, so should be avoided. For patients in shock, as this one, intravenous insulin is recommended as a bolus of 15 - 20 units followed by infusion of 0.1 units/kg/hr or approximately 5 - 10 units per hour. Subcutaneous insulin is not recommended in patients in shock, since poor perfusion impairs the absorption of insulin. Once the blood sugar falls to 200 - 250 mg/dl and acidosis is resolved, standard insulin therapy is started. Frequent labs and bedside blood sugar testing are done, including the complete metabolic panel and arterial blood gases. Potassium, phosphate and bicarbonate are replaced as needed. Insulin causes metabolism of ketones and regeneration of bicarbonate, so bicarbonate is given only if arterial pH goes below 7.1 or there is life-threatening hyperkalemia, since bicarbonate drives the potassium into the cells. Antibiotics are given for the underlying pneumonia in this case.

A 52-year-old Caucasian woman who has a longstanding history of Addison's disease is scheduled for an elective cardiothoracic surgery in 4 weeks. The planned surgical procedure is expected to produce significant surgical and medical stress. The patient currently takes hydrocortisone 21 mg PO daily in 2 doses; 7 mg is taken in the morning, and 14 mg is taken in the evening. Question What would be the most logical plan of action in regards to her hydrocortisone perioperatively? Answer Choices 1 Keep at the current amount of daily replacement dose 2 Stop hydrocortisone 2 days before the surgery, and restart the daily replacement dose 2 days after the procedure 3 Double the daily replacement dose 2 days before the surgery 4 Intravenous hydrocortisone 75 mg/daily on day of procedure, then taper over next 1 - 2 days to the patient's usual outpatient daily replacement dose 5 Intravenous hydrocortisone 150 mg/daily on day of procedure, then taper over next 2 - 3 days to the patient's usual outpatient daily replacement dose

Explanation The correct response is intravenous hydrocortisone 150 mg/daily on day of procedure, with a taper over next 2 - 3 days to usual daily replacement dose if the postoperative period is otherwise uncomplicated. Cortisol has many critical metabolic and endocrine functions that are necessary for human survival, particularly during times of stress. Everyday health and illness, surgery, anesthesia, trauma, and severe illnesses result in elevated plasma ACTH and cortisol levels. Those that are adrenally insufficient (such as patients diagnosed with Addison's disease) have very little or no function of this type of endocrine function, thereby predisposing them to inappropriate body reactions to stress that can result in devastating morbidities. External corticosteroids are prescribed to these patients to take on a daily basis. For this reason, patients with any type of adrenal insufficiency must have an appropriate treatment plan in regards to their corticosteroid therapy in order for them to produce an appropriate immune response to the stress of surgery. There is no universally accepted regimen for corticosteroid supplementation during the surgical and postoperative time period. A recommended dosing plan should be based on the severity of the surgery stress as well as the medical stress that the patient will most likely endure intra- and post-operatively. Cardiothoracic surgery and its postoperative time period are categorized as severe in terms of the effects on the body. For this reason, the most appropriate plan would be Intravenous hydrocortisone 150 mg/daily on day of procedure, with a taper over next 2 - 3 days to usual daily replacement dose if the postoperative period is otherwise uncomplicated.

Your patient in the labor and delivery unit is a 26-year-old primigravida; she is at 36 weeks of gestation. She is experiencing fever and abdominal tenderness. Her pregnancy has been normal; however, 2 days ago she noticed clear fluid vaginal leaking. There are no changes in fetal movements, and there has been no vaginal bleeding. Today her temperature is 38.5 C, blood pressure is 120/80 mm Hg, heart rate is 92/min, and respirations are 18/min. Fetal heart rate is 160/min (tachycardia), with good variability. Contractions are registered every 10 minutes. On examination her abdomen is diffusely tender, the cervix is closed, and there is turbid vaginal pooling that shows an elevated pH and a ferning pattern. Question After administering antibiotics, what will be your next step in the management of this patient? Answer Choices 1 Cesarean section 2 Oxytocin 3 Magnesium sulfate 4 Indomethacin 5 Steroids

Explanation The correct response is oxytocin. Your patient most probably has chorioamnionitis secondary to the prolonged rupture of membranes. Delivery is indicated for the benefit of both mother and fetus. Her cervix is closed; therefore, the induction of labor is the best next step in the management. A cesarean section is not indicated in this case; instead, you should try vaginal delivery. You should only consider a cesarean section if there is significant maternal and/or fetal distress and a vaginal delivery is remote. Magnesium sulfate is not indicated; your patient has chorioamnionitis, and you do not want tocolysis. Indomethacin is not indicated; you do not want to delay the labor. Steroids (e.g., betamethasone) are used to improve fetal lung function and are given up to 34 weeks of gestation (the greatest benefit is at less than 32 weeks of gestation). Your patient is at 36 weeks of gestation; therefore, steroids are not indicated.

A 36-year-old woman presents for fertility counseling. She and her husband have been trying to conceive for over 1 year, during which time her periods have been irregular. Her last menses was 4 months ago. Serum hCG is negative. Serum levels of LH and FSH are elevated, and serum estradiol is low. What is the most likely diagnosis? Answer Choices 1 Tubal obstruction 2 Polycystic ovarian disease 3 Endometriosis 4 Premature menopause 5 Panhypopituitarism

Explanation The correct response is premature menopause. Low levels of estradiol associated with high levels of gonadotropic hormones indicate a functioning pituitary, therefore excluding panhypopituitarism, and primary ovarian failure. Average age of menopause in the United States remains at age 52, with a perimenopausal period averaging 5 years. Tubal obstruction, commonly the result of endometriosis, will result in infertility, but it does not affect menstruation.

A 49-year-old Caucasian man who is well known to your practice presents due to his history of hypertriglyceridemia. He has come in to see you today for evaluation of his recent cholesterol lab values. He has a significant family history of cardiovascular disease; his mother had a heart attack at the age of 57, and his father had open heart surgery at the age of 60. The patient has had low HDL levels in several past cholesterol screening tests. At the last office visit, lifestyle modifications were implemented by the patient. He has been extremely conscientious about his diet; for the past year, he has also been participating in physical activity 6 days a week. In addition, he has significantly limited his alcohol intake. Current lab values for the patient are as follows: total cholesterol of 235 mg/dL, triglycerides of 350 mg/dL, HDL of 35, and a LDL of 275 mg/dL. You decide to initiate pharmacologic therapy in order to treat the patient's cholesterol and triglyceride levels. Question What medication should you prescribe? Answer Choices 1 Simvastatin 2 Ezetimibe 3 Colestipol 4 Colesevelam 5 Cholestyramine

Explanation The correct response is simvastatin. Most patients with high cholesterol levels present with no specific symptoms or signs, although this is a significant contributor to the number 1 killer of adults in the United States, which is cardiovascular disease. There are a multitude of modifiable as well as non-modifiable risk factors present that contribute to overall cholesterol levels. If a patient has any abnormal levels of cholesterol levels, the initial component of any treatment is appropriate patient education with lifestyle modifications, which many times include limiting consumption of alcohol and cholesterol-containing foods, smoking cessation, and increasing physical activity. If a valiant effort of modifiable risk factors is taken and fails to bring cholesterol levels to normal ranges, pharmaceutical intervention should be initiated. The patient described in the scenario above would most benefit from the initiation of simvastatin. This medication has been found to significantly decrease triglycerides; it also causes up to a 40% reduction of LDLs and up to a 10% increase of HDL levels. Financially, it is on the lower end of the spectrum in terms of price per dose. Ezetimibe, cholestyramine, colestipol, and colesevelam are not good choices due to the fact that they have a varying effect on triglyceride levels, which is one of this patient's major abnormalities.

Your patient is a 42-year-old woman with a 1-year history of amenorrhea and galactorrhea. She also has been experiencing fatigue, somnolence, and easy bruising. She takes Metformin for type II diabetes and an ACE inhibitor for hypertension. On examination, you find an obese woman with peripheral edema (ankles and hands), hirsutism, and diabetic retinopathy; the rest of the examination is normal. Her prolactin levels are 144.8 ng/mL (normal range is 4.8 to 23.3 ng/mL); repeated prolactin is 110 ng/mL. TSH and free T4 are normal. Question What should be your next step in regards to making a diagnosis? Answer Choices 1 Obtain serum testosterone levels 2 Perform kidney function tests 3 Schedule pituitary MRI 4 Perform visual field testing 5 Evaluate coagulation status

Explanation The correct response is that you should perform kidney function tests. Your patient has galactorrhea, amenorrhea, and signs and symptoms that suggest a renal insufficiency (fatigue, somnolence, easy bruising, peripheral edema); the insufficiency was probably caused by long-standing diabetes (diabetic retinopathy develops after a long history of diabetes). Chronic renal failure elevates prolactin by decreasing peripheral clearance of the hormone. Evaluation of hyperprolactinemia should include a review of medications, including estrogen therapy, and history of fertility or gonadal dysfunction. Elevated prolactin levels can result in secondary hypogonadism. In general, signs and symptoms of hyperprolactinemia are due to either the excess hormone secretion (i.e., galactorrhea and amenorrhea) or local compression (e.g., new-onset or persistent headache, dizziness, visual changes, and vision loss). Since your patient has no signs of any of them, biochemical tests should be ordered before the imaging. Laboratory evaluation should include a repeat serum prolactin test, measurements of TSH and free T4, and a pregnancy test. If the results come back normal and if other diagnoses are excluded, the most likely diagnosis is a prolactinoma. A pituitary MRI should only be obtained in such cases. Elevated prolactin levels can result in secondary hypogonadism. Serum testosterone levels should be checked in men with galactorrhea. Visual field testing can be performed in individuals with specific visual complaints, especially loss or impairment of peripheral vision.

A 33-year-old woman presents with fatigue, weight gain, and non-specific aches and pains for the past year or so. She denies fever, cough, abdominal pain, dysuria, or visual changes, but she has had constipation recently. She feels dizzy on and off, and she grows increasingly tired with each passing day. Her weight gain of 20 pounds or so in the last year has made her depressed and tired. Her best friend repeatedly tells her that her voice is deeper and raspier on the phone. After further questioning, she claims that she has had quarrels with her husband and children regarding the thermostat in her house. She would like to keep it at 80°F, and they want it at 60°F in this month of May. On examination, she has a temperature of 96°F, pulse 56/min, and BP 110/70 mm of Hg. She is 5'3" tall, and her weight is 155 pounds. Lungs are clear, abdomen shows no abnormal findings, and heart sounds are regular. Her skin appears somewhat dry and scaly. Her hair is coarse. You order tests. Question What is the most important test at this point? Answer Choices 1 Metabolic panel 2 Complete blood count and peripheral smear 3 Thyroid stimulating hormone 4 FSH and LH 5 Lipid profile

Explanation The correct response is thyroid-stimulating hormone. This woman has hypothyroidism. She has the classic symptoms, which are weight gain, depression, constipation, myalgias, cold intolerance, fatigue, deep voice, bradycardia, and dry skin and hair. It is a very common condition that is seen mostly in middle aged and elderly women; however, it can also be seen in young patients. 1.5 - 2% women above 60 years and up to 5 - 15% women above 65 years of age may be affected. Hashimoto's thyroiditis is an autoimmune condition, and it is the most common cause of hypothyroidism in the western world, whereas iodine deficiency is the etiological factor in the developing countries. Hashimoto's thyroiditis may present as hypothyroidism, goiter, silent thyroiditis, or post-partum thyroiditis. Other symptoms of hypothyroidism include puffy eyes from periorbital edema, shortness of breath, exercise intolerance, decreased appetite, somnolence, blurred vision, and decreased sweating. Anemia may be present, which can be normocytic normochromic due to decreased red cell mass or, in 10% of cases, autoimmune thyroiditis. Macrocytic anemia due to pernicious anemia may be seen. Menstrual irregularities are common and could be amenorrhea, oligomenorrhea, or menorrhagia. Infertility and early pregnancy loss may be seen. Muscle weakness and carpal tunnel syndrome may occur. TSH will be high and T3, T4 low. TSH is the earliest and most sensitive indicator of primary hypothyroidism. Sensitive third generation TSH assays are now available for easy diagnosis. Anti-thyroid peroxidase (TPO) antibodies and antibodies to thyroglobulin may also be done. Treatment is by replacement of thyroxine for life, since it is a permanent condition. The recommended replacement dose for an average 70 kg adult is 1.6 mcg/kg/day, but the dose may vary from 50-200 mcg daily. In the elderly, it is a good idea to start at a lower dose, since thyroxine can aggravate coronary ischemia. It should be taken in the morning on an empty stomach. Metabolic panel is non-diagnostic, and so is a complete blood count. Anemia may be present occasionally. FSH and LH are done when perimenopause is suspected. Some of her symptoms, such as depression, myalgias, and fatigue, are also seen in perimenopausal women; however, weight gain, cold intolerance, and bradycardia are not seen. Lipid profile is a screening test for adults and should be done in this patient also. It is not a diagnostic indicator of the underlying condition, which is hypothyroidism.

A 23-year-old presents with frequent, painful urination and lower back pain. History and physical exam reveals suprapubic pain, no evidence of systemic illness or fever, and no history of hospitalization. A presumptive diagnosis of an urinary tract infection (UTI) is made, and a urine culture yields lactose-fermenting beta-hemolytic Gram-negative rods. What is the most likely etiology of the UTI? Answer Choices 1 Escherichia coli 2 Klebsiella pneumoniae 3 Enterobacter spp. 4 Proteus mirabilis 5 Pseudomonas sp.

Explanation The most common cause of an uncomplicated UTI is Escherichia coli, responsible for up to 85% of outpatient cases. Proteus sp. and Pseudomonas sp. are most often the etiologic agents of UTIs in hospitalized patients. Pseudomonas does not ferment lactose. Klebsiella pneumoniae and Enterobacter spp. are rarely beta-hemolytic.

A 24-year-old woman presents with severe diarrhea that she has been experiencing for 3 days, with no medical complaints before then. She now feels dizzy upon standing, her tongue is dry, and her eyes appear glazed. Her serum sodium concentration is 130 mEq/L. Which of the following findings is most likely? Answer Choices 1 Decreased serum aldosterone 2 Increased serum atrial natriuretic peptide 3 Increased intravascular volume 4 Increased serum ADH concentration 5 Urine osmolality less than serum osmolality

Explanation The patient has obvious symptoms of dehydration. She also has a low serum sodium concentration, which shows that the body's drive to conserve water supersedes that of maintaining an adequate sodium concentration. ADH makes the normally impermeable collecting tubules permeable to water. Hence ADH promotes water reabsorption in the collecting tubules, so urine volume is decreased, and urine osmolality is increased.The patient's condition can be readily explained in view of an increased level of serum ADH. Serum aldosterone will be increased, as the body will also be trying to conserve sodium. Atrial natriuretic peptide will be decreased for the same reason.

A 32-year-old man presents with multiple symptoms. He states that his feet have become wider, requiring him to purchase shoes in a larger size. His hands have also increased in size, and a friend recently noted that his handshake felt doughy. His facial features have become coarse, and his brow also appears abnormal (as seen in the image). Question What is the most common cause of the patient's condition? Answer Choices 1 MEN-1 syndrome 2 Pituitary tumor 3 Pancreatic tumor 4 Lymphoma 5 Hypothalamic tumor

Explanation The patient most likely has a pituitary tumor. The patient is suffering from acromegaly, the excess secretion of growth hormone (GH). GH typically stimulates growth, and when produced in excess amounts, causes abnormal growth of the soft tissue and skeleton. As a result, patients develop symptoms of increased hand and foot size, a prominent brow, and coarse facial features. Most cases of acromegaly are caused by a pituitary adenoma that secretes excessive amounts of GH. MEN-1 syndrome, pancreatic tumors, lymphoma,and hypothalamic tumors may all cause acromegaly, but they are not the most common causes.

A 58-year-old man presents to discuss his recent annual screening blood work results. His past medical history includes obesity and hypertension. He wants to discuss whether he has additional risk factors for Coronary Heart Disease (CHD). His Total Cholesterol is 240mg/dL, his Low-Density Lipoprotein Cholesterol (LDL) is 191mg/dL, his High-Density Lipoprotein Cholesterol (HDL) is 32mg/dL, and his Triglycerides are 140mg/dL. Question What is the most likely diagnosis? Answer Choices 1 Hypertriglyceridemia with high risk for CHD 2 Normal cholesterol profile with low risk for CHD 3 Hypercholesterolemia with low risk for CHD 4 Hypercholesterolemia with high risk for CHD 5 Hypertriglyceridemia with low risk for CHD

Explanation The patient would be diagnosed with hypercholesterolemia with high risk for Coronary Heart Disease (CHD) for several reasons. His total cholesterol is considered elevated, as it is more than 200mg/dL, and his Low-Density Lipoprotein Cholesterol (LDL) is also elevated, as it is more than 160mg/dL. In addition, his High-Density Lipoprotein Cholesterol (HDL) is less than 35mg/dL, which increases his risk for CHD. In order to be diagnosed with hypercholesterolemia with low risk for CHD, the patient would essentially have to have total cholesterol and LDL levels that are elevated, but not to the extent that his are. In particular, an LDL level above 100mg/dL, but still under 160mg/dL, is considered high, but does not have the same risk as a level that is over 160mg/dL, as his is. In addition, he could have lower risk for CHD had his HDL level been over 60mg/dL. The patient would not be diagnosed with hypertriglyceridemia, with or without risk for CHD, because his triglycerides are actually within normal limits. Triglyceride levels under 200mg/dL are considered desirable. Levels between 200 and 400mg/dL are considered borderline elevated, and levels over 400mg/dL are considered high. The patient would definitely not be diagnosed with a normal cholesterol profile. A normal cholesterol profile would include a total cholesterol under 200mg/dL, an HDL level over 60mg/dL, an LDL level under 130mg/dL, and a triglyceride level under 200mg/dL. Of the 4 serum levels, he only has a normal triglyceride level. The rest are abnormal and thus increase his risk for CHD.

A 45-year-old man presents with what he refers to as "anxiety attacks". He describes them as recurring bouts of palpitations, headaches, anxiety, and sweating. One evening, while having dinner with his wife, his wife witnesses an episode. She is concerned and immediately takes his blood pressure. His blood pressure is 195/105 mmHg. The next day, his wife takes him to see their family doctor. On further questioning, he comments that he sometimes gets light headed when he stands up too rapidly. He comments that his mother had similar problems. On physical examination, his blood pressure is 165/90 mmHg and his heart rate is 80 beats/min. A 24-hour collection of his urine test is positive for vanillylmandelic acid. Question What is the most likely diagnosis? Answer Choices 1 Pheochromocytoma 2 Essential hypertension 3 Cushing's syndrome 4 Renal artery stenosis 5 Phenylketonuria

Explanation This patient exhibited signs and symptoms of pheochromocytoma. He has paroxysmal hypertension and paroxysmal episodes of sweating and anxiety. The paroxysmal episodes that he experiences are due to intermittent release of catecholamines from the pheochromocytoma. Postural hypotension can sometimes be seen with pheochromocytoma. Urinary vanillylmandelic acid (VMA) is a metabolite of catecholamines and is found in pheochromocytoma. Essential hypertension, Cushing's syndrome, and renal artery stenosis can all cause hypertension; however, they would not cause paroxysmal hypertension, as this patient had. In addition, VMA would not be found in the urine. Phenylketonuria is not a cause of hypertension. It is a metabolic disease that results in problems processing phenylalanine.

A 32-year-old woman presents with a solitary thyroid nodule. A biopsy confirms that she has thyroid cancer. She subsequently has a thyroidectomy. However, during the procedure, the surgeon accidentally removes most of her parathyroid glands as well. As a consequence, she develops hypoparathyroidism. Her laboratory results are in the chart. What symptom would be consistent with her history and findings? TEST RESULTS REFERENCE RANGE RBC 4.8 x 106/µl 4.5 - 5.7 x 106/µl (male) 3.9 - 5.0 x 106/µl (female) Hematocrit 42 40 - 50 (male) 36 - 44 (female) Hemoglobin 15 gm/dL 13.8 - 17.2 gm/dL (male) 12.1 - 15.1 gm/dL (female) Platelets 252,000/mm3 130,000 - 400,000/mm3 Potassium 4.1 mEq/L 3.5 - 5.0 mEq/L Sodium 144 mEq/L 135 - 145 mEq/L Calcium 7.8 mg/dL 8.5 - 10.5 mg/dL Question What is a sign of her secondary condition? Answer Choices 1 Chvostek's sign 2 Kernig's sign 3 Babinski's sign 4 Kussmaul's sign 5 Quincke's sign

Explanation This patient has hypocalcemia secondary to hypoparathyroidism. Paresthesias can be seen with hypocalcemia due to the increase in neuromuscular irritability. Common sites for the paresthesias are around the mouth and on the fingertips. Tetany can also be seen. Chvostek's sign can be seen with tetany. When tapping on the facial nerve produces contraction on that side of the face, it is called Chvostek's sign. Kernig's sign is a sign of meningeal irritation. The patient lies on his back and his leg is raised and knee bent at a 90-degree angle. When the examiner straightens the knee, if there is pain or resistance to further extension, Kernig's sign is said to be positive. Babinski's sign is an abnormal response to stimulation on the sole of the foot; it results in dorsiflexion of the big toe and fanning of the other toes. Babinski's sign is seen with pyramidal disease. Kussmaul's sign is an increase in venous pressure during inspiration; it can be seen with cardiac tamponade. Quincke's sign (Quincke's pulse) is an alteration seen in nails with each heartbeat. Quincke's sign can be seen with aortic insufficiency.

A 62-year-old woman presents to her OB/GYN clinic for an annual exam. She reports she has not felt well for several months, but her family practice physician retired, and she kept hoping her symptoms would improve. She reports stretch marks and a weight gain of about 20 pounds, which is especially noticeable in her face. She feels fatigued; she has no sex drive, and she feels more irritable. She notes increased dark hairs growing on her chin. The patient reports easy bruising, but she denies blood in the stools. She denies fever, chills, foreign travel, changes in sexual partners, and suicidal thoughts. Her past medical history is significant for asthma, with periodic hospitalizations for asthma attacks. She was menopausal at the age of 51. She has had no surgeries. She takes prednisone 40 mg daily, an inhaler with fluticasone propionate/salmeterol 2 times daily, and an albuterol inhaler as needed for her asthma. She has no allergies. She is retired; she lives at home with her husband, and she denies use of tobacco, alcohol, and street drugs. BP is 162/92 mm Hg, and pulse is 98. Respiratory rate is 12. Weight is 170 lbs, and height is 63", making her BMI 30.11. On physical exam, the patient has a rounded, full face; she appears fatigued, but she is pleasant and cooperative. Striae are noted on her abdomen, and hirsutism is present on her chin and abdomen. There appears to be a large fat deposit on the upper back, and she has an obese abdomen, with some muscle wasting on the patient's extremities. The remainder of her exam is normal. Question What is the most appropriate intervention for this patient's current condition? Answer Choices 1 Counsel the patient about normal post-menopausal symptoms 2 Perform dexamethasone suppression test 3 Refer to dietitian for weight loss 4 Stop the inhaled medications 5 Taper off the daily prednisone

Explanation This patient is presenting with Cushing syndrome, an excess of glucocorticoids; it is most likely iatrogenic, caused by her daily prednisone. The most appropriate intervention is to taper off her daily prednisone. This common steroid is often given for acute flares of chronic diseases. However, chronic daily use can lead to a glucocorticoid excess known as Cushing syndrome. Cushing syndrome is characterized by central obesity, muscle wasting, hirsutism, a 'buffalo hump' on the upper back, 'moon facies', weight gain, hypertension, decreased libido, irritability, and several other possible signs/symptoms. Unrecognized and untreated Cushing syndrome can lead to osteoporosis. Tapering off the prednisone (rather than immediate discontinuation) helps prevent sudden adrenocortical insufficiency. While it would be acceptable to counsel the patient about normal post-menopausal symptoms at her GYN annual exam, her current condition cannot be attributed to normal menopausal changes. Normal post-menopausal changes include vasomotor symptoms (such as hot flashes and night sweats) and vulvovaginal changes (such as atrophy, vaginal dryness and dyspareunia). Some women may experience some mild weight gain and irritability, but the remainder of this patient's symptoms are not related to a drop in sex hormones. When Cushing syndrome is suspected, it is common to perform a dexamethasone suppression test. The dexamethasone is an exogenous glucocorticoid that will negatively feedback on the pituitary and should lower the adrenocorticotropic hormone and result in lowered cortisol levels in the serum. However, while the patient is taking daily corticosteroids, the test will be altered. This patient should be tapered off the prednisone, and if symptoms persist, a dexamethasone test can be done. This patient's concerns should not be attributed to a weight-gain only issue and simply referred to a dietitian for weight loss. Weight gain is a common side effect from exogenous corticosteroids and should reverse with discontinuation of the medication. A dietitian referral could be offered in addition to treatment of this patient's Cushing syndrome, but it should not be the primary treatment. This patient's inhaled fluticasone propionate is also a steroid medication, and it is given to reduce inflammation in the airways. Typically, the inhaled and intranasal steroids at usual doses are considered low enough strength that they will not lead to corticosteroid excess and Cushing syndrome. The systemic steroid, prednisone, is much more potent and is the likely culprit for this patient's symptoms. The albuterol inhaler is a short-acting beta-agonist and should not be discontinued in patients with severe asthma; therefore, it would be inappropriate to stop this patient's inhaled medications.

A 35-year-old man with no significant past medical history presents with multiple skin lesions in various parts of his body. These evolved gradually over the past year. He describes them as small, painless, non-pruritic, and red-yellow, as well as flesh-toned. They are distributed on his buttocks, posterior ankles, and on his knees. There is no discharge to these lesions. He denied any allergies, trauma, history of surgeries, or instrumentation. He further denied recent fever, chills, travel, insect bites, sick contacts, chest pain, cough, abdominal pain, nausea, vomiting, diarrhea, edema, or other rashes. The physical examination was notable for an overweight man with multiple red-yellow papules on the buttocks. Additionally, there were several flesh-toned, firm, nodular lesions distributed over the bilateral Achilles and patellar tendons. All skin lesions were nontender and without discharge. The remainder of the physical exam was unremarkable. Question What is the most likely diagnosis? Answer Choices 1 Hyperlipidemia 2 Squamous cell carcinoma 3 Folliculitis 4 Psoriasis 5 Nummular eczema

Explanation This patient is presenting with hyperlipidemia. Extremely high levels of chylomicrons or VLDL particles (triglyceride level above 1000 mg/dL) result in the formation of eruptive xanthomas. These lesions are described as red-yellow papules, which are commonly found on the buttocks. High LDL concentrations result in tendinous xanthomas on certain tendons, such as the Achilles, patellar, and those on the back of the hand. The lesions associated with squamous cell carcinoma occur primarily in chronic sun-exposed areas and appear as small red, conical, hard nodules that occasionally ulcerate without healing. Folliculitis typically presents as pustules of hair follicles, with symptoms ranging from slight burning and tenderness to intense itching. Psoriatic lesions may be pruritic and are most commonly found on the scalp, elbows, knees, palms, and soles. They are red, sharply defined plaques covered with silvery scales. Pitting and onycholysis are common nail findings. Nummular eczema is a chronic, pruritic inflammatory dermatitis occurring as coin-shaped plaques composed of grouped small papules and vesicles on an erythematous base, which is common on the lower legs. These are seen in atopic individuals.

A 28-year-old woman with a 4-year history of end stage renal disease secondary to diabetes has had intact parathyroid hormone (PTH) levels of 600-800pg/ml (ref 150-300 pg/ml) for the past 15 months. She does not take prescribed phosphorus binders with meals; she routinely misses her dialysis treatments, during which she receives intravenous paricalcitol. What radiologic findings are expected in this patient? Answer Choices 1 Normal DEXA scan 2 Cinnamon skull 3 Rugger jersey spine 4 Thumbprinting 5 Quasifractures

Explanation This patient will most likely have a 'rugger Jersey' appearance to her vertebrae if she has a back or skeletal X-ray. She has hyperparathyroidism. End stage kidney disease patients (stage 5 chronic kidney disease) should have PTH levels between 150 and 300 pg/ml. She is likely hyperphosphatemic because of impaired renal phosphorus excretion, inadequate dialysis, and failure to take phosphorus binders when she eats. Hyperphosphatemia stimulates PTH release. Vitamin D (i.e., paricalcitol) can suppress PTH release, but she may be missing adequate doses when she skips her dialysis sessions. Excess PTH leads to abnormal bone remodeling, including excessive resorption. Lucencies may be seen in poorly mineralized areas. 'Pepper pot skull' refers to such scattered lucent findings on skull films; 'cinnamon skull' is not a term used to describe the radiologic findings of hyperparathyroidism. 'Pseudofractures' are the straight lucent bands of poorly mineralized bone seen in hyperparathyroidism, particularly in long bones. Quasifractures are not radiologic findings seen in hyperparathyroidism. DEXA (Dual X-ray absorptiometry) scans are quantitative measures of bone mineralization. Low bone mineralization is inversely related to PTH levels (Brenner). This patient will not likely have a normal DEXA scan because her bones are diseased. DEXA is not a diagnostic test for renal bone disease; measures of PTH, together with calcium, phosphorus, and alkaline phosphatase levels, are better diagnostic tests for renal bone disease. Thumbprinting refers to bowel wall edema noted on CT or abdominal radiographs in certain conditions, such as mesenteric ischemia (Harrison's ch. 279) and toxic megacolon. Ischemia often requires surgical intervention. Toxic megacolon is treated surgically and with antibiotics.

A 52-year-old woman presents to her primary care physician with weight gain and poor wound healing. Examination reveals round face, central obesity with muscle wasting, striate, and hypertension. Imaging reveals a pituitary adenoma. Question What is the recommended treatment? Answer Choices 1 Hydrocortisone 2 Prednisone 3 Transsphenoidal adenoma resection 4 Pituitary radiosurgery 5 Adrenalectomy

Explanation Transsphenoidal adenoma resection is correct. The patient is showing signs of Cushing syndrome (hypercortisolism). Cushing syndrome is a term used to describe the effects of excessive corticosteroid production. Cushing disease refers to hypercortisolism caused by excessive ACTH secretion. Most cases of Cushing disease are caused by a pituitary adenoma. The preferred treatment is transsphenoidal resection. Hydrocortisone and prednisone are incorrect. Both of these medications would worsen Cushing disease. Pituitary radiosurgery and adrenalectomy are both used in the treatment of Cushing disease, but transsphenoidal adenoma resection is the preferred treatment modality.

A 16-year-old boy presents for a physical. The physician notices a large amount of truncal fat and relatively thin limbs. What is the most likely diagnosis? Answer Choices 1 Turner's syndrome 2 Marfan's syndrome 3 Cushing's syndrome 4 Klinefelter's syndrome 5 Male hypogonadism

Explanation Truncal fat with relatively thin limbs is characteristic of Cushing's syndrome. Turner's syndrome is characterized by very short stature and webbed neck. People with Marfan's syndrome have very long limbs, fingers, and toes in proportion to the trunk. Klinefelter's syndrome is a chromosomal abnormality in which males have a eunuchoid habitus. Males with hypogonadism have long limbs in proportion to the trunk.

A 78-year-old woman is diagnosed with a duodenal neuroendocrine tumor. The attending surgeon, prior to undertaking surgery, reviews her past medical and surgical history. She underwent a total parathyroidectomy at the age of 55 for multiple hyperplastic parathyroid adenomas. Since then, she has been on calcium supplementation. She has no other comorbidities. Her younger brother had similar adenomas. CT scans are ordered to look for any metastases, and they reveal a small pituitary adenoma. She undergoes surgery and recovers with no complications. Question What diagnostic measure could have detected this in her youth? Answer Choices 1 Yearly serum calcitonin levels 2 BRCA mutation analysis 3 Wermer's syndrome genetic testing 4 RET protooncogene testing 5 Radio iodine uptake scanning

Explanation Wermer's syndrome genetic testing is the correct answer. Also known as Multiple Endocrine Neoplasia syndrome type 1 (MEN1), it is characterized by the occurrence of varying combinations of more than 20 endocrine and non-endocrine tumors. Endocrine tumors are represented mainly by the 'classic' P-triad originally described by Wermer: parathyroid, pituitary, and pancreatic tumors. Asymptomatic MEN1 mutant carriers need to be clearly recognized because the gene-related mutations confer a high risk of multiple primary cancers, which occur at younger ages, and they affect multiple family members who inherit the cancer-predisposing genetic mutation. A DNA test of the MEN1 gene detects mutations in 80 - 90% of familial MEN1 and in 65% of individuals with simplex MEN1. Yearly serum calcitonin levels is incorrect. Calcitonin levels help to diagnose Medullary carcinoma of the thyroid, but yearly levels are not an effective way of diagnosing MEN1 syndrome. In addition, it is not a preventive measure as Calcitonin levels are elevated only after the development of the malignancy. BRCA mutation analysis is incorrect. It is not associated with MEN1 syndrome. RET protooncogene testing is incorrect. It is diagnostic for MEN2a syndrome. Radioiodine uptake scanning is incorrect. In today's setting, its utility is mainly in the evaluation of hyperthyroidism and in identifying cold nodules.

A 3-year-old girl is brought to the clinic by her mother; she tells you that the child has not been eating well over the past month and has developed swelling in the abdomen. On exam, the child has a smooth abdominal mass that is the size of a baseball on the left side. Vital signs reveal a blood pressure of 134/82 mm Hg, temperature of 99.8 degrees F, and respirations of 16 breaths per minute. Urinalysis shows only 1+ red blood cells, and CBC and CMP are within normal limits. Question What is the most likely diagnosis? Answer Choices 1 Multicystic dysplastic kidney disease 2 Splenomegaly 3 Lymphoma 4 Wilms' tumor 5 Renal cell carcinoma

Explanation Wilm's tumors account for 95% of all urinary tract malignancies in childhood. The median age of diagnosis is 3 years. These tumors are usually a solitary, unilateral mass. Children typically present with an enlarging smooth abdominal mass confined to one side of the abdomen. Other associated symptoms include: abdominal pain, fever, hypertension (60%), and hematuria (25%). Multicystic dysplastic kidney disease is more common on the left side and is more common in boys. 60% of unilateral multicystic dysplastic kidneys involute in the first 3 years of life. Therefore, instead of getting bigger to reveal a palpable mass, these typically get smaller over time in children. Splenomegaly in children is usually indicative of hepatic or hematologic disease. This child's CBC and CMP are within normal limits, making this diagnosis unlikely. Lymphoma often presents with painless, firm lymphadenopathy typically confined to 1 or 2 lymph node areas (usually the supraclavicular and cervical nodes). Mediastinal lymphadenopathy is also a common presentation. It can manifest with coughing and shortness of breath. Abdominal masses are not a common presentation of lymphoma, and CBC is typically abnormal in these patients. Renal cell carcinoma accounts for only 3% of childhood urinary tract malignancies. Wilms' tumor is much more likely to be the diagnosis in a child.

A 2-year-old boy has a solid mass of tumor on the lower pole of his right kidney. Histopathological studies show whorls of mesenchymal cells and clusters of disorganized epithelial cells rather than renal corpuscles and renal tubules, which confirm the diagnosis of Wilms tumor. Question What is true regarding treatment/prognosis of this diagnosis? Answer Choices 1 If tumor penetrates kidney capsule, post-treatment survival is less than 50%. 2 Combination chemotherapy is appropriate after surgery. 3 Bilateral involvement is incurable. 4 There are no associated congenital abnormalities. 5 The tumor is not radiosensitive.

Explanation Wilms tumor is one of the earliest examples where adjuvant systematic chemotherapy and surgical reduction of the tumor is employed with successful outcome even for advanced disease. Even with metastases to liver and lungs, or with bilateral tumors, patients still have better than a 50% chance of survival. Some cases of Wilms tumor are associated with mutations in the WT-1 gene, a tumor suppressor gene that is also involved in the development of the urogenital system. Consequently, Wilms tumor is commonly associated with developmental defects in the gonads. The tumor is radiosensitive, and advanced cases can be treated with radiotherapy, albeit with a high complication rate in young children.

A 32-year-old woman in her 34th week of pregnancy comes to the labor floor after 2 - 3 hours of vaginal bleeding. Labor and vaginal delivery of her previous 3 pregnancies were normal. She is frightened, but oriented. While taking her medical history, what points are relevant to ask? Answer Choices 1 Cigarette use 2 The latest pap smear result 3 Recent sexual intercourse 4 Desire for future pregnancies 5 Recent vulvar pruritus

Explanation You should ask about recent sexual intercourse. The woman in this case exhibits 3rd-trimester bleeding, which is an obstetric emergency. History, physical exams, and lab test should be performed to assess maternal and fetal well-being. The differential diagnosis consists of placenta previa, abruptio placenta, and local vaginal or cervical bleeding from trauma. In medical history, you should ask about bleeding problems and sexual intercourse, as they can cause cervical trauma. In physical examinations, Leopold's Maneuvers outline the fetal lie and indicate fetal malposition. Postural hypotension suggests major uncompensated blood loss. After the history and physical exams have been completed, measurement of blood loss and urine output should be done. However, a type and cross match is an essential precaution. At this time, amniocentesis is appropriate to look for blood in the amniotic fluid and to determine fetal lung maturity. Internal examinations must not be performed in women suspected of having placenta previa, unless immediate delivery is required. The use of oxytocin challenge test in the face of 3rd-trimester bleeding is contraindicated. The best management for this woman would be careful monitoring of vaginal bleeding. Blood should be set up in the blood bank. Cesarean section may be required later, but not at this point. The use of Ritodrine to inhibit early labor is not indicated. Magnesium sulfate is used to control preeclampsia, however it is thought that magnesium sulfate can worsen bleeding in a woman with placenta previa by preventing blood coagulation.

A 35-year-old woman presents because of weight loss and palpitations. She lost 10 kg over 5 months despite having a good appetite. Her heart pounds and her hands tremble "all the time." She feels hot, is sweating profusely, and has difficulties going to and maintaining sleep; the slightest stimulus wakes her. Her job is suffering because of her nervousness, and her supervisor became concerned because she uses the bathroom 3 - 4 times a day in a need to move her bowels. She thinks that poor sleep quality and frequent bowel movements make her weak; she cannot climb stairs anymore and has to take a rest every 10 steps or so. Physical examination reveals a slim, anxious woman with pronounced stare, fine postural and tremor at rest, and slight proximal weakness. Her thyroid is diffusely enlarged and non-tender; her pulse is 100/min; and the rest of examination is within normal limits. Question What is the most likely diagnosis? Answer Choices 1 Hashimoto's thyroiditis 2 Toxic goiter 3 Grave's disease 4 Plummer's disease 5 Thyroid storm

Explanation Your patient has signs and symptoms suggestive of hyperthyreosis. The most common cause of hyperthyreosis is Grave's disease. It is an autoimmune disease caused by anti thyroid-stimulating hormone (TSH) receptor, can be familial, and is associated with other autoimmune diseases. Grave's disease typically presents with diffuse enlargement of the gland and exophthalmos. Exophthalmos is seen in about half of patients; it is a distinctive feature of Grave's disease and cannot be seen in other thyroid diseases. In the case of inflammation, as in Hashimoto's thyroiditis, the gland will be tender on palpation and there will be no exophthalmos. Patients with toxic goiter usually have a history of nontoxic enlargement of the gland and no exophthalmos. Plummer's disease, or toxic multinodular goiter, is the second most common cause of hyperthyroidismin the US and the most common cause of hyperthyroidism in the developing world, where the population is iodine-deficient. Patients usually have a history of prior nontoxic goiter with multiple nodules in the gland and no exophthalmos. Thyroid storm is a medical emergency. Patients with thyroid storm usually have a history of untreated hyperthyroidism and stress (most commonly trauma and infection) as a trigger. Typical presentation is a hypermetabolic state with high fever, tachycardia, nausea, vomiting, tremulousness, agitation, and changes in mental state.

A 16-year-old boy is gripped with sudden abdominal pain at school and starts throwing up. The pain is initially central around the umbilicus without any radiation then becomes more vaguely centered in the right lower abdomen. He is brought to the ER where his vitals are: temperature 101.1 F, BP 110/72 mm Hg, pulse 106/min, and SPO2 92%. Chest is clear and abdominal exam reveals tenderness in the right lower quadrant with rebound and hypoactive bowel sounds. Labs show HB of 14g/dl, WBC 15500, and platelets 420,000/uL. Obstructive series shows non specific bowel dilatation. CT scan of the abdomen is ordered. What is the likely diagnosis? Answer Choices 1 Acute gastroenteritis 2 Acute cholecystitis 3 Acute appendicitis 4 Acute diverticulitis 5 Incarcerated hernia

Explanation Acute appendicitis is a common cause of acute abdominal pain. It can occur at any age and is caused by the obstruction of the lumen of the appendix by foreign body, inflammation, fecolith, or tumor. Increased intraluminal pressure causes necrosis and perforation with signs of localized peritonitis. Pain starting in the umbilical area and later migrating to the right lower quadrant at McBurney's point is classic. Fever, vomiting, diarrhea, and leukocytosis support the diagnosis. CT scan in adults and abdominal ultrasound in children are diagnostic, and treatment is immediate surgical removal. In a female patient, endometriosis, ovarian cyst, and ovarian torsion are the other differential diagnoses. Acute gastroenteritis can mimic appendicitis but watery diarrhea is usually the predominant symptom with vomiting in some infections. Abdominal pain is diffuse and crampy and is not localized to McBurney's point. Fever and leukocytosis may be present. CT scan rules out localized pathology and is necessary only in limited cases. Treatment is supportive, since it is a self-limiting condition. Acute cholecystitis is typically characterized by right upper quadrant pain with radiation to the back, nausea and vomiting, fever, and right upper quadrant tenderness, also known as Murphy's sign. Jaundice may be present occasionally, with elevated liver enzymes and leukocytosis. It is caused by inflammation of the gall bladder wall, usually with one or multiple gallstones. Infection within the biliary system also plays a role in acute cholecystitis. Ultrasound reveals gallstones in most cases. Gall bladder wall thickening more than 4-5 mm, edema (causing double wall sign), and sonographic Murphy's sign (tenderness in the right upper quadrant while palpation with the ultrasound transducer) are also seen in the ultrasound. Patients need hospitalization for proper management. Treatment is conservative with hydration, pain control, bowel rest, and correction of any electrolyte disturbances. Antibiotics are not recommended as a rule but are generally given. Cholecystectomy is recommended for empyema, gangrene, rupture of the gall bladder, recurrent disease, or gallstones. Asymptomatic gallstones, however, are not treated surgically. Acute diverticulitis is usually left-sided and rarely right-sided. It presents with left lower quadrant pain, constipation or diarrhea, and left lower tenderness. Fever and mild leukocytosis are common but liver enzymes are normal. If right sided it is frequently mistaken for appendicitis. Diverticulitis can occur anywhere in the GI tract and may be mistaken for other common conditions. For example, diverticulitis of the transverse colon may be confused with cholecystitis, pancreatitis, and peptic ulcer disease. A tender mass may be present in a few cases. But it is very rare in less than 30 years of age and CT scan differentiates it from other conditions. Treatment is usually conservative in the first episode, with bowel rest and antibiotics. Colonic resection is advised in recurrent episode, complications like perforation, fistula or abscess, intractable symptoms, or suspicion of carcinoma. Incarcerated hernia presents with severe pain in the groin area, with a mass that may be purplish in color due to ischemia and gangrene. It should be suspected when a reducible hernia becomes painful and non-reducible. Treatment is surgical.

The most common cause of tender testicular swelling is: Answer Choices 1 Tuberculosis. 2 Hematocele. 3 Neoplasm. 4 Orchitis. 5 Hydrocele.

Explanation Acute orchitis is the most common cause of tender testicular swelling. It may result from mumps and other infectious diseases, and may be confused with spermatic cord torsion. Tuberculosis, hematoceles, neoplasms, hydroceles, chyloceles, and gummas all may produce nontender testicular swellings.

The mother of a 2-year-old Caucasian boy explains that the boy has had a 4-day history of fever and cries during urination. She has been giving him acetaminophen, but the fever keeps coming back. She also says that his urine "smells funny." Family history is positive for a 4-year-old brother with Down syndrome. His temperature is 39°C. Urinalysis and culture reveal urinary tract infection. A VCUG and a renal bladder ultrasonography reveal vesicourethral reflux (VUR). Question What is a risk factor for VUR in this boy? Answer Choices 1 His gender 2 Acetaminophen treatment 3 His age 4 His ethnicity 5 Family history of Down syndrome

Explanation Ethnicity is a significant risk factor for vesicourethral reflux (VUR) because it is 10 times more common in Caucasians than African-Americans. VUR is the retrograde flow of urine from the bladder towards the kidneys due to a dysfunctional vesicoureteric junction. It may occur alone or in association with other urological conditions like posterior urethral valves. Although prevalence is more common in male neonates, VUR is 5 times more common in female children compared to male children after the age of 1 year. Acetaminophen treatment and family history of Down syndrome have no association with VUR.

Which one of the following bacteria is carried by about 20% of women of child-bearing age and is a cause of sepsis and meningitis of the newborn? Answer Choices 1 Lactobacilli species 2 Bacteroides 3 Bifidobacterium 4 Group B streptococci 5 Clostridium

Explanation Group B streptococci are carried by about 20% of women of childbearing age in their vagina and are a cause of sepsis and meningitis of the newborn and are acquired during passage through the birth canal. Lactobacilli, Bacteroides, Bifidobacterium and Clostridium are normally found in the colon and do not cause sepsis and meningitis of the newborn.

A 56-year-old man presents for a routine follow-up regarding his positive HIV status. He is compliant with his medications and has been feeling well. In addition to his antivirals, he takes daily trimethoprim/sulfamethoxazole for Pneumocystis jiroveci pneumonia (PCP) prophylaxis. Question What detail of his history would warrant the prophylaxis? Answer Choices 1 CD4 cell count <400 2 History of previous PCP infection 3 CD4 cell count >200 4 History of any previous pneumonia infection 5 HIV viral load >100,000 copies/mL

Explanation History of previous Pneumocystis jiroveci pneumonia (PCP) infection is the correct answer. Patients who are HIV positive should take antibiotic prophylaxis for PCP if they have a history of PCP infection, if their CD4 cell count is below 200, or if they have evidence of immunodeficiency, such as oral candidiasis. Prophylaxis for PCP is typically daily trimethoprim/sulfamethoxazole double strength, but it can also be done with daily dapsone or monthly aerosolized pentamidine. If patients are on antiretroviral therapy and their CD4 cell count remains above 200 for at least 3 months, then they can stop prophylaxis. If their CD4 cell count drops below 200 again, they need to restart prophylaxis. CD4 cell count <400 is not the correct answer. Patients who are HIV positive need to be on PCP prophylaxis for various reasons, one of which is a CD4 count <200. A CD4 cell count between 200 and 400 would not warrant prophylaxis, until the point that the count drops below 200. CD4 cell count >200 is not the correct answer. A CD4 cell count over 200 would actually be a reason NOT to have the patient on PCP prophylaxis, as one of the criteria to start prophyhlaxis is a CD4 cell count below 200. In addition, when a patient has been on prophylaxis and antiretrovirals and their CD4 cell count is over 200 for 3 months, they can stop the prophylaxis. History of any pneumonia infection is not the correct answer. Only patients who have had previous PCP infections need to be treated with prophylaxis for PCP. Having had other types of pneumonia does not warrant PCP prophylaxis. HIV viral load >100,000 copies/mL is not the correct answer. HIV viral loads over 100,000 warrant antiretroviral treatment of HIV, but PCP prophylaxis is not given based on viral load. Rather, it is given based on CD4 cell count, previous PCP infection, or evidence of immunodeficiency.

A 14-year-old girl had primary amenorrhea, a webbed neck, a broad, shield-like chest with widely spaced nipples, and lack of secondary sexual characteristics. A karyotype of this patient is shown in Figure G3.6. The MOST likely diagnosis is which of the following? Answer Choices 1 Turner syndrome 2 hermaphroditism 3 androgen-insensitivity syndrome 4 Down syndrome 5 Sheehan syndrome

Explanation In classic Turner syndrome (ovarian dysgenesis), there are only 45 chromosomes, 1 sex chromosome being absent (45, X). This syndrome includes bilateral ovarian agenesis (streak ovaries), infantile genitalia, short stature, short webbed neck, high arched palate, cardiovascular and renal defects (such as coarctation of the aorta and horseshoe kidney), and cubitus valgus (an outward deviation of the extended forearm).

A 16-year-old girl has had recent onset of painful menstrual periods. Age of menarche was at 12 years. Her periods were irregular for about 8-10 months, but have been fairly regular since then, with occasional mild crampy pains on the first 2 days of her period. She rates the pains now as being 8-9 on a scale of 10. Cramps will start about a half a day before the onset of her periods, worsen the following day, and then gradually subside over the next day. There is no change with bowel movements. She also describes having a headache, looser stools, mild nausea, and low back and thigh pain. Bleeding is moderate on the first 2 days. She is otherwise healthy without history of abdominal surgery. Privately she denies any sexual activity. She is an average student and has missed 1-2 days of school with each period for the past 3 months. She also denies any fever or dysuria. There is also no family history of gynecological problems. On exam she has diffuse midline lower abdominal tenderness with some mild soreness to lower back and thighs, bowel sounds are normal, and there is no rebound tenderness. An external genital exam is normal. Based on this information, which of the following is the most likely cause of this girl's dysmenorrhea? Answer Choices 1 Primary dysmenorrhea 2 Endometriosis 3 Irritable bowel syndrome 4 Imperforate hymen 5 Ovarian cysts

Explanation Primary dysmenorrhea is associated with ovulatory cycles and typically occurs at an earlier age than secondary dysmenorrhea, as 1/3 of adolescents continue to experience anovulatory cycles in the first few years after menarche due to the time required for the hypothalamic-pituitary-ovarian cycle to mature. Disability is common with a reported 14% of girls reporting missing school or work because of pain and nearly 50% of those have pain described as moderate or severe. The physiologic basis relates to cell membrane phospholipids, endomyometrial prostaglandins, and leukotrienes. Defining symptoms include crampy midline lower abdominal pain beginning with menstrual flow or a short time before. As in this patient, cramps are typically intense on the first or second day and resolve before the end of the menstrual flow. The pain may be referred to the lower back and anterior thigh. Nausea or vomiting may occur as well as near-syncope or dizziness and complaints of weakness. Breast tenderness, bloating, headache, and mood changes can also be seen. Management involves the use of NSAIDs, which reduce the production of prostaglandins. Oral contraceptives should be considered for teens that have not experienced sufficient relief of symptoms with NSAIDs. Endometriosis occurs when functioning ectopic endometrial glands and stroma are present outside the uterine cavity, causing pain, irregular bleeding, and frequently infertility, though it may also be asymptomatic. The pelvic pain often cycles with menstruation and is associated with dysmenorrhea, dyspareunia, and infertility. The pain can be a deep constant ache with bilateral patterns of distribution and radiate to the buttock and perianal region. Pain may be associated with bladder or bowel function and there can be menstrual spotting. There is usually nonspecific pelvic tenderness on exam. Unlike this patient, it occurs more frequently in first-degree relatives of women whose endometriosis has been confirmed surgically. The pain of irritable bowel syndrome in adolescents is similar to that described in adults, with paroxysmal periumbilical abdominal pain that, unlike in this patient, is relieved by defecation or associated with an irregular pattern of defecation, including a change in frequency or consistency of stool, straining or urgency, feeling of incomplete evacuation, passage of mucus, and bloating or abdominal distention. Left lower quadrant fullness over the left colon may also be present. Periods of diarrhea alternate with periods of constipation. Imperforate hymen is recognized as a membrane covering the vaginal opening seen on external exam not evident in this patient. This may be bulging or a have a bluish hue from blood retained. There is a history of cyclic abdominal pain and a midline abdominal mass. A large enough mass may cause urinary obstruction. Ovarian cysts are fluid or semisolid filled sacs that develop on or within the ovary and are typically functional and disappear on their own. These develop from follicles that fail to rupture and release an egg and instead of being reabsorbed, the fluid within the follicle persists and forms a cyst. This may cause a sudden onset of pain that is constant or intermittent, dull or sharp aching pelvic pain that occurs, unlike the pattern seen in this patient, shortly after beginning or ending a menstrual period. There may be abnormal uterine bleeding, either longer, shorter or absent, abdominal bloating or distention, or no symptoms. Large or persistent cysts may require surgical removal.

A 17-year-old high school basketball player had been hospitalized with a throat infection, fever, and a question of pneumonia. She had been taking a number of antibiotics and her physician noted edema and an elevated blood pressure. Ten days after being discharged, she began to note blood in her urine. You order an urinalysis and red blood cells and protein are noted in her urine. What is the most likely diagnosis? Answer Choices 1 Chronic Renal Failure 2 Nephrolithiasis 3 Cystitis 4 Glomerulonephritis 5 Nephrotic Syndrome

Explanation The clinical picture is suggestive of glomerulonephritis. Signs and symptoms of glomerulonephritis include hematuria, proteinuria, edema, and hypertension usually occurring 7 to 10 days after the onset of acute pharyngitis. Chronic renal failure is most commonly caused by untreated or poorly-controlled diabetes mellitus and untreated or poorly-controlled hypertension. This is no indication of untreated or poorly-controlled diabetes mellitus and untreated or poorly-controlled hypertension in this patient. Nephrolithiasis, or kidney stones, would present with flank pain, ± fever, hematuria, and oliguria. There is no indication of flank pain or oliguria in this patient. Cystitis or bladder infection or inflammation would present with suprapubic pain, dysuria, nocturia, odd or foul smelling urine, an increase in urinary frequency, and no fever. On urinalysis, a cloudy appearance and white blood cells would be seen on microscopic examination. These symptoms are not present in this patient. Nephrotic syndrome presents with massive proteinuria, > 3.5g per 24 hour urine, hypoalbumenemia, edema, and hyperlipidemia. Oval fat bodies may be seen on urinalysis. These symptoms, with the exception of edema, are not present in this patient.

A 40-year-old woman presents to the emergency room with flank pain and fever of 102°F. Upon interview of the patient, she also noted that her urine output has decreased over the last few days. On further inquiry, the patient stated that she has been trying to lose weight by increasing her protein in her diet, exercising and decreasing her normal fluid intake. On urinalysis, red blood cells are noted. What is the most likely diagnosis? Answer Choices 1 Acute renal failure 2 Nephrolithiasis 3 Cystitis 4 Chronic renal failure 5 Nephritic syndrome

Explanation The clinical picture is suggestive of nephrolithiasis(renal stones). Most patients with renal stones present with flank pain and hematuria with or without fever. There are numerous causes of renal stones but dehydration favors stones formation and a high protein diet can predispose a patient to stone formation. Acute renal failure can be associated with renal stones. Renal stones are categorized as a postrenal cause of acute renal failure. But with acute renal failure, a rapid rise in BUN and creatinine would be seen on serum chemistries, which are not seen in this patient. Cystitis or bladder infection or inflammation would present with suprapubic pain, dysuria, nocturia, odd of foul smell to urine, and an increase in urinary frequency, and no fever. On urinalysis, a cloudy appearance and white blood cells would be seen. These symptoms are not present in this patient. Chronic renal failure is most commonly caused by untreated or poorly-controlled diabetes mellitus and untreated or poorly-controlled hypertension. This is no indication of untreated or poorly-controlled diabetes mellitus and untreated or poorly-controlled hypertension in this patient. Nephritic syndrome is characterized by proteinuria, hematuria, azotemia, red blood cell casts, oliguria, and hypertension.

A 66-year-old man presents to the office with polyuria and erectile dysfunction. He denies any other symptoms or significant past medical history. Physical examination reveals Tanner stage 5 of the external genitalia, balanitis of an uncircumcised penis, and slightly enlarged, symmetrical and smooth prostate. His condition is most likely the result of: Answer Choices 1 Benign prostatic hypertrophy 2 Diabetes insipidus 3 Diabetes mellitus 4 Hypogonadism 5 Prostate cancer

Explanation The correct answer is diabetes mellitus since the presence of polyuria would indicate hyperglycemia and the associated erectile dysfunction and/or balanitis may be the only other presenting symptom or sign of diabetes mellitus in a male patient. Erectile dysfunction is a common vascular and neurological complication of diabetes and occurs in up to 75% of male diabetics. Elevated blood sugars result in autonomic neuropathy of the cavernous nerve of the penis so that erectile dysfunction serves as one of the earliest indications of neuropathy. Likewise, hyperglycemia results in microvascular damage to the dorsal and cavernous arteries, in the same way retinopathy, nephropathy, and neuropathy develop, further contributing to poor perfusion and erectile dysfunction. Hyperglycemia also results in the colonization of skin organisms, commonly Candida, resulting in typical superficial yeast infections seen in diabetics such as balanitis in men and vulvovaginitis in women. Benign prostatic hypertrophy (BPH) typically occurs in the periurethral zone of the prostate and usually presents with lower urinary symptoms (LUTS) that suggest obstruction (i.e. hesitancy, weak stream, straining, post-void leaking) or irritation (i.e. nocturia, frequency, urgency). Digital rectal examination of prostatic hyperplasia typically reveals a smooth, firm enlargement of the gland which may be asymmetrical or indurated. Early BPH is not typically associated with erectile dysfunction or Candidaskin infections. Prostate cancer most often develops in the peripheral zone of the prostate and is usually asymptomatic. Locally advanced prostate cancer may encroach on the central transition zone of the prostate and present with irritative urinary symptoms. Prostate cancer that extends outside the prostate capsule may result in erectile dysfunction. Carcinomas in the peripheral zone are often palpable and typically a hard, irregular nodule or induration. Prostate cancer is not typically associated with Candidaskin infections. Hypogonadism may present with fatigue, decreased libido, diminished erections, gynecomastia, or decreased testicular size, muscle mass, or hair growth associated with secondary sexual characteristics. It is typically not associated with an enlargement of the prostate, urinary complaints, or Candidaskin infections. The characteristic presentation of diabetes insipidus (DI) is abnormally large amounts of dilute urine - insipidus means tasteless. Polyuria is massive, often associated with nocturia and enuresis, and results in dehydration, which is often not evident due to a compensatory increase in thirst and polydipsia. DI is the result of the posterior pituitary's failure to secrete antidiuretic hormone (ADH) resulting in central diabetes insipidus (DI) or the kidney's resistance to ADH resulting in nephrogenic DI. DI is not typically associated with Candidaskin infections.

A 20-year-old female (G1, P0) presents to your office for her 36th week antenatal care visit. The woman's blood group is O, Rh negative. Her husband's blood type is B, Rh positive. The fetal ultrasound showed mild enlargement in the cardiac chambers and pericardial effusion. The spectrophotometric analysis of the amniotic fluid sample showed increased absorbance at 450nm. The most likely mechanism regarding these findings is Answer Choices 1 Cardiac malformation 2 Hemolytic disease in the fetus 3 Hepatic dysfunction in the fetus 4 Hypoglycemia in the fetus 5 Polycystic kidney

Explanation The fact that the mother is Rh negative and that the father is Rh positive means that the likelihood that the mother will be carrying an Rh positive baby can either 0% or 50% depending on whether the father is heterozygous or homozygous for the Rh gene locus. The antenatal screening of the mother at 36 weeks gestation showed dilatation of the cardiac chambers and pericardial effusion in the fetus, which are signs of heart failure. Since the mother is Rh negative and has likelihood of 50% of carrying an Rh positive baby, hemolytic disease of the newborn should be on the top of your list while working up the differential diagnosis in this case. The presence of hyperbilirubinemia in the amniotic fluid as detected by spectrophotometry also should steer you towards hemolytic disease in the fetus due to Rh incompatibility with alloimmunization of the mother. It is a common misunderstanding that the first pregnancy should be safe in an RH neg. mother carrying and RH pos. fetus. Feto-maternal transfusion on a small scale can occur during pregnancy and depending on how much has been exchanged between the maternal and the fetal circulation, RH antibodies can be found in third trimester of primigravidas, that is why re-screening for RH antibodies around the 28th week is recommended. The heart failure and jaundice are not signs of hypoglycemia in the fetus and the mother is not diabetic making this possibility highly unlikely. The ultrasonic evaluation of the fetus showed only mild signs of heart failure and did not indicate the presence of cardiac malformation in the fetus. In addition the Rh-negative status of the mother should alert you to the possibility of an Rh incompatibility specially knowing that the husband is Rh positive. Hepatic dysfunction also will be associated with increased bilirubin in the amniotic fluid but will not explain the cardiac findings. The ultrasound examination would have detected the polycystic kidneys if it were present.

A 59-year-old patient with end-stage renal disease presents via ambulance to the emergency department. Initially complaining of severe weakness, he quickly developed respiratory failure, for which he was intubated shortly after arrival. Stat chemistry labs reveal the following: Na 131 mEq/L; K 6.1 mEq/L; Cl 98 mEq/L; CO3- 19 mEq/L; BUN 42 mEq/L; Cr 6.6 mEq/L. Question Which of the following electrocardiographic (ECG) findings is most likely to be present? Answer Choices 1 Flat P waves 2 Peaked T waves 3 Shortened QT interval 4 Shortened PR interval 5 ST segment elevation

Explanation The patient is presenting with hyperkalemia. This is relatively common in patients with end-stage renal disease, especially among those who are noncompliant with medications and/or dialysis therapy. Signs and symptoms of hyperkalemia include weakness, which may escalate to muscle paralysis and respiratory failure. A number of characteristic electrocardiographic (ECG) findings are present in the setting of hyperkalemia. Tall, so-called "peaked" T waves are a classic early finding. As potassium levels rise, there is flattening of P waves, prolongation of the PR interval, and widening of the QRS complex. Untreated, a sinus wave pattern may develop followed by life-threatening dysrrhythmias and/or cardiac arrest.

You are the resident on call in the ER when a multiparous patient presents in the first stage of labor, with contractions less than one minute apart. The chief resident calls you to assist in the delivery of the child and states that "the fetal head is engaged." In reference to this phrase, which of the following is true? Answer Choices 1 the widest portion of the fetal head has successfully passed through the pelvic inlet 2 forceps will need to be used because the widest portion of the fetal head is engaged at the pelvic inlet 3 in a multiple birth, the head of the first fetus is not presenting first, but is positioned correctly 4 the fetus is presenting in a frank breech fashion with the head positioned correctly in the pelvis 5 the crown of the head of the fetus is seen to bulge the perineum indicating birth is imminent

Explanation When the fetal head is engaged, the widest portion of the fetal head has successfully passed through the pelvic inlet. This phrase signals that the delivery of the infant is proceeding normally, that the presentation is head first, and is not a breech presentation. After this stage, with the complete dilation and effacement of the cervix, the child will descend down the birth canal. Only at that time would the fetal head be noted to be crowning, e.g., presenting as a bulge at the perineum.

An unresponsive 20-year-old female college student is brought to the emergency room. Respiratory rate is 50/min and systolic blood pressure is 60mm Hg. She is intubated and transferred to the ICU. The patient's roommate reports that the patient had been feeling ill for about 2 days and had been experiencing diarrhea. The physical examination reveals lungs that are clear; she has an elevated body temperature of 39.8°C, and there is a desquamation of the skin around the fingernails and underlying toes. A malodorous tampon is removed. Culture and Gram stain is obtained from exudate material from the cervix. Blood is drawn for a CBC and for coagulation studies. The CBC is significant for an elevated white blood cell count with a pronounced left shift and a platelet count that is significantly decreased. Prothrombin time is abnormally high. The Gram stain is significant for Gram-positive cocci in clusters. The culture is positive for Gram-positive cocci that are beta-hemolytic on 5% sheep blood agar, catalase-positive, and coagulase-positive. Refer to the image. This patient is diagnosed as having menstrual toxic shock syndrome due to what infection? Answer Choices: A Streptococcus agalactiae B Clostridium perfringens C Bacillus cereus D Streptococcus intermedius group (millerii) E Streptococcus pyogenes F Staphylococcus aureus

F

You are providing care to a woman who is at 33 weeks gestation. Her pregnancy is complicated by gestational diabetes. She is being provided education by the dietician, and has weekly obstetrical appointments. What fasting blood sugar (FBS) readings should necessitate switching from diet control to insulin therapy? A FBS > 70 mg/dL B FBS > 90 mg/dL C FBS > 95 mg/dL D FBS > 110 mg/dL E FBS > 126 mg/dL

FBS >95 mg/dL Gestational diabetes has different risks associated with it, in particular for the fetus. Stricter glycemic controls are recommended for pregnant versus non pregnant women by the ACOG and the ADA.

A 22-year-old non-obese female presents to the clinic with sign and symptoms of deep vein thrombosis (DVT). She denies recent trauma, prolonged immobilization, smoking, use of oral contraceptives, or changes in her health. Her last menstrual period was one week ago. Which of the following is the most likely diagnosis that contributed to the development of her DVT? A Antithrombin deficiency B Factor V Leiden C Protein C deficiency D Protein S deficiency E Hyperhomocysteinemia

Factor V Leiden B Factor V Leiden mutation is one of the most common genetically inherited prothrombotic states. Deficiencies of antithrombin (A), protein C (C) and protein S (D) when homozygous in nature typically present as fetal death or severe thrombosis at birth. When heterozygous, they carry a much lesser risk of developing thrombosis as compared to Factor V Leidin. Hyperhomocysteinemia (E) also carries a much lesser risk of developing thrombosis.

A 20-year-old Lebanese woman presents to a family practice office because she wants to start birth control. She has never been sexually active, and she is engaged to be married in 2 months. She feels well and has no complaints. She thinks she wants "the pill". Her fiancé is also a virgin, and they are not interested in condoms or other barrier methods of contraception. She wants to delay childbearing for at least 2 years. A summary of her past medical history includes: Medications: occasional over-the-counter ibuprofen for menstrual cramps and headaches Allergies: Penicillin Surgical history: Tonsillectomy Medical history: No known conditions. OB/GYN history: Menarche age 12. Regular monthly menses, with mild-moderate dysmenorrhea. Family history: Patient's older sister had a blood clot in her lung that followed delivery of a child. Her paternal grandfather has diabetes and hypertension. Her maternal grandmother had a stroke. Her mother had a deep venous thrombosis in a leg, and her maternal grandfather had prostate cancer. Social history: The patient works as a waitress part-time and is attending college. She currently lives in a dorm. She denies the use of tobacco, alcohol, and recreational drugs. Vitals are obtained from your medical assistant prior to physical exam. Weight 123 lbs. Height 64" Body mass index (BMI) 21.1 Pulse 88 Blood pressure 134/86 Temperature 97.9 °F Question Based upon this patient's presentation, what test should be ordered before prescribing birth control pills? Answer Choices 1 Bleeding time 2 Complete blood count (CBC) 3 Complete metabolic panel (CMP) 4 Factor V Leiden 5 Pap smear

Factor V Leiden Explanation Generally, no screening tests are recommended prior to starting most patients on most forms of contraceptives. However, when risk factors suggest a possible contraindication, the clinician should delve deeper and individualize testing. In this case, the patient is healthy, but her family history is concerning for a heritable form of hypercoagulability. She should be tested for a Factor V Leiden mutation. Factor V Leiden mutations are estimated to be present in approximately 5% of the Caucasian population, and it is highest in those of Lebanese ethnicity, with a prevalence of over 14% of the affected population. This mutation is the most common of heritable causes of hypercoagulability. If the patient tests positive, then she should not use estrogens in either contraceptives or menopausal hormone therapy, due to increased risk of thromboembolism. This patient should be offered alternate methods of contraception. Bleeding time is a test of platelet function, not coagulation factors. Bleeding time is useful in evaluation of patients at risk of bleeding too much, not clotting. A complete blood count (CBC) is useful in evaluating several conditions, such as anemia and infection and alterations of platelet numbers. If the clinician ordered a CBC and it returned normal, it could provide false reassurance that this patient could safely use combination hormonal contraceptives; the CBC can be entirely normal in an individual with a Factor V Leiden mutation. There is another condition of inherited hypercoagulability, familial or essential thrombocytosis, which would show up on the CBC, but this condition is very rare; also, it often presents at birth, so this patient is unlikely to have it. A complete metabolic panel (CMP) provides information on liver and kidney function, glucose levels, and electrolytes. It does not provide information related to this patient's family history of hypercoagulability. Theoretically, the CMP could be done to ensure normal liver enzymes because many of the hormonal contraceptives are hepatically metabolized. However, routine testing of liver function is not recommended prior to starting birth control. The Pap smear (short for Papanicolaou) is a test for cervical cancer and pre-cancerous dysplasia. Many years ago, it was thought that birth control increased risk of cervical cancer. That myth has been disproved. There is no compelling reason to obtain a Pap smear in order to initiate birth control. Routine cervical cancer screening guidelines should be followed, which would indicate that this patient should wait until the age of 21 years before beginning Pap testing.

A 23-year-old male presents with uncontrolled mucocutaneous bleeding following a wisdom tooth extraction. Traditional methods of controlling the bleeding have not significantly reduced the bleeding. On history, he reports epistaxis that takes longer than usual to stop bleeding. He also states that he seems to bruise more frequently than his friends. What laboratory test would be the most beneficial given his history? A. Bleeding time and platelet count B. Bleeding time and VWF antigen C. Factor VII assay and factor VIII assay D. Factor VII assay and platelet count E. Factor VIII assay and VWF antigen

Factor VIII assay and VWF antigen In a patient suspected of having von Willebrand's disease, routinely performed tests include the assay of factor VIII activity, VWF antigen, and VEW Ristocetin cofactor activity. Bleeding time was previously used as a screening test, but is inconsistent. Factor VII assay would be used for someone suspected of having VII deficiency. Platelet counts are usually normal in a patient with von Willebrand's disease.

A 46-year-old man presents for a routine check up. He drinks alcohol occasionally and quit smoking 20 years ago. His BP is 124/76 mmHg. He works as a salesman, but he wants to change to a desk job because he finds the commuting too stressful. His new employer requests that he have a full health check up prior to starting. He is concerned because his maternal grandmother and his father had diabetes. Question What is the best screening test to use? Answer Choices 1 HbA1c 2 Fasting Plasma Glucose 3 Post Prandial Plasma Glucose 4 75 gram Glucose Tolerance Test 5 100 gram Glucose Tolerance Test

Fasting Plasma Glucose Explanation The FPG is the recommended screening test. The OGTT may be necessary for the diagnosis of diabetes when the FPG is normal. A fasting blood glucose level of over 126 is considered diagnostic of diabetes. A level of 110-125 is considered impaired glucose tolerance. The FPG is preferred for screenings because it is faster and easier to perform, more convenient, acceptable to patients, and less expensive. These are the American Diabetes Association Guidelines. HbA1c, Post Prandial Glucose, and OGTT are incorrect. The American Diabetic Association has approved the use of PPBS and HBA1c in the diagnosis of diagnosis of diabetes. However, fasting plasma glucose is preferred for the reasons stated above.

A 55-year-old Hispanic woman presents to establish care. She recently went to a health fair, where she had some basic serum chemistries drawn. Her serum creatinine was 1.5 mg/dl. On a questionnaire she completed in your waiting room, she noted that she has no known past medical history, except for occasional muscular aches, for which she takes indomethacin (about 2 or 3 times in the past 8 months). There is no family history of renal disease. Her BP is 142/82 mm Hg, and her body mass index is 31 kg/m2. Question What additional study would be most helpful in determining the cause of her elevated creatinine? Answer Choices 1 Complete blood count 2 Renal ultrasound 3 Fasting serum glucose level 4 Urinalysis to evaluate for the presence of hematuria 5 Urine culture

Fasting serum glucose level Explanation The most likely cause of chronic kidney disease in the United States is diabetes, so a history of elevated blood sugars is important to know. This patient is noted to be somewhat overweight, putting her at higher risk for diabetes. A fasting serum glucose level would be useful to rule of diabetes mellitus. Complete blood count may reveal anemia related to chronic kidney disease, but would not be useful in identifying the cause of kidney dysfunction. Chronic urinary infections can lead to scarring of the renal interstitium and to chronic kidney disease. It is important to ask about this risk, particularly if other risk factors for kidney disease are not noted. A urine culture can be used to rule out urinary tract infection. However, in this patient with no signs or symptoms of urinary tract infection, urine culture is not likely to be a useful diagnostic test. Hematuria can be caused by a variety of disorders including glomerular disease, acute and chronic infections, nephrolithiasis, renovascular disease, cystic kidneys, and urogenital cancers. It is prudent to ask about hematuria. Still, these disorders are less common than diabetic and hypertensive nephropathies, so they are not the most likely cause of her elevated creatinine. Renal ultrasound can identify renal scarring, tumors, kidney stones, and other structural abnormalities that could be causing kidney dysfunction, but ultrasound is not a usual first-line screening test.

A 4-year-old girl has always been below 3% for her height and weight. Her mother says she eats 3 meals a day with healthy snacks. The family tries to maintain a low-fat, high-fiber diet. Her parents and sisters are of above average height and average weight. Other than occasional upper respiratory infections and 1 episode of pneumonia last winter, the girl has been fairly healthy. Her failure to thrive workup is normal other than a sweat test, which reveals a high chloride concentration. Question What is an important modification to her diet in light of her probable diagnosis? Answer Choices 1 Fat-soluble vitamin supplements 2 Low-protein foods 3 Low-fat foods 4 High-calcium foods 5 Water-soluble vitamin supplements

Fat-soluble vitamin supplements A positive sweat test indicates cystic fibrosis. Positive results should be confirmed, and a negative result should be repeated if there is still a suspicion of cystic fibrosis. Patients with cystic fibrosis usually have loss of exocrine pancreatic function and inadequate digestion of proteins and fats. The optimal diet includes fat-soluble vitamin supplements (A, D, E, and K), pancreatic enzyme replacements, and high-protein and high-calorie foods. Zinc and iron supplements are also sometimes needed. A higher number of calories may be required due to the increased work of breathing and higher metabolic demands. Weight gain is usually not achieved unless lung infection is controlled.

A 39-year-old woman presents for an annual appointment. She has a history of diabetes mellitus, but she is otherwise healthy. Her recent lab work reveals that her serum glucose is well controlled by a combination of lifestyle and 1 medication. Although this is a routine appointment, she does present with recent onset of some unusual symptoms. Physical examination of the patient also reveals some abnormalities. Question What history and physical examination combination would prompt a workup for Addison's disease? Answer Choices 1 Cramping, numbness around mouth, and tingling in distal extremities combined with carpopedal spasm and exaggerated deep tendon reflexes 2 Weight gain, decreased libido, menstrual irregularity combined with moon facies, and increased adipose tissue in neck 3 Fatigue, dizziness, salt craving combined with increased pigmentation of her lips and hand creases, as well as hair loss 4 Bone pain, flank pain, and generalized anxiety combined with hypertension and mental confusion 5 Hair loss, decreased energy, and constipation combined with periorbital puffiness and dry, coarse skin

Fatigue, dizziness, salt craving combined with increased pigmentation of her lips and hand creases, as well as hair loss The correct answer is the combination of fatigue, dizziness, and salt craving with increased pigmentation of her lips and hand creases, as well as hair loss. Addison's disease is the result of adrenal cortex destruction, which causes deficiencies in cortisol, aldosterone, and androgens. Patients can have any combination of various systemic symptoms, including fatigue, weakness, nausea, vomiting, abdominal pain, dizziness, chronic diarrhea, depression, salt craving, and a decreased tolerance for cold. In addition, the physical examination findings can also vary. Some possible findings are hypotension, weight loss, vitiligo, hair loss, and increased pigmentation. The increased pigmentation can occur in the hand creases, dental-gingival margins, buccal and vaginal mucosa, lips, areolas, scars, and pressure points. The increased pigmentation is a result of increased secretion of β-lipoprotein. The combination of weight gain, decreased libido, and menstrual irregularity with moon facies and increased adipose tissue in neck are indicative of a patient with Cushing's disease. Cushing's disease is essentially the opposite of Addison's disease; its signs and symptoms are caused by an excess of adrenocorticotropin rather than the deficiency seen in Addison's disease. Patients with Cushing's disease can also experience hypertension, hirsutism, depression, easy bruising, muscle weakness, obesity, thin skin, skeletal growth retardation, and acne. The combination of cramping, numbness around mouth, and tingling in distal extremities with carpopedal spasm and exaggerated deep tendon reflexes could occur in a patient with hypoparathyroidism. This disorder is a result of a deficiency of parathyroid hormone from congenital absence, injury, surgery, or other diseases. Most of the symptoms associated with hypoparathyroidism are a result of hypocalcemia. The combination of bone pain, flank pain, and anxiety with hypertension and mental confusion could occur in a patient who has hyperparathyroidism. This disorder is characterized as dysfunction in the body's regulatory system for parathyroid hormone. While up to 75% of patients can be asymptomatic, the hypercalcemia associated with hyperparathyroidism can result in the classic complaints of painful bones, renal stones (causing flank pain), abdominal groans, and psychic moans. The combination of hair loss, decreased energy, and constipation with periorbital puffiness and dry, coarse skin is indicative of hypothyroidism. Hypothyroidism is typically an autoimmune disease or a result of prior radiation therapy or thyroid surgery.

A 32-year-old, vegetarian female in mid-pregnancy complains of lack of energy and says she becomes easily fatigued. Upon any strenuous movement, her heart pounds rapidly and she becomes short of breath. Nutritional supplement of what mineral may alleviate the female's symptoms? Answer Choices 1 Mg2+ 2 Ca2+ 3 Fe2+ 4 Cu2+ 5 Zn2+

Fe2+ Explanation Pregnant females can experience iron deficiency anemia due to increased demands on their blood. Oxygen (O2) in the lungs binds to the iron ion, Fe2+ while complexed with the heme cofactor of hemoglobin in red blood cells. Iron supplements or foods with abundant iron, such as liver, lean meats, or vegetarian alternatives such as spinach, carrots, and raisins can alleviate anemic symptoms. The other minerals have biological roles but are not associated with anemic symptoms. Magnesium (Mg2+) coordinates with the negatively charged backbone of DNA and interacts with neurotransmitter receptors at excitatory synapses in the central nervous system. Mg2+ deficiency affects the nervous system, resulting in vasodilation, tremors, and depression. Calcium phosphate forms a hard material in bone and teeth. In addition, Ca2+ is a ubiquitous second messenger ion in cellular signaling coupled to G-protein signaling, hormone signaling, and ion channel activity. Ca2+ deficiency can give rise to muscle twitching or cramping and cardiac arrhythmias. Copper (Cu2+) participates in bone and blood formation and is an electron carrier in mitochondrial electron transfer proteins. Cu2+ deficiency is uncommon since the trace amounts needed are satisfied by most diets. Zinc (Zn2+) is a cofactor of many DNA and RNA binding proteins, including many transcription factors. Severe zinc deficiency can retard growth in children, can cause low sperm count in males, and can slow wound healing.

A 73-year-old man with hypothyroidism has been in hospitalized with a spinal cord injury is evaluated at the bedside. He is unable to have bowel movements; he has only passed a stool 2 times within the past 10 days. He states that he has had involuntary passage of small loose or liquid stools during this time; it has been associated with abdominopelvic discomfort. His dietary history is remarkable for a low-fiber diet that lacks raw fruits and vegetables. Of note, he takes oxycodone for chronic back pain. He denies any abdominal or pelvic pain, weight loss, hematochezia, melena, fever, chills, or urinary complaints. His rectal exam reveals good sphincter tone, but a firm, immovable mass is detected. Bedside pelvic radiograph displays the following image. Question What is the most likely diagnosis? Answer Choices 1 Rectal cancer 2 Prostatitis 3 Acute colitis 4 Rectal abscess 5 Fecal impaction

Fecal Impaction This patient's diagnosis is a rectal impaction; the attached X-ray displays colonic distension due to fecal impaction. Risk factors include being elderly, immobile, having a spinal cord injury, chronic use of opioids, hypothyroidism and a low fiber diet. Dilated bowel as well as a large amounts of stool may be visible via radiography. Impaction may also occur due to primary colorectal disorders, such as obstruction, slow colonic motility, outlet obstruction, Hirschsprung's disease in children, and Chagas' disease. Adenocarcinomas, the most common form of rectal cancer, most commonly present with rectal bleeding. Other signs and symptoms include a change in bowel habits, often in the form of diarrhea, occult bleeding, tenesmus, and a feeling of incomplete evacuation. Other manifestations include abdominal or back pain, urinary symptoms, malaise, pelvic pain, or (rarely) perforation. Prostatitis is an infection or inflammation of the prostate gland that presents with fever, chills, malaise, arthralgias or myalgias, perineal/prostatic pain, dysuria, obstructive urinary tract symptoms (including frequency, urgency, dysuria, nocturia, hesitancy, weak stream, and incomplete voiding), low back or abdominal pain, urethral discharge, or may have a history of sclerotherapy for rectal prolapse. The physical exam reveals a tender, nodular, hot, boggy, or normal-feeling gland on digital rectal examination, suprapubic abdominal tenderness, or an enlarged tender bladder due to urinary retention. Colitis is an inflammation of the colon. It may be associated with enteritis (inflammation of the intestine), proctitis (inflammation of the rectum), or both. The most common symptoms upon presentation include abdominal pain and diarrhea, with or without occult blood. The pain is frequently colicky and, in Crohn's disease, may localize to the right lower quadrant or periumbilical area. Frank rectal bleeding is more common in ulcerative colitis than Crohn's disease. On the physical, pallor, tachycardia, abdominal tenderness, blood in the stool, hyperthermia, weight loss, and signs of dehydration may be noted. Patients with pseudomembranous colitis typically present with profuse watery or mucoid diarrhea, tenesmus, fever, abdominal cramps, and tenderness, usually within 1 week of antibiotic therapy. The stools may be frankly bloody or guaiac-positive. Perirectal abscesses are associated with indolent, dull perirectal pain exacerbated by movement and increased perineal pressure from sitting or defecation. There may be associated pruritis. Those with an ischiorectal abscess often present with systemic fevers, chills, and severe perirectal pain and fullness consistent with the more advanced nature of this process. External signs are minimal and may include erythema, induration, or fluctuance. Physical examination demonstrates a small, erythematous, well-defined, fluctuant, subcutaneous mass near the anal orifice. On digital rectal examination (DRE), a fluctuant, indurated mass may be encountered.

A nulliparous 25-year-old woman has had an uneventful, spontaneous labor at term. She has an epidural anesthetic and has been in the second stage of labor for 2 hours. The fetal heart tones show some signs of deterioration. You decide to expedite the delivery with the use of forceps. What criterion defines the application of low forceps? Answer Choices 1 Fetal head is on the perineum 2 Fetal skull has reached the pelvic floor 3 Fetal skull is at station +2, but not on the pelvic floor 4 Head is engaged, but above station +2 5 Scalp is visible at the introitus without manually separating the labia

Fetal skull is at station +2, but not on the pelvic floor Explanation Low forceps is defined by the skull being at station +2 or greater, but not on the pelvic floor. Outlet forceps includes situations where the scalp is visible, the fetal skull has reached the pelvic floor, or the head is on the perineum. In addition, rotation must not exceed 45 degrees. Midforceps is the head engaged but above a station of +2.

A 48 year-old male presents to the clinic with a history of productive cough for the past 3 days. Which of the following factors if present best predicts the diagnosis of pneumonia in this patient? A Adenopathy B Double sickening C Dyspnea D Fever E Hoarseness

Fever Cough, fever (D) and sputum production are classic symptoms of pneumonia. Adenopathy (A) and hoarseness (E) may coexist in some cases of viral pneumonia. Dyspnea (C) may develop as pneumonia progresses. Double sickening (B) is a clinical consideration in the differentiation of acute sinusitis from upper respiratory infection that typically occurs after approximately 7 days of illness.

A 27-year-old man has been experiencing heartburn on and off for approximately 6 years. At this current visit, the patient is also describing symptoms of chest pain, dysphagia, and even early satiety. Because of the persistence of his symptoms despite pharmacologic therapy with both proton pump inhibitors and various histamine blockers, you decide to consult with a local gastroenterologist group. The patient undergoes an endoscopy, with biopsies of tissue. Results of the biopsy show no ulcers and are H. pylori negative; the tissue samples themselves are prominently infused with an abundant amount of eosinophils. Question Based on the history and physical exam findings above, what is the most effective pharmaceutical intervention at this time? Answer Choices 1 Oral omeprazole 2 Oral ampicillin 3 Oral cetirizine 4 Fluticasone metered dose inhaler 5 Topical fluconazole

Fluticasone metered dose inhaler The patient described in the above scenario has the diagnosis of eosinophilic esophagitis (EE). Because this is a relatively new diagnosis, the prevalence of EE is unknown, although the incidence is increasing in both children as well as adults. These patients will have symptoms typically 4 - 5 years before they have a confirmed diagnosis. EE is more commonly seen in male patients who are Caucasian, and up to 70% of patients with EE have personal history of atopy to environmental or foods. The most common symptoms found in adults with EE are dysphasia, refractory heartburn, chest pain, and food impaction. Usually, no physical exam abnormalities will be detected. An endoscopy with esophageal biopsy is the only definitive method to diagnose EE. The presence of eosinophils of 15 or more per high power field is required for a diagnosis. Other physical findings that may be visualized during an endoscopy that are highly suggestive of EE include stacked circular rings ('feline esophagus'), proximal strictures, linear furrowing, as well as white exudates or even papules that signify eosinophilic purulent material. Currently there are no FDA approved medications for the treatment of EE. The most efficient treatment for symptoms is topical corticosteroids, either via fluticasone metered dose inhaler or viscous budesonide. These medications are swallowed and therefore coat the esophagus; the symptoms as well as the overall eosinophilia improve fairly quickly. Patient response is usually in the first few days of treatment; however, symptoms will reoccur once the medication regimen is discontinued. Topical fluconazole is not the correct option; EE is not a fungal infection. Oral ampicillin is not correct because EE's underlying pathology is not a bacterial infection. Oral omeprazole is not specifically recommended for treatment of EE; however, it may treat the co-existent GERD. Oral cetirizine is also not an appropriate choice; if there is evidence of severe atopy, therapy should be initiated, but no evidence presents in the case scenario showing that the treatment of airborne allergic disorders improves EE.

A 25-year-old female presents for an ultrasound after having a positive home pregnancy test. She has an unremarkable past medical history and physical exam. She states she has been feeling fine without any abdominal discomfort or vaginal bleeding noted. On ultrasound you determine she is 10 weeks pregnant. You note a noncomplex unilateral mass on her left ovary measuring 2 cm in diameter. What is the most likely diagnosis? A Benign cystic teratoma B Serous cystadenoma C Functional ovarian cyst D Hemorrhagic ovarian cyst E Torsion of the adnexa

Functional Ovarian cyst C More than 90% of unilateral, noncomplex masses that are measured to be less than 5 centimeters in diameter that are identified in the first trimester are functional ovarian cysts. Benign cystic teratomas (A) make up 21% of pathologic ovarian neoplasms and serous cystadenomas (B) make up 21%. A hemorrhagic cyst (D) is a non-functional cyst. Torsion of the adnexa (E) is most commonly seen between 6-14 weeks gestation or in immediate puerperium but symptoms include abdominal pain and tenderness.

A 68-year-old African-American woman with a past medical history of obstructive sleep apnea, hypertension, and COPD presents with chronic, progressive dyspnea, which occurred upon exertion initially, but is now noted at rest over the last 8 months. She is maintained on home oxygen for COPD. There is associated fatigue, substernal exertional chest pain, and 2 episodes of syncope, which has occurred during exertion. She denies other symptoms. Physical exam reveals an oxygen saturation of 90%, a left parasternal lift, narrow splitting of the second heart sound, accentuation of the pulmonary component of the second heart sound, an early systolic ejection click, and an S4 gallop. No murmurs are identifiable. There is also +1 pitting edema noted bilaterally to the lower extremities to the mid-calf level. An EKG and chest x-ray were performed, which revealed the results shown. Question What treatment is considered beneficial in the management of this patient at this time? Answer Choices 1 Propranolol 2 Warfarin 3 Furosemide 4 Digitalis 5 Vasopressin

Furosemide This patient has a presentation most consistent with cor pulmonale. Chronic obstructive pulmonary disorder is the most common cause of secondary cor pulmonale while primary pulmonary hypertension is idiopathic. The electrocardiogram demonstrates right axis deviation, right ventricular hypertrophy, and right atrial enlargement. The chest x-ray indicates cardiac enlargement, with prominence of the pulmonary artery, right atrium, and right ventricle. Diuretics such as furosemide are used in the management of chronic cor pulmonale, particularly when the right ventricular filling volume is markedly elevated and in the management of associated peripheral edema. These agents may result in improvement of the function of both the right and left ventricles. Propranolol is a nonselective beta adrenergic receptor blocker whose use is contraindicated in patients with bronchospastic disease (such as asthma), COPD and congestive heart failure. Beta-selective agonists, not antagonists, demonstrate the advantages of bronchodilator and mucociliary clearance effect in the treatment of cor pulmonale. Warfarin is recommended in patients at high risk for thromboembolism. The beneficial role of anticoagulation in improving the symptoms and mortality in patients with primary PAH has been demonstrated in several studies. The evidence of benefit, however, has not been established in patients with secondary PAH. Therefore, anticoagulation therapy may be used in patients with cor pulmonale secondary to thromboembolic phenomena and with underlying primary PAH. Digitalis should not be used during the acute phases of respiratory insufficiency when large fluctuations in levels of hypoxia and acidosis may occur. Patients with hypoxemia or acidosis are at increased risk of developing arrhythmias due to digitalis through different mechanisms, including sympathoadrenal stimulation. Vasopressin possesses both vasoconstrictor and antidiuretic properties, both of which contribute to elevating arterial pressure. Its use is not appropriate for cor pulmonale. Rather, vasodilators have been advocated in the long-term management of chronic cor pulmonale with modest results. For instance, calcium channel blockers (particularly oral sustained-release nifedipine and diltiazem) can lower pulmonary pressures.

A 1 day-old boy develops progressing abdominal distension, bilious vomiting and failure to pass a meconium stool. Abdominal radiographs show dilated loops of small bowel. Which of the following genetic mutations should the patient be evaluated for? A ΔF508 B G551D C Q1412X D R117H E W1282X

G551D Mutations in CFTR protein function resulting from genotype G551D mutations are amenable to treatment with ivacaftor (B). Approximately 5% of CF patients have the G551D mutation and all patients should be assessed for potential ivacaftor therapy. ΔF 508 (A) is the most common genotype occurring in 60-66% of all CF patients and is not amenable to ivacaftor therapy.

A 36-year-old software consultant presents with complaints of a burning sensation in his chest for the last two weeks. He has had similar symptoms in the past, on and off for about 6 months. His wife has noticed episodes of regurgitation and coughing at night. There is no dysphagia, weight loss, hematemesis, or melena. He has no other past medical history. He is a nonsmoker and does not drink alcohol. Family history is significant only for HTN in father. On exam, his BP is 120/80 mm of Hg, pulse 76/min, SPO2 92%, height 6'4, and weight 242 pounds. Physical exam is unremarkable. What would be the likely diagnosis? Answer Choices 1 Peptic ulcer disease 2 Gastroesophageal reflux disease 3 Esophageal stricture 4 Acute gastritis 5 Cholelithiasis

GERD

A 5-year-old child presents for her kindergarten checkup. The clinician notes that over the past couple of years, her height decreased from the 50th percentile to the 5th percentile. On examination, the clinician also notes truncal adiposity. Her CBC and lead levels were normal. Which of the following is the most likely diagnosis? A growth hormone deficiency B Cushing disease C congenital hypothyroidism D congenital adrenal hyperplasia

GH deficiency Growth hormone (GH) deficiency is defined as a decreased growth velocity, delay in skeletal maturation, absence of other explanations for poor growth (lack of intake), and laboratory tests demonstrating decreased GH secretion. Etiology of GH deficiency can be congenital, genetic, acquired, or idiopathic, which is the most common. Infants usually have a normal birth weight and may have a slightly decreased length. In addition, most infants present with other endocrine deficiencies like hypoglycemia, hypothyroidism, and/or adrenal insufficiency. Children may present with truncal adiposity because growth hormone promotes lipolysis. Serum GH or intrinsic growth factor levels may or may not be decreased. In patients who do not have a demonstrated decrease in these hormones, a trial period with GH is indicated. These patients and positive GH-deficient patients receive a once-daily subcutaneous injection of recombinant human GH. Congenital hypothyroidism typically presents with short stature (typically noted after the 4-month newborn visit), delayed epiphyseal development, delayed closure of fontanelles, and retarded dental eruption in addition to other signs of hypothyroidism. Cushing disease typically presents with truncal adiposity with thin extremities, muscle wasting, decreased growth rate, and moon facies. Laboratory results show elevated adrenocorticosteroids both in urine and serum, hypokalemia, eosinopenia, and lymphocytopenia. Typically, in patients younger than the age of 12, Cushing disease is secondary to administration of ACTH or glucocorticoids. Congenital adrenal hyperplasia typically presents with pseudohermaphroditism in females or salt-losing crisis in males with or without isosexual precocity. There is an increased linear growth and advanced skeletal maturation.

A 60-year-old Caucasian man comes to your office to establish care. He has no known medical problems and his only medication is daily ibuprofen for 5 years for musculoskeletal aches associated with his work as a handyman. He is at most risk for which following due to chronic NSAID use? Answer Choices 1 Chronic kidney disease not requiring dialysis 2 Chronic kidney disease requiring dialysis 3 Gastrointestinal bleed 4 Accidental job injury 5 Chronic joint pain

GI bleed The patient described in this question is at most risk for a gastrointestinal bleed secondary to his chronic non-steroidal use. Patients in their 60s are at an approximate 3-fold increased risk for a gastrointestinal bleed when they take non-steroidal anti-inflammatory medications compared to not taking them. The non-steroidals inhibit the prostaglandins responsible for homeostasis, and thus can lead to gastrointestinal mucosal injury. Chronic analgesic consumption is associated with chronic kidney disease. Analgesic nephropathy is still a relatively uncommon cause of chronic kidney disease; its prevalence is higher in Europe and Australia than in the United States, where it accounts for only 2-4% of the cases of end stage kidney disease. End stage kidney disease is the most advanced stage of kidney disease, for which patients typically require dialysis. Some studies have debated causality, however a recent New England Journal of Medicine article addressed the risk in patients with chronic consumption and found a 2.5-5 fold increase risk of chronic renal failure in patients who chronically used acetaminophen vs. those who did not. Cumulative lifetime use increased the odds of renal failure, and results were similar for patients who took aspirin. The highest risk was noted among patients who took >500g/yr acetaminophen (>1.4g/day). Other studies have found increased risk among users of >3-5 gram per lifetime and >6 tablets/day for 3 years. The National Kidney Foundation currently classifies 5 stages of chronic kidney disease in patients who have structural or functional damage of the kidneys for greater than or equal to 3 months, as evidenced by abnormal pathology or laboratory markers of kidney damage (blood, urine, imaging studies). Patients who have a GFR less than 60ml/min/1.73m3 for at least 3 months, with or without such markers, are also considered to have chronic kidney disease. This patient presumably already has chronic aches (for which he takes the ibuprofen) and should not be at any additional risk for accidental injury secondary to the ibuprofen.

A 62-year-old woman presents with excruciating pain, burning, and swelling in her left forearm and wrist. She reports that symptoms initially began with a fracture 4 months ago. The fracture was casted, and the patient was told it had healed well with cast removal at 8 weeks. She is frustrated because her symptoms have persisted and worsened, rather than improving, as she was told they would. The patient has continued to use a sling and limit use of the left arm so she doesn't worsen her condition. She is unable to wear a jacket or long sleeves, as even the fabric touching her skin causes pain. She denies fevers, pain in other areas, and new trauma. She denies polyuria and polydipsia. Her past medical history is unremarkable; she is a menopausal woman, with no known medical conditions, no history of surgeries, no regular medications and no allergies. She lives with her husband, and she is a homemaker. She denies drug, alcohol, and tobacco use. On physical exam, she is a small, thin, pleasant woman, and she is fully oriented. Vitals are normal. No gait or balance abnormalities are noted when she walks or gets onto the exam table. Her left forearm has some mild edema and erythema, as well as tenderness with even light touch. Distribution of findings includes the region from the elbow to wrist, both the anterior and posterior surfaces. Left wrist strength and range of motion are decreased compared to the right. Distal pulses, capillary refill, and reflexes are normal. The remainder of the exam, including mental status, is normal. Question What medication would be most appropriate for this patient's likely diagnosis? Answer Choices 1 Gabapentin 2 Heparin 3 Methadone 4 Probenecid 5 Vancomycin

Gabapentin This patient's diagnosis is most likely a complex regional pain syndrome (CRPS). CRPS most often develops after a minor trauma and classic characteristics include pain out of proportion with findings and history, allodynia (pain sensation with normally non-painful stimuli) and motor and sensory disturbances in the affected extremity. The mechanism for CRPS development is not well understood. Prolonged immobilization following injury is a risk factor for development of CRPS. Diagnosis is clinical, and testing is done to rule out other disorders. Treatment is multi-modal and primarily consists of physical therapy, focusing on mobilization and desensitization. Other treatments are often off-label and targeted at chronic pain relief; they include corticosteroids, bisphosphonates, tricyclic antidepressants, anticonvulsants, and topical anesthetics. Gabapentin would be a reasonable choice for this patient. Heparin would be used if this patient's symptoms were attributed to a deep venous thrombosis (DVT). DVT is rare in the upper extremities; it is not associated with the hyperesthesia and allodynia shown in this patient. Methadone, a longer-acting opioid medication, can be used for chronic pain. However, for several reasons, this is not the most appropriate medication to use in CRPS. Opiates carry addiction potential and increase fall risk in the elderly. CRPS is often categorized as early (<6 months' duration) versus late (>6 months). For early CRPS, opiates should be avoided and even with late CRPS, other medications options should be utilized before initiating opiates. Probenecid is a medication which inhibits urate resorption. It is used for gout, a painful inflammatory condition which may cause acute pain, erythema and swelling in the affected joint(s). Gout tends to be more acute and affect joints (not the forearm, as with this patient). Vancomycin is an IV glycopeptide antibiotic; it is used for severe bacterial infections, including cellulitis. Cellulitis, which commonly presents with erythema, edema and tenderness in an extremity, may be considered on the differential for this patient. Cellulitis tends to occur more acutely. This patient had a normal 8 week exam with symptoms of CRPS, but no findings of cellulitis at that time.

A 42-year-old man presents with severe abdominal pain in the mid-epigastric region. The pain began 3 days ago with an intensity of 10/10, concomitant nausea, and vomiting (on 2 occasions). The patient denies any significant past medical history. BP is 86/58, and pulse is 112; temp is 101.5° F. There is rebound tenderness in the midepigastric area; bowel sounds are absent, and there is a positive Cullen's sign. The rest of the physical exam is within normal limits. What are the 2 most common causes of acute pancreatitis? Answer Choices 1 Hyperlipidemia and hypercholesterolemia 2 Pancreatic cancer and chemotherapeutic drugs 3 Gallstones and alcohol abuse 4 Abdominal trauma and postoperative period 5 Peptic ulcer disease and cystic fibrosis

Gallstones and alcohol abuse More than 70% of cases of acute pancreatitis are due to alcohol abuse or gallstones. Less common causes include postoperative pancreatitis, abdominal trauma, hyperlipidemia, certain drugs, hypercalcemia, peptic ulcer disease, uremia, cystic fibrosis, and viral infections. Complications such as adult respiratory distress syndrome, hemorrhagic pancreatitis, pancreatic abscess, pancreatic pseudocyst, and pancreatic ascites account for the 10% mortality rate associated with acute pancreatitis. Hemorrhagic pancreatitis can lead to retroperitoneal hemorrhage and widespread tissue necrosis, and it may require surgical intervention or peritoneal lavage. This retroperitoneal bleeding can be responsible for the discoloration or hematoma in the periumbilical area (Cullen's sign).

A 65-year-old Caucasian man presents due to losing 15 lbs over the past 6 months; he is also experiencing abdominal pain and bloody stools. He also has a 35-year history of daily alcohol intake and smoking. On examination, the patient is found to have pallor and several hyperpigmented velvety papillomatous plaques on the back of his neck and axilla, as shown in the image. Question What condition is suspected based on patient's signs and symptoms? Answer Choices 1 Chronic liver disease 2 Gastric carcinoma 3 Inflammatory bowel disease 4 Pancreatitis 5 Colon cancer

Gastric carcinoma Gastric carcinoma is the correct answer. The sudden appearance of extensive skin lesions suggestive of seborrheic keratosis or acanthosis nigricans must include workup for intra-abdominal malignancy; in particular, gastric cancer should be considered. It is a para-neoplastic syndrome; it has been given the eponym 'Leser-Trelat sign'. Chronic liver disease is incorrect. Skin manifestations of cirrhosis include spider naevi and caput medusae. Inflammatory bowel disease is incorrect. The skin manifestations most commonly seen with the condition are erythema nodosum and pyoderma gangrenosum. Pancreatitis is incorrect. Severe acute pancreatitis is sometimes associated with ecchymoses of the abdominal wall; they are known as Grey-Turner's sign and Cullen's sign. Colon cancer is incorrect. It is rarely associated with dermatomyositis.

A 72-year-old woman presents because she "threw up fresh blood" 1 hour ago. She denies any heartburn or gastric symptoms. Her past medical history includes type II diabetes controlled with diet and glyburide and ibuprofen use for severe, chronic osteoarthritis. On initial examination, she appears anxious, but she is ambulatory and has no noticeable pallor. Family history is unremarkable. Vital signs are as follows: BP 140 x 80 mmHg without postural hypotension, HR 104 bpm, and RR 18 bpm. The rest of the physical examination shows no abnormalities. The patient is admitted and undergoes upper endoscopy. What finding is most probable? Answer Choices 1 An actively bleeding duodenal ulcer 2 A malignant gastric ulcer 3 Brisk arterial bleeding originating from a point in the gastric wall 4 Gastritis 5 A positive test for Helicobacter pylori

Gastritis Non-steroidal anti-inflammatory agents (NSAIDs) cause ulcer diathesis because they inhibit prostaglandin synthesis. Virtually all NSAID users develop some degree of gastritis, and hemorrhagic gastritis is a common cause of upper gastrointestinal bleeding in these patients. Erosions and ulceration, sometimes with multiple ulcers, are also common. Endoscopic studies have shown that 15-20% of chronic NSAID users develop ulcers; however, most of these lesions are not associated with serious complications (e.g. bleeding, perforation, etc), and screening is not indicated. Chronic NSAID users have a 2 - 4% per year risk of upper gastrointestinal bleeding. It is estimated that NSAID-associated bleeding causes 2,600 deaths annually in the United States. Its incidence is rising because of population aging, which has led to an increased number of people with chronic rheumatic conditions.

A 32-year-old woman presents due to "I'm feeling odd." Further questioning reveals the patient experiencing a significant amount of involuntary movements and a severe lack of coordination; she recently had a significant decrease in cognitive abilities. Because she is an established patient, you recall that the patient got married about a year ago; past family history is unknown due to the fact that the patient was adopted as a child. Physical examination reveals rapid, involuntary movements of fingers bilaterally, impaired ability to concentrate, minor slurred and disorganized speech, as well as difficulty responding with the appropriate words or phrases when prompted. Question What would be the recommended testing at this time to confirm the patient's diagnosis? Answer Choices 1 Magnetic resonance imaging (MRI) of the head 2 Computed tomography (CT) of the head 3 Acid fast bacilli (AFB) smear 4 Genetic testing 5 Rapid plasma reagin (RPR) testing

Genetic testing The correct response is genetic testing. The patient in the above scenario is displaying signs of Huntington's disease. This fatal progression of chorea, dementia, and behavioral changes is an autosomal dominant disorder that occurs in all ethnic groups with a prevalence rate of 5 per 100,000. More commonly than not patients will have a strong family history of this disease state; the unknown family history in our patient makes this information, although missing, extremely valuable. Onset of symptoms typically occurs between the ages of 30 - 50 years old and usually leads to death within 15 - 20 years after symptoms begin occurring. Early symptoms and signs include abnormal movements or intellectual changes, both of which are evident in the patient above. Severe progression of rigidity, chorea, akinesia, and severe psychiatric changes will accompany the progression of the disease. Diagnostic testing at this time to confirm diagnosis of Huntington's disease should be via genetic testing with ultimate guidance from a genetic counselor. The gene responsible for causing Huntington's disease has been identified exclusively on chromosome 4. CT scanning or MRI imaging will not confirm the diagnosis, but rather support it; cerebral atrophy as well as caudate nucleus atrophy will be found as the disease progresses. Acid fast bacilli smear is the inappropriate choice for this scenario; this diagnostic testing may be utilized in a patient to support the diagnosis of tuberculosis. Rapid plasma reagin testing is also the incorrect choice; this diagnostic study along with the serologic test for syphilis (STS), venereal disease research laboratory (VDRL), and the fluorescent treponemal antibody (FTA) assist health care providers in diagnosis as well as successful treatment of syphilis.

A 34-year-old man presents for his first doctor's appointment. He had always neglected his health, but he finally sought medical attention at his wife's insistence. He explains that he was from and "unhealthy family" and that he was tired of being around doctors while growing up; therefore, he avoided medical attention. His father died at the age of 30 of "very high blood pressure" and "heart failure". His older brother was recently operated on for the removal of a cancer from his neck. Prior to that, his brother had had surgery to remove a mass from his adrenal gland. He wants to know what he can do to be healthy. His blood pressure and BMI are within normal limits. He does not smoke or use alcohol, and he exercises regularly. The physician suspects a possible familial syndrome. Question What is the most appropriate next diagnostic step? Answer Choices 1 An annual colonoscopy starting now 2 Measure T3, T4, and TSH levels 3 Genetic testing for possible familial cancer syndrome 4 Reassurance and suggest he return for yearly physical exam 5 Order a renal ultrasound to look for renal cysts

Genetic testing for possible familial cancer syndrome Explanation The vignette suggests a possible familial cancer syndrome. MEN 2a syndrome fits the picture of 2 generations and multiple members of the family affected. MEN 2a is an autosomal dominant disease, characterized by primary hyperparathyroidism, medullary carcinoma of thyroid, and pheochromocytoma. The father has features suggestive of pheochromocytoma, one of the causes of early onset hypertension and sudden cardiac death. The older brother has an adrenal mass that was excised, indicating a possible pheochromocytoma, and a malignant neck mass, indicating possible medullary cancer of the thyroid. MEN 2a is most commonly caused by mutations in the RET proto oncogene. The mutations are more commonly familial, although they may occur spontaneously. Genetic analysis for the mutation is done as a screening test in particularly high risk families, and a prophylactic thyroidectomy may be advocated. Mutation testing has replaced the regular measurement of serum calcitonin as a screening test. Other tests that may be done include: Serum Ca, parathyroid hormone, and plasma free metanephrine or urinary catecholamine levels Pheochromocytoma localization with MRI or CT A routine colonoscopy is incorrect. The US Preventive Services Taskforce recommends a routine colonoscopy starting at age 50 and repeated every 10 years. With a family history of colonic malignancy, the guidelines suggest earlier screening. However, this patient has no family history of colon cancer and is only 34 years of age. Thyroid function testing is incorrect. There is no indication at the present time for a thyroid function testing. Calcitionin may be measured to screen for early medullary cancer, but the thyroid hormones themselves are of no benefit. Reassurance is incorrect. With such a strong family history, reassuring the patient and sending him home would not be appropriate. Renal ultrasound to look for cysts is incorrect. It may be recommended if suspecting polycystic kidney disease, but there is no history suggestive PKD.

Your patient is a full-term newborn with facial defects affecting her eyes, nose, and upper lips. She is the first child of non-consanguineous parents. Her mother has a history of gestational diabetes, which began at the start of her pregnancy. She contracted German measles a month before the delivery. During the course of infection, her self-prescribed daily treatment was 3 tablets of aspirin, at least 6 cups of herbal tea, and a double dose of folic acid. Imaging studies showed prosencephaly. Question What risk factor is responsible for the condition of this child? Answer Choices 1 Rubella 2 Aspirin 3 Herbal tea 4 Gestational diabetes 5 Folic acid

Gestational diabetes Explanation The only possible risk factors during the period of the development of face and forebrain is gestational diabetes, because the forebrain is formed and the face begins to develop in the fifth and sixth weeks of human pregnancy. Other known risk factors for prosencephaly include transplacental infections, bleeding during the first trimester, a history of miscarriages, use of some drugs potentially unsafe in pregnancy (insulin, birth control pills, lithium, anticonvulsants, retinoic acid, cholesterol-lowering agents, and maternal hypocholesterolemia), and toxins (alcohol, nicotine). Rubella, or German measles, is not likely the cause of birth defects because infection occurred after organogenesis was finished. Rubella can cause miscarriage, stillbirth, or birth defects (most often deafness, brain damage, heart defects, and cataracts) if infection occurs during the first 16 weeks of pregnancy. Aspirin taken during the last trimester may increase the risk of bleeding in the newborn, but cannot cause birth defects. Herbal tea is not the cause of birth defects after organogenesis is finished. All women capable of pregnancy should take a daily vitamin supplement of folic acid to prevent neural tube defects. In the late pregnancy, however, folic acid can neither prevent nor cause birth defects of the brain and face.

A 13-year-old boy presents with a long history of intermittent jaundice without other signs and symptoms. Yesterday he took several paracetamol tablets, and the headache and jaundice appeared again. His parents have a consanguineous marriage, and nobody in the family has similar symptoms. His physical examination today is within normal limits, except for mild scleral icterus. The initial laboratory examinations show: RBC 4.5mill/mm3; WBC 6000/mm3; total bilirubin 2.2 mg/dl; indirect bilirubin 2.0 mg/dl; direct bili 0.2 mg/dl; liver enzymes, serum copper, and all other parameters are normal; hepatitis B (-); and the full set of his autoimmune markers (-) and Coombs test (-). Blood smear is normal. Ultrasonography of liver, gallbladder, pancreas, spleen, and for both kidneys are normal. His urine appears very yellow. Question What is the most likely the diagnosis? Answer Choices 1 Gilbert's syndrome 2 Intravascular hemolysis 3 Chronic viral hepatitis 4 Crigler-Najjar syndrome 5 Budd-Chiari syndrome

Gilbert's disease Your patient most probably has Gilbert's syndrome (GS), also known as Gilbert-Meulengracht syndrome. It is a relatively common genetic disease found in up to 5% - 10% of the population and generally does not need special treatment. Inherited non-haemolytic hyperbilirubinemic conditions include Dubin-Johnson, Rotor, and GB syndromes, and all are important differential diagnoses indicating benign disease that requires no immediate treatment. GB can be diagnosed by clinical presentation, biochemistry, and genotyping, and is significant because of the presence of the disposition towards drug-associated toxicity. A major characteristic is jaundice, caused by elevated levels of unconjugated bilirubin in the bloodstream. The cause of this hyperbilirubinemia is the reduced activity of the enzyme glucuronyltransferase, which conjugates both bilirubin and some lipophilic molecules, including drugs. Intravascular hemolysis, with resulting hemoglobinemia, hemoglobinuria, and bilirubinemia, will show fragments of the red blood cells ("schistocytes") and sometimes spherocytes in peripheral blood smear, reticulocytosis, elevated unconjugated bilirubin that may lead to jaundice, elevated lactate dehydrogenase (LDH) in the blood, and decreased haptoglobin levels. If the direct Coombs test is positive, hemolysis is caused by an immune process. Hemosiderin in the urine indicates chronic intravascular hemolysis. There is also urobilinogen in the urine. Viral hepatitis with jaundice will have elevated liver function tests (AST and ALT elevated out of proportion to alkaline phosphatase, usually with hyperbilirubinemia), and viral serologic testing will be positive. Crigler-Najjar syndrome is a rare inherited form of non-hemolytic jaundice, which results in high levels of unconjugated bilirubin and often leads to brain damage in infancy. Budd-Chiari syndrome is caused by occlusion of the hepatic veins. It presents with the classical triad of abdominal pain, ascites, and hepatomegaly. The syndrome can be fulminant, acute, chronic, or asymptomatic.

A 13-year-old boy presents with a long history of intermittent jaundice without other signs and symptoms. Yesterday he took several paracetamol tablets, and the headache and jaundice appeared again. His parents have a consanguineous marriage, and nobody in the family has similar symptoms. His physical examination today is within normal limits, except for mild scleral icterus. The initial laboratory examinations show: RBC 4.5mill/mm3; WBC 6000/mm3; total bilirubin 2.2 mg/dl; indirect bilirubin 2.0 mg/dl; direct bili 0.2 mg/dl; liver enzymes, serum copper, and all other parameters are normal; hepatitis B (-); and the full set of his autoimmune markers (-) and Coombs test (-). Blood smear is normal. Ultrasonography of liver, gallbladder, pancreas, spleen, and for both kidneys are normal. His urine appears very yellow. Question What is the most likely the diagnosis? Answer Choices 1 Gilbert's syndrome 2 Intravascular hemolysis 3 Chronic viral hepatitis 4 Crigler-Najjar syndrome 5 Budd-Chiari syndrome

Gilbert's syndrome Your patient most probably has Gilbert's syndrome (GS), also known as Gilbert-Meulengracht syndrome. It is a relatively common genetic disease found in up to 5% - 10% of the population and generally does not need special treatment. Inherited non-haemolytic hyperbilirubinemic conditions include Dubin-Johnson, Rotor, and GB syndromes, and all are important differential diagnoses indicating benign disease that requires no immediate treatment. GB can be diagnosed by clinical presentation, biochemistry, and genotyping, and is significant because of the presence of the disposition towards drug-associated toxicity. A major characteristic is jaundice, caused by elevated levels of unconjugated bilirubin in the bloodstream. The cause of this hyperbilirubinemia is the reduced activity of the enzyme glucuronyltransferase, which conjugates both bilirubin and some lipophilic molecules, including drugs. Intravascular hemolysis, with resulting hemoglobinemia, hemoglobinuria, and bilirubinemia, will show fragments of the red blood cells ("schistocytes") and sometimes spherocytes in peripheral blood smear, reticulocytosis, elevated unconjugated bilirubin that may lead to jaundice, elevated lactate dehydrogenase (LDH) in the blood, and decreased haptoglobin levels. If the direct Coombs test is positive, hemolysis is caused by an immune process. Hemosiderin in the urine indicates chronic intravascular hemolysis. There is also urobilinogen in the urine. Viral hepatitis with jaundice will have elevated liver function tests (AST and ALT elevated out of proportion to alkaline phosphatase, usually with hyperbilirubinemia), and viral serologic testing will be positive. Crigler-Najjar syndrome is a rare inherited form of non-hemolytic jaundice, which results in high levels of unconjugated bilirubin and often leads to brain damage in infancy. Budd-Chiari syndrome is caused by occlusion of the hepatic veins. It presents with the classical triad of abdominal pain, ascites, and hepatomegaly. The syndrome can be fulminant, acute, chronic, or asymptomatic.

A 23-year-old male with a recent diagnosis of Non-Hodgkin's lymphoma presents complaining of swelling of the neck and face, cough, and dyspnea on exertion. On exam you note dilated neck veins. Which of the following is the most appropriate initial treatment for this patient? A Beta-blockers B Glucocorticoids C IV fluids D Trendelenburg position E Vena cava stenting

Glucocorticoids The classic presentation signs and symptoms of superior vena cava syndrome (SVCS) are present in this patient. Patients with lymphoma often respond with a decrease in tumor mass and improvement in SVCS with glucocorticoid therapy. Beta-blockers (A) and vena cava stenting (E) aren't routinely used in the management of SVCS. Other initial symptomatic therapies include low salt diet, diuretics, head elevation, and diuretics. IV fluids (C) and Trendelenburg position (D) are contraindicated.

A 55-year-old woman presents with a slight cough. She has had the cough for about a week. She is a non-smoker, and she does not remember having a fever or feeling sick. Auscultation of the chest reveals clear lung fields. A chest X-ray shows a subpleural 'coin lesion' in the right upper lobe. What is the most likely diagnosis? Answer Choices 1 Small cell anaplastic carcinoma 2 Bronchiectasis 3 Silicosis 4 Granuloma 5 Exogenous lipid pneumonia

Granuloma The most likely diagnosis is granuloma. The differential diagnosis of a solitary coin lesion includes: Lung carcinoma (most commonly an adenocarcinoma) Granuloma Hamartoma A small cell carcinoma (or oat cell carcinoma) tends to spread very quickly and not remain localized. Moreover, it is common with a history of smoking. Silicosis results from years of inhaling dust containing silica, and it produces diffuse scattered nodules in the lungs. Bronchiectasis results from inflammation, destruction, and dilatation of the bronchi; it does not produce a round discreet lesion. It gives a typical honeycomb appearance on chest X-ray. Exogenous lipid pneumonia is due to aspiration of a substance that is oily or contains lipids. It is not typically localized.

A 33-year-old woman presents with weight loss in spite of a hearty appetite. At first glance, the physician notices her large eyes. She gives a history of increased anxiety over the last few months; it has been associated with palpitations and diaphoresis. During the examination, the physician notes tachycardia, hypertension, sweaty palms, and trembling outstretched hands. There is a diffuse swelling in the neck with a bruit heard on auscultation. What is the most likely diagnosis? Answer Choices 1 Hashimoto's thyroiditis 2 Hypothyroidism 3 Parathyroid adenoma 4 Lateral aberrant thyroid 5 Graves' disease 6 Hyperthyroidism

Graves' Explanation Graves' disease is an idiopathic autoimmune disorder associated with HLA-B8 and HLA-DR3. It is characterized by hyperthyroidism and a diffuse swelling of the thyroid (which moves with deglutition); hence, it also called diffuse toxic goiter. Hashimoto's thyroiditis and hypothyroidism are associated with lowered levels of thyroxine and its related symptoms, whereas this patient is showing signs of hyperthyroidism. The features that distinguish graves disease from other causes of hyperthyroidism are given below. Lateral aberrant thyroid is a unique presentation of carcinoma thyroid wherein the lymph node metastasis is detected before the primary. The most common clinical features of hyperthyroidism are: GIT: Wt. Loss Fatigue Anorexia Cardiorespiratory: Dyspnea on exertion Palpitations CNS: Emotional lability Tumor Hyperreflexia Dermatological: Palmar erythema Alopecia Increased sweating Reproductive: Amenorrhea / oligomenorrhea Infertility Impotence Decreased libido Other: Heat intolerance Hyper defecation Apathy Clinical features specific to Graves' Disease: Ocular: Lid lag Excess lacrimation Chemosis Exophthalmos Corneal ulceration Diplopia and ophthalmoplegia Papilledema Decrease visual acuity Dermatological: Vitiligo Clubbing Pretibial myxedema Other: Lymphadenopathy Diffuse goitre with bruit

A 35-year-old woman presents with a 2-month history of palpitation and nervousness. She mentions that she always feels hot, even if the weather is cold. Her menses have been irregular lately. She has had no fevers recently. She was also told that her eyes are "weird looking". On examination, her blood pressure is 150/70 mmHg, her pulse rate 89 beats per minute. Her eyes show exophthalmos, and she also has lid lag on looking down. Thyrotoxicosis is suspected. What follows is the thyroid scan result. Question What is the most likely diagnosis? Answer Choices 1 Graves' disease 2 Toxic multinodular goiter 3 Hashimoto thyroiditis 4 Factitious hyperthyroidism 5 Toxic adenoma

Graves' disease Explanation The most likely diagnosis in this patient is Graves' disease. Thyrotoxicosis occurs due to thyroid hormone excess, the etiology of which varies from Graves' disease, toxic multinodular goiter, toxic adenoma, thyroiditis, and even functioning metastasis. Symptoms of thyrotoxicosis include hyperactivity, irritability, dysphoria, heat intolerance and sweating, palpitations, fatigue, weakness, weight loss with increased appetite, diarrhea, polyuria, and menstrual irregularities. Graves' disease is the most common cause of thyrotoxicosis. It is an autoimmune disease in which thyroid-stimulating autoantibodies stimulate thyroid receptors to secrete thyroxine. It is more common in women and can be identified by radionucleotide scan in which the whole thyroid shows increased uptake. Here the patient has all Graves' disease characters: she is middle aged with thyrotoxic characters and her Tc-99 scan shows the characteristic Graves' pattern. Toxic multinodular goiter is another cause of thyrotoxicosis in which there are multiple nodules that can be felt with palpation, and it is shown to be hot or active with the radionucleotide scan. It shows localized or patchy uptake in the scan pictures. Hashimoto thyroiditis is an inflammatory condition that follows viral infection. It is associated with a brief period of hyperthyroidism that is followed by hypothyroidism, which may persist throughout life. On Tc scan, it shows decreased uptake. Factitious hyperthyroidism is due to ingestion of the thyroid hormone. Usually this happens in nurses, physicians, or those who have access to medicine. It can be detected by measuring T3 and T4. There is also decreased uptake on thyroid scan. Toxic adenoma is a benign neoplastic nodule, which actively secretes thyroxine. It shows localized or patchy uptake of Tc 99 on thyroid scan.

A 42-year-old man presents with acute abdominal pain. He describes the pain as constant and severe. Upon further questioning, he affirms that the pain radiates to his back. He leans forward in attempt to alleviate the pain. On physical examination, his temperature is 101 F and his pulse is 94. His abdomen is tender, but there is no guarding or rigidity. There is discoloration in the flanks and discoloration around his umbilicus. His lab results are as follows: TEST RESULTS REFERENCE RANGE Potassium 4.2 mEq/L 3.5 - 5.0 mEq/L Calcium 8.1 mg/100 mL 8.7 - 10.5 mg/100 mL Amylase 378 U/L 25 - 125 U/L Lipase 630U/L 30 - 210 U/L Question The discoloration of the flanks in this patient is indicative of what sign? Answer Choices 1 Grey Turner's sign 2 Cullen's sign 3 Lisker's sign 4 Kehr's sign 5 Blumberg's sign

Grey Turners Grey Turner's sign is ecchymosis of the flanks that can sometimes be seen with acute pancreatitis. This patient has signs and symptoms of acute pancreatitis,which include non-colicky abdominal pain that radiates to the back and fever. An elevated amylase, an elevated serum lipase,and hypocalcemia are all also consistent with pancreatitis. The most frequent underlying cause of pancreatitis is alcoholism or biliary disease. Cullen's sign is ecchymosis around the umbilicus that can sometimes be seen with acute pancreatitis. Lisker's sign is tibial bone tenderness that can sometimes be elicited with deep vein thrombosis. Kehr's sign is pain in the left shoulder secondary to splenic rupture. Blumberg's sign is abdominal rebound tenderness. This is an indication of peritoneal irritation.

Patients with a certain type of dwarfism (Laron dwarf) have normal or elevated levels of growth hormone; however, these patients still have the characteristics of dwarfs. What is the most likely explanation for their condition? Answer Choices 1 Growth hormone does not activate its receptor 2 Insulin-like growth factor receptor is defective 3 Somatostatin levels are very low 4 Insulin-like growth factor is overproduced 5 Insulin-like growth factor is rapidly degraded

Growth hormone does not activate its receptor Explanation Human growth hormone, also known as somatotropin, is a 191 amino acid polypeptide with a molecular weight of 22,000 and is the major hormone regulating growth in humans. It is synthesized as a 28,000-dalton precursor that is not biologically active. The gene for pre-growth hormone is found on chromosome 17. Growth hormone is part of a family of hormones that include prolactin and human chorionic somatomammotropin (human placental lactogen). Of the 3, only growth hormone has growth promoting activity. Growth hormone directly acts on cells via a receptor-signaling pathway to cause differentiation and the release of insulin-like growth factors (IGF's). The IGF's promote cell division. Growth hormone synthesis and secretion are regulated by somatostatin and growth hormone-releasing hormone (GHRH). GHRH is produced by the hypothalamus and acts on the anterior pituitary to promote growth hormone release. Somatostatin (growth hormone release-inhibiting hormone) inhibits growth hormone synthesis and secretion. Therefore, excess somatostatin could lead to decreased amounts of growth hormone. A deficiency in growth hormone causes a form of dwarfism. A 2nd type of dwarfism (Laron dwarf) is caused by defects in the growth hormone receptors. These patients have normal or elevated growth hormone levels but do not secrete IGF's due to the inability of growth hormone to properly signal an increase in IGF secretion. If these patients are administered IGF, growth stimulation is observed indicating the presence of functional IGF receptors.

A 29-year old patient presents with a 9-month history of recurrent hemoptysis. The patient suffered from cavitary tuberculous infection 5 years ago and was effectively treated with antituberculous drugs. A thin-walled cavity was seen over the right upper lobe when first diagnosed. At a later date, a progressive opacification of tuberculous cavity was seen. What organism is most likely responsible for such radiologic changes of a tuberculous cavity? Answer Choices 1 Secondary growth of staphylococci 2 Growth of Candida albicans might have filled the cavity 3 Growth of Aspergillus fumigatus might have filled the cavity 4 The cavity filled with granulation tissue and subsequent fibrosis 5 The cavity filled with desquamated epithelial cells of alveoli

Growth of Aspergillus fumigatus might have filled the cavity One of the complications of tuberculous cavity is the growth of Aspergillus fumigatus within the cavity (fungal ball). Diagnosis is made by isolation of Aspergillus hyphae in the sputum. IgG antibody against Aspergillus can be demonstrated in such patients.

A 55-year-old man with no significant past medical history presents for a routine evaluation and fasting bloodwork. He does not note any symptoms at this time. His physical examination reveals an obese body mass index with a waist circmference of 120 cm and a blood pressure of 140/90 mm Hg, but he is otherwise unremarkable. His fasting bloodwork is drawn. Question What laboratory finding would qualify a diagnosis of metabolic syndrome in this patient? Answer Choices 1 HDL value of 35 mg/dL 2 Total cholesterol of 230 mg/dL 3 Triglyceride value of 125 mg/dL 4 LDL measurement of 110 mg/dL 5 Fasting plasma glucose of 95 mg/dL

HDL value of 35 mg/dL Explanation The correct response is an HDL value of 35 mg/dl. According to the National Cholesterol Education Program and Adult Treatment Panel III, the diagnostic criteria of the metabolic syndrome include 3 or more of the following: • Central obesity: waist circumference >102 cm (M), >88 cm (F) • Hypertriglyceridemia: triglyceride level ≥150 mg/dL or specific medication • Low HDLcholesterol: <40 mg/dL in men and <50 mg/dL in women, or specific medication • Hypertension: blood pressure ≥130 mm Hg systolic or ≥85 mm Hg diastolic or specific medication • Fasting plasma glucose level ≥100 mg/dL or specific medication or previously diagnosed type II diabetes

A 72-year-old woman presents with a 2-week history of fever, cough, and excessive diuresis. The woman has diabetes mellitus, which is treated with glimepiride (Amaryl). Her fluid and food intake have been poor during this time as well. On physical examination, blood pressure is 98/58 mm Hg; pulse is 112/min; temperature is 100.6° F; and respirations are shallow and regular at 20/minute. On physical assessment, the patient is stuporous; skin and mucous membranes are dry; heart has a regular rate and rhythm without murmurs; and auscultation reveals rales in the left lung base. Question Based on this information, what is most likely causing the patient's condition? Answer Choices 1 Diabetic ketoacidosis 2 Hyperglycemic hyperosmolar state 3 Hypoglycemic coma 4 Lactic acidosis 5 Chronic renal failure

HHS Explanation The correct answer is a hyperglycemic hyperosmolar state (HHS). Most commonly seen in the elderly with type II diabetes mellitus, it is typically preceded by a few weeks of polyuria, weight loss, fatigue, and poor oral intake; this results in a change in mental status that can progress from confusion to lethargy, stupor, and coma. It is associated with dehydration and hyperosmolality that presents on physical examination with hypotension and tachycardia. It is usually precipitated by a serious infection (sepsis or pneumonia, which is most likely the case with this patient) or acute illness (stroke or myocardial infarction) in the postoperative period, or it is due to noncompliance. Lactic acidosis is an accumulation of lactate due to excess formation and decreased utilization. Although an increase in lactic acid may occur in HHS, it is not the cause of this patient's condition. It may be seen in patients with type II diabetes treated with glucophage (metformin) in renal insufficiency or when taken during periods of illness, when bedridden, or in conjunction with intravenous radiographic contrast. It is the most common cause of metabolic acidosis in hospitalized patients; it occurs with excess production (with shock due to poor perfusion), poor utilization (hepatocellular dysfunction), or decreased clearance of lactate (cirrhosis). The metabolic acidosis is accompanied by a characteristic, compensatory hyperpnea with long, deep breaths, which this patient does not exhibit. HHS results in a prerenal azotemia or acute renal failure, which is also a metabolic acidosis, but renal failure is not the underlying cause of this patient's condition. The fact that the patient has had polyuria over the past 2 weeks makes chronic renal failure unlikely. Hypoglycemia has an acute onset when blood glucose is <70 mg/dL, which triggers secretion of counter-regulatory hormones (glucagon, epinephrine, cortisol) and growth hormone to raise blood glucose levels. This response produces adrenergic symptoms such as palpitations, tremor, anxiety, hunger, or nausea. A blood glucose decrease to <45 - 50 mg/dL will result in neuroglycopenia, with symptoms of weakness, confusion, and combativeness, and if untreated, it may result in seizure, coma, or death. This patient had an insidious onset of symptoms that signify hyperglycemia, not hypoglycemia. Diabetic ketoacidosis is seen primarily in type I diabetics, and it is associated with nausea, vomiting, abdominal pain, acetone-odor breath, and Kussmaul breathing.

A 5-year-old boy presents with history of low-grade fever, headache, and intermittent colicky pain in the abdomen, which has been localized mainly around the umbilicus since yesterday. The child has vomited once. In between the episodes of pain, the child is playful. His symptoms are also accompanied by a maculopapular rash that is more confluent over the lower extremities and the buttocks. There is no itching. Some areas of the rash are turning purple and red, which is suggestive of a hemorrhagic rash. Both knees and ankles are swollen and tender, and there is edema of the hands and feet mainly in the dependent areas. Examination of the cardiovascular, respiratory system, and abdominal examination are essentially normal. Laboratory investigations show: Hb. - 10gm%, WBC. 11,000/cm Platelet count - 550,000/cmm. Serum IGA - 500 mg /dL (normal 14-159 mg/dLfor 2-5 years age group) Urine - Proteinuria++, RBCs++ Stool - RBC+ Question What is the most likely diagnosis? Answer Choices 1 Kawasaki disease 2 Systemic onset Juvenile rheumatoid arthritis 3 Systemic lupus erythematosis 4 Henoch-Scholein purpura 5 Polyarteritis nodosa

HSP Explanation The most likely diagnosis is Henoch-Scholein purpura, which is also known as anaphylactoid purpura. It is the most common cause of non-thrombocytopenic purpura in children. Boys are affected twice as frequently as girls. It is a common vasculitis of small vessels, with cutaneous and systemic manifestations. The systems primarily involved are the skin, gastrointestinal tract (GIT), and kidneys. The characteristic manifestation of the disease is the rash, which presents initially as a pink maculopapular rash, but progresses to petechiae and purpura, which typically is characterized clinically as palpable purpura. The rash may continue to appear intermittently for 3 or 4 months, or even up to 1 year. Edema and vasculitis of the GIT may lead to GI hemorrhage, manifesting with colicky pain in abdomen, vomiting, and hematemesis. There may be enlargement of mesenteric lymph nodes. Stool is positive for occult blood. Swelling of knee and ankle joints is frequently seen due to serous effusion. There may be edema of the dependent areas. Renal involvement, which is the most important cause of morbidity and mortality, manifests as hematuria, proteinuria, and hypertension. CNS and cardiac involvement may rarely occur. Laboratory findings include thrombocytosis, leukocytosis, and elevated ESR. Serum IgA levels are elevated. Urine examination shows albuminuria, hematuria, and presence of white blood cells and casts in the urine. Renal biopsy may show mesangial deposition of IgA. Diagnostic criteria of Kawasaki disease are fever of more than 5 days duration and presence of at least 4 of the following conditions: (1) Strawberry tongue (protuberance of tongue papillae) suggestive of streptococcal infection (2) Diffuse reddening of the oral and pharyngeal mucosa, dry and cracking lips. (3) Conjunctivitis without any discharge. (4) Edema/erythema of the hands and feet and later desquamation of the skin of the fingers and toes. (5) Polymorphous rash. (6) Cervical lymphadenopathy (at least one lymph node >1.5 cm). These features are not present in the above child. Systemic lupus erythematosis (SLE) is a multisystem disease involving nearly all the organs. It is an autoimmune disorder that causes inflammation of the blood vessels and connective tissue, resulting in multisystem involvement. It is seen more commonly in girls in contrast to HSP, which is more common in boys. Joints may be merely stiff or there may be active inflammation. Cutaneous manifestations include malar, or butterfly, rash involving the cheeks and nasal bridge. Rash may be photosensitive and may involve all sun exposed areas. This rash is quite different from the rash of Henoch-Schonlein purpura. Hepatosplenomegaly and lymphadenopathy are often present. Cardiac involvement may include pericarditis, valvular thickening, myocarditis, conduction abnormalities, and congestive cardiac failure. Pulmonary involvement includes pulmonary hemorrhage and fibrosis. This is in contrast to the index case. Renal involvement may manifest as hypertension, edema, electrolyte abnormalities, nephrosis, or acute renal failure. Systemic onset juvenile rheumatoid arthritis (JRA) may be characterized by spiking fevers, arthritis, hepatosplenomegaly, lymphadenopathy, and serositis leading to pericardial effusion. Fever is accompanied by a faint transient, evanescent salmon-colored macular rash more commonly over the trunk and proximal limbs. It is non-pruritic and may last for a few hours. Heat, even that of a warm bath, may cause reappearance of the rash. Lab investigation includes raised ESR, leukocytosis, thrombocytosis, and C-reactive proteins (CRP) and anemia of chronic disease. JRA is the most common chronic rheumatologic disease in children, with a minimum duration of 6 weeks. The new nomenclature juvenile idiopathic arthritis (JIA) is being increasingly used to better define various subgroups. Clinical manifestations of Polyarteritis nodosa (PAN) is a necrotizing vasculitis involving small and medium sized arteries. Boys and girls are equally affected. It is believed to be a post-infective autoimmune response in susceptible individuals commonly occurring after upper respiratory infection by group A streptococcal infection, chronic hepatitis B infection, infectious mononucleosis, and tuberculosis. Common features include fever, weight loss, and abdominal pain. Skin manifestations include purpura, edema, and painful nodules along the course of arteries. Cardiac involvement occurs as myocarditis, pericarditis, and arrhythmias. Angiography may show aneurismal dilatation and segmental stenosis.

A 32-year-old man is admitted to the hospital following loss of consciousness. The patient had been ill with fever and headache for several days, then developed double vision, confusion, and loss of consciousness. He has been otherwise healthy with no past medical conditions. Imaging reveals edema of the frontal lobe. Question Which virus most commonly causes the patient's infection? Answer Choices 1 Herpes simplex virus 2 Adenovirus 3 Influenza A 4 West Nile Virus 5 Epstein-Barr Virus

HSV Herpes simplex virus is correct. The patient is suffering from encephalitis, which is typically viral in nature. Herpes simplex virus is the most common viral cause of encephalitis. Adenovirus is a rare cause of encephalitis. Adenovirus more commonly causes respiratory and eye infections. Encephalitis is a rare complication of these conditions. Influenza A is a rare cause of encephalitis. Influenza A typically causes respiratory infection, which is infrequently complicated by encephalitis. West Nile Virus is a rare cause of encephalitis. While about half of all patients who are diagnosed with West Nile Virus have meningitis and encephalitis, the overall rate of West Nile Virus is low. Epstein-Barr Virus is incorrect. Epstein-Barr virus causes infectious mononucleosis. It is estimated that only 1-5% of cases of infectious mononucleosis have nervous system involvement.

A 32-year-old man presents with a 2-day history of high fever and progressive, severe, headaches. It is associated with blurred vision and increasing confusion. The patient is normally healthy, and he does not have a remarkable past medical history. He is married. He does not drink alcohol, smoke, or use illicit drugs. He has not had any blood transfusions, and he takes no medications. On physical examination, he appears ill and disorientated to time, place, and person. His pulse is 110/minute, temperature 39 degrees Celsius (102 degrees Fahrenheit), respirations 22/minute and blood pressure is 115/70 mmHg. He is well-hydrated. He has no scleral icterus or oral candidal infection. Pupils are equal and reactive. There are no palpable adenopathy or rashes. His cranial nerve examination is intact. His fundi are normal. He can move all of his limbs. Deep tendon reflexes are normal; plantar reflexes are equivocal. His neck is supple. His lab work includes Hemoglobin = 13. 1 g/dL; hematocrit = 39%; white blood cell count (WBC) = 18x109/L with 70% polymorphonuclear neutrophils and 30% lymphocytes; platelets = 200x109/L; erythrocyte sedimentation rate = 90 mm/hour; sodium = 138 mEq/L; potassium = 4 mEq/L; chloride = 102 mEq/L; blood urea nitrogen (BUN) = 29 mg/dL; creatinine = 1mg/dL; glucose = 101mg/dL; calcium = 9. 2 mg/dL; total bilirubin = 1mg/dL; alanine transaminase (ALT) = 26 units/L; aspartate transaminase (AST) = 22 units/L; alkaline phosphatase = 112 units/L. Results of drug screening are negative and his urinalysis is normal. The lumbar puncture results are as follows: Cerebrospinal pressure Increased Protein level 180 mg/dL Glucose 92 mg/dL White blood cell count 116 per microliter (all lymphocytes) Red blood cell count 80 per microliter Gram-staining results White blood cells with no organisms Oligoclonal bands Absent IgG index Normal Question An MRI scan of the brain demonstrates low-density lesions in the brain; they represent areas of gross demyelination. What is the most likely diagnosis? Answer Choices 1 Acute multiple sclerosis 2 Central nervous system lymphoma 3 Herpes simplex virus (HSV) encephalitis 4 Multifocal leukoencephalopathy 5 Viral meningitis

HSV encephalitis The patient has a fever, severe headache, altered mental status and has abnormal levels of lymphocytes in the CSF. These findings, along with the results of the MRI scan, suggest herpes simplex virus, encephalitis. HSV encephalitis occurs in 10% of all cases of encephalitis in the United States. There is a high mortality rate and morbidity rate. The latter is normal in the form of neurological damage. Herpes simplex type 1 is the most common cause and type 2 causes neonatal encephalitis (acquired by mother). The illness typically occurs over several days. The CSF findings in this patient typify the condition. Specifically, the CSF pressure is increased with pleocytosis. The red blood cells reflect the hemorrhagic nature of the CNS lesions. The protein level is increased; the glucose level may be normal or decreased. Tuberculosis or fungal meningitis is associated with decreased glucose levels, which can create confusion. Multiple sclerosis (MS) is a chronic CNS disease affecting young adults. There is the presence of demyelination and T-cell predominant perivascular inflammation in the brain white matter. Some axons may be spared from these pathological processes. Early symptoms may include numbness and/or paresthesia, mono- or paraparesis, double vision, optic neuritis, ataxia, and bladder control problems. Subsequent symptoms include upper motor neuron signs. Vertigo, incoordination and other cerebellar problems, depression, emotional lability, abnormalities in gait, dysarthria, fatigue and pain are also commonly seen. The presence of oligoclonal bands and/or elevated IgG index constitutes the basis for diagnosis. The most typical presentation of a primary CNS lymphoma is in an immunocompetent patient with progressive symptoms indicative of a mass lesion. Seizures may occur. Patients with AIDS are more likely to present with an encephalopathy than other patients with a primary CNS lymphoma. A careful sexual and drug abuse history is necessary. Isolated, ocular, or meningeal tumor may occur in the absence of any focal abnormalities on MRI. Multifocal leukoencephalopathy is a lethal secondary viral infection; it primarily occurs in AIDS patients with advanced immunodeficiency. The JC virus is a ubiquitous virus, able to enter the brain and lytically infect oligodendrocytes (the cells making myelin in the brain). This causes demyelination of the brain, which results in a wide variety of focal neurologic symptoms, including weakness, loss of sensation, visual loss, and changes in balance and coordination. Because it is usually relentlessly progressive, severe neurologic disability develops over a period of 2 to 6 months; death follows rapidly from general disability. A small percentage of persons stabilize or spontaneously improve. The lack of meningeal signs and the presence of demyelination on the MRI scan would not typically back up the diagnosis of viral meningitis.

A 76 year-old woman with steroid dependent chronic obstructive pulmonary disease is hospitalized with fever, chills, and a productive cough. The sputum gram stain shows many WBCs and small, pleomorphic gram-negative rods. Which of the following is the most likely causative agent? A Chlamydia pneumoniae B Haemophilus influenzae C Mycoplasma pneumoniae D Staph aureus E Strep pneumoniae

Haemophilus influenzae Haemophilus influenzae (B) is a gram-negative pleomorphic coccobacillus. Strep pneumonia (E) and Staph aureus (D) are gram positive organisms. Mycoplasma pneumonia (C) and Chlamydia pneumoniae (A) aren't visible on gram stain.

A 43-year-old male farmer from the Southwest United States has been working in a very contaminated barn with rodent feces for the last week. He presents to your office with complaints of fever, non-productive cough, malaise, and decreased appetite. His physical exam reveals a temperature of 102 0 F, pulse rate of 98, blood pressure of 98/62, and O 2 saturation of 93%. Lung sounds have diffuse crackles throughout, and the rest of the exam is unremarkable. Based on the history and exam findings, what is the most likely pathogen for this type of illness? A Influenza pneumonia B Varicella pneumonia C Hantavirus pneumonia D Streptococcal pneumoniae E Cytomegalovirus

Hantavirus The history of the patient being exposed to the rodent feces is a typical presentation of a pneumonia caused by the hantavirus. There is no treatment for this type of pneumonia, only supportive care.

A 10-year-old boy is referred to you for being fidgety at school, though he makes good grades. Prior history is unremarkable, and there has been no recent illness. The father had a similar history as a child. While speaking with the patient, you notice that he clears his throat several times per minute. Examination is otherwise normal, except for rapid, nonrhythmic jerking movements of the face, neck, and shoulders while at rest. Question What is true of this patient's case? Answer Choices 1 Does not have Tourette syndrome due to the absence of coprolalia 2 Has a factitious disorder 3 Has a condition that clusters in families 4 Must start a regimen of levodopa 5 Suffers from Sydenham chorea

Has a condition that clusters in families The combination of vocal tics (here, throat clearing) and motor tics should suggest Gilles de la Tourette (Tourette) syndrome, which is a common tic disorder that clusters in families. Once thought to be a single gene, autosomal dominant condition, it appears that multiple genes and nongenetic factors may be at play. Coprolalia is but one of many vocal tics seen in Tourette syndrome (throat clearing, humming, whistling) and, although a notorious one, is not a necessary feature. The positive family history makes it unlikely that the condition described is factitious. Pimozide, SSRIs, and other agents have been used in the treatment of Tourette syndrome, but levodopa is not used. The movement disorder of Sydenham chorea, as seen after a streptococcal infection, is choreiform - that is, tends to be slower and rhythmic, as opposed to the rapid, nonrhythmic jerks seen in Tourette syndrome. Also, vocal tics are not typically seen with Sydenham chorea.

A 32-year-old woman presents with constipation, weight gain, and dry skin. She has been experiencing the symptoms for a few months. Examination findings include dry rough skin, diffuse thyroid enlargement, bradycardia, and edema of hands and feet. A thyroid profile is performed and shows elevated thyroid stimulating hormone (TSH) and the presence of thyroid antibodies-antithyroid peroxidase (anti-TPO) and anti-thyroglobulin (anti-Tg) antibodies. The tissue biopsy microscopy is revealed in the image, with areas of interest indicated by the arrows. Question What is the most likely cause of these findings? Answer Choices 1 Hashimoto's thyroiditis 2 Riedel's Thyroiditis 3 Subacute painless thyroiditis 4 Suppurative thyroiditis 5 De Quervain's Thyroiditis

Hashimoto's Thyroiditis The presence of antithyroid antibodies and the typical histology points to a diagnosis of Hashimoto's thyroiditis (chronic lymphocytic thyroiditis). It is the most common form of thyroiditis, and most common cause of hypothyroidism in the US. This condition is characterized by thyroid cell destruction by various antibody-mediated immune processes. The gland appears diffusely enlarged and firm. Increased circulating levels of antithyroid peroxidase or anti-thyroglobulin antibodies is a diagnostic feature. Hashimoto's thyroiditis is a histological diagnosis characterized by infiltration of lymphocyte and plasma cells, reduced size of the thyroid follicles containing sparse colloid, and fibrosis along with presence of follicle with oxyphilic change called Hurthle cells. The image shows replacement of thyroid follicles by lymphoplasmacytic infiltrate with germinal centers (indicated by arrows). Riedel's thyroiditis is characterized by dense fibrosis, which results in a stony hard enlargement of the thyroid gland. It is the rarest form of thyroiditis. Histological features of Riedel's thyroiditis include a fibro-inflammatory process involving a portion or the entire thyroid gland and the presence of fibrotic extension into the adjacent structures beyond the thyroid capsule. Subacute painless thyroiditis is a self-limiting disease process having triphasic clinical presentation of hyperthyroidism, hypothyroidism, and return to euthyroid state. Thyroid biopsy shows presence of lymphocytic infiltration. Suppurative thyroiditis is rare and associated with severe anterior neck pain, tenderness, and redness in the region. It is an acute thyroiditis characterized by high fever and a swollen and tender thyroid gland. Lab findings include leukocytosis and increased erythrocyte sedimentation rate (ESR). De Quervain's thyroiditis, also called subacute granulomatous thyroiditis, is a painful enlargement of the thyroid gland. It is believed to be caused by viral infection or a post-viral inflammatory response. The histological finding in De Quervain's thyroiditis includes the presence of multinucleated giant cell granulomas.

A 32-year-old woman presents with constipation, weight gain, and dry skin. She has been experiencing the symptoms for a few months. Examination findings include dry rough skin, diffuse thyroid enlargement, bradycardia, and edema of hands and feet. A thyroid profile is performed and shows elevated thyroid stimulating hormone (TSH) and the presence of thyroid antibodies-antithyroid peroxidase (anti-TPO) and anti-thyroglobulin (anti-Tg) antibodies. The tissue biopsy microscopy is revealed in the image, with areas of interest indicated by the arrows. Question What is the most likely cause of these findings? Answer Choices 1 Hashimoto's thyroiditis 2 Riedel's Thyroiditis 3 Subacute painless thyroiditis 4 Suppurative thyroiditis 5 De Quervain's Thyroiditis

Hashimotos The presence of antithyroid antibodies and the typical histology points to a diagnosis of Hashimoto's thyroiditis (chronic lymphocytic thyroiditis). It is the most common form of thyroiditis, and most common cause of hypothyroidism in the US. This condition is characterized by thyroid cell destruction by various antibody-mediated immune processes. The gland appears diffusely enlarged and firm. Increased circulating levels of antithyroid peroxidase or anti-thyroglobulin antibodies is a diagnostic feature. Hashimoto's thyroiditis is a histological diagnosis characterized by infiltration of lymphocyte and plasma cells, reduced size of the thyroid follicles containing sparse colloid, and fibrosis along with presence of follicle with oxyphilic change called Hurthle cells. The image shows replacement of thyroid follicles by lymphoplasmacytic infiltrate with germinal centers (indicated by arrows). Riedel's thyroiditis is characterized by dense fibrosis, which results in a stony hard enlargement of the thyroid gland. It is the rarest form of thyroiditis. Histological features of Riedel's thyroiditis include a fibro-inflammatory process involving a portion or the entire thyroid gland and the presence of fibrotic extension into the adjacent structures beyond the thyroid capsule. Subacute painless thyroiditis is a self-limiting disease process having triphasic clinical presentation of hyperthyroidism, hypothyroidism, and return to euthyroid state. Thyroid biopsy shows presence of lymphocytic infiltration. Suppurative thyroiditis is rare and associated with severe anterior neck pain, tenderness, and redness in the region. It is an acute thyroiditis characterized by high fever and a swollen and tender thyroid gland. Lab findings include leukocytosis and increased erythrocyte sedimentation rate (ESR). De Quervain's thyroiditis, also called subacute granulomatous thyroiditis, is a painful enlargement of the thyroid gland. It is believed to be caused by viral infection or a post-viral inflammatory response. The histological finding in De Quervain's thyroiditis includes the presence of multinucleated giant cell granulomas.

The most common type of thyroiditis in the US is associated with increased circulating levels of antithyroid peroxidase or antithyroglobulin antibodies. The tissue biopsy is revealed in the image. With what condition is it most likely to be associated? Answer Choices 1 Hashimoto's thyroiditis 2 Riedel's Thyroiditis 3 Subacute thyroiditis 4 Suppurative thyroiditis 5 De Quervain's Thyroiditis

Hashimotos Explanation Hashimoto's thyroiditis is also known as chronic lymphocytic thyroiditis. It is the most common form of thyroiditis and probably any thyroid disorder in the US. The gland appears diffusely enlarged and firm. Increased circulating levels of antithyroid peroxidase or antithyroglobulin antibodies are a diagnostic feature. It tends to be familial, and it is 6 times more frequent among females. The image shows Hurtle cells with abundant pink cytoplasm and vesicular nuclei. The red arrow points to the lymphoid cell whereas the black arrow points to a hurtle cell. Riedel's thyroiditis is often a manifestation of a multifocal systemic fibrosis syndrome with a stony hard enlargement of the thyroid gland. It is the rarest form of thyroiditis. De Quervain's thyroiditis is also called subacute thyroiditis; it is an acute, painful enlargement of the thyroid gland. If there is no pain, it is called a silent thyroid. Suppurative thyroiditis is rare and associated with severe pain, tenderness, redness, and fluctuation in the region.

A 60-year-old hypertensive man presents with constipation; he states that he has not had a single bowel movement in the past 2 days. 1 month ago, he had a myocardial infarction, but he is now stable and is on a low-fat, low-salt diet. He He refuses a docusate sodium enema and is prescribed docusate tablets. Question What precaution should the patient take while using this laxative? Answer Choices 1 He should not use it long-term 2 It should be consumed with fruit juice only 3 He will have to increase his salt intake for the docusate to be effective 4 He should also consume mineral oil for the docusate to be effective 5 He may get dehydrated while using docusate

He should not use it long term Docusate should not be used long-term; generally it should not be used for more than 1 week. Docusate is a stool softener which acts in the small and large intestines by increasing the water and fat content in the stool; this facilitates its easier movement. It is ideally suited for short-term treatment of constipation because long-term use may increase the risk of a deficiency of fat-soluble vitamins due to decreased absorption. Other contraindications include appendicitis, acute surgical abdomen, intestinal obstruction, fecal impaction, and undiagnosed abdominal pain. Mineral oil should not be prescribed along with docusate as the former may be absorbed, leading to toxicity. Docusate tablets are taken with a glass of water. A high-salt diet is not required for docusate to be effective. Dehydration is a side effect of sodium picosulfate, which is a contact laxative used to treat constipation or to prepare the bowel prior to surgery.

A 24-year-old man is brought in by his roommates for a seizure. The patient has alternative episodes of consciousness and is not currently seizing. The roommates give a history of the patient being in previously excellent health and studying exercise science in college. He first complained of feeling ill about 3-4 days ago, with headache, fever, and malaise. By last night, the roommates noted the man was not making sense in conversation, having trouble swallowing, and appeared more lethargic. Early this morning, he also had a witnessed seizure. Vitals are shown in the table. Weight 182# Height 72" Pulse 106 Blood pressure 102/58 Temperature 104.1°F/40.1°C On physical exam, the patient appears lethargic and intermittently responds to loud voice stimulus. He does not verbalize any answers and appears to have a hemiparesis. Other than his abnormal neurologic status and mild tachycardia, the remainder of his physical exam is normal. Multiple tests are done and results are shown in the table. Urinalysis Normal Complete blood count Mildly elevated white blood cell count (WBC) Comprehensive metabolic panel Normal C-reactive protein Elevated Toxicology screen Negative Cerebrospinal fluid analysis Increased lymphocytes and protein, normal glucose MRI of head Some areas of increased signal, no masses Brain biopsy Edema, inflammation, Negri bodies Question If the patient was able to supply an accurate, complete history by himself, which of the following aspects in this patient's history would best explain his current condition? Answer Choices 1 He had experienced multiple concussions while playing football as a teen. 2 He had recently started a new oral medication for acne. 3 He was the victim of a bat bite while spelunking. 4 His father had Huntington disease. 5 His mother consumed large amounts of alcohol throughout his fetal gestation.

He was bit by a bat This patient presents a very concerning clinical picture for encephalitis. Encephalitis classically presents with fever, headache, altered mental status, and a variety of other neurologic signs and symptoms. It is caused by a variety of viral organisms, ranging from herpesvirus, influenza, HIV, adenovirus, and many others. It can be transmitted by a variety of vectors worldwide, and history is vital to establishing possible etiologies. If this patient was able to report he was the victim of a bat bite while spelunking, that history would be most supportive of a diagnosis of rabies encephalitis. Encephalitis is a complicated condition with a variety of findings, but the Negri bodies on brain biopsy are pathognomonic for rabies. If he had experienced multiple concussions while playing football as a teen, the patient may be suspected of having postconcussion syndrome and/or mild traumatic brain injury (TBI). Both are associated with headaches and possible seizures, but the timing would be much different. Neither is associated with fever, as with this patient. Some medications, such as isotretinoin, could produce neurologic and psychiatric adverse effects ranging from psychosis, seizures, and stroke. If this patient had recently started a new oral medication for acne, medication side effects would not explain his fever and inflammatory response or the Negri bodies. If his father had Huntington disease, an autosomal dominant neurodegenerative disorder, this patient would be expected to have a much less acute presentation. While seizures and cognitive deficits are possible with Huntington disease, this patient's presentation is much more suggestive of encephalitis. If his mother consumed large amounts of alcohol throughout his fetal gestation, fetal alcohol syndrome may be suspected. Cognitive deficits and seizures are associated with this syndrome, but abnormalities would have been present since infancy and not with an acute onset.

Some temporary disease processes are unique to pregnancy and may occur in one or all of a women's pregnancies. Which of the following would raise suspicion for the first phase of toxemia referred to as pre-eclampsia? Answer Choices 1 Headache, visual disturbances, abdominal pain, vomiting 2 Sinus pain, chest pain, bowel disturbances 3 Chest pain, shortness of breath, pain in arms 4 Bowel disturbances, pain in arms, pain in back 5 Face swelling, edema of the limbs, renal failure

Headache, visual disturbances, abdominal pain, vomiting Explanation Pre-eclampsia, the first stage of toxemia, exhibits the symptoms of headache, visual disturbances, abdominal pain, and vomiting. These are often overlooked as normal pregnancy symptoms. The patient should be monitored for further symptoms. Chest pain, shortness of breath, and pain in the arms would raise the suspicion of cardiac problems and should not be ignored.

A 36-year-old woman with a past medical history of diabetes mellitus presents to her family medicine office with complaints of weight gain and skin changes. Her review of systems is positive for menstrual irregularities with extended periods of amenorrhea, infertility, depression, cognitive dysfunction, and emotional lability. Of late, her fasting glucose levels have been above normal. Her physical exam notes increased adipose tissue in the face, upper back, and above the clavicles. Her skin reveals ecchymoses, telangiectasias, and purpura along her back and lower extremities, facial acne, and cutaneous atrophy. Her abdominal exam reveals the following image. Question Which of these is correct regarding this patient's condition? Answer Choices 1 Strengthened bone and enhanced immune function are typical. 2 Bilateral adrenal hyperplasia is the leading cause of this presentation. 3 This illness is more commonly encountered in males. 4 In males with this condition, basal gonadotropin function is unaffected. 5 Headaches, polyuria, vision loss, or galactorrhea suggest a pituitary source.

Headaches, polyuria, vision loss, or galactorrhea suggest a pituitary source. This patient is exhibiting signs and symptoms consistent with Cushing's syndrome. Most cases of Cushing syndrome are due to exogenous glucocorticoids. Endogenous glucocorticoid overproduction or hypercortisolism that is independent of ACTH is usually due to a primary adrenocortical neoplasm (usually an adenoma but rarely a carcinoma). ACTH-secreting neoplasms cause ACTH-dependent Cushing syndrome. In 80% of cases, this is due to an anterior pituitary tumor and referred to as classic Cushing disease. Patients with an ACTH-producing pituitary tumor present with headaches, polyuria, nocturia, visual problems, or galactorrhea . Exposure to excess glucocorticoids results in multiple medical problems, including hypertension, obesity, osteoporosis, fractures, impaired immune function, impaired wound healing, glucose intolerance, and psychosis. Bilateral micronodular hyperplasia and macronodular hyperplasia are rare causes of Cushing syndrome. In men, inhibition of LHRH and FSH/LH function may lead to decreased libido and impotence.

A 48-year-old woman presents after a seizure. Prior to the seizure, she experienced confusion and disorientation preceded by nausea, vomiting, and blurred vision. Symptoms appeared after working for several hours in the garden under the sun. Her medical history is significant for the presence of schizophrenia, for which she takes chlorpromazine at bedtime. Her temperature is 41 C; BUN and creatinine are elevated; and there is neutrophilia, hemoconcentration, and lactic acidosis. You think that the event is possibly drug-related. Question What is the most likely diagnosis? Answer Choices 1 Heat cramps 2 Neuroleptic malignant syndrome 3 Heat stroke 4 Malignant hyperthermia 5 Heat exhaustion

Heat stroke Heat disorders can be exertional and nonexertional. Both can be drug-related. Neuroleptics (e.g., phenothiazines, thioxanthenes) may impair thermoregulation due to both anticholinergic and antidopaminergics effects. Anticholinergics inhibit sweating, therefore disturbing thermoregulation during exercise or under conditions of environmental heat stress. Antidopaminergics elevate the set point of the temperature regulation center in hypothalamus. Your patient most probably suffered heat stroke. It is a life-threatening condition characterized by elevated body temperature with nausea, blurred vision, confusion, disorientation, and seizures. Hemoconcentration, anuria, rhabdomyolysis, kidney dysfunction, lactic acidosis, and even disseminated intravascular coagulation may result. Heat cramps are a mild disorder characterized by painful muscle contractions due to temporary fluids and electrolytes depletion. There are no signs and symptoms of neurological dysfunction, and body temperature is normal. Neuroleptic malignant syndrome is an idiosyncratic reaction to neuroleptics, most commonly phenothiazines and butyrophenones, and it is characterized by rigidity, fever, and autonomic instability. It is not connected with the exposure to heat and exertion. Men under 40 are at greatest risk. Malignant hyperthermia is a nonexertional idiosyncratic reaction to the anesthetic. Heat exhaustion is a condition with a severity that lies between heat cramps and heat stroke. Body temperature might be slightly elevated, and there may be neurological signs like headache, but there will be neither severe confusion nor seizures.

A 50-year-old man presents with weakness and abdominal discomfort. Upon questioning for medical history, he acknowledges a lack of sexual desire. He denies any photosensitivity. On physical examination, his liver is enlarged. He has an abnormal skin pigmentation that is located on his face, neck, and elbows; it gives his skin a gray-brown hue. His laboratory results are in the chart. TEST RESULTS REFERENCE RANGE TIBC 275 μg/dL 250 - 375 μg/dL Plasma iron 220 μg/dL 50 - 150 μg/dL Transferrin saturation 90% 20 - 40% Question What is the most likely diagnosis? Answer Choices 1 Hemochromatosis 2 Lead poisoning 3 Wilson's disease 4 Protoporphyria 5 Iron deficiency anemia

Hemochromatosis This patient has signs and symptoms consistent with hemochromatosis. Hemochromatosis is due to an increase in iron within the tissues. The liver is commonly affected, with hemochromatosis; hepatomegaly can be seen. There is an abnormal skin pigmentation that can be seen with hemochromatosis. Plasma iron will be elevated, total iron binding capacity will be normal or low, and transferrin saturation will be elevated. Lead poisoning can cause abdominal pain; however, the laboratory results of this patient are not consistent with lead poisoning. Wilson's disease is due to an accumulation of copper. The liver is also affected with Wilson's disease. The laboratory tests in this scenario do not support a diagnosis of Wilson's disease. Ceruloplasmin is the blood test that should be done in evaluation of a patient for Wilson's disease. Protoporphyria is due to a defect in heme synthesis and metabolism. Photosensitivity is seen with protoporphyria. The lab results are not consistent with iron deficiency anemia. Serum TIBC would likely be increased in iron deficiency anemia, not normal (as in the case here). With iron deficiency anemia there also would be a decrease in serum iron level, not an increase (as is seen in this case).

A 12-year-old Jewish girl is brought in by her father. The father reports that the child started to bleed profusely following tooth extraction 2 hours ago. Further questioning reveals that there is a history of bleeding disorder in the child's maternal uncle and aunt. Examination reveals tachycardia and bleeding from the site of tooth extraction. Coagulation profile reveals prolonged activated partial thromboplastin time (aPTT), normal prothrombin time (PT) and thrombin time (TT), and deficiency of factor XI. The child was treated with fresh frozen plasma (FFP) and fibrin glue. Question What is the likely diagnosis? Answer Choices 1 Thrombocytopenia 2 Hemophilia B 3 Von Willebrand's disease 4 Hemophilia A 5 Hemophilia C

Hemophilia C Hemophilia C is an inherited autosomal disorder characterized by deficiency of factor XI and is common in Ashkenazi or Iraqi Jews. Patients are usually asymptomatic and spontaneous bleeds into joints, soft tissues, and muscles are rarely seen in Factor XI deficiency. Bleeding usually occurs after tooth extractions, trauma, or surgery, such as tonsillectomy. Hemophilia C should be suspected in patients with prolonged aPTT and a positive family history of life-long bleeding disorder affecting both the male and female family members. This condition is primarily treated with FFP and other adjunctive measures such as fibrin glue and antifibrinolytics. Replacement with Factor XI concentrates can be done when available.

A 55-year-old female presents with a several-month history of increasing cough and dyspnea. She also has increased serum urea, nitrogen, and serum creatinine. A chest X-ray shows multiple bilateral small nodules. A renal biopsy shows a focal necrotizing vasculitis; her antineutrophil cytoplasmic autoantibody (ANCA) test is positive at 1:160. What additional finding would be most likely to occur? Answer Choices 1 Angina 2 Hemorrhagic pericarditis 3 Endocarditis 4 Hemoptysis 5 Hemothorax

Hemoptysis The history of upper airway involvement (cough with dyspnea), the nodular pattern on X-ray, and the renal biopsy showing focal necrotizing vasculitis suggests the diagnosis of Wegener's granulomatosis. Pulmonary (as well as upper respiratory tract) involvement with Wegener's granulomatosis commonly leads to hemorrhage, which manifests as hemoptysis. Although the etiology is unknown, it is believed to represent an antigen-triggered immunologic reaction. Pulmonary manifestations include cough, dyspnea, chest discomfort, and hemoptysis. In most cases, the extrarenal disease precedes the onset of renal disease. Although the heart may be involved with Wegener's granulomatosis, it does not usually lead to symptoms of ischemia (angina). The vasculitis does not typically involve the pericardium extensively; therefore, hemorrhagic pericarditis would not be a finding. The renal failure could lead to a fibrinous pericarditis. Endocarditis is not a feature of Wegener's granulomatosis. The vasculitis of Wegener's granulomatosis involves very small arteries and capillaries. Hemothorax is more typical of trauma.

A 42-year-old man presents with severe abdominal pain in the mid-epigastric region. The pain began 3 days ago with an intensity of 10/10; there is concomitant nausea and vomiting (on 2 occasions). The patient denies any significant past medical history, but he states that he "only" drinks 10 beers a day, and he has been doing so for the past 15 years. BP is 86/58. Pulse is 112, and temp is 101.5F. There is rebound tenderness in the midepigastric area, absent bowel sounds, and a positive Cullen's sign. The rest of the physical exam is within normal limits. In cases of acute pancreatitis, a positive Cullen's sign suggests what complication? Answer Choices 1 Perforated viscus 2 Pancreatic abscess 3 Hemorrhagic pancreatitis 4 Pancreatic ascites 5 Pancreatic pseudocyst

Hemorrhagic pancreatitis The correct response is hemorrhagic pancreatitis. More than 70% of cases of acute pancreatitis are due to alcohol abuse or gallstones. Complications, such as adult respiratory distress syndrome, hemorrhagic pancreatitis, pancreatic abscess, pancreatic pseudocyst, and pancreatic ascites, among others, account for the 10% mortality rate associated with acute pancreatitis. Hemorrhagic pancreatitis can lead to retroperitoneal hemorrhage and widespread tissue necrosis, and it may require surgical intervention or peritoneal lavage. This retroperitoneal bleeding can be responsible for the discoloration or hematoma in the periumbilical area (Cullen's sign).

A 37-year-old woman presents with a persistent flu-like illness, malaise, anorexia, nausea, and vomiting. The patient is a non-smoker; she drinks alcohol moderately, and she has no remarkable travel history. She admits to using parenteral drugs during her college life. Examination shows icteric sclerae, splenomegaly, right upper quadrant tenderness, and dark urine. What has the highest risk of chronic infection in adults? Answer Choices 1 Hepatitis A virus 2 Hepatitis B virus 3 Hepatitis C virus 4 Hepatitis D virus 5 Hepatitis E virus

Hep C Hepatitis C is a viral infection caused by an enveloped RNA virus belonging to the flaviviridae family. It is transmitted parenterally by sharing needles during parenteral drug use as well as from mother to child. The medical staff is also at risk of infection and sexual transmission can also occur. Other groups at risk of infection are those undergoing hemodialysis and recipients of organ transplant before 1992. Blood, blood products, and organ donors have been screened for hepatitis C since 1992 and the risk has been minimized in recipients now. Acute hepatitis C is asymptomatic in 60 - 70% of patients. Chronic infection develops in 55 - 85% of patients, chronic liver disease in 70% of patients, and cirrhosis in 5 - 20% of patients. Combination therapy with pegylated interferon and ribavirin is the treatment of choice resulting in sustained response rates of 40 - 80%. Hepatitis C is the leading indication for liver transplant in the US. Hepatitis A is transmitted by the feco-oral route and it has no chronic form. Once infected, it provides immunity for life. A vaccine is also available for travelers to endemic areas and other susceptible groups. Hepatitis B is also transmitted parenterally and sexually. The risk of chronic infection varies with age; for those over 5 years of age, it is about 2 - 6%. Infants have a risk of chronic infection of 90%, and between 1 - 5 years it is about 30%. Hepatitis D occurs as a co-infection or superinfection in patients infected with hepatitis B. It cannot infect a person by itself because it requires the envelope protein of hepatitis B. Chronic HBV carriers who acquire HDV superinfection usually develop chronic HDV infection. Hepatitis E is transmitted feco-orally, and the most common source is contaminated water. The acute hepatitis is most severe in pregnancy, especially the 3rd trimester. It has no chronic form. Hepatitis A and E are rare in the United States.

A 58-year-old woman with a past medical history of hypertension, hyperlipidemia, breast cancer, hip fractures, and coronary artery disease is being evaluated for acute-onset, severe left-sided pleuritic chest pain over the course of the last 2 hours. The pain is associated with feelings of anxiety, hemoptysis, shortness of breath, and nausea. She "feels warm" but denies chills, palpitations, wheezing, cough, edema, vomiting, abdominal pain, abnormal bowel habits, or dietary intolerances. She endorses a 30 pack/year smoking history, but denies drug or alcohol use. Upon physical exam, she is found to be febrile, hypotensive, tachycardic, tachypnic, diaphoretic, and in acute painful distress. There is perioral cyanosis and a pleural friction rub to the left lung fields; the remainder of the exam is normal. Question What is the most appropriate therapeutic intervention for this patient at this time? Answer Choices 1 Indomethacin 2 Doxycycline 3 Heparin 4 Prednisolone 5 Albuterol

Heparin This patient's presentation is significant for a pulmonary embolism. Predisposing underlying conditions are almost always present; venous thrombosis may result from a generalized hypercoagulable state, venous endothelial injury, or local stasis (Virchow triad). Most commonly, the initial manifestations of pulmonary embolism include an abrupt dyspnea and chest pain. Tachycardia and hypoxia are the most common clinical signs. Associated manifestations include fever, hypotension, cyanosis, pleural friction rub, and findings consistent with pulmonary consolidation. Anticoagulation is the foundation for successful treatment of DVT and PE. Unfractionated heparin, low-molecular-weight heparin, or fondaparinux all achieve effective anticoagulation immediately.

A 28-year-old woman presents with a 5-day history of progressive jaundice, vomiting, nausea, and malaise. She has developed a fever and has a 2-day history of dark urine. She has no history of intravenous drug use and has had no recent transfusions; she denies any sexual contact in the last 3 months. The patient has recently returned from a trip to Mexico where she consumed various foods from street vendors. Physical examination reveals a jaundiced patient. An abdominal examination is significant for a palpable liver edge 2 cm below the right costal margin and a total liver span of 12 cm below the mid-clavicular line. Blood is drawn for cultures, a complete blood cell count, and liver function tests. Blood studies are significant for a direct bilirubin of 13.0 mg/dL, aspartate aminotransferase of 1,800 U/liter, and alanine aminotransferase of 2,500 U/liter. Blood cultures are negative and serological studies are conclusively diagnostic. What is the most probable cause of the patient's symptoms? Answer Choices 1 Hepatitis A 2 Norwalk virus 3 Rotavirus 4 Adenovirus 5 Parvovirus

Hepatitis A Hepatitis A is a member of the Picornaviridae family; it is a nonenveloped virus approximately 27 - 28 nm in diameter. It is spherical, with a surface structure that suggests icosahedral symmetry. It is a single-stranded RNA virus. Signs and symptoms of an infection include diarrhea, dark urine, jaundice, and flu-like symptoms. Incubation is 15 - 50 days, with an illness duration of 2 weeks - 3 months. The virus is associated with shellfish harvested from contaminated waters, raw produce, uncooked foods, and cooked foods that are not reheated after contact with an infected food handler. Confirmation is based on the presence of anti-hepatitis A antibodies, positive IgM, and increases in serum ALT and bilirubin. Treatment is supportive, and prevention is by vaccination. Norwalk virus is a single-stranded RNA virus that is approximately 29 - 35 nm in diameter with spiky ill-defined outlines. The virus produces gastroenteritis-like symptoms that include nausea, vomiting, and watery/large volume diarrhea. Fever is usually absent. The incubation period is 24 - 48 hours, and the illness duration is 24 - 60 hours. Sources of infection include poorly cooked shellfish and ready-to-eat foods (e.g., salads and sandwiches) touched by infected food handlers. Diagnosis is based on negative stool cultures, stool that is negative for white blood cells, and serological studies for the presence of fourfold or greater increases in antibody titers of Norwalk virus antibodies. Treatment is mainly supportive; bismuth sulfate may be given. Rotavirus is a double-stranded RNA virus belonging to the Reoviridae family. The virus is 70 nm in diameter and appears as a wheel. The virus infection has signs and symptoms that include vomiting, watery diarrhea, and low-grade fever. There may be temporary lactose intolerance. The virus is especially common in infants and children, the elderly, and the immunocompromised. The incubation period is 1 - 3 days, and the duration of illness is 4 - 8 days. The virus is associated with ready-to-eat foods touched by infected food handlers (e.g., salads and fruits) and fecally-contaminated foods. Confirmation is based on the detection of the virus antigen in stool. Stool routine cultures are negative, and stools are negative for the presence of white blood cells. Treatment is supportive in nature. Fluid electrolyte replacement may be needed in severe cases of diarrhea, especially in infants. Adenovirus is a virus with DNA as the genetic material; it is approximately 70 nm in diameter. The virus causes gastroenteritis; symptoms include vomiting, nausea, malaise, diarrhea, headache, and fever. Incubation is 10 - 70 hours, with a duration of 2 - 9 days for symptoms. The organism is associated with ingesting food contaminated with feces and ready-to-eat foods touched by infected food workers. The virus is also found to be transmitted through the ingestion of some shellfish. The virus can be detected antigenically in acute stool samples. Stool staining for white blood cells is negative. Supportive care is the treatment of choice. The disease is usually mild and self-limiting. Parvovirus is a single-stranded DNA virus. Infections with this virus are mostly asymptomatic, although the virus produces erythema infectiosum (fifth disease) in small children (a benign rash appears on the face, trunk, and extremities). A fever also develops. The disease can cause intrauterine infection and fetal death due to the vertical transmittance of the virus from mother to fetus. In individuals with hemoglobinopathies, there can be transient aplastic crisis. Immunodeficient patients can develop chronic anemia.

A 50-year-old man presents with a 3-month history of weakness, fatigue, and abdominal discomfort. Upon further questioning, he acknowledges a lack of sexual desire. He denies any photosensitivity. On physical examination, his liver is enlarged, and his spleen is palpable. He has abnormal skin pigmentation on his face, neck and his elbows and which gives his skin a metallic gray hue. His laboratory results are in the chart. TEST RESULTS REFERENCE RANGE TIBC 275 μg/dL 250 - 350 μg/dL Plasma iron 220 μg/dL 80 - 160 μ/dL Transferring saturation 90% 16 - 57% Question What serious complication is associated with the patient's condition? Answer Choices 1 Hepatocellular carcinoma 2 Bronchogenic carcinoma 3 Pancreatic carcinoma 4 Lymphoma 5 Leukemia

Hepatocellular carcinoma This patient has signs and symptoms consistent with hemochromatosis, which is associated with hepatocellular carcinoma. Hemochromatosis is due to an increase in iron within the tissues. Hemochromatosis is a disorder of iron overload; it could be due to genetic or non-genetic causes. In hereditary hemochromatosis, there is absorption of a few milligrams of iron each day, in excess of need. As such, clinical manifestations often occur only after the age of 40 years, when body iron stores have reached 15 to 40 g. The symptoms of hemochromatosis are nonspecific and include arthralgia; abdominal pain; fatigue, weakness; impotence; weight loss; amenorrhea and early menopause; abnormal skin pigmentation; damage to the pancreas leading to diabetes; cardiomyopathy; and cirrhosis. The liver is commonly affected with hemochromatosis, and hepatosplenomegaly is commonly seen. There is an abnormal skin pigmentation that is seen with hemochromatosis. In hemochromatosis, the plasma iron will be elevated; total iron binding capacity will be normal or low; and transferrin saturation will be elevated. Hepatocellular carcinoma is the most serious complication, and it is a major cause of death in patients with hemochromatosis. A large percentage of patients with hemochromatosis will develop problems with their pancreas; however, the pancreatic pathology commonly seen with hemochromatosis is insulin dependent diabetes, not pancreatic cancer. Bronchogenic carcinoma, pancreatic carcinoma, lymphoma, and leukemia are not known complications of hemochromatosis.

A 41-year-old woman presents due to worsening symptoms. She was diagnosed with idiopathic pulmonary hypertension about 2 years prior to presentation; she is on home oxygen therapy. She has longstanding fatigue and dyspnea, but she is now experiencing profound dyspnea with exertion; swelling in her ankles; some discomfort in her right, upper abdomen; and the inability to breathe well when lying down. She has always been thin, but her weight has increased by 10 pounds in the last month. She denies fever and chills. She recently had an electrocardiogram (ECG), but she has not seen a healthcare provider to discuss the results. The ECG report indicates peaked p waves, right axis deviation, and tall R wave in V1. Question Based on this patient's history and test results, what physical exam findings would be expected? Answer Choices 1 Abdominal bruit 2 Absent breath sounds in right lung 3 Dry mucus membranes and reduced skin turgor 4 Hepatojugular reflux 5 Virchow's node enlargement

Hepatojugular reflux This patient is presenting with a progression of a primary pulmonary disease (pulmonary hypertension) into cor pulmonale, which is also known as pulmonary heart disease. Cor pulmonale, when moderate to severe, will present with signs and symptoms of right heart failure, such as hepatojugular reflux, peripheral edema, prominent P2, hepatomegaly, cyanosis, wheezing, and ascites. The ECG findings indicate right ventricular hypertrophy, which is a complication of progressing primary lung disease. Depending on the location in which the bruit was auscultated, an abdominal bruit might indicate atherosclerosis in the renal arteries, aorta, or iliac arteries. Atherosclerosis is typically seen in much older patients. This patient's pulmonary hypertension is not directly associated with atherosclerosis, and bruits would not be expected with her physical exam. Absent breath sounds are a finding associated with several pulmonary conditions, such as pneumothorax or a severe pneumonia; they are also seen in patients with a history of pneumonectomy. A pneumothorax or pneumonia might have some aspects in common with this patient's history (such as dyspnea), but they would not be associated with the fluid retention and ECG changes. Dry mucus membranes and reduced skin turgor are indicators of severe dehydration. Nothing about this patient's history is consistent with dehydration. On the contrary, this patient is in a fluid-overload state. Virchow's node enlargement indicates left-sided supraclavicular lymphadenopathy. This finding is a harbinger for malignancy of the thorax. The patient's symptoms do not suggest malignancy. If malignancy was present, depending on the primary type of cancer, the patient might exhibit unexplained weight loss and possibly night sweats.

You are treating a 14-year-old female for gonorrhea. She has a past history of Chlamydia. Her LMP was 3 weeks ago. She is allergic to penicillin. Her only medication is oral combination birth control pills. Why should Levofloxin be avoided in this patient? A Her age B Her LMP status C Her history of Chlamydia D Her oral contraceptives E Her penicillin allergy

Her Age Fluoroquinolones are contraindicated in pediatric patients. The risk-benefit assessment indicates that levaquin is only appropriate in pediatric patients at least 6 months of age, for treatment of inhalational anthrax (post-exposure). There are other medications, both oral and injectable, with better safety profiles that are available.

A 16-year-old girl presents with papillary thyrocarcinoma. Your surgical plan is bilateral thyroidectomy. The procedure is uneventful. Histopathology indicates that the tumor was completely excised along with the thyroid. You schedule your patient for post-surgical I131 to destroy any remaining tumor cells and thyroid tissue. 2 days post-operatively, your patient has difficulty speaking and swallowing. While you are examining her, she begins to have laryngeal spasms, experiences difficulty breathing, and goes into respiratory arrest. What happened? Answer Choices 1 She had an allergic reaction to the I131 2 Perturbation of certain tumors can stimulate them to grow rapidly. The tumor probably spread to the respiratory centers of the brain 3 Lack of T3/T4 in the blood has slowed her metabolism causing laryngeal spasms 4 Her parathyroid glands were either removed during the procedure or severely damaged 5 Because most anesthetic agents are lipid soluble, she is probably reacting to anesthetic agents that were stored in her body fat and released as she began to breakdown her fat stores

Her parathyroid glands were either removed during the procedure or severely damaged Explanation Lack of parathyroid activity will result in rapid depression in serum concentrations of calcium. I131 does not have the characteristics of an allergen. Although some tumors may grow rapidly when manipulated, papillary thyrocarcinoma is very slow-growing and not likely to metastasize in 2 days. The half-life of T3/T4 in the body is approximately 6 weeks. Although some anesthetic agents may be stored in body fat, they are released in relatively small amounts.

A 37-year-old female presents to the clinic for her obstetrical appointment. She is a G5P4 African American female. On physical exam her uterus is larger than expected for dates by 5 cm. What is her risk of having twins increased by? A Her increasing age B Her increasing parity C Her race D Her use of clomifine E Her late age of menarche

Her use of clomifine Incidence of multiples is not related to age, parity, or menarche; the increase seen in the last few decades is solely related to the increased use of fertility drugs.

A 72-year-old man presents to the outpatient clinic in follow-up for his dyspnea and cough. He reports shortness of breath, especially with activity, and a cough, which is non-productive. Symptoms have been present for 1 year, and they are getting worse. He initially went to the cardiologist for heart concerns, but no cardiovascular disease was found. A chest X-ray was ordered, and the patient reports it showed no masses in his chest. The patient denies any other symptoms, including fever, chills, night sweats, chest pain, and weight loss. The patient is a retired salesman; he fishes as a hobby. He lives at home with his wife; he denies use of tobacco, alcohol, and drugs. He denies any out of the country travel. On physical exam, the patient sits comfortably with normal respiratory effort. Auscultation of his lungs reveals fine crackles in both bases. A dry cough is noted a few times. Cardiovascular exam, including heart and extremities, is normal, except for clubbing of the fingers bilaterally. Question What test is the next most appropriate in evaluation of this patient's condition? Answer Choices 1 Echocardiogram 2 High-resolution computed tomography (CT) 3 Mantoux test or purified protein derivative (PPD) 4 Ultrasound of the thorax 5 Ventilation-perfusion (VQ) scan

High resolution CT This patient has a likely diagnosis of idiopathic pulmonary fibrosis (IPF). This is a relatively rare disease and can be difficult to diagnose. Patients typically have symptoms for 1 - 2 years prior to definitive diagnosis. IPF is a chronic, progressive restrictive pulmonary disease of the lung parenchyma. Unlike pneumoconiosis, there are no known occupational exposures (hence the term idiopathic). IPF presents with exertional dyspnea and a non-productive cough. A high-resolution computed tomography (CT) and/or a lung biopsy are the means for definitive diagnosis. The CT may show reticular opacities, honeycombing, and perhaps a 'ground glass' appearance in the lung tissue. There are heterogeneous areas of diseased lung interspersed with healthy tissue. An echocardiogram is not helpful in diagnosing IPF. It can be assumed that evaluation by a cardiologist would have determined if an echocardiogram was necessary. This patient was cleared by a cardiologist and has only pulmonary complaints (dyspnea and cough, with findings of lung crackles and clubbing). The CT should be done next, and if not consistent with IPF, further testing with an echocardiogram should be considered. The Mantoux test or purified protein derivative (PPD) is a skin test for tuberculosis. This patient's presentation is not consistent with tuberculosis (TB), so a PPD would not be recommended. Chronic cough, fever, night sweats, weight loss, along with TB risk factors (such as nursing home setting, foreign birth or travel, etc.) would indicate PPD. Ultrasound of the thorax would not be helpful in evaluating possible IPF. In fact, ultrasound of the thoracic structures (outside of echocardiography) is not often done. A plain chest film (X-ray) is a good starting point for pulmonary complaints, followed by a CT as the next appropriate test. A ventilation-perfusion scan is a test that is primarily used in the diagnosis of pulmonary embolism (PE). This patient is not presenting with a history consistent with a PE (which may include acute dyspnea and chest pain).

Which of the following sets of lab values is most consistent with the diagnosis of Hashimoto's thyroiditis? A High serum TSH, low serum total T4, and high thyroidal peroxidase antibodies B Low serum TRH, low serum TSH, and low serum free T3 levels C Normal TSH, normal serum total T4, and normal radioactive iodine uptake D Low serum TSH, high serum total T4, and high thyroid stimulating antibodies E High serum TSH, high radioactive iodine uptake, and high serum free T3

High serum TSH, low serum total T4, and high thyroidal peroxidase antibodies The correct choice is A, high serum TSH, low serum total T 4 , and high thyroidal peroxidase antibodies. Hashimoto's thyroiditis is the most common cause of primary hypothyroidism and is autoimmune in nature. The serum thyroid hormone levels are low, secondary to the destruction occurring in the thyroid gland. The negative feedback loop causes the pituitary to respond by increasing production and secretion of TSH. Thyroidal peroxidase, thyroglobulin, and TSH receptor blocking autoantibodies can be found in these patients. Choice B, low serum TRH, low serum TSH, and low serum free T 3 levels, is seen in patients with secondary hypothyroidism relating to pathology in the hypothalamus. Choice C, normal TSH, normal serum total T 4 , and normal radioactive iodine uptake, is seen in patients who are euthyroid. Choice D, low serum TSH, high serum total T 4 , and high thyroid stimulating antibodies, are findings in patients with primary hyperthyroidism (e.g. Graves' disease). Choice E, high serum TSH, high radioactive iodine uptake, and high serum free T 3 can be seen in patients with secondary hyperthyroidism, as a result of anterior pituitary pathology.

A 52-year-old woman presents because her menopausal symptoms have been extremely distressing. Over the past 4 months, she has experienced severe mood swings, hot flashes, night sweats, breast tenderness, and changes in her appetite. She has never smoked; she has an occasional drink. She had an IUD that was removed at 35 years of age. There is no family history of cancer. After a prolonged discussion, a decision to start hormone replacement therapy is made. Question What is a legitimate concern with the use of an estrogen-only supplement? Answer Choices 1 Vaginal dryness and itching 2 Worsening mood swings 3 Lowering of serum HDL level 4 Higher risk of vertebral fractures 5 Higher risk of endometrial cancer

Higher risk of endometrial cancer Explanation A higher risk of endometrial cancer is the right answer. When estrogen is administered as the sole therapy without cyclical progesterone, it causes unopposed proliferation of the endometrium. Over a period of time, this increases the chance of dysplasia, leading to endometrial cancer. To prevent this, estrogen is usually administered with cyclical progesterone. Vaginal dryness and itching is incorrect. Estrogen helps to maintain the epithelial lining of the vagina and increases lubrication, thereby decreasing the symptoms of postmenopausal atrophic vaginitis. Worsening of mood swings is incorrect. Hormone replacement therapy during menopause is beneficial in mitigating mood symptoms. When used, they are prescribed in the lowest dose possible for the shortest period of time. Lowering of serum HDL level is incorrect. Estrogen does exactly the opposite; it raises HDL and lowers LDL levels. It acts as a cardioprotective agent in that respect. Higher risk of vertebral fractures is incorrect. Estrogen influences bone metabolism and prevents osteoporosis.

A 36-year-old Jewish man presents with abdominal pain and diarrhea, as well as a 2-day history of a low-grade fever. He is initially treated with fluids and antibiotics, but his symptoms worsen. On evaluation, he is diagnosed with Crohn's disease. He is a nonsmoker, but has alcoholic drinks on alternate days; he underwent an appendectomy 2 years ago. His mother was diagnosed with irritable bowel syndrome (IBS) 10 years ago. Question What in his history is a risk factor for Crohn's disease? Answer Choices 1 His age 2 Alcohol consumption 3 His Jewish ancestry 4 Family history of IBS 5 Appendectomy

His jewish ancestry Crohn's disease (CD) is more common in the Jewish population; it is more common in whites than African-Americans. Environmental factors may play a role in its etiopathogenesis because African-Americans seem to have the same risk as whites, but African blacks have a lower risk. The age of onset is between 15 to 30 years with a second peak between 60 to 80 years. CD is an inflammatory bowel disease affecting any part of the GIT from mouth to anus; the most commonly affected region is the ileocecal region. Symptoms include abdominal pain, diarrhea (which may be bloody), vomiting, fever, and weight loss. Extraintestinal symptoms (e.g., arthritis, uveitis) may also develop. Smoking, not alcohol, is a risk factor for CD. A family history of IBS is unrelated to CD; however, a family history of CD itself is a risk factor. An appendectomy neither increases the risk nor is protective in CD.

A 56-year-old woman presents with heartburn and dyspepsia. She was diagnosed with osteoarthritis 4 years prior to presentation, and for the past 18 months, she has been managing pain with naproxen. An upper gastrointestinal endoscopy shows several areas of gastric irritation and a single ulcer about 3 mm in diameter. The gastroenterologist suggests that the patient be tested for Helicobacter pylori infection. What is the gold standard for diagnosis of this infection? Answer Choices 1 Culture 2 Histology 3 Antigen test 4 Western blot serology 5 Urease breath test

Histology The long term use of non-steroidal antiinflammatory drugs (NSAIDS) in treatment of arthropathies carries about a 4-fold increase in the relative risk of duodenal or gastric ulcers. This risk is multifactorial; 1 risk factor for development of gastric ulcers in patients taking NSAIDS is infection with Helicobacter pylori. Histologic examination of gastric biopsies obtained during endoscopy is the gold standard for diagnosis of this infection based on sensitivity and specificity (refer to the table). This approach has the added benefit of allowing simultaneous assessment of the pathology of the lesion, which can rule out neoplasms. Culture of H. pylori is difficult and requires considerable microbiologic expertise. However, indicators in the cultivation medium can detect urease. Invasive procedures can be preceded by the urease breath test as a screening for the presence of the organism. Helicobacter serology by ELISA is limited to screening and detection of a past or present infection. An antigen test is also available that detects H. pylori protein in the stool.

A 27-year-old woman presents with cramping abdominal pain and vaginal bleeding. Further history reveals amenorrhea for the past 2 menstrual cycles. On examination, she is found to have left lower abdominal tenderness and an adnexal mass. Lab values reveal an elevated Beta HCG level. Question A further history would possibly reveal what associated risk factor? Answer Choices 1 History of PID 2 Diaphragm use 3 Use of condoms 4 Oral contraceptive pills 5 Multiple sexual partners

History of PID Explanation A history of pelvic inflammatory disease is the correct answer. The vignette describes a potential ectopic pregnancy, evidenced by the triad of amenorrhea, adnexal mass, and an elevated B-HCG. Inflammation of the fallopian tubes secondary to bacterial infection resolves with fibrosis and adhesion formation. The fertilized egg gets trapped on its transit through the fallopian tube, resulting in an ectopic pregnancy. Being thin walled, implantation in the fallopian tube invariably results in a rupture. Diaphragm use is incorrect. Diaphragms sit atop the cervix, preventing sperm entering the uterine cavity. They are used as a temporary measure and are not associated with ectopic implantation. Condom use is incorrect. It is used as a temporary measure, and condom use is not associated with ectopic implantation. Oral contraceptive use is incorrect. Regular OCP use is associated with an extremely low failure rate. In the event of contraceptive failure, there is no increased risk of ectopy with OCP use. Multiple sexual partners is incorrect. While a common risk factor for PID, by itself it is not a risk factor.

A 44-year-old Caucasian woman is admitted to the hospital with a severe nosebleed. The patient states that there was no history of trauma and she never has had nosebleeds before. She also reports a history of upper respiratory infection (URI) symptoms 1 week ago. The patient's blood work is notable for a platelet count of 10,000/cmm. A presumptive diagnosis of idiopathic thrombocytopenic purpura (ITP) is made, and the patient is treated with steroids. The patient's platelet count is refractory and a splenectomy is performed. After a few months, what would you expect to see in this patient's peripheral blood smears? Answer Choices 1 Burr cells 2 Howell-Jolly bodies 3 Hypersegmented neutrophils 4 Schistocytes 5 Spherocytes

Howell jolly bodies Howell-Jolly bodies, which is the correct response,are basophilic red cell inclusions commonly found in splenectomy patients. This patient has had a splenectomy after being diagnosed with ITP, and therefore, the most likely finding in this patient's peripheral smear would be Howell-Jolly bodies. Howell-Jolly bodies are round, basophilic staining nuclear fragments of DNA in the red blood cell. This finding is classic in patients who have had splenectomy, and in the case of sickle cell disease in which the spleen auto-infarcts.

A 25-year-old woman is seeking medical attention because she is concerned about a genetic disease that is running in her family. Both her father and her paternal grandfather are affected. Her 50-year-old father has suffered for several years with coordination difficulties, chorea, decline in cognitive ability, and aggressive outbursts. His symptoms are getting progressively worse. This patient has a 50% chance of developing what disorder? Answer Choices 1 Creutzfeldt-Jakob disease 2 Early-onset familial Alzheimer disease 3 Hallervorden-Spatz disease 4 Huntington's disease 5 Lesch-Nyhan disease

Huntington Disease Huntington's disease is a progressive disorder that affects motor skills and cognition. It is caused by a CAG trinucleotide repeat expansion in the HD gene, located on chromosome 4p16. It is inherited in an autosomal dominant manner. The majority of affected individuals present with neurological manifestations; symptoms include coordination problems and involuntary movements. Oculomotor disturbances, dysarthria, and hyperreflexia occur early in the disease. It is characterized by a progressive deterioration of both voluntary and involuntary muscle control, cognitive decline, and psychiatric disturbances. The average age of onset is between 35 and 44 years. Median survival time is 15 to 18 years. Eventually, the patient becomes severely disabled due to a complete loss of motor control, muteness, and incontinence. Molecular genetic diagnosis is available. Creutzfeldt-Jakob disease is a subacute spongiform encephalopathy caused by a prion, which is a proteinaceous infectious particle. These infectious agents are thought to act by transforming normal protein molecules into deadly ones. This disorder can be inherited, transmitted through infection, or occur sporadically. Inherited cases (15% of CJD cases) show a mutation in the gene that codes for the normal prion protein. The disease is characterized by rapid and progressive dementia and neuromuscular symptoms. Initial symptoms include insomnia, personality or behavioral changes, problems with memory, incoordination, and vision disturbances. This is followed by dementia and myoclonus. Eventually, the patient becomes bedridden and enters a coma. Duration is usually 1 year or less. Average age of onset is between 50 and 75 years of age. Presently, there is no treatment. Early-onset familial Alzheimer disease is an adult onset progressive dementia. It is associated with cerebral cortical atrophy, β-amyloid plaques and neurofibrillary tangles. As the name implies, patients with this disorder have a positive family history. Mutations have been found in 2 genes: the amyloid precursor protein gene (APP), presenilin 1 (PSEN1), and presenilin 2 (PSEN2). Inheritance occurs in an autosomal dominant manner. Mean age of onset is less than 65 years. Initial symptoms are vague and include memory failure, confusion, language disturbance, agitation, and withdrawal. Progression of the disorder is slow, taking place over the course of years (mean range is 8-10 years). Alzheimer disease is the most common cause of dementia in North America and Europe. Eventually, the disease worsens, and death occurs due to malnutrition and pneumonia. Diagnosis is based upon the progressive dementia with the presence of β-amyloid plaques and neurofibrillary tangles. Molecular genetic testing is available for all 3 genes involved. Hallervorden-Spatz disease is a rare neurodegenerative disorder inherited in an autosomal recessive manner. It is characterized by a progressive rigidity that starts in the lower extremities and moves to the upper extremities. Rigidity may be accompanied by involuntary movements. Muscle tone and voluntary movement deteriorate progressively, leading to feeding difficulties and emaciation. During the late stages of the disorder, mental deterioration and progressive dementia are present. Average age of onset is the 1st or 2nd decade; mean survival time is approximately 10 years. Although the responsible gene has not been identified, it has been mapped to chromosome 20p13-p12.3. Diagnosis has generally been made postmortem. Lesch-Nyhan syndrome is a neurological disorder caused by overproduction of uric acid. It is caused by mutations in the hypoxanthine-guanine phosphoribosyl transferase (HPRT) gene located on chromosome Xq26.1. It is inherited in an X-linked recessive manner. Only men are affected (with exception). The syndrome is characterized by neurologic dysfunction, cognitive disturbances, behavioral disturbances, and the overproduction of uric acid. Initially, affected individuals show developmental delay and hypotonia, usually within the 1st year of life. Most never walk. Over the next few years, spasticity and hyperreflexia emerge, as does the hallmark sign of self-injurious behavior. This includes biting of the hands, lips, and cheeks, as well as head or limb banging. Survival into the 2nd or 3rd decade is common. The disorder can be diagnosed by molecular genetic testing.

A 28-week-premature infant is noted to have increasing tachypnea and difficulty breathing, with diffusely decreased breath sounds on exam shortly after delivery. A chest x-ray reveals diffuse, bilateral atelectasis and air bronchograms. Which of the following is the most likely diagnosis? A Acute asthma exacerbation B Hyaline membrane disease C Meconium aspiration D Pleural effusion E Spontaneous pneumothorax

Hyaline membrane disease Premature infants, due to a lack of surfactant, develop marked atelectasis and decreased lung compliance, with acute respiratory distress. This is termed hyaline membrane disease and is the most common cause of respiratory distress in preterm infants. Meconium aspiration is more likely in full-term or near-term infants who experience fetal distress, and the x-ray may demonstrate hyperexpansion and irregular infiltrates. Spontaneous pneumothorax can occur at birth. Exam findings will include decreased breath sounds on the affected side, and x-ray findings should indicate the pneumothorax or pneumomediastinum. Pleural effusion may be present in hydropic infants, or it may be due to an underlying disorder or chylothorax. The x-ray would reveal opacity of the affected side, with blunting of the costophrenic recess. Asthma exacerbation should involve airway hyperresponsiveness in relation to a trigger exposure, and would demonstrate bilateral hyperinflation with diaphragm flattening on x-ray.

A 4-month-old infant presents for a well child check up. She was a spontaneous vaginal delivery at 39.5 weeks without complications. The mother notes the baby has been more irritable and has not been eating well. No cough or fever is described. There are no smokers at their home. On exam, you note impaired extraocular movements, especially in the upward gaze, and a bulging anterior fontanel. There is increased tone of the legs. Skin exam is normal. Similar to her last visit, the length and weight are at the 50th percentile. The head circumference was formerly at the 75th percentile and is now at the +99th percentile. What clinical diagnosis do you make for this child? Answer Choices 1 Positional plagiocephaly 2 Craniosynostosis 3 Hydrocephalus 4 Neurofibromatosis 5 Catch-up growth

Hydrocephalus The clinical diagnosis of hydrocephalus can be made by macrocephaly, impaired upward gaze, and hypertonia of the extremities. These signs would warrant further workup for definitive diagnosis. Craniosynostosis results when there is premature closure of the cranial sutures, usually resulting in elongation of the head. Positional plagiocephaly is occipital flattening, most often from infants lying on their back to sleep. The skin exam is normal in this baby, while neurofibromatosis will often display café au lait spots. Catch-up growth can be seen in babies born prematurely, which is not the case with this patient. Also, only the head percentile is changing, not the length and weight.

A 29-year-old woman presents with a 3-month history of excessive fatigue and weakness. Hyperpigmentation is noted over her knuckles, elbows, and knees. Her labs are significant for mild leukocytosis, hyponatremia, and hyperkalemia. There is no elevation of cortisol on the cosyntropin stimulation test. Question What medication is recommended in this patient? Answer Choices 1 Levothyroxine 2 Hydrocortisone 3 Propylthiouracil 4 Octreotide 5 Progesterone

Hydrocortisone Explanation Hydrocortisone is correct. The patient's history and her lack of cortisol elevation on the cosyntropin stimulation test indicate Addison's disease or primary adrenal insufficiency. Addison's disease is caused by adrenal dysfunction, most commonly from autoimmune disease. The treatment of Addison's disease requires cortisosteroid replacement therapy. Hydrocortisone is the treatment of choice. Levothyroxine is incorrect. Levothyroxine is used in the treatment of hypothyroid. Propylthiouracil is incorrect. Propylthiouracil is used in the treatment of hyperthyroid. Octreotide is incorrect. Octreotide is not used in the treatment of Addison's disease. Octreotide is used in the treatment of intestinal tumors, carcinoid, tumors, and acromegaly. Progesterone is incorrect. Progesterone is used for many conditions, including infertility and amenorrhea. It is not indicated in the treatment of Addison's disease.

Your 52-year-old male patient has chronic renal failure on dialysis for 1 year. He is suffering from anemia and hypoproteinemia. He complains of bone pains, which have developed recently. Renal osteodystrophy is diagnosed after appropriate tests and you plan vitamin D therapy. What is the role of the kidneys in the synthesis of vitamin D? Answer Choices 1 Hydroxylation of 25-hydroxy vitamin D3 at the number 1 position 2 Hydroxylation of 1,-hydroxy vitamin D3 at the number 25 position 3 Activiation of cholesterol to cholecalciferol 4 Conversion of calcitriol to cholecalciferol 5 Providing the cholesterol for synthesis of vitamin D

Hydroxylation of 25-hydroxy vitamin D3 at the number 1 position Explanation Activation of Vitamin D is completed by the kidneys when they hydroxylate it at the number 1 position. The 25-hydroxy cholecalciferol is converted to 1, 25 dihydroxycholecalciferol (calcitriol) by the kidneys by 1-alpha-hydroxylase. This activates vitamin D for use in calcium and phosphorus uptake from the intestinal lumen. Hydroxylation of vitamin D3 at the number 25 position is accomplished by the liver. Cholesterol is activated to cholecalciferol by sunlight (UV light) in the skin. Once active, vitamin D3 is placed in the intestinal epithelial cells. Excess can be lost through the lumen. Cholecalciferol cannot be converted to calcitriol without the intermediate hepatic step. Calcitriol is not converted to cholecalciferol. Cholesterol is synthesized by the liver or from dietary sources.

An 11-month-old African-American male presents to the pediatric office with lethargy, jaundice and splenomegaly. A CBC reveals hemoglobin of 8.0 mg/dl and a hematocrit of 25%. Peripheral smear appearance is available below. (shows sickle cells) Which of the following treatments will reduce hemolysis and increase hemoglobin levels for this patient? A Allogeneic stem cell transplant B Folic acid supplementation C Hydroxyurea D Prophylactic penicillin E Pneumococcal vaccine

Hydroxyurea Hydroxyurea directly reduces hemolysis and increases levels of fetal hemoglobin and reduces complications and transfusion frequency. Sickle cell disease can be cured in 80% of individuals who receive a suitable transplant (A), but this procedure carries great risk and donors are difficult to identify. Folic acid supplementation (B) and pneumococcal vaccine (E) are indicated for preventive therapy of all patients with sickle cell disease, and prophylactic antibiotics (D) are used to prevent recurrent infections.

A 22-year-old woman presents with an 8-month history of amenorrhea. Further questions elicit additional pertinent positives of backaches, headaches, and acne. Physical examination reveals a female patient with a moon-shaped face, multiple purple striae, and significant central obesity (body mass index of 36). Question Considering the given history and physical exam discoveries above, what is the main cause of these findings? Answer Choices 1 Hyperglycemia 2 Hypothyroidism 3 Hypercortisolism 4 Hyperthyroidism 5 Low antidiuretic hormone

Hypercortisolism Explanation This patient likely is suffering from Cushing syndrome, also known as hypercortisolism. Consequences of excessive levels of circulating cortisol, no matter the etiology, will lead to signs and symptoms such as central obesity, thin extremities, a moon face, a buffalo hump, supraclavicular fat pads, protuberant abdomen, and complaints of oligomenorrhea, amenorrhea, or possibly erectile dysfunction in men. Backaches, headaches, hypertension, acne, purple striae, and impaired wound healing may also be found in these patients. Diabetes type 2 is caused by insulin resistance to circulating endogenous insulin in tissues. Eventually insulin is unable to be produced by the pancreatic islet cells, and an exogenous supply is required to prevent significant hyperglycemia and its related co-morbidities. Features of these patients include central obesity (specifically visceral obesity) with less fat noted on extremities. Patients will have a history or evidence of acanthosis nigricans, eruptive xanthomas on flexor surfaces, skin tags, chronic skin infections, frequent candidal vulvovaginitis in women, and erectile dysfunction or balanoposthitis in men. Hypothyroidism and hyperthyroidism both are related to dysfunction of the thyroid gland; hypothyroidism is due to failure of the thyroid gland itself or deficient supply of pituitary TSH (thyroid stimulating hormone). Symptoms exhibited with hypothyroidism are weight gain, fatigue, lethargy, depression, weakness, menses irregularities, athralgias, muscle cramps, cold intolerance, dry skin, and constipation; signs present may include thinning hair, brittle nails, bradycardia, or even peripheral edema. Hyperthyroidism is the disease state wherein there is an excessive serum level of T3 and/or T4 along with suppressed TSH levels. Signs and symptoms may include: restlessness, nervousness, heat intolerance, increased sweating, weight loss, palpitations, atrial fibrillation, exophthalmos, or even pretibial myxedema, depending on the severity. Diabetes insipidus (DI) is antidiuretic hormone deficiency. Signs and symptoms of DI include an extreme, unquenchable thirst with a very specific craving for ice water. Patients also will have significant polyuria or hypernatremia.

A 73-year-old frail-appearing woman is brought in by her daughter who is concerned about her mother's increasingly poor memory. The mother complains of fatigue and weakness so profound "her bones hurt" as well as chronic constipation. She is being treated for depression and osteoporosis, but she is otherwise in good health. Given the lab values, what is the most likely diagnosis? Labs WBC 5.3 thous/ml CBC 4.2 mill/mcl Hct 35% Hemoglobin 12.1 g/dl Glucose 80 mg/dl BUN 15 mg/dl Creatinine 0.6 mg/dl Sodium 140 mEq/L Potassium 4.2 mq/L Chloride 148 mEq/L Phosphate 2.0 mg/dl Calcium 12 mg/dl Alkaline Phosphatase 25 U/L Answer Choices 1 Bone metastasis 2 Multiple myeloma 3 Hyperparathyroidism 4 Hyperthyroidism 5 Addison's disease

Hyperparathyroidism Explanation Painful bones, renal stones, abdominal groans, and psychic moans are the sine qui non-symptoms of hyperparathyroidism. In this case, the arthralgias and myalgias; constipation and depression along with hypercalcemia; and elevated and low serum phosphate levels substantiate the diagnosis. Bone metastasis, multiple myeloma, and some hematological malignancies will present with hypercalcemia, but with an elevated alkaline phosphatase and hematological abnormalities. Addison's disease presents with hyponatremia, hyperkalemia, and sometimes hypoglycemia as a result of a deficiency in the secretion of adrenocortical hormones.

A 25-year-old woman presents because of changes in her eyes, heat sensitivity, constant sweating, and nervousness. She has lost 15 pounds recently. Her doctor suspects that she has Graves' disease. She asks her doctor why her thyroid is enlarged. What is the correct term for the underlying phenomenon in her illness? Answer Choices 1 Hyperplasia 2 Hypertrophy 3 Metaplasia 4 Dysplasia 5 Anaplasia

Hyperplasia Hyperplasia is an increase in the number of cells in the tissue. Hypertrophy is an increase in the organ size. Hypertrophy is not due to an increase in the number of cells; rather, it is due to an increase in the size of the cells. Metaplasia is the replacement of one adult tissue type with another adult tissue type. Dysplasia is a disorderly arrangement to the tissue. Anaplasia is characteristic of malignancy. It refers to cancerous cells being undifferentiated.

A 47-year-old woman presents because she feels "weak and dizzy". She was feeling well up until about an hour ago, at which point she began to feel as if she were going to "pass out." Her husband states that she appeared nervous and confused; she was also trembling. When he asked her what was wrong, she did not seem to understand him. However, she did not seem to lose consciousness. She had been healthy, and she has no significant past medical history. On examination, she is found to have tachycardia and generalized weakness. She is conscious, but slightly disoriented. She is afebrile. Her laboratory tests are normal with the exception of a hemoglobin level of 10.4g/dL and a blood glucose level of 47 mg/dL. The patient receives treatment and improves immediately. Question What is the most likely cause of this patient's symptoms? Answer Choices 1 Hypoglycemia 2 Transient ischemic attack 3 Anxiety disorder 4 Sepsis 5 Seizure disorder

Hypoglycemia Explanation Hypoglycemia results from low blood glucose. The symptoms of hypoglycemia can vary depending on the glucose level. It also varies from person to person. Symptoms can include weakness, pallor, hunger, palpitations, sweating, nervousness, and trembling. These symptoms usually resolve when the glucose levels return to normal. Anyone who has experienced an episode of hypoglycemia describes a sense of urgency to eat and resolve the symptoms. The symptoms are usually easily recognized by patients. If the patient does not eat or take an agent to increase their blood glucose level, the brain will eventually begin to suffer. The patient can develop drowsiness, confusion, behavioral changes, seizures, and coma. Causes of hypoglycemia include insulin resistance in patients who are pre-diabetic, insulin producing tumors, and medications. True hypoglycemia is found in patients with diabetes. Patients who are pre-diabetic may have low blood glucose if they are fasting and their insulin level is high. Diagnosis is usually made through history and physical and is confirmed with laboratory measurements. Plasma insulin, plasma C-peptide, and serum glucose should be checked. Treatment consists of recognizing the symptoms and resolving the hypoglycemia. The patient's symptoms can usually be resolved with food and drink containing sugar. If the patient does not improve after consumption of sugar, he or she should seek medical care for further evaluation. The other choices are incorrect since the symptoms and laboratory results do not match. The patient's labs are normal with the exception of low hemoglobin and blood glucose levels. Transient ischemic attack is an acute episode stroke that last usually less than 24 hours. It can be thought of as a stroke in which the symptoms resolve. It is caused by a temporary occlusion of a blood vessel. Symptoms are not affected by food or drink. Anxiety disorders can result in excessive worry and the feeling of being overwhelmed. Lab results, such as blood glucose levels, would not be affected. Sepsis results from a bacterial infection that has spread to the blood. Patients may have fever, chills, tachycardia, decreased urination, confusion, dizziness, and an elevated white cell count. This patient's white blood cell count was normal. Depending on the type of seizure, seizure disorders can produce disorientation, uncontrollable movements, or staring into space. Unlike symptoms of hypoglycemia, they do not end when the diet is changed.

A 21-year-old insulin-dependent diabetic college student is acutely agitated and is verbalizing expletives. Prior to admitting him to an acute psychiatry inpatient service and administering intra muscular chlorpromazine (Thorazine) or haloperidol (Haldol), for what should he be evaluated? Answer Choices 1 Hyponatremia 2 Hypokalemia 3 Hypocalcemia 4 Hypovolemia 5 Hypoglycemia 6 Hypomagnesemia 7 Hypothermia

Hypoglycemia Explanation Hypoglycemia, reduced blood glucose level below the laboratory reference ranges, is most likely to present with a toxic-metabolic state emulating an acute agitation and/or an acute psychosis; this patient demonstrates these symptoms. Given the known history of diabetes, and even in the absence of a history of a insulin-dependent diabetes disorder, the health care provider should check the patient's blood glucose level; if found to be low, glucose should be administered intravenously. Hyponatremia, which is reduced serum sodium below the laboratory references ranges, may present with seizures; however, none are evident in this case. Hypokalemia, which is reduced serum potassium below the laboratory reference ranges, may present with muscular weakness; however, that is not a listed symptom. Hypocalcemia, which is reduced serum calcium below the laboratory reference ranges, may present with neuromuscular hyperirritability; it is not mentioned in this case. Hypovolemia, which is reduced circulating volume, may present with cardiovascular collapse verging on shock. This patient is not in such a condition. Hypomagnesemia, which commonly results from poor nutrition, would be unlikely in this setting. Hypothermia, which is reduced body temperature, may present with slow mentation; it is not a condition pertinent to this case. The core ethical and legal principle is to be reasonably certain that all efforts have been provided. Afterwards, it is important to identify, diagnose, treat, and correct all acute medical issues and crises before simply providing injections of intra-muscular, anti-agitation, and/or anti-psychotic medications that necessarily carry an element of risk and are of no benefit in this setting.

A 5-month-old female infant presents with a 3-day history of vomiting. She is exclusively breastfed, and her mother states that today she has vomited within 15 minutes of each feeding. Her last wet diaper was 10 hours ago. On physical examination, she is afebrile, tachycardic, irritable, and does not express tears when crying. She was a full-term infant born via vaginal delivery. She has no significant past medical history. Her 3-year-old sister has had gastroenteritis for the past few days. Question What is the most likely laboratory finding? Answer Choices 1 Decreased blood urea nitrogen 2 Hyponatremia 3 Hypokalemia 4 Decreased urine specific gravity 5 Hypernatremia

Hypokalemia This infant is showing signs of moderate dehydration due to emesis and limited breast milk intake. Gastric and urinary potassium losses can both contribute to hypokalemia. Dehydration leads to hemoconcentration with an increased hemoglobin and increased urine specific gravity. BUN and creatinine could be increased with volume depletion, and they could be increased further with renal insufficiency. Not eating for an extended period of time could lead to hypoglycemia in a 5-month-old infant. Hypernatremia implies a deficit of total body water. Major symptoms are thirst, confsion, neuromuscular excitability, seizures, and coma.

Your patient is a 65-year-old man presenting with a 3-week history of progressive leg swelling, dyspnea on exertion, and increased thirst. A couple days ago, he started having nausea, headache, vomiting, and his gait became unsteady. He does not have chest pain or shortness of breath at rest. His medical history is significant for hypertension, type II diabetes, and chronic kidney failure. His medications include a beta blocker/alpha-1 blocker (carvedilol), aspirin, angiotensin-converting enzyme (ACE) inhibitor (lisinopril), biguanide (metformin), and a loop diuretic (furosemide). Physical examination reveals bilateral pitting edema up to his knees, swelling in the face, peripheral pulses 2+ in all extremities, and an unsteadiness during Romberg testing with both eyes open and closed. Auscultation reveals bilateral crackles at the lung bases, and the rest of his physical examination is not contributing. Laboratory findings are: Test Results Reference Range (males) Sodium Potassium Chloride CO2 Creatinine Blood urea nitrogen Fasting Glucose Serum osmolality 125 mEq/L 3.1 mEq/L 97 mEq/L 22 mmol/L 2.6 mg/dL 40 mg/dL 189 mg/dL 270 mOsm/kg H2O 135-145 mEq/L 3.5-5.0 mEq/L 95-108 mEq/L 20-32 mmol/L 0.7-1.4 mg/dL 7-30 mg/dL <110 mg/dL 280-295 mOsm/kg H2O Total protein Albumin Total bilirubin Alkaline phosphatase Alanine aminotransferase Aspartate aminotransferase 6.0 g/dL 2.9 g/dL 1.1 mg/dL 90 IU/L 34 IU/L 37 IU/L 6.0-8.5 g/dL 3.5-5 g/dL 0.1-1.3 mg/dl 40-120 U/L 0-35 U/L 0-37 U/L Urine Sodium <10 mEq/L Variable (depending on volume status) Red blood cell count White blood cell Hemoglobin Platelets 4 000 000/uL 6.8 000 /mm 11.0 g/dL 300 000/mm3 4,7 000 000- 6,1 000 000/uL 3.8000-10.8000/mm3 13.8-17.2 g/dL 150 000-450 000/mm3 Question What is the most probable cause of his neurological symptoms and signs? Answer Choices 1 Hyponatremia 2 Hypokalemia 3 Hypervolemia 4 Hyperglycemia 5 Anemia

Hyponatremia Explanation Nausea, headache, vomiting, unsteady gait, and Romberg showing vestibular disfunction are most probably caused by hyponatremia, the most common electrolyte abnormality in clinical practice. Hyponatremia is defined as a serum sodium concentration less than 135 mEq/L and is severe when the serum sodium level falls below 125 mEq/L. It is often relatively asymptomatic. Hyponatremia manifests mostly by neurological symptoms secondary to cerebral edema. When the serum sodium level drops, water moves from the extracellular space into cells and causes cellular swelling. Brain swelling is probably the first to manifest, given the fact that the brain contains around 80% water and has no space within a skull to accommodate swelling. Early symptoms of cerebral edema are nausea, headache, and vomiting. In the acute cases (those that happen over several hours), hyponatremia may cause acute cerebral edema, with seizures, brain herniation, respiratory failure, coma, and death. However, hyponatremia that develops over several days gives a chance for the brain to adapt to the excess water by extruding osmolytes (potassium, chloride, and small organic molecules), thus equalizing the osmolality between the intra- and extracellular fluid. Therefore, the symptoms of chronic hyponatremia are less severe, even when laboratory shows severe hyponatremia, slightly decreased cognitive functions, mild nausea, and mild gait and balance disturbances. Physical examination in patients with hyponatremia is generally not specific and usually relates to the underlying condition - in this case, probably renal failure. Hypokalemia will not present with neurological symptoms. Mild hypokalemia is often without symptoms, and severe hypokalemia, with serum potassium concentrations of 2.5-3 mEq/L, causes mostly muscle symptoms, such as weakness, pain, cramps, constipation (disturbance in the smooth muscles function), and, when severe, can cause flaccid muscle paralysis. Volume overload, primarily salt and water, will manifest as subcutaneous edema, ascites, and/or pulmonary edema, presenting with the symptoms of the failure of those organs. Your patient has volume overload, but neurological symptoms in hypervolemia with hyponatremia are also caused by sodium disbalance. Hypervolemia with hyponatremia is most commonly caused by chronic heart failure, liver, or kidney disease. Hyperglycemia of 189 mg/dL is unlikely the cause of progressive leg swelling, dyspnea on exertion, increased thirst, nausea, headache, vomiting, unstaeady gait, and balance disturbances. Severe anemia may present with dypnea, thirst, and dizziness, but anemia in this case is very unlikely to be the cause and is not the cause of progressive leg swelling and neurological symptoms and signs.

A 26-year-old athlete presents complaining of scant menses x 4 months. She is a G0P0, menarche was at age 13, and her menses have been mostly regular. She is a non-smoker and non-drinker, and has been trying to achieve pregnancy x 8 months. Physical exam reveals a thin, white female in no distress. Vitals are normal, BMI is 17.5, her pelvic exam is normal, and STD cultures are negative. A pregnancy test is negative in clinic. What is the most likely cause for her amenorrhea? A Idiopathic B Hypothalamic C Hypothyroid D Polycystic ovarian syndrome E Androgenic

Hypothalamic A young healthy woman with a low BMI, no other signs of virilization, and a history of normal menarche is most likely to have induced amenorrhea, due to suppression of the hypothalamic axis from low weight and fat index.

A 26-year-old athlete presents complaining of scant menses x 4 months. She is a G0P0, menarche was at age 13, and her menses have been mostly regular. She is a non-smoker and non-drinker, and has been trying to achieve pregnancy x 8 months. Physical exam reveals a thin, white female in no distress. Vitals are normal, BMI is 17.5, her pelvic exam is normal, and STD cultures are negative. A pregnancy test is negative in clinic. What is the most likely cause for her amenorrhea? A Idiopathic B Hypothalamic C Hypothyroid D Polycystic ovarian syndrome E Androgenic

Hypothalamic B A young healthy woman with a low BMI, no other signs of virilization, and a history of normal menarche is most likely to have induced amenorrhea, due to suppression of the hypothalamic axis from low weight and fat index.

Release of which of the following substances is triggered by pituitary growth hormone and promotes growth of other tissues in the body? A C-pepide B IL-I C IGF-I D Thyroxine E Catecholamines

IGF-I The correct choice is C, IGF-I or insulin like growth factor I. This growth factor leads to increased DNA, RNA, and protein synthesis, which leads to overgrowth of bone, soft tissue, and cartilage. Choice A, c-peptide, is a part of the prohormone of insulin. Choice B, IL-I or interleukin I, is an important cytokine that promotes cell activation. Choice D, thyroxine, potentiates the actions of growth hormone on tissues.

A 50-year-old man with a history of myocardial infarction (MI) is on aspirin, atenolol, and atorvastatin; he presents with pain in the knee joints. He is a diabetic controlled by diet and exercise. He is diagnosed with osteoarthritis, and he is prescribed ibuprofen. Question What is a cause for concern with regard to the ibuprofen prescription for this patient? Answer Choices 1 Ibuprofen increases the risk of hyperglycemia 2 His age is inappropriate to prescribe ibuprofen 3 Ibuprofen reduces aspirin's cardioprotective benefits 4 Ibuprofen decreases atorvastatin's hypocholesterolemic effects 5 Ibuprofen is ineffective in treating osteoarthritis

Ibuprofen reduces aspirin's cardioprotective benefits Explanation Ibuprofen is a nonsteroidal anti-inflammatory drug (NSAID), which when used concomitantly with aspirin, may reduce the latter's cardioprotective effect. Ibuprofen has been shown to inhibit the irreversible platelet inhibition brought about by aspirin. A clinically and statistically significant increase in the risk of mortality in persons using ibuprofen and aspirin, compared to those using ibuprofen alone, has been noted. It is recommended that ibuprofen not be prescribed for the long-term in patients on aspirin. Another strategy is to ingest aspirin 2 hours before ibuprofen. Ibuprofen has not been shown to cause hyperglycemia or interact with atorvastatin. The greater risk of gastrointestinal complications should be borne in mind while prescribing NSAIDS in persons aged 65 years or more. Ibuprofen is 1 of the NSAIDs effective in treating osteoarthritis.

A 7-year-old boy presents with headaches, nausea, and lethargy. According to his parents, the boy has been consuming large amounts of fluids. He also urinates frequently. A previously healthy child, he has no significant past medical history. His symptoms began approximately 3 weeks ago. His parents thought that it was his increased activity level that led to his increased fluid intake. However, over the past few days, he has been becoming more lethargic and is still consuming large amounts of water and juice. On physical exam, the boy appears to be thin, ill-appearing, and lethargic. His eyes appear sunken and his skin is dry. His vital signs are as follows: blood pressure 100/54 mm of Hg, temperature 99.0 degrees Fahrenheit, pulse 120 beats per minute, and a respiratory rate of 22 per minute. Laboratory tests show dilute urine with a specific gravity of 1.002 and a urine osmolality of 199mOsm/kg. His plasma osmolality is 296mOsm/kg. Question Which of the following statements is true? Answer Choices 1 The genetic form of diabetes insipidus is rare and is usually in an autosomal recessive pattern 2 The drug of choice for treatment is chlorpropamide 3 Idiopathic DI is associated with destruction of cells in the hypothalamus, often as part of an autoimmune process 4 Patients with nephrogenic DI have low serum ADH levels 5 An individual with DI cannot normally drink enough fluid to keep up with fluid loss

Idiopathic DI is associated with destruction of cells in the hypothalamus, often as part of an autoimmune process Explanation Idiopathic DI is associated with destruction of cells in the hypothalamus, often as part of an autoimmune process. Lymphocytes infiltrate the posterior pituitary and the stalk. These infiltrates may show up on Magnetic Resonance Imaging (MRI) and help in the diagnosis. In addition, antibodies may form against vasopressin cells. These antibodies can also cause the development of the disease. The drug of choice for the treatment of DI is usually an analogue of vasopressin, such as desmopressin. There is a genetic form of DI that is rare and is inherited in an autosomal dominant pattern. Patients with nephrogenic DI have a normal-to-elevated serum ADH level, and the kidneys fail to respond to exogenous ADH. Under normal circumstances, an individual with DI is able to keep up with fluid loss by increasing their fluid intake. Stress, lack of availability, or illness may make this more difficult, and the signs of dehydration will become evident.

A 64-year-old man presents with progressive dyspnea, fatigue, chronic dry cough, and exercise intolerance. His symptoms have been progressing over the past several months to a year. As part of your work up, you obtain pulmonary function testing; it reveals a FEV1/FVC ratio of >0.7, a decreased total lung capacity, and a decreased residual volume. Question What is the most likely diagnosis? Answer Choices 1 Anemia 2 Asthma 3 Chronic obstructive pulmonary disease 4 Pulmonary embolism 5 Idiopathic pulmonary fibrosis

Idiopathic Pulmonary Fibrosis This patient in this case presents with dyspnea, fatigue, dry cough, and exercise intolerance, which are fairly common symptoms. The clue to making this diagnosis lies in his pulmonary function tests (PFT's). The findings of an increased FEV1/FVC ratio with a decreased total lung capacity and residual volume is indicative of a restrictive pathology.1 Idiopathic pulmonary fibrosis (IPF) is a slowly progressive fibrosing lung disease; it commonly affects people 50 - 70 years old, and there is a slight male predominance.1 In addition, physical exam may reveal late inspiratory crackles and digital clubbing.2 Anemia can cause similar symptoms, but this is due to decreased circulation of oxygenated blood and not a result of an underlying lung disorder; therefore, there will be no abnormality in PFT's. Both asthma and chronic obstructive pulmonary disease (COPD) are obstructive pathologies. PFT's would show an obstructive pattern, with an FEV1/FVC ratio of <0.7 as well as an increased total lung capacity and residual volume.1 Pulmonary embolism is an unlikely diagnosis in a patient who has been symptomatic for several months to a year.

What is the most accurate statement concerning the treatment of migraine headache with sumatriptan? Answer Choices 1 Its effectiveness is related to its action as a serotonin antagonist 2 Its most common side effect is irritability 3 The recommended oral dose is 6 mg 4 It is indicated for both treatment and prophylaxis of migraine headache 5 If it is effective in the initial therapy, it is often effective in aborting a recurrence of symptoms

If it is effective in the initial therapy, it is often effective in aborting a recurrence of symptoms f it is effective in the initial therapy, it is often effective in aborting a recurrence of symptoms. Sumatriptan is a serotonin agonist; it is believed that its effectiveness is related to the importance of serotonin in the regulation of cranial and intracranial vasoactivity. The most common side effect is nausea with or without vomiting; however, these are frequent symptoms of a migraine headache itself. It may also cause sedation and drowsiness, but irritability is not common. The recommended oral dose is 25 mg, and the recommended subcutaneous dose is 6 mg. Sumatriptan is indicated for the treatment of a migraine headache, but it is not recommended for prophylaxis. Clinical trials have shown that sumatriptan is effective in reducing the severity of a migraine headache in approximately 70 - 80% of patients, but recurrence of the headache is seen in as many as 35 - 45%; a 2nd dose of sumatriptan is effective in treating the recurrence in the majority of patients.

A 20-year-old woman was just told by her new sexual partner that she needed to be checked for a sexually transmitted infection because he has developed dysuria and a profuse urethral discharge. She herself has had a subjective fever for the past two days, some nausea but no vomiting, diffuse lower abdominal pain, and a severe backache. On examination, she has a temperature of 100.5˚F, hypoactive bowel sounds, bilateral lower abdominal quadrant tenderness, a profuse mucopurulent cervical discharge and pronounced cervical motion tenderness. Serum pregnancy testing is negative. She is given an injection of ceftriaxone and a prescription for doxycycline for 14 days and an appointment for follow up the next day. Under which of the following conditions should metronidazole be added to her regimen? A If she does not appear improved by the following morning B If she develops vomiting or diarrhea C If she has a probable tubo-ovarian abscess D If she has an allergic reaction to the doxycycline E If she has had more than one partner in the past month

If she has a probable tub-ovarian abscess C Recommended regimens for treatment of pelvic inflammatory disease include ceftriaxone or another parenteral third-generation cephalosporin and doxycycline or cefoxitin, probenecid, and doxycycline. Metronidazole (or clindamycin) should be added to either regimen if a tubo-ovarian abscess is present. If being treated as an outpatient and she has not improved in 3 days (A), or if she develops vomiting (B), she should be hospitalized rather than given metronidazole. Metronidazole is not a satisfactory substitute for doxycycline (D). The patient's condition rather than the number of sexual partners (E) determines the appropriate therapy.

A 55-year-old African-American man presents as febrile with massive swelling of the abdomen and diarrhea. He has a 20-year history of heavy alcohol use. A fluid wave is elicited on physical examination of the abdomen by striking one flank and feeling the transmitted wave on the opposite flank. Question In what case would ascitic fluid analysis suggest cirrhosis as a cause of ascites in this patient? Answer Choices 1 If the acidic fluid and leukocyte count is more than 500/mm3 2 If the fluid has a protein concentration below 3 g/dL 3 If there is a large number of red blood cells in the fluid 4 If fluid is designated as a exudate 5 If the Serum Ascites Albumin Gradient is lower than 1.1 g/dL

If the fluid has a protein concentration below 3 g/dL The correct response is if the fluid protein concentration below 3/dL. After the diagnosis of ascites is made by physical examination, all patients with new-onset ascites should undergo abdominal paracentesis and ascitic fluid analysis. The most important tests to order for fluid analysis include protein concentrationand cell count. Fluids with protein concentration above 3 g/dL are designated as exudates. Those with values below 3 g/dL are designated as transudates. Diseases usually associated with transudates include congestive heart failure, cirrhosis, constrictive pericarditis, inferior vena cava obstruction, hypoalbuminemia, Meigs syndrome, and some cases of nephrotic syndrome. The amount of albumin in the ascitic fluid compared to the serum albumin (the Serum Ascites Albumin Gradient, SAAG) can be indicative of the cause of ascites. Ascites related to hypertension, cirrhosis, or congestive heart failure generally shows a SAAG greater than 1.1 g/dL. Exudates are more commonly seen with peritoneal neoplasm, pancreatic ascites, myxedema, and tuberculous peritonitis. A large number of red blood cells in the fluid or grossly bloody ascites suggests a diagnosis of neoplasm. An acidic fluid and leukocyte count of more than 500/mm3 strongly suggests a peritoneal infection or inflammatory process. Other tests that should be ordered in the appropriate clinical setting include cytologic examination, lactic dehydrogenase (LDH), specific tumor markers, glucose, and cultures for bacteria, mycobacteria, and fungi.

A 50-year-old male with an elevated WBC of 143,000, hematocrit 47% (45 to 52%), platelet count of 247,000 (150,000 to 4000,000), and BCR/ABL found in peripheral blood. He complains of fatigue, night sweats, and abdominal fullness. The best initial course of therapy would include which of the following? A Bone marrow transplant B Cytarabine C Dasatinib D Hydroxyurea E Imatinib mesylate

Imatinib mesylate (GLEEVAC) This patient has classic signs of chronic myelogenous leukemia. The first line of therapy for patients in the chronic phase of this disease is Imatinib mesylate, starting at 400 mg/day. For patients that demonstrate to Imatinib alone, Dasatinib may be an adjunct therapy. Hydroxyurea and Cytarabine were used prior to formulation of Imatinib, and may play a role for resistant CML. Bone marrow transplant would be reserved for patients that are resistant to current therapy or are in relapse.

On Thanksgiving day, a 5-month-old infant presents with wheezing, rapid respirations (>45 breaths/min), and chest retractions. The patient has a 2-day history of rhinorrhea and low-grade fever. Breath sounds are normal, and there is no cyanosis. What test can confirm the most likely diagnosis? Answer Choices 1 Chest X-ray 2 Immunofluorescence of nasal secretion 3 Gram stain of the sputum 4 Blood gas analysis 5 White blood cell count and differential

Immunofluorescence of nasal secretion The correct answer is Immunofluorescence of nasal secretion. The clinical presentation, time of year, and age of the child strongly suggest bronchiolitis caused by respiratory syncytial virus. A chest X-ray may indicate interstitial pneumonia, but it can be relatively normal. Gram stain is needed in case of suspected bacterial infection, and sputum usually is difficult to obtain from children. Blood gas analysis may detect abnormalities when cyanosis is absent, but it cannot confirm a specific diagnosis. The white blood cell count and differential are usually normal; however, immunofluorescence of nasal secretion can lead to rapid viral identification.

A 24-year-old man presents with a painful ulcer on his left leg and systemic symptoms. The lesion started 1 week ago as a small pustule that developed at the site of mosquito bite, and it rapidly developed into a painful ulcer. Patient describes pain as "stabbing" and debilitating. He also develops symmetrical joints pain, muscle pain, fever, and malaise. His past medical history is significant for ulcerative colitis, which is currently in remission. On examination, you find a deep exudative ulceration, with a well-defined violet border and a worn, erythematous, and indurated edge. There are signs of pathergy as well. Question What will you recommend in addition to meticulous wound care? Answer Choices 1 Immunosuppressant (systemic corticosteroids) 2 Antiviral (systemic acyclovir) 3 Surgical debridement 4 Antifungal (systemic fluconazole) 5 Painkiller (systemic paracetamol)

Immunosuppressants (corticosteroids) Your patient most probably has Pyoderma gangrenosum, which is a ulcerative skin lesion of an uncertain etiology; in more than 50% of cases, it is associated with systemic diseases, most commonly inflammatory bowel disease. It usually develops rapidly and can progress from a pimple to an ulcer in 1 or 2 days. In a process termed pathergy, new ulcerations may occur after trauma or injury to the skin. Pain is the predominant symptom, but symmetrical arthritis, myalgias, and malaise are also common. When the lesions heal, they usually leave the scars that are often cribriform. Immunosuppression is the mainstay of treatment; it is believed that dysregulation of the immune system (specifically, altered neutrophil chemotaxis) is involved. Most clinicians use both topical and systemic therapy. The most commonly used treatments include topical potent corticosteroids or tacrolimus to treat early lesions; systemic corticosteroids, TNF-α inhibitors, or other anti-inflammatories or immunosuppressants are used to treat more severe manifestations. Surgery or debridement is contraindicated because of the presence of pathergy. Even if there was no pathergy, surgery is contraindicated because skin trauma can trigger the pathergy. Acyclovir is not indicated; the clinical picture is not consistent with acute herpes infection and the patient is not immunocompromised. Fluconazole is not indicated; the clinical picture is not consistent with tinea (or any other fungal infection). Paracetamol alone probably will not relieve the debilitating pain. Sufficient pain medication in this case will probably include paracetamol and some other medication, sometimes opioids.

A 27-year-old man is seen for evaluation of a small mass in his right groin. His history reveals that this mass has been present for about 2 years, but he has suddenly been motivated to seek medical evaluation by his wife who has fears of cancer. He believes he 1st became aware of the mass when getting dressed, and notes occasional aching in the right groin. He denies trauma to the region. His past medical history is significant for chronic low back pain, secondary to a motor vehicle accident (MVA) 9 years ago. He takes over-the-counter ibuprofen as needed, and has no known drug allergies. He has had a tonsillectomy, appendectomy, and L4-L5 fusion, post-MVA. There is a family history of lung cancer in his paternal grandfather and diabetes in his maternal grandparents. The patient smokes, but denies alcohol and illicit drugs. He is married with 3 children, but admits to >10 sex partners previous to marriage. He works as a pipe fitter, which involves heavy lifting and frequent bending. His physical exam is as follows: BP 115/80, pulse 72, temp 98.0 degrees, weight 190 pounds, and height 69". Inspection of the patient in the supine position reveals no visible mass in the right inguinal region, corresponding with his description. There is no erythema or swelling in either inguinal region or abdomen. Abdominal exam reveals no tenderness, masses, hepatosplenomegaly, acites, or bruits. Inspection of the scrotum and penis shows no lesions, erythema, or tenderness. The patient is asked to stand for a testicular exam, which reveals normal-sized testes without nodules or masses. With a finger placed in the inguinal ring an impulse is felt on the right, but not on the left, when the patient is asked to cough. Although many conditions comprise the differential for a mass in the groin, you suspect he has an inguinal hernia. Question What aspect of this patient's history or physical is most specific to the diagnosis of hernia? Answer Choices 1 Absence of fever 2 Chronic nature of the mass 3 History of heavy lifting 4 History of multiple sex partners 5 Impulse felt with cough 6 Unilateral groin mass

Impulse felt with cough The impulse the examiner felt with the cough most strongly supports the diagnosis of hernia; this is a classic finding associated with inguinal hernias. Other conditions in the differential would be much more likely to present as an inguinal mass that does not come and go with straining, coughing, or Valsava maneuvers. The individual's history and exam are most consistent with the indirect type of inguinal hernia, but distinguishing direct versus indirect does not significantly change management. While absence of fever may help to exclude acute infectious etiology (e.g., epididymitis, orchitis, and cat-scratch disease), the differential of a groin mass in an afebrile patient still includes malignancy, hydrocele, varicocele, as well as hernia. The chronic nature of this patient's mass helps the examiner focus away from the acute scrotal emergencies, such as testicular torsion and infectious conditions. Patients with hernias may exhibit either chronic or acute presentations. Other items on the chronic groin mass differential include malignancy, undescended testes, and dermatologic conditions; the acute versus chronic course of the complaint is not helpful in fully narrowing the differential. While long-term heavy work seems to have an association with groin hernia, this patient's history of heavy lifting is not specific enough upon which to base the diagnosis of hernia. The patient's history of multiple sex partners would be associated with a risk for epididymitis or orchitis. This aspect of the patient's history does not support the suspected diagnosis of inguinal hernia. Unilateral groin mass suggests a very broad differential (e.g., lymphadenopathy, scrotal mass, vascular mass, and infectious processes). This aspect of the patient's history does not narrow the differential to hernia alone.

An 83-year-old man presents to his urologist's office with "problems down there." He is a poor historian, but his wife accompanies him and helps with his review of systems. She does not report that he has any voiding issues and reports that he has never had to see a urologist before now. Genitourinary examination is suggestive of phimosis. Question What physical examination finding would support this diagnosis? Answer Choices 1 Entrapment of the foreskin behind the glans penis in an uncircumcised male patient 2 Erythema and edema of the glans penis in an uncircumcised male patient 3 Erythema and edema of the phallus in a circumcised male patient 4 Erythema and edema of the glans penis in a circumcised male patient 5 Inability to retract the foreskin over the glans penis in an uncircumcised male patient

Inability to retract the foreskin over the glans penis in an uncircumcised male patient Explanation Inability to retract the foreskin over the glans penis in an uncircumcised male patient is the correct answer. Phimosis can be the result of recurrent infections or irritation, advanced age, diabetes, and poor hygiene. Patients can experience painful erections, recurrent balanitis, and voiding difficulties. Treatment can be with topical steroids followed by gradual retraction of the foreskin or circumcision. Entrapment of the foreskin behind the glans penis in an uncircumcised male patient is not the correct answer. This is a description of a patient with paraphimosis. Typically, this results in retraction of the foreskin for medical or hygiene purposes that is not followed by properly pulling the foreskin back in place over the glans penis. This can be very painful, and needs to be manually reduced as soon as possible to prevent necrosis of the glans penis. A dorsal slit or circumcision may be necessary if manual reduction is not possible. Erythema and edema of the glans penis in an uncircumcised male patient is not the correct answer. This description most closely describes balanitis and does not address the placement or status of the foreskin, which is the main concern with phimosis. Erythema and edema of the phallus in a circumcised male patient is not the correct answer, as patients who are circumcised cannot experience phimosis due to their lack of foreskin. Erythema and edema of the glans penis in a circumcised male patient is not the correct answer, as patients who are circumcised cannot experience phimosis due to their lack of foreskin.

A 34-year-old woman requests a prescription for Synthroid (levothyroxine) 125 mcg, which she takes 1 X day. The patient has no medical records, but you are able to ascertain she underwent previous transsphenoidal surgery for an enlarged pituitary gland. She was subsequently placed on thyroid hormone replacement medication and advised she must remain on this as a life-long medication routine. She admits to some fatigue and difficulty concentrating; otherwise, she is in her usual state of health. Further review of syptoms is unremarkable. Physical exam reveals coarse hair, patchy areas of dry skin, no periorbital edema or lid lag, no thyroid enlargement, and ankle reflexes +1/4. The remainder of the physical examination is within normal limits. Subsequent laboratory studies are as follows: Result Reference range TSH 0.25 0.40 - 4.5 mU/L Free T4 0.6 0.8 - 1.8 ng/dL Question What is the best therapeutic intervention? Answer Choices 1 Increase the dose of Synthroid (levothyroxine) 2 Decrease the dose of Synthroid (levothyroxine) 3 Continue the current dose of Synthroid (levothyroxine) 4 Discontinue Synthroid and initiate treatment with Tapazole (methimazole) 5 Obtain a thyroid Iodine131 uptake and scan

Increase the dose of Synthroid (levothyroxine) Explanation The correct choice is to increase the dose of Synthroid (levothyroxine) for the treatment of secondary hypothyroidism. TSH secretion is controlled by hypothalamic thyrotropin-releasing hormone (TRH) and negative feedback from circulating free thyroid hormones FT4 and FT3. It is the most important test in the diagnosis of thyroid disorders. However, in this patient (who previously underwent surgical resection of the pituitary), postoperative TSH secretion was found to be insufficient; this resulted in hypothyroidism secondary to pituitary dysfunction, thus requiring life-long thyroxine replacement. In patients with secondary hypothyroidism, the TSH levels remain low or inappropriately normal (for the circulating thyroid hormone levels), so the dosage of thyroxine is titrated to maintain circulating thyroid hormone levels within the normal range. Since the patient has a low serum free T4, she is hypothyroid; therefore, it is inappropriate to decrease, discontinue, or continue her current dose of Synthroid. Based upon her history, the low serum TSH is the result of surgical intervention, not indicative of hyperthyroidism, so Tapazole and an I131 uptake and scan are not indicated. It would also be prudent to obtain past medical records, including post-operative and recent laboratory results, to ensure the remainder of the pituitary hormone levels are within normal limits. Levels that should be looked at include prolactin (PRL), follicle stimulating hormone (FSH), luteinizing hormone (LH), human growth hormone (HGH), and adrenocorticotropic hormone (ACTH).

A 43-year-old Caucasian woman presents for evaluation of multiple concerns, including: a 6-month history of menstrual irregularities; weight gain with increased abdominal girth; easy bruising without a history of trauma; new "stretch marks" on her torso; mood swings; decreased libido, and weakness in her arms and legs. She had previously regular menses, a bilateral tubal ligation (BTL), and she reports multiple negative home pregnancy tests. She denies hot flashes, night sweats, and frank depression. She denies any health changes, medications, or stressors in relation to these changes. The patient is frustrated that her health fair labs (which include complete blood count, complete metabolic panel, lipid panel and thyroid stimulating hormone) were all normal, and they do not explain her symptoms. Her past medical history is unremarkable, with no chronic conditions and no medication use; her only surgery was the BTL. She lives with her husband and 3 children. She works in retail, and she walks for exercise. She denies use of tobacco, alcohol, and drugs. Her blood pressure is 154/92. Chart review demonstrates weight gain of 15 pounds over 6 months, with normal blood pressures in the past. On physical exam, you observe an overweight woman with an especially rounded, full face. She has a fatty fullness to her neck and some central obesity, but her arms show some muscle wasting. Purple striae are noted on her torso. Her skin also appears thinned, with multiple bruises. Hirsutism is observed on the patient's chin, abdomen, and breasts. The remainder of her exam is unremarkable. Question What would best explain the pathophysiologic basis for this patient's condition? Answer Choices 1 Autoimmune destruction of adrenal glands, causing decreased cortisol production 2 Exogenous administration of corticosteroids, causing increased adrenal production of cortisol 3 Hypoplastic adrenal cortex, causing low serum levels of cortisol 4 Increased adrenocorticotropic hormone (ACTH) secretion from pituitary, causing increased adrenal production of cortisol 5 Low adrenal production of cortisol, causing increased renal retention of cortisol

Increased adrenocorticotropic hormone (ACTH) secretion from pituitary, causing increased adrenal production of cortisol Explanation This patient presents with a multitude of signs and symptoms (weight gain, menstrual irregularity, mood swings, poor libido, moon facies, fat depositions, proximal limb weakness, central obesity, skin thinning, purple striae, and easy bruising) that support a diagnosis of Cushing syndrome. The most common cause of Cushing syndrome is considered iatrogenic, from administering exogenous steroids, such as prednisone. The next most common cause is known as Cushing disease and results from increased adrenocorticotropic hormone (ACTH) secretion from pituitary, causing increased adrenal production of cortisol. This patient should be evaluated with laboratory tests, and then imaging, to look for a pituitary adenoma. Autoimmune destruction of adrenal glands, causing decreased cortisol production, is the most common mechanism for development of Addison's disease. Addison's disease also has multiple, somewhat nonspecific signs and symptoms, but they differ from those of excess cortisol production. A patient with Addison's may have fatigue, weakness, hypotension, hyperkalemia and hyponatremia, hyperpigmentation of the skin, anorexia, nausea, vomiting, diarrhea, and weight loss. As indicated above, exogenous administration of corticosteroids is the most common cause of iatrogenic Cushing syndrome. However, the effect of administering corticosteroids is suppression of (not increased) adrenal cortisol secretion. This is also the rationale for tapering off prescribed corticosteroids, in order to allow the patient's natural adrenal production of cortisol to return to normal. Otherwise, an Addison's-type situation occurs with sudden discontinuation of high-dose steroids. Hypoplastic adrenal cortices would be unable to produce normal cortisol levels, thus causing low serum cortisol levels. Congenital adrenal hypoplasia is another (less common) cause of Addison's disease. Renal retention, or elimination, of cortisol is not a mechanism for controlling cortisol levels in the body. Instead, corticotropin-releasing hormone (CRH) from the hypothalamus and ACTH from the pituitary gland, and to a lesser degree, arginine vasopressin (AVP), regulate physiologic cortisol levels. It would be expected that when low adrenal production of cortisol occurs, there would be an increase in CRH and ACTH in order to stimulate more adrenal production of cortisol.

A 28-year-old female presents with a gradual progression of fatigue and pallor over the last few months. Initial CBC results show a hemoglobin of 10.4 mg/dL, hematocrit of 32%, an MCV of 112 fL, and a reticulocyte count of 0.1%. Which of the following is the most likely pathophysiologic mechanism responsible for her anemia? A Chronic blood loss B Defective bone marrow/stem cell function C Defective DNA production D Defective hemoglobin production E Increased destruction of red blood cells

Increased destruction of red blood cells The time course of the patient's presentation is consistent with multiple episodes of acute hemolysis. Defects in bone marrow (B) or red blood cell precursors (C and D) are refuted by the elevated reticulocyte count. Chronic blood loss (A) would have a more insidious, gradual onset and likely result in a decreased MCV.

A 37-year-old man has a 5-hour history of left-sided flank pain. He was previously healthy and takes no medications or supplements. He passed several grainy pieces in his urine 2 hours ago and brought them to the emergency room. Calcium oxalate crystals were identified. What dietary modifications will help reduce chances of future episodes of nephrolithiasis? Answer Choices 1 Decreased intake of citric acid foods 2 Increased intake of oxalate rich foods 3 Increased intake of sodium 4 Increased intake of fluids 5 Increased intake of proteins

Increased fluid intake Explanation This patient has passed calcium oxalate stones in his urine. Increasing his intake of fluids will help prevent future episodes of nephrolithiasis. At least 2 liters/day of fluid intake is suggested to prevent recurrent nephrolithiasis. Increased intake of water, coffee, and beer will all lead to decreased urine concentration of stone-forming solutes; alcoholic beverages inhibit the action and secretion of anti-diuretic hormone, leading to decreased urine solute concentration. In addition, reducing the intake of oxalate-rich foods, such as rhubarb, green leafy vegetables, chocolate, tea, liver, nuts, and seeds, is recommended. Crystals form in urine when the urine is supersaturated with crystal-forming solutes such as calcium, phosphate, and uric acid. Some patients overexcrete solutes, while others drink inadequate amounts of fluids to keep solutes dissolved. Stones also occur when the urine is infected with urea-splitting bacterium (Proteus). Here, urea is broken down into ammonia and bicarbonate, which then forms ammonium hydroxide and bicarbonate, the components of struvite stones. Struvite stones consist of a triple phosphate of calcium, magnesium, and ammonium Certain stone inhibitors, such as pyrophosphate, citrate, and magnesium, prevent crystal growth. In patients who have low levels of these inhibitors, stones are more likely to form. We do not know if this patient is deficient in urinary stone inhibitors, only that he passed calcium oxalate stones. 24-hour urine collections, preferably 2 samples drawn 6 weeks apart, are recommended to assess urinary citrate excretion. Decreased intake of oranges (being a type of citrus fruit) will likely decrease urinary citrate levels. This modification will be of little benefit if his urine citrate levels are normal and may be detrimental if his levels are low. Increased intake of liver is not recommended; liver is an oxalate-rich food. Increased sodium is not advised in a patient who is prone to developing urinary calcium stones. Increased sodium intake leads to increased calcium excretion, which may promote further calcium stone formation. Increased protein intake is not recommended for patients with risk or recurrent renal stones and should be limited to less than 80/g day. Nephrolithiasis is a common problem, affecting some ~ 2 - 9% of the population. Without preventative treatment, 50% of patients having an episode of nephrolithiasis will likely have recurrent episodes over the next 10 years. Because of its likely recurrence, measures to prevent recurrent nephrolithiasis (such as increasing water intake, decreasing sodium intake, and moderating calcium intake) are all important in a patient such as this one who has documented calcium phosphate stones.

A 50-year-old woman presents for her annual pelvic examination. She states her last menstrual period was over 6 months ago; the last few occurrences of menses were extremely irregular. The patient also describes having the sensation of intense heat in her face and trunk; the sensation is accompanied by sweating. She further states that these "heat episodes" have been occurring 1 or 2 times a week for the last several months. She has no other complaints at this time. She has received her annual pap and pelvic examination yearly, as well as a clinical breast exam, without any issues. During the pelvic examination, you note obvious vaginal thinning and excessive dryness; there is also apparent vaginal wall atrophy. Question Laboratory findings in this patient would include a decreased serum estradiol as well as what? Answer Choices 1 Decreased serum thyroid stimulating hormone (TSH) 2 Increased serum follicle stimulating hormone (FSH) 3 Decreased serum luteinizing hormone (LH) 4 Increased serum prolactin level 5 Decreased serum aldosterone level

Increased serum follicle stimulating hormone (FSH) Explanation This patient scenario above is most likely to be caused from menopause. In the most pure sense of explanation, menopause is a cessation of menstruation from either natural aging (usually amenorrhea for at least 6 months) or an external cause (surgical). There is usually a 1 - 3 year time period during which women will typically adjust physiologically to the diminished hormonal and menstrual actions as well as the effects this has on their body. These effects may include hot flashes, night sweats, vaginal dryness, and in the later stages, osteoporosis. The average age of menopause in western societies is around 51 years old. Laboratory findings that indicate natural menopause include elevated levels of FSH, LH, decreased levels of estradiol (estrogen), and normal levels of TSH and prolactin. Decreased levels of TSH would be seen in hyperthyroidism, which is also referred to as thyrotoxicosis. One of the many symptoms seen with hyperthyroidism includes menstrual irregularities; however, if TSH levels are corrected, the menstrual regularity will most likely return to its normal state. Serum prolactin levels will remain unchanged in naturally occurring menopause. Levels may be increased if amenorrhea is a consequence of a prolactin-secreting pituitary adenoma. Decreased levels of aldosterone in relation to amenorrhea may be found in patients who are experiencing toxemia of pregnancy, which is not likely in this patient.

A 28-year-old man presents with a 2-week history of an inguinal lump. He states he was lifting a heater, felt a "pop" in his groin, and began to notice a bulge in his lower abdomen that recently became mildly tender. Resting or lying flat provides some relief, while standing or lifting aggravates it. He denies any fever(s), nausea, vomiting, or changes in bowel habits, as well as any previous abdominal surgeries or procedures. Upon examination, a palpable soft, reducible mass in the lower abdomen is found, and hernia examination reveals a mass pushing against your finger. You order an ultrasound of the lower abdomen and find the intestinal sac has traversed the deep inguinal ring. Question What type of hernia does this patient have? Answer Choices 1 Direct inguinal hernia 2 Femoral hernia 3 Indirect inguinal hernia 4 Umbilical hernia 5 Incisional hernia

Indirect Inguinal hernia Explanation The correct answer is indirect inguinal hernia, as these hernias are characterized by the intestinal sac entering through the deep inguinal ring. Typically due to a congenital defect, there is incomplete obliteration of processes vaginalis. A direct inguinal hernia enters through the weakened abdominal fascia and into the anatomic region known as Hesselbach's Triangle. This area is bordered by the rectus abdominus, the inferior epigastric artery, and the inguinal ligament. Femoral hernias and umbilical hernias do not traverse through the deep inguinal ring, and femoral hernias occur lower on the body than inguinal hernias, near the leg crease. Umbilical hernias occur higher in the abdomen around the umbilicus. There was no previous surgery for an incisional hernia to occur.

A 28-year-old man presents with a 2-week history of a lump in his groin. While lifting a heater unit, he felt a "pop" in his groin and began to notice an outpouching in his lower abdomen; it has become mildly tender over the last week. Resting and lying flat appears to help, and standing and lifting aggravates it. He denies any fevers, nausea, vomiting, or changes in bowel habits. Patient denies any previous abdominal surgeries or procedures. Upon examination, you identify a soft, reducible mass in the lower abdomen, and hernia examination reveals a mass pushing against your finger. An ultrasound of the lower abdomen shows the intestinal sac has traversed the deep inguinal ring. Question What is the patient's condition called? Answer Choices 1 Direct inguinal hernia 2 Femoral hernia 3 Indirect inguinal hernia 4 Umbilical hernia 5 Incisional hernia

Indirect inguinal hernia The correct answer is indirect inguinal hernia; these hernias are characterized by the intestinal sac entering through the deep inguinal ring. They are typically the result of a congenital defect, and there is incomplete obliteration of processes vaginalis. A direct inguinal hernia enters through the weakened abdominal fascia and into the anatomic region known as Hesselbach's Triangle. This area is bordered by the rectus abdominus, the inferior epigastric artery, and the inguinal ligament. Femoral hernias and umbilical hernias do not traverse through the deep inguinal ring, and femoral hernias occur lower on the body than inguinal hernias near the leg crease. Umbilical hernias occur higher in the abdomen around the umbilicus. The patient did not have surgery; therefore, an incisional hernia could not have occurred.

A 67-year-old man presents for a routine follow-up visit. His past medical history includes chronic obstructive pulmonary disease (COPD) for the past 3 years and hypertension (HTN) for the past 5 years. Daily medications include a tiotropium inhaler and hydrochlorothiazide. He has smoked a pack of cigarettes a day for 50 years, and he has no desire to quit at this time. Physical exam remains unchanged from 3 months ago, and the patient has no complaints. His last pneumococcal and influenza vaccinations were 2 years ago. Question What should be offered at this visit? Answer Choices 1 Bupropion 2 Influenza vaccine 3 Fluticasone and salmeterol 4 Nicotine patch 5 Pneumococcal vaccine

Influenza vaccine Chronic obstructive pulmonary disease (COPD) affects approximately 32 million Americans and is the 4th leading cause of death in the US (1). The main cause of COPD is smoking. For this reason, COPD is considered a preventable disease. "The Global Initiative for Chronic Obstructive Lung Disease (GOLD) guidelines define COPD as a disease state characterized by airflow limitation that is not fully reversible, is usually progressive, and is associated with an abnormal inflammatory response of the lungs to inhaled noxious particles or gases" (Mosenifar, 2011, Overview). Treatment of COPD is aimed at limiting exacerbations and progression of disease. Smoking cessation is an important component of the treatment of COPD, but it will only be effective if the patient is willing to attempt to quit. Nicotine patches or gum can be used as nicotine replacement therapy to aid in smoking cessation. The dose of nicotine is gradually decreased. Bupropion (Zyban) is an antidepressant that can be used in conjunction with behavioral therapy to help in smoking cessation (1). Pharmacological treatment of stable COPD includes treating the airflow restriction and inflammation. Inhaled bronchodilators, such as beta-agonists and anticholinergic agents, are the mainstay of therapy. "Tiotropium is the only long-acting muscarinic agent available at this time and has become a first-line therapy in patients with persistent symptoms" (Mosenifar, 2011, Treatment). Combination therapy of inhaled beta-agonist and corticosteroids or inhaled anticholinergic and corticosteroids are indicated when a patient's symptoms are not controlled by monotherapy alone (1). In this case, the patient is controlled on tiotropium, so the addition of fluticasone and salmeterol (corticosteroid and long-acting beta-agonist) is not warranted. Pulmonary infections can lead to a COPD exacerbation; therefore, patients should routinely receive pneumococcal and influenza vaccines. The CDC recommends adults receive 1 dose at age 65 years. In the case presented, the patient had his vaccine at age 65 and does not need another one. The current recommendation for the influenza vaccine is that everybody receives an annual dose (2). Therefore, the patient in this case should be given an influenza vaccine at this visit.

You are currently on an inpatient hospitalist team in a local pediatric hospital. First thing this morning, your team is called in to evaluate an infant born at 27 weeks gestation 50 minutes ago. Upon initial inspection of the newborn, you observe rapid, labored grunting respirations, flaring nostrils, and retractions that are present above and below the breastbone. Auscultation reveals diminished air movement, and a chest radiograph reveals a ground glass appearance in the lung fields bilaterally. The patient is diagnosed with respiratory distress syndrome (RDS). Question What pharmacological agent should be initiated as soon as possible to help alleviate the patient's signs and symptoms? Answer Choices 1 Intravenous antivirals 2 Hepatitis B immunoglobulin 3 Intravenous antibiotic 4 Inhaled surfactant replacement 5 Inhaled corticosteroids

Inhaled surfactant replacement Respiratory distress syndrome (RDS) is also known as hyaline membrane disease (HMD) and is a condition that causes infants to need extra oxygen and assistance in breathing. It is one of the most common problems seen in premature infants; the more premature the baby, the higher the risk and the more severe the HMD. Many times HMD typically worsens over the first 48 to 72 hours after birth and then improves with treatment; more than 90 percent of babies with HMD survive. HMD occurs when there is not enough surfactant in the newborn's lungs. Surfactant is made by the cells in the airways and consists of phospholipids and protein. It begins to be produced in the fetus at about 24 to 28 weeks of pregnancy and is found in amniotic fluid between 28 and 32 weeks. By about 35 week's gestation, most babies have developed adequate amounts of surfactant. In healthy lungs, surfactant is released into the lung tissues to help lower surface tension in the airways and this helps keep the lung alveoli open. When there is not enough surfactant, the tiny alveoli collapse with each breath. As the alveoli collapse, damaged cells collect in the airways and this makes it even harder to breath. The newborn works harder and harder to breath with each breath, trying to re-inflate the collapsed airways. This vicious cycle can eventually lead to a build-up of carbon dioxide, eventual acidosis, and/or eventual physical exhaustion from trying to attempt to take breaths. The most common symptoms of HMD include difficulty breathing at birth that gets progressively worse, cyanosis, flaring of the nostrils, tachypnea, grunting sounds, and chest retractions. Chest radiographs will reveal a characteristic "ground glass" appearance". Treatment of HMD consists of placing an endotracheal tube, supplemental oxygen, continuous positive airway pressure (CPAP), and inhaled surfactant replacement with artificial surfactant. This treatment has been shown to reduce the severity of HMD and is most effective if started within the first 6 hours of birth. Artificial surfactant comes as a powder that is mixed with sterile water and given through the ET tube. RDS stemming from HMD is not a viral or bacterial infection; therefore, intravenous antivirals or antibiotics would be an inappropriate choice. This also is not specifically an issue with Hepatitis B infection, therefore, the Hepatitis B immunoglobulin answer is incorrect. Inhaled corticosteroids may give some minor symptomatic relief, but they are not an indicated or efficient treatment for patients with hyaline membrane disease or respiratory distress syndrome.

A 27-year-old woman has a history of recurrent ovarian cysts; she is being treated with a combination oral contraceptive (norethindrone/ethinyl estradiol 1mg/35mcg). The norethindrone in this drug acts to suppress ovulation by what process? Answer Choices 1 Decreasing circulating sex-hormone binding globulin (SHBG) 2 Increasing gonadotropin production 3 Inhibiting release of follicle-stimulating hormone (FSH) secretion from the anterior pituitary 4 Inhibiting release of luteinizing hormone (LH) secretion from the anterior pituitary 5 Stimulating proliferation in the endometrial lining

Inhibiting release of luteinizing hormone (LH) secretion from the anterior pituitary Explanation Norethindrone is the progestin component of the oral contraceptive, which "primarily suppresses luteinizing hormone (LH) secretion (and thus prevents ovulation)."1 The LH 'surge' is responsible for triggering ovulation. The LH is released from the anterior pituitary and is suppressed in a negative feedback cycle when progestin levels are increased. The norethindrone would be expected to increase circulating sex-hormone binding globulin (SHBG), resulting in decreased free androgens in the serum.2 Both the estrogen and progestin components of this drug decrease gonadotropin (LH and FSH) production by negative feedback to the anterior pituitary. The estrogenic agents in oral contraceptives (ethinyl estradiol) suppress follicle-stimulating hormone (FSH), as well as provide stability to the endometrium and potentiation of the progestin's actions. The effect on FSH is primarily attributed to the estrogen, while the effect on LH is primarily attributed to the progestin, such as norethindrone. Estrogen stimulates proliferation in the endometrial lining. Progestins, such as norethindrone, are associated with endometrial thinning and a shift to the secretory phase.1Neither effect on the endometrium leads to ovulation suppression.

A 43 year-old woman with a past medical history of HIV infection presents to the clinic due to a 3-month history of dry, non-productive cough and progressive dyspnea. The patient is subsequently prescribed trimethoprim-sulfamethazole. What is the mechanism of action of this sulfamethazole? A Binds to the 30S ribosomal subunit to inhibit protein synthesis B Binds to the 50S ribosomal subunit to inhibit protein synthesis C Inhibits dihydropteroate synthase and folate production D Inhibits DNA replication by binding to DNA gyrase and topoisomerase IV E Inhibits the transpeptidation reaction

Inhibits dihydropteroate synthase and folate production Sulfamethoxazole inhibits dihydropteroate synthase and folate production (C). Tetracycline binds to the 30S ribosomal subunit to inhibit protein synthesis (A). Macrolide antibiotics bind to the 50S ribosomal subunit to inhibit protein synthesis (B). Ciprofloxacin inhibits DNA replication by binding to DNA gyrase and topoisomerase IV (D). Beta lactam antibiotics inhibit the transpeptidation reaction leading to cell wall destruction (E).

A 38-year-old Caucasian man with a long-standing diagnosis of acromegaly has come to you with a concern regarding his chance of developing colon cancer. He had read some articles on the internet suggesting that patients with the diagnosis of acromegaly have an increased incidence of developing colon cancer, and this has caused him tremendous worry. The patient has no known personal or family history of diverticular disease, colon polyps, or colon cancer. He had a successful selective trans- sphenoidal surgical resection of a pituitary micro adenoma over 20 years ago and has had routine periodic serum insulin-like growth factor-1(IGF-1) levels drawn that have always been within normal range. Question Based on the information above, what would be the recommendation for performing a colonoscopy in this patient at this time? Answer Choices 1 Initial colonoscopy now and every 5 years thereafter 2 Initial colonoscopy at 40 years old and every 5 years thereafter 3 Initial colonoscopy at 40 years and every 3 years thereafter 4 Initial colonoscopy at 50 years and every 5 years thereafter 5 Initial colonoscopy at 50 years and every 3 years thereafter

Initial colonoscopy at 50 years and every 3 years thereafter Explanation There is conflicting evidence indicating that patients with the diagnosis of acromegaly have an increased risk of developing colon cancer. There is a higher incidence of colon cancer mortality in patients who have a history of uncontrolled growth hormone or IGF1 levels. The key risk factors in this patient population that increase one's chance include the patient being over 50 years old and male, having 3 or more skin tags, having a personal history of colon polyps, and having a family history of colon cancer. The patient in the above scenario has only 1 of these risk factors, as he is male. The current recommendation for screening follows the 2009 guidelines from the Acromegaly Consensus Group, which recommends the following: baseline colonoscopy at the age of 50 and then every 3 years thereafter. This should be strictly followed, especially if the patient has a history of poorly controlled growth hormone levels.

A 42-year-old schoolteacher presents with a hand tremor; the tremor has been present for several years, but it is getting more bothersome. The patient would like treatment for her tremor. It appears to be present only in her right hand, and she tells you that it "comes and goes". It is especially embarrassing for her because she writes on the board in front of her students. She denies pain, motor weakness, and abnormal sensation in the right hand. She reports that her father's hand used to "shake" when he drank his coffee. She otherwise feels well, and she denies other symptoms. She has not noticed memory problems, incoordination, or balance problems. Her past medical history is unremarkable. She had 3 uncomplicated pregnancies and deliveries; she has no chronic medical conditions. Except for a mild tremor on the patient's right hand when held extended, the physical exam is unremarkable. Her neurological exam is otherwise normal. Question What is the most appropriate intervention for this patient's condition? Answer Choices 1 Advise the patient her condition is benign and no treatments are available 2 Initiate carbidopa/levodopa 3 Initiate methylphenidate 4 Initiate propranolol 5 Referral for physical therapy

Initiate Propranolol Based on her history and exam, this patient has a benign essential (familial) tremor, for which it is appropriate to initiate propranolol. Propranolol is a non-selective beta-blocker. This patient has no contraindications to the therapy; she is troubled by the worsening tremor, which interferes with her daily activities. Essential tremor is indeed a benign condition. When the tremor is mild and does not cause embarrassment or interfere with the patient's activities, it would be reasonable to avoid medications, but it is inappropriate to tell the patient that no treatments exist. Especially in this case, the patient desires treatment. Carbidopa/levodopa would be an appropriate medication for an individual with tremor due to Parkinson's disease; however, this patient has no other neurological complaints and no history to suggest she currently has Parkinson's disease. Methylphenidate is a stimulant; it is typically used for the treatment of attention deficit hyperactivity disorder. Not only would it be ineffective in this patient's case, it would also likely make her tremor worse. Physical therapy has not been proven to have a role in treating essential tremor. It would not be helpful in this case.

A 48-year-old man presents to the family practice clinic for evaluation of fatigue, weakness, and nausea. He reports that his symptoms have progressively worsened over the last 6 - 8 months. He reports fatigue despite adequate sleep; he is also experiencing an overall feeling of muscle weakness, nausea with occasional vomiting, a weight loss of about 12 pounds, headaches, and muscle aches. His wife thinks he appears tanned year-round, despite a lack of sun exposure. He admits feeling anxious and somewhat irritable, but he denies any major psychosocial or traumatic events surrounding onset of symptoms. Prior to the onset of his symptoms, he was healthy and active. His past medical history reveals no chronic medical conditions, no medication use, and no history of surgery; he does not have any allergies. His family history is significant for thyroid disease in a sister and his mother; there is also a history of diabetes mellitus type I in a brother. He teaches high school, and he lives with his wife and children; he denies the use of tobacco, alcohol, and drugs. On physical exam, he is noted to be hypotensive and hyperpigmented. The remainder of his physical exam is normal. Several labs are performed, and the results are as follows: Urinalysis- Normal Complete blood count- Mildly decreased hemoglobin and hematocrit Comprehensive metabolic panel- Mildly decreased sodium and elevated potassium, rest normal Adrenocorticotropic hormone (ACTH)- Elevated Cortisol (morning level)- Decreased ACTH stimulation test- Decreased cortisol Question Once this patient is diagnosed, what choice represents the most important intervention for his presumed condition? Answer Choices 1 Increase dietary sodium 2 Initiation of antidepressant with anti-anxiety properties 3 Initiation of steroids 4 Refer to oncologist 5 Restrict dietary potassium intake

Initiate Steroids Explanation This patient is presenting with a corticoadrenal insufficiency, which is also known as Addison's disease. In the United States, this rare disease is most often linked with autoimmune causes. The affected patient will have a variety of symptoms related to low glucocorticoid and mineralocorticoid production. Signs and symptoms include fatigue, nausea, vomiting, headaches, anorexia, myalgias, arthralgias, muscle weakness, anxiety, mental irritability, hyperpigmentation, hypotension, hyponatremia, hyperkalemia, anemia, and many more. The mainstay of treatment is initiation of steroids, often with oral hydrocortisone, unless urgent IV treatment is needed. Patients with Addison's disease commonly have hyponatremia and hyperkalemia. They can be instructed to increase dietary sodium, especially in times of exercise and hot weather. However, this alone will not address the underlying adrenal insufficiency, and failure to initiate steroid treatment will risk patient deterioration. With a variety of somatic and psychiatry complaints, a patient presenting like this may be falsely given a psychiatric diagnosis, and he may be treated with initiation of an antidepressant with anti-anxiety properties; however, the anxiety and irritability are symptoms of the adrenal disorder, and they should improve with treatment of the underlying disorder. It would be reasonable to monitor this patient and possibly treat him with antidepressants if his symptoms do not resolve. While Addison's disease is serious and chronic, it is not malignant. This patient's presentation with weight loss, nausea, and fatigue could be suggestive of cancer. However, with the laboratory values indicating adrenal insufficiency, it would be inappropriate to refer the patient to an oncologist. Even as a mild hyperkalemia is indicated on this patient's lab values, restriction of dietary potassium is not an appropriate treatment of his primary condition. The hyperkalemia is a result of the adrenal insufficiency. With appropriate treatment with steroids, such as hydrocortisone, the electrolyte abnormalities are likely to improve without additional intervention.

A 27-year-old woman and her male partner come to the emergency department for assistance with emergency contraception. They experienced condom failure during intercourse an hour ago and neither desires pregnancy. Her last menstrual period was approximately two weeks ago and her cycles occur every 28-30 days. Her medical history includes a deep venous thrombosis during labor and delivery 5 years ago. What is the most appropriate course of action at this time? A Administration of an ethinyl estradiol and levonorgestrel combination now and in 12 hours B Dilation and curettage C Insertion of a copper-containing intrauterine device D Serial beta hCG determinations E Testing for factor V Leiden

Insertion of a copper-containing intrauterine device Insertion of a copper-containing IUD is an effective means of preventing an unintended pregnancy in this case. The woman's history of clotting is a contraindication to use of combination oral contraceptives (A). Dilation and curettage (B) is neither appropriate nor effective for emergency contraception. Performing beta hCG determinations (D) would merely detect pregnancy if it were to occur. Given her history of clotting, testing for factor V Leiden (E) may be appropriate but will not affect management at this time.

A 32-year-old Caucasian woman has a past medical history of Hashimoto's thyroiditis, type I diabetes mellitus, and pernicious anemia; she presents with a 2-year history of insidious and intermittent fatigue, anorexia, involuntary weight loss, nausea, abdominal pain, vomiting, and dizziness that is associated with position changes. Her physical exam is noteworthy for postural hypotension, with a maximum systolic blood pressure of 104 in the supine position. Additionally, she has a low-grade fever and a generalized pigment change to her skin, as indicated in the image below. Question What health maintenance recommendation should be provided? Answer Choices 1 Periodic DEXA scans are useful in detecting steroid-induced increased bone density 2 Close clinical monitoring during treatment is unnecessary 3 A decrease in salt intake is necessary in hot weather 4 A reduction of steroid replacement doses is needed in stressful situations 5 Instructions on self-administration of IM hydrocortisone on an as-needed basis

Instructions on self-administration of IM hydrocortisone on an as-needed basis Explanation This patient's most likely diagnosis is primary adrenal insufficiency, which is known as Addison's disease. Treatment involves replacement of corticosteroids and mineralocorticoids. Patients should be instructed on how to give themselves IM injections. They should be given a prescription for parenteral hydrocortisone for use on occasions when oral intake may not be possible or when marked vomiting or diarrhea occurs. A periodic bone dual-energy radiographic absorptiometry scan may be useful in detecting early reductions of bone density and osteoporosis in patients who are inadvertently over-replaced with maintenance steroids. Patients on steroid replacement therapy need to be closely monitored by their primary care physician and by an endocrinologist. Close monitoring for any signs of inadequate replacement (e.g., morning headaches, weakness, and dizziness) and any signs of over-replacement (e.g., cushingoid features) should prompt appropriate dosage adjustment. Patients may need to be advised to increase salt intake in hot weather. Patients should be instructed to double or triple their steroid replacement doses in stressful situations, such as a common cold or tooth extraction.

A patient was recently diagnosed with type 1 diabetes mellitus. A treatment plan was initiated, with a combination regimen of insulin. Which of the following types of insulin works well with a rapidly acting insulin, such as insulin lispro, to provide 24-hour coverage for the patient? A NPH insulin B Regular insulin C Insulin aspart D Insulin glargine E Humalog 75/25

Insulin glargine The correct choice is D, insulin glargine. This is the only long acting insulin listed. The combination of a long acting insulin with a rapidly acting insulin provides physiologic insulin replacement to the patient. This regimen provides postprandial control after meals and basal coverage throughout the day and night. Choice A NPH insulin, can be used by itself in two or more injections throughout the day. Choice B, regular insulin, can be used instead of rapid acting insulin, and not in combination with it. Choice C, insulin aspart, is a type of rapidly acting insulin and would not be used in combination with another rapidly acting insulin. Choice E, Humalog 75/25, is a combination insulin preparation with 75% intermediate acting insulin and 25% insulin lispro.

A 51-year-old woman presents with a long history of blurry vision, palpitations, and weakness. Her lab results are as follows: TEST RESULTS REFERENCE RANGE Calcium 8.9 mg/dL 8.4-10.2 mg/dL Potassium 3.8 mEq/L 3.5-5.0 mEq/L Sodium 144 mEq/L 135-145 mEq/L Glucose 41 mg/dL 70-110 mg/dL You suspect a pancreatic islet cell tumor. What is the most likely diagnosis? Answer Choices 1 VIPoma 2 Insulinoma 3 Gastrinoma 4 Glucagonoma 5 Somatostatinoma

Insulinoma Her low blood glucose levels indicate an insulinoma. The endocrine portion of the pancreas is housed in the islets of Langerhans. Tumors that arise from this area are referred to as islet cell tumors. The specific type of tumor depends on the cell type involved. Typically, the tumor is named based on the hormone it is secreting. An insulinoma is an islet cell tumor. Insulin is produced by the beta cells of the islets of Langerhans. Therefore, an insulinoma is a tumor of the beta cells. An insulinoma is the most frequently occurring islet cell tumor. Hypoglycemia, secondary to the insulin secretion, is a symptom. Verner-Morrison syndrome is due to a VIPoma. This is an endocrine tumor of the pancreatic islets of Langerhans. It secretes vasoactive intestinal polypeptide. Vasoactive intestinal polypeptide is produced by the D1 cells of the islets of Langerhans. Therefore, a VIPoma is a tumor of the D1 cells. Other names for a VIPoma include pancreatic cholera and WDHA-syndrome (watery diarrhea, hypokalemia and achlorhydria). Gastrinomas are most often due to islet cell tumors. On occasion, gastrinomas can arise outside of the pancreas. Peptic ulcers, secondary to the excessive gastrin secretion, are frequent. Gastrinomas and peptic ulcerations are referred to as the Zollinger-Ellison syndrome. A glucagonoma is an islet cell tumor. Glucagon is produced by the alpha cells of the islets of Langerhans. Therefore, a glucagonoma is a tumor of the alpha cells. Diabetes can be seen with a glucagonoma. Other symptoms include a skin rash, weight loss, and anemia. A glucagonoma is a rare islet cell tumor. A somatostatinoma is an islet cell tumor. Somatostatin is produced by the delta cells of the islets of Langerhans. Therefore, a somatostatinoma is a tumor of the delta cells. Somatostatinoma is a rare islet cell tumor.

What condition is characterized by fine movements that are absent at rest, occur with activity, and worsen as the target is neared? Answer Choices 1 Postural tremor 2 Asterixis 3 Intention tremor 4 Chorea 5 Static tremor

Intention Tremor Intention tremors are fine movements of the hands that are absent at rest, occur with activity, and worsen as the target is neared. They occur with multiple sclerosis and cerebellar disease. Choreiform movements are brief, rapid, jerky, unpredictable motions of the hands. They occur both at rest and during normal actions. They are associated with both Huntington's and Sydenham choreas. Asterixis is non-rhythmic flapping movements of the hands, especially if the wrists are dorsiflexed. It is associated with liver failure, renal failure, and pulmonary insufficiency. Static, or resting, tremors are coarse movements of the hands that are present at rest, disappear with movement, and may involve alternation of the fingers in a "pill-rolling" manner. They are associated with Parkinson's syndrome and extrapyramidal dysfunction. Postural tremors are fine movements of the hands that occur when the hands are held in an active position, usually against gravity. Examples of postural tremors are those resulting from anxiety, fatigue, and hyperthyroidism, as well as familial postural tremors.

A 40-year-old man presents with a 2-year history of severe, burning epigastric pain. A detailed history reveals that the pain is greatest in the early hours of the morning and wakes him up from sleep. The pain is also felt 2-3 hours after meals. He reports diarrhea for the past 2 years and black stools for the past 3 days. On examination, his pulse is 74/min and blood pressure 136/84 mm Hg. There is slight epigastric discomfort on palpation. Lab examination shows occult blood in the stool and hyperchlorhydria. What could be a complication in this patient? Answer Choices 1 Gallstones 2 Intestinal ulcers 3 Pernicious anemia 4 Intestinal alkalosis 5 Weight gain

Intestinal Ulcers The diagnosis in this case is gastrinoma (Zollinger-Ellison disease). Gastrinomas are small tumors that continually release excessive amounts of gastrin. Hypergastrinemia has a trophic effect on the gastric mucosa and enhances the secretion of hydrochloric acid from gastric parietal cells in both basal and stimulated states. The continued delivery of acid to the upper small intestine exceeds the capacity of acid-neutralizing mechanisms and results in the erosion and ulceration of the intestinal mucosa. In addition to acid secretion, parietal cells release intrinsic factor in response to gastrin. Intrinsic factor is a glycoprotein required for the normal intestinal absorption of vitamin B12. Red blood cell maturation is retarded in the absence of intrinsic factor resulting in pernicious anemia. Intestinal acidosis in patients with hypergastrinemia also reduces the activity of pancreatic enzymes, particularly lipases. This inhibits digestion and reabsorption of fat resulting in steatorrhea (fat in the feces) and weight loss. Gastrinomas may be either sporadic or associated with autosomal dominantly inherited Multiple Endocrine Neoplasia (MEN) type I.

A 54-year-old woman presents to the emergency department with nausea, vomiting, right-upper quadrant abdominal pain, fever, and jaundice starting 4 hours ago. During the last 6 months, she suffered several bouts of upper-abdominal pain accompanied by nausea, vomiting, and occasional jaundice, for which medical attention was sought. Her past medical history includes hyperlipidemia, for which she first took simvastatin; she switched to cholestyramine because of side effects. Vital signs on admission are as follows: BP 110 / 80 mm Hg, HR 90 bpm, RR 20 rpm, temperature 38.1°C (100.6 F). She is alert and oriented, and mildly jaundiced. Her right-upper abdomen is diffusely tender to palpation. An upper abdominal ultrasound is performed and reveals a thickening and calcifications of the gallbladder wall, but there are no signs of air within the peritoneal cavity or the bile ducts. What is the most appropriate next step in management? Answer Choices 1 Intravenous hydration and antibiotics 2 Endoscopic papillotomy 3 Give analgesics and refer for elective cholecystectomy 4 Magnetic resonance cholangiography to look for choledocholithiasis 5 Emergency cholecystectomy

Intravenous hydration and antibiotics The presence of fever, jaundice, and right-upper quadrant pain defines Charcot's triad, which is the classical presentation of acute cholangitis. Reynolds' pentad consists of Charcot's triad plus sepsis/shock and mental status changes. 95% of patients presenting with these syndromes have common duct stones. In most cases, there is a favorable response to conservative treatment, which consists of interrupting oral feeding, analgesia, intravenous hydration, and antibiotics. The following antibiotic schemes can be used: A third-generation cephalosporin + an aminoglycoside Piperacillin or ampicillin + metronidazole + an aminoglycoside Monotherapy with imipenem, meropenem, mezlocillin, ampicillin-sulbactam, ticarcillin-clavulanate, or piperacillin-tazobactam After clinical improvement, cholecystectomy is undertaken in the first few days after the initial hospitalization in order to avoid recurrence. Surgery is warranted because the recurrence rate is as high as 10% per year in patients whose gallbladders are not removed. The use of cholestyramine is another risk factor for recurrence in this patient, and gallbladder calcifications (porcelain gallbladder) are thought to pose a risk of malignization and are considered an indication for cholecystectomy. When the disease progresses despite the initial conservative treatment (i.e., there is worsening fever, leukocytosis, abdominal pain, and guarding), emergency biliary drainage is warranted because of the possibility of gallbladder perforation or gangrene. Some possible approaches are cholecystectomy (conventional or laparoscopic), cholecystostomy, or percutaneous drainage. The latter 2 are usually reserved for sicker patients, who are less likely to tolerate surgery well.

A 2-year-old boy presents with acute abdominal pain. The boy has not passed a stool for 2 days. On examination, the abdomen is distended and tender. Bowel sounds are absent. The boy is taken to surgery. Refer to the image for the gross appearance of the surgical specimen. What is the most likely diagnosis in this case? Answer Choices 1 Acute appendicitis 2 Meckel's diverticulitis 3 Intussusception 4 Volvulus 5 Ischemic bowel disease

Intussusception Intussusception is the telescoping of a segment of intestine into the segment below due to peristalsis. The telescoped segment is called the intussusceptum and lower receiving segment is called the intussuscipiens. The condition commonly occurs in infants and young children, more often in the ileocecal region. Less commonly, ileoileal and coli-colic intussusception occurs. In children, the cause is not known, though enlargement of the lymphoid tissue in terminal ileum has been suggested. In adults, the usual causes are foreign bodies and tumors. The main complications include intestinal obstruction, infarction, gangrene, perforation, and peritonitis. The picture shows cecum, which has been cut open to show the ileum that has intussuscepted inside. Acute appendicitis, the acute inflammation of the appendix, is the most common acute abdominal condition confronting the surgeon. The most common etiological factor is the obstruction of the lumen that leads to increased intraluminal pressure. This presses upon the blood vessels to produce ischemic injury, which in turn favors the bacterial proliferation and acute appendicitis. Grossly, the appendix is swollen and hyperemic during early stages. The serosa then is covered with fibrinopurulent exudate, called acute suppurative appendicitis. As the disease advances, there is necrosis and ulceration of the mucosa, which extends through the wall so that the appendix becomes soft and friable and the surface shows greenish black gangrenous necrosis (known as acute gangrenous appendicitis). Microscopically, the diagnostic criterion is the neutrophilic infiltration of the muscularis. Clinically, the patient presents with acute abdominal pain. If not adequately managed, complications like peritonitis, appendicular abscess, adhesions, mucocele, and portal pylephlebitis occur. Meckel's diverticulum is the most common congenital anomaly of the gastrointestinal tract. It occurs in 2% of population, and it is more common in men. It is commonly situated on the antimesenteric border of the ileum, about 1 meter above the ileocecal valve. Meckel's diverticulum is an out pouching containing all the layers of the intestinal wall in their normal orientation. It is usually lined by small intestinal type of epithelium. At times, it may contain gastric mucosa or pancreatic tissue. Meckel's diverticulitis is the common complication of the diverticulum. If not treated, it may lead to perforation and hemorrhage. Volvulus is the twisting of loop of intestine upon itself through 180 degrees or more. This leads to obstruction of the intestine as well as cutting of vascular supply to the affected region. It is usually due to bands, adhesions, and long mesenteric attachment. The condition affects mainly the sigmoid colon. Ischemic bowel disease applies to the structural changes in the colon that occur due to deprivation of blood supply. The causes for deprivation of blood could be arterial occlusion, venous occlusion, and non-occlusive ischemia. The causes of non-occlusive ischemia include cardiac failure, shock, dehydration, and vasoconstrictive drugs. Depending on the severity and the presentation, it is classified into transmural, mural and mucosal infarction, and chronic ischemic colitis. In transmural infarction, there is hemorrhagic infarction of the bowel with gangrenous changes, and a risk of perforation. In the mucosal and mural type, the mucosa is hemorrhagic and oedematous but the remaining layers will remain intact. In chronic ischemia, mucosal ulceration and inflammation develops, and sub-mucosal chronic inflammation and fibrosis may lead to stricture formation. In the setting of transmural infarction, patients generally present with acute abdominal pain, vomiting, fever, and bloody diarrhea. The patient may progress to shock and vascular collapse within hours.

What mineral is essential to prevent goiter? Answer Choices 1 Iodine 2 Vitamin B12 3 Calcium 4 Iron 5 Magnesium

Iodine Although the body needs only a trace amount of the mineral iodine, it is essential for the synthesis of the thyroid hormone, thyroxine. The thyroid gland enlarges in iodine deficiency and sometimes become enlarged enough to be visible as a lump in the neck called a goiter. Vitamin B12 is essential in maintaining and protecting the myelin sheaths of nerve fibers. Calcium is vital to bone formation. Most of Calcium is stored in teeth and bones. 1% of this mineral is in cellular fluid, which aids in cell membrane transport, blood clotting, muscle contraction, and maintaining blood pressure. Iron aids in transporting oxygen throughout the body. Only 1% of magnesium can be found in cellular fluids. The majority of magnesium is found in bones and muscles, and the rest of the mineral can be found in the liver, muscle, and heart. Magnesium is essential in numerous cellular functions such as potassium, calcium, and vitamin D metabolism.

A 68-year-old woman presents with a 4-hour history of severe left-sided abdominal pain. The pain was initially associated with several episodes of diarrhea with some hematochezia. Since then, she has had a few more stools, but she has not seen any more frank blood. Her pain is somewhat improved from what it was at the onset. Past medical history is positive for renal lithiasis and atrial fibrillation. Current medications are digoxin and aspirin. She is a recovering alcoholic who last had a drink 15 years prior. She recently returned home from a trip to Mexico; she was visiting relatives for 2 months. On exam, you see a well-nourished woman in extreme discomfort. Auscultation of her lungs reveals good breath sounds bilaterally; her heart has an irregularly irregular rhythm with a rate of 92; there is a soft II/VI systolic murmur. Her abdomen is mildly obese and tender on the left side; there is no appreciable mass or rebound. There is no flank tenderness. Rectal exam reveals guaiac positive mucus mixed with flecks of bright red blood. 12 hours after presentation, her left upper quadrant pain is still present; however, it is much improved, and she had only 2 more stools with small flecks of blood in them. Question What is the most likely diagnosis? Answer Choices 1 Left ureteral lithiasis 2 Diverticulitis 3 Infectious colitis 4 Small bowel ischemia 5 Ischemic colitis

Ischemic Colitis This is a classic presentation of ischemic colitis. Ischemic colitis occurs when there is obstruction of the colonic blood supply, causing ischemia with inflammation and ulceration of the colonic mucosa. The area of the splenic flexure is most at risk for ischemia because it is the watershed area of the arterial supply to the colon. Ischemic colitis is most commonly seen in elderly individuals. These patients typically present with an acute onset of pain associated with bloody diarrhea, and they have an area of tenderness corresponding to the ischemic segment of colon. Patients may experience recurrent bouts of ischemic colitis. Sigmoidoscopy makes the diagnosis in 85% of patients; 15% will have ischemia in areas proximal to the reach of a sigmoidoscopy. The majority of these patients will improve with supportive care (hydration and prophylactic antibiotics, in case of bacterial transmigration). However, some patients have much more severe disease, which may even require emergency colectomy. The most important condition to differentiate from ischemic colitis is that of small bowel ischemia. Small bowel ischemia can lead to gangrene of the intestine. Most commonly, these patients have developed occlusion of some of the distribution of the superior mesenteric artery, either from acute thrombosis in an area of atherosclerosis, or from embolic disease. These patients classically have pain out of proportion to their exam, at least early in the process, and they do not usually have an area of localized tenderness on palpation. These patients can have leukocytosis on the level of 20,000 to 30,000/μL, and they may exhibit a lactic acidosis and elevated amylase levels. Their stool is usually positive for occult blood, but frank blood is only rarely seen. Early diagnosis is very important, although often difficult, and urgent surgery is the only treatment. Renal or ureteral lithiasis can cause severe abdominal pain, and these patients can have diarrhea as well, but it should not cause hematochezia or localized anterior abdominal tenderness. Diverticulitis is more common on the left, which is the case with this patient's pain, but it typically does not cause hematochezia. Additionally, the sudden onset of very severe pain would not be typical of diverticulitis. Infectious colitis can cause abdominal pain and hematochezia, but the improvement in her pain over such a short time period would be atypical for an enterotoxic infection. Infection should be considered in this patient because of her recent travel history.

A 77-year-old woman presents with weakness, impaired taste, and poor wound healing. She resides in a nursing home and began noticing a change in her usual state of health about 3 weeks ago. She has a past medical history significant for chronic renal failure. She gets dialysis 3 days a week and has done so for the past 2 years. Physical examination reveals a thin, frail woman in no acute distress. She has 3 small sores on her right leg; they are in various stages of healing. There is no sign of infection. The rest of her exam is normal. Laboratory analysis shows that she has a normal urinalysis, CBC, and WBC; however, her zinc level is low. The rest of her blood work is within normal limits. Question What concerning this condition is true? Answer Choices 1 It is very common in the United States 2 Diagnosis is usually made by laboratory analysis 3 Cardiac irregularities are common 4 It is common in children 5 It is responsible for fetal malformations

It can be dx with lab analysis This patient has a zinc deficiency, which is established by low zinc level and the associated clinical presentation. Diagnosis is usually made by laboratory analysis. World Health Organization (WHO), the United Nations Children's Fund (UNICEF), the International Atomic Energy Agency (IAEA), and the International Zinc Nutrition Consultative Group (IZiNCG) have set the standard recommendations for functional indicators of zinc status in populations. The recommended indicator is serum zinc concentration less for the specific cut-offs for sex, age, etc. The causes of zinc deficiency include malnutrition, chronic debilitating diseases, chronic renal disease, alcoholism, drugs such as penicillamine and diuretic, and genetic disorders, such as sickle cell disease. Clinical manifestations in severe cases include alopecia, diarrhea, weight loss, infections, dermatitis, hypogonadism in men, and intercurrent infections. Supplementation with zinc is the treatment of choice. Zinc deficiency is not very common in the United States. The dietary intake, especially in children, exceeds the new dietary reference intakes, and the present level intake seems adequate. The World Health Organization/Food and Agriculture Organization/International Atomic Energy Agency (WHO/FAO/IAEA) and the Food and Nutrition Board/US Institute of Medicine (FNB/IOM), as well as recently the International Zinc Nutrition Consultative Group (IZiNCG) have set the average requirement for dietary zinc intake. Zinc deficiency is not commonly known to cause cardiac irregularities, unlike low levels of copper, in which the incidence of cardiovascular disease is increased. The effects of zinc deficiency in pregnancy, such as causing fetal skeletal malformations, have not been determined definitively; however, marginal deficiencies are found to be associated with preterm delivery, prolonged labor, etc. Zinc deficiency seems to be more prevalent in adults, as not many studies have been made in children, except where it is associated with disease conditions, such as diarrhea and malnutrition.

A 38-year-old man presents with upper abdominal pain. More recently, he has experienced nausea. A stool sample is tested for occult blood, and it is positive. An upper endoscopy reveals no esophageal lesions, but there is a solitary 2-cm diameter ulceration of the stomach. The ulceration is sharply demarcated. What is the most appropriate statement regarding this lesion? Answer Choices 1 It is probably located in the antrum 2 It is probably associated with increased gastric acid production 3 A gastrinoma of the pancreas is probably present 4 It is probably malignant 5 Because of its small size, a biopsy is not necessary

It is probably located in the antrum. From the findings described in the above patient with a sharply demarcated area of ulceration suggests that it is a benign peptic ulcer. The gastric antrum is a typical location for benign peptic ulcer. There is usually decreased acid production in patients with peptic ulceration, but not a total absence of acid production. A gastrinoma of the pancreas (Zollinger-Ellison syndrome) will be accompanied by multiple gastric ulcerations. The small size with the sharp demarcation suggests that it is a benign ulcer; however, the size is not a reliable clue that it is benign. All gastric ulcers seen on endoscopys hould be biopsied.

A 23 year-old male with cystic fibrosis inquires about the availability of treatments that can help improve his lung function. Which of the following treatments is most effective at reversing the pulmonary effects of cystic fibrosis? A Albuterol B Azithromycin C Inhaled hypertonic saline D Inhaled levofloxacin E Ivacaftor

Ivacaftor Ivacaftor (E) is the only treatment that restores function of the CFTR protein in cystic fibrosis patients with a G551D mutation thereby reversing the effects of the disease, approximately 5% of all cystic fibrosis patients have the G551D mutation. Albuterol (A) and hypertonic saline (C) are indicated to improve lung function and mucous clearance. Azithromycin (B) and inhaled levofloxacin (D) are used to treat chronic infection/colonization with pseudomonous.

You are on duty in the ER when a young man is brought in by ambulance. The paramedics report that they were called by this man's roommate who reported the patient was confused and dazed, and obviously very ill. The patient is acutely febrile and delirious; there is evidence of meningeal involvement. Subsequent diagnosis reveals viral encephalitis. What is the most common cause of viral encephalitis worldwide? Answer Choices 1 La Crosse 2 Herpes simplex 3 Japanese B 4 Eastern equine 5 Western equine

Japanese B Viral encephalitis is secondary to viral infection of the brain tissue itself, causing widespread death of nerve cells and glia. Approximately 1,000-5,000 cases per year are reported in the U.S. Worldwide, Japanese B encephalitis is the most common cause of epidemic encephalitis. However, herpes simplex is the most common cause of sporadic encephalitis in the U.S. Arthropod-borne viral encephalitides are transmitted through mosquitoes (most often), and ticks and are named for the region of the US affected. These included LaCrosse, St. Louis, Venezuelan equine, Western equine, and Eastern equine encephalitides. LaCrosse, a type of California encephalitis, is the most common arthropod-borne encephalitis in the U.S. Eastern equine encephalitis (EEE) is a summertime disease that affects children in the east, south and on the Gulf Coast. The mortality is 50-75%, with 80% of patients having neurological deficits. Western equine encephalitis (WEE) occurs also in the summer months in states west of the Mississippi River. The mortality is 5-15%, with infants being the most susceptible. Venezuelan equine encephalitis (VEE) occurs in the southwestern U.S. and Central and South America and has very low mortality and morbidity rates. This infection has been linked to epidemics in tens of thousands of cases, however. St. Louis encephalitis occurs in the central, western and southern U.S. and affects older adults in the autumn months.

A mother brings her 16-year-old son to your medical office for a comprehensive history and physical examination. She tells you she is concerned about his immature physical development and insecure behavior. She thinks these characteristics are markedly different from her other children. His IQ is 70, and he is in special education for a language-based learning disability. On physical examination, he is tall and thin; he has sparse body hair and a high-pitched voice. Heart, lungs, abdomen, and neurologic exam are unremarkable. Pertinent positive findings include disproportionately long arms and legs, gynecomastia, as well as small testes and phallus. Question What is the most likely diagnosis? Answer Choices 1 Fragile X syndrome 2 Klinefelter syndrome (XXY) 3 Turner syndrome (XO) 4 Triple X syndrome (XXX) 5 XYY syndrome (XYY)

Kleinfelter Syndrome Explanation The combination of hypogonadism, long extremities, decreased intelligence, and behavioral problems makes Klinefelter syndrome (also referred to as XXY syndrome, 47,XXY, and Klinefelter's syndrome) the most likely diagnosis. The original syndrome, as described by Dr. Klinefelter, consisted of gynecomastia, testicular atrophy, and infertility. Intelligence profiles can range from specific learning disabilities (language learning or reading impairment most common) to frank mental retardation/intelligence disability (MR/ID). The only constant feature of the syndrome is testicular atrophy with resulting azoospermia and infertility. The atrophy of the testis is the result of fibrosis, which begins to appear in childhood and progresses until all the seminiferous tubules are replaced by fibrous tissue. In males presenting with gynecomastia, MR/ID, and eunuchoidism (i.e., loss of male secondary sexual characteristics, small penis, loss of body hair, and a high-pitched voice), Klinefelter syndrome should be at the top of the list in the differential diagnosis. Most patients with Klinefelter's syndrome have 47 chromosomes instead of the normal 46 chromosome karyotype. The extra chromosome is an X chromosome, making the sex chromosome constitution XXY instead of XY. Klinefelter's syndrome is one of the most common chromosome abnormalities seen in males and occurs in 1 in 300 of the male population. Patients with this syndrome show that the Y chromosome is strongly sex-determining; thus, a patient who has an XXY chromosome constitution may have the appearance of a normal male, with infertility being the only incapacity, while the loss of a Y chromosome leads to the development of a bodily form that is essentially feminine. Fragile X syndrome is incorrect, as it is characterized by prominent jaw, large ears with soft cartilage, and macroorchidism in pubertal male patients. Turner syndrome (XO) is incorrect, as it is a genetic condition of females patients; it is usually characterized by a short stature, increased distance between the nipples, low hairline, low set ears, a webbed neck, amenorrhea, and sterility. Triple X syndrome (XXX) is incorrect. This condition only occurs in female patients. XYY syndrome (XYY) is incorrect. In this condition, IQ is normal, and there is normal sexual development as well as normal fertility.

A 15-year-old boy is seen by his primary care physician for delayed sexual changes during puberty. Upon physical exam, the physician notes an overall normal appearance, with small testes and some gynecomastia. Behaviorally, the patient seems to be somewhat shy and reserved, with occasional social outbursts. Although he is tall for his age, he is uncoordinated and inactive. Measured testosterone levels are decreased. A karyotype is done, and found to contain an extra chromosome. The most probable diagnosis for this child is which of the following? Answer Choices 1 Klinefelter syndrome 2 X-linked adrenoleukodystrophy 3 X-linked MR hypotonic facies syndrome 4 Fragile X syndrome 5 XYY syndrome

Klinefelter syndrome Explanation Fragile X syndrome results from a mutation in the FMR1 gene, located on the long arm of the X chromosome (Xq27). It is characterized by moderate mental retardation. The phenotypic manifestations of fragile X syndrome vary, but often include developmental delay, hyperactivity, abnormal craniofacials, and macro-orchidism (in post-pubertal males). Greater than 99% of affected individuals have what is known as a "full mutation" in the FMR1 gene. This mutation is caused by an increased number of CGG trinucleotide repeats in the 5' end of the gene (> 230 CGG repeats), which causes aberrant methylation of the gene, and aberrant expression of the gene product. Mothers of affected children that have this full mutation are obligate carriers of a "premutation" in the FMR1 gene. This premutation or "intermediate" allele has between 55 and 230 CGG repeats, and can expand upon transmission to offspring. (The normal allele has between 6 and 54 CGG repeats). These women, and their family members, are at an increased risk to have children affected with fragile X syndrome. Molecular genetic testing is available on a clinical basis to determine the status of the FMR1 gene allele(s). Klinefelter syndrome affects males. It is diagnosed by an abnormal karyotype. Individuals with this syndrome have an extra X chromosome, with the karyotype being 47, XXY (Variations occur, but this karyotype is the most common). The extra X chromosome seems to affect the functioning of the testes and testosterone production. Adolescent boys with this disorder may undergo gynecomastia. Most are tall, but not particularly coordinated. The penis is of normal length; however the testes are small. Treatment with male sex hormones can be helpful. X-linked adrenoleukodystrophy is caused by a mutation in the ALD gene, located on the long arm of the X chromosome (Xq28). The childhood form most commonly presents between the ages of four and eight years. It begins with symptoms of attention deficit and hyperactivity disorder, but gets progressively worse, with symptoms including difficulty with previously mastered subjects such as speech and reading. The affected individual also becomes clumsy and has visual disturbances. Brain MRI is often abnormal, even while symptoms are still mild. The rate of progression of the disorder varies, but leads to death in a matter of years. Along with the childhood form of the disease, there are several other types. Type two usually presents during middle age (possibly as early as in the twenties), and includes leg stiffness and weakness and sexual dysfunction. This, too, is progressive, usually over decades. Type three, which is found in approximately 10% of cases, is characterized by adrenal insufficiency. Presentation can be anywhere between two years of age to adulthood. X-linked mental retardation hypotonic facies syndrome is caused by a mutation in the XNP gene, also located on the X chromosome (Xq13.3). Affected individuals have a distinct phenotype that includes genital abnormalities, a common set of facial features, and severe developmental delays with mental retardation. All patients have a normal 46, XY karyotype; however their appearance at birth can range anywhere from a male with hypospadius to a normal appearing female. Common craniofacial features include a small head circumference, small triangular nose, tented upper lip, prominent lower lip and open mouth. Short stature is common. Developmentally, milestones are delayed to a marked degree. Hypotonia is commonly present. Interestingly, the mutated gene appears to down regulate expression of the alpha-globin gene, leading to a microcytic and hypochromic anemia in some affected individuals. Individuals with XYY syndrome have an abnormal karyotype - 47, XYY. Physical development is normal, although these individuals have an increased likelihood for learning problems and tend to be tall.

A 44-year-old man with a history of ulcerative colitis presents with bloody diarrhea. The patient has had a long history of poorly controlled ulcerative colitis despite being on optimal medical management. The patient subsequently is surgically managed with a proctocolectomy with permanent ileostomy. However, complications in his procedure call for a long hospital stay. After a few months, the patient is found to have ascites, pitting edema, and a fatty liver. Question What is the most likely diagnosis? Answer Choices 1 Hypovitaminosis B1 2 Kwashiorkor 3 Total parenteral nutrition-induced steatosis 4 Marasmus 5 Primary sclerosing cholangitis

Kwashiorkor This patient is having protein deficiency, and therefore the most likely answer is kwashiorkor. Kwashiorkor is secondary to total protein deficiency. This finding is typically found in patients who are severely malnourished. The typical findings in these patients are ascites and pitting edema due to hypoalbuminemia. In this patient, his long hospital stay with a complicated proctocolectomy, has led to protein deficiency. Marasmus is a total caloric and protein deficiency. These patients typically present with broomstick extremities (muscle wasting). These patients do not present with ascites and pitting edema. Total parenteral nutrition (TPN) related microvesicular steatosis is a common finding in patients who are on TPN. However, there is no mention in the question stem that this patient was on TPN. Primary sclerosing cholangitis (PSC) is commonly associated with patients who have ulcerative colitis. PSC is a chronic liver disease, which presents as inflammation, destruction, and fibrosis of both the intrahepatic and extrahepatic bile ducts, with the resultant destruction leading to cirrhosis of liver. Hypovitaminosis B1 (thiamine) is not the correct answer choice. This deficiency is known as wet beri beri. Though it may cause edema, there will be dilated cardiomyopathy, which is not found in this case.

What hormone is responsible for ovulation? Answer Choices 1 Follicle stimulating hormone 2 Luteinizing hormone 3 Estrogen 4 Testosterone 5 Insulin-like growth factor

LH Explanation Luteinizing hormone is the hormone responsible for initiation of ovulation. Follicle stimulating hormone is responsible for follicle maturation. Estrogen increases the number of luteinizing hormone (LH) receptors, which also also leads to unregulation of LH. Ultimately, a rapid rise in LH (LH surge) occurs approximately 24 - 36 hours prior to ovulation. Estrogenis produced by the conversion of testosterone to estradiol by aromatase in the granulosa cells.

A 25-year-old woman, G0P0, presents to your office with 1-year history of oligomenorrhea. Her most obvious physical exam findings are hirsutism and obesity, having a calculated BMI of 30. What would you expect to find on her workup results? Answer Choices 1 LH:FSH ratio of 2:1 or greater 2 TSH less than 0.4 mIU/mL 3 Fasting glucose less than 100 mg/dl 4 Free testosterone level less than 0.6ng/mL 5 Prolactin level greater than 40ng/mL

LH:FSH ratio of 2:1 or greater Explanation The correct answer is LH:FSH ratio of 2:1 or greater as this is the most positive indicator of polycystic ovarian syndrome (1). TSH less than 0.4 mIU/mL would indicate the possibility of hyperthyroidism and therefore is incorrect. The only symptom this patient has that would correlate with hyperthyroidism is the oligomenorrhea. A fasting glucose less than 100 mg/dl would indicate insulin sensitivity and a patient suspected of having PCOS usually demonstrates insulin resistance. A free testosterone level less than 0.6ng/mL is incorrect as 25-50% of patients with PCOS have elevations in free testosterone (1). A prolactin level greater than 40ng/mL indicates hyperprolactinemia, which usually is not a finding in PCOS and is indicative of a separate disorder (1).

A 58-year-old man presents with a 1-day history of severe abdominal pain, nausea, and vomiting. He initially thought he had some indigestion, with pain located in the epigastric region, and tried some calcium carbonate (Tums) with no relief. The pain and vomiting progressed through the night and kept him from sleeping and going to work. He feels the pain boring through to his back. He denies hematemesis, fever, diarrhea, out-of-the-country travel, and contact with sick people. Prior to onset of pain, he reports good health. He has no known medical conditions and takes no medications. He has had no surgeries. He smokes cigarettes (1ppd x 40 years), admits "moderate" alcohol use, and denies drug use. He is married and works as a welder. Vitals are: BP: 102/56 mmHg; HR: 116bpm; RR: 15; Temp: 98.9F; O2Sat: 95% on room air. On physical exam, the patient appears uncomfortable on the exam table and grimaces when changing position for exam. He is cooperative, alert, and oriented. Abnormal physical exam findings include: abdomen distended, decreased bowel sounds, and tender epigastric region, with guarding. He is tachycardic. No jaundice is noted. The remainder of the exam is normal. This patient's test results are shown in the table. Alk Phosphatase150 (50-136) Amylase 272 (20-110) Lipase 290 (0-160) Question What pharmacologic treatment is the most important intervention for this patient's likely condition? Answer Choices 1 Ertapenem 2 Hyoscyamine 3 Lactated Ringer's 4 Pancrelipase 5 Promethazine

LR

A 58-year-old man presents with a 1-day history of severe abdominal pain, nausea, and vomiting. He initially thought he had some indigestion, with pain located in the epigastric region, and tried some calcium carbonate (Tums) with no relief. The pain and vomiting progressed through the night and kept him from sleeping and going to work. He feels the pain boring through to his back. He denies hematemesis, fever, diarrhea, out-of-the-country travel, and contact with sick people. Prior to onset of pain, he reports good health. He has no known medical conditions and takes no medications. He has had no surgeries. He smokes cigarettes (1ppd x 40 years), admits "moderate" alcohol use, and denies drug use. He is married and works as a welder. Vitals are: BP: 102/56 mmHg; HR: 116bpm; RR: 15; Temp: 98.9F; O2Sat: 95% on room air. On physical exam, the patient appears uncomfortable on the exam table and grimaces when changing position for exam. He is cooperative, alert, and oriented. Abnormal physical exam findings include: abdomen distended, decreased bowel sounds, and tender epigastric region, with guarding. He is tachycardic. No jaundice is noted. The remainder of the exam is normal. This patient's test results are shown in the table. Alk Phosphatase150 (50-136) Amylase 272 (20-110) Lipase 290 (0-160) Question What pharmacologic treatment is the most important intervention for this patient's likely condition? Answer Choices 1 Ertapenem 2 Hyoscyamine 3 Lactated Ringer's 4 Pancrelipase 5 Promethazine

LR This patient is presenting with an episode of acute pancreatitis. Pancreatitis is characterized by epigastric pain, nausea, and vomiting. Many other acute abdomen conditions may present similarly. Significant elevations of lipase and amylase, as well as CT evidence of pancreatic inflammation, fluid collections, and/or necrosis, will establish the diagnosis. Pancreatitis can be acute or chronic, and those with chronic disease may have periodic acute flares. In acute pancreatitis, early fluid resuscitation is one of the "few medical interventions that appears to affect outcome," so (of the choices listed) administering lactated Ringer's (or saline) would be the best answer for this patient. Also, ensuring the patient has "nothing by mouth" (NPO) until symptoms decrease is standard treatment. Ertapenem is a carbapenem antibiotic, indicated for complicated intra-abdominal, skin, and urinary tract infections. Although this patient's white blood cell count (WBC) is elevated, the primary process is not infectious. The WBC increases in inflammatory states. Rarely are antibiotics needed in acute pancreatitis. Hyoscyamine is an anticholinergic medication, with a wide range of uses, including many gastrointestinal conditions. It may be used for reducing spasm in irritable bowel syndrome and for biliary colic. Before the test results pointed to pancreatitis, as in this patient's diagnosis, biliary colic and obstruction would be on the differential. However, hyoscyamine would not be a primary treatment for pancreatitis. Pancrelipase is a digestive enzyme, taken with meals, and indicated for pancreatic insufficiency, a common complication of many years of chronic pancreatitis. This patient may be at risk for future pancreatitic disease, but the pancrelipase would not benefit him at this time. Promethazine is a common antiemetic with antihistamine and anticholinergic properties. Although it may be reasonable to consider antiemetic treatment for a patient with acute pancreatitis, it is an adjunct treatment and does not have a vital impact on the course of the disorder.

A 35-year-old Asian man presents with diarrhea; the diarrhea has worsened over the past year since getting married and changing his diet. It is associated with abdominal discomfort and the feeling of a lot of gas in his stomach. On examination, his abdomen is distended; there is a tympanic note on percussion. There is no area of tenderness. The hydrogen breath test reveals elevated hydrogen in his expired air. Question What is the most likely diagnosis? Answer Choices 1 Celiac disease 2 Whipple's disease 3 Short bowel syndrome 4 Tropical sprue 5 Lactose intolerance

Lactose intolerance Lactose intolerance results from a deficiency of lactose, a disaccharidase in the mucosal cells of the small intestine which splits the disaccharide lactose into glucose and galactose. Patients note borborygmi, flatulence, nausea, abdominal cramps, pain, and diarrhea after ingesting lactose-containing food (e.g., milk). Laboratory investigations reveal that the diarrheal stools are acidic. When the hydrogen breath test is administered, there is elevated hydrogen content in the expired air because the colonic flora digests the unabsorbed lactose. Treatment includes following a lactose-free diet. Celiac disease is a hereditary disorder caused by gluten intolerance; gluten is found in wheat, rye, barley, and oats. Patients may be asymptomatic or may present with diarrhea, abdominal discomfort, distention, and steatorrhea; their stools are pale, malodorous, and difficult to flush because they float on the toilet water. Patients may have anemia from iron and folate deficiency, osteomalacia and bone pains from calcium deficiency, and edema from hypoproteinemia. Diagnosis is confirmed by a small intestine mucosal biopsy which shows a flat mucosa due to villous atrophy, and by subsequent improvement on a gluten-free diet. Specific treatment includes a gluten-free diet. Whipple's disease mainly affects men; it is caused by the bacterium Tropheryma whippelii. It is a multisystemic disease which affects the small intestines, joints, brain, heart, and eyes. Patients can present with diarrhea, steatorrhea, abdominal pain, weight loss, and joint pains. On examination, they may be pale and have lymphadenopathy. Histological examination of a small bowel mucosal biopsy reveals PAS-positive foamy macrophages. Treatment is with trimethoprim-sulfamethoxazole or chloramphenicol. Short bowel syndrome is usually the result of surgical resection of the intestines or a jejunoileal bypass. The malabsorption is a result of inadequate absorptive surface. Malabsorption of vitamin B12 results in paresthesias; malabsorption of calcium results in bone pain and carpopedal spasms. Tropical sprue is an acquired disease that affects both visitors and natives of tropical areas (e.g., the Caribbean and South India). Its etiology is unknown. Patients usually present with diarrhea and weight loss; they report that their stools are soft and bulky (steatorrhea). They may also develop deficiencies of folate and cobalamin. Stool microscopy should be done to look for cysts and trophozoites. Histological examination of a small bowel mucosal biopsy aids in making the diagnosis. Treatment is with tetracycline or oxytetracycline.

A 30-year-old woman comes in for evaluation of infertility. She and her husband have been having unprotected intercourse for the past year. Her menstrual cramps have become increasingly painful, and she has a severe low backache for several days before and during her menses. She complains that intercourse is painful when her husband "goes deep." Physical examination reveals multiple tender nodules of various sizes in the posterior vaginal fornix. Definitive diagnosis is best accomplished using what methodology? A Abdominal radiography B CA-125 measurement C CT scanning of the abdomen D Laparoscopy E Pelvic ultrasonography

Laparoscopy D This woman has a classic presentation for endometriosis. Definitive or final diagnosis can only be made at laparoscopy or laparotomy, allowing direct visualization of the endometrial implants. Abdominal radiography (A), computed tomography scanning (C), and ultrasonography (E) are not usually helpful in making a definitive diagnosis. CA-125 (B) may be elevated in endometriosis, but lacks specificity.

A 33-year-old gravida 3, para 1011, whose last menstrual period (LMP) was 6 weeks ago, presents with a 3-day history of vaginal bleeding as well as intermittent, non-radiating right lower quadrant pain. She states that her pain scale is a 7/10 and that the pain is worse today than it has been previously. Her past medical history is significant for gonorrhea at age 16; she was treated with antibiotics. Vital signs are: BP, 80/42 mm Hg; pulse is 120 BPM, and respirations are 22/min. The patient is afebrile, and oxygen saturation is 95% on room air. The abdomen is distended, and bowel sounds are absent. There is rebound tenderness and guarding. Pelvic exam demonstrates a mass in the right adnexa. The uterus is approximately 4 - 6 weeks size. A serum beta human chorionic gonadotrophin (HCG) level is 3,723 mIU/ml. Hematocrit is 24%. Ultrasound demonstrates free fluid in the pelvis, and there is a 6 cm complex mass in the right adnexa. There is no evidence of an intrauterine gestation. Question What is the most appropriate treatment for the patient at this time? Answer Choices 1 Laparoscopic salpingostomy 2 Methotrexate, 75 mg IM x 1 dose 3 Serial quantitative beta HCG levels every 48 hours 4 Laparotomy 5 Laparoscopic right salpingo-oophorectomy

Laparotomy Explanation Approximately 2% of all pregnancies are ectopic pregnancies, and most are located in the Fallopian tube. The triad of amenorrhea, vaginal spotting, and unilateral adnexal pain is strongly suggestive of an ectopic pregnancy. This patient is hemodynamically unstable. Such patients should undergo exploratory laparotomy as soon as is possible after the presumptive diagnosis has been made. Laparoscopic surgery of any kind is unacceptable in the face of a hemodynamically unstable patient with an ectopic pregnancy. Methotrexate is an acceptable alternative to surgical management of an ectopic pregnancy, but is contraindicated when an ectopic pregnancy is ruptured, or when the adnexal mass is >3.5 cm on ultrasound. Serial quantitative beta HCG levels may be obtained every 48 hours in order to confirm the diagnosis of ectopic pregnancy; however, in this case, the patient's unstable status makes this treatment plan dangerous and unnecessary.

A 22-year-old woman presents with a history of chronic diarrhea. She gives a history of large volumes of watery fecal output that is non-bloody and can be easily flushed; the diarrhea is painless and persists with fasting. The patient's mother complains that she eats less and has lost weight. She has no history of fever, flushing, wheezing, intolerance to heat/cold, or infection. Her family history and menstrual history are insignificant. On examination, her weight is 130lb; height is 5 feet 5 inches; pulse is 80/min; BP is 100/74mmHg; and temperature is 98.8°F. The tongue is dry, but the rest of her physical examination is within normal limits. Question What is the most likely diagnosis? Answer Choices 1 Carcinoid tumor 2 Laxative abuse 3 Celiac disease 4 Hyperthyroidism 5 Inflammatory diarrhea

Laxative abuse The most likely diagnosis is laxative abuse. Large volumes of watery fecal output that is non-bloody, painless, and persists with fasting suggests secretory diarrhea that can result from use of stimulant laxatives like senna, bisacodyl, or castor oil1. The patient is young, dehydrated, and based on her symptoms, is likely taking laxatives to lose weight and get rid of unwanted calories. There are no signs and symptoms of other causes of secretory diarrhea, such as carcinoid syndrome, and the rest of the options do not have conditions that cause secretory diarrhea. Many patients might not admit the use of laxatives to lose weight. Carcinoid tumor can cause secretory diarrhea1, but it is unlikely in this patient; she has no associated symptoms, such as episodic wheezing or flushing. Hyperthyroidism results in diarrhea due to dysmotility1. However, the patient does not have any symptoms of hyperthyroidism, such as unexplained weight loss or heat intolerance. Inflammatory diarrhea can be ruled out in this patient; there is no fever or signs of inflammation or bloody stools. Celiac disease causes diarrhea due to mucosal malabsorption. Patients present with multiple nutritional deficiencies and fatty diarrhea, which are not present in this patient, making this diagnosis unlikely

A 64-year-old man presents with difficulty speaking, trouble swallowing, and right arm weakness. Physical examination reveals dysarthria, tongue deviation to the right, paralysis of the left soft palate, left facial droop, left facial sensory loss, and right arm weakness. These findings suggest involvement in what area? Answer Choices 1 Medulla 2 Left brain stem 3 Left cerebral hemisphere 4 Pons 5 Right midbrain

Left Brain Stem Although precise localization of neurologic deficits is often difficult, certain patterns are important to recognize. Brain stem involvement results in 'crossed findings' (e.g., right facial weakness and left arm weakness) because the lesions affect the brain stem nuclei directly (uncrossed), and the corticospinal tract as it is crossing to the opposite side of the body. Because of the unique anatomic arrangement of the brainstem, a unilateral lesion within the structure can cause "crossed findings" in which ipsilateral dysfunction of 1 or more cranial nerves is associated with hemiplegia and hemisensory loss on the contralateral side of the body, as described in this case. Choice B is the only option mentioning the brain stem; therefore, it is the correct choice. Medullary syndromes, such as lateral medullary syndrome, are associated with vertigo, nausea, vomiting, nystagmus as well as hiccups, and diplopia; they were not observed in this patient. Medial medullary syndrome is associated with tongue deviation toward the lesion, which is opposite to what is described in this case. Cerebral (cortical) lesions produce contralateral motor and sensory findings in the limbs and contralateral cranial nerve deficits (e.g., right arm and right facial paralysis). Pontine lesions result in coma, miosis, gaze paresis, and altered respiratory patterns. Cerebellar lesions produce nystagmus, dizziness, nausea and vomiting, and the inability to stand or walk if midline cerebellar areas are involved.

A 55-year-old woman presents with a 2-month history of gross hematuria. She states she has no pain with urination, but the hematuria is persistent. On questioning, she states that she has had a 1-month history of some progressively worsening left flank pain and persistent back pain. The pains are not debilitating, but they are nagging. She has no chronic medical problems. She admits to a 50-pack/year smoking history, and she states she is currently retired from her job as a teacher. Vital signs are within normal limits, and physical exam reveals a left side abdominal mass. Urine dipstick only shows too numerous to count RBCs, and urine cultures are negative. CT scan of the abdomen and pelvis with and without contrast reveals a 4.2 cm solid enhancing lesion in the left renal parenchyma. Question What is the most effective treatment for this patient? Answer Choices 1 Radiation of the kidney 2 Chemotherapy 3 Left nephrectomy 4 Hormonal therapy 5 Observation

Left Nephrectomy Explanation This patient likely has renal cell carcinoma of the left kidney. Surgery is most effective treatment for primary RCC. Partial nephrectomies are common in peripheral tumors that are less than 4 cm in size, but total nephrectomies are the treatment of choice in larger or more centrally located tumors. Radiation preoperatively and/or postoperatively has not been shown to change overall survival with RCC. Chemotherapy currently available is not beneficial in RCC. Hormone therapy, including progesterone therapy, has not been shown to be effective in treatment of RCC. Observation is a viable option in elderly patients and poor surgical candidates if the tumor is less than 3 cm in size. These patients are often followed with CT scans every 6 months.

A 74-year-old woman presents for management of an ischemic stroke. She reports difficulty seeing objects on her right side. You perform confrontational visual field testing as part of your neurological examination, and you discover she has a right inferior homonymous quadrantanopsia. This lesion localizes to what part of the body? Answer Choices 1 Left optic tract 2 Optic chiasm 3 Left optic nerve 4 Left parietal lobe 5 Right optic tract

Left parietal lobe Quadrantanopsia (visual loss in a quadrant of the visual field) may result from several causes, of which stroke is the most common. Accurate visual field testing can often localize the region of ischemia. An inferior homonymous quadrantanopsia localizes to the contralateral parietal lobe where the superior optic radiation fibers run. Other symptoms such as hemiplegia or hemisensory loss may also be seen. An optic tract lesion would cause a contralateral homonymous hemianopsia. An optic chiasmal lesion would cause a temporal non-homonymous hemianopsia. A left optic nerve lesion would cause unilateral, not bilateral visual field loss.

A 29-year-old woman has Wilson's disease, for which she is taking D-penicillamine. Today, her laboratory findings (including liver function tests) are within normal limits; there are no neurological signs. Her work requires frequent (3 - 5 days per week) travel, including international overnight travel all over the world. She wants contraception that is both effective and convenient in terms of application. Question What is your advice? Answer Choices 1 Combined oral contraceptive pill 2 Progestin-only pill 3 Nonhormonal copper intrauterine device 4 Levonorgestrel-releasing Intrauterine system 5 Contraceptive patch

Levonorgestrel-releasing Intrauterine system Explanation Only spermicide, barrier contraceptives, and progesterone-only preparations can be considered in Wilson's disease. Levonorgestrel is a 2nd-generation progestin (synthetic progestogen). A levonorgestrel-releasing intrauterine system causes thickening of cervical mucus, thereby reducing sperm motility and penetration; over time it decreases proliferation of the endometrium. The local inflammatory response could be toxic to sperm, thus providing an additional contraceptive effect. Combination oral contraceptive pills contain both estrogen and progestins. Estrogen-containing contraceptives are not recommended since estrogen may interfere with copper excretion. The progestogen-only pill is efficient; however, it must be taken at or around the same time every day (within 3 - 12 hours). Therefore, the pill may be inconvenient for a woman with a job that requires international travel all over the world. Nonhormonal copper intrauterine devices should be avoided in a disease that has the main characteristic of copper accumulation in the body. Copper IUDs primarily work by disrupting sperm mobility and damaging sperm. Contraceptive patches also contain estrogen; therefore, they should be avoided in Wilson's disease.

An 8-year-old boy presents with acute abdominal pain. On examination, there is mild hepatosplenomegaly. Biochemical investigations reveal elevated serum amylase and lipase levels. The plasma sample of the patient shows a thick creamy layer when kept overnight at 4 C. Fasting serum triglycerides level are 2200mg/dl. There have been similar complaints of abdominal pain in the past 4-5 years. The Apo CII levels were normal. What is the most likely diagnosis? Answer Choices 1 Abetalipoproteinemia 2 Apo CII deficiency 3 Familial combined hyperlipidemia 4 Familial hypercholesterolemia 5 Familial type III hyperlipoproteinemia 6 Lipoprotein lipase deficiency

Lipoprotein lipase deficiency Lipoprotein lipase deficiency is characterized by a defect in the degradation of chylomicrons. Lipoprotein lipase is required for the catabolism of triacylglycerol present in chylomicron to free fatty acids and glycerol. Lipoprotein lipase is present in the endothelium of blood vessels of the adipose tissue and muscle. A high serum triglyceride level characterizes this disorder. The serum triglyceride level is usually >1500mg/dl. The serum cholesterol is marginally raised. Lipoprotein lipase may be released from the tissues by administration of intravenous heparin. Diagnosis is confirmed by measuring lipoprotein lipase using specific immunologic techniques. Serum triglycerides levels are markedly elevated (>1500mg/dl). The most frequent presentation is as recurrent abdominal pain accompanied by hepatosplenomegaly. In many cases, patients present with acute pancreatitis. Observation of plasma, after keeping it overnight at 4 C, shows the presence of a creamy layer on the plasma. Management involves the administration of medium-chain triglycerides in the diet that enter the portal circulation directly without formation of chylomicrons. Familial hypercholesterolemia (FH) is an autosomal dominant genetic defect in which there is deficiency of functional LDL receptors. These LDL receptors recognize the Apo B100 present in IDL and LDL. LDL/IDL binds to the receptor and is taken up into the cell by receptor-mediated endocytosis. In FH, there is decreased peripheral uptake of LDL and IDL, which causes a marked increase in the serum cholesterol level. Serum triglycerides are within normal limits. There is a large accumulation of cholesterol in the macrophages of the tendons that causes xanthomas, indicating that the scavenger cells are taking up and degrading large amounts of LDL. Patients manifest with an increased risk of atherosclerosis and coronary artery disease at an early age. There is cholesterol accumulation in the arteries, especially at the root of the aorta, causing aortic stenosis. Apo C II is the apolipoprotein present in chylomicrons that is required for the activation of lipoprotein lipase. Absence of Apo CII is inherited as a recessive disorder. It is characterized by high levels of chylomicrons and serum triglycerides. It is distinguished from lipoprotein lipase deficiency by the assay of lipoprotein lipase after the administration of intravenous heparin. In this disorder, lipoprotein lipase levels are normal; however, Apo CII levels are low. Symptoms of presentation and management of the condition are similar to lipoprotein lipase deficiency. Familial combined hyperlipidemia (Frederick son's type IIB hyperlipoproteinemia) is characterized by an increase in LDL and VLDL. In these patients, there is an increased fasting serum cholesterol and serum triglycerides level. There is an increased risk of coronary artery disease in these patients. Serum cholesterol level is usually in the range of 200-300 mg/dl and serum triglyceride is in the range of 200-500mg/dl. Abetalipoproteinemia is characterized by an absence of lipoprotein species containing apoprotein B, which are LDL, chylomicrons, and VLDL. The levels of triglycerides and total cholesterol are very low (<50mg/dl). The predominant features include malabsorption of dietary fat and steatorrhea. There is also malabsorption of the fat-soluble vitamins. Lack of vitamin E causes progressive degeneration of the CNS, which manifests as neurological abnormalities by the first decade. There is decreased visual acuity and night blindness as a result of vitamin A deficiency. Familial type III hyperlipoproteinemia (dysbetalipoproteinemia) is characterized by increased levels of IDL in circulation. There is also an increase in the chylomicron remnant particles. There is a defect in apolipoprotein E, which is required for the uptake of chylomicron remnants and IDL by the liver. In these individuals, the fasting serum cholesterol and triglycerides are raised. Patients present with an increased risk of premature atherosclerosis and palmar xanthomas.

A 34-year-old diabetic female complains of amenorrhea for the past 2 months. Last week she tested positive for strep pharyngitis and is currently being treated. She admits that she has not been using birth control. Her urine HCG is positive. Her current medications are listed in the choices below. Which of the following medications should you discontinue? A amoxicillin B lisinopril C acetaminophen D humalog insulin E methyldopa

Lisinopril B The correct answer is (B). Lisinopril, an ACE inhibitor, is contraindicated in pregnancy due to known problems with fetal toxicity and should be stopped as soon as possible once pregnancy is confirmed. If a patient is planning on becoming pregnant the ACE inhibitor should also be discontinued. ARBs should also be avoided. Choices (A), (C), and (D), and (E) can be used safely in pregnancy and are considered category B. Methyldopa is preferred in the treatment of hypertension in pregnancy and its safety is supported by evidence.

What exposure is linked to Listerosis

Listeriosis has been linked to exposures to contaminated food, particularly dairy products and hot dogs.

A 25-year-old man presents with increased urination and thirst. Over the past 3 days, he has been unable to satisfy his thirst and has to urinate up to 20 times per day. He noticed that his urine is very clear and colorless. In general, he feels very weak. He has never had any problems with urination before. His medical history is remarkable for a recently diagnosed psychiatric condition for which he began medical treatment. On physical exam, he appears to be lethargic, dehydrated, and pale. His vital signs are as follows: blood pressure 96/52 mm of Hg, temperature 101.2 degrees fahrenheit, pulse 108 beats per minute, and a respiratory rate of 26 per minute. Question What is a common cause of diabetes insipidus? Answer Choices 1 Lithium 2 Carbamazepine 3 Amitriptyline 4 Valproic acid 5 Vasopressin

Lithium Explanation Lithium, used in the treatment of bipolar disorder, is a common cause of diabetes insipidus. It is freely filtered through the glomerulus and reabsorbed in the proximal tubule along with sodium and water. Even small doses of lithium may cause diabetes insipidus. Lithium can also make the distal renal tubules resistant to the action of vasopressin. This patient had recently started this medication, which resulted in his new symptoms. Carbamazepine, amitriptyline, and valproic acid do not cause diabetes insipidus. Analogues of vasopressin are used to treat diabetes insipidus. Diabetes Insipidus (DI) exists in 1 of 2 forms: central and nephrogenic. In central diabetes insipidus, there is a decrease in the secretion of antidiuretic hormone (ADH). This results in polyuria and polydipsia and the patient has a diminished ability to concentrate urine. Decreased or deficient ADH can be caused by a defect in the hypothalamic osmoreceptors, supraoptic or paraventricular nuclei, or the supraoptico-hypophyseal tract. Nephrogenic DI is defined as a decrease in the ability to concentrate urine due to a resistance to ADH on the kidney. It can be seen in chronic renal insufficiency, lithium toxicity, hypercalcemia, hypokalemia, and tubulointerstitial disease. There is a hereditary form of nephrogenic DI that is rare and is transmitted as an X-linked genetic defect of the V2 receptor gene. ADH's main target is the kidney. It causes an alteration of permeability to water of the collecting tubules in the kidney's medulla and cortex. Water is reabsorbed by osmotic equilibration and is put back into the bloodstream. The actions of ADH are mediated through at least 2 receptors. The first is known as V1. V1 mediates vasoconstriction and enhances the release of corticotrophin. It also takes part in the synthesis of renal prostaglandins. The second one, known as V2, is a mediator of the antidiuretic response. The diagnosis of DI is usually made clinically. If a patient is able to match fluid loss with adequate fluid intake, their condition may appear to be otherwise normal. If they cannot keep up with fluid demand, they will begin to show signs of dehydration (sunken eyes, dry skin and mucous membranes, fever, tachycardia, unintentional weight loss). Other symptoms include headache, fatigue, muscle aches and irritability. A urine specific gravity of 1.005 or less and a urine osmolality less than 200 mOsm/kg is the hallmark of DI. Random plasma osmolality generally is greater than 287mOsm/kg. It is important to rule out other causes such as diabetes mellitus. One way to do this is to use the water deprivation test. A patient is monitored, and his/her water intake is withheld. The patient's weight and urine osmolality are measured on an hourly basis. If two urine osmolality readings are less than or equal to 30 mOsm on two consecutive readings, or if the person's weight decreases by 3% or more, then 5 units of vasopressin is given and another urine sample is checked after an hour. A normal reading will show a urine osmolality that is 2 to 4 times greater than the plasma osmolality. In patients with central DI, urine osmolality can increase by 50% or more. ADH levels will be extremely low in central DI. If a patient has nephrogenic DI, their response to the test will be an ADH level that is either normal or increased. There will be no response to the ADH by the kidney. The disease is very uncommon in the U.S., and men and women are affected approximately equally. The most common causes are head trauma and surgery. There is a genetic form of DI that is rare and is inherited in an autosomal dominant pattern. It is caused by a mutation in the AVP-neurophysin gene. Medications used to treat DI include vasopressin analogues in either oral, nasal, or subcutaneous forms. Other medications used include indomethacin, carbamazepine, thiazide diuretics, and chlorpropamide.

An 8-month-old infant presents with shortness of breath, wheezing, intercostal retractions, and respiratory rate of 50. What would further suggest a diagnosis of bronchiolitis? Answer Choices 1 Wheezing improved after nebulizer 2 Peak flow 300 prior to treatment 3 Liver and spleen palpable 4 Clubbing of the finger

Liver and spleen palpable Enlargement of the liver and spleen occurs due to hyperinflation of the lungs with bronchiolitis. With this diagnosis, a nebulizer that improves wheezing is not a specific criterion, but it is more so with asthma. A peak flow of 300 is good; therefore, it would likely be lower with bronchiolitis. Clubbing of the fingers is seen with prolonged COPD in adults, not in the pediatric population.

A 51-year-old woman who has been in a road traffic accident presents with multiple injuries. She has a urine output of 350 mL over the previous 24 hours. Her serum creatinine level has increased from 4.2 mg/dL to 4.8 mg/dL over the same period. What drug can be used to convert this patient's oliguric renal failure to the non-oliguric type, and to facilitate further management? Answer Choices 1 Potassium sparing diuretics 2 Thiazide diuretics 3 Carbonic anhydrase inhibitors 4 Loop diuretics 5 Osmotic diuretics

Loop diuretics Loop diuretic drugs produce diuresis in patients with an acute renal failure (ARF), congestive heart failure, and acute pulmonary edema. Loop agents can increase the rate of urine flow and enhance potassium excretion in cases of acute renal failure. They are used to convert oliguric renal failure to non-oliguric renal failure, and to facilitate fluid and electrolyte management. They do not, however, shorten the duration of renal failure or reduce mortality. Potassium sparing diuretic drugs are mild efficacy drugs with a relatively weak diuretic action; they are often used in combination with loop diuretics or thiazides in patients with congestive heart failure. Thiazide diuretics are used to treat hypertension, congestive cardiac failure, and edema. Carbonic anhydrase inhibitors produce a mild diuretic effect; they are used to treat glaucoma. Osmotic diuretics may be used to reduce intracranial pressure in cerebral edema.

A 19-year-old woman presents to the emergency department complaining of headache. The headaches are generalized and increasing in intensity. They have not responded to over-the-counter (OTC) medications. She complains of approximately 1 week of blurred vision, intermittent diplopia, and vague dizziness. Her medical history includes obesity and acne. She takes Accutane and oral contraceptives. She is found to have bilateral papilledema, visual acuity of 20/30 on physical examination, and a normal MRI of the brain. The next most appropriate step would be

Lumbar Puncture The presence of headache associated with papilledema raises the concern for a brain tumor. The MRI excluded a mass lesion, raising a strong suspicion of pseudotumor cerebri. This is also known as benign intracranial hypertension. It is not a benign condition, however, since it causes severe headache and may result in visual loss. It is particularly frequent in obese adolescent girls and young women. The etiology is unknown but may be associated with the use of oral contraceptives, vitamin A, and tetracycline. The presentation consists of headaches caused by an increase in intracranial pressure and blurring of vision. There may be diplopia, but the remainder of the neurologic examination is unremarkable. Papilledema is virtually always part of the presentation. The mental status is normal. The differential diagnosis includes venous sinus thrombosis, sarcoidosis, and tuberculosis or carcinomatous meningitis. The last two are excluded by lumbar puncture. An abnormal cerebrospinal fluid is not consistent with pseudotumor cerebri. The diagnosis is made by excluding mass lesions with CT scan or MRI and demonstrating markedly increased intracranial pressure by lumbar puncture. The treatment involves weight loss, diuretics, and steroids. Repeat lumbar punctures to remove cerebrospinal fluid and decrease intracranial pressure are effective. In cases that are unresponsive to these measures, lumbar-peritoneal shunting is effective, as is unilateral optic nerve sheath fenestration. Effective treatment can improve headaches and prevent vision loss.

Which of the following is the most prevalent fatal cancer in the United States? A Cervical B Colon C Esophageal D Liver E Lung

Lung Lung cancer continues to lead as the major cause of cancer deaths in both men and women in the United States, with cigarette smoking causing greater than 90% of cases. Despite educational campaigns highlighting the risks of smoking, lung cancer continues to kill more individuals that colorectal, breast, and prostate cancers combined.

A 68-year-old man presents to your pulmonology practice for long-standing dyspnea and non-productive cough. The patient has had the cough and progressively worsening dyspnea for about 1.5 years. He denies other symptoms, and he is not a smoker. He denies fevers, chills, and night sweats. He denies unusual travel, hobbies, and occupational exposures. The rest of his review of systems is negative. His past medical history is unremarkable. He had a workup through his family practice, and he was then sent to a cardiologist, who ruled out cardiovascular causes of his dyspnea. He was given trials of various antibiotics, inhalers, and steroids; there was no improvement in symptoms, despite good compliance. Several screening blood tests (including blood count, metabolic panel, HIV, and autoimmune markers) are done, revealing no abnormalities. A chest X-ray shows a few reticular opacities. A high-resolution computed tomography (HSCT) scan shows some scattered areas of reticular opacities and honeycombing. Your office runs a pulmonary function test (PFT), which shows some restrictive impairment and reduced perfusion of carbon monoxide. Physical exam is significant for fine inspiratory bibasilar crackles and clubbing in the fingers. An occasional dry cough is noted. Question What intervention is recommended for this patient's condition? Answer Choices 1 Avoidance of exertion 2 Daily inhaled steroids 3 Long-term oral steroid use 4 Lung transplant 5 Ventilation-perfusion scan

Lung Transplant This patient is presenting with idiopathic pulmonary fibrosis (IPF), and the only intervention currently associated with improvements in mortality is lung transplant. IPF is a chronic, progressive restrictive pulmonary disease of the lung parenchyma. IPF presents with exertional dyspnea and non-productive cough; it also presents with the imaging characteristics described in this patient. Avoidance of exertion may prevent the exertional dyspnea in this patient, but it is not recommended. Respiratory therapy to improve exercise tolerance is recommended in cases of IPF. Daily inhaled steroids and long-term oral steroid use are not proven as effective in IPF. The pathophysiologic basis for IPF is much more a disorder of fibroblastic disease, in which there is an abnormal accumulation of extracellular matrix, and not inflammatory in nature. Anti-inflammatory medications do not alter the course of IPF. A ventilation-perfusion scan is a test that is primarily used in the diagnosis of pulmonary embolism (PE). This patient is not presenting with a history consistent with a PE (which may be acute dyspnea and chest pain). The imaging that has already been done (high-resolution CT) is sufficient for diagnosis of IPF. No further studies are recommended. If the provider needs further confirmation, a lung biopsy should be the confirmatory test of choice.

MCC of CKD

MC cause of CKD is DM!!! HTN is 2nd leading cause!!!!

A new patient to the practice reports that his paternal grandfather had a tumor of the pituitary gland, his father had hyperparathyroidism, one uncle had a pancreatic tumor and another a thyroid cancer. His first cousin has "some kind of facial tumors" and another relative had treatment for "producing too much stomach acid." Given this family history, he should be screened for which of the following? A Carney complex B Cowden disease C McCune-Albright syndrome D MEN1 E Sipple syndrome

MEN1 This family history strongly suggests Multiple Endocrine Neoplasia 1, which are inherited in an autosomal dominant manner and involve mutations on the long arm of chromosome 11 (11a13). Carney complex (A) is another syndrome of multiple endocrine neoplasia (MEN) with tumors of the adrenal cortex, pituitary gland, thyroid, and gonads and with hyperpigmentation and cardiac myxomas. Cowden disease (B) is also a syndrome of MEN including thyroid abnormalities, breast cancer and hamartomas. Another MEN, McCune-Albright syndrome (C) is associated with precocious puberty, Cushing syndrome, hyperthyroidism, and acromegaly. Sipple syndrome (E), also known as MEN2a. may develop medullary thyroid carcinoma, pheochromocytomas, or Hirschsprung disease.

A 32-year-old man presents with muscle weakness, lack of coordination, and tingling in his fingertips; he states that he has not been able to control his urinary function for the past few weeks. He also states that he has double vision sometimes. What is the most likely diagnosis? Answer Choices 1 Parkinson's disease 2 Multiple sclerosis 3 Multiple myeloma 4 Transient ischemic attack 5 Stroke

MS The clinical picture is suggestive of multiple sclerosis. The common clinical presentation is weakness, numbness, tingling, or unsteadiness in a limb, spastic paraparesis, retrobulbar neuritis, diplopia, disequilibrium, or a sphincter disturbance such as urinary urgency or hesitancy. Common findings in Parkinson's disease are tremor, rigidity, bradykinesia, and postural instability. Parkinson's disease is commonly seen in older patients. Multiple myeloma is a malignancy of plasma cells characterized by replacement of bone marrow, bone destruction, and paraprotein formation with a median age-of-onset of 65 years old. Most patients complain of lower back pain. Transient ischemic attacks (TIA) are characterized by focal ischemic cerebral neurological deficits that last for less than 24 hours. Symptoms vary from patient to patient, but they include abrupt or sudden onset; recovery usually occurs within minutes. Some common symptoms include weakness and heaviness in the contralateral arm, leg, or face. Strokes are similar to TIA, but symptoms last longer than 24 hours and most patients have a history of hypertension, diabetes mellitus, valvular heart disease, or atherosclerosis.

A 24-year-old male with thalassemia major who has received adequate transfusions, chelation therapy, and regular health checkups is in to establish care. He is 5'4" with a BMI of 17. He eats a balanced healthy diet and gets regular exercise. You know that there are complications of this disease. What is the best next step for this patient?

Maintained sustained reduction in body iron. Maintaining sustained reductions in body iron has demonstrated increased overall survival rates through reductions in cardiac disease specifically due to siderosis. While these patients are at increased risk for osteoporosis and cardiac siderosis, the next best step in this patient is to maintain reduced iron levels. There is no place for increased blood transfusion or obtaining regular testosterone levels.

A 69-year-old man presents with a 7 - 10 day history of increasing dyspnea and inspiratory chest pain. He has a 2-pack-per-day smoking history and abuses alcohol. He has hypertension, diabetes, coronary artery disease, and chronic kidney disease. Upon further questioning, you discover that he was discharged from the hospital 2.5 weeks ago; he was diagnosed with congestive heart failure after presenting with similar symptoms. The patient's breathing appears labored, and you note diminished breath sounds on auscultation and dullness to percussion in the lower half of the lung fields bilaterally. A chest X-ray reveals bilateral moderate pleural effusions. Thoracentesis reveals pleural fluid with the following characteristics: (1) turbid in appearance; (2) 2500 white blood cells/microliter; (3) glucose equal to serum levels; (4) ratio of pleural fluid protein to serum protein of 0.75; and (5) ratio of pleural fluid LDH to serum LDH of 0.72. Question Based on the information above, what is the most likely etiology of the pleural effusions? Answer Choices 1 Congestive Heart Failure 2 Nephrotic Syndrome 3 Cirrhosis 4 Malignancy 5 Bacterial pneumonia

Malignancy is the correct response. A pleural effusion is an abnormal accumulation of fluid in the pleural space. Patients with pleural effusions most often report dyspnea, cough, and pleuritic chest pain. Large pleural effusions are more likely to be symptomatic than smaller effusions. Physical findings are usually absent in small effusions. Larger effusions may present with dullness to percussion and diminished or absent breath sounds over the affected area. A diagnostic thoracentesis should be performed whenever there is a new pleural effusion and no clinically apparent cause, when there is an atypical presentation, or when an effusion fails to resolve as expected. Sampling allows visualization of the fluid in addition to chemical and microbiologic analyses to help identify the etiology of the effusion. Pleural samples should be sent for measurement of protein, glucose, and LDH, in addition to total and differential white blood cell counts. These chemistry tests are used to classify effusions as a transudate or an exudate. This distinction is important because the differential diagnosis for each is entirely different. A pleural exudate is an effusion that has 1 or more of the following laboratory features: (1) ratio of pleural fluid protein to serum protein > 0.5; (2) ratio of pleural fluid LDH to serum LDH > 0.6; and (3) pleural fluid LDH greater than 2/3 the upper limit of normal serum LDH. Additionally, samples with low glucose levels usually indicate a bacterial or significant inflammatory etiology. In contrast, transudative effusions have none of these features. Transudates are also distinguished by fewer than 1000 white blood cells/microliter and a pleural glucose level equal to serum. These types of effusions occur in the setting of normal capillary integrity and also suggest the absence of local pleural disease. The characteristics of the pleural fluid from the patient in the clinical scenario presented above indicate an exudative pleural effusion. Malignancy is the most likely etiology in this patient, as a bacterial cause would typically present with a decreased glucose level in the pleural fluid compared with the serum; there is an equal glucose level in this case. Congestive heart failure accounts for >90% of transudative pleural effusions. Nephrotic syndrome and cirrhosis with ascites can also lead to transudative pleural effusions. Bacterial pneumonia and cancer are the most common causes of exudative effusions.

A patient with a history of alcohol abuse presents with a histopathological report; he is asking about the 'bodies found in his liver cells'. What type of bodies is he talking about? Answer Choices 1 Russell bodies (protein inclusions) 2 Lewy bodies (protein aggregates) 3 Negri bodies (eosinophilic inclusions) 4 Heinz bodies (hemoglobin precipitations) 5 Mallory bodies (hyaline inclusions) 6 Guarnieri bodies (intracytoplasmic inclusions)

Mallory bodies (alcoholic hyaline) are cytoplasmic inclusions found most often in alcoholic hepatitis. They are abnormal aggregations of cytoskeletal proteins. Russell bodies are found in reactive or malignant plasma cells, not in liver cells. They represent excess immunoglobulin. Lewy bodies are abnormal aggregates of protein inside nerve cells in Parkinson's disease (PD) and some other degenerative brain disorders. They are not found in liver cells. Negri bodies are eosinophyllic, viral, intracytoplasmic inclusions found in hippocampal nerve cells. They are pathognomonic for rabies infection. Heinz bodies are inclusions within red blood cells that are composed of denatured hemoglobin. Although they can be found in liver disease, as well as alpha thalassemia and glucose-6-phosphate dehydrogenase, they are not found in liver cells. Guarnieri bodies, or B-type inclusions, are the sites of viral replication; they are found in all poxvirus-infected epithelial cells, but not in the hepatocytes.

A 25-year-old man presents with a 3-hour history of retrosternal chest pain. He reports that he has been vomiting since the previous evening and his last vomitus had fresh blood in it. He denies any trauma to the chest. On examination his BP is 90/60 mm Hg and PR 110/min. Question What is the most likely cause of this patient's chest pain? Answer Choices 1 Tietze's syndrome 2 Myocardial infarction 3 Panic disorder 4 Mallory-Weiss Syndrome 5 Tuberculous pleuritis

Mallory weiss Syndrome In cases of Mallory-Weiss Syndrome, patients present with hematemesis, especially after repeated severe retching and vomiting, which results in a mucosal tear at the gastroesophageal junction. They may also have retrosternal chest pain. Tuberculous pleuritis chest pain ispleuritic and aggravated by coughing. It is described as sharp. Other symptoms (e.g., fever, dyspnea, and weight loss) may also be present. On examination, there may be dullness on percussion as well as absent breath sounds on the affected side. In Tietze's syndrome, patients present with anterior chest pain that is aggravated by taking a deep breath, sneezing, and turning motions. On examination, the affected costochondral junctions are erythematous, warm, and tender on palpation. The most commonly affected are the 2nd or 3rd costochondral joints. Patients with myocardial infarction usually present with a left-sided or retrosternal pain that may radiate to the jaw, neck, and shoulder. They describe it as heaviness or a squeezing sensation. It is of variable duration and often lasts for more than 30 minutes. There is a gradual intensification of the pain. The onset of the pain may be during physical exertion or at rest. It is not relieved by nitroglycerine. On examination, they may be dyspneic and diaphoretic. Chest pain due to a panic disorder has a variable presentation. It can be retrosternal or localized, brief or over 30 minutes in duration. Numerous terms, such as aching and sharp, can be used to describe it. Other symptoms of a panic disorder include lightheadedness, shortness of breath, nausea, paresthesias, palpitations, derealization, and the fear of losing control. The history helps elicit precipitating factors and prior panic attacks.

Breast cancer is one of the most frequently occurring cancers in women. Which of the following methods is the most effective for detecting breast cancer? Answer Choices 1 Mammography 2 Breast Self Examination 3 Yearly physical examination 4 Thermography 5 Ultrasonography

Mammography Explanation The breast is the most frequent site of occurrence for cancer in women. One out of nine women will have breast cancer in their lifetime. Breast self examination (BSE) is one of the effective early detection modalities for this disease, but recent studies show that mammography is the only clinically effective screening measure for the detection of breast cancer. The American Cancer Society recommends, in the United States, annual screening mammography with clinical examination in women aged 40 years. Research has shown that BSE plays a small role in finding breast cancer compared with finding a breast lump by chance or simply being aware of what is normal for each woman. Some women feel very comfortable doing BSE regularly (usually monthly) which involves a systematic step-by-step approach to examining the look and feel of one's breasts. Doing BSE regularly is one way for women to know how their breasts normally feel and to notice any changes. Thus, more than actual detection and decrease in deaths due to early detection of cancer, BSE helps create familiarity for the patient with her breasts and thereby report any changes earlier to the doctor.

A 32-year-old woman comes to your office for contraception. She has divorced her husband two weeks ago and is now dating several new boyfriends. She states she has started a new company just over a week ago and has sunk all of her money into it. Although she knows that businesses like hers have not done well lately, she is certain the market is about to shift in her favor. She states she must hurry and get back to work, that she hasn't even had time to sleep for over a week. She speaks very rapidly and is pacing back and forth, seemingly unable to sit still. Conversation is difficult as she is easily distractible. She denies use of drugs or alcohol. She also denies any history of depression. What is her most likely diagnosis? A Bipolar disorder B Delusional disorder C Hypomanic episode D Manic episode E Neurosis

Manic episode D The patient described is exhibiting behaviors consistent with a manic episode (D). Her symptoms have been present for over a week and are impairing her ability to function, so it is not hypomania (C). As she has no clear history of depression, it is likely that she will have a depressive episode in the future, which will allow for a diagnosis of bipolar disorder (A). She does exhibit grandiosity, but there are no clear delusions (B). Neurosis (E) is a more general term for cognitive distress including some form of anxiety.

You receive notice that one of your patients, a 2-month-old boy, was found dead in his parents' bed. Sudden Infant Death Syndrome (SIDS) is suspected. He was just seen for his 2-month well baby check and seemed healthy and thriving, although he was recovering from a mild upper respiratory infection. Everything looked consistent with a mild cold on his exam and he was afebrile. You recall that his mother mentioned she had returned to work part-time and found a family care home for him while she worked. You remember counseling about back to sleep care and other risks for SIDS. Although she is a smoker, she had cut back during her pregnancy and said she now only smokes outside. She seemed receptive, but she was also preoccupied with her 2 other small children present and seemed very tired. What major risk factor in almost every epidemiologic study of SIDS may have influenced this case? Answer Choices 1 Bedsharing 2 Care by secondary caregivers 3 Presence of soft objects in crib 4 Pacifier use 5 Maternal smoking during pregnancy

Maternal smoking during pregnancy SIDS is considered the sudden death of an infant under the age of 1 year that is unexplained by thorough case investigation, autopsy, and review of case history. The occurrence peaks between 2 - 3 months of age. Since 1994, when the Back to Sleep campaign began recommending placing infants in a supine position, there has been a consistent decrease in SIDS rate; however, SIDS is still responsible for more infant deaths in the United States than any other cause of death during infancy beyond the neonatal period. The predominant hypothesis concerning the etiology remains that certain infants may have a maldevelopment or delay in maturation of the brainstem neural network responsible for arousal, and this affects the physiologic response to life-threatening challenges during sleep. Independent risk factors for SIDS that have been consistently identified across studies are prone sleep position, sleeping on a soft surface, overheating, late or no prenatal care, young maternal age, preterm birth and/or low birth weight, male gender, and maternal smoking during pregnancy. Almost every epidemiologic study has shown maternal smoking during pregnancy to be a major risk factor. Smoking after birth has emerged as a separate risk factor, but separating this variable from maternal smoking before birth is difficult. Bed sharing is highly controversial. Although it facilitates breastfeeding and enhances the infant-maternal bond, it can be hazardous under certain conditions. SIDS risk seems to be particularly high with multiple bed sharers, especially if alcohol is consumed. It can be extremely hazardous when sleeping with an infant on a couch. The longer the duration of bed sharing is during the night, the higher the risk. There is, however, evidence that room sharing without bed sharing is the most protective sleep setting for the 1st 6 months and should be recommended. Approximately 20% of SIDS deaths occur while in the care of a non-parental caregiver. Many of these deaths have been associated with the prone sleep position, especially when the infant is accustomed to sleeping supine. This has shown to actually increase the risk of SIDS 18-fold. Some centers are unaware of the dangers of sleeping prone and/or are misinformed of the risks and benefits of sleep positions. Studies have shown that infants dying of SIDS were more likely to have used a pillow or soft mattress, and they were found with their nose and mouth completely covered by bedding or assumed a facedown posture. Soft objects such as stuffed toys, quilts, blankets, and loose bedding should be kept out of the crib. Blankets should be tucked in around the crib mattress. Several new studies have reported an actual protective effect of pacifiers on the incidence of SIDS. The mechanism is unclear, but lowered arousal threshold among users has been proposed. Others theorize that pacifier use may enhance an infant's ability to breathe through the mouth if the nasal airway becomes obstructed. Retroposition of the tongue can lead to obstructive apnea and asphyxiation; sucking of the pacifier requires forward positioning of the tongue, decreasing risk of oropharyngeal obstruction.. There has also been a concern raised about an approximate 1 - 2 fold increased risk of otitis media associated with pacifier use, but this incidence is generally lower in the 1st year of life, especially in the 1st 6 months, which is when the risk of SIDS is highest.

A 23-month-old boy presents with a sudden onset of bright red blood in his bowel movements. He is afebrile and does not seem to be in any pain. A technetium-99m pertechnetate scan after enhancement with cimetidine shows a 'hot spot' in the area of his ileum. Question What is the most likely cause? Answer Choices 1 Bleeding arteriovenous ileal malformation > 1 cm 2 Ileal bleeding secondary to factor VIII deficiency (hemophilia) 3 An ileal polyp 4 Intussusception 5 Meckel diverticulum

Meckel Diverticulum This patient is most likely suffering from a common congenital abnormality of the development of the ileum called a Meckel's diverticulum. In the embryo, the vitelline duct is a communication between the yolk sac and the lumen of the gastrointestinal tract at the midgut. Normally, it degenerates completely, but the persistence of a portion of the vitelline duct leads to the development of a cul-de-sac on the ileum (Meckel's diverticulum). This congenital birth defect follows a rule of 2s. It occurs in 2% of the population, but only 2% show symptoms. It is usually located about 2 feet from the ileocecal valve and is about 2 inches long. It can contain 2 types of ectopic tissue: gastric or pancreatic. It usually presents by 2 years old. The presence of ectopic gastric mucosa in the diverticulum can lead to secretion of stomach acid downstream from the duodenum, which has bicarbonate-secreting submucosal Brunner glands to neutralize gastric acid. Ectopic gastric tissue in the ileal diverticulum can lead to ulceration and bleeding of the adjacent ileal mucosa. Vascular malformations, intussusception, and coagulation disorders are more likely to manifest themselves in the first year of life. Cow's milk colitis is a problem of the first year of life and spontaneously resolves by the end of that year for most children. NEC is a problem of the stressed, usually premature, newborn. Rectal polyps are likely to present in older children, as is HSP. Babies and older children can have gastric bleeding from gastritis or gastric ulcers. Duodenal ulcers are much more common in older children.

An 82-year-old man presents with difficulty breathing and chest pain. He admits to poor follow-up with his primary care physician; he has not been seen by his doctor in several years. When last seen, his health issues included hypertension, hyperlipidemia, diabetes mellitus type II, and being grossly overweight. His prescriptions ran out, so he has not been on any treatment for these issues for several years. Chest X-ray done at the bedside reveals significant bilateral effusions that are initially felt to be secondary to congestive heart failure based on various other findings. A pulmonologist is consulted and performs bilateral thoracentesis to help alleviate the patient's symptoms. The right side is significantly more affected than the left, with over 1000mL drained versus about 625mL on the left. Question During initial testing, what finding was most indicative of these thoracentesis results? Answer Choices 1 Bilateral decreased tactile fremitus 2 Mediastinum shifted to the right 3 Mediastinum shifted to the left 4 Pleural friction rub heard throughout 5 Bilateral inaudible breath sounds

Mediastinum shifted to the left Mediastinum shifted to the left is the correct answer. Patients with a pleural effusion that is over 1000mL can experience a shift in the mediastinum away from the side with the larger effusion. A pleural effusion is an accumulation of fluid within the pleural cavity, which is the area between the chest wall and the lung. The increased pressure within the affected side of the chest causes the mediastinum, including the trachea, to shift away from the side with the larger amount of fluid (and therefore larger amount of pressure). In this case, the mediastinum would shift towards the left, as the right side has almost twice the amount of fluid as the left. Decreased tactile fremitus is not the correct answer. Tactile fremitus is vibration felt on a patient's chest while the patient vocalizes at low frequency. The patient is asked to repeat the same phrase over and over while the examiner moves their palm throughout the various lung fields. Tactile fremitus can be decreased in cases of any pathology separating the lung tissue from the chest wall. Pleural effusion is a common cause of decreased tactile fremitus. It is incorrect in this case because a physical examination is very nonspecific. The examiner would not necessarily know from assessing tactile fremitus the amount of pleural effusion or the differentiation between the sides. Mediastinum shifted to the right is not the correct answer. If the patient's mediastinum was shifted toward the right in this case, then it would be shifted toward the affected side. In patients with large pleural effusions, the side with the higher amount of pressure pushes the shift away from the affected side. Patients with atelectasis, pleural fibrosis, or who are status post-pneumonectomy would experience a decreased pressure within the pleural cavity on 1 side; therefore, the mediastinum would be shifted toward that affected side. Pleural friction rub is not the correct answer. A pleural friction rub is heard on cardiac auscultation and is the result of the pleural linings rubbing together. In cases when the pleural linings are inflamed (such as in pneumonia, pleural effusion, or pulmonary embolism), a pleural friction rub can be heard. The finding is nonspecific in terms of the amount of fluid or differentiation between sides; therefore, it would not necessarily be indicative of these particular thoracentesis findings. Bilateral inaudible breath sounds is not the correct answer. Due to the increased fluid and pressure associated with pleural effusion, the breath sounds may not be heard during physical examination. However, not hearing the sounds on either side does not give any indication as to how severe the pleural effusion is (although this typically happens in the larger effusions) and certainly does not show the differentiation between sides.

An 82-year-old man presents with difficulty breathing and chest pain. He admits to poor follow-up with his primary care physician; he has not been seen by his doctor in several years. When last seen, his health issues included hypertension, hyperlipidemia, diabetes mellitus type II, and being grossly overweight. His prescriptions ran out, so he has not been on any treatment for these issues for several years. Chest X-ray done at the bedside reveals significant bilateral effusions that are initially felt to be secondary to congestive heart failure based on various other findings. A pulmonologist is consulted and performs bilateral thoracentesis to help alleviate the patient's symptoms. The right side is significantly more affected than the left, with over 1000mL drained versus about 625mL on the left. Question During initial testing, what finding was most indicative of these thoracentesis results? Answer Choices 1 Bilateral decreased tactile fremitus 2 Mediastinum shifted to the right 3 Mediastinum shifted to the left 4 Pleural friction rub heard throughout 5 Bilateral inaudible breath sounds

Mediastinum shifted to the left is the correct answer. Patients with a pleural effusion that is over 1000mL can experience a shift in the mediastinum away from the side with the larger effusion. A pleural effusion is an accumulation of fluid within the pleural cavity, which is the area between the chest wall and the lung. The increased pressure within the affected side of the chest causes the mediastinum, including the trachea, to shift away from the side with the larger amount of fluid (and therefore larger amount of pressure). In this case, the mediastinum would shift towards the left, as the right side has almost twice the amount of fluid as the left. Decreased tactile fremitus is not the correct answer. Tactile fremitus is vibration felt on a patient's chest while the patient vocalizes at low frequency. The patient is asked to repeat the same phrase over and over while the examiner moves their palm throughout the various lung fields. Tactile fremitus can be decreased in cases of any pathology separating the lung tissue from the chest wall. Pleural effusion is a common cause of decreased tactile fremitus. It is incorrect in this case because a physical examination is very nonspecific. The examiner would not necessarily know from assessing tactile fremitus the amount of pleural effusion or the differentiation between the sides. Mediastinum shifted to the right is not the correct answer. If the patient's mediastinum was shifted toward the right in this case, then it would be shifted toward the affected side. In patients with large pleural effusions, the side with the higher amount of pressure pushes the shift away from the affected side. Patients with atelectasis, pleural fibrosis, or who are status post-pneumonectomy would experience a decreased pressure within the pleural cavity on 1 side; therefore, the mediastinum would be shifted toward that affected side. Pleural friction rub is not the correct answer. A pleural friction rub is heard on cardiac auscultation and is the result of the pleural linings rubbing together. In cases when the pleural linings are inflamed (such as in pneumonia, pleural effusion, or pulmonary embolism), a pleural friction rub can be heard. The finding is nonspecific in terms of the amount of fluid or differentiation between sides; therefore, it would not necessarily be indicative of these particular thoracentesis findings. Bilateral inaudible breath sounds is not the correct answer. Due to the increased fluid and pressure associated with pleural effusion, the breath sounds may not be heard during physical examination. However, not hearing the sounds on either side does not give any indication as to how severe the pleural effusion is (although this typically happens in the larger effusions) and certainly does not show the differentiation between sides.

A 45-year-old man presents with what he refers to as "anxiety attacks". He describes them as recurring bouts of palpitations, headaches, anxiety, and sweating. One evening, while having dinner with his wife, his wife witnesses an episode. She is concerned and immediately takes his blood pressure. His blood pressure is 195/105 mmHg. The next day, his wife takes him to see their family doctor. On further questioning, he comments that he sometimes gets light headed when he stands up too rapidly. He comments that his mother had similar problems. On physical examination, his blood pressure is 165/90 mmHg and his heart rate is 80 beats/min. A 24-hour collection of his urine test is positive for vanillylmandelic acid. Question There is a familial association of this condition with what other disease? Answer Choices 1 Medullary carcinoma of the thyroid 2 Follicular carcinoma of the thyroid 3 Papillary carcinoma of the thyroid 4 Pancreatic islet cell carcinoma 5 Pituitary adenoma

Medullary carcinoma of the thyroid This patient exhibits signs and symptoms of a pheochromocytoma. He has paroxysmal hypertension and paroxysmal episodes of sweating and anxiety. The paroxysmal episodes that he experiences are due to intermittent release of catecholamines from the pheochromocytoma. Postural hypotension can sometimes be seen with pheochromocytoma. Urinary vanillylmandelic acid (VMA) is a metabolite of catecholamines and is found in pheochromocytoma. Pheochromocytoma is 1 of the diseases seen with multiple endocrine neoplasia, type II. Multiple endocrine neoplasia, type II (Sipple's syndrome) consists of pheochromocytoma, medullary carcinoma of the thyroid, and parathyroid hyperplasia. Pituitary adenoma (or hyperplasia), pancreatic islet cell tumors, and parathyroid tumors are components of multiple endocrine neoplasia type I (Wermer's syndrome). Follicular carcinoma of the thyroid and papillary carcinoma of the thyroid are not associated with pheochromocytoma.

A 45-year-old man presents with episodic attacks of headache, recurring bouts of palpitations, anxiety, and sweating. He also gives history of a severe attack 1 week prior; it occurred while he was having wine and cheese with his wife. On further questioning, he comments that he gets light headed when he stands up too rapidly. He comments that his mother had similar problems. On physical examination, his blood pressure is 165/90mmHg and his heart rate is 80/min. A 24-hour collection of his urine tests positive for vanillylmandelic acid. Imaging studies showed bilateral adrenal medullary hyperplasia. Further work-up showed hypercalcemia, hypophosphatemia, and increased parathyroid hormone levels. The positive family history and investigative findings are suggestive of multiple endocrine neoplasia (MEN) in this patient. Question MEN type II is characterized by pheochromocytoma (PCC), hyperparathyroidism and which of the following conditions? Answer Choices 1 Medullary carcinoma of the thyroid 2 Pituitary adenoma 3 Pancreatic islet cell carcinoma 4 Follicular carcinoma of the thyroid 5 Papillary carcinoma of the thyroid

Medullary carcinoma of the thyroid Explanation This patient exhibits signs and symptoms of a pheochromocytoma. Orthostatic hypotension can be seen with pheochromocytoma. He has hypertension and paroxysmal episodes. The paroxysms that he experiences are due to intermittent release of catecholamines from the pheochromocytoma. Urinary vanillylmandelic acid (VMA) is a metabolite of catecholamines and is commonly found in pheochromocytoma. Multiple endocrine neoplasia type II (Sipple's syndrome) is a rare autosomal dominantly inherited syndrome; it is characterized by pheochromocytoma, medullary carcinoma of the thyroid, and primary hyperparathyroidism or parathyroid hyperplasia. In contrast to sporadic PCC, the familial variety within MEN-IIA begins with adrenal medullary hyperplasia. It is multi-centric and bilateral in more than 50% of the cases. Normal adrenal medulla is composed of roughly 85% epinephrine, in comparison; most PCCs predominantly contain norepinephrine. The familial pheochromocytomas are an exception because they secrete large amounts of epinephrine. Diagnosis involves genetic testing; hormonal assay, and imaging tests to locate the tumors. Pituitary adenoma and pancreatic islet cell tumors are components of multiple endocrine neoplasia type I (Wermer's syndrome). Follicular carcinoma of the thyroid and papillary carcinoma of the thyroid are not associated with pheochromocytoma.

A 38-year-old male with a history of frequent urinary tract infections (UTIs) and kidney stones is following up with the urology physician assistant. His urine today continues to reveal persistent hematuria, which he had the last few times he saw his primary care physician. His vitals are 120/80, P = 72, T = 96.7˚F. He denies any discomfort at this time. GFR is normal. You review a urogram, which reveals a paint-brush appearance of the papillae. The kidneys are normal in size. What is the most likely diagnosis? A chronic renal insufficiency B renal cell carcinoma C end stage renal disease D medullary sponge disease E polycystic kidney disease

Medullary sponge disease The correct answer is (D). The patient's history of hematuria, recurrent UTIs, and kidney stones are a common presentation of this inherited disease with medullary cysts. The findings on the urogram of a paint-brush appearance of the papillae are pathognomonic of the disease. Choices (A) and (C) are unlikely due to the normal GFR. The patient's history is not suggestive of choice (B), renal cell carcinoma. A solitary solid renal mass would suggest renal cell carcinoma.

A 45-year-old man presents with what he refers to as "anxiety attacks". He describes them as recurring bouts of palpitations, headaches, anxiety, and sweating. One evening, while having dinner with his wife, his wife witnesses an episode. She is concerned and immediately takes his blood pressure. His blood pressure is 195/105 mmHg. The next day, his wife takes him to see their family doctor. On further questioning, he comments that he sometimes gets light headed when he stands up too rapidly. He comments that his mother had similar problems. On physical examination, his blood pressure is 165/90 mmHg and his heart rate is 80 beats/min. A 24-hour collection of his urine test is positive for vanillylmandelic acid. Question There is a familial association of this condition with what other disease? Answer Choices 1 Medullary carcinoma of the thyroid 2 Follicular carcinoma of the thyroid 3 Papillary carcinoma of the thyroid 4 Pancreatic islet cell carcinoma 5 Pituitary adenoma

Medullary thyroid cancer Explanation This patient exhibits signs and symptoms of a pheochromocytoma. He has paroxysmal hypertension and paroxysmal episodes of sweating and anxiety. The paroxysmal episodes that he experiences are due to intermittent release of catecholamines from the pheochromocytoma. Postural hypotension can sometimes be seen with pheochromocytoma. Urinary vanillylmandelic acid (VMA) is a metabolite of catecholamines and is found in pheochromocytoma. Pheochromocytoma is 1 of the diseases seen with multiple endocrine neoplasia, type II. Multiple endocrine neoplasia, type II (Sipple's syndrome) consists of pheochromocytoma, medullary carcinoma of the thyroid, and parathyroid hyperplasia. Pituitary adenoma (or hyperplasia), pancreatic islet cell tumors, and parathyroid tumors are components of multiple endocrine neoplasia type I (Wermer's syndrome). Follicular carcinoma of the thyroid and papillary carcinoma of the thyroid are not associated with pheochromocytoma.

Which of the following sets of disorders is commonly found in multiple endocrine neoplasia (MEN) 2A? A Islet cell tumor; renal cell carcinoma; pheochromocytoma; B Pheochromocytoma; medullary thyroid carcinoma; mucosal neuromas C Medullary thyroid carcinoma; parathyroid hyperplasia; pheochromocytoma D Parathyroid adenoma; islet cell hyperplasia; pituitary adenoma E Visceral lipomas; Marfanoid features; retinal angiomas

Medullary thyroid carcinoma; parathyroid hyperplasia; pheochromocytoma The correct choice is C. The three primary features of MEN type 2A include medullary thyroid carcinoma, parathyroid hyperplasia or adenoma, and pheochromocytoma. In choice A, islet cell tumor and renal cell carcinoma are disorders found in various other MEN syndromes, but not in MEN type 2A. Choice B, pheochromocytoma, medullary thyroid carcinoma, and mucosal neuroma are disorders found in MEN 2B. Choice D, parathyroid adenoma, islet cell hyperplasia, and pituitary adenoma are found in MEN I. Choice E, visceral lipomas, Marfanoid features, and retinal angiomas are features found in various MEN syndromes, but not in MEN 2A.

A 22-year-old woman presents with an 8-hour history of headache and vomiting. She resides in a college dormitory; she has no remarkable travel history, and reports eating food from a street vendor 24 hours before becoming ill. The patient is lethargic and disoriented. On examination, she is found to have a temperature of 38.5° C and a petechial rash on her legs. Gram stain of exudate from a skin lesion shows Gram-negative diplococci. A lumbar puncture is performed; the cerebral spinal fluid (CSF) reveals a leukocyte count of 5/μL consisting of primarily neutrophils. A Gram stain of CSF is negative for bacteria. What is the most likely diagnosis? Answer Choices 1 Rocky Mountain spotted fever 2 Neisseria gonorrhoeae bacteremia 3 Listeria monocytogenes meningitis 4 Meningococcemia 5 Viral meningitis

Meningococcemia Meningococcemia may evolve rapidly, with death ensuing before the development of meningitis. The presence of Gram-negative diplococci and the clinical picture are consistent with a diagnosis of bacteremia caused by Neisseria meningitidis. While also a Gram-negative diplococcus, N. gonorrhoeae is an unlikely cause of this clinical presentation. Listeria monocytogenes, transmitted by contaminated food, is also a common cause of meningitis. This organism is not typically associated with a petechial rash; it appears as a Gram-positive coccobacillus in the CSF, where mononuclear cells predominate. Rocky-Mountain spotted fever is caused by the Gram-negative obligate intracellular pathogen Rickettsia rickettsii, which is not detectable by Gram stain. Infection with R. rickettsii results in a macropapular rash. Viral meningitis is usually accompanied by monocytes in the CSF, although neutrophils may appear early in the disease.

A 50-year-old woman presents for her annual pelvic examination; her previous examination took place about 18 months ago. She tells you that her last menstrual period was over 12 months ago; the last few occurrences of menses were extremely irregular. The patient also describes having multiple daily episodes of severe, intense heat in her face and trunk that is accompanied by sweating. She further states that these "heat episodes" have been occurring 4 - 6 times a day for the last 4 weeks. They have been interfering with her everyday activities, as well as her sleep. She has no other complaints at this time. She has received her annual pelvic examinations yearly as well as clinical breast exams and mammograms; there have been no significant findings. During the pelvic examination, you note obvious vaginal thinning and excessive dryness; there is also apparent vaginal wall atrophy. Question What pharmacologic intervention can be prescribed in order to assist in ameliorating the patient's symptoms? Answer Choices 1 Menopausal hormone therapy 2 Low-dose oral contraceptives 3 Phytoestrogens 4 Progestin-releasing intrauterine devices 5 Testosterone replacement therapy

Menopausal hormone therapy Explanation This patient scenario above is most likely the result of menopause. Menopause in the most pure sense of explanation is a cessation of menstruation from either natural aging (usually amenorrhea for at least 6 months) or an external cause (surgical). There is usually a 1 -3 year time period that women will typically adjust physiologically to the diminished hormonal and menstrual actions and the effects this has on their body. These effects may include hot flashes, night sweats, vaginal dryness, and in the later stages, osteoporosis. In western societies, the average age at which women achieve menopause is about 51 years old. The patient described in the scenario above is suffering from moderate-to-severe symptoms of the physiological menopausal transition. Given that this patient does not have any obvious risk factors, menopausal hormone therapy (MHT) is the best pharmacological option at this time. At one time, MHT was referred to hormone replacement therapy. Benefits of the use of MHT include reducing hot flashes, night sweats, and other related issues, such as poor sleeping and irritability. MHT will also treat vaginal symptoms related to menopause, such as vaginal dryness and discomfort. Another benefit to its use is slowing of bone loss and easing mood swings or depressive symptoms. An extremely critical component that must be remembered by the healthcare provider and the patient if MHT is started is that the use of MHT is only recommended for a short period of time and at the lowest effective dose. Low-dose oral contraceptives (or birth control pills) may be an option for menopausal symptom relief only if the patient is perimenopausal. Because this patient has not had her menstrual period in over 8 months, she is categorized as being menopausal, so this treatment would not be appropriate. Phytoestrogens are substances found in plants, such as soy and red clover extracts, that bind to estrogen receptors. They do not appear to significantly improve menopausal hot flashes, and they would not be the best option for this patient. Progestin-releasing intrauterine devices (IUDs) would not be an appropriate treatment option as this is only used for patients using MHT to reduce the incidence or dysfunctional uterine bleeding and endometrial carcinoma. Testosterone replacement therapy in women is reserved for those patients suffering from mainly hypoactive sexual desire disorder (HSDD); other signs and symptoms that may benefit from testosterone replacement therapy include hot flushes, pubic atrophy, muscle atrophy, and osteoporosis; however, this is not the correct treatment option for this patient.

A 65-year-old man presents with fatigue, shortness of breath, chest pain, and weight loss. Physical exam findings include diminished breath sounds on auscultation, and there is dullness to percussion. You order a chest X-ray; it reveals a pleural-based mass. What type of lung cancer is most likely? Answer Choices 1 Adenocarcinoma 2 Squamous cell carcinoma 3 Small cell carcinoma 4 Large cell carcinoma 5 Mesothelioma

Mesothelioma The clinical picture is suggestive of a mesothelioma lung cancer. The mean age at onset of symptoms is 60 years old with a history of asbestos exposure. Common findings include weight loss, shortness of breath, fatigue, and chest pain. Other findings include diminished breath sounds, digital clubbing, and dullness to percussion. A key feature is a pleural-based mass or thickening of the pleura on imaging. Adenocarcinomas usually present as peripheral nodules or masses on imaging. Squamous cell carcinoma usually presents as central masses on imaging and are more likely to present with hemoptysis. Small cell carcinomas typically begin centrally with concomitant hilar and mediastinal abnormalities present. Large cell carcinomas on imaging present as central or peripheral masses, but they do not usually involve the pleura.

A 50-year-old woman presents with right-sided pleural effusion. Thoracentesis shows the presence of exudative serosanguineous pleural fluid and positive cytology. For what condition is this finding most typical? Answer Choices 1 Metastatic infiltrating ductal carcinoma 2 Cor pulmonale 3 Systemic lupus erythematosus 4 Staphylococcus aureus septicemia 5 Pulmonary infarction

Metastatic infiltrating ductal carcinoma A blood-tinged effusion in anyone past the age of 40 should put neoplasia into the differential diagnosis. Neoplastic pleural invasion usually involves both the visceral and the parietal surfaces and can cause a bloody pleural effusion. Breast cancers often metastasize to the lung and pleura. Carcinomas of the lung and breast are the most common primary sites from which pleural metastases arise. Stomach and ovarian carcinomas are next in frequency for the development of malignant pleural effusion. Spread of malignancy to the pleura from breast, stomach, and ovarian cancers, usually occurring indirectly from hepatic metastases. However, contiguous spread through the chest wall in breast cancer, and through the diaphragm in ovarian or stomach cancer, can occur occasionally. When malignancy is suspected, cytological examination of the fluid helps in establishing the diagnosis. Pleural biopsy, or pleural fluid immunocytochemistry, is used as an adjunct to cell morphology to aid diagnosis. Cor pulmonale results in peripheral edema including pleural effusion. These are usually serous effusions, which is transudative in nature. Systemic lupus erythematosus (SLE) can be associated with exudative pleural effusions, but they are usually serous and present bilaterally. Bacterial sepsisdue to S. aureus could lead to pleuritis with purulent exudates. Pulmonary embolism (a common cause of pulmonary infarction) results in bloody pleural effusions; however, the cytological examination will be negative.

A 27-year-old woman complains of years of menstrual irregularity and increasing facial and chest hair. PMH: significant for ovarian cyst and left cytectomy. She is a non-smoker and non-drinker. Labs include a negative uCG, elevated LH, and low FSH. She desires fertility and she has not responded to three cycles of clomiphene. What would be the next choice of medication that may return ovulation? A Insulin B Metformin C Dexamethasone D Spironalactone E Finasteride

Metformin B Dexamethasone, finasteride, and spironalactone all treat symptoms of hirsutism, but do not treat the underlying cause of PCO or improve fertility outcomes. PCO has an underlying insulin resistance that can be treated with oral hypoglycemics and improve sensitivity to insulin. Adding insulin does not improve the resistance.

An 88-year-old female patient has been advised by her primary care physican that she needs a computed tomography (CT) scan of her abdomen and pelvis due to persistent abdominal pain, bloating, and weight loss. She was told that she needs to hold one of her medications the day of the procedure and that she may resume the medication 48 hours later. She can't remember which medication she should discontinue. Which medication listed below should she discontinue temporarily as specified above due to the diagnostic test ordered? A glyburide B glipizide C pioglytizone D metformin E acarbose

Metformin The correct answer is (D). A CT of the abdomen and pelvis requires p.o. and IV iodinated contrast unless ordered specifically without IV contrast. There is an increased risk of acute renal failure with IV iodinated contrast. The risk to the patient may increase with metformin and therefore should ideally be held prior to and for 48 hours after any radiocontrast IV study to avoid the added possibility of lactic acidosis. The other treatments for non-insulin-dependent diabetes mellitus (NIDDM) do not require discontinuation with IV contrast studies for these time periods. However, the am dose of a sulfonylurea (glyburide or glipizide) may be held until after the study that day when the patient resumes eating to avoid hypoglycemia in some patients.

A 21-year-old woman presents with urinary frequency; her BMI is 41. A urinalysis is positive for glucose. Her random blood sugar is 257 and hemoglobin A1C is 8.5%. She is diagnosed with type II diabetes, diet and exercise are recommended. What is the drug of choice for managing her diabetes? Answer Choices 1 Metformin (Glucophage) 2 Glipizide (Glucotrol) 3 Pioglitazone (Actos) 4 Insulin 5 Acarbose (Precose)

Metformin is the only hypoglycemic agent that has been shown to reduce mortality rates in patients with type II diabetes. Insulin resistance, decreased insulin secretion, and increased hepatic glucose output are the hallmarks of type II diabetes. Metformin decreases hepatic glucose output and, to a lesser extent, sensitizes peripheral tissues to insulin. Metformin can reduce A1C levels by an average of 1 - 2.5%. Metformin has also been shown to slow progression of impaired glucose tolerance to diabetes. Metformin causes weight loss and is not associated with hypoglycemia. Gastrointestinal symptoms associated with metformin can be minimized by starting at a low dose and titrating slowly. It is usually administered 2 times a day. Long-acting preparations that are administered once daily are also available. Metformin rarely causes lactic acidosis, and current guidelines recommend against using metformin when creatinine levels are ≥1.4 in women and ≥1.5 in men. Metformin is approved for use in children over 10 years. Glipizide is a sulfonylurea insulin secretagogue, and it has not been shown to have any effect on cardiovascular morbidity or mortality. Severe hypoglycemia can occur with glipizide. Actos causes fluid retention and weight gain. It increases the incidence of heart failure, but it has not been shown to affect cardiovascular mortality. Insulin is typically introduced when glucose control is no longer possible with oral agents, or when contraindications to oral agents exist. Newly diagnosed patients may also benefit from short-term insulin use if sugars are extremely high. Such patients would typically have A1C levels greater than 9%. Acarbose is an alpha-glucosidase inhibitor and has been shown to reduce cardiovascular events; average A1C reduction is only 0.5 - 1%. It has to be taken 3 times a day with meals; gastrointestinal side effects can be troubling.

A 24-year-old G2P2 delivered a viable female infant (8 lb 4 oz) via caesarean section, after a failed 20-hour induction for post date pregnancy. On day 2, she developed a postoperative fever of 101F (38.3C). She had slightly increasing abdominal cramping and pain, no change in loci, is voiding well, and has passed flatulence. Her WBC is 19,000. What is the mostly likely cause for her fever? A Urinary tract infection B Ileus C Metritis D Atelectasis E Tubo-ovarian abscess

Metritis C The patient is passing urine and flatulence well, making choices A and B less likely. Fever greater the 38C is the most important indicator of metritis. Fever is not usually indicative of mild atelectasis. Tubo-ovarian abscess is usually a complication from PID. The patient has many risk factors for metritis including c-sect, prolonged induction, and fever.

A 52-year-old man presents with a 3-day history of persistent diarrhea, having up to 7 watery bowel movements without bleeding a day. He has associated lower-abdominal cramping and notes mild nausea. He denies travel outside of the country. He does not recall eating anything unusual, and none of his family members are sick. Past medical history is significant for GERD, for which he takes protonix daily. He recently had an upper respiratory infection and completed a course of oral levofloxacin approximately 3 weeks ago. A stool sample is obtained. Stool is negative for ova and parasites, but PCR testing is positive for Clostridium difficile. Question What is the appropriate first-line treatment in this patient? Answer Choices 1 Loperamide 2 Lomotil 3 Prednisone 4 Metronidazole 5 Clindamycin

Metronidazole Metronidazole is correct. The patient is infected with Clostridium difficile, a Gram-positive, spore-forming bacillus. It is manifested by watery diarrhea, which left untreated can progress to fulminant colitis. Previous antibiotic use is the leading cause of Clostridium difficile, as the use of antibiotics alters the normal colonic flora, allowing Clostridium difficile to proliferate. Patients who have confirmed infection should first discontinue any offending antibiotics. The patient is then treated with either oral metronidazole or oral vancomycin. Due to its cost, oral vancomycin is typically reserved for severe or recurrent cases of Clostridium difficile. Loperamide and lomotil are incorrect. These medications are both anti-motility agents. Their use is not recommended in the treatment of Clostridium difficile, as anti-motility agents have been associated with the development of toxic megacolon and systemic infection. Prednisone is incorrect. While corticosteroids have been used in severe Clostridium difficile colitis as adjunctive treatment to reduce inflammation, their use is not recommended for first-line treatment. In order to clear Clostridium difficile, the patient must undergo treatment with either metronidazole or vancomycin. Clindamycin is incorrect. Clindamycin is not active against Clostridium difficile. In fact, its use has been associated with development of Clostridium difficile.

A young woman presents with pneumonia; a course of cephalosporins is given to treat this condition. A few days after starting the treatment, she develops a non-bloody, watery diarrhea with abdominal cramps. She returns to the doctor's office. Her stool is sent to the lab, and a colonoscopy is performed to find out the cause of this diarrhea. The colonoscopy shows yellow-white plaques, and membranes on colonic mucosa. What antibiotic best treats this condition? Answer Choices 1 Ampicillin 2 Metronidazole 3 Tetracycline 4 Trimethoprim-sulfamethoxazole 5 Clindamycin

Metronidazole The correct response is metronidazole. This young woman is presenting with a typical presentation of antibiotic-associated pseudomembranous colitis. The cause of this condition is Clostridium difficile, which is a Gram-positive, spore-forming rod. It is found in the gastrointestinal tract in approximately 3% of the general population. Antibiotics (e.g., clindamycin, ampicillin, and cephalosporins) suppress drug-sensitive members of the normal flora, allowing C. difficile to multiply and produce cytopathic toxins called cytotoxin A and B. Initially, the treatment is to withdraw the causing antibiotic; if the diarrhea is severe and patient is ill, treatment with metronidazole is recommended. Vancomycin can be used as well, but it is not preferred over metronidazole because of the risk of developing resistant enterococci. Cytotoxin A is an enterotoxin causing an outpouring of fluid resulting in watery diarrhea. Cytotoxin B causes damage to the colonic mucosa, leading to the pseudomembranous formation. This is a heat and acid-labile protein, and it attaches to the intestinal epithelial cells at the microvilli level; it causes cell death. Clindamycin, ampicillin, and cephalosporins can cause pseudomembranous colitis, so they cannot be used for treatment. Tetracycline and trimethoprim-sulfamethoxazole do not have any effect on Clostridium difficile.

A 19-year-old presents complaining of vaginal discharge and itching for 3 days. She is sexually active and uses condoms most of the time. A physical exam reveals the following: vitals are normal, abdomen is soft and non-tender, + bowel sounds, the pelvic exam is notable for moderate discharge with no masses or tenderness. Her wet mount/KOH prep reveals 20 wbcs, 2+ bacteria, no hyphae, 5 to 7 clue cells. What is the most appropriate treatment? A Terconazole vaginal cream daily x 3 days B Cefixime 400 mg x one dose C Ciprofloxin 500 mg BID x 7 days D Metronidazole 500 mg BID x 7 days E Azithromycin 1 gram x 1 dose

Metronidazole 500 mg BID x 7 days D Clues cells with bacteria and white blood cells are indicative of Gardnerella vaginitis, which is best treated with metronidazole.

A 36-year-old man with a past medical history of diabetes mellitus presents with weight gain and skin changes. His review of systems is positive for a diminished libido, impotence, depression, cognitive dysfunction, and emotional lability. Lately, his fasting glucose levels have been above normal. His physical exam notes increased adipose tissue in the face, upper back, and above the clavicles. His skin reveals ecchymoses, telangiectasias, and purpura along his back and lower extremities; there is also facial acne and cutaneous atrophy. His abdominal exam reveals the following findings. Question What is the most appropriate pharmacotherapeutic treatment for this patient? Answer Choices 1 Ketoconazole 2 Metyrapone 3 Mupirocin 4 Prednisone 5 Tacrolimus

Metyrapone Explanation This patient is demonstrating signs and symptoms consistent with Cushing's syndrome. Medications used in the management of Cushing syndrome include somatostatin analogs (pasireotide), adrenal steroid inhibitors such as metyrapone, ketoconazole, and etomidate, glucocorticoid receptor antagonists (mifepristone), and adrenolytic agents (mitotane). Metyrapone blocks 11-beta-hydroxylase activity, the final step in cortisol synthesis. Therapy is begun at 1 g/d divided into 4 doses and increased to a maximum dose of 4.5 g/d. Adverse effects present from increases in androgen and mineralocorticoid precursors and include hypertension, acne, and hirsutism. Ketoconazole has been the most popular and effective of these agents for long-term use and usually has been the agent of choice. However, the FDA has issued a black box warning that states clinicians should no longer prescribe ketoconazole, except to treat some life-threatening fungal infections; this is due to increased risk for severe hepatotoxicity, including fatalities and liver transplantation, adrenal insufficiency, and adverse drug interactions. Prednisone is not indicated in the treatment of Cushing syndrome. Cushing syndrome is caused by prolonged exposure to elevated levels of either endogenous glucocorticoids or exogenous glucocorticoids. Exogenous use of glucocorticoids should always be considered and excluded in the etiology of Cushing syndrome. Tacrolimus is a topical immunosuppressant agent useful in the treatment of psoriasis, while mupirocin is a topical antibiotic useful in the treatment of topical skin infections such as impetigo.

A 28-year-old woman presents with a 6-hour history of severe pulsatile hemicranial headache. She had numbness of the right upper limbs and lower limbs prior to the onset of the headache. The numbness lasted for 45 minutes. The headache worsens with exertion. She had a similar episode of headache, with nausea and vomiting, 2 weeks prior to presentation. She does not have a history of head injury. She has been married for 2 months, and she is on oral contraceptives. She gives a history of febrile convulsions in her childhood, and she has a sister with epilepsy. On examination, her blood pressure is 150/84 mm Hg, and neurological examination is normal. Her serum cholesterol level is 384 mg/dl. Question What is the most likely diagnosis? Answer Choices 1 Sensory epilepsy 2 Migraine 3 Cluster headache 4 Thromboembolic transient ischemic attack 5 Hypertension

Migraine The correct answer is migraine. The patient presents with a headache which is hemicranial and pulsatile, which are characteristics commonly seen with migraines. In addition, the association of nausea/vomiting and thepresence of a precipitating factor (i.e., being on oral contraceptives) increases the likelihood of it being a migraine. The numbness lasted for less than 60 minutes, which is probably an aura of sensory symptoms. Sensory epilepsy is an incorrect response. The numbness lasted 45 minutes and disappeared; however, in cases of sensory epilepsy, there is usually a distinct progression of symptoms. Cluster headache is an incorrect response. Cluster headaches are usually accompanied by at least 1 of the following ipsilateral autonomic symptoms: conjunctival injection or lacrimation, nasal congestion or rhinorrhea, eyelid edema, forehead and facial sweating, miosis or ptosis, and restlessness or agitation. Thromboembolic transient ischemic attack is an incorrect response. In cases of transient ischemic attacks, the maximum deficit is present immediately, and headache is unusual. Hypertension is an incorrect response. Systolic blood pressure of 150 mm Hg may probably be due to pain (headache). Mild hypertension {(140 - 159)/ (90 - 99) mm Hg} is usually asymptomatic.

A 92-year-old woman with Alzheimer's disease is brought in from her long term care facility. She was found holding her abdomen and screaming in pain. She is unable to provide any history, but her aide states that the patient was incontinent of liquid stool shortly before being brought to the hospital. On examination, her abdomen is distended; there are no palpable masses. On rectal examination, there are firm feces in the rectal vault. Question What is the best initial treatment? Answer Choices 1 Fiber supplementation 2 Decompressive colonoscopy 3 Magnesium citrate 4 Mineral oil enema 5 Colectomy

Mineral oil enema Mineral oil enema is the correct response. The patient is suffering from fecal impaction. Initial treatment requires resolving the impaction by the use of enemas, digital removal, or a combination of the 2. This is necessary to avoid the development of large bowel obstruction. Fiber supplementation is incorrect. Once the patient's obstruction has resolved, fiber may be used to help prevent the development of constipation and further episodes of fecal impaction. Unless the impaction has resolved, fiber supplementation could actually harm the patient Decompressive colonoscopy is incorrect. A colonoscope would not be able to make it pass the stool involved in the fecal impaction. Magnesium citrate is incorrect. The patient should have therapy aimed at disrupting the fecal impaction. Colectomy is incorrect. Surgery would not be warranted in this patient unless she showed signs of perforation.

A 60-year-old man presents with a 6-month history of pain in his knee joints. It has been gradually increasing, and now limits his activities. X-ray reveals osteoarthritic changes in both knee joints. You consider treating him with a non-steroidal anti-inflammatory drug (NSAID); however, you note that he has a history of gastric ulcers. In the past, the patient has not tolerated proton pump inhibitors well; he has had several episodes of severe headache, diarrhea, and dizziness when treated with them. Question What is the best way to reduce the risk of NSAID-induced gastric mucosal injury in this patient? Answer Choices 1 Misoprostol 2 Milk of Magnesia 3 Octreotide 4 Ranitidine 5 Sucralfate

Misoprostol Misoprostol is a prostaglandin analogue that acts on the gastric parietal cells to reduce gastric acid secretions. Proton pump inhibitors (PPI) are better tolerated than misoprostol and may be preferred over misoprostol, even in NSAID-induced gastric injury. Ideally, this patient should be treated with a selective COX-2 inhibitor NSAID with a PPI; however, considering his poor tolerance to PPI (which is rare), misoprostol is the next best choice. Misoprostol causes nausea, diarrhea, abdominal pain, and dysmenorrhea. It should be avoided in pregnancy, as it may cause an abortion. Milk of magnesia is an antacid that acts by neutralizing the acid in the stomach; it is commonly used in dyspepsia. Octreotide is a somatostatin analogue that inhibits gastric and pancreatic secretion. It is used in Zollinger-Ellison syndrome. H2 receptor blockers (e.g., ranitidine) are used to treat gastroesophageal reflux disease, as well as gastric, duodenal, and stress ulcers; however, they are not the 1st choice in NSAID-induced injury because a double dose would be required. Sucralfate is an ulcer protective; it forms a coating on the ulcer crater and acts as a barrier to acid and pepsin.

A 17-year-old female athlete presents with easy fatigability and weakness. She has been training daily, and she has no significant medical history. Her pulse is 55/min, regular, BP 120/80 mmHg. There are no remarkable findings in a physical examination. Lab reports show the following: Serum Na+ 140 mEq/L Cl- 86 mEq/L K+ 2.3 mEq/L HCO3- 34 mEq/L pH 7.50 Urine Na+ 82 mEq/24 h K+ 168 mEq/24 h What is the most likely cause of her condition? Answer Choices 1 Bulimic vomiting 2 Hyperventilation induced by anxiety 3 Misuse of diuretics 4 Somatoform disorder 5 Use of anabolic steroids

Misuse of diuretics The patient has hypokalemia and alkalosis. Even though her serum potassium is low, she is still losing potassium in the urine. This is consistent with the action of a diuretic drug. Surreptitious use of diuretics by athletes is a common occurrence, as they use it to reduce weight. Abuse of diuretics is also a form of bulimia nervosa. Bulimic vomiting can also cause a hypokalemic alkalosis, but in that case urinary excretion of potassium would not be elevated. Additional sign of induced vomiting, like eroded dental enamel, would be evident on physical examination. Hyperventilation would lead to a respiratory alkalosis with a compensatory mild metabolic acidosis. In case of a somatoform disorder, there would not be any abnormal lab values. Anabolic steroids are unlikely to produce a picture of electrolyte imbalance.

A 4 year-old girl presents to the clinic due to severe allergic rhinitis and recurrent asthma exacerbations. Which of the following leukotriene pathway inhibitors is indicated for use in this child? A Beclomethasone B Montelukast C Salmeterol D Zafirlukast E Zileuton

Montelukast Montelukast (B) is indicated for use in children 1 year of age or older, zafirlukast (D) is indicated for children > 5 years of age, and zileuton (E) is indicated for children > 12 years of age. Beclomethasone (A) is an inhaled corticosteroid and salmeterol (C) is an inhaled long-acting beta-2 agonist.

Hyponatremia evaluation

Most often, hyponatremia is due to excessive water retention rather than a true sodium deficiency. The first step in evaluating hyponatremia is to determine serum osmolality. Knowing whether the serum is isotonic (normal osmolality), hypotonic (low osmolality), or hypertonic (high osmolality) can help determine the etiology of the hyponatremia, and therefore, treatment. The most common causes of isotonic hyponatremia are hyperproteinemia and hyperlipidemia. The most common causes of hypertonic hyponatremia are hyperglycemia, presence of radiocontrast agents, and the presence of inactive metabolites, that is, mannitol, sorbitol, glycerol, and maltose. Treatment is aimed at correcting the underlying disorder. Most commonly, hyponatremia occurs in the setting of low osmolality (hypotonic). To further evaluate the etiology of the hyponatremia, it must be determined if the patient is hypovolemic, euvolemic, or hypervolemic. Hypovolemic hyponatremia is usually due either to extrarenal or intrarenal sodium losses. Extrarenal losses occur from dehydration, diarrhea, and vomiting. Urinary sodium measures <10 mEq/L (normal, >20 mEq/L), as the kidneys are avidly retaining sodium in an attempt to restore volume. Treatment is directed at restoring volume. Intrarenal sodium losses occur from the use of diuretics and ACE inhibitors, nephropathies, and mineralocorticoid deficiency. Urinary sodium measures >20 mEq/L. Treatment is directed at reversing the underlying cause. The most common causes of euvolemic hyponatremia are SIADH, postoperative hyponatremia, hypothyroidism, psychogenic polydipsia, and endurance exercise. In these cases, electrolyte-free water is retained, which results in a true physiologic hyponatremia. Treatment is directed at correcting the underlying abnormality and replacing sodium losses. Hypervolemic hyponatremia is caused by congestive heart failure, liver disease, nephrotic syndrome, and advanced CKD in general, anything that causes fluid retention. Treatment is directed at treating the underlying disease, restricting water intake, and facilitating excretion of water.

A 28-year-old woman who was born and brought up just outside of Washington, DC, comes in for evaluation of vague "problems with swallowing." She has no other symptoms except "my neck is bigger than it used to be." Examination reveals only a diffuse, somewhat irregular, nontender enlargement of the thyroid gland with distinct masses palpable within it. What is the most likely diagnosis? A endemic goiter B Graves disease C Hashimoto thyroiditis D multinodular goiter E thyroid carcinoma

Multinodular goiter Multinodular goiter is the most likely in a woman with these findings in the United States. It may be nontoxic as in this case or toxic, i.e., producing excessive thyroid hormones which cause symptoms of hyperthyroidism. Endemic goiter (A), which may present as a simple enlargement of the thyroid or as a multinodular one, is found almost entirely in iodine-deficient areas of the world and is extremely rare in the U.S. In Graves disease (B), the thyroid is enlarged and may exhibit a thrill and a bruit. In addition, the patient would have other signs of hyperthyroidism. The thyroid in Hashimoto thyroiditis (C) is diffusely enlarged and firm with fine nodules. A thyroid carcinoma (E) usually presents as a firm, nontender nodule in the gland.

A 70-year-old African American male was admitted to the hospital with complaints of a high grade fever, nausea, infection and severe back pain. Nearly two years prior while walking down his steps, he fell without incurring any apparent fractures. Since then he has been complaining of mild back pain, which worsened during cough. Lab reports reveal low red blood cell count but normal neutrophil and platelet count. The serum protein electrophoresis reveals the presence of paraprotein. Bone marrow biopsy reveals 50% infiltration by plasma cells. Based on this information, what would be your diagnosis? Answer Choices 1 multiple myeloma 2 Waldenström's macroglobulinemia 3 monoclonal gammopathy of unknown significance (MGUS) 4 primary amyloidosis 5 polyclonal hypergammaglobulinemia

Multiple Myeloma Explanation Multiple myeloma is a malignant disorder of the plasma cells. The disease occurs very frequently in people above 60 years of age. Males are more frequently affected than females. In the patients suffering from multiple myeloma, bone marrow biopsy shows heavy penetration with atypical plasma cells. Anemia is nearly universal in these patients. The hallmark of myeloma is the finding of a paraprotein on serum protein electrophoresis (SPEP). The majority of patients have a monoclonal spike in the beta or gamma globulin region. Overall, approximately 60% of myeloma patients have an IgG paraprotein, 25% have an IgA, and 15% have light chains only. Monoclonal gammopathy of unknown significance (MGUS) closely matches multiple myeloma in the immunoelectrophoresis patterns. Thus, if one considers all patients with paraproteins, MGUS is far more common than myeloma. However, myeloma is distinguished from MGUS by findings of replacement of the bone marrow, bone destruction and progression over time. Although the height of paraprotein spike alone is not used to distinguish benign from malignant conditions. In practice all the patients with IgG spikes greater than 3.5g/dL prove to have myeloma. In polyclonal hypergammaglobulinemia there is a polyclonal instead of a monoclonal spike.

A 45-year-old woman presents with a 1-month history of double vision, tripping, urinary incontinence, and restless legs; the symptoms are accompagnied by weakness, numbness, and tingling sensations. She denies any headaches, difficulty with speech, or memory impairment. Her physical examination does not demonstrate impairments in mental status, speech, or memory. Her muscular strength is 2+ bilaterally in the upper and lower extremities, although it is diminished from a previous exam 1 year ago. Deep tendon reflexes are also diminished compared to 1 year ago. What is the most likely diagnosis? Answer Choices 1 Multiple sclerosis 2 Vitamin E deficiency 3 Vitamin B12 deficiency 4 Friedreich's ataxia 5 Charcot-Marie-Tooth Disease

Multiple sclerosis Multiple sclerosis is correct because this patient demonstrates most of the distinguishing characteristics; spasticity (what she describes as restless legs), diplopia, unsteadiness, numbness, tingling, and urinary urgency/hesitancy (sphincter abnormalities). Vitamin E deficiency is incorrect because its main features are ataxia, slurred speech, sensory loss, and absent deep tendon reflexes. This patient doesn't report or exhibit speech impairment. Vitamin B12 deficiency is incorrect because its main features are polyneuropathy, mental status changes, and optic neuropathy. These patients do not exhibit or report mental status issues. Friederich's ataxia is incorrect because its main features are clumsy hands, ataxia, pes cavus, and sensory disturbances. Charcot-Marie-Tooth Disease is incorrect because its main features are polyneuropathy, distal weakness, impaired deep tendon reflexes, and sensory loss.

A 5-year-old boy presents with an altered gait. His mother reports that he first walked independently at 15 months of age, but that for the past 3 months his gait has become waddling and he has been taking a less active part in physical activities with his friends. He has even begun to fall on occasion. Past medical history is unremarkable. The mother recalled that a maternal uncle died in his late teens of an unknown condition. Physical examination reveals normal vital signs and growth parameters. Gait is waddling. Running is awkward, and hopping on either foot can only be performed with great difficulty. Strength of the deltoids and hip flexors was 3/5, with normal strength in the distal muscle groups. The calves appear enlarged bilaterally. He needs to climb up upon himself to arise from a sitting position on the floor. Question What test is most likely to yield a specific diagnosis? Answer Choices 1 Aldolase level 2 Creatinine kinase level 3 Electromyography 4 Muscle biopsy 5 Nerve conduction velocities

Muscle Biopsy Male gender, age of onset, gait alteration, proximal weakness, pseudohypertrophy of the calves, and positive Gower sign (needing to climb up upon himself), as well as the family history, strongly suggest Duchenne muscular dystrophy, which is an X-linked recessive condition caused by any of a number of mutations of the dystrophin gene. The condition is progressively debilitating. In some cases, it is associated with mental retardation and myocardial disease. Muscle biopsy will usually show muscle necrosis with replacement of muscle tissue by fat and fibrous tissue (contributing to the pseudohypertrophy of the calves). An elevated aldolase level is nonspecific for any number of muscle disorders. Similarly, an elevated creatinine kinase is also nonspecific for any number of muscle disorders. Electromyography will help to distinguish between neuropathy and myopathy, but it rarely provides an exact diagnosis. Nerve conduction velocities will help distinguish between neuropathy and myopathy, but they will not provide a specific diagnosis.

A 23-year-old woman presents with increasing fatigability and weakness. On further questioning, she reveals that, in the past month, she has experienced difficulty swallowing; there have also been episodes of double vision as the day progresses. Several of her professors have expressed concern because she looks sleepy during the day. A CT of the chest is advised. It reveals a mass that, once removed surgically, significantly lessens symptoms. The pathology slide made from the removed tissue can be found below. Based on the combined aspects of her history, findings upon investigation, and her recovery, what did the patient likely have? Answer Choices 1 Myasthenic syndrome 2 Myasthenia gravis 3 Amyotrophic lateral sclerosis 4 Botulism 5 Hyperthyroidism 6 Inclusion body myositis

Myasthenia Gravis Explanation Myasthenia gravis is an autoimmune disorder in which patients present with the cardinal symptoms of weakness and fatigue.This condition is more common among young women. Clinical features of this condition include: Ptosis and diplopia Chewing and swallowing difficulties Respiratory difficulties Asymmetrical proximal limb weakness with normal deep tendon reflexes The initial symptoms are often related to muscles that are controlled by cranial nerves, as these are affected first. Confirmation of this condition and the elimination of differential diagnosis can be done using the edrophonium test. The administration of edrophonium, a short-acting anticholinesterase, results in a marked increase in the muscle strength for a short duration in patients with myasthenia gravis. Investigations done in patients with myasthenia gravis are: The acetylcholine receptor antibody test CT scan of the thorax to detect a thymoma if present Electromyogram (EMG) The patient in this question was found to have a mediastinal mass, which was found to be a mixed thymoma. (See image) Treatment of myasthenia gravis involves: Neostigmine and other anticholinesterase drugs, improving symptoms. Avoidance of drugs such aminoglycosides. Thymectomy(removal of the thymus), which produces an improvement in over 85% of the patients between puberty and 55 years of age. It is usually performed in cases of moderate-to-marked generalized myasthenia gravis, and in some, it can produce complete remission of the disease. A thymoma is an indication for surgery. Steroids provide for symptom management as well; improvement is the most dramatic with high-dose corticosteroids. Plasmapheresis and IV immunoglobulins. Amyotrophic lateral sclerosis is a type of motor neuron disease. This is a mixed upper and lower motor neuron deficit, which is found in the limbs and is often associated with Parkinsonism and dementia. This condition does not present with ptosis, and it does not have the characteristic variation of muscle power seen in myasthenia gravis. Myasthenic syndrome, otherwise referred to as the Lambert-Eaton Syndrome, is the result of the defective release of acetylcholine in response to a nerve impulse. The resulting weakness is more prominent in the proximal muscles of the limbs. Muscle contraction strength increases with sustained contraction. Diagnosis of the disease is by electrophysiological studies. It is usually associated with small cell carcinoma of the lung. Treatment is aimed primarily at the tumor, but plasmapheresis and immunosuppression have been successful. Botulism is due to consumption of the toxin of Clostridium botulinum, which prevents the release of acetylcholine at the neuromuscular junctions and the autonomic synapses. This is an acute condition. Symptoms appear within 72 hours of ingestion, and they require immediate medical attention. Inclusion body myositis is a disorder of middle age. It results in weakness of the proximal muscles of the lower and then upper limbs. The diagnosis of this condition is also by means of a muscle biopsy, and it does not respond to corticosteroids. This patient does not show any of the other clinical signs of hyperthyroidism, such as weight loss, increased appetite, preference for cold weather, nervousness, increased bowel movements, an enlarged thyroid, etc.

A 65-year-old woman presents with general weakness. She had the weakness for the past few days, and it gets worse as the day progresses. She says she has been having double vision, and you notice she has poor posture. A blood test reveals presence of antibodies to acetylcholine receptor. A repetitive nerve stimulation test showed a decremental response. What is the most likely diagnosis? Answer Choices 1 Marfan's syndrome 2 Myasthenia gravis 3 Duchenne's muscular dystrophy 4 Rheumatoid arthritis 5 Systemic lupus erythematosus

Myasthenia Gravis Myasthenia gravis, a disorder more common in women, is characterized by chronic weakness of voluntary muscles. This weakness improves with rest and worsens with activity. The body produces an immune response (autoantibodies) against the nicotinic acetylcholine receptors at the neuromuscular junction. The most common presentation is muscle weakness that progressively worsens as the day progresses. The clinical features include ocular manifestations such as ptosis and diplopia, extra-ocular manifestations including limb and trunk weakness, dysphagia, generalized weakness, depressed gag reflex, limp body, slackening of the jaw, and respiratory depression. The diagnostic tests are anti-ACh receptor antibody assay, repetitive nerve stimulation (shows a decremental response), and electromyography. Treatment consists of the administration of acetylcholinesterase inhibitors, immunosuppressive agents like corticosteroids, cyclosporine, azathioprine, plasmapheresis, and intravenous gamma globulin. Thymectomy can also be done. Marfan syndrome occurs due to mutation in the fibrillin-1 gene, located on chromosome 15. The cardinal features occur in 3 systems--skeletal, ocular, and cardiovascular. Skeletal manifestations include an increase in height, disproportionately long limbs and digits, joint laxity, anterior chest deformity, vertebral column deformity such as scoliosis and thoracic lordosis, and high-arched palate with crowding of the teeth. Ocular manifestations include myopia and subluxation of the lenses (ectopia lentis). Cardiovascular features are mitral regurgitation, mitral valve prolapse, dilatation of the aortic root, and aortic regurgitation. Molecular studies of fibrillin-1 gene and imaging studies should be carried out in patients suspected of Marfan syndrome. Duchenne muscular dystrophy is an X-linked recessive disorder. It is characterized by delay in gross motor milestones, symmetrical proximal progressive muscle weakness, pseudohypertrophy of calf muscles, and lordotic posture. Positive Gower sign and waddling gait are diagnostic of Duchenne muscular dystrophy. Confirmation of diagnosis is by elevated levels of creatine kinase, DNA testing, and muscle biopsy. Rheumatoid arthritis is a chronic inflammatory systemic disease that primarily involves the articular surfaces and synovial membranes of multiple joints. It is characterized by the presence of rheumatoid nodules and rheumatoid factor. Assay for rheumatoid factor and imaging studies should be carried out in patients suspected of having rheumatoid arthritis. Systemic lupus erythematosus (SLE) is a chronic, inflammatory, multi-system disorder. SLE is an autoimmune disorder associated with the production of antinuclear antibodies (ANA). The manifestations include polyarthralgias, polyarthritis, malar rash, discoid lupus erythematosus, alopecia, aphthous stomatitis, anemia, and lupus nephritis. The clinical presentation and the investigations are indicative of myasthenia gravis and not Marfan syndrome, Duchenne muscular dystrophy, rheumatoid arthritis, or systemic lupus erythematosus.

A 22 year-old male presents with a 2-week history of an upper respiratory infection that hasn't improved after taking amoxicillin for 6 days. He notes persistent sore throat, intermittent fever, and a worsening nonproductive cough. Physical examination reveals bilateral diffuse crackles. What is the most likely causative agent of this patient's symptoms? A Streptococcal pneumoniae B Klebsiella pneumoniae C Mycoplasma pneumoniae D Pneumocystis jiroveci E Staphlococcal aureus

Mycoplasma pneumoniae C The patient presentation is consistent with atypical pneumonia most commonly caused by viruses, mycoplasma pneumonia (C), chlamydia pneumonia, or legionella. The presentation of streptococcal pneumoniae (A) is more acute with productive cough and fever being early symptoms. Klebsiella pneumoniae (B) also presents with fever and a cough classically noted to be productive of "currant jelly" sputum. Pneumocystis jiroveci (D) causes pneumonia of insidious onset in immunocompromised patients. Staphlococcal (E) pneumonia also presents more acutely with productive cough.

A 42-year-old woman works full-time as a data entry clerk and often puts in many hours of overtime. She is experiencing numbness and tingling in her right thumb, index finger, middle finger, and half of her ring finger. The numbness and tingling initially comes and goes; however, after a few months, it is constantly present. Question What could be considered a predisposing condition for the most likely diagnosis? Answer Choices 1 Myxedema 2 Hyperthyroidism 3 Hyperparathyroidism 4 Hypoglycemia 5 Cushing syndrome

Myxedema Explanation This patient has symptoms consistent with carpal tunnel syndrome. In carpal tunnel syndrome, the median nerve is compressed in the wrist. The distribution of her complaints indicates distribution of the median nerve. Myxedema is a predisposing condition for carpal tunnel syndrome. Myxedema is seen with hypothyroidism that causes mucopolysaccharides and fluid accumulation in the tissues. This leads to compression of the median nerve. None of the other choices are associated with an increased risk in carpal tunnel syndrome.

A 55-year-old man is admitted to the hospital with abdominal pain. He has a past medical history of hypertension. A CT scan with contrast reveals the presence of a small bowel obstruction. An NG tube is placed; IV narcotics are given for pain control; and IV fluids are started. A few days into his hospitalization, he develops confusion. Arterial blood gas is drawn, revealing the following: pH- 7.55 CO2- 42 HCO3- 28 Question What in the patient's history is contributing to his acid base disorder? Answer Choices 1 Anxiety 2 Hypertension 3 CT contrast administration 4 NG tube placement 5 IV narcotic administration

NGT Explanation NG tube placement is correct. The patient's ABG values indicate that he is suffering from metabolic alkalosis. Metabolic alkalosis is associated with elevated pH. The pH becomes elevated due to decreased hydrogen ion concentration or increased bicarbonate concentration. NG tube placement causes the loss of HCl from the stomach, and this is the most likely cause of metabolic alkalosis in this patient. Anxiety is incorrect. During anxiety attacks, patients may develop respiratory alkalosis due to hyperventilation. Carbon dioxide levels decrease during hyperventilation due to the patient's rapid breathing. Hypertension is incorrect. Hypertension itself is not associated with metabolic alkalosis. Certain medications, such as diuretics, may lead to metabolic alkalosis due to the associated fluid loss. However, there is no mention in the patient's history of diuretic use. IV narcotic administration is incorrect. The use of IV narcotics may cause CNS depression, which would lead to respiratory acidosis. Respiratory acidosis results from hypoventilation. CT contrast administration is incorrect. CT contrast administration may lead to renal failure in certain individuals. However, this would likely be associated with a metabolic acidosis and not metabolic alkalosis.

A 20-year-old female college student presents due to a 7-day history of daily heartburn. She has never experienced heartburn as bad as she does currently. She is treating this with over-the-counter histamine-2 receptor antagonist, famotidine, 1 tablet daily. The patient denies any other significant past medical history and is currently taking a daily multivitamin and an antihistamine for seasonal allergies. She recently admits to having increased episodes of headaches that she believes are due to stress. For this reason, she has been taking ibuprofen 600 mg every 8 hours. She states that she has been taking this consistently every 8 hours for the last 10 days; she hopes that after finals are finished, her headaches will subside. She also states that she was given amoxicillin 2 weeks ago for a middle ear infection, which resolved without any further intervention. She denies any difficulty swallowing, weight loss, night sweats, chest pain, black tarry stool, use of tobacco/alcohol, or coughing up blood. Physical examination is unremarkable for any abnormalities. Question Which of the following medications that the patient mentioned is most likely causing her increased gastrointestinal symptoms? Answer Choices 1 Antibiotic 2 H2-receptor antagonist 3 Multivitamin 4 H1-receptor antagonist 5 NSAID

NSAID Patients classified as having mild or intermittent symptoms of gastroesophageal reflux disease (GERD) typically are seen as not adversely affecting the patient's quality of life. Initial action includes modification of behaviors, such as eating smaller meals and eliminating acidic foods or known foods that precipitate the reflux (fatty foods, alcohol, chocolate, or peppermint). Weight loss has also been shown to help decrease issues, as well as advising patients to avoid lying down at least 3 hours after eating meals. Elevation of the head of the bed is also suggested to initiate relief. The next step in terms of treatment would be to discontinue any medications that may be increasing the symptoms of GERD. Medications that potentially irritate the esophagus and cause gastritis symptoms include: tetracycline, bisphosphonates, iron supplements, Non-Steroidal Anti-inflammatories (NSAIDs), and potassium supplements. Medications that increase acid reflux and worsen the condition of GERD include: anticholinergics, calcium channel blockers, narcotics, progesterone, quinidine, benzodiazepines, or even theophylline.

A 23-year-old woman is brought to the office due to her "heart racing". Her episodes have been intermittent over the past 2 weeks. She reports feeling tired, and she has not worked for 2 days. She says she had 'the flu' about a month ago and has not quite recovered. On examination, her pulse is 102/min, BP 110/80, and her palms feel sweaty and shaky. All other systems appear to be normal. Her lab investigations are given below: Creatinine: 0.9 mg/dl AST: 45 IU/dl ESR: 68 mm/hr Hemoblobin: 11.9 g/dl TSH: 0.01 mIU/mL T3: 5 mcg/dL T4: 160 mcg/dL Her radioiodine uptake scan shows diffusely reduced uptake throughout the gland. Question What is the best initial therapy? Answer Choices 1 Propylthioruacil 2 Radiolabeled Iodine ablation 3 High dose prednisone 4 NSAIDs 5 Levothyroxine

NSAIDS Explanation The clinical picture is suggestive of subacute thyroiditis, also called DeQuervain's thyroiditis. The etiology is unclear, but it is usually a result of viral infection with coxackie, mumps, and adenovirus, among others. NSAIDs are the drug of choice, with steroids being a backup drug for severe cases. The pathophysiology involves inflammation of the thyroid gland with release of colloid and preformed thyroid hormone. Histology shows infiltration of the gland with mononuclear cells and non-caseous granulomas. Due to destruction of the follicular cells, colloid and thyroid hormones are released into the general circulation, resulting in hyperthyroidism. The hyperthyroid phase lasts as long as the preformed colloid is release. A transient hypothyroid phase while the follicular cells regenerate commonly follows the depletion of colloid. The hallmark of the inflammation is pain, usually described by the patient as being vaguely in the front of the neck and presenting while swallowing, chewing, or radiating to the jaw. Hyperthyroid symptoms commonly manifest as palpitations, sweating, fatigue, weight loss, and diarrhea. Examination typically reveals tenderness over the thyroid gland, which may sometimes be severe. Lab investigations show an elevated ESR, which is evidence of inflammation. Thyroid hormone levels reflect the hyper or hypothyroid phase of the illness. Thyroglobulin levels may be elevated due to the release of colloid. Radio iodine uptake studies show diffusely reduced uptake due to the destruction of cells. This is in direct contrast to Grave's disease, where there is diffusely increased uptake. NSAIDs are usually sufficient for treatment. Rarely steroids are required for severe disease. Propylthiouracil is incorrect. The hyperthyroidism in thyroiditis is due to inflammation and destruction of cells with the release of preformed thyroid hormones, with an eventual return to normal. Administering anti-thyroid medication is unnecessary. Radioiodine ablation is incorrect, as subacute thyroiditis is a reversible condition, with thyroid function eventually coming back to normal. High dose prednisone is incorrect. It may be used in severe thyroiditis not responding to NSAIDs, but should be avoided as a first line medication due to the associated side effects. Levothyroixine is incorrect. This patient is hyperthyroid at present, and thyroxine would exacerbate the problem.

A 28-year-old man presents with rectal bleeding. The patient had noticed blood with bowel movements 3 times. The blood is described as bright red in color and small in amount. He also complains of rectal pain, especially with passing hard stools. He has tried some over-the-counter hemorrhoid creams without relief. The patient admits episodic constipation. He denies dark, tarry stools, easy bruising, and prior episodes of rectal bleeding. He has not noticed blood in his urine or with brushing his teeth. He denies nausea, vomiting, diarrhea, fevers, and weight loss. He has no known medical conditions. Family history is negative for gastrointestinal disorders. Social history reveals he is in a heterosexual relationship and denies anal intercourse. On physical exam, abdomen is normal. The anus has no visible protrusions or rash; however, there is a very small, erythematous and tender area that appears like a "paper cut" or crack in the skin. The patient experiences pain with digital rectal exam (DRE). No masses are noted in the rectal vault. Question What is the most appropriate prescription treatment for this patient's current condition? Answer Choices 1 Bacitracin ointment 2 Minoxidil topical 3 Nitroglycerin ointment 4 Nystatin topical 5 Tretinoin topical

NTG ointment

A 40-year-old woman presents with a 7-day history of pain in her right arm. The patient denies any trauma or injury to this extremity just prior to the pain starting, but she does admit to having a Colles' fracture in this arm about 2 months ago. She denies any injury to her back, neck, or other components of the musculoskeletal system prior to the event of pain. She describes the pain burning and throbbing with an extremely diffuse, uncomfortable ache accompanying it. She further states that this limb has become extremely sensitive to touch and cold; she adds that it appears somewhat more swollen than her left arm. The patient is very upset because she does not know why her arm is so painful when she has not done anything to it. She is a nonsmoker. She does not drink alcohol, and she exercises 3 times a week. Physical examination of the extremity reveals a slightly cyanotic hue and generalized pain of the entire right upper extremity. Pulses are strong (2+), but due to the pain, the patient states ROM is limited. Question Considering the most likely diagnosis, what is the initial pharmaceutical regimen that should be initiated? Answer Choices 1 Acetaminophen 500 mg 2 tablets BID 2 Acyclovir 800 mg TID 3 Naprosyn 500 mg 2 tablets BID 4 Cephalexin 500 mg BID

Naprosyn The correct response is Naprosyn 500 mg 2 tablets BID. The patient being described is most likely in stage I of complex regional pain syndrome (CRPS). This disorder is rare, causing instability of the vasomotor and autonomic components. CRPS is a disorder of a body region, most commonly the extremities, that is characterized by pain, swelling, limited range of motion, vasomotor instability, skin changes, and even patchy bone demineralization. Frequently symptoms will begin after the patient has sustained an injury, had surgery, or experienced a vascular event, such as a stroke. 35% of patients with CRPS report not having a distinct precipitating event. Stage 1 of CRPS characteristically has findings such as pain in the limb (burning/throbbing), diffuse, uncomfortable aching, sensitivity to touch or cold, or even localizededema. The distribution of pain is not compatible with a single peripheral nerve, trunk, or root lesion. There will also be signs of variable vasomotor disturbances that result in altered color and temperature. At this stage if a radiograph was completed, most likely it will be normal but could show signs of patchy demineralization. Early treatment gives the best prognosis for this patient. The accepted first line treatment would be NSAIDs (Naprosyn 250 - 500 mg 2 times daily). More severe cases that have significant edema (our patient above states there is swelling, but pulses are strong on PE) will be started on daily prednisone, 30 - 60 mg/day for a 4 week taper. Other options that may be considered for treatment include (but are not limited to) gabapentin, topical capsaicin, topical dimethylsulfoxide, IV bisphosphonates, IV immunoglobulin, and intrathecal baclofen. Acetaminophen, acyclovir, colchicine, and cephalexin are not indicated for treatment of CRPS.

A 22-year-old male presents with his girlfriend. She is concerned because of her boyfriend's behavior and she feels that he needs help. During the visit, you learn that the behaviors of concern include excessive demands for attention and grandiosity. He is also preoccupied with power and shows little interest in others. What is the most likely classification of this personality disorder? A Antisocial B Avoidant C Histrionic D Narcissistic E Schizoid

Narcissistic D This patient is exhibiting all of the characteristics of narcissistic personality disorder. Antisocial personality disorder is characterized by selfishness, callousness, promiscuousness, and impulsive behavior, and an inability to learn from experience and legal problems. Avoidant personality disorder presents clinically as someone who fears rejection, overreacts to rejection and failure, and has poor social endeavors and low self-esteem. Clinical findings of histrionic personality disorder include being dependent, immature, seductive, egocentric, vain, and emotionally labile. Schizoid personality disorder is characterized by shyness, introversion, being withdrawn, and avoiding close relationships.

A 27-year-old woman presents for the evaluation of hypersomnia. Her symptoms began approximately 5 years prior to presentation. The hypersomnia persisted despite the fact that she was sleeping 6 to 8 hours per night during the week and up to 10 hours on the weekends. She has a difficult time maintaining wakefulness while at work. It has been difficult developing a social life due to the fact that she often goes to bed early on weekends to catch up on her sleep. She often has vivid dreams and frequent hypnagogic hallucinations. She has sleep paralysis, which is long standing; it typically occurs 1 or 2 times a month. She has had episodes of cataplexy, with several episodes within a 1-week period. An episode would occur if she got excited or stressed. Her neck muscles would weaken, and she would have an irresistible urge to sleep. During these episodes, she was able to hear everything that was going on around her, but she could not respond. The episodes typically lasted a few minutes. She denies a history of head trauma or depression. She undergoes an overnight sleep study that demonstrates an apnea-hypopnea, and she slept for 7.4 hours. Question What is the most likely cause of the patient's symptoms? Answer Choices 1 Narcolepsy 2 Absence seizures 3 Simple partial seizures 4 Frontal lobe epilepsy 5 Delayed sleep-phase syndrome

Narcolepsy Narcolepsy is characterized by daytime sleepiness, cataplexy, hallucinations, and sleep paralysis. It is a chronic sleep disorder caused by the brain's inability to regulate sleep-wake cycles normally. Narcolepsy is thought to result from genetic predisposition, abnormal neurotransmitter functioning and sensitivity, and abnormal immune modulation. Human leukocyte antigen subtypes and abnormalities in monoamine synaptic transmission are thought to be involved. The cause of narcolepsy remains unknown. Men and women are affected equally, and children as young as 2 years old may also be affected. The age of onset distribution is biphasic; the highest peak occurs at the age of 15, and a smaller peak occurs at around age 36. Dysfunction and inappropriate regulation of rapid eye movement (REM) sleep are thought to cause narcolepsy. At various times throughout the day, people with narcolepsy feel strong urges to sleep. Patients may fall asleep anywhere from a few seconds to several minutes. In rare cases, some people may remain asleep for an hour or longer. Patients suffer from extreme daytime sleepiness, sudden loss of voluntary muscle tone (cataplexy), vivid hallucinations during sleep onset or upon awakening, and brief periods of total paralysis at the beginning or end of sleep. The condition may not be diagnosed until 10 to 15 years after the first symptoms appear. Absence seizures are brief episodes of staring. Awareness and responsiveness are impaired. Patients usually do not realize when they've had one. There is no warning before a seizure, and the person is alert immediately afterward. Simple partial seizures are seizures that usually affect the temporal lobes and/or hippocampi. Simple partial seizures are usually followed by larger seizures later on in life; the abnormal electrical activity spreads to a larger area of the brain, usually resulting in a complex partial seizure or a tonic-clonic seizure Frontal lobe epilepsy is either simple partial or complex partial, mostly with secondary generalization, and it is characterized by recurrent seizures. The clinical presentation ranges from postural, behavioral, tonic, to motor manifestations. Status epilepticus is usually associated with frontal lobe seizures. Delayed sleep-phase syndrome (DSPS), which is also known as delayed sleep-phase disorder or delayed sleep-phase type, is a circadian rhythm sleep disorder, a chronic disorder of the timing of sleep, peak period of alertness, hormonal, and other rhythms. People with DSPS tend to fall asleep well after midnight, and they also have difficulty waking up in the morning.

A 34-year-old man presents with a right-sided periorbital headache of 20 minutes duration. The pain is described as excruciating and bore-like, and it is associated with tearing and redness of the right eye, right eyelid swelling, and nasal congestion. The headaches have been occurring at multiple times nearly daily over the past 2 weeks, and each episode lasts approximately 30 to 60 minutes. The pain is not relieved with acetaminophen or ibuprofen. There is no nausea, vomiting, or photophobia. There is no history of amaurosis, visual scintillations, diplopia, focal weakness, numbness, neck stiffness, or other common neurologic complaints. The patient has had similar bouts of headaches over the past 5 years; the bouts typically last 3 to 4 weeks. The patient has an otherwise unremarkable medical history. General exam is remarkable for right eye conjunctival injection. The neurologic exam is significant for a mild right ptosis; the right pupil is 2 mm and reactive to light; the left pupil is 4 mm and reactive to light. There are no other neurologic findings. Question What is the most effective abortive therapy for this patient's headache? Answer Choices 1 Aspirin 2 Meperidine 3 Naproxen 4 Nasal oxygen 5 Oral sumatriptan

Nasal O2 This patient has cluster headaches (CH). CH is a relatively rare primary headache disorder characterized by recurrent periorbital or temporal headaches; they are associated with ipsilateral conjunctival injection, lacrimation, rhinorrhea, eyelid edema, and oculosympathetic signs (ptosis and miosis). The pain of CH is severe, typically characterized as "excruciating" in nature. CH occurs in cycles (the cluster period), usually lasting several weeks to months. The headaches are relatively brief in duration (15 to 180 minutes), but they can occur multiple times daily (up to 8) during the cluster period. Unlike other primary headaches, CH is more common in men. CH is classified as episodic or chronic, depending upon the duration and frequency of the cluster period. In episodic CH, the cluster period lasts from 7 days to 1 year; cycles are separated by pain-free remissions of at least 2 weeks. In chronic CH, the cluster period is longer than 1 year; remissions last less than 2 weeks. Abortive therapies for acute CH include nasal oxygen (5 to 10 L/minute), sumatriptan (subcutaneous), and dihydroergotamine (DHE, subcutaneous or intramuscular). Oral analgesics (acetaminophen, aspirin, ibuprofen, narcotics) are much less effective in aborting acute CH. A brief course of corticosteroids is often effective in shortening the CH cycle. Prophylactic agents in CH include lithium, methysergide, verapamil, and valproic acid.

A 4-year-old male has been experiencing a significant cough for the last 12 to 14 days, and initial episodes of coughing are characterized as frequent outbursts of 5 to 10 spastic coughs in a row. The patient does not report any fever, but does note that the coughing is worse at night. On examination, the patient is alert, awake, and oriented. His temperature is 97.7 0 F, pulse rate is 89, respiratory rate is 25, and blood pressure is 110/56. The HEENT is unremarkable, and lung sounds are clear to auscultation. You suspect that the patient may have an acute case of pertussis. Based on the history and physical exam findings, which is the test of choice for confirming a diagnosis of pertussis? A Complete blood count (CBC) B Throat culture C Chest x-ray D Nasopharyngeal culture E Sputum gram stain and culture

Nasopharyngeal culture A special medium culture plate (such as a Bordet-Gengou agar) is required for the nasopharyngeal swab for the diagnosis of pertussis. Throat culture, chest x-rays, and complete blood counts are helpful in ruling out other disease patterns.

Septic arthritis in adults younger than 30 years is usually caused by A Neisseria gonorrhea B Staphylococcus aureus C Pseudomonas aeruginosa D Streptococcus pyogenes E Salmonella species

Neisseria gonorrhea In patients younger than 30 years, gonococcus is the most common cause of septic arthritis. When all patients are considered, Staphylococcus aureus is the most common cause. Patients with prevalent joint disease and intravenous drug users are especially susceptible to Staphylococcus. Pseudomonas is also a common cause of septic arthritis in intravenous drug users. Salmonella is not a common cause of joint infection.

A 43-year-old Caucasian man with a 20-year history of bipolar disorder comes to your office for the first time. He presents with long term polyuria and polydipsia. He previously took lithium for mood stabilization for 15 years before initiating divalproex sodium therapy. He stopped using lithium because of the polyuria, but felt that the polyuria never fully subsided. His weight is stable, and he has no other urinary complaints. His blood pressure is 115/80 mm Hg and his physical exam is normal. His urinalysis shows no blood, cells, protein, glucose, nitrate, casts, or crystals. Question What is the most likely cause of his polyuria? Answer Choices 1 Central diabetes insipidus 2 Nephrogenic diabetes insipidus 3 Polyuria secondary to hyperglycemia 4 Polyuria following acute kidney injury 5 Polyuria secondary to polydipsia

Nephrogenic diabetes insipidus

A 43-year-old Caucasian man with a 20-year history of bipolar disorder comes to your office for the first time. He presents with long term polyuria and polydipsia. He previously took lithium for mood stabilization for 15 years before initiating divalproex sodium therapy. He stopped using lithium because of the polyuria, but felt that the polyuria never fully subsided. His weight is stable, and he has no other urinary complaints. His blood pressure is 115/80 mm Hg and his physical exam is normal. His urinalysis shows no blood, cells, protein, glucose, nitrate, casts, or crystals. Question What is the most likely cause of his polyuria? Answer Choices 1 Central diabetes insipidus 2 Nephrogenic diabetes insipidus 3 Polyuria secondary to hyperglycemia 4 Polyuria following acute kidney injury 5 Polyuria secondary to polydipsia

Nephrogenic diabetes insipidus This patient's symptoms of excessive production of urine (polyuria) is most likely caused by nephrogenic diabetes insipidus secondary to lithium use. Lithium impairs the distal water reabsorption in the collecting ducts, mediated by vasopressin (ADH), leading to the production of large quantities of dilute urine. Unfortunately, lithium use, even for as short a period as 1 year, can lead to irreversible damage of the renal tubules (via down-regulation and production of receptors and channels responsible for water reabsorption). Treatments for lithium-induced nephrogenic diabetes insipidus include A) amiloride, a distal-tubule acting diuretic which competes with lithium for access to ion channels and thus prevents the lithium-induced polyuria, and B) hydrochlorothiazide with a low-salt diet, in order to effectively decrease the quantity of urine produced. Depakote can be substituted for lithium,but as mentioned above, lithium may cause irreversible tubular damage. In central diabetes insipidus, decreased levels of ADH are produced by the posterior pituitary. Nephrogenic, not central diabetes insipidus is the expected complication of lithium use. ADH is usually released in response to increases in serum osmolality, and/or decreases in arterial volume. Non-osmotic causes such as nausea in post-opertive setting can also elicit ADH release. Head injury, granulomas, and other central nervous system abnormalities can lead to impaired ADH production and release. Central and nephrogenic diabetes are distinguished by a water deprivation test as described in the following table: Disorder Response to ADH Treatment Central DI >50% increase in urine osmoles Vasopressin, DDAVP, Decreased salt intake Nephrogenic DI <10% increase in urine osmol. Diuretics, Removal of offending medications, Decreased salt intake Primary polydipsia <10% increase in urine osmol. Fluid restriction Central diabetes insipidus is treated by the administration of vasopressin (DDAVP), intra-nasally, orally, or intravenously. The differential diagnosis for polyuria also includes the presence of excess serum solutes, ie: glucose and mannitol than can be reabsorbed by the kidney tubules. Hyperglycemia is an unexpected cause of this patient's polyuria since no glucose was found in this urinalysis. Following acute kidney injury (previously termed acute renal failure), damaged renal tubules may temporarily have difficulty concentrating and diluting urine. The benign urine (lack of blood, protein, cells and casts), makes this cause of polyuria unlikely. Polyuria secondary to polydipsia is intuitive. Unlike patients with diabetes insipidus, patients with primary polydipsia (also termed psychogenic polydipsia) will have decreased urine production when they decrease their fluid intake, will increase their plasma ADH levels, and concentrate their urine appropriately in response to decreased fluid intake. Primary polydipsic patients are also less prone to abnormalities of sodium. Note that polyuria is not equivalent to urinary urge, the sudden need to void, or to urinary frequency, voiding in the order of 7-15 times/day.

A 44-year-old diabetic woman presents with a 2-week history of lower extremity edema. She has no other symptoms. Physical examination of her lower extremities reveals bilateral edema with 2+ pitting. Urinalysis: Appearance Pale yellow and cloudy Glucose Negative Bilirubin Negative Ketones Negative Specific gravity 1.025 Blood Negative pH 7.0 Protein 4+ Urobilinogen Normal Nitrite Negative Leuk. esterase Negative Microscopic examination: RBCs 0 - 2 cells/hpf WBCs 0 - 2 cells/hpf Few squamous epithelial cells noted Moderate oval fat bodies noted Question What is the most likely diagnosis? Answer Choices 1 Cirrhosis 2 Cystitis 3 Pyelonephritis 4 Nephrotic syndrome 5 Nephritic syndrome

Nephrotic syndrome Explanation The clinical picture is suggestive of nephrotic syndrome. Peripheral edema is the hallmark of nephrotic syndrome; this occurs when serum albumin levels drop to below 3 g/dL. Other findings include positive urinary protein (large amounts); oval fat bodies may be seen on microscopic urine exam. Clinical findings associated with cirrhosis include symptoms of weakness, easily fatigability, disturbed sleep, muscle cramps, and weight loss. In 70% of cases the liver is palpable. Jaundice is not an early sign, but rather a later symptom occurring when the disease progresses. Urine findings include darkened urine color, positive bilirubin and variable urobilinogen, and negative protein. These findings are not present in this patient. Clinical findings in cystitis include irritative voiding symptoms, being afebrile, and positive urine cultures. On urinalysis, leukocyte esterase is usually present with positive or negative nitrites and a cloudy appearance; it may have a noxious smell. Microscopic exam may reveal WBCs and bacteria. These symptoms are not present in this patient. Findings with pyelonephritis include fever, flank pain, and symptoms similar to cystitis. These findings are not present in this patient. Nephritic syndrome can present with proteinuria, hematuria, azotemia, RBC casts, oliguria, and hypertension. These findings are not present in this patient.

A 45-year-old woman is constantly thirsty and consumes large amounts of water. She has a history of sarcoidosis. In addition, her urine output is in the range of 7 liters per day; she wakes up several times a night to urinate. Upon administration of aqueous vasopressin, her urine osmolarity rises above the value of her plasma osmolarity. Question What structure is most involved in the pathophysiology of the patient's condition? Answer Choices 1 Neurohypophysis 2 Adenohypophysis 3 Glomerulus 4 Loop of Henle 5 Collecting duct

Neurohypophysis Explanation This patient has signs and symptoms of diabetes insipidus. Central diabetes insipidus occurs when there is deficient antidiuretic hormone (ADH) secretion from the posterior pituitary (neurohypophysis). In central diabetes insipidus there will be a low ADH level. Nephrogenic diabetes insipidus occurs when the kidney fails to respond to ADH. The collecting duct is the portion of the kidney that is actually responsive to ADH In nephrogenic diabetes insipidus; ADH levels will be normal to elevated. In a patient with central diabetes insipidus, administration of aqueous vasopressin will result in an increase in urine osmolarity above plasma osmolarity. This will not be the cause in a patient with diabetes of nephrogenic origin. Central diabetes insipidus can be caused by granulomatous disease (e.g., sarcoidosis), head trauma, intracranial neoplasm, and some other conditions. The anterior lobe of the pituitary is the adenohypophysis. The anterior lobe of the pituitary secretes follicle-stimulating hormone (FSH), luteinizing hormone (LH), prolactin, growth hormone (GH), thyroid-stimulating hormone (TSH), and adrenocorticotropic hormone (ACTH) (and the corticotropin-related peptides). The glomerulus is a specialized capillary tuft. The endothelium of the glomerulus is fenestrated; the glomerulus is surrounded by Bowman's capsule. A renal corpuscle consists of the glomerulus and Bowman's capsule. A renal corpuscle is sometimes called a Malpighian corpuscle. Renal corpuscles are seen in the kidney cortex. The loop of Henle is part of the nephron. It is shaped like a hairpin; the proximal convoluted tubule leads into the descending limb of the loop of Henle. The ascending limb of the loop of Henle goes to the distal convoluted tubule.

A 4-year-old girl, adequately immunized for her age, has a history of repeated pyogenic infections. She had normal antibody responses following childhood immunizations. A defect in what cells best explains the cause of her infections? Answer Choices 1 B-lymphocytes 2 T lymphocytes 3 Eosinophils 4 Macrophages 5 Neutrophils 6 Basophils

Neutrophils Pyogenic infections are generally caused by bacteria. The entry of bacteria into the body triggers a number of responses: activation of the complement pathway and phagocytosis by neutrophils and macrophages. Macrophages play a major role in chronic granulomatous infections, and neutrophils predominate in acute pyogenic infections. An adequate response to childhood immunizations indicates an adequate capability of antibody production, and recovery from viral infections like measles indicates appropriate cell-mediated immunity. Therefore, the defect must lie in the neutrophils

A 4-year-old girl, adequately immunized for her age, has a history of repeated pyogenic infections. She had normal antibody responses following childhood immunizations. A defect in what cells best explains the cause of her infections? Answer Choices 1 B-lymphocytes 2 T lymphocytes 3 Eosinophils 4 Macrophages 5 Neutrophils 6 Basophils

Neutrophils Pyogenic infections are generally caused by bacteria. The entry of bacteria into the body triggers a number of responses: activation of the complement pathway and phagocytosis by neutrophils and macrophages. Macrophages play a major role in chronic granulomatous infections, and neutrophils predominate in acute pyogenic infections. An adequate response to childhood immunizations indicates an adequate capability of antibody production, and recovery from viral infections like measles indicates appropriate cell-mediated immunity. Therefore, the defect must lie in the neutrophils.

A 42-year-old Hispanic man presents because his employee health fair lab results returned with several 'out of normal' range results. He is an established patient in your practice; you have seen him 4 times for illness or minor injury over the past 10 years. He reports that he is in generally good health and feels well; he does not see any other healthcare providers. A summary of his past medical history includes: Medications: occasional over-the-counter ibuprofen for joint pain Allergies: none Surgical history: open reduction of left ankle at the age of 22 years Medical history: mild osteoarthritis Social history: patient denies the use of tobacco or illicit drugs. He drinks 3 - 4 beers per week. He is married and has 5 kids; he works in industrial hygiene at a lab facility. He plays rugby on the weekends as a hobby. Family history: no chronic diseases are known to the patient. His vital signs at check-in were all in normal ranges. This patient's laboratory results from the health fair are shown in the chart. Complete Metabolic Panel (CMP) Total Protein 6.3 6.4 - 8.2 g/dL AST (SGOT) 21 15 - 37 U/L ALT (SGPT) 17 5 - 43 U/L Alk Phosphatase 55 50 - 136 U/L Total Bilirubin 0.8 0.1 - 1.2 mg/dL Anion Gap 6 6 - 16 mmol/L eGFR >60 >60 mL/min/1.73m2 Lipid Panel Cholesterol 226 </=200 mg/dL Triglyceride 864 </=150 mg/dL HDL 42 40 - 59 mg/dL LDL Unable to calculate </=100 mg/dL Question Assuming you have counseled the patient on his condition and recommended lifestyle changes, what is the most appropriate prescription for him at this time? Answer Choices 1 Cholestyramine (Questran or Prevalite) 2 Clopidrigel (Plavix) 3 Ezetimibe (Zetia) 4 Lisinopril (Zestril or Prinivil) 5 Nicotinic acid (niacin)

Niacin Out of the listed options, nicotinic acid (niacin) is the most appropriate to quickly lower this patient's highly elevated triglycerides (TG). Niacin, which is also known as vitamin B3, is inexpensive, and it is available in multiple prescription and over-the-counter preparations. Additionally, a fibrate, such as gemfibrozil or fenofibrate, is considered another first-line treatment for significantly elevated TG. This patient has other minor abnormalities in his lab results, but they do not warrant immediate treatment. Cholestyramine is a bile acid sequestrant. It can be useful for decreasing elevated low-density lipoproteins (LDL), but it can actually increase TG. Clopidrigel reduces platelet activation and binding. It is indicated in acute coronary syndromes and for the prevention of thrombotic events, but it is not indicated for the treatment of elevated TG. This patient exhibits a mild thrombophilia, but with a negative clinical history for thrombotic risk, no such treatment is indicated. Ezetimibe inhibits cholesterol absorption and has a role in treating elevated LDL both alone and in combination with an HMG CoA reductase inhibitor (statin). This patient may have somewhat elevated total cholesterol and LDL (the calculation for LDL is not valid when the triglycerides are so dramatically elevated), but it is more important that his hypertriglyceridemia is treated initially. Ezetimibe is not recommended solely for hypertriglyceridemia; it can also cause pancreatitis, for which this patient is already at risk, as a side effect. Lisinopril is an angiotensin-converting enzyme (ACE) inhibitor. It is used primarily for hypertension, and it is also used in diabetics due to its beneficial effects on preserving renal function. It can cause hyperkalemia, but it should not be used for raising this patient's potassium. Even though it protects renal function, lisinopril can also raise the BUN and creatinine.

A 37-year-old man presents with diarrhea, nausea, vomiting, and a rash. He also complains of memory problems and depression. He states that he and his family live on a farm that produces corn. Due to recent financial problems, his diet has become more limited. He eats mainly corn products and only small amounts of fish and vegetables. He takes no vitamin supplementation, and he states that his symptoms began about 1 month ago. He began having trouble with his memory about 1 week ago. He has no significant past medical history, but he does admit to daily alcohol consumption. His physical examination is significant for stomatitis, glossitis, and a rash over his arms and legs. His mini mental status exam score is 24. Question What deficiency does this patient most likely have? Answer Choices 1 Niacin 2 Zinc 3 Folate 4 Vitamin B12 5 Vitamin B1

Niacin (B3) This patient probably has niacin deficiency. Niacin (nicotinamide, nicotinic acid) deficiency is uncommon in the United States. It is often found in people who live on a diet that consists mainly of corn (maize). This is due to the fact that the niacin in corn cannot be absorbed unless it is chemically treated with alkali first. If a person consumes a diet rich in tryptophan, but low in niacin, they are able to compensate since tryptophan can be converted into niacin. Deficiency may also result from alcoholism, cirrhosis, or diarrhea. Men and women are affected equally. Symptoms of niacin deficiency include nausea, vomiting, diarrhea, rash, glossitis, stomatitis, depression, and psychosis. Niacin deficiency, also known as pellagra, manifests as the '3 Ds': diarrhea, dermatitis, and dementia. Treatment consists of niacin supplementation. The causes of zinc deficiency include malnutrition, chronic debilitating diseases, chronic renal disease, alcoholism, drugs such as penicillamine and diuretic, and genetic disorders, such as sickle cell disease. Clinical manifestations in severe cases include alopecia, diarrhea, weight loss, infections, dermatitis, hypogonadism in men, and intercurrent infections. Supplementation with zinc is the treatment of choice. Folate deficiency causes megaloblastic macrocytic anemia, as folate plays a key role in nucleic acid synthesis. The early manifestation of folate deficiency, especially in its suboptimal state, predisposes to occlusive vascular disease and thrombosis. These manifestations are linked to increased homocysteine levels found in folate deficiency. Neurological disturbances, such as mood disturbance and spinal cord syndromes, are also seen. It is also associated with predisposition to neoplasia and interferes with immunologic status. Folate replacement is the option to prevent and to treat the deficiency. Vitamin B1 (or thiamine deficiency) causes beri beri, occurring mostly in the malnourished and alcoholics. The deficiency manifests with acute heart failure, neurologic deficits, and epilepsy. Empiric use of thiamine and prophylactic use in high-risk population is strongly recommended even before blood reports are obtained, as the treatment is inexpensive and prevents major catastrophes. Vitamin B12 (or cobalamin deficiency) manifests as megaloblastic macrocytic anemia, pancytopenia, and a spectrum of neuropsychiatric disorders such as peripheral neuropathy, parasthesias, and demyelination of corticospinal tract. Nutritional deficiency, alcoholism, and malabsorption syndromes are some causes of B12 deficiency. It is also associated with homocysteinemia and atherosclerosis. Diagnosis is by serum estimation of B12, and oral supplementation is safe and effective. Intramuscular injections may also be used.

Pellagra, known by many medical students to produce the "4 D's" (diarrhea, dermatitis, dementia, and death), is caused by what deficiency? Answer Choices 1 Vitamin E 2 Vitamin B6 3 Vitamin B1 4 Vitamin B12 5 Niacin

Niacin Deficiency has been linked to pellagra. This condition, now quite rare, consists of nausea with vomiting, diarrhea, dermatitis, and cognitive decline (such as irritability and memory loss). Vitamin B6 (pyridoxine) deficiency and excess has been associated with neurological disease. Both lead to a distal symmetrical polyneuropathy. A deficiency of B6 is commonly found in patients taking INH, an anti-tuberculosis drug. INH inhibits pyridoxine phosphorylation, thereby decreasing the concentration of the active compound-pyridoxal phosphate. Excessive use of B6 leads to an excess of pyridoxine, which competes with pyridoxal phosphate for binding sites on the enzyme. Vitamin B12 deficiency causes both a peripheral neuropathy and a degeneration in the spinal cord known as subacute combined degeneration. This affects the dorsal column sensory functions (position sense and vibration) and the lateral corticospinal tract. The resulting symptoms include paresthesias of the feet and hands with subsequent weakness and stiffness of the legs and a spastic gait. Vitamin B1 (Thiamine) deficiency has been linked to the neurological syndrome of Wernicke's encephalopathy. This condition, although often associated with alcoholism, is a true nutritional deficiency, causing neuropathological lesions in areas such as the thalamus, mammillary bodies, and the periaqueductal areas. A polyneuropathy has also been linked to Vitamin B1 deficiency, especially in alcoholism. This deficiency plays at least a role in the development of this disease, along with the possible neurotoxic role of alcohol. Vitamin E deficiency occurs usually with malabsorption of the fat-soluble vitamins A, D, E, and K. This can occur with conditions such as cholestatic liver disease or celiac disease. Neurological symptoms include a peripheral neuropathy and spinocerebellar degeneration. The spinocerebellar degeneration can occur as a defect in abetalipoproteins, as occurs in Bassen-Kornzweig syndrome. This is also associated with acanthocytes and retinitis pigmentosa.

A 75-year-old African-American man has a past medical history that is significant for severe and uncontrolled hypertension; he is brought into the emergency room by his family due to a 30-minute history of change of mental status. The patient had just climbed stairs when he first developed a headache that has become progressive; it is associated with nausea, non-bilious vomiting, and unilateral upper and lower extremity numbness. His physical exam reveals an alert and oriented times zero patient. He is afebrile, with a blood pressure of 185/108 mm Hg, and there is nuchal rigidity. His neurological exam notes contralateral sensory loss, contralateral hemiparesis, gaze paresis, homonymous hemianopia, and miosis. A CT scan without contrast was performed, and it is shown in the image. Question What medication would be most appropriate in this patient? Answer Choices 1 Heparin 2 Dobutamine 3 Nicardipine 4 Pyrimethamine 5 Methotrexate

Nicardipine The correct response is nicardipine. This patient is experiencing an intracerebral hemorrhage, with underlying hypertension as a major contributory etiologies. Antihypertensive agents reduce blood pressure to prevent exacerbation of intracerebral hemorrhage. Pressure should be cautiously lowered to a mean arterial pressure (MAP) less than 130 mm Hg. Excessive hypotension should be avoided. Early treatment in patients presenting with spontaneous intracerebral hemorrhage is important as it may decrease hematoma enlargement and lead to better neurologic outcome. Nicardipine is a calcium channel blocker with a potent rapid onset of action, ease of titration, and lack of toxic metabolites. It (as is Labetalol) is an agent that reduces blood pressure in order to prevent exacerbation of intracerebral hemorrhage. Patients with intracerebral bleeding should not be given blood-thinning agents. Instead, vitamin K and protamine may be used to restore normal coagulation parameters. Furthermore, recombinant factor VIIa (rFVIIa) within 4 hours after the onset of intracerebral hemorrhage limits the growth of the hematoma, reduces mortality, and improves functional outcomes at 90 days. Dobutamine should be avoided; it is a sympathomimetic agent. It has beta and alpha-1 adrenergic effects in the body, causing tachycardia and vasoconstriction. Pyrimethamine is the most effective agent in the treatment of toxoplasmosis, and it is included in most drug regimens. Methotrexate is the single most effective chemotherapeutic agent for primary central nervous system lymphoma.

A 74-year-old man presents after his wife witnessed him grab his head in pain and fall to the floor. He has not regained consciousness. His current blood pressure is 150/96 mm Hg, and his heart rate is 65 bpm. Emergent head CT shows a subarachnoid hemorrhage. Question In addition to life saving interventions, what prescription medication will most benefit this patient at this time? Answer Choices 1 Furosemide (loop diuretic) 2 Prednisone (glucocorticoid) 3 Tranexamic acid (antifibrinolytic agent) 4 Labetolol (beta blocker) 5 Nimodipine (calcium channel blocker)

Nimodipine (CCB) Nimodipine, a calcium channel blocker, has been shown to improve outcomes in patients following aneurysmal SAH. The mechanism of action is thought to be prevention of ischemia. Glucocorticoids (e.g., prednisone) are often utilized in patients with SAH because they offer symptomatic relief of headache and neck pain, but they have not been proved to decrease cerebral edema. Antifibrinolytic agents (e.g., Tranexamic acid) can be utilized in patients with a diagnosed aneurysm who cannot undergo directed treatment, but they are not routinely used following aneurysmal rupture. Labetalol (a beta blocker) may be utilized to treat elevated BP, but it must be used with caution because it can also decrease cerebral perfusion. Diuretics (e.g., furosemide) have no identified role in the treatment of aneurysmal SAH.

A 23-year-old man presents after being thrown from a motorcycle onto a cement median. He is awake and appears alert on exam. He believes that his helmet was knocked off at point of impact, but he cannot recall the specifics of his crash. A head CT scan is consistent with a cerebral contusion. What findings on a CT scan would you see with this diagnosis? Answer Choices 1 A peripheral ring of enhancement around areas of hematoma 2 Biconvex shaped hematoma over the cerebral hemisphere 3 No cortical disruption, but tissue necrosis, edema, and punctate hemorrhages 4 No cortical disruption or necrosis, but multiple small isolated hemorrhages throughout the cortex, with edema 5 Enlarged lateral ventricles, third and fourth ventricles, and basal cistern; normal or absent sulci

No cortical disruption or necrosis, but multiple small isolated hemorrhages throughout the cortex, with edema The correct response is no cortical disruption or necrosis, but multiple, small, isolated hemorrhages throughout the cortex, with edema. A cerebral contusion is a focal lesion to the brain, secondary to trauma. Contusions usually occur over the crest of gyri, largest at the surface of the brain and tapering in size into the white matter. Cerebral contusions can occur under the site of impact, or on the contralateral side of the brain (a contrecoup lesion). The CT scan of a cerebral contusion characteristically reveals punctate hemorrhages and edema under the site of impact. A cerebral contusion is defined as "bruising" of the brain without interruption of the cortex. Frequently, a subarachnoid hemorrhage overlies the area of contusion. A biconvex-shaped hematoma over the cerebral hemisphere is indicative of epidural hematoma. A peripheral ring of enhancement around areas of a hematoma is seen 1 to 6 weeks after a traumatic intracerebral hematoma, not in cerebral contusion. Panventricular enlargement is a feature of hydrocephalus.

A 51-year-old man presents with a 3-day history of severe pain associated with bowel movements. The pain lasts up to 1 hour following each bowel movement. His stools are described as "hard", but they retain their normal brown color, with the occasional presence of a few drops of bright red blood. His dietary history is remarkable for a low-fiber diet lacking in raw fruits and vegetables. He denies any sexual contact, drug use, or other gastrointestinal symptoms. He takes ferrous sulfate for anemia and oxycodone for chronic joint pain. His rectal exam reveals the following finding in the attached image. Question What is the most appropriate diagnostic test to order at this time? Answer Choices 1 CT scan of the abdomen and pelvis 2 No diagnostic tests are necessary 3 HIV ELISA test 4 Anoscopy 5 Rectal biopsy

No diagnostic test needed This patient's diagnosis is a posterior anal fissure. Along with a history, the diagnosis can usually be made based on findings from a gentle perianal examination with inspection of the anal mucosa. In this case, no diagnostic procedures are required. A digital rectal examination is painful and often can be deferred. Anoscopy may be required for lesions that are not well-visualized; however, this is not well tolerated by a patient with an acute anal fissure. In this case, anoscopy can often be deferred; the patient can be treated based only on the symptoms. If an ordinary anal fissure is suspected, and if it is located in the posterior or anterior midline, then no laboratory or imaging tests are necessary. If the fissure is off the midline or irregular, or if an underlying illness (e.g., Crohn disease, squamous cell cancer, AIDS) may be present, then the appropriate tests should be ordered. These tests include erythrocyte sedimentation rate, stool and viral cultures, human immunodeficiency virus (HIV) testing, and biopsy of the lesion/fissure.

A 42-year-old African American male is admitted to the hospital with heme positive urine and anemia. He recently completed a course of trimethoprim/sulfamethoxazole for a urinary tract infection. Treatment for his condition should include which of the following? A Begin vancomycin 1 GM IV q 12 for untreated infection B No treatment C Restart bactrim IV for an undertreated UTI D Transfuse packed red cells E Transfuse platelets

No treatment No treatment is necessary, except to avoid known oxidant drugs. The patient is not exhibiting signs of an untreated infection, so A is wrong. The anemia is transient so no transfusions are needed.

What is the most common cause of treatment failure in tuberculosis? A Drug resistance B Noncompliance to therapy C Inappropriate selection of medication D Sepsis

Noncompliance to therapy The usual reason for failure is simply due to the patient not continuning their treatment plan, regardless of the severity of the disease. Drug resistence, while present in some cases does not preclude the patient from treatment failure. Ongoing sepsis is not a reason to have treatment failure.

A 30-year-old woman presents with episodic headache, palpitations, and sweating. She has had multiple episodes, each lasting approximately 20 minutes, over the last few weeks. Her primary care physician noted hypertension on her last several visits, and her blood pressure today is 210/98 mm Hg. She has no past medical history. Question The production of what hormone is associated with her condition? Answer Choices 1 Growth hormone 2 Gastrin 3 Norepinephrine 4 Insulin 5 Cortisol

Norepinephrine Explanation Norepinephrine is the correct response. The patient's history of headache, palpitations, diaphoresis, and hypertension suggests pheochromocytoma. Pheochromocytoma is a tumor that originates in the adrenal glands. Pheochromocytomas produce catecholamines, such as norepinephrine, epinephrine, and dopamine, causing its symptoms. Growth hormone is an incorrect response. Growth hormone may be secreted by pituitary tumors, leading to acromegaly. Acromegaly results in abnormal growth of soft tissue and the skeleton. Patients present with increase in hand and foot size, coarse facial features, and a prominent brow. Gastrin is an incorrect response. Gastrin-producing tumors are associated with Zollinger-Ellison syndrome. Zollinger-Ellison syndrome causes abdominal pain, diarrhea, and ulcers. Insulin is an incorrect response. Rare pancreatic tumors called insulinomas secrete insulin. Insulinomas are associated with symptoms of hypoglycemia. Cortisol is an incorrect response. Cortisol may be overproduced due to pituitary adenomas, resulting in Cushing syndrome. Cushing syndrome is associated with hypertension; patients also present with round face, central obesity, muscle wasting, striae, and muscle wasting.

While performing a Doppler ultrasound on a woman at 30 weeks gestation, you notice that the fetal heart rate is consistently within the range of 130 - 140 beats/minute. What can you conclude about the heart rate of the fetus? Answer Choices 1 Structural heart disease 2 Normal heart rate 3 Asphyxia 4 Bradycardia 5 Tachycardia

Normal HR Explanation The fetus has a normal fetal heart rate, as it is within the range of 110 - 160 beats/minute. The diagnosis of structural heart disease cannot be established by a fetal heart rate. The fetus has neither tachycardia (an early sign of fetal asphyxia), nor bradycardia (a late sign of asphyxia). Fetal bradycardia is defined as a decrease in the baseline fetal heart rate to <100 beats per minute. Suspicious tachycardia is defined as the heart rate being between 150 and 170; a pathological pattern is above 170 beats/minute.

A 5 year-old girl presents to the clinic with her mother who is concerned about her daughter's energy level since starting full day kindergarten. CBC reveals a hemoglobin of 12.3 g/dl, hematocrit of 36%, MCV of 62 fL, and an MCHC of 34 g/dL. Serum ferritin levels are normal. What is the most likely hemoglobin electrophoresis results for this patient? A Decreased HbA2 and increased HbF levels B Decreased HbA2 and HbF levels C Increased HbA2 and normal HbF levels D Increased HbA2 and HbF levels E Normal HbA2 and HbF levels

Normal HbA2 and HbF levels Alpha thalassemia presents with a normal hemoglobin electrophoresis, since all adult hemoglobin are alpha-containing and affected equally. Beta thalassemia major may present with increased HbA 2 and/or HbF levels (C and D). Beta thalassemia minor typically presents with increased HbA 2 (C).

A 28-year-old man with no significant past medical history is rushed to the local trauma center following a stab wound to his chest. Paramedics report that there was significant blood loss. The patient has lost consciousness, is oliguric, and extremities are cool and moist to touch. His physical exam is also remarkable for tachycardia, tachypnea, a depressed systolic pressure, an immeasurable diastolic blood pressure. Question What is the preferred initial pharmacologic agent of choice for this patient? Answer Choices 1 Epinephrine 2 Hypertonic saline and dextran 3 Normal saline 4 Somatostatin 5 Vasopressin

Normal Saline Explanation This patient is presenting with signs and symptoms consistent with hypovolemic shock due to acute traumatic blood loss. 3 goals exist in the emergency department treatment of the patient with hypovolemic shock as follows: (1) maximize oxygen delivery - completed by ensuring adequacy of ventilation, increasing oxygen saturation of the blood, and restoring blood flow, (2) control further blood loss, and (3) fluid resuscitation. Current recommendations are for aggressive fluid resuscitation with lactated Ringer solution or normal saline in all patients with signs and symptoms of shock, regardless of underlying cause. If a patient is moribund and markedly hypotensive (class IV shock), both crystalloid and type O blood should be started initially. These guidelines for crystalloid and blood infusion are not rules; therapy should be based on the condition of the patient. Epinephrine is indicated as initial resuscitation management in cardiac arrest, anaphylactic shock, symptomatic bradycardia, and hypotension refractory to volume replacement. Epinephrine is associated with a host of adverse effects such as induction of pulmonary hypertension, tachyarrhythmia, myocardial ischemia, lactic acidosis, hyperglycemia and compromise hepatosplanchnic perfusion, oxygen exchange, and lactate clearance. The combination of hypertonic saline and dextran also has been studied because of previous evidence that it may improve cardiac contractility and circulation. Studies in the US and Japan have failed to show any difference when this combination was compared with isotonic sodium chloride solution or lactated Ringer solution. Somatostatin and octreotide infusions have been shown to reduce gastrointestinal bleeding from varices and peptic ulcer disease. These agents possess the advantages of vasopressin without the significant side effects. In the patient with GI bleeding, intravenous vasopressin and H2 blockers have been used. Vasopressin commonly is associated with adverse reactions, such as hypertension, arrhythmias, gangrene, and myocardial or splanchnic ischemia. Therefore, it should be considered secondary to more definitive measures. H2 blockers are relatively safe but have no proven benefit.

A 66-year-old man presents with vomiting. He has lung cancer, but he has declined treatment. His father died of lung cancer at the age of 65. On examination, he is confused and dehydrated with a BP of 100/50 mm Hg and PR of 100 beats/minute. Laboratory analysis reveals a calcium level of 16 mg/dL. His renal function tests are normal. Question What is the most appropriate initial therapy in the management of this patient? Answer Choices 1 Normal saline 2 Prednisone 3 Calcitonin 4 Hemodialysis 5 Plicamycin

Normal Saline The clinical presentation is suggestive of a hypercalcemia of malignancy. It is most frequently associated with multiple myeloma, breast, lung, and kidney cancers, as well as T-cell lymphomas. The hypercalcemia is usually a result of increased bone resorption by osteoclasts. Patients present with anorexia, nausea, vomiting, constipation, fatigue, and headache. When the plasma calcium level rises above 12 mg/dL, they may also present with confusion, delirium, and coma. Laboratory results may also reveal an elevated blood urea nitrogen and creatinine levels. The 1st step in the management of severe hypercalcemia of malignancy, when the renal function is normal, is aggressive rehydration with intravenous normal saline while closely monitoring urine output and electrolytes. The bisphosphonate pamidronate, which acts as a bone resorption inhibitor, can then be given intravenously in addition to the normal saline. Calcitonin, another bone resorption inhibitor, is also used to manage severe hypercalcemia of malignancy when it is combined with prednisone; however, some patients are unresponsive to this combination. Hemodialysis, along with a calcium-free dialysate and peritoneal dialysis, is also used to lower the calcium levels (especially in patients with renal failure). Plicamycin, which inhibits bone resorption, is also effective in lowering plasma calcium; however, due to its potentially severe adverse effects, it should be given if there is insufficient response to the other medications.

A 3-year-old boy presents for his annual well child check. He and his family moved to the United States from Africa 4 months ago. He is a thin boy; he is in no acute distress. His heart rate and rhythm are regular; his lungs are clear, and he has no hepatosplenomegaly. Due to his recent immigration, he is given a tuberculin skin test. He has no known chronic medical conditions and no known contacts with tuberculosis disease. The test shows an induration of 6 mm. Question What is the next step in the management of this patient? Answer Choices 1 Chest X-ray and isoniazid 2 Isoniazid, rifampin, pyrazinamide, and ethambutol 3 Hospitalization for further workup 4 Normal well child care 5 Gastric aspirate sampling

Normal child well care visit For a child who has immigrated from a high-prevalence area of the world but has no close contacts with tuberculosis disease, is not immunocompromised, and has no signs or symptoms of the disease, the induration must be 10 mm or greater to be positive. Therefore, this child has a negative test and should receive normal well child care. If the test were positive, he would require a chest X-ray and further workup. If he were found to have latent infection without active disease, he would be treated with isoniazid alone. If he were found to have a pulmonary or extrapulmonary disease, except meningitis, he would be treated with isoniazid, rifampin, pyrazinamide, and ethambutol. Gastric aspirate sampling is one way of isolating the organism if a patient has active tuberculosis disease.

A 2-month-old infant who is breastfeeding presents with low hemoglobin levels. The infant was born at home and the mother received no prenatal care; she did not, and does not, take any medications. Family history is unremarkable. On examination, the infant appears healthy. Question What is the underlying mechanism of low hemoglobin levels in this infant? Answer Choices 1 Folate deficiency 2 Vitamin K deficiency 3 Breastfeeding 4 Infection 5 Normal process

Normal process st 2 - 3 months of life is the signal to the infant's body to increase erythrocyte production. Called physiologic anemia of infancy, this is considered normal and no treatment is needed. Folate deficiency in the mother would more likely result in low birth weight, prematurity, and/or neural tube defects. Vitamin K deficiency is the cause of hemorrhagic diathesis; its manifestations would be obvious. Breastfeeding when the mother did not, and does not, take any drugs cannot be the cause of anemia in a 2-month-old infant. Common infections in a 2-month-old infant include the digestive tract, ear, skin, and upper respiratory tract; all will have relevant symptoms.

A 35-year-old woman just found out she is pregnant. She is experiencing polyuria, but she denies dysuria and incontinence. Her urinalysis is unremarkable. Her fetal ultrasound is normal, and her renal ultrasound shows normal physiological hydronephrosis of pregnancy. Her pre-pregnancy weight was 155 lbs, and she is 5 feet tall. Her calculated body mass index (BMI) is 30.3 kg/m2. She takes no medications. She smokes ½ pack of cigarettes/day. Question Assuming that the patient has a normal vaginal delivery and no episiotomy, how can you best prevent urinary incontinence post partum? Answer Choices 1 Normalize weight 2 Avoid episiotomy 3 Vaginal childbirth 4 Prescribe Duloxetine 5 Prescribe oxybutynin

Normalize weight Explanation This patient will benefit from lifestyle modification to normalize her body weight post-pregnancy. Dietary modifications, exercise, breastfeeding, and nutritional counseling may be helpful. She was overweight pre-pregnancy, with a BMI > 30 kg/m2. Obesity is a known risk factor for incontinence. It is also a risk factor for insulin resistance and diabetes, which may also contribute to various forms of incontinence. None of the other options listed below will help prevent incontinence; in fact, they may be risks for incontinence. Incontinence is common in pregnancy. Fetal compression of the bladder plus large volumes of urine due to suggested volume intake and increased glomerular filtration rates may contribute to this. Postpartum, vaginal birth, and changes in the laxity/strength of the pelvic floor may contribute to stress incontinence. Stress incontinence is characterized by the involuntary leaking of urine during stress or increases in abdominal and bladder pressure, such as coughing and sneezing. Bladder pressure at these times exceeds urethral pressure, allowing urine to leak through the urethra. Treatments for stress incontinence include pelvic floor exercises. By repeatedly contracting and relaxing the vagina and pelvic floor, leaking may decrease. In this patient's case, treating her cough with cough suppressants may additionally help with the urine leaks. In obese patients, 5 - 10% weight loss may also improve symptoms. Pessaries may be inserted into the vagina to increase urethral support. Urethral support can also be increased surgically by inserting a fascial sling or vaginal tape to support the urethra. Smoking cessation is laudable on many accounts. It may contribute to low birth weight in the baby and is risk factor for a variety of cardiovascular diseases in the mother. It is not currently considered a risk factor for incontinence. Episiotomy may be a risk factor for fecal incontinence, but it is not a known risk for urinary incontinence. Oxybutynin is an anticholinergic amine used in the treatment of neurogenic bladder and overactive bladder/urge incontinence. Reports of its use during pregnancy and lactation are not available (Micromedex). Duloxetine is used in the management of stress incontinence. It is a reuptake inhibitor of serotonin and noradrenaline. Its use is not suggested in pregnancy because it is category C and may have teratogenic effects.

Anemia of CKD and tx

Normocytic normochromic anemia caused by low epo production: • Tx erythropoetic agents like Epogen or Aranesp • Target Hgb 10-11.5 (normal is 13 g/dL) • Make sure the patient is not iron deficient anemic aswell because Epogen will not work if IDA is not treated

A 35-year-old woman presents with watery diarrhea. It began 1 day after eating at a party. Her 6-year-old daughter ate the same food and is vomiting, but she does not have diarrhea. On examination, the woman is not dehydrated or febrile. A stool examination is negative for leukocytes and trophozoites. Question What is the causative organism? Answer Choices 1 Entamoeba histolytica 2 Enterotoxigenic Escherichia coli 3 Shigella dysenteriae 4 Bacillus cereus 5 Norwalk virus

Norwalk virus Norwalk virus can cause water and food borne outbreaks. The incubation period is 1 to 3 days. Patients present with watery diarrhea and vomiting. They may also report abdominal and muscle pains. In adults, diarrhea is usually more prominent than vomiting. In children, the reverse is true. On examination, they may be febrile. The stool examination is negative for leukocytes, ova, and trophozoites. The mainstay of therapy is hydration, as symptoms usually resolve within 2 days. Entamoeba histolytica colitis develops 2 to 6 weeks after ingesting contaminated water or food. Patients usually present with bloody and mucoid diarrhea. Stool examination reveals hematophagous trophozoites and cysts. Patients with Enterotoxigenic Escherichia coli food poisoning present with watery diarrhea 12 to 72 hours after ingesting contaminated water or raw fruit salads. Examination of the stool reveals no fecal leukocytes or trophozoites. Patients with Shigella dysenteriae food poisoning present with bloody diarrhea; symptoms usually start 16 hours after ingesting contaminated foods like potato salad. A stool examination reveals polymorphonuclear leukocytes. In emetic Bacillus cereus food poisoning, patients develop vomiting 1 to 6 hours after ingesting contaminated food like reheated fried rice. In the diarrheal form, they develop watery diarrhea 8 to 16 hours after ingestion. There are no fecal leukocytesor trophozoites.

A 23-year-old G1P0 woman presents for her first prenatal visit. The first day of her last menstrual period (LMP) was February 23. Generally, she feels well and has no complaints. The patient has started a prenatal vitamin, is getting the appropriate amount of sleep, and is keeping up with a moderate exercise routine. A complete physical exam is performed, and there are no abnormal findings. Initial ultrasound documents fetal cardiac activity. Question Using Nägele's rule, what is the patient's estimated date of confinement (EDC)? Answer Choices 1 November 16 2 November 30 3 November 23 4 September 23 5 September 30

November 30 Explanation Nägele's rule is a mathematical way of calculating a pregnant patient's EDC using the first date of her LMP: EDC = LMP - 3 months + 7 days. Therefore, using this equation, this patient's EDC is November 30.

A 33-year-old G1P0 presents for evaluation of her inability to conceive a pregnancy for six months. She menstruates monthly. Her past medical history is significant for PID x 2, for which she was hospitalized for IV antibiotics. Her Chlamydia, GC, and pap smear are normal. When should a more comprehensive evaluation for her infertility begin? A Now B 6 months C 12 months D 18 months E 24 months

Now A A comprehensive work up should begin now, due to her advancing age and history of significant PID, which may require surgical treatment.

A 33-year-old G1P0 presents for evaluation of her inability to conceive a pregnancy for six months. She menstruates monthly. Her past medical history is significant for PID x 2, for which she was hospitalized for IV antibiotics. Her Chlamydia, GC, and pap smear are normal. When should a more comprehensive evaluation for her infertility begin? A Now B 6 months C 12 months D 18 months E 24 months

Now A comprehensive work up should begin now, due to her advancing age and history of significant PID, which may require surgical treatment.

A 51-year-old female presents to her primary care provider for her annual physical. She is a healthy white female and a non-smoker. She has mild HTN, but an otherwise negative health Hx. In addition, FHx is negative and ROS is negative. Her LMP was 6 months ago. Her last mammogram was 3 years ago. When should she have her next mammogram? A In 2 years B In 1 year C Now D Only if self breast exam reveals abnormality E Only if provider breast exam reveals abnormality

Now C The preponderance of data strongly supports the benefits of a screening mammography. New analyses of older randomized studies have suggested that screening may not work. While the design defects in some older studies cannot be retrospectively corrected, most experts, including panels of the American Society of Clinical Oncology and the American Cancer Society, continue to believe that screening conveys substantial benefit.

A 26 year-old man presents to the clinic after developing a severe productive cough, fever, chills, night sweats and weight loss. Which is the most definitive diagnostic test for determining the presence of Mycobacterium tuberculosis as the causative agent of his active pulmonary tuberculosis and yields results in less than 24 hours? A Acid-fast bacilli smear of sputum B Blood culture C Mycobacterial culture of sputum D Nucleic acid amplification of sputum E Tuberculin skin test

Nucleic acid amplification of sputum The most definitive diagnostic tests for confirmation of tuberculosis are mycobacterial culture (C) which requires at least 2 weeks to grow mycobacterium, and nucleic acid amplification (D) of a respiratory specimen that can be completed in a matter of hours. AFB smear (A) has a sensitivity of only 40-60%, but positive results help establish a presumptive diagnosis. Tuberculin skin testing (D) is effective in screening patients for latent TB. Routine blood cultures (B) are ineffective in diagnosing TB.

A 33-year-old woman presents seeking advice. She is concerned regarding the appearance of spider angiomas that are present on her trunk and face. She states that spider angiomas appeared during pregnancy and that they persist now, 9 months after delivery. Before pregnancy, she was treated with imipramine because of depression and she is currently still taking this drug. She denies alcohol intake, however she tells you that 3 years ago, she had acute infection with hepatitis C virus. She was also frequently treated with flucloxacillin during the past couple of years for recurrent respiratory tract infections. In addition, she started taking oral contraceptives after delivery. The remainder of her personal history is unremarkable. Physical examination reveals the presence of multiple spider angiomas on the patient's face, forearms, and back. The remainder of the patient's general physical findings are unremarkable. Routine laboratory analyses reveal normal AST (26 IU/l, reference values 8 to 27 IU/l) and ALT (22 IU/l, reference value 8 to 23 IU/l) levels. Alkaline phosphatase level is normal (43 IU/l, reference value 23-71 IU/l) and other routine laboratory analyses reveal no abnormalities. HbSAg, HbeAg, and antiHbc antibodies are not present in patient's serum. IgG antibodies to hepatitis C are present, but testing for hepatitis C virus (HCV) does not reveal the presence of HCV RNA in patient's serum. Question What is the most likely cause of the patient's spider angiomas? Answer Choices 1 Flucloxacillin-induced hepatitis 2 Imipramine-induced hepatitis 3 Oral contraceptives 4 Chronic hepatitis caused by hepatitis C virus 5 Pregnancy-related appearance

OCP The most likely cause of this patient's spider angiomas are Oral contraceptives. In conditions when estrogen hormones are present in excess such as during pregnancy, contraceptive intake, or liver disease (due to decreased degradation of estrogens), the surplus of estrogens cause the appearance of spider nevi. In pregnant women, spider angiomas appear from the second to fifth month of pregnancy and disappear in a short period of time (within 2 months) after the delivery. Since in the presented case, spider angiomas are present for a longer period of time, it is unlikely that they are pregnancy-related. In untreated patients with acute Hepatitis C, disease evolves towards chronicity in about 80% of patients, while only about 20% of patients recover completely. Patients who have chronic hepatitis C also have HCV RNA in their serum, but those who have recovered completely have no HCV RNA in serum and have IgG antibodies against hepatitis C virus as the marker of past hepatitis C infection. As the presented patient satisfies criteria for the patient who had hepatitis C in the past, her spider angiomas are not related to chronic hepatitis C infection. Imipramine seldom causes liver damage. In addition, hepatitis caused by imipramine administration is of a cholestatic type, i.e. it is associated with increased serum levels of transaminases and alkaline phosphatase. Since the alkaline phosphatase level in the presented patient is normal, imipramine-induced hepatitis is not likely. Similar to imipramine, hepatitis caused by flucloxacillin administration is also of a cholestatic type. For the reasons presented above, the possibility of flucloxacillin-induced hepatitis is also unlikely.

A 23-year-old female presents with a history of rigid thought patterns and a need for control. She sees herself as a perfectionist. She discloses that she feels a need to check the locks on her doors at home once every 30 minutes. She is consumed with these thoughts about locking the doors. What is the most likely classification for her personality disorder? A Histrionic B Narcissistic C Obsessive-compulsive D Paranoid E Schizotypal

Obsessive-compulsive C Clinical findings of obsessive-compulsive disorder include being a perfectionist, egocentric, and indecisive, with rigid thought patterns and need for control. Clinical findings of histrionic personality disorder include being dependent, immature, seductive, egocentric, vain, and emotionally labile. Narcissistic personality disorder presents with the clinical findings of grandiosity, a preoccupation with power, lacking interest in others, and excessive demands for attention. Clinical findings of someone who has paranoid personality disorder would include defensiveness, being overly sensitive, secretive, suspicious, hyper-alert, and with a limited emotional response. Schizotypal clinical findings include being superstitious, socially isolated, and suspicious, and having limited personality ability, odd speech, and eccentric behaviors.

A 45-year-old woman presents with yellow discoloration of the skin, clay-colored stool, and dark urine. She is obese, but otherwise healthy. She has long-standing digestive troubles and an intolerance to fatty meals for which she takes antispasmodic medications. On examination, there is mild tenderness in the upper right abdominal quadrant; serum bilirubin shows elevated total and conjugated types, as well as normal unconjugated bilirubin. Question What is the most likely cause of her jaundice? Answer Choices 1 Obstructive reasons 2 Hemolytic anemia 3 Hepatic enzyme deficiency 4 Hepatitis 5 Drug-induced hemolysis

Obstructive reasons With obstructive jaundice, bilirubin is formed and conjugated normally but the process of biliary drainage is impaired; an increase in serum conjugated bilirubin is seen. This can be due to a stone or tumor in the bile ducts, liver, or pancreas. Jaundice (or icterus) is a yellowish discoloration of the skin and mucous membranes due to increased serum bilirubin. Jaundice can occur due to a deficit in the bilirubin clearance pathway. Normally, bilirubin is unconjugated in the blood; it undergoes conjugation in the liver and is secreted in the bile. Increased unconjugated serum bilirubin occurs due to increased hemolysis, and it is usually mild. When there is a defect in liver conjugation due to congenital or acquired reasons (e.g., hepatitis), conjugation decreases; this results in increased unconjugated serum bilirubin. In hepatitis, there is also some defective biliary drainage, so both unconjugated and conjugated bilirubin increases. There is also clay-colored stool, dark urine, and itching.

Clinical signs of Wernicke's Encephalitis

Ocular Signs-Nystagmus, b/l lateral rectal palsies, conjugate gaze palsies Ataxia- affects stance and gait, no response to caloric stimulus Confusion- Apathy, disorientation, stupor, coma

A 29-year old patient presents with a 9-month history of recurrent hemoptysis. The patient suffered from cavitary tuberculous infection 5 years ago and was effectively treated with antituberculous drugs. A thin-walled cavity was seen over the right upper lobe when first diagnosed. At a later date, a progressive opacification of tuberculous cavity was seen. What organism is most likely responsible for such radiologic changes of a tuberculous cavity? Answer Choices 1 Secondary growth of staphylococci 2 Growth of Candida albicans might have filled the cavity 3 Growth of Aspergillus fumigatus might have filled the cavity 4 The cavity filled with granulation tissue and subsequent fibrosis 5 The cavity filled with desquamated epithelial cells of alveoli

One of the complications of tuberculous cavity is the growth of Aspergillus fumigatus within the cavity (fungal ball). Diagnosis is made by isolation of Aspergillus hyphae in the sputum. IgG antibody against Aspergillus can be demonstrated in such patients.

A 49-year-old Caucasian woman presents with a 2-month history of worsening abdominal pain. The patient describes the pain as crampy, unrelenting abdominal pain that begins in the right upper and middle quadrant of the abdomen; very occasionally, it will go into right shoulder area. The pain usually occurs shortly after especially heavy meals, usually within 30 minutes; it then eventually subsides, although this takes anywhere from 60 - 120 minutes. Physical examination reveals significant right upper quadrant tenderness and rigidity with a palpable, firm mass. Ultrasound performed in the office reveals complete calcification of the gallbladder wall; there is a hyperechoic semilunar appearance with posterior acoustic shadowing. Question A symptomatic patient as described above with such severe calcification as seen on ultrasound would most likely be treated via what method? Answer Choices 1 Reassurance only 2 Periodic ultrasound imaging study 3 Open cholecystectomy 4 Laparoscopic cholecystectomy 5 Percutaneous biliary stent placement

Open cholecystectomy Patients withsymptomatic porcelain gallbladdersare much more commonly female than male, with the typical age range being from 38 - 70 years old. In general, patients will describe a history of biliary type pain. Diagnostically, an ultrasound or CT scan will most efficiently give the visualization of the calcification necessary to confirm a porcelain gallbladder.Those found to have complete mural calcification (complete type) generally will need to have it treated with an open cholecystectomy. This is because the mucosal calcification that creates the porcelain characteristic makes the gallbladder very thick and fibrotic, which in turn makes it potentially difficult to grab by forceps and dissect out in a laparoscopic approach. There have been recent studies suggesting a laparoscopic cholecystectomy would be acceptable in patients who have evidence of a long cystic duct and biliary anatomy that is well-defined perioperatively, as well as in those who have evidence of a less advanced form of porcelain gallbladder, although there is no confirmed change of recommended treatment plan at this time. Anytime a porcelain gallbladder is found, there has to be a heightened alert for the suspicion of gallbladder carcinoma. The more advanced and severe the calcification is of the gallbladder, the heightened potential of it being malignant. Reassurance only and periodic ultrasounds would not be appropriate in this scenario. This approach may be followed in some part in a patient with less severe signs and symptoms. Percutaneous biliary stent placement is not the appropriate choice for this clinical scenario, as it is not treating the main pathology being presented.

A 63 year-old woman developed severe thrombocytopenia after undergoing her first treatment of a cisplatin-based chemotherapy regimen. Which of the following agents may be administered to prevent thrombocytopenia if the patient continues her cisplatin-based regimen? A Darbepoetin alfa (DARB) B Erythropoietin (EPO) C Filgrastim (G-CSF) D Oprelvekin (IL-11) E Sargramostim (GM-CSF)

Oprelvekin, or recombinant interleukin-11, is a megakaryocyte growth factor that promotes platelet production. Erythropoietin (B) and darbepoetin alfa (A) are both stimulators of red blood cell production, while filgrastim (C) and sargramostim (E) stimulate granulocyte production.

A 63-year-old woman presents with a 6-month history of worsening difficulty in swallowing. Shortly after swallowing, she feels like something is getting stuck in her upper chest, and the sensation lasts long enough to begin causing significant chest discomfort just behind her breastbone. The difficulty swallowing is often extremely variable and intermittent; it has not been progressive. It gets to the point that she feels like she is going to regurgitate her food, and she is also experiencing substantial episodes of acid reflux. The difficulty swallowing seems to be worsened by when the patient is extremely stressed and when she eats hot or cold food. She denies any weight loss, night sweats, or other significant issues. Physical examination of the patient is otherwise noncontributory. An extensive gastrointestinal evaluation, which included a comprehensive endoscopic evaluation, is negative. Question What is the first-line prescribed medication that would help alleviate this patient's signs and symptoms? Answer Choices 1 Injected Botulinum toxin 2 Intravenous nitroglycerin 3 Oral proton pump inhibitor (PPI) 4 Oral prednisone 5 Oral Metoclopramide

Oral PPI

A 63-year-old woman presents with a 6-month history of worsening difficulty in swallowing. Shortly after swallowing, she feels like something is getting stuck in her upper chest, and the sensation lasts long enough to begin causing significant chest discomfort just behind her breastbone. The difficulty swallowing is often extremely variable and intermittent; it has not been progressive. It gets to the point that she feels like she is going to regurgitate her food, and she is also experiencing substantial episodes of acid reflux. The difficulty swallowing seems to be worsened by when the patient is extremely stressed and when she eats hot or cold food. She denies any weight loss, night sweats, or other significant issues. Physical examination of the patient is otherwise noncontributory. An extensive gastrointestinal evaluation, which included a comprehensive endoscopic evaluation, is negative. Question What is the first-line prescribed medication that would help alleviate this patient's signs and symptoms? Answer Choices 1 Injected Botulinum toxin 2 Intravenous nitroglycerin 3 Oral proton pump inhibitor (PPI) 4 Oral prednisone 5 Oral Metoclopramide

Oral PPI The correct response is oral proton pump inhibitor. The patient in this presented scenario is showing signs and symptoms that are most consistent with esophageal dysphagia. Difficulty swallowing is the main complaint and is typically caused by 1 of 2 entities: localized neuromuscular disorders or obstructive lesions. Our patient specifically is showing issues of diffuse esophageal spasm. This is a motility disorder that is defined by simultaneous uncoordinated contraction of several esophageal segments. This can lead to potentially retrosternal pain and is worsened or precipitated by acid reflex, rapid eating, stress/anxiety, and hot/cold food. The dysphagia found in these patients is intermittent, non-progressive and does not cause weight loss. These patients have no documented abnormality in the distribution of myenteric neurons, normal lower esophageal sphincter relaxation, and no evidence of obstruction. All of these components are consistent with the patient described. In terms of a first-line treatment, acid suppression with a proton pump inhibitor (PPI) is the first-line pharmaceutical intervention that should be initiated. Sublingual nitroglycerin and calcium channel blockers may also be considered as treatment, but they are not typically considered first-line options. Botulinum toxin is not considered first-line treatment for diffuse esophageal spasms; it may be considered to be administered/injected in the lower esophageal sphincter and lower esophagus to reduce chest pain caused by diffuse esophageal spasm but again is not the first line treatment. Also, the patient described above is experiencing other symptoms, not just the chest pain. Prokinetic agents, such as metoclopramide, are considered treatment options for reflux esophagitis and Barrett esophagus-related causes of dysphagia. Oral corticosteroids may be considered in patients with diagnosed eosinophilic esophagitis.

A 33-year-old woman presents seeking advice. She is concerned regarding the appearance of spider angiomas that are present on her trunk and face. She states that spider angiomas appeared during pregnancy and that they persist now, 9 months after delivery. Before pregnancy, she was treated with imipramine because of depression and she is currently still taking this drug. She denies alcohol intake, however she tells you that 3 years ago, she had acute infection with hepatitis C virus. She was also frequently treated with flucloxacillin during the past couple of years for recurrent respiratory tract infections. In addition, she started taking oral contraceptives after delivery. The remainder of her personal history is unremarkable. Physical examination reveals the presence of multiple spider angiomas on the patient's face, forearms, and back. The remainder of the patient's general physical findings are unremarkable. Routine laboratory analyses reveal normal AST (26 IU/l, reference values 8 to 27 IU/l) and ALT (22 IU/l, reference value 8 to 23 IU/l) levels. Alkaline phosphatase level is normal (43 IU/l, reference value 23-71 IU/l) and other routine laboratory analyses reveal no abnormalities. HbSAg, HbeAg, and antiHbc antibodies are not present in patient's serum. IgG antibodies to hepatitis C are present, but testing for hepatitis C virus (HCV) does not reveal the presence of HCV RNA in patient's serum. Question What is the most likely cause of the patient's spider angiomas? Answer Choices 1 Flucloxacillin-induced hepatitis 2 Imipramine-induced hepatitis 3 Oral contraceptives 4 Chronic hepatitis caused by hepatitis C virus 5 Pregnancy-related appearance

Oral contraceptives The most likely cause of this patient's spider angiomas are Oral contraceptives. In conditions when estrogen hormones are present in excess such as during pregnancy, contraceptive intake, or liver disease (due to decreased degradation of estrogens), the surplus of estrogens cause the appearance of spider nevi. In pregnant women, spider angiomas appear from the second to fifth month of pregnancy and disappear in a short period of time (within 2 months) after the delivery. Since in the presented case, spider angiomas are present for a longer period of time, it is unlikely that they are pregnancy-related. In untreated patients with acute Hepatitis C, disease evolves towards chronicity in about 80% of patients, while only about 20% of patients recover completely. Patients who have chronic hepatitis C also have HCV RNA in their serum, but those who have recovered completely have no HCV RNA in serum and have IgG antibodies against hepatitis C virus as the marker of past hepatitis C infection. As the presented patient satisfies criteria for the patient who had hepatitis C in the past, her spider angiomas are not related to chronic hepatitis C infection. Imipramine seldom causes liver damage. In addition, hepatitis caused by imipramine administration is of a cholestatic type, i.e. it is associated with increased serum levels of transaminases and alkaline phosphatase. Since the alkaline phosphatase level in the presented patient is normal, imipramine-induced hepatitis is not likely. Similar to imipramine, hepatitis caused by flucloxacillin administration is also of a cholestatic type. For the reasons presented above, the possibility of flucloxacillin-induced hepatitis is also unlikely.

A 13-year-old boy presents with right breast development over the last 5 to 6 months. He is on the swim team, so this has caused him much consternation. The breast swelling is slightly tender and without any drainage. He has been healthy and denies taking any medications, he denies any substance abuse or trauma, and he is doing well at school. On exam, his height is 63 in (160 cm), 75th percentile and weight is 115 lbs. (53kg) also 75th percentile, BP is 94/68, pulse 68 beats/min. There is slightly tender 7-8 cm of right breast elevation and swelling extending from the areolar that is non-erythematous. The left side is normal. There is no axillary lymphadenopathy. The testes are descended bilaterally and measure 3 cm in size. Pubic hair shows sparse growth of long, downy hair at the base of the penis. There is no axillary or facial hair. He does have scant acne. The mother is very anxious and wants laboratory tests. Which of the following would be the most appropriate initial evaluation to rule out any underlying pathology? Answer Choices 1 Obtain surgery referral 2 Order ultrasound 3 Order LH, FSH, testosterone and estradiol 4 Order a urine drug screen 5 Obtain nutrition consult

Order LH, FSH, testosterone and estradiol Explanation Approximately 70% of pubertal boys develop at least breast buds during adolescence, though some will have more extensive growth. This is probably a result of low testosterone in relation to estradiol levels thus laboratory evaluation should include testosterone, estradiol and gonadotropins to assess levels. This would effectively rule out conditions that could cause markedly abnormal levels, such as Klinefelter syndrome, the most common sex chromosome disorder, and the most common cause of hypogonadism and infertility in men. There are decreased levels of testosterone and elevated follicle-stimulating and luteinizing hormones. Karyotype analysis will reveal 1 or more extra X chromosomes. Physiologic gynecomastia usually resolves by 18 years when adult androgen/estrogen ratios are achieved. The incidence of breast enlargement peaks between the ages of 13-15 years during Sexual Maturity rating or Tanner staging 2 to 3 and may be unilateral or bilateral. Reassurance concerning the benign nature of the condition is usually all that is needed. Gynecomastia that occurs at other times in age or sexual development (excluding newborns) or that evolves rapidly would warrant further investigation. Surgical removal of the enlarged breast glandular tissue should be considered in boys who have had very persistent gynecomastia and who have completed puberty. Ultrasound imaging of the breast can be used to distinguish between solid tumors and fluid-filled cysts. Breast cancer in adolescents is extremely rare. Tumors tend to be slow growing, well differentiated, firm and painless. A urine drug screen would point to possible marijuana use that can be seen with gynecomastia when it is smoked heavily. There is little concern in this patient who is doing well at school, denies substance abuse, and whose parent does not express this concern. Obese pubertal boys commonly present with gynecomastia due to the accumulation of fat in the upper segment of the body both subcutaneously and intra-abdominally. The breasts are often greater in size and more persistent than in usual physiologic gynecomastia. In such a case, a nutrition consult would be indicated; however, on exam this patient was seen to be proportional for both height and weight.

A patient was prompted to visit his health care provider after his wife started to notice that he was not interested in eating, has lost weight, and has been suffering from nausea for the last few weeks. The practitioner notes hyperpigmentation of the patient's skin, although the patient denies any recent sun or tanning salon exposure. Routine non-fasting blood work reveals the following: Sodium = 130 meq/L Potassium = 5.2 meq/L Chloride = 105 meq/L Glucose = 135 mg/dL Hemoglobin = 13.0 g/dL Hematocrit = 39.0 WBC count = 8,000/mm 3 Which of the following physical exam findings would you expect to see in this patient? A Orthostatic hypotension B Wide, purple striae C Central obesity D Full facial features E Exophthalmous

Orthostatic hypotension The correct choice is A, orthostatic hypotension. The first step in the discussion of this patient is the suspected diagnosis of adrenal insufficiency. Patients with this disorder will have an excess of ACTH, which will act like melanocyte stimulating factor on the skin and cause hyperpigmentation. In adrenal insufficiency, aldosterone is deficient, thereby causing a decrease in sodium retention and potassium excretion. Hypotension is found in approximately 90% of these patients, sometimes associated with syncope as well. Choices B, C, and D are found in patients with cortisol excess. Choice E can be found in patients with Graves' disease.

A 34-year-old male has a one and one-half day history of fever, chills, a non-productive cough, and malaise. He is otherwise healthy with no long-standing medical history, and is taking no chronic medications. On examination, the patient has a temperature of 101.3 0 F, BP 123/63, P 78, R 18. His HEENT reveals mild rhinorrhea, moist mucous membranes, clear lung sounds, and a regular rate and rhythm. The rapid nasal viral test for influenza B is positive. Based on this information, what is the medication treatment for this patient? A Amantadine B Oseltamavir C Ramantadine D Famciclovir E Azithromycin

Oseltamavir Oseltamivir is the best antiviral medication for the treatment of acute influenza. This medication is ideally started within the first onset of illness, usually within the first 24 to 36 hours. Amantadine and Ramantidine have been shown to not be effective, and there is growing resistance to the medication.

A 45-year-old male with asthma and diabetes is diagnosed with influenza B by nasal swab. He has been ill for one and a half days. Which of the following is indicated for treating this patient? A Acyclovir B Amantadine C Nevirapine D Oseltamivir E Zanamivir

Oseltamavir The neuraminidase inhibitors, including oseltamivir and zanamivir, are associated with a reduction in duration of illness, and secondary complications for both influenza A and B viral strains. However, zanamivir, due to the oral inhalation delivery route, is relatively contraindicated in this patient, due to his history of asthma and an associated increased risk of bronchospasm. Amantadine is inactive against influenza B, as well as certain influenza A strains. Nevirapine is an antiretroviral agent used in the treatment regimen for HIV. Acyclovir is an antiviral agent, but is not indicated for influenza.

A previously healthy 30-year-old woman, G1 P1, presents with amenorrhea, weight loss, shortness of breath, and increasing abdominal circumference. Menstrual irregularity started about 1 year ago, and her last menstruation was 3 months ago. She has lost around 5 kilos over the last few months, but at the same time, her waist has enlarged. Shortness of breath started a week ago, and it is worse when she is lying down; she now sleeps using at least 2 pillows. An examination of the lungs shows dullness to percussion, decreased tactile fremitus, and inaudible breath sounds bilaterally. Physical examination of the abdomen shows bulging of the flanks in the reclining position, and there is a difference in percussion in the flanks that shifts when she is turned on the side. Pelvic examination shows a normal uterus and left adnexa; the right adnexum appears enlarged, smooth, and tender. Complete blood count and chemistry is normal, and a chest X-ray confirms the presence of pleural effusion on the right side. Fluid obtained from peritoneal cavity shows heterogeneously bloody content that clots; the leukocyte number is normal, and serum-ascites albumin gradient (SAAG) is 0.8 g/dL (low). The sample is negative of malignancy. Question What is the most likely cause of her symptoms? Answer Choices 1 Extrauterine pregnancy 2 Acute heart failure 3 Acute liver failure 4 Intra abdominal hemorrhage 5 Ovarian fibroma

Ovarian fibroma Explanation Ascites in a young woman is most often caused by liver, pancreatic or renal disease, pelvic or abdominal tumours, and infection. The SAAG less than 1.1 g/dL points to the non-portal hypertensive ascites. This case is Meigs syndrome; it is characterized by the triad of benign ovarian tumor (usually fibroma), ascites, and pleural effusion that resolves after resection of the tumor. Menstrual irregularity and amenorrhea are probably caused by a fibrous growth in this woman's ovary, which causes abnormal levels of sex hormone production. An extrauterine pregnancy will not present with the ascites and pleural effusion. The clinical presentation of ectopic pregnancy occurs at approximately 2 months after the last normal menstrual period; it typically causes vaginal bleeding and abdominal pain. Acute heart failure may present with shortness of breath, and ascites and ovarian tumor may be the accidental finding; however, SAAG >1.1 g/dL rules out non-portal hypertensive ascites. The SAAG is calculated by subtracting simultaneously obtained the ascitic fluid albumin value from the serum albumin value, and it correlates directly with portal pressure. Therefore, it is the best single test for classifying the ascites into the ascites caused by portal hypertension (SAAG >1.1 g/dL) and non-portal hypertension (SAAG < 1.1 g/dL). Its accuracy is about 97%. Acute liver failure is not likely when there is a normal CBC and chemistry panel with non-portal hypertensive SAAG. Visible bloody ascites requires the presence of more than 20,000 red blood cells/µL. It is usually a consequence of either malignancy or a traumatic tap. A traumatic tap results in a heterogeneously bloody fluid that will clot. Therefore, a traumatic tap is most probably the case of bloody peritoneal fluid.

A 72-year-old man is evaluated at the bedside following hospital admission for a 1-year history of progressive dyspnea, nonproductive cough, weight loss, low-grade fevers, fatigue, and myalgias. His past medical history is remarkable for atrial fibrillation, for which he takes amiodarone, hypercholesterolemia, and recurrent urinary tract infections, for which his urologist prescribed nitrofurantoin on a chronic, prophylactic basis. He denies any cigarette use, history of murmurs or coronary artery disease, chills, fatigue, rhinitis, otalgia, chest pain, wheezing, hemoptysis, syncope, abdominal pain, rashes, peripheral edema, diaphoresis, arthralgias, vomiting, or urinary complaints. Arterial blood gas analysis demonstrated a PaO2 of 50 mmHg and pulse oximetry of SpO2 of 87%; bedside echocardiogram and electrocardiograms are unremarkable for abnormalities; a chest x-ray revealed peripheral reticular opacities at the lung bases and a generalized honeycombing pattern. Question What treatment is the most beneficial in the management of this patient? Answer Choices 1 Prednisone 2 Colchicine 3 Oxygen 4 N-acetylcysteine 5 Bosentan

Oxygen This patient's most likely diagnosis is idiopathic pulmonary fibrosis (IPF). It is defined as a specific form of chronic, progressive fibrosing interstitial pneumonia of unknown cause, primarily occurring in older adults, limited to the lungs, and associated with the histopathologic and/or radiologic pattern of usual interstitial pneumonia (UIP). Patients with hypoxemia (PaO2 < 55 mmHg or oxygen saturation as measured using pulse oximetry [SpO2] < 88%) at rest or with exercise should be prescribed oxygen therapy to maintain a saturation of at least 90% at rest, with sleep, and with exertion. Corticosteroids have not been evaluated in a randomized, placebo-controlled trial to determine their benefit in treating patients with idiopathic pulmonary fibrosis. Retrospective uncontrolled studies have reported no survival benefits. Evidence-based guidelines recommend that patients with idiopathic pulmonary fibrosis should not be treated with corticosteroid monotherapy. Colchicine has been shown to inhibit fibroblast proliferation and collagen synthesis in vitro; however, evidence-based guidelines recommend that patients with idiopathic pulmonary fibrosis should not be treated with colchicine. Evidence-based guidelines recommend that the majority of patients with IPF should not be treated with N-acetylcysteine monotherapy. Bosentan is an endothelin receptor A and B antagonist that is approved for the treatment of pulmonary hypertension. While bosentan has been shown to have antifibrotic effects in an animal model of pulmonary fibrosis, current evidence-based guidelines recommend that patients with idiopathic pulmonary fibrosis should not be treated with bosentan.

A 69-year-old man with a past medical history of obstructive sleep apnea, hypertension, and COPD presents with dyspnea on exertion that has been slowly progressive over the course of the last year. He notes impairment in climbing stairs and walking short distances. His review of systems is positive for fatigue, palpitations, intermittent retrosternal chest pain, swelling of his lower extremities, dizziness, and "feeling faint;" his associated symptoms are also known to occur upon exertion. His cardiac exam is remarkable for an oxygen saturation of 90%, accessory muscle usage, an increased pulmonic component of the second heart sound (P2), wide, inspiratory splitting of S2 over the cardiac apex, right-sided S3 and S4 gallops, a left parasternal lift, a loud diastolic murmur that increases with inspiration and diminishes with the Valsalva maneuver, prominent "A" waves in jugular venous pulsations, and increased JVD. He has an enlarged liver with hepatojugular reflux, peripheral edema, and ascites. A bedside EKG and chest x-ray were performed, which revealed the results in the image. Question What is the most appropriate intervention in the management of this patient? Answer Choices 1 Lisinopril 2 Warfarin 3 Digoxin 4 Oxygen therapy 5 Nitroglycerin

Oxygen therapy This patient has a presentation most consistent with cor pulmonale secondary to chronic obstructive pulmonary disease (COPD). The electrocardiogram demonstrates right axis deviation, right ventricular hypertrophy, and right atrial enlargement. The chest x-ray indicates cardiac enlargement, with prominence of the pulmonary artery, right atrium, and right ventricle. Treatment should center upon the underlying contributory disorder. For patients with COPD, bronchodilation and infection treatment should be considered. Oxygen therapy is of great importance in patients with underlying COPD, especially when administered on a continuous basis. Oxygen therapy relieves hypoxemic pulmonary vasoconstriction, which then improves cardiac output, lessens sympathetic vasoconstriction, alleviates tissue hypoxemia, and improves renal perfusion. Anticoagulation with warfarin is recommended in patients at high risk for thromboembolism. The evidence of benefit has not been established in patients with secondary PAH. Therefore, anticoagulation therapy may be used in patients with cor pulmonale secondary to thromboembolic phenomena and with underlying primary pulmonary artery hypertension. The use of cardiac glycosides, such as digitalis, in patients with cor pulmonale has been controversial, and the beneficial effect of these drugs is not as obvious as in the setting of left heart failure. This drug must be used cautiously and should not be used during the acute phases of respiratory insufficiency, when large fluctuations in levels of hypoxia and acidosis may occur. Patients with hypoxemia or acidosis are at increased risk of developing arrhythmias due to digitalis through different mechanisms, including sympathoadrenal stimulation. Some classes of vasodilators, such as beta agonists, nitrates, and angiotensin-converting enzyme (ACE) inhibitors, have been tried, but, in general, vasodilators have failed to show sustained benefit in patients with COPD and are not routinely used.

A 15-year-old male presents complaining of a sore throat, headache, and mild cough that started 8 days ago and has progressed to include a worsening cough and increasing fatigue. His chest x-ray reveals bilateral hilar infiltrates and a CBC is normal. Which of the following diagnostic tests will most likely confirm the suspected diagnosis? A Acid-fast bacilli smear and culture B Blood culture C PCR testing of sputum D Sputum culture E Sputum gram stain

PCR Testing of the sputum The patient most likely has atypical pneumonia secondary to mycoplasma pneumoniae which is best confirmed by PCR testing of sputum (C), oropharyngeal or nasal secretions and isn't detected through standard cultures (B, D) or staining techniques (A, E).

A patient recovering from hip surgery begins to ambulate for the first time about 2 hours postoperatively. Suddenly, she experiences shortness of breath. She becomes tachypneic and tachycardic; she experiences pain on inspiration. Prior to getting out of bed, her postoperative course was unremarkable. There is no swelling; there is no palpable thrill at the incision site. What is your initial diagnosis? Answer Choices 1 Postoperative pneumonia 2 Atelectasis 3 Arteriovenous fistula at surgical site 4 Pulmonary embolism 5 Abdominal aortic aneurysm rupture

PE The clinical picture is suggestive of a pulmonary embolism (PE). Pulmonary embolisms are a serious and potentially fatal complication of thrombi formation, usually in the lower extremities. Risk factors include venous stasis, hypercoagulability, and injury to the vessel wall; they are known as Virchow's triad. Venous stasis increases with immobility, in this case bed rest, especially postoperatively. Clinical findings include dyspnea, chest pain, syncope, hemoptysis, tachypnea, and pain on inspiration. Postoperative pneumonia usually occurs after 48 hours postoperatively, and the patient will present with fever on day 3 or 4 postoperatively. Atelectasis is a common postoperative complication (collapse of lung tissue), especially after chest or abdominal surgery. Arteriovenous fistula is an abnormal connection between an artery and a vein; a palpable thrill can be felt at the incision site. An abdominal aortic aneurysm rupture presents with severe pain, a palpable abdominal mass, and hypotension.

A 46-year-old woman presents with a 2-month history of heartburn, epigastric discomfort, nausea, and occasional vomiting. She has a history of hyperlipidemia, controlled with diet and exercise, as well as asthma, for which she takes inhalers as needed. She takes no other medications, including over-the-counter analgesics. Family history is noncontributory. On exam she is afebrile, BP120/70 mm Hg, pulse 74/min, and SPO2 92%. Lungs are clear, and she has minimal epigastric tenderness. Otherwise, physical examination is unremarkable. She is advised by her physician to take lansoprazole once daily, which provides only partial relief. Endoscopy is then recommended, which shows a duodenal ulcer. Biopsy reveals infection with Helicobacter pylori. What would be the recommended regimen at this time? Answer Choices 1 Lansoprazole, amoxicillin, and metronidazole twice daily for 2 weeks 2 PPI (proton pump inhibitor), amoxicillin, and clarithromycin twice daily for 2 weeks 3 Omeprazole, tetracycline, and clarithromycin twice daily for 2 weeks 4 Bismuth, metronidazole, and lansoprazole twice daily for 2 weeks 5 Bismuth, metronidazole, and tetracycline 4 times daily for 2 weeks

PPI (proton pump inhibitor), amoxicillin, and clarithromycin twice daily for 2 weeks

A 46-year-old woman presents with a 2-month history of heartburn, epigastric discomfort, nausea, and occasional vomiting. She has a history of hyperlipidemia, controlled with diet and exercise, as well as asthma, for which she takes inhalers as needed. She takes no other medications, including over-the-counter analgesics. Family history is noncontributory. On exam she is afebrile, BP120/70 mm Hg, pulse 74/min, and SPO2 92%. Lungs are clear, and she has minimal epigastric tenderness. Otherwise, physical examination is unremarkable. She is advised by her physician to take lansoprazole once daily, which provides only partial relief. Endoscopy is then recommended, which shows a duodenal ulcer. Biopsy reveals infection with Helicobacter pylori. What would be the recommended regimen at this time? Answer Choices 1 Lansoprazole, amoxicillin, and metronidazole twice daily for 2 weeks 2 PPI (proton pump inhibitor), amoxicillin, and clarithromycin twice daily for 2 weeks 3 Omeprazole, tetracycline, and clarithromycin twice daily for 2 weeks 4 Bismuth, metronidazole, and lansoprazole twice daily for 2 weeks 5 Bismuth, metronidazole, and tetracycline 4 times daily for 2 weeks

PPI (proton pump inhibitor), amoxicillin, and clarithromycin twice daily for 2 weeks There are several regimens recommended for H. pylori infection, which is an important cause of peptic ulcer disease and should be treated if found associated with symptoms. The choice of the regimen depends on considerations such as cost, side effects, and ease of administration. Allergy to one of the medications, as well as intolerance, should also be taken into account. Any proton pump inhibitor (PPI) with amoxicillin 1000 mg twice daily and clarithromycin 500 mg twice daily for 2 weeks or PPI with metronidazole 500mg twice daily and clarithromycin 500mg twice daily for 2 weeks are recommended. These are the triple drug therapies available. The other regimens suggested are bismuth, metronidazole, and tetracycline 4 times daily for 2 weeks along with PPI twice daily for 2 weeks or H2 receptor antagonist twice daily for 4 weeks (quadruple drug therapy). Dual therapy with a proton pump inhibitor and an antibiotic (amoxicillin or clarithromycin) is not recommended as primary therapy, since eradication rates are much lower than the above regimens. The most common side effect is a metallic taste in the mouth due to clarithromycin or metronidazole. Amoxicillin can cause diarrhea or a rash. Clarithromycin can also cause nausea, vomiting, abdominal pain, and (rarely) QT prolongation. Metronidazole can cause peripheral neuropathy, seizures, and a disulfiram-like reaction when taken with alcohol. Tetracycline is teratogenic and causes photosensitivity.

A 63-year-old woman presents with a 6-month history of worsening difficulty in swallowing. Shortly after swallowing, she feels like something is getting stuck in her upper chest, and the sensation lasts long enough to begin causing significant chest discomfort just behind her breastbone. The difficulty swallowing is often extremely variable and intermittent; it has not been progressive. It gets to the point that she feels like she is going to regurgitate her food, and she is also experiencing substantial episodes of acid reflux. The difficulty swallowing seems to be worsened by when the patient is extremely stressed and when she eats hot or cold food. She denies any weight loss, night sweats, or other significant issues. Physical examination of the patient is otherwise noncontributory. An extensive gastrointestinal evaluation, which included a comprehensive endoscopic evaluation, is negative. Question What is the first-line prescribed medication that would help alleviate this patient's signs and symptoms? Answer Choices 1 Injected Botulinum toxin 2 Intravenous nitroglycerin 3 Oral proton pump inhibitor (PPI) 4 Oral prednisone 5 Oral Metoclopramide

PPIs The correct response is oral proton pump inhibitor. The patient in this presented scenario is showing signs and symptoms that are most consistent with esophageal dysphagia. Difficulty swallowing is the main complaint and is typically caused by 1 of 2 entities: localized neuromuscular disorders or obstructive lesions. Our patient specifically is showing issues of diffuse esophageal spasm. This is a motility disorder that is defined by simultaneous uncoordinated contraction of several esophageal segments. This can lead to potentially retrosternal pain and is worsened or precipitated by acid reflex, rapid eating, stress/anxiety, and hot/cold food. The dysphagia found in these patients is intermittent, non-progressive and does not cause weight loss. These patients have no documented abnormality in the distribution of myenteric neurons, normal lower esophageal sphincter relaxation, and no evidence of obstruction. All of these components are consistent with the patient described. In terms of a first-line treatment, acid suppression with a proton pump inhibitor (PPI) is the first-line pharmaceutical intervention that should be initiated. Sublingual nitroglycerin and calcium channel blockers may also be considered as treatment, but they are not typically considered first-line options. Botulinum toxin is not considered first-line treatment for diffuse esophageal spasms; it may be considered to be administered/injected in the lower esophageal sphincter and lower esophagus to reduce chest pain caused by diffuse esophageal spasm but again is not the first line treatment. Also, the patient described above is experiencing other symptoms, not just the chest pain. Prokinetic agents, such as metoclopramide, are considered treatment options for reflux esophagitis and Barrett esophagus-related causes of dysphagia. Oral corticosteroids may be considered in patients with diagnosed eosinophilic esophagitis.

A patient seen at the prenatal clinic develops Graves disease at 25 weeks' gestation. Which of the following is the most appropriate treatment? A PTU 100 mg po tid B methimazole 10 to 30 mg po qd C propranolol 80 mg po qid D radioactive iodine therapy (RAI, 131I) E levothyroxine 0.1 mg po qd

PTU 100 mg po tid In nonpregnant patients, PTU and methimazole are the drugs of choice for the management of Graves disease. During pregnancy, PTU has a lower incidence of crossing the placental barrier than does methimazole. It also is excreted into breast milk to a lesser degree than is methimazole. Propranolol will help with the symptoms of Graves but not treat it. It can also cause low birth weight in the infant. RAI is contraindicated in pregnancy. Levothyroxine will worsen a Graves patient's hyperthyroidism

A patient seen at the prenatal clinic develops Graves disease at 25 weeks' gestation. Which of the following is the most appropriate treatment? A PTU 100 mg po tid B methimazole 10 to 30 mg po qd C propranolol 80 mg po qid D radioactive iodine therapy (RAI, 131I) E levothyroxine 0.1 mg po qd

PTU 100 mg po tid In nonpregnant patients, PTU and methimazole are the drugs of choice for the management of Graves disease. During pregnancy, PTU has a lower incidence of crossing the placental barrier than does methimazole. It also is excreted into breast milk to a lesser degree than is methimazole. Propranolol will help with the symptoms of Graves but not treat it. It can also cause low birth weight in the infant. RAI is contraindicated in pregnancy. Levothyroxine will worsen a Graves patient's hyperthyroidism.

A 20-year-old man presents due to developing sudden onset of shortness of breath. He is experiencing chest pain that worsens with inspiration. He has no history of pulmonary disease, but he does smoke half a pack of cigarettes daily. Vital signs reveal blood pressure 130/82, heart rate 140, respirations 28, and temperature of 98.2 degrees Fahrenheit. Heart sounds are normal with no murmurs, gallops or rubs. Lung examination reveals absent breath sounds and hyperresonance to percussion on the left side. Question What is the most likely diagnosis? Answer Choices 1 Pleural effusion 2 Pneumonia 3 Lung cancer 4 Pneumothorax 5 Asthma attack

PTX Pneumothorax occurs when air accumulates in the pleural spaces, causing the lung to collapse. Pneumothorax may be associated with underlying lung disease, or it can occur spontaneously. Spontaneous pneumothorax is more common in young men with a history of smoking. The presentation is dependent on the size of the pneumothorax. The patient in this case most likely has a large pneumothorax, as he presents with chest pain, tachypnea, tachycardia, hyperresonance to percussion, and absent breath sounds on the affected side. Diagnosis can be confirmed with chest X-ray. Pleural effusion is abnormal fluid collection in the pleuritic space. Patients will complain of pleuritic chest pain, as in this case, but they will have findings of dullness to percussion on examination. Pneumonia is unlikely; the patient is not demonstrating any signs of infection. Lung cancer may be associated with dyspnea, but it typically has gradual onset. Although the patient does have a risk factor for lung cancer (history of smoking), his young age makes the diagnosis unlikely. A patient having an asthma attack may complain of dyspnea and chest pain, but he or she would likely have wheezing on examination.

A 75-year-old woman presents to your office complaining of intense pruritis of the vulva and occasional bleeding. She is unsure if the bleeding is caused by her scratching in attempts to alleviate the itching. She has tried some OTC preparations to alleviate the itching and has not had any relief from them. She denies any vaginal discharge or dysuria. On physical examination you notice excoriations and some scattered lesions that look like eczema on the vulva and they do not scrape off. Also noted was inguinal lymphadenopathy. You decide to do a punch biopsy. The pathology report reveals large eosinophilic cells. Question What is the most likely diagnosis? Answer Choices 1 Lichen sclerosis 2 Vulvar carcinoma 3 Paget's disease 4 Melanoma 5 Candida albicans

Paget's disease Explanation Paget's diseaseis associated with intense pruritus of the vulva, along with lesions that resemble eczema. Pathology also characteristically shows large eosinophilic Paget's cells (1). Lichen sclerosis is not correct because the pathology would reveal changes associated with chronic inflammation. Vulvar carcinoma is incorrect as it generally shows squamous cell characteristics. (1). Melanoma is incorrect as the lesion will generally appear hyperpigmented (1). Candida albicans is incorrect because the lesions that appear on the vulva do not scrape off and she does not have any vaginal discharge. Candida albicans is also not associated with inguinal lymphadenopathy.

A 64-year-old man presents with a 3-week history of weight loss, abdominal pain, loss of appetite, nausea, weakness, fatigue, vomiting, diarrhea, indigestion, back pain, and depression. Past medical history is negative. Social history includes smoking 2 packs of cigarettes a day for 35 years. Clinical exam includes abdominal pain in the upper quadrants and pallor. Fecal fat test is positive. What is the most likely diagnosis? Answer Choices 1 Acute pancreatitis 2 Chronic pancreatitis 3 Pancreatic carcinoma 4 Pacreatic abscess 5 Insulinoma

Pancreatic carcinoma is the correct response. Itis the 4th most common cancer causing death in the U.S. The disease is more common in men, especially those between 60 and 70 years. The cause is unknown; however, the incidence is greater in smokers. High fat diet and chemical exposures may increase the risk. Symptoms include weight loss, abdominal pain, loss of appetite, jaundice, nausea, weakness, fatigue, vomiting, diarrhea, indigestion, back pain, stools (clay colored), pallor, and depression. Tests should include a pancreatic biopsy, an abdominal CT scan, and abdominal ultrasound. Only 20% of the tumors are operable at the time of diagnosis. Palliation is generally the treatment, along with chemotherapy and radiation. The chief causes of acute pancreatitis in adults are gallstones, other biliary diseases, or alcohol use. Viral infection (mumps, Coxsackie B, mycoplasma pneumonia, and Campylobacter), injury, pancreatic or common bile duct surgical procedures, and certain medications (especially estrogens, corticosteroids, thiazide diuretics, acetaminophen, tetracycline), are other causes. After the triggering event, the process continues with autodigestion that causes swelling, hemorrhage, and damage to the blood vessels. An attack may last for 48 hours. Symptoms include abdominal pain (mainly located in the upper abdomen) nausea, vomiting, weakness, sweating, anxiety, fever, clammy skin, and mild jaundice. General examination may show a low blood pressure and a heart rate above 90. Most cases resolve within 1 week with supportive measures such as fluid replacement. Chronic pancreatitis is caused by alcohol abuse, hemochromatosis (a condition of excess iron in the blood), and other unknown factors. Inflammation and fibrosis cause the destruction of functioning glandular tissue in the pancreas. This results in an inability to properly digest fat due to a lack of pancreatic enzymes. The production of insulin is also affected. Symptoms include abdominal pain (mainly in the upper abdomen), nausea, vomiting, weight loss, and fatty stools. Additional symptoms may include swelling (overall), clay-colored stools, and abdominal indigestion. Pancreatic abscess occurs in 5 to 10% of people with acute pancreatitis. An abscess may be caused by inadequate drainage of a pancreatic pseudocyst; a complication associated with pancreatitis. Symptoms include fever, chills, abdominal pain, and abdominal mass. Physical exam will show signs of pancreatitis, and tests should include abdominal CT and ultrasound. Treatment will include laparotomy with drainage and possible resection of dead tissue. Insulinomas are generally benign tumors of the insulin-secreting cells of the pancreas, which secrete excess amounts of insulin. Risk factors include a prior history of multiple endocrine neoplasia Type I (MEN I). Symptoms include sweating, tremor, rapid heart rate, anxiety, hunger, dizziness, headache, clouding of vision, confusion, behavioral changes, convulsions, and loss of consciousness. Surgery is the treatment of choice to remove the tumor. If the tumor is not found during surgery, diazoxide may be given. A diuretic is always given with this medication to avoid retaining too much salt.

A 42-year-old woman presents for annual examination. She is in excellent health; there are no current medical issues or medications. On examination, a firm, nontender thyroid nodule is palpated. TSH, T3, and T4 are all within normal limits. The patient undergoes a fine needle aspiration. Question What is the most common type of thyroid cancer likely in this patient? Answer Choices 1 Papillary carcinoma 2 Follicular carcinoma 3 Medullary carcinoma 4 Anaplastic carcinoma 5 Metastatic disease from another location

Papillary Carcinoma Explanation Papillary carcinoma is the most common form of thyroid cancer, accounting for approximately 80% of all cases. Follicular carcinoma accounts for approximately 14% of thyroid cancers. Medullary carcinoma is responsible for about 3% of thyroid cancers. Anaplastic carcinoma causes approximately 2% of thyroid cancers. Metastatic disease from another location is a rare presentation of thyroid cancer. It is most commonly seen with lung, breast, renal, and skin cancers.

Psammoma bodies are concentric calcified structures and are found in several tissue pathologies. In the image, psammoma bodies can be seen. Psammoma bodies are characteristic of which of the following? Answer Choices 1 Papillary carcinoma of the thyroid 2 Mucinous cyst adenocarcinoma of the ovaries 3 Medullary carcinoma of thyroid 4 Follicular adenoma 5 Pheochromocytoma

Papillary carcinoma of the thyroid Explanation Single necrotic cells may form a nidus for deposition of calcium. Subsequently, the progressive acquisition of an outer layer of calcium may form a lamellated structure, which resembles grains of sand. This is called a psammoma body. Psammoma bodies are seen in some neoplastic conditions, such as papillary carcinoma of the thyroid, serous cyst adenocarcinoma of the ovary, adenocarcinoma of the endometrium, and meningioma. Rarely, it may be seen in adenocarcinoma of the lung. In the case of the ovary, the possible mode of formation is by neoplastic and histiocytic cellular degeneration. Psammoma bodies (blue arrow) - calcific concretions with well-defined concentric laminations

Which agent is most responsible for croup infections? A Parvovirus B Adenovirus C Parainfluenza D Bocavirus E Streptococcus

Parainfluenza Laryngotracheobronchitis, or croup, is mostly caused by the parainfluenza virus. Other agents that can cause this are respiratory syncytial virus (RSV), human metapneumovirus, influenza virus, rubeola virus, adenovirus, and Mycoplasma pneumoniae. Croup, also known as laryngotracheobronchitis, is associated with upper tracheal narrowing and edema, which is visible on an anteroposterior soft tissue neck x-ray. This is termed the "steeple sign." Epiglottitis is associated with a thickened epiglottis on a lateral soft-tissue neck x-ray, termed the "thumb" sign. Foreign body aspiration and tracheal carcinoma may have x-ray findings based on the location, size, and components present. Peritonsillitis is best visualized on physical exam. If assessing for a potential peritonsillar abscess, a contrasted CT is recommended.

A 72-year-old man presents with the inability to comprehend spoken language. His speech is fluent and has a normal cadence and rhythm; however, when he talks, it is gibberish. He frequently substitutes one word for another. What is this phenomenon called? Answer Choices 1 Paraphasia 2 Echolalia 3 Alexia 4 Apraxia 5 Agraphia

Paraphasia Paraphasia is a type of aphasia. The substitution of a similar sounding word for another word is called paraphasia. With paraphasia, the words can also be jumbled. If a patient is able to hear things and repeat them, it is called echolalia. With echolalia, the patient does not understand what he has heard. This is also referred to as echophrasia. Alexia is a type of aphasia. An aphasia in which there is a problem with reading is called alexia. Alexia is word blindness or text blindness. Alexia is also called optical aphasia or visual aphasia. Apraxia refers to the condition in which a patient has difficulty performing motor acts, despite having the muscular capacity and coordination to do so. A patient with apraxia cannot execute the intended movement. A writing disturbance is called agraphia. There are various forms of agraphia. With absolute agraphia, even simple letters cannot be written. This is also referred to as literal agraphia.

A 56-year-old woman presents with severe left-sided flank pain. She has been treated for kidney stones twice in the past year. Review of systems is positive for polydipsia, fatigue, overall weakness, and depression. She denies fever, hematuria, vaginal discharge, constipation, or diarrhea. Her past medical history is significant for hypertension, which is controlled on hydrochlorothiazide. Her serum laboratory values are listed in the table. Serum Reference range Result Blood urea nitrogen 8 - 20 mg/dL 19 mg/dL Creatinine 0.6 -1.2 mg/dL 1.1 mg/dL Glucose 60 - 110 mg/dL 82 mg/dL Phosphorus 2.5 - 4.5 mg/dL 1.9 mg/dL Ionized calcium 4.6 - 5.3 mg/dL 5.9 mg/dL Parathyroid hormone, intact 11 - 54 pg/mL 87 pg/mL Question What is the underlying diagnosis contributing to her recurrent nephrolithiasis? Answer Choices 1 Chronic kidney disease 2 Diabetes mellitus 3 Metastatic breast cancer 4 Parathyroid adenoma 5 Thiazide diuretic overuse

Parathyroid Adenoma Explanation Parathyroid adenoma is the correct response. The patient has hypercalcemia that is contributing to her recurrent nephrolithiasis. Patients with hypercalcemia may be asymptomatic or present with "bones, stones, abdominal groans, psychic moans, with fatigue overtones." The most common causes of hypercalcemia are primary hyperparathyroidism and malignancy, accounting for over 90% of cases of hypercalcemia. To determine the cause of hypercalcemia, serum ionized calcium, phosphate, intact parathyroid hormone (iPTH), and urine calcium levels should be evaluated. If malignancy is suspected, appropriate workup should be performed. This patient has elevated ionized calcium, elevated calcium in her urine, and hypophosphatemia, with elevated parathyroid hormone levels consistent with primary hyperparathyroidism. Primary hyperparathyroidism is caused by unregulated PTH secretion. In 80% of cases, it is caused by a single parathyroid adenoma. It is more common in patients over the age of 50 and affects women more than men. Laboratory results show increased serum and urine calcium, increased iPTH, and decreased or normal serum phosphorous. Imaging is not necessary, but it may be used in a preoperative mapping of the parathyroid glands. Medical treatment includes fluids, bisphosphonates, or cinacalcet. Symptomatic patients can be cured with a parathyroidectomy. Calcium and vitamin D supplementation may be required for a reactive secondary hyperparathyroidism occurring after surgery. Patients with chronic kidney disease will have increased blood urea nitrogen and creatinine levels. They develop a secondary hyperparathyroidism due to decreased vitamin D metabolism, resulting in decreased serum calcium levels and increased PTH levels with increased serum phosphorus levels. Patients with hypercalcemia can develop polyuria and polydipsia due to nephrotic diabetes insipidus. These symptoms can be similar to diabetes mellitus (DM). In DM, the serum calcium, phosphorus, and parathyroid levels will be normal with elevated blood glucose levels. Thiazide diuretic use can result in hypercalcemia, but the urine calcium level will be low. PTH and phosphorus levels will be normal. Malignancy should be evaluated as a possible cause of hypercalcemia. If the patient has not undergone routine screenings, they should be done. The PTH level will be low in a patient with malignancy-related hypercalcemia.

Your patient has a history of primary hyperparathyroidism. Recently she has been hospitalized due to obstructing kidney stones. She has had several fractures including her hip, sacrum, and forearm in the past year, all on separate occasions. She is constantly complaining of a lack of energy. What is the recommended treatment for her symptomatic hyperparathyroidism? A surgical removal of the pituitary B high-dose calcium supplementation C parathyroidectomy D thyroidectomy E thiazide diuretics

Parathyroidectomy The correct answer is (C). Primary hyperparathyroidism is most commonly secondary to a single parathyroid adenoma. The recommended treatment for symptomatic primary hyperparathyroidism is parathyroidectomy. High-dose calcium supplementation and thiazide diuretics, choices (B) and (E), can worsen the hypercalcemia associated with hyperparathyroidism. Neither surgical removal of the pituitary, choice (A), nor or a thyroidectomy, choice (D), is an indicated treatment for this condition.

A patient is recovering from having a total thyroidectomy two days ago for medullary thyroid cancer. An extensive neck dissection was required during the surgery. Post-operative lab testing reveals a low serum calcium level. Which of the following clinical presentations will most likely occur in this patient? A Constipation B Anorexia C Polyuria D Bone pain E Paresthesias

Paresthesias The correct choice is E, paresthesias. Circumoral paresthesias are signs of hypocalcemia. Hypocalcemia can occur after any type of neck surgery that may have resulted in destruction of the parathyroid glands. Choices A through D are symptoms of hypercalcemia and may be seen in hyperparathyroidism.

A colleague asks you to provide a neurosurgical consultation on a 75-year-old man. The consultation request form informs you that, "This man is having difficulty tolerating his medications as well as difficulty with his day-to-day activities". Your colleague wants to know if neurosurgery could help the patient's neurologic disorder. As a prudent provider, you plan to visit this man prior to recommending or denying definitive surgery. For what disorder may neurosurgery become a reasonable therapeutic option? Answer Choices 1 Alzheimer's 2 Parkinson's 3 Amyotrophic lateral sclerosis 4 Down's syndrome 5 Multiple sclerosis

Parkinson's As per Svennilson E, Torvik A, Lowe R, and Leskell L, "Treatment of Parkinsonism by Stereotactic Thermo-lesions in the Pallidal Region," Acta Psychiatry Neurology Scandinavia, 35:358-377, 1960, neurosurgery was provided for patients afflicted with Parkinson's disorder (PD) 50 years ago. Providers now know that dopamine replacement, carbidopa-levodopa (Sinemet) for example, is not a cure for PD. Eskandar, EN, Cosgrove GR, and Shinobu LA, "Surgical Treatment of Parkinson's Disease," The Journal of the American Medical Association, Volume 286, Number 24, pages 3056 - 3059, December 26, 2001, endorse that thalamotomy and pallidotomy can improve tremor and rigidity in selected patients with PD, sparing patients the risk of the adverse effects of the dopamine receptors. Neurosurgery has not been reasonably considered for patients with Alzheimer's disorder, ametropic lateral sclerosis, multiple sclerosis, or Down's syndrome, all of which represent a diffuse involvement of the brain and central nervous system in contrast to PD, which represents a focal involvement of the brain and central nervous system. In addition, the reduced intellectual functioning commonly associated with Down's syndrome would prevent achieving scholastic and academic rank.

A 38-year-old male presents with red-brown urine in the mornings. The discoloration has come and gone several times over the last several months. He has also noted hard, painful areas on his skin, which seem to be located over veins, which have occurred intermittently. Laboratories reveal nonspherocytic red cells and a Coombs negative intravascular hemolysis. What is the most likely diagnosis? A Acute Myelogenous Leukemia B Aplastic Anemia C Iron Deficiency Anemia D Paroxysmal Nocturnal Hemoglobinuria E Von Willebrand's Disease

Paroxysmal Nocturnal Hemoglobinuria Urine that appears to be bloody, primarily in the morning and episodic in nature, along with thrombosis of dermal veins, nonspherocytic red cells and Coombs negative intravascular hemolysis are classic signs of PNH. PNH may be a symptom of aplastic anemia, but this patient doesn't have a pancytopenia. He also does not have a microcytic anemia, which rules out iron deficiency. Von Willebrand's is a coagulation disorder.

A 36-year-old woman with a past medical history of diabetes mellitus and hepatitis presents to her family medicine office with complaints of weight gain and skin changes. Her review of systems is positive for menstrual irregularities with extended periods of amenorrhea, infertility, depression, cognitive dysfunction, and emotional lability. Of late, her fasting glucose levels have been above normal. Her physical exam notes increased adipose tissue in the face, upper back, and above the clavicles. Her skin reveals ecchymoses, telangiectasias, and purpura along her back and lower extremities, facial acne, and cutaneous atrophy. Her abdominal exam reveals the following image. At this time, she refuses any surgical interventions. Question Which of the following is the most appropriate pharmacologic treatment for this patient at this time? Answer Choices 1 Deltasone (Prednisone) 2 Pasireotide (Signifor) 3 Ketoconazole (Nizoral) 4 Sandostatin (Octreotide) 5 Declomycin (Demeclocycline)

Pasireotide (Signifor) Explanation This patient is demonstrating signs and symptoms consistent with Cushing's syndrome. Pasireotide (Signifor) is a somatostatin analog that binds and activates human somatostatin receptors, resulting in inhibition of ACTH secretion, which leads to decreased cortisol secretion. It is indicated for treatment of adults with Cushing disease in whom pituitary surgery is not an option or has not been curative. Treatment for exogenous Cushing syndrome is gradual withdrawal of glucocorticoid; therefore, treatment with prednisone is inappropriate. Ketoconazole has been the most popular and effective of these agents for long-term use and has been the agent of choice. However, the FDA has issued a warning that states clinicians should no longer prescribe ketoconazole, except to treat some life-threatening fungal infections; this is due to increased risk for severe liver injury, adrenal insufficiency, and adverse drug interactions. Agents that decrease CRH or ACTH release have been studied for the treatment of Cushing disease. Such agents include bromocriptine, cyproheptadine, valproic acid, and octreotide. Currently, use of these agents is investigational. Demeclocycline is not indicated in the treatment of Cushing syndrome; it is used in the treatment of syndrome of inappropriate ADH secretion refractory to furosemide and water restriction.

Which of the following is one standard of care to treat a first-time deep vein thrombosis (DVT) without pulmonary embolism (PE)? A Begin the patient on warfarin 5 mg PO once daily and check an INR in five days and adjust to therapeutic levels continuing warfarin for six months. B Begin the patient on enoxaparin 1 mg/kg SQ QD while also starting warfarin 5 mg PO once daily and check INR in five days and adjust to therapeutic levels continuing warfarin for six months and enoxaparin until therapeutic on the warfarin. C Begin the patient on enoxaparin 2 mg/kg SQ QD while also starting warfarin 5 mg PO once daily and check INR in five days and adjust to therapeutic levels continuing warfarin for six months and enoxaparin until therapeutic on the warfarin. D Begin the patient on heparin sodium IV and monitor and adjust levels to achieve therapeutic levels Q six hours and transition to warfarin as soon as possible. E Initiate a hypercoaguability work up and treat accordingly.

Patients with deep vein thrombosis (DVT) without pulmonary embolism (PE) do not necessarily require hospitalization. Outpatient therapy would include treatment with a low molecular weight heparinoid (LMWH), such as enoxaparin, subcutaneously while the patient becomes therapeutic on oral warfarin. The LMWH can be discontinued once therapeutic on warfarin, with continued monitoring of INRs for six months before considering discontinuation.

A 48-year-old woman presents complaining of vaginal fullness and difficulty passing stool. Upon exam she is found to have a stage 3 rectocele by the Baden Walker System. What is the most common cause for this? A Increasing age B Genetic disposition C Obesity D Pelvic floor injury E Constipation

Pelvic floor injury D While all of the answer choices are risk factors, the most common cause of pelvic organ prolapse remains pelvic floor injury, usually related to child birth or trauma.

A 37-year-old gravida 4, para 1, whose last normal menstrual period was 6 months ago, presents with 5-month history of irregular menses. She states her menstrual period is now 10 days in length. She uses approximately 10-12 pads or tampons daily during menses. She also has unpredictable bleeding between menstrual periods. This bleeding may begin anywhere from 2-12 days after the end of her regular period, and may last for between 2-15 days. She uses between 5-10 pads or tampons a day during this time. She denies syncope but admits to headaches and occasional dizziness. She also complains of bilateral pelvic pain that is present constantly and that radiates down both thighs. The pain is crampy, and it is relieved to some degree by naproxen sodium. On exam, the patient's vital signs are within normal limits. Abdominal exam demonstrates normal bowel sounds. There is a globular and irregular pelvic mass that extends about 5 cm above the umbilicus. The mass is nontender. There is no rebound or guarding. The pelvic exam reveals scant blood in the vaginal vault, no vaginal or cervical lesions, no cervical motion tenderness, and confirms the presence of the pelvic mass. Question What is the next best step in management for this patient? Answer Choices 1 Dilation and curettage 2 Pelvic ultrasound 3 CT scan of abdomen and pelvis, with and without contrast 4 Follicle-stimulating hormone (FSH), luteinizing hormone (LH), prolactin 5 Total and free testosterone

Pelvic ultrasound Explanation The patient's age and her history of menometrorrhagia, pelvic pain, and a globular, irregular pelvic mass are strongly suggestive of leiomyomata uteri. Fibroids are the most common solid pelvic tumors in women. The best study to evaluate the pelvic mass is ultrasound. Dilation and curettage is not indicated at this time. The patient should have an endometrial biopsy performed, because she is over the age of 35 and is experiencing abnormal vaginal bleeding, though it is probably due to leiomyomata uteri. If an endometrial biopsy is unsuccessful, then hysteroscopy and dilation and curettage would be indicated. CT scan of the abdomen and pelvis, with and without contrast, is not indicated at this time. If ultrasound exam were suspicious for malignancy, a CT scan of the abdomen and pelvis would be indicated to evaluate further the mass as well as to determine the extent of any lymphadenopathy. Hormone studies (FSH, LH, prolactin, free and total testosterone) are not generally useful in the diagnosis of a pelvic mass. They may be of use in a patient with abnormal vaginal bleeding whose pelvic exam does not reveal any masses.

A 58-year-old woman with no significant past medical history presents with a 6-month history of "heartburn", sometimes occurring after meals. There is associated fatigue, bloated abdominal sensation, early satiety, and alternating constipation and diarrhea. She denies fever, chills, changes in weight, chest pain, shortness of breath, abdominal pain, nausea, vomiting, melena, hematochezia, and vaginal discharge. Her last menstrual period was 4 years ago. She is unmarried and does not have any children. Her physical exam reveals normal vital signs and a normal cardiopulmonary exam. Her abdomen is protuberant; there is a shifting dullness and a fluid wave noted. The pelvic exam reveals a solid, irregular fixed lesion in the left lower abdomen. Question What is the most appropriate initial intervention for this patient at this time? Answer Choices 1 Perform a transvaginal ultrasound 2 Prescribe a proton pump inhibitor 3 Refer the patient for an upper endoscopy 4 Prescribe medication for irritable bowel syndrome 5 Observation and reassessment in 1 to 3 months

Perform a transvaginal ultrasound Explanation This patient's presentation suggests ovarian cancer. Doppler transvaginal ultrasonography is used in the initial evaluation, and it is the most useful initial investigational tool in the assessment of adnexal masses. Ultrasonography may define the morphology of the pelvic tumor. In addition, it can determine whether the suspected tumor has metastasized to other abdominal organs, such as the liver. Sonographic features suggestive of cancer include complexity with solid and cystic areas, extramural fluid, echogenicity, wall thickening, septa, and papillary projections. Proton pump inhibitors and irritable bowel therapeutics should not be initiated without adequate investigation into the presence of abdominopelvic malignancy. In patients with diffuse carcinomatosis and gastrointestinal symptoms, a GI tract workup and endoscopy may be indicated as an adjunctive modality to a transvaginal sonogram.

A 40-year-old woman presents for a consultation. She takes a multivitamin, iron, and calcium as advised. She has never smoked cigarettes and does not drink alcohol at all. After she gave birth to her fourth child about 7 years ago, she underwent bilateral tubal ligation because both she and her husband decided not to have more children. Afterwards, she started gaining weight (now her BMI is 30). In the last several months, her periods have come every 2 weeks and last about 5-7 days. Before that they were regular: every 28-30 days, with 3-5 days of bleeding. Her mother died from breast cancer at age 70. Physical examination reveals pallor and tachycardia. Pelvic examination is normal. Question What is the next best diagnostic step? Answer Choices 1 Prescribe oral contraceptive pills 2 Check FSH 3 Perform endometrial biopsy 4 Schedule mammogram 5 Check LH

Perform endometrial biopsy Explanation Your patient most probably has dysfunctional uterine bleeding (DUB). DUB is irregular uterine bleeding that occurs in the absence of pathology or medical illness. It is a diagnosis of exclusion. Laboratory studies for patients with DUB include human chorionic gonadotropin (HCG), complete blood count (CBC), Pap smear, endometrial sampling, thyroid functions and prolactin, liver functions, coagulation studies/factors, and testing for the presence of uterine fibroids or polyps and hormone assays when indicated. Most important, however, is to exclude endometrial hyperplasia or carcinoma because your patient has several risk factors: she is older than 35 years; she is obese; and she has prolonged periods of unopposed estrogen stimulation and probably chronic anovulation. Endometrial biopsy is the most important and most commonly used diagnostic test for DUB in such a patient. In DUB there is usually a hormonal imbalance (too much estrogen thickens endometrium, and progestrone causes excessive irregular bleeding). The effect of oral contraceptive pills on endometrium mimics that of an ovulatory cycle. However, before you start oral contraceptive pills, you should confirm the diagnosis of DUB, which means excluding the other reasons. Follicle-stimulating hormone (FSH) testing may be ordered when a woman's menstrual cycle has stopped or become irregular to determine if the woman has entered menopause. That is not the case in your patient. Mammography is indicated in this woman. The National Cancer Institute recommends that women age 40 or older should have screening mammograms every 1 to 2 years. However, it is not the most important test you should order at this point; instead, you should find a reason for bleeding because of all the consequences of losing blood and because of the risk of endometrial cancer. Checking luteinizing hormone (LH) will be indicated in women having difficulties getting pregnant, having irregular or heavy menstrual periods, or having symptoms of pituitary, hypothalamic, or ovarian disorders. Again, endometrial biopsy done in the office is probably more important.

You provide care to a 37 kg, 10-year-old boy with Duchenne muscular dystrophy, which was diagnosed 4 years ago. For the past 18 months, he has been treated with prednisolone (0.75 mg/kg/day). At this time, he has limited ambulation, and tendon release surgery is planned. What regarding his perioperative care is true? Answer Choices 1 Forced vital capacity <30% is an absolute contraindication to surgery 2 Optimal anesthesia is delivered with inhalational anesthesia drugs, such as halothane, enflurane, isoflurane, methoxyflurane, sevoflurane, or desflurane 3 Peri-operative steroid coverage with hydrocortisone should be started 1 - 3 days prior to the planned surgery 4 Preoperative echocardiography is a reliable determinant of intraoperative cardiac function 5 Succinylcholine is an appropriate medication for muscle relaxation during surgery

Peri-operative steroid coverage with hydrocortisone should be started 1 - 3 days prior to the planned surgery The rationale for stress steroid coverage is that chronic corticosteroid therapy suppresses the hypothalamic-pituitary-adrenal (HPA) axis, although this has never been well-documented in studies. Patients who are not taking glucocorticoids activate HPA axis with surgery, trauma, and severe illness. Adults who do not experience stress secrete cortisol equivalent to 5 mg/day of prednisone or 20 mg hydrocortisone/day. Generally, the recommended dose has been 3 to 5 times the amount secreted daily, which is up to 200 - 400 mg/day of hydrocortisone. More recent studies suggest that lower doses may be used. Suggested lower doses for major surgery, such as cardiopulmonary bypass, are up to 100 - 150 mg/day hydrocortisone for 2 - 3 days, and 75 - 100 mg/day for moderate procedures, such as orthopedic surgery, for 1 - 2 days. Even when a patient has taken glucocorticoids in the past and has stopped, the HPA axis may recover quickly or not fully recover for 9 - 12 months. Succinylcholine, a depolarizing agent, may induce increased muscle contractions and may cause rhabdomyolysis. With muscle breakdown, creatine phosphokinase, potassium, and myoglobin may be released. The large proteins of the myoglobin may cause renal damage. Elevated potassium levels may cause cardiac arrest. Both succinylcholine and inhalational anesthetics may cause a malignant hyperthermia-like response. High intracellular calcium levels lead to activation of actin and myosin fibers with continuous contractions of the muscles producing muscular rigidity and spasm. A hypermetabolic state occurs in the muscle cells resulting in increased oxygen consumption and metabolic acidosis. Respiratory acidosis may occur with elevated CO2 levels. As a result of the hypermetabolic state, hyperthermia occurs, along with tachycardia and possible cardiac arrhythmia. Nitrous oxide is a safe inhalational agent. Also safe are non-depolarizing muscle relaxants, including pancuronium, cisatracurium, vecuronium, rocuronium, atracurium, and mivacurium. Echocardiography should be done pre-operatively in all patients with Duchenne muscular dystrophy. In the past, patients commonly died from respiratory problems. As care for these conditions has improved, dilated cardiomyopathy has become an increasingly common morbidity and cause of death in these patients. The time course for development of cardiomyopathy is not clearly defined. There may be limited ability to increase cardiac output, with stress resulting in impaired oxygen delivery. Clinical symptoms of cardiac dysfunction may be subtle, vague or unrecognized, as these children have limited physical activity. Scoliosis may limit the usefulness of echocardiography, making it difficult get appropriate windows. In addition, echocardiography may not detect the myocardium's ability to respond to stress. It has been suggested that pre-operative stress echocardiography may be a more reliable test. Intraoperative transesophageal echocardiography, pulmonary arterial catheterization, and pulse contour analysis (PCCO) may be of great benefit for intraoperative monitoring. There is evidence that specific dystrophin gene mutations may be predictive of cardiomyopathy, while other mutations may be protective or inhibitory toward its development1. Forced vital capacity should be determined before surgery. There are no absolute contraindications to surgery based on pulmonary function. Good results have been reported in patients with FVC 20% of predicted, although best prognosis for recovery seems to be with FVC >40% predicted. Pre-operative sleep studies or nocturnal oximetry can be helpful if abnormal, noninvasive nighttime ventilation can be instituted prior to surgery.

A 6-year-old girl presents because her teacher is concerned about behaviors at school. The teacher has noticed the girl "staring off into space" frequently throughout the day, and the teacher is able to get her attention only occasionally. There are also periods when she appears to be talking to herself, but there is no sound coming from her mouth. The girl's mother states that there are times when the patient does not seem to be paying attention to what the mother is saying. Shortly after these "episodes", the child engages in conversation without any problem; therefore, the mother did not think the episodes were an issue. There is no concern about other abnormal behavior or discipline issues at home or at school. Question What is the most likely diagnosis? Answer Choices 1 Attention-deficit disorder (ADD) 2 Syncopal episodes 3 Petit mal (absence seizures) 4 Narcolepsy 5 Childhood disintegration disorder (CDD)

Petit mal (absence) seizures The patient is probably experiencing petit mal (absence) seizures. Absence seizures are a form of generalized seizures seen in children, and they usually cease by age 20. They are characterized by abrupt onset of impaired consciousness and can be associated with enuresis or automatisms. Patients often appear as though they are "staring off into space" during the seizure, and the episodes often terminate as quickly as they came. These children are often categorized as "being in their own world", leading to a delay in diagnosis or a misdiagnosis of attention deficit disorder. These types of seizures are diagnosed using an electroencephalogram (EEG), showing crusts of bilaterally synchronous and symmetric 3-Hz spike-and-wave activity. Attention-deficit disorder (ADD) is a form of Attention-deficit hyperactivity disorder (ADHD) that is characterized more by inattentiveness without the presence of restless or impulsive behavior. This is the most common emotional, cognitive, and behavioral disorder treated in children/adolescents. The diagnosis is made by careful clinical history, revealing a significant level of inattentiveness and distractibility, plus-or-minus impulsivity, and hyperactivity that is inappropriate for the developmental stage of the child. These patients often change activities frequently, have a hard time with organizational skills, and are commonly caught daydreaming. The symptoms of ADD are usually pervasive, even though they may not occur in all settings. ADD/ADHD also does not involve any impairment of consciousness. Syncopal episodes are better known as "fainting spells." These are episodes of diminished or complete loss of consciousness accompanied by flaccidity that usually occur in relation to postural change, emotional stress, instrumentation, pain, straining, cardiac arrhythmias, or other physiologic disturbance. Associated signs/symptoms include pallor, sweating, nausea, and malaise. While this could be in the differential diagnosis for our patient, the cause of the syncopal episode would still need to be determined by conducting a battery of tests. In addition, our patient is not noted to lose postural tone during her episodes; this can distinguish her presentation from true syncope. Narcolepsy is a condition that is associated with "sleep attacks" that are uncontrollable and occur in inappropriate and occasionally dangerous situations. Most of these attacks are brief; however, this is not always true. Associated symptoms can include cataplexy (a sudden loss of muscle tone), hypnagogic hallucinations (dream-like experiences while falling asleep), and sleep paralysis. Childhood disintegration disorder (CDD) is a rare syndrome on the autism spectrum. It is characterized by normal development until the age of 2 that is followed by a major loss of skills, both social and verbal. Motor skills are often preserved. The presentation is often very abrupt and very dramatic. This differs greatly from the child presented above; she has had a more gradual onset of symptoms that have almost gone "unnoticed".

A 21-year-old woman presents with a severe headache; it is accompanied by sweating and palpitations. She was seen by her primary care physician earlier in the day, and he admitted her from the office when her blood pressure was recorded several times at 210/98. The patient has no known medical or surgical history, and she is currently on a daily multivitamin. She states that she has been having her symptoms episodically over the last 2 weeks, but this is the first time she has been seen by a physician. The patient is found to have elevated urinary and plasma metanephrines and catecholamines. Question What antihypertensive agent should be used for this patient? Answer Choices 1 Lisinopril 2 Phenoxybenzamine 3 Metoprolol 4 Lasix 5 Hypertension should not be treated at this time

Phenoxybenzamine The correct answer is phenoxybenzamine. The patient's young age suggests that she is suffering from a form of secondary hypertension. Given her paroxysmal headaches, diaphoresis, and palpitations accompanied by hypertension, she is likely suffering from a pheochromocytoma. Pheochromocytomas are benign tumors that arise from the adrenal glands. The tumors secrete norepinephrine, which leads to hypertension, tachycardia, headache, and diaphoresis. Diagnosis is made based on elevation in urinary and plasma metanephrines and catecholamines. and it is followed by tumor location on CT or MRI. The patient requires removal of the pheochromocytoma, but preoperative control of her blood pressure is necessary. Alpha blockers, such as phenoxybenzamine or prazosin, are the preferred antihypertensive agents in pheochromocytoma. Lisinopril is an ACE inhibitor and would not be considered first-line treatment in the control of hypertension in pheochromocytoma. Metoprolol is a beta blocker. Beta blockers may only be used in the treatment of pheochromocytoma after the patient has received an alpha blocker. If a patient with pheochromocytoma receives a beta blocker as first-line treatment, the patient will develop unopposed alpha agonism, causing worsened hypertension. Lasix is a diuretic and is not recommended as first-line treatment in the control of hypertension in pheochromocytoma. Hypertension should not be treated at this time is an incorrect answer; the patient's blood pressure is severely elevated and she requires control of her blood pressure prior to the removal of her pheochromocytoma.

A new patient is in your office for a routine physical. He informs you that he has a genetic disease. Upon questioning he tells you that it was identified at birth and its complications were minimized with a specific diet. He asks if it is okay to drink diet soda drinks. You advise him against consuming foods containing aspartame. You do this because he has what disease? Answer Choices 1 Lupus erythematosus 2 Type I diabetes mellitus 3 Maple syrup urine disease 4 Phenylketonuria 5 Alkaptonuria

Phenylketonuria Aspartame is a commercially synthesized dipeptide used as an artificial sweetener in many foods, including soft drinks. The structure of aspartame isL-aspartyl-L-phenylalanyl methyl ester. This can be broken down in the body to release phenylalanine. The disease phenylketonuria (PKU) results from decreased levels of the enzyme phenylalanine hydroxylase that is part of the phenylalanine degradation pathway. Untreated, PKU leads to mental retardation. Appropriate intervention with a very low phenylalanine diet, especially in the very young, can prevent the retardation greatly. Maple syrup urine disease is another disease that results from a defect in an enzyme involved in amino acid degradation, in this case that of the aliphatic amino acids leucine, isoleucine, and valine. The block is in the oxidative decarboxylation (α-keto acid dehydrogenase is the enzyme) of these amino acids. Similarly, alkaptonuria also results from a defect in an enzyme involved in an amino acid degradation pathway. This is due to a defect in the enzyme homogentisate oxidase. Foods containing aspartame are often recommended for patients with diabetes, since it is a way to have something that tastes sweet, but does not contain sugar.

A 32-year-old woman presents with paroxysmal episodes of headache, palpitations, and diaphoresis; the episodes last 20 minutes. During 1 episode, her blood pressure was checked and found to be 194/110 mm Hg. These episodes can occur with activity or at rest and she denies known precipitating events. On examination, blood pressure is 120/88 mm Hg, pulse is 82/min, temperature 98.6° F, and respirations 14/minute. Physical assessment, including cardiovascular and neurologic examination, is unremarkable. What is the most likely diagnosis? Answer Choices 1 Hyperthyroidism 2 Anxiety attack 3 Pheochromocytoma 4 Renal artery stenosis 5 Coronary artery disease

Pheochromocytoma Explanation Headache, palpitations, and diaphoresis are the classic triad of a pheochromocytoma crisis. These catecholamine-secreting tumors, most commonly found in young to mid-adult women, present with paroxysmal episodes in over 50% of patients. The episodes are of sudden onset, last minutes to hours, and they sometimes occur as a result of activity affecting the abdomen; often, they occur without a clear precipitating event. Hypertension with tachycardia occurs in most patients, presenting only during attacks in 40% of those patients, while the remaining 60% have sustained hypertension with half of those patients also experiencing hypertensive crises. In the absence of sustained hypertension or a paroxysmal episode, physical exam is often unremarkable. Diagnosis is made based on a 24-hour urine specimen for vanillylmandelic acid (VMA), metanephrines, and free catecholamines. Urine collection should occur when the patient is hypertensive or initiated at the time of a paroxysmal crisis. Associated symptoms during a paroxysmal episode may include apprehension or a sense of doom suggesting an anxiety attack. Although anxiety attacks present as recurrent, unpredictable episodes of intense fear that can be associated with palpitations and sweating, they lack the malignant hypertensive response of a pheochromocytoma. A pheochromocytoma crisis may precipitate chest pain with nausea, vomiting, pallor, and arrhythmias. The catecholamine surge, rather than coronary artery disease, may induce myocardial ischemia, resulting in angina and myocardial infarction. Signs and symptoms of increased metabolic rate associated with a pheochromocytoma such as weight loss, palpitations, and sweating may also suggest hyperthyroidism. The most common cardiac manifestation in thyrotoxicosis is an arrhythmia, such as sinus tachycardia or atrial fibrillation in patients > 50 years of age, and not the malignant hypertension associated with a pheochromocytoma. Renal artery stenosis presents as an abrupt onset of hypertension, malignant and/or refractory, typically associated with abdominal and/or flank bruits. When it occurs in younger women, it is most often the result of fibromuscular dysplasia of the arterial wall. When the onset occurs at > 50 years of age, it is most often the result of atherosclerotic plaques.

A 73-year-old, male with a 30 pack-year smoking history presents to the clinic with complaints of headache and dizziness. Labs reveal a hemoglobin of 21.3 g/dl, hematocrit of 63%, and platelet count of 498,000. He tests positive for the JAK2 mutation. Which of the following is the most appropriate treatment for the patient's suspected diagnosis? A Allopurinol B Anagrelide C Diphenhydramine D Hydroxyurea E Phlebotomy

Phlebotomy E The first-line, and primary treatment for polycythemia vera is chronic phlebotomy. Patients who fail to improve or don't tolerate chronic phlebotomy can be treated with hydroxyurea (D), with anagrelide (B) being a third-line treatment. Allopurinol (A) may be necessary to treat hyperuricemia and gout. Diphenhydramine (C) may be used as symptomatic treatment for pruritus that doesn't improve with phlebotomy.

A 15-year-old boy presents for a high school basketball physical. After his mother leaves the room, he asks if you can 'take a look at something'. He then lifts his shirt and tells you that he is developing 'man-breasts'. He is very embarrassed. There is firm, mobile, tender tissue palpable under both nipples. This tissue is approximately 75 mm in size on the right side and 1 cm on the left. There is no nipple discharge on either side. The patient's height and weight are 66 inches and 125 lbs (BMI 20.2). Question What is the most important next step in the diagnostic workup? Answer Choices 1 Drug screen and careful medication history 2 Liver function tests and abdominal CT scan 3 Mammogram 4 Physical examination (especially genitalia) 5 Thyroid function tests

Physical examination especially genitalia Explanation The correct response is a physical examination (especially genitalia). Gynecomastia may be either physiologic or pathologic. Physiologic gynecomastia has normal peaks in the neonatal period and in adolescence due to a relative increase in estrogen. Age-related gynecomastia (usually between 50 and 70 years of age) results from an increase in fatty tissue and decrease in hormone levels. Pseudogynecomastia (usually seen in obese men) is enlarged breasts without a palpable mass. The most reassuring aspect of this patient's gynecomastia is his age. If his symptoms were occurring outside the neonatal or adolescent period, or if they were associated with physical or laboratory abnormalities, a more aggressive workup would be warranted. The examiner should pay particular attention to the signs of puberty, particularly testicular size, in order to rule out conditions such as Klinefelter's syndrome (which may present with gynecomastia). Therefore, the most important next step in this patient's workup is a thorough physical examination; this will ensure normal puberty and rule out a testicular mass as a cause of a hormone imbalance and gynecomastia. Physiologic gynecomastia generally resolves within 2 years without any treatment. Both prescription and illicit drugs can cause gynecomastia. Common offenders include some antidepressants, antibiotics, medication used to treat HIV or heart disease, marijuana, anabolic steroids, heroin, and chronic alcohol abuse. Asking about medications and drug use would certainly be an important part of the medical history in the workup of gynecomastia. Liver disease (cirrhosis) is a very common cause of gynecomastia. Liver disease causes impaired metabolism of androstenedione, a precursor of estrogen and testosterone. In addition, patients with liver disease may be treated with spironolactone, a medication that is known to cause gynecomastia. This patient has no history or signs of liver disease. A mammogram would not be indicated in this patient because the breast changes are bilateral and there is a lack of concerning features (e.g., a firm, discrete mass, bloody nipple discharge, and/or skin changes). Breast cancer is rare in men, and this patient likely does not have pathologic gynecomastia. Thyroid disease (both hypo and hyper) can cause gynecomastia. In hyperthyroidism, this is a result of an increased conversion of androgen to estradiol. Hypothyroidism results in a lower level of testosterone with a resulting increase in estradiol production. This particular patient has no signs of symptoms of thyroid disease.

A 54-year-old woman presents with nervousness, a tremor, and irritability. She indicates that she has lost some weight over the last few weeks even though her appetite has increased. She also says that she is feeling feverish. You suspect that she may have a thyroid condition and order the appropriate labs. The lab results are as follows: TSH 7.8 (0.5 - 5.5 mU/L) T4 4.2 (0.8 - 1.8 ng/dl) T3 5.3 (1.4 - 4.2 pg/ml) Question What is the most likely diagnosis? Answer Choices 1 Thyroid storm 2 Pituitary adenoma 3 Graves' Disease 4 Hashimoto's thyroiditis 5 Iodine deficiency

Pituitary Adenoma Explanation The clinical picture is of a pituitary adenoma. The diagnosis is suggestive by clinically evident hyperthyroidism with elevated T4 and T3 and elevated serum TSH levels. Untreated hyperthyroidism may progress into a thyroid storm. Patients may present with tachycardia, fever, agitation, nausea, vomiting, diarrhea, and restlessness or psychosis. Graves' disease would produce decreased TSH and elevated T4 and T3 levels. Hashimoto's thyroiditis would generally produce elevated TSH and decreased T4 and T3 levels. In iodine deficiency or goiter, there is diffuse enlargement of the gland as well as elevated TSH and decreased T4 levels.

A 40-year-old woman presents with anxiety, difficulty sleeping, rapid heartbeat, and a tremor in her hands for the past 3 months. You note the presence of bulging eyes and suspect Graves' disease. You confirm that she has this disease by demonstrating the presence of thyroid stimulating immunoglobulin (TSI), which mimics the action of thyrotropin; you also use thyroid function tests. What gland releases thyrotropin hormone? Answer Choices 1 Adrenal gland 2 Pancreas 3 Pituitary gland 4 Thyroid gland 5 Hypothalamus

Pituitary Gland Explanation Thyrotropin is released by the pituitary gland. The pituitary gland releases thyrotropin/Thyroid Stimulating Hormone (TSH) in response to insufficient levels of thyroid hormone. Communication between the pituitary gland and the thyroid gland through TSH levels controls the levels of thyroid hormone in the blood. If the levels of thyroid hormone are low, then the levels of TSH will rise. The measurement of TSH in the blood is taken as a measure of thyroid function. High levels of TSH would indicate low levels of thyroid hormone. Thyrotropin hormone is not released by pancreas, hypothalamus adrenal, or the thyroid gland. Graves' disease is a type of hyperthyroidism caused by a generalized overactivity of the entire thyroid gland. The patient appears hot and flushed, and the thyroid gland enlarges in this condition. It is believed that Graves' disease is an autoimmune disorder. Antibodies are produced against 4 thyroid antigens, namely thyroglobulin, thyroid peroxidase, sodium-iodide symporter, and the thyrotropin receptor. The antibody against the thyrotropin receptor-TSI is the principal autoantibody of Graves' disease responsible for the manifestations of hyperthyroidism. Autoantibodies are produced against certain proteins on the surface of thyroid cells, stimulating those cells to overproduce thyroid hormones, resulting in the overactive thyroid. The onset of the disease is gradual, and the symptoms may be mistaken for nervousness due to a stressful life situation. Weight loss occurs, and this is followed by other symptoms such as trembling, muscle weakness of the upper arms and thighs, and insomnia. Thyroid function tests and the presence of TSI help in establishing the diagnosis of Graves' disease.

A 35-year-old male presents complaining of increasingly constant headaches, double vision centrally, and a progressive loss of peripheral vision for two weeks. He has no previous headache history and denies any other medical conditions. Physical examination reveals bitemporal hemianopsia without additional neurologic findings. What is the most likely diagnosis? A Acute ischemic stroke B Circle of Willis ruptured aneurysm C Migraine headache D Multiple sclerosis E Pituitary adenoma

Pituitary adenoma Pituitary adenomas, benign neoplasms associated with pituitary hormone secretory changes, may enlarge and become symptomatic. Symptoms are based upon the location and size of the tumor, and may include bitemporal hemianopsia, double vision, color desaturation, and visual acuity loss. Headaches may occur, due to associated pressure changes within the intrasellar space. Additional evaluation should include a T1-weighted MRI, screening laboratory tests, and a full ophthalmologic evaluation. These tests will also help evaluate for potential differential diagnoses, such as those listed. The patient's history is not consistent with an acute ischemic stroke or migraine headache. Although an unruptured aneurysm may have very similar findings to a pituitary tumor, ruptured aneurysms present with acute headache, nausea, vomiting, and potential changes in consciousness. Multiple sclerosis (MS) should remain on the differential for this patient and will also be evaluated through MRI (although the current findings are more consistent with a pituitary adenoma), and additional neurologic findings would be likely with MS.

A 40-year-old female presents to your office with symptoms of weight gain, hirsuitism, and easy bruising. Past medical and surgical history is noncontributory. She drinks one glass of wine on weekends and does not smoke cigarettes. She takes one multivitamin daily. Upon physical exam, you note facial fullness, central obesity, and thin skin. Which of the following is the most common cause of her symptoms? A. Pituitary Adenoma B. Iatrogenic C. Adrenal Micronodular Hyperplasia D. Adrenocortical adenoma E. Ectopic ACTH syndrome

Pituitary adenoma The correct choice is A, pituitary adenoma. This patient's clinical presentation is typical in Cushing's syndrome. The most common cause of Cushing's syndrome (other than ingestion of oral steroid medications) is Cushing's disease. This disease is caused by a benign, ACTH secreting pituitary adenoma. Choice B, iatrogenic, refers to the ingestion of prescribed (or non-prescribed) oral corticosteroid medications. This is frequently seen in patients requiring long-term oral steroid medications. This patient does not have this type of history. Choice C, adrenal micronodular hyperplasia, and choice D, adrenocortical adenoma, can cause Cushing's syndrome at less frequent incidence. Choice E, ectopic ACTH syndrome, presents more commonly in males with extremely elevated levels of plasma cortical and ACTH. These patients commonly have a positive history of an ectopic source of the ACTH, such as in small cell carcinoma of the lung.

A 23-year-old primigravida is referred to her obstetrician by a community nurse. The nurse noted 2 blood pressure readings of 150/90 and 154/90 taken 15 minutes apart. Physical examination reveals a uterus at roughly 24 weeks gestation and grade 2 pitting edema. Urine analysis is 1+ positive for albumin. A sonogram at eight weeks gestation showed a single live intra-uterine gestation. She has had regular antenatal check-ups, and has no past history of hypertension or diabetes. Both her parents and grandparents are hypertensive. She is unaware of whether her mother or grandmother faced similar problems during their pregnancies. Question What is the most likely initiating event for her condition? Answer Choices 1 Over activation of her renin-angiotensin system 2 Overproduction of B-HCG 3 Placental ischemia 4 Renal artery stenosis 5 Essential hypertension

Placental ischemia Placental ischemia is the correct answer. The vignette describes a woman with classic features of pregnancy induced hypertension, as evidenced by the blood pressure readings, edema, time of onset beyond 20 weeks and proteinuria. The initiating event appears to be abnormal cytotrophooblast invasion of the spiral arterioles, which leads to activation of the maternal vascular endothelium. The cascade that follows includes enhanced formation of endothelin and thromboxane, increased vascular sensitivity to angiotensin II, reduced nitric oxide, and prostacyclin (vasodilator) synthesis. Over activation of her renin-angiotensin system is incorrect. While it is an important component of the response, it does not initiate the process. Overproduction of B-HCG is incorrect. It may result in high blood pressure as seen in hydatiform mole and multiple gestations. However, there is no indication of either in this vignette. Renal artery stenosis is incorrect. It is a common cause of hypertension in young individuals. It would be present through the duration of pregnancy. Essential hypertension is incorrect. Essential hypertension is a pathology associated with aging, the pathogenesis of which is unclear. It is certainly unlikely in a 23-year-old. In addition, the patient would have a past history of hypertension.

A 30-year-old woman presents with a 2-day history of fever, cough with sputum, and chest pain; there is also a 1-day history of frank blood in sputum (75 cc in the last 24 hours). She is a non-smoker and does not give a history of any recent inhalation exposure or illicit drug use. There is no past history of malignancy or autoimmune disease. On examination, pulse is 92/min; BP is 106/70mmHg; and temperature is 101 F. Oxygen saturation is 97%. On auscultation, S1, S2 normal, decreased breath sounds, and increased tactile vocal fremitus are noted on the right middle lobe. There is no pallor, icterus, cyanosis, edema, or lymphadenopathy. Question What is the next best step in evaluation of her hemoptysis? Answer Choices 1 Computed Tomography (CT) scan of chest 2 Sputum for Gram stain 3 Plain X-ray of chest 4 Bronchoscopy 5 Complete blood count (CBC)

Plain X-ray of chest Plain X-ray of chest is the correct answer. It is the next step in the evaluation of hemoptysis to identify the source of bleeding1. If the chest X-ray does not help identify the source of bleeding, then a CT scan of the chest should be done1. CT scan helps to identify the source of bleeding, as well as gives information on mediastinal lymhadenopathy, which may support the diagnosis of thoracic malignancy1. Both Gram stain and CBC should be obtained for complete evaluation1. Gram stain helps evaluate for the presence of infection, and CBC is used to assess platelet count and hematocrit1. These investigations should be done following chest X-ray. If all these procedures are unrevealing, then bronchoscopy should be considered1.

A mother brings her 2-year-old boy for the evaluation of frequent febrile urinary tract infections (UTIs); the infections have been present since the birth. He has just finished a 10-day prescribed trimethoprim/sulfamethoxazole (Bactrim) course, and he now has no problems. The mother thinks that main reason for the frequent UTIs is because a boy is neither interested nor willing to use the toilet; he is almost always wet. Physical examination today is unremarkable. He is not circumcised. You ordered dipstick urine analysis and perform ultrasound that came back normal. Question What will be your next step? Answer Choices 1 Plan voiding ureterocystogram 2 Order antibiotics prophylaxis 3 Arrange circumcision 4 Address dysfunctional voiding 5 Perform radionucleotide cystography

Plan voiding ureterocystogram Explanation Urinary tract infections (UTIs) are common in children and may cause permanent kidney damage. Urinary tract anomalies are risk factors for UTIs, and you should search for them in a boy of this age. While voiding cystourethrography (VCUG) is not recommended routinely after the first UTI, it should be performed if there is a recurrence, particularly a recurrence of febrile UTIs. Kidneys and bladder ultrasound are performed in younger children (3 - 5 years of age) as the initial step to evaluate anatomy, but they cannot relieve vesicoureteral reflux (VUR). Recommendations regarding antimicrobial prophylaxis still lack evidence, both for and against regardless, in this case, your approach should be based on the diagnosis of the presence of eventual urinary tract anomalies. You should tell the patient's mother that circumcision reduces UTIs, especially in high-risk boys; however, it is more important to exclude the presence of anatomical abnormalities, which put this boy at an even bigger risk. Voiding dysfunction is defined as daytime voiding disorders in children who do not have neurologic, anatomic, obstructive, or infectious abnormalities of the urinary tract. It is recommended that toilet training begin when a child is 18 months old and shows the interest. A child's interest usually appears around 24 - 25 months. Daytime dryness is usually achieved by 3 years of age. Dysfunctional voiding can lead to VUR, accidental urinary leakage, and UTIs; however, it is too early to think about the presence of dysfunctional voiding in this child. Both radionuclide cystography and voiding cystourethrography are used in detecting and grading vesicoureteral reflux. While VCUG is suggested for both girls and boys, radionuclide cystography is suggested only for girls because voiding cystourethrography is needed for adequate anatomic imaging of the urethra and bladder in boys.

A 55-year-old woman was diagnosed with small cell cancer of the lung 2 months ago; she presents with increasing dysphagia, respiratory difficulties, and weakness of the upper limb. Her vital signs are: pulse 85/min, BP 120/90 mm Hg, resp. 12/min, and temp. 37.7 C. On examination, she has ptosis of both eyes, and she reports diplopia. Her pupillary responses are normal. The strength in the muscles of her arm on testing is 2/5; on repeated testing, the strength improves to 4/5. Sensation is intact in both upper limbs. What is the most appropriate treatment for her condition? Answer Choices 1 Neostigmine 2 Atropine 3 Plasmapheresis 4 Amikacin 5 Cisapride

Plasmapheresis The Lambert-Eaton syndrome is caused by antibodies to the presynaptic calcium channel, which decreases the release of acetylcholine. Repeated stimulation of the nerve ending increases the intracellular calcium concentration, which allows enough acetylcholine to be released to cause muscle contraction. Anticholinergics would not help in this condition. The treatment lies in removing the circulating antibodies using plasmapheresis or immunosuppressants, such as prednisone or azathioprine. The Lambert-Eaton (or myasthenic syndrome) may occur as a paraneoplastic syndrome of small cell lung cancer. The presentation is similar to myasthenia gravis; however, the weakness is made worse by sustained movement in myasthenia gravis. It is improved in the Lambert-Eaton syndrome. Myasthenia gravis is caused by antibodies to the acetylcholine receptor; consequently, the treatment lies in increasing the acetylcholine concentration in the synaptic cleft using an anti-cholinergic medication such as neostigmine. Amikacin is an aminoglycoside; it can cause weakness of skeletal muscles. Cisapride is a promotility agent use for dysphagia or gastro-esophageal reflux. It has been withdrawn from the US market, and it is available only to patients who meet specific criteria. Atropine is a cholinergic antagonist at muscarinic receptors and would not be useful in this case.

A 63-year-old female presents with deep bone pain in both legs. Laboratory evaluation reveals Bence-Jones proteinuria and a monoclonal spike on serum electrophoresis. Which of the following vaccines should this patient receive as part of her management? A Diphtheria B Human Papilloma Virus (HPV) C Pertussis D Pneumococcal E Varicella

Pneumococcal D Patients with multiple myeloma are at risk from infections, especially from encapsulated organisms such as Haemophilus influenzae and pneumococcus. Pneumococcal vaccines should be administered, but the patient response is decreased based on their current immunodeficiency. DTaP (A and C), HPV (B), and varicella (E) vaccines are appropriate for this patient based on general immunization guidelines for adults, but not of increased necessity based on her diagnosis.

You are examining a 6-month-old male infant who has not received any immunizations since birth. He was born at home, and he has suffered respiratory tract infections and chronic diarrhea since birth. His parents are not related. 2 of his older brothers died of pneumonia at the age of 8 and 12 months, respectively; however, his 4-year-old sister is healthy. His parents moved to United States from an underdeveloped country 3 months ago. They are worried because 1 of their neighbors' children, who is 6 years of age, developed a pruritic skin rash and fever 2 days ago; another child, who is 6 months of age, is in the hospital because of severe vomiting and diarrhea. The parents of your patient are worried. On examination, your patient's vitals are normal; his weight is <5th percentile, his height is at the 5th percentile, and his head circumference is at the 50th percentile. Eczematous skin rash and mucocutaneous changes suggestive to a fungal infection are noted. His thyroid, heart, lungs, abdomen, genitalia, and nervous system are within normal limits; you are not able to palpate lymph nodes, and there are no signs of hepatosplenomegaly. Question What vaccine should you recommend? Answer Choices 1 Rotavirus vaccine (RV) 2 Pneumococcal vaccine (PCV13) 3 Measles, mumps, and rubella vaccine (MMR) 4 Bacille Calmette-Guerin vaccine (BCG) 5 Variccella vaccine (Var)

Pneumococcal vaccine (PCV13) Your patient suffers recurrent respiratory infections and diarrhea; he also has skin changes and a family history highly suggestive of an X-linked immune deficiency, most likely a severe combined immunodeficiency (SCID). SCID is a group of congenital diseases caused by different genetic mutations resulting in the severe deficiency of T and B- lymphocytes. Onset before age 6 months suggests a T-cell defect, as maternal antibodies are present for the 1st 6 to 9 months. Live vaccines are contraindicated in SCID because introducing an infectious agent, even a weakened one, can cause actual disease. However, pneumococcal vaccine PCV13 is considered risk-specific and is recommended in a 4-dose series at ages 2, 4, and 6 months (minimum age of 6 weeks), as well as at age 12 through 15 months. Rotavirus (RV) is a live vaccine and cannot be given to SCID patients. The measles, mumps, and rubella (MMR) vaccine is a live vaccine and cannot be given to patients with SCID. BCG, or Bacille Calmette-Guerin, is not generally recommended for use in the United States because of the low risk of infection with Mycobacterium tuberculosis, the variable effectiveness of the vaccine against adult pulmonary TB, and the potential interference with tuberculin skin test reactivity. More importantly, in regard to this case, it is a live vaccine and should not be given to any immunosuppressed person. Variccella (Var) is also a live vaccine and cannot be given to SCID patients.

A 45-year-old man is on chemotherapy. He starts to notice that he is short of breath and has developed a nonproductive cough. Bronchial lavage confirms that he has an infection. What is the most likely pathogen? Answer Choices 1 Pneumocystis jiroveci 2 Giardia lamblia 3 Babesia microti 4 Trichophyton tonsurans 5 Trypanosoma cruzi

Pneumocystis jiroveci Pneumocystis jiroveci (formerly called Pneumocystis carinii) is a fungus that can cause pneumonia in an immunosuppressed host. It is transmitted by inhalation. Giardia lamblia is a protozoon that can cause gastrointestinal discomfort, malabsorption, and diarrhea. It is transmitted by the fecal oral route and by drinking infected water. Babesia microti is a protozoon that can cause fever. In a vulnerable host, it occasionally causes a more severe illness. It is transmitted by a tick. Trichophyton tonsurans is fungus that causes tinea capitis. This is a dermatophytosis of the scalp and other areas with hair. Trypanosoma cruzi is a protozoon that causes Chagas' disease. It is transmitted by bugs.

A 30-year-old immunocompromised patient presents with a 2-week history of breathlessness and a nonproductive, dry cough. The patient is afebrile, pulse is 100, and BP is 110/70 mm Hg. On auscultation, scattered rales all over the chest are heard. A chest X-ray shows diffuse air space and interstitial shadowing in both lungs. The shadowing is more prevalent in the apical region. What is the most likely diagnosis? Answer Choices 1 Tuberculosis 2 Mycoplasma pneumoniae 3 Viral pneumonia 4 Bacterial pneumonia 5 Pneumocystis pneumoniae

Pneumocystis pneumoniae The patient being immunocompromised narrows the choices to Pneumocystis pneumoniae and tuberculosis; diffuse infiltrate is more common in Pneumocystis infections of the lung. The fungus Pneumocystis jirovecii causes pneumocystis pneumonia. Tuberculosis shows localized infiltrate; it is commonly seen at the apices with hilar lymphadenopathy. Bacterial pneumonia will have an acute history of 2 - 4 days with fever and a nonproductive cough. The X-ray will show lobar infiltrate; it is rarely found throughout the lung. Viral pneumonia will have fever before the onset of the respiratory symptoms. Diagnosis is usually by isolating the organisms as well as serological tests. The presence of cavities in the lung indicates bacterial causes, fungal (Mycoplasma) causes, tuberculous infection, or neoplasm.

A 25 year-old HIV positive white male presents with dyspnea and a nonproductive cough. The patient's temperature is 103.5 degrees F, respirations 40, and pulse 140. Physical exam reveals a thin, dyspneic male, height 5'10" and weight 150 pounds, in moderate respiratory distress with minimal bibasilar crackles and peripheral cyanosis. CXR reveals a diffuse interstitial infiltrate and his LDH is elevated. Your most likely diagnosis is: A Streptococcal pneumoniae B Klebsiella pneumoniae C Mycoplasma pneumoniae D Pneumocystis jiroveci E Staphlococal aureus

Pneumocystitis jiroveci D The insidious onset of atypical pneumonia symptoms in a patient with HIV is consistent with pneumocystis jiroveci (D). Streptococcal pneumoniae (A), Klebsiella pneumonia (B), and Staphlococal aureus (E) present as "typical" pneumonia with the acute onset of fever and productive cough.

A 42-year-old woman presents for an evaluation of back pain. She states that she has a history of recurrent UTIs and that on occasion, she has noted that her urine appears red. Upon physical examination, she demonstrates CVA tenderness and a palpable abdominal mass. Her vital signs include blood pressure of 145/90 mmHg, respiratory rate of 16 breaths/min, and heart rate of 72 bpm. You order a urinalysis which has the following abnormal results: leukocyte esterase 1+, protein trace, and blood 3+. The microscopic examination reveals 5-10 WBCs/hpf and 20-30 RBCs/hpf. An abdominal ultrasound demonstrates multiple, bilateral fluid collections within the kidneys. Question What, if any, is the most likely underlying renal pathology? Answer Choices 1 Normal kidneys 2 Renal cell carcinoma 3 Goodpasture's syndrome 4 Polycystic kidney disease 5 Medullary sponge kidneys

Polycystic Kidney Disease The clinical picture is suggestive of acute pyelonephritis in the setting of polycystic kidney disease. Abdominal or flank pain, blood in the urine, recurrent UTIs, and hypertension are common symptoms of polycystic kidney disease. Cystic lesions on abdominal ultrasound are a hallmark of the disorder. These lesions would not be present in a patient with normal kidneys. A solid renal mass is most often seen in the context of renal cell carcinoma. Goodpasture's syndrome is defined as a combination of glomerulonephritis and pulmonary hemorrhages. The disease is usually preceded by an upper respiratory tract infection. Patients usually present with dyspnea and hemoptysis. This is not seen in this patient. Medullary sponge kidney is a common and benign disorder presenting at birth and is not usually diagnosed until the 4th of 5th decade. Common findings include gross hematuria, recurrent UTIs, and renal stones. The kidneys will have irregular enlargements of the medullary and interpapillary collecting ducts giving a "Swiss cheese" appearance. This is not the finding present on this patient's abdominal ultrasound.

A 24-year-old woman, gravida 0, presents with heavy bleeding between her periods, with cycles that seem irregular and more than 40 to 60 days apart. She has never had regular monthly periods. She is also concerned that she has acne and hair growth on her face, chest, and abdomen. She would like to become pregnant. She denies any chronic medical problems, although she states that her previous physician has advised her to lose weight because she has borderline diabetes. She is 5 feet 5 inches tall and weighs 240 lbs. Her gynecologic examination is unremarkable. Question What is the most likely explanation for her abnormal uterine bleeding? Answer Choices 1 A testosterone-secreting tumor of the ovary or adrenal 2 Increased exogenous estrogen 3 Increased exogenous progesterone 4 Loss of pulsatile GnRH due to thyroid dysfunction 5 Polycystic ovary syndrome (PCOS)

Polycystic ovary syndrome (PCOS) Explanation This patient has polycystic ovary syndrome. PCOS, also known by the name Stein-Leventhal syndrome, is an endocrine condition present in 5% to 10% of women of reproductive age. It is the most common cause of infertility in women. It is referred to as polycystic because most women with the condition have a number of small cysts in the ovaries; however, it is the characteristic constellation of signs, symptoms, and biochemical aberrations, rather than the presence of the cysts themselves, that is important in establishing the diagnosis, including dysfunctional uterine bleeding (DUB) due to estrogen breakthrough bleeding, hyperandrogenism, insulin resistance, and often obesity. Each plays a role in the evolution of an oligo-ovulatory state. It most commonly affects women ages 15 - 20 and is characterized by high estrogen and androgen levels, resulting in virilization (hair growth and acne), chronic menstrual irregularities, and infertility. Although the exact mechanism is yet to be determined, it is thought to involve a disorder of the hypothalamic-pituitary axis, in which access luteinizing hormone (LH) and androgen production causes virilization and anovulation with cyst formation in the ovary. Increased endogenous estrogen associated with obesity is likely to contribute to her anovulatory state, but there is no evidence to suggest that she is exposed to exogenous estrogen. Hyperandrogenism and, thus, virilization can be seen in the presence of a testosterone-secreting tumor of the ovary or adrenal, but her long history of oligomenorrhea makes this less likely. Thyroid dysfunction may also cause anovulation through dysregulation of a feedback loop that result in increased prolactin levels. However, her long history, including hyperandrogenism (acne and excess hair growth) and obesity, make this diagnosis less likely. A similar bleeding pattern could be seen with progestin-only contraception, such as Depo-Provera (depot medroxyprogesterone acetate/DMPA, or the "mini-pill," progestin-only birth control pill), but such hormonally-related bleeding would be due to progesterone breakthrough rather than estrogen breakthrough, as seen here. This patient desires pregnancy and is not exposed to exogenous progestin. Women with PCOS are at risk for cardiovascular illness and diabetes mellitus. They are also at increased risk for endometrial hyperplasia and endometrial cancer because of long-term unopposed estrogen stimulation of the endometrium.

A 64-year-old, nonsmoking male presents to the clinic with complaints of headache and dizziness. Initial labs reveal hemoglobin of 20.1 g/dl, hematocrit of 60%, and platelet count of 567,000. Which of the following is the most likely diagnosis in this patient? A Acute myeloid leukemia B Chronic myeloid leukemia C Chronic obstructive pulmonary disease (COPD) D Multiple myeloma E Polycythemia vera

Polycythemia Vera E Polycythemia vera often presents with complaints related to increased red blood cell mass, such as, headache, fatigue, and dizziness with elevated red blood cell mass with leukocytosis and/or thrombocytosis noted on CBC. Chronic hypoxia from conditions such as COPD (C) can also cause elevated RBC mass, but is unlikely in this non-smoker. The patient lacks white blood cell abnormality symptoms (e.g., recurrent infections, elevated white count) making acute myeloid leukemia (A), chronic myeloid leukemia (B), or multiple myeloma (D) unlikely diagnoses.

A 49-year-old man presents with chronic fatigue. He states that, despite obtaining 7 - 9 hours of sleep nightly, he wakes up feeling unrested and sometimes with a headache. Occasionally, he falls asleep in the middle of the day. He is not sure if he snores at night, but he has been told that he snores when he is napping. He reports no other changes in general condition. His other medical problems are hypertension and hypercholesterolemia, for which he takes metoprolol and simvastatin, respectively. Physical exam reveals a moderately overweight man in no apparent distress. Blood pressure, heart rate, and resting oxygen saturation are within normal limits. Question Based on the patient's symptoms, what diagnostic studies would be most useful? Answer Choices 1 24-hour Holter monitoring 2 Echocardiogram 3 Electrocardiogram 4 Electroencephalogram 5 Polysomnography

Polysomnography

You are reviewing incoming test reports while your supervising physician (SP) is out of town. There are 2 reports for a 46-year-old man; they are shown in the table. Vasopressin challenge test: Positive response Magnetic resonance imaging (MRI) brain: Mild thickening of the pituitary stalk, no masses or lesions Your SP left a note that he planned to initiate desmopressin after the test results confirmed his suspicion. Question What symptom would you expect to find in reviewing the history portion of this patient's chart? Answer Choices 1 Agnosia 2 Diplopia and blurred vision 3 Headaches 4 Memory loss 5 Polyuria and polydipsia

Polyuria and polydipsia Explanation This patient most likely has diabetes insipidus (DI). The most common presenting symptoms are polyuria and polydipsia. Diabetes insipidus is primarily a disorder in which the arginine-vasopressin (AVP) secreting cells are destroyed, causing a deficiency in antidiuretic hormone (ADH). The condition can be controlled with desmopressin, but in patients with very mild symptoms, no treatment may be preferred. Agnosia is the inability to recognize objects. It can be associated with dementia, head trauma, and brain tumors. The vasopressin challenge test, which should reduce thirst and urine output in patients with diabetes insipidus, would not be an appropriate test; desmopressin would not be suggested as a treatment. An MRI may be part of the workup for agnosia, and there could be variable findings. Diplopia and blurred vision should trigger a thorough neurologic exam and eye exam, with a possible MRI for further workup. As is the case with agnosia, the vasopressin challenge test and use of desmopressin have no role in regard to the patient's visual symptoms. Headaches are common, and most do not require specialized testing or imaging. If they present with 'red flags' that might indicate a more serious disorder (additional neurologic symptoms, new onset later in life, etc.), MRI of the brain is reasonable. Vasopressin challenge would not be used. Diabetes insipidus very rarely presents with headaches. Memory loss is not associated with these tests, findings, or treatments. A complaint of memory loss in a 46-year-old should be evaluated with a thorough neurologic exam; psychological testing and brain imaging should be considered as possibilities.

A 42-year-old woman is noted to have a solitary parenchymal pulmonary nodule in the right upper lobe on a routine plain chest film. This was not evident on a chest film taken 1 year earlier. The patient has a history of invasive ductal carcinoma of the right breast; she underwent a radical mastectomy 6 years ago. At that time, no lymph nodes were involved. 2 years later, she had a chest-wall recurrence and underwent wide local excision followed by radiation therapy. She was disease free for the past 4 years, until the plain chest film confirmed the pulmonary nodule. She had no history or risk factors of HIV or tuberculosis, and there is no history of travel that involved exposure to pulmonary mycosis. The patient smokes 5 cigarettes a day. She denies hemoptysis or other pulmonic or constitutional symptoms. On physical exam, her temperature is 98 degrees Fahrenheit (37 Celsius). She has no lymphadenopathy or abnormal palpable breast masses on the left side. Her complete blood count is normal. A CT scan of the patient's chest revealed only a nodular opacity in the right upper lobe measuring 2 cm x 1.5 cm. Question What is the most appropriate next step in the patient's evaluation? Answer Choices 1 Bone scan 2 Bronchoscopy 3 Positron emission tomography (PET) scan 4 Thoracotomy 5 Transthoracic needle aspiration biopsy

Positron emission tomography (PET) scan A chest CT scan is an important part of a solitary lung nodule. It is more sensitive than plain radiography and can determine whether a nodule identified on a chest film is indeed solitary. It provides better visualization of the nodule, is more sensitive for calcification, and can be used to guide thoracic needle aspiration biopsies. After the CT of the chest, the next step would be the PET scan. This study uses uptake of 2-fluoro-dexy-D-glucose (FDG) to measure glucose metabolism. The uptake of FDG by lung tumors is greater than that of normal lung parenchyma. In a patient with a solitary nodule, the sensitivity and specificity of PET for malignancy are about 95% and 90% respectively. With nodules less than 1.5 centimeters, PET is less reliable and also produces a higher rate of false-negatives. PET is particularly useful in evaluating indeterminate lesions in patients who are a poor surgical risk. Other benefits of PET scanning are detection of occult metastases and improved staging. A CT-PET strategy has been shown to be more cost-effective and also prevents more unnecessary thoracotomies than conventional approaches. CT-PET can demonstrate nodule enhancement. PET scanning is also the preferred approach in patients who have a solitary lung nodule classified as "indeterminate" based on the initial chest film and CT scan. If the PET scan is negative, follow up with serial CT scans. A positive PET scan indicates that malignancy is likely. PET is particularly useful in evaluating lesions in patients who are a poor surgical risk. Other benefits include detection of occult metastases and improved staging. A bone scan would be useful when the patient has symptoms of bone pain, mid-thoracic pain, or night fevers. An incidental finding of a high serum calcium level in this clinical setting may also warrant a bone scan. Bronchoscopy and transthoracic needle aspiration biopsy (TNAB) are the traditional options. Bronchoscopy may be useful if the lesion is 2 centimeters or more in diameter and can be accessed via a bronchus. In other settings, the false-negative rate is high; thus, in most patients, bronchoscopy is of limited value and has a relatively low diagnostic yield. TNAB is considered when the solitary nodule is smaller than 2 centimeters. The diagnostic yield is high for peripheral lesions. A negative result on TNAB may reliably rule out pulmonary mycosis in patients who live in areas where the incidence of this infection is high. TNAB has a higher complication rates than bronchoscopy. Although thoracotomy is the most definitive way to establish a diagnosis in patients with a solitary lung nodule, operative mortality is 3% to 7%. If it is video-assisted, then there is lower perioperative morbidity and a shorter hospital stay. In 25% of cases, a video-assisted thoracoscopic procedure needs to be converted to an open thoracotomy.

A 16-year-old previously healthy boy fell on a cemented surface while skateboarding 3 weeks ago, and he lost consciousness for 2 minutes. He was not wearing a helmet. A CT scan showed no abnormalities. He is now back at school and complains of difficulty concentrating in class, frequent spells of dizziness, and headaches. His neurologic exam is normal. Question What condition should you consider as the likely explanation for his symptoms? Answer Choices 1 Traumatic Brain Injury 2 Dysthymia 3 Complex partial seizures 4 Post Concussion Syndrome 5 Subdural hemorrhage

Post concussion syndrome This adolescent displays the characteristic features of post-concussion syndrome, which is usually diagnosed on the basis of presence of 3 out of the following 8 features after a significant injury event: 1) headache; 2) dizziness; 3) fatigue; 4) irritability; 5) insomnia; 6) concentration or 7) memory dif?culty; and 8) intolerance of stress, emotion, or alcohol. It is unlikely to be a traumatic brain injury because he had a short duration of loss of consciousness, normal head CT, and his current neurologic exam is normal. Symptoms of dysthymia or depression can seem to overlap with those of PCS, but he had no such symptoms prior to the traumatic event. Complex partial seizures are unlikely; they are usually associated with focal seizure activity in the form of tonic/tonic-clonic movements. Subdural hemorrhage is usually evident on head CT, but delayed subdural hemorrhages are rarely seen. In those situations, they are associated with abnormalities on neurologic examination.

A 7-year-old girl presents for a 1-day history of bloody urine. The grossly bloody urine scared both the girl and her parents, but she denies dysuria and frequency. No trauma or sexual abuse has occurred. The parents deny recent fever in the patient, but note that she had a fever for a few days accompanying a sore throat. She was given acetaminophen at an appropriate dosage for her weight, and about 3 days of some leftover amoxicillin; both the fever and pharyngitis then resolved. Her past medical history is unremarkable for any chronic illnesses. Her only medication is a multivitamin, and she has NKDA. She has had no surgeries, and family history is unremarkable for urinary tract disorders or any bleeding disorders. On physical exam, she appears interactive and in no apparent distress; she is well-nourished, non-obese, and perhaps mildly edematous. Vitals are a temperature of 99.0° F and a BP of 138/85 mm Hg; P is 98, and RR is 20. No rashes are found. Cardiac exam reveals normal rate and rhythm; there are no murmurs or rubs. On abdominal exam, her abdomen is non-distended, non-tender, and without masses or hepatosplenomegaly. She has no CVA tenderness. A urinalysis is performed; the significant findings are as follows: Protein 2+ Glucose Negative Ketones Negative Blood 4+ Nitrites Negative Leukocyte esterase 1+ Microscopic analysis is performed, which confirms the presence of red blood cells (RBCs) that are dysmorphic and indicate RBC casts. Blood tests are ordered, which reveal a complete blood count within normal limits and a complete metabolic panel with elevated creatinine. The patient's antistreptolysin O level is elevated, and her serum complement level is low. Question What is the most likely diagnosis for this patient? Answer Choices 1 Acute cystitis 2 Analgesic abuse nephropathy 3 Diabetic nephropathy 4 Postinfectious glomerulonephritis 5 Renal cell carcinoma

Postinfectious glomerulonephritis Explanation Of the listed choices, this pediatric patient most likely has postinfectious glomerulonephritis (PIGN), or poststreptococcal glomerulonephritis. The diagnosis is supported by both her history and the test results. "An estimated 5 to 10% of patients with streptococcal pharyngitis" may develop this condition.1 Though the test is not specific, the antistreptolysin O, along with the history of pharyngitis, help establish a recent streptococcal infection for this patient. PIGN falls under the broader category of nephritic syndromes characterized by hematuria and RBC casts, with possible proteinuria, edema, hypertension, and elevated serum creatinine. Treatment for PIGN is supportive, and normal renal function returns for the vast majority of patients. While the hematuria and presence of leukocyte esterase are consistent with acute cystitis, the rest of the history and findings are not. The classic presentation of acute cystitis includes dysuria and frequency. The remainder of the patient's presentation (i.e., edema, elevated BP, RBC casts, and serum tests) is inconsistent with acute cystitis. The leukocyte esterase is not specific for infection; in this case, it indicates inflammation in the urinary tract. Analgesic abuse nephropathy is a chronic tubulointerstitial nephritis "caused by cumulative lifetime use of large amounts (i.e., ≥ 2 kg) of certain analgesics."1 Though this patient was dosed with acetaminophen, her use was in small amounts and only for a few days. The history can rule out this diagnosis. Diabetic nephropathy is an important diagnosis to keep in mind when evaluating renal dysfunction, as it is the most common cause of nephrotic syndrome. The diagnosis includes a history of diabetes and demonstrated proteinuria, which can be confirmed by various methods. This patient has no history of diabetes, and even if she presented with elevated glucose, diabetic nephropathy would not be expected to develop for many years. Renal cell carcinoma (RCC) is suggested by persistent hematuria, especially in individuals over age 50 or in those at high risk. The serum findings in this patient are not consistent with the diagnosis of RCC. CT, MRI, and sometimes renal biopsy help confirm the diagnosis for RCC.

An 18-year-old woman presents with nausea, vomiting, drowsiness, and abdominal pain. She has Kussmaul respirations, ketotic breath, dry tongue, and loss of skin turgor. Her laboratory studies show the following results: sodium, 126 mEq/L; potassium, 3.1 mEq/L; bicarbonate, 8 mEq/L; chloride, 98 mEq/L, glucose, 320 mg/dL; BUN, 26 mg/dL; creatinine, 1.0 mg/dL; pH 7.02; pCO2, 13 mm Hg; and pO2, 86 mm Hg. In addition to intravenous fluid administration, which of the following therapies should be administered as part of initial treatment? Answer Choices 1 Calcium 2 Magnesium 3 Phosphate 4 Potassium 5 Bicarbonate

Potassium Explanation The correct response is potassium. Diabetic ketoacidosis is marked by an absolute insulin deficiency resulting in metabolic acidosis, ketonemia, and hyperglycemia. It is most commonly observed in patients with type 1 diabetes as a result of noncompliance with insulin therapy. Patients in diabetic ketoacidosis (DKA) often have normal or elevated serum potassium levels because of acidosis and dehydration. These normal or elevated potassium levels occur despite a large total body potassium deficit caused by osmotic diuresis and vomiting. Increased urinary output following fluid resuscitation significantly increase potassium losses. Withholding potassium from a patient in severe DKA can potentially result in cardiac arrest, severe dysrhythmias, respiratory arrest, and generalized paralysis. In contrast, bicarbonate therapy is controversial in DKA and may worsen hypokalemia. Several studies have failed to show any benefit with bicarbonate in DKA, even with a pH as low as 6.9. Although most patients have total body depletion of phosphate, immediate phosphate repletion has not been shown to alter the course of DKA. Replacement with magnesium should occur only in the presence of significant hypomagnesemia or refractory hypocalemia or hypokalemia. Calcium is not indicated in this setting in the immediate treatment of DKA

An 18-year-old woman presents with nausea, vomiting, drowsiness, and abdominal pain. She has Kussmaul respirations, ketotic breath, dry tongue, and loss of skin turgor. Her laboratory studies show the following results: sodium, 126 mEq/L; potassium, 3.1 mEq/L; bicarbonate, 8 mEq/L; chloride, 98 mEq/L, glucose, 120 mg/dL; BUN, 26 mg/dL; creatinine, 1.0 mg/dL; pH 7.02; pCO2, 13 mm Hg; and pO2, 86 mm Hg. In addition to intravenous fluid administration, what therapeutic agent is indicated pending further laboratory results? Answer Choices 1 Calcium 2 Dextrose 3 Phosphate 4 Potassium 5 Bicarbonate

Potassium Patients in DKA may be euglycemic, and they often have normal or elevated serum potassium levels because of acidosis and dehydration. These normal or elevated potassium levels occur despite a large total body potassium deficit caused by osmotic diuresis and vomiting. Administration of insulin, correction of acidosis with bicarbonate, and increased urinary output following fluid resuscitation significantly increase potassium losses. Withholding potassium from a patient in severe DKA with a normal potassium level can potentially result in cardiac arrest, severe dysrhythmias, respiratory arrest, and generalized paralysis. In contrast, bicarbonate therapy is controversial in DKA and may worsen hypokalemia. Several studies have failed to show any benefit with bicarbonate in DKA, even with a pH as low as 6.9. Although most patients have total body depletion of phosphate, immediate phosphate repletion has not been shown to alter the course of DKA. Calcium is not indicated in this setting. Dextrose is not indicated because the patient has a normal blood sugar. Relative contraindications to dextrose include hyperosmolar coma and stroke.

A 22-year-old woman presents with amenorrhea for the past 8 months. Further questions elicit additional pertinent positives of backaches, headaches, and acne. Physical examination reveals a female patient with a moon-shaped facies, multiple purple striae, and significant central obesity (body mass index of 36). Question Given this history and physical exam findings, what pharmaceutical treatment should be initiated after the appropriate clinical intervention has occurred? Answer Choices 1 Insulin 2 Levothyroxine 3 Prednisone 4 Propylthiouracil (PTU) 5 Desmopressin

Prednisone Explanation This patient likely is suffering from Cushing's syndrome, which is also known as hypercortisolism. Consequences of excessive levels of circulating cortisol, no matter the etiology, will lead to certain signs and symptoms, such as central obesity, thin extremities, a moon face, a buffalo hump, supraclavicular fat pads, protuberant abdomen, and oligomenorrhea/amenorrhea; in men, erectile dysfunction is a possibility. The most common etiology of Cushing's syndrome is a benign pituitary adenoma secreting excessive ACTH. Primary clinical intervention is transsphenoidal selective resection of the pituitary adenoma. Once this has been completed, patients typically suffer from a suppression of pituitary corticotrophs; therefore, treatment with hydrocortisone or prednisone replacement therapy is necessary for a range of 6 - 36 months post-surgery. Diabetes Type II is caused by insulin resistance in tissues to circulating endogenous insulin. Eventually insulin is unable to be produced by the pancreatic islet cells and an exogenous supply is required to prevent significant hyperglycemia and its related co-morbidities. Features of these patients include central obesity (specifically visceral obesity), with less fat noted on extremities. Insulin, although usually not the common initial medication considered at for patients with diabetes, is one of the many types of pharmaceutical treatments available for these patients. Hypothyroidism and hyperthyroidism both are related to dysfunction of the thyroid gland; hypothyroidism is due to failure of the thyroid gland itself or deficient supply of pituitary TSH (thyroid stimulating hormone). Symptoms exhibited with hypothyroidism are weight gain, fatigue, lethargy, depression, weakness, menses irregularities, athralgias, muscle cramps, cold intolerance, dry skin, and constipation; signs present may include thinning hair, brittle nails, bradycardia, or even peripheral edema. Typical treatment for hypothyroidism is a dose of daily levothyroxine. Hyperthyroidism is the disease state in which there is an excessive serum level of T3 and or T4 along with suppressed TSH levels. Signs and symptoms may include (depending on the severity) restlessness, nervousness, heat intolerance, increased sweating, weight loss, palpitations, atrial fibrillation, exophthalmos, or even pretibial myxedema. The medication that is commonly utilized in treatment of hyperthyroidism is primarily propylthiouracil, which is also frequently referred to as PTU. Diabetes insipidus (DI) is antidiuretic hormone deficiency. A sign/symptom of DI is an extreme, unquenchable thirst with a very specific craving for ice water. Patients also will have significant polyuria or hypernatremia. Pharmaceutical treatment for DI is typically desmopressin.

A 72-year-old man presents with an 8-month history of progressive dyspnea, which has been accompanied by a dry and persistent hacking cough. While the dyspnea now occurs at rest, he denies fever, chills, palpitations, chest pain, or peripheral edema. He states that he has worked for many years at a local chemical plant. His physical exam is remarkable for digital cyanosis and clubbing, while his pulmonary exam reveals diffuse fine, dry inspiratory crackles. His cardiac exam was positive for a prominent pulmonary valve closure sound (P2) and an elevated jugular venous pressure of 6 cm. A chest x-ray noted small lung volumes, with increased densities in the lung periphery and a honeycombing pattern; pulmonary function testing measured reductions in TLC, FEV1, and FVC with a preserved FEV1/FVC ratio. Question What is the best treatment for this patient at this time? Answer Choices 1 Erythromycin 2 Furosemide 3 Lung transplantation 4 Prednisone 5 Cyclophosphamide

Prednisone This patient most likely has hypersensitivity pneumonitis (HP), also known as extrinsic allergic alveolitis. This is an inflammatory disorder of the lung involving alveolar walls and terminal airways that is induced by repeated inhalation of a variety of organic agents. The chronic form of HP typically results from low-grade or recurrent exposure over many months to years, and the lung disease may already be partially or completely irreversible. These patients are usually advised to avoid all possible contact with the offending agent. In addition to identification of the causative agent and its avoidance, institution of glucocorticoid treatment is indicated. Prednisone at a dosage of 1 mg/kg per day or its equivalent is continued for 7 - 14 days. It is then tapered to 0.25-0.5 mg/kg and is maintained at this level for an additional 4 - 12 weeks at a rate that depends on the patient's clinical status. While patients with chronic HP may gradually recover without therapy following environmental control, a trial of prednisone may be useful to obtain maximal reversibility of the lung disease. Following initial prednisone therapy (1 mg/kg per day for 2 to 4 weeks), the drug is tapered to the lowest dosage that will maintain the functional status of the patient. Many patients will not require or benefit from long-term therapy if there is no further exposure to the antigen. Improvement of lung function may continue over a few months to years. If the patient's condition continues to decline on glucocorticoids, a second agent should be introduced while lowering or maintaining the prednisone dose at 0.25 mg/kg per day. Glucocorticoid therapy is recommended for symptomatic interstitial lung disease (ILD) patients with eosinophilic pneumonias, cryptogenic organizing pneumonia, connective tissue diseases, sarcoidosis, hypersensitivity pneumonitis, acute inorganic dust exposures, acute radiation pneumonitis, diffuse alveolar hemorrhage, and drug-induced ILD. In organic dust disease, glucocorticoids are recommended for both the acute and chronic stages.

A 23-year-old G1P0 presents to the office complaining of headache, nausea, swelling, and generally not feeling well. She is at 33 weeks gestation. A physical exam reveals a 5-pound weight gain in 2 weeks, BP 148/90, P 84, T 98.1°F, and UA concentrated with 1+ protein. What is the most likely diagnosis? A Pregnancy induced hypertension B Preeclampsia C Eclampsia D Gastroenteritis E Third space fluid retention

Preeclampsia B Preeclampsia has a classic triad of weight gain, elevated blood pressure, and proteinuria. A woman with this triad of symptoms has preeclampsia until proven otherwise.

Vaginal trichomoniasis has been associated with adverse pregnancy outcomes. What is an example of such an outcome? Answer Choices 1 Hyperemesis gravidarum 2 Premature rupture of membranes 3 Preeclampsia 4 Gestational Diabetes 5 Eclampsia

Premature rupture of membranes Trichomoniasis infection is associated with premature rupture of membranes, preterm delivery, and low birth weight. There is no proven association between trichomoniasis and hyperemesis gravidarum, preeclampsia, eclampsia, or gestational diabetes.

A 42-year-old morbidly obese woman is referred back to her primary care provider to supervise her weight loss program. A surgeon evaluated her for a 'gallbladder attack' 1 month ago, which then resolved; she currently denies abdominal pain. When examined via ultrasound, she still has visible stones in the gallbladder. She declined to have surgery until she achieves her initial weight loss goal of 50 pounds. She is on a supervised healthy diet of about 1200 kilocalories daily; she exercises 1 hour a day 6 days per week. She is losing about 15 - 16 pounds a month. Question What intervention would be most effective at preventing the recurrence of her gallbladder "attack" until she achieves her weight loss goal? Answer Choices 1 Prescribe orlistat 2 Prescribe ursodiol 3 Recommend over-the-counter omeprazole 4 Reduce patient's exercise 5 Restrict caloric consumption further

Prescribe Ursodiol Both morbid obesity and rapid weight loss are risk factors for development of cholecystitis. Gallstones may be present within the gallbladder and remain asymptomatic, or the gallbladder walls may become inflamed, resulting in cholecystitis. Ursodiol is approved for prevention of gallstones in obesity patients with rapid weight loss. It would be the best choice in health maintenance and delaying or preventing need for surgical cholecystectomy. Orlistat is a prescription medication approved for treatment of obesity. Its mechanism is to block fat absorption. It does not have a direct role in health maintenance or prevention of cholecystitis or cholelithiasis. In fact, it has pronounced gastrointestinal side effects. Over-the-counter omeprazole is a commonly used proton-pump-inhibitor (PPI); it is helpful in acid lowering in the stomach and treatment of gastroesophageal reflux disease. It has not been shown to help gallbladder disease, and there is some evidence that PPIs may worsen it. Reducing the patient's exercise may aggravate her condition and negatively impact her weight loss. Exercise has an inverse relationship with cholecystitis and rates of cholecystectomies, so this patient should be encouraged to continue frequent exercise. This patient should not be instructed to restrict caloric consumption further. She is already losing weight quite rapidly at around 4 pounds per week. Her daily caloric intake is low; recommending further restriction will likely discourage her, and there will not be any benefit in regard to her gallbladder disease.

A 31-year-old man presents with constipation, fatigue, and weight gain of 20 pounds over the last 6 months. He has no significant past medical history. He is a smoker. His family history is notable for a myocardial infarction in his father at the age of 42. His physical examination is unremarkable, except for a weight of 220 pounds and a height of 6 feet. You order laboratory tests, which reveal normal electrolytes, a total cholesterol of 223, and a triglyceride level of 205. His thyroid function tests reveal a thyroxine of 2.1 (4.5 - 11.0), a free T4 index of 1.7 (4.5 - 11.0), and a TSH of 193 (0.35 - 5.50). Question What is the best next step in managing this patient's condition? Answer Choices 1 Telling him to stop smoking and start on a low cholesterol diet 2 Telling him to stop smoking, start on a low cholesterol diet, and start taking an HMG-CoA reductase inhibitor 3 Prescribing oral levothyroxine 4 Prescribing oral levothyroxine and an HMG-Co reductase inhibitor 5 Being admitted to the hospital for intravenous levothyroxine

Prescribing oral levothyroxine Explanation The patient has frank primary hypothyroidism. His thyroid function tests confirm this diagnosis by showing a markedly elevated TSH and a depressed thyroxine level. Elevated total cholesterol levels and triglycerides can be seen in these patients. These abnormalities usually resolve with thyroid replacement. The best way to manage these abnormalities is with thyroid hormone replacement orally (oral levothyroxine). Although his cholesterol is elevated and he is a smoker, telling him to stop smoking and start a low cholesterol diet would not address his underlying disorder. After his thyroid condition has been addressed, a fasting lipid profile should be obtained and smoking cessation counseling should begin. Adding an HMG-CoA reductase inhibitor to diet and smoking cessation or to levothyroxine therapy is premature. Again, a fasting lipid profile should be obtained after thyroxine has been adequately replaced. Admission to the hospital for intravenous levothyroxine is only indicated in cases of myxedema coma where extreme hypothermia, mental status change, seizures, and respiratory depression are seen. These are not present in this patient.

A 17-year-old female presents to the emergency department complaining of watery vaginal discharge for 6 hours. She is found to be at 35 weeks gestation. An external fetal monitor reveals fetal heart tones in the 130s, good variability, and no contractions. What is the most likely diagnosis? A Preterm labor B Preterm rupture of membranes C Premature rupture of membranes D Preterm labor and premature rupture of membranes E Preterm and premature rupture of membranes

Preterm and premature rupture of membranes Preterm rupture of membranes is defined as rupture before 37 weeks gestation; premature is defined as before the onset of labor. The absence of pain or contractions decreases the likelihood of labor.

A 45-year-old man is admitted to the medical floor of a hospital with increasing jaundice, swollen legs, and episodes of disorientation for the last several weeks. His abdomen is distended and he looks ill. According to the family, he is an alcoholic and has recently been diagnosed with cirrhosis of liver. On examination, he has a temperature of 99 F, BP 100/72 mm Hg, and his pulse is 86/minute. Sclerae are icteric, and he has 2+ pitting pedal edema. Lungs are clear, and heart sounds are normal. Abdomen is distended with moderate ascites, caput medusae, and no tenderness. Liver and spleen are not palpable. He is awake but drowsy and oriented to person and place but not to time. He does have a fine tremor in his hands. Labs show WBC 8000/uL, platelets 100,000/uL, Hb 12g%, AST 76 U/L, ALT 56 U/L, AP 62 U/L, and ammonia is 124 mg/dL. Question At this time which of the following is most important to avoid full decompensation into hepatic encephalopathy? Answer Choices 1 Increase dietary protein 2 Prevent constipation 3 Sedate the patient to avoid injury from disorientation 4 Start IV antibiotics empirically anticipating infection 5 Add thiamine and folic acid for nutritional support

Prevent Constipation Explanation This patient is already in the initial phases of hepatic encephalopathy due to alcoholic liver disease as evidenced by the disorientation and tremors, also known as asterixis. The primary cause of hepatic encephalopathy is unclear. Metabolic abnormalities due to liver dysfunction, resulting in a spectrum of neuropsychiatric signs and symptoms, are seen. High levels of ammonia are found in the blood. Constipation causes increased ammonia production and absorption due to prolonged intestinal contact and aggravates the condition. Lactulose should be administered frequently to eliminate the ammonia in the stool. It is an indigestible sugar that acts as an osmotic laxative by increasing the water content of the stool and promoting bowel movements. It is digested by the colonic bacteria, and the acidic remains convert ammonia into ammonium ions in the colon, which are then excreted in the stool. Oral antibiotics can lower blood ammonia levels by decreasing ammonia production and absorption. The commonly used ones are neomycin, metronidazole, vancomycin, and, lately, rifaximin. The last three are better tolerated than neomycin. However antibiotics have their side effects and can cause bacterial overgrowth syndromes. Their main use continues to be in patients who cannot tolerate disaccharides like lactulose. Acarbose and fermentable fiber can also cause decrease in intestinal ammonia production and absorption. Newer studies with sodium benzoate are ongoing. Benzoate and glycine react to form hippurate, and for every mole of benzoate utilized this way, one mole of nitrogen is excreted in the urine, thereby enhancing ammonia metabolism. This, however, still needs to be studied further to be used widely. Studies with ornithine-aspartate are also being done as a stimulator of ammonia metabolism. All the products mentioned are yet to replace lactulose as the first line of treatment but are potentially useful once more studies are done. Other precipitating factors for hepatic encephalopathy include azotemia, hypokalemia, gastrointestinal bleeding, high protein diet, alkalosis, infection, sedatives, and other hepatotoxic agents. High protein diet is a contraindication in this condition, as protein catabolism causes increase in ammonia levels. Daily protein should be restricted to 40 g/day. Sedation of the patient should also be avoided, since sedatives cause cerebral depression and worsening of encephalopathy. These drugs are also not metabolized adequately by the diseased liver. Though infection is an important precipitating factor for hepatic encephalopathy, empiric treatment is not recommended. However, early and adequate treatment of an infection should be done, especially for spontaneous bacterial peritonitis. In fact multiple randomized control trials have been done regarding antibiotic prophylaxis for SBP, and they have shown not only a decrease in bacterial infections but also a significant reduction in mortality. Prophylaxis is recommended in cirrhotic patients with risk factors for SBP like GI bleeding, prior history of SBP, and low ascitic fluid protein. Thiamine and folic acid should be added for nutritional support to all alcoholic patients, since they are malnourished and vitamin depleted. However, this will not change the outcome in hepatic encephalopathy as quickly or as much as avoiding constipation.

A premature infant is born with the inability to produce adequate amounts of pulmonary surfactant. Which of the following is the primary function of pulmonary surfactant? A increase the solubility of carbon dioxide in the alveoli B increase the solubility of oxygen in the alveoli C prevent infectious organisms from infiltrating the alveoli D prevent the collapse of small lung alveoli E stimulate the unloading of carbon dioxide from hemoglobin

Prevent the collapse of lung alveoli Because of their small size, many lung alveoli are prone to collapse. Pulmonary surfactant contains a high concentration of amphipathic phospholipid molecules, which lowers the surface tension of alveoli. According to the law of Laplace, a reduction of surface tension reduces the collapsing pressure on small alveoli and allows them to remain open. Pulmonary surfactant production does not typically begin until the 24th week of gestation; hence, an infant born before this time is at great risk for having collapsed alveoli.

A middle-aged woman presents with elevated cholestatic liver enzyme levels. She is not taking any medications, does not drink alcohol, and does not complain of abdominal pain. She has not had any previous biliary tract surgery. Which of the following is the most likely diagnosis? A primary biliary cirrhosis B pancreatitis C cholecystitis D fatty liver

Primary Biliary Cirrhosis A Primary biliary cirrhosis affects women typically between ages 40 and 60. It is often discovered incidentally when the serum alkaline phosphatase level is found to be elevated. Many patients do not have pain, which is more common in cholecystitis or pancreatitis. Primary sclerosing cholangitis is more likely to occur in a patient with known inflammatory bowel disease.

A 54-year-old man is discovered to have hypertension (165/110 mm Hg) on a routine physical exam. On questioning, the patient reveals that he had been suffering from headaches and leg cramps in addition to a feeling of generalized weakness. A careful history further reveals an increased urinary volume. Complete investigations show hypokalemic alkalosis with high serum sodium (154 mEq/L) and low serum potassium (2-4 mEq/L). The ECG shows flattened T waves, long QT, and U waves. The lab reports also show low plasma renin. What is the likely diagnosis? Answer Choices 1 Primary hyperaldosteronism 2 Secondary hyperaldosteronism 3 Primary hypoaldosteronism 4 Secondary hypoaldosteronism 5 Cushing's Syndrome

Primary Hyperaldosteronism Explanation Primary and secondary hyperaldosteronism are both characterized by increased aldosterone production from the adrenal cortex. Primary hyperaldosteronism results from adrenal adenomas (Conn's Syndrome) in 60% of cases. 30% are due to bilateral adrenal hyperplasia. Secondary hyperaldosteronism results from increased activation of the renin-angiotensin system, leading to intensive stimulation of the adrenal cortex. Secondary hyperaldosteronism is generated by extravascular loss of Na+ and water (e.g. cirrhosis, nephrosis, congestive heart failure), renal artery stenosis, or by tumor of the juxtaglomerular cells. Renin secretion from the juxtaglomerular cells is increased under these conditions, leading to increased formation of angiotensin II. Thus, in secondary hyperaldosteronism, both renin and angiotensin levels are high. In primary hyperaldosteronism the renin and angiotensin levels are low, being reduced by the expansion of the extracellular space. Aldosterone stimulates K+ secretion in the kidney; therefore, plasma K+ level is decreased in both primary and secondary hyperaldosteronism. Plasma Na+ level is usually within the physiological range, because the increased Na+ reabsorption is always followed by increased water accumulation. The plasma level of angiotensinogen is normal in both primary and secondary hyperaldosteronism.

A patient describes an insidious onset of vague gastrointestinal symptoms including nausea, vomiting, and abdominal pain. She has been losing weight over the last few weeks as well. Past medical history reveals a new diagnosis of anemia. The patient is not currently taking any medications. Physical exam findings include hypotension and dark freckles with dark, bluish black pigmentation of the mucosal membranes. Serum sodium is decreased, while the potassium and plasma ACTH levels are elevated. What is the most likely diagnosis for this patient? A Acute abdomen B Cushing's syndrome C Primary adrenal insufficiency D Disseminated intravascular coagulation E Diabetic ketoacidosis

Primary adrenal insufficiency The correct choice is C, primary adrenal insufficiency. This disorder presents with symptoms resulting from progressive destruction of the adrenal glands and resultant decrease in aldosterone and cortisol secretion. The presentation can range from subtle to fulminant adrenal crisis. Common clinical findings include hyperpigmentation, weakness, anorexia, nausea, vomiting, diarrhea, and hypotension. Some patients with adrenal insufficiency may initially be treated for an acute abdomen, choice A. In this patient, ACTH levels are known to be elevated, pointing the cause to the pituitary-adrenal axis. Choice B, Cushing's syndrome, presents with signs and symptoms related to an excess in plasma cortisol, such as truncal obesity, hyperglycemia, hypertension, skin changes, and weight gain. Choice D, disseminated intravascular coagulation, is a coagulation disorder presenting with bleeding and thrombosis. Choice E, diabetic ketoacidosis, is an acute complication seen in patients with type 1 diabetes mellitus.

A 60-year-old female presents to her primary care practitioner for her yearly check up. Her past medical history is significant only for a history of repeat kidney stones. She enjoys gardening outside, has no significant family history, and is not taking any medications. Routine mammograms have been normal. During the review of her systems, the patient describes feeling tired lately and has noticed muscle aches over the last few weeks. Initial lab values are shown below: Hemoglobin = 12.0 gm/dL Hematocrit = 36% BUN = 10 mg/dL Creatinine = 0.7 ng/mL Calcium = 12 mg/dL Phosphate = 2.0 mg/dL PTH = 100 pg/mL TSH = 2.0 UIU/mL What is the most likely cause of her symptoms? A Vitamin D deficiency B Hypercalcemia of malignancy C Primary hyperparathyroidism D Factitious hypercalcemia E Hyperthyroidism

Primary hyperparathyroidism The correct choice is C, primary hyperparathyroidism. This patient has the characteristic signs and symptoms of hypercalcemia, along with the lab findings of primary hyperparathyroidism with elevated calcium, low phosphate, and elevated PTH. Other common presenting features include bone pain, paresthesias, polyuria, and gastrointestinal symptoms. Females are affected three times as much as males, and it is typically found accidentally. Choice A, vitamin D deficiency, is unlikely since she enjoys gardening outside in the sun. Vitamin D deficiency can be seen is association with primary hyperparathyroidism, but is not the cause of this disorder. Choice B, hypercalcemia of malignancy, is unlikely with her history of normal mammograms. Serum PTH is typically normal in this disorder as well. Choice D, factitious hypercalcemia, is unlikely since she doesn't take any calcium supplements or other medications. Choice E, hyperthyroidism, can be a cause of hypercalcemia if the patient is suffering from thyrotoxicosis, but this patient's TSH is within the reference range.

Which of the following pairs of conditions constitute 90% of the causes of adult hypercalcemia? A Vitamin D intoxication and renal failure B Lithium ingestion and hyperthyroidism C Prolonged immobilization and thiazide use D Hypothyroidism and hypoparathyroidism E Primary hyperparathyroidism and malignancy

Primary hyperparathyroidism and malignancy The correct choice is E, primary hyperparathyroidism and malignancy. Together, they constitute 90% of the causes of hypercalcemia. Choices A, B, and C can cause hypercalcemia, but at a lower rate. Choice D, hypothyroidism and hypoparathyroidism, do not cause hypercalcemia.

A 35-year-old man presents with a 4-week history of difficulty swallowing food. He has developed chest pain over the last few days, and he has also noticed regurgitation of food. On detailed history, he says that the regurgitation contains undigested fragmented food material; it is unassociated with a foul smell. He has lost 3 kg over the past month. On examination, his pulse is 94 bpm and blood pressure is 130/86 mm Hg. Mild pallor is noted, but there is no icterus. Epigastric tenderness is present. CBC shows a hemoglobin of 9.5 g/dL (total count 7400 cells/mm3 and ESR 12 mm/hr). An X-ray of the abdomen shows an absence of a fundic air shadow. The patient undergoes barium studies which show a dilated esophagus; the lower end appears beak-shaped. Question What is the most common cause for the above condition? Answer Choices 1 Diabetes mellitus 2 Chagas disease 3 Eosinophilic gastroenteritis 4 Neurodegenerative disorder 5 Primary idiopathic

Primary idiopathic Explanation Gradual onset of difficulty in swallowing food, regurgitation of undigested food, absence of fundic air on plain abdominal X-ray, and dilated esophagus with beak shaped lower end are diagnostically indicative of Achalasia cardia. It is due to degeneration of inhibitory neurons in the lower end of esophagus. The most common cause of Achalasia is primary idiopathic. There is a deficiency of inhibitory neurons, vasoactive intestinal peptide, and nitric oxide synthase in this condition. Other rarer causes of achalasia include Chagas' disease, lymphoma, neurodegenerative disorders, esophageal gastroenteritis, and certain viral infections.

A 48-year-old man presents with a 5-year history of hand tremor. The tremor was initially mild and has progressed over the last year. It diminishes at rest, but it intensifies with intentional movements and upon emotional stress and fatigue. He is unable to write or drink from a cup during stressful situations. He never drinks alcohol, and he does not currently take any drug except metformin for non-insulin dependent diabetes mellitus. Family history is negative. Physical exam is unremarkable. His gait, speech, and posture are normal. Question What is the best first-line agent for treatment of this condition? Answer Choices 1 Nimodipine 2 Alprazolam 3 Amitriptylline 4 Primidone 5 Theophylline

Primodone Essential tremor is the most common idiopathic movement disorder and may occur in at least 14% of adults (Schwartz and Andrasik, 2003). The disease is more common in men than in women, with a mean age of onset of 45 years and 60% positive family history (Schwartz and Andrasik, 2003). According to the consensus statement of the Movement Disorder Society (MDS, 1998), the core criteria for diagnosing essential tremor include the presence of a visible, bilateral postural tremor (predominantly symmetric, with or without kinetic tremor or head tremor) of the hands or forearms that does not feature abnormal posturing and/or other neurological signs (Deuschl et al., 1998). The tremor may have duration greater than 3 years, and it may diminish with alcohol consumption (Deuschl et al., 1998; Chen and Swope, 2003). Primidone (and propranolol) are the best initial agents for treatment of essential tremor (Louis, 2001). Relative contraindications for the use of propranolol are asthma, congestive heart failure, diabetes mellitus, and atrioventricular block (Louis, 2001).

A 23-year-old woman presents to clinic complaining of amenorrhea for 3 months. She also complains of increasing facial hair and weight gain. On exam, PMH: menarche age 13. Physical exam reveals a well-developed, slightly obese female with a BMI of 29. Her amenorrhea can likely be improved with which therapy? A Testosterone administration B Progesterone administration C Estrogen administration D Estrogen suppression E Testosterone suppression

Progesterone administration B Progesterone administration slows GnRH pulses, thereby improving FSH secretion and follicular maturation.

A 25-year-old woman presents to discuss her options of available contraception. You review her history and note menses onset at 12 years old, duration of menses typically around 6 days in length, and it occurs every 30 days. She is G0P0, and she has no history of abnormal pap smears or diagnosed STIs. The patient is a non-smoker, single, and in a monogamous relationship with 1 partner for the past year. All other medical history is noncontributory with the exception of the patient having a deep venous thrombosis at age 19 and a pulmonary embolism at age 21. Other than anticoagulation therapy for the appropriate amount of time, no other hematological evaluation was pursued after these events. Question Given the most likely inherited diagnosis, what would be the safest form of contraception at this time? Answer Choices 1 Estrogen/progestin combination oral contraception pill 2 Progestin-only oral contraception pill 3 Etonogestrel/ethinyl estradiol vaginal ring 4 Norelgestromin/ethinyl estradiol patch-transdermal 5 Estradiol/medroxyprogesterone monthly injection

Progestin-only oral contraception pill Explanation The correct response is progestin-only oral contraception pill. Our patient above is overall healthy 25-year-old female patient. However, her history of multiple hypercoagulable events without any substantial risk factors puts her in the likely category of possessing an inherited hypercoagulable state. The most common hypercoagulable state that is inherited currently is Factor V Leiden. Despite the fact that this patient is very highly likely to have Factor V Leiden, she can still be offered a type of contraceptive method to prevent unplanned pregnancy. The common pre-thrombotic contraceptive component is estrogen. For this reason, the patient above would be safest at beginning any method of contraception that only contains progestin. There are several types of contraception that are progestin-only: progestin-only implants, a monthly injection of only progestin, levonorgestrel based intrauterine device, and the progestin-only pills, which are also referred to as the "mini-pill." The estrogen/progestin combination oral contraception pill, the etonogestrel/ethinyl estradiol vaginal ring, the norelgestromin/ethinyl estradiol patch-transdermal, and the estradiol/medroxyprogesterone monthly injection all contain a form of estrogen, which use of this should be strictly avoided in patients such as the one described in the above scenario.

A 36-year-old woman presents for her annual gynecological examination. In addition to her routine testing, she is seeking advice on contraception. She currently takes no medication and has no known medical conditions. She smokes a 1/2 pack of cigarettes daily; she drinks 1 - 3 alcoholic drinks weekly, and she is currently sexually active with 2 partners. Her past PAP smears have all been normal. She is G2P1 and is uncertain whether she wants children in the future. Question What is your recommendation for contraception at this time? Answer Choices 1 Combination estrogen-progestin pill 2 Progestin-only pill 3 Ortho Evra (ethinyl estradiol/norelgestromin) patch 4 NuvaRing (etonogestrel/ethinyl estradiol) 5 Tubal ligation

Progestin-only pill Explanation Out of the above options, the patient should receive progestin-only pills. The patient is a smoker, which is a contraindication for estrogen containing products. The patient should be informed that she needs to take progestin-only pills at the same time daily; otherwise, she needs to use a back-up form of contraception. The patient cannot receive the combination estrogen-progestin pill, the Ortho Evra (ethinyl estradiol/norelgestromin) patch, or the NuvaRing (etonogestrel/ethinyl estradiol); they all contain estrogen. The provider should not recommend tubal ligation because the patient is uncertain whether she wants children in the future; tubal ligation is considered a permanent sterilization option.

A 28-year-old woman presents because she has not gotten her period in 2 months. She is worried that she may be pregnant, but she is concerned because she and her boyfriend broke up about 2 months ago. She does not want to be a single mother. The patient mentions a discharge from both breasts. You order a urinary HCG, which is negative. Question What diagnostic study would you order next? Answer Choices 1 Prolactin level 2 Free T4 level 3 Pelvic ultrasound 4 Thyroid ultrasound 5 Serum calcium

Prolactin level Explanation The clinical picture is suggestive of amenorrhea/galactorrhea caused by increased levels of prolactin. Of women with amenorrhea and galactorrhea, 70% have hyperprolactinemia. So based on statistics, prolactin levels would be the next study to order. Free T4 levels are to measure for hypothyroidism, which can cause hyperprolactinemia. But the patient does not demonstrate signs of hypothyroidism (cold intolerance, skin, hair and nail changes, weight gain, etc.). A pelvic ultrasound would be for detection of pregnancy/ectopic pregnancy, but the HCG is negative. A thyroid ultrasound would be to detect thyroid masses, but this patient does not have a thyroid condition. Serum calcium would be used to detect hyperparathyroidism, which can cause hyperprolactinemia; however, the patient does not demonstrate signs of hyperparathyroidism (not feeling well, abdominal pain, kidney stones, bone pain and mental changes).

Over a period of several months, a 62-year-old man has developed erectile dysfunction. He has no history of neurologic, kidney, or cardiovascular disease or diabetes mellitus. He takes a multivitamin and an occasional ibuprofen for aches and pains. He has never smoked cigarettes, drinks 1-2 glasses of wine with dinner on weekends, and uses no mind-altering drugs. Physical examination is remarkable only for bilateral gynecomastia. What is the most likely diagnosis? A breast cancer B depression C prolactinoma D steroid abuse E testicular cancer

Prolactinoma Men with prolactinomas may experience erectile dysfunction, infertility, and, less commonly, gynecomastia. Breast cancer in men (A) presents as a usually as a unilateral mass. Men with depression (B) may have erectile dysfunction, but not gynecomastia. Steroid abuse (D) is associated with gynecomastia, but the patient would likely be showing other signs and symptoms. Testicular cancer (E), specifically germ cell cancer, is associated with gynecomastia in 5% of cases but this man has no testicular mass or swelling.

A 45-year-old patient came in to see his health care provider today, to discuss the results of his last annual assessment. He was told that he had developed type 2 diabetes mellitus. One of the recommendations from the physician assistant included a visit to an ophthalmologist. The physician assistant was concerned after seeing new capillaries, macular edema, and fibrous tissue within the retina during his funduscopic exam. What type of ocular complication does this patient most likely have at this time? A Background retinopathy B Closed angle glaucoma C Macular degeneration D Diabetic cataracts E Proliferative retinopathy

Proliferative Retinopathy The correct choice is E, proliferative retinopathy. The distinguishing factor in the patient's presentation, which signals this disorder, is the development of newly formed vessels. Proliferative retinopathy is the leading cause of blindness in the United States. Up to 20% of patients with type 2 diabetes have retinopathy at the time of diagnosis. Choice A, background retinopathy, or simple retinopathy includes retinal microaneurysms, hemorrhages, exudates, and edema, without new vessel formation. Choice B, closed angle glaucoma, is relatively uncommon in patients with diabetes, except after cataract extraction. Choice C, macular degeneration, is not associated with diabetes mellitus specifically. Choice D, diabetic cataracts, tends to occur in patients with diabetes earlier than the general population, and may correlate with the severity of the disease.

A 45-year-old patient came in to see his health care provider today, to discuss the results of his last annual assessment. He was told that he had developed type 2 diabetes mellitus. One of the recommendations from the physician assistant included a visit to an ophthalmologist. The physician assistant was concerned after seeing new capillaries, macular edema, and fibrous tissue within the retina during his funduscopic exam. What type of ocular complication does this patient most likely have at this time? A Background retinopathy B Closed angle glaucoma C Macular degeneration D Diabetic cataracts E Proliferative retinopathy

Proliferative retinopathy The correct choice is E, proliferative retinopathy. The distinguishing factor in the patient's presentation, which signals this disorder, is the development of newly formed vessels. Proliferative retinopathy is the leading cause of blindness in the United States. Up to 20% of patients with type 2 diabetes have retinopathy at the time of diagnosis. Choice A, background retinopathy, or simple retinopathy includes retinal microaneurysms, hemorrhages, exudates, and edema, without new vessel formation. Choice B, closed angle glaucoma, is relatively uncommon in patients with diabetes, except after cataract extraction. Choice C, macular degeneration, is not associated with diabetes mellitus specifically. Choice D, diabetic cataracts, tends to occur in patients with diabetes earlier than the general population, and may correlate with the severity of the disease.

Your patient is a 26-year-old mother of two young children whose second child was just delivered two weeks ago and she is breast-feeding. She complains of insomnia and depression. She denies the use of any drugs or alcohol. You would like to hold off on pharmacologic therapy unless her symptoms progress. Which of the following will be your first line of treatment? A Encourage more time with the baby B Encourage her to begin planning to get back to work C Promote adequate sleep D Suggest she get out of the house more often E Suggest her husband help more around the house

Promote adequate sleep The correct answer is to promote adequate sleep (C). Postpartum "blues" are very common. Depression can occur in some cases, but most resolve without therapy. When required, SSRIs may be used even when a woman is breast feeding, though no studies have been done.

A 56-year-old African American man with a history of hypercholesterolemia and obesity presents with a 6-week history of reduced urinary stream. Upon further questioning, he also admits to generalized weakness, loss of appetite, and a dull lower backache during this time, which he attributes to being "out of shape." He denies fever, chills, chest pain, shortness of breath, abdominal pain, hematuria, frequency, hesitancy, flank pain, dysuria, abnormal penile discharge, a history of trauma, alcohol use, or smoking. He states that he had an extramarital affair about 2 months ago and did not use any barrier methods during sexual intercourse. The physical exam is notable for a male in no acute distress with normal vital signs. He is found to have a distended bladder on abdominal exam, vertebral tenderness of the lumbosacral spine, and firm, nontender nodularity of prostate upon digital rectal exam. Question What is the most likely diagnosis in this patient? Answer Choices 1 Acute bacterial prostatitis 2 Benign prostatic hypertrophy 3 Bladder cancer 4 Prostate cancer 5 Urolithiasis

Prostate Cancer Explanation This patient's presentation is most consistent with prostate cancer. Risks associated with prostate cancer include a high-fat diet, family history, and African American ethnicity. Upon digital rectal exam (DRE), it may manifest as focal nodules or areas of induration within the prostate. Obstructive voiding symptoms can occur with prostate cancer or benign prostatic hypertrophy; however, the prostate is more likely to demonstrate the absence of the median sulcus in BPH, not nodularity, as is observed in cancer. Manifestations of metastatic and advanced prostate cancer may also include weight loss and loss of appetite, anemia, bone pain (with or without pathologic fracture, most likely of the lumbar spine), neurologic deficits from spinal cord compression, and lower extremity lymphedema secondary to lymph node metastasis. Acute bacterial prostatitis typically presents with fever, chills, malaise, arthralgias, myalgias, perineal or prostatic pain, dysuria, and obstructive and irritative urinary tract symptoms, including frequency, urgency, dysuria, nocturia, hesitancy, weak stream, and incomplete voiding. There may also be lower abdominal or back pain and spontaneous urethral discharge. The prostate will be tender, nodular, hot, boggy, or normal-feeling on digital rectal examination in acute prostatitis. Suprapubic abdominal tenderness and an enlarged tender bladder due to urinary retention may also be present. Absence of systemic symptoms and persistence of pain for at least 3 months indicates chronic prostatitis. Risk factors that favor the development of bladder cancer include cigarette smoking and exposure to industrial dyes or solvents. Common presenting findings include gross or microscopic hematuria and irritative voiding symptoms such as frequency and urgency. Metastasis may cause hepatomegaly, lymphadenopathy, and lymphedema associated with involvement of pelvic lymph nodes. A form of nephrolithiasis, ureterolithiasis is caused by calculi in the ureters. It presents as abrupt, severe, colicky pain in the flank and ipsilateral lower abdomen. There is often radiation to testicles or vulvar area with intense nausea with or without vomiting. There is significant costovertebral angle tenderness; pain can move to the upper/lower abdominal quadrant coinciding with the migration of the ureteral stone. Patients typically are constantly changing body positions, such as writhing and pacing about. Tachycardia, hypertension, and microscopic hematuria are common.

A 30-year-old man presents with a 1-month history of severe indigestion. He has tried over-the-counter antacids for the last 2 weeks to help relieve the pain, but they provided only mild relief. He describes the pain as a dull, aching, "hunger-like pain". He also states that a few hours after eating the pain lessens, and then worsens a few hours later. His physical examination is unremarkable except for epigastric pain on deep palpation of the abdomen. His stool guaiac test is negative. Question What therapeutic regimen should be prescribed? Answer Choices 1 H2-blocker therapy and bismuth therapy 2 Dietary modifications 3 Continue antacids and modify diet 4 Stress reduction, bismuth therapy, and dietary modifications 5 Proton pump inhibitor therapy, dietary modifications, and stress reduction

Proton pump inhibitor therapy, dietary modifications, and stress reduction The clinical picture is suggestive of peptic ulcer disease (PUD). The treatment of choice is proton pump inhibitors due to their ease of use and efficacy. Dietary modifications and stress reduction will assist in the decrease of stomach acid production. The hallmark symptom of PUD is epigastric pain (dyspepsia). The pain can be described as aching, dull, or gnawing. About 50% of patients do get some relief with over-the-counter antacids or other treatments. Some patients may have gastric pain that awakens them at night. Duodenal ulcers are usually indicated when there is relief of pain for a few hours after eating. H2-blocker therapy is effective in treating PUD, but it has been replaced by proton pump inhibitors. Dietary modifications alone are not helpful in this situation due to the patient experiencing severe dyspepsia. Continuing antacids would not be helpful since they offer only mild relief; they are no longer used as first-line therapy for active ulcers. Bismuth therapy (Pepto-Bismol), which is another over-the-counter antacid, is no longer used as first-line therapy for active ulcers; however, it is included in quadruple first-line therapy.

A 6-year-old boy presents with fever and cough. He has history of several episodes of pneumonia. A sweat test reveals an increased amount of chloride, indicating that he has cystic fibrosis. He is coughing up thick, greenish sputum. Temperature is 37.6° C. A Gram stain of the sputum reveals Gram-negative rods and a culture grows a Gram-negative rod that is oxidase-positive and produces a blue-green pigment. What is the most likely cause of the infection? Answer Choices 1 Pseudomonas aeruginosa 2 Haemophilus influenzae 3 Bordetella pertussis 4 Legionella pneumophila 5 Streptococcus pneumoniae

Pseudomonas Pseudomonas aeruginosa is a Gram-negative, oxidase-positive rod that does not ferment glucose, with a typical metallic sheen of the growth on TSI agar, coupled with the blue-green pigment on ordinary nutrient agar and a fruity aroma. P. aeruginosa produces 2 pigments: (1) pyocyanin which can color the pus in a wound blue, and (2) pyoverdin (fluorescein), a yellow-green pigment that fluoresces under ultraviolet light. P. aeruginosa is the only species of Pseudomonads that synthesizes pyocyanin. Pseudomonads are able to grow in water containing only traces of nutrients, and they have a remarkable ability to withstand disinfectants. Strains of P. aeruginosa isolated from cystic fibrosis patients have a prominent slime layer (glycocalyx), which gives their colonies a very mucoid appearance. P. aeruginosa is primarily an opportunistic pathogen that causes infections in hospitalized patients, those with chronic respiratory disease in whom the normal clearance mechanisms are impaired, immunosuppressed patients with neutrophil counts of less than 500/μL, and patients with indwelling catheters. P. aeruginosa can cause infections virtually anywhere in the body, but urinary tract infections, pneumonia (especially in cystic fibrosis patients) and wound infections (especially burns) predominate. Haemophilus influenzae is a small Gram-negative rod with a polysaccharide capsule. It is the leading cause of meningitis in young children, and it is an important cause of upper respiratory tract infections and sepsis in children; also, it causes pneumonia in adults, particularly those with chronic obstructive lung disease. Bordetella pertussis is a small, coccobacillary, encapsulated Gram-negative rod. It causes whooping cough (pertussis). Legionella pneumophila is Gram-negative rod that stain faintly with the standard Gram stain. Sputum Gram stains reveal many neutrophils, but no bacteria. The organism fails to grow on ordinary media in a culture of blood or sputum, but it will grow on medium supplemented with iron and cysteine. L. pneumophila causes atypical pneumonia, both in the community and in hospitalized immunocompromised patients. Streptococcus pneumoniae causes pneumonia, bacteremia, meningitis and infections of the upper respiratory tract. Pneumococci are Gram-positive, lancet-shaped cocci arranged in pairs or short chains.

A 70-year-old woman with long-standing type II diabetes mellitus presents with complaints of pain in the left ear with purulent drainage. On physical examination, the patient is afebrile. The pinna of the left ear is tender, and the external auditory canal is swollen and edematous. The white blood cell count is normal. Question What organism is most likely to grow from the purulent drainage? Answer Choices 1 Candida albicans 2 Haemophilus influenza 3 Moraxella catarrhalis 4 Pseudomonas aeruginosa 5 Streptococcus pneumoniae

Pseudomonas Aeruginosa Explanation Ear pain and drainage in an elderly diabetic patient must raise concern about malignant external otitis. The swelling and inflammation of the external auditory meatus strongly suggest this diagnosis. This infection usually occurs in older, poorly controlled diabetics, and it is almost always caused by P. aeruginosa. It can invade contiguous structures, including the facial nerve or temporal bone, and it can even progress to meningitis. S. pneumoniae, H. influenzae, and M. catarrhalis frequently cause otitis media, but not external otitis. Candida albicans almost never affects the external ear.

A 52-year-old man presents with rectal pain during bowel movements. Patient describes his pain as tearing. Physical examination reveals a tear in the anal mucosa. What is considered first-line treatment in this case? Answer Choices 1 Psyllium 2 Topical nitroglycerin 3 Oral ciprofloxacin 4 Botox injection 5 Surgery

Psyllium is the correct response. The patient has an anal fissure. Anal fissure is exacerbated by constipation. First-line treatment is fiber supplementation, such as psyllium and sitz baths. Topical nitroglycerin is an incorrect response. Nitroglycerin is used in the treatment of anal fissure, but is not a first line treatment. Oral ciprofloxacin is an incorrect response. The patient has no signs of infection, so there is no role for antibiotics. Botox and surgery are incorrect responses. These therapies are reserved for refractory cases.

A 44-year-old healthy man, who is a non-smoker, has a 3-cm coin lesion in the right upper lobe. The lesion was revealed on his chest radiograph. The patient did not have any clinical symptoms. The fine-needle aspiration did not reveal any cells. What is the most likely tumor in this case? Answer Choices 1 Pulmonary hamartoma 2 Bronchial carcinoid 3 Mesothelioma 4 Metastatic adenocarcinoma 5 Large cell undifferentiated carcinoma

Pulmonary Hamartoma A peripheral 'coin lesion' could be a granuloma, carcinoma, or a hamartoma. The most likely malignancy in a non-smoker would be adenocarcinoma. Pulmonary hamartoma is the most common benign lung tumor, although it is a relatively uncommon lesion. It is likely an acquired lesion from peribronchial mesenchyme. The patients are generally asymptomatic. The lesion is almost always solitary and is usually peripheral; hence, it does not reveal any cells on fine-needle aspiration. Histologically, hamartomas are composed of both mesenchymal and epithelial elements. A bronchial carcinoid is (usually) a central benign tumor with some of the properties of malignant growth. Mesothelioma is a bulky pleural mass. It is rare, even in smokers with asbestos exposure. Metastatic adenocarcinoma usually produces multiple lung nodules. Large cell undifferentiated carcinoma is a large peripheral mass.

A 33-year-old African-American woman with no significant past medical history, who is in her 38th week of a normal pregnancy, presents with a 1-hour history of shortness of breath. She does not recall any precipitating activities or events that may have provoked these symptoms and recalls that she was laying in her bed for several hours since waking when the symptoms developed. She admits to an associated sharp, nonradiating pleuritic chest pain, as well as lower extremity swelling, which she states has been "persistent throughout the course of her pregnancy". She denies palpitations, chest pressure, cough, sputum, fever, chills, changes in weight, rashes, diaphoresis, abdominal pain, nausea, or a history of allergies. Her physical exam is noteworthy for tachypnea and tachycardia; however, the rest of the vital signs are normal. Her lungs are clear to auscultation bilaterally, without wheezing, rhonchi, or crackles. Her lower extremities are remarkable for 2+ pitting edema up to the level of her knees; there is no calf tenderness, venous cords, or Homan's sign appreciated. Her skin and mucous membranes were without diaphoresis or cyanosis. A bedside EKG performed reveals sinus tachycardia at 120 bpm with prominent S waves in lead I and Q waves in lead III. Question What is the most likely diagnosis? Answer Choices 1 Acute myocardial infarction 2 Bacterial pneumonia 3 Pulmonary edema 4 Pulmonary embolism 5 Bronchial asthma

Pulmonary embolism This patient has a presentation most consistent with pulmonary embolism. Nearly all PEs arise from deep venous thrombosis (DVT) in the lower extremity or pelvic veins. Risk factors for DVT and PE are similar in children and adults and include conditions that impair venous return, conditions that cause endothelial injury or dysfunction, and underlying hypercoagulability disorders. Bed rest and confinement without walking, even for a few hours, are common precipitators. Specific risks include an age of greater than 60, atrial fibrillation, cigarette smoking (including passive smoke), estrogen receptor modulators, exogenous estrogens, and progestins (including oral contraceptives and estrogen therapy), extremity or pelvic trauma, heart failure, hypercoagulability disorders, immobilization, indwelling venous catheters, myeloproliferative disorders, nephrotic syndrome, obesity, pregnancy and postpartum states, sickle cell anemia, recent surgeries, and prior venous thromboembolism. Larger emboli cause acute dyspnea, pleuritic chest pain, or both. Dyspnea may be intermittent or occur only with exercise. Less common symptoms include cough and hemoptysis. The most common signs of PE are tachycardia and tachypnea. Less commonly, patients have hypotension, a loud 2nd heart sound (S2) due to a loud pulmonic component (P2), and crackles or wheezing. In the presence of right ventricular failure, distended internal jugular veins and a right ventricular heave may be evident, and right ventricular gallop (3rd and 4th heart sounds [S3 and S4]), with or without tricuspid regurgitation, may be audible. ECG most often shows tachycardia and various ST-T wave abnormalities, which are not specific for PE. An S1Q3T3 or a new right bundle branch block may indicate the effect of abrupt rise in right ventricular pressure on right ventricular conduction; these findings are moderately specific but insensitive for PE. Pulmonary edema typically causes severe dyspnea, the production of pink, frothy sputum, and diaphoresis and cyanosis. Rales are present in all lung fields, as are generalized wheezing and rhonchi. The chest X-ray usually indicates vascular redistribution, blurriness of vascular outlines, increased interstitial markings, and, characteristically, the butterfly pattern of distribution of alveolar edema. The patient's younger age, quality of chest pain, lack of gastrointestinal and skin findings, and absence of characteristic EKG findings make acute myocardial infarction a less likely diagnosis. The rapid onset of symptoms and denial of fever, cough, and sputum suggest a diagnosis other than bacterial pneumonia, as does the absence of an allergic history and adventitious lung sounds.

A 20-year-old Caucasian man presents with a 1-month history of tingling sensations in his legs. He is on isoniazid treatment for pulmonary tuberculosis. On examination you note a greasy yellow scale over his scalp and eyebrows. Laboratory investigations reveal a hemoglobin level of 9 g/dl. Question What is the most likely diagnosis? Answer Choices 1 Vitamin A deficiency 2 Pyridoxine deficiency 3 Vitamin C deficiency 4 Niacin deficiency 5 Vitamin E deficiency

Pyridoxine deficiency In cases of pyridoxine or (vitamin B6 deficiency), patients can present with peripheral neuropathy, seborrheic dermatosis, glossitis, and cheilosis. Laboratory investigations reveal anemia with lymphopenia. Causes include malabsorption as well as medications, such as isoniazid and penicillamine. Dietary sources of vitamin B6 include liver, legumes, whole grain cereals, and meats. Vitamin E deficiency may cause a hemolytic anemia in premature infants. Laboratory investigations reveal low plasma tocopherol levels, a low hemoglobin level, reticulocytosis, hyperbilirubinemia, and creatinuria. Causes of vitamin E deficiency in premature infants include limited placental transfer of vitamin E and the resultant low levels at birth combined with its relative deficiency in the infant diet. Dietary sources for older children and adults include wheat germ, vegetable oils, egg yolk, and leafy vegetables. In cases of vitamin A deficiency, patients can present with inability to see well in dim light or night blindness. There may also be conjunctival and corneal xerosis, as well as pericorneal and corneal opacities, and Bitot's spots. Bitot's spots are a collection of keratin appearing as triangular foamy spots on the conjunctiva. Patient may also have xeroderma, hyperkeratotic skin lesions, and increased susceptibility to infections. Causes include inadequate dietary intake and malabsorption. Dietary sources of vitamin A include fish, liver, egg yolk, butter, cream, dark green leafy vegetables, and yellow fruits and vegetables. Niacin deficiency causes pellagra, which is characterized by: A symmetrical dermatitis, usually on parts of the body exposed to sunlight Scarlet glossitis and stomatitis Diarrhea Mental aberrations, such as memory impairment, depression, and dementia. These may appear alone or in combination. Causes include inadequate dietary intake, especially in patients with corn-based diets or alcoholism. Dietary sources include legumes, yeast, meat, and enriched cereal products. In cases of vitamin C (or ascorbic acid) deficiency, patients can present with bleeding tendencies (as a result of weakened capillaries) and impaired wound healing due to impaired formation of connective tissue. On examination, the gums may be swollen and friable; the teeth may be loose. There may also be multiple splinter hemorrhages on the nails and ecchymoses, especially over the lower limbs. Causes include inadequate dietary intake and certain conditions, such as pregnancy and lactation, which increase vitamin C requirements. Dietary sources of vitamin C include citrus fruits, such as oranges, lemons, and tangerines, as well as tomatoes and potatoes.

A 28-year-old woman presents because she is concerned about her 2, 4 and 6-year-old children. She doesn't have any symptoms, and there is nothing abnormal in her medical records. However, her father was recently diagnosed with medullary thyroid cancer due to the presence of MEN 2 A, and she is asking if the condition hereditary; if so, she wants to know what can be done in order to prevent her children from developing the malignancy. Question You explain the autosomal dominant trait of the disease; what advice should you provide? Answer Choices 1 RET mutation test in the patient 2 RET mutation test in children 3 Plasma calcitonin levels in children 4 Prophylactic total thyroidectomy 5 Carcinoembryonic antigen test in children

RET mutation test in the patient Multiple endocrine neoplasia type 2A (MEN 2A) is a rare familial autosomal dominant multiglandular syndrome. MEN 2A is diagnosed clinically by the occurrence of 2 or more specific endocrine tumors in a single individual or in close relatives: medullary thyroid carcinoma; hyperparathyroidism (hyperplasia or multiple adenomas), pheochromocytomas (often bilateral); or Hirschsprung disease. Patients with MEN 2A should have genetic testing for the RET proto-oncogene (RET) mutation. RET mutation test identifies about 95% of affected individuals. If the mutation is present, the first-degree relatives may be tested for the specific RET mutation; each kindred has a specific RET codon mutation that determines MEN 2 A variation (e.g., the age of onset or the aggressiveness of medullary thyroid cancer). Both the specific mutation and family history should guide you in the further management. Because of the incomplete penetrance, about 30% of those with such mutations never manifest endocrine tumors. In the affected family members, you may consider regular check-ups for MEN 2-associated pheochromocytomas (24 hours urine for elevated catecholamines and catecholamine metabolites, especially vanillyl-mandelic acid at the earliest age possible), medullary thyroid carcinoma (serum calcitonin or serum calcitonin/pentagastrin-stimulated calcitonin levels), thyroid tumours (ultrasound and fine-needle aspiration), and annual testing for parathyroid abnormalities (simultaneously elevated serum calcium and parathyroid hormone levels with an elevated urinary calcium to creatinine ratio). The results should guide you in the scheduling profilactic thyroidectomy. An RET mutation test should be performed in children if the mother has RET mutation. Such testing is highly accurate and sensitive for presymptomatic identification of at-risk individuals and test for the specific mutation will determine the best next step in order to reduce morbidity and mortality. Plasma calcitonin levels in children to test for the presence of medullary thyroid carcinoma should be considered if they are positive for the specific RET mutation in order to decide the further steps of management. This test is also used to look for the possible recurrence of MTC after treatment. Prophylactic thyroidectomy shall be planned only in the presence of the specific mutation and case histories of family members. Children harboring this mutation are advised to have a prophylactic total thyroidectomy by the age 6 years. While carcinoembryonic antigen (CEA) is often elevated in thyroid cancers, it is not specific; higher than normal levels may also be due to the breast, colon, lung and pancreatic cancers; other causes include liver and gallbladder disorders, heavy smoking, inflammatory bowel disease, lung infections, pancreatitis, peptic ulcer disease etc. However, CEA can be useful after the thyroidectomy if one suspects a reccurrence of the tumor.

What is the most common pathogen in pediatric bronchiolitis? A Mycoplasma B Chlamydia C Parainfluenza D Respiratory syncytial virus (RSV) E Streptococcus

RSV Viruses are by far the most common reasons for bronchiolitis to occur. RSV is the clear leader in causative agents.

What is the definitive treatment of choice for elderly patients diagnosed with Graves' disease? A Beta blocking agents B Levothyroxine C Methimazole D Total thyroidectomy E Radioactive iodine

Radioactive Iodine The correct choice is E, radioactive iodine. This is the treatment of choice in the elderly because it is efficient, easy to take, and inexpensive. Choice A, beta blocking agents, are useful in the treatment of symptoms of hyperthyroidism, such as palpitations, but they are not a definitive treatment for the disorder. Choice B, levothyroxine, is used for thyroid hormone supplementation in patients with hypothyroidism. Choice C, methimazole, is an anti-thyroid drug that has increased toxicity in the elderly and is more useful in younger patients with mild hyperthyroidism. Choice D, total thyroidectomy, has a limited role as a treatment for hyperthyroidism, and is associated with increased morbidity in the elderly.

A 19-year-old woman presents with weight loss, tiredness, and a decreased appetite. She mentions some nausea, and she vomited "once", but she denies diarrhea, illicit alcohol, and drug use, palpitations, tremor, and skin changes. Her urine dips positive for glucose and ketones. What would you likely see on physical exam? Answer Choices 1 Rapid deep breathing 2 Respiratory rate is unchanged 3 Moist mucous membranes 4 Heart rate is decreased 5 Respiratory rate is decreased

Rapid deep breathing Explanation The patient described here likely has diabetic ketoacidosis, a form of metabolic acidosis. Her respiratory response to metabolic acidosis should be rapid deep breathing (Kussmaul breathing), leading to respiratory alkalosis. Symptoms of diabetic ketoacidosis may be highly variable; symptoms include nausea, vomiting, thirst, polyuria, abdominal pain, and shortness of breath. Physical findings may include hypotension (secondary to volume depletion), tachycardia, dry mucous membranes, hypercapnia, abdominal tenderness, and lethargy. In patients with known diabetes, precipitating events may include inadequate insulin intake, infections (e.g., urinary), myocardial infarctions, and pregnancy; history and related diagnostic testing should focus on these precipitating factors. In patients with both known and new onset diabetes, suspicion of diabetic ketoacidosis warrants an evaluation of serum glucose, ketones, and basic chemistries. Potassium is often elevated due to cellular shift and may fluctuate with insulin treatment; bicarbonate is often decreased secondary to the acid buffering, and urea nitrogen is often elevated due to volume depletion. Management focuses on correcting salt and water deficits over the first 24 hours (first with saline and then with 1/2 normal saline when hemodynamically stable); administering intravenous insulin (0.15 units/kg bolus, then 0.1 unit/kg/hour); and frequent monitoring of serum glucose, potassium and bicarbonate, and acetone during treatment.

Which condition is suggested by urethritis, arthritis, and conjunctivitis? A chlamydial infection B gonococcal infection C reactive arthritis D tertiary syphilis

Reactive arthritis Both chlamydia and gonorrhea infections can result in urethritis. Gonococci can disseminate to the joints and cause septic arthritis. Chlamydia is typically asymptomatic but can cause chronic conjunctivitis in adolescents and young adults. Reactive arthritis (also known as Reiter syndrome) is a result of an untreated chlamydia infection, and although typically characterized, in texts, by the triad of urethritis, arthritis, and conjunctivitis, all of the symptoms may not be present or not identified at the time of presentation. Tertiary syphilis is characterized by neurologic and cardiovascular disease, gumma, auditory and ophthalmic involvement, and cutaneous lesions.

A middle-aged woman presents with diarrhea and vomiting that started this afternoon. She does not remember eating any meat, chicken, pudding, or ice cream the day before. She lives alone and says she warmed up leftover rice for supper last night. She looks sick from vomiting, but she is not dehydrated. She has no fever, and her blood pressure and pulse are within normal limits. What would be the best next step? Answer Choices 1 Admit and observe 2 Start oral fluids and metronidazole 3 Reassure the patient and send home with oral rehydration 4 Send stool specimen to the lab and ask patient to come the next day for follow up 5 Admit for parenteral treatment

Reassure the patient and send home with oral rehydration This patient is suffering from acute food poisoning. The history gives us the clue that she acquired this food poisoning from reheated rice. The most common organism that might cause this condition in this case would be Bacillus cereus. Bacillus cereus is a Gram-positive, spore-forming rod. Spores on grains, such as rice, survive steaming and rapid frying. The spores germinate when rice is kept warm for many hours. B. cereus produces 2 enterotoxins. The mode of action of 1 of the enterotoxins is the same as that of cholera toxin ( i.e., ADP-ribosylates G protein, which stimulates adenylate cyclase and leads to an increased concentration of cyclic AMP within the enterocyte). The mode of the other enterotoxin is uncertain. Clinically, it is a self-limiting situation and requires supportive treatment only. The incubation period is about 18 hours. The best advice for this patient would be to reassure her that it will get better and give her some oral rehydration supplements.

A 40-year-old man presents in January complaining of a 4-day history of fevers, chills, myalgias, headache, productive cough, and mild sinus congestion. He has no significant past medical history. His physical examination reveals a temperature of 102.7° F, pulse 96/min, respiratory rate 20/min, and blood pressure of 128/80. There is mild maxillary sinus tenderness. His oral cavity and oropharynx are clear. His tympanic membranes are pearly gray with normal light reflex. His chest is clear to auscultation; cardiac exam is unremarkable. What is the best course of therapy for this patient? Answer Choices 1 Admit to the hospital for antibiotics and intravenous fluids 2 Treat with oral trimethoprim/sulfamethoxazole for 7 days 3 Prescribe amantadine 4 Prescribe zanamivir 5 Recommend bed rest, analgesics, and topical decongestants only

Recommend bed rest, analgesics, and topical decongestants only The best course of treatment is supportive and symptomatic care with hydration, analgesics, and decongestants. This patient presents with classic symptoms of an influenza infection. He has no evidence of a severe bacterial infection; therefore, antibiotics are not indicated. It is important to note that the flu can cause sinus congestion as well as rhinorrhea. Although amantadine and zanamivir are approved for treatment of influenza infections, they have only been found to be helpful if treatment has begun within 48 hours of the initial symptoms. If prescribed within this period, the duration of illness can be shortened by about 1 day.

A 17-year-old boy presents with intermittent bouts of shortness of breath, coughing, and chest tightness. The symptoms most often occur during football practice, but sometimes they also occur when he is just standing outdoors in cold weather. He denies palpitations, fever, and chills. Past medical history is noncontributory. He is a nonsmoker. Physical exam is unremarkable. Vital signs are as follows: BMI 19 kg/m2; BP 116/70 mmHg; HR 80 bpm; and SpO2 99% on room air. His physician orders pulmonary function testing (PFTs) and a chest X-ray (CXR). Question What set of PFTs is most consistent with the diagnosis of asthma? Answer Choices 1 Reduced forced expiratory volume in 1 minute (FEV1), reduced FEV1/Forced vital capacity (FVC) ratio, and Reduced Residual Volume (RV) 2 Reduced FEV1, Reduced FEV1/FVC ratio, and Increased RV 3 Reduced FEV1, Increased FEV1/FVC ratio, and Increased RV 4 Reduced FEV1, Increased FEV1/FVC ratio, and Reduced RV 5 Increased FEV1, Increased FEV1/FVC ratio, and Reduced RV

Reduced FEV1, Reduced FEV1/FVC ratio, and Increased RV Asthma is a chronic condition that affects pulmonary function. The pathophysiologic process underlying asthma is inflammation. In response to specific triggers, an immune-mediated cascade occurs that leads to recruitment of inflammatory mediators and airway constriction. Although this is a normal response to airway irritants, the airway of patients with asthma is hyperresponsive. Bronchial constriction increases the work of breathing, particularly the exhalation phase. This is manifested by reduced FEV1 (volume of air that can be forcibly exhaled in the first second of exhalation after maximal inspiration) and reduced FEV1/FVC ratio (FVC = volume of air that can be forcibly blown out after a maximal inspiration). Inability to adequately exhale leads to air trapping, which causes an increase in residual volume. Common treatments for asthma include inhaled beta-2 agonists, which promote relaxation of airway smooth muscle as well as inhaled corticosteroids that minimize inflammatory-mediated bronchoconstriction.

A 68-year-old man with a history of cirrhosis presents due to weight gain, increased girth, and shortness of breath. He denies fever or chills, cough, melena, hematemesis, hemoptysis, and confusion. He takes furosemide on a daily basis and has not missed any doses. On physical exam, his blood pressure is 120/80; pulse is 78, and respiratory rate is 18. He is alert and cooperative. Heart exam: regular rate and rhythm without murmurs. Lung exam: decreased breath sounds on right side. Abdominal exam: positive fluid wave, no tenderness to palpation. There is no hepatosplenomegaly appreciated. Chest X-ray shows a right-sided pleural effusion; it is free flowing, as is evidenced on a decubitus film. Question What is the etiology of his pleural effusion? Answer Choices 1 Altered permeability of pleural membrane 2 Reduced intravascular oncotic pressure 3 Increased hydrostatic pressure 4 Decreased lymphatic drainage 5 Reduced pressure in pleural space

Reduced intravascular oncotic pressure Pleural effusions, an abnormal collection of fluid in the pleural space, is related to an underlying pathology causing excess fluid production or decreased absorption. Effusions are classified as transudative or exudative depending on the characteristics of the fluid. Patients with liver disease/cirrhosis have decreased protein and albumin production, leading to decreased oncotic pressure of the intravascular fluid. This contributes to ascites and dependent edema. Peritoneal ascites may travel across the diaphragm and lead to pleural effusions (hepatic hydrothorax), most commonly on the right side. Pleural effusions caused by decreased oncotic pressure are transudative. Treatment is aimed at reducing the ascitic fluid using diuretics (loops and/or spironolactone). Severe cases may require treatment with a transjugular intrahepatic portosystemic shunt (TIPS) or liver transplant. Other causes of hypoalbuminemia, such as nephrotic syndrome, also cause pleural effusions due to reduced intravascular pressure. Altered permeability of the pleural membrane causing a pleural effusion is seen with localized inflammation, malignancy, or pulmonary embolism. Decreased lymphatic drainage is commonly seen in malignancy (lung, breast, or lymphoma). The effusion is exudative and almost always unilateral on the side with the associated pathology. Pleural effusions caused by congestive heart failure are due to increased hydrostatic pressure. They are usually bilateral and transudative. Reduced pressure in the pleural space prevents full lung expansion, allowing pleural fluid accumulation. Causes of this include atelectasis (transudative) or mesothelioma (exudative).

A woman brings her 13-year-old daughter to clinic, concerned about the fact that she has not yet had her first menstrual cycle. The patient is PMH negative, Social Hx negative, FMH: mothers' age of menarche was 13. Physical exam reveals the child to be a well-developed, well-nourished female, height and weight at the 50% for age and gender. Thelarche is present and has sparse pubic hair. The remainder of her physical exam is normal for age. The most appropriate course of action is? A Reevaluation in 6 months B Thyroid evaluation C Pelvic exam D Pelvic ultrasound E hCG

Reevaluation in 6 months A The child is in the expected age for puberty, and she shows signs of hormonal changes. Thelarche- onset of breast development

A 55-year-old man presents with increased urinary frequency; he is also experiencing difficulty starting and maintaining a steady stream, nocturia, and the inability to achieve an erection for the past few months. During rectal examination, you note a non-tender, non-enlarged prostate with an isolated left posterior lobe nodule. What option is the most appropriate? Answer Choices 1 Refer the patient for a transrectal ultrasound of the prostate and order a PSA level 2 Order a serum acid phosphatase 3 Reassure the patient and schedule a follow-up appointment in 6 months 4 Initiate prazosin and schedule a follow-up appointment in 4 weeks 5 Initiate norfloxacin therapy for 7 days and schedule a follow-up in 2 weeks

Refer the patient for a transrectal ultrasound of the prostate and order a PSA level Explanation The clinical picture is suggestive of prostate cancer. Signs and symptoms include those similar to benign prostatic hypertrophy (BPH) such as urinary frequency, urinary hesitancy, nocturia, hematuria, and difficulty achieving erections. Most patients are asymptomatic. Prostate cancer may be manifested as focal nodules or areas of induration within the prostate. Modern transrectal ultrasound provides high-definition images of the prostate. It also guides biopsy. Measurement of the PSA is useful in detecting and staging prostate cancer. Serum acid phosphatase would not be useful since it is found in many organs. A prostatic acid phosphatase would be more specific in diagnosing prostate cancer. Since a nodule was found in this patient, it needs to be investigated further, and reassuring the patient would be considered inappropriate medical practice. Prazosin is an alpha-blocker used in the treatment of BPH. BPH usually results in a smooth, firm, elastic enlargement of the prostate, which is not seen in this patient. Norfloxacin is an antibiotic used in the treatment of urinary tract infections. Signs and symptoms of a UTI include painful urination, increased frequency, and an odd smell to the urine. Urinalysis may show positive leukocyte esterase, nitrates, and blood, with WBCs and bacteria on microscopic examination. None of these symptoms/reults are indicated in this patient.

A 21-year-old woman presents with double vision that occurs only when she looks to her right side. The double vision began when she woke up in the morning. She had an episode of left leg numbness while at summer camp 6 years ago, but it resolved over 3 days and she never told anyone. Also, 3 years ago she saw her private physician after a 7-day episode of right eye pain and visual blurring. He attributed her symptoms to an ocular migraine. There is no history of head trauma. She hasn't had any infections, fevers, or immunizations recently. She is awake, alert, and in no acute distress. Positive findings include mild pallor and atrophy of the right optic disc. Bedside visual fields and acuity are normal. Testing external ocular motion in both eyes together reveals that there is no left eye movement beyond midline when attempting to look to her right, accompanied by right eye lateral nystagmus. When the left eye is tested with the right eye closed, eye movements are full. No other motor signs are found. No skin, sensory, or hearing findings are found. A magnetic resonance imaging scan (MRI) of the head with gadolinium enhancement reveals a 2 x 3 cm lucency in the region of the right parietal white matter without swelling or enhancement. Multiple sclerosis (MS) is suspected. Question Given this history, how would this patient's condition best be subtyped? Answer Choices 1 Relapsing Remitting MS (RRMS) 2 Secondary Progressive MS (SPMS) 3 Primary Progressive MS (PPMS) 4 Progressive Relapsing MS (PRMS) 5 There is insufficient data at this time to subclassify

Relapsing remitting MS The patient has had 3 discrete clinical exacerbations (right parietal white matter/left leg numbness, right optic nerve/ visual blurring, eye pain with residual disc pallor, and median longitudinal fasciculus/resolved diplopia), with full clinical resolution. It should be kept in mind that this is a clinical descriptive and disability rating. Therefore, residual abnormalities on exam and ancillary lab testing do not define the subtype. The patient is best subtyped at this time as Relapsing Remitting MS. Subtyping may change over time with new clinical events. This patient might later develop persistent clinical disability (e.g., severe ataxia, persistent urinary incontinence, extremity weakness, diplopia, etc.). She would then best be reclassified as Secondary Progressive MS, whether these occurred as residua of discrete relapses or as an independent progressive deterioration independent of relapses. Unlike the classic Relapsing/Remitting presentation of MS, some patients present with a progression of neurological disability with no clear relapses, per se. These are subtyped as Primary Progressive MS. Some patients may initially rate a Primary Progressive MS rating, yet later develop episodes of superimposed relapses and remissions. These patients are then reclassified as Progressive Relapsing MS.

A 50-year-old woman has type 2 diabetes mellitus; she has been taking 500 mg of metformin daily for approximately 1 year. She has a hemoglobin A1c of 6.5%. She states that she carefully watches her diet, walks 1 mile almost every day, and monitors her blood sugar levels in the evening and before eating breakfast. Her blood sugar levels are often higher in the morning than they were the previous evening even though she does not eat anything. To what process can this be attributed? Answer Choices 1 Release to the bloodstream of glucose from glycogen in muscle 2 Release to the bloodstream of glucose from glycogen in the liver 3 Release to the bloodstream of glucose (from gluconeogenesis) from amino acids occurring in the liver 4 Continuous absorption of glucose into the bloodstream formed by the digestive system 5 Release to the bloodstream of glucose (from gluconeogenesis) from lactate occurring in the muscle

Release to the bloodstream of glucose from glycogen in the liver Explanation The correct response is the release to the bloodstream of glucose from glycogen in the liver. The major source of glucose for the body during an overnight fast is the glucose released from glycogen stored in the liver. As blood glucose levels decrease, glucagon levels increase. The binding of glucagon to receptors in the liver triggers the activation of a cascade, resulting in the activation of the enzyme glycogen phosphorylase. This enzyme catalyzes the release of glucose 1-phosphate from glycogen. The glucose-1-phosphate is converted to glucose-6-phosphate by the action of phosphoglucomutase; the resulting glucose 6-phosphate is converted to glucose by glucose-6-phosphatase. The glucose formed is then released to the bloodstream; therefore, even though the patient did not eat overnight, the action of glucagon on the liver generates glucose which could increase the patient's serum blood glucose. Although muscle contains large amounts of glycogen which can be broken down, the muscle lacks the enzyme glucose 6-phosphatase; therefore, glucose 6-phosphate remains in the muscle to be used to generate ATP for muscle contraction. Most individuals have adequate liver glycogen stores to provide glucose for an overnight fast; they will not be breaking down protein to generate amino acids which could be used for gluconeogenesis (the generation of glucose from non-carbohydrate sources).

A 22-year-old woman develops fever, rash, arthralgias, and decreasing urine output 2 weeks after completing a course of penicillin to treat streptococcal pharyngitis. Her physical exam is normal. She takes no other medications or supplements and has no other past history or symptoms. Her laboratory work results are as follows: Total white blood cell count 7.0x103/cmm Eosinophils elevated Serum creatinine 1.5 mg/dl Serum potassium 4.0 meq/l Serum bicarbonate 23 meq/l Fasting Blood glucose 80 mg/dl Urinalysis no casts, no bacterial growth, 2 red cells per high powered field, numerous white cells, trace protein Erythrocytes sedimentation rate elevated Question What diagnostic study will confirm her diagnosis? Answer Choices 1 Renal ultrasound 2 Renal biopsy 3 Renal gallium scan 4 Urine eosinophil count 5 24 hour urine for creatinine clearance

Renal biopsy Explanation This patient most likely has acute interstitial nephritis (AIN) secondary to her penicillin exposure. Of the listed choices, only renal biopsies yield information specific to acute interstitial nephritis. Renal biopsy is the criterion standard for diagnosing AIN. Lymphocytic and plasma cell infiltrates in the peritubular areas of the interstitium are noted. However, being an invasive procedure, it is not used in all patients, especially if the condition is mild or if the patient improves rapidly after removing the offending cause. Renal ultrasound may show slight increases in renal size and cortical echogenicity in AIN, but this may also occur with other renal conditions. A clinician may order this test to evaluate for other forms of injury (such as acute renal obstruction) in the evaluation of acute kidney injury. 24-hour urine samples are used to assess urine output, daily protein, electrolyte excretion, and creatinine clearance, a measure of renal filtering ability. Currently, calculations of creatinine clearance are made using the modification of diet in renal disease (MDRD) equation. Here, a 24-hour urine collection will tell us how well her kidneys are working, but it will not show the cause of their decline in function. Elevated Urinary eosinophils may be found in a variety of other diseases, including pyelonephritis and prostatitis. The positive predictive value of urine eosinophils for diagnosing AIN is low (Kodner). Gallium scans have limited predictive value for diagnosing AIN (Markowitz). Cortical necrosis (i.e., secondary to ischemia) unrelated to AIN and other diseases may cause similar patterns of uptake as AIN. If the patient was still on the medication suspected to cause the problem, it should be discontinued and never again used. This alone may cause resolution of her illness in a couple of weeks. Her symptoms, urine output, volume status, serum creatinine, and electrolytes should be monitored to evaluate for the need for dialysis. She can be started on prednisone therapy for 2 weeks, to be tapered thereafter.

A 55-year-old woman presents with a 2-month history of gross hematuria. She states she has no pain with urination, but the hematuria is persistent. On questioning, she states that she does have some progressively worsening left flank pain and persistent back pain over the past month. The pains are not debilitating, but they are nagging. She has no chronic medical problems. She admits to a 50-pack year smoking history, and she states she is currently retired from her job as a teacher. Vital signs are within normal limits, and physical exam reveals a left side abdominal mass. Urine dipstick only shows too numerous to count RBCs, and urine cultures are negative. Question What is the most likely diagnosis? Answer Choices 1 Acute cystitis 2 Bladder cancer 3 Renal cell carcinoma 4 Urethritis 5 Ureteral calculi

Renal cell carcinoma Explanation Hematuria is the most common presenting sign of urinary tract cancer. Silent or painless hematuria suggests tumor or renal parenchymal disease. Renal cell carcinoma can present with flank pain, hematuria, persistent back pain, and an abdominal mass; also, it can be found incidentally on CT scan; therefore, the clinical picture points to cancer with renal cell carcinoma as the most likely diagnosis. Smoking is a risk factor for renal cell cancer. RCC is more common in men than women (2:1), and it has a peak incidence in the 6th decade of life. Bladder cancer often presents with gross hematuria, but it is most typically painless. The left flank pain and back pain are more characteristic of RCC. Bladder cancer is the 2nd most common urologic cancer, and the mean age at diagnosis is 65 years. It is more common in men than women (2.7:1), and 98% of primary bladder cancers are epithelial malignancies (majority urothelial cell carcinomas). Acute cystitis typically presents with irritative voiding symptoms (frequency, urgency, dysuria) and suprapubic discomfort in addition to possible hematuria. Urinalysis will show pyuria, bacteriuria, and varying degrees of hematuria. Urine cultures will show specific organisms. Urethritis is inflammation of the urethra that presents with urethral discharge, dysuria, and itching. Urethritis is most often caused by a STD. This patient does not have a history of unprotected sexual intercourse or any other symptoms that would indicate this diagnosis. Ureteral calculi can present with hematuria, but it also typically presents with flank or abdominal pain as well, but no abdominal mass. If the stone is in the ureter, it often causes some hydroureter with or without hydronephrosis, both of which cause some pain or discomfort. The patient has no past history of forming stones.

A 62-year-old man is hospitalized because of a 1-week history of extreme malaise and painful skin changes. At the beginning, it was "just a rash", but it then developed into clusters of clear vesicles; new vesicles appear while old ones dry and crust over. His past medical history is significant for diabetes mellitus type II, for which he is on the diet-controlled. You find the rash distributed over his body, and you make a diagnosis of disseminated herpes zoster. Acyclovir IV is introduced, together with the supportive therapy, but after 2 days, the patient starts experiencing nausea and swelling; his urine output dramatically decreased with laboratory studies showing elevated BUN and creatinine. Question What is the most probable mechanism of renal failure in this patient? Answer Choices 1 Renal tubular obstruction 2 Endothelial injury 3 Hypersensitivity reaction 4 Interstitial nephritis 5 Kidney infection

Renal tubular obstruction Explanation Acyclovir precipitates in renal tubules because it is poorly soluble in urine. In that way, it causes the obstruction of renal tubules and acute renal failure. Endothelial injury is characterized by reduced vasodilation, a proinflammatory state, and prothrombic properties; it may be associated with hypertension and diabetes, particularly in type II diabetes with insulin resistance. However, in a patient with acute kidney failure who has been exposed to the high dose of parenteral therapy with a poorly soluble nephrotoxic drug, endothelial injury should not be your initial choice. Renal tubular cells dysfunction due to hypersensitivity reaction to drugs or by infection will cause acute interstitial nephritis. It is often associated with obstruction or reflux, so that you can include this in your list of differential diagnoses. However, the development of acute renal failure during the therapy with acyclovir makes renal tubular obstruction more likely. In acute interstitial nephritis, renal tubular cells dysfunction is caused primarily by a hypersensitivity reaction. When caused by an allergic reaction, the symptoms of acute tubulointerstitial nephritis are fever, rash, and enlarged kidneys. Besides, acyclovir-induced crystalluria that causes mechanical tubular obstruction is the better option in this case. Kidney infection with acute renal failure is not a probable diagnosis in a patient that has no back pain and no signs of a urinary tract infection.

A 29-year-old man has been trying to have a child with his wife for the past 3 years; they have not met with any success. The wife was thoroughly evaluated for infertility; the workup revealed no abnormalities. Examination of his inguinal region reveals an ill-defined tortuous swelling that increases with standing and coughing. Ultrasonography is suggestive of being a varicocele. His initial semen analysis reveals mild oligospermia. Question What is the best next step in management? Answer Choices 1 Embolization of dilated veins 2 Surgical mesh repair 3 Low dose testosterone 4 Repeat semen analysis 5 Surgical excision of dilated veins

Repeat serum analysis Explanation Repeat semen analysis is the correct answer. The results of a single semen analysis are inadequate in making a diagnosis of infertility. For a multitude of reasons, counts vary over time. 2 or 3 separate counts should be taken at least 2 - 4 weeks apart. Despite the presence of a varicocele, if a repeat semen analysis is normal and the varicocele remains asymptomatic, there is no reason for operative intervention. Embolization of dilated veins is incorrect. Embolization is an option in treatment of varicocele. The dilated veins of the pampiniform plexus are embolized using interventional radiology. It is used in the presence of a symptomatic varicocele or if infertility is confirmed. Surgical mesh repair is incorrect. Mesh repair is used in the management of a hernia, not in a varicocele. Low dose testosterone is incorrect. It may be an option if infertility is due to hypogonadism due to low testosterone. It is not an initial step in management. Surgical excision of dilated veins is incorrect. If the varicocele is symptomatic or if infertility is confirmed, surgical excision is an option.

A 32 year-old obese woman presents to the clinic complaining of swelling, warmth, and redness of her left lower extremity. Doppler ultrasound confirms a deep venous thrombosis is the greater saphenous vein. Which of the following best describes the most likely pathophysiology underlying her thrombophilia? A Antithrombin deficiency B Increased plasma prothrombin C Protein C deficiency D Protein S deficiency E Resistance to activated protein C

Resistance to activated protein C Factor V Leiden is a common genetic mutation that results in resistance to activated protein C. Increased plasma prothrombin concentration results from a prothrombin gene mutation (B), while deficiencies of antithrombin (A), protein C (C) and protein S (D) are rarer causes of prothrombotic states and are not directly associated with factor V Leiden mutation which is the most common cause of thrombophilia in this clinical scenario

A 48-year-old female presents with a gradual progression of fatigue, pallor, and dyspnea on exertion over the last few months. Initial CBC results show a hemoglobin of 10.2 mg/dL, hematocrit of 31%, an MCV of 74 fL, and a reticulocyte count of 0.1%. After treatment for her anemia begins, which of the following diagnostic studies can be ordered first (earliest) to demonstrate a response to therapy? A Bone marrow aspirate B Ferritin C Hemoglobin/hematocrit D Iron binding capacity E Reticulocyte count

Reticulocyte count The patient most likely has an underlying iron deficiency anemia (IDA) which requires confirmation and then evaluation for sources of chronic blood loss. IDA is treated with iron replacement (e.g., ferrous sulfate) and response to therapy can be assessed in 7-10 days via a reticulocyte count. The elevation of the reticulocyte count (after initial reticulocyte count was relatively low) at this time demonstrates the return of normal hemoglobin production (E). Bone marrow aspirate (A) would be an inappropriately invasive test to perform in this scenario. The patient's hemoglobin, hematocrit (C), MCV, and iron studies (B and D) will take a couple to a few months to return to normal.

A 42-year-old man presents to the emergency department with a severe headache. He has been getting several of these headaches recently and has tried all over-the-counter pain relievers and headache medicines with no relief. His current headache started 15 minutes ago. He describes the pain as located next to and behind his left eye and "stabbing/excruciating" in nature. He feels like his left eye tears up profusely with these headaches. He reports he has been healthy otherwise, with no chronic medical conditions, no history of surgery, no medications, and no drug allergies. He denies recent stressors that may have caused his headaches. On physical exam, the patient appears slightly agitated and appears uncomfortable. His left eye's conjunctiva is mildly injected, and lacrimation is noted. His right eye is normal. Cranial nerves II-VII are intact, although the patient expresses discomfort when the light is shown in his left eye. Speech, gait, coordination, and reflexes are all normal. The remainder of his exam is normal. Head MRI is performed and reported as normal. Question Which of the following aspects of patient history would be most consistent with this patient's suspected condition? Answer Choices 1 Head injury prior to onset of headaches 2 Periodic episodes of aphasia and gait disturbances 3 Recent exposure to outdoor pollens 4 Rhinorrhea associated with headaches 5 2 to 3-day duration of each headache

Rhinorrhea associated with HA This patient is suffering from cluster headaches. Cluster headaches are classified as trigeminal autonomic cephalalgias and are most common in men. Cluster headaches may be triggered by alcohol, histamine, and nitroglycerin, but often no cause is identified. There may be a genetic component. Classic presentation of cluster headache includes episodic, unilateral, severe headache, usually located in the temporal and/or orbital region, along with associated lacrimation, injection, eyelid edema, miosis or ptosis of the ipsilateral eye, and nasal congestion and rhinorrhea associated with the headaches. Imaging studies, such as head MRI, should be performed to rule out other primary causes of headache, such as vascular malformation, neoplasm, and infection. A head injury prior to the onset of headaches is not associated with cluster headaches. Post-concussion headaches tend to have longer duration, but less severe intensity compared to cluster headaches. Other clues to a post-concussion headache may be memory problems, "fogginess," and nausea, and/or vomiting. The eye symptoms and signs this patient displayed are not consistent with a prior head injury. Periodic episodes of aphasia and gait disturbances would be concerning for a severe disorder, ranging from vascular, infectious, neurodegenerative, and neoplastic in etiology. Patients with cluster headaches would not be expected to have these types of neurologic symptoms. Recent exposure to outdoor pollens would be expected in a patient experiencing allergy-related headaches. A patient with allergy-related headaches would be expected to have much less intense severity and bilateral eye and nasal symptoms. 2 to 3-day duration of each headache can be seen with tension-type headaches and migraines. However, cluster headaches are noted to last 15-180 minutes of duration.

A 30-year-old woman presents with a 2-week history of small wounds on the sides of her mouth. She recently adopted a strict vegetarian diet. On examination, you note she has pale conjunctivae, a magenta tongue, and macerated lips (in addition to the angular stomatitis). Question What is the most likely diagnosis? Answer Choices 1 Thiamine deficiency 2 Riboflavin deficiency 3 Vitamin K deficiency 4 Vitamin D deficiency 5 Iron deficiency

Riboflavin deficiency In cases of riboflavin (or Vitamin B2) deficiency, patients present with angular stomatitis and cheilosis. On examination, they are pale, have atrophic glossitis, and the tongue may appear magenta. They may also have a sebaceous dermatosis with greasy material in their nasolabial folds, alae nasi, and genitals. Causes include inadequate dietary intake. Dietary sources of riboflavin include milk, cheese, meat, and enriched cereal products. Vitamin B1 (or thiamine) deficiency results in beriberi, which is characterized by: A bilateral symmetric peripheral neuropathy beginning in the legs. Wernicke-Korsakoff syndrome, which is comprised of nystagmus, ophthalmoplegia, ataxia, memory loss, and confabulation. Congestive heart failure with tachycardia, peripheral edema, and cardiomegaly. In cases of vitamin K deficiency, patients present with bleeding tendencies, which include epistaxis, menorrhagia, and hematuria. The prothrombin time (PT) and the activated partial thromboplastin time (aPTT) are usually prolonged. In cases of vitamin D deficiency, children can present with inability to walk unsupported due to muscle weakness and lower limb skeletal deformities, such as genu varum and genu valgum. They may also have prominent costochondral junctions. In cases of iron deficiency, patients can present with feeling weak, dizzy, and tired; they may experience syncope. On examination, they have pale conjunctivae and koilonychia (spooning of the nails).

A 57-year-old male is being monitored for Binet Stage A CLL. He is emergently seen in the clinic with rapid lymph node enlargement, fever, weight loss, and hepatosplenomegaly. On laboratory examination, he is found to have an elevated serum lactate dehydrogenase and a monoclonal gammopathy on serum protein electrophoresis. A retroperitoneal ultrasound reveals bulky adenopathy. What is the most likely diagnosis? A. Acute lymphadenitis secondary to HSV B. Epstein-Barr viral infection C.Hodgkin lymphoma D Rapidly advancing CLL E Richter transformation

Richter transformation All findings are classic for Richter transformation of CLL to an aggressive large B-cell, high-grade lymphoma. While HSV cannot be totally ruled out without a node biopsy, given the patient history and findings the most likely diagnosis is Richter transformation. Rapidly advancing CLL does not usually develop retroperitoneal adenopathy. Occasionally, Richter syndrome with Hodgkin lymphoma features is seen, but accounts for less than one-fifth of all cases of Richter transformation. EBV may play a role in this later syndrome.

A 32-year-old woman presents with fatigue, pruritus, and yellowing of the white of her eyes. Investigations reveal anti-mitochondrial antibodies and elevated alkaline phosphatase (ALP), γ -glutamyl transpeptidase (GGTP) levels. Primary biliary cirrhosis is diagnosed, and she is treated with ursodeoxycholic acid, corticosteroids, and cholestyramine. 1 month later, she returns with abdominal discomfort and constipation. Her symptoms cease when she stops taking cholestyramine. However, the itching is severe when the drug is stopped. Question In order to relieve this patient's itching, what drug may be used as an alternative to cholestyramine? Answer Choices 1 Isoniazid 2 Ethambutol 3 Pyrazinamide 4 Streptomycin 5 Rifampin

Rifampin Rifampin has been found to relieve pruritus in biliary disease. Rifampin is an antimycobacterial agent that can be used in patients who cannot tolerate cholestyramine. It is started at a dose of 150 mg per day and may be gradually increased to 600 mg per day. The mechanism of action of rifampin in pruritus of biliary disease is explained as follows: Being a strong enzyme inducer of the microsomal enzyme inducer system, it promotes the metabolism of endogenous pruritogenic compounds. It competes with the uptake of bile salts in the hepatocytes. It may alter the synthesis of secondary bile acids in the intestinal lumen by its antimicrobial action and thus reduce the amount of lithocholic acid. The other choices are all antimycobacterial agents with no role in pruritus of biliary disease.

A 61-year-old man with hypertension presents with sudden onset of right eye blindness while shaving this morning. He describes a "curtain falling over my right eye," which cleared spontaneously after 10 minutes. He had no other concurrent neurologic symptoms and no prior episodes of similar description. His general examination reveals a blood pressure of 140/90 mmHg, but it is otherwise unremarkable. Neurologic exam is normal. Question What is the most likely diagnosis? Answer Choices 1 Right internal carotid artery transient ischemic attack (TIA) 2 Basilar artery TIA 3 Left posterior cerebral TIA 4 Left internal carotid artery TIA

Right ICA TIA The diagnosis in this patient is amaurosis fugax, or transient monocular blindness. This disorder is most often due to ipsilateral internal carotid artery (ICA) atherosclerosis, leading to a transient ischemic attack (TIA) of the retina. A TIA is defined as a focal neurologic deficit (e.g. amaurosis fugax, hemiplegia, aphasia) lasting less than 24 hours, which is referable to the distribution of the carotid or vertebral-basilar circulation. The vascular supply of the retina is through the ophthalmic artery, which is an intracranial branch of the ICA. Thus, this patient had a TIA of right ICA origin. Visual disturbances of the basilar or posterior cerebral circulation include diplopia, bilateral (binocular) blindness, and hemianopia. Atheroembolism from the ICA to the ipsilateral ophthalmic artery is the most likely mechanism of amaurosis fugax. Other causes of TIA include hemodynamic (low perfusion) impairment, arteritis, thrombocytosis, polycythemia vera, and cardiogenic embolus. Rarely, a cerebral mass lesion (neoplasm, abscess, vascular malformation) produces syndromes that masquerade as TIA's. Management of patients with carotid TIA is directed at identifying an accountable vascular (or cardiac) lesion, since the annual risk of ipsilateral stroke is approximately 15%. Appropriate workup for a patient with TIA includes: brain CT or MRI to visualize silent infarcts or mass lesions; carotid duplex, magnetic resonance or CT angiography, and conventional angiography to determine the presence of carotid stenosis; and cardiac evaluation (ECG, echocardiography, Holter monitor) to exclude a cardiogenic source of embolism. Medical therapy for stroke prevention includes antiplatelet agents (aspirin, ticlopidine) for patients with primary carotid disease with less than 70% stenosis; anticoagulation has been shown to be effective for cardiogenic stroke prevention (e.g. atrial fibrillation). Surgical treatment, i.e. carotid endarterectomy, is superior to medical therapy in patients with 70% or greater stenosis of the carotid artery. The latter was confirmed by the North American Symptomatic Carotid Endarterectomy Trial (NASCET) in 1991. The benefits of carotid endarterectomy decline in patients with significant comorbidity (i.e. cancer, coronary artery disease), and this procedure should be performed by experienced surgeons. The treatment of asymptomatic carotid stenosis remains controversial.

You meet with a 15-year-old boy who has a diagnosis within the autism spectrum disorder. The patient is accompanied by his mother. This patient is considered high functioning: he attends a public school, goes to daily cognitive/behavioral sessions, and he attends weekly counseling sessions with a provider who specializes in treating adolescent patients with the diagnosis that is within the autism spectrum disorder. Despite this fact, mom states she has noted a significant increase in episodes of what she can only describe as temper tantrums; severe irritability and quickly changing moods are also present. These occurrences have gotten so severe that they have begun to disrupt his daily activities. Question Ico-delete Highlights What is the only pharmacologic agent approved by the US Food and Drug Administration (FDA) to treat irritability in children and adolescents with autism spectrum disorder? Answer Choices 1 Sertraline 2 Paroxetine 3 Atomoxetine 4 Clonidine 5 Risperidone

Risperidone Adolescents classified as having a diagnosis within the autism spectrum disorder (ASD) will have varying degrees of impairment in their social and behavioral function. Family education, behavioral and educational interventions, and counseling have a significant place in this treatment plan. Pharmacotherapy may be considered, but it should be used as adjunctive therapy to those mentioned above. In 2006 the US Food and Drug Administration (FDA) approved risperidone, an atypical antipsychotic, in the oral disintegrating tablet form, for the symptomatic treatment of irritability in both children and adolescents with autism spectrum disorder. This is considered the first FDA approved drug treatment for behaviors specifically associated with ASD, and it has been used off-label for many years for these symptoms. Other symptoms that risperidone could be considered to help treat in patients with ASD include aggression and deliberate self-injury. When beginning any pharmacotherapy for these patients, having a "start low and go slow" regimen is strongly recommended, with consistent follow-up visits for evaluation of alleviation of the symptoms. Such pharmacologic agents as selective serotonin reuptake inhibitor (SSRI), such as sertraline or fluoxetine, are generally introduced to help alleviate anxiety symptoms. The patient in the above scenario is currently not experiencing or expressing any excessive anxiety. Although paroxetine is a type of SSRI that is used to treat patients with depression, obsessive-compulsive disorder, anxiety disorder, post-traumatic stress disorder, or premenstrual dysphoric disorder, it is currently not a recommended first-line treatment option for symptomatic behavior in ASD patients. Patients who express symptoms consistent with hyperactivity and inattention should be treated with methylphenidate, atomoxetine, or clonidine; this is not consistent with the symptoms that were discussed in this patient.

A 28-year-old G2P2 woman has just delivered a term male infant via normal spontaneous vaginal delivery. There was spontaneous rupture of the membranes 2 hours prior to delivery. Meconium stained amniotic fluid and thick particulate meconium was noted. The infant is brought under the radiant warmer where he appears peripherally cyanotic. He is crying and moving vigorously, and his heart rate is 160 beats per minute. Question What is the most appropriate next step in the resuscitation of this infant? Answer Choices 1 Bulb suction of the oropharynx and nasopharynx 2 Positive pressure ventilation via bag-valve mask 3 Removal of meconium by bulb suction 4 Routine neonatal evaluation since infant is vigorous 5 Suction the trachea via an endotracheal tube

Routine neonatal evaluation since infant is vigorous Explanation The recommendations have changed with the publication of the 2005 AHA/AAP Neonatal Resuscitation Program guidelines. Although the presence of meconium in the amniotic fluid should alert the caregiver to the possibility of neonatal distress, a vigorous infant is a good sign and further intervention is not warranted. Routine suctioning of the nose and mouth as the head is delivered is no longer recommended because it does not reduce the risk of meconium aspiration syndrome. In the presence of meconium stained fluid, suctioning is recommended if the infant has a low heart rate (<100 bpm), depressed respirations, or poor muscle tone. In an infant with one or more of these signs of distress, suctioning should be performed. Prior to drying, the infant should have visible, residual meconium removed from the nose and mouth by bulb suction. Then, under direct visualization, the trachea should be suctioned with an appropriately sized endotracheal tube. The infant in this case scenario is clearly vigorous and therefore does not require suctioning. He should only receive routine care (standard positioning, warming, etc.).

A 28-year-old pregnant woman who is currently at 33 weeks gestation is being evaluated for contractions at labor and delivery in the hospital. Laboratory testing on vaginal secretions from this patient reveals the following: Fetal fibronectin Positive Dried secretions on glass slide Ferning present Nitrazine paper Color change of yellow to blue Question What do these tests indicate? Answer Choices 1 Severe chorioamnionitis is present. 2 Fetal lungs are mature. 3 Rupture of membranes has taken place. 4 Estimated date of delivery (EDD) is confirmed. 5 The patient has a mild preeclampsia.

Rupture of membranes has taken place Explanation While direct observation of fluid leaking from the cervix is the best support, the above 3 tests (positive fetal fibronectin, ferning present on microscope slide and increased pH on nitrazine paper) indicate presence of amniotic fluid and thus rupture of membranes. Fetal fibronectin is not present in normal vaginal secretions. Normal vaginal pH during pregnancy is 4.5 - 4.7.1 However, amniotic fluid is 7 - 7.5, so after rupture of membranes, an alkaline pH is noted that causes corresponding color changes in the nitrazine paper. The laboratory testing that would most aid in a diagnosis of severe chorioamnionitis includes Gram stain for organisms and a white blood cell count of the amniotic fluid. 1 Often, the diagnosis is confirmed after pathology examination of the placenta. Fetal lung maturity is best evaluated with a lecithin to sphingomyelin (L:S) ratio. Some laboratories use different values for a level corresponding to fetal lung maturity (>/= to 2.0 or >3.0). A very low L:S ratio indicates fetal lungs are immature. 1 Confirmation of estimated date of delivery (EDD) cannot be accomplished with laboratory testing. Very early in a pregnancy, human chorionic gonadotropin (hCG) levels rise in a predictive manner in normal singleton, intrauterine pregnancy. 1 However, the level of hCG can neither be used to predict nor confirm the EDD. The best methods of confirming an accurate EDD include calculation based upon known date of last menstrual period (in a woman with regular menstrual cycles) and ultrasound in the 1st trimester.2 Preeclampsia is a syndrome in pregnancy consisting of hypertension and proteinuria. The testing that indicates a diagnosis of preeclampsia includes a blood pressure measurement and evaluation of the urine for protein, preferably from a 24-hour urine collection.3 None of the above listed tests on vaginal secretions indicate preeclampsia; furthermore, they do not indicate severity level.

A 46-year-old woman presents with nausea, vomiting, crampy abdominal pain, and loud bowel sounds for the past several hours. She denies weight loss. She has had 1 normal bowel movement since the symptoms began, but this did not help her symptoms. She has a past surgical history of an abdominal hysterectomy 7 years prior. On physical exam, she is afebrile, with hyperactive and high-pitched bowel sounds localized to the left upper quadrant. She also has mild, diffuse abdominal tenderness. Question What is the most likely diagnosis? Answer Choices 1 Small bowel obstruction 2 Whipple's disease 3 Diverticulitis 4 Acute paralytic ileus 5 Irritable bowel syndrome

SBO The correct answer is small bowel obstruction, as it typically presents with localized high pitched bowel sounds with crampy abdominal pain, nausea, and vomiting. The bowel sounds will diminish if complete obstruction occurs. Acute paralytic ileus typically presents with very diminished or no bowel sounds. Patients with diverticulitis often have more severe, localized abdominal tenderness on exam and typically present with fever and possibly peritoneal signs. Whipple's disease is an infectious disorder characterized by fever, lymphadenopathy, arthralgias, weight loss, and chronic diarrhea. Irritable bowel syndrome is not typically associated with nausea and vomiting, and the abdominal pain is relieved with defecation.

A 17 year-old male is training for an Ironman triathlon and notes excessive coughing, chest tightness and wheezing when running. Which of the following is the most appropriate treatments for this patient? A Albuterol B Cromolyn C Fluticasone D Ipratropium bromide E Salmeterol

Salmeterol The mainstay of treatment for exercise-induced asthma are beta-2 agonists. Due to the duration of physical activity this patient should be treated with a long-acting beta-2 agonist, salmeterol (E) instead of albuterol (A). If his symptoms are persistent then addition of a daily inhaled corticosteroid (C) is required, and daily ICS should be added if his use of salmeterol is frequent to avoid asthma-related death associated with the lone use of long-acting beta-agonists. Cromolyn (B) is an additional treatment options for exercise-induced bronchospasm, but is less commonly used compared to beta-2 agonists.

A 63-year-old woman presents to you with a 5-year history of stage-3 chronic kidney disease. She states that she has not been very good about following her provider's orders, and wants to know what things she can do to help her condition. Question What is the appropriate dietary management for this patient? Answer Choices 1 Salt, water, and protein restriction, potassium supplementation, and magnesium restriction 2 Salt, water, and protein restriction, with phosphorus, potassium, and magnesium restriction 3 Salt and water restriction, with magnesium supplementation, and potassium restriction 4 Salt, water, and protein restriction, with phosphorus and magnesium supplementation 5 Salt, water, and protein restriction, with magnesium and phosphorus restriction

Salt, water, and protein restriction, with phosphorus, potassium, and magnesium restriction Explanation The correct answer is restriction of salt, water, protein, phosphorus, and magnesium, as well as potassium. Some studies have shown that protein restriction will slow the progression to end-stage renal disease. Overload of sodium and water can lead to congestive heart failure and edema. Phosphorus and magnesium restriction is needed, as hyperphosphatemia and hypermagnesemia can be seen in chronic renal failure; this is due to decreased excretion of phosphate and magnesium. The other answers are not correct, as potassium supplementation could cause a hyperkalemic state: it should be avoided in chronic renal failure unless otherwise indicated. Phosphorus and magnesium should be restricted as indicated above.

A 51-year-old woman presents after discovering a new breast mass on her left side. It has not been tender, and she discovered it 2 weeks ago. Her last exam and mammogram were normal 2 years ago. Her past medical history is significant only for being perimenopausal. She takes no medications. Your physical exam confirms a firm, non-tender, fixed 1.5 cm mass in the left breast. Right breast exam is normal. Question What is the next most appropriate step for the management of this breast mass? Answer Choices 1 Refer for lumpectomy 2 Schedule a bilateral diagnostic mammogram 3 Order BRCA test 4 Reassure patient and re-examine in 6 months 5 Order follicle stimulating hormone (FSH)

Schedule a bilateral diagnostic mammogram The next most appropriate step in the management of this patient is to schedule a bilateral diagnostic mammogram because the mass has characteristics suspicious for malignant breast disease (such as being unilateral, non-tender, and fixed). Especially in patients over 40 years of age, the workup of a breast mass should include mammography. A mammogram can further characterize the breast mass and potentially detect additional smaller lesions and lymph node involvement that were not detected by physical exam. Ultrasonography may add additional diagnostic information in the evaluation of this mass. The imaging results can guide the next best treatment approach, such as fine-needle aspiration if the mass appears to be a simple cyst, or open biopsy with node sampling if the mass appears to be cancerous. Eventually, pathology on the tissue will be needed for definitive diagnosis. At this point, a referral for lumpectomy could have injurious effects regardless of whether the breast mass is benign in nature or if it is malignant and invasive. If the mass turns out to be a simple breast cyst, the patient would have been subjected to an invasive procedure when watchful waiting or needle-drainage may have sufficed. If the mass turns out to be invasive with lymph node involvement, the lumpectomy alone would have missed the opportunity to surgically sample lymph nodes, leading to an incorrect breast cancer staging or an additional surgery to remove additional cancer. BRCA testing is a serum test (for BRCA1 and BRCA2) to detect chromosomal mutations (on chromosomes 17 and 13) that are associated with an increased risk of breast and ovarian cancer. The results of a BRCA test will not influence the need to evaluate the symptomatic patient. Only about 5%-10% of breast cancer cases have a known genetic mutation. If the patient tests negative for BRCA, she still has a chance that her breast mass is malignant; conversely, a patient who tests positive for the BRCA mutation(s) may develop benign breast masses. Reassurance of the patient may be helpful, but because of the suspicious characteristics of the mass and the patient's age, delaying evaluation could be deleterious to her outcome. Further workup is warranted immediately. Checking the patient's FSH level might yield some information as to her ovarian status. The FSH rises as menopausal ovarian production of estrogen and progestin declines. However, the level does not directly influence management of the breast mass.

A patient presents with bilateral spastic paresis of his legs. His gait is noticeably stiff and his legs are advanced slowly. He uses short steps and crosses his legs in front of the other as he advances. What type of gait does he have? Answer Choices 1 Scissors gait 2 Steppage gait 3 Sensory ataxia gait 4 Cerebellar ataxia gait 5 Myopathic gait

Scissors Gait Scissors gait is apparent when each leg is advanced slowly and the thighs tend to cross each other. The gait is stiff and the steps are short. This is associated with bilateral spastic paresis of the legs. Steppage gait is associated with a foot drop, often secondary to lower neuron disease. The patient drags his foot, or lifts the knee high and slaps the foot against the ground. This gait may be unilateral or bilateral. Sensory ataxia gait is unsteady and wide-based. The feet are thrown forward, out, and then down. There is a distinct 2-slap sound, secondary to the heel striking first, then the forefoot. This is associated with polyneuropathy or posterior column damage. The gait characterized by cerebellar ataxia is associated with a disease of the cerebellum or affiliated tracts. The gait is staggering, unsteady, and wide-based. The patient has difficulty with turns, and cannot stand steady when feet are together and eyes closed (Romberg's sign). A myopathic gait is where there is weakness on one side. This displays a a drop in the pelvic area on the contralateral side of the pelvis while walking (Trendelenburg sign). When there is a bilateral weakness, it will display as a dropping of the pelvic girdle on both sides of the body while walking that results in a waddle. This is seen in patients whom are afflicted with myopathies, such as muscular dystrophy.

A 50-year-old male is seen with a routine check-up. He is concerned about the possibility of developing diabetes mellitus. He has a negative family history of diabetes. He has no signs or symptoms of diabetes and he is not overweight. Without any risk factors for diabetes, what is the recommended screening protocol for this patient according to the American Diabetes Association (ADA)? A screen all men over 25 years of age every five years B screen all men over 35 years of age every two years C screen everyone over 45 years of age every three years D no screening is necessary without risk factors E no screening is necessary without a family history of diabetes

Screen everyone over 45 yo every 3 years The correct choice is C, screen everyone older than 45 years of age every three years. In addition, the ADA recommends screening for younger people if they are overweight and have at least one additional risk factor, such as positive family history, hypertension, and/or vascular disease. The other choices are not recommended by the ADA for screening the general population for diabetes mellitus.

An 88-year-old woman informs you that it has been over 10 years since her last colonoscopy (which was negative), and it is time for another. She reminds you that she has always kept up with all her health screening, and she wants you, as her primary care physician, to refer her to "a good gastroenterologist to make sure [she does not] have any colon cancer". You review her history, and you note that she has been in generally good health. She has had no weight loss, no change in her bowel habits, and no dark stool or rectal bleeding. There is no family history of colon cancer. She is eating well and continues to remain active, although slowed down a bit due to arthritis. She takes acetaminophen, an occasional ibuprofen, a multivitamin, and olmesartan/hydrochlorothiazide for her high blood pressure. Question Based on the most recent recommendations from the US Preventive Services Task Force, what should you tell your patient? Answer Choices 1 Fecal occult testing is adequate for colon cancer screening given her negative history. 2 CT colonography would be preferable due to the risks of colonoscopy at her age. 3 Colonoscopy should be done, but she should be aware of the increased risks of the procedure in patients over age 70. 4 Screening for colon cancer in asymptomatic patients over age 85 is not recommended because the risks outweigh the benefits. 5 Flexible sigmoidoscopy with fecal DNA testing would be the best screening method for her.

Screening for colon cancer in asymptomatic patients over age 85 is not recommended because the risks outweigh the benefits. The most recent guidelines from the US Preventive Services Task Force (USPSTF) recommend against screening for colon cancer in asymptomatic persons over 85 who have had adequate screening in the past. The Task Force states that screening should increase the number of life years gained with the least amount of risk. It is felt that for persons over 85, death rates would be higher due to the likely presence of other conditions, reducing any potential benefit screening would have on mortality. In these cases, the risks of screening would outweigh the benefits. It is estimated that 18,800 lives per year could be saved if the goals for colorectal cancer screening were met. Colorectal cancer is the 3rd most common cancer in the United States. Evidence shows that persons aged 50 - 75 gain the greatest benefit from screening for colon polyps that may grow into cancer. These guidelines refer to persons who are asymptomatic, and they do not apply to persons with inflammatory bowel disease or known inherited diseases, such as familial adenomatous polyposis. The same recommendations apply to all ethnic groups (for example, African Americans, even though they appear to have an increased mortality from colorectal cancer, and 1st-degree relatives of persons with colon cancer or adenomas. However, it may be prudent to start screening at a younger age if adenomas or cancer was found in a younger 1st-degree relative). Additionally, when cancer or large adenomas are found, persons enter a surveillance program and screening recommendations would not be applicable. The 3 tests that are recommended for screening are colonoscopy, flexible sigmoidoscopy, and high sensitivity fecal occult blood testing. Evidence shows that any of these methods is effective in reducing mortality, but the benefits of sigmoidoscopy and fecal testing are mostly in reducing the need for colonoscopy if results are negative. Positive results would still require confirmation. Significant complications occur in about 3.4 per 10,000 sigmoidoscopies. There are not yet adequate figures to quantify harms associated with fecal testing, but the USPSTF estimates that any risk is probably no greater than small. The risk would most likely be due to colonoscopy done for a false positive test. Potential harm from colonoscopy is significant; death can be associated with the procedure, and serious adverse effects, including major bleeding, cardiovascular events, severe pain, and perforation, are possible. These occur at a rate of about 25 per 10,000. Perforation alone occurs in 3.8 per 10,000 tests done. While there is as yet no direct evidence as to which of the recommendations using these 3 screening methods (see table - Screening for Colorectal Cancer) is most effective, USPSTF states that any of them are an effective strategy as compared to no screening. Screening methods that USPSTF found insufficient evidence to support include CT colonography and fecal DNA testing. Possible benefits of CT colonography include a fewer adverse effects, and if a colonography screening test became available that did not require bowel preparation, then screening could be more acceptable and more common. CT colonography may also benefit persons who refuse colonoscopy. However, risk vs. benefit for screening could be underestimated since the studies included persons for screening and surveillance. Also, the CAT scan would include visualization of other areas in the abdomen. Finding abnormalities incidentally may be useful, but there may also be a risk for unnecessary work ups. CT colonography may also not be as good in picking up smaller adenomas. The rate of false positives (and potential for unneeded colonoscopy) is still unclear. Risks from radiation exposure may also be significant in an asymptomatic population. According to available studies, a potential benefit for DNA stool testing may include increased specificity. However, the cost of testing would probably be high, and information and research are still very limited. Colorectal screening for regularly followed asymptomatic persons from 76 to 85 is not recommended routinely, as researchers found little added benefit; however, USPSTF does add that there may be reasons to screen particular persons in this age group.

A 64-year-old man with chronic renal failure presents for a follow up. His physical examination is unremarkable. His pulse, BP, and temperature are as follows: 80/min, 130/90 mm Hg, and 98.9 F. The patient's lab results are as follows: Total serum Ca++: 7.5 mg/dl Serum ionized Ca++: 3.5 mg/dl (reference range- 4.5-5.3 mg/dl) Serum phosphate: 5.2 mg/dl Serum PTH (parathyroid hormone): 13 pmol/L (reference range- 1.0-6.8 pmol/L) Serum urea: 40 mg/dl Serum creatinine: 2.2 mg/dl Question What is the best explanation for the patient's results? Answer Choices 1 Tertiary hyperparathyroidism 2 Primary hyperparathyroidism 3 Primary hypoparathyroidism 4 Secondary hyperparathyroidism 5 Secondary hypoparathyroidism

Secondary Hyperparathyroidism Explanation The candidate in this case is a patient with chronic renal failure who presents for follow up. His serum urea and creatinine are elevated. The patient's serum calcium is decreased. The decreased serum total and ionized calcium in the presence of increased PTH is consistent with secondary hyperparathyroidism. The presence of chronic renal failure suggests that the parathyroid hyperfunctioning is arising in the setting of impaired renal function. In chronic renal failure, hyperphosphatemia and decreased renal production of dihydroxy vitamin D3 leads to hypocalcemia. This stimulates the parathyroid glands to increase PTH secretion in order to increase the level of serum calcium by increasing intestinal absorption and bone resorption. In spite of the increased level of PTH, both the total and ionized serum calcium are decreased. These findings are consistent with the diagnosis of secondary hyperparathyroidism in the setting of chronic renal failure. Primary hyperparathyroidism is excluded by decreased serum calcium levels. Tertiary hyperparathyroidism is marked by hypercalcemia and elevated PTH levels, which are not present in this patient. Hypoparathyroidism is excluded by the increased levels of PTH values.

You have determined that a multiple sclerosis patient with relapsing-remitting disease has not returned to her baseline functional status with her most recent exacerbations, despite her returning to baseline with exacerbations for many years. Her exam also reveals residual neurologic dysfunction. Which of the following multiple sclerosis clinical types would you classify this patient within? A Clinically isolated syndrome B Primary progressive C Progressive/relapsing D Relapsing/remitting E Secondary progressive

Secondary progressive This patient would be classified within the secondary progressive clinical type. Secondary progressive MS follows a relapsing/remitting course initially, during which the patient returns to baseline neurologically and functionally between exacerbations, then evolves into a course of the patient having deterioration in function and neurologic status, which is not associated with acute exacerbations. The patient is also noted to not return to baseline following acute exacerbations. Primary progressive MS is associated with a decline in functional and neurologic status from the onset of disease, and disability is common. Progressive/relapsing MS is associated with progressive neurologic and functional decline from the onset of disease, but with associated exacerbations during the clinical course as well. The designation of clinically isolated syndrome indicates that a patient has had a single clinical event suggestive of MS but does not meet MS diagnostic criteria, thus requiring a waiting period and additional studies.

A 56-year-old man presents with a 10-day history of epigastric pain and vomiting. He states that similar symptoms have appeared frequently throughout the past year and that he was diagnosed with a duodenal ulcer 8 months ago. He was given eradication therapy for a Helicobacter pylori infection and urea breath test confirmed successful eradication. With the discontinuation of protein pump inhibitors, his symptoms reappeared. He has a surgical history of an incomplete vagotomy at age 22 for the treatment of duodenal ulcer. He denies intake of salicylates or any other drugs that could damage duodenal mucosa or be a causative agent for recurrent ulcers. General physical findings are unremarkable, except for the presence of epigastric and periumbilical pain on palpation. Neither routine laboratory analyses nor an abdominal echo-sonogram reveal abnormalities. Endoscopy reveals the presence of an ulcer in the first portion of the duodenum. His BAO is 15.3 mEq/hr, and his BAO/MAO (basal/maximal acid output) ratio is 0.75. His serum gastrin level is 268 pg/mL (non-fasting normal is 200 pg/mL). What test should be used first to confirm the suspected diagnosis of Zollinger-Ellison syndrome? Answer Choices 1 Repeated gastrin determination 2 Secretin stimulation test 3 Calcium infusion test 4 Endoscopic echo-sonography 5 CT scan of pancreas

Secretin Stimulation test Hypergastrinemia is always present in patients with Zollinger-Ellison syndrome but may also be present in other diseases and conditions. In order to discriminate patients with Zollinger-Ellison syndrome from patients with hypergastrinemia caused by other factors, a stimulatory test with secretin is performed. The test, which is highly sensitive and specific, is based on the fact that secretin administration causes a striking elevation of serum gastrin level in patients with Zollinger-Ellison syndrome that cannot be noted in persons with hypergastrinemia caused by other factors. The repeated determination of serum gastrin level is of no diagnostic value, particularly if vagotomy (which influences serum gastrin level) was formerly performed. Calcium infusion test may also be utilized for the diagnosis of Zollinger-Ellison syndrome. However, this test is less sensitive and less specific than secretin test, and its performance is associated with more side effects and potential false negatives. Endoscopic echo-sonography may be of valuein detection of tumor site once diagnosis of Zollinger-Ellison syndrome is firmly established. Also, CT scan of the pancreas may be helpful in detecting tumor lesiona, but it is of no value in detection of hormonal potential of the tumor.

A 47-year-old man presents with numbness, tingling, and pain anterior and to the left side of his thigh. Pain is provoked even with light touch, and tingling continues for several minutes after the touch. Symptoms started a couple of months ago and worsen when he wears a belt and walks down slopes and stairs, as well as after prolonged standing. Symptoms are relieved when he puts a pillow between his thighs. Patient also assumes hunched posture while standing to avoid unpleasant sensory symptoms. His BMI is 40, and he has a protruding, pendulous abdomen. Question What else do you expect to find on examination? Answer Choices 1 Sensory loss in anterolateral thigh down to left upper knee 2Sensory loss on the dorsum of the foot 3 Foot drop 4 Loss of foot inversion in plantar flexion 5 Loss of m. quadriceps reflex

Sensory loss in anterolateral thigh down to left upper knee Your patient most probably has entrapment syndrome caused by the compression of lateral femoral cutaneous nerve and suffers from the condition called meralgia paresthetica. Pendulous abdomen and tight belt compress the inguinal ligament downward and onto the nerve. Lateral femoral cutaneous nerve is pure sensory nerve conducting sensory signals from the region of anterolateral thigh down to the upper knee. On examination, you can find decreased sensitivity to pinprick together with a hyperpathic reaction to touch. This angulation of the nerve is further exaggerated with extension of the thigh and relaxed with flexion. Foot drop is a sign of common peroneal nerve entrapment. Loss of foot inversion in plantar flexion is a sign of L5 radiculopathy, in which posterior tibialis function is disturbed. The quadriceps, patellar reflex, or knee-jerk is a deep tendon reflex that can be lost in the case of the lesion of L2, L3, and L4 root. This myotatic reflex tests motor function. Your patient has no signs and symptoms of radiculopathy or motor function disturbance.

A 54 year-old female returns for pathology results after being diagnosed with ovarian cancer. Which of the following is the most likely cause of ovarian epithelial malignancy? A Serous tumors B Endometrioid tumors C Mucinous tumors D Clear cell tumors E Transitional cell tumors

Serous Tumors A The most common of the ovarian epithelial malignancies are serous tumors (50%); tumors of mucinous (25%) (C), endometrioid (15%) (B), clear cell (5%) (D), and transitional cell (E) histology or Brenner tumors (1%) represent smaller proportions of epithelial ovarian tumors. Over half of all epithelial ovarian cancers have serous histology. The second most common histologic type of epithelial ovarian cancers are endometrioid adenocarcinomas.

The patient is a 19-year-old woman who presents due to lower abdominal pain, chills, and vaginal discharge. She admits to being sexually active and has a history of irregular menstruation. Her pelvic examination reveals adnexal and cervical motion tenderness, as well as purulent discharge seen at cervical os. Question What other factor in the patient's case would influence you to admit her for inpatient intravenous antibiotic therapy as opposed to outpatient antibiotic therapy? Answer Choices 1 Oral temperature of 100.8 ° F 2 Lower abdominal tenderness on exam 3 Serum β-hCG level of 28,000 mIU/mL 4 Evidence of cervical infection with N. gonorrhoeae 5 Elevated C-reactive protein

Serum B-hCG level of 28,000 mIU/mL Explanation Given the patient's history and physical examination, the most likely diagnosis is pelvic inflammatory disease (PID). Most cases of PID are treated on an outpatient basis, but there are certain criteria that warrant inpatient intravenous antibiotics. One of the indications for hospitalization is pregnancy, and a serum β-hCG of 28,000 mIU/mL is laboratory evidence that this patient is at least 5 weeks pregnant. In addition, the chance for ectopic pregnancy as a cause of the symptoms and elevated serum β-hCG still exists. Ectopic pregnancy is treated surgically, so this would be an added reason to treat the patient as an inpatient. Oral temperature of 100.8°F is not abnormal for a patient with PID. In fact, one of the additional criteria used to diagnose PID is an oral temperature over 101°F. If a patient has a high fever associated with the disease, then the health care provider may opt to treat as an inpatient. High fever may be a sign that there is a pelvic abscess or another undiagnosed surgical emergency, such as appendicitis. Lower abdominal tenderness is a symptom of PID and not a criterion for admission. N. gonorrhoeae can be found in the cervical culture of a patient with PID, and a C-reactive protein can be elevated. Neither is necessary for diagnosis, but both are common in patients with PID and are not reasons for inpatient treatment in themselves.

A 40-year-old patient presents to your walk-in clinic with symptoms of hyperhydrosis, oily skin, daytime sleepiness, and snoring. Upon exam, you note large fleshy heel pads and hands with sweaty palms. The patient also has coarse facial features. When asked, the patient isn't aware of any major changes in her face or body. She has not seen another health care provider in many years and has not kept up with any health care maintenance schedule. The patient lives with her ill mother and is not currently employed. Which of the following screening tests would best aid in the diagnosis of this patient? A CT of the chest and abdomen B Thyroid scan C 24-hour urine for catecholamines D Serum calcitonin level E Serum IGF-I level

Serum IGF I level The correct choice is E, serum IGF-I level. Age and gender matched levels of IGF-I are elevated in patients with acromegaly. IGF-I is the mediator of most of the effects of GH on the body, and lead to the proliferation of bone, cartilage, and soft tissue. Although GH levels may be elevated in patients with acromegaly, they are secreted in a pulsatile fashion and are not consistently elevated. Serum GH levels are not the best screening test for acromegaly. Choices A, a CT of the chest and abdomen, and B, a thyroid scan, are expensive imaging studies that are not usually used as screening tests. They also have no role in the routine workup of patients with suspected acromegaly. Choice C, 24-hour urine for catecholamines, is a test that can be used in the work up of patients with suspected pheochromocytoma. Choice D, serum calcitonin levels, are associated with medullary thyroid cancer and other thyroid disease.

A 40-year-old woman presents for check-up. She is married with 2 sons, and she experiences occasional constipation; she attributes it to her diet. She mentions having mild anxiety. She does not smoke; she consumes alcohol only recreationally, and she has a body weight of 85 kg. She is well nourished and communicates well. Lung, cardiac, abdominal, and neurological examinations are unremarkable. Her thyroid is normal in size and consistency. Her menses are normal, and her drug history is negative, except for "hormone pills" for contraception. Her family history is remarkable for a mother with hypothyroidism and osteoporosis. Question What screening option is appropriate for this woman at present? Answer Choices 1 Follow-up 2 Free T3 assay 3 Total T4 assay 4 Serum TSH assay 5 Total T4 and T3 resin uptake assays

Serum TSH assay Explanation The American Thyroid Association recommends that adults, particularly women, high-risk individuals, and those with symptoms and signs attributable to thyroid dysfunction, should be screened for thyroid dysfunction by measuring serum TSH concentration beginning at the age of 35 years; the screening should be repeated every 5 years (Ladenson et al., 2001). The absence of classic manifestations of hypo- or hyperthyroidism or lack of risk factors would not defer a screening TSH assay.

A sexually active, 17-year-old patient presents alone to your office and requests contraceptive counseling. Because she is under the age of consent, what is an ethical concern of which you must be aware in seeing her? Answer Choices 1 You are obligated to refuse to see her. 2 You may only see her with parental consent. 3 You do not need parental consent if she is accompanied by an adult. 4 She can give consent for contraceptive counseling and evaluation of STDs. 5 You must counsel abstinence as contraception at her age.

She can give consent for contraceptive counseling and evaluation of STDs. Explanation This case highlights several ethical issues. Autonomy indicates that an individual may make his/her own health care decisions based on their own values. It is usually limited by a determination of whether the individual is competent and, in pediatrics, at least in part determined by the age of the individual. Competence is an individual's ability to understand the possible consequences of his/her decision and the available alternatives. Patients are generally considered to have a right to confidentiality, i.e., trust that information about them will not be disclosed. Depending on the age of the patient, there may be a conflict of interest between the physician's duty to the patient and the physician's duty to the parents. States either have laws allowing individuals who would otherwise be considered minors to obtain contraceptive services or have no law precluding the provision of such services.However, most states have placed the interests of the adolescents above those of the parents in issues relating to the management of sexually transmitted diseases and other issues of sexuality. Physicians have a duty to tell the truth to their patients. This duty can create quite a conflict when other family members, especially parents, request information about another family member that the physician is obligated to keep confidential.

A 45-year-old woman presents for initial history and physical at her new primary care provider's office. She is currently asymptomatic, but based on her sexual history with multiple partners, you decide to obtain screening hepatitis lab tests in addition to other screening tests. The following results are obtained: Hep A Ab- negative HBcAb-positive HBeAg-negative HBsAg-positive HBsAb-positive Hep C Ab-positive Hep D Ab-negative Question What is the most likely diagnosis based on these results? Answer Choices 1 She has not been infected with or been vaccinated against Hepatitis B virus but has been exposed to Hepatitis C 2 She has been vaccinated against Hepatitis B in the past but is not currently infected with Hepatitis B nor exposed to Hepatitis C 3 She has had Hepatitis B infection in the past, is currently infected with Hepatitis B, and has been exposed to Hepatitis C 4 She has had Hepatitis B infection in the past but is not currently infected and has been exposed to Hepatitis C 5 She has been vaccinated against Hepatitis B but has not been infected with it and has been exposed to Hepatitis C

She has had Hepatitis B infection in the past, is currently infected with Hepatitis B, and has been exposed to Hepatitis C Explanation: HBcAb, HBsAg, and HBsAb become positive with Hepatitis B virus infection, but over time HBsAg will clear from the serum only if the patient is not chronically infected. These results demonstrate chronic infection, as all three are positive. She also shows exposure to Hepatitis C, as the surface antibody is positive. The remaining answers are incorrect because vaccination alone will cause only the HBsAb to become positive. All Hepatitis B results would be negative if the patient had not had Hepatitis B virus infection or been vaccinated for Hepatitis B virus. Also, only HBsAb would be positive if the patient was never infected but had received the vaccine.

A 35-year-old Caucasian female patient who has been diagnosed with cervical cancer receives chemotherapy as an outpatient. You send her home with a prescription for metoclopramide. What do you need to tell her about the drug? Answer Choices 1 Nausea and vomiting are common side effects 2 She may experience dry mouth 3 She may experience drowsiness 4 The drug should be taken after meals 5 If she misses a dose to just skip it and go back to schedule with the next dose

She may experience drowsiness Explanation Drowsiness is a known side effect of metoclopramide, therefore you need to tell the patient not to operate equipment before she knows her response to the drug. Nausea and vomiting are the indications for prescribing the drug. The effectiveness of the drug is not affected by food. Since nausea and vomiting often occur with food intake, you should advise the patient to take the medication before meals. Doses of metoclopramide should not be skipped. If forgotten it should be taken as soon as possible, except if it is already time for the next dose. As metoclopramide is an anti-emetic which acts directly on the gastrointestinal tract there is no known side effect of dry mouth.

A 20-year-old woman presents with a 1-day history of passing bloody diarrhea. On examination, her temperature is 39 degrees centigrade. A stool examination reveals polymorphonuclear leukocytes, but no trophozoites. Question What is the causative organism? Answer Choices 1 Entamoeba histolytica 2 Enterotoxigenic Escherichia coli 3 Shigella dysenteriae 4 Bacillus cereus 5 Norwalk virus

Shigella dysenteriae In cases of food poisoning caused by Shigella dysenteriae, patients present with bloody diarrhea; symptoms usually start 16 hours after ingesting contaminated foods. They may also be febrile. Common sources of infection include potato salad and raw vegetable salads. A stool examination reveals polymorphonuclear leukocytes. The diagnosis is confirmed by culturing the organism from stool. Management includes rehydration and, in severe cases, antibiotics like trimethoprim-sulfamethoxazole or ampicillin. In cases of emetic Bacillus cereus food poisoning, patients develop vomiting 1 to 6 hours after ingesting contaminated food (e.g., reheated fried rice). In the diarrheal form, they develop watery diarrhea 8 to 16 hours after ingestion. There are no fecal leukocytesor trophozoites. In adults with Norwalk virus food poisoning, the watery diarrhea is more prominent than vomiting. In children, the reverse is true. The incubation period is 1 to 3 days. There are no fecal leukocytes or trophozoites. Entamoeba histolytica colitis develops 2 to 6 weeks after ingesting contaminated water or food. Patients usually present with bloody and mucoid diarrhea. Stool examination reveals hematophagous trophozoites and cysts. Patients with enterotoxigenic Escherichia colifood poisoning present with watery diarrhea 12 to 72 hours after ingesting contaminated water or raw fruit salads. Examination of the stool reveals no fecal leukocytes or trophozoites.

What statement is true concerning balloon tamponade of esophageal varices? Answer Choices 1 An excellent measure for long term control of esophageal varices 2 Need not be preceded by upper endoscopy in a patient with a prior history of bleeding 3 Should be preceded by endotracheal intubation to protect the airway from aspiration 4 Position confirmation can be made by auscultation of the abdomen 5 Is associated with minimal morbidity and mortality

Should be preceded by endotracheal intubation to protect the airway from aspiration The use of balloon tamponade, most commonly with the Sengstaken-Blakemore tube, for esophageal varices is temporarily effective in controlling bleeding in 80-90% of patients. However, rebleeding will occur in 60% of patients and therefore is not effective for long term control. This is not a benign procedure, there is a significant risk of aspiration and esophageal or gastric perforation. Therefore, elective endotracheal intubation prior to placement of the tube is preferential. Careful placement of the tube with meticulous monitoring of balloon pressures is essential to minimize the risks of perforation. Radiologic confirmation of the tube position should always precede inflation of the balloons and frequent radiologic studies should be performed to assure maintenance of this position. In a patient with a history of esophageal varices with a new upper gastrointestinal bleed, 25% of the time it will be from a source other than the varices. Therefore, if the patient is not having exsanguinating hemorrhage, it is useful to perform upper endoscopy to confirm the source of bleeding and possibly treat the varices with sclerotherapy, which can control the bleeding in 90-95% of cases.

Over the past year, a 43-year-old man has been experiencing episodes of palpitations, anxiety, and sweating. Multiple endocrine neoplasia (MEN) syndrome runs in his family. On physical examination, his blood pressure is elevated. His body habitus is normal. His mucosal membranes are unremarkable. A 24-hour collection of his urine tests positive for vanillylmandelic acid. He is diagnosed with a pheochromocytoma. What is the most likely diagnosis? Answer Choices 1 Wermer's syndrome 2 Sipple's syndrome 3 Sturge-Weber syndrome 4 Mucosal neuroma syndrome 5 Carcinoid syndrome

Sipple's Syndrome Explanation Sipple's syndrome (multiple endocrine neoplasias, type IIA) consists of pheochromocytoma, hyperparathyroidism, and medullary carcinoma of the thyroid. Multiple endocrine neoplasias refer to a group of familial syndromes in which various pathologies of the endocrine glands can be found. Multiple endocrine neoplasias, type I (Wermer's syndrome): Hyperparathyroidism Pancreatic islet cell tumors Pituitary tumors Multiple endocrine neoplasias, type IIA (Sipple's syndrome): Hyperparathyroidism Pheochromocytoma Medullary carcinoma of the thyroid Multiple endocrine neoplasias, type IIB (mucosal neuroma syndrome): Pheochromocytoma Medullary carcinoma of the thyroid Mucosal neuromas Marfanoid body habitus Sturge-Weber syndrome usually includes a facial port wine stain and seizures. There can also be other neurologic findings, such as weakness. Carcinoid syndrome presents with flushing, respiratory wheezing, and diarrhea. Carcinoid syndrome is due to the activity of tumors from neuroendocrine cells.

A 46-year-old woman presents with nausea, vomiting, crampy abdominal pain, and loud bowel sounds for the past several hours. She denies weight loss. She has had 1 normal bowel movement since the symptoms began, but this did not help her symptoms. She has a past surgical history of an abdominal hysterectomy 7 years prior. On physical exam, she is afebrile, with hyperactive and high-pitched bowel sounds localized to the left upper quadrant. She also has mild, diffuse abdominal tenderness. Question What is the most likely diagnosis? Answer Choices 1 Small bowel obstruction 2 Whipple's disease 3 Diverticulitis 4 Acute paralytic ileus 5 Irritable bowel syndrome

Small bowel obstruction The correct answer is small bowel obstruction, as it typically presents with localized high pitched bowel sounds with crampy abdominal pain, nausea, and vomiting. The bowel sounds will diminish if complete obstruction occurs. Acute paralytic ileus typically presents with very diminished or no bowel sounds. Patients with diverticulitis often have more severe, localized abdominal tenderness on exam and typically present with fever and possibly peritoneal signs. Whipple's disease is an infectious disorder characterized by fever, lymphadenopathy, arthralgias, weight loss, and chronic diarrhea. Irritable bowel syndrome is not typically associated with nausea and vomiting, and the abdominal pain is relieved with defecation.

A 65-year-old man presents with a 25-pound unexplained weight loss. He also has noticed a change in his usual cough. He sees his family doctor. Upon questioning for the medical history, the doctor discovers that he has a 60-pack/year history of smoking as well as dyspnea. On physical examination, he appears to have Cushing's syndrome. X-rays show a central lesion with no cavitation. A hilar mass is seen on chest X-ray. What is the most likely diagnosis? Answer Choices 1 Small cell carcinoma 2 Adenocarcinoma 3 Metastatic Wilms tumor 4 Large cell carcinoma 5 Squamous cell carcinoma

Small cell carcinoma Small cell carcinoma is most frequently a central lesion not associated with cavitation. A hilar mass is associated with small cell carcinoma. Small cell carcinoma is associated with paraneoplastic syndromes. Cushing's syndrome secondary to ectopic ACTHproduction can sometimes be seen with small cell carcinoma, as can syndrome of inappropriate antidiuretic hormone production (SIADH). An adenocarcinoma is usually seen peripherally on X-ray. An adenocarcinoma is characterized pathologically by mucus production. On the rare occasion when lung cancer occurs in somebody who has never smoked, it is most often classified as an adenocarcinoma. Adenocarcinomas may develop in areas of the lung where there is scarring or fibrosis. For example, a cause of the scarring can be a granulomatous infection, such as tuberculosis. For this reason, adenocarcinoma is sometimes referred to as "scar carcinoma". Wilms tumor is a pediatric malignancy of the kidney. It can metastasize to the lungs. Large cell carcinoma is usually seen peripherally on X-ray. Large cell carcinoma shows poorly differentiated cells. It does not show squamous features or glandular features. Large cell carcinoma does not react to mucicarmine staining. Large cell carcinoma is also called undifferentiated carcinoma or anaplastic carcinoma. The cell of origin is unknown. Squamous cell carcinoma is centrally located. Cavitation can also sometimes be seen with X-ray. Keratin formation, keratin pearl formation, and intercellular bridges can be seen with squamous cell carcinoma.

A 63-year-old woman presents with shortness of breath, cough, and proximal muscle weakness of 1-month duration. On clinical exam, she is noted to have a blood pressure of 156/102 mm Hg, facial flushing, mild hirsutism, truncal obesity, marked proximal muscle weakness of both the upper and lower extremity, and hyperpigmentation over the palms and back of the neck. Laboratory exam reveals hypercortisolism and increased ACTH. Which of the following would be the most likely primary diagnosis in this patient? A lymphoma B ovarian cancer C renal cell carcinoma D small cell lung carcinoma

Small cell lung cancer Tumor cells may secrete hormones that have the same biologic actions as the normal hormone. This patient's symptoms are consistent with adrenocorticoid hyperfunction. The most common cause of ectopic ACTH syndrome is small cell lung carcinoma. This should be suspected in any patient with risk factors for lung cancer.

A 65-year-old man presents with flank pain, blood in his urine, and an unexplained weight loss. His past medical history is significant for numerous infections, kidney stones, cigarette use, and alcohol use. On physical exam, there is a palpable abdominal mass, as well as a low-grade fever. Diagnostic tests determine that he has cancer. Question What puts this patient at risk for the development of his particular type of cancer? Answer Choices 1 Eschericia coli infection 2 Urolithiasis 3 Smoking 4 Interstitial nephritis 5 Schistosoma haematobium infection

Smoking Explanation This patient has renal cell carcinoma. Smoking is a risk factor for many neoplasms, including renal cell carcinoma. There is a classic triad associated with renal cell carcinoma: hematuria, flank pain, and a palpable abdominal mass; however, the classic triad is not seen in most patients. Other presenting symptoms include a fever and weight loss. Cells from the proximal convoluted tubule are the most common cells that renal carcinoma arises from. There is an increased incidence of renal cell carcinoma with von Hippel-Lindau disease. Escherichia coli infection, urolithiasis, and interstitial nephritis are not known risk factors. Schistosoma haematobium infection is associated with bladder tumors.

A 53-year-old man with a 40-pack/year smoking history presents with a 10-month history of intermittent cough with productive sputum. He admits to progressive exertional shortness of breath, which recently has limited his activity to climbing 1 flight of stairs or walking 3 city blocks. He denies diaphoresis, fever, chills, chest pain, palpitations, audible wheezing, pleurisy, peripheral edema, hemoptysis, abdominal pain, reflux, regurgitation, diarrhea, melena, or hematochezia. He also denies travel, sick contacts, and drug or alcohol use. His general survey reveals an overweight male with an odor of smoke and nicotine staining of his fingernails. His nails also demonstrate digital clubbing. His pulmonary exam reveals prolonged expiratory phase expiratory, barrel chest, poor diaphragmatic excursion, and wheezing to auscultation. Pulmonary function testing shows airflow obstruction with a reduction in FEV1 and FEV1/FVC ratio; an increase in total lung capacity, functional residual capacity, and residual volume were noted. Question What clinical intervention has been demonstrated to influence the natural history of this patient's illness? Answer Choices 1 Intravenous diuretics 2 Inhaled anticholinergic agents 3 Smoking cessation 4 Long-acting inhaled -agonists and glucocorticoids 5 Antibiotic prophylaxis

Smoking cessation This patient's diagnosis is most consistent with chronic obstructive pulmonary disease. Only 3 interventions: smoking cessation, oxygen therapy in chronically hypoxemic patients, and lung volume reduction surgery in selected patients with emphysema have been demonstrated to influence the natural history of patients with COPD. All other current therapies are directed at improving symptoms and decreasing the frequency and severity of exacerbations. Diuretics are not indicated in the management of COPD. Long-acting inhaled -agonists (e.g., salmeterol) and anticholinergic agents, such as tiotropium or ipratropium bromide, have been shown to improve symptoms and reduce exacerbations, but have failed to demonstrate that either influences the rate of decline in FEV1. Despite the frequent implication of bacterial infection in COPD exacerbations, chronic suppressive, or "rotating" antibiotics, are not beneficial in patients with COPD.

The patient is a 35-year-old woman who presents as a new patient with urinary frequency, urgency, dysuria, and suprapubic discomfort for several months. Repeated urinalysis and clean catch urine cultures ordered by her primary care physician have been unremarkable. The urologist does not find any significant physical exam findings and decides to perform a cystoscopy under IV sedation. Findings include velvety red patches known as Hunner's ulcers, and a bladder biopsy is negative for cancer. Passive hydrodistention of the bladder is performed at the time of the cystoscopy and is found to provide the patient with minimal relief from her symptoms following the procedure. Question What medication would be an appropriate next step in this patient's treatment? Answer Choices 1 Ciprofloxacin (Cipro) 500mg BID x 7 days 2 Sodium Pentosanpolysulfate (Elmiron) 100mg TID 3 Bisacodyl (Dulcolax) 5mg once daily 4 Hydrocodone (Vicodin) 5/500 q 4-6 hours prn 5 Acetaminophen/Aspirin/Caffeine (Excedrin) 1-2 tablets daily

Sodium Pentosanpolysulfate (Elmiron) 100mg TID Explanation The scenario is describing a patient with interstitial cystitis (IC). Patients with IC have a 10:1 female to male ratio and are typically in the third decade of life. Symptoms usually include urinary frequency, nocturia, urgency, and bladder or pelvic pain. Physical examination is usually unremarkable and helpful at ruling out other causes of the patient's symptoms. The urinalysis and urine culture are usually unremarkable, which also rules out other differential diagnoses. Cystoscopy with hydrodistention under sedation is often used to diagnose IC by both the appearance of the bladder and the bladder capacity (not usually over 350cc). Hunner's ulcers seen during cystoscopy with hydrodistention are pathognomonic for interstitial cystitis, although they do not have to be present for a patient to have this diagnosis (only present in 5-10% of cases). The hydrodistention can also help to relieve symptoms, and can be an effective treatment for many patients with IC. However, if symptoms persist, then other treatment options are warranted. Altering diet and avoiding foods and beverages that are bladder irritants can be helpful in improving symptoms in patients with IC. Beyond these measures, there are various medications that can offer relief. Elmiron stands alone in its class of medications, but is similar to a class of medications called low molecular weight heparins. It prevents the irritation of the bladder wall that is the cause behind the patient's symptoms. This medication is prescribed 100mg TID and is a first-line treatment. It is the best choice of those listed as potential answers. Ciprofloxacin (Cipro) is an antibiotic commonly used to treat urinary tract infections (UTI). While UTI would have been high on the list of differential diagnoses for this patient, it was ruled out by the negative urinalysis and urine culture. Bisacodyl (Dulcolax) is a medication commonly used to treat constipation and would therefore not be an appropriate treatment for this patient. Hydrocodone (Vicodin) and acetaminophen/aspirin/caffeine (Excedrin) are both commonly used to treat pain. Hydrocodone is often prescribed to patients with IC, as chronic opioid use is not uncommon due to the occasional extreme nature of the pelvic pain. However, it would not be the next best treatment and is essentially masking symptoms and not treating the IC. Excedrin is a pain reliever, but it contains caffeine. Caffeine is a bladder irritant and should be avoided by patients with IC, as it can potentiate the symptoms.

A 13-year-old boy has a 3-year history of type I diabetes mellitus without significant problems; he presents for an acute visit. The patient states his morning glucose readings have risen from an average of 100 mg/dL to over 200 mg/dL over the past 2 weeks. He is currently taking 25 units of glargine (Lantus) at bedtime and 8 units of aspart (Novolog) before meals. The patient states that he has been compliant with his insulin and diet. His mother states that he recently has been having nightmares and night sweats, but he denies any other complaints. ROS and physical exam are otherwise unremarkable. What is the patient most likely experiencing? Answer Choices 1 A tonic seizure 2 Diabetic ketoacidosis 3 Somogyi effect 4 Dawn phenomenon 5 The onset of puberty

Somoygi Explanation The correct answer is that the patient is most likely experiencing the Somogyi effect. The Somogyi effect begins with an episode of nocturnal hypoglycemia; this triggers the body's counter-regulatory response, secreting glucagon, epinephrine, and cortisol plus growth hormone to raise the blood sugar, resulting in elevated morning levels or rebound hyperglycemia. The nocturnal hypoglycemia is due to hyperinsulinemia as a result of insulin therapy in conjunction with decreased evening caloric intake or extra calories utilized for activity, especially in the evening hours. The symptoms of nocturnal hypoglycemia (restlessness, profuse sweating, and nightmares) occur most often when the patient is asleep; it is possible that only a parent or partner may notice them. Patients also tend to awaken with a severe morning headache. Adrenergic symptoms of hypoglycemia, such as palpitations, tremor, anxiety, hunger, and nausea, are the result of the counter-regulatory hormones secreted at blood glucose levels < 70 mg/dL; they may be noticeable by the patient. Neuroglycopenia, with symptoms of weakness, confusion, and combativeness, occurs if the blood glucose falls below 45 - 50 mg/dL; if untreated, they may result in seizure, coma, or death. Checking blood sugars at 3:00 am or continuous glucose monitoring are useful to detect nocturnal hypoglycemia. This patient would benefit from correcting the nocturnal hyperinsulinemia with a bedtime snack, limiting evening exercise, decreasing the dinner dose of aspart, or decreasing the daily dose of glargine if his basal blood sugars, fasting, and pre-meal are low. The Dawn phenomenon also results in morning hyperglycemia, but this is due to the physiologic secretion of growth hormone in the early morning hours, which produces insulin resistance. In this case, there is insufficient insulin to overcome this inherent insulin resistance, so patients arise with elevated morning blood sugars. This can also be documented by checking 3:00 am blood sugars or continuous glucose monitoring, and it is corrected by adding insulin to the evening or nighttime regimen. The onset of puberty would result in accelerated secretion of growth hormone, resulting in morning hyperglycemia, but it would not explain the symptoms of nocturnal hypoglycemia. Tonic seizures occur most frequently in childhood and during sleep. Although his mother did not specifically note any tonic (sustained) or clonic (rapid contraction/relaxation) activity, other forms of seizures can present with night terrors. Likewise, untreated hypoglycemia can result in seizures. Although a seizure disorder would be included in the differential diagnosis, it would not explain the recent onset of morning hyperglycemia. The symptoms of diabetic ketoacidosis include nausea, vomiting, abdominal pain, polyuria, acetone-odor breath, and Kussmaul breathing.

A 40-year-old white male patient presents to your office to ask for advice regarding the inability of his wife to conceive. She is younger than he is by eight years and is believed by him to be in good health. Additionally, she has two children from a previous marriage, and both pregnancies were without complications. He was also married once before but his first wife did not conceive children. He has not yet undergone any fertility tests. The most important factor concerning male fertility is Answer Choices 1 Lung and liver dysfunctions 2 Spermatogenesis, varicocele, infections, drugs, and heat 3 Mucous and sperm interactions 4 Endometrial cavity and shape 5 The size of the genital tract 6 Personal and social history as it regards smoking, drugs and eating habits 7 Past infections of the male genitalia

Spermatogenesis, varicocele, infections, drugs, and heat Explanation Factors involved in fertility: The male factor---Spermatogenesis, infections (prostatitis, epididymis, mumps) varicocele, heat (decrease sperm count and motility) drugs (marijuana, tobacco, alcohol). The ovarian factor---Ovulation The cervical factors---Mucus and sperm interaction The uterine factor---Endometrial integrity and cavity size and shape The tubal factor---oviductal potency and anatomic relationships to the ovary The coital factor---Insemination

A 64-year-old, nonsmoking male presents to the clinic with complaints of headache and dizziness. Initial labs reveal hemoglobin of 20.1 g/dl, hematocrit of 60%, and platelet count of 567,000. Which of the following is the most likely physical exam finding in this patient? A Erythromelalgia B Hepatomegaly C Loss of vibratory sense D Petechiae E Splenomegaly

Splenomegaly Splenomegaly is the most common physical exam finding in patients with polycythemia vera (PCV) and may be accompanied by hepatomegaly (B). Erythromelalgia (A) also occurs in patients with PCV due to increased red blood cell mass, but occurs less commonly than splenomegaly. Patients with PCV often have thrombocytosis, which most commonly contributes to thrombotic events, but less commonly may result in abnormal platelet function and petechiae (D) and increased bleeding risk. PCV is associated with elevated B 12 levels and not likely to result in the classic neurologic findings of B 12 deficiency including peripheral neuropathy assessed by vibratory sensation (C), ataxia and dementia.

A 64-year-old, nonsmoking male presents to the clinic with complaints of headache and dizziness. Initial labs reveal hemoglobin of 20.1 g/dl, hematocrit of 60%, and platelet count of 567,000. Which of the following is the most likely physical exam finding in this patient? A Erythromelalgia B Hepatomegaly C Loss of vibratory sense D Petechiae E Splenomegaly

Splenomegaly Splenomegaly is the most common physical exam finding in patients with polycythemia vera (PCV) and may be accompanied by hepatomegaly (B). Erythromelalgia (A) also occurs in patients with PCV due to increased red blood cell mass, but occurs less commonly than splenomegaly. Patients with PCV often have thrombocytosis, which most commonly contributes to thrombotic events, but less commonly may result in abnormal platelet function and petechiae (D) and increased bleeding risk. PCV is associated with elevated B 12 levels and not likely to result in the classic neurologic findings of B 12 deficiency including peripheral neuropathy assessed by vibratory sensation (C), ataxia and dementia.

A 25-year-old male basketball player presents with acute onset shortness of breath associated with right-sided chest pain. The pain is unaffected by position and is worse with inspiration. He was grocery shopping when it started. He denies chest trauma. He had an upper respiratory infection earlier in the month that had resolved without incident. He smokes 1 pack of cigarettes per day and has no significant past medical history. On PE he is afebrile; BP is 138/80; P is 124; R is 24; and pulse oximetry is 94% on room air with mild respiratory distress. Trachea is midline. Lungs are clear to auscultation bilaterally with no wheezing or rhonchi; tactile fremitus and percussion are equal throughout posterior lung fields. Heart is tachycardic with normal S1 and S2, no murmur, rubs, or gallops. Question What is the diagnosis? Answer Choices 1 Pericarditis 2 Pulmonary embolism 3 Spontaneous pneumothorax 4 Acute chest syndrome 5 Community acquired pneumonia

Spontaneous Pneumothorax The patient has a primary spontaneous pneumothorax (PSP). There is a higher incidence in males and usually occurs between 20-40 years of age. PSP is heavily associated with smoking. Patient presentation and physical exam findings will vary depending on the size of the pneumothorax. Most commonly, patients present with unilateral chest pain, dyspnea, and cough occurring with minimal activity, and tachycardia and tachypnea. A large PSP may become a tension pneumothorax, and the patient may have a deviated trachea and no breath sounds on affected side with increased resonance to percussion with respiratory distress and shock. Imaging of choice for a PSP is a chest radiograph, usually an upright inspiratory film. A white visceral pleural line is diagnostic. Supine or lateral decubitus views can also be used. Pericarditis is a differential diagnosis, but the chest pain is usually relieved with positioning. Pulmonary embolism is another differential, but the patient does not have risk factors of hypercoagulability, venous stasis, or vascular injury. A patient with CAP would usually be febrile and complain of associated symptoms such as a productive cough and malaise. Acute chest syndrome is associated with sickle cell disease.

A thin 26-year-old woman presents with sudden onset of dyspnea, non-productive cough, and vague chest pain radiating to the left shoulder. She is a soccer player, and her symptoms initially occurred 24 hours ago during her usual 5-mile training run. PMH is unremarkable. EKG shows sinus tachycardia, 105 bpm. Respiratory rate equals 30 breaths per minute. Decreased breath sounds and hyperresonance are noted on the left thorax; otherwise, the physical exam is unremarkable. What is the most likely diagnosis? Answer Choices 1 Myocardial infarction 2 Spontaneous pneumothorax 3 Exercise-induced asthma 4 Dissecting aortic aneurysm 5 Atypical pneumonia

Spontaneous pneumothorax is more likely to occur in young, tall, thin individuals; a history of smoking is commonly present. Initial symptoms include sudden onset of chest pain, dyspnea, and cough; symptoms are usually associated with exertion. Chest X-rays will show air in the pleural space along the lung border on the affected side, without evidence of pleural effusion. Myocardial infarction is less likely in this age group, but it cannot be ignored and must be ruled out. In addition to the symptoms mentioned in this case, exercise-induced asthma typically presents with diffuse expiratory wheezing and prolonged expiratory phase. Dissecting aortic aneurysm may occur in individuals with Marfan's syndrome, which is also seen in tall, thin individuals. However, the classic presentation is that of sudden onset of severe chest pain, radiating to the back, with or without symptoms of hypovolemic shock. The lack of fever, prodromal period, fatigue, malaise, or other constitutional symptoms makes an infectious agent unlikely.

A 2-year-old male presents with a 10-day history of fever, cough, and decreased appetite and fluid intake. He is normally healthy. On examination, the child appears ill, has a temperature of 102.2 0 F, a pulse rate of 122, and a respiratory rate of 36. On auscultation of the lungs there are rhonchi heard on the right lung fields, as well as a small amount of wheeze. A chest x-ray is ordered, which reveals the presence of pneumatoceles. Based on these findings, what is the most likely pathogen causing this patient's infection? A Streptococcus pneumoniae B Hemophilus influenza C Staphylococcal aureus D Pseudomonas aeurginosa E Chlamydia pneumoniae

Staph Aureus Pneumatoceles, pyopneumothorax, and empyemas are frequently encountered in pediatric Staphylococcal aureus pneumonias.

A 26-year-old woman who is nursing presents to clinic complaining of 2 to 3 days of increasing pain and redness in her left breast. She continues to feed her infant and has good milk supply. She is 4 weeks postpartum. She complains of some general fatigue, but no headaches, body aches, or fever. On physical exam, results are as follows: P 80, T 99.1°F, and BP 120/70, CV RRR, and lungs CTA. Her left breast has a 5-cm area of induration, in the upper outer quadrant a 2-cm mass is noted, which is tender to the touch. In addition, milk expressed is non-purulent. What is the most likely cause of the infection? A Staphylococcus aureus B Streptococcus viridians C Blastomycosis D Sporotrichosis E Streptococcus pyogenes

Staph aureus A The discrete nodules indicate a breast abscess, which can commonly occur in nursing women. Fungal infections can occur, but are rare, and Streptococcal infections are not as likely as staph infections.

A 75-year-old man was recovering from a pneumonia caused by Streptococcus pneumoniae. The patient seemed to be doing fine, but his condition suddenly deteriorated. He presents after developing a persistent fever, chills, cough, and diaphoresis. A CBC reveals leukocytosis with a left-shift. A chest X-ray demonstrates an air-fluid level in the pleural space, which is suggestive of an abscess. Question What organism caused the patient's deterioration? Answer Choices 1 Pseudomonas aeruginosa 2 Chlamydia trachomatis 3 Coccidioides immitis 4 Staphylococcus aureus 5 Legionella pneumophila

Staphylococcus Aureus The clinical picture is suggestive of a pleural empyema. Of the organisms listed, empyemas are commonly caused by Staphylococcus aureus, Strep pneumoniae, Strep pyogenes, and H. influenza in the adult patient. The other organisms listed either do not cause an empyema, or they infrequently cause an empyema.

A 33-year-old woman presents with multiple symptoms; they have been worsening over several months. She notes fatigue, loss of appetite, weight loss, abdominal pains, nausea, as well as an "achiness" throughout her muscles and joints. The patient's family has noticed her skin appears very tanned, despite lack of sun exposure; she also uses more salt on food than ever before. The patient has stable hypothyroidism and takes daily medication. She denies any fever/chills, bowel changes, recent travel, psychosocial changes, or other triggers for her various symptoms. Her past medical history is remarkable for hypothyroidism. She takes 88 mcg of levothyroxine. She has had 3 pregnancies and a tubal ligation as her only surgery. She has no allergies; she denies the use of tobacco, alcohol, and drugs. She works in retail. Her family history is unremarkable. Vitals are normal with a BP of 104/64 mm Hg. On physical exam, the patient does have some hyperpigmentation, especially in the nipple/areola region. The remainder of her exam is normal. Weight loss of 8 lbs. is noted from a prior visit. Fasting, morning labs are drawn with results below. Urinalysis - Normal Complete blood count - Normal Comprehensive metabolic panel- Mildly decreased sodium and elevated potassium, rest normal Thyroid stimulating hormone (TSH)- Normal (1.8 uIU/mL) Adrenocorticotropic hormone (ACTH)- Elevated (>200 pg/mL) C-reactive protein- Normal Cortisol (morning level)- Decreased (<3 mcg/dL) Question What pharmacologic intervention is the most appropriate for this patient's condition? Answer Choices 1 Decrease the levothyroxine dose 2 Increase the levothyroxine dose 3 Start furosemide 4 Start hydrocortisone 5 Start sodium bicarbonate

Start hydrocortisone This patient is presenting with Addison disease, a deficiency in the adrenal corticosteroids. The low cortisol and the elevated ACTH confirm this. The symptoms of Addison disease are many and can be confused with other conditions, including some thyroid conditions. They may include: fatigue, weakness, abdominal pain, nausea, anorexia, weight loss, anxiety, irritability, depression, arthralgias and myalgias. Addison disease is a considered an autoimmune condition, and it is often linked with other autoimmune disorders, such as hypothyroidism. Once the condition is identified, the treatment of choice is to start hydrocortisone. The treatment may be IV in acute conditions; oral treatment is appropriate in less urgent cases, and it is usually lifelong. This patient reports stable hypothyroidism and compliant medication-taking. Furthermore, her TSH result is normal, so it would be inappropriate to decrease the levothyroxine dose or increase the levothyroxine dose. Some common symptoms of hypothyroidism include fatigue, depression, weight gain, constipation, and nail/hair changes. Symptoms of over-replaced thyroid medication can be anxiety, palpitations, and/or weight loss. In certain instances, when hyperkalemia (elevated potassium) is identified, one treatment may be to start furosemide, due to its effects on renal potassium excretion. However, this patient's hyperkalemia is mild, and once the underlying condition (Addison disease) is treated, the hyperkalemia should resolve without other measures. This patient has a mild hyponatremia (low sodium); likewise, it should resolve with appropriate replacement of steroids, such as hydrocortisone. It would be inappropriate to start sodium bicarbonate for several reasons. This medication is not intended to raise serum sodium. (Hyponatremia has many potential causes, and fluid status needs to be addressed. Intravenous saline is often part of acute treatments of hyponatremia.) Furthermore, the sodium bicarbonate is primarily used for acidosis, and as an antacid, it is not appropriate for hyponatremia.

A 55-year-old man presents for a follow-up on recent cholesterol tests. He has a past medical history of 2 coronary stents placed 3 days ago after presenting to the emergency department with chest pain and a 40 pack-year history of smoking (he quit smoking 2 years ago). He was not on any medications when he presented to the ER. Cholesterol tests reveal: total cholesterol 200 mg/dL; LDL 100 mg/dL; HDL 40 mg/DL; and triglycerides 395 mg/dL. Question What is the most appropriate treatment to reduce future cardiovascular risk in this patient? Answer Choices 1 Fibrates 2 Fish oil 3 Nicotinic acid 4 Plasmapheresis 5 Statin

Statin Explanation In patients with mild to moderate hypertriglyceridemia (150-500 mg/dL), the main indication for therapy is reduction of cardiovascular (CV) risk. Statins are the best therapy for CV risk reduction, and although they are not the most effective medication for reducing elevated triglycerides, they should be the first line therapy for a patient with a high CV risk. Many studies have shown that fibrates, nicotinic acid, and fish oil can lower triglyceride levels, but they do not lower the cardiac risk as much as statins. This patient is a high-risk cardiac patient because of his recent history of blocked coronary arteries, and his best option at this time is treatment with a statin drug. Plamapheresis is reserved for patients with severe hypertriglyceridemia who are at risk for pancreatitis.

A patient with HIV positive status, a CD4 count of 277/mcL, and is on chronic HIV medication therapy is diagnosed with pneumonia. What is the most common pathogen that would result in pneumonia for this patient? A Pneumocystosis jiroveci B Streptococcus pneumoniae C Pseudomonas aeurginosa D Klebsiella pneumoniae E Hemophilus influenzae

Strep Pneumo Due to the tight control of most of the HIV population, the most common pathogen is actually the most common community-acquired pathogen in the United States, Streptococcus pneumoniae. When the patient has lower CD4 counts and higher viral loads they are more susceptible to pathogens such as pneumocystosis jiroveci.

A 46-year-old man is hospitalized due to pneumonia. Over the course of 24 hours, he develops a very bad headache and stiff neck followed by a seizure. On physical examination, he is febrile with signs of meningeal irritation. A lumbar puncture results are as follows: TEST RESULTS and REFERENCE RANGE: CSF pressure- 250 mm H2O (70-80 mm H2O) CSF leukocytes- 8,700 cu mm (0-10 cu mm) CSF glucose- 22 mg/dL (45-80 mg/dL) CSF protein- 233 mg/dL (15-45 mg/dL) CSF gram stain- pending What organism is the most likely cause of his meningitis? Answer Choices 1 Streptococcus pneumoniae 2 Streptococcus agalactiae 3 Cryptococcus neoformans 4 Staphylococcus epidermidis 5 Staphylococcus aureus

Strep Pneumonaie The most likely pathogenic organism for bacterial meningitis depends on the age and health of the patient. Streptococcus pneumoniae is a common pathogen for meningitis. Cryptococcus neoformans is a cause of meningitis in immunocompromised patients. Staphylococcus epidermidis is a common cause of meningitis in people with CNS shunts. Staphylococcus aureus meningitis is associated with trauma, neurosurgery, or some other underlying illness or debilitating condition. Streptococcus agalactiae is a likely cause of meningitis in neonates and infants. It is the most common pathogen of neonatal meningitis because it is often found in the genital tract of women. Streptococcus agalactiae is a Gram-positive coccus.

A 56-year-old man presents with fatigue, fever, chills, and a productive cough. Symptoms began 3 days ago. The patient is drenched in sweat and has shaking chills. The patient has a 2 pack/day smoking habit. He has dyspnea (respiratory rate of 23/min) and pleuritic chest pain to his left side. Crackles are heard over the right middle, left middle, and left lower lung fields. X-rays are significant for right lower lobe, left lingular, and left lower lobe infiltrates. The patient has an elevated body temperature of 40.0° C. Sputum is collected for culture and Gram stain that is brown-green in color. A CBC and blood cultures are also ordered. The CBC results are significant for an elevated WBC of 14.0 x 109/L with a neutrophilic left shift on the differential. The sputum Gram stain contains >25 WBCs per lower-power field with no epithelial cells and many Gram-positive diplococci (refer to the image). The next day, the culture grows 4+ of an alpha hemolytic organism that is catalase negative and bile salt soluble. Blood cultures also turn positive that next day for Gram-positive diplococci. What is causing the pneumonia with septic complications? Answer Choices 1 Haemophilus influenzae 2 Streptococcus pneumoniae 3 Chlamydia pneumoniae (TWAR) 4 Candida albicans 5 Moraxella catarrhalis

Streptococcus pneumoniae is a Gram-positive diplococcus; it is catalase-negative, alpha hemolytic on BAP, PYR-negative, P-disc (optochin) sensitive, and bile salts soluble. The organism has an external capsule. It is a major cause of lobar pneumonia. It is also an important cause of a host of other diseases, such as otitis media, septicemia, sinusitis, and meningitis. Patients with pneumococcal pneumonia usually have an appearance that is described as grayish and noxious. They usually present with a body temperature of 102° -103° F, an elevated pulse rate, and a respiratory rate of 20-24 per minute. Chest radiographs usually reveal an area of infiltration involving less than a full segment; several areas may also be involved. Consolidation that involves a full segment or lobe is detected in the majority of bacteremic cases, and is more common when pneumococcal pneumonia occurs in otherwise healthy young adults. There is diminished respiratory excursion (splinting) on the affected side because of pain. Dullness to percussion is present in half of all cases. Crackling sounds are heard on careful auscultation in nearly all cases. WBC is elevated. In cases where there is no fever present and there is a decrease in WBC and the patient is a young adult, the pneumonia can progress rapidly with grave results.

A 44-year-old woman G5P5 presents for her annual pelvic examination. Her menses are regular, and she is currently mid-cycle. She notes 'leaking urine' when she coughs, sneezes, or strains. There is a bulge into the anterior vaginal wall; it is exacerbated when the patient is asked to 'bear down'. A urinalysis is unremarkable. There is no cervical motion tenderness or discharge noted on pelvic examination. The patient is afebrile and in no distress. No lesions are noted on the external genitalia and the pelvic examination is unremarkable except for the bulge noted previously. Vaginal cultures for gonococcus (GC) and chlamydia are pending. Question What is the most likely diagnosis? Answer Choices 1 Bladder infection 2 Menopause 3 Multiple sclerosis 4 Pelvic inflammatory disease 5 Stress urinary incontinence

Stress urinary incontinence Incontinence and incomplete emptying of the bladder are both common symptoms in patients with multiple sclerosis because the disease involves a dysfunction of muscle coordination and strength. The patient does not give a history of previous symptoms suggestive of MS, but a workup may be warranted if symptoms worsen or persist without another obvious etiology. The presence of an anatomical defect suggests incontinence resulting from a cystocele in this patient. Pelvic inflammatory disease may also cause dysuria, often in association with cervical wall motion tenderness. GC and chlamydia cultures are pending; if there is a suspicion of PID, empiric treatment should be considered. There are no lesions consistent with herpes or chanchroid, and the pelvic examination does not suggest the presence of an infection.

An aspirated peanut that is causing a partial obstruction of the trachea in a child is most likely to cause which of the following physical exam findings? A Aphonia B Inability to cough C Stridor D Progressive cyanosis E Rhonchi

Stridor Incomplete airway obstruction due to a foreign body will cause turbulent air flow in the airway and an inspiratory wheeze sound, known as stridor. Attempts should be made to remove the foreign body, leaving the patient with a partial obstruction to utilize the cough reflex to remove the foreign body. If unsuccessful, or if findings such as aphonia, an inability to cough, progressive cyanosis, or unconsciousness occur, a complete obstruction is present and definitive intervention must convene. Age appropriate measures, utilizing back blows and chest compressions in infants younger than the age of one and abdominal thrusts in children over the age of one, should be attempted.

A 9-year-old boy presents to a neurologist's office accompanied by his mother. She reports that he has had 3 episodes of what she thinks are seizures. She describes these episodes in detail to the neurologist. Question What would lead the neurologist to consider the diagnosis of tonic-clonic seizures? Answer Choices 1 Sudden loss of consciousness with convulsions that do not cease without medical attention and recur on a semi-regular basis 2 Sudden loss of consciousness with convulsions and confusion that lasts about 15 minutes each time, then resolves spontaneously 3 Sudden episodes of staring into space lasting 2-4 seconds each time and occurring 3-6 times per day 4 Alternating jerking of each of his extremities while maintaining consciousness in episodes that last several minutes 5 Episodes of blank staring that last a few minutes, during which he walks the same pattern in the room and does not remember it afterwards

Sudden loss of consciousness with convulsions and confusion that lasts about 15 minutes each time, then resolves spontaneously The correct answer is the type of seizure that involves a sudden loss of consciousness with convulsions and confusion that lasts about 15 minutes each time, then resolves spontaneously. These types of seizures are also known as generalized convulsive seizures, or tonic-clonic seizures. They are associated with loss of consciousness, convulsions, and muscle rigidity lasting minutes to hours. Sudden loss of consciousness with convulsions that do not cease without medical attention and recur regularly is not the correct answer because this description is closest to status epilepticus given the recurrent and severe nature. Epilepsy is a condition during which a person has recurrent seizures that typically need medical attention. Any seizure that lasts longer than 5 minutes should be treated medically. Episodes of starting into space that last only a few seconds occur in patients with absence seizures. Absence seizures are a type of generalized seizure known as a generalized non-convulsive seizure. Tonic-clonic seizures involve loss of consciousness and some sort of convulsion, so this type of seizure is not a tonic-clonic seizure. Alternating jerking of each of his extremities while maintaining consciousness in episodes that last several minutes is not the correct answer, as this description closest describes simple partial seizures. Patients with simple partial seizures do not experience loss of consciousness. 'Episodes of blank staring that last a few minutes, during which he walks the same pattern in the room and does not remember it afterwards,' is not the correct answer, as this describes a complex partial seizure. This type of seizure includes impairment of awareness and some form of automatism, such as lip smacking, chewing, walking, fidgeting, or another involuntary, repetitive coordinated movement.

A 15-year-old boy was healthy until 3 months ago. He has been having episodes of crampy abdominal pain and explosive liquid stools 2-3 times a day. He has had intermittent fevers and has noted blood in the stool. He has a documented 15-lb weight loss. Question What is the best way to start treatment? Answer Choices 1 Diphenoxylate hydrochloride with atropine sulfate 2 Eliminate lactose-containing foods 3 Loperamide HCL 4 Eliminate all high fiber foods 5 Sulfasalazine

Sulfasalazine

A 15-year-old boy was healthy until 3 months ago. He has been having episodes of crampy abdominal pain and explosive liquid stools 2-3 times a day. He has had intermittent fevers and has noted blood in the stool. He has a documented 15-lb weight loss. Question What is the best way to start treatment? Answer Choices 1 Diphenoxylate hydrochloride with atropine sulfate 2 Eliminate lactose-containing foods 3 Loperamide HCL 4 Eliminate all high fiber foods 5 Sulfasalazine

Sulfasalazine This patient has ulcerative colitis. Therefore, it would be most appropriate to start treatment with sulfasalazine at 50-75 mg/kg/day in 2-4 divided doses. Sulfasalazine is a sulfa drug that has important anti-inflammatory properties because it inhibits the synthesis of mediators of the inflammatory response. In some patients, sulfasalazine is poorly tolerated, in which case mesalamine (50-100 mg/kg/day) and balsalazide (110-175 mg/kg/day) are preferable treatments. Diphenoxylate hydrochloride with atropine sulfate at 1-2 mL t.i.d. and loperamide HCl at 0.04-0.08 mg/kg/day in 2-4 divided doses are contraindicated because they would suppress the symptoms of ulcerative colitis without amelioration of the underlying cause of the condition. Elimination of lactose-containing food would have no effect because the patient is lactose-tolerant. In addition, elimination of dietary fiber would not have any effect.

A 35-year-old woman is being treated for an acute migraine headache. She received an injection approximately 20 minutes ago; she is now experiencing significant chest pain. What medication do you suspect the patient received? Answer Choices 1 Promethazine 2 Metoclopramide 3 Ketorolac 4 Sumatriptan 5 Trimethobenzamide

Sumatriptan The clinical picture is suggestive of a side effect of sumatriptan. Sumatriptan is commonly used for treatment of acute migraine attacks. This drug can cause coronary vasospasms in 1 - 5% of patients; this is perceived as chest pain or discomfort. Promethazine has sedative and antiemetic properties; it does not cause chest discomfort as a side effect. Metoclopramide is commonly used to treat nausea and vomiting, but it does not cause chest discomfort as a side effect. Ketorolac is an NSAID and does not cause chest discomfort. Trimethobenzamide is an antiemetic used for the treatment of nausea and vomiting; it does not cause chest discomfort as a side effect.

A 45-year-old African-American man with no significant past medical history presents with a 1-hour history of left retroorbital headache. The headache was of a sudden onset and began upon waking that morning. It is described as excruciating, stabbing, sharp, and lancinating; it is rated as severe in intensity. He denies any preceding infections, nausea, vomiting, photophobia, or osmophobia; he also denies fever, chills, stiff neck, focal weakness, numbness, tingling, vision, hearing, gait, or speech changes. He recalls a similar episode several months ago; it lasted about a week, and it dissipated without complications. His physical exam is remarkable for painful distress, lacrimation with conjunctival injection, nasal congestion, rhinorrhea, left ocular miosis, and left forehead diaphoretic flushing. Question What pharmacologic agent is the most beneficial for this patient at this time? Answer Choices 1 Sumatriptan 2 Verapamil 3 Lithium 4 Topiramate 5 Prednisone

Sumatriptan The correct response is sumatriptan. This patient's most likely diagnosis is most likely a cluster headache. Pharmacologic management of cluster headache may be divided into abortive/symptomatic and preventive/prophylactic strategies. Abortive agents are used to stop or reduce the severity of an acute attack, and include oxygen, triptans, ergot alkaloids, and anesthetics. Inhalation of high-flow concentrated oxygen is extremely effective for aborting attacks. 5-Hydroxytryptamine-1 (5-HT1) receptor agonists, such as triptans or ergot alkaloids with metoclopramide, are often the first line of treatment. Stimulation of 5-HT1 receptors produces a direct vasoconstrictive effect and may abort the attack. Subcutaneous injection of sumatriptan can be effective, in large part because of the rapidity of onset. Studies have indicated that intranasal administration is more effective than placebo but not as effective as injections. Prophylactic agents are used to reduce the frequency and intensity of individual headache exacerbations. Preventive and prophylactic medications include calcium channel blockers, mood stabilizers, and anticonvulsants. Verpamil is the most effective calcium channel blocker for prophylaxis. It inhibits calcium ions from entering slow channels, select voltage-sensitive areas, or vascular smooth muscle, thereby producing vasodilation and preventing the initial vasoconstrictive phase of cluster headaches. It can be combined with ergotamine or lithium. Preliminary evidence suggests that prophylactic lithium may interfere with substance P and vasoactive intestinal peptide (VIP)-induced arterial relaxation. Anticonvulsants such as Divalproex and Topiramate are preventative medications whose mechanism of action may involve regulation of central sensitization. Prednisone is very effective for aborting the cluster headache cycle or providing intermediate prophylaxis as bridging therapy between acute and prophylactic agents. It is effective for treatment that does not respond to lithium.

Your patient is a 78-year-old woman who is an inpatient status post-colectomy for colon cancer. On post-operative day 3, her oral temperature is noted to be elevated to 100.6° F. Chest x-ray and urinalysis are both negative for signs of infection. An infectious disease consult is placed in order to better define the patient's new fever. Question What physical examination findings would support a diagnosis of superficial thrombophlebitis?

Superficial thrombophlebitis is an inflammation of a vein and can be related to a septic condition. One risk factor for superficial thrombophlebitis is presence of an IV catheter, which this patient would have due to being an inpatient and post-operative. 65-78% of cases are caused by Staphylococcus aureus, and about 88 out of 100,000 patients experience a superficial thrombophlebitis while admitted. The physical examination would reveal tenderness, erythema, and edema of the vein where the IV is present. 84% of patients would also have signs of systemic sepsis, including fever in about 70% of cases.

Which of the following is a major contraindication for surgical resection of a lung carcinoma? A Chest wall invasion B Non-malignant pleural effusion C Superior vena cava syndrome D Unilateral endobronchial tumor E Vagus nerve involvement

Superior Vena Cava syndrome Surgical resection of lung carcinoma is contraindicated in cases of superior vena cava syndrome, extrathoracic metastases, heart, pericardial or great vessel involvement, recurrent laryngeal or phrenic nerve involvement, esophageal or carina involvement, malignant effusion, or contralateral mediastinal lymph nodes. Other contraindications are patient and staging dependent.

A 23-year-old male with a recent diagnosis of Non-Hodgkin's lymphoma presents complaining of swelling of the neck and face, cough, and dyspnea on exertion. On exam you note dilated neck veins. Which of the following is the most likely diagnosis? A Angioedema B Carotid artery dissection C Cushing's syndrome D Myxedema E Superior vena cava syndrome

Superior vena cava syndrome E The classic presentation signs and symptoms of superior vena cava syndrome (SVCS) are present in this patient. Although the most common cancer associated with SVCS is lung cancer, it also occurs in Non-Hodgkin's lymphoma. Angioedema (A), carotid artery dissection (B), Cushing's syndrome (C) and myxedema (D) can result in facial swelling, but should not lead to the presence of dilated neck veins.

A 1-year-old female is having a 2-day history of fever (102 0 F oral), rhinorrhea, and dry cough, with a decreased appetite. The mother states that her daughter has been less active, and her fluid intake has decreased for her age. On exam, the child is non-toxic appearing, has a rectal temperature of 100.2 0 F, and has nasal flaring and a respiratory rate of 45, rhinorrhea, moist mucous membranes, and a minimal wheeze heard bilaterally. Her chest x-ray has no specific findings. What is the initial treatment of choice for this patient with these symptoms? A Antibiotics B Oxygen therapy C Supportive care D Antiviral medications E Plasmaphoresis

Supportive care The choice for treatment of acute bronchiolitis is supportive care. Oxygen therapy is only reserved for those patients who are hypoxic, and antiviral medications have not proven to be effective in shortening or eradicating the infection.

A 49-year-old man presents to the office complaining of general malaise with muscle aches, anorexia, fever, and severe pain over his anterior neck radiating to his ears. He states that he was ill about 2 weeks ago with a sore throat, but it resolved within a few days. On palpation, the thyroid gland is enlarged and tender. His laboratory workup shows a high T4 level and increased erythrocyte sedimentation rate (ESR). What is the most appropriate therapy for this patient's disease? A levothyroxine sodium B PTU therapy C radioiodine ablation D surgery E supportive therapy only

Supportive therapy only This is subacute, painful thyroiditis. This is a self-limiting disorder that at most requires symptomatic therapy. In mild cases, analgesics (ASA) are sufficient for pain relief and to decrease the inflammation. Prednisone may bring more relief if needed. Transient hypothyroidism should be treated as well.

A 30-year-old woman with a past medical history of Sheehan's syndrome and a craniopharyngioma, for which she has followed up with her neurologist, presents to her primary care office with increasing polyuria, urinary frequency, enuresis, and nocturia. She states that as busy office worker, she finds it difficult to drink water throughout the day and has developed an "unquenchable thirst." She also finds that her sleep is disturbed and, as a result, has mild daytime fatigue and somnolence. She denies a family and personal history of diabetes mellitus, thyroid dysfunction, illicit drug use, smoking, and alcohol use. She denies fever, chills, polyphagia, weight changes, vaginal discharge, hematuria, or abnormal urinary odor. Question What physical exam finding would be most likely in this patient? Answer Choices 1 Suprapubic distension 2 Hypertension 3 Adnexal mass 4 Cervical motion tenderness 5 Diminished vibratory sensation

Suprapubic distension Explanation This patient's presentation and past medical history are suggestive of central diabetes insipidus. Decreased secretion or action of antidiuretic hormone usually manifests as diabetes insipidus, a syndrome characterized by the production of abnormally large volumes of dilute urine. The 24-hour urine volume is >50 mL/kg body weight, and the osmolarity is <300 mosmol/L. The polyuria produces symptoms of urinary frequency, enuresis, and/or nocturia, which may disturb sleep and cause mild daytime fatigue or somnolence. It also results in a slight rise in plasma osmolarity that stimulates thirst and a commensurate increase in fluid intake (polydipsia). Overt clinical signs of dehydration are uncommon unless fluid intake is impaired. The physical examination findings vary with the severity and chronicity of DI. The examination findings may be entirely normal. Hydronephrosis, with pelvic fullness, bladder enlargement, flank pain or tenderness, or pain radiating to the testicle or genital area may be present. Unless the thirst mechanism is impaired or access to fluid is restricted, dehydration is not seen. Aside from an enlarged bladder, no specific signs of diabetes insipidus exist. Hypotension, not hypertension, would be anticipated with significant and chronic dehydration. Adnexal masses and cervical motion tenderness reflect underlying gynecological illnesses not present in this patient. Diminished vibratory sensation suggests peripheral neuropathy, which is not a known manifestation of diabetes insipidus.

What is the definitive treatment for acute cholecystitis? Answer Choices 1 Antibiotics 2 Antibiotics and observation 3 Surgery 4 Cholecystostomy 5 Observation alone

Surgery The definitive treatment for acute cholecystitis is surgery. Antibiotics and cholecystostomy may be temporizing maneuvers to calm down an acute situation or to temporize a patient that is otherwise too ill to undergo surgery, but the only treatment that will cure the pathology causing the problem is surgery, be it an open cholecystectomy or a laparoscopic cholecystectomy.

At her routine annual pelvic exam, a 39-year-old female presents to the clinic complaining of pelvic pressure and bloating for several months. She is a G3P2 who delivered vaginally. She is a nonsmoker. Her maternal aunt had a history of ovarian cancer. Her pelvic exam reveals an 8-cm ovarian mass in the right adnexal area. What is the most appropriate evaluation of the ovarian mass? A Pelvic ultrasound B Pelvic CT scan C Surgical evaluation D CA 125 level E Repeat pelvic exam 1 month

Surgical evaluation C The patient is high risk, as she is premenopausal, has a family history of cancer, and the mass is large. Therefore, surgical evaluation should be undertaken. CA 125 can be negative in early disease, and pelvic US and CT are not sensitive enough. Repeat examination should be reserved for low risk women with smaller ovarian masses.

A 69-year-old woman with a 2-year history of asymptomatic hyperparathyroidism presents with a 1-week history of generalized anxiety and intermittent confusion. 6 months ago, she had normal results on serum and urine protein electrophoresis, mammography, radiography of the chest, and ultrasonography of the neck. The immunoreactive parathyroid hormone level was 5 times the normal level, and the ionized serum calcium level was also increased. She has no other major medical problems. Question Which of the following is the preferred treatment for patients with this presentation? Answer Choices 1 Surgical exploration of the neck 2 Low-calcium diet with oral phosphate supplementation 3 Oral bisphosphonate therapy 4 Subcutaneous calcitonin therapy 5 Estrogen therapy

Surgical exploration of the neck Explanation Over 90% of cases of hypercalcemia in the elderly are caused by primary hyperparathyroidism or malignancy. The rest are caused by medications (e.g. thiazide diuretics), vitamin D intoxication, sarcoidosis, renal failure, hyperthyroidism, and milk-alkali syndrome. Most patients with primary hyperparathyroidism are asymptomatic and are identified incidentally. These patients should be monitored every 6 to 12 months. Most patients with primary hyperparathyroidism do not have disease progression; therefore, immediate therapy is unnecessary. Symptoms of primary hyperparathyroidism manifest in the form of fatigue, weakness, alterations in mental status, hypertension, kidney stones, and gastrointestinal symptoms such as anorexia, nausea, and constipation. Osteoporosis is also common finding. This patient now has symptoms of anxiety and confusion that warrant intervention. Surgery in the form of bilateral neck exploration, and parathyroidectomy is the preferred method of treatment in patients with symptomatic hyperparathyroidism. Medical management is an alternative to surgery in asymptomatic patients, patients who refuse surgery, and those who are considered poor surgical candidates. Adequate hydration should be undertaken, and medications that increase serum calcium levels (e.g., thiazide diuretics, lithium) should be avoided or discontinued. Maintaining adequate vitamin D intake is also important; vitamin D deficiency stimulates parathyroid hormone secretion and further bone resorption. Oral bisphosphonates, such as alendronate, can be used in patients with primary hyperparathyroidism in order to help preserve bone density; however, this is not the preferred treatment method. Subcutaneous calcitonin is also effective in this regard, but it does not treat the primary cause of the disorder. Oral phosphate supplementation and a low calcium diet can help lower serum calcium levels, but surgery remains the most definitive treatment. In postmenopausal women, estrogen replacement therapy suppresses bone turnover, decreases urinary calcium excretion, and increases bone mineral density. However, estrogen therapy also increases the risk of stroke, coronary artery disease, and breast cancer. Therefore, it is not first-line therapy for primary hyperparathyroidism in patients who are good surgical candidates.

Which of the following is the most common cause of hypoparathyroidism? A Familial hypoparathyroidism B Idiopathic hypoparathyroidism C Severe magnesium depletion D Surgical removal of the parathyroid E Iron deposition in the parathyroid

Surgical removal of the parathyroid Choice D, surgical removal of the parathyroid glands, is the correct answer. Surgery for head and neck cancer, thyroidectomy, and parathyroidectomy are the most common causes of hypoparathyroidism. Choices A, B, C, and E are all causes of hypoparathyroidism that occur more infrequently.

2 weeks after an upper respiratory infection, an adolescent presents with diarrhea, sweating, and increased heart rate. Physical examination reveals a tremor and a swollen, tender, and painful thyroid gland. Pulse rate is 110/min and blood pressure is 130/60 mm Hg. Refer to the attached laboratory studies. What is the appropriate treatment? Answer Choices 1 Symptomatic (NSAID, beta-blocking drug) 2 L-Thyroxin sodium 3 Thyroidectomy 4 Methimazole 5 Propylthiouracil

Symptomatic (NSAID, beta-blocking drug) The correct response is symptomatic (NSAID, beta-blocking drug). The clinical picture indicates hyperthyroidism caused by subacute thyroiditis. Sweating, diarrhea, elevated pulse rate, and wide pulse pressure with higher systolic and lowered diastolic pressures are characteristic signs of hyperthyroidism. Subacute thyroiditis usually follows a viral infection and has a benign course. A very low radioiodine uptake, while serum T3 and T4 are high, confirms the diagnosis of subacute thyroiditis. This reflects follicular cell injury and the consequential inability to trap iodine. Symptomatic treatment with non-steroidal anti-inflammatory drugs and beta-blockers is sufficient. Thyroid function returns to normal within a few weeks. The benign course of the disease makes other, more active, treatments of hyperthyroidism unnecessary.

What is the definitive treatment for the majority of patients presenting with mild symptoms of hyperthyroidism secondary to subacute thyroiditis? A Subtotal thyroidectomy B Oral methimazole C Symptomatic treatment D Radioactive iodine E Antibiotics

Symptomatic treatment The correct choice is C, symptomatic treatment. Most patients with subacute thyroiditis and symptoms of hyperthyroidism require only symptomatic treatment, with non-steroidal anti-inflammatory medications and/or beta blockers, for any cardiac symptoms including palpitations and tachycardia. Occasionally, patients may require a course of prednisone for this acute inflammatory condition. Most patients will recover spontaneously within a few months. Choices A, B, D, and E are not necessary in this condition. Most cases of subacute thyroiditis are associated with viral infections, and resolve without additional thyroid medications.

A 65-year-old female presents to clinic for her annual pap smear. She is in good health but has mild hyperlipidemia, which is controlled with diet. She had a hysterectomy more than 10 years ago for dysfunctional uterine bleeding. How often should she have a pap smear? A Annually B Every 2 years C Every 3 years D Symptomatically

Symptomatically D According to the new 2010 ACOG guidelines, women who have no high-grade lesions or cervical cancer history, and are over 65, may discontinue cervical cancer screening due to the decrease risk and slow progression if disease does occur.

A 45-year-old man presents with significant weight loss (10 pounds in 4 months), cough with hemoptysis, and pleuritic chest pain. The chest X-ray shows ill-defined opacities in both the lungs; the opacities have a reticulonodular pattern. A transbronchial biopsy is performed, and it microscopically shows a few epithelioid cells with necrotic debris. What is the most likely diagnosis? Answer Choices 1 Aspergillosis 2 Squamous cell carcinoma 3 Pneumocystis carinii pneumonia 4 Oat cell carcinoma 5 Tuberculosis

TB The epithelioid cells suggest that it is a granuloma. The hemoptysis suggests that the granuloma has eroded enough parenchyma and involved a bronchus. A granulomatous infection along with hemoptysis is typical for tuberculosis. The history of significant weight loss and hemoptysis is also typical of tuberculosis, but it can be seen in other cases also. Aspergillosis is more likely to produce a fungus ball, which is defined on an X-ray. The inflammatory reaction would vary from acute to mixed to granulomatous. Squamous cell carcinoma is usually a central mass lesion. It is unlikely to produce epithelioid cells, which are seen in the biopsy of this patient. Pneumocystis carinii pneumonia has a granulomatous pattern in rare cases, but the granulomas are small and unlikely to produce hemoptysis. Oat cell (or small cell) carcinoma is centrally located and is unlikely to be found with epithelioid cells.

A 40-year-old woman presents with anxiety, difficulty sleeping, rapid heartbeat, and tremor in her hands. You note the presence of bulging eyes and suspect Graves' disease. What blood levels should be taken so that the disease can be confirmed? Answer Choices 1 Thyroid stimulating hormone 2 Thyroid peroxidase 3 Protein-bound iodine 4 Thyroglobulin 5 Thyrotropin-releasing hormone

TSH Explanation Graves' disease is a type of hyperthyroidism caused by a generalized over-activity of the entire thyroid gland. The patient appears hot and flushed, and the thyroid gland enlarges in this condition. It is believed that Graves' disease is an autoimmune disorder. Antibodies are produced against certain proteins on the surface of thyroid cells, stimulating those cells to overproduce thyroid hormones. In this condition antibodies are produced against the thyrotropin receptor, thyroglobulin, thyroid peroxidase, and sodium-iodide symporter. The circulating autoantibodies continuously stimulate the thyroid gland via the thyrotropin receptor. Associated suppression of the pituitary thyrotropin level is due to increased production of thyroid hormones. The onset of the disease is gradual, and the symptoms may be mistaken for nervousness due to a stressful life situation. Weight loss occurs, and it is followed by other symptoms, such as trembling, muscle weakness of the upper arms and thighs, and insomnia. The pituitary gland releases Thyroid Stimulating Hormone (TSH) in response to insufficient levels of thyroid hormone. Communication between the pituitary gland and the thyroid gland through TSH levels controls the levels of thyroid hormone in the blood. If the levels of thyroid hormone are low, then the levels of TSH will rise. The measurement of TSH in the blood is taken as a measure of thyroid function. In Graves' disease, there is a suppressed level of thyrotropin along with elevated levels of free T4 or T3 hormone levels. The new generation (III generation) assay of TSH is very sensitive and has revolutionized diagnosis of Graves' disease by providing accurate measurements of very low TSH levels. Suppression of TSH is an early and highly sensitive marker of thyrotoxicosis. Estimation of thyroid peroxidase, protein-bound iodine, thyroglobulin, and thyrotropin-releasing hormone are not as reliable as TSH assay.

A 43-year-old woman has an upper respiratory viral infection, with runny nose, fever, sneezing, and congestion. 2 days later, she notices pain in the front of her neck with swelling in that area. She notices more fatigue, palpitations, sweating, and anxiety; as a result, she has been unable to go to work. She has a temperature of 100.2 F, pulse 106/min, BP 110/70 mm of Hg, and respirations 16/minute. She looks nervous and tired. Neck exam reveals a tender vague mass anteriorly. Oropharynx is erythematous without exudates. No other masses are appreciated in the neck bilaterally. Lungs are clear and the abdomen normal. She is alert and oriented, and other than a fine tremor in her hands, no neurological deficit can be elicited. Lab work is ordered. What is the next best step in the work up of this patient? Answer Choices 1 Rapid streptococcal test and if negative, throat culture 2 TSH, T4, ESR 3 Serum thyroglobulin and thyroid ultrasound 4 Heterophile antigen test 5 Complete blood count with peripheral smear

TSH, T4, ESR Explanation This patient has a viral infection followed by infection of the thyroid gland, causing subacute thyroiditis, which is also known as de Quervain's syndrome. Neck pain and a tender mass in the neck anteriorly indicate thyroid inflammation. Sometimes, the swollen thyroid may cause obstructive symptoms. Sudden release of thyroid hormone may cause transient hyperthyroidism; this is the case in this patient who is experiencing nervousness, tremors, palpitations, and sweating. Lab tests show low TSH, high free T4, high ESR, high C reactive protein, and raised serum thyroglobulin. Treatment is symptomatic; anti-inflammatory medication, steroids, and beta-blockers should be given for hyperthyroid symptoms. A reasonable approach is to relieve the pain with high doses of a non-steroidal, such as aspirin or naproxen. If the symptoms are not relieved in 2 - 3 days, prednisone at 30 - 40 mg daily should be given in the absence of contraindications. The usual treatment can last for 4 - 8 weeks. Once the pain is controlled, prednisone should be tapered. If the patient has severe pain, a steroid can be started right away. In the absence of relief from these 2 modes of treatment, the diagnosis should be questioned. Symptoms from release of thyroid hormones need not be treated if they are mild; however, if they are more severe, propranolol or atenolol help in symptomatic relief from anxiety, tremors, and palpitations. The differential diagnosis of this case includes group A beta hemolytic streptococcal infection, infectious mononucleosis, cervical lymphadenitis, and occasionally scrofula. In streptococcal infections of the throat, tonsils are enlarged, exudates are seen, and cervical lymphadenopathy is obvious. Infectious mononucleosis is also characterized by high fever, pharyngitis, and lymphadenopathy, especially posterior cervical, with hepatosplenomegaly in many cases. Fatigue is prominent without tremors, anxiety, or palpitations. The heterophile antibody test is positive, with atypical lymphocytes in the peripheral smear. Scrofula is a tuberculous lymphadenitis caused by mycobacterium scrofulaceum. It is characterized by single gradually enlarging mass in neck on 1 side; the mass is non-tender, rarely indurated, erythematous, or with a draining sinus. Cervical lymphadenitis has multiple etiologies and can be caused by viral, bacterial, fungal, or (rarely) mycobacterial organisms. Multiple lymph nodes are usually palpable, and they may be tender. The underlying condition is usually obvious.

Tardive dyskinesia

Tardive dyskinesia usually appears months or years after starting neuroleptic medication, and consists of involuntary stereotyped movements of the face, mouth, tongue, trunk, and limbs.

Patients with suspected familial hypercholesterolemia have serum cholesterol levels > 300 mg/dL and are at increased risk of atherosclerosis. Which of the following physical exam findings are nearly pathognomonic for familial hypercholesterolemia? A Tendon xanthomas B Lipomas C Bouchard's nodes D Carotid bruits E Visceral obesity

Tendon xanthomas The correct choice is A, tendon xanthomas. These are depositions of cholesterol rich substances that can present in any tendon as a mass-like lesion. They are most commonly found in the Achilles, patellar, and hand extensor tendons. Choice B, lipomas, are benign, soft, moveable subcutaneous tumors made from fat cells. Choice C, Bouchard's nodes, are painless nodules on the PIP joints, commonly seen in patients with osteoarthritis. Choice D, carotid bruits, may be heard with auscultation of the neck during the physical exam in patients with artherosclerosis of the carotid arteries. Choice E, visceral obesity, is a risk factor for diabetes and atherosclerosis, but is not pathognomonic of familial hypercholesterolemia.

A 20-year-old female college student presents to the student health center with a 1-week history of daily headaches. She has no significant past medical history. Upon further questioning, she admits to headaches occasionally over the past 2 years, but it is lasting longer this time. She classifies the pain as a 4/10 that is generalized, but it is worse in the back of the head. The headaches are not debilitating or throbbing, and she describes it as an annoying pain. Acetaminophen helps some, but does not make the headache go away completely. On physical exam, there are no neurologic deficits, vision is 20/20 uncorrected, and vital signs are within normal limits. She does complain of tenderness upon palpation of the muscles of the head, neck, and shoulders. Question What is the most likely diagnosis for this patient? Answer Choices 1 Cluster headache 2 Giant cell arteritis 3 Intracranial mass 4 Migraine headache 5 Tension headache

Tension HA Tension Headache is the most common type of primary headache disorder. Common complaints include: pericranial tenderness, poor concentration, and daily headaches (often vice-like or tight). These can be exacerbated by emotional stress, fatigue, noise, or glare. Cluster headaches affects mostly middle-aged men and typically present with unilateral periorbital pain. Migraine headaches usually present with a lateralized throbbing pain that occurs episodically following its onset in adolescence or early adult life. These headaches can be associated with nausea, vomiting, anorexia, phonophobia, photophobia, and blurred vision. Giant cell arteritis affects mostly elderly patients and can cause headaches that are preceded by myalgia, malaise, anorexia, and weight loss. Loss of vision is a common manifestation. Intracranial masses can also cause headaches that can range from mild to severe. Often these types of headaches are accompanied by neurologic deficits.

A 37-year-old Hispanic man presents with a 4-month history of mild-to-moderate headaches; on average, he gets them 3 - 4 days per week. He has tried over-the-counter analgesics with minimal relief. He is seeking care now because he had been promoted to store manager several months prior to presentation, and he is worried that his headaches are affecting his concentration. His headaches are generalized in location, described as starting at the base of his head and extending all over, feeling "tight" in nature. He denies memory loss, photophobia, nausea/vomiting, rhinorrhea, lacrimation, and upper respiratory symptoms associated with the headaches. He also denies seizures, syncope, incoordination, vertigo, weakness, and paresthesias. The patient mentions his concerns for his work several times. Although he enjoys his work, he admits to having some anxiety about being able to handle his new duties. His family history is negative for headache. Physical exam is performed; vitals, neurological, cardiovascular, and HEENT findings are all normal. Question What is the most likely diagnosis? Answer Choices 1 Cluster headache 2 Headache secondary to brain tumor 3 Migraine headache 4 Subarachnoid hemorrhage 5 Tension headache

Tension HA This patient presents with a most likely diagnosis of tension headache. Tension headaches, the most common type of headache, present as rather diffuse/generalized head pain, without the classic characteristics of the other types of headaches. A component of psychological stress is often present. Cluster headaches are a severe type of headache, most often occurring in middle-aged men, in which the severe headache is accompanied by several other symptoms, such as lacrimation, nasal rhinorrhea, and congestion. The pain is usually localized in the periorbital region. About 1/3 of patients with a brain tumor will present with a headache; however, tension-type headaches are far more common in the general population. Headaches secondary to brain tumor can vary in character and intensity. New onset headaches later in life, especially those without a coinciding significant psychosocial stressor, are of greater concern; they have a larger possibility of being caused by an intracranial mass. Nausea and vomiting or neurologic symptoms may accompany headaches caused by brain tumor. Migraine headaches are also quite common. They are classically described as throbbing in nature and are accompanied by nausea (and possibly vomiting), photophobia (sensitivity to light), and phonophobia (sensitivity to sounds)' visual changes and aura are also possible. Migraines are more common in women than men, but they can occur in men. They are usually a chronic condition, beginning in adolescence or the early adult years. Stress can be among many triggers for migraines. This patient's pain pattern and age of onset are inconsistent with migraine, and he lacks the associated symptoms. Subarachnoid hemorrhage (SAH) is classically described by patients as an acute headache that is the "worst headache of my life." There may be preceding trauma and associated nausea and vomiting; nuchal rigidity (due to meningeal irritation) and mental status changes are also seen. This patient's headache history is inconsistent with SAH.

A 29-year-old Caucasian man with a prior history significant for right cryptorchidism presents for a routine physical examination. His prior cryptorchidism was corrected by orchiopexy at the age of 6 months. He has no complaints at present. His physical exam reveals no abnormalities other than bilateral gynecomastia and a right testicular mass which is painless and firm; it measures approximately 1.5 cm in diameter. The right testicular mass does not transilluminate, nor does it disappear when the patient lies supine. There is no femoral or inguinal lymphadenopathy, nor are there palpable hernias. Question What is the most likely diagnosis? Answer Choices 1 Testicular malignancy 2 Scrotal hydrocele 3 Epididymitis 4 Orchitis 5 Varicocele

Testicular malignancy Explanation This patient's physical exam findings suggest testicular malignancy. In the United States, the incidence of testicular cancer in African-Americans is approximately 1/4 of that in Caucasians. Within a given race, individuals in the higher socioeconomic classes have approximately twice the incidence of those in the lower classes. Testicular cancer is slightly more common on the right side than on the left, coinciding with higher rates of right cryptorchidism. Of all acquired and congenital risk factors, cryptorchidism is the strongest associated risk factor associated with testicular cancer. Placement of the cryptorchid testis into the scrotum (orchiopexy) does not alter the malignant potential of the cryptorchid testis; however, it does facilitate examination and tumor detection. The most common symptom of testicular cancer is a painless enlargement of the testis. Enlargement is usually gradual, and a sensation of testicular heaviness is not unusual. Acute testicular pain is seen in approximately 10% of cases, and may be the result of intratesticular hemorrhage or infarction. Approximately 10% of patients are asymptomatic at presentation, and the tumor may be detected incidentally following trauma or by the patient's sexual partner. On physical exam, a testicular mass or diffuse enlargement is found in most cases. The mass is typically firm and nontender, and the epididymis should be easily separable from it. A hydrocele may accompany the testicular tumor and help to camouflage it. Transillumination of the scrotum can help to distinguish between these entities. Palpation of the abdomen may reveal bulky retroperitoneal disease; assessment of supraclavicular, scalene, and inguinal nodes should be performed. Gynecomastia is present in 5% of all germ cell tumors, but may be present in 30 - 50% of Sertoli and Leydig cell tumors. Its cause seems to be related to multiple complex hormonal interactions involving testosterone, estrone, estradiol, prolactin, and hCG. Hemoptysis may be seen in advanced pulmonary disease. Findings consistent with epididymitis are an enlarged, tender epididymis associated with fever, urethral discharge, and irritative voiding symptoms. A translucent, fluid-filled hydrocele should be visualized by transillumination of the scrotum. Orchitis presents with inflamed, painful, tender, and swollen testes. A varicocele, which is an engorgement of the pampiniform plexus of veins in the spermatic cord, should disappear when the patient is in the supine position. It is frequently described as a 'bag of worms' that is separate from the testes.

GCS values

The GCS is correlated with degree of brain injury: GCS ≥13, minor injury; GCS 9 -12, moderate injury; and GCS 3 - 8, severe injury (coma). A GCS ≤ 8 generally indicates the need for airway protection via endotracheal intubation.

A 45-year-old overweight man presents for a routine physical. He relates that he is always thirsty and urinates frequently at night. You order several blood tests, including a test for hemoglobin A1C. The levels of hemoglobin A1C level are 8.0, and you conclude that he has type II diabetes mellitus. You prescribe Glyburide, a sulfonylurea drug. After 3 months, his hemoglobin A1C level is 7.5. You add the drug Metformin, which drops the hemoglobin A1C to 6.8. What is the primary cause of the reduction in the patient's hemoglobin A1C level? Answer Choices 1 Reduction of fatty acid oxidation by metformin 2 The ability of metformin to decrease the production of glucose by the liver 3 Increased processing of proinsulin to insulin 4 Increased incorporation of GLUT4 into adipose tissue plasma membranes 5 Reduced clearance of glyburide from patient's serum

The ability of metformin to decrease the production of glucose by the liver Explanation Metformin (Glucophage®) is a hypoglycemic drug that is effective only in the presence of insulin because it acts to increase the action of insulin. It does not alter insulin production. One important mechanism by which metformin works is decreasing the production of glucose by the liver. Metformin acts by activating the AMP-activated protein kinase (AMPK), a major cellular regulator of lipid and glucose metabolism. In addition, glucose utilization by muscle is increased. Given in combination with a sulfonylurea, metformin lowers blood glucose concentrations more than either drug alone. This treatment is not effective for patients with insulin-dependent diabetes (Type I) because the beta cells are not viable. Type II diabetes mellitus, also called non-insulin-dependent diabetes mellitus (NIDDM), has a slow onset and is typically seen in patients over the age of 40. Insulin is present, but it is not properly utilized. Controlling diet can often control NIDDM. In cases where diet does not lower blood glucose levels, drugs called oral hypoglycemic agents (such as the sulfonylureas and metformin) are prescribed. Sulfonylurea agents (glyburide, glipizide, and glimepiride) act to lower blood glucose levels by causing the pancreas to secrete more insulin and by allowing target cells to better use the insulin. The main mechanism of action of the sulfonylurea drugs is to stimulate the release of endogenous insulin from the beta cells of the pancreas. The major insulin responsive glucose transporter, GLUT4, is not affected directly by metformin. The conversion of proinsulin to insulin occurs in the secretory granules as they mature, and it is regulated by glucose levels. Metformin has been shown to have no effect on the oxidation of fatty acids such as oleate.

A 24-year-old man presents with a 1-week history of shortness of breath and a nonproductive cough. On physical exam, he is tachycardic, tachypneic, and febrile. He has lost weight without a change in dietary habits. Auscultation of this chest reveals bibasilar crackles. A chest X-ray is ordered and demonstrates diffuse interstitial infiltrates. You collect an arterial blood gas, and the results show moderate hypoxemia. A metabolic panel is ordered, and the only abnormality is an isolated elevated lactate dehydrogenase (LDH) enzyme. Question What is the most likely diagnosis? Answer Choices 1 Bowen's disease 2 Streptococcal pneumoniae 3 Mycoplasma pneumoniae 4 Stevens-Johnson syndrome 5 Pneumocystis jiroveci

The clinical picture is suggestive of Pneumocystis jiroveci pneumonia. Signs and symptoms include fever, dyspnea, and a nonproductive cough. Chest examination reveals bibasilar crackles on auscultation, and X-rays reveal diffuse interstitial infiltrates. Isolated elevated LDH is often a finding on serum chemistries. The infectious agent is Pneumocystis jiroveci. This type of pneumonia is common in AIDS patients. Bowen's disease is a form of early squamous cell carcinoma. Lesions can appear as solitary or multiple, and they are pink or red in color with a slightly scaling surface, small erosions, and possible crusting. Streptococcal pneumoniae and/or Mycoplasma pneumoniae pneumonia are community-acquired pneumonias that present similarly to Pneumocystis jiroveci pneumonia, except bibasilar crackles are not present and isolated LDH is not a finding. Steven-Johnson syndrome is a mucocutaneous drug-induced or idiopathic reaction of the skin that is characterized by skin tenderness and erythema of skin and mucosa, followed by extensive cutaneous and mucosal epidermal necrosis and sloughing.

A 42-year-old gravida 1, para 1 presents with a 4-month history of menorrhagia. She is having shortened menstrual cycles that are sometimes only 15 days in length, with menstrual bleeding for 5 - 6 days. She is using approximately 12 - 14 pads or tampons per day. She admits to fatigue, headaches, and occasional dizziness, but denies syncope. There is no dysmenorrhea. A thyroid-stimulating test last month was within normal limits. Abdominal and pelvic exams are normal. What statement is true regarding dysfunctional uterine bleeding? Answer Choices 1 It is seen in all age groups equally 2 Dilatation and curettage is the only treatment 3 The condition is most common after 40 years of age 4 The condition is not seen in adolescents 5 It is never associated with ovulatory cycles

The condition is most common after 40 years of age Explanation Dysfunctional uterine bleeding (DUB) is a condition of irregular uterine bleeding in a patient who does not have an anatomic uterine lesion. It is most common above the age of 40 years (50% of the cases), but it is also seen in adolescents (20%), in whom it is associated with anovulatory cycles. Anovulatory cycles are characterized by abnormal levels of estrogen and may be due to estrogen withdrawal or breakthrough. A deteriorating ovarian follicular function is responsible for anovulatory bleeding during the climacteric. Other etiologies, such as polycystic ovarian disease, fibroids, and thyroid disease need to be ruled out before making the diagnosis. An endometrial biopsy or dilation and curettage (if the patient cannot undergo an endometrial biopsy in the office) can be diagnostic, but it is not curative or even therapeutic in a patient with DUB. Medical therapy, including estrogens, progestational agents, progesterone-impregnated intrauterine devices, and combination oral contraceptives are used to treat the condition.

The patient is a 47-year-old male who presents to an urgicare center with 18 hours of abdominal pain, nausea, vomiting, and chills. He is a single construction worker, denies smoking, and has at least a 10-year history of drinking 2-4 alcoholic beverages daily. A series of lab work is performed on the patient to evaluate his abdominal pain prior to abdominal imaging. Question What laboratory results would be most indicative of the patient suffering from acute pancreatitis? Answer Choices 1 serum amylase 310 U/L and serum lipase 760 U/L 2 serum amylase 250 U/L and serum lipase 110 U/L 3 serum aspartate aminotransferase 32 U/L and serum alanine aminotransferase 29 U/L 4 serum white blood cell count 14,000/mm3 and serum total bilirubin 1.8 mg/dl 5 serum aspartate aminotransferase 120 U/L and serum alanine aminotransferase 40 U/L

The correct answer is a serum amylase of 310 U/L and a serum lipase of 760 U/L, as these levels are significantly elevated, which is indicative of acute pancreatitis. The normal range for serum amylase is 30-220 U/L, and the normal range for serum lipase is 0-160 U/L. Both levels being significantly elevated is typically seen in a patient with acute pancreatitis. Serum lipase is both more sensitive and more specific than serum amylase for diagnosis of acute pancreatitis, but more so when the serum lipase is at least 3 times the normal level (as is with this case). However, various other biliary and intestinal diseases can also alter these lab results. Blood work results with a serum amylase of 250 U/L and a serum lipase of 110 U/L do not indicate acute pancreatitis. The serum amylase is slightly elevated, but the serum lipase is normal. This can occur for many gastrointestinal-related disorders and renal failure, but these lab results could also commonly be found in a patient with chronic (rather than acute) pancreatitis. Lab results of serum aspartate aminotransferase (AST) 32 U/L and serum alanine aminotransferase (ALT) 29 U/L are actually normal results for these particular lab tests. The AST and ALT can be used to identify hepatocellular diseases of the liver. However, mildly increased levels can also be seen in patients with acute pancreatitis. Lab results of serum aspartate aminotransferase 120 U/L and serum alanine aminotransferase 40 U/L can be seen in patients with various hepatocellular levels as both are increased from normal. These results are possible in a patient with acute pancreatitis, but are not the most indicative of the answer choices. Cirrhosis is a disease of the liver that can often occur in patients with large alcohol intake and often causes lab results such as these. Specifically, results that show an AST level that is 3 times that of the ALT level. Lab results of serum white blood cell count 14,000/mm3 and serum total bilirubin 1.8 mg/dl could be seen with various infections separately. However, the combination of both results limits the possible disorders. The white blood cell count could be elevated in a patient with acute pancreatitis, and the serum bilirubin could also be elevated in patient with acute pancreatitis when it is associated with alcoholic hepatitis. However, these results are not the MOST indicative of acute pancreatitis when compared to the other options.

A 32-year-old male presents with odynophagia, dysphagia, and chest pain. His past medical history consists of him being HIV positive. He is currently not taking any medications, as he cannot afford to pay for them. An endoscopy is ordered and the results show several large, deep ulcerations. Initial treatment consists of which of the following medications? A Nystatin suspension B Acyclovir C Famciclovir D Ganciclovir E Fluconazole

The correct answer is ganciclovir. This patient has cytomegalovirus esophagitis, and the initial treatment is ganciclovir 5 mg/kg IV every 12 hours for 3 to 6 weeks. Nystatin suspension is used to treat oropharyngeal candidiasis. Acyclovir and famciclovir are used to treat herpetic esophagitis. Fluconazole is used to treat candidal esophagitis.

A 28-year-old man presents with acute onset of dyspnea and left-sided chest pain. He denies any trauma or previous similar complaints in the past. Chest X-ray demonstrates a visceral pleural line just under the left hilum. Question What left-sided findings would you expect on physical exam? Answer Choices 1 Positive whisper pectoriloquy 2 Egophony 3 Increased tactile fremitus 4 Hyperresonance to percussion 5 Pleural rub

The correct answer is hyperresonance to percussion over the affected area, which is the most consistent finding on physical exam for a pneumothorax. Based on history, physical exam and radiographic findings, spontaneous pneumothorax is the most likely diagnosis. Tactile fremitus is decreased over the affected area on physical exam. Whisper pectoriloquy is absent over the affected area. Egophony is heard when consolidation is present, not with a pneumothorax. A pleural rub is not typically heard with a pneumothorax, but it is heard in conditions such as pleuritis, pleural effusion, and possibly pneumonia.

A 75-year-old man presents with a 4-month history of dyspnea on exertion and a productive cough. He also unintentionally lost 10 pounds in 2 months. His past medical history is significant for coronary artery disease and a myocardial infarction. He has smoked an occasional cigar the last few years. He has been retired for 12 years, but for 30 years he worked odd jobs in the construction industry. He also helped his father in the family's car garage shop. Vital signs are normal. His physical exam is remarkable for decreased breath sounds in left lower lung fields and dullness to percussion. A chest radiograph is ordered, and it shows a left sided pleural effusion. Question What condition does this patient have? Answer Choices 1 Lung cancer 2 Congestive heart failure 3 Malignant mesothelioma 4 Pneumonia 5 Recurrent postmyocardial infarction pericarditis

The correct answer is mesothelioma. His work history predisposed him to asbestos. Mesothelioma can take more than 40 years to surface, and the most common industries that expose people to asbestos are construction, mining, brake linings, roofing, and working in shipyards. This patient worked in both construction and a car garage shop, which would have likely exposed him routinely to asbestos (1). Lung cancer is not likely as he does not have the classic risk factors for developing it. His cigar smoking history is not significant. Congestive heart failure is an incorrect choice; his work history is the most important thing to focus on in this case. There is no mention of any significant cardiac findings on exam. Also, there is a history of loss of weight, which favors mesothelioma. Pneumonia is incorrect. Even though he could have this as an underlying disease, the pleural effusion seen on the chest radiograph is indicative of malignant mesothelioma. Recurrent postmyocardial infarction pericarditis is incorrect because there was no mention of pleuritic or inspiratory chest pain.

A 39-year-old woman presents for an annual appointment. She has a history of diabetes mellitus, but she is otherwise healthy. Her recent lab work reveals that her serum glucose is well controlled by a combination of lifestyle and 1 medication. Although this is a routine appointment, she does present with recent onset of some unusual symptoms. Physical examination of the patient also reveals some abnormalities. Question What history and physical examination combination would prompt a workup for Addison's disease? Answer Choices 1 Cramping, numbness around mouth, and tingling in distal extremities combined with carpopedal spasm and exaggerated deep tendon reflexes 2 Weight gain, decreased libido, menstrual irregularity combined with moon facies, and increased adipose tissue in neck 3 Fatigue, dizziness, salt craving combined with increased pigmentation of her lips and hand creases, as well as hair loss 4 Bone pain, flank pain, and generalized anxiety combined with hypertension and mental confusion 5 Hair loss, decreased energy, and constipation combined with periorbital puffiness and dry, coarse skin

The correct answer is the combination of fatigue, dizziness, and salt craving with increased pigmentation of her lips and hand creases, as well as hair loss. Addison's disease is the result of adrenal cortex destruction, which causes deficiencies in cortisol, aldosterone, and androgens. Patients can have any combination of various systemic symptoms, including fatigue, weakness, nausea, vomiting, abdominal pain, dizziness, chronic diarrhea, depression, salt craving, and a decreased tolerance for cold. In addition, the physical examination findings can also vary. Some possible findings are hypotension, weight loss, vitiligo, hair loss, and increased pigmentation. The increased pigmentation can occur in the hand creases, dental-gingival margins, buccal and vaginal mucosa, lips, areolas, scars, and pressure points. The increased pigmentation is a result of increased secretion of β-lipoprotein. The combination of weight gain, decreased libido, and menstrual irregularity with moon facies and increased adipose tissue in neck are indicative of a patient with Cushing's disease. Cushing's disease is essentially the opposite of Addison's disease; its signs and symptoms are caused by an excess of adrenocorticotropin rather than the deficiency seen in Addison's disease. Patients with Cushing's disease can also experience hypertension, hirsutism, depression, easy bruising, muscle weakness, obesity, thin skin, skeletal growth retardation, and acne. The combination of cramping, numbness around mouth, and tingling in distal extremities with carpopedal spasm and exaggerated deep tendon reflexes could occur in a patient with hypoparathyroidism. This disorder is a result of a deficiency of parathyroid hormone from congenital absence, injury, surgery, or other diseases. Most of the symptoms associated with hypoparathyroidism are a result of hypocalcemia. The combination of bone pain, flank pain, and anxiety with hypertension and mental confusion could occur in a patient who has hyperparathyroidism. This disorder is characterized as dysfunction in the body's regulatory system for parathyroid hormone. While up to 75% of patients can be asymptomatic, the hypercalcemia associated with hyperparathyroidism can result in the classic complaints of painful bones, renal stones (causing flank pain), abdominal groans, and psychic moans. The combination of hair loss, decreased energy, and constipation with periorbital puffiness and dry, coarse skin is indicative of hypothyroidism. Hypothyroidism is typically an autoimmune disease or a result of prior radiation therapy or thyroid surgery.

Patients prescribed which of the following medications may present with a dorsocervical fat pad, thin extremities, and central obesity, which may mimic Cushing's syndrome? A ACE inhibitors B HIV antiretrovial medications C Loop diuretics D Opioid analgesics E Aminoglycosides

The correct choice is B, HIV antiretroviral medications. Patients using highly active antiretroviral therapy (HAART) can develop partial lipodystrophy and changes in body fat distribution. This complication occurs more commonly with long-term use, and affects 1/3 to 2/3 of patients on therapy for longer than one year. The other medication classes noted are not associated with lipodystrophy. Opioids, choice D, are associated with a decline of cortisol secretion.

A patient was recently diagnosed with type 1 diabetes mellitus. A treatment plan was initiated, with a combination regimen of insulin. Which of the following types of insulin works well with a rapidly acting insulin, such as insulin lispro, to provide 24-hour coverage for the patient? A NPH insulin B Regular insulin C Insulin aspart D Insulin glargine E Humalog 75/25

The correct choice is D, insulin glargine. This is the only long acting insulin listed. The combination of a long acting insulin with a rapidly acting insulin provides physiologic insulin replacement to the patient. This regimen provides postprandial control after meals and basal coverage throughout the day and night. Choice A NPH insulin, can be used by itself in two or more injections throughout the day. Choice B, regular insulin, can be used instead of rapid acting insulin, and not in combination with it. Choice C, insulin aspart, is a type of rapidly acting insulin and would not be used in combination with another rapidly acting insulin. Choice E, Humalog 75/25, is a combination insulin preparation with 75% intermediate acting insulin and 25% insulin lispro.

A patient is being treated for hypothyroidism. His condition has been stable for the past year. What blood test should be ordered and monitored yearly in this patient? A Total T4 B T3 resin uptake C Thyroid releasing hormone D Thyroid stimulating hormone E Free T3

The correct choice is D, thyroid stimulating hormone. This test will help to monitor patient adherence with thyroid hormone supplementation, as well as to fine tune the dose so that the TSH remains within the reference range. Choices A, B, and E can be used in the work up of patients for primary hypothyroidism, but alone each test is not helpful to monitor chronic disease in patients. Choice C, serum thyroid releasing hormone, is used more commonly when investigating secondary hypothyroidism

A 40-year-old patient presents to your walk-in clinic with symptoms of hyperhydrosis, oily skin, daytime sleepiness, and snoring. Upon exam, you note large fleshy heel pads and hands with sweaty palms. The patient also has coarse facial features. When asked, the patient isn't aware of any major changes in her face or body. She has not seen another health care provider in many years and has not kept up with any health care maintenance schedule. The patient lives with her ill mother and is not currently employed. Which of the following screening tests would best aid in the diagnosis of this patient? A CT of the chest and abdomen B Thyroid scan C 24-hour urine for catecholamines D Serum calcitonin level E Serum IGF-I level

The correct choice is E, serum IGF-I level. Age and gender matched levels of IGF-I are elevated in patients with acromegaly. IGF-I is the mediator of most of the effects of GH on the body, and lead to the proliferation of bone, cartilage, and soft tissue. Although GH levels may be elevated in patients with acromegaly, they are secreted in a pulsatile fashion and are not consistently elevated. Serum GH levels are not the best screening test for acromegaly. Choices A, a CT of the chest and abdomen, and B, a thyroid scan, are expensive imaging studies that are not usually used as screening tests. They also have no role in the routine workup of patients with suspected acromegaly. Choice C, 24-hour urine for catecholamines, is a test that can be used in the work up of patients with suspected pheochromocytoma. Choice D, serum calcitonin levels, are associated with medullary thyroid cancer and other thyroid disease.

A 54-year-old Caucasian man presents with sudden severe shortness of breath. The patient is well known to you due to a 5-year history of severe emphysema. He states that he has been a basketball player all his life; he was practicing about 1 hour prior to presentation, at which point he experienced sudden chest pain and immediate shortness of breath. It is still bothering him. He describes the chest pain in the middle of the chest, more so on the right anterior side. The patient admits to smoking 2.5 packs of cigarettes daily. The patient has had at least 5 episodes similar to this one since he was 19 years old. Physical examination reveals a tall, thin, well-developed man in moderate distress. Other abnormalities include mild tachycardia (120 beats per minute) and diminished breath sounds in the posterior right lower lobe. There is noted wheezing and crackles heard throughout the rest of the lung fields in both the inspiratory and expiratory phases of breathing. Question Based upon the most likely diagnosis at this time, what pharmaceutical option may be used to help decrease this occurrence in the future? Answer Choices 1 Inhaled corticosteroids 2 Intravenous antibiotics 3 Talc sclerotherapy 4 Inhaled short-acting β2-agonist 5 Oral antibiotics

The correct response is Talc sclerotherapy. The clinical scenario is highly suggestive of a (recurrent) secondary pneumothorax. Components that lead to this diagnosis include unilateral, sudden chest pain with dyspnea and minimal physical exam findings, in this case the mild tachycardia and diminished breath sounds. The above scenario most likely fits what is considered a secondary pneumothorax, which usually occurs as a complication of various pathologies, including COPD, asthma, cystic fibrosis, tuberculosis, Pneumocystis pneumonia, and various interstitial lung diseases. Diagnosis is usually confirmed with chest radiograph. Treatment is typically observation and potential surgical intervention after about 5 days if the pneumothorax fails to resolve itself. However, the ability of the patient to withstand a surgical intervention also must be considered. There are various thought processes in terms of management of recurrent pneumothorax especially. The correct option out of the above is the talc sclerotherapy; this is commonly reserved for patients who may not necessarily be surgical candidates. Intravenous antibiotics would be correct if the pneumothorax were actually caused by the sequelae of staphylococcal pneumonia. Oral antibiotics, short-acting β2-agonist, and inhaled corticosteroids are inappropriate in the management of a secondary pneumothorax.

A 30-year-old woman presents with a 2-day history of severe pain on defecation; the patient has not had a problem with bowel movements in the past. Recently, she has noticed some increasing constipation; this may be due to starting vitamins that contain calcium. She has had 3 bowel movements in the past 2 days, which she describes as feeling like she was being cut with glass each time she passed them. The pain is excruciating and usually lets up in intensity when the bowel movement ends, but the anal area throbs for 1 - 2 hours afterwards. There is a small amount of blood noted when wiping, but none is noted in the toilet. The patient's past medical history is negative. She takes no prescription medications. There is no family history of colon cancer. On exam, what would you would expect to find?

The correct response is an anal fissure in the posterior anal verge. This patient presents with a common anorectal problem, which is an anal fissure. Most anal fissures are the result of constipation and hard stools; occasionally they are due to diarrhea. The history presented is typical, and the vast majority of fissures lie posteriorly in the anoderm; occasionally they are found anteriorly.

A 30-year-old woman presents with a 2-day history of severe pain on defecation; the patient has not had a problem with bowel movements in the past. Recently, she has noticed some increasing constipation; this may be due to starting vitamins that contain calcium. She has had 3 bowel movements in the past 2 days, which she describes as feeling like she was being cut with glass each time she passed them. The pain is excruciating and usually lets up in intensity when the bowel movement ends, but the anal area throbs for 1 - 2 hours afterwards. There is a small amount of blood noted when wiping, but none is noted in the toilet. The patient's past medical history is negative. She takes no prescription medications. There is no family history of colon cancer. On exam, what would you would expect to find? Answer Choices 1 An anal fistula 2 An anal fissure in the anterior anal verge 3 An anal fissure in the lateral anal verge 4 An anal fissure in the posterior anal verge 5 A thrombosed external hemorrhoid

The correct response is an anal fissure in the posterior anal verge. This patient presents with a common anorectal problem, which is an anal fissure. Most anal fissures are the result of constipation and hard stools; occasionally they are due to diarrhea. The history presented is typical, and the vast majority of fissures lie posteriorly in the anoderm; occasionally they are found anteriorly. Laterally positioned fissures are highly unusual and should raise a red flag. Conditions that may be associated with abnormally located fissures include Crohn's disease, HIV infection, anal sepsis, and trauma. An anal fistula (fistula in ano) is a communicating tract between the anal canal and the perineal skin. Patients with this condition would complain of a perianal discharge which may be persistent or intermittent; there may also be soreness. Most fistulas are the result of a persistent internal opening in the anal canal from a previous anorectal abscess. A thrombosed external hemorrhoid also presents as acute anal pain; however, in this instance the pain is continuous for 2 - 3 days. In addition, it is usually uncomfortable for the patient to sit down. The patient may report the presence of a highly tender lump in the anal area.

A 65-year-old Caucasian man presents for a routine physical. He states that he is concerned about the development of prostate cancer. His history is significant for benign prostatic hyperplasia (BPH), for which he underwent a transurethral resection of the prostate (TURP) 3 years ago. His social history is significant for a 50 pack-year history of smoking, and he worked for 40 years as a coal miner. His father died of prostate cancer at age 76. What factor would most likely contribute to his risk of developing prostate cancer? Answer Choices 1 Race 2 History of BPH 3 Smoking history 4 Family history 5 Occupational history

The correct response is family history. Several risk factors have been identified for prostate cancer, including race and nationality. African-Americans are twice as likely to develop prostate cancer as are Caucasian Americans, and the incidence is higher in North America and western Europe than in other areas of the world. Age is the single largest risk factor, with rates of prostate cancer increasing rapidly after the age of 50. High fat diets and a sedentary lifestyle have also been linked to prostate cancer. Men with first-degree relatives with prostate cancer are twice as likely to develop prostate cancer as are other men, and the risk is even higher if multiple relatives are affected. Other factors, such as a history of smoking and occupational exposures, have not been conclusively linked to prostate cancer. Benign prostatic hypertrophy arises in cells in a different area of the prostate gland, and a history of BPH does not increase the risk of developing prostate cancer.

A 63-year-old Caucasian woman 3 weeks status post total thyroidectomy presents with "twitching". The patient has noted significant bilateral muscle cramps as well as not being able to feel objects in her hands appropriately. She is also experiencing muscle spasms. Physical examination reveals a blood pressure of 90/50 mm Hg in both the left and right arms (with the patient sitting); there is a positive Chvostek sign, and there is also a positive Trousseau sign. ECG reveals a prolonged QT interval. Lab results are drawn, and they are as follows: Sodium 139 mEq/L (135 - 145 mEq/L) Potassium 4.0 mEq/L (3.5 - 5.0 mEq/L) Calcium 7.3 mg/dL (8.5 - 10.5 mg/dL) Magnesium 1.8 mEq/L (1.5 - 2.5 mEq/L) Question What is the most appropriate first line intervention for this patient? Answer Choices 1 Oral magnesium supplement 2 Intravenous sodium replacement 3 Intravenous calcium replacement 4 Oral calcium supplement 5 Oral potassium supplement

The correct response is intravenous calcium replacement. The patient described in the above scenario is having signs and symptoms consistent with hypocalcemia. This electrolyte deficiency is most likely due to the fact that she had a total thyroidectomy 3 weeks ago; this has led to what is termed acquired hypoparathyroidism. The parathyroid glands are primarily responsible for secreting parathyroid hormone (PTH). PTH's primary actions include increasing osteoclastic activity, renal tubular reabsorption of calcium, stimulation of the synthesis of 1,25-dihydroxycholecaldiferal by the kidneys. PTH also inhibits the absorption of phosphate and bicarbonate by the renal tubules; all of these actions lead to the increase of serum calcium. Removal of the thyroid and inadvertently the removal of the parathyroid glands (and PTH) has led to hypocalcemia. Acquired hypoparathyroidism is most commonly seen following surgical thyroidectomies and could be a transient or permanent condition. Signs and symptoms of hypocalcemia typically are not seen until serum levels fall below 7.5 mg/dL or lower. Patients may experience paresthesias, hyperreflexia, tetany, muscle spasms, muscle cramps, or even seizures. Physical examination may reveal a positive Chvostek sign (facial spasms following percussion on the facial nerve) and a positive Trousseau sign (carpopedal spasms of the hand by inflammation of a sphygmomanometer above systolic BP for several minutes.) Refractory hypotension may also be found. Severe cases of hypocalcemia may also lead to prolonged QT intervals on ECG, which potentially can lead to torsades de pointes. Oral supplementation of calcium is utilized in patients who are asymptomatic; however, our patient is displaying significant signs and symptoms. For this reason, the answer choice of intravenous calcium replacement is suitable at this point in time. Oral magnesium, oral potassium, and intravenous sodium supplementation are all inappropriate treatments at this time; these electrolytes are still in acceptable ranges.

A 45-year-old Caucasian woman presents with a 2-month history of a lump in her right breast. She has had no children and has been on oral contraceptive pills for 20 years. Her menstrual periods began at the age of 15 and are still regular. She is a successful business woman who does not smoke cigarettes and drinks 6 cups of coffee per day. On examination, she has a nontender lump in the upper outer quadrant of her right breast. Several matted right axillary lymph nodes are also palpable. Question Given her history, what is her greatest risk factor for developing breast cancer? Answer Choices 1 Use of oral contraceptive pills 2 Menarche at 15 years of age 3 Menopause at age 45 years of age 4 Excessive caffeine consumption 5 Nulliparity

The correct response is nulliparity. The clinical features are suggestive of breast cancer. Although it occurs mainly in women, a small percentage develops in men. Breast cancer has no single cause, but multiple risk factors have been identified. These include: Family history of premenopausal breast cancer in a first-degree relative like one's mother, daughter, or sister. Having the first full-term pregnancy after the age of 30 Prior personal history of breast cancer Chest exposure to high dose of ionizing radiation Nulliparity Early menarche at or before 12 years Late menopause after 50 years Current or recent use of oral contraceptive pills (for women who stopped using oral contraceptive pills 10+ years ago, there is no apparent increase in risk).

An 82-year-old woman living in a nursing home has her routine biannual CBC, sequential multiple analysis-7 (SMA-7), and urinalysis (UA). She is a widowed nonsmoker and nondrinker with 3 grown children who visit her weekly. Her CBC and SMA-7 come back normal. Her UA shows several WBCs and 105 colony-forming units per mL of urine. The patient has no symptoms and no known drug allergies. Question What is the most appropriate next step? Answer Choices 1 Send specimen for culture and sensitivity and base treatment on results 2 Treat with ciprofloxin b.i.d. for 7 days 3 Treat with ciprofloxin b.i.d. for 3 days 4 Observation without antibiotic treatment 5 Treat with ciprofloxin, trimethoprim, orsulfamethoxazole for 3 days then repeat UA to confirm resolution

The correct response is observation without antibiotic treatment. In 2004, the United States Preventive Services Task Force recommended against screening for asymptomatic bacteriuria in men and non-pregnant women. This recommendation was based on studies showing a lack of benefit in treating these patients, coupled with the potential harm in treating (in the form of adverse effects from antibiotics and development of bacterial resistance). The USPSTF updated this recommendation in 2008 based on further studies between 2004 and 2008; it corroborated previous studies showing a lack of benefit in treating asymptomatic bacteriuria. It is still recommended to screen pregnant women between 12 - 16 weeks gestation, or at the 1st prenatal visit. This recommendation is based on studies that show the detection and treatment of asymptomatic bacteriuria in pregnant women reduces the incidence of symptomatic maternal urinary tract infections and low birth weight.

A 43-year-old woman is found to have a palpable thyroid nodule. It is 1.5 cm in size, and it is located in the right lobe without regional lymphadenopathy. Upon questioning, the patient denies noticing this or any increase in the size of her thyroid. She denies hoarseness, a personal or family history of thyroid disease, and thyroid cancer. Question What is the most common form of thyroid cancer for which she is at risk? Answer Choices 1 Follicular 2 Papillary 3 Medullary 4 Hürthle cell 5 Anaplastic

The correct response is papillary. Papillary thyroid cancer (PTC) is the most common type of thyroid cancer in the United States; it has a prevalence rate of 80 - 90% of all thyroid cancers. It arises from the follicular epithelial cells of the thyroid, as does follicular thyroid cancer (FTC), which is the 2nd most frequently occurring thyroid cancer (accounting for 5 - 15% of all cases). Together, they are defined as well-differentiated thyroid carcinomas; when identified early they have a good, and frequently curable, prognosis. Psammoma bodies (PBs) are concentric lamellated calcified structures, observed most commonly in papillary thyroid carcinoma (PTC) Hürthle cell carcinoma (oncocytic) is a variant of FTC. Also arising from the follicular epithelial cells is undifferentiated (anaplastic) thyroid carcinoma; this accounts for < 5% of thyroid cancers and carries a worse prognosis than the well-differentiated histologic forms. It is aggressive and often refractory to treatment. Medullary thyroid cancer (MTC) arises from the parafollicular cells (C cells), which are calcitonin-producing and account for 5 - 10% of thyroid carcinomas. 75% of MTC patients will have sporadic cases, whereas 25% will have hereditary forms such as multiple endocrine neoplasia 2A (MEN 2A), MEN 2B, or familial (FMTC). MEN 2A is associated with hyperparathyroidism, a high risk of MTC, and pheochromocytoma. MEN 2B is associated with MTC, an increased risk of pheochromocytoma, and mucosal neuromas on the lips, tongue, and GI tract.

A 40-year-old woman presents with rectal pain. She reports that the pain is especially severe with bowel movements, and on exam she is noted to have an anal fissure. What is the best initial treatment? Answer Choices 1 Application of hydrocortisone cream to the area 3 times a day 2 Stool softeners 3 Internal anal sphincterotomy 4 Hemorrhoidectomy 5 Fissurectomy

The correct response is stool softeners. Anal fissures are usually caused by trauma to the anal canal due to the passage of hard stool. Most are found in the posterior midline of the anus, but some are found in the anterior midline. The ones found laterally may be associated with TB, syphilis, or AIDS. Hydrocortisone creams are used to decrease the inflammatory process usually associated with inflamed hemorrhoids. Anal fissures are initially treated conservatively using stool softeners and bulking agents to ease the passage of a bowel movement. If there is no improvement after 6 weeks of conservative therapy, surgery is usually required. The operation for non-healing anal fissures is an internal anal sphincterotomy. Fissurectomy is not a procedure used for the treatment of anal fissures. Hemorrhoidectomy is the operative resection of hemorrhoids, and it is not associated with the treatment of anal fissures.

A 55-year-old man presents with a 4-hour history of severe left-sided chest pain. His wife tells you that they went out to dinner earlier in the evening, but the patient felt ill shortly after returning home. He had several episodes of vomiting, followed by severe pain. On exam he appears ill; he is cool and diaphoretic. Blood pressure is 90/60, heart rate is 126, respiratory rate is 32, and temperature is 39.8o C. There is no JVD. Cardiac exam is tachycardic, but there is no murmur. Lung sounds are decreased in the left base, with dullness to percussion. The abdominal exam reveals tenderness, but there is no guarding or rebound. Laboratory studies reveal a white blood cell count of 14,000, a hemoglobin of 14 gm/dL, and a hematocrit of 44 gm/dL. EKG reveals a sinus tachycardia without any Q waves. Chest X-ray reveals a left pleural effusion. The described case scenario represents a patient with Boerhaave's syndrome, which is a spontaneous esophageal rupture after forceful vomiting. Once the diagnosis is confirmed, what is the optimal definitive treatment? Answer Choices 1 Intravenous antibiotics 2 Tube thoracostomy 3 Surgical exploration 4 Observation 5 Pulmonary function tests

The correct response is surgical exploration. A gastrografin swallow will usually confirm the diagnosis and delineate the site of rupture. If this is unsuccessful, a barium swallow should follow. It is important to delineate the site of rupture since this will impact the surgical approach. Chest X-ray findings that are more suggestive of the diagnosis include hydropneumothorax, pneumomediastinum, and mediastinal widening. The most definitive treatment is surgical exploration, identification, and repair of the perforation. Since the time frame is relatively short in this case, suture repair with an omental or pleural flap should suffice. Wide chest tube drainage is necessary to prevent postoperative sequelae. Antibiotic therapy should be also initiated, but will never be the definitive treatment alone. The mortality rate from this condition is directly related to the time delay. In patients where the perforation is repaired within 24 hours, the mortality rate is 10 - 15%; the mortality rate is over 50% in patients where the therapy is delayed beyond 24 hours.

What anticonvulsant is most effective for primary generalized seizures? Answer Choices 1 Phenytoin 2 Phenobarbital 3 Valproic acid 4 Gabapentin 5 Carbamazepine

The correct response is valproic acid. A seizure is defined as a discrete neurologic disorder in which there is excessive and synchronous discharge of cortical neurons. Epilepsy is a syndrome characterized by recurrent, unprovoked seizures. A seizure may be classified as generalized or partial. In a generalized seizure, there is bilateral cerebral hemisphere disturbance; in partial seizures, the abnormal neuronal discharge is limited to a specific brain area (e.g., right anterior temporal lobe). Generalized seizures may be primary (bilateral brain involvement at seizure onset) or secondary generalized (seizure starts in a focal brain area, then spreads to encompasses both cerebral hemispheres). Valproic acid is usually the first-line medication for primary generalized seizures. Phenytoin, phenobarbital, carbamazepine, and gabapentin have efficacy in partial and secondary generalized seizures. Phenytoin and phenobarbital have a role in the treatment of primary generalized seizures, but valproic acid is superior.

A 43-year-old woman was diagnosed with type II diabetes mellitus 6 months ago; she presents for follow up. The patient is presently asymptomatic. She currently receives oral hypoglycemics. The patient has no other significant medical history. Her glucose, acetone, lactate, and glycated hemoglobin are performed, and they are reported as follows: Serum Lactate 0.5 mEq/L Serum Acetone 0.4 mg/dl Fasting serum Glucose 130 mg/dl Glycated Hemoglobin 6% What is the best way to describe the patient's diabetic state? Answer Choices 1 The diabetic state is in good glycemic control 2 The diabetic state is not well controlled and the patient needs addition of another oral hypoglycemic agent 3 The diabetic state is not controlled and the patient's oral hypoglycemic dose needs to be increased 4 The patient's diabetic state is not in control and requires initiation of insulin therapy 5 The patient's diabetic state is well controlled and requires reduction in the dose of the oral hypoglycemic drug

The diabetic state is in good glycemic control

A 33-year-old African American woman develops a loss of appetite and has a few episodes of vomiting. She reports constipation and lethargy. She thinks that her urine output has increased. Her history is significant for the presence of sarcoidosis. On physical examination, some muscle weakness is present. Laboratory tests are ordered and are pending. An EKG is done and shows a shortened QT interval. Question Based on the EKG changes, what electrolyte abnormality would be expected? Answer Choices 1 Hypercalcemia 2 Hyperkalemia 3 Hypocalcemia 4 Hypokalemia 5 Hyponatremia

The four causes of a short QT interval: Hypercalcemia Digitalis Thyrotoxicosis Increased sympathetic tone Explanation Hypercalcemia refers to an elevated calcium level. A shortened QT interval is seen with hypercalcemia. The time from the beginning of the QRS complex to the ending of the T wave is the QT interval. Sarcoidosis can be associated with hypercalcemia. Symptoms of hypercalcemia include anorexia, vomiting, constipation, and polyuria. Muscle weakness can be seen. None of the other electrolyte abnormalities listed are associated with sarcoidosis. Hyperkalemia refers to an elevated potassium level. Hyperkalemia can result in peaked T waves on EKG. The T wave is a reflection of ventricular repolarization. There can be widening of the QRS complex as well. The QRS complex is seen when the ventricle depolarizes. Hypocalcemia refers to a depressed calcium level. There would be a prolonged QT interval with hypocalcemia. The time from the beginning of the QRS complex to the ending of the T wave is the QT interval. Hypokalemia refers to a depressed potassium level. Hypokalemia can result in a depressed ST segment and the presence of U waves. The time from the end of the QRS complex to the beginning of the T wave is the ST segment. Hyponatremia refers to a depressed sodium level. Hyponatremia is not associated with EKG changes.

Headaches may be considered primary in origin (migraine, tension-type, and cluster) or due to underlying metabolic or organic disease. Primary headaches are not associated with any structural disease, but it is believed they are caused by biochemical dysfunction. What statement correctly describes cluster headaches? Answer Choices 1 Patients usually have a positive family history for cluster headaches 2 The headache lasts an average of 30 minutes to 3 hours 3 Cluster headaches occur approximately with equal frequency in males and females 4 Cluster headache pain is typically a moderate, bilateral, steady pain 5 Menstruation is an exacerbating factor for cluster headaches

The headache lasts an average of 30 minutes to 3 hours Cluster and migraine headaches are attributable to cerebrovascular imbalances and are both referred to as vascular headaches. The cerebrovascular dysfunction in cluster headaches is likely different from that of migraine headaches. Cluster headaches usually last from 30 minutes to 3 hours; they occur in groups of 1 to 3 daily over a period of several weeks to a few months. Headache-free intervals range from several months to several years. Clusters occur more often in the spring. The pain is described as excruciating and as seeming to bore from one eye back into the head. Accompanying characteristics include ipsilateral tearing, conjunctival congestion, nasal stuffiness, and a partial Horner's syndrome. Aggravating factors are mostly unknown, but alcohol and sleep may be triggers. Physical activity is known to ameliorate cluster headaches. There is rarely a positive family history and cluster headaches occur more frequently in males. Migraine headaches occur more often in females. Alcohol, chocolate, menstruation, weather changes, and physical activity are considered triggering factors. Migraine headaches usually have moderate to severe pain described as a unilateral throbbing, and they can last from several hours to several days. Associated symptoms include nausea, photophobia, phonophobia and, less commonly, aura. Most patients have a family history positive for migraines

A 40-year-old man presents with right upper quadrant pain, generalized weakness, and weight loss. He gives a past history of hepatitis B infection. On examination, he has hepatomegaly; there are arterial bruits, ascites, jaundice, and signs of cirrhosis. What investigation would you value most in the diagnosis of this patient? Answer Choices 1 Human chorionic gonadotrophin 2 α-fetoprotein 3 Carcinoembryonic antigen 4 Alkaline phosphate 5 S-100 antigen

The history and the findings are congruent with hepatocellular carcinoma. α-fetoprotein is the major tumor marker for hepatocellular carcinoma, and is elevated in over 70% of patients with this disease. High levels of this enzyme carry a poor prognosis. Human chorionic gonadotropin is raised in cases of hydatidiform moles and chorionic carcinoma. Carcinoembryonic antigen (CEA) is useful in following the progression of gastrointestinal neoplasms, but it is not of diagnostic value. A rise in alkaline phosphatase indicates cholestatic liver disease, bone diseases (e.g., Paget's), osteomalacia, hyperparathyroidism, or renal failure. It may also be elevated in growing children.

A 76-year-old woman presents because her children are concerned that that she might have dementia. She states that she is doing reasonably well, except that she sometimes sleeps less deeply and wakes up more often than she did several years ago. According to her children, she is slower than before, and her memory has been getting worse over the last 3 years; she has difficulties recalling the specific date of an event (although she can describe the event itself). She also has a great deal of trouble with names, but she can easily recognize people. She always says: "It is on the tip of my tongue, but..." Aside from hypertension that is under control, she does not have any other health problems. She has been a widow for about 10 years. Her older brother was diagnosed with dementia. Physical examination today is within normal limits for the age, and neurological examination is nonfocal. Question What should be your next diagnostic step? Answer Choices 1 EEG 2 Stanford-Binet Intelligence Test 3 Mini-mental status examination 4 MRI of the brain 5 TSH and T4

The mini-mental state examination (MMSE) is a brief 30-point screening test for cognitive impairment. In persons older than 70, the ability to memorize, acquire, and retain new information and recall names is diminished. Memory function may be disturbed in this way despite the relative retention of other intellectual abilities. It particularly applies to episodic memory, and it is manifested by difficulties with recalling names and/or specific dates. This disturbance is known as benign senescent forgetfulness or age-associated memory impairment. It progress very little or not at all over a period of many years, and it does not interfere significantly with the person's work performance or activities of daily living. Your patient most probably has this sort of age related cognitive decline. MMSE can be used also as follow-up of her condition. Electroencephalography (EEG) is recording of spontaneous electrical activity of the brain; it is primarily used in the diagnosis of epilepsy, and it is sometimes used in the diagnosis of coma, encephalopathies, sleep disorders, and brain death. Your patient's sleep complaints are normal in advanced age. Stanford-Binet Intelligence Scales cannot be used to score adults. MRI of the brain is indicated when structural changes are suspected; however, they are not likely in this patient due to her normal neurological examination. You should perform a thyroid function test if you find out that a patient has dementia - slowness is a part of normal aging.

A 35-year-old woman presents with a 2-month history of palpitation and nervousness. She mentions that she always feels hot, even if the weather is cold. Her menses have been irregular lately. She has had no fevers recently. She was also told that her eyes are "weird looking". On examination, her blood pressure is 150/70 mm Hg, and her pulse rate 89 beats per minute. Her eyes show exophthalmos, and she also has lid lag on looking down. Thyrotoxicosis is suspected. What follows is the thyroid scan result. Question What is the most likely diagnosis? Answer Choices 1 Graves' disease 2 Toxic multinodular goiter 3 Hashimoto thyroiditis 4 Factitious hyperthyroidism 5 Toxic adenoma

The most likely diagnosis in this patient is Graves' disease. Thyrotoxicosis occurs due to thyroid hormone excess, the etiology of which varies from Graves' disease, toxic multinodular goiter, toxic adenoma, thyroiditis, and even functioning metastasis. Symptoms of thyrotoxicosis include hyperactivity, irritability, dysphoria, heat intolerance and sweating, palpitations, fatigue, weakness, weight loss with increased appetite, diarrhea, polyuria, and menstrual irregularities. Graves' disease is the most common cause of thyrotoxicosis. It is an autoimmune disease in which thyroid-stimulating autoantibodies stimulate thyroid receptors to secrete thyroxine. It is more common in women and can be identified by radionucleotide scan in which the whole thyroid shows increased uptake. Here the patient has all Graves' disease characters: she is middle aged with thyrotoxic characters and her Tc-99 scan shows the characteristic Graves' pattern. Toxic multinodular goiter is another cause of thyrotoxicosis in which there are multiple nodules that can be felt with palpation, and it is shown to be hot or active with the radionucleotide scan. It shows localized or patchy uptake in the scan pictures. Hashimoto thyroiditis is an inflammatory condition that follows viral infection. It is associated with a brief period of hyperthyroidism that is followed by hypothyroidism, which may persist throughout life. On Tc scan, it shows decreased uptake. Factitious hyperthyroidism is due to ingestion of the thyroid hormone. Usually this happens in nurses, physicians, or those who have access to medicine. It can be detected by measuring T3 and T4. There is also decreased uptake on thyroid scan. Toxic adenoma is a benign neoplastic nodule, which actively secretes thyroxine. It shows localized or patchy uptake of Tc 99 on thyroid scan.

A 20-year-old woman (G1, P0) presents for her 28th week antenatal care visit. The woman's blood group is O and is Rh-. Her husband's blood type is B and is Rh+. Her indirect Coombs test in the first antenatal care visit was negative; however, the indirect Coombs test at 28 weeks is positive. What does the mother's positive indirect Coombs test mean? Answer Choices 1 The fetus is Rh- 2 The fetus has antibodies against the mother's red cells 3 The mother has formed antibodies against Rh antigens 4 The mother has an autoimmune disorder 5 The mother is at risk for hemolysis

The mother has formed antibodies against Rh antigensExplanation The mother's positive indirect Coombs test means that she has formed antibodies against Rh antigens. The fact that the mother is Rh- and that the father is Rh+ means that the likelihood that the mother will be carrying an Rh+ baby can be either 50% or 100% depending on whether the father is heterozygous or homozygous for the Rh gene locus respectively. The antenatal screening of the mother at 28 weeks gestation was negative (showing anti-Rh antibodies), indicating her alloimmunization. The indirect Coombs test evaluates for the presence of circulating antibodies in the mother's serum and in this case showed the presence of anti-Rh antibodies in the mother's blood. Since the mother's Coombs test was negative initially and then positive at 28 weeks of her pregnancy, indicating that her child is Rh+ and has transported some of its red cells to her circulation, likely through a minor fetomaternal hemorrhage, which is most likely to occur in third trimester. Upon encounter of the Rh+ red blood cells, the mother formed anti-Rh antibodies in her blood, which were detected by the indirect Coombs test. The presence of the anti-Rh antibodies in the mother does not mean that she is at risk for hemolysis since her red cells are Rh-. The presence of an antibody in the mother's blood against a foreign antigen upon exposure to it is a normal immune response and does not represent an autoimmune disorder. The indirect Coombs test is performed on the mother and therefore can only show the presence or absence of the antibody under study (in this case anti-Rh) in the mother's blood.

A 35-year-old Hispanic man is too embarrassed to explain his chief symptom. You enter the room and coerce him to give you the reason he has come for medical treatment. He admits to having a sore that will not heal around his rectum and anal area. This lesion has been draining pus consistently for the last week. The patient further explains that he has only had intermittent pain with this lesion (4/10 on a 0 - 10 pain scale) that is made slightly worse when he has a bowel movement (ranking it a 6/10). More recently, especially in the last few days, he has noted that simply sitting increases the pain. The patient also admits to intermittent periods of itching. He denies fever, diarrhea, or ever being diagnosed with inflammatory bowel disease. Physical examination of the anorectal area reveals excoriated and inflamed perianal skin with a palpated induration. Question What is this patient's most likely diagnosis? Answer Choices 1 Anal fistula 2 Crohn's disease 3 Anal fissure 4 Anal cancer 5 Internal hemorrhoid

The patient in this scenario is experiencing signs and symptoms from an anorectal fistula. Anorectal fistulas are commonly a chronic manifestation of an acute perirectal process that eventually comes from an anorectal abscess. Many fistulas originate from an infected anal crypt gland. Patients with anorectal fistulas usually will present with a "non-healing" area (an anorectal abscess that is draining) or may describe chronic purulent drainage and a pustule-like lesion on the perianal or buttock area. They will only have intermittent pruritus or rectal pain; pain is increased during defecation, sitting, or standing. Physical examination reveals excoriated and inflamed perianal skin. An external opening of the fistula may be visualized. If no opening is seen, palpation will reveal induration just below the skin. Anal fissure is not the most likely diagnosis in this case scenario. Anal fissures most often affect infants as well as middle-aged individuals. The majority of fissures are considered primary and caused by local trauma such as passage of hard stool, prolonged diarrhea, vaginal delivery, or anal sex. Presentation of anal fissures is a tearing pain accompanying bowel movements as well as bright red rectal bleeding that is limited to a small amount noted on the toilet paper or surface of the stool. The patient described all of these components. Patients will also complain of perianal pruritus or irritation, which he also admits to experiencing. Patients with Crohn's disease often have extremely variable signs and symptoms and possibly will have these for many years prior to diagnosis. Fatigue, prolonged diarrhea and abdominal pain, weight loss, and fever with or without gross bleeding are considered hallmarks of Crohn's disease. Signs and symptoms of anal carcinoma include rectal bleeding (occurring in 45% of patients), anorectal pain or sensation of a rectal mass (30%), or even no issues (20%). Very often issues are confused with hemorrhoids and therefore patients may delay evaluation. Over 80% of patients developing anal cancer have detected human papilloma virus. Internal hemorrhoids create principle issues of bleeding, prolapse, and mucoid discharge.Bleeding may range from bright red blood that is seen as streaks on the toilet paper to actual dripping of it into the toilet bowl after a bowel movement. Over time, hemorrhoids potentially will develop into a prolapsed state achieve a point in time when patients will note a sense of fullness or discomfort along with mucoid drainage that results in further irritation and soiling of underclothes.

A 23-year-old woman presents with dysuria and left flank pain. She is 27 weeks pregnant. Physical examination reveals a temperature of 38°C (100.4°F); there is tenderness with percussion over the left costovertebral angle. Urinalysis shows 15 to 20 WBC's/hpf and 15 to 20 bacteria/hpf. What is the most appropriate management plan? Answer Choices 1 Schedule renal ultrasound 2 Order urine culture and arrange follow-up in 24 hours 3 Obtain IV pyelogram 4 Treat with oral trimethoprim/sulfamethoxazole 5 Admit for IV antibiotics

The patient should be admitted for IV antibiotics. Urinary tract infections in pregnant patients are associated with significant risks greater than in non-pregnant patients. Mechanical pressure on the ureters and bladder by the enlarging uterus, progesterone-mediated relaxation of smooth muscle, incomplete bladder emptying, and increased urinary tract volume contribute to the increased incidence of urinary tract infections during pregnancy as well as the increased severity of associated complications. Approximately 5% to 7% of pregnant women have asymptomatic bacteriuria. Untreated bacteriuria is associated with a high incidence of prematurity and fetal wastage as well as a 20% to 40% incidence of acute maternal pyelonephritis. Patients with uncomplicated bacteriuria should be treated with ampicillin, amoxicillin, nitrofurantoin, or a cephalosporin. However, pregnant patients with pyelonephritis (i.e., fever, chills, and flank tenderness) are at increased risk for sepsis and preterm labor, and they require hospitalization for aggressive therapy with parenteral antibiotics. Sulfonamide antimicrobial agents increase serum bilirubin levels, thereby increasing the risk of neonatal kernicterus; they should be avoided during the 3rd trimester.

A 42-year-old man presents following an episode of loss of consciousness. The patient was in excellent health until the evening of admission; his wife found him unconscious on the concrete deck surrounding their swimming pool. His swim trunks were fully wet and there was vomitus on the deck near him. He regained consciousness in the ambulance, at which time he was confused, and complained of severe generalized headache. Past medical history is positive for well-treated hypertension, 2 packs per day smoking history of 24 years duration, occasional social drinking, and a 2-month history of intermittent right ocular pain. Physical exam reveals a temperature of 98.5 F by mouth, blood pressure of 190/110 mm Hg, pulse rate of 116/min and regular, with respirations 22/min and unlabored. He is a well-developed, well-nourished, middle-aged man. He is awake, but drowsy, and he has a severe headache. There is mild nuchal rigidity. Fundi are clear. The rest of the neurological exam is normal. There are no signs of trauma. Labs reveal a white blood cell count of 11,500/mm3, with a normal differential. The chemistry profile is normal. Electrocardiogram reveals prolonged QRS complexes, prolonged QT interval, and deeply inverted and symmetric T waves. Unenhanced computed tomography of the head reveals a large collection of blood in the right Sylvian fissure and lesser amounts in the basal cisterns. Question The patient is identified as having an aneurysmal subarachnoid hemorrhage (SAH). What statement about this condition is most valid? Answer Choices 1 Early mortality after SAH is about 10% 2 Normal head CT rules out the diagnosis of SAH 3 The risk of rebleeding is highest in the first 6 to 24 hours after the initial bleed 4 He has an increased risk of developing right-sided hemiparesis 5 He has a worse prognosis because of his history of hypertension

The risk of rebleeding is highest in the first 6 to 24 hours after the initial bleed The patient has had a hemorrhagic stroke due to a ruptured saccular ("berry") aneurysm. Rebleeding is most common in the first 6 to 24 hours. Early mortality from this condition approaches 50%, and those who survive often suffer significant permanent deficits (4). Noncontrast head CT is the most common modality for diagnosis of SAH and is nearly 100% sensitive, especially in the first few hours after a bleed. However, when the bleed is more remote or more minor, the CT scan may be negative. In this case, a lumbar puncture should be performed and will reveal red blood cells (6). Hypertension is seen in almost all patients following a bleed; it tends to be mild to moderate and fluctuate with the degree of head pain (1). His history of hypertension does not worsen his prognosis; a history of preexisting hypertension is only slightly more likely in patients with ruptured saccular aneurysms as compared to matched controls. Hemiparesis certainly may occur, but it would likely occur on his left side based on the location of the intracranial bleed.

A woman brings in her exclusively breast-fed, 7-month-old male infant for a routine check up. You see a well-fed, African-American boy with pigeon breast, depression along the line of diaphragmatic insertion into rib cage, and costochondral thickening that looks like a string of beads. On an X-ray the bones are translucent and the skull bones are thinning. What is the most likely diagnosis? Answer Choices 1 Osteomalacia 2 Primary hyperparathyroidism 3 Osteitis deformans 4 Rickets 5 Silverman syndrome

These symptoms are classical for rickets. Pigeon breast = pectus carinatum (sternum protrusion), Harrison's groove = depression along the line of diaphragmatic insertion into rib cage, rachitic rosary = costochondral thickening, craniotabes = thinning of skull bones, and radiolucency of bones. This child has 2 risk factors for rickets. His dark skin pigmentation interferes with the penetration of ultraviolet light in the skin, and he is exclusively breast-fed. Breast milk of African American women contains only about 35 IU/l vitamin D, which is far below the 400 IU/d recommended for infants by the American Academy of Pediatrics. Osteomalacia would be the right diagnosis for an adult showing these symptoms. Primary hyperparathyroidism is caused by either tumor or hyperplasia of the parathyroid glands. In this case, it would be secondary hyperparathyroidism caused by hypocalcemia. Osteitis deformans or Paget's disease is an idiopathic increase of osteoblast and osteoclast activity, usually in elderly people. It presents with diffuse fractures and bone pain. Silverman syndrome is congenital pigeon breast often combined with heart abnormalities.

A 78-year-old male presents to the office due to increasing exertional dyspnea and cough for the past week. Physical exam reveals an S3 gallop, mild JVD, and 2+ pitting edema of the bilateral lower extremities. The patient has had mild congestive heart failure in the past. A chest x-ray reveals prominent pulmonary vasculature without any additional complications. The patient's current medications are aspirin and simvastatin daily. He has no drug allergies. Which of the following is the most appropriate initial management for this patient? A Angiotensin II receptor blocker B β blocker C Calcium channel blocker D Nitrate E Thiazide diuretic plus ACE inhibitor

Thiazide diuretic plus ACE-I ACE inhibitors are recommended, as first line treatment for symptomatic congestive heart failure, based upon clinical trials, reveal an approximately 20% reduction in CHF mortality in symptomatic heart failure patients. Diuretics provide CHF symptom improvement and promote water and sodium excretion to decrease intravascular volume. blockers and Angiotensin II receptor blockers are also beneficial for CHF patients. Nitrates are usually reserved for acute or decompensating patients. Calcium channel blockers may accelerate CHF progression and should be used with caution.

The picture below is a view of the inferior aspect of the penis and scrotum of a 2-month-old male infant. The mother brought the infant to a pediatric clinic because, during micturation, urine ran from the opening at the bottom of the midline groove instead of from the tip of the penis. Question What is the most likely diagnosis? Answer Choices 1 Epispadias 2 Cleft scrotum 3 Cryptorchism 4 Hypospadias 5 Bifid penis

This is an example of hypospadias. It occurs frequently (about 1/300 male infants) and is caused by inadequate midline fusion of the urogenital folds in male embryos; consequently, there is incomplete formation of the penile urethra. Hypospadias often results from inadequate production of androgens by the testes or inadequate receptors for the hormones. The external urethral orifice may be located anywhere along the ventral midline aspect of the glans penis, shaft of the penis, scrotum, or perineum. Epispadias is an abnormal urethral orifice on the dorsal side of the penis. Cleft scrotum is due to failure of fusion of the labialscrotal swellings. Cryptorchism is either unilateral or bilateral failure of descent of the testes into the scrotum. A bifid penis is often associated with bladder exstrophy.

A 32-year-old man presents with severe headaches. He reports that he had experienced 2 of these severe headaches about a month ago, and now has had 2 more, which have awoken him from sleep. The headaches lasted anywhere from 15 minutes to about an hour, then resolved entirely. His wife urged him to seek care for this one, as she was worried about a stroke. He complains of severe head pain, and his left eye appears "droopy," slightly swollen, and is watering. His nose is runny. He denies history of head trauma, memory loss, gait disturbances, muscle weakness, nausea, and vomiting. Over-the-counter acetaminophen and ibuprofen have not been helpful. He has no known medical conditions, takes no regular medications, and has no known allergies. He has never had surgery. He denies any family history of headaches or neurological conditions. He smokes 1 pack/day and drinks alcohol 3-4 times/week. He denies the use of drugs. On physical exam, the patient appears restless. Ptosis and pupillary constriction are observed in his left eye, and some clear nasal discharge is noted. The remainder of his exam is normal. Question What test results best support this patient's likely diagnosis? Answer Choices 1 Inflammation on temporal artery biopsy 2 Normal magnetic resonance imaging (MRI) 3 Polyspikes and slow wave pattern on electroencephalogram (EEG) 4 Positive toxicology for cocaine 5 Thickened sinus mucosa on computed tomography (CT)

This patient is most likely experiencing cluster headaches, in which he would be expected to have normal magnetic resonance imaging (MRI). Cluster headaches are intense, episodic headaches, which occur more commonly in men than women. Symptoms are unilateral, often accompanied by rhinnorhea and lacrimation, miosis, and ptosis of the eye. Inflammation on temporal artery biopsy indicates temporal arteritis, which can cause severe unilateral headache. Temporal arteritis is more likely to occur in an older person and is not associated with the rhinorrhea and these optic symptoms. (There may be amaurosis fugax, diplopia, and vision loss.) Polyspikes and slow wave pattern on electroencephalogram (EEG) indicates seizure activity. This patient's symptoms and headache pattern are not associated with any seizure disorder. If the patient had positive toxicology for cocaine, he could present with restlessness and mydriasis (dilation of the pupils - bilaterally), rather than unilateral pupillary constriction. Thickened sinus mucosa on computed tomography (CT) indicates chronic sinusitis, which can cause headaches. The headaches associated with chronic sinusitis are much less intense/acute and episodic than those of cluster headaches.

A 41-year-old woman presents due to worsening symptoms. She was diagnosed with idiopathic pulmonary hypertension about 2 years prior to presentation; she is on home oxygen therapy. She has longstanding fatigue and dyspnea, but she is now experiencing profound dyspnea with exertion; swelling in her ankles; some discomfort in her right, upper abdomen; and the inability to breathe well when lying down. She has always been thin, but her weight has increased by 10 pounds in the last month. She denies fever and chills. She recently had an electrocardiogram (ECG), but she has not seen a healthcare provider to discuss the results. The ECG report indicates peaked p waves, right axis deviation, and tall R wave in V1. Question What is the most appropriate intervention for her current condition? Answer Choices 1 Prescribe a calcium channel blocker, such as verapamil 2 Prescribe a diuretic, such as furosemide 3 Prescribe a fluoroquinolone, such as levofloxacin 4 Prescribe a lipase inhibitor, such as orlistat 5 Prescribe a thiazolidinedione, such as pioglitazone

This patient is presenting with a progression of a primary pulmonary disease (pulmonary hypertension) into cor pulmonale, which is also known as pulmonary heart disease. Cor pulmonale, when moderate to severe, will present with signs and symptoms of right heart failure, such as the severely fluid overloaded state. In addition to treatment of the underlying pulmonary disorder, cor pulmonale is treated much like right-sided heart failure is treat. It would be most appropriate to prescribe a diuretic, such as furosemide, at this time. If possible, this patient should also be referred to a cardiopulmonary specialist.

A 50-year-old man with a past medical history of hyperlipidemia presents with a recurrent "bitterness and sour sensation in his mouth" and a nonproductive cough; both have been occurring over the last year. There is associated progressive dysphagia and odynophagia to solid foods, but not to liquids; as a result, he has lost 15 pounds over the past 5 months. He admits to drinking 1 to 2 beers per week, but he denies alcohol use beyond this. He also denies cigarette smoking and illicit drug use. He denies fever, chills, sputum production, chest pain, palpitations, dyspnea, shortness of breath, wheezing, peripheral edema, abdominal pain, nausea, vomiting, diarrhea, melena, hematochezia, and changes in bowel habits. The physical exam reveals normal vital signs. Oropharyngeal, neck, cardiopulmonary, and abdominal exams are normal. Question What is the most appropriate initial intervention for this patient at this time? Answer Choices 1 Perform a chest X-ray 2 Upper endoscopic evaluation 3 Prescribe a trial of esomeprazole 4 Barium esophagography 5 Ambulatory esophageal pH monitoring

This patient's most concerning diagnosis is a malignant esophageal lesion. However, nonmalignant etiologies may also be responsible for this presentation. A barium esophagography is obtained as the first study to evaluate dysphagia. In patients with progressive esophageal dysphagia, a radiographic barium study is used to differentiate between mechanical lesions (such as peptic stricture and esophageal cancer) and motility disorders (such as achalasia). The sensitivity of barium radiography for detecting esophageal strictures is greater than that of endoscopy. Barium studies also provide an assessment of esophageal function and morphology that may be undetected on endoscopy. Hypopharyngeal pathology and disorders of the cricopharyngeal muscle are better appreciated on radiographic examination, particularly with videofluoroscopic recording. Either a positive or a negative study is usually followed by an endoscopic evaluation; it is done in order to clarify findings in the case of a positive examination or to add a level of certainty in the case of a negative one. A chest X-ray may reveal adenopathy, a widened mediastinum, metastatic lesions to the lungs or bone, or signs of tracheoesophageal fistula, such as pneumonia. It will not, however, reliably identify esophageal lesions. Endoscopy with biopsy establishes the diagnosis of esophageal carcinoma with high reliability. It is the study of choice for evaluating persistent heartburn, odynophagia, and structural abnormalities detected on barium esophagography. Ambulatory esophageal pH monitoring is the preferred study for documenting acid reflux. It is indicated in the following clinical scenarios: for patients being considered for anti-reflux surgery with a normal endoscopy; in patients with a normal endoscopy but with continued reflux symptoms following therapy with a proton pump inhibitor; and in the evaluation of atypical reflux symptoms that continues despite anti-reflux therapy. Patients with alarm symptoms, such as unexplained weight loss, odynophagia, jaundice, recurrent vomiting, blood loss, a palpable mass or lymph node, or a family history of GI malignancy, should have diagnostic testing performed promptly.

A 72-year-old man is evaluated at the bedside following hospital admission for a 1-year history of progressive dyspnea, weight loss, low-grade fevers, fatigue, and myalgias. His past medical history is remarkable for atrial fibrillation, for which he takes amiodarone, hypercholesterolemia, and recurrent urinary tract infections, for which his urologist prescribed nitrofurantoin on a chronic, prophylactic basis. He denies any cigarette use, history of murmurs or coronary artery disease, chest pain, wheezing, hemoptysis, syncope, abdominal pain, rashes, peripheral edema, diaphoresis, or vomiting. Bedside echocardiogram and electrocardiograms are unremarkable for abnormalities. A chest x-ray revealed peripheral reticular opacities at the lung bases and a generalized honeycombing pattern (refer to image). Question Which of these statements regarding the clinical presentation of this patient is correct? Answer Choices 1 Acute, severe shortness of breath is the most common presenting symptom. 2 Most patients are asymptomatic at the time of diagnosis. 3 The most common symptoms are fever and a productive cough. 4 Physical exam findings of pulmonary hypertension commonly occur. 5 The pulmonary exam routinely reveals generalized wheezing and rhonchi.

This patient's most likely diagnosis is idiopathic pulmonary fibrosis. It is critical to obtain a complete history, including medication history, social history, occupational history, exposure history, and review of systems, to ensure other causes of interstitial lung disease are excluded. Amiodarone, bleomycin, and nitrofurantoin are notable medications associated with pulmonary fibrosis. Most patients with idiopathic pulmonary fibrosis present with a gradual onset, often for longer than 6 months duration. Dyspnea, which is the most prominent symptom in idiopathic pulmonary fibrosis (IPF), usually begins insidiously and is often progressive. Approximately 5% of patients have no presenting symptoms when idiopathic pulmonary fibrosis is diagnosed. The clinical symptoms of idiopathic pulmonary fibrosis are nonspecific; symptoms often precede the diagnosis by a median of 1-2 years. Most patients present with exertional dyspnea and a nonproductive cough. Pulmonary hypertension is a common comorbidity in patients with idiopathic pulmonary fibrosis, and an estimated 20-40% of patients with idiopathic pulmonary fibrosis who are evaluated or listed for lung transplantation have pulmonary hypertension at rest. Patients may have a loud P2 component of the second heart sound, a fixed split S2, a holosystolic tricuspid regurgitation murmur, and pedal edema. As right ventricular hypertrophy ensues, a right ventricular heave may be palpated at the lower left sternal border and increased right atrial pressure may cause elevation of the jugular venous pressure. The chest radiograph lacks diagnostic specificity for idiopathic pulmonary fibrosis. In most patients with idiopathic pulmonary fibrosis, the physical examination reveals fine bibasilar inspiratory crackles (Velcro crackles). Additionally, digital clubbing is seen in 25-50% of patients with IPF.

A patient presents to the office at 5 weeks gestation. She has been spotting for several days, and the flow is increasing slightly. She has mild, crampy pain; no fetal heart tones are auscultated. Ultrasound reveals an intrauterine gestational sac with a fetal pole. What is the most likely diagnosis? A Threatened abortion B Spontaneous abortion C Incomplete abortion D Septic abortion

Threatened abortion A While she is at risk for a spontaneous abortion, one cannot determine fetal viability with one exam. Serial exams are necessary. Fetal heart tones are not always present at 5 weeks.

A 4-year-old boy presents with bloody diarrhea, anemia, thrombocytopenia, fever, and lethargy following the exposure to the homemade hamburgers. His urine output is decreasing. Question What is the acute mechanism of kidney failure in this child? Answer Choices 1 Consumptive coagulopathy 2 Autoimmune coaguloptahy 3 Thrombotic microangiopathy 4 Segmental glomerulosclerosis 5 Tubulointerstitial nephritis

Thrombotic microangiopathy Based on the symptoms in the case, the patient is diagnosed with hemolytic uremic syndrome (HUS). E coli serotype O157:H7, which causes more than 80% of infections leading to HUS, produces shiga-like toxin. Toxin enters the circulation and causes the endothelial injury with the formation of arteriolar and capillary thrombi and red cell fragmentation. Thrombi damage glomerular filtration system in the kidneys and cause kidney failure; therefore, the progressive renal failure in HUS is the result of microangiopathic non-immune hemolytic anemia, and thrombocytopenia and the main mechanism is endothelial injury. Consumptive coagulopathy,which is also knowndisseminated intravascular coagulation (DIC), is a result of a pathological activation of coagulation cascade that causes intravascular clot formation. Signs and symptoms depend on the organ involved, but the most common is bleeding due to the consumption of platelets and clot factors in the blood. Consumptive coagulopathy is rarely seen in HUS. Autoimmune mechanism can also cause coagulopathy with bleeding symptoms, but the formation of autoantibodies is not the characteristic of the hemolytic uremic syndrome. Focal segmental glomerulosclerosis is caused by a variety of factors (infections, inflammations, toxins, hemodynamic and genetics) that cause nephrotic syndrome. Proposed mechanisms are either primary alteration of epithelial cells (idiopathic, viral-associated, drug-induced, and genetic) or secondary to reduction in nephron mass or hemodynamic adaptation (obesity, a single kidney, renal dysplasia/agenesis, reflux nephropathy, sickle cell disease, hereditary nephropathies, and other primary glomerular diseases). It is not characteristic for the HUS. In acute interstitial nephritis, renal tubular cells dysfunction is caused primarily by a hypersensitivity reaction to drugs or by infection. In hemolytic uremic syndrome, the initial damage is in the vascular endothelium.

A 70-year-old woman presents with a 1-week history of palpitations, dyspnea, and generalized weakness. She also gives history of decreased oral intake and weight loss. The patient has no significant previous medical history. On admission, the patient is afebrile. Pulse is 130/min, BP is 100/68 mm Hg, and RR is14/min. Chest X-ray is normal, and electrocardiogram shows an atrial fibrillation. Skin appears warm and smooth. Question What is the next diagnostic step that would be useful in management? Answer Choices 1 Carcinoembryonic antigen (CEA) test 2 Gastroscopy 3 Colonoscopy 4 Thyroid tests 5 Adrenal functional tests

Thyroid Tests Explanation Other than sinus tachycardia occurring in patients with hyperthyroidism, atrial fibrillation is the most common cardiac arrhythmia. It occurs in 10 -15% of patients with hyperthyroidism and increases significantly in patients over 60 years of age. In the elderly, atrial fibrillation occurs in 25 - 30% of patients with hyperthyroidism. Thyrotoxic atrial fibrillation is associated with increased risk of thromboembolic complications involving the central nervous system. Therefore, it is imperative to perform thyroid function tests to rule out hyperthyroidism in all cases of new-onset atrial fibrillation. The mainstay of treatment in patients with atrial fibrillation and hyperthyroidism is restoration of euthyroid status by using antithyroid drugs. Restoration of the euthyroid status is often associated with conversion to sinus rhythm. However, the rate of conversion is slower in the elderly. Treatment options for atrial fibrillation are rate-control agents, oral anticoagulants, and defibrillation. Other tests, such as CEA antigen test, gastroscopy, colonoscopy, and adrenal function tests, would not be useful for ascertaining the cause of atrial fibrillation.

A mother expresses concern for her teen son after feeling a lump in his neck. He has no history of trauma to his neck. Surgical history is negative, and the patient does not take any medications. The mother tells you that thyroid problems run in the family. The patient has not been ill recently. Upon exam you feel a nontender, firm nodule on the right side of his thyroid with associated cervical lymphadenopathy. His serum TSH level is within the reference range. Radionuclide thyroid scanning demonstrates a "cold" nodule in the right side of the thyroid. What is the most appropriate next step in the work up of this patient? A MRI of the anterior pituitary B CT of the thyroid C MRI of the thyroid D Thyroid nodule fine needle aspiration E Emergent thyroidectomy

Thyroid nodule fine needle aspiration The correct choice is D, thyroid nodule fine needle aspiration. With the advent of fine needle aspiration, it has become much easier, safer, and more reliable to obtain a specimen for biopsy. This patient has several characteristics that increase his risk of malignancy including his gender, young age, firmness of the nodule, and related lymphadenopathy. These, along with the ease of biopsy, suggest this path for diagnostic work up. Not enough information is known to warrant an emergent thyroidectomy, choice E. Choice A, MRI of the anterior pituitary, would be warranted if there was a suspicion of a pituitary cause of the thyroid nodules. Since the TSH is normal and the patient is not presenting with headaches or other pituitary related symptoms, this is not suggested. Choice B, CT of the thyroid, and choice C, MRI of the thyroid, would not provide any additional information after the thyroid scan. They may be helpful prior to any surgery if needed.

A patient is being treated for hypothyroidism. His condition has been stable for the past year. What blood test should be ordered and monitored yearly in this patient? A Total T4 B T3 resin uptake C Thyroid releasing hormone D Thyroid stimulating hormone E Free T3

Thyroid stimulating hormone The correct choice is D, thyroid stimulating hormone. This test will help to monitor patient adherence with thyroid hormone supplementation, as well as to fine tune the dose so that the TSH remains within the reference range. Choices A, B, and E can be used in the work up of patients for primary hypothyroidism, but alone each test is not helpful to monitor chronic disease in patients. Choice C, serum thyroid releasing hormone, is used more commonly when investigating secondary hypothyroidism.

A 42-year-old teacher presents with a 1-year history of hand tremor. She tells you that it appears to be only in her right hand and comes and goes. It is especially embarrassing for her because she writes on the board in front of her students. She denies pain, motor weakness, and abnormal sensation in her right hand. She reports that her father's hand used to "shake" when he drank his coffee. Otherwise, she feels well, and she denies other symptoms. She has not noticed memory, incoordination, or balance problems. Her past medical history is unremarkable, with 3 uncomplicated pregnancies and deliveries; she has no chronic medical conditions. Except for a mild tremor on the patient's right hand when held extended, her physical exam is unremarkable. Her neurological exam is otherwise normal. Question What lab/diagnostic study would be most appropriate for this patient? Answer Choices 1 Computed tomography (CT) of the head 2 Electromyography (EMG) 3 Magnetic resonance imaging (MRI) of the head 4 Serum ceruloplasmin 5 Thyroid stimulating hormone (TSH)

Thyroid stimulating hormone (TSH) would be the most appropriate test at this time. This patient presents with a history and physical consistent with benign essential (or familial) tremor. A TSH would help rule out hyperthyroidism as the cause of this patient's tremor. It is non-invasive, readily accessible, and relatively inexpensive. Unless the history suggests a more serious cause of tremor, no further testing is recommended. A CT or MRI of the head would be recommended if the patient presented with more neurologic findings on exam or a history concerning for tumor growth, such as new headaches. Both tests are quite expensive, and they are unnecessary in cases of benign essential tremor. EMG is done in evaluation of multiple nerve and muscle disorders, such as carpal tunnel syndrome, muscular dystrophy, and myasthenia gravis. Nerve conduction can be measured on EMG. EMG is not recommended or routinely done in the evaluation of essential tremor. Serum ceruloplasmin is useful in the evaluation of the relatively rare Wilson's disease. The tremor of Wilson's disease is a 'wing-beating tremor' and several other findings, such as dysarthria, dystonia, splenomegaly, pallor due to anemia, and Kayser-Fleischer rings (brown to grey-green rings around the cornea), are often present. This patient's history does not suggest Wilson's disease.

A 35-year-old woman presents with weight loss, tremor, and palpitations. She also feels hot, is sweating, and has sleep difficulties and frequent arousals, in addition to frequent bowel movements (3-4 times a day), nervousness, and weakness. Physical examination reveals a slim anxious female with pronounced stare, fine postural and tremor in rest, and slight proximal weakness. Her thyroid is diffusely enlarged and non-tender; her pulse is 100/min; and the rest of examination is within normal limits. Her TSH is low, and free T4 is high. Question What is the most accurate diagnostic test for hyperthyroidism in this patient? Answer Choices 1 MRI of pituitary gland 2 Free T3 3 Fine needle biopsy 4 Color Doppler flow 5 Thyrotropin receptor antibody assay

Thyrotropin receptor antibody assay Explanation Your patient has signs and symptoms of Grave's disease. Grave's disease is an autoimmune disease caused by anti thyroid-stimulating hormone (TSH) receptor. Thyrotropin receptor antibody assay is not required for the diagnosis of Grave's disease, but it will confirm the diagnosis. Sometimes this test is needed to establish the cause of exophthalmos (e.g., in the absence of thyrotoxicosis) or to predict fetal or neonatal thyrotoxicosis. Among the choices offered, it is the most accurate test for Graves' disease. You can order an image of pituitary gland in secondary hyperthyroidism, when both TSH and T4 are high; that is not the case in your patient. You can order free T3 levels in the case of hyperthyroidism when TSH is low and T4 normal and you think that patient might have T3 thyrotoxicosis. Usually those patients will have mild disease, but, otherwise, the clinical picture may be similar to other forms of hyperthyroidism. Elevation of T3 alone can also produce symptoms of thyrotoxicosis, but there will be no exophthalmos. Fine needle biopsy is indicated to rule out or confirm cancer of the thyroid gland. Patients with thyroid cancer usually are euthyroid and have normal TSH and non-functional ("cold") nodule. Color Doppler flow exam may help when a possible nodule is detected on physical exam or may show hypoechogenicity to confirm the diagnosis of Hashimoto's thyroiditis or demonstrate intense vascularity in the case of Graves' disease. However, ultrasound of thyroid gland rarely has a value in the diagnosis of hyperthyroidism and is not specific.

A 60-year-old man presented with a mass in the left lobe of the thyroid. Fine needle aspiration was consistent with papillary carcinoma. There was no evidence of locally invasive or metastatic disease. Which of the following treatments is recommended for this patient? A Chemotherapy B External beam radiation C Preoperative radioiodine ablation D Total thyroidectomy

Total thyroidectomy Papillary carcinoma is the most common type of thyroid malignancy. Treatment includes a thyroid lobectomy and isthmusectomy or total thyroidectomy. The decision regarding the extent of the surgery is based on the extent of the disease, the tumor size, and histiologic grade. A poor prognosis is seen in males, patients older than 50 years of age, primary tumors greater than 4 cm in size, tumors that are less well differentiated, or evidence of locally invasive or metastatic disease. Accordingly, the recommended treatment for this patient is a total thyroidectomy. Radioiodine ablation is recommended postoperatively.

A 42-year-old woman comes in with a "lump in her neck." When she was a teenager, she underwent radiation treatment for "some kind of tumor" in her neck. Ultrasound reveals a 1.5 cm lesion in the left lobe of the thyroid; biopsy confirms papillary carcinoma. Thorough evaluation reveals no evidence of metastasis. Which of the following is the initial treatment of choice for her? A chemotherapy B lobectomy with isthmectomy C radiation therapy D radioactive iodine therapy E total thyroidectomy

Total thyroidectomy Total or near-total thyroidectomy is indicated for this woman. No chemotherapy (A) is available for thyroid cancer. Lobectomy with isthmectomy is reserved for papillary carcinoma that is less than 1 cm in size in persons with no history of radiation exposure and no evidence of metastasis. Radiation therapy (C) is used to treat bone metastasis and anaplastic carcinoma. Radioactive iodine therapy (D) may be used following thyroidectomy to ablate any remnant of the gland and to treat cancer that has metastasized or is otherwise high risk.

A 67-year-old man was treated for bacterial pneumonia and prescribed erythromycin for treatment. He presents a week later with severe diarrhea, lower abdominal cramping, fever, nausea, and dehydration. Stool examination and enzyme immunoassay of the patient's serum indicate infection by Clostridium difficile, which is a Gram-positive bacilli. The patient is prescribed metronidazole for the infection. Question What component of this infection is the cause of the symptoms in the patient? Answer Choices 1 Capsule 2 Cell wall 3 Flagella 4 Spores 5 Toxins

Toxins released by the organism are responsible for the patient's symptoms. The symptoms of a C. difficile infection are due to the release of toxins produced by the organism into the body of humans. C. difficile produces 2 types of toxins, A and B; they enter the intestinal mucosal cells via specific receptors on their cell wall and result in inflammation of the colon, mucosal damage, and fluid and mucous secretions. These changes cause diarrhea. Toxin A has enterotoxicity and toxin B has cytotoxicity. A strain of C. difficile, NAP 1, is found to produce another type of toxin known as binary toxin. Along with toxins, colonization of the organism is also found to be an essential factor in the onset of symptoms. The cell wall of C. difficile does not disrupt the intestinal mucosal cells. The cell wall of the organism helps in adhering to the mucosal cells at specific sites and in colonization of the organism at the site of adherence. The organisms have various proteins on their cell wall that function as adhesins and attach the organism at particular sites on the surface of mucosal cells. The proteins of the cell wall may help the organism in invading the cells. These proteins can be targeted when producing vaccines against C. difficile. Capsules are generally not produced by Clostridium species, but a capsule-like substance has been found around C. difficile cells. The capsules were found to be composed of glycocalyx and were detected by antibody stabilization methods. These structures protect the organism from phagocytosis during infection and help in colonizing on the mucosal layer of the intestine. A few strains of C. difficile are known to have flagella, which help the cells in motility. In this organism the flagella are also known to possess virulence factors, which result in production of antibodies against them in infected humans. The flagella are found to help the organisms in adhering to the cells of the intestinal mucosa. The flagella of C. difficile are also found to be involved in cross-reactions among serogroups. Spores produced by Gram-positive bacteria are resistant to adverse conditions like high temperature, antibiotics, disinfectants, etc. These spores can survive in an environment for months or years without germination. C. difficile is a major source of nosocomial infections pertaining to its spores. The spores enter the human body when inhaled or through the oral route. They then geminate in the small intestine, thereby producing viable cells of C. difficile and starting the cycle of infection.

A 54-year-old man presents after having a generalized seizure. The patient is HIV positive, but he has been unable to afford antiretroviral therapy since losing his job 2 years ago. Other than cachexia, the physical exam is unremarkable. Upon further inquiry, the patient also notes that he has become short-tempered and hypercritical; at times, he seems confused. An MRI of the brain is performed, and it reveals several cortical ring-enhancing lesions. Question What is the most likely diagnosis? Answer Choices 1 AIDS dementia complex 2 Cryptococcal meningitis 3 Cytomegalovirus encephalitis 4 Progressive multifocal leukoencephalopathy 5 Toxoplasma encephalitis

Toxoplasma encephalitis The patient's symptoms and MRI findings are most consistent with the diagnosis of toxoplasma encephalitis. Toxoplasmosis is the most common cerebral mass lesion among HIV-positive patients. Infection with the Toxoplasma gondii parasite is relatively common and usually asymptomatic. Reactivation occurs in HIV positive patients due to failing cellular immunity, and it causes a multifocal necrotizing encephalitis. Seizures may be the initial manifestation of central nervous system (CNS) infection; other common clinical manifestations include focal neurologic deficits, such as impaired speech and hemiparesis. Personality change, lethargy, headache, and confusion are also observed. The MRI in patients with toxoplasma encephalitis characteristically reveals multiple, ring-enhancing lesions with surrounding edema; these lesions usually occur bilaterally in the frontal and parietal cortices. AIDS dementia complex describes a constellation of cognitive symptoms seen among HIV positive patients. The condition occurs when HIV virus disseminates to the CNS. Within the CNS, the virus tends to concentrate in the basal ganglia and subcortical regions. Symptoms include a constellation of cognitive, behavioral, and motor disturbances that cause varying degrees of functional impairment. Characteristic MRI findings include non-enhancing white matter, cerebral atrophy, and ventricular enlargement. The diagnosis requires that other central nervous system infections, carcinoma, as well as general medical conditions and substance abuse have been excluded. Cryptococcal meningitis is caused by the encapsulated fungus Cryptococcus neoformans. Among HIV positive patients, the illness may be the result of new infection or reactivation of latent infection. Presenting signs are often nonspecific; they include headache, fever, change in mental status, and nausea or vomiting. Nuchal rigidity and photophobia may also be present, and elevated intracranial pressure is not uncommon. MRI findings vary, but they include lesions in the basal ganglia; meningeal enhancement, cerebral edema, and shrunken ventricles may also be seen. Cytomegalovirus (CMV) infection causing encephalitis is usually observed in patients with evidence of CMV infection at other sites. MRI findings vary, but they often show areas of focal necrosis within the brain parenchyma, meninges, or periventricular regions. Symptoms typically reflect progressive dementia, with episodes of confusion, apathy, and focal neurologic deficits. Progressive multifocal leukoencephalopathy is often a fatal disorder; it is caused by reactivation of a latent JC viral infection. Focal neurologic deficits such as hemiparesis and gait disturbance are often the initial presenting symptoms; they are followed by progressive cognitive decline, coma, and death. The MRI commonly reveals multiple, non-contrast enhancing foci in cerebral white matter.

A 3-month-old male infant presents with history of noisy breathing since birth; the noise is gradually increasing. There is no history of fever, cough, or running nose. Physical examination reveals a low-pitched, inspiratory wheeze; it is more prominent over the central airways and loudest over the trachea. Wheezing increases during crying, feeding, and when the infant is laid in supine position. There is no cyanosis, subcostal or intercostal retraction, or hoarseness of voice. Wheezing has not shown any response to bronchodilators. Question What is the most likely diagnosis? Answer Choices 1 Bronchiolitis 2 Congenital subglottic stenosis 3 Tracheomalacia 4 Tracheoesophageal fistula 5 Vocal cord paralysis

Tracheomalacia Tracheomalacia is a common cause of persistent wheezing in early infancy, with male preponderance in the ratio of 2:1. In primary tracheomalacia, there is insufficient cartilage to maintain the patency of the airway throughout the respiratory cycle. It commonly occurs in premature infants. Secondary tracheomalacia occurs when trachea is compressed by structures like vascular rings or deficient cartilage due to tracheoesophageal fistula. The dominant finding is a monophonic, low-pitched wheeze most prominent over central airways. There is persistent wheeze even in the absence of any viral respiratory infection, and it is loudest over the trachea. Subcostal retractions are absent unless there is asthma or some other cause of small airway obstructions. Wheeze does not respond to bronchodilators. Diagnosis is confirmed by flexible or rigid bronchoscopy. Bronchiolitis is usually caused by viral infection. More than 50% of cases are caused by respiratory syncytial virus (RSV). Other viruses include parainfluenza virus, adenovirus, and mycoplasma. It commonly occurs in male infants who have not been breast-fed. The infant first develops a mild upper respiratory tract infection with sneezing and rhinorrhea, which is accompanied by fever up to 101 - 102 degrees Fahrenheit. Respiratory distress, cough, and wheezing appear gradually; symptoms may interfere with feeding. Apnea is more common in infants less than 2 months of age. The infant is tachypneic with nasal flaring and chest retractions. Auscultation may reveal fine crackles and wheeze. A trial with bronchodilators, albuterol, or epinephrine may be considered and continued only if there is a clinical improvement with epinephrine used in an inpatient setting. Congenital subglottic stenosis is the 3rd most common congenital anomaly of the larynx causing stridor, which is usually biphasic or primarily inspiratory. Recurrent or persistent croup occurs in these infants. The 1st symptom often occurs during an episode of respiratory infection, as edema and thick secretions cause narrowing of the already compromised airway. Esophageal atresia is a frequent congenital anomaly affecting 1/4000 neonates. Of these, over 90% have associated tracheoesophageal fistula. It typically manifests with frothing and bubbling at the mouth and nose and episodes of coughing, respiratory distress, choking, and cyanosis. These symptoms are exacerbated by feeding. Regurgitation of gastric contents through the distal fistula causes more damaging pneumonia than aspiration of pharyngeal secretions from the blind upper pouch of esophagus. Vocal cord paralysis is the 2nd most common cause of neonatal stridor. It is often associated with central nervous system lesions like meningomyelocele, Arnold-Chiari malformation, and hydrocephalus. Bilateral vocal cord paralysis typically produces airway obstruction manifested by high-pitched inspiratory stridor. Unilateral paralysis causes aspiration, coughing, and choking. Cry is weak. Other symptoms of airway obstruction are less common. Diagnosis of vocal cord paralysis is made by awake flexible laryngoscopy with ultrasonography, which is a useful adjunctive examination.

A 19-year-old woman gives birth to an apparently healthy 8-pound female infant. During her first attempt at feeding, choking, coughing, and cyanosis occur. Historically, the mother had polyhydramnios during her pregnancy. What is the most likely explanation for the newborn's distress? Answer Choices 1 Choanal atresia 2 Respiratory distress syndrome 3 Polycystic kidney disease 4 Structural heart disease, cyanotic 5 Tracheoesophageal fistula

Transesophageal fistula The clinical picture is suggestive of tracheoesophageal fistula. Commonly, the infant presents with choking, cyanosis, copious secretions, and respiratory distress. Polyhydramnios is common. Symptoms worsen during feedings; stomach distention and pneumonia are possible complications. Continuous esophageal suctioning should persist until definitive repair. Diagnosis is made by placing a NG tube to where resistance is met. An X-ray is taken, and the tube should be visible in a blind pouch. Choanal atresia is a disorder where the back of the nasal passage is blocked. It is defined as bilateral or unilateral. Bilateral is a life-threatening condition that results in severe respiratory distress at birth; it requires immediate placement of an oral airway. Unilateral choanal atresia usually presents later and is confused with chronic sinusitis. Respiratory distress syndrome infants present with tachypnea (respiratory rate > 60 breaths/minute), cyanosis on room air, expiratory grunting, and intercostal/sternal retractions. Infants with polycystic kidney disease are usually diagnosed by prenatal ultrasound. It is most severe when the cystic kidneys are nonfunctional in utero. Hypertension is a common finding. Renal masses may be felt upon palpation of the abdomen. Structural heart disease with cyanotic presentation typically presents with cyanosis, without associated respiratory distress and failure to increase PaO2 with supplemental oxygen.

A 22-year-old patient with sickle cell disease presents to the emergency department complaining of chest pain, fever, and non-productive cough. On physical exam his temperature is 100.6˚F, BP is 144/88, pulse is 110, respiratory rate is 24, and pulse oximetry is 84%. CBC shows a WBC of 11,500, hemoglobin of 8.3%, and hematocrit of 28%. What is the most critical/emergent treatment for this patient? A Acetaminophen B Bone marrow transplant C Furosemide D Hydroxyurea E Transfusion

Transfusion Patients with sickle cell disease are prone to acute chest syndrome resulting from sickling of cells within the lung that typically presents with chest pain, tachypnea, cough, fever, and oxygen desaturation. Transfusion is indicated when the hematocrit drops below 30% or oxygen saturation drops below 90%. Bone marrow transplant is only an option in children (B) and not indicated for emergent therapy. Furosemide (C) would promote dehydration and further exacerbation of cell sickling. Hydroxyurea (D) is used prophylactically to reduce the frequency of sickle cell crises and/or acute chest syndrome. Acetaminophen (A) may be used for symptomatic treatment, but is less critical than transfusion and may not provide adequate pain management.

A 22-year-old woman presents with an 8-month history of amenorrhea. On further questioning, she admits to backaches, headaches, and acne. Physical examination reveals a female patient with a moon-shaped facies, multiple purple striae, and significant central obesity (body mass index of 36). Question Based on the history and physical exam findings, what is the most appropriate initial clinical intervention? Answer Choices 1 Radiation 2 Chemotherapy 3 Oral prednisone 4 Transsphenoidal selective resection 5 Lumpectomy

Transsphenoidal selective resection Explanation This patient likely is suffering from Cushing's syndrome, which is also known as hypercortisolism. Consequences of excessive levels of circulating cortisol, no matter the etiology, will lead to certain signs and symptoms, such as central obesity, thin extremities, a moon face, a buffalo hump, supraclavicular fat pads, a protuberant abdomen, oligomenorrhea, amenorrhea, and/or (in men) possibly erectile dysfunction. Backaches, headaches, hypertension, acne, purple striae, and impaired wound healing may also be found in these patients. The most commonly used clinical intervention is transsphenoidal selective resection of the adenoma. An adenoma of the pituitary gland is one of the more common etiologies of Cushing's syndrome. Pituitary corticotrophs remain suppressed after this surgery despite the pituitary function returning to normal; for this reason, patients require 6 - 36 months of oral prednisone therapy after surgery. Radiation, chemotherapy,and lumpectomy are not indicated treatment plans when dealing with the type of tumor that leads to Cushing's syndrome.

An 18-year-old woman comes in for evaluation of a bad smelling vaginal discharge. She says that it is so heavy that she must wear a pad to absorb it and that it is "yellowy green and bubbly." She also has a great deal of itching of vulva. Evaluation confirms the presence of a discharge and a vaginal pH of 5.5. Her vagina and cervix are diffusely erythematous with scattered petechiae. What is the most likely diagnosis? A Atrophic vaginitis B Bacterial vaginosis C Candidiasis D Gonorrhea E Trichomoniasis

Trichomoniasis E Trichomoniasis vaginitis is the most common STD in the United States that is not caused by a virus. It causes a profuse and frothy discharge that is greenish and can be foul smelling. Pruritus may occur. The vaginal pH is usually more than 5.0 and the vagina and cervix are red and may demonstrate small petechiae. Atrophic vaginitis (A) occurs primarily in postmenopausal women and is characterized by a thin watery or serosanguinous discharge, while bacterial vaginosis (B) has a greyish homogeneous discharge with a characteristic "fishy" odor released when potassium hydroxide is mixed with vaginal secretions. The discharge of candidiasis (C) is typically white and clumpy and is accompanied by intense itching in the vulva. More than three quarters of women who have gonorrhea (D) have no symptoms, but they may have a mucopurulent discharge.

A female patient presents with a history of hypertension and low plasma HDL. She is asking you if she has metabolic syndrome. She does not have diabetes and she is not obese. Which of the following NCEP ATP III criteria would be needed to confirm that diagnosis? A LDL > 70 mg/dL B LDL > 90 mg/dL C Triglycerides > 100 mg/dL D Triglycerides ≥ 150 mg/dL E Triglycerides > 300 mg/dL

Triglycerides ≥ 150 mg/dL The correct choice is D, triglycerides ≥ 150 mg/dL. The NCEP ATP III criteria for metabolic syndrome include three or more of the following: Central obesity with waist circumference > 102 cm in men and > 88 cm in women Hypertriglyceridemia with serum triglycerides ≥ 150 mg/dL, or the patient is taking medication for hypertriglyceridemia Low HDL level < 40 mg/dL in men and < 50 mg/dL in women, or the patient is taking medication for low HDL Hypertension with blood pressure ≥ 130 mm systolic or ≥ 85 mm diastolic, or the patient is taking medication for high blood pressure Fasting plasma glucose ≥ 100 mg/dL, or a positive diagnosis for diabetes mellitus, or the patient is taking medication for hyperglycemia

A 30-year-old G1P0 woman who is 15 weeks pregnant undergoes "triple screening." The maternal alphafetoprotein (AFP), human chorionic gonadotropin (hCG), and unconjugated estriol (uE3) levels are all lower than normal. This suggests which of the following possible problems with the fetus? A cystic fibrosis B Down syndrome C homocystinuria D G6PD deficiency E trisomy 18

Trisomy 18 E The triple screen detects possible Down syndrome and trisomy 18. When the fetus has Down syndrome (B), the AFP and uE3 are low and the hCG is high, while trisomy 18 is suggested by low values in all three. A positive screen must be followed up by fetal karyotyping. Cystic fibrosis (A), homocystinuria (C), and G6PD deficiency (D) are not detected by the triple screen.

A 3-year-old girl presents with a persistent cough. A few weeks ago, she had a low-grade fever with rhinorrhea and cough for several days. The rhinorrhea and fever resolved, but the cough has continued. It bothers her primarily during the night and when she wakes in the morning. Her medical history is significant only for mild seasonal allergies that appeared right after she turned 3. Her vaccines are current, and her parents deny recent travel. She is normally a thriving, happy child who attends preschool 3 days a week. For the past few weeks, she has been tired and fussy during the day because the cough keeps her up at night. On exam, she appears well. She is in no respiratory distress, and her lungs are clear to auscultation. Question What intervention will most likely cut down on the coughing and improve the girl's sleep? Answer Choices 1 Give diphenhydramine before bed 2 Encourage extra fluids and humidifier for the room 3 Discourage milk consumption late in the day 4 Give Robitussin as needed 5 Try an MDI with bronchodilator

Try an MDI with Bronchodilator This girl's symptoms and history of allergic rhinitis are consistent with cough-variant asthma. Respiratory symptoms are characteristically worse at night and often triggered by respiratory infections. A short-acting bronchodilator before bed and in the morning may help ease the cough and allow her to sleep better. Her primary cold symptoms have resolved, so diphenhydramine, Robitussin, and cutting down on milk are unnecessary. They are ineffective in treating a cough caused by asthma. Fluids and humidification are always a good recommendation, but in this case they will not directly cut down on the cough as much as a bronchodilator.

A 16-year-old sexually active girl is seen for a 2-month history of amenorrhea. She denies having unprotected sex, but always relies upon her partner to use a condom. She has vomited in the early morning twice in the past week. She has also had vaginal spotting for 3 days, accompanied by cramping lower abdominal pain that became sharp. Onset of menses was at 12 years, with normal, regular periods since then. There is no history of sexually transmitted disease. Physical examination revealed normal vital signs. Slight right and left lower quadrant abdominal tenderness without guarding and rebound was present. The cervix was closed. No blood was seen in the vaginal vault. The uterus was not palpable. Serum β-HCG: 5,200 mIU/ml. Vaginal spotting has increased, and abdominal pain has become more frequent. Repeat examination 3 days after the initial visit is unchanged. The uterus is still not palpable. Repeat serum β-HCG is 6,800 mIU/ml. Transvaginal ultrasound failed to reveal an intrauterine pregnancy or gestational sac. Question What is the most likely diagnosis? Answer Choices 1 Cervical ectopic pregnancy 2 Choriocarcinoma 3 Pseudocyesis 4 Tubal ectopic pregnancy 5 Very early intrauterine gestation

Tubal ectopic pregnancy Explanation The absence of fetal pole or gestational sac in the uterus with elevated serum β-HCG indicates an ectopic pregnancy, of which a tubal ectopic is the most common. The full triad of abdominal/pelvic pain, amenorrhea and irregular vaginal bleeding is present in only one-half of patients. Abdominal/pelvic pain may be unilateral or bilateral, usually worse on the affected side. Cervical ectopic pregnancies are rare. Serum β-HCG is not adequately elevated to suggest choriocarcinoma. Serum β-HCG is not elevated in pseudocyesis (false pregnancy), a condition in which the female patient truly believes she is pregnant and may exhibit signs and symptoms of true pregnancy such as amenorrhea, hyperemesis, breast swelling and tenderness, weight gain, abdominal tenderness, and even quickening. Transvaginal ultrasound failed to reveal even a fetal pole or gestational sac in the uterus, which should be detected by 36 to 40 days.

What exposure is linked to Tularemia

Tularemia is associated with contact with rabbits, other rodents, and biting arthropods.

A 19-year-old female patient presents to her family practice office for her annual Pap test and her first dose of the quadravalent human papillomavirus (HPV) vaccine. As her physician assistant, when would you schedule her to come in for her second dose of the HPV vaccine? A two weeks B two months C three months D four months E six months

Two months B People are frequently confused by differences between the quadravalent and bivalent versions of this vaccine. But dosing schedules are not one of the confusing issues. The Centers for Disease Control and Prevention (CDC), Advisory Committee on Immunization Practices (ACIP), and the manufacturers of both the HPV4 and the HPV2 vaccines all agree: The dosing and administration schedules are the same for HPV4 and HPV2. Each dose is 0.5 mL, administered intramuscularly, preferably in a deltoid muscle. The vaccines are administered in a three-dose schedule. The second dose is administered one to two months after the first dose, and the third dose is administered six months after the first dose.

Consideration should be given to screening patients with type 1 diabetes mellitus should also be screened for which of the following: A sarcoidosis B Sheehan's syndrome C Sjögren's Syndrome D thyroid disease

Type 1 diabetes mellitus (T1DM) is an autoimmune disease. As such, patients have a significantly higher risk of other autoimmune diseases, including celiac and thyroid disease. Most recommendations include screening for both diseases in patients with T1DM.

A 30-year-old woman presents to the office with polyuria, fatigue, and a chronic white vaginal discharge with vaginal pruritis. She has been having the discharge off and on for the past 6 months with recurrent treatment failures. Which of the following is the most likely diagnosis? A type 2 diabetes mellitus B hyperthyroidism C hypothyroidism D diabetes insipidus

Type 2 DM Polyuria, polydipsia, and fatigue are all findings that can be consistent with both type 1 and type 2 diabetes. Any woman who presents with a chronic vaginal discharge or chronic vaginal pruritis should be screened for type 2 diabetes.

A 25-year-old woman is seen today in your office for vague abdominal pain, nausea, anorexia, weight loss, anxiety, and dizziness. Her past medical history is significant for type 1 diabetes mellitus, and her family history is significant for hypothyroidism in several family members. A review of systems reveal a history of amenorrhea. Upon exam, you note hyperpigmentation of her skin and areas of vitiligo, but no mucocutaneous candidiasis. You are not surprised to find that her serum ACTH level is elevated and her serum cotisol is low. Which of the following syndromes should be investigated in this patient? A Type 1 polyglandular autoimmune syndrome type 1 B Type 2 polyglandular autoimmune syndrome type 2 C Multiple endocrine neoplasia type 2A D Multiple endocrine neoplasia type 1 E Metabolic syndrome

Type 2 polyglandular autoimmune syndrome type 2 The correct choice is B, type 2 polyglandular autoimmune syndrome. This patient is presenting with signs and symptoms of adrenal insufficiency including abdominal pain, nausea, anorexia, vomiting, weight loss, anxiety, and hyperpigmentation. Type 2 polyglandular autoimmune syndrome presents most commonly in young women between 20 and 40 years old, with evidence of adrenal insufficiency, type 1 diabetes mellitus, and autoimmune thyroid disease. Because of the strong family history of thyroid disease, it would be prudent to conduct an investigation into this disorder, starting with a serum TSH level. Primary ovarian failure and vitiligo may be symptoms of the autoimmune polyglandular syndrome as well. Choice A, type 1 autoimmune polyglandular syndrome, presents more commonly in childhood with adrenal insufficiency, hypoparathyroidism, and mucocutaneous candidiasis. Choices C and D are inherited disorders, characterized by the development of several different types of endocrine organ neoplasias. Choice E, metabolic syndrome, includes a constellation of several metabolic disorders that increase the risk of cardiovascular disease and diabetes mellitus in the patient.

A 25-year-old woman is seen today in your office for vague abdominal pain, nausea, anorexia, weight loss, anxiety, and dizziness. Her past medical history is significant for type 1 diabetes mellitus, and her family history is significant for hypothyroidism in several family members. A review of systems reveal a history of amenorrhea. Upon exam, you note hyperpigmentation of her skin and areas of vitiligo, but no mucocutaneous candidiasis. You are not surprised to find that her serum ACTH level is elevated and her serum cotisol is low. Which of the following syndromes should be investigated in this patient? A Type 1 polyglandular autoimmune syndrome type 1 B Type 2 polyglandular autoimmune syndrome type 2 C Multiple endocrine neoplasia type 2A D Multiple endocrine neoplasia type 1 E Metabolic syndrome

Type 2 polyglandular autoimmune syndrome type 2 The correct choice is B, type 2 polyglandular autoimmune syndrome. This patient is presenting with signs and symptoms of adrenal insufficiency including abdominal pain, nausea, anorexia, vomiting, weight loss, anxiety, and hyperpigmentation. Type 2 polyglandular autoimmune syndrome presents most commonly in young women between 20 and 40 years old, with evidence of adrenal insufficiency, type 1 diabetes mellitus, and autoimmune thyroid disease. Because of the strong family history of thyroid disease, it would be prudent to conduct an investigation into this disorder, starting with a serum TSH level. Primary ovarian failure and vitiligo may be symptoms of the autoimmune polyglandular syndrome as well. Choice A, type 1 autoimmune polyglandular syndrome, presents more commonly in childhood with adrenal insufficiency, hypoparathyroidism, and mucocutaneous candidiasis. Choices C and D are inherited disorders, characterized by the development of several different types of endocrine organ neoplasias. Choice E, metabolic syndrome, includes a constellation of several metabolic disorders that increase the risk of cardiovascular disease and diabetes mellitus in the patient.

A 28-year-old woman presents with bloody diarrhea for 1 week. After further questioning, she reveals that she had similar attacks in the past that subsided on their own. Her stool specimen is negative for ova and parasites. Stool culture did not grow any pathogens. A sigmoidoscopy is performed. There is friable erythematous mucosa extending from the rectum to the mid-descending colon, with broad-based ulcers in the descending colon. A biopsy taken from the rectum shows diffuse mononuclear inflammatory infiltrates in the lamina propria with crypt abscesses. However, no granulomas are seen. What is the most likely diagnosis in her case? Answer Choices 1 Ulcerative colitis 2 Crohn's disease 3 Ischemic colitis 4 Shigellosis 5 Amebic colitis

Ulcerative Colitis Ulcerative colitis is an ulcera-inflammatory disease limited to the colon and affecting only the mucosa and submucosa, except in the most severe cases. Ulcerative colitis extends in a continuous fashion proximally from the rectum in contrast to the skip lesions in Crohn's disease. Histologically, there is mononuclear inflammatory cell infiltration of the lamina propria collection of neutrophils in the crypts, called crypt abscesses and epithelial cell necrosis. However, there are no well-formed granulomas. Ischemic bowel disease applies to the structural changes in the colon that occur due to deprivation of blood supply .The causes for deprivation of blood could be arterial occlusion, venous occlusion and non-occlusive ischemia. The causes of non-occlusive ischemia include cardiac failure, shock, dehydration, and vasoconstrictive drugs. Depending on the severity and the presentation, it is classified into transmural, mural and mucosal infarction, and chronic ischemic colitis. The diagnosis is based on the natural history of the disease and endoscopic findings with supportive biopsy. Crohn's disease is a chronic, relapsing inflammatory disorder of obscure origin. It is a granulomatous disease and most often involves the small intestine and colon. It is characterized by transmural inflammation affecting all layers, presence of non-caseating granulomas, fissuring with formation of fistulas, and skip lesions. Shigellosis is a bacterial enterocolitis caused by shigella. It primarily involves distal colon with acute mucosal inflammation and erosion with purulent exudates. Stool culture is diagnostic. Amebic colitis is caused by Entamoeba histolytica, a protozoan parasite that spreads by faeco-oral route. They form flask shaped ulcers with a narrow neck and broad base. A stool examination for ova and cyst shows trophozoites and cysts of ameba. These parasites penetrate portal vessels, embolize to the liver to produce solitary discrete hepatic abscess, some exceeding 10 cm.

A 60-year-old man presents with a 2-week history of nausea and several episodes of epigastric pain. The pain is characteristically dull and lasts for several hours. His symptoms are not worsened by any particular food or activity. What is the most appropriate initial test? Answer Choices 1 Upper endoscopy 2 Gastrointestinal series 3 CT Scan of the abdomen 4 Ultrasonography of the upper abdomen 5 MRI of the abdomen

Ultrasonography of the upper abdomen Ultrasonography (USG) of the upper abdomen is the 1st choice in the investigation of acute or subacute abdominal pain. A common cause of acute abdominal pain in the elderly is cholecystitis, which can be investigated through USG. Changes occurring in the biliary system because of aging make older patients susceptible to cholecystitis, the most common indication for surgery in this population. Differential diagnosis would include diverticulitis (the most common cause of severe abdominal pain in patients over the age of 50), as well as bowel obstruction, pancreatitis, and peptic ulcer disease. Catastrophic conditions (e.g., abdominal aortic aneurysm rupture and mesenteric ischemia) must also be considered. This patient's symptoms are not very intense (his symptoms have been present for 2 weeks) and, therefore, do not necessitate urgent invasive investigations. If ultrasonography of the upper abdomen shows no significant findings, then a number of other tests can be performed (e.g., an upper endoscopy to rule out peptic ulcer disease, a gastrointestinal series to rule out bowel obstruction, and a CT scan or MRI of the abdomen to rule out pancreatitis or diverticulitis); any other undetermined pathology may be undertaken to exclude other possibilities.

A 60-year-old man with a past medical history of hypertension, hyperlipidemia, and prostate cancer is being evaluated in the emergency room. He has complained of new-onset acute severe left-sided pleuritic chest pain over the course of the last 2 hours. The pain is associated with feelings of anxiety, hemoptysis, shortness of breath, and nausea. He "feels warm" but denies chills, palpitations, wheezing, cough, edema, vomiting, abdominal pain, abnormal bowel habits, or dietary intolerances. He endorses a 40 pack-year smoking history, but denies drug or alcohol use. Upon physical exam, he is found to be febrile, hypotensive, diaphoretic, and in acute painful distress. There is perioral cyanosis and a pleural friction rub to the left lung fields. There is no peripheral edema, and the remainder of the exam is normal. A bedside electrocardiogram revealed the attached image. S1Q3T3 Question What is the most appropriate initial intervention for this patient? Answer Choices 1 Surgical embolectomy 2 Oral warfarin 3 Unfractionated heparin 4 Recombinant tissue plasminogen activator 5 Inferior vena caval filter placement

Unfractioned Heparin Unfractionated heparin is the standard initial treatment for isolated, unconfirmed PE. The FDA has not approved any low molecular weight heparin (LMWH) for primary symptomatic PE without associated DVT. Patients at high risk for an adverse outcome with anticoagulation alone should be considered for embolectomy if they have contraindications to thrombolytic therapy. Clinical success has been reported, but randomized trials are lacking. This vitamin K antagonist prevents carboxylation activation of coagulation factors II, VII, IX, and X. The full effect of warfarin requires at least 5 days, even if the prothrombin time, used for monitoring, becomes elevated more rapidly. If warfarin is initiated as monotherapy during an acute thrombotic illness, a paradoxical exacerbation of hypercoagulability can increase the likelihood of thrombosis rather than prevent it. Pulmonary embolism thrombolysis with Tissue plasminogen activator (tPA) remains controversial because large clinical trials using survival as an end point have not been conducted. Tissue plasminogen activator (tPA) may be considered in radiographically-confirmed pulmonary embolism. There are 2 indications for placing an inferior vena caval filter for patients with acute PE: (1) concomitant major bleeding, requiring transfusion or any intracranial hemorrhage that precludes the use of anticoagulant therapy, or (2) recurrent PE despite prolonged intensive anticoagulation.

A 37-year-old man with an unremarkable past medical history presents during a cold winter's day with a 10-day history of acute onset of productive cough with a moderate amount of yellow sputum. There is associated fever, shortness of breath, and malaise. He denies recent travel, sick contacts, occupational exposure, and a history of smoking or alcohol use. He denies arthralgias, chills, wheezing, abdominal pain, nausea, vomiting, diarrhea, edema, or rashes. His physical exam is remarkable for fever, tachypnea, reduced fremitus, dullness to percussion, and basilar crackles in the right lower lung field. Question What additional presentation finding would be most likely in this patient? Answer Choices 1 Chest pain precipitated by exertion and an S4 gallop 2 Unilateral, sharp inspiratory chest pain and thoracic friction rub 3 Chest pain relieved by antacids and provoked by food intake 4 A vesicular rash on the thorax preceded by paresthesias 5 Hemoptysis, tachypnea, unilateral lower extremity edema

Unilateral, sharp inspiratory chest pain and thoracic friction rub This patient's diagnosis represents a pleural effusion, a complication of bacterial pneumonia; an exudative effusion is most likely. Common findings of a pleural effusion include unilateral, sharp, or knife-like chest pain that is provoked by breathing, coughing, or movements of the trunk. Physical exam findings include dullness to percussion, a deviated trachea away from the side in large effusions, decreased to absent breath sounds, a pleural friction rub, and reduced tactile fremitus. Chest pain precipitated by exertion and an S4 gallop is consistent with angina. Chest pain relieved by antacids and provoked by food intake is likely representative of reflux esophagitis or gallbladder disease; a vesicular rash on the thorax in a dermatomal distribution, often preceded by paresthesias, is consistent with herpes zoster. Hemoptysis, tachypnea, and unilateral lower extremity edema should raise suspicion of a pulmonary embolus.

A 10-year-old boy presents with a 2-month history of intermittant burning pain in the epigastrium. Pain is felt more during the night and between meals and is partly relieved by eating food or by taking antacids. Pain usually lasts for about ½ to 1 hour and is accompanied by nausea and vomiting. The patient often has a feeling of bloating and burping. The child remains asymptomatic for several days in between. There is no history of taking analgesics or anti-inflammatory drugs. Physical examination shows epigastric tenderness. The rest of the examination is essentially normal. Stool examination for occult blood is positive. Question What is the investigation of choice for establishing the diagnosis? Answer Choices 1 Ultrasound abdomen 2 Upper GI barium studies 3 CT Scan abdomen 4 Upper GI endoscopy 5 Stool microscopy

Upper GI endoscopy The patient's history, clinical examination, and stool testing positive for occult blood are suggestive of peptic ulcer disease. Upper GI endoscopy is the investigation of choice in children with suspected peptic ulcer disease. Endoscopy allows direct visualization of the esophagus, stomach, and duodenum, identifies the specific lesions, and also screens for the presence of H. pylori in the biopsy specimens. H. pylori is a gram negative S-shaped rod. Biopsy should always be obtained from the antrum of the stomach regardless of endoscopic findings. Endoscopic findings may vary from grossly normal mucosa to nonspecific gastritis with prominent rugal folds to nodularity and ulcers. Stool microscopy has no role in the diagnosis of H. pylori infection. However, stool antigen testing identifies active H. pylori infection by detecting the presence of H. pylori antigen in the stool. Single contrast barium studies have overall sensitivity of 75%, but double contrast studies have sensitivity of 95% in detection of gastric ulcer. These results are comparable to endoscopy. However, barium studies have the disadvantage that the biopsy specimen cannot be obtained and ulcers smaller than 5mm may not be detected on barium studies. CT san has no primary role in the detection of gastric ulcers. It is helpful in the detection of subphrenic collection that may occur after perforation of gastric ulcer. Ultrasound abdomen has no role in the detection of gastric ulcer but is useful in the detection of other causes of upper abdominal pain like gallstones, pancreatitis, and subphrenic, as well as other collections due to perforated gastric ulcer.

A 70-year-old man presents with a 3-month history of urinary leaking. He notes that he suddenly feels the need to urinate. He notes symptoms when at rest and when asleep; he has not had any symptoms when coughing or bearing down. He has not experienced urinary dribbling, dysuria, foul smelling urine, or fever. He has daily bowel movements. He has no other known medical problems, and he takes no medications. His temperature is 98 degrees Fahrenheit. His bladder is not distended, and he walks without difficulty. His post void residual is 30 ml. His rectal examination is normal, and his urinalysis is unremarkable. Question What is the most likely diagnosis? Answer Choices 1 Stress incontinence 2 Urge incontinence 3 Overflow incontinence 4 Mixed incontinence 5 Fecal impaction

Urge Incontinence Explanation This patient most likely has urge incontinence. In urge incontinence, patients typically have involuntary leaks, increased urinary frequency, and nocturnal incontinence either during or just after the sensation of needing to void. Symptoms are not exacerbated by increased abdominal pressure or the stress of coughing/sneezing. Bladder detrusor muscles may be overactive, leading to the unexpected release of urine. Treatments include scheduled voiding and anticholinergic medications (oxybutynin, etc.). Stress incontinence is characterized by the involuntary leaking of urine during stress or increases in abdominal and bladder pressure, such as coughing and sneezing. Bladder pressure at these times exceeds urethral pressure, allowing urine to leak through the urethra. Urinary tract deficits are found commonly in older patients; both men and women have decreased bladder sensation, decreased contractility, and involuntary bladder contractions, which predispose them to incontinence. Obesity, pregnancy, and vaginal births may increase the risk for stress incontinence. In such cases, the pelvic floor muscles may be insufficiently strong to support the urethra and overcome pressure of urine flowing from the bladder. Mixed incontinence refers to the presence of symptoms of both stress and urge incontinence. It may be seen in 1/3 of patients. Overflow incontinence refers to urinary leaks that occur due to an obstruction of urine flow. In the absence of urinary retention, post void residuals are typically elevated; normal post-void residuals in the absence of retention are less than 200 ml. His 30 ml post-void urine volume is not consistent with urinary retention. Initially, patients with urinary retention may experience dribbling after voids, straining, the sensation of a full bladder, and a constant urge to void. Prostatic hypertrophy, atonic bladders, etc. can impede urine flow. Once urine volume exceeds bladder capacity, it may spill out, causing a leak. Overflow incontinence may be distinguished from urge incontinence by urodynamic testing, and it may be treated with terazosin and finasteride. Many medical problems and medications can contribute to incontinence. Delirium, restricted mobility, urinary infection, fecal impaction, polyuria, and medications that decrease urethral pressure (e.g., alpha blockers, neuroleptics, and benzodiazepines) or increased bladder pressure (e.g. anticholinergics, beta blockers, anti-Parkinson's medications, and bethanechol) may contribute to incontinence. The normal rectal examination and the history of regular bowel movements make fecal impaction a less likely diagnosis in this case.

A 6-year-old boy presents with his mother, who is concerned because he wets his bed 2 or 3 times a week at night. He has no urinary symptoms during the daytime. He is the second child of 3. He is in the 1st grade and is struggling with his performance. He has had no medical problems; there is no history of developmental delay, and he was the product of a normal uncomplicated pregnancy and delivery. Since the birth of the 3rd child, his behavior has been poor. The vital signs are normal, and examination of other body systems is unremarkable. Question What investigation is most important in this child's workup? Answer Choices 1 Complete cystometric evaluation 2 Intravenous pyelography (IVP) 3 Renal ultrasound 4 Urine culture 5 Urinalysis

Urinalysis Enuresis means any involuntary loss of urine. If it is used to denote incontinence during sleep, it should always be qualified with the adjective "nocturnal." Nocturnal enuresis is defined as involuntary urination in sleep without urological or neurological causes after the age of 5 years, at which time bladder control would normally be expected. The classification of enuresis is based on whether the child has ever achieved bladder control. Primary enuresis refers to a child who has never been dry; whereas, secondary enuresis means the child has been dry for a period but becomes enuretic later. Another useful classification is based on a period when the child does not have bladder control: • Nocturnal enuresis: Enuresis at night only. • Diurnal enuresis: Enuresis during the day only. • Nocturnal and diurnal enuresis: Enuresis during both day and night. The pathophysiology of enuresis is not completely clear; however, immaturity on the part of the autonomic nervous system that controls the bladder is present in the vast majority of cases. Only 20% of children with enuresis have a psychodevelopmental disorder (lower intelligence quotient or behavioral disorder). Secondary enuresis is often associated with a stressful environmental event. Urinalysis is the only mandatory investigation for nocturnal enuresis. Treatment: the therapeutic approach is still based on empirical data. Therapy is aimed at alleviating the symptoms of nocturnal enuresis rather than at curing the condition. A behavior-modification program is the treatment of choice, including buzzer/bell and pad, positive reinforcement, charting progress to increase confidence and self-esteem, urinating before bedtime, avoiding liquids after the evening meal, and avoiding psychological trauma through blame or belittling the child. Complete cystometric evaluation is incorrect. Cystometric analysis is used to evaluate the bladder's capacity to contract and expel urine. Intravenous pyelography (IVP) is incorrect. IVP refers to a series of X-rays taken of the kidneys, their collecting or drainage system (the ureters), and the bladder. It is done to locate a suspected obstruction to the flow of urine through the collecting system. Renal ultrasound is incorrect. If enuresis also occurs in the daytime or if urinary flow is small or interrupted, a renal ultrasound and a careful neurologic examination are indicated. Urine culture is incorrect. Urine culture may be appropriate, but is not most important. Urine culture is useful mainly when history, physical examination, or both suggests infection.

A 32-year-old woman presents with weight gain, acne, and purple striae. Question What test should be ordered to confirm the most likely diagnosis? Answer Choices 1 Serum electrolytes 2 Urinary free cortisol level 3 Blood LH/FSH ratio 4 Blood androgen level 5 Urinary osmolality

Urinary free cortisol level Explanation Cushing syndrome is due to an elevation of corticosteroids in the blood stream; it is diagnosed by measuring urinary free cortisol. Cushing presents with moon faces, facial plethora, supraclavicular fat pads, buffalo hump, truncal obesity, and purple striae. Hypertension, impaired glucose tolerance, and osteopenia may be seen. Causes of Cushing syndrome include administration of exogenous corticosteroids such as prednisolone, pituitary adenoma, adrenal adenoma or carcinoma, and exogenous production of ACTH (such as oat cell tumor of lung). Urinary free cortisol estimates the 24-hour cortisol in urine. A value about 3 - 4 times higher than normal suggests Cushing syndrome. Values higher than normal, but not 3 - 4 times, and values within the normal range will need repeat testing in suspected cases. Other tests are available, such as the dexamethasone suppression test. During this test, 1 mg of dexamethasone is given at 11 pm, and the cortisol level is measured at 8 am on the subsequent morning. Serum electrolytes, androgen level, luteinizing hormone level, and urine osmolality are not useful in the diagnosis of Cushing syndrome.

A 62-year-old man is being evaluated in an internal medicine clinic for persistent hematuria. He denies dysuria, urgency, and frequency. The isolated hematuria was first discovered 4 months ago on a routine urinalysis for a workplace physical. A repeat urinalysis 6 weeks later again showed hematuria, confirmed by microscopic evaluation. The remainder of his urinalysis was within normal limits. His past medical history is remarkable for COPD and obesity. He has smoked 1.5 ppd x 45 years. He uses inhaled medications for his COPD and has NKDA. His physical exam was significant for a temperature of 100.5 degrees F. Abdominal exam elicited mild tenderness in the left upper quadrant, but no masses were palpable (although his exam was limited by his obesity). Chest and abdominal CTs with contrast were ordered. The report showed a normal chest and a left renal mass enhanced by radiocontrast, suggestive of renal cell carcinoma (RCC). No metastatic disease was noted on imaging. A renal biopsy is pending. Question Assuming the biopsy supports the diagnosis of renal cell carcinoma, what referral would be most appropriate for this patient? Answer Choices 1 Medical oncologist for traditional cytotoxic chemotherapy 2 Radiation oncologist for external beam radiation therapy 3 Urologic surgeon for radical nephrectomy 4 Urologist for retrograde pyelogram 5 Hospice for palliative care only

Urologic surgeon for radical nephrectomy Explanation The most appropriate intervention for this patient is referral to a urologic surgeon for radical nephrectomy, which is the "standard treatment for localized RCC and provides a reasonable chance for cure."1 The nephrectomy serves to diagnose, stage, and treat the cancer. "Surgical removal of the kidney is the most accurate method to definitively diagnose and to stage RCC."2 A referral to a medical oncologist for traditional cytotoxic chemotherapy is not recommended, as "none of the cytotoxic chemotherapy agents have been shown to improve survival in renal cell carcinoma."2 Immunotherapy (interleukon or interferon) is considered a reasonable addition to nephrectomy in patients with metastatic RCC, but there is no current evidence suggesting this patient has metastatic disease. RCC is considered a radiation-resistant tumor, so external beam radiation is rarely used as a primary treatment. Radiation may have a role in palliative treatment in special circumstances. Referral to a urologist for a retrograde pyelogram would not address the RCC. This type of imaging allows visualization of the bladder, ureters, and pelvicaliceal collecting system by administering contrast through a catheter to flow up toward the kidneys. This type of imaging may be helpful in the diagnosis of urethral strictures, trauma, and reflux, but adds no additional information and certainly no treatment value for this patient with RCC. A referral to hospice for palliative care only suggests there are no reasonable treatments for this patient and that he is expected to die within 6 months. However, 5-year survival rates are around 66% for stage I RCC, and this patient should be offered the referral for treatment.

A 23-year-old primiparous woman with an estimated gestational age of 27 weeks presents following a motor vehicle accident. Another car struck the vehicle that she was driving on the passenger side at 40 mph. She was wearing her seat belt. She cannot recall striking her head, but her chest hurts where the belt was, and her chin is scraped from the air bag. Review of her prenatal charts shows that her laboratory work is up to date. Her Rh factor is negative and her initial antibody screen was negative. She has gained 18 pounds during the pregnancy, and her blood pressure and urine dipstick readings have been normal. Her dates were confirmed by an ultrasound done at 12 weeks, and the fundal height measurements have fit with the dates throughout the pregnancy. Her physical exam reveals a blood pressure of 130/85 mm Hg, pulse of 96 BPM, respirations of 16/minute, and temperature of 98 °F (36 °C). There is an irregular abrasion on the chin. All extremities move fully without pain. The fundal height is 28 cm and the abdomen is non-tender. The fetal head is ballotable above the symphysis. Question What would be a major warning sign of minor trauma in this patient? Answer Choices 1 Fetal heart rate of 150 beats/minute 2 Presence of short and long-term fetal heart rate variability 3 2 fetal heart rate accelerations of about 22 beats per minute within a 20 minute strip 4 2 uterine contractions per hour 5 Vaginal bleeding

Vaginal bleeding Explanation Pregnant women who are involved in motor vehicle accidents need to be evaluated for the likelihood of injury that could have adverse effects on the pregnancy. The workup should include a careful physical exam and lab testing to check for maternal or fetomaternal hemorrhage. Rhesus-negative mothers may need Rh° (D) immune globulin. The most important component of the evaluation is fetal heart monitoring of at least 4 hours duration. After this initial monitoring period, if the mother is stable and has no specified warning signs, she may be discharged. Vaginal bleeding may be indicative of placental abruption and doing a speculum examination to evaluate for the presence of amniotic fluid and/or blood is prudent. An urgent ultrasound to assess for abruption is not likely to be helpful (sensitivity is 40% and specificity is 25). Differentiating between normal 'venous lakes' and extravasation, indicating abruption, is difficult. Frequent uterine contractions with or without vaginal bleeding are a good indicator; later, a detailed biophysical profile might be helpful if initial cardiac monitoring is equivocal. In a recent related study, frequent uterine contractions of greater than 4 contractions/hr are associated with poorer outcomes (especially within the initial 4 hours after trauma). Those with fewer than 4 uterine contractions per hour had the same live birth rates and Apgar scores as the control patients. Evidence of a reactive fetal strip would include the presence of short and long-term beat-to-beat variability, and the presence of at least 2 accelerations in fetal heart rate of at least 20 beats/minute within a 20 minute strip. In addition, a fetal heart rate of between 160/minute and 120/minute (above or below which would be classed as a fetal tachycardia and fetal bradycardia, respectively). Other warning signs would include the presence of abdominal tenderness and the presence of fetal decelerations, especially late decelerations.

A 62-year-old diabetic hypertensive woman is evaluated for incontinence. She is found to have a cystocele and grade 3 uterine prolapse. Her diabetes is poorly controlled; so is her hypertension. On examination, the uterus can be repositioned with firm digital pressure. In addition to being declared unfit for surgery, she is not keen on undergoing any procedure. She has no history of any urinary infections or pelvic inflammatory disease. Question What is the next best step in management? Answer Choices 1 Anticholinergic agents to reduce bladder reactivity 2 Vaginal pessary placement 3 Laparoscopic ventral suspension of the uterus 4 Topical estrogen creams 5 Daily manual replacement of uterus

Vaginal pessary placement Explanation Vaginal pessary placement is the correct answer. Vaginal pessaries are the mainstay of conservative management of uterine prolapse. Pessaries are manually placed around the cervix, which prevents the descent of the uterus. Acute pelvic inflammatory disease and recurrent vaginitis are the only real contraindications to pessary placement. Anticholinergic agents is incorrect. Anticholinergic agents are used in the management of overactive bladders and act by reducing the bladder tone. Cholinergic nerve supply is responsible for detrusor contraction; thus, anticholinergics are useful in reducing bladder tone. Laparoscopic ventral suspension of the uterus is incorrect. While the treatment of choice for symptomatic prolapse is surgery, it is contraindicated in patients that are unfit for surgery, as is this patient. Topical estrogen creams is incorrect. It does not help uterine prolapse. Estrogen creams are useful in treating vaginal dryness and atrophic vaginitis.

What is the treatment of choice for primary generalized seizures? Answer Choices 1 Phenytoin 2 Phenobarbital 3 Valproic acid 4 Carbamazepine 5 Gabapentin

Valporic Acid Epilepsy is defined as a syndrome characterized by recurrent, unprovoked seizures. A seizure is best defined as a discrete neurologic event caused by abnormal, excessive, spontaneous discharge of cerebral cortical neurons. The phenomenology of a single seizure is dependent upon the origin and subsequent spread of neuronal discharge. Epileptic seizures, as opposed to symptomatic seizures provoked by acute brain disturbances (e.g. hyponatremia, meningitis, cocaine use), are treated medically with anticonvulsants. Specific anti-epileptic drugs are efficacious for different types of seizure. Primary generalized epilepsy is commonly associated with developmental brain disorders of childhood (congenital and acquired). Types of primary generalized seizures include tonic-clonic, absence, tonic, clonic, myoclonic, and atonic. Anticonvulsants indicated for primary generalized seizures include valproic acid, clonazepam, and ethosuximide. Localization-related epilepsy results in partial (focal) seizures with or without secondary generalization. These are best treated with phenytoin, carbamazepine, phenobarbital, and valproic acid. The newer anticonvulsants, gabapentin, felbamate, and lamotrigine are also effective for partial seizures.

A 70-year-old man presents with memory loss that has been worsening over the past few months. He also has history of slurred speech and walking difficulty. There is a history of getting lost in familiar surroundings. His history is also significant for a stroke 2 years ago. On examination, there are focal neurological signs including plantar response, gait abnormalities, exaggeration of deep tendon reflexes, and weakness of the right lower limb. Brain imaging studies show multiple large-vessel infarcts and extensive periventricular white matter lesion. What is indicated by evidence of relevant cerebrovascular disease on brain imaging and by focal signs, along with temporal relationship of stroke and dementia, or abrupt cognitive deterioration and stepwise progression? Answer Choices 1 Lewy body dementia 2 Vascular Dementia 3 Dementia of the Alzheimer's Type 4 Dementia Due to Parkinson's disease 5 Delirium

Vascular dementia Dementia is defined as a multifaceted decline in cognitive functioning that causes impaired functioning in daily life. Memory impairment is generally regarded as a necessary aspect, but decline in one or more other cognitive domains (such as language, praxis, gnosis, visuoconstructive function, frontal-executive functions) also should be demonstrated. Vascular dementia is the second most common cause of dementia after Alzheimer's. Multi-infarct dementia is the most common type of vascular dementia. The criteria for the diagnosis of vascular dementia includes evidence of relevant cerebrovascular disease on brain imaging (brain imaging must show evidence of multiple large-vessel infarcts, a single strategically placed infarct, multiple basal ganglia, white-matter lacunae's or hemorrhages, extensive periventricular white-matter lesions, or combinations of these), by focal signs along with temporal relationship of stroke and dementia, or abrupt cognitive deterioration and stepwise progression. The course must come on gradually and be accompanied by continuing cognitive decline, and there must be no evidence that the dementia is due to a physical condition. Frequently, laboratory studies, MRI, or CT scans will confirm the diagnosis. Lewy body dementia is caused by the abnormal microscopic deposition of protein in nerve cells, called Lewy bodies, which cause destruction of the nerve cells over time. These deposits can cause symptoms typical of Parkinson disease, such as tremor and muscle rigidity, as well as dementia similar to that of Alzheimer's disease. Delirium is a state of transient global disorder of cognition. White matter underlying the cortex can aid in determining the etiology of this disorder. Alzheimer's dementia is the most common type of dementia that is caused by a loss of brain cells. It is a form of degenerative brain disease resulting in progressive mental deterioration with disorientation, memory disturbance, and confusion. Parkinson's disease is a progressive neurological disorder in which there is limb stiffness, speech problems, and tremor. Dementia may develop late in the disease, but not everyone with Parkinson's disease has dementia.

A 37-year-old woman presents with increased thirst. She states that she is always thirsty and has cravings for ice water. She has been urinating frequently, typically with large amounts, and notes nocturia. She finds herself unable to restrict her fluid intake. Urinalysis shows pale urine with osmolarity of 200 mOsm/L and no presence of bacteria, glucose, or ketones. Question What testing would be appropriate in this patient? Answer Choices 1 Oral glucose tolerance test 2 Vasopressin challenge test 3 Cosyntropin stimulation test 4 Dexamethasone suppression test 5 Radioactive iodine uptake test

Vasopressin challenge test Explanation Vasopressin challenge test is correct. The patient is showing signs of diabetes insipidus. Diabetes insipidus is caused by one of two mechanisms: (1) impaired release of antidiuretic hormone (ADH) from the hypothalamus (central diabetes insipidus) or (2) impaired response of the kidneys to ADH (nephrogenic diabetes insipidus). The vasopressin challenge test is performed by obtaining urine measurements before and after administering desmopressin acetate (a synthetic vasopressin). Those with central diabetes insipidus will have an increase in urine osmolality following desmopressin administration. Those with nephrogenic diabetes insipidus will not have any change in urine osmolality following desmopressin administration. Oral glucose tolerance test is incorrect. The oral glucose tolerance test is used in the diagnosis of diabetes mellitus. Cosyntropin stimulation test is incorrect. The cosyntropin stimulation test is used in the diagnosis of Addison's disease (adrenal insufficiency). Dexamethasone suppression test is incorrect. The dexamethasone suppression test is used in the diagnosis of Cushing syndrome (hypercortisolism). Radioactive iodine uptake test is incorrect. The radioactive iodine uptake test is used in the work-up of hyperthyroidism.

A 4-year-old boy presents with a 3-day history of fever with chills, cough, and fast breathing. His parents report decreased oral intake and increased difficulty breathing with retractions over the last 24 hours. His initial vital signs are: heart rate 144 bpm; respiratory rate 32/min; temperature 101.3 degrees Fahrenheit (38.5 degree Celsius); and oxyhemoglobin saturation 89% on room air. He is immediately started on supplemental oxygen, and his oxyhemoglobin saturation improves to 95%. Subsequent evaluation, including a chest X-ray, is suggestive of right middle and lower lobe pneumonia. Question What is the mechanism for the low oxyhemoglobin saturation in this patient? Answer Choices 1 Hypoventilation 2 Ventilation/Perfusion mismatch 3 Diffusion Block 4 Shunt 5 Increased dead space in lungs

Ventilation/Perfusion mismatch Of all the causes of hypoxemia in children, ventilation perfusion mismatch is the most common cause in most clinical situations. In a child with pneumonia, the middle and lower lobes of the right lung will continue to receive pulmonary blood flow, but the exchange of gases will be impaired due to poor air entry in those parts of the lung. This generates a mismatch between the ventilation (which is reduced) and perfusion (which is maintained), causing most of the blood from those areas to return to the left atrium in a relatively deoxygenated state. This leads to an overall drop in the percentage of hemoglobin molecules that are saturated with oxygen, thereby manifesting as a drop in the measured oxyhemoglobin saturation. Hypoventilation occurs in patients with neuromuscular weakness and can cause significant elevation of carbon dioxide levels before it causes hypoxemia. Diffusion block is associated with lung disorders, such as idiopathic pulmonary fibrosis and interstitial lung diseases, which increase the thickness of the alveolar basement membrane, thereby reducing the exchange of gases. Shunt causes hypoxemia due to the mixing of deoxygenated blood with oxygenated blood when the former has bypassed the lung completely (such as right to left shunt in a patient with Tetralogy of Fallot). A patient with pneumonia will not have an increase of dead space in the lungs.

Verbigeration

Verbigeration is a symptom of schizophrenia and other psychotic disorders that consists of repetition of senseless words or phrases, but is not a side effect of neuroleptic medication.

6 hours after eating raw fish and other seafood at a Japanese restaurant, a couple developed nausea and vomiting, watery diarrhea, abdominal cramps, and fever. What organism is likely involved in the patients' condition? Answer Choices 1 Salmonella enteritidis 2 Bacillus cereus 3 Clostridium perfringens 4 Staphylococcus aureus 5 Vibrio parahaemolyticus

Vibrio Bacillus cereus cause 2 syndromes: (1) one involves a short incubation period with nausea and vomiting and is similar to staphylococcal food poisoning; (2) the other involves a long incubation period (18 hours) with watery, non-bloody diarrhea and resembles clostridial gastroenteritis. Clostridium perfringens cause food poisoning with an 8- to 16-hour incubation period. It is characterized by watery diarrhea with cramps and little vomiting. It resolves in 24 hours. Staphylococcus aureus cause food poisoning characterized by vomiting being more prominent than diarrhea. The disease is due to ingestion of enterotoxin, which is performed in foods and hence has a short incubation period (1-8 hours). Vibrio parahaemolyticus are a marine organism transmitted by contaminated seafood, especially when raw fish is eaten. The clinical picture varies from mild to quite severe watery diarrhea, nausea and vomiting, abdominal cramps, and fever. The illness is self-limited, lasting about 3 days. Salmonella enteritidis cause enterocolitis after an incubation period of 6-48 hours. The disease begins with nausea and vomiting and then progresses to abdominal pain and diarrhea, which can vary from mild to severe and present with or without blood. Usually, the disease lasts a few days and is self-limited.

A 50-year-old woman with a family history of colon cancer underwent a total colonoscopy as per screening guidelines for colon cancer. 3 small (<1cm) pedunculated polyps were removed using hot-wire biopsy. Pathology reported all 3 polyps to be tubular adenomas. What type of colon polyp has the highest risk of carcinoma? Answer Choices 1 Hyperplastic 2 Hamartomatous 3 Tubular adenoma 4 Tubulovillous adenoma 5 Villous adenoma

Villous Cancer Hyperplastic polyps are benign with no malignant potential. Hamartomatous polyps are normal tissues in an abnormal configuration with no malignant potential; juvenile polyps are the most common type of hamartomatous polyps. Tubular adenomas are the most common neoplastic polyp, and they have a 1-33% risk of carcinoma (depending upon their size). Tubulovillous adenomas have a higher risk of carcinoma (5-45%). Villous adenomas are usually sessile polyps with the highest risk of carcinoma (10-55%); this is partially because 60% of them are >2cm at discovery with a 55% malignant potential.

1-year-old boy presents with colicky abdominal pain, vomiting, and a 1-day history of irritability and lethargy. The child has vomited once, and the vomitus contained only food particles, without blood or bile. Rectal bleeding and "currant jelly" stool are also noted. The child has been healthy, and this is the first such episode since birth. Vitals were as follows: PB 110/70 mm Hg, PR 100/min, RR 20/min, temperature 98.8° F. On examination, a soft, slightly tender abdomen is noted. A vague vertical mass is palpable in the right upper quadrant. Other systems are normal on examination. Plain abdominal film shows evidence of obstruction, and barium enema detects a coiled-spring appearance to the bowel. Intussusception is diagnosed. Question What disease would you suspect if the child is under the age of 2 years and has hypertrophy of Peyer's patches? Answer Choices 1 Viral infection 2 Meckel's diverticulum 3 Peutz-Jeghers polyps 4 Juvenile polyposis 5 Lymphoma

Viral Infection Intussusception is the 'telescoping' of bowel into the adjacent segment of the bowel. It is most common in male children of the age group 3 months-1 year. The triad of colicky abdominal pain, vomiting, and rectal bleeding in this age group is a reliable hint to the diagnosis. Rare presentations, such as those with neurological symptoms, subacute, or chronic forms, can also occur. The most commonly involved sites are the terminal ileum and ileocecum. A mechanical lead point can be identified in children above 2 years, such as abdominal trauma, hemangioma, Meckels diverticulum, and foreign body; however, in younger children it is assumed to be a previous viral infection causing hypertrophy of Peyer's patches. This leads to intussusception, strangulates venous return, and results in the classic consequences of bowel swelling, ischemia, and perforation. A contrast enema is both diagnostic and therapeutic. Reduction can be achieved in most cases. There is a chance of recurrence in 10% of children, which can be treated again by contrast barium enema.

A 1-year-old female is having a 2-day history of fever (102 0 F oral), rhinorrhea, and dry cough, with a decreased appetite. The mother states that her daughter has been less active, and her fluid intake has decreased for her age. On exam, the child is non-toxic appearing, has a rectal temperature of 100.2 0 F, and has nasal flaring and a respiratory rate of 45, rhinorrhea, moist mucous membranes, and a minimal wheeze heard bilaterally. Her chest x-ray has no specific findings. Based on these findings, what is the initial ancillary test to confirm the diagnosis? A Acid fast bath test B Viral nasal washings C Sputum culture and sensitivity D Blood cultures E Throat culture

Viral Nasal Washings B Viral nasal washings are the best choice for determining RSV infection that causes bronchiolitis. Sputum and blood cultures do not grow the agents that would typically cause the infection to occur, and you are obtaining a sample from a different part of the respiratory tract and blood.

A 60-year-old man presents with problems seeing at night, especially when driving. You suspect he is suffering from night blindness. A deficiency in what vitamin can cause night blindness? Answer Choices 1 Vitamin D 2 Vitamin C 3 Vitamin A 4 Vitamin B6 5 Vitamin K

Vitamin A Vitamin A is involved in the general health of epithelial tissue. It exists in various forms that have different functions in the organism. The 11-cis-retinal form functions as a component of the vision cycle. It combines with the protein opsin to form rhodopsin, which is the photosensitive molecule in the discs of the rod cells. Upon absorption of light, the 11-cis form is converted to trans-retinal that dissociates from the opsin protein. This cascade converts light into atomic motion and then into a nerve signal. A single photon is absorbed by a rod cell and leads to hyperpolarization of the plasma membrane by closing the cation-specific channels. The trans-retinal is then isomerized back to 11-cis retinal, which can then bind opsin again and go through another cycle. A deficiency in vitamin A leads to night blindness and eventually deterioration of the outer rod segments. Vitamin A (all-trans-retinol) is converted to 11-cis-retinal in several steps. Retinyl phosphate functions in the synthesis of some glycoproteins and mucopolysaccharides as a glycosyl donor. This is especially true for certain glycoproteins necessary for normal growth regulation and mucous secretion. Retinol and all-trans-retinoic acid bind to specific receptors in the cell, the nuclear receptor superfamily. This family of receptors includes those that bind estrogen, progesterone, glucocorticoids, vitamin D3, and thyroxine, in addition to retinol and all-trans-retinoic acid. All of these receptors contain a highly conserved DNA binding domain, a hormone-binding domain, and a variable activation domain. β-Carotene functions as an antioxidant in the cell. β-Carotene is the precursor of retinal, the chromophore in visual pigments. Synthetic forms of vitamin A are also now available and used as drugs. 13-cis-retinoic acid is used in the treatment of acne. Etretinate, an aromatic derivative of all-trans- retinoic acid, is used in the treatment of psoriasis. The structures of some of these important retinoids are shown in the included image.

A 44-year-old emaciated man is brought to the emergency room towards late morning by his friends. They report he was difficult to rouse. He has not eaten anything since a large meal the night before. His friends report a 25-year history of alcohol abuse. Examination shows severely restricted horizontal eye movements and ataxia of both lower limbs. What nutritional deficiency can cause this patient's symptoms? Answer Choices 1 Folate 2 Vitamin A 3 Vitamin B1 4 Vitamin B4

Vitamin B1 (Thiamine) This patient is a classic example of Wernicke's encephalopathy, which results due to a deficiency of vitamin B1 or thiamine. Alcohol abuse is one of the most important contributing causes. The deficiency results in ischemic damage to the brain stem and its connections. The typical triad is comprised of ocular signs, ataxia, and a confused state. Management involves the immediate administration of thiamine parenterally - 50 mg IV and 50 mg IM, followed by daily doses of 50 mg IM. This leads to prompt reversibility of signs and symptoms. Note: Intravenous glucose solutions should not be given to such patients, as they may exhaust the patient's reserve of B vitamins and cause a rapid worsening.

A 48-year-old woman underwent gastric bypass surgery. She followed the prescribed diet and lost approximately 100 pounds in 18 months. She presents with numbness in her feet, and blood tests reveal a deficiency in 1 vitamin. Based on these observations, this woman is most likely deficient in what vitamin? Answer Choices 1 Biotin 2 Thiamine 3 Vitamin B12 4 Vitamin D 5 Vitamin K

Vitamin B12 The gastric bypass procedure bypasses the duodenum and much of the jejunum and can lead to malabsorption problems, particularly of vitamin B12, iron, folate, and calcium. The gastric bypass procedure done on morbidly obese patients is an operation that makes the stomach smaller and allows food to bypass part of the small intestine. This bypass of the intestine allows fewer calories to be absorbed, resulting in weight loss, but can also lead to insufficient absorption of some needed nutrients. The deficiency in vitamin B12 leads to problems associated with numbness and tingling of the hands and feet, weakness, fatigue, and anemia. Vitamin B12 is normally absorbed from the digestive system after it binds to a protein called intrinsic factor. An individual lacking intrinsic factor can also develop vitamin B12 deficiency. Vitamin B12 is required for 2 reactions in humans: the methylation of homocysteine to form methionine and the conversion of methylmalonyl CoA to succinyl CoA. The metabolism of vitamin B12 is tied to folic acid metabolism; therefore, a deficiency in vitamin B12 can lead to a metabolic deficiency of folic acid. This lack of usable folic acid leads to deficiencies in purine and dTMP synthesis. This in turn affects nucleotide synthesis needed for DNA synthesis. Rapidly dividing cells, such as blood cells, are affected, leading to pernicious anemia in the cases of vitamin B12 deficiency. The absorption of biotin, thiamine, and vitamins K are usually less affected by the gastric bypass procedure. A deficiency in vitamin D is also possible but would not cause the numbness in her feet. Deficiencies in vitamin D would manifest themselves in bone and calcium problems.

A 24-year-old woman presents with a 2-week history of persistent fatigue and decreased functional capacity. The woman has been consuming a strict total vegan diet (without dairy products or eggs) for the past 8 years. She also has a 3-week history of numbness and pins and needle sensation in her feet. The patient is having other problems, such as increased irritability and forgetfulness. Physical exam reveals pallor, peripheral neuropathy of her lower extremity, and memory deficits. CBC shows a macrocytic, normochromic anemia. There is also an elevation in the levels of methylmalonic acid. Question What therapy is indicated? Answer Choices 1 Folate supplementation 2 Iron supplementation 3 Vitamin B12 supplementation 4 Packed RBCs transfusion 5 Erythropoietin therapy

Vitamin B12 supplementation A strict vegan diet consists of fruits, vegetables, and grains. Meat (including fish, seafood, and poultry), dairy products, and eggs are excluded. This commonly results in vitamin B12 deficiency, presenting as macrocytic, normochromic anemia/megaloblastic anemia. The clinical manifestations associated with vitamin B12 deficiency include hematological, neurological, and psychiatric manifestations. The hematological manifestations include macrocytic (megaloblastic) anemia and pancytopenia in advanced cases. Paresthesias, peripheral neuropathy, and demyelination of the corticospinal tract and dorsal columns (subacute combined systems disease) are the neurological sequelae associated with vitamin B12 deficiency. Psychiatric disorders associated with vitamin B12 deficiency include impaired memory, irritability, depression, and dementia; rarely, psychosis is a result of the deficiency. Treatment of vitamin B12 deficiency includes administration of cobalamin through oral or parenteral route. Cobalamin is administered as 1000 mcg daily intramuscular (IM) 2 times per week for 2 weeks during the initial period, which is followed by 1000 mcg/wk IM for 5 weeks; afterwards, the patient must receive maintenance therapy of 100 - 1000 mcg IM every month for life. If iron studies indicate the need, iron supplementation may be considered in a hypochromic, microcytic anemia. Folate supplementation should be considered in macrocytic anemia with decreased serum folate levels. A normocytic, normochromic anemia may occur due to blood loss, aplastic anemia, sickle cell anemia, or end-stage renal disease. If severe, transfusion may be necessary. End-stage renal disease may result in decreased production of erythropoietin (EPO), and EPO therapy may be necessary.

A 32-year-old woman presents with a 1-month history of bleeding gums when brushing her teeth. She also reports that her wounds are taking longer than usual to heal. She is a stay-at-home mother and is breastfeeding her 6-month old twins. On examination, you note multiple splinter hemorrhages on her nails and ecchymoses over her lower limbs. Question What is the most likely diagnosis? Answer Choices 1 Vitamin A deficiency 2 Pyridoxine deficiency 3 Vitamin C deficiency 4 Niacin deficiency 5 Vitamin E deficiency

Vitamin C

A 32-year-old woman presents with a 1-month history of bleeding gums when brushing her teeth. She also reports that her wounds are taking longer than usual to heal. She is a stay-at-home mother and is breastfeeding her 6-month old twins. On examination, you note multiple splinter hemorrhages on her nails and ecchymoses over her lower limbs. Question What is the most likely diagnosis? Answer Choices 1 Vitamin A deficiency 2 Pyridoxine deficiency 3 Vitamin C deficiency 4 Niacin deficiency 5 Vitamin E deficiency

Vitamin C In cases of vitamin C (or ascorbic acid) deficiency, patients can present with bleeding tendencies (as a result of weakened capillaries) and impaired wound healing due to impaired formation of connective tissue. On examination, the gums may be swollen and friable; the teeth may be loose. There may also be multiple splinter hemorrhages on the nails and ecchymoses, especially over the lower limbs. Causes include inadequate dietary intake and certain conditions, such as pregnancy and lactation, which increase vitamin C requirements. Dietary sources of vitamin C include citrus fruits, such as oranges, lemons, and tangerines, as well as tomatoes and potatoes. Vitamin E deficiency may cause a hemolytic anemia in premature infants. Laboratory investigations reveal low plasma tocopherol levels, a low hemoglobin level, reticulocytosis, hyperbilirubinemia, and creatinuria. Causes of vitamin E deficiency in premature infants include limited placental transfer of vitamin E and the resultant low levels at birth combined with its relative deficiency in the infant diet. Dietary sources for older children and adults include wheat germ, vegetable oils, egg yolk, and leafy vegetables. In cases of vitamin A deficiency, patients can present with inability to see well in dim light or night blindness. There may also be conjunctival and corneal xerosis, as well as pericorneal and corneal opacities, and Bitot's spots. Bitot's spots are a collection of keratin appearing as triangular foamy spots on the conjunctiva. Patients may also have xeroderma, hyperkeratotic skin lesions, and increased susceptibility to infections. Causes include inadequate dietary intake and malabsorption. Dietary sources of vitamin A include fish, liver, egg yolk, butter, cream, dark green leafy vegetables, as well as yellow fruits and vegetables. Niacin deficiency causes pellagra, which is characterized by: A symmetrical dermatitis, usually on parts of the body exposed to sunlight Scarlet glossitis and stomatitis Diarrhea Mental aberrations, such as memory impairment, depression, and dementia. These may appear alone or in combination. Causes include inadequate dietary intake, especially in patients with corn-based diets or alcoholism. Dietary sources include legumes, yeast, meat, and enriched cereal products. In cases of pyridoxine or (vitamin B6 deficiency), patients can present with peripheral neuropathy, seborrheic dermatosis, glossitis, and cheilosis. Laboratory investigations reveal anemia with lymphopenia. Causes include malabsorption as well as medications, such as isoniazid and penicillamine. Dietary sources of vitamin B6 include liver, legumes, whole grain cereals, and meats.

A 4-month-old female infant has been gaining weight well and achieving normal milestones. She was a full-term infant born via vaginal delivery without complication. Her past medical history is significant only for a mild URI the previous month, and she is current with her vaccines. She has been exclusively breastfed since birth. Her mother eats a well-rounded diet that includes meat and dairy. Question What supplementation does the infant require? Answer Choices 1 Vitamin A 2 Vitamin C 3 Vitamin D 4 Vitamin E 5 Vitamin K

Vitamin D Breastfed-only infants require vitamin D supplementation to help prevent rickets, infantile tetany, and osteomalacia. The other vitamins are transferred in adequate quantity through breastmilk. Infant formula includes vitamin D. Children and adults get vitamin D through fortified milk, margarine, and fish liver oils. Vitamin D can also be obtained through exposure to sunlight or other ultraviolet sources.

A 3-year-old boy presents with his mother; she reports that he is unable to walk without support. He is an only child who rarely leaves his 10th floor apartment home. On examination, he has a rachitic rosary and genu valgum. Question What is the most likely diagnosis? Answer Choices 1 Thiamine deficiency 2 Riboflavin deficiency 3 Vitamin K deficiency 4 Vitamin D deficiency 5 Iron deficiency

Vitamin D Deficiency In cases of vitamin D deficiency, children can present with an inability to walk unsupported due to muscle weakness and lower limb skeletal deformities such as genu varum (bow legs) and genu valgum (knock knees). On examination, they may have prominent costochondral junctions (rachitic rosary) and indentation of the lower ribs at their diaphragmatic attachment (Harrison's groove). Causes include inadequate dietary intake and inadequate exposure to sunlight. They should be encouraged to be exposed to ultraviolet irradiation of the skin as well as increase their dietary sources of vitamin D which include fortified milk, fish liver oils, butter, egg yolk, and liver. In cases of riboflavin (or Vitamin B2) deficiency, patients present with angular stomatitis and cheilosis. On examination, they are pale, have atrophic glossitis, and the tongue may appear magenta. Vitamin B1 (or thiamine) deficiency results in beriberi, which is characterized by: A bilateral symmetric peripheral neuropathy beginning in the legs. Wernicke-Korsakoff syndrome, which is comprised of nystagmus, ophthalmoplegia, ataxia, memory loss, and confabulation. Congestive heart failure with tachycardia, peripheral edema, and cardiomegaly. In cases of vitamin K deficiency, patients present with bleeding tendencies, which include epistaxis, menorrhagia, and hematuria. The prothrombin time (PT) and the activated partial thromboplastin time (aPTT) are usually prolonged. In cases of iron deficiency, patients can present with feeling weak, dizzy, and tired; they may experience syncope. On examination, they have pale conjunctivae and koilonychia.

A 3-year-old boy presents with his mother; she reports that he is unable to walk without support. He is an only child who rarely leaves his 10th floor apartment home. On examination, he has a rachitic rosary and genu valgum. Question What is the most likely diagnosis? Answer Choices 1 Thiamine deficiency 2 Riboflavin deficiency 3 Vitamin K deficiency 4 Vitamin D deficiency 5 Iron deficiency

Vitamin D deficiency In cases of vitamin D deficiency, children can present with an inability to walk unsupported due to muscle weakness and lower limb skeletal deformities such as genu varum (bow legs) and genu valgum (knock knees). On examination, they may have prominent costochondral junctions (rachitic rosary) and indentation of the lower ribs at their diaphragmatic attachment (Harrison's groove). Causes include inadequate dietary intake and inadequate exposure to sunlight. They should be encouraged to be exposed to ultraviolet irradiation of the skin as well as increase their dietary sources of vitamin D which include fortified milk, fish liver oils, butter, egg yolk, and liver. In cases of riboflavin (or Vitamin B2) deficiency, patients present with angular stomatitis and cheilosis. On examination, they are pale, have atrophic glossitis, and the tongue may appear magenta. Vitamin B1 (or thiamine) deficiency results in beriberi, which is characterized by: A bilateral symmetric peripheral neuropathy beginning in the legs. Wernicke-Korsakoff syndrome, which is comprised of nystagmus, ophthalmoplegia, ataxia, memory loss, and confabulation. Congestive heart failure with tachycardia, peripheral edema, and cardiomegaly. In cases of vitamin K deficiency, patients present with bleeding tendencies, which include epistaxis, menorrhagia, and hematuria. The prothrombin time (PT) and the activated partial thromboplastin time (aPTT) are usually prolonged. In cases of iron deficiency, patients can present with feeling weak, dizzy, and tired; they may experience syncope. On examination, they have pale conjunctivae and koilonychia.

A 23-year-old man presents with difficulty in walking and frequent falls for the last 3 months. He gives history of occasional alcohol consumption and regular cigarette smoking (6 per day). He also has 3-month history of increased thirst and increased frequency of urine. There is no history of fever, weight loss, and no history of any major illness in the past. He has mild palpitations on walking. On examination, pulse 86/min and blood pressure 130/80mm of Hg are noted. Abdominal, respiratory, and cardiovascular system examination is within normal limits. CNS examination shows nystagmus, loss of fast saccadic eye movements, truncal titubation, absent deep tendon reflexes, and upward plantar reflexes. There is weakness in distal limb muscles and some sensory loss of fine touch and vibrations. He has ataxic gait, but no spasticity. His CBC was within normal limits, blood sugar fasting is 120mg%, and Postprandial is 168 mg%. MRI of brain is normal. Question This condition is associated with a defiency of what vitamin? Answer Choices 1 Vitamin A 2 Vitamin B1 (Thiamine) 3 Vitamin D 4 Vitamin E 5 Vitamin K

Vitamin E Friedreich's ataxia may be associated with vitamin E deficiency. 2 types of vitamin E deficiency can occur in Friedreich's ataxia: The first is abetalipoproteinemia (Bassen-Kornzweig syndrome), which is associated with a defect in the microsomal triglyceride transfer protein. This results in the impairment of formation and secretion of VLDL in the liver and a deficiency of delivery of vitamin E to the tissues, including CNS. The second type, called ataxia with vitamin E deficiency (AVED), is due to a defect in a tocoferol transfer protein and an impaired ability to bind the vitamin E to VLDL. There are 2 forms of Friedrich's ataxia- the classic form and the form associated with Vitamin E deficiency.

A 23-year-old man presents with difficulty in walking and frequent falls for the last 3 months. He gives history of occasional alcohol consumption and regular cigarette smoking (6 per day). He also has 3-month history of increased thirst and increased frequency of urine. There is no history of fever, weight loss, and no history of any major illness in the past. He has mild palpitations on walking. On examination, pulse 86/min and blood pressure 130/80mm of Hg are noted. Abdominal, respiratory, and cardiovascular system examination is within normal limits. CNS examination shows nystagmus, loss of fast saccadic eye movements, truncal titubation, absent deep tendon reflexes, and upward plantar reflexes. There is weakness in distal limb muscles and some sensory loss of fine touch and vibrations. He has ataxic gait, but no spasticity. His CBC was within normal limits, blood sugar fasting is 120mg%, and Postprandial is 168 mg%. MRI of brain is normal. Question This condition is associated with a defiency of what vitamin? Answer Choices 1 Vitamin A 2 Vitamin B1 (Thiamine) 3 Vitamin D 4 Vitamin E 5 Vitamin K

Vitamin E deficiency Friedreich's ataxia may be associated with vitamin E deficiency. 2 types of vitamin E deficiency can occur in Friedreich's ataxia: The first is abetalipoproteinemia (Bassen-Kornzweig syndrome), which is associated with a defect in the microsomal triglyceride transfer protein. This results in the impairment of formation and secretion of VLDL in the liver and a deficiency of delivery of vitamin E to the tissues, including CNS. The second type, called ataxia with vitamin E deficiency (AVED), is due to a defect in a tocoferol transfer protein and an impaired ability to bind the vitamin E to VLDL. There are 2 forms of Friedrich's ataxia- the classic form and the form associated with Vitamin E deficiency.

A 6-month-old infant is brought to your office. The mother indicates that the infant had several scratches that took a long time to stop bleeding. After ruling out various forms of hemophilia, you suspect that the infant has a vitamin deficiency that is causing this problem. The patient is deficient in what vitamin? Answer Choices 1 Vitamin C 2 Folic acid 3 Vitamin D 4 Vitamin K 5 Niacin

Vitamin K Prothrombin and many of the other blood clotting factors contain the modified amino acid γ-carboxyglutamic acid (gla). This modification occurs via an enzymatic reaction requiring vitamin K and converts glutamic acid, a weak calcium chelator, to gla, a much stronger calcium chelator. Vitamin K is an isoprenoid compound, which is classified as a fat-soluble vitamin; it is found in green plant leaves and is also synthesized by intestinal bacteria. The formation of gla residues is required for the 'active' form of prothrombin. These gla residues are found in the amino terminal region of the protein and function by chelating calcium ions. This forms part of the binding site to phospholipid membranes, where the conversion from the zymogen prothrombin to active thrombin takes place. A vitamin K deficiency results in an increased coagulation time. A 2,3-epoxide derivative of vitamin K is an intermediate in the formation of gla residues. Other proteins that contain gla residues include Factors VII, IX, and X, several other anticoagulation proteins, and the bone proteins (Osteocalcin and matrix-Gla protein). Dicumarol is a naturally occurring anticoagulant that is an antagonist of vitamin K. It inhibits the reductase that converts the epoxide form of vitamin K back to the active vitamin.

A 32-year-old woman presents with a pins and needles sensation on the sides and the bottom of her feet, as well as severe burning sensation in the feet. The burning sensation manifests primarily at night. Examination reveals a profoundly cachectic but alert patient with atrophy of muscles of the leg. Examination of the eye indicates that she also has nystagmus. Further questioning reveals that she has a history of homelessness and chronic alcoholism. What vitamin deficiency is most commonly associated with chronic alcoholism? Answer Choices 1 Vitamin C 2 Vitamin B1 3 Niacin 4 Vitamin E 5 Vitamin B6

Vitamin b1 (thiamine) Wernicke's encephalopathy is a result of the inadequate intake or absorption of thiamine (Vitamin B1) coupled with continued carbohydrate ingestion. It is most common among chronic alcoholics who suffer from poor nutrition. It is therefore imperative that all chronic alcoholics receive a dose of thiamine before they receive any glucose solution IV. It is also seen in patients on dialysis and during hyperemesis gravidarum of pregnancy. Clinical features appear acutely and include ataxia, confusion, nystagmus, partial ophthalmoplegia, apathy, confusion, severe memory loss, as well as autonomic dysfunction. Treatment of suspected or borderline cases consists of giving 5 - 30 mg of thiamine orally as a single dose for a period of 1 month. Parenteral thiamine of 50 - 100 mg is given to prevent Wernicke's encephalopathy, and it is also used in its treatment. Thiamine has to be given as IV, especially before administering fluids containing glucose. Rehydration of the patient along with supplementation of electrolytes and nutritional therapy is also required. Advanced cases require hospitalization and close supervision. Other vitamins deficiencies are not commonly associated with chronic alcoholism.

A 17 year-old male notes acute onset of dyspnea, excessive non-productive coughing, "wheezing" and upper chest tightness when running. Symptoms usually resolve in a few minutes and he is able to resume running. He is prescribed albuterol with no improvement. Which of the following is the most likely diagnosis in the patient? A Angioedema B Bronchiectasis C Cystic fibrosis D GERD E Vocal Cord Dysfunction

Vocal Cord Dysfunction E Vocal cord dysfunction (E) is commonly misdiagnosed as asthma or is a comorbidity in patients with asthma. The history of acute onset and rapid disappearance of symptoms is consistent with vocal cord dysfunction, and not typical of cystic fibrosis (C), or GERD (D). The presence of urticarial or swelling of the face or lips would support the life-threatening diagnosis of angioedema (A). Bronchiectasis (B) typically presents with a chronic productive cough and additional pulmonary symptoms based on the underlying cause and severity.

A 1-month-old infant boy presents with a 101.5 fever. He was a full term baby born via vaginal delivery. His mother was GBS (group B strep) positive and treated adequately. Since birth he has been exclusively breastfed and has been gaining weight slowly. He has no URI symptoms and is active and feeding well. He receives a sepsis workup and is admitted for 48-hour sepsis rule out. At 24 hours, his urine culture grows e. coli. His other cultures are negative, and he is sent home on antibiotics. Question What further testing should be done as an outpatient when he has recovered? Answer Choices 1 Monthly urine cultures 2 No testing required until he has a second UTI 3 CT of the pelvis 4 MRI of the abdomen 5 Voiding cystourethrogram

Voiding Cystourethrogram Explanation Any boy who has a urinary tract infection should undergo an initial urethral catheterization to collect a urine sample as the infant or child is unable to void upon request. A sensitivity test could also be ordered, so that the health care provider can test the bacteria to confirm which medication is best for treating the UTI infection. Immediately following the clearing of infection, additional testing may be recommended to rule out abnormalities in the urinary tract. Kidneys can be damaged with repeated infections. The age of the child and the type of urinary infection will determine the types of tests that will be ordered. As this child is less than 2 months, a voiding cystourethrogram should be ordered to confirm that hydronephrosis, scarring, obstructive uropathy, or masses are not associated with the urinary tract infection. As an individual test cannot detect all that needs to be known about the urinary tract that could prove useful, multiple tests might be necessary. Such tests could include: Kidney and bladder ultrasound, computerized tomography scan, magnetic resonance imaging, radionuclide scan, or urodynamics.

A 60-year-old man with a history of recurrent sinus infections presents with hemoptysis and hematuria. Physical examination shows a temperature of 101 degrees Fahrenheit, a blood pressure of 145/85 mmHg, decreased breath sounds on his right lower lobe, and palpable purpura on his bilateral lower legs. No warm or swollen joints were noted. Anti-neutrophil cytoplasmic antibody Positive Anti-double-stranded DNA Negative Anti-Glomerular Basement Antibody Negative Erythrocyte sedimentation rate 50mm/hour Hemoglobin 11 g/dL Serum creatinine 1.5 mg/dl Serum potassium 5 meq/l Serum chloride 21 meq/l Urinalysis 1.020, pH 6, numerous red cells, 500 mg albumin/g creatinine, red blood cell casts Chest x-ray Right lower lobe effusion Question What is the most likely diagnosis? Answer Choices 1 Systemic lupus erythematosus 2 Wegener's Granulomatosis 3 Rheumatoid arthritis 4 Goodpasture's syndrome 5 Henoch-Schönlein Purpura (HSP)

Wegener's Granulomatosis Wegener's Granulomatosis is a small vessel vasculitis of unknown pathogenesis that causes inflammation and necrotizing granulomas in the upper and lower respiratory tract and kidneys, as well as rash; 15 - 50% of patients have cutaneous involvement (Brenner, chapter 31). Palpable reddish-purple lesions on the lower extremities should raise suspicion of vasculitis; macules and nodules, may also erupt. Some 50 - 95% of patients with Wegener's will eventually have renal involvement (Brenner), which may range from mild findings to fulminant dialysis-requiring nephritis. On renal biopsy, focal segmental necrotizing glomerulonephritis is noted; crescents may be seen. Immune staining is generally negative. Treatment involves immunosuppressive therapy, as is the case with SLE, and dialysis as needed. In some resistant cases, plasmapheresis may be helpful. Patients with Wegener's are often Anti-Neutrophil Cytoplasmic Antibody (ANCA) positive. Patients with Goodpasture's syndrome typically present with a triad of glomerulonephritis, pulmonary hemorrhage, and anti-glomerular basement membrane (GBM), type IV collagen antibodies. Many patients with Goodpasture's syndrome are ANCA positive. SLEis a systemic inflammatory disease that causes a variety of signs and symptoms affecting many organs. Renal involvement is frequent in SLE and may include a variety of glomerular diseases, causing hematuria, proteinuria, and urinary casts. Henoch-Schönlein Purpura (HSP) is a vasculitic syndrome that is seen more often in children than adults; it may cause petechiae, purpura, gastrointestinal bleeding, and hematuria/glomerulonephritis. Complement counts and anti-double-stranded DNA are normal, and elevated IgA antibody levels may be seen. Rheumatoid arthritis (RA) is an inflammatory arthritis that causes joint stiffness and swelling that takes a long time to loosen up after prolonged sitting or sleeping. Symmetrically swollen joints and rheumatoid factor may be noted, but vasculitis is uncommon in RA (Harrison's chapter 314).

A 55-year-old man with a history of heavy alcohol use presents for treatment of an infected gash in his hand. He is not currently intoxicated. His clothes fit loosely, and he mentions that he thinks he has lost weight since becoming homeless earlier in the year. In preparation for giving him antibiotics, he receives an intravenous infusion of 5% dextrose solution. 2 hours later, he is confused and ataxic, with bilateral 6th nerve palsies (intranuclear ophthalmoplegia) and nystagmus. What is the patient most likely exhibiting? Answer Choices 1 Alcoholic ketoacidosis 2 Wernicke encephalopathy 3 Friedreich ataxia 4 Sepsis 5 Malnutrition

Wernicke encepahlopathy Ablation or destruction of the limbic areas highlights the role of limbic areas in the processing and storage of memory. Because of the protection afforded by the redundancy of having 2 hemispheres with overlapping functions, tumors, injuries or infarctions affecting limbic areas are often silent unless they cause pressure effects or electrical excitation. Toxic or metabolic damage, which typically affects symmetrical areas, can have a dramatic impact by contrast. The most common, clinically significant example is that of thiamine deficiency, which occurs in malnutrition, especially among alcoholics. Acute thiamine deficiency produces Wernicke encephalopathy. The cardinal features of this condition are delirium with ataxia, oculomotor dysfunction (especially 6th nerve palsy- intranuclear ophthalmoplegia), and sometimes anisocoria. Because thiamine is a cofactor for the cerebral metabolism of glucose, administering glucose to someone on the verge of thiamine deficiency can precipitate acute encephalopathy. Wernicke encephalopathy is prevented or rapidly reversed by administration of thiamine. Chronic thiamine deficiency causes alcohol-induced persisting amnestic disorder (Korsakoff's syndrome). The main sites of damage are paraventricular, symmetrical lesions in the mammillary bodies, the thalamus, and the hypothalamus. Lesions may also occur in the midbrain, pons, medulla, fornix, and cerebellum. Cerebellar and ocular signs may not be present. Diagnosis depends on a careful mental status examination. Patients with Korsakoff's syndrome have relatively preserved long-term memory, are socially appropriate, use language normally, and can do simple calculations. Their profound recent memory loss seems so disproportionate to the rest of their mental status examination that observers may misdiagnosis malingering, i.e., willfully faking abnormality for personal gain. Patients with Korsakoff's syndrome will occasionally try to conceal their deficits by confabulation, making up or agreeing to untrue scenarios given by the examiner. This is not the same thing as lying, but it is often mistaken for it. The mild macrocytosis is a common incidental finding in chronic alcoholics, but it does not account for the observed behavioral changes.

What is Chronic Kidney Disease

What is CKD? • > 3 months with Low GFR <60 ml/min

A 3-year-old girl presents with progressive abdominal enlargement associated with abdominal pain and occasional vomiting. Physical examination shows a palpable mass over the right upper quadrant extending to the right flank. She looks pale and the BP is slightly elevated. Urinalysis shows microscopic hematuria. What is the most likely diagnosis? Answer Choices 1 Wilm's Tumor 2 Neuroblastoma 3 Nephroblastomatosis 4 Renal Cell Carcinoma 5 Mesoblastic Nephroma

Wilm's tumor Explanation The clinical picture is suggestive of Wilm's Tumor. It accounts for most renal tumors in childhood during the first 5 years of life. It affects both sexes equally. It is a solitary growth that affects either part of the kidneys. There are congenital anomalies associated with it, most commonly the GUT anomalies, hemihypertrophy, sporadic aniridia, and mental retardation. It is commonly manifested by an abdominal mass that is described as generally smooth, firm and rarely crosses the midline, and it causes abdominal pain and vomiting. Hypertension is seen in 60% of the patients either due to elaboration of renin by the tumor cells or due to compression of the renal vasculature by the tumor. Hematuria is also uncommon and mostly microscopic. CT scan confirms the diagnosis which will show an intrarenal tumor, therefore ruling out Neuroblastoma. Treatment is by surgical removal. Chemotherapy is indicated post-operatively for the residual tumor. Neuroblastoma is a malignancy of the neural crest. It is the most common solid tumor in children outside the CNS. It is slightly more common in males and whites and median age of diagnosis is 2 years old. It arises mostly in the abdomen either in the adrenal gland or retroperitoneal sympathetic ganglia followed by the thoracic area mostly seen in the posterior mediastinum. Other sites are the head, neck, and epidural area. Tumors in the head and neck region are sometimes associated with Horner's Syndrome (Mioisis, Ptosis, Anhidrosis, and Enophthalmos). Diagnosis is by CT scan or MRI but pathologic diagnosis is made by biopsy. Tumor markers such as VMA and HVA (Homovanillic Acid) help confirm the diagnosis. Treatment is surgery, chemotherapy, and radiation depending on the stage of the tumor. Nephroblastomatosis are immature renal elements called Nephrogenic rest. It is a Wilm's tumor precursor lesion that is both unifocal and deep within the Renal parenchyma (intralobar rest) or multi-focal (perilobar rest). Subsequent development of Wilm's tumor in the other kidney is more likely in patients with this feature; therefore prompt inspection of the contralateral kidney is necessary during surgery of the neprhogenic rest. CT scan follow-up should also be done. Renal Cell Carcinoma is rare during the first decade of life but can occur occasionally in teenagers. Initial presentations are abdominal mass and hematuria. Surgical resection may offer cure, but prognosis is poor with post-operative residual disease. Mesoblastic Nephroma is a massive, firm, solitary renal mass and is generally thought to be benign. It resembles Leiomyoma or low-grade leiomyosarcoma grossly and microscopically. It also accounts for the majority of congenital renal tumors. It is more often seen in males and noted to produce renin. Treatment is surgical resection.

A 17-year-old adolescent male presents with unexplained neurological symptoms. His liver is enlarged on palpation, and he has other symptoms of hepatitis. Blood work reveals depressed ceruloplasmin levels. An ophthalmological examination reveals Kayser-Fleischer rings. What is the most likely diagnosis? Answer Choices 1 Krabbe disease 2 Lesch-Nyan syndrome 3 Marfan syndrome 4 Wilson disease 5 Hunter syndrome

Wilson Disease This young man is suffering from Wilson's disease, a genetic disorder of copper metabolism. It is inherited as an autosomal recessive mutation in the ATP7B genes located on chromosome 13. The protein of the ATP7B gene is a copper-transporting ATPase The frequency of the heterozygous carriers is relatively high (1/90). The incidence of homozygous recessive affected individuals is about 1/30,000. It affects all ethnic groups and both sexes equally. Neurological symptoms, hepatitis, and Kayser-Fleisher rings, greenish-brown deposits of copper in the corneal endothelial (Descemet membrane) basement membrane near the peripheral cornea where it meets the iris (limbus), are characteristic of Wilson's disease. About 40% of the dietary copper is absorbed in the gastrointestinal tract, and makes its way to the liver bound to albumin. In the liver, it is complexed with ceruloplasmin, a blood protein that carries most of the copper. Ceruloplasmin levels are abnormally low in patients with Wilson disease, although the disease is not, per se, a mutation in the ceruloplasmin gene. Ceruloplasmin is recycled in the liver by the usual lysosomal degradation pathway, and the unused copper is excreted in bile. When excessive copper is absorbed in the gut, it accumulates in the brain (producing neurological symptoms), the liver (producing hepatitis and hepatomegaly), and the cornea. Penicillamine can be used to treat this disease. It chelates copper and provides some symptomatic relief. Unfortunately, when used appropriately in pregnant women to treat potentially life-threatening Wilson disease, it is harmful to the fetus and can produce cutis laxa in the newborns of penicillamine-treated patients (see Figure H6.3).

A 43-year-old woman presents for the evaluation of possible seizures. The patient was well until the day of admission to the hospital. She works as a secretary at a law firm. She was having a heated argument about work assignments with her supervisor when she slumped back in her chair, became pale, and then had a 5-10 second spell of generalized tonic-clonic jerking. She was placed on the floor, and she began to rouse after 15-30 seconds. Past medical history reveals a history of 3 generalized tonic-clonic seizures at age 4. Electroencephalogram at that time revealed intermittent generalized spike and wave discharges. She was started on divalproex sodium (Depakote) and had no further seizures. At age 8, a repeat electroencephalogram was normal and divalproex was discontinued. Seizures did not reoccur. She was in a major automobile accident at age 27, was in the hospital for 3 months, and in rehabilitation for another 3 months. She made a full recovery. The patient describes her health now as 'excellent'. Physical examination reveals a well-developed, well-nourished woman; she is awake, alert, fully-oriented, and in no acute distress. Temperature is 98.8 F orally. Blood pressure is 129/85mmHg. Heart rate is 76/min and regular. Respirations are 14/min and unlabored. General neurological and medical exams are normal. Lab studies include a normal complete blood count and differential, blood chemistry panel, urinalysis, chest X-ray, and cardiogram. A portable electroencephalogram is done and reported as normal. Syncope and seizures are being considered, but pseudoseizures are too because of the relationship of this spell to emotional distress. Question What profile is best correlated with the likelihood of pseudoseizures (psychogenic non-epileptic seizures) in patients where pseudoseizures are a consideration? Answer Choices 1 Young (20 - 30 years old) women with history of sexual abuse 2 Middle-aged (40 - 60 years old) men with a history of seizures 3 Young (20 - 30 years old) men with serious medical problems 4 Middle-aged (40 - 60 years old) men with a history of fibromyalgia 5 Middle-aged (40 - 60 years old) males with a history of drug abuse 6 Elderly men with a history of physical abuse

Young (20 - 30 years old) women with history of sexual abuse While pseudoseizures (Psychogenic non-epileptic seizures) can occur at any age, the great bulk of them occur in the 20-30 year old age group. Women predominate (75%) in this age group, although this gender difference fades with years and nears 50% in the elderly. A history of sexual abuse can be obtained in 32% of women with pseudoseizures in this age group. The incidence of reported sexual abuse decreases steadily with age (4% in the over 55 age group). The middle years (20-40) are relatively quiet for the incidence of pseudoseizures. Although the percentage of men among patients with pseudoseizures increases with age, it never reaches 50%. While incidence of true epilepsy is frequent among patients with seizures (it is felt that 5-20% of those with a diagnosis of seizures have pseudoseizures), this correlation is seen mostly in the young and is not a serious factor over 55 years of age (2). Although the 20-30 year old age group contains the bulk of patients with pseudoseizures, the men in this age group are a distinct minority (25%). Among the elderly who have pseudoseizures, there is a relatively high incidence of major health problems (42%), especially serious cardiovascular issues (2), but this is not seen in the young age group. Again, the middle years are relatively quiet with respect to pseudoseizures. Men remain in the minority. However, a history of fibromyalgia does have a predictive value of 85% for pseudoseizures (3). Still again, the middle years are relatively quiet for pseudoseizures and men remain in the minority. There is, however, a strong relationship between drug abuse and pseudoseizures, but not as strong as among the young. While the elderly can have pseudoseizures, it is relatively uncommon. Again, men are in the minority. A history of physical abuse is uncommon in males. Summary of pseudoseizure profile ('risk factors'-none in themselves are absolute indicators) 1) Ironically, a group at increased risk for pseudoseizures is epileptics, where epilepsy and pseudoseizures frequently coexist (5). While 0.5-1.0% of the population may have a diagnosis of epilepsy, it is felt that 5-20% of these actually have pseudoseizures. This association decreases with age. 2) Age; predominantly younger patients (10-30) but can be seen in any age. 3) Gender; 78% of younger patients are women (1), but this difference fades with advancing age. 4) A diagnosis of fibromyalgia has an 8% predictive value that the patient has pseudoseizures (3). 5) A diagnosis of chronic pain syndrome also has a predictive value of 85% that the patient's spells represent pseudoseizures (3). 6) The rule of 2's; a history of 2 normal EEGs, 2 seizures per week, and resistance to 2 antiepileptic drugs have a positive predictive value for of 85%. 7) A history of sexual abuse is seen in about 32% of young women, but much less so in men, and drops to a major degree in both genders with age (2). 8) A history of major health problems becomes increasingly common with age with a cardiovascular predominance (2). 9) A history of illicit drug abuse is common (1). 10) Pending compensation issues (5).

An immigrant worker presents with a chronic cough, hemoptysis, and night sweats. Physical exam reveals a wasted, middle-aged man with bronchial breath sounds in the right upper lobe. The patient is asked for 2 sputum samples; 1 is sent for culture, and the other is directly examined under a microscope. What technique will be the best choice for microscopic examination for rapid initial diagnosis? Answer Choices 1 Gram stain 2 Giemsa stain 3 Ziehl-Neelsen stain 4 Direct immunofluorescence 5 Darkfield examination

Ziehl-Neelsen stain This patient has the clinical features of pulmonary tuberculosis; in the United States, TB is common in immigrants from developing countries. For rapid diagnosis of pulmonary TB, direct sputum examination with Ziehl-Neelsen stain is the best choice. The culture should be sent at the same time to identify the type of Mycobacterium and the sensitivity to the drugs. Mycobacterium are aerobic, acid-fast bacilli. They are neither Gram-positive nor Gram-negative, but stain red with Ziehl-Neelsen stain because of the N-glycolyl muramic (mycolic) acid in their cell walls. The lipid content of the cell wall is very high; therefore, Mycobacterium is hydrophobic and impermeable to basic aniline bacteriologic stains. Gram staining is the most common stain used for most of the bacteria, but it is not used for Mycobacterium. Giemsa stain is used for tissue, blood, or secretions to see the microorganisms. It is commonly used for Neisseria species, malaria, and other blood parasites. Direct immunofluorescence is useful in the identification of many microorganisms such as Bordetella pertussis and Chlamydia trachomatis. Fluorescein-labeled monoclonal or polyclonal antibodies against specific microbial antigens are added to a tissue or cell suspension; this enables it to detect a specific microorganism. Darkfield examination uses a microscope that illuminates the object over a dark background. It is used extensively to identify spirochetes, which are too thin to be seen easily by staining.

A progestin-only contraceptive, or "minipill," would be most appropriate for which of the following patients? A a 25-year-old woman in excellent overall health B a 28-year-old woman with a history of epilepsy C a 32-year-old woman with a history of pelvic inflammatory disease D a 37-year-old woman who smokes 2 packs per day and has a history of hypertension E a 38-year-old woman with a history of asthma and bronchitis

a 37-year-old woman who smokes 2 packs per day and has a history of hypertension In the majority of cases, a combined hormonal contraceptive (ie, one that contains both an estrogen and progestin) is the preferred method of oral contraception because of its efficacy when used perfectly (>99%). However, for women older than 35 years of age who are smokers or are obese, or who have a history of hypertension or vascular disease, progesterone-only contraceptives are recommended. Ethinyl estradiol (EE), the most common estrogen found in combined hormonal contraceptives, has been associated with an increased risk of myocardial infarction in women older than 35 years of age who are smokers. Additionally, EE has also been shown to cause increases in blood pressure in both normotensive and mildly hypertensive women. Progestin-only contraceptives, however, tend to be less effective than the combined hormonal contraceptives.

A 7-year-old male is suspected to have a Wilms' tumor (nephroblastoma). What is the most common symptom at presentation? A Abdominal mass B Hypertension C Hematuria D Coagulopathy E Fever

abdominal mass The diagnosis of a Wilms' tumor is most commonly made after the discovery of an asymptomatic mass, by a family member or a physician, during a routine physical examination. The most common sign is an abdominal mass (A). Other symptoms at presentation include abdominal pain and distention, anorexia, nausea and vomiting, fever (E) and hematuria. Hypertension (B) is seen in 25-60% of cases and is caused by elevated renin levels. As many as 30% of patients demonstrate hematuria (C) and coagulopathy (D) can occur in up to 10%.

A 64-year-old man presents with a history of progressive dysphagia to solids; recently, he has been having trouble with liquids. He also experiences occasional regurgitation of undigested food. His physical exam is unremarkable. A barium esophagram reveals a distinctive bird's beak appearance of the distal esophagus. Question What is the most likely diagnosis? Answer Choices 1 Gastroparesis 2 Gastroesophageal reflux disease 3 Mallory-Weiss syndrome 4 Achalasia 5 Candidal esophagitis

achalasia The correct answer is achalasia; it often presents with progressive dysphagia to liquids and solids as well as complaints of regurgitation of undigested food. Classic radiographic findings include a bird's beak appearance in the distal esophagus, demonstrating the narrowing of lumen. Gastroparesis is delayed gastric emptying, not an esophageal disorder that would cause the progressive dysphagia noted by the patient. The vagus nerve is not functioning properly in these cases. Gastroesophageal reflux disease can cause dysphagia, but it is not typical; this condition causes regurgitation of undigested food and does not have the classic appearance on esophagram. Mallory-Weiss syndrome presents typically with hematemesis, typically due to forceful vomiting or retching, which causes tears in the esophagus. It is associated with alcoholism. Candidal esophagitis is a fungal infection of the esophagus often seen in immunocompromised patients, such as those who are HIV infected. It presents with progressive dysphagia, but oral thrush is often present in these patients, and an esophagram will not show the classic appearance of achalasia.

A 30-year-old female presents to your office for a routine physical exam. She has not seen a health care provider in many years. Upon talking with the patient, you find out that she had been diagnosed with hypertension several years ago, but was unable to afford the antihypertensive medications that were prescribed to her. She has no complaints at this time. Upon exam of the head and neck, you note widened spaces between her lower incisor teeth and a large, fleshy nose. Her skin is oily and she demonstrates mild proximal muscle weakness. Her EKG reveals a left axis deviation and widened QRS. What is the most likely rationale for her clinical presentation? A Diabetes mellitus B Cushing's syndrome C Hypothyroidism D Acromegaly E Clinical depression

acromegaly The correct choice is D, acromegaly. Patients with acromegaly have an abundance of growth hormone secretion. This leads to excessive growth of many areas of the body including soft tissue. Patients with acromegaly also have an increased incidence of hypertension and left ventricular hypertrophy. None of the other choices will cause this patient's constellation of symptoms. Patients with many endocrine disorders may develop weaknesses as seen in this patient, but the large nose and widely spaced teeth are characteristic of acromegaly.

A 23-year-old woman presents with abdominal pain which began last night. It started as a dull ache in the lower abdomen, but it has localized to the right lower quadrant over the past several hours. She has experienced anorexia and 2 episodes of vomiting. She recently entered into a sexual relationship with a new boyfriend. Her last menstrual period was 1 week prior to presentation. On exam, her temperature is 37.8° C, pulse is 90, and blood pressure is 120/72. Abdominal exam reveals a soft abdomen with tenderness to moderate palpation in the right lower quadrant. There appears to be guarding and mild rebound tenderness. Pelvic exam reveals a normal-appearing cervix with clear cervical discharge; there is no cervical motion tenderness. CBC reveals a white blood cell count of 11,800/μL.; urinalysis reveals a few squamous cells, but no bacteria or blood. What is her most likely diagnosis? Answer Choices 1 Acute salpingitis 2 Acute tubo-ovarian abscess 3 Acute appendicitis 4 Renal lithiasis 5 Ectopic pregnancy

acute appendicitis Acute appendicitis can affect all ages and is a common cause of emergency surgery; there is a lifetime incidence of approximately 6%. While appendicitis has some very classic features, it still can be very difficult to diagnose. It is especially dfficult to diagnose in very young or old patients, in patients with concomitant pregnancy, or in patients with other diseases (e.g., diabetes or AIDS). This patient's presentation is classic for appendicitis. The classic presentation is one of anorexia, with abdominal pain that begins in the periumbilical area. The pain becomes steady and migrates to the right lower quadrant in a 24-hour period. Nausea, emesis, and/or diarrhea are common. Rebound tenderness can be present and reflects peritoneal inflammation; rebound is less apparent in elderly or obese patients. Fever may or may not be present, although over 80% of patients with acute appendicitis will have an elevated white blood cell count. Radiographic exams that may be useful, if positive, include plain films of the abdomen (to look for a fecalith or gas in the appendix), a barium enema (to look for non-filling of the appendix), ultrasound (especially in pregnancy, to look for a tender, immobile, non-compressible structure), or a CT scan (to look for an inflamed appendix). However, surgical evaluation with laparoscopy or laparotomy is the definitive test for appendicitis, and is the treatment of choice if suspicion is high. Acute salpingitis and a tubo-ovarian abscess are often high on the differential diagnosis list in a young female with abdominal pain, particularly in this patient with her recent menses and new sexual partner. However, pelvic inflammatory disease is usually associated with cervical motion tenderness; the patient will usually have bilateral lower abdominal pain, since PID is an ascending infection from the cervix up through the fallopian tubes. The normal appearing cervix and cervical discharge also make PID less likely, although this patient should be tested for Neisseria gonorrhoeae and Chlamydia trachomatis during the pelvic exam. Renal lithiasis seems less likely since the patient does not complain of flank pain and has a normal urinalysis. Ectopic pregnancy should be a concern with any sexually active female, but the history of her recent menses makes that diagnosis less likely; a serum pregnancy test is still indicated, however, to entirely rule out the potentially life-threatening entity.

Early one afternoon, a male adolescent presents with abdominal pain, nausea, and vomiting. The pain has been worsening since the onset of symptoms in the morning. There is no known gastrointestinal disease in the history; no one in the immediate environment has one either. Physical examination finds no abdominal tenderness, but Psoas sign and tenderness on rectal examination are detected. Temperature and pulse are slightly elevated. Skin turgor is reduced, and there is a 10 mm Hg drop in postural blood pressure. Laboratory studies find 18,000 white blood cells per microliter. What is the most likely diagnosis? Answer Choices 1 Acute mesenteric lymphadenitis 2 Acute appendicitis 3 Salmonella gastroenteritis 4 Meckel's diverticulitis 5 Regional enteritis

acute appendicitis The course and signs of the disease indicate appendicitis. The lack of abdominal tenderness and positive rectal examination indicate inflammation of retrocecal or pelvic appendix. Salmonella gastroenteritis would likely affect other persons in the child's environment. Regional enteritis is associated with a prolonged history. Acute mesenteric lymphadenitis poses a differential diagnostic challenge more frequently among children than adults. The diagnosis is impossible clinically, although the temperature tends to be higher and the pain is more diffuse. Culture of mesenteric nodes and serologic titers confirms Yersinia infection in some patients. Meckel's diverticulitis is rare, but it is impossible to distinguish from appendicitis. Considering the risk of perforation and limited observation, if unavoidable, overdiagnosis is preferred in cases of suspected appendicitis.

A 60-year-old woman presents with a history of persistent cough. She is confined to her bed; walking over 10 paces causes severe breathlessness. She has no energy to carry out any of her regular activities. She has never smoked, and she drinks the occasional glass of wine. On physical examination, she is found to have decreased breath sounds and dullness to percussion over her right lower thorax. Further evaluation reveals an irregular mass in the periphery of the right lung base with a right sided pleural effusion. A needle is inserted into the pleural space and divulges blood stained fluid. Question If results prove to be a malignancy, what is the most likely sub-type considering she has never smoked? Answer Choices 1 Small cell carcinoma 2 Large cell carcinoma 3 Adenocarcinoma 4 Mesothelioma 5 Squamous cell carcinoma

adenocarcinoma Adenocarcinoma is the correct answer. Adenocarcinoma accounts for about 40% of all lung cancers. While smoking is a strong risk factor, it is the most common lung malignancy in non-smokers. It is seen more commonly in women, and it starts typically at an earlier age. Formerly known as bronchoalveolar carcinoma, adenocarcinoma sometimes produces mucus or other such substances. The peripheral location is also typical, as compared to the more central squamous cell carcinoma. Small cell carcinoma is incorrect. Also known as oat cell carcinoma, it is extremely unusual in non-smokers, and it accounts for about 10% to 15% of lung cancers. They are typically central in location, and they respond poorly to treatment. Large cell carcinoma is incorrect. It accounts for about 10% to 15% of lung cancers. It can occur in any part of the lung, and it tends to grow and spread rapidly. Being more uncommon than adenocarcinoma, in this patient, the likelihood of it being a large cell tumor is lower. Mesothelioma is incorrect. Mesothelioma is a pleural malignancy associated strongly with asbestos exposure. They typically present as pleural effusions with pleural plaques. Smoking is not strongly associated with mesothelioma. Squamous cell carcinoma is incorrect. It accounts for about 25 to 30% of all cases of lung cancer, and it is strongly associated with smoking. Squamous cell carcinoma tends to be more central in location and is most commonly associated with para-neoplastic syndromes.

A 69-year-old edentulous, alcoholic man who lives alone presents for evaluation of a shoulder wound that is not healing well. On physical examination, numerous ecchymoses are noted on the posterior aspect of his legs and thighs. Careful examination of the man's skin reveals minute hemorrhages around hair follicles, and splinter hemorrhages in the nail beds. Laboratory analysis is remarkable for a hemoglobin of 10 (normal 14 - 18 g/dL); no other hematologic abnormalities are noted. Question What treatment should be included in therapy? Answer Choices 1 Administration of factor VIII 2 Administration of iron 3 Administration of vitamin B12 4 Administration of vitamin C 5 Administration of vitamin K

administration of vitamin C The patient suffers from scurvy due to a deficiency of dietary vitamin C. Absence of vitamin C leads to impaired hydroxylation of proline residues in the nascent procollagen chains leading to weakness of blood vessel walls. Clinically, the deficiency syndrome is characterized by perifollicular hemorrhages, fragmentation of hairs, purpura, ecchymoses, splinter hemorrhages, and hemorrhages into muscle. In patients with normal dentition, gum changes (e.g., swelling, bleeding, and loosening of teeth) are also noted. Without supplementation with vitamin C, death may eventually occur. Administration of factor VIII would be indicated for factor VIII deficiency. This deficiency would lead to a prolonged PTT (partial thromboplastin time), which was not noted in this patient. Administration of iron would be of benefit in iron-deficiency anemia, but there is no indication of a hypochromic, microcytic anemia in this patient. The anemia of scurvy is typically normochromic and normocytic due to bleeding. Administration of vitamin B12 would be indicated for a megaloblastic anemia. Although a macrocytic anemia may be observed in scurvy (due to concomitant dietary folate deficiency or perturbations in the folate pool), this patient did not show macrocytosis. Administration of vitamin K would be appropriate for vitamin K deficiency. This deficiency would produce prolongations of the prothrombin time (PT), followed by prolongation of the PTT as the vitamin K-dependent factors (II, VII, IX, X, protein C, and protein S) are depleted.

A 30-year-old man is brought to the emergency department by his wife for abdominal pain, nausea, vomiting, and diarrhea. She says he has been getting "worse and worse" for at least the past two months. He is "too tired" and his "muscles hurt too much" to play golf, formerly their favorite leisure activity. He has been depressed and reticent, instead of his usual talkative self. He won't eat much of anything, even when she cooks his favorite meals and she is sure he has lost considerable weight because his clothes "hang off him." He has refused medical evaluation until today, when the abdominal pain of approximately a week increased and he began having the vomiting and diarrhea. Initial impression is that the man is quite tanned, although it is the middle of winter. Closer examination reveals dark pigmentation in his skin folds and on the buccal mucosa. This presentation most strongly suggests which of the following? A adrenocortical insufficiency B diabetic ketoacidosis C hypercortisolism D hyperglycemic hyperosmolar state E hypothyroidism

adrenocortical insufficiency This is a fairly classic presentation of adrenocortical insuffiency (Addison's disease). Diabetes mellitus, type 1, that has led to ketoacidosis (B) could also present with weight loss, nausea, and vomiting; but is also accompanied by hyperphagia and polydipsia, and does not have the hyperpigmentation associated with Addison's disease. Patients with hypercortisolism (Cushing Syndrome) (C) present with central obesity, hirsutism, thin skin, poor wound healing, and a host of other problems including emotional lability. Those with hyperglycemic hyperosmolar state (D) are typically older, have a high body mass index, and present with lethargy, confusion, and dehydration. Patients with hypothyroidism (E) usually have weight gain and constipation along with lethargy, fatigue, and weakness.

Which of the following statements regarding diabetic medications is/are most correct? A Incretin-mimetics like exenatide commonly result in weight loss. B Thiazolidinediones (TZDs) should be held prior to and for 48 hours after administration of ionidated contrast material. C Sulfonylureas have the highest risk of hypoglycemia of all oral diabetic agents. D Sulfonylureas help preserve beta-cell function. E All of the above.

all of the above Incretin-mimetics act upon the satiety center of the brain and as a result promote weight loss, averaging approximately six pounds. Metformin may cause fatal lactic acidosis when given to patients on concomitantly-administered iodinated contrast dye. Sulfanylureas, in part because of first-pass metabolism, have the highest risk of hypoglycemia of all the oral anti-diabetic agents. TZDs have been found to preserve beta-cell function more so than any other agent.

A 37-year-old female presents to the labor and delivery department complaining of intermittent pain and contractions. Upon arrival, she also complains of vaginal bleeding. She is a G3P2 at 39 weeks gestation; no other prenatal complications are noted. She is a non-smoker. A physical exam reveals the following: P 90, BP 130/80, T 98.7°F, abdomen gravid, positive bowel sounds, and left lower quadrant tenderness noted. A sterile speculum exam reveals the cervix to be dilated 8, fetus is cephalic, and membranes are intact. The fetal monitor reveals heart tones in the 140s with mild, decreased variability and good quality contractions noted. Delivery is felt to be imminent, and vaginal delivery has been determined to be the best course of action. What will likely decrease bleeding and shorten time to delivery? A Increased activity level B Amniotomy C Oxytocin therapy D Epidural placement E IV sedation

amniotomy If the fetus is mature and vaginal delivery (versus c-section) has been determined to be the best course of action, then amniotomy may diminished amnionic fluid volume. This might also allow for better spiral artery compression, and serve to both decrease bleeding from the implantation site and reduce entry of thromboplastin into the maternal circulation.

A 27-year-old man is admitted to the emergency department with frank bleeding per rectum. Large bleeding polyps are noted in the colon, for which a subtotal colectomy is performed. On careful review, it comes to light that both his father and grandfather had problems with colonic polyps, resulting in surgery. Shown below is a picture of his colon. Question What measures should have been initiated in this patient? Answer Choices 1 Annual fecal occult blood testing 2 Annual upper GI endoscopy 3 Annual colonoscopy 4 Annual CEA levels 5 Prophylactic colectomy at 1 year of age

annual colonoscopy The picture shows an excised portion of colon with multiple polyps. With the given history, a diagnosis of Familial Adenomatious Polyposis (FAP) is likely. Annual colonoscopy or sigmoidoscopy should be initiated early in patients with a history of familial adenomatous polyposis. Being an autosomal dominant disease with a high degree of penetrance, screening is essential. Although not clearly defined, most authorities recommend yearly endoscopy, starting with the onset of symptoms or in early teenage years if asymptomatic. A colectomy is usually performed if dysplasia is present or based on clinical judgment. Annual fecal occult blood testing is incorrect. It is not as sensitive as colonoscopy in detecting polyps and does not allow a biopsy. Annual upper GI endoscopy is incorrect. Polyps are usually in the colon. Annual CEA levels is incorrect. Annual CEA levels are not used in screening at all. They are used in the follow up of colonic malignancy. Prophylactic colectomy at 1 year is incorrect. Colectomy is done when dysplasia is noted, or when polyps are numerous.

A 19-year-old woman is forced to seek medical attention by her parents. She is 5'5" in height and weighs 100 lbs. Her parents tell you that she has been dieting seriously for the last 6 months and refuses to eat most meals with the rest of the family. Normally, she eats only small amounts of vegetables and an occasional piece of fruit. Recently though, she has started stuffing herself with junk food for an entire evening, crying herself to sleep, and then exercising ferociously every night for the next week. Her parents say that they are tired of arguing with her about her thinking that she's too fat. Recent complaints of hypothermia, amenorrhea, and fainting spells led them to seek medical care for her. What disorder does this woman display? Answer Choices 1 Bulimia nervosa 2 Anorexia nervosa 3 Pica 4 Rumination 5 Eating disorder NOS

anorexia nervosa Anorexia nervosa is characterized by" A refusal to maintain a minimally normal body weight (defined as 85% of what is expected) An intense fear of gaining weight A disturbance in one's subjective experience of body weight or shape A denial of the seriousness of low weight and self-starvation In females, amenorrhea for at least 3 consecutive cycles 90% of patients reported with this disorder are female, although incidence in males is growing somewhat This case shows evidence of binge eating, but it is considered "binge-eating/purging type" of anorexia and not bulimia; this is due to the low body weight. The purging common to bulimia (self-induced vomiting, misuse of laxatives and diuretics, or enemas) can be present in anorexia also, as may other inappropriate compensatory behaviors (fasting, excessive exercise). Pica and rumination are usually seen earlier in childhood; they involve, respectively, the eating of nonnutritive substances and the regurgitation and rechewing of food. The NOS designation would be used if not all the criteria for any specific eating disorder were met. In this case, it would have been used if all the criteria for anorexia nervosa were met except that she still had regular menses. Refer to the image - anorexic perception. Much research has found that anorexics have a highly distorted perception of what they see in the mirror, which is similar to what happens in body dysmorphic disorder. However, when this distortion occurs only with reference to "fatness," it is classified as an eating disorder.

A 14-year-old girl is brought to the office by her mother. She passed out at gymnastics practice yesterday, and her mother is concerned. She sustained no head injury and regained consciousness immediately. She admits to some dizziness, but states that otherwise she feels fine and is eager to return to practice. The patient appears thin despite wearing baggy clothes. On examination, her skin is dry and cool to touch. There is soft downy hair covering her body. Heart rhythm is regular, but she is mildly tachycardic and slightly hypotensive. Neurological examination is normal. Question What is the likely diagnosis? Answer Choices 1 Cushing's syndrome 2 Addison's disease 3 Anorexia nervosa 4 Bulimia nervosa 5 Absence seizure

anorexia nervosa Anorexia nervosa is correct. The patient is a thin-appearing female who had an episode of syncope. This paired with the presence of soft downy hair over her body (lanugo) is highly suggestive of anorexia nervosa. Participation in sports where thin body image is idealized, such as gymnastics, puts an individual at increased risk for anorexia nervosa. This patient requires careful intervention in order to avoid the complications of anorexia nervosa, including death. Cushing's syndrome is incorrect. Cushing's syndrome is associated with high levels of cortisol. It causes symptoms such as weight gain, striae, round face, fatigue, and poor wound healing. Addison's disease is incorrect. Addison's disease is due to insufficient production of cortisol by the adrenal glands. It is associated with symptoms such as fatigue, nausea, vomiting, and hyperpigmentation of the skin. Bulimia nervosa is incorrect. Patients with bulimia nervosa do not typically experience extreme weight loss. Patients are more likely to present with dental erosion (due to frequent vomiting) and enlargement of the parotid salivary glands. Lanugo would not be present in a patient with bulimia nervosa. Absence seizure is incorrect. The patient did not suffer any seizure-like activity and has symptoms suggestive of anorexia nervosa.

A 48-year-old Caucasian woman with a chronic history of inability to tolerate oral intake is admitted to the hospital for J-tube placement. The patient also complains of a rash that she has developed on her upper extremities. Examination reveals a ring of hemorrhage around hair follicles in the upper extremities. What is the most likely etiology of the rash? Answer Choices 1 Niacin deficiency 2 Cobalamin deficiency 3 Biotin deficiency 4 Ascorbic acid deficiency 5 Riboflavin deficiency

ascorbic acid deficiency Explanation The correct answer choice is ascorbic acid deficiency. This patient has scurvy due to ascorbic acid or Vitamin C deficiency. Patients with scurvy typically report a history of a poor diet (tea and toast), or in this case, malnutrition secondary to a chronic inability to tolerate oral intake of foods. Patients with scurvy also present with skin ecchymosis and the characteristic perifollicular hemmorhage. Note that lysyl oxidase, an important enzyme involved in cross-linking collagen, utilizes Vitamin C. Niacin deficiency usually presents as the 3 D's: diarrhea, dermatitis, and dementia. Patients on corn diets or those with an inability to reabsorb tryptophan (Hartnup disease) present with this condition. Cobalamin deficiency presents with megaloblastic anemia. Patients may also present with neurologic disease (subacute combined degeneration). Biotin deficiency presents with dermatitis, alopecia, and lactic acidosis. Riboflavin deficiency presents with cheilosis, angular stomatitis, and glossitis.

A 16-year-old girl presents with excessive fatigue and constipation that has developed over the past 3 months. Her eyes have become puffy and her weight has increased approximately 4 lbs. Her skin is dry despite daily moisturizer application, and there is considerable hair loss. Her mother is under treatment for rheumatoid arthritis, and her father has hypertension. On physical examination, her thyroid is diffusely enlarged (twice the normal size) with a rubbery consistency. There is a slight peripheral edema and pallor of skin. Thyroid function tests show free T4 levels of 0.2 ng/dL and a TSH level of 96 mlU/L. Question What is the most likely underlying cause of her disease? Answer Choices 1 Thyroid hormone resistance 2 TSH-producing pituitary tumor 3 Autoimmunity 4 Iodine deficiency 5 Impaired thyroid response to TSH

autoimmunity Explanation This patient has a clinical picture that suggests hypothyroidism, which is confirmed by the low levels of free T4 and elevated TSH. A family history of autoimmune disease puts her at an increased risk to develop autoimmune hypothyroidism. Spontaneous hypothyroidism in iodine-sufficient areas is almost exclusively caused by autoimmunity. Hashimoto's thyroiditis and (in some rare cases) TSH-receptor blocking antibodies can also cause hypothyroidism that may be reversible.These blocking antibodies occupy the TSH receptor on the thyroid cells, preventing the gland from receiving TSH signals to produce T3 and T4. Treatment is levothyroxine replacement (1.6 mg/kg body weight), aiming for TSH levels within the low/normal range. Thyroid hormone resistance is a genetically inherited condition in which mutations in the thyroid hormone receptor (THR) gene render the receptor less interactive with other cofactors in the signaling cascade; there can also be a lower affinity for T3, the active thyroid hormone. Most commonly, these patients are asymptomatic and the condition is detected during a routine laboratory analysis or after a family member is diagnosed. Thyroid hormone levels are elevated along with paradoxically elevated TSH levels. There is no current treatment for this condition. TSH-producing pituitary tumors are rare pituitary adenomas that present more commonly as large lesions producing mass effects (e.g., headache, visual field impairment). Patients have elevated TSH and elevated thyroid hormones. The diagnosis is suspected in a patient with signs and symptoms of hyperthyroidism, elevated T3, free T4, and TSH levels, and a pituitary lesion on magnetic resonance imaging. Iodine deficiency resulting in goiter and/or thyroid dysfunction is currently limited to endemic areas located far from the sea and where salt is not supplemented with iodine. Iodine is the main substrate used in the production of thyroid hormones and is mainly supplemented by diet. In its absence, a decreased production of T3 and T4 will result in increased levels of TSH and the development of a goiter. Treatment is with levothyroxine and an adequate supplementation of iodine. Impaired thyroid hormone response to TSH is a condition in which the thyroid responds poorly to TSH due to mutations in the TSH receptor gene. Patients affected by this disorder present in 2 ways: if the resistance is partial, they present with inadequate production of thyroid hormones in lieu of increased TSH levels; if the resistance is complete, they present with congenital hypothyroidism, decreased response to TSH stimulation, and undetectable thyroglobulin levels. Treatment is levothyroxine replacement.

A patient describes a desire for close relationships and to be more successful at work. However, she views herself as being undesirable and inferior. Because of these feelings she avoids social activities and extra occupational projects out of fear of criticism, rejection, and embarrassment. Which diagnosis would best fit this description? A avoidant personality disorder B borderline personality disorder C histrionic personality disorder D schizoid personality disorder

avoidant PD A An individual with avoidant personality disorder differs from schizoid in that they desire interaction and closeness but are unable to overcome their deep seated self-beliefs and fears. They tend to be less impulsive and more stable than borderline personality disorder patients and have less of a need to be the center of attention than those with histrionic personality disorders.

An 18-month-old boy presents with a 1-day history of colicky abdominal pain and blood in the stools. He had been playing normally until the onset of his symptoms. According to his parents, he has vomited several times and has been febrile. The infant has been lethargic for 1 hour. No history of accidental ingestion of drugs or alcohol was elicited. Past medical history is unremarkable. He is on no medications and has no known drug allergies. Physical exam shows a blood pressure of 110/76 mm Hg, pulse of 126 BPM, and a respiratory rate of 20/min. HEENT is within normal limits and his chest is clear with no murmurs, rubs, or gallops in the heart. Rate and rhythm are regular. His abdomen is soft; there is tenderness in the right upper quadrant, diminished bowel sounds, and an ovular mass palpated in the right upper quadrant. Lab results show normal electrolytes and CBC; the drug screen is negative. The infant is given intravenous fluids and nasogastric decompression is done. Question What investigation will help confirm the diagnosis? Answer Choices 1 Barium enema 2 Plain radiograph of abdomen 3 Upper GI endoscopy 4 Colonoscopy 5 CT scan of the abdomen

barium enema Intussusception, in which a part of the bowel telescopes into an adjacent part of the bowel, is the most common cause of bowel obstruction in children between ages 6 months and 3 years of age. Typical presenting signs include intermittent colicky, abdominal pain, vomiting, and bloody, mucousy stools sometimes described as "currant-jelly" in appearance. Some children may present with only irritability and progressive or alternating lethargy. On abdominal examination, a slightly tender, sausage-shaped palpable mass is characteristic. Most cases are idiopathic; however, it is felt that hypertrophied Peyer's patches in the ileum stimulate peristalsis and cause the intussusception. Other causes, usually found in older children, include polyp, lymphoma, intestinal parasites, Meckel diverticulum, intramural hematoma, and hemangioma. Ileocolic intussusceptions are by far the most common, but cecocolic and ileoileal presentations may occur. The telescoping of the bowel causes diminished venous blood flow resulting in edema and hemorrhage, leading to decreased arterial blood flow, ischemia, and infarction. Left untreated, death occurs in most cases. The diagnostic test of choice is to perform a barium enema; it helps in confirming the diagnosis because the enema shows a typical "cervix-like mass" or a "coiled spring" appearance on the evacuation film. Treatment by hydrostatic reduction with the same barium enema is successful in 50 - 90% of cases. Air contrast enemas are being used widely now, but barium enemas are still standard as well. If the intussusception is not reducible by the enema, then surgical treatment is indicated. Imaging with CT scan is usually not indicated for establishing the diagnosis of intussusception, unless a lead point is suspected. A lead point is found in only 10% of children less than 2 years old. Plain radiographs may be normal or show a pattern of small bowel obstruction with no gas in the right colon, indicating intussusception; however, it needs to be followed up with barium enema for confirmation. An upper GI endoscopy would be unlikely to demonstrate any pathology at the ileocolic level, and a colonoscopy would not be as helpful in either identifying the intussusception or providing therapeutic relief.

What is the preferred treatment for infantile intestinal obstruction due to intussusception? Answer Choices 1 Colonoscopy 2 Emergent colon resection 3 Nasogastric decompression 4 Barium or air enema 5 Intravenous fluids only

barium or air enema In cases of childhood intussusception, the most common location is in the right colon. The treatment of choice is a low-pressure barium enema to reduce the intussusception. If this procedure is not successful, then laparotomy with manual reduction is indicated. Reduction of the intussusception is necessary in order to prevent ischemia and gangrene of that segment of intestine.

A 57-year-old African-American woman presents with retrosternal burning sensation. The symptoms started during the day several months ago, and lately they have been occurring at night. Heavy meals and sweets seem to trigger them. She also mentions that swallowing solid foods is uncomfortable and she needs to "wash" everything down with water. The burning starts in the epigastrium, which is sensitive to pressure; it radiates to the lower jaw, and it is accompanied by heartburn. The patient describes her job as very stressful, and she complains about never having enough time for herself. She is 165cm tall, weighs 95kg, her blood pressure reads 145/90, pulse is 75/min, and respiration rate is 23. A stool sample tested positive for hemoglobin. What step should be taken next in order to corroborate the suspected diagnosis? Answer Choices 1 ECG 2 Abdominal ultrasound 3 Test for Helicobacter pylori 4 Esophageal manometry 5 Gastric secretory testing and serum gastrin measuring 6 Barium swallow

barium swallow The symptoms described let you suspect GERD (gastroesophageal reflux disease). Diagnostic measurements depend upon symptoms. If symptoms are of short duration and there are no systemic manifestations, empiric treatment should be tried. With good response to that, chronic maintenance treatment should be considered. Poor response should lead to endoscopy of the upper gastrointestinal tract. With dysphagia, the 1st diagnostic step is barium swallow. If it is normal, treatment should be empiric; if it is abnormal, an endoscopy should be performed. If the endoscopy shows reflux, chronic treatment for GERD will be necessary. With an unclear diagnosis after history and endoscopy, if empiric treatment does not show any response or surgical treatment is considered, esophageal manometry with Bernstein test and 24-hour esophageal pH monitoring need to be performed. An ECG would be appropriate if you suspected angina pectoris as the cause of the symptoms. Angina pectoris, a common manifestation of coronary artery disease, can present as heartburn with radiating pain from the epigastrium to jaw, left shoulder, and/or left axilla. It can be triggered by physical and mental stress, cold temperatures, or heavy meals. One kind of angina, Prinzmetal's angina, typically occurs in bed at night; it is caused by coronary spasms, and it is not precipitated by cardiac work. Dysphagia, however, is not a symptom of angina. When there is no dysphagia, but there are chronic symptoms and/or signs of blood-loss (heme-positive stool, anemia, or hematemesis), an esophagogastroscopy would be proper. Helicobacter pylori (HP) plays a role in gastritis and peptic ulcers. There is no proof of any connection to GERD. Acute HP gastritis is characterized by epigastric pain, nausea, and vomiting. Ulcus duodeni classically presents with burning epigastric pain 1 - 3 hours after meals and at night. It tends to wax and wane over months. Additionally, ulcer patients may present with hemorrhages. Esophageal manometry, which measures intraesophageal pressure, is performed if a patient with symptoms of GERD has a normal upper endoscopy after barium swallow or empiric treatment for GERD. Gastric secretory testing and serum gastrin measuring is indicated in patients with refractory ulcer disease; it is also indicated if Zollinger-Ellison syndrome (caused by a gastrin producing endocrine tumor, which is usually located in pancreas or duodenum) is suspected, or in patients who will undergo elective surgery for duodenal ulcers. Treatment of GERD First line lifestyle modification (avoid spicy foods and foods that delay gastric emptying, smaller meal sizes, weight loss, avoid lying down after meals Second line H2-receptor antagonist for 8 - 12 weeks Third line Proton pump inhibitors Fourth line Surgical fundoplication

A 32-year-old woman presents with a 2-day history of having a vaginal "bump"; the bump is painful to sit on. She has never had this problem before, and she has been monogamous with the same sex partner for 7 years. On physical exam (refer to image), you notice a solitary 2 cm smooth, slightly tender mass at the introitus. A KOH/wet mount demonstrates squamous cells with no white blood cells (WBCs), hyphae, or motile organisms. Whiff test is negative. Question What is the most likely diagnosis? Answer Choices 1 Vaginal candidiasis 2 Herpes simplex, type 2 3 Lichen sclerosis 4 Bartholin gland cyst 5 Vaginal intraepithelial neoplasia (VAIN)

bartholin gland cyst Explanation This patient most likely has a Bartholin gland cyst. The Bartholin glands are located at approximately the 4:00 and 8:00 positions at the introitus, and secretions help with vaginal lubrication. A cyst may form when the gland outlet is obstructed, and occasionally infection develops, leading to an abscess. Both Bartholin cysts and abscesses can be painful. Initial conservative treatment consists of warm compresses to encourage drainage. If this is unsuccessful, incision and drainage is necessary. If Neisseria gonorrhea is implicated as a causative organism, appropriate antibiotic treatment should be prescribed. Vaginal candidiasis (also known as a "yeast infection") would be characterized by symptoms of vaginal pruritus and discharge. Vulvar pain is common, and patients may note pain upon sitting. However, a physical exam would reveal vaginal and vulvar erythema, thick white discharge, and hyphae on the wet mount. The fungal hyphae are especially noticeable when potassium hydroxide (KOH) is applied to the microscope slide to lyse the other cells. Vaginal candidiasis is not associated with a solitary vulvar mass. Herpes simplex, type 2 is typically the strain associated with genital herpes. A symptomatic patient with herpes may describe vulvar lesions causing pain with sitting. When lesions are visible, herpes is characterized by multiple, painful vesicles and/or ulcerated lesions. Herpes lesions are typically much smaller than the described single mass. Lichen sclerosis is a chronic, progressive dermatological problem; it often involves the vulvar and perineal epithelium. It is characterized by pruritus, dyspareunia, and fissuring of the skin. Lichen sclerosis is more common in post-menopausal women, and physical exam reveals thin, white-appearing tissue. Masses are not associated with lichen sclerosis. Vaginal intraepithelial neoplasia (VAIN) is typically located at the upper 1/3 of the vagina. Its presence is usually detected by an abnormality on the Pap pathology from asymptomatic patients. This patient's 2 cm mass at the introitus could potentially be a malignant tumor, but VAIN is an unlikely diagnosis. Bartholin gland carcinoma is a possibility, but it is very rare.

A 73-year-old woman presents to you for the first time for an initial history and physical. She states that over the past year she has been hospitalized twice for pneumonia that required mechanical ventilation for 2-3 weeks each episode and bacteremia requiring several weeks of antibiotics. Which of the following nail abnormalities would you most likely expect to see? Answer Choices 1 Paronychia 2 Beau's lines 3 Pitting 4 Onycholysis 5 Splinter hemorrhages

beau's lines Explanation The correct answer is Beau's lines as this condition is typically seen in patients with a history of severe, systemic illness such as the example given. Paronychia is a bacterial infection of the proximal and lateral nail folds that causes pain and inflammation, typically with pus formation. Pitting is seen in patients with a history of psoriasis and alopecia areata. Onycholysis is the separation of the nail plate from the nail bed and is more commonly seen in women with long nails. Splinter hemorrhages are a characteristic sign in patients with infectious endocarditis.

A 33-year-old African-American woman presents with a new onset of left facial droop. The droop was present when she awoke this morning, and it has worsened since then. She reports that she had been feeling fine. She denies headaches, visual changes, and weakness or tingling in the extremities. Exam reveals an anxious, overweight woman. She is afebrile. Cranial nerve exam reveals weakness of the entire left side of the face, including the forehead. The left eye does not close entirely, and there is a widened palpebral fissure. Sensation is intact. All cranial nerves, other than VII, appear intact. Her tympanic membranes are clear, and there are no skin lesions around the face or head. The rest of her exam is unremarkable. Labs reveal normal CBC, serum electrolytes, thyroid function tests, and liver transaminases. A chest X-ray is clear. Question What is the most likely cause of this patient's facial droop? Answer Choices 1 Bell's palsy 2 Sarcoidosis 3 Ramsay-Hunt syndrome 4 Lyme disease 5 Cerebrovascular accident (CVA)

bell's palsy This patient has a classic presentation of Bell's palsy. Bell's palsy is a sudden, unilateral facial weakness and affects about 1 out of every 60 or 70 persons in their lifetime. The etiology is unknown at this time, but herpes simplex virus type 1 has been associated with some cases. The presumed pathogenesis of the palsy is that inflammation of the facial nerve causes swelling then secondary compression, and ischemia occurs where the nerve courses through the temporal bone. Pain behind the ear may precede the paralysis by a day or 2. Hyperacusis and loss of taste may occur, if the lesion is proximal enough; lacrimation may be affected as well. Sensation is not affected, although patients may experience numbness or heaviness in the face. In some cases, there is a mild lymphocytosis of the CSF. MRI may reveal swelling and uniform enhancement of the geniculate ganglion and facial nerve. Also, in some cases, MRI may reveal a swollen facial nerve entrapped within the temporal bone. Prognosis depends upon the degree of paralysis. Approximately 80% of patients will recover within a few weeks to a month. Of those with an incomplete paralysis in the 1st week, nearly all will experience complete recovery within several months. Of those with a more complete paralysis, like this patient, nerve conduction studies and electromyography can indicate prognosis. The likelihood of completely recovery is 90% if the nerve branches in the face retain normal excitability to supramaximal electrical stimulation; however, the likelihood is only about 20% if electrical excitability is absent. Treatment includes symptomatic care, such as taping the upper eyelid closed during sleep and using lubricating agents in the eye In order to prevent corneal desiccation. Some studies have shown improvement in patients treated with glucocorticoids; other studies showed improvement with prednisone and acyclovir together. Sarcoidosis can cause a form of bilateral facial paralysis (facial diplegia) called uveoparotid fever (Heerfordt's syndrome). Given this patient's African American background, sarcoidosis should be considered as a possible etiology of her symptoms. However, the unilateral nature of the paralysis, combined with a negative chest X-ray, makes sarcoidosis unlikely. A serum angiotensin converting enzyme (ACE) could be drawn if sarcoidosis is a concern. Ramsay-Hunt syndrome is a facial palsy associated with a vesicular eruption in the pharynx, external auditory canal, and other surrounding areas of skin. This is thought to be secondary to herpes zoster infection of the geniculate ganglion; often the 8th cranial nerve is affected, as well. This patient did not have any skin lesions around the ear, thus making this diagnosis very unlikely. However, this demonstrates the importance of a thorough examination in facial palsy patients, including careful examination inside the ears. Lyme disease is a frequent cause of facial paralysis in areas where Lyme infection is endemic. However, the majority of patients who have facial palsy caused by Lyme disease note an antecedent rash adjacent to the site of a tick bite. Blood tests for Lyme disease can help identify that as an etiology. However, this patient denied any risk factors for tick bites such as camping/ travel, making this diagnosis less likely. The family history of CVA and the fact that she is currently taking oral contraceptives do confer some increased risk for thrombotic events in this patient. However, with a stroke, the forehead is usually spared from facial palsy, since the upper facial muscles (the frontalis and orbicularis oculi muscles) are innervated by corticobulbar pathways from both motor cortices. The lower facial muscles are innervated only by the contra-lateral hemisphere. This patient's facial palsy is complete and includes the forehead because the facial nerve itself is affected in Bell's palsy.

You are caring for a 33-year-old G3P2 women. Her PMH is significant for obesity and allergies. She receives routine obstetrical care. When is she due for her glucose challenge testing to rule out gestational diabetes? A Between 12 and 16 weeks gestation B Between 16 and 20 weeks gestation C Between 20 and 24 weeks gestation D Between 24 and 28 weeks gestation E Between 28 and 32 weeks gestation

between 24 and 28 weeks gestation Routine surveillance for gestational diabetes of a pregnant women is between 24 and 28 weeks; it is not modified due to obesity or other risk factors for diabetes. Gestational diabetes is a hormone-mediated intolerance. Surveillance is modified in the presence of pre-existing diabetes.

A 32-year-old woman is 2-hours status post cesarean delivery of a twin gestation at 36 weeks. Her pregnancy was uncomplicated. She presented in early labor, which became prolonged despite oxytocin infusion. A cesarean section was performed when her labor became non-progressive. In the recovery area, she notes nausea and lightheadedness. On exam, her heart rate is 133 beats per minute, and blood pressure is 76/42 mm Hg. Significant vaginal bleeding is noted; abdominal palpation reveals a soft uterus. Question What intervention would be most appropriate? Answer Choices 1 Administration of intravenous magnesium 2 Bimanual uterine massage 3 Immediate surgical exploration 4 Transfusion with fresh frozen plasma 5 Transvaginal ultrasound

bimanual uterine massage Explanation The most likely cause of bleeding in this patient is uterine atony. Uterine atony occurs when the uterine myometrium fails to contract following delivery. Contractions of the uterine muscles after delivery normally tamponade bleeding from uterine arterioles. Absence of this response causes continued bleeding, which usually becomes evident early after delivery. Bimanual uterine palpation revealing a soft, 'boggy' uterus confirms the diagnosis. Initial treatment involves bimanual uterine massage, which helps promote uterine contractions. In addition, uterotonic agents are administered. The first-line of therapy is intravenous oxytocin. Second-line therapies including ergot alkaloid derivates and prostaglandins (e.g., Hemabate) are used when oxytocin therapy is unsuccessful. When these measures are unsuccessful, surgical interventions are required, the most common of which is bilateral uterine artery ligation. Uterine atony can also occur as a result of retained products of conception; these products inhibit uterine contraction. In this case, manual or surgical extraction is necessary. Intravenous magnesium is a uterine relaxant used to inhibit uterine contractions in settings of premature labor. It has the opposite effect than that desired to treat uterine atony. Transfusion with fresh frozen plasma may be indicated as a temporizing measure in the setting of disseminated intravascular coagulation accompanied by severe bleeding or massive uncontrolled hemorrhage from uterine rupture, neither of which is suspected in this clinical scenario. Transvaginal ultrasound is rarely needed to diagnose postpartum hemorrhage, and it is not the most appropriate next step in light of the patient's condition.

A 60-year old woman has a history of stress related migraine headaches. Many conventional treatments have failed. What treatment would you now try on this client? Answer Choices 1 Aversion therapy 2 Biofeedback 3 Flooding 4 Systematic desensitization 5 Token economy

biofeedback There are several different types of behavioral therapies, and each one is unique in regards to the type of problem being treated. Biofeedback is a form of behavior therapy involving relaxation and the power of suggestion. It is used to treat psychosomatic-related ailments such as asthma, depression, migraine headaches, hypertension, stress, chronic pain, and digestive problems. Token economy involves the use of rewards to condition behavior. These rewards may include money, privileges, food passes, and gifts. This therapy is utilized a lot in inpatient or residential settings, such as prisons, mental hospitals, nursing homes, and substance abuse treatment centers. Aversion therapy is a type of conditioning in which a negative stimulus is connected with an undesired behavior. For example, to assist an individual to stop smoking, one might add an unpleasant taste or smell when a smoker lights up a cigarette. This type of therapy is used frequently in smoking, weight loss, or other undesired behaviors. Systematic desensitization is a behavioral technique in which a client with some type of avoidant behavior linked to a specific stimulus is asked to construct a hierarchy of anxiety-provoking images until the avoidant behavior diminishes. Examples of fears the technique may work on include heights, snakes, airplane travel, elevators, escalators, and many others. Flooding is a technique in which the client is exposed to the highest possible anxiety-provoking experience related to whatever they fear the most. For example, instead of proceeding slowly as in systematic desensitization, the client is immediately exposed to a tall building, snake, elevator, escalator, or whatever they fear the most. This is a highly controversial therapy, and it should only be used with a great degree of caution and knowledge. Treatment of major depression in older adults is most successful with a combination of antidepressant medications plus interpersonal supportive psychotherapy. Behavioral, cognitive, and family therapy has shown less promising results. Brief or short therapy has shown to be as effective, or more effective, compared to long-term psychotherapy. In addition, most insurance programs do not fund long-term therapies. Older adults with adjustment reactions usually have much less severe symptoms, and these usually occur after a major stress such as a death, change in residence, or other psychosocial stressors.

A 62-year-old woman presents with bright red gross hematuria for the past 2 months. She states it is painless, but persistent. She denies other symptoms. She has no chronic medical problems. Upon further questioning, she admits to a 50-pack year smoking history, and she states she is currently retired from her job in a rubber factory. Vital signs are within normal limits, and physical exam is normal. Urine dipstick only shows too numerous to count RBCs, and urine cultures are negative. Question What is the most likely diagnosis? Answer Choices 1 Acute cystitis 2 Bladder cancer 3 Renal cell carcinoma 4 Urethritis 5 Ureteral calculi

bladder cancer Hematuria is the most common presenting sign of urinary tract cancer, and bright red gross hematuria is usually of lower urinary tract origin. Silent or painless hematuria suggests tumor or renal parenchymal disease; therefore, the clinical picture points to cancer, with bladder cancer as the most likely diagnosis. Smoking and exposure to industrial dyes or solvents (like in a rubber plant) are risk factors for bladder cancer. Bladder cancer is the 2nd most common urologic cancer, and the mean age at diagnosis is 65 years. It is more common in men than women (2.7:1), and 98% of primary bladder cancers are epithelial malignancies (majority urothelial cell carcinomas). Ordering cytology of the urine sample is often helpful with higher grade and stage bladder cancers. Patients can become anemic with chronic blood loss, so a CBC is justified. Diagnosis is made by cystoscopy with biopsy. Acute cystitis typically presents with irritative voiding symptoms (frequency, urgency, dysuria) and suprapubic discomfort in addition to possible hematuria. Urinalysis will show pyuria, bacteriuria, and varying degrees of hematuria. Urine cultures will show specific organisms. Renal cell carcinoma can present with flank pain, hematuria, persistent back pain, and an abdominal mass; also, it can be found incidentally on CT scan. RCC is more common in men than women (2:1), and it has a peak incidence in the 6th decade of life. This could be a possible option for diagnosis, but the significant history of smoking and previous work history points more toward bladder cancer. Urethritis is inflammation of the urethra that presents with urethral discharge, dysuria, and itching. Urethritis is most often caused by a STD. This patient does not have a history of unprotected sexual intercourse or any other symptoms that would indicate this diagnosis. Ureteral calculi can present with hematuria, but it also typically presents with flank or abdominal pain as well. If the stone is in the ureter, it often causes some hydroureter with or without hydronephrosis, both of which cause some pain or discomfort. The patient has no past history of forming stones.

A previously healthy 26-year-old primiparous woman presents after 2 days of prolonged rupture of the membranes at 36 weeks of gestation with breech fetal presentation. Because of frequent fetal heart rate decelerations, the child is delivered by caesarean section. Prophylactic antibiotics were not given. Post-operatively, the woman starts complaining of increasing fatigue and muscle pain. On the 3rd day post op, she starts experiencing headache, insomnia, anxiety, generalized abdominal pain, and fever. Her temperature is 41 C. Pulse rate and respiration rate are increased (120/min and 20/min respectively). On examination, there are no signs of infection of the abdominal wall; however, there is a red rash, with swelling and tenderness, on both of her thighs. Question What is your initial step? Answer Choices 1 Antibiotics IV 2 Blood culture x 3 3 Painkillers as needed 4 Ultrasound examination 5 Antipyretics IV

blood cultures x 3 Explanation The patient most likely has puerperal sepsis. Puerperal sepsis is an infection of the genital tract that occurs at any time between the onset of the rupture of the membranes or labor and the 42nd day postpartum. Diagnostic essentials are also systemic signs of infection, abdominal and/or pelvic pain, vaginal discharge, peritoneal signs, abnormal smell/foul odor of discharge, and a delay in the rate of reduction of size of uterus (<2 cm/day during first 8 days). Collecting blood cultures prior to antibiotic administration is mandatory to identify the organism that caused sepsis. Most organisms that cause puerperal sepsis are considered normal vaginal flora, but they can cause puerperal infection in a patient with predisposing factors like: prolonged (more than 24 hours) or traumatic labor, Caesarean section, frequent and unsanitary vaginal examinations, unsanitary delivery practices, retained products of conception, hemorrhage or certain maternal conditions (anemia, malnutrition etc.), and prolonged and premature rupture of the membranes. Premature rupture of membranes happens when rupture of the membrane of the amniotic sac and chorion happens more than 1 hour before the onset of labor; it is prolonged when it occurs more than 18 hours before labor, and it is preterm when it occurs before 37 weeks gestation. This patient was at high risk for developing puerperal sepsis, and she was a good candidate for prophylactic antibiotic therapy (it was not given). You should definitively give antibiotics in the case of sepsis, but failure to check blood cultures prior to an antibiotic regimen can affect the growth of blood borne bacteria and prevent a culture from becoming positive later. You may give her painkillers, but it is essential that you deal with the cause of the sepsis. Ultrasound examination should be considered after you stabilize the patient - it is not your 1st step in the management. Antipyretics are also indicated in this patient, but blood culture is the 1st step in management of sepsis.

A 56-year-old chronic alcoholic presents with signs of anemia. Laboratories reveal a moderate anemia with a hematocrit of 30% (45 to 62%) and a dimorphic population of red cells, one normal and one hypochromic. The MCV (mean corpuscular volume) is normal, the serum iron level is elevated, and the transferrin saturation is high. The diagnosis was made using which diagnostic study? A 24-hour urine for porphyrin B Bone marrow biopsy C Echocardiogram D Ferritin level E Liver biopsy

bone marrow biopsy Diagnosis of sideroblastic anemia is made by examination of the bone marrow, using Prussian blue staining and noting the presence of ringed sideroblasts, which are cells with iron deposits encircling the red cell nucleus. None of the other studies are useful in making this diagnosis.

A 31-year-old woman presents with fever, malaise, and a mass in the right breast. On physical examination, a 4 cm (in diameter) cystic mass is present in the right breast under the nipple. The mass is tender to palpation. Her vital signs are pulse 95/min, BP 120/80 mm Hg, and temperature 99.9°. A fine needle aspirate is performed on the breast cystic lesion. A representative field of the aspirated material is represented in the image. What is the most likely diagnosis? Answer Choices 1 Breast abscess 2 Fat necrosis 3 Fibroadenoma 4 Ductal adenocarcinoma of the breast 5 Lobular adenocarcinoma of the breast

breast abscess Explanation The aspirated material is predominantly purulent with numerous polymorphonuclear leukocytes, debris, and few scattered benign epithelial cells. Abscess of the breast occurs commonly during lactation due to injury to the mammary ducts. Microscopically, the abscess cavity consists of numerous polymorphonuclear leukocytes and secretions surrounded by inflamed and (eventually) fibrotic breast tissue with obliteration of the lobular pattern. Clinically, a localized abscess may mimic carcinoma. The abscess is treated by incision and drainage with administration of antibiotics. Fat necrosis most often presents with a hard mass in the breast that can mimic carcinoma; it results from trauma to the breast or following a surgical procedure. A history of trauma can be elicited in 1/2 of the cases, usually 1 - 2 weeks prior to the occurrence of the fat necrosis. Microscopically, it is characterized by broken-down fat surrounded by clusters of foamy macrophages, with the presence of occasional hemosiderin-laden macrophages and dystrophic calcification. Fibroadenoma is a common benign breast lesion, usually occurring in patients between the ages of 20 and 35 years. It increases in size during pregnancy and tends to regress as the age of the patient increases. It is usually a single lesion, but in 20% of the cases there are multiple lesions in the same breast or bilaterally. Microscopically, the fibroadenoma varies from case to case depending on the relative amounts of glandular and stromal tissue. The fine needle aspirates of fibroadenoma show branching monolayers of epithelium, with well-defined outlines and stromal elements represented by amorphous background material and numerous naked oval stromal nuclei in the background. The epithelial cells are cytologically bland. Breast adenocarcinoma, whether ductal or lobular, is ruled out by the inflammatory background and the benign bland cytologic appearance of the few epithelial cells present in the smear.

You note an irregular hard mass with ill-defined margins on the right breast of a 37-year-old woman. In addition, the nipple has bloody discharge. What is the most likely diagnosis? Answer Choices 1 Mastitis 2 Breast cancer 3 Paget's disease 4 Fibroadenoma

breast cancer Explanation Breast cancer is manifested as a single, irregular, stellate, fixed and firm nodule as is presented in the scenario. A mastitis is warm and erythematous along the ducts, and it is usually associated with breast feeding. Paget's disease presents with a nodule at the epidermis of the nipple/areola, with an eczema rash, scaling, and excoriation. Fibroadenoma is a single, nontender, freely movable, round, and lobular nodule; it typically has clear margins.

A 19-year-old woman presents to the Accident and Emergency Department following a road traffic accident 20 minutes prior. She states her car was just lightly bumped from behind, but she is concerned she might have whiplash, as her neck is stiff and sore. She also says she's experiencing tingling in her left arm down to her thumb. On physical examination, her bicep strength is +3/5 on the left and +5/5 on the right. Her bicep tendon reflex is 2+ on the left and 2+ on the right. In addition, her brachioradialis tendon reflex is 1+ on the left and 2+ on the right. The remainder of the physical examination is normal. Question Based on the above presentation, what cervical nerve root is most likely affected? Answer Choices 1 C4 2 C5 3 C6 4 C7 5 C8

c6 The most likely nerve affected in this case is the C6 nerve root. Impingement of this nerve can cause numbness and tingling down the arm into the thumb, with weakness in the bicep muscle and diminished brachioradialis tendon reflex in the affected extremity. Impingement of the C4 nerve root may cause neck and upper shoulder numbness and pain. Impingement of C5 nerve root may cause deltoid and shoulder numbness and pain, and bicep tendon reflex may be diminished. Impingement of C7 also causes numbness and pain down the affected arm, but reflex may be diminished into the middle finger and the triceps,. Impingement of C8 may cause numbness and tingling primarily on the lateral surface of the arm and into the lateral hand into the fourth and fifth digits. It may also cause dysfunction of the hand as it innervates the small hand muscles.

Which of the following drugs can cause syndrome of inappropriate antidiuretic hormone (SIADH)? A carbamazepine B glyburide C lithium carbonate D metoprolol

carbamazepine Many medications can enhance the release or potentiate the effects of ADH. Carbamazepine may increase ADH release.

A 73-year-old man presents after a 15-minute episode of right eye vision loss, which he described as being "like a shade being pulled down". What diagnostic test is most likely to be abnormal? Answer Choices 1 Carotid ultrasound 2 Computed tomography 3 Electrocardiogram 4 Erythrocyte sedimentation rate 5 Electroencephalogram

carotid u/s The case described above is a classic example of amaurosis fugax, which is caused by transient occlusion of the ophthalmic artery, a branch of the internal carotid artery. This occurs most often in the setting of internal carotid artery stenosis. Like other transient ischemic attacks, it is often a harbinger of an impending stroke.

A 56-year-old male patient is diagnosed with prostatitis. Which of the following is the least appropriate antibiotic to prescribe in the family practice setting? A ceftriaxone B doxycycline C levofloxacin D trimethoprim-sulfamethoxazole (TMP-SMX) E All are appropriate antibiotics for this patient in this setting.

ceftriaxone A The response to antibiotics in acute bacterial prostatis is usually prompt, perhaps because drugs penetrate readily into the acutely inflamed prostate Antibiotic selection should be guided by results of urine cultures and susceptibility results. Appropriate empiric antibiotics include a fluroquinolone (i.e.levofloxacin 500 mg once daily) or TMP/SMX (one double-strength tablet every 12 hours). Patients who are too ill for oral therapy or are septic on presentation should be hospitalized for initial parenteral treatment (intravenous quinolones with or without an aminoglycoside). Ceftriaxone would not be recommended as first-line.

A 4-year-old boy presents with erratic movements of his left arm and leg. These movements have been present since birth and occur on the left side. They have not been getting worse. The boy's mother admits that he has been using his right hand for almost all activities since the age of 1. Further questioning reveals that the child exhibited delayed milestones, such as rolling over at 6 months, sitting up at 1 year, and walking at 2 years. He does not speak, and he does not respond to his mother's verbal commands. His physical exam is noteworthy for spasticity, hyperreflexia, ataxia, involuntary movements, and weakness of his left arm and leg. His left arm and left leg are measured to be shorter and are atrophied in comparison to the right. Additionally, the child does not respond to verbal stimuli. Question What is the most likely diagnosis? Answer Choices 1 Duchenne muscular dystrophy 2 Lesch-Nyhan syndrome 3 Cerebral palsy 4 Huntington's disease 5 Rett syndrome

cerebral palsy The most likely diagnosis in this patient is cerebral palsy. Cerebral palsy is a chronic, nonprogressive impairment of muscle tone, strength, coordination, or movements. Cerebral palsy is due to cerebral insult or injury before birth, during delivery, or in the perinatal period. The most common form of cerebral palsy (75% of cases) involves spasticity of the limbs. Depending on the type and severity of the motor deficits, associated neurologic deficits or disorders may occur as follows: seizures in up to 50%; mild mental retardation in 26%; and severe retardation in up to 27%. Disorders of language, speech, vision, hearing, and sensory perception are found in varying degrees and combinations. Gross motor milestones of concern with cerebral palsy include head control at age 2 months, rolling at age 4 months, sitting at age 6 months, and walking at age 1 year. Infants with cerebral palsy may have significantly delayed gross motor milestones or show an early hand preference when younger than 1.5 years, suggesting the relative weakness of 1 side. While this patient demonstrates a hemiplegic pattern, other patterns are possible, such as monoplegia or quadriplegia. The findings on physical examination in those with spastic cerebral palsy are variable and are predominantly those of spasticity, clonus, hyperreflexia, ataxia, and involuntary movements. Physical indicators of cerebral palsy include joint contractures secondary to spastic muscles, growth delay, and persistent primitive reflexes. Microcephaly is frequently present. In patients with hemiplegia, the affected arm and leg may be smaller and shorter than the unaffected limbs. Duchenne-type muscular dystrophy, which is also called pseudohypertrophic muscular dystrophy, is a progressive disease that affects boys. Early signs of disease include pseudohypertrophy of the calf, planovalgus deformity of the feet, and proximal muscle weakness. Muscle weakness in the hips may be exhibited by the Gower sign, in which the patient uses the arms to support the trunk while attempting to rise from the floor. Other signs are hesitance when climbing stairs, acceleration during the final stage of sitting, and shoulder weakness. Lesch-Nyhan syndrome is characterized by hyperuricemia, self-mutilative behavior, choreoathetosis, spasticity, and mental retardation. Hyperuricemia results from urate overproduction and can cause uric acid crystalluria, nephrolithiasis, obstructive uropathy, and gouty arthritis. The onset of Huntington's disease is usually between 30 and 50 years of age. The initial symptoms may consist of either abnormal movements or intellectual changes, but ultimately both occur. The earliest mental changes are often behavioral. Symptoms may include irritability, moodiness, antisocial behavior, and a psychiatric disturbance. A more obvious dementia subsequently develops. Dyskinesia may initially manifest as fidgetiness or restlessness, but eventually choreiform movements and some dystonic posturing may occur. Progressive rigidity and akinesia (rather than chorea) sometimes occur in association with dementia, especially in cases with childhood onset. Rett syndrome affects 1 of every 10,000 to 15,000 girls. After 6 to 18 months of normal development, motor skills and mental abilities seem slowly to regress. Certain handwringing and other stereotyped hand movements appear as the disease progresses and are characteristic of the disease.

A 72-year-old male is found to have an absolute lymphocyte count of 13,500 (1000 to 3500). Marrow examination demonstrates infiltration with leukemic lymphocytes, with a lacy or interstitial pattern. What is the most common physical finding that would be associated with this patient? A Cervical and supraclavicular lymphadenopathy B Proptosis and headache C Rhinitis and polyneuropathy D Splenomegaly and hepatomegaly E Weight loss and night sweats

cervical and supraclavicular lymphadenopathy

A 19-year-old college freshman living in a dormitory presents with fever and chills, intense headache, vomiting, and stiff neck. His temperature is 39.2°C and pulse 140/min. After examination, the clinical diagnosis of meningitis is made. Lumbar puncture reveals fluid that is turbid and under pressure. The cerebrospinal fluid (CSF) and blood are sent for microbiological studies. The patient is placed on intravenous ceftriaxone. Gram stain of smears made from centrifuged deposit of CSF show plenty of polymorphonuclear leucocytes, intracellular gram-negative diplococci, and a few extracellular Gram-negative diplococci. A culture of CSF on chocolate agar media incubated at 37°C in presence of 5% CO2 grows colonies of Gram-negative diplococci that are oxidase-positive. By conventional tests and agglutination reaction, the isolated pathogen is identified as Neisseria meningitidis group B. Question What prophylactic measure should be taken by other students staying in the dormitory? Answer Choices 1 Chemoprophylaxis with vancomycin 2 Immunization with tetravalent meningococcal polysaccharide vaccine 3 Chemoprophylaxis with oral penicillin 4 Chemoprophylaxis with rifampin 5 Immunization with tetravalent meningococcal conjugate vaccine

chemoprophylaxis with Rifampin At present, chemoprophylaxis is the only prophylactic measure available against N. meningitidis group B. Among the drugs listed, rifampin is the one used for the purpose. Vancomycin is not effective against most Gram-negative bacteria, including neisseria species. Oral penicillin does not eradicate N. meningitidis from the nasopharynx. The tetravalent vaccines do not contain group B polysaccharide and have no effect against group B meningococcal infection. Close contacts of the patient with meningococcal disease are at greater risk of acquiring the infection. Prophylaxis is indicated in those individuals and in populations with increased carrier rates. Nasopharyngeal mucosa is the site of colonization of N. meningitidis. Overcrowding, as is often found dormitories, causes an increase in carrier rates. Prevalence of carriage is reported to be highest among adolescents and young adults. Transmission occurs through direct contact with large droplets of respiratory secretions from patients or asymptomatic carriers. After colonization, development of invasive disease depends on the strain, environmental, and host factors. Chemoprophylaxis eradicates the carrier state and thus prevents spread of infection. It should be administered as early as possible, ideally within 24 hours of identifying the index case, in order to be effective in preventing secondary cases. Rifampin is one of the drugs commonly used. Mass prophylaxis with rifampin during large outbreak settings is a cause for concern as it might lead to circulation of rifampin-resistant strains. However, mass chemoprophylaxis with rifampin might be considered in outbreaks involving limited populations, such as in a dormitory, single school, or college. It would be especially advisable in cases caused by serogroup B, for which an effective vaccine is not yet available. Rifampin resistance develops due to mutations in the rpoB gene. Alternative drugs that are recommended for chemoprophylaxis are ciprofloxacin and ceftriaxone. Ceftriaxone is administered intramuscularly. Ciprofloxacin is not indicated in small children. N. meningitidis is now the leading cause of bacterial meningitis in the US, and each year 1,400-2,800 cases occur. More than 98% are sporadic infections. Localized outbreaks involving colleges, schools, and nursing homes have been reported. Groups B, C, and Y are major causes of meningococcal disease in the US, and group B is responsible for approximately 30% of cases. The disease affects mainly infants, adolescents, and young adults. No vaccine is yet available against N. meningitidis group B. Development of effective polysaccharide vaccine is hindered by the poor immunogenicity of group B polysaccharide. The lack of immunogenicity is due to immunological cross reactivity with fetal neural antigens. A vaccine containing group B polysaccharide might trigger autoimmunity because of the molecular mimicry. Vaccines using non-capsular antigens like outer membrane proteins (outer membrane vesicle vaccine) and factor H-binding protein (multi-component group B recombinant protein vaccine) are under development. These require standardization and validation. Vancomycin is not used for chemoprophylaxis of meningococcal infection. Most of the Gram-positive normal flora is susceptible to this drug. So this antibiotic is used as a constituent of modified Thayer martin medium for selective isolation of pathogenic neisseria. The medium can be used for isolation of N. meningitidis from nasopharyngeal swabs for detection of carriers. Immunization with tetravalent meningococcal polysaccharide vaccine (MPSV4) is not useful as a measure of prophylaxis for infection by group B meningococcus. The preparation contains only polysaccharides from groups A, C, Y, and W-135 and protects against infection by these 4 serogroups. MPSV4 is effective in older children and adults, and it is not immunogenic in children less than 2 years of age. Oral penicillin is not used for chemoprophylaxis of meningococcal meningitis, although most strains of meningococci are sensitive to penicillin. It is not found effective in eradicating meningococci from the nasopharynx. Tetravalent meningococcal conjugate vaccine (MCV4) is not useful against group B meningococcal infection. It contains polysaccharides from A, C, Y, and W-135 serogroups of N. meningitidis conjugated with diphtheria toxoid, and it is useful for preventing infection by these 4 serogroups. This vaccine is considered more effective than MPSV4. It stimulates both B-cell and T-cell dependent immune responses. As a result, the vaccine induces a strong booster effect that confers long-term protection, and it helps in reducing nasopharyngeal carriage. It is also reported to be safe and immunogenic in 2- to 10-year-olds. Centers for Disease Control and Prevention recommends this vaccine for routine immunization of adolescents, college freshmen living in dormitories, military recruits, and other populations at increased risk of developing invasive meningococcal disease. MCV4 has been licensed for use in individuals 11-55 years old.

A 55-year-old man starts to develop behavioral and mental changes. In addition, he is having coarse, spasmodic, involuntary movements that involve his face and his extremities. What is the classification of this abnormal movement? Answer Choices 1 Chorea 2 Athetosis 3 Ballism 4 Asterixis 5 Dystonia

chorea Chorea is a spasmodic involuntary movement that involves the face and limbs. Chorea can be seen with Huntington's disease (Huntington's chorea). Athetosis is slow, writhing, twisting involuntary movement. Almost all parts of the body can be affected by athetosis. Speech can also be affected when the athetosis is affecting the tongue. Depending on the characteristic of the movements that are seen, it is sometimes modified and called choreoathetosis or athetotic dystonia. Athetosis can be seen with cerebral palsy and some other conditions. Ballism is a flailing, jerky movement of the limbs. It affects the proximal muscles. If the abnormal movement is on one side, it is called hemiballism. Asterixis is also called "liver flap". It is a flapping movement of the hands or arms. During the physical examination to test for asterixis, have the patient hold up his hands (dorsiflex) and spread his fingers. Asterixis is seen with metabolic encephalopathies. Dystonia is an abnormal movement where there is abnormal contraction of muscles. Dystonia can be divided into primary and secondary. Spasmodic torticollis is a form of dystonia.

A hydatidiform mole is removed from the uterus of a 20-year-old woman. Subsequent to evacuation, her serum human chorionic gonadotropin (hCG) concentrations are monitored, which continue to rise. What is most likely diagnosis? Answer Choices 1 Choriocarcinoma 2 Ectopic pregnancy 3 Ovarian failure 4 Pituitary insufficiency 5 Sarcoma of the uterus

choriocarcinoma Explanation Hydatidiform mole is a form of gestational trophoblastic disease. It is a benign proliferation of trophoblastic tissue with cystic (hydropic) swelling of chorionic villi. It commonly occurs in women younger than 20 or over 45. Once a hydatidiform mole has been diagnosed, it must be removed. This can be done by either evacuating the uterus (an option preferred in younger women) or by hysterectomy. Since most trophoblastic cells secrete hCG, its levels are indicative of tumor mass, a persistently high or increasing level of hCG after removal of the mole indicates development of an invasive mole or choriocarcinoma. Hydatidiform moles are essentially due to a failed pregnancy; a second, ectopic pregnancy is extremely unlikely. Since only trophoblastic tissue produces hCG, none of the other mentioned conditions would cause an increase in its level.

The placenta has 2 portions that are identified with either the fetus or the mother. Each portion has its origin from different cells or tissues. Which of the following is the fetal portion of the placenta? Answer Choices 1 Chorion 2 Amnion 3 Yolk sac 4 Allantois 5 Decidua

chorion Explanation The fetal portion of the placenta is derived from the chorion which contains the chorionic plate and chorionic villi. The maternal portion is formed by the decidua basalis. The yolk sac develops in the first trimester to aid in the transfer of nutrients. It also has a role in blood development, is the primitive gut, and the walls contain the primordial germ cells. Usually the yolk sac detaches by the 6th week, but can persist in the adult as Meckel's diverticulum. The decidua is the endometrium found in the pregnant uterus and is shed at partition. There are several forms of decidua that occur during pregnancy and a decidual cast that can occur independent of pregnancy. The amnion can be found surrounding the fetus and amniotic fluid. It is derived from trophoblast and can be seen in the first trimester on a sonogram. Allantois is found in most mammals and serves a different function in each species. In humans its blood vessels give rise to those found in the umbilical cord. In the term fetus the allantois becomes the median umbilical ligament.

What is the most common pulmonary complaint of a patient with tuberculosis? A Hemoptysis B Chronic cough C Wheezing D Dyspnea E Chest pain

chronic cough Chronic cough is the most common complaint. Hemoptysis can occur but is rare, and a productive cough becomes more prevalent after the chronic cough has been established for a while.

A 52-year-old man presents with vomiting and epigastric distress for the past few hours. He has been drinking alcohol for over 20 years, and he has been a moderate-to-heavy drinker. 5 years ago, he was diagnosed with a "gastric/duodenal ulcer", for which he has been taking cimetidine and antacids. The pain now radiates towards the left along the costal margin. He has noticed his appetite has been reduced lately, and his stools are bulky and foul smelling. His friends have commented on his sickly look and weight loss. What is the most likely diagnosis? Answer Choices 1 Chronic pancreatitis 2 Acute recurrent pancreatitis 3 Myocardial infarction 4 Acute cholecystitis 5 Left sided ureteric colic

chronic pancreatitis Chronic pancreatitis is common in alcoholics when pancreatic proteins become denatured and cause destruction of glandular and ductal tissue in the pancreas. There is recurrent epigastric or upper abdominal colicky pain; it radiates towards the left, usually along left costal margin in the direction of the tail of the pancreas. There may be nausea and vomiting along with weight loss, appetite disturbance, and bowel disturbance. Due to fat malabsorption, the stools are bulky and foul smelling, which is called steatorrhea. There may be varying degrees of liver and gall bladder problems accompanying chronic pancreatitis. ERCP is the best diagnostic tool. Acute recurrent pancreatitis is usually a more acute event that is initiated by pathologic activity within the glandular/ductal system. Deep epigastric pain radiates to the back, and it is relieved by leaning forward. There may be an accompanying fever with a history of similar attacks in the past. An inferior MI can mimic this clinical picture, but pain direction, radiation, and EKG would be diagnostic. Acute cholecystitis is more common in women, and it can be excluded by direction and radiation of the pain leaning towards the right hypochondrium. In left ureteric colic, the pain would not present in the epigastrium; instead, it would be located more in the left lumbar region and flank, and it would radiate towards the scrotum on that side.

A 32-year-old woman presents with a solitary thyroid nodule. A biopsy confirms that she has thyroid cancer. She subsequently has a thyroidectomy. However, during the procedure, the surgeon accidentally removes most of her parathyroid glands as well. As a consequence, she develops hypoparathyroidism. Her laboratory results are in the chart. What symptom would be consistent with her history and findings? TEST RESULTS REFERENCE RANGE RBC 4.8 x 106/µl 4.5 - 5.7 x 106/µl (male) 3.9 - 5.0 x 106/µl (female) Hematocrit 42 40 - 50 (male) 36 - 44 (female) Hemoglobin 15 gm/dL 13.8 - 17.2 gm/dL (male) 12.1 - 15.1 gm/dL (female) Platelets 252,000/mm3 130,000 - 400,000/mm3 Potassium 4.1 mEq/L 3.5 - 5.0 mEq/L Sodium 144 mEq/L 135 - 145 mEq/L Calcium 7.8 mg/dL 8.5 - 10.5 mg/dL Question What is a sign of her secondary condition? Answer Choices 1 Chvostek's sign 2 Kernig's sign 3 Babinski's sign 4 Kussmaul's sign 5 Quincke's sign

chvostek's sign This patient has hypocalcemia secondary to hypoparathyroidism. Paresthesias can be seen with hypocalcemia due to the increase in neuromuscular irritability. Common sites for the paresthesias are around the mouth and on the fingertips. Tetany can also be seen. Chvostek's sign can be seen with tetany. When tapping on the facial nerve produces contraction on that side of the face, it is called Chvostek's sign. Kernig's sign is a sign of meningeal irritation. The patient lies on his back and his leg is raised and knee bent at a 90-degree angle. When the examiner straightens the knee, if there is pain or resistance to further extension, Kernig's sign is said to be positive. Babinski's sign is an abnormal response to stimulation on the sole of the foot; it results in dorsiflexion of the big toe and fanning of the other toes. Babinski's sign is seen with pyramidal disease. Kussmaul's sign is an increase in venous pressure during inspiration; it can be seen with cardiac tamponade. Quincke's sign (Quincke's pulse) is an alteration seen in nails with each heartbeat. Quincke's sign can be seen with aortic insufficiency.

A 30-year-old G1P1 Caucasian woman with a desire to become pregnant. She has been having unprotected intercourse for 12 months without pregnancy. She typically has 3 - 4 menstrual cycles a year. She reports that she had similar problems becoming pregnant with her 1st child. Her prior doctor did a complete workup for her infertility and amenorrhea. Records have been sent for your review. She successfully conceived on a medication 3 years ago, but she cannot recall the name of the medications. She has never used any method of contraception. This patient would like you to treat her infertility. Question What is the drug of choice for this patient? Answer Choices 1 Clomiphene 2 Danazol 3 Leuprolide 4 Mifepristone 5 Spironolactone

clomiphene Explanation The drug of choice for this patient is clomiphene (Clomid and others), which has estrogen agonistic and antagonistic effects, which increase gonadotropin release and follicular maturation. This medication also improves the luteinizing hormone release and estradiol secretion. Spironolactone is the medication that inhibits aldosterone and dihydrotestosterone, functioning as an androgen receptor blocker. Its primary use in gynecology is for acne, hirsutism, and polycystic ovarian syndrome. Leuprolide is a medication that causes inhibition of gonadotropin release, which reduces ovarian steroidogenesis. It is used in women for the treatment of uterine fibroids and endometriosis; it treats prostate cancer in men. It produces the opposite desired effect when given to a patient with infertility. Mifepristone is a medication with progesterone and glucocorticoid antagonistic effects, which reduce endometrial development. It is used for early pregnancy termination; off-label, it is used for endometriosis, uterine fibroids, and postcoital contraception. This medication should not be given to this patient. Danazol is an older medication that suppresses follicle stimulating hormone (FSH) and luteinizing hormone (LH). It reduces estrogen production and is weakly androgenic. This medication would worsen the patient's chances of conception.

A 34-year-old multiparous woman presents for a routine PAP smear after being "too busy" to have annual exams for the past 7 years. 3 Pap smears in her 20's have all been normal. She has had 1 episode of venereal warts in her late teens; there was no recurrence. She had 2 vaginal deliveries. She does not smoke. Remainder of her history is negative. Her Pap smear is reported as "atypical squamous cells of undetermined significance". What is the most appropriate next step in regards to evaluating the patient? Answer Choices 1 Colposcopy 2 Cone biopsy 3 Hysterectomy 4 Loop electrosurgical excision procedure (LEEP) 5 Repeat PAP in one year

colposcopy Explanation Under the Bethesda system of reporting Pap smear results, the term atypia is reserved for abnormalities which are neither clearly reactive in nature nor meet the criteria for squamous intraepithelial lesions. The evaluation then must obtain tissue for further specificity. Colposcopy with directed biopsies of any identified abnormality is the appropriate next step. In a patient with a negative history for HPV disease and annual Pap smears, and who can be counted on for compliance, a follow up PAP smear at a shorter interval (6 months) would be sufficient surveillance after a first atypical Pap smear. Cone biopsy and LEEP are not appropriate until a more specific tissue diagnosis is available. Hysterectomy is not indicated for an as yet undetermined cervical lesion.

A 62-year-old woman with breast cancer is terminally ill and is receiving hospice care. She receives around-the-clock opioids for continuous pain and liquid morphine as rescue medication. She is currently experiencing abdominal discomfort and pain. A rectal exam is performed, which excludes stool impaction. Question What is the initial step to treat her present symptoms? Answer Choices 1 Methylnaltrexone 2 Dietary fiber 3 Adequate hydration 4 Combination of senna and docusate 5 Physical activity

combination of senna and ducosate The correct answer is a combination of senna and docusate (laxative and stool softener). The patient is suffering from opioid-induced constipation (OIC). Studies strongly recommend routine prescription of laxatives for the management or prophylaxis of opioid-induced constipation1. A combination of drugs with different modes of action is likely to be more effective in resistant constipation than a single agent1. Methylnaltrexone administered by subcutaneous injection should be considered in the treatment of opioid-related constipation when traditional laxatives are not effective1; therefore, it is not used for initial treatment of OIC. Methylnaltrexone is a drug that targets opioid induced constipation by blocking peripheral opioid receptors, but not central receptors for analgesia2. Neither adequate hydration nor dietary fiber intake alone is effective in treating OIC2. Similarly, physical activity has little role in the treatment of OIC. In addition, given that the patient is terminally ill, physical activity is likely to be limited by fatigue.

A 48-year-old woman presents with a 1-week history of diarrhea with fever. Stool is watery and associated with abdominal pain. She has no other medical problems other than an osteomyelitis, for which she completed a course of clindamycin 3 weeks earlier. The patient was started on oral metronidazole after a diagnosis of diarrhea from Clostridium difficile was made 1 week ago. The patient's condition has not resolved after 7 days of metronidazole treatment. What is the best way to treat her? Answer Choices 1 Switch to vancomycin 2 Continue metronidazole 3 Switch to ampicillin 4 Switch to ciprofloxacin 5 Switch to clindamycin

continue metroniadazole The patient has an uncomplicated case of Clostridium difficile colitis. Current guidelines call for her to continue metronidazole for a course of 10-14 days. C. difficile is a Gram-positive, spore-forming rod; it is accountable for 15 to 20% of antibiotic-related cases of diarrhea and almost all cases of pseudomembranous colitis. Disruption of the normal colonic micro flora is usually the precipitating event for C. difficile colitis. This disruption is generally caused by the use of broad-spectrum antibiotics, such as clindamycin, broad-spectrum penicillin, and cephalosporins. Colonization of C. difficile occurs after the micro floral disruption, during which the heat-resistant spores are converted into vegetative spores. Asymptomatic carrier state or clinical manifestations of C. difficile colitis develop depending on the host immune factor. Manifestations range from mild diarrhea to life-threatening C. difficile pseudomembranous colitis. The use of clindamycin prior to the occurrence of the diarrhea strongly suggests pseudomembranous colitis as the cause of the diarrhea. Pseudomembranous colitis is an inflammatory bowel disorder associated with antibiotic use. It is due to toxins of Clostridium difficile. The toxins bind to intestinal mucosal epithelial cells and cause watery, secretory diarrhea, abdominal pain, fever, and dehydration. The mucosa shows yellow plaques, focal ulceration, and exudates, which form the characteristic pseudomembrane. Stool samples are positive for fecal leukocytes, and the toxin and the organism may be cultured. Metronidazole is a first line therapeutic agent in the treatment of C. difficile colitis, given in the dose of 500 mg orally, 3 times a day for 10-14 days. It is inexpensive and has an efficacy of > than 90%. The response to treatment with metronidazole is usually rapid, with fever resolution occurring within 24 hours and diarrhea resolution within 4-5 days. Vancomycin is approximately equally effective as metronidazole in the treatment of C. difficile colitis. Vancomycin should be initiated in patients with severe symptoms (e.g., leukocytosis >15,000 cells/uL or serum Cr 1.5 x normal). It is administered orally in the dose of 125 mg 3 times a day for 10 - 14 days. Initial treatment of severe cases of pseudomembranous colitis must be aggressive, with intravenous metronidazole and oral vancomycin given in combination. In cases where medical therapy fails, surgical intervention, such as colectomy and ileostomy, may be necessary. Ampicillin and clindamycin, being the offending agents, should not be administered; they may cause the condition to worsen. Ciprofloxacin is not the drug of choice for treating unresolved cases of pseudomembranous colitis following metronidazole treatment.

A 17-year-old male adolescent presents with unexplained neurological symptoms. His liver is enlarged on palpation and he has other symptoms of hepatitis. Blood work reveals depressed ceruloplasmin levels. An ophthalmological examination reveals Kayser-Fleischer rings. The most likely diagnosis is Wilson's disease. What abnormality is the primary cause of the disease? Answer Choices 1 Copper metabolism 2 Selenium deficiency 3 Glycolipid metabolism 4 Glycogen storage 5 Collagen biosynthesis

copper metabolism This young man is suffering from Wilson's disease, a genetic disorder of copper metabolism. It is inherited as an autosomal recessive mutation in the ATP7B genes located on chromosome 13. The protein of the ATP7B gene is a copper-transporting ATPase. The frequency of the heterozygous carriers is relatively high (1/90). The incidence of homozygous recessive affected individuals is about 1/30,000. It affects all ethnic groups and both sexes equally. Neurological symptoms, hepatitis, and Kayser-Fleisher rings, greenish-brown deposits of copper in the corneal endothelial (Descemet membrane) basement membrane near the peripheral cornea where it meets the iris (limbus), are characteristic of Wilson's disease. About 40% of the dietary copper is absorbed in the gastrointestinal tract, and makes its way to the liver bound to albumin. In the liver, it is complexed with ceruloplasmin, a blood protein that carries most of the copper. Ceruloplasmin levels are abnormally low in patients with Wilson's disease, although the disease is not, per se, a mutation in the ceruloplasmin gene. Ceruloplasmin is recycled in the liver by the usual lysosomal degradation pathway, and the unused copper is excreted in bile. When excessive copper is absorbed in the gut, it accumulates in the brain (producing neurological symptoms), the liver (producing hepatitis and hepatomegaly), and the cornea. Penicillamine can be used to treat this disease. It chelates copper and provides some symptomatic relief. Unfortunately, when used appropriately in pregnant women to treat potentially life-threatening Wilson's disease, it is harmful to the fetus and can produce cutis laxa in the newborns of penicillamine-treated patients

A 27-year-old woman presents in active labor. She is G3P2 and at 39 weeks of gestation. She has been receiving prenatal care since 6 weeks gestation, and her pregnancy has been uncomplicated. Both of her prior births were normal spontaneous vaginal deliveries. Her cervix is 6 cm, 90% effaced, mid-position, and soft. The fetus is not engaged and is thought to be vertex. Initial fetal monitoring shows a heart rate in the 140s with good accelerations, and it is reassuring. Contractions are 4 minutes apart, and she is comfortable. 20 minutes later, she experiences a large gush of clear fluid, and severe variable decelerations appear on the fetal heart rate monitor. Question What is the most likely diagnosis? Answer Choices 1 Cord prolapse 2 Placental abruption 3 Placenta previa 4 Uterine rupture 5 Vasa previa

cord prolapse Explanation The most likely diagnosis is cord prolapse. Cord prolapse occurs in 2 out of 1000 deliveries. It is diagnosed when fetal monitor recordings show that severe variable decelerations or bradycardia occur after membrane rupture. The cord is often palpable in the vagina. Cord prolapse happens most often at 5-cm cervical dilatation and in nonvertex presentations. 3 types of prolapse can occur. Overt cord prolapse is diagnosed when the membranes are ruptured and the umbilical cord falls through the cervix into the vagina ahead of the fetal presenting part. Funic presentation describes loops of umbilical cord between the presenting part and the cervical os prior to rupture of membranes. Occult cord prolapse is diagnosed when the cord is palpable alongside the presenting part on digital cervical exam. Treatment: Immediate delivery is essential to prevent fetal compromise. Cesarean delivery is generally preferred when the cervix is not fully dilated. Vacuum or forceps delivery may be attempted if the cervix is completely dilated, although manual elevation of the fetal part and emergent cesarean are the most common management. Uterine rupture is less likely since the patient has no history of cesarean delivery. Abruption is often associated with tetanic contractions and bleeding. An increase in uterine tone is seen between contractions. Vasa previa usually presents with painless bleeding at rupture of membranes. The cervical exam is not consistent with a diagnosis of placenta previa.

Upon inspection of an 18-year-old man's scrotum, you note that the left side is underdeveloped, and a testis is not palpable. There is no scrotal tenderness, swelling, or nodularity. What is the patient's condition most likely due to? Answer Choices 1 Cryptorchidism 2 Acute epididmitis 3 Hydrocele 4 Testicular cancer 5 Strangulated inguinal hernia

cryptorchidism Explanation An underdeveloped scrotum, either unilateral or bilateral, is likely due to undescended testicles, which is also known cryptorchidism. Acute epididymitis and strangulated inguinal hernia are usually tender to the touch and cause swelling. Hydroceles are commonly accompanied by scrotal swelling. If painless nodules are palpated in the testes, testicular cancer should be ruled out, especially in men between the ages of 15 to 35.

A neonate presents with meconium ileus that is successfully unobstructed. The infant returns at her 4-month appointment with signs of failure to thrive. Which of the following is the most likely diagnosis for this patient? A cystic fibrosis B Wilson disease C intussusception D volvulus

cystic fibrosis Cystic fibrosis (CF) is a major cause of gastrointestinal and pulmonary morbidity in children due to mutations in the CF genes. The mutations lead to a deficiency in cystic fibrosis transmembrane conductance regulator protein that controls movement of salt and water into and out of epithelial cells and results in production of abnormally thick mucus. About 15% of patients with CF present with meconium ileus at birth. This is typically treated with enema for disimpaction and rarely surgery. Approximately half of the infants with CF will present with failure to thrive, which is diagnosed by lack of growth for 2 consecutive months in patients younger than 6 months of age. They may also present with respiratory compromise. However, not all patients present in childhood. Diagnosis of CF is confirmed by a sweat chloride level above 60 meq/L or with genetic testing. Treatment for patients with CF is mainly symptomatic therapy for obstructions of the digestive and respiratory tract. In addition, there is pancreatic enzyme supplementation to aid in digestion and vitamin and calorie supplementation for deficiencies in the diet. Gene therapy is now being looked at for future treatment. Intussusception (telescoping of the small intestine) typically presents in an infant with paroxysmal abdominal pain, vomiting, and diarrhea that may progress into bloody stools. Volvulus is normally the result of intestinal malrotation that causes occlusion of the superior mesenteric artery and eventual bowel necrosis. Infants typically present within 3 weeks of life with bile-stained vomiting and bowel obstruction. Wilson's disease is the defect in the ability to excrete copper in the bile that results in accumulation of copper in the liver.

A 24-year-old male is brought to the clinic by his mother, who is concerned because her son believes that a local TV news anchor is in love with him. The mother states that this thought has been persistent for the last two to three months, and that he goes around town telling everyone about their relationship; however, she knows that her son does not even know the TV news anchor. The mother notes that this belief has not impaired his daily functioning, but has significantly affected his social functioning. The patient is exhibiting signs most consistent with which of the following psychiatric disorders? A Delusional disorder B Histrionic disorder C Paranoid disorder D Schizoid disorder E Schizotypal disorder

delusional disorder A This patient is exhibiting signs most consistent with delusional disorder. Delusional disorder is a psychosis in which the person has persistent beliefs that are non-bizarre, such as being persecuted, being related to or loved by a well-known person, or that their partner is unfaithful. In this disorder, the delusions tend not to affect the patient's intellectual and occupational activities, but social and marital functioning are significantly affected. Clinical findings of histrionic personality disorder include being dependent, immature, seductive, egocentric, vain, and emotionally labile. Clinical findings of someone who has paranoid personality disorder would include defensiveness, being overly sensitive, secretive, suspicious, and hyper-alert, and having a limited emotional response. Schizoid personality disorder is characterized by shyness, introversion, being withdrawn, and avoiding close relationships. Schizotypal disorder is characterized by being superstitious, socially isolated, and suspicious, with limited interpersonal ability, odd speech, and eccentric behaviors.

A 38-year-old female is at a follow-up visit for hypertension. She is accompanied by her husband. During the clinical visit the patient is noted to be passive, letting her husband do most of the talking. She also appears to lack confidence and self-esteem. The husband ends up making the decisions, and she is over-accepting of his dominance. What is the most likely classification of this personality disorder? A Avoidant B Borderline C Dependent D Histrionic E Schizotypal

dependent C The clinical findings of dependent personality disorder are the most consistent with the given clinical scenario. Avoidant personality disorder presents clinically as someone who fears rejection, overreacts to rejection and failure, has poor social endeavors, and low self-esteem. Borderline personality disorder clinical findings include impulsiveness, unstable and intense interpersonal relationships, lack of self control, suicidal ideations, aggressive behavior, and a high drug abuse rate. Clinical findings of histrionic personality disorder include being dependent, immature, seductive, egocentric, vain, and emotionally labile. Schizotypal clinical findings include being superstitious, socially isolated, and suspicious, and having limited personality ability, odd speech and eccentric behaviors.

A 73-year-old man is brought into your office by his adult children with a concern of memory loss. They report their father's memory has been declining since the death of their mother a few months ago but are now concerned because he is losing weight, sleeping during the daytime, and is not keeping up with current events like he usually does. This type of behavior is most associated with which of the following? A Pick disease B Creutzfeldt-Jakob disease C depression D Alzheimer disease E vitamin B12 deficiency

depression C This patient's symptoms are most consistent with situational depression over the loss of his spouse. Transient memory problems can be a component of depression as a result of decreased attention and interest. Dementia is a progressive impairment of higher cognitive function, and initially, the patient's social graces are preserved. It has many causes, of which Pick disease, Creutzfeldt-Jakob disease, and Alzheimer disease are irreversible. Vitamin B 12 deficiency can cause reversible form of cognitive impairment, in which the elderly are susceptible, so serum analysis of vitamin B 12 should be performed in diagnostic evaluations of dementia in this population

An 70-year-old woman is brought to your attention because of the slowly progressive gait disorder; she is also experiencing forgetfulness and problems with micturition. About 1 year ago, she started having weakness and tiredness in her legs, followed by unsteadiness and her steps became shorter and shorter. Sometimes, she forgets where she put things; she has forgotten to turn off the oven 2 times. Over the last month, she has started experiencing urinary urgency, the need for frequent urination, and the involuntary leaking of urine that worries her. She notes no other symptoms. Question What is the most likely cause of her urinary problems? Answer Choices 1 Increased cerebrospinal fluid pressure 2 Urinary tract infection 3 Frontal lobe dysfunction 4 Detrusor muscle instability 5 Pelvic sphincter weakness

detrusor muscle instability The correct response is detrusor muscle instability. A clinical triad of slowly progressive gait disorder, followed by impairment of mental function and sphincteric incontinence, highly suggests the presence of normal-pressure hydrocephalus. Ventricular expansion in hydrocephalus involves frontal horns, affecting the functions of frontal lobes and of basal ganglionic-frontal motor control. The central white matter is affected more than cortical gray matter, thalami, basal ganglia, and brainstem. The distortion of the central portion of the corona radiata by the distended ventricles affects the sacral motor fibers that innervate legs and the sacral bladder center, causing the abnormal gait and incontinence. Decreased inhibition of bladder contractions leads to the instability of bladder detrusors. In cases of normal-pressure hydrocephalus, the formation of CSF equilibrates with absorption; there might be variations in the CSF pressure, but overall, pressure is considered normal. A urinary tract infection will probably manifest with a strong, persistent urge to urinate, a burning sensation when urinating, and passing frequent, small amounts of urine that has unusual smell. Its appearance may be cloudy; it may be red, pink, or cola-colored if there is blood in the urine. Dementia is a consequence of the distortion of the periventricular limbic system and frontal lobes in the cases of normal-pressure hydrocephalus. However, incontinence is only a result of a lack of concern for micturition in extremely advanced cases, in which there is frontal lobe dysfunction. Your patient has insight into her problems and worries; therefore, frontal lobe dysfunction resulting in a lack of concern for micturition is probably not the cause of her urinary problems. Pelvic sphincter weakness will probably manifest as stress incontinence; incontinence happens when a weak sphincter cannot hold the urine and physical pressure is placed on the urinary system, such as when a patient coughs, sneezes, or laughs.

42-year-old female complains of weight gain (especially in her abdomen) over the past 8 months. She also has noted that her skin bruises easily. Her husband has noted she seems to be very moody lately and she is worried about their relationship. Furthermore, her hair seems to be getting thinner and she is now getting acne like she had in her teenage years. She wonders if this is due to menopause since her periods have stopped suddenly about a year ago. On physical examination her BP = 170/50, P = 82, T = 98.2˚F. You note the following findings on examination (see picture). What diagnostic test is indicated initially to confirm your suspected diagnosis? A cosyntropin stimulation test B MRI pituitary C dexamethasone suppression test D 24-hour urine for protein E serum protein electrophoresis

dexamethasone suppression test The correct answer is (C). The patient's symptoms are consistent with a diagnosis of Cushing's syndrome (or disease). Her physical examination findings of hypertension and abdominal obesity with the classic purple striae also support the diagnosis. The initial diagnostic test of choice would be the dexamethasone suppression test. If the test is positive, further confirmatory testing is done which would also help to identify the cause. An MRI of the pituitary is appropriate if further testing suggests the possibility of a pituitary adenoma as the cause of the Cushing's syndrome, but is not used as an initial diagnostic test for Cushing's disease. A cosyntropin stimulation test, choice (A), is indicated for the diagnosis of Addison's disease.

A 22-year-old man is brought to the Emergency Department with nausea, fever, loss of appetite, and right lower quadrant pain. He was in his usual state of good health until about 1 day prior to admission when he began having poorly localized abdominal pain. The pain worsened and became sharp and localized to the right lower quadrant. His fever also worsened, and he was found to have an elevated white blood cell count. Question What is the most common early symptom of the patient's condition? Answer Choices 1 Fever 2 Diffuse abdominal pain 3 Nausea 4 Pain localized to the lower right quadrant 5 Vomiting

diffuse abdominal pain Appendicitis is an inflammation of the appendix. Appendicitis is a common infection of the abdomen that can lead to surgical intervention. The appendix is attached to the first part of the colon, called the cecum. It is a blind pouch and is actually called the vermiform appendix (which means "worm-like appendage"). The appendix is part of the immune system and contains lymphatic tissue. The appendix produces mucus, and as objects (usually stool) enter the appendix, they can become trapped. In addition, mucus can thicken and accumulate. As a result, the appendix can become blocked by the resulting fecalith. After the appendix becomes blocked, bacteria can increase in numbers and cause infection. The infection can spread from the appendix out into the abdomen. The appendix can rupture, and the spread of infection starts as an abscess around the appendix. Occasionally, the body can contain the infection and resolve. Differential diagnosis includes: kidney disease, pelvic inflammatory disease, gall bladder disease, right-sided diverticulitis, and Meckel's diverticulitis. The most common complication of appendicitis is perforation, which causes the infection to spread outside the appendix. Other complications include peritonitis, sepsis, and intestinal blockage. Symptoms of appendicitis include abdominal pain, loss of appetite, fever, nausea, vomiting, and fever. The main symptom is abdominal pain which is poorly localized at first. The initial periumbilical pain is due to obstruction and inflammation of the appendix and is mediated through the visceral pain fibres as a mid-gut pain. When appendicitis becomes transmural, the serosa of the appendix and the parietal peritoneum are involved, causing a localised pain mediated through the somatic pain fibres in the right iliac fossa. It is usually located at McBurney's point. This is located 1/3 of the distance from the ASIS (anterior superior iliac spine) to the umbilicus. Diagnosis is usually made by taking a thorough history and performing a physical exam. Blood and urine samples should be taken, and elevated white blood cell count can often be seen. Urinalysis may show bacteria and red and white blood cells. An abdominal radiograph may show the presence of a fecolith, but may be negative in many patients. An ultrasound should also be done. Appendicitis is usually treated by surgery.

Mr. Smith leaves home and does not return nor does he go to work. A friend of Mr. Smith sees him in another state while on vacation. When he approaches Mr. Smith, he does not recognize him and has a total different demeanor. What type of disorder does Mr. Smith have? A amnesia B dissociative fugue C schizophrenia D dissociative identity disorder E depersonalization

dissociative fugue Dissociative or psychogenic fugue is precipitated by a stressful event that causes the patient to develop amnesia, leave home, and assume another identity.

A 34-year-old woman presents with worsening headaches. She says that the headaches are present throughout the day and that she has been feeling nauseous. Lately she has also noticed difficulty in seeing vehicles overtake her on the freeway. She has had several close calls due to this handicap. Her periods, which were previously regular are now irregular, with heavy bleeding every 3 - 4 months. She has also noticed a milky discharge from both nipples. Her pregnancy test is negative. An MRI of the brain shows a mass compressing the hypothalamic pituitary axis. Question What is the best course of treatment? Answer Choices 1 Irradiation of the tumor 2 Surgical excision 3 Catheter ablation 4 Dopamine agonists 5 External beam laser coagulation

dopamine agonist he most likely diagnosis is a pituitary adenoma compressing the HP axis. The standard of care at the present time is the use of dopamine agonist drugs, such as bromocriptine and cabergoline. These drugs reduce prolactin levels to normal in 70 - 100% of patients. The presence of the pituitary mass blocks the action of dopamine on the prolactin-secreting cells and increases the secretion of prolactin. Cabergoline is an alternative to bromocriptine with fewer side effects. However, it is a much more expensive drug, and it is usually used when bromocriptine is not tolerated. Pergolide has been taken off the market due to associated heart valve damage with its use. Followup may be done by measurement of the fasting serum prolactin level. Surgery is incorrect, as it is usually indicated only in failure of medical therapy, continued visual field loss despite medication, or failure to tolerate medical therapy. Radiation therapy is incorrect. Although effective, it is seldom used due to the risk of permanent hypopitutarism after treatment. External beam coagulation and ablation are both incorrect. They are distractors that have no role in therapy.

A 13-year-old boy is diagnosed with Autism Spectrum Disorder. The boy repeats phrases in a parrot-like fashion; he repeats whatever he hears, but comprehension is absent. What condition is exhibited by the child in the above description? Answer Choices 1 Paraphasia 2 Echolalia 3 Alexia 4 Apraxia 5 Agraphia

echolalia Echolalia is a pathological, parrot-like, and apparently senseless repetition or echoing of a word or phrase spoken by a person or heard on TV; the repetition/echoing is done without actual comprehension. This is also referred to as echophrasia. Echolalia occurs in autism, catatonic schizophrenia, Gilles de la Tourette's Syndrome, and in transcortical aphasia. Paraphasia is a type of aphasia. The substitution of a similar sounding word for another word is called paraphasia. With paraphasia, the words can also be jumbled. Alexia is the inability to read and understand written language. Alexia is word blindness or text blindness. Alexia is also called optical aphasia or visual aphasia. Apraxia refers to the condition where a patient has difficulty performing a learned motor activity despite having the understanding, muscular capacity, coordination, and normal sensations to do so. A patient with apraxia cannot execute the purposeful movement. A writing disturbance is called agraphia. It is the inability to express thought in written language that is not due to mechanical dysfunction.

A 33-year-old man with no past medical history presents with groin mass. He denies pain and trauma to the region. When the patient stands, there is the presence of a round swelling in the inguinal area. If the patient is supine, the mass disappears. Question What is the appropriate intervention in this patient? Answer Choices 1 Immediate surgical repair 2 Elective surgical repair 3 Oral antibiotic therapy 4 IV antibiotic therapy 5 No intervention is needed

elective surgical repair Elective surgical repair is correct. The patient has an inguinal hernia, which can be reduced easily. Immediate surgical repair is incorrect. The patient's hernia should be repaired. However, the hernia is not currently incarcerated and can be treated electively. Oral antibiotic therapy and IV antibiotic therapy are not indicated in the treatment of hernias. No intervention is needed is incorrect. The presence of a hernia signals a risk for potential incarceration in the future. The patient is young with no comorbidities. He should be scheduled for elective repair to avoid the development of incarceration.

A 30-year-old G 2 P 1 woman comes to the emergency department at 37 weeks gestation with a chief complaint of worsening abdominal pain for the past two hours. On examination, the uterus is tense and tender. External fetal monitoring indicates frequent uterine contractions with late decelerations. An ultrasound suggests a retroplacental hemorrhage. What is the most appropriate course of action at this time? A Emergency operative delivery B Expectant management at home C Monitoring in the labor suite with the mother on her side D Tocolytic administration until the infant is 38 weeks gestation E Vaginal delivery with induction

emergency operative delivery A Indications for emergency cesarean delivery in a woman with placental abruption include fetal heart tracings that are not reassuring. Furthermore, a retroplacental hemorrhage carries a direr prognosis than a subchorionic one. Expectant management (B), monitoring (C), attempting to stop labor (D), and attempting a vaginal delivery (E) put both mother and fetus at severe risk of death.

A 25-year-old Caucasian woman and her husband have been trying to have a baby for the last 2 years. The patients' medical history included cyclical pelvic pain, dysmenorrhea and dyspareunia. The physical exam reveals the following: diffuse abdominal or pelvic pain of variable location, nodular thickening and tenderness along the uterosacral ligaments, on the posterior surface of the uterus, and in the posterior cul-de-sac, scarring and narrowing of the posterior vaginal fornix, and adnexal enlargement and tenderness. Question What is the most likely diagnosis? Answer Choices 1 Endometriosis 2 Ectopic pregnancy 3 Adnexal mass 4 Pelvic relaxation 5 Leiomyomatous uterus

endometriosis Explanation Endometriosis is the growth of tissue outside the uterus. The ectopic endometrial tissue can proliferate, and infiltrate, and spread to remote sites elsewhere in the body. Sites that endometriosis is most often found are, ovarian, pelvic peritoneum, anterior and posterior cul-de-sac, uterosacral, round, and broad ligaments, and fallopian tubes. An ectopic pregnancyis one that develops at any site other than the endometrium. Symptoms of an ectopic pregnancy are abdominal pain, possibly a missed period, abnormal bleeding at the time of presentation is not uncommon. Physical characteristics of an adnexal mass that may give a presumptive diagnosis: consistency (cystic, solid or both), size (in centimeters) morphology and surface contour (smooth, nodular, vague or sharp borders), location around the uterus, mobility (free or fixed to adjacent structures) tenderness, unilateral or bilateral. The structures supporting the urethra (urethrocele), bladder (cystocele), uterus, posterior wall of the vagina (enterocele), and rectum (rectocele) weaken. Often symptoms fo this pelvic relaxation occur at or after menopause because of the lack of hormonal effect, causing atrophy of these tissues. Each of these structures has different presenting symptoms. A leiomyoma is a benign uterine tumor. Other names for leiomyoma include myoma, fibroid, and fibromyoma. Symptoms associated with leiomyoma are bleeding, pressure, and pain.

A 27-year-old G1 P1 woman has recently given birth by caesarean section to a 36-week male infant. She did well throughout her pregnancy up until 34 weeks gestation. At that time, she presented with fever, abdominal pain, and wetness. She was diagnosed with preterm premature rupture of membranes (PPROM) and chorioamnionitis. She was treated with steroids and antibiotics, stabilized, and she then delivered by C-section with no complications. On postpartum day 7, she presents with sore breasts from breast-feeding and a sore abdomen. She also admits to an odorous vaginal discharge, but she denies any associated vaginal bleeding. On physical examination, she is having a moderate amount of lochia alba, and her temperature is 101.2°F. Question What is the most likely diagnosis at this time? Answer Choices 1 Deep vein thrombosis (DVT) 2 Endometritis 3 Infected retained placenta 4 Mastitis 5 Septic pelvic thrombophlebitis

endometritis Explanation Postpartum endometritis is most likely in this patient given her history of chorioamnionitis. Prolonged rupture of membranes, cesarean delivery, prolonged labor, and multiple cervical examinations are all risk factors for postpartum endometritis. The presence of intra-amniotic infection increases the risk of postpartum endometritis further. Antibiotics are not routinely continued for chorioamnionitis after a delivery because the "source" of the infection (the placenta) has been removed. Whenever fever occurs in the immediate postpartum period, endometritis should be suspected. Broad-spectrum antibiotics should be administered promptly by the parenteral route. Similar to chorioamnionitis, multiple bacterial organisms (usually normal vaginal and perineal flora) are likely to be responsible for this infection. Therefore, uterine cultures are unlikely to be helpful for guiding antibiotic therapy. Parenteral therapy should be continued until the patient has been afebrile for at least 24 hours. Mastitis is characterized by a swollen, firm, tender breast with systemic symptoms including inflamed breast, fevers, chills, and flu-like symptoms. Staphylococcus aureus is the typical pathogen. However, in the immediate postpartum period, breast engorgement without infection is the most likely reason for the patient's sore breasts. Pregnancy and the postpartum period increase a woman's risk of thrombogenesis. However, DVT is not a likely source of the fever. Septic pelvic thrombophlebitis is a diagnosis of exclusion and is usually entertained when fever spikes continue following treatment for endometritis. Infected retained placenta is unlikely in the absence of vaginal bleeding. Lochia This is the uterine discharge following delivery and lasts for 3 or 4 weeks. Foul-smelling lochia suggests infection. Types: (1) Lochia rubra. This blood-stained fluid lasts for the first few days. (2) Lochia serosa. This discharge appears 3 to 4 days after delivery. It is paler than lochia rubra because it is admixed with serum. (3) Lochia alba. After the 10th day, because of an admixture with leukocytes, the lochia assumes a white or yellow-white color. Puerperium: This period of 4 to 6 weeks starts immediately after delivery and ends when the reproductive tract has returned to its non-pregnant condition. Multiple anatomic and physiologic changes occur during this time; the potential exists for significant complications, such as infection or hemorrhage. Puerperal infection is defined as any infection of the genitourinary tract during the Puerperium accompanied by a temperature of 100.4oF (38 oC) or higher that occurs for at least 2 of the first 10 days postpartum, exclusive of the first 24 hours. Prolonged rupture of the membranes accompanied by multiple vaginal examinations during labor is a major predisposing cause of puerperal infection.

A 23-year-old Hispanic female delivers a healthy male child by normal vaginal delivery. She had a small episiotomy which was clean cut. She is discharged home after 48 hours but returns to the ER on the fifth post partum day with fever and chills. She says she has had foul-smelling vaginal discharge and lower abdominal pain for the last 24 hours. On exam, she has a temperature of 102° F, BP 110/60 mm of Hg, pulse 118/min, and SPO2 92%. Lungs are clear, and she has no pallor, cyanosis, or icterus. Abdominal exam demonstrates tenderness in the suprapubic area and both lower quadrants. Pelvic exam shows foul smelling purulent vaginal discharge and tender uterus. The episiotomy site is non-tender. Labs show Hb 0f 11g/dl, WBC 13,500, bands 11%, and platelets 350,000/uL. Urinalysis is pending. Question The most likely diagnosis is: Answer Choices 1 Endometritis 2 Pyelonephritis 3 Wound cellulitis 4 Cervicitis 5 Cystitis

endometritis Explanation This female patient is suffering from post partum endometritis as obvious by recent delivery, fever, foul-smelling vaginal discharge, and tender pelvic exam. Other risk factors for endometritis include invasive gynecological procedures, retained products of conception, intrauterine devices, submucosal fibroids, multiple sexual partners, unprotected intercourse with infected partner who may be asymptomatic, etc. Leukocytosis with bandemia supports diagnosis. Blood cultures, urine culture, and cervical culture may be done but treatment should not be delayed. Broad spectrum antibiotics to cover beta lactamase and anaerobic organisms like clindamycin with an aminoglycoside are given. Pyelonephritis is unlikely since she has no back pain or costovertebral angle tenderness, and foul vaginal discharge is absent in pyelonephritis. UA and urine culture should rule it out. Wound infections are rare in a small, clean-cut episiotomy. Fever with foul vaginal discharge does not occur, and the wound area will be obviously erythematous, indurated, and tender. Even in third or fourth degree lacerations antibiotic therapy is usually unnecessary. Cervicitis is either asymptomatic or has purulent vaginal discharge without fever, chills, tender abdominal exam, or leukocytosis. Cystitis may be associated with low grade fever and suprapubic tenderness, but dysuria is predominant and vaginal discharge absent. UA confirms diagnosis.

A 62-year-old male diabetic patient presents to your family practice office with a fever and cough, and after a physical examination you perform a chest x-ray, which reveals a right middle lobe pneumonia. You prescribe levofloxacin 750 mg PO for five days and give the patient his first dose in the office prior to sending him home. Within five minutes, he develops generalized urticaria, shortness of breath with a pronounced wheeze, and becomes hypotensive. While your staff calls 911, what is the most important agent to administer first? A cimetidine B dipenhydramine C epinephrine D methylprednisolone E none of the above

epinephrine Early recognition of an anaphylactic reaction is mandatory, since death occurs within minutes to hours after the first symptoms. Mild symptoms such as pruritus and urticaria can be controlled by administration of 0.3 to 0.5 mL of 1:1000 (1.0 mg/mL) epinephrine SC or IM, with repeated doses as required at 5- to 20-minute intervals for a severe reaction.

A 12-year-old girl, diagnosed using a quick antigen test as having a pharyngeal infection due to Streptococcus pyogenes, also presents with a rash that is on the upper part of the chest and trunk. This rash is caused by Streptococcus pyogenes producing what? Answer Choices 1 Exfoliative toxin 2 Elastase 3 Enterotoxin 4 Erythrogenic toxin 5 Streptolysin

erythrogenic toxin Erythrogenic toxin (streptococcal pyrogenic exotoxin) is produced by Streptococcus pyogenes. The toxin is responsible for the rash of "scarlet fever". The toxin has been shown to exhibit pyrogenicity and cytotoxicity. It usually appears at the 2nd day of infection, on the upper part of the chest, spreading to the rest of the trunk out towards the rest of the body, with the palms, soles, and face being spared. Exfoliative toxin, produced by Staphylococcus aureus, is responsible for scalded skin syndrome (SSS), wherein the patient appears to have acquired a burn of the skin. There is extensive scalding and flaking desquamation of the epidermis. The syndrome is especially common in infants and small children. Elastase is an extracellular protease that is produced by Pseudomonas aeruginosa. The protease is associated with the organism's virulence due to tissue destruction and bacterial invasion. Elastase is necrotizing to the skin, cornea, and lung; it is capable of producing hemorrhage. Enterotoxin is produced by various bacteria. In Clostridium perfringens, the production of this toxin causes the symptoms of food poisoning. Clostridium perfringens type A is most associated with this toxin production. Enterotoxin is thought to act as a superantigen, which causes a massive release of inflammatory mediators and induces a calcium ion-dependent breakdown of permeability. Exotoxin (diphtheria toxin) is produced by Corynebacterium diphtheriae. The toxin inhibits protein synthesis in mammalian cells, but not bacteria. It affects all cells in the body; the heart, nerves, and kidney are impacted the most.

A 9-year-old girl presents with a sore throat. The mother states that she began to run a fever a few days ago, and she complained that her throat hurt. On physical exam, you note a red throat, a red, beefy tongue, tonsillar exudates, and swollen anterior cervical lymph nodes. You order a rapid strep test which comes back positive. It is noted in the patient's records that she has had a severe anaphylactic reaction to penicillin. What antibiotic would treat this infection while minimizing the risk of invoking an allergic reaction? Answer Choices 1 Augmentin 2 Cephalexin (Keflex) 3 Ciprofloxacin 4 Mupirocin (Bactroban) 5 Erythromycin

erythromycin The clinical picture is suggestive of a streptococcal bacterial infection. Penicillins are the 1st-line antibiotics in the treatment of strep pharyngitis. Since the patient is allergic to penicillins, erythromycin is an effective alternative and has no allergic cross-reactivity with the penicillins. Augmentin contains amoxicillin, a member of the penicillin family with allergic cross-reactivity; it is contraindicated for patients allergic to penicillin. Cephalexin, a cephalosporin, can be used to treat strep throat. However, approximately 7% of patients who are allergic to penicillin are also allergic to cephalosporins. Therefore, compared to erythromycin, use of this agent poses a slightly higher risk of causing an allergic reaction in this patient. Ciprofloxacin is effective against Gram-negative organisms. Since streptococcal species are Gram-positive, it would not be an appropriate treatment in this scenario. Mupirocin is a topical antibiotic and is not indicated in the treatment of strep throat.

A 53-year-old man presents with increased difficulty swallowing and occasional regurgitation of his meals. His symptoms have been occurring with greater frequency and severity over the last 4 months. He also gets some shortness of breath, but attributes that to his weight and lack of physical activity. His past medical history is remarkable for chronic heartburn, which he treats intermittently with over the counter antacids. He takes no regular medications, and he has no allergies. He has not had any surgeries. He is smoker, but he denies use of alcohol and drugs. He works as a building inspector, and he lives with his wife and children. The patient is obese, but the rest of his physical exam is normal. Blood tests, electrocardiogram, and chest X-ray are done in the clinic; they are normal. He is referred for endoscopy, and esophageal biopsy shows specialized intestinal metaplastic cells (of columnar epithelium). Question What is the medication of choice for this patient's condition? Answer Choices 1 Alendronate 2 Dicyclomine 3 Esomeprazole 4 Famotidine 5 Simethicone

esomeprazole This patient is presenting with Barrett's esophagus, which is a type of chronic esophagitis in which the normal squamous epithelium is replaced with columnar epithelium. Barrett's esophagus is a complication of chronic gastroesophageal reflux disease (GERD) and can develop into esophageal adenocarcinoma. Use of proton pump inhibitors (PPIs), such as esomeprazole, reduces the risk of cancer. In order to monitor for the development of cancer, a routine endoscopy should be periodically performed in patients with known Barrett's esophagus. Alendronate is a bisphosphonate; it is used for both treatment and prevention of osteoporosis. Alendronate can cause multiple gastrointestinal adverse effects, including esophagitis and esophageal ulcerations and strictures. This medication could dramatically worsen this patient's current condition. Dicyclomine is an anticholinergic medication that relaxes smooth muscle. It is used for spasms associated with irritable bowel syndrome. It does not have a role in treating GERD or Barrett's esophagus. Famotidine antagonizes H2 histamine receptors (H2 blocker). It is used in treating duodenal and gastric ulcers as well as GERD. H2 blockers are usually considered less potent than PPIs. PPIs are the treatment of choice for Barrett's esophagus. Simethicone is sold in various over-the-counter products for flatulence. It has no role in treating GERD or Barrett's esophagus.

A patient comes in to a small evening clinic with a complaint of dysphagia. The dysphagia started three or four months ago and has progressively gotten worse. He also complains of losing weight. He is African American, 60 years old, and weighs 170 pounds at six feet tall. He appears older than his stated years. He wants something to help him in swallowing. He does not complain of heartburn. You note he does not eat on a regular basis and when he does eat, it is usually fast food. He has smoked for the last forty years, two packs a day. He drinks 12 cans of beer on weekdays and approximately 48 cans of beer during the weekend. He uses recreational drugs occasionally. He is not married, and lives alone. You know that his history of smoking and alcohol use make him at risk for a life threatening condition. You decide to get an endoscopy with biopsy. While waiting for results what is your most likely diagnosis? Answer Choices 1 Gastroesophageal Reflux Disease 2 Esophagus Neoplasm 3 Esophagus Stricture 4 Esophagus Varices 5 Mallory-Weiss syndrome

esophageal neoplasm Esophageal neoplasm is 60% squamous cell, carcinomas arising from the surface epithelium, most commonly in the middle and lower esophagus. Adenocarcinomas, which constitute 35%, arise from the gastric fundus and develop in the lower third of the esophageal tract. This cancer is most common in older adults, with African Americans affected three times as often as Caucasians , and men three times as often as women. Risks factors: Smoking and tobacco use (chewing) Alcohol abuse Drug abuse (e.g., morphine, opium) Malnutrition Environmental carcinogens (e.g., Nitrosamines, silica, fungi) History of cancer of the larynx or pharynx History of chronic inflammation of the esophagus, achalasia (failure of esophageal sphincter to relax), tylosis, or caustic burns to esophagus. Dysphagia is the most common presenting symptom. Regurgitation and weight loss may also occur. The etiology is not well defined but is associated with chronic esophageal irritation.

A 49-year-old woman presents with intermittent vaginal bleeding and anemia for 3 months. After her endometrial biopsy, you confirm that she has endometrial carcinoma. What is the largest risk factor for endometrial cancer? Answer Choices 1 Estrogen exposure 2 Oral contraceptive pill 3 Polycystic ovary 4 Granulosa-theca cell ovarian tumor 5 Obesity

estrogen exposure Increased risk of endometrial cancer has been associated with the following: Estrogen exposure is the largest risk factor: Sources of estrogen exposure may be endogenous or exogenous. There is a significant correlation between use of oral estrogen and endometrial cancer, when estrogen therapy is administered without the protective effects of cyclic progestin. Obesity: Women who are 20 - 50 lbs. over their ideal body weight have a 3-fold increased risk for endometrial cancer. Polycystic ovary disease: This is the disease risk attributed to stimulation release of unopposed estrogen from the ovaries. Granulosa-theca cell ovarian tumors: A hormonally active estrogen-producing stromal tumor of the ovary.

A 26-year-old woman presents with a 3-month history of amenorrhea. She has been sexually active with her husband, and she has been trying to conceive for the past 1 year. A pregnancy test is obtained, and it is negative. On examination, she appears overweight; there is acne and hirsutism. Her past menstrual history is significant for irregular menstrual cycles; they are not associated with dysmenorrhea or menorrhagia. Question What is the best management for her infertility? Answer Choices 1 Weight loss and exercise 2 Androgen receptor antagonist 3 Metformin 4 Estrogen receptor modifiers 5 InVitro fertilization

estrogen receptor modifiers Explanation Estrogen receptor modifiers is the correct answer. Clomiphene citrate is a commonly used drug in the management of anovulatory infertility. The vignette describes a classic case of polycystic ovarian syndrome, which is the most common cause of anovulatory infertility. It acts centrally by inhibiting the negative feedback of estrogen on the release of gonadotropins. This results in higher levels of follicle stimulating hormone (FSH) and luteinizing hormone (LH), which promotes the development and release of an ovum. Weight loss and exercise is incorrect. They are key to the management of PCOS, and they do so by reducing Insulin resistance and improving glycemic control. However, they do not correct anovulation. Metformin is incorrect. Metformin reduces insulin resistance and improves peripheral glucose uptake while promoting weight loss. However, it does not correct infertility. In vitro fertilization is incorrect. It is a viable option, and it is often used in infertility after medical management has failed. It is expensive, invasive, and time consuming; there is no guarantee of success. Therapy with clomiphene results in fertility in a number of women with PCOS and should be the first line of therapy.

For the past year, a 30-year-old woman and her husband have been trying unsuccessfully to become pregnant. Over-the-counter ovulation tests have indicated that she is ovulating. Neither partner smokes cigarettes, uses any mind-altering drugs, or has a history of sexually transmitted infection. She has no history of abdominal surgery or pelvic procedures, diethylstilbestrol (DES) exposure, or major illness. On examination, she is 66" tall, weighs 135#, and appears healthy. Her thyroid is nonpalpable, and pelvic examination is unremarkable. What is the most appropriate next step in evaluation of this couple's infertility? A Endometrial biopsy B Examination of the husband C Pelvic ultrasonography D Performing a hysterosalpingogram E Referral to an infertility clinic

examination of the husband Male factors account for 25-40% of infertility. Since the woman's initial evaluation appears normal and ovulation tests are positive, the next step is to evaluate the husband before embarking on a more detailed evaluation of the wife. An endometrial biopsy (A) may be appropriate during the late luteal phase to evaluate the endometrial lining. Pelvic ultrasonography (C) may help identify ovarian cysts, endometrial implants, leiomyomas and other treatable conditions. Hysterosalpingography (D) is useful in evaluating the patency of the fallopian tubes and the size and shape of the uterine cavity. Since this woman is only 30 years of age and many other studies can be performed in ob-gyn setting, referral to an infertility clinic (E) is premature.

A 65-year-old woman is concerned about a non-tender lump she discovered in her right breast last month. When performing a breast self-exam this month, she noticed the lump persists, but it is unchanged. On examination, you feel a firm, unfixed, non-tender 2 cm mass in the right upper outer quadrant; there are no palpable axillary nodes. Mammography and a fine-needle aspirate (FNA) are inconclusive. What is the next most appropriate step? Answer Choices 1 Repeat mammography in 3 months 2 Ultrasound 3 Partial mastectomy 4 Tamoxifen therapy 5 Excisional biopsy

excisional biopsy Explanation Establishing a diagnosis under these circumstances requires excisional biopsy and pathological examination of the lesion. In this case, the mammogram and FNA do not explain the clinical findings despite the usual sensitivity of the FNA (>90%). Ultrasound studies may add some information but are typically not useful in establishing a definitive diagnosis. Initiating surgical or medical therapy is inappropriate as would be delaying a diagnosis for 3 months.

A 30-year-old healthy female's labs return showing an HDL of 28 mg/dl, LDL of 132 mg/dl, and total cholesterol of 185mg/dl. She is concerned due to a family history of coronary artery disease (CAD) on her father's side in his 60s. She denies tobacco use. Her blood pressure is 108/50, P-64. She requests advice on what she can due to increase her HDL. What do you advise her? A atorvastatin B cholestyramine C ezetimibe D gemfibrozil E exercise

exercise The correct answer is (E). This patient is at very low risk of CAD based on the 10-year Framingham projections. Based on her < 1%, 10-year risk drug therapy is not indicated. Exercise has been shown to increase HDL. HDL is lower in patients who have a sedentary lifestyle or are obese. Her LDL may also be reduced with exercise, weight loss, and dietary modifications.

An unidentified motorcyclist is transported to the emergency department via helicopter from the scene of a highway accident. According to eyewitnesses, he appears to have been trying to avoid a piece of debris in the road; however, in doing so, he lost control of his bike and hit the highway median. On arrival, the emergency medical team conducts a rapid assessment, and he is assigned a Glascow Coma Score of 12. Question In addition to motor and verbal responsiveness, what needs to be assessed in order to determine the patient's Glascow Coma Score? Answer Choices 1 Appearance (skin color) 2 Eye response 3 Reflex response 4 Pulse 5 Respirations

eye response The Glascow Coma Score (GCS) is a neurological scale used to assess level of consciousness. It is used as an initial measure of consciousness following head injury, and it is also used to monitor patients following such injuries. There are 3 elements to the assessment: motor response, verbal response, and eye response. Scores range from 3 (deep unconsciousness) to 15 (normal), and are determined as follows: The GCS correlates to degree of brain injury: GCS ≥13, minor injury; GCS 9-12, moderate injury; and GCS 3-8, severe injury (coma). A GCS ≤ 8 generally indicates the need for airway protection via endotracheal intubation.

A 42-year-old woman presents for a post-operative visit 6 weeks following abdominal hysterectomy for fibroids. Her only current medications include the Norco (acetaminophen and hydrocodone) that she was prescribed after surgery, and she continues to take it sparingly. Today, she notes several new symptoms; they include decreased appetite, nausea, "bloating", and mild abdominal pain. These symptoms have been gradually getting worse over the course of the last 2 weeks. Her bowel movements have consisted of small amounts of liquid stools every few days. Question What is the most likely diagnosis? Answer Choices 1 Clostridium difficile infection 2 Postoperative ascites 3 Fecal impaction 4 Endometriosis 5 Small bowel obstruction

fecal impaction Fecal impaction presents with abdominal pain, nausea, vomiting, decreased appetite, abdominal distention, and diarrhea from liquid stool escaping around the impaction. Opiates can predispose a patient to fecal impaction. The remaining diagnoses are incorrect because they present differently than the patient in this case. C. diff infection presents with copious watery diarrhea many times a day. Ascites may present with bloating and abdominal pain, but it is typically associated with chronic liver disease and is not associated with diarrhea. Endometriosis is often asymptomatic, but it can be associated chronic pelvic pain that worsens with menses. While a bowel obstruction can be associated with fecal impaction, in such cases, it is typically a large bowel obstruction. A small bowel obstruction would present with paroxysmal severe mid-abdominal pain.

A 19-year-old G1P0 presents for her routine obstetric exam. She is at 34 weeks gestation. When completing the physical assessment you perform a Leopold maneuver. What does this maneuver assess? A Fetal lie and station B Fetal position and presentation C Fetal lie and position D Fetal lie and presentation E Fetal position and station

fetal lie and position C The maneuver is used to examine the abdomen and determine the lie (first) and position (second). While the abdominal exam can be used to determine breech versus cephalic, it does not differentiate between, face, brow, and footling presentations. A vaginal exam must be done to determine the presenting part and decent into the pelvis.

A 41-year-old woman presents due to rectal pain. The pain is brought on by bowel movements and is described as an intense tearing pain. She notes a small amount of blood on the toilet paper, but none in the stool itself. She has a bowel movement approximately every 2 to 3 days and notes that her stool is somewhat hard. On examination, a small crack is noted in the posterior midline. Hemoccult is negative. There are no masses noted on rectal examination. Question What is the recommended initial treatment for the patient's condition? Answer Choices 1 Fiber supplementation and sitz baths 2 Oral miralax 3 Topical nitroglycerin ointment 4 Oral prednisone 5 Surgical intervention

fiber supplementation and sitz bath Fiber supplementation and sitz baths is correct. The patient has an anal fissure. Anal fissures are associated with tearing pain during defecation, constipation, hematochezia, and the presence of a crack in the epithelium on examination. Since anal fissures are associated with constipation, fiber treatment is recommended to make the stool easier to pass, therefore placing less strain on the affected area. Sitz baths help alleviate the discomfort and irritation associated with anal fissures. Oral miralax is incorrect. Fiber should be trialed before using miralax to treat constipation. Topical nitroglycerin ointment is effective in the treatment of anal fissure, but it is considered a second line therapy. Oral prednisone is incorrect. Anal fissures that occur off the midline can potentially be caused by Crohn's disease, which could be treated with oral prednisone. However, the patient's fissure is located in the posterior midline, and she does not have any of the other typical symptoms of Crohn's disease, such as diarrhea and abdominal pain. Surgical intervention may ultimately be required for anal fissures, but is not considered a first line therapy.

A 61-year-old male presents with acute onset dyspnea, cough with thick, blood tinged sputum, and fever. He is a known, longstanding alcoholic and has Type II Diabetes, for which he takes no medication. A chest x-ray reveals a right upper lobe infiltrate with early fibrotic changes. Which of the following is an appropriate first-line medication(s) for the most likely etiologic organism? A Macrolide B Doxycycline C Second generation cephalosporin D Beta-lactam plus fluoroquinolone E Fluoroquinolone

fluorquinolone Klebsiella pneumoniae, a gram negative bacteria, is associated with pneumonia in immunocompromised patients, especially alcoholics. Community-acquired pneumonia due to Klebsiellae is associated with a broad range of presentations. This may range from mild acute bronchitis or bronchopneumonia to acute onset of high fevers, chills, and cough with blood tinged, thick sputum (often referred to as "currant jelly sputum"), and aggressive chest x-ray findings, such as cavitation and empyema. Community-acquired pneumonia empiric treatment is based upon the likely causative organisms within patient subsets. This includes appropriate selections for individuals able to be handled as outpatients, but with significant past medical history and risk factors for drug-resistant bacteria, such as alcoholism. The guidelines currently indicate that these patients should be treated with a beta-lactam plus a macrolide or doxycycline, or a fluoroquinolone, or amoxillin-clavulanate. The guidelines also include treatment recommendations for other patient subsets, and alternative medications to consider (which can be viewed in Table 251-4).

Your patient is a 77-year-old man complaining of stiffness and slowness that started several months ago and is more pronounced on the left side of his body. Neurological examination reveals tremor in rest, bradykinesia, cogwheel rigidity in his neck and limbs, and postural instability. His face is expressionless (mask-like) and he has a stooped posture; when he walks, there is mild retropulsion and no arm swings. Question What transient factor would you also expect to see? Answer Choices 1 Freezing 2 Titubation 3 Hemiparesis 4 Chorea 5 Paraparesis 6 Slapping

freezing your patient most probably has Parkinson disease or parkinsonism (tremor in rest, rigidity, bradykinesia, postural instability). Parkinsonian gait is characterized by the absence of arm swings, small shuffling steps, hypokinesia, and reduced walking speed. The patient has difficulty starting and stopping after starting. Freezing of gait is most often seen in the advanced stages of the disease. It is typically transient and lasts less than a minute, during which gait is halted and the patient feels that his/her feet are glued to the ground. When the patient overcomes the block, walking can be performed relatively smoothly. The most common form of freezing is "start hesitation", which is when the patient wants to start walking. It is provoked by placing a patient in a narrow space or in stressful situations, such as in the doctor's office. Later in the course of the disease, it can appear spontaneously.

A man presents with pulmonary edema, and he is treated with intravenous furosemide and oxygen. Clinical improvement is seen. He is also taking levothyroxine (Synthroid), acyclovir, triamcinolone acetonide (Nasacort), and azelastine hydrochloride (Astelin) spray. The pulmonary congestive symptoms are alleviated, and there is a significant increase in urine output. What medicine contributed most to the client's improvement as well as the significant increase in urine output? Answer Choices 1 Levothyroxine (Synthroid) 2 Acyclovir 3 Triamcinolone acetonide (Nasacort) 4 Azelastine Hydrochloride (Astelin) 5 Furosemide

furosemide The early effect of furosemide is thought to be due to direct pulmonary venous dilation. It may also redistribute pulmonary blood flow away from fluid-filled alveoli to well-aerated alveoli. This improves blood oxygen saturation and alleviates symptoms of congestive heart failure. Taken orally, the drug has an onset of action with 1 hour, and its diuretic effect lasts for 6 to 8 hours. Administered intravenously, it has a diuretic effect beginning within minutes and lasting approximately 2 hours. Levothyroxine (Synthroid) is used for hypothyroidism. Hypothyroidism is a slowing of the thyroid gland. Symptoms can be dry hair, dry skin, and puffiness of the face or hands and feet. The client can also suffer from depression, fatigue, weight gain or loss, and not sleeping. Triamcinolone Acetonide (Nasacort) is a nasal spray often used for allergies or chronic sinusitis. It is a steroid, and it works by shrinking the mucous membranes. Azelastine hydrochloride (Astelin) is a nasal spray used to shrink the mucous membranes, enabling the sinus to drain, or for seasonal allergies. Azelastine hydrochloride exhibits histamine H1-receptor antagonist activity in isolated tissues. Acyclovir is an antiviral medicine often used to treat the symptoms of herpes (both type 1 and type 2), herpes zoster (shingles), and chicken pox (varicella).

A 63-year-old man presents with a 6-month history of symptoms of urinary frequency, hesitancy, and nocturia. In addition, burning dysuria has occurred on 2 occasions, requiring treatment with antibiotics. He has a 1-year history of angina pectoris, for which he takes occasional nitroglycerin. On physical examination, the blood pressure is 130/90; heart rate is 90/min and regular, and an enlarged prostate is palpable per rectum. Laboratory data, including EKG, yields no contraindication to surgery; he is operated upon for a transurethral resection of the prostate. Anesthesia and surgery are uneventful, and blood loss is minimal. 6 hours postoperatively, he experiences a shaking chill, a temperature of 40 degrees C, and his blood pressure is 90/60 mm Hg. What is the most likely diagnosis? Answer Choices 1 Gram-negative bacteremia 2 Myocardial infarction 3 Postoperative bleeding 4 Arrhythmia 5 Lobar pneumonia

gram negative bacteremia Explanation The correct response is Gram-negative bacteremia. Some degree of enlargement of the prostate is extremely common from the age of 50 onwards, but this type of enlargement often produces either minor symptoms, or no symptoms at all. However, benign hypertrophy of the gland results in elongation and tortuosity of the prostatic urethra, and the median lobe may become a large, rounded, swelling overlying the posterior aspect of the internal urinary meatus. Here, it can act like a ball valve, producing urinary obstruction. The deranged anatomy in the region of the internal meatus, may allow urine into the prostatic urethra. The urine in this situation sets up a desire to micturate and this produces one of the most common symptoms of prostatism, namely, frequency. This is particularly worrisome to the patient at night, as it interferes with his sleep. The obstruction, and instrumentation to relieve it, predisposes to urinary infection. The obstruction to the outflow of the bladder may result in renal failure and uremia. Gram-negative enteropathogens are the most common cause of urinary tract infections and intra-abdominal sepsis, especially post-operatively, in the acute abdomen. Septicemia causes high fever, shivering, headache, and rapid breathing; it may progress to delirium, coma, and death. Myocardial infection gives rise to chest pain, which is usually of greater severity and duration than in angina, and is associated with nausea, vomiting, sweating, and extreme distress. The patient may be cold and clammy with tachycardia, hypotension, cyanosis, and mild pyrexia (Postoperative bleeding may lead to hypotension and hypovolemic shock, unless fluid volume is rapidly replenished. Arrhythmias may give rise to tachy- and bradycardias, which are sometimes felt as palpitations. They may also present with their hemodynamic consequences: dyspnea, angina, collapse, or 'funny turns'. Corresponding EKG changes are diagnostic. Pneumonia is relatively slow in onset, with symptoms of systemic upset, fever, pleuritic pain, cough, and green sputum (may be scanty at first, or, 'rusty' in color, if due to pneumococcal). On examination, there will be signs of consolidation, or just localized crepitations. Tachypnea is a valuable sign, especially in the elderly, in whom there is high index of suspicion.

The parents of a 3-year-old boy are worried about his delayed language development; they tell you that the boy always appears to be "in his own little world". You are told that his previous doctor diagnosed the child with autism. The parents want a second opinion. What finding correlates strongly with autism? Answer Choices 1 Ability to develop peer relationships 2 Ability for spontaneous play 3 Communication through gestures 4 Microcephaly 5 Stereotypical hand flapping

hand flapping Restrictive and stereotyped patterns of behavior such as hand flapping are common in autistic children; of the choices given, this finding most correlates with autism. The diagnosis of autism would require impaired social interaction, such as failure to develop peer relationships. A lack of varied, spontaneous play is common in autism. A child who is able to communicate through gestures is unlikely to be autistic. In fact, an autistic child has an absence of any type of communication and has a qualitative impairment of communication with delay in, or complete lack of, development of the spoken language. Microcephaly is uncommon in autism; however, macrocephaly is prevalent in autism.

One of your patients is requesting your help. He has seen three physicians in the past 3 months and is still having symptoms. He states, "I keep having these episodes of feeling like I am going to die. Out of the blue I feel real nervous, I get a splitting headache, break out in a sweat, and even feel like I am trembling. Sometimes I feel my heart beating out of my chest but don't have any pain or shortness of breath." He has had numerous tests including a cardiac stress test, multiple EKGs, complete blood count (CBC), basic metabolic panel (BMP), thyroid tests, and computed tomography (CT) scans of his head, which were normal. His last physician told him that he had an anxiety disorder and should try medications; he declined. His only medical problem is hypertension, which has worsened recently. He is currently taking lisinopril 20 mg QD, amlodipine 10 mg QD, and HCTZ 25mg QD. His physical examination is unremarkable except a BP of 190/92, P = 74. What is the best test to order to confirm your suspected diagnosis? A dexamethasone suppression test B sleep study C renal artery ultrasound D plasma fractionated free metanephrines E thyroid uptake scan

he correct answer is (D). The patient's symptoms and uncontrolled hypertension with a previous negative evaluation for cardiac, electrolyte, or thyroid causes highly suggests a pheochromocytoma as the possible cause. Plasma fractionated free metanephrines is a very sensitive test for the diagnosis of pheochromocytoma. Another test to consider would be evaluation of the total urinary metanephrines. A dexamethasone suppression test, choice (A), is used to r/o Cushing's syndrome, which is unlikely due to the normal physical examination findings. A sleep study, choice (B), (suggesting sleep apnea) and renal artery ultrasound, choice (C), (suggesting renal artery stenosis) are used to diagnose secondary hypertension, but would unlikely explain all the symptoms in the scenario. A thyroid uptake scan, choice (E), is not indicated since there is no indication of abnormal thyroid testing suggesting hyperthyroidism, or examination stated findings suggestive of a thyroid nodule.

A 23-year-old primigravida is admitted to your ward because of vaginal bleeding and cramping at 26 weeks. The ultrasound shows fetal heartbeat, and you try to stop a miscarriage by ordering terbutaline. What do you tell the nurse to watch for? Answer Choices 1 Constipation and dry mouth 2 Headache, tachycardia, and nausea 3 Sedation, urinary retention, and dry mouth 4 Anxiety, headache, and mydriasis 5 Cold extremities, arrhythmia, and insomnia

headache, tachycardia and nausea Explanation Terbutaline relaxes smooth muscle in the bronchial system and the uterus by stimulating β2-receptors. It is a tocolytic drug, and most commonly used in patients with bronchospasms caused by reversible obstructive airway disease. Headache, tachycardia, and nausea are common side effects of beta-mimetic drugs, which relax the uterine muscle. Other side effects are shown in the following table. Central nervous system Drowsiness, dizziness, headache, tremor, nervousness Gastrointestinal Nausea, vomiting Cardiovascular Palpitations, tachycardia, arrhythmia Respiratory Paradox bronchospasm Metabolic Hypokalemia Skin Diaphoresis Other Dry mouth and throat Constipation and dry mouth are side effects of skeletal muscle relaxants, like cyclobenzaprine, which is used as short-term treatment of muscle spasms. Sedation, urinary retention, and dry mouth are side effects of antihistamine drugs, which effect the peripheral H1-receptors. Urinary retention and sedation however are side effects of the "older" drugs like brompheniramine, chlorpheniramine, clemastine, promethazine, and triprolidine and not the "newer" ones like fexofenadine, loratadine, and cetirizine. Anxiety, headache, and mydriasis are signs of atropine overdosage. Atropin is an anticholinergic that inhibits acetylcholine at the parasympathica neuroeffector junction. It blocks vagal effects on the sinuatrial and atrioventricular nodes thereby enhancing conduction through the AV node and increasing the heart rate. It is used to treat bradycardia and to dilate the pupils. Cold extremities, bronchospasm, and insomnia are side effects of beta-blockers. Beta-blockers (or symppatholytics) decrease myocardial contractility, heart rate, blood pressure, and cardiac output (less blood getting to the periphery, which explains cold extremities). Myocardial oxygen use gets reduced. Since it decreases the effect of catecholamines on the bronchial system, it can trigger an asthma attack by increasing contraction of bronchioli and alveoli.

A 50-year-old obese woman underwent a cholecystectomy and T-tube drainage of the common bile duct. On the 7th postoperative day, she develops sudden epigastric and left-sided chest pain. She is short of breath and is sweating profusely. A review of the chart shows that her temperature is between 99 F and 100 F for the last 2 days. The patient's vitals: pulse 90/min, BP 110/60 mm Hg, and RR 24/min. The ECG reading shows tachycardia, and the ventilation-perfusion lung scan shows a VQ mismatch. Question What is the initial management of this patient? Answer Choices 1 Pulmonary embolectomy 2 Urokinase therapy 3 Exploratory laparotomy and drainage of abscess 4 Heparinization 5 Inferior vena caval filter

heparinization The diagnosis is pulmonary embolism, which should always be suspected if the patient suddenly collapses 1 - 2 weeks after surgery. Pulmonary embolism develops due to blockage of blood flow to the lungs caused by venous thrombi from deep vein thrombosis passing into the pulmonary circulation. It can be prevented by mobilizing the patient early following surgery and considering prophylaxis with heparin. Classically, it presents around 10 days post-op with collapse and sudden breathlessness while straining at stool; however, it may occur after any period of immobility or with no predisposing factors. Other features include pleuritic pain, dyspnea, shock, tachypnea, cyanosis, hemoptysis, gallop rhythm, pleural rub, right ventricular S3 gallop, and loud P2. Chest X-ray is often normal and sometimes with decreased vascular markings. Diagnosis is usually made clinically and confirmed by ventilation-perfusion mismatch on lung VQ (ventilation-perfusion) scan. An isotope lung scan using the macroaggregated albumin followed by ventilation scan using 133 Xenon can provide complementary evidence that the perfusion problem is due to pulmonary emboli. Initial management of this patient is adequate heparinization. Most patients with pulmonary emboli would resolve on adequate anticoagulation. The initial dose would be given intravenously and could be up to 10,000 units. Pulmonary embolectomy is indicated in only a very small group of patients who have massive embolus and are in a critical state. Exploratory laparotomy would be fruitless, as there will be no pathology inside the abdomen because the cause of this incident lies in insufficient physical mobilization after the operation. Treatment with streptokinase or urokinase is not recommended in this postoperative patient. Vena cava filter is indicated in patients who continue to throw emboli in spite of adequate anticoagulation or in whom heparinization is not advisable.

A 20-year-old woman presents with anorectal pain; there are streaks of blood on her stool and toilet paper. She states that "she has a tearing pain during each bowel movement". She dreads having a bowel movement, and she attempts to hold it as long as she can. The symptoms have been occurring over the past 2 weeks. Her history is also significant for breaking her leg in a skiing accident 4 weeks ago; she was prescribed Percocet for the first few days due to her pain. An anoscope is used; you find an acute anal fissure. Question What should be the next step in regard to treatment? Answer Choices 1 High fiber content/stool softeners 2 Topical Nitroglycerin 3 Lateral internal sphincterotomy 4 Botulinum Toxin 5 Continue Percocet for her anal rectal pain

high fiber content/stool softeners A high fiber diet/stool softeners should be tried first for about 6 weeks; doing so typically heals acute anal fissures. Percocet use is known to cause constipation and constipation is a cause for anal fissures. Unless she continues to have significant pain from her leg fracture, her Percocet should be discontinued to avoid further constipation. Topical nitroglycerin should be considered if the high fiber diet fails to heal the fissure. Botulinum toxin can be used in conservative treatment, but it is limited in regard to availability and expense. When all conservative therapies have been exhausted, lateral internal sphincterotomy should be used as a last resort.

The mother of a 2-year-old Caucasian boy explains that the boy has had a 4-day history of fever and cries during urination. She has been giving him acetaminophen, but the fever keeps coming back. She also says that his urine "smells funny." Family history is positive for a 4-year-old brother with Down syndrome. His temperature is 39°C. Urinalysis and culture reveal urinary tract infection. A VCUG and a renal bladder ultrasonography reveal vesicourethral reflux (VUR). Question What is a risk factor for VUR in this boy? Answer Choices 1 His gender 2 Acetaminophen treatment 3 His age 4 His ethnicity 5 Family history of Down syndrome Your Answer: Correct His ethnicity

his ethnicity Ethnicity is a significant risk factor for vesicourethral reflux (VUR) because it is 10 times more common in Caucasians than African-Americans. VUR is the retrograde flow of urine from the bladder towards the kidneys due to a dysfunctional vesicoureteric junction. It may occur alone or in association with other urological conditions like posterior urethral valves. Although prevalence is more common in male neonates, VUR is 5 times more common in female children compared to male children after the age of 1 year. Acetaminophen treatment and family history of Down syndrome have no association with VUR.

A young child and his parents have been adhering to the treatment plan for type 1 diabetes, as discussed with their health care provider. It includes a change in diet, as well as blood glucose and ketone monitoring. They noticed that the amount of insulin needed decreased after the first two weeks. What is this time period commonly called? A Postprandial control phase B Glucose tolerance time C Pre-diabetic period D Mature onset diabetes of youth E Honeymoon phase

honeymoon phase The correct choice is E, honeymoon phase. During this time, some pancreatic beta cell function may recover, although within eight weeks to two years most patients will show absent or negligible pancreatic beta cell function. Choice A, postprandial control phase, choice B, glucose tolerance time, and choice C, pre-diabetic period, are not true time periods. Choice D, mature onset diabetes of youth, is a subgroup of autosomal dominant inherited disorders, characterized by diabetes in non-obese older children that are not ketosis prone and generally do not need insulin therapy to control their disease. This type of diabetes accounts for up to 5% of diabetes in North America and Europe.

Your supervising physician asks you to advise him which finding is least likely to be suggestive of a thyroid malignancy in your 49-year-old female with a small palpable thyroid nodule. Which of the following choices would be least likely to suggest malignancy in this patient? A ultrasound showing lesion with microcalcifications B ultrasound showing a lesion of > 1 cm C hot nodule on 123I uptake scan D ultrasound showing a solid lesion E cold nodule on 123I uptake scan

hot nodule on 123I uptake scan C The correct answer is (C). A hot nodule, which is a hyperfunctioning thyroid nodule, suggests a benign etiology. The other choices, including ultrasound findings of microcalcifications, solid lesions, and lesions > 1 cm, should increase your index of suspicion for possible malignancy. Cold nodules are nonfunctioning thyroid nodules, which should increase your suspicion, especially in combination with suspicious ultrasound and/or clinical examination findings.

A 60-year-old man presents for evaluation of loss of recent and remote memory, psychomotor slowing, and difficulty with complex tasks. On exam, the patient also shows some tremor tics, dystonia, and impaired coordination. Family history reveals he had 2 brothers who both died of a strange "movement" disorder. What is the provisional diagnosis? Answer Choices 1 Delirium 2 Dementia of the Alzheimer's type 3 Vascular dementia 4 Huntington's disease 5 Substance induced dementia

huntington's disease Dementia due to medical conditions include loss of memory and other cognitive deficits due to diseases such as Huntington's disease, Parkinson's, Pick's disease, multiple sclerosis, HIV disease, vitamin B-12 deficiencies, and hypothyroidism. Head traumas and tumors can also cause dementia, and all physical causes must be ruled out first. Vascular dementia presents with the same signs and symptoms as other dementias; however, the patient usually has a strong history of cerebrovascular disease, multiple infarcts, or history of arteriosclerotic disease. Vascular dementia usually has a sudden acute onset after the infarcts. Vascular dementia may also include behavioral disturbances in the patient. In patients with a delirium, there are usually rapid fluctuations in cognitive functioning, brief durations of normalcy, and disturbances in the sleep-wake cycle. In addition, deliriums come on acutely and rapidly. Deliriums are somewhat common in older patients who have had heart surgery and in those who have experienced a change of environments, such as a hospital intensive unit. Clients with substance-induced dementia have cognitive deficits related to a drug or alcohol use or abuse and/or toxicity of prescription drugs; the toxicity can be accidental or intentional. This appears to be more common in the elderly than once thought. Many elderly take prescription drugs incorrectly or mix them with contraindicated over-the-counter drugs. Also, patients may suffer side effects and not report them to their physicians. Dementia of the Alzheimer's type involves gradual loss of memory and other cognitive functioning, which results in the inability to accomplish activities of daily living. Before a diagnosis is made, all other physiological causes should be ruled out.

A 33-year-old African American woman develops a loss of appetite and has a few episodes of vomiting. She reports constipation and lethargy. She thinks that her urine output has increased. Her history is significant for the presence of sarcoidosis. On physical examination, some muscle weakness is present. Laboratory tests are ordered and are pending. An EKG is done and shows a shortened QT interval. Question Based on the EKG changes, what electrolyte abnormality would be expected? Answer Choices 1 Hypercalcemia 2 Hyperkalemia 3 Hypocalcemia 4 Hypokalemia 5 Hyponatremia

hypercalcemia Hypercalcemia refers to an elevated calcium level. A shortened QT interval is seen with hypercalcemia. The time from the beginning of the QRS complex to the ending of the T wave is the QT interval. Sarcoidosis can be associated with hypercalcemia. Symptoms of hypercalcemia include anorexia, vomiting, constipation, and polyuria. Muscle weakness can be seen. None of the other electrolyte abnormalities listed are associated with sarcoidosis. Hyperkalemia refers to an elevated potassium level. Hyperkalemia can result in peaked T waves on EKG. The T wave is a reflection of ventricular repolarization. There can be widening of the QRS complex as well. The QRS complex is seen when the ventricle depolarizes. Hypocalcemia refers to a depressed calcium level. There would be a prolonged QT interval with hypocalcemia. The time from the beginning of the QRS complex to the ending of the T wave is the QT interval. Hypokalemia refers to a depressed potassium level. Hypokalemia can result in a depressed ST segment and the presence of U waves. The time from the end of the QRS complex to the beginning of the T wave is the ST segment. Hyponatremia refers to a depressed sodium level. Hyponatremia is not associated with EKG changes.

A 38-year-old woman presents for treatment of a minor burn on her left hand. She is nervous, hyperexcitable, and experiencing palpitations. Her pulse is up to 110/min; her blood pressure is 120/80, and you notice a fine tremor of her hands. Upon further questioning, she tells you that she has been losing weight even though she's been eating a lot; she also sweats a lot, especially at night, and she has problems sleeping. She also notes frequent bowel movements, occasional diarrhea, and irregularities in her menstruation. What is the most likely diagnosis? Answer Choices 1 Hyperthyroidism 2 Hypothyroidism 3 Menopause 4 Malignant tumor 5 Pheochromocytoma

hyperthyroidism

Excessive doses of some vitamins can cause toxicity. For a certain vitamin, early manifestations of chronic toxicity include dermatological findings like dry scaly skin and hair loss. Anorexia, vomiting, and hyperostosis may also be seen. What hypervitaminosis can cause these symptoms? Answer Choices 1 Hypervitaminosis A 2 Hypervitaminosis B6 3 Hypervitaminosis C 4 Hypervitaminosis E 5 Hypervitaminosis B3

hypervitaminosis A Very large doses of the fat-soluble vitamins are definitely toxic. Acute vitamin A intoxication was first described by Arctic explorers, who developed headache, diarrhea, and dizziness after eating polar bear liver. The liver of this animal is particularly rich in vitamin A. Nausea, vomiting, papilledema, and lethargy may be seen in acute toxicity. Chronic toxicity can occur when a person consumes 50,000 units per day for 3 months or more. Mouth sores may also be seen as an early manifestation along with those mentioned in the question. Hypercalcemia, increased intracranial pressure with papilledema, and decreased cognition may be seen in more serious cases. Hepatomegaly leading to cirrhosis may occasionally occur. Excessive vitamin A has also recently been related to increased risk of hip fracture. Elevated vitamin A in the blood is the only clue to the diagnosis. Elimination of vitamin A from the diet usually results in rapid recovery. Large doses of water-soluble vitamins (C and B) have been thought to be less likely to cause problems because they can be rapidly cleared from the body; however, it has been demonstrated that ingestion of large doses of vitamin B6 can produce peripheral neuropathy and large doses of niacin (B3) may cause skin flushing. Very large doses of vitamin C can cause gastric irritation, flatulence, or diarrhea. There is a theoretical risk of oxalate stones, as oxalate is metabolized by vitamin C; however, not many such cases have been reported. Vitamin E is the least toxic of the fat-soluble vitamins. Large doses taken over extended periods of time have not produced toxicity, except for a few GIT disturbances.

A 5-month-old female infant presents with a 3-day history of vomiting. She is exclusively breastfed, and her mother states that today she has vomited within 15 minutes of each feeding. Her last wet diaper was 10 hours ago. On physical examination, she is afebrile, tachycardic, irritable, and does not express tears when crying. She was a full-term infant born via vaginal delivery. She has no significant past medical history. Her 3-year-old sister has had gastroenteritis for the past few days. Question What is the most likely laboratory finding? Answer Choices 1 Decreased blood urea nitrogen 2 Hyponatremia 3 Hypokalemia 4 Decreased urine specific gravity 5 Hypernatremia

hypokalemia This infant is showing signs of moderate dehydration due to emesis and limited breast milk intake. Gastric and urinary potassium losses can both contribute to hypokalemia. Dehydration leads to hemoconcentration with an increased hemoglobin and increased urine specific gravity. BUN and creatinine could be increased with volume depletion, and they could be increased further with renal insufficiency. Not eating for an extended period of time could lead to hypoglycemia in a 5-month-old infant. Hypernatremia implies a deficit of total body water. Major symptoms are thirst, confsion, neuromuscular excitability, seizures, and coma.

Addison's disease, also known as adrenocortical insufficiency, manifests itself by which one of the following? Answer Choices 1 Hyponatremia 2 Truncal obesity 3 Glucose intolerance 4 Hypertension 5 Osteoporosis

hyponatremia Explanation Deficiency of adrenocorticosteroids is the hallmark of Addison's disease, which represents a failure of the adrenal cortex to secrete steroids in adequate amounts to satisfy metabolic needs. 1 of the functions of these adrenal steroids is to regulate sodium metabolism; specifically, they do so by enhancing sodium reabsorption in the presence of hyponatremia or volume depletion. Without this function, a common manifestation of adrenal insufficiency is hyponatremia. The other clinical manifestations listed are typical of the opposite syndrome, hyperadrenocorticism, or Cushing's syndrome.

A 30-year-old patient presents 2 months postthyroidectomy. The patient has had symptoms of increased irritability, muscle spasms, and hair loss for the past month. On physical examination, a positive Chovstek sign is noted. Which of the following is the most likely diagnosis? A hypothyroidism B hypopituitarism C hypoparathyroidism D hypogonadism

hypoparathyroidism Hypoparathyroidism commonly presents following thyroidectomy surgery. This patient has classic signs and symptoms of a low calcium level and hypoparathyroidism. Chovestek sign is a physical exam finding that is positive after tapping in front of the ear in the facial nerve region. When doing this, the muscle contracts. When the calcium level is low, this occurs. Hypothyroidism can occur following a thyroidectomy but the symptoms are not the same.

A 72-year-old man is evaluated following admission to the hospital for a 1-year history of progressive dyspnea, nonproductive cough, weight loss, low-grade fevers, fatigue, and myalgias. His past medical history is remarkable for atrial fibrillation (for which he takes amiodarone), hypercholesterolemia, and recurrent urinary tract infections, for which his urologist prescribed nitrofurantoin on a chronic, prophylactic basis. He denies cigarette use, a history of murmurs or coronary artery disease, chills, fatigue, rhinitis, otalgia, chest pain, wheezing, hemoptysis, syncope, abdominal pain, rashes, peripheral edema, diaphoresis, arthralgias, vomiting, and urinary complaints. A bedside echocardiogram and electrocardiograms are unremarkable for abnormalities; a chest X-ray revealed peripheral reticular opacities at the lung bases and a generalized honeycombing pattern. Question What is the most likely diagnosis? Answer Choices 1 Idiopathic pulmonary fibrosis 2 Pulmonary embolism 3 Sarcoidosis 4 Goodpasture's syndrome 5 Wegener's granulomatosis

idiopathic pulmonary fibrosis This patient's most likely diagnosis is idiopathic pulmonary fibrosis. Amiodarone, bleomycin, and nitrofurantoin are notable medications associated with pulmonary fibrosis. Most patients with idiopathic pulmonary fibrosis present with a gradual onset, which is often greater than 6 months. The clinical symptoms of idiopathic pulmonary fibrosis are nonspecific; symptoms often precede the diagnosis by a median of 1 - 2 years. Most patients present with exertional dyspnea and a nonproductive cough. Associated constitutional symptoms are uncommon. The physical examination reveals fine bibasilar inspiratory crackles (Velcro crackles) and digital clubbing in 25 - 50% of cases. Typical chest X-ray findings include peripheral reticular opacities (netlike linear and curvilinear densities) that are predominantly located at the lung bases, honeycombing (coarse reticular pattern), and lower lobe volume loss. The diagnosis of pulmonary embolism almost always occurs with underlying predisposing conditions present; venous thrombosis may result from a generalized hypercoagulable state, venous endothelial injury, or local stasis (Virchow triad). Most commonly, the initial manifestations of pulmonary embolism include an abrupt dyspnea and chest pain. Tachycardia and hypoxia are the most common clinical signs. Associated manifestations include fever, hypotension, cyanosis, pleural friction rub, and findings consistent with pulmonary consolidation. ECG most often shows tachycardia and various ST-T wave abnormalities, which are not specific for PE. Common chest radiographic abnormalities include atelectasis, pleural effusion, parenchymal opacities, and elevation of a hemidiaphragm. The classic radiographic findings of pulmonary infarction include a wedge-shaped, pleura-based triangular opacity with an apex pointing toward the hilus (Hampton hump) or decreased vascularity (Westermark sign). Sarcoidosis is a multisystem inflammatory disease of unknown etiology that manifests as non-caseating granulomas, predominantly in the lungs and intrathoracic lymph nodes. The presentation commonly includes systemic complaints of fever, anorexia, and arthralgias, as well as pulmonary complaints such as dyspnea on exertion, cough, chest pain, and (rarely) hemoptysis. Extrapulmonary findings are common, and include erythema nodosum, lower-extremity panniculitis, lupus pernio, a facial violaceous rash, maculopapular plaques, granulomatous uveitis, conjunctival lesions, scleral plaques, cardiomyopathy, and cranial nerve palsies. The chest X-ray commonly reveals air trapping, lymphadenopathy, and infiltrates. Goodpasture's disease is a condition of glomerulonephritis, with or without pulmonary hemorrhage, and the presence of circulating anti-glomerular basement membrane (anti-GBM) antibodies. Constitutional symptoms, such as malaise, chills and fever, and/or arthralgias, may precede or be concurrent with pulmonary or renal manifestations. Hemoptysis, cough, dyspnea, and shortness of breath describe pulmonary involvement, while hematuria, edema, high blood pressure, and uremia signify renal affliction. Significant anemia and chest pain may also occur. Wegener granulomatosis is a rare multisystem autoimmune disease of unknown etiology; its hallmark features include necrotizing granulomatous inflammation and pauci-immune vasculitis in small- and medium-sized blood vessels. A wide spectrum of extrapulmonary manifestations, which include recurrent respiratory infection in adults and upper and lower respiratory tract problems in children, is expected. Manifestations include constitutional complaints, conjunctivitis, episcleritis, uveitis, optic nerve vasculitis, retinal artery occlusion, nasolacrimal duct occlusion, proptosis, chronic sinusitis, epistaxis, saddle nose deformity, serous otitis media, hearing loss, strawberry gingival hyperplasia, stridor, myalgias, arthritis, arthralgias, glomerulonephritis, and renal failure. Later-onset findings reveal neuropathic and cranial nerve abnormalities. Other manifestations include CNS small- to medium-sized vessel vasculitis, a palpable purpura, splanchnic vasculitis, myocardial infarction, and/or cardiac friction rubs.

A 50-year-old man presents with episodes of flushing. He is quite disturbed because it starts in his face and goes from red to deep violet. He also experiences diarrhea, cramps, and nausea. It is determined that he has a carcinoid tumor that has metastasized. Where is the most likely site of the primary tumor? Answer Choices 1 Prostate 2 Sympathetic chain ganglia 3 Adrenal medulla 4 Adrenal cortex 5 Ileum

ileum This patient has signs of carcinoid syndrome. The ileum is a common site for a carcinoid tumor; however, they can be found in other areas of the body as well. Other common sites for carcinoid tumors are other regions of the gastrointestinal tract and the bronchi. Of the choices given, the ileum is the most likely primary site of his tumor, not the prostate, sympathetic chain ganglia, adrenal medulla, or adrenal cortex. Carcinoids produce and secrete 5-hydroxytryptamine, which is serotonin. Carcinoids can produce other things as well; however, 5-hydroxytryptamine (serotonin) is the major product seen with carcinoid tumors. Urinary 5-hydroxyindoleacetic acid (5-HIAA) will be elevated with carcinoid syndrome and can be used in diagnosis.

A mother brings in her 20-month-old female child to the office because she noticed pubic hair growing. On examination, the clinician notices that the clitoris is enlarged; the rest is unremarkable. Which of the following is an expected laboratory finding on this patient? A increased aldosterone B increased estrogen C increased androstenedione D increased luteinizing hormone

increased androstenedione C Infant girls presenting with signs of precocious puberty need to be screened for congenital adrenal hyperplasia (CAH). CAH most commonly presents with pseudohermaphroditism in females—urogenital sinus, enlarged clitoris, or other signs of virilization. In males, there tends to be isosexual precocity in older males and salt-losing crisis in infant males. Both children show increased linear growth and skeletal maturation. The most common type of CAH is a deficiency in the enzyme 21-hydroxylase and laboratory tests demonstrate increased urinary and plasma androgens (DHEA, androstenedione). There may be elevated progesterone, but typically there is no effect on estrogen. There is also decreased aldosterone and elevated urinary ketosteroids. There is also no effect on the levels of leuteinizing hormone or follicle-stimulating hormone. Treatment usually involves glucocorticoids, mineralocorticoids, and reconstructive surgery, if needed.

A 28-year-old female presents with a gradual progression of fatigue and pallor over the last few months. Initial CBC results show a hemoglobin of 10.4 mg/dL, hematocrit of 32%, an MCV of 112 fL, and a reticulocyte count of 0.1%. Which of the following is the most likely pathophysiologic mechanism responsible for her anemia? A Chronic blood loss B Defective bone marrow/stem cell function C Defective DNA production D Defective hemoglobin production E Increased destruction of red blood cells

increased destruction of RBCs The time course of the patient's presentation is consistent with multiple episodes of acute hemolysis. Defects in bone marrow (B) or red blood cell precursors (C and D) are refuted by the elevated reticulocyte count. Chronic blood loss (A) would have a more insidious, gradual onset and likely result in a decreased MCV.

A 3-year-old boy presents following a car accident. He had not been buckled in, and he was thrown against the windshield. When the paramedics arrived at the scene, he was crying and clinging to his mother. Upon arrival at the emergency department, he is sobbing and does not react to her any more. He sustained lacerations of the face and both arms. His blood pressure is 150/84, pulse 62, and the pupils react sluggishly. He also has a temperature of 39.2° C. What do his symptoms indicate? Answer Choices 1 Viral infection 2 Reye's syndrome 3 Increased intracranial pressure 4 Anxiety attack 5 Guillain-Barré syndrome

increased intracranial pressure This child most certainly sustained head injuries with increased intracranial pressure in the accident. It is very likely, since he was not buckled in, that his face and arms were cut when he was thrown against the windshield. His pupil reaction, disorientation, and not recognizing his mother all support that theory. Elevated temperature is common in patients with head injuries and is called 'central fever'. The only symptom that points to viral infection is the fever, but there would not be any changes in pupil reaction. Reye's syndrome is caused by aspirin medication with a viral infection being present. It affects the central nervous system, but it is not related to trauma. A child may cry and be very anxious in a hospital setting, but that would not cause temperature elevation and sluggish pupils. Guillain-Barré syndrome, or acute idiopathic polyneuritis, is a rapidly progressing ascending motor neuron paralysis of unknown etiology, which often follows viral infections.

A 1-year-old boy presents with increasing lethargy. He is barely responsive, and his parents deny any trauma or injury. What is the most common cause of nontraumatic altered levels of consciousness? Answer Choices 1 Seizure disorder 2 Diabetic ketoacidosis 3 Inborn errors of metabolism 4 Toxic ingestion 5 Infection

infection Awareness of self and the surrounding environment or consciousness may be altered into different abnormal states of consciousness. Consciousness can shift from loss of clear thinking or confusion, usually accompanied by disorientation, to delirium, a succession of confused and unconnected ideas manifested in children as extreme mental and motor excitement, to lethargy, a profound type of slumber where movement or speech is limited, to stupor or deep sleep where arousal is achieved only by repeated vigorous stimuli, finally to coma, unresponsiveness to even painful stimuli. Non-traumatic coma is most common in infants and toddlers with another smaller peak of occurrence in adolescence. The most common cause of non-traumatic altered level of consciousness in children is infection of either the brain (encephalitis), meninges (meningitis), or both; infections account for more than 1/3 of cases. Prolonged seizures, anticonvulsive therapy, and postictal states can also lead to altered levels of consciousness. The most common metabolic cause of alteration of consciousness is diabetic ketoacidosis, which can occur at any age, but is most common in adolescence. Caused by severe insulin deficiency, hyperglycemia and ketogenesis lead initially to polyuria, polydipsia, hyperpnea, vomiting, and abdominal pain. As the process progresses, hyperosmolar dehydration and acid/base and electrolyte disturbances occur. Advanced stages alter level of consciousness and can lead to coma. Alterations of consciousness due to inborn errors of metabolism that present with electrolyte and glucose abnormalities typically present in infancy. The availability of gluconeogenic precursors or the functions of the enzymes required for production of hepatic glucose are affected. Metabolic defects causing hypoglycemia include glycogen storage disease, galactosemia, fatty acid oxidation defects, carnitine deficiency, several of the amino acidemias, hereditary fructose intolerance, and defects of other gluconeogenic enzymes. Toxic ingestion and exposure are very common in toddlers and adolescents, with a toddler's ability to explore his environment filled with often brightly colored medications and intentional ingestion by adolescents typically involving over-the-counter medications or psychotropic drugs such as antidepressants.

Patients suffering from asthma, no matter the classification, should be prescribed what medication? Answer Choices 1 Inhaled corticosteroid 2 Inhaled long-acting β2-agonist 3 Leukotriene receptor antagonist 4 Inhaled short-acting β2-agonist 5 Omalizumab

inhaled short acting B2 agonist Inhaled short-acting β2-agonist (SABA) is the appropriate answer. SABA is considered a quick-relief medication that acts by relieving and reversing acute airflow obstruction. Whether the patient is diagnosed with intermittent or persistent asthma, SABA is the drug chosen to be used for acute symptomatic periods and is typically instructed to be used on an as needed basis. Patients are explicitly educated on the "Rule of 2's" in relation to the use of SABA: if using SABA >2 days a week, if waking up at night due to asthma or cough more than 2 times in a month, or if having to refill their prescribed SABA more than 2 times a year, this is highly suggest that there is inadequate control, and a step up treatment plan needs to be made in terms of asthma management. SABA can be administered via an inhaler or nebulizer machine. Inhaled corticosteroids, long-acting β2-agonists, and Leukotriene receptor antagonists are used in varying degrees, dosages, and combinations for long-term control (maintenance) of asthma. Maintenance medications are taken daily despite whether the patient is symptomatic or not. Depending on the medication, the route of administration could be via inhalation or an oral tablet. Omalizumab is a subcutaneous injection indicated for patients with moderate to severe persistent allergic asthma caused by year-round airborne allergens. This injection is given every 2-4 weeks in the provider's office.

A 32-year-old man presents due to occasional shortness of breath and associated cough, especially when he is working outside. He has associated chest tightness, which resolves within minutes when he sits down and rests. These symptoms occur 1-2 days a month. He is otherwise healthy and does not smoke. Blood pressure is 128/74; pulse is 76; respiration is 14; and pulse oximetry is 100% on room air. His FEV1 is 96%. Question What is the treatment of choice? Answer Choices 1 Inhaled corticosteroid 2 Inhaled short acting beta agonist 3 Inhaled long acting beta agonist 4 Inhaled anticholingeric 5 Oral beta agonist

inhaled short acting beat agonist This patient has intermittent asthma, which is defined as symptoms ≤ 2days per week, nighttime awakenings ≤ 2 times a month, and no interference with normal activities with a normal FEV1 between exacerbations. The treatment of choice is inhaled short acting beta agonist (SABA), albuterol, when needed. If the asthma occurs due to an allergen or in a predictable manner, the patient can use the SABA prior to the exposure and avoid triggers as much as possible. A patient with persistent asthma requires treatment with an inhaled corticosteroid as a controller medication. The dose (low, medium, or high) depends on the severity of the asthma and the level of asthma control. A long-acting beta agonist may be added for controller medication. The patient would continue to use a SABA as needed for acute symptoms. Inhaled anticholinergic agents are used to treat chronic obstructive disease (COPD), but may be used as an adjunctive therapy to SABA for acute asthma exacerbations. Oral beta agonists have a delayed therapeutic response and are not useful for acute attacks. Inhaled SABA is more effective.

A 22-year-old man presents with fever and a transient maculopapular rash of 1 week duration. His serum creatinine and blood urea nitrogen (BUN) are elevated. The urinalysis is significant for hematuria, pyuria, white blood cell casts, and eosinophiluria. What is the most likely diagnosis? Answer Choices 1 Acute tubular necrosis 2 Diabetic nephropathy 3 Hypertensive nephrosclerosis 4 Interstitial nephritis 5 Lupus nephritis

interstitial nephritis The clinical picture of fever, recent etiological exposure (e.g., drugs, infection), maculopapular rash with serum and urinalysis findings of elevated creatine, elevated BUN, hematuria, pyuria, white blood cells casts and eosinopiluria is suggestive of interstitial nephritis. Interstitial nephritis typically occurs following medication administration, but can also occur in response to viruses or bacterial infections. It is an allergic reaction of the kidney that results in fevers, rash, arthralgias, hematuria, and eosinophilia. The eosinophils in the urine are pathognomonic for this disease. In acute tubular necrosis, the BUN and creatinine are elevated but the urinalysis may show a brown color. On microscopic examination, muddy brown casts (pigmented granular casts), epithelial cell casts, and renal tubular cells would be seen, which is not the case in this patient. There is no history of diabetes or hypertension in this patient; therefore, diabetic nephropathy and hypertensive nephrosclerosis are ruled out. Lupus nephritis is a complication of systemic lupus erythematosus. It is an autoimmune inflammatory disorder that affects many organs. 85% of patients are women. On urinalysis, hematuria and proteinuria are common findings.

A 19 year old female presents for her first pelvic examination and is noted to have a palpable, non-tender right ovary. She is sexually active, and uses condoms for birth control and protection from STDs. Her LMP was 20 days ago. She underwent transvaginal ultrasound and was found to have a 3-cm follicular cyst. What does this right ovarian mass represent? A Retained hemorrhagic products B Intrafollicular fluids C Solid intracellular material D Germ cell layers E Keratinized squamous epithelium

intrafollicular fluids B Follicular cysts occur prior to ovulation and cause expansion of the follicular antrum, serious fluid collection, and subsequent follicular cyst formation. Corpus luteum cysts form after ovulation and may accumulate blood products. Germ cell layers and keratinized squamous epithelium are solid components of teratomas.

A 5-month-old male infant presents after a seizure involving all 4 limbs. His mother tells you that he was born full term without any complications, and he was well until 2 days ago when he developed a fever. He vomited multiple times yesterday and was irritable. He has not had diarrhea or a cough. He was given antipyretic medication for his fever. He has no known allergies. His immunizations are up-to-date. His developmental milestones have been in accordance with his age. On physical exam, his temperature is 102.7 F, and his pulse is 154/min; BP is 90/50 mmHg, and RR is 20/min. He is lethargic, pale, and focal neurological deficits are present. His anterior fontanel is bulging. You suspect that he has bacterial meningitis. Question After drawing blood samples for investigations, what is the most appropriate next step? Answer Choices 1 Intravenous phenytoin 2 Intravenous empirical antibiotics 3 MRI of the head 4 Lumbar puncture 5 Intravenous glucose

intravenous emiprical antibiotics The infant in the vignette appears to have bacterial meningitis. The initial approach to the patient should be the "ABCs." After assessing and stabilizing the patient's airway and obtaining IV access, intravenous antibiotics should be given immediately. As bacterial meningitis is associated with high morbidity and mortality, prompt initiation of empirical antibiotics is crucial for better prognosis. The choice of antibiotics is dependent on the patient's age and specific predisposing conditions. Use of broad-spectrum cephalosporins, such as ceftriaxone or cefotaxime with vancomycin, may be used in infants more than 1 month old. Ideally, serum glucose, blood culture, complete blood count, and serum chemistries should be drawn when IV access is obtained; however, drawing labs should not delay beginning antibiotics. Intravenous glucose is necessary if the patient is found to be hypoglycemic; bedside serum glucose is mandatory in any patient that presents with a seizure. Intravenous phenytoin and an MRI of the head might also be necessary for a patient such as the one in the vignette, but would not emergently precede antibiotics. The diagnosis of bacterial meningitis rests on CSF examination performed after lumbar puncture. However, LP is deferred in patients with evidence of increased intracranial pressure, new onset seizure, cardiorespiratory compromise, or focal neurological deficits. Antibiotics should be given, and CT scan of the head should be performed. If CT scan is negative, LP can be performed.

A 20-year-old woman presents with a 1-week history of dizziness. She reports that her menstrual periods have been heavy since menarche at the age of 14 years. On examination, she is pale and has koilonychia. Laboratory investigations reveal normal prothrombin time, normal activated partial thromboplastin time, and a normal platelet count. Question What is the most likely diagnosis? Answer Choices 1 Thiamine deficiency 2 Riboflavin deficiency 3 Vitamin K deficiency 4 Vitamin D deficiency 5 Iron deficiency

iron deficiency In cases of iron deficiency, patients can present with feeling weak, dizzy, and tired; they may experience syncope. On examination, they have pale conjunctivae and koilonychia (spooning of the nails). Causes include inadequate dietary intake as well as blood loss in menorrhagia and during delivery, or after trauma. Dietary sources of iron include beef, liver, and dark green vegetables. In riboflavin or vitamin B2 deficiency, patients present with angular stomatitis and cheilosis. On examination, they are pale, have atrophic glossitis, and the tongue may appear magenta. Vitamin B1 (or thiamine) deficiency results in beriberi, which is characterized by: A bilateral symmetric peripheral neuropathy beginning in the legs. Wernicke-Korsakoff syndrome, which is comprised of nystagmus, ophthalmoplegia, ataxia, memory loss, and confabulation. Congestive heart failure with tachycardia, peripheral edema, and cardiomegaly. In cases of vitamin K deficiency, patients present with bleeding tendencies such as epistaxis, menorrhagia, and hematuria. The prothrombin time (PT) and the activated partial thromboplastin time (aPTT) are usually prolonged. In cases of vitamin D deficiency, children can present with inability to walk unsupported due to muscle weakness and lower limb skeletal deformities, such as genu varum and genu valgum.

A 38-year-old man presents with upper abdominal pain. More recently, he has experienced nausea. A stool sample is tested for occult blood, and it is positive. An upper endoscopy reveals no esophageal lesions, but there is a solitary 2-cm diameter ulceration of the stomach. The ulceration is sharply demarcated. What is the most appropriate statement regarding this lesion? Answer Choices 1 It is probably located in the antrum 2 It is probably associated with increased gastric acid production 3 A gastrinoma of the pancreas is probably present 4 It is probably malignant 5 Because of its small size, a biopsy is not necessary

it is probably located in the antrum From the findings described in the above patient with a sharply demarcated area of ulceration suggests that it is a benign peptic ulcer. The gastric antrum is a typical location for benign peptic ulcer. There is usually decreased acid production in patients with peptic ulceration, but not a total absence of acid production. A gastrinoma of the pancreas (Zollinger-Ellison syndrome) will be accompanied by multiple gastric ulcerations. The small size with the sharp demarcation suggests that it is a benign ulcer; however, the size is not a reliable clue that it is benign. All gastric ulcers seen on endoscopys hould be biopsied.

A 48-year-old HIV positive man starts to develop headaches. At first, he attributes the headaches to stress. However, they persist and become worse over the next few weeks. He develops nausea and vomiting, and he thinks he has a fever. He starts to become confused, so he seeks medical attention. On physical examination, his temperature is 100°F. He has signs of meningeal irritation. A lumbar puncture is performed, and there is an elevation of his CSF pressure. Cerebrospinal fluid is sent to the lab. The CSF is centrifuged. A drop of India ink is placed on the slide along with a drop of the spun CSF. The India ink test is positive. Question What diagnostic sign may be found in this patient? Answer Choices 1 Kernig's sign 2 Babinski's sign 3 Kussmaul's sign 4 Quincke's sign 5 Chvostek's sign

kerning sign Kernig's sign is a sign of meningeal irritation. The patient lies on his back, and his leg is raised and knee bent at a 90-degree angle. If there is pain or resistance to further extension when the examiner straightens the knee, Kernig's sign is said to be positive. This patient has signs and symptoms of cryptococcal meningitis. Cryptococcus neoformans is a yeast with a capsule made of polysaccharides. India ink exam is positive for Cryptococcus neoformans. When visualized on high dry magnification, encapsulated organisms (Cryptococcus neoformans) have capsules that look like halos because they exclude the India ink. Cryptococcus neoformans can cause meningitis, and there is an increased risk of Cryptococcus neoformansmeningitis in immunosuppressed individuals. Babinski's sign is an abnormal response to stimulation on the sole of the foot that results in dorsiflexion of the big toe and fanning of the other toes. Babinski's sign is seen with pyramidal disease. Kussmaul's sign is an increase in venous pressure during inspiration; it can be seen with cardiac tamponade. Quincke's sign is also called Quincke's pulse. This sign is seen in the nails. It consists of whitening and reddening of the nail bed coinciding with each heartbeat. Quincke's sign can be seen with aortic insufficiency. Chvostek's sign can be seen with tetany. When tapping on the facial nerve produces contraction on that side of the face, it is called Chvostek's sign.

A 53-year-old man presents with a 2-day history of jaundice and malaise. His history is significant for mucosal candidiasis, for which the patient is on oral ketoconazole 200 mg daily for the past 3 weeks. He occasionally has headaches, and sometimes takes paracetamol in a daily dose of 1 gram to achieve headache relief. Two weeks before the appearance of symptoms, the patient was treated with flucloxacillin for respiratory tract infection. Except for the appearance of jaundice and malaise, the patient denies the presence of any other symptoms, and the remainder of his personal history is unremarkable. Physical examination reveals a mildly jaundiced patient, 180 centimeters tall, 82 kilograms in weight. His blood pressure is 110/86 mmHg, and the remainder of his general physical examination revealed no abnormalities. Laboratory analyses reveal the presence of hyperbilirubinemia and elevated serum transaminases, normal alkaline phosphatase and γ-Glutamyl transferase levels. Other routine laboratory analyses reveal no abnormalities. Serological testing does not reveal the presence of antibodies against human immuno-deficiency, hepatitis A, C, D and E, or viruses in patient's serum. Also, HBsAg, anti-HBs or anti-HBc antibodies are not present in patient's serum. Anti-LKM-1, antinuclear, anti-thyroid, antimitochondrial, and anti-smooth muscle antibodies are absent. Question What is the most likely cause of liver dysfunction in this patient? Answer Choices 1 Paracetamol-induced hepatitis 2 Primary sclerosing cholangitis 3 Ketoconazole-caused hepatitis 4 Flucloxacillin-induced hepatitis 5 Primary autoimmune hepatitis

ketoconazole causing hepatitis Ketoconazole is an antifungal imidazole used in treating systemic mycoses. The common adverse effect of ketoconazole is hepatotoxicity. The severity of ketoconazole-induced hepatotoxicity is linked to the exposure level of the drug. Ketoconazole-induced hepatotoxicity is probably mediated through a reactive metabolite N- deacetyl ketoconazole (DAK). The latter appears to be the major metabolite, which is a hepatic cytotoxic. Ketoconazole administration results in a significantly dose-dependent increase in serum transaminase activities, as well as cloudy swelling, ballooning degeneration and centrilobular confluent necrosis of the hepatocytes. The histological feature ranges from acute hepatitis to confluent centrilobular, or massive necrosis. Paracetamol hepatotoxicity is dose-related (i.e., signs of liver dysfunction appear only when a high dose of paracetamol, which exceeds liver metabolizing capacity for this drug, is taken). In such a case, toxic intermediary metabolites do accumulate, causing liver damage. It is considered that a daily paracetamol dose of 10 - 15 mg/kg may be considered as a safe, therapeutic dose. Therefore, liver dysfunction, in the presented case, is not the result of paracetamol-induced hepatitis. Primary sclerosing cholangitis is frequently accompanied with elevation of serum alkaline phosphatase, and gamma-glutamyl transferase levels. Since increased serum levels of those enzymes are not noted in the presented case, it is unlikely that the patient suffers from primary sclerosing cholangitis. Flucloxacillin administration causes cholestatic hepatitis (i.e., liver dysfunction in patients with flucloxacillin-induced hepatitis is associated with elevated alkaline phosphatase and gamma-glutamyl transferase levels, as well as with disproportionate rise of conjugated serum bilirubin level). Since hepatitis in the presented patient is not of cholestatic type, it is unlikely that he suffers from flucloxacillin-induced hepatitis. Autoimmune hepatitis is characterized with the appearance of auto-antibodies, including liver kidney microsomal (LKM) antibodies. LKM antibodies in patients with genuine (primary) autoimmune hepatitis are of LKM1 type, not of LKM2 type.

A 53-year-old man presents with a 2-day history of jaundice and malaise. His history is significant for mucosal candidiasis, for which the patient is on oral ketoconazole 200 mg daily for the past 3 weeks. He occasionally has headaches, and sometimes takes paracetamol in a daily dose of 1 gram to achieve headache relief. Two weeks before the appearance of symptoms, the patient was treated with flucloxacillin for respiratory tract infection. Except for the appearance of jaundice and malaise, the patient denies the presence of any other symptoms, and the remainder of his personal history is unremarkable. Physical examination reveals a mildly jaundiced patient, 180 centimeters tall, 82 kilograms in weight. His blood pressure is 110/86 mmHg, and the remainder of his general physical examination revealed no abnormalities. Laboratory analyses reveal the presence of hyperbilirubinemia and elevated serum transaminases, normal alkaline phosphatase and γ-Glutamyl transferase levels. Other routine laboratory analyses reveal no abnormalities. Serological testing does not reveal the presence of antibodies against human immuno-deficiency, hepatitis A, C, D and E, or viruses in patient's serum. Also, HBsAg, anti-HBs or anti-HBc antibodies are not present in patient's serum. Anti-LKM-1, antinuclear, anti-thyroid, antimitochondrial, and anti-smooth muscle antibodies are absent. Question What is the most likely cause of liver dysfunction in this patient? Answer Choices 1 Paracetamol-induced hepatitis 2 Primary sclerosing cholangitis 3 Ketoconazole-caused hepatitis 4 Flucloxacillin-induced hepatitis 5 Primary autoimmune hepatitis

ketoconazole induced hepatitis Ketoconazole is an antifungal imidazole used in treating systemic mycoses. The common adverse effect of ketoconazole is hepatotoxicity. The severity of ketoconazole-induced hepatotoxicity is linked to the exposure level of the drug. Ketoconazole-induced hepatotoxicity is probably mediated through a reactive metabolite N- deacetyl ketoconazole (DAK). The latter appears to be the major metabolite, which is a hepatic cytotoxic. Ketoconazole administration results in a significantly dose-dependent increase in serum transaminase activities, as well as cloudy swelling, ballooning degeneration and centrilobular confluent necrosis of the hepatocytes. The histological feature ranges from acute hepatitis to confluent centrilobular, or massive necrosis. Paracetamol hepatotoxicity is dose-related (i.e., signs of liver dysfunction appear only when a high dose of paracetamol, which exceeds liver metabolizing capacity for this drug, is taken). In such a case, toxic intermediary metabolites do accumulate, causing liver damage. It is considered that a daily paracetamol dose of 10 - 15 mg/kg may be considered as a safe, therapeutic dose. Therefore, liver dysfunction, in the presented case, is not the result of paracetamol-induced hepatitis. Primary sclerosing cholangitis is frequently accompanied with elevation of serum alkaline phosphatase, and gamma-glutamyl transferase levels. Since increased serum levels of those enzymes are not noted in the presented case, it is unlikely that the patient suffers from primary sclerosing cholangitis. Flucloxacillin administration causes cholestatic hepatitis (i.e., liver dysfunction in patients with flucloxacillin-induced hepatitis is associated with elevated alkaline phosphatase and gamma-glutamyl transferase levels, as well as with disproportionate rise of conjugated serum bilirubin level). Since hepatitis in the presented patient is not of cholestatic type, it is unlikely that he suffers from flucloxacillin-induced hepatitis. Autoimmune hepatitis is characterized with the appearance of auto-antibodies, including liver kidney microsomal (LKM) antibodies. LKM antibodies in patients with genuine (primary) autoimmune hepatitis are of LKM1 type, not of LKM2 type.

A 54-year-old man presents to his family physician with complaints that he has very large urine output and that he is constantly thirsty. On questioning, he states that his urine output is many liters per day and that he awakens at night to urinate. His history is also remarkable for a 25-year history of a bipolar disorder, treated effectively with lithium. His lab results are as follows: TEST RESULTS REFERENCE RANGE BUN 17 mg/dL 10-20 mg/dL Calcium 9.9 mg/dL 8.5-10.5 mg/dL Potassium 4.2 mEq/L 3.5-5.0 mEq/L Sodium 149 mEq/L 135-145 mEq/L Glucose (fasting) 109 mg/dL 65-110 mg/dL The results of his urinalysis are as follows: TEST RESULTS REFERENCE RANGE Urine dipstick blood negative negative Urine dipstick glucose negative negative Urine dipstick ketones negative negative Urine dipstick protein negative negative 24 hour urine protein 124 mg/24 hour <150 mg/24 hour Urine osmolality 40mOsm/kg 50-1400mOsm/kg Urine specific gravity 1.001 1.001-1.035 Urine pH 6.2 4.5-8.5 Question A water deprivation test with exogenous vasopressin administration is done under meticulous supervision. His urine osmolality does not increase. What organ is most involved in the pathophysiology of this condition? Answer Choices 1 Kidney 2 Pituitary 3 Bladder 4 Ureter 5 Urethra

kidney A known side effect of lithium is nephrogenic diabetes insipidus. Nephrogenic diabetes insipidus can be seen in a significant minority of the patients taking lithium. Nephrogenic diabetes insipidus results when the collecting duct of the kidney does not respond to antidiuretic hormone (ADH). In complete nephrogenic diabetes insipidus, the kidney is unresponsive to ADH. Therefore, exogenous vasopressin (ADH) will not increase the urine osmolality. The pituitary gland secretes numerous hormones. The posterior lobe of the pituitary is the neurohypophysis. One of the hormones secreted by the posterior pituitary is antidiuretic hormone (ADH). The production of the posterior pituitary hormones actually occurs in the hypothalamus. Antiduiretic hormone (ADH) is also known as arginine vasopressin. It works on the collecting tubules of the kidney to conserve water. When the pituitary does not secrete ADH, the condition is called central diabetes insipidus. The urinary bladder is a muscular sac. The bladder stores urine and it is not responsible for concentration or production of urine. Pathology of the bladder does not result in nephrogenic diabetes insipidus. The ureter is a tube that transmits urine from the kidney to the bladder. The ureter is not responsible for concentration or production of urine. Pathology of the ureter does not result in nephrogenic diabetes insipidus. The urethra is a tube that transmits urine from the bladder out of the body. The urethra is not responsible for concentration or production of urine. Pathology of the urethra does not result in nephrogenic diabetes insipidus.

A 14-year-old boy presents to the emergency department with his parents. He has a history of type 1 diabetes, and has had bronchitis for the last few days. He is now presenting with difficulty breathing, worsening fatigue, polydipsia, and polyuria. His last fingerstick glucose at home this morning was 350 mg/dL. Which of the following patterns of breathing are characteristic of this complication of diabetes? A Cheyne stokes respiration B Bradypnea C Biot breathing D Kussmaul breathing E Painful respiration

kussmaul breathing The correct choice is D, Kussmaul breathing, which is deep regular breathing or hyperpnea. It can be seen as a compensatory action of metabolic acidosis and hypoxia. Choice A, Cheyne-Stokes respiration, is a waxing and waning pattern of rate and volume that includes periods of apnea. This can be seen in patients at high altitudes, and with severe left sided heart failure or neurologic disease. Choice B, bradypnea, is noted with a slower than usual respiratory rate and can be seen with use of CNS depressant drugs, uremia, or structural intracranial lesion. Choice C, Biot breathing, is an uncommon variant of Cheyne-Stokes respiration, with periods of apnea alternating with a series of equal breaths that end abruptly. It can be seen in patients with meningitis. Choice E, painful respiration, is relatively normal in pattern, but interrupted by pain during breathing from such disorders as pleurisy, fractured ribs, or subphrenic inflammation.

A 44-year-old man with a history of ulcerative colitis presents with bloody diarrhea. The patient has had a long history of poorly controlled ulcerative colitis despite being on optimal medical management. The patient subsequently is surgically managed with a proctocolectomy with permanent ileostomy. However, complications in his procedure call for a long hospital stay. After a few months, the patient is found to have ascites, pitting edema, and a fatty liver. Question What is the most likely diagnosis? Answer Choices 1 Hypovitaminosis B1 2 Kwashiorkor 3 Total parenteral nutrition-induced steatosis 4 Marasmus 5 Primary sclerosing cholangitis

kwashiorkor This patient is having protein deficiency, and therefore the most likely answer is kwashiorkor. Kwashiorkor is secondary to total protein deficiency. This finding is typically found in patients who are severely malnourished. The typical findings in these patients are ascites and pitting edema due to hypoalbuminemia. In this patient, his long hospital stay with a complicated proctocolectomy, has led to protein deficiency. Marasmus is a total caloric and protein deficiency. These patients typically present with broomstick extremities (muscle wasting). These patients do not present with ascites and pitting edema. Total parenteral nutrition (TPN) related microvesicular steatosis is a common finding in patients who are on TPN. However, there is no mention in the question stem that this patient was on TPN. Primary sclerosing cholangitis (PSC) is commonly associated with patients who have ulcerative colitis. PSC is a chronic liver disease, which presents as inflammation, destruction, and fibrosis of both the intrahepatic and extrahepatic bile ducts, with the resultant destruction leading to cirrhosis of liver. Hypovitaminosis B1 (thiamine) is not the correct answer choice. This deficiency is known as wet beri beri. Though it may cause edema, there will be dilated cardiomyopathy, which is not found in this case.

A 22-year-old woman presents with a history of chronic diarrhea. She gives a history of large volumes of watery fecal output that is non-bloody and can be easily flushed; the diarrhea is painless and persists with fasting. The patient's mother complains that she eats less and has lost weight. She has no history of fever, flushing, wheezing, intolerance to heat/cold, or infection. Her family history and menstrual history are insignificant. On examination, her weight is 130lb; height is 5 feet 5 inches; pulse is 80/min; BP is 100/74mmHg; and temperature is 98.8°F. The tongue is dry, but the rest of her physical examination is within normal limits. Question What is the most likely diagnosis? Answer Choices 1 Carcinoid tumor 2 Laxative abuse 3 Celiac disease 4 Hyperthyroidism 5 Inflammatory diarrhea

laxative abuse The most likely diagnosis is laxative abuse. Large volumes of watery fecal output that is non-bloody, painless, and persists with fasting suggests secretory diarrhea that can result from use of stimulant laxatives like senna, bisacodyl, or castor oil1. The patient is young, dehydrated, and based on her symptoms, is likely taking laxatives to lose weight and get rid of unwanted calories. There are no signs and symptoms of other causes of secretory diarrhea, such as carcinoid syndrome, and the rest of the options do not have conditions that cause secretory diarrhea. Many patients might not admit the use of laxatives to lose weight. Carcinoid tumor can cause secretory diarrhea1, but it is unlikely in this patient; she has no associated symptoms, such as episodic wheezing or flushing. Hyperthyroidism results in diarrhea due to dysmotility1. However, the patient does not have any symptoms of hyperthyroidism, such as unexplained weight loss or heat intolerance. Inflammatory diarrhea can be ruled out in this patient; there is no fever or signs of inflammation or bloody stools. Celiac disease causes diarrhea due to mucosal malabsorption. Patients present with multiple nutritional deficiencies and fatty diarrhea, which are not present in this patient, making this diagnosis unlikely1.

A 28-year-old man presents for a barium enema; he has a 6-month history of abdominal cramping, pain, tenesmus, bloody diarrhea, and painful urgency. He also reports arthralgias, but denies a history of traveling abroad or the use of antibiotics. On physical exam, he has indurated and painful erythematous plaques and nodules on both shins. An abdominal exam reveals a mild tenderness in the lower left quadrant (LLQ). Stool exam is positive for occult blood and negative for ova and parasites. Lab analysis reveals mild leukocytosis, microcytic anemia, positive p-ANCA antibodies, and elevated ESR. What would be the most likely result of a barium enema? Answer Choices 1 Apple core appearance 2 Cobblestone appearance 3 Lead pipe appearance 4 String sign 5 Thumb printing

lead pipe appearance on barium enema Lead pipe colon is the appearance on barium enema of a foreshortened, narrow colon with loss of redundancy and haustral markings. This sign is specific for ulcerative colitis (UC), an inflammatory disease of the rectum and colon. The peak incidence of UC is around 25 to 45 years. P-ANCA antibodies are associated with UC. Iron deficiency can lead to microcytic anemia. Common extra intestinal manifestations of UC are erythema nodosum (described as erythematous plaques and nodules on pretibial areas), arthritis, pyoderma gangrenosum, uveitis, episcleritis, and sclerosing cholangitis. Toxic megacolon is a complication of UC characterized by the dilation of the colon; there is a risk of perforation. There is an increased incidence of colon cancer (10%) after 10 years of this disease. The inflammation involves the mucosal tissues only, which are friable and show ulcerations on colonoscopy. There is also a characteristic continuous involvement and pseudopolyp appearance caused by mucosal regeneration. Sulfapyridine, mesalamine, corticosteroids, 6-mercaptopurine, and azathioprine are used for treatment. Colectomy is curative and is indicated for intractable disease, or in the case of dysplasia. An apple core appearance on barium enema is seen in patients with colon cancer. This sign refers to the shouldered margins of the stricture caused by the neoplasmic invasion. A cobblestone appearance and string sign are characteristic for Crohn's disease, another inflammatory disease of the digestive tract. The cobblestone appearance refers to the discontinuous areas of inflammation separated by healthy bowel. String sign, seen in the small bowel series, is the appearance caused by stricture areas. Thumb printing is a radiological sign seen in ischemic colitis, a disease most likely found in elderly patients with left lower abdominal pain and gross rectal bleeding.

A 24-year-old man presents with head trauma received in a motor vehicle accident (MVA) 30 minutes ago. Paramedics relate loss of consciousness for 1 minute, with a complete recovery. At the initial evaluation, the physical exam reveals a patient with a Glasgow Coma Scale (GCS) of 12, right hemiparesis, and a left fixed dilated pupil. What is the most probable finding in the brain CT scan? Answer Choices 1 A left epidural hematoma 2 A right epidural hematoma 3 A left subdural hematoma 4 A right subdural hematoma 5 A subarachnoid hemorrhage

left epidural hematoma The patient presents with a history of head trauma, a brief initial period of unconsciousness, a lucid interval lasting minutes, subsequent deterioration in neurological status revealed in a Glasgow coma scale of 12, right hemiparesis, and left fixed dilated pupil. This is a classical presentation of left epidural hematoma, which is seen as a biconvex hyper-density (whiteness) on a CT scan. A right epidural hematoma would produce a left hemiparesis and a right fixed dilated pupil.

A 43-year-old Caucasian woman presents for evaluation of menstrual irregularities over the past 6 months, despite of history of prior regular menses. She had a bilateral tubal ligation (BTL) and reports multiple negative home pregnancy tests. The patient also notes weight gain and increased girth in her abdomen. She reports easy bruising without a history of trauma; she also notes the new development of "stretch marks" on her torso, and there is also weakness in her arms and legs. She denies any health changes, medications, or stressors in relation to these changes. She denies hot flashes, night sweats, and frank depression; however, she admits to some mood swings and poor libido. The patient is frustrated that her health fair labs (which include complete blood count, complete metabolic panel, lipid panel, and thyroid stimulating hormone) were all normal and do not explain her symptoms. Her past medical history reveals no chronic conditions; she does not take any medications, and she has no known drug allergies. Her only surgery was the BTL. She lives with her husband and 3 children. She works as a retail clerk and walks for exercise. She denies the use of tobacco, alcohol, and drugs. Her blood pressure is 154/92. Chart review demonstrates weight gain of 15 pounds over 6 months. She has had normal blood pressures in the past. On physical exam, you observe an overweight woman with an especially rounded, full face. She also has a fatty fullness to her neck region, and there is some central obesity; however, her arms show some muscle wasting. She has purple striae on her torso. Her skin also appears thinned, with multiple bruises. Hirsutism is observed on the patient's chin, abdomen, and breasts. The remainder of her exam was unremarkable. Testing confirms elevated ACTH and high cortisol levels in the evening and after dexamethasone suppression, as well as a pituitary macroadenoma as the cause. Question What prescription medication may be indicated after the surgical removal of her tumor? Answer Choices 1 Aldosterone 2 Insulin 3 Levothyroxine 4 Parathyroid hormone 5 Prolactin

levothyroxine Explanation This patient presents with a multitude of signs and symptoms (weight gain, menstrual irregularity, mood swings, poor libido, moon facies, fat depositions, proximal limb weakness, central obesity, skin thinning, purple striae, and easy bruising) that support a diagnosis of Cushing's syndrome or Cushing's disease. The most common cause of Cushing syndrome is considered iatrogenic, from administering exogenous steroids, such as prednisone. The next most common cause is known as Cushing's disease, and it results from increased adrenocorticotropic hormone (ACTH) secretion from pituitary, which causes increased adrenal production of cortisol. This patient's tests indicate Cushing's disease, with a pituitary macroadenoma. Surgery is indicated. If damage occurs to the anterior pituitary, it can affect hormone secretion. Often, replacement of levothyroxine, estrogen, progesterone, and possibly desmopressin may be needed. Aldosterone, which regulates sodium balance, is a hormone produced by the adrenal glands. Rarely, Cushing's disease can be caused by an adrenal neoplasm, but those cases are considered non-ACTH dependent. This patient would not be expected to have any adrenal issues following pituitary surgery. Insulin, which regulates glucose metabolism, is produced in the pancreas. This patient's Cushing's disease and subsequent pituitary surgery should not lead to the requirement of insulin prescription. Parathyroid hormone, which regulates calcium balance, is produced by the parathyroid glands. These glands can be damaged during thyroid surgeries and require subsequent management of calcium. However, this patient's parathyroid hormone should not be altered by her pituitary surgery. Prolactin, which primarily regulates milk production, is produced by the anterior pituitary gland. This patient would potentially experience problems with prolactin production following pituitary surgery if she were lactating. However, she has had surgical sterilization; therefore, she has no need for prolactin production.

A 31-year-old man with a known generalized seizure disorder is brought to your emergency department. His friends tell you that the patient had a seizure and did not wake up. When he did not wake up after 30 minutes, his friends called 911. On examination, he is breathing and his heart is beating. He is warm, dry, and pink. His basic laboratory values are within normal limits, and the computerized axial tomography (C.A.T.) scan of his brain is unremarkable. An emergency electroencephalogram (E.E.G.) is not available. His only medications are phenytoin and phenobarbital. What is the best initial treatment in this case? Answer Choices 1 Glucose 2 Lorazepam (Ativan) 3 Phenytoin (Dilantin) 4 Phenobarbital 5 Craniotomy with burr hole placement by a neurosurgeon

lorazepam (Ativan) According to consulting neurologist Wade S. Smith, Audio-Digest Family Practice, Vol. 49, 2001, "Consciousness is usually regained in a few minutes following a generalized seizure. When consciousness is not promptly regained following a seizure, then the diagnosis of status epilepticus becomes a reasonable consideration." When the emergency electroencephalogram (E.E.G.) is not readily available on a 24-hour basis, Professor Smith recommends that empiric pharmacotherapy for a working diagnosis of status epilepticus be provided. The first pharmacotherapy recommended for status epilepticus is lorazepam (Ativan). The second line pharmacotherapy for status epilepticus is phenytoin (Dilantin). The third line pharmacotherapy for status epilepticus is phenobarbital. Note that the absence of tonic-clonic movements does not exclude the diagnosis of status epilepticus, as it is possible that it will present with the solitary manifestation of persistent loss of consciousness. With his normal laboratory values, glucose would not be of value for this patient. Additionally, with the normal computerized axial tomography (C.A.T.) scan, the neurosurgical placement of burr holes by craniotomy would be nonfunctional

A 50-year-old Caucasian man is in persistent cardiac arrest despite 2 electrical shocks and adrenaline medication. He had collapsed on a local golf course while playing with his wife. When the paramedics arrived, he had a pulse and blood pressure, but he arrested on the way to the hospital. The only medication he takes on a regular basis is an ACE inhibitor for high blood pressure. At the base of his neck, you feel a hump; you note purple stretch scars on his abdomen; in addition to his obesity, he has a moon face. You draw some blood for electrolytes. What do you expect to find? Answer Choices 1 Low potassium 2 High potassium 3 High sodium 4 Low sodium 5 Low magnesium

low potassium Trunk obesity, moon face, purple stretch marks on the abdomen, and buffalo hump are typical symptoms of Cushing's syndrome due to chronic exposure to corticosteroids. Other symptoms are muscle wasting and weakness, thin and atrophic skin, poor wound healing, easy bruising, hypertension, osteoporosis, renal stones, glucose intolerance, psychiatric problems, and virilism in females. Due to increased secretion or administration of ACTH, too much aldosterone is produced. The latter results in increased secretion of K+ and H+ ions through the kidney, and ends in hypokalemia (low potassium) and metabolic alkalosis. Cardiac symptoms are usually minimal until serum potassium levels are <3 mEq/l. ECG changes include ST-depression; decreased and sometimes negative T; U-wave, and extrasystole (remember: "no pot [assium], no t [ea], but 'U' "). In severe hypokalemia, you can find premature ventricular and atrial contractions, as well as arrhythmia, which can lead to cardiac arrest. Hyperkalemia presents in the ECG as shortening of the QT-interval and tall, peaked T-waves and then proceeds to nodal and ventricular arrhythmia, widening of the QRS complex, PR prolongation, disappearance of P-wave, and finally degeneration of the QRS complex to sine wave and asystole. Clinical symptoms are arrhythmia, paresthesias, hyporeflexia, muscle weakness, obstipation, and acidosis. Hypernatremia is usually generated when water losses exceed sodium losses in combination with inadequate water intake. The symptoms are CNS dysfunction-like confusions, seizures, coma, hemorrhages, and thrombosis. Hyponatremia is usually caused by renal retention of water, while sodium is excreted. Typical symptoms are confusion, lethargy, and altered mental status. Cerebral edema, brain herniation, coma, and death can occur. Hypomagnesemia is often caused by inadequate intake. The symptoms include tetanus, tremor, seizures, nausea, vomiting, lethargy, weakness and mental alteration.

A 68-year-old man presents to your pulmonology practice for long-standing dyspnea and non-productive cough. The patient has had the cough and progressively worsening dyspnea for about 1.5 years. He denies other symptoms, and he is not a smoker. He denies fevers, chills, and night sweats. He denies unusual travel, hobbies, and occupational exposures. The rest of his review of systems is negative. His past medical history is unremarkable. He had a workup through his family practice, and he was then sent to a cardiologist, who ruled out cardiovascular causes of his dyspnea. He was given trials of various antibiotics, inhalers, and steroids; there was no improvement in symptoms, despite good compliance. Several screening blood tests (including blood count, metabolic panel, HIV, and autoimmune markers) are done, revealing no abnormalities. A chest X-ray shows a few reticular opacities. A high-resolution computed tomography (HSCT) scan shows some scattered areas of reticular opacities and honeycombing. Your office runs a pulmonary function test (PFT), which shows some restrictive impairment and reduced perfusion of carbon monoxide. Physical exam is significant for fine inspiratory bibasilar crackles and clubbing in the fingers. An occasional dry cough is noted. Question What intervention is recommended for this patient's condition? Answer Choices 1 Avoidance of exertion 2 Daily inhaled steroids 3 Long-term oral steroid use 4 Lung transplant 5 Ventilation-perfusion scan

lung transplant This patient is presenting with idiopathic pulmonary fibrosis (IPF), and the only intervention currently associated with improvements in mortality is lung transplant. IPF is a chronic, progressive restrictive pulmonary disease of the lung parenchyma. IPF presents with exertional dyspnea and non-productive cough; it also presents with the imaging characteristics described in this patient. Avoidance of exertion may prevent the exertional dyspnea in this patient, but it is not recommended. Respiratory therapy to improve exercise tolerance is recommended in cases of IPF. Daily inhaled steroids and long-term oral steroid use are not proven as effective in IPF. The pathophysiologic basis for IPF is much more a disorder of fibroblastic disease, in which there is an abnormal accumulation of extracellular matrix, and not inflammatory in nature. Anti-inflammatory medications do not alter the course of IPF. A ventilation-perfusion scan is a test that is primarily used in the diagnosis of pulmonary embolism (PE). This patient is not presenting with a history consistent with a PE (which may be acute dyspnea and chest pain). The imaging that has already been done (high-resolution CT) is sufficient for diagnosis of IPF. No further studies are recommended. If the provider needs further confirmation, a lung biopsy should be the confirmatory test of choice.

A 56-year-old woman is being seen for regular assessment and monitoring of her type 2 diabetes mellitus. She has been following a strict diet and exercise plan for 2 years with the addition of metformin 6 months ago for an increased HgA 1c level. Her HgA 1c at today's visit is 7.1. What is the appropriate management for this patient? A add exenatide to her current therapy B change her oral therapy to rosiglitazone C add insulin to her current therapy D maintain her current therapy and recheck in 6 months

maintain her current therapy and recheck in 6 months D The HgA 1c goal for this patient is less than 6.5, with action at a level of greater than 8.0. The appropriate action at this time is to continue her current therapy and reassess in 6 months.

Which of the following is the most common cause of short stature worldwide? A Acromegaly B Malnutrition C Prader-Willi syndrome D Congenital growth hormone deficiency E IGF-I receptor deficiency

malnutrition The correct choice is B, malnutrition. When not associated with chronic diseases, this is the most common cause of short stature worldwide. Children with malnutrition commonly present with failure of weight gain before growth rate decreases. A dietary history is key to the diagnosis, as well as a history of any parasites in the local area. Choice A, acromegaly, is a disorder of growth hormone excess. Choice C, Prader-Willi syndrome, choice D, congenital growth hormone deficiency, and choice E, IGF-I receptor deficiency, have been found to cause short stature, but are not seen as commonly as malnutrition.

which of the following is the most common cause of short stature worldwide? A Acromegaly B Malnutrition C Prader-Willi syndrome D Congenital growth hormone deficiency E IGF-I receptor deficiency

malnutrition The correct choice is B, malnutrition. When not associated with chronic diseases, this is the most common cause of short stature worldwide. Children with malnutrition commonly present with failure of weight gain before growth rate decreases. A dietary history is key to the diagnosis, as well as a history of any parasites in the local area. Choice A, acromegaly, is a disorder of growth hormone excess. Choice C, Prader-Willi syndrome, choice D, congenital growth hormone deficiency, and choice E, IGF-I receptor deficiency, have been found to cause short stature, but are not seen as commonly as malnutrition.

A 34-year-old woman, an immigrant from South America, presents with an acute onset of chest palpitations and shortness of breath. The patient also gives a 1-week history of fever, fatigue, and weakness. Echocardiography reveals that her heart is dramatically increased in size. Electrocardiogram (EKG) shows low voltage QRS with atrioventricular (AV) conduction abnormality. It is determined that she has Chagas' disease. Question What is a complication that occurs during the acute phase of this patient's condition? Answer Choices 1 Megaesophagus 2 Thromboembolism 3 Gastric dilatation 4 Meningoencephalitis 5 Megaureter

meningoencephalitis Chagas' disease is also referred to as American trypanosomiasis. Chagas' disease is seen in the Western Hemisphere, primarily in South America, Central America, and Mexico. It is caused by the protozoa Trypanosoma cruzi. It can be acute or chronic. The manifestations in the acute phase include prolonged fever, tachycardia, fatigue, anemia, weakness, mild hepatosplenomegaly, and lymphadenopathy. Meningoencephalitis is the complication occurring during the acute phase. The acute stage resolves spontaneously by 4-8 weeks in 90% of the individuals who are infected. The persistence leads to development of intermittent or chronic phase. Chronic infection can lead to cardiomyopathy. The cardiac changes seen with chronic Chagas' disease include an enlarged heart and EKG findings of right bundle branch block and premature ventricular contractions. Chronic infection can also lead to megaesophagus, megacolon, and thromboembolism. Gastric dilatation and megaureter may occur, but they are rare.

A 78-year-old male returns to the FP office for a follow up of non-insulin-dependent diabetes mellitus (NIDDM) as a new patient to you, although he has been an office patient for the past year. He denies any problems this visit and says his blood sugars are in the 90-120 mg/dl range. He is currently taking the medications listed in the following choices. You receive his labs and note that his creatinine is 2.0 mg/dl and on the previous few labs this creatinine was also in the 1.8-2.0 mg/dl range. What medication should be discontinued? A glipizide B metformin C omeprazole D sitagliptin E atenolol

metformin The correct answer is (B). Metformin is contraindicated in this diabetic patient with chronic renal failure due to an increased risk of lactic acidosis and should be discontinued. Sitagliptin requires a dosing adjustment in renal failure but is not contraindicated in this patient. Glipizide (sulfonylurea), omeprazole (a PPI for GERD), and atenolol (a beta blocker for hypertension) are not contraindicated in this patient. Beta blockers should be used with caution in diabetics due to the potential of masking symptoms of hypoglycemia, but are not contraindicated.

The most common cause of nephrotic syndrome in children is A post-streptococcal glomerulonephritis B minimal change disease C diabetes mellitus D NSAIDs E polycystic kidney disease

minimal change disease B The most common cause of nephrotic syndrome in children is minimal change disease. Diffuse injury to the capillaries is the underlying cause, resulting in significant proteinuria, edema, hypoalbuminemia, and hyperlipidemia. It accounts for 65% of cases of nephrotic syndrome in children; however, 10% of adults with nephrotic syndrome have minimal change disease. Treatment is with corticosteroids for 2 to 4 weeks, dietary sodium restriction, and sometimes diuretics to reduce the edema. Relapse and lack of response to corticosteroids can occur. If the latter occurs, renal biopsy is indicated to rule out other causes of the nephrotic syndrome, such as focal glomerulosclerosis and membranoproliferative glomerulonephritis.

A sexually active 19-year-old woman presents with clusters of painful vesicles on an erythematous base on the vulva and cervix, accompanied by temperature of 100°F and mild malaise. She reports a history of a similar outbreak last month, which resolved in 10 days. Microscopic examination of cells from the basement of a blister treated with Giemsa stain is likely to reveal A multinucleated giant cells B gram-positive cocci in clusters C gram-positive cocci in chains D gram-negative rods E hyphae and buds

multinucleate giant cells A The clinical presentation is consistent with herpes simplex. The appropriate microscopic study is a Tzanck smear, prepared by staining cells from the floor of a vesicle using Papanicolau, Giemsa, or Wright methods. The Tzanck smear will show multinucleated giant cells. It has a sensitivity of 60% to 70% and as a result should be confirmed by viral culture. Gram-positive cocci are consistent with staphylococcal or streptococcal infection and gram-negative rods are usually enteric pathogens. Hyphae and buds are seen on KOH prep with candidal infection.

Your patient is a 77-year-old male with a history of hypertension. For about the past 50 years has smoked a pipe daily. He feels great but admits that his cholesterol was elevated for the past 5 years, and has declined treatment. His best friend died of an myocardial infarction last week and the patient now agrees to treatment for his hyperlipidemia. His LDL is 285 mg/dl and HDL is 30 mg/dl. You decide to put him on simvastatin 80 mg QD. Prior to initiation, you advise the patient to notify you about which of the following potential side effects of this medication? A cough B double vision C myalgias D elevated blood pressure E restlessness

myalgias The correct answer is (C). Myalgias are common side effects of statins, which may result in a patient discontinuing the medication. If the patient develops significant myalgias a CPK may be ordered to rule out myositis and if elevated the statin may need to be discontinued. Elevated liver enzymes may occur, which may result in discontinuation of the medication. The other options are unlikely side effects of statins.

A 32-year-old woman starts to notice that she is having difficulty brushing her hair. She also notices that her eyes are extremely tired after reading the evening newspaper. One night, her husband comments that she has droopy eyelids. She is relieved the next morning when her eyelids appear normal. Her condition deteriorates, and she has weakness and fatigability to the point that she finally sees her family doctor. On history, she denies pain. Her physical exam demonstrates weakness in her limbs. Her deep tendon reflexes are within normal limits. Her doctor wants to run a diagnostic test by injecting a drug. What is the most likely diagnosis? Answer Choices 1 Myasthenia gravis 2 Duchenne's dystrophy 3 Polymyositis 4 Familial periodic paralysis 5 Botulism

myasthenia gravis This patient's symptoms indicate myasthenia gravis. Ocular muscle weakness and ptosis are common. Muscular weakness and fatigue are consistent with this condition. Myasthenia gravis occurs due to the presence of antibodies to the acetylcholine receptor. A diagnostic aid is the Tensilon test. The Tensilon test entails the administration of a rapid acting anticholinesterase, edrophonium. By administering an anticholinesterase, any acetylcholine released will be present longer and will have a longer period of time to act at the neuromuscular junction. Duchenne's dystrophy would present at a very young age. It is X-linked. Polymyositis can present with weakness. However, the weakness is proximal; it is usually of the hips and thigh, but sometimes it is of the shoulder girdles. Polymyositis would spare the ocular muscles. Familial periodic paralysis can present with weakness. However, the weakness would be in the extremities and the trunk. It would also spare the ocular muscles. Botulism can present with weakness. In addition, the ocular muscles can be involved. However, botulism would present with a progressive weakness, rather than an intermittent weakness. There would also be a loss of deep tendon reflexes. Furthermore, there is nothing in her history to suggest ingestion of botulism toxin.

A person with an exaggerated sense of entitlement and uniqueness and who believes they can only be understood by people of significance is described to you by a colleague. They go on to state the person is arrogant, is lacking in empathy, and can be manipulative with relationships. What personality disorder best fits this scenario? A histrionic B narcissistic C antisocial

narcissistic B The scenario represents a typical "snapshot" of this diagnosis. These persons typically have fantasies of unlimited success and have a strong need for admiration from others. They can be jealous of others but commonly assume that others are extremely jealous of them. Treatment is made difficult as they do not accept criticism or any attack on their "narcissistic supply."

A 54-year-old man notices that he has very large urine output and he is constantly thirsty. In addition to urinating large volumes during the day, he awakens at night to urinate. His history is remarkable for a 25-year history of a bipolar disorder, which is treated effectively with lithium. His lab results are as follows: TEST RESULTS REFERENCE RANGE BUN 17 mg/dL 10 - 20 mg/dL Calcium 9.9 mg/dL 8.5 - 10.5 mg/dL Potassium 4.2 mEq/L 3.5 - 5.0 mEq/L Sodium 149 mEq/L 135 - 145 mEq/L Glucose (fasting) 109 mg/dL 65 - 110 mg/dL The results of his urinalysis are as follows: TEST RESULTS REFERENCE RANGE Urine dipstick blood Negative Negative Urine dipstick glucose Negative Negative Urine dipstick ketones Negative Negative Urine dipstick protein Negative Negative 24 hour urine protein 124 mg/24 hour <150 mg/24 hour Urine osmolality 40mOsm/kg 50-1400mOsm/kg Urine specific gravity 1.001 1.001 - 1.035 Urine pH 6.2 4.5 - 8.5 Question A water deprivation test with exogenous vasopressin administration is done under meticulous supervision. His urine osmolality does not increase. What is most likely the case? Answer Choices 1 Syndrome of inappropriate ADH secretion 2 Central diabetes insipidus 3 Diabetes mellitus 4 Nephrogenic diabetes insipidus 5 Psychogenic polydipsia

nephrogenic diabetic insipidus Explanation A known side effect of lithium is nephrogenic diabetes insipidus. This side effect is very common. Nephrogenic diabetes insipidus results when the collecting duct of the kidney does not respond to the antidiuretic hormone (ADH). Antiduiretic hormone (ADH) is also known as arginine vasopressin. It is secreted from the pituitary. It works on the collecting tubules of the kidney to conserve water. Nephrogenic diabetes insipidus is characterized by renal resistance to ADH. This results in a large output of dilute urine. There will be polyuria and polydipsia. Hypernatremia would be present. In contrast to central diabetes insipidus, plasma ADH level would be elevated and there is no response to the administration of vasopressin. Nocturia can be present. In complete nephrogenic diabetes insipidus, the kidney is unresponsive to ADH. Therefore, exogenous vasopressin (ADH) will not increase the urine osmolality. The syndrome of inappropriate ADH secretion (SIADH) is due to excess ADH (antidiuretic hormone) or an ADH like substance. The syndrome of inappropriate ADH secretion is characterized by concentrated urine. There will be an accompanying serum hyponatremia. Hyponatremia can cause central nervous system symptoms (obtundation, seizure, and coma). Central diabetes insipidus is characterized by a lack of antidiuretic hormone (ADH) secretion from the pituitary. Because of the deficiency of ADH, there is an inability of the kidney to concentrate the urine. This results in a large urine output of dilute urine. There will be polydipsia and polyuria. Nocturia can be present. Hypernatremia would be present. In contrast to nephrogenic diabetes insipidus, there is an increase in urine osmolality in response to the administration of vasopressin. Diabetes mellitus is due to an absolute or relative insulin deficiency. Untreated, it is characterized by polyuria, polydipsia, and polyphagia, and there will be an elevated blood glucose and glucosuria. Glucosuria is the presence of glucose in the urine. Psychogenic polydipsia is a psychological condition. A patient with psychogenic polydipsia drinks an enormous amount of water. Because of this large water intake, there is polyuria. However, with psychogenic polydipsia nocturia is usually absent. There will also be hyponatremia.

A 45-year-old woman comes to her physician for her regular visit. She has a history of diabetes mellitus for 10 years and is on 10 units of lantus insulin daily at night. She has no complaints at this time and has no other medical history. Family history is unremarkable. She is a nonsmoker and drinks 2-3 beers on the weekends. On examination, she is afebrile, BP 120/80 mmHg, pulse 76/minute, and has no jaundice, pallor, or cyanosis. Lungs are clear, heart sounds are normal, and abdominal exam reveals no hepatosplenomegaly, ascites, tenderness, or mass. Routine labs show a Hb of 13 g%, WBC 7000/uL, platelets 350,000/uL, AST 90 U/L, ALT 102 U/L, AP 36 U/L, bilirubin 1.1 mg/dL, and albumin 4.0 mg/dL. Fasting blood sugar is 116 mg/dL. Further tests show negative HBsAg and anti-HCV. ANA, ESR, and gamma GT are also normal. Ultrasound of the liver is pending. Question What is the most likely diagnosis? Answer Choices 1 Alcoholic liver disease 2 Primary biliary cirrhosis 3 Nonalcoholic steatohepatitis 4 Autoimmune hepatitis 5 Cholestatic hepatitis

nonalcoholic steatohepatitis Explanation Nonalcoholic steatohepatitis (NASH) is a diagnosis of exclusion in patients who have an asymptomatic increase in liver enzymes where no other etiological factor can be determined. It is a definite clinical entity seen most commonly in patients with diabetes, obesity, or hyperlipidemia. In this patient, her alcohol intake is not enough to cause alcoholic hepatitis. Also her ALT is higher than AST, the reverse of that seen in alcohol-related liver disease. Primary biliary cirrhosis and cholestatic hepatitis have an increase in alkaline phosphatase due to cholestasis, which is normal in this patient. Autoimmune hepatitis is usually associated with a high ANA. NASH can also be seen, though uncommonly, with other conditions like extensive abdominal surgery, rapid weight loss or severe starvation, hyperalimentation, and certain drugs like tamoxifen and amiodarone. The pathogenesis of NASH is not fully understood, but there are several theories. Insulin resistance is the most widely accepted one, which leads to hepatic steatosis and steatohepatitis. Other oxidative injuries can also cause inflammation and stimulate steatohepatitis like hepatic iron, intestinal bacteria, deficiency of anti-oxidants, etc. Ultrasound may show fatty liver, and liver biopsy will reveal fatty change with or without fibrosis and cirrhosis. There is no particular treatment for NASH, though better diabetic control, gradual weight loss, and treatment of hyperlipidemia are recommended. Less than 25% progress to cirrhosis of the liver. It has a much better prognosis than alcoholic hepatitis. Primary biliary cirrhosis is a chronic liver disease of middle-aged women of unknown etiology. An immunologic basis has been proposed. Alkaline phosphatase is increased due to cholestasis as well as IgM level. Early symptoms include pruritus and fatigue. Later jaundice, arthralgias, hyperpigmentation, keratconjunctivitis, steatorrhea, osteomalacia, and vitamin deficiencies may occur. Anti-mitochondrial antibodies are positive in more than 90% of patients and are diagnostic. Liver biopsy shows granulomatous inflammation of the bile ducts. Treatment includes replacement of vitamins, ursodeoxycholic acid, cholestyramine for itching, cyclosporine, colchicines, and methotrexate. Studies are ongoing with lamivudine, zidovudine, and benzafibrate. Autoimmune hepatitis is a heterogeneous condition characterized by the presence of autoantibodies, abnormal liver enzymes, absence of other etiological factors, positive ANA, marked increase in serum gamma globulins, and piecemeal necrosis on liver biopsy. Antismooth muscle antibody, soluble liver antigen antibodies, and antibodies to liver/kidney microsomal (LKM) antigens are seen. Steroids and azathioprine are used for treatment.

A mid-level manager complains that he is having problems at work because his employees just will not take the care he does in doing their jobs. Consequently, he says, he fears that their lack of interest in their work will prevent him from getting ahead because his attention to detail and devotion are not seen by his boss when overall production is low. Probing reveals that there are actually more problems here than just the attitudes of this man's employees. His attention to detail often gets in the way, and it prevents work from being accomplished in a timely manner. You formulate further questions to determine if this man might have what personality disorder? Answer Choices 1 Obsessive-compulsive 2 Narcissistic 3 Borderline 4 Dependent 5 Avoidant

obsessive compulsive Obsessive-compulsive personality disorder is characterized by a lifelong pattern of being preoccupied with perfectionism, orderliness, and control. Individuals with obsessive-compulsive disorder have difficulty being open with others, they are unable to be flexible because it means giving up control, and their perfectionism usually makes them very inefficient in their accomplishments. Other symptoms include excessive devotion to work or to other tasks, being overly conscientious about moral or ethical issues, and having restricted expression of affect. The major features of narcissistic personality disorder are grandiosity (an inflated sense of self importance), a need for admiration, and a lack of empathy for others. If criticized, the narcissistic individual often reacts with rage, and he/she often exploits others. Feelings of grandiosity usually preoccupy the individual, and there is often a profound sense of entitlement. Borderline personality disorder is most often associated with a pattern of unstable interpersonal relationships. Self-image is poor, affect is often labile or depressed, and these individuals are highly impulsive - especially with regard to self-destructive behaviors. Individuals with borderline personality disorder are chronically bored and empty feeling. They alternate between overly idealizing others and devaluing them. Their emotions are intense. Although individuals with borderline personality disorder are said to have a need to be taken care of, they have difficulty accepting the help of others because they mistrust their intentions. Individuals with dependent personality disorder, on the other hand, tend be clingy and submissive in their behaviors because their need to be taken care of is so pervasive. They have difficulty making everyday decisions without consulting others, and they agree with others even if they believe the person to be wrong due to their intense fear of rejection. They have difficulty doing things on their own, and their most pervasive fear is that of being abandoned. Avoidant personality disorder is characterized by social inhibition and feelings of inadequacy. Individuals who suffer from this disorder are extremely sensitive to negative feedback from others. They avoid social activities due to their fear of having to participate.

A 66-year-old man with a history of obesity and hyperlipidemia presents with an 8-month history of progressing "burning in his chest". This sensation is noted in the midline of his chest, and it is provoked when he bends over, wears tight clothing, after he eats a large meal, and when lying supine. He denies any chest pressure, cough, shortness of breath, palpitations, dizziness, lightheadedness, and diaphoresis. His physical examination is unremarkable. An upper endoscopy is performed, which reveals the attached image. Question What is the preferred pharmacotherapeutic intervention for this patient? Answer Choices 1 Omeprazole 2 Nystatin 3 Cisapride 4 Cimetidine 5 Sucralfate

omeprazole The correct response is Omeprazole. This patient's presentation is most consistent with gastroesophageal reflux disease (GERD). Previously, histamine-2 receptor antagonist (H2RA) therapy was recommended as the initial treatment for esophagitis associated with GERD; however, newer evidence in cost-effectiveness analysis and symptomatic relief suggests proton pump inhibitors (PPIs) (such as omeprazole, pantoprazole or lansoprazole daily for 4 - 8 weeks) to be superior to ranitidine, cimetidine, and placebo. This patient is not experiencing fungal esophagitis (which is most commonly caused by Candida albicans). Medical therapy includes topically active agents, such as nystatin or clotrimazole. Oral amphotericin B, fluconazole, or itraconazole are also useful. Gastroprokinetic agents, such as metoclopramide or cisapride, and sucralfate, a coating agent, are less effective, but they may be useful in selected patients or as second-line agents. Cisapride is only available through an investigational limited-access program because of its potential for risk of serious cardiac arrhythmias and death.

Serosanguineous ascites is generally associated with which of the following processes? Answer Choices 1 Ovarian cancer 2 Lymphoma 3 Congestive heart failure 4 Cirrhosis 5 Infection

ovarian cancer Explanation The correct answer is ovarian cancer. Bloody or serosanguineous ascites is associated with intraperitoneal cancers, of which ovarian cancer is one. Lymphoma is incorrect because it will generally produce a chylous ascites. Congestive heart failure is incorrect because it will generally produce serous ascites. Cirrhosis is incorrect because it will generally produce serous ascites. Infection is incorrect because it will generally produce cloudy ascites.

A 40-year-old otherwise healthy woman, nulligravida, presents with involuntary loss of urine that occurs after drinking a small amount of water, when washing the dishes, when hearing water running, and sometimes for no reason discernable to the patient. It is preceded by suddenly feeling the need to urinate and happens both during the day and at night. Urine analysis and culture, pelvic, gynecological, and neurological examinations are normal. Cystometric studies show residual volume of 45 mL (normal) with involuntary detrusor contractions starting already with 200 mL. Question What will be your therapeutic approach? Answer Choices 1 Obtain psychiatric consultation 2 Antibiotic trial 3 Oxybutinin 4 Neostigmine 5 Kegel exercise

oxybutinin Your patient most probably suffers from idiopathic hypertonic incontinence. Her bladder is best described as instable, spastic, or overactive. The most common cause of urge incontinence is involuntary and inappropriate detrusor muscle contractions (normally there are no detrusor contractions during cystometry studies). Cholinergic receptors are present mostly in the detrusor muscle, and their stimulation enhances urination. Therefore, their inhibition with use of an anticholinergic such as oxybutynin will prevent involuntary urgency and incontinency for no apparent reason. Psychiatric consultation will probably not prevent involuntary activity of the detrusor. It can be performed after the organic cause is excluded. Patients with normal urine analysis do not need antibiotics. You will not use neostigmine because it is an inhibitor of acethylcholinesterase. Acethylcholinesterase inhibitors such as neostigmine indirectly stimulate receptors in the detrusor and enhance urination. They could worsen her symptoms. Kegel exercises with voluntary contractions of pubococcygeus muscle will help the patient with some forms of stress incontinence. Your patient is nulligravida and has no signs of problems with pelvic relaxation.

Which serological test can be performed for Dx for Ulcerative Colitis?

pANCA

Your patient is asked to see you in followup for his fasting labs. His total cholesterol = 230 mg/dl, triglycerides = 1200 mg/dl, unable to calculate LDL. Your patient should be advised that due to these findings he is at risk for which of the following? A diabetes mellitus B pancreatitis C gout D diabetes insipidus E hypertension

pancreatitis B The correct answer is (B). Very elevated triglycerides, especially over 1000 mg/dl, increase the patient's risk of acute pancreatitis. This patient requires prompt treatment. Hypertriglyceridemia may be a component of metabolic syndrome but does not directly cause diabetes mellitus or hypertension. Hypertriglyceridemia does not increase the risk of gout and diabetes insipidus.

A 30-year-old man presents with periodic sensations of choking, numbness in his feet, tingling in his legs, cold sweats, and dizziness. He reveals that he used to feel this way as a child whenever he went to the mall with his mother. These feelings have been increasing over the past few years. The patient's episodes occur while checking out at the grocery store, while driving or walking over bridges, and while looking out his 19th-floor office window. What psychiatric diagnosis fits this patient best, after ruling out substance use and any precipitating medical condition? Answer Choices 1 Panic disorder with agoraphobia 2 Social Phobia 3 Posttraumatic stress disorder 4 Generalized anxiety disorder 5 Obsessive-compulsive disorder

panic attack with agoraphobia All of the answer choices are anxiety disorders, i.e., they present as anxiety far out of proportion to the actual situation. What distinguishes them is the pattern of each disorder's symptomatic manifestation. Panic disorder with agoraphobia is described as panic attacks not due to direct physiological effects of a substance or general medical condition. Phobias manifest as intense anxiety that is routinely elicited by specific situations (crowds, escalators, standing in lines, high places) or the presence of specific things (bees, water, snakes, spiders). A social phobia is a marked and persistent fear of 1 or more social performance situations in which the person is exposed to unfamiliar people or the possible scrutiny of others. Posttraumatic stress disorder would only be indicated if all of the fearful situations were related to the same general traumatic event(s). Generalized anxiety disorder is "free-floating" and not relative to specific situations. Obsessive-compulsive disorder involves behavioral or ideational loops that seem intended to "ward off" the experience of anxiety.

Psammoma bodies are concentric calcified structures and are found in several tissue pathologies. In the image, psammoma bodies can be seen. Psammoma bodies are characteristic of which of the following? Answer Choices 1 Papillary carcinoma of the thyroid 2 Mucinous cyst adenocarcinoma of the ovaries 3 Medullary carcinoma of thyroid 4 Follicular adenoma 5 Pheochromocytoma

papillary carcinoma Single necrotic cells may form a nidus for deposition of calcium. Subsequently, the progressive acquisition of an outer layer of calcium may form a lamellated structure, which resembles grains of sand. This is called a psammoma body. Psammoma bodies are seen in some neoplastic conditions, such as papillary carcinoma of the thyroid, serous cyst adenocarcinoma of the ovary, adenocarcinoma of the endometrium, and meningioma. Rarely, it may be seen in adenocarcinoma of the lung. In the case of the ovary, the possible mode of formation is by neoplastic and histiocytic cellular degeneration. Psammoma bodies (blue arrow) - calcific concretions with well-defined concentric laminations

A 21-year-old female presents to clinic complaining of mild, low abdominal ache and intermittent dysuria. She denies N/V/D, and she is sexually active and uses condoms some of the time. Her LMP was 10 days ago, and she is a G0P0. Physical exam reveals a healthy female in no acute distress. Vitals are as follows: P 70, BP 120/80, T 99.9°F. Lungs are clear, CV RRR, abd soft non-tender, + BS. Pelvic exam reveals normal external genitalia, scant discharge, moderate cervical motion tenderness, and no adnexal masses. What is her most likely diagnosis? A Tubo-ovarian abscess B Gastroenteritis C Ectopic pregnancy D Cervicitis E Pelvic inflammatory disease

pelvic inflammatory disease Suspicion for PID should be very high in a young, healthy, and sexually active woman with cervical motion tenderness. She is not spotting and just menstruated, making ectopic pregnancy much less likely.

A 30-year-old patient presents to labor and delivery complaining of bright red vaginal bleeding. She has no pain. The fetus is still active. She is 37 weeks pregnant. PMH is significant for in vitro fertilization. What is the most likely diagnosis? A Placental abruption B Placenta acreata C Placenta previa D Disseminated intravascular coagulopathy E Active labor

placenta previa C The most likely diagnosis is placenta previa, as the bleeding is bright red and painless. Labor and abruption are associated with discomfort and pain.

A 29-year-old woman, G4P2011, LMP 9 months prior, presents with severe lower abdominal pain. The pain is sharp and tearing. She began to have vaginal spotting prior to presentation. She denies any contractions. There is no history of medical problems or surgery, and she is on no medications. All previous deliveries were vaginal. She has smoked 1 pack of cigarettes a day over the past 10 years, denies alcohol use, but does admit to a remote history of heroin abuse by insufflation. On physical examination: T = 100.4° F; BP= 110/70 mmHg; P= 85/min; RR= 20/min. Pertinent findings on the PE were relegated to the pelvic exam; fundal height measures 39 cm, and there is profuse bleeding from the vagina. Fetal monitor shows contractions every minute with elevated baseline uterine tone. Fetal tachycardia is evident at 180 beats/minute, and late decelerations are also present. Question What is the most likely diagnosis? Answer Choices 1 Cervical cancer 2 Chorioamnionitis 3 Placenta previa 4 Placental abruption 5 Uterine rupture

placental abruption Explanation This patient has placental abruption. Abruption is a leading cause of 2nd and 3rd trimester bleeding. A meta-analysis has demonstrated that smoking increases the risk of abruption by 90%. Maternal and paternal smoking increases the risk of abruption 2-fold. Risk increases 5-fold when both parents smoke. Women with a history of abruption have a 15% increased risk during future pregnancies. Placenta previa, cocaine use, preeclampsia, and preterm premature rupture of membranes (PPROM) are also associated with placental abruption. Placental abruption is defined as complete or partial separation of the placenta prior to delivery. The incidence of abruption is 5 or 6 out of 1000 deliveries. Obvious vaginal bleeding occurs if the hemorrhage develops between the membranes and the uterus. Concealed presentations occur when blood collects behind the placenta. Abruption is a leading cause of 2nd and 3rd trimester bleeding, and it causes significant maternal and neonatal morbidity and mortality. Classically placental abruption presents as painful third-trimester bleeding. Treatment:Emergent cesarean is generally indicated when the patient is not in labor and hemodynamic compromise is present. If delivery is imminent and the abruption is mild, vaginal delivery may be attempted. Cervical cancer is incorrect.Although cervical cancer can coexist with pregnancy, it is very unlikely in this case. The severity of the signs and symptoms in this case would be incongruent with achieving 9 months gestation. Chorioamnionitis is incorrect.Clinical features include maternal fever and uterine tenderness in the presence of confirmed premature rupture of membrane (PROM). Profuse vaginal painful bleeding is not a finding. Placenta previa is incorrect, as it classically presents with painless 3rd trimester bleeding. Uterine rupture is incorrect. A uterine rupture typically occurs during active labor. As this patient denies any contractions, it is very unlikely.

A 72-year-old man presents with longstanding but increasing dyspnea, and a 3 day history of shortness of breath, coughing, unilateral sharp chest pain and mild temperature elevation. Pain is worse when he takes a deep inspiration and when he coughs. PMH is significant for a smoking history of 2 packs/day for 40 years equating to 80 pack years. The patient has other co-morbidities including CAD, COPD, CHF, hypertension and dyslipidemia. He denies recent travel, sick contacts, occupational exposure, drug or alcohol use. Vital signs are BP 190/96, HR 140, RR 48, 02 68% on room air. Chest examination of the left posterior chest reveals a dull percussion note, inaudible bronchovesicular breath sounds, decreased tactile fremitus, a pleural friction rub on inspiration and diminished voice sounds below the 6th intercostal space. Significant bilateral lower extremity edema is also noted. A chest x-ray reveals blunting of the costophrenic angle. Question What is the most likely diagnosis? Answer Choices 1 Pneumothorax 2 Pulmonary thromboembolism 3 Pulmonary fibrosis 4 Pleural effusion 5 Bronchiectasis

pleural effusion The signs and symptoms of the patient are consistent with pleural effusion. The key finding is blunting of the costophrenic angle on chest x-ray. Fluid in the pleural space pushes the lung upward resulting in a blunting of the costophrenic angle that is normally seen as a sharply pointed downward indentation. Results of the chest examination show decreased or absent breath sounds, dullness to percussion over affected area, decreased tactile fremitus, and friction rub. The most common cause of pleural effusion is left-ventricular failure. Pneumothorax is characterized by hyperresonance of percussion note, JVD due to compression of the superior vena cava, falling 02 saturation, and hypotension. Chest x-ray shows a visceral pleural line and/or lung retraction from the chest wall. Pulmonary thromboembolism is characterized by sudden onset dyspnea, pleuritic chest pain, hemoptysis, tachypnea, unilateral lower extremity edema, hypoxia and hypocarbia. Pulmonary fibrosis is associated with exertional dyspnea, non-productive cough and crackles heard on inspiration. CT scan shows patchy opacities at lung bases with honeycombing. Bronchiectasis is associated with dilation of the bronchi. Patients present with productive cough with yellow or green sputum, dyspnea, hemoptysis. Exam reveals rales, rhonchi, wheezes, purulent mucus. Chest x-ray reveals increased bronchovascular markings; tram lines and areas of honeycombing.

A 45-year-old man presents with a 2-day history of sharp left-sided chest pain. It is aggravated by taking a deep breath. He denies any trauma to the chest. On examination, he is febrile, dyspneic, and has rales on auscultation. Question What is the most likely cause of his chest pain? Answer Choices 1 Esophageal reflux 2 Pneumonia 3 Aortic dissection 4 Pulmonary embolism 5 Unstable angina

pneumonia Pneumonia pain is usually described as a sharp pain that is aggravated by inspiration. Other signs and symptoms of pneumonia, such as cough, fever, dyspnea, and rales on auscultation, may also be present. Esophageal reflux pain is a substernal pain that is usually described as burning. It may last for up to 1 hour. It is relieved by antacids and may be aggravated by alcohol and postprandial recumbency. Aortic dissection pain is of abrupt onset that is usually described as severe and tearing. It may radiate from the anterior chest wall posteriorly to the interscapular region. On examination, the blood pressure may be elevated, and a pericardial rub and the diastolic murmur of aortic insufficiency radiating along the right sternal border may be auscultated. Pulmonary embolism pain is usually of abrupt onset and pleuritic in nature. It has a duration of several minutes to a few hours. There may be predisposing factors, such as deep venous thrombosis or long periods of immobilization. Patients may also report hemoptysis. On examination, the patients are dyspneic and tachypneic with tachycardia. Unstable angina pain is usually described as tightness or pressure on the chest and lasts for 10 -20 minutes. It is usually left-sided or retrosternal, and it may radiate to the jaw and neck. It often occurs at rest, but it may be precipitated by physical exertion and emotional stress. On examination, a systolic murmur of mitral regurgitation may be auscultated during the pain.

A 47-year-old man presents with abdominal pain and difficulties breathing. He has a history of alcohol abuse and confirmed cirrhosis of the liver. On examination, you see a malnourished, jaundiced patient with a distended belly. Percussion of the abdomen reveals a huge amount of fluid and wave sign. What is the primary cause of the ascites? Answer Choices 1 Increased albumin production 2 Increased ammonia production 3 Portal hypertension 4 Decreased fluid intake 5 Blockage of the common bile duct

portal hypertension In cirrhosis the fibrotic changes in the liver tissue increase the resistance to blood flow through the organ and this results in increased portal pressure. The lesion is intrahepatic and sinusoidal. Hepatic synthetic failure causes hypoalbuminemia and decreased albumin production, which leads to reduced portal oncotic pressure in cirrhotic patients. Ammonia levels may be elevated, but elevated levels do not cause ascites. Ammonia is a highly toxic metabolic product of the urea cycle. The urea cycle is the only major pathway to remove waste nitrogen, and it usually gets converted to non-toxic urea in the liver. Increased portal pressure, reduced portal oncotic pressure, and elevated aldosterone level with sodium retention cause ascites. Decrease in fluid intake is actually one of the therapeutic strategies for ascites. Blockage of the common bile duct results in jaundice, but it has nothing to do with the development of ascites.

In addition to insulin and fluid replacement with 0.9% saline, which electrolyte is commonly infused in the type 2 diabetic patient who arrives in the emergency department in a hyperglycemic, hyperosmolar, nonketotic state? A bicarbonate B potassium C calcium D magnesium E sulfate

potassium Insulin not only causes cellular uptake of glucose but also of potassium. Hypokalemia may develop when insulin is infused to correct either a hyperglycemic hyperosmolar state or a diabetic ketoacidosis. Hence, in order to avoid hypokalemia, potassium chloride can be added to a saline solution, as long as the serum potassium is not elevated.

A 50-year-old man with no significant past medical history presents for a follow-up. He notes that, over the past week, he has developed increased nonproductive cough and shortness of breath. He has had a chronically-progressive cough and shortness of breath upon exertion over the past year. His occupational history reveals coal mining for 25 years. He denies any smoking, alcohol consumption, travel history, or sick contacts. He also denies fever, chills, sore throat, otalgia, chest or abdominal pain, peripheral edema, rashes, or pruritus. A chest radiograph was performed which revealed the following imaging. Question What treatment would be most beneficial in this case? Answer Choices 1 Trexall (Methotrexate) 2 Oseltamivir (Tamiflu) 3 Amoxicillin-clavulanate (Augmentin) 4 Prednisone (Deltasone) 5 Acyclovir (Zovirax)

prednisone The correct response is prednisone (Deltasone). This patient's most likely diagnosis is silicosis. His acute manifestations signify the development of acute silicosis on top of chronic silicosis. The X-ray demonstrates miliary mottling with perihilar and mediastinal lymphadenopathy showing 'egg shell calcification'. Corticosteroids may be of benefit in acute silicosis. In chronic silicosis, they are unlikely to be of any benefit, although pulmonary function improvement was noted in 1 study. This patient is not experiencing a bacterial or viral infection; therefore, antibiotics (Augmentin) or antiviral agents (Tamiflu and Zovirax) are inappropriate. Methotrexate is indicated in the treatment of ectopic pregnancy, autoimmune disorders (such as psoriasis and rheumatoid arthritis), as second-line therapy in sarcoidosis, and several malignancies.

A 20-year-old woman is brought to the emergency room (ER) by the police. She was assaulted and raped 3 hours ago. Her last menstrual period (LMP) was 22 days ago, and her period lasts for 4 - 5 days with a 28-day menstrual cycle. She is not using any contraceptive method. What hormone is mainly produced by the ovary at this point of her menstrual cycle? Answer Choices 1 Estrogen 2 Follicle stimulating hormone (FSH) 3 Inhibins 4 Luteinizing hormone (LH) 5 Progesterone

progesterone Explanation The correct response is progesterone. The 5th day of the menstrual period corresponds to the follicular phase of the cycle, which starts from the first day of bleeding to the day of the LH peak. In this phase, the FSH, which is produced by the pituitary, stimulates the development of follicles in the ovaries, with only 1 follicle dominant at the end. During the second half of this phase, the granulosa cells of the follicle begin to produce estrogen. Ovulation occurs approximately at the middle of the menstrual cycle (day 14). At the end of the follicular phase, a rise in the level of estrogens takes place, preceding the LH peak. The ovulation will succeed by an average of 30 hours after this peak. The 22nd day of the menstrual period corresponds to the luteal phase of the menstrual cycle. The luteal phase starts from the day of ovulation to the first day of the menstrual period. In this phase, the ovarian corpus luteum begins to produce progesterone. If there is fecundation fertilization of the ovum, the production of progesterone continues for 4 to 5 weeks, until the placenta can take over and produce enough of the hormone to maintain pregnancy. If there is no fertilization, progesterone levels decrease and the menstruation occurs. Estrogens are produced by the granulosa cells of the follicle. They are very important in the follicular phase; their level is very low at the beginning of this phase and reaches a maximum point at the middle of the cycle, just before the LH peak. They are the feedback for the LH production and (in part) for the FSH production. Inhibins are also hormones produced by granulosa cells of the follicle, and they are part of the feedback for the FSH. Inhibin B levels rise during the luteal-follicular transition, are highest during the mid follicular phase, then go up again during the LH peak, and finally decrease in the late follicular phase. Inhibins A levels decrease during the late luteal phase.

A 31-year-old woman is being evaluated for irregular, infrequent menstrual periods. On further questioning, she complains of headaches, fatigue, and breast discharge. She takes ibuprofen only occasionally. Which of the following labs would most likely be elevated in this patient? A BUN and creatinine B luteinizing hormone (LH) and follicle-stimulating hormone (FSH) C oxytocin D prolactin E TSH

prolactin This patient's symptoms are consistent with a pituitary adenoma. Prolactinomas account for about half of all functioning pituitary tumors and may secrete PRL, GH, and ACTH.

A 6-month-old infant is brought to your office. The mother indicates that the infant had several scratches that took a long time to stop bleeding. After ruling out various forms of hemophilia, you suspect that the infant has a vitamin deficiency that is causing this problem. What proteins would be affected by this vitamin deficiency? Answer Choices 1 Protransglutaminase 2 Urokinase 3 Prekallikrein 4 Prothrombin 5 Fibrinogen

prothrombin Prothrombin and many of the other blood clotting factors contain the modified amino acid γ-carboxyglutamic acid (gla). This modification occurs via an enzymatic reaction requiring vitamin K and converts glutamic acid, a weak calcium chelator, to gla, a much stronger calcium chelator. Vitamin K is an isoprenoid compound, classified as a fat-soluble vitamin; it is found in green plant leaves and is also synthesized by intestinal bacteria. The formation of gla residues is required for the "active" form of prothrombin. These gla residues are found in the amino terminal region of the protein and function by chelating calcium ions. This forms part of the binding site to phospholipid membranes, where the conversion from the zymogen prothrombin to active thrombin takes place. A vitamin K deficiency results in an increased coagulation time. A 2,3-epoxide derivative of vitamin K is an intermediate in the formation of gla residues. Other proteins that contain gla residues include Factors VII, IX, and X, several other anticoagulation proteins, and the bone proteins (Osteocalcin and matrix-Gla protein). Dicumarol is a naturally occurring anticoagulant that is an antagonist of vitamin K. It inhibits the reductase that converts the epoxide form of vitamin K back to the active vitamin.

A 28-year-old Caucasian woman presents with worsening shortness of breath with activity. The patient states that, over the last 4 weeks, it has become increasingly difficulty for her to walk up a flight of stairs without stopping after a few steps to rest. Currently, she cannot even walk across a room without feeling out of breath. She admits to significantly increased fatigue and dizziness as well as shortness of breath. The patient has a history of systemic lupus erythematosus; she was diagnosed with the condition about 2 years ago. Physical examination reveals a young woman who appears to be her stated age, but she is in a mild amount of distress due to being short of breath while just sitting on the examination table. Accessory muscle use is present. The patient's skin has a faint bluish hue to it; her lips are also a shade of blue. Ankles reveal 2+ pitting edema. Question What is the most likely diagnosis? Answer Choices 1 Pulmonary embolism 2 Pulmonary hypertension 3 Congestive heart failure 4 Dilated cardiomyopathy 5 Pericarditis

pulmonary htn The patient above is most likely suffering from restrictive lung diseasedue to the lupus, and specifically signs and symptoms of pulmonary hypertension. Pulmonary hypertension is a lung disorder the pulmonary arteries become narrowed. As a result, the blood pressure in these arteries rises above normal levels. The high pressure strains the right ventricle of the heart, causing it to expand in size. Overworked and enlarged, the right ventricle gradually becomes weaker and loses its ability to pump enough blood to the lungs. This can lead to the development of right heart failure. Symptoms of pulmonary hypertension do not usually occur until the condition has progressed. The initial symptom is usually shortness of breath with everyday activities, such as climbing stairs. Fatigue, dizziness, and fainting spells also can be symptoms. Edema in the ankles, abdomen, or legs may be present; bluish lips may also be present. Chest pain may occur as strain on the heart increases. Causes of pulmonary hypertension may include, but are not limited, to dexfenfluramine and phentermine (fen-phen has been taken off the market, but former fen-phen users have a 23-fold increase risk of developing pulmonary hypertension, possibly years later), liver diseases, rheumatic disorders (which include systemic lupus erythematosus), other lung conditions, heart diseases, and low-oxygen conditions; it may be idiopathic in origin. Congestive heart failure is usually a disease state that initially affects the left side of the heart, causing signs and symptoms relating to that. Pulmonary embolism is incorrect; it usually presents with dyspnea and pain on exertion, possible tachypnea, and (potentially) even hemoptysis or syncope. It is also usually seen in a more acute presentation scenario. Pericarditis is incorrect, although may be extremely hard to differentiate from pathologies that cause right-sided heart failure. The cause of pericarditis is a major difference; etiologies include tuberculosis, radiation therapy, cardiac surgery, viral pericarditis, or histoplasmosis. Dilated cardiomyopathy is incorrect; this condition will also occur in the left portion of the heart initially.Causes include (but are not limited to) ETOH abuse, catecholamine excess, and myocarditis; they can also be idiopathic.

What is the definitive treatment of choice for elderly patients diagnosed with Graves' disease? A Beta blocking agents B Levothyroxine C Methimazole D Total thyroidectomy E Radioactive iodine

radioactive iodine

A 5-year-old girl is seen in your office with a several week history of increased thirst, weight loss, and blurred vision. She has a positive family history for diabetes mellitus, hypertension, and stroke. Her urine dipstick chemical testing reveals positive glucose and negative ketones, protein, blood, and nitrites. Which of the following laboratory test results would support a diagnosis of diabetes mellitus in this patient? A random plasma glucose > 200 mg/dL B random urine glucose dipstick > 1+ C plasma hemoglobin A1c < 7% D fasting plasma glucose > 110 mg/dL E 2-hour postprandial plasma glucose > 135 mg/dL

random plasma glucose > 200 mg/dL

A newborn baby boy is listless and febrile. He has diarrhea and vomiting as well. There is no nuchal rigidity, however CSF examination confirms meningitis. He is being treated with gentamicin and ampicillin. Question What is a common side effect he may experience? Answer Choices 1 Rash 2 Constipation 3 Thrombocytopenia 4 Leukopenia 5 Elevated liver enzymes

rash Explanation Skin rash is a common side effect of ampicillin. Rash occurs at a rate of approximately 3-7%. Constipation is not a side effect of ampicillin. Diarrhea is a side effect of ampicillin, rather than constipation. Thrombocytopenia, leukopenia, and elevated liver enzymes can be seen but less frequently than a rash. References:

A 12-year-old boy is being seen for concerns of development of breast tissue. Upon physical exam, he is noted to have a firm, slightly tender mass under the left areola. What is the most appropriate action at this time? A referral to pediatric surgery for resection B measurement of serum hCG C measurement of testosterone and estrogen levels D reassurance and observation

reassurance and observation Type 1 idiopathic gynecomastia in adolescent men presents with a firm mass under the areola ("breast bud") typically during sexual maturation stages (SMR), stages II to III. This is a result of normal estrogen and androgen activity at the breast tissue level. Appropriate action is observation and to reassure the patient that the condition will likely resolve in 1 to 2 years.

A 53-year old-man presents with a history of indolent, dull pain in his rectal area; it is associated with pruritus. The pain is increased upon movement, defecation, and prolonged sitting. He has experienced constipation and describes his stools are described as "hard", but they retain their normal brown color without evidence of blood. He denies ever having had these symptoms before. He denies any sexual contact, drug use, or fever, chills, or other gastrointestinal symptoms. Upon inspection, erythema is observed, and a digital rectal exam reveals a fluctuant, tender, and indurated mass, which is seen in the attached image. Question What is the most likely diagnosis? Answer Choices 1 External hemorrhoid 2 Rectal fistula 3 Acute proctitis 4 Rectal abscess 5 Necrotizing fasciitis

rectal abscess This patient's presentation is most consistent with an anorectal abscess. Perirectal abscesses are associated with indolent, dull perirectal pain that is exacerbated by movement and increased perineal pressure from sitting or defecation. There may be associated pruritis. Those with an ischiorectal abscess often present with systemic fevers, chills, and severe perirectal pain and fullness consistent with the more advanced nature of this process. External signs are minimal and may include erythema, induration, or fluctuance. Physical examination demonstrates a small, erythematous, well-defined, fluctuant, subcutaneous mass near the anal orifice. On digital rectal examination (DRE), a fluctuant, indurated mass may be encountered. The most common presentation of hemorrhoids is rectal bleeding, pain, pruritus, or prolapse, with rectal bleeding being the most common presenting symptom. An anal fistula is an inflammatory tract between the anal canal and the skin. A patient with an anal fistula may complain of recurrent malodorous perianal drainage, pruritus, recurrent abscesses, fever, or perianal pain due to an occluded tract. Inflammation of the mucosal lining of the rectum is defined as proctitis. The presentation includes persistent bright red rectal bleeding, a decrease in stool volume and an increase in mucoid consistency, tenesmus, fecal urgency or constipation, and abdominal cramping. Necrotizing fasciitis is a rapidly progressive inflammatory infection of the fascia, with secondary necrosis of the subcutaneous tissues. It may be caused by aerobic, anaerobic, or mixed flora bacteria. Fournier gangrene is a form of necrotizing fasciitis that is localized to the scrotum and perineal area. The hallmark symptom of necrotizing fasciitis is intense pain and tenderness over the involved skin and underlying muscle. Pain usually develops prior to other symptoms, such as fever, malaise, and myalgias. Other findings include edema extending beyond the area of erythema, skin vesicles, and crepitus.

A 45-year-old man presents with hematemesis. He has had 2 episodes of vomiting 'coffee-ground'-appearing material; the vomiting began 45 minutes prior to presentation. Additionally, he reports passing black, sticky stools for the past 3 or 4 days. Past medical history is positive for occasional headaches; they have been coming more frequenly lately. Social history reveals alcohol use (1 case of beer each weekend) and tobacco (1 pack per day). Medications include ibuprofen as needed for headaches; he has been taking 800 mg 3 times a day for the past week. You place a nasogastric tube and find bright red blood that fails to clear with saline irrigation. Hemoglobin is 8.9 g/dL. Evaluation of his blood pressure and pulse reveals orthostatic changes that resolve with an intravenous fluid bolus of 500 cc of Lactated Ringer's solution. What should you do next? Answer Choices 1 Transfuse 2 units of packed red blood cells and recheck CBC in 8 hours 2 Check serum for Helicobacter pylori antibody 3 Order a double contrast barium swallow to evaluate for a gastric ulcer 4 Refer for emergency upper endoscopy 5 Discontinue his ibuprofen use and encourage cessation of alcohol and tobacco

refer for emergency endoscopy He should be referred for an emergency upper endoscopy. This patient is most likely bleeding from a gastric ulcer. His recent NSAID use, as well as his alcohol and tobacco habits, make him at risk for peptic ulcer disease. His symptoms of melena and hematemesis, along with his anemia, make the diagnosis quite straightforward. It appears that this patient is still actively bleeding based on the results of the nasogastric tube irrigation; therefore, the priority should be getting the ulcer to stop bleeding. Upper endoscopy should be performed so that the bleeding site can be identified and treated with electrocautery, coagulation, or injection of epinephrine or a sclerosing agent. If the bleeding cannot be stopped with endoscopic interventions, angiographic embolization should also be tried. If these interventions do not succeed, the patient has rapid deterioration, or if he requires more than 6 units of blood in a 24-hour period, then emergency surgery may be indicated. The other choices are not the best options for immediate management. This individual cannot be followed simply with transfusions and serial CBC's because he appears to still be actively bleeding. Helicobacter pylori infection may very well be playing a part in the etiology of this man's ulcer, but evaluation for H. pylori can be done with a biopsy at the time of his endoscopy; it will not help in his immediate management. A barium esophagram will not identify actively bleeding ulcers and cannot treat active bleeding. While NSAID, alcohol, and tobacco use may have precipitated this man's GI bleed, counseling about his use of these substances will not sufficiently treat his immediate bleed.

A 15-year-old female comes into your office with multiple symptoms of an eating disorder. What is the single feature that would lead you to diagnose anorexia rather than bulimia or binge eating disorder? A Binge eating behaviors B Compensatory measures such as purging C Preoccupation with body weight D Refusal to maintain normal body weight E Strict focus on dietary intake

refusal to maintain normal body weight D The diagnosis of anorexia requires that the patient refuse to maintain a normal body weight (D). All the other behaviors mentioned (A, B, C, E) may be shared between anorexia and bulimia. People with binge eating disorder (not fully recognized in the DSM IV-TR) display binge eating without compensatory measures, resulting in obesity.

A 17-year-old male arrives comatose in the emergency department where you are on duty. There is no history on your patient and no visible signs of trauma. You draw blood for gases. If the blood gas results are pH 7.6, H+ 38 nEq/L, PCO2 32 mmHg, and HCO3- 23 mEq/L, what acid-base disturbance does your patient have? Answer Choices 1 Respiratory acidosis 2 Respiratory alkalosis 3 Metabolic acidosis 4 Metabolic alkalosis 5 No disturbance, he is normal

respiratory alkalosis With respiratory acidosis, his blood pH should be below 7.4 and his pCO2 should be greater than 40 mmHg. Rarely does respiratory alkalosis occur. The underlying mechanism is increased respiration decreases CO2 in the body. Respiratory alkalosis can occur if an individual ascends to a high altitude and, in an effort to increase O2 levels in the blood, the respiratory rate increases. With metabolic acidosis, his blood pH should be below 7.4. While the patient is alkalotic, you would expect his pCO2 to increase as his respiratory system compensates for metabolic alkalosis. Normal pH is roughly 7.4.

A 21-year-old woman presents for a Pap smear. On speculum exam, the cervix is in a normal position. During a bimanual examination of the pelvis, the body of the uterus is unable to be palpated by the abdominal or pelvic hand. When a rectovaginal exam is performed, the body of the uterus is palpable. What position is this patient's uterus in? Answer Choices 1 An anteverted uterus 2 A retroflexed uterus 3 A retroverted uterus 4 A second-degree prolapsed uterus 5 A third-degree prolapsed uterus

retroflexed uterus Explanation When the uterus that cannot be palpated by a bimanual exam but can be felt rectovaginally, it is tilted backwards. A retrodisplaced uterus wherein the cervix is normally positioned, as in this case, is in a retroflexed position. A retrodisplaced uterus with a forward-facing cervix is in a retroverted position. An anteverted uterus is a forward-facing uterus, the most common position found. In a 2nd-degree prolapsed uterus, the cervix is in the vaginal introitus with the body of the uterus descending from its normal placement in the pelvic. In a 3rd-degree prolapsed uterus, the cervix and uterus are outside the vaginal introitus.

A 28-year-old woman presents with abdominal pain and vaginal bleeding. The pain began last night as a dull ache in the right lower quadrant, but this morning it became much more severe. She also complains of dizziness and nausea. She cannot recall when her last menstrual period was, but she says that she began bleeding yesterday and has a light menstrual flow. On examination, the patient is afebrile; pulse is 100/min, BP is 86/60 mm Hg, and RR is 20/min. Physical exam reveals moderate to severe tenderness in the right lower quadrant with rebound. Pelvic exam reveals a small amount of blood at the cervical os with cervical motion tenderness. You order a variety of laboratory tests on this patient. Question What is the most likely diagnosis? Answer Choices 1 Acute appendicitis 2 Acute salpingitis 3 Tubo-ovarian abscess 4 Ruptured ectopic pregnancy 5 Ruptured ovarian cyst

ruptured ectopic pregnancy Explanation Ruptured ectopic pregnancy is probably the most life-threatening condition in gynecology. Any practitioner that cares for reproductive-aged women must keep a high index of suspicion for this dangerous event in order to avoid a delay in diagnosis that may be fatal. Ectopic pregnancy is defined as extra-uterine implantation of a pregnancy; it occurs most commonly in the fallopian tube (96%). Most tubal pregnancies are found in the distal 2/3 of the tube. 2% of ectopic pregnancies are uterine ectopic pregnancies (interstitial); other sites include the cervix, ovary, or pelvic and abdominal cavities, although these implantations are rare. The major risk factors for ectopic pregnancy are a prior tubal infection or tubal surgery, a prior ectopic pregnancy (giving a risk of 10-25+% for recurrence), a history of diethylstilbestrol (DES) exposure, or presence of an IUD (intrauterine device). Ectopic pregnancy occurs in 0.5 - 1.0% of pregnancies; however, the incidence of ectopic pregnancy is increasing in recent years. This increase is probably a reflection of the increasing incidence of pelvic inflammatory disease, increased use of IUDs, and increasing incidence of tubal ligation for sterilization and other tubal surgeries. A tubal pregnancy will usually present with a history of vaginal spotting and crampy pain as early as 4 to 5 weeks after the last menstrual period (LMP). The tubal pregnancy will usually bleed slowly at first, but when the pregnancy expands to the point of tubal rupture, patients can present with rapid hemorrhage, experiencing shock and then death. On physical exam, the patient will exhibit tenderness and often peritoneal irritation, especially if blood has leaked out of the fallopian tube. The uterus will be enlarged, but it will be smaller than expected for the age of the pregnancy. There is often cervical motion tenderness, and an adnexal mass may be felt. Abdominal pain with vaginal bleeding accompanied by hypotension, marked abdominal guarding, and rebound tenderness is suggestive of ruptured ectopic pregnancy. A serum pregnancy test may reveal a smaller quantitative value than expected by LMP. If the diagnosis is in question and the patient stable, serial beta-hCG values may be estimated. In ectopic pregnancy, the serial beta-hCG does not rise appropriately with respect to the gestational age (doubling every 48 - 72 hours). However, transvaginal ultrasound is usually utilized to look for the absence of an intrauterine gestational sac and the presence of an adnexal mass, which confirms the diagnosis of ectopic pregnancy. If a patient has a beta-hCG level of 1,500 mIU per mL or greater, but the transvaginal ultrasonography does not show the presence of an intrauterine gestational sac, then ectopic pregnancy should be suspected. Culdocentesis, with the finding of old, non-clotting blood, used to be performed commonly in the assessment of tubal pregnancy, but it has largely been replaced by ultrasound because this imaging technique has increased sensitivity. An untreated ectopic pregnancy is usually fatal. Early diagnosis is the key to appropriate treatment, and the mortality rate for ectopic pregnancy in the US is currently 10-15%. The treatment for ectopic pregnancy is usually surgical, especially if there has been tubal rupture. However, some early ectopic pregnancies may be treated with methotrexate. These patients must be followed carefully with serial serum pregnancy tests and ultrasounds, and up to 1/3 of these patients will still require eventual surgical intervention. While appendicitis, salpingitis, ovarian cyst, and tubo-ovarian abscess may also present with abdominal pain, the absence of specific symptoms for each of these helps in ruling them out; symptoms of abdominal pain, vaginal bleeding, and amenorrhea are characteristic for ectopic pregnancy.

A 30-year-old woman has been acting strangely for several weeks. She attends conscientiously and regularly to her work in an insurance office, but lately she has been talking about her co-workers "plotting against her". She claims that she is the sole possessor of a digital encryption key that, if discovered by the wrong people, could result in the collapse of the entire U.S. economy. Despite the dangerous implications of this belief, if it were true, she appears to be oddly unaffected by it. In the absence of evidence of substance abuse or of a precipitating general medical condition, what psychiatric diagnosis might be suggested at this time? Answer Choices 1 Dementia 2 Schizophreniform disorder 3 Schizophrenia 4 Autism 5 Hypomania

schizophreniform disorder Schizophreniform disorder has symptoms in common with schizophrenia: Delusions (unrealistic beliefs, e.g., grandeur, persecution) Hallucinations (unrealistic sensations or perceptions) Disorganized speech Disorganized or catatonic behavior Negative symptoms (flattened affect, alogia, avolition) The primary difference between the 2 is duration. Schizophreniform disorder is diagnosed when psychotic signs have been present less than 6 months. Consequently, it is often designated as a "provisional" diagnosis. Dementia is always correlated with a physiological condition. Autism is a developmental disorder that is primarily neuromuscular and is evident from early childhood. Hypomania refers to a disorder of elevated or irritable mood. Hallucinations and delusions are not symptoms of dementia, autism, or hypomania.

A 48-year-old woman presents with a 6-month history of difficulty in swallowing. A barium swallow reveals an area of stricture in the lower esophagus just above the gastroesophageal junction. She has an upper endoscopy performed, and a biopsy of the lower esophagus is taken; it shows no acute inflammation or ulceration, only submucosal atrophy along with fibrosis of smooth muscle. What do these findings most strongly suggest? Answer Choices 1 Iron-deficiency anemia 2 Scleroderma 3 Barrett's esophagus 4 Portal hypertension 5 Mallory-Weiss syndrome

scleroderma Patients with progressive systemic sclerosis, with either the CREST syndrome or diffuse scleroderma, have fibrosis in the gastrointestinal tract; most often, it is located in the esophagus. Scleroderma is a serious, progressive disease that affects the skin and connective tissue (e.g., cartilage, bone, fat, and the tissue that supports the nerves and blood vessels throughout the body). Scleroderma is also frequently called systemic sclerosis. Iron-deficiency anemia can be associated with esophageal webs. This is called Plummer-Vinson syndrome, and it is quite rare. Barrett's esophagus is a columnar metaplasia of the esophageal epithelium that occurs with chronic reflux; the esophagus is lined with gastric mucosa, and it is usually diagnosed endoscopically. Portal hypertension leads to the appearance of esophageal varices in the lower esophagus that can erode and bleed profusely. Mallory-Weiss syndrome is an esophageal mucosal laceration; it usually presents with marked vomiting, and it is typically found in alcoholics. Mallory-Weiss syndrome is bleeding from an arterial blood vessel in the upper gastrointestinal tract, and it is caused by a mucosal gastric tear at or near the point where the esophagus and stomach join.

A 50-year-old male is seen with a routine check-up. He is concerned about the possibility of developing diabetes mellitus. He has a negative family history of diabetes. He has no signs or symptoms of diabetes and he is not overweight. Without any risk factors for diabetes, what is the recommended screening protocol for this patient according to the American Diabetes Association (ADA)? A screen all men over 25 years of age every five years B screen all men over 35 years of age every two years C screen everyone over 45 years of age every three years D no screening is necessary without risk factors E no screening is necessary without a family history of diabetes

screen everyone over 45 years of age every three years The correct choice is C, screen everyone older than 45 years of age every three years. In addition, the ADA recommends screening for younger people if they are overweight and have at least one additional risk factor, such as positive family history, hypertension, and/or vascular disease. The other choices are not recommended by the ADA for screening the general population for diabetes mellitus.

A 50-year-old male is seen with a routine check-up. He is concerned about the possibility of developing diabetes mellitus. He has a negative family history of diabetes. He has no signs or symptoms of diabetes and he is not overweight. Without any risk factors for diabetes, what is the recommended screening protocol for this patient according to the American Diabetes Association (ADA)? A screen all men over 25 years of age every five years B screen all men over 35 years of age every two years C screen everyone over 45 years of age every three years D no screening is necessary without risk factors E no screening is necessary without a family history of diabetes

screen everyone over 45 yo every 3 years The correct choice is C, screen everyone older than 45 years of age every three years. In addition, the ADA recommends screening for younger people if they are overweight and have at least one additional risk factor, such as positive family history, hypertension, and/or vascular disease. The other choices are not recommended by the ADA for screening the general population for diabetes mellitus.

The mechanism of sumatriptan in migraine involves what neurotransmitter? Answer Choices 1 Serotonin (5HT1) 2 Serotonin (5HT2) 3 Norepinephrine 4 Dopamine 5 Substance P

serotonin (5HT1) The headache and associative symptoms of migraine are postulated to result from the spontaneous activation of the trigeminovascular system. The peripheral fibers of the trigeminal nerve (ophthalmic division) supply innervation to the supratentorial meninges (dura mater, pia mater) and the intracranial blood vessels (arteries, veins, and venous sinuses). When activated, the peripheral axons of these bipolar neurons release vasoactive peptides (e.g. substance P, calcitonin gene-related protein, prostaglandins) adjacent to craniovascular structures, resulting in sterile inflammation, vasodilation, and pain. Central stimulation of the trigeminovascular system disrupts activity in the brainstem and hypothalamus, which is likely responsible for the associated nausea, vomiting, photophobia, and phonophobia of migraine. Treatment of migraine can be abortive or prophylactic. The trigeminovascular neurons are inhibited by serotonin 5HT1D- receptors, which are located both centrally and peripherally. Activation of these receptors aborts acute migraine headache and associative symptoms. Sumatriptan and dihydroergotamine (DHE) are specific 5HT1D- agonists used as abortive agents for migraine. Other medications used for acute migraine include acetaminophen, aspirin, naproxen sodium, ketorolac, caffeine, and butalbital. These drugs presumably act by reducing inflammation and promoting vasoconstriction within and around cranial blood vessels. Migraine prophylaxis is appropriate in patients who have frequent attacks (more than 3 per month) that interfere with social and occupational functioning. Prophylactic agents include tricyclic antidepressants (e.g. amitriptyline), β-blockers (e.g. propranolol, atenolol, nadolol), verapamil, and valproic acid.

A 25-year-old female presents for an ultrasound after having a positive home pregnancy test. She has an unremarkable past medical history and physical exam. She states she has been feeling fine without any abdominal discomfort or vaginal bleeding noted. On ultrasound you determine she is 10 weeks pregnant. You note a noncomplex unilateral mass on her left ovary measuring 2 cm in diameter. Which additional history would support your suspected diagnosis? A History of herpes simplex virus B She used clomiphene to conceive C Two previous Caesarian sections D 20-pound unintentional weight loss

she used clomiphene to conceive B Patients who used assisted reproduction, such as clomiphene, present a special subgroup as their ovaries frequently have ovarian cysts. This is common during the first trimester due to ovarian hyperstimulation. Herpes (A), Cesarean sections (C), and miscarriages (E) are not proven to increase the risk of functional ovarian cysts. A 20-pound unintentional weight loss (D) would have you consider a malignant cause, which is much rarer than a functional ovarian cyst.

An extremely heavy 12-year-old girl comes to the practice with her grandmother for new patient evaluation, bringing old records with her. Her blood pressure today is mildly elevated. Which of the following parameters will help determine whether her overweight and elevated blood pressure are due to Cushing syndrome (adrenocortical hyperfunction) rather than exogenous obesity? A advanced skeletal maturity B heavy thighs and legs C pinkish striae D short stature E slightly increased growth rate

short stature Children with Cushing syndrome typically have short stature, while those who are obese due to exogenous factors have normal or tall stature. Likewise, they tend to have delayed skeletal maturity (A), truncal obesity with thin extremities (B), purplish striae (C), and a slowed growth rate (E), while obese children have advanced maturation, heavy extremities, pinkish striae, and an increased growth rate.

In Western society, diverticulosis most often occurs in which portion of the colon? A transverse B sigmoid C descending D ascending E equally common in all parts of the colon

sigmoid Diverticulosis may arise anywhere in the large intestine, from the cecum to the end of the sigmoid colon. In Western societies, diverticula most often occur in the sigmoid colon where there is greatest intraluminal pressure.

Consideration should be given to screening patients with type 1 diabetes mellitus should also be screened for which of the following: A sarcoidosis B Sheehan's syndrome C Sjögren's Syndrome D thyroid disease

sjogren syndrome

A 32-year-old pregnant woman presents for a screening ultrasound at 16 weeks gestation. The ultrasound notes that the fetus is male, has shortened long bones (all below the 5%), and a large head circumference. Question You refer the patient to a tertiary care center for further evaluation and explain that you suspect that the fetus may have what condition? Answer Choices 1 Tiisomy 13 2 Trisomy 18 3 Trisomy 21 4 Skeletal dysplasia 5 45, X

skeletal dysplasia Explanation In this case, there are signs of a skeletal dysplasia where bony growth of the long bones of the skeleton is not normal. Increased head circumference size can also be observed in the skeletal dysplasias. While achondroplasia is the most common recognized skeletal dysplasia, over 100 disorders have been reported and additional ultrasound imaging and possibly genetic testing will be needed to make a specific diagnosis. Trisomy 13 is often fatal in utero or in the first few days or weeks of life. Global growth retardation (not just long bone shortening), renal anomalies, cleft lip/palate, and severe central nervous system malformations may be noted on ultrasound. Trisomy 18 is often fatal in utero or in the first few days or weeks of life. Growth retardation may be present, along with renal anomalies, severe cardiac malformations, and omphaloceles. Skeletally, there may be shortening of the sternum and prominent calcanei on the feet. Trisomy 21 sometimes leads to shortening of the long bones in utero. although head circumference is often decreased. Ultrasound markers would include the presence of congenital heart disease (1/3 of all live born cases), duodenal atresia, and increased nuchal translucency. 45 X, or Turner syndrome, has few major structural problems in fetal development in spite of the fact that many 45 X fetuses will spontaneously miscarry. Ultrasound imaging may identify a short neck, increased nuchal translucency (or even a cystic hygroma), and/or webbing of the neck. Additionally, the phenotype of the fetus in this problem is male, and the phenotype of 45 X is female.

A surgery resident moves into a new apartment. His new roommate is a healthy 28-year-old man. With some embarrassment, the roommate asks for medical advice for a problem he has not yet discussed with any doctor. He reports that he is unable to urinate in a public urinal; however, he experiences no difficulty at home. What is the most likely diagnosis? Answer Choices 1 Agoraphobia 2 Generalized anxiety disorder 3 Obsessive-compulsive disorder 4 Benign prostatic hypertrophy 5 Social phobia 6 Stress reaction

social phobia A phobia is an intense, irrational and persistent fear of a specific object, activity, or situation. Social phobia involves such a fear of being embarrassed or humiliated while in social situations. Many men are uncomfortable urinating in public restrooms because of the lack of privacy. This rarely manifests as anything more than a mild apprehension; however, the degree of apprehension in this patient is obviously very high, and it qualifies the situation as a social phobia. Agoraphobia is a fear of open spaces. In generalized anxiety disorder, anxiety dominates the clinical symptoms. In obsessive-compulsive disorder, patients experience recurrent, intrusive thoughts (obsessions) that are relieved to some extent by engaging in repetitive, ritualistic behaviors (compulsions). In the case of benign prostatic hypertrophy, hesitancy and inability to urinate does not improve in the absence of an audience. A stress disorder rarely manifests in such a manner. There are no other symptoms related to stress.

A 55-year-old man, a smoker with a 30-pack-year history of cigarette smoking, presents with a 3-month history of cough productive of blood and sputum. The patient admits a weight loss of 25 pounds over the past year. The patient also complains of diffuse 'bone' pain, abdominal pain, polydipsia, polyuria, and anxiety over the past month; all are unusual symptoms for him. The patient denies recent travel. The patient was treated in the emergency room twice over the past 2 months for nephrolithiasis. On chest radiography, a 3cm cavitary lesion with an air-fluid level is noted centrally in the left upper lung field; according to the radiologist it is considered suspicious for a pulmonary abscess. The patient also exhibits clubbing on physical examination. The patient's PPD is negative, and there is no history of anti-tuberculosis medication use in this patient's history. The patient's laboratory values are as follows: Calcium: 13.2 (Normal: 8.5-10.8 mg/dl) Phosphorus: 1.8 (Normal: 2.5-4.5 mg/dl) Urine Cyclic Adenosine Monophosphate (cAMP): 9.2 nmol/mL (Normal: 1.6-6.2 nmol/mL) Question What is the most likely diagnosis? Answer Choices 1 Reactivation of Latent Tuberculosis Infection 2 Small Cell Carcinoma 3 Squamous Cell Carcinoma 4 Bronchoalveolar Cell Carcinoma 5 Large Cell Carcinoma

squamous cell Squamous cell carcinoma is the most likely diagnosis in this patient with signs and symptoms of hypercalcemia; it is a notorious paraneoplastic syndrome in pulmonary squamous cell carcinoma due to the secretion of parathyroid hormone related peptide (PTHRP). The patient also has a strong history of cigarette smoking, clubbing on physical examination, and a centrally-located cavitary lung lesion that resembles a pulmonary abscess; all findings suggest squamous cell carcinoma. This question combines clinical features of both hyperparathroidism and squamous cell carcinoma. Hyperparathyroidism is a syndrome of hypercalcemia resulting from excessive release of parathyroid hormone and in most cases is due to adenoma in a single parathyroid gland. Parathyroid gland hypertrophy accounts for the remaining cases of hyperparathyroidism, and paraneoplastic syndromes such as those associated with squamous cell carcinoma are indeed rare. The symptoms of hyperparathyroidism are simple if you remember the mnemonic: "Painful bones (and tenderness), renal stones (nephrolithiasis), abdominal groans (abdominal pain), and psychic moans (changes in mental status)." This patient has abdominal pain, bone pain, a history of nephrolithiasis, and a recent change in mental states (new onset anxiety and depression). Combine these features with a high serum calcium, low serum phosphate, and high urinary cAMP (PTH exerts its effects though the second messenger cyclic AMP (cAMP)), and the diagnosis of hyperparathyroidism is assured. Bronchoalveolar cell carcinoma is a subtype of adenocarcinoma, and although an association with smoking has not been established, a substantial percentage of patients have a significant smoking history. The incidence of bronchoalveolar cell carcinoma is increased in patients who have underlying interstitial lung disease, parenchymal scaring, and exogenous lipoid pneumonia. Bronchoalveolar carcinoma may appear as a solitary pulmonary nodule, multiple nodules, or consolidation. Large cell pulmonary carcinomas account for only a small percentage of bronchogenic carcinomas and are also associated with cigarette smoking. The lesion occurs peripherally and grows rapidly, with early metastases and a poor outcome. Small cell carcinomas also arise centrally in peribronchial locations and infiltrate the bronchial submucosa. Small cell carcinoma of the lung (Oat Cell Carcinoma) is strongly associated with cigarette smoking. Widespread metastases occur early in the course of the disease, with common spread to mediastinal lymph nodes, liver, bones, adrenal glands, and brain. Also, several paraneoplastic syndromes are associated with oat cell carcinoma secondary to oncogenic cell production of peptide hormones. The most common paraneoplastic syndromes are the syndrome of inappropriate secretion of antidiuretic hormone (SIADH) and the syndrome of ectopic adrenocorticotropic hormone (ACTH) production, which can lead to Cushing Disease.

A 35-year-old woman presents with vomiting and watery diarrhea that began 4 hours after eating a chicken sandwich at her workplace cafeteria. She reports that 2 other workmates have also developed similar symptoms. There are no leukocytes or cysts on examination of her stool. Question What is the causative organism? Answer Choices 1 Staphylococcus aureus 2 Clostridium perfringens 3 Escherichia coli serotype O157:H7 4 Giardia lamblia 5 Salmonella typhi

staph aureus In cases of Staphylococcus aureus food poisoning, patients present with severe nausea and vomiting. They may also develop diarrhea, abdominal pain, and occasionally headache and fever. Stools are watery but not blood stained. The onset of symptoms is 1 to 6 hours after ingesting the contaminated food. Common culprits include processed meat, dairy products, and potato salads, especially when prepared by food handlers with skin infections that are left at room temperature. Several persons may be similarly affected. There are no fecal leukocytes or trophozoites on examination of the stool. Management includes intravenous replacement of fluid and electrolytes. In cases of Clostridium perfringens, food poisoning, patients present with watery diarrhea 8 to 24 hours after ingesting contaminated food like inadequately cooked meat or legumes. No fecal leukocytes or trophozoites are present on stool examination. In cases of Escherichia coli serotype O157:H7 food poisoning, patientspresent with watery diarrhea, which can become bloodstained. Sources of infection include undercooked beef. A stool examination reveals polymorphonuclear leukocytes. Giardia lamblia can be acquired by drinking contaminated water. Patients present with watery diarrhea. The incubation period is around 1 week. There are trophozoites in the stool, but no leukocytes. Patients with Salmonella typhi food poisoning usually present with bloody diarrhea. Sources of infection include beef, poultry, eggs, and dairy products. Examination of the stool reveals mononuclear leukocytes.

An 11-year-old boy presents after waking with a new onset of right hemiparesis. He is also experiencing slurring of speech, double vision, numbness of the face, dizziness, neck pain, and headache. His cognition is normal. He had been playing hockey on the evening prior to presentation and was struck by a puck on his neck. He has no significant past medical history. On physical examination, blood pressure is 110/60 mmHg, pulse 72/minute, and respirations 16/minute. His heart sounds and breath sounds are normal. The power in the left upper and lower limbs is 3/5, with sensory loss on the left side of the face and arms. What caused the boy's hemiparesis? Answer Choices 1 Brachial plexus injury 2 Cervical cord injury 3 Conversion reaction 4 Stroke due to carotid dissection 5 Transverse myelitis

stroke due to carotid dissection Dissection of the craniocervical arteries is the most common non-atherosclerotic cause of strokes in young people. He probably has experienced a stroke due to carotid dissection. Blunt trauma to the neck, such as being struck with a hockey puck, can cause clotting at the site of the dissection, resulting in an embolic stroke in the area perfused by the middle cerebral artery. Carotid angiography in such patients reveals internal carotid arterial occlusion. This type of stroke is not uncommon in childhood, and it may also occur following intraoral trauma, chiropractic manipulation, and cervical spinal injury following motor vehicle accidents. It usually presents with neurological deficits preceded by headache and neck pain. The patient may present with dizziness, dysarthria, diplopia, nystagmus, numbness, and features of Horner's syndrome. Diagnosis is by imaging methods such as Magnetic Resonance Imaging (MRI), Computed Tomography (CT), or Magnetic Resonance Angiography (MRA). Initial treatment is with anticoagulants. An injury of the cervical cord should cause bilateral symptoms and possibly respiratory compromise, rather than hemiparesis and sensory loss over the face and arms. A lesion of the brachial plexus should not cause weakness of the leg. Transverse myelitis usually causes bilateral symptoms, and a specific sensory level deficit is present. Conversion reaction is a psychopathological condition characterized by the presence of bodily symptoms that have no discernable physical cause. Conversion reaction is a diagnosis of exclusion and would not be the most likely explanation of these findings. Another risk factor for stroke in childhood is sickle cell disease. The incidence of stroke among children who have sickle cell disease is estimated to be as high as 6%. Transfusion to decrease the percentage of hemoglobin S has been found to lower that risk significantly. Hemorrhagic stroke in childhood may be due to arteriovenous malformations, arterial aneurysm, or cavernomas. Underlying hematologic conditions, including coagulopathy (e.g. hemophilia) or thrombocytopenia (e.g. idiopathic thrombocytopenic purpura), also may be a cause. Strokes have also been associated with cocaine use or trauma. Despite careful and complete evaluation, the etiology in some cases remains cryptogenic.

A 6-week-old male infant presents with a 4-day history of cough and nasal congestion. According to his mother, he occasionally has a bluish tint around his lips while sleeping. There is no history of fever. His older siblings have an upper respiratory infection. The patient's appetite has been decreased somewhat, mostly due to the copious nasal secretions; however, he has been maintaining a normal urine output. He was delivered pre-term at approximately 34 weeks gestation. He had mild respiratory distress syndrome, spending 2 days on a ventilator in the neonatal intensive care unit (NICU). He went home in 10 days and has done well since. He has had no immunizations. A physical exam reveals an infant in mild respiratory distress, respirations of 52/min, with slight intercostal retractions. Temperature is 100.2 F with a HR of 130/min. Perioral duskiness is seen. Oxygen saturation at room air is 83%, and HEENT exam is otherwise normal. His chest exam shows coarse rhonchi and expiratory wheezes. Heart rate and rhythm are regular. No murmurs appreciated. Abdomen is soft and non-tender. Neurological is intact. Chest X-ray shows mild hyper expansion, but no consolidation. Nasal swab for respiratory syncytial virus (RSV) is positive. Question What is the most appropriate treatment? Answer Choices 1 Aerosolized albuterol 2 Intravenous steroids 3 Empiric ampicillin and cefotaxime 4 Aerosolized ribavirin 5 Supplemental oxygen and supportive therapy

supp O2 and supportive treatment Respiratory syncytial virus (RSV) is the most important etiology of viral lower respiratory disease in infants and children; bronchiolitis is the most common manifestation. Infection is initiated in the upper respiratory tract and may spread to the lower tract; this causes obstruction of smaller airways by edema, necrotic tissue, and inflammatory cells. This leads to the typical findings of nasal discharge, cough, retractions, and wheezing or rhonchi. The peak incidence is during the first 2 years of life. In the United States, it occurs during the winter months in an epidemic fashion. RSV is a member of the Paramyxovirus family. Complications include apnea in the very young and premature infant, pneumonia, croup, respiratory failure, otitis media, and dehydration. The severity of bronchiolitis is diagnosed and assessed based on history and physical examination. Infants with mild bronchiolitis can be treated symptomatically at home. The following are the criteria for hospitalization in RSV bronchiolitis: Age less than 12 weeks Gestational age at birth of less than 34 weeks Cardiopulmonary disease or immunodeficiencies Wheezing and respiratory distress associated with oxygen saturation below 92 percent on room air (hypoxemia) History of significant apnea before assessment. The management in moderate-to-severe bronchiolitis involves supplemental oxygen therapy and supportive measures such as prevention of dehydration and respiratory support. Supplemental oxygen is the single most useful therapy, usually delivered via nasal prongs when oxygen saturation (SpO2) falls persistently below 90% in previously healthy infants. Mechanical ventilation may be needed for respiratory failure or severe apnea. Ribavirin, an antiviral agent, has not been shown to reduce hospital stays or mortality. The agent is very expensive and may be teratogenic in humans. It may still have a place for treatment of infants at significant risk for complications (i.e., those with cardiopulmonary disease). Bronchodilators, such as albuterol, have not been shown to be effective; however, their empirical use in the setting of the hospitalized patient is still commonly seen. Likewise, corticosteroids are not effective, and their use is contraindicated. Antibiotics, such as ampicillin and cefotaxime, are not routinely used in the majority of patients, but they are indicated in cases of clinical evidence of superimposed bacterial pneumonia or acquired nosocomial infection during the hospitalization.

A 9-year-old girl developed erythema of the cheeks; subsequently, an erythematous maculopapular rash started spreading from her arms to her trunk and legs. She complains of itching. Her medical history is non-contributory, and the results of physical examination are otherwise unremarkable. What is the appropriate therapy? Answer Choices 1 Tetracycline 2 Vidarabine or acyclovir 3 Orally administered penicillin 4 Chloramphenicol or tetracycline 5 Supportive antipruritics

supportive antipruritics The pattern of rash progress indicates erythema infectiosum, a mild, self-limited systemic disease which is caused by a human parvovirus. The usual therapy consists only of supportive measures, such as antipruritics, to relieve itching. Antibiotics are irrelevant, and the mild, self-limited nature of the disease makes antiviral therapy unnecessary in immunocompetent patients.

A 62-year-old obese woman presents due to urine leakage. She has had some symptoms for about 2 years, but they are getting worse. She leaks urine when she coughs or sneezes. She is not very active, but if she jumps, she also leaks urine. She wears a pad daily, as she has leakage daily. The amount varies from a few drops to a gush. The patient denies hematuria, dysuria, and pelvic pain. She sometimes feels vaginal pressure and fullness. She is considering quitting her job because she is embarrassed to be in public when she has urine leakage. Her past medical history reveals she is menopausal; she has had 4 vaginal deliveries. She has no other known medical conditions; she has not had any surgeries; she takes no medications and has no allergies. She is married and works part-time at a call center; she denies the use of tobacco, alcohol, and drugs. On physical exam, she is obese, with an atrophic vulva/vagina. The pelvic examination reveals downward and forward rotation of the vaginal wall, with an anterior bulging when the patient is asked to strain. The remainder of her exam is normal. A dipstick urinalysis is normal. What is the most appropriate intervention for this patient's current condition? Answer Choices 1 Antibiotic therapy 2 Daily cranberry juice or supplement 3 Start prescription oxybutynin 4 Surgical repair 5 Use over-the-counter phenazopyridine (Azo) as needed

surgical repair Explanation This patient is presenting with a cystocele, a herniation of the bladder wall into the vagina. Common symptoms may include stress urinary incontinence (SUI) and a feeling of vaginal fullness; other symptoms include incomplete voiding and dyspareunia. Of the choices listed, surgical repair is the most appropriate intervention. Other possible approaches to cystocele treatment include pelvic floor physical therapy and use of a pessary. The type of surgical approach to cystocele varies, from a less-invasive mesh sling to a more invasive colpopexy. Antibiotic therapy would be appropriate if this patient's urinary symptoms were due to a cystitis, or urinary tract infection (UTI). Common signs and symptoms of a UTI include urinary frequency and/or hesitancy, dysuria, and (possibly) gross hematuria; urinalysis shows white blood cells, red blood, cells and nitrites. Some women have both a cystocele and a UTI, so it is important to rule out infection when evaluating urinary tract problems. Daily cranberry juice and/or supplements are often used for urinary tract health. However, limited evidence supports cranberry's benefits related to UTI prevention. There is no evidence that cranberry juice or supplements is helpful for cystoceles and stress urinary incontinence. If this patient were suffering overactive bladder (detrusor instability) with resulting urge incontinence, it would be helpful to start prescription oxybutynin, an anticholinergic medication. However, this patient's problem is primarily anatomic, and most medications are ineffective in treating cystocele. If this patient was experiencing both stress and urge incontinence (mixed incontinence), the medication (along with surgery) would be indicated. Patients with frequent UTI may be instructed to use over-the-counter phenazopyridine (Azo) as needed. The phenazopyridine helps with bladder pain and has mild bacteriostatic properties. However, it does not have a role in cystocele and SUI. This medication also turns the urine bright orange, and it can interfere with urinalysis dipstick interpretation.

A 15-year-old adolescent boy is seen in your office at 11:30 A.M. with a complaint of left scrotal pain and swelling; it started at 7 A.M. that same day, which is when he woke up. He recalls no trauma; when questioned, he says that he has never had intercourse. He has been feeling nauseated, and he vomited once. Physical examination demonstrates a well-nourished, well-developed boy, appearing moderately uncomfortable. Vital signs are normal. with the exception of a temperature of 37.9 degrees centigrade orally. Pain assessment score (Wong-Baker scale) is 6/10. He is Tanner Stage III puberty. The remaining physical examination is normal, except for the following findings: The left testicle is approximately 1.5 times the size of the right testicle. The skin is diffusely erythematous. Due to tenderness when touched, it is difficult to palpate the scrotum. Cremasteric reflex is absent. There are small, soft, pea-sized lymph nodes in both inguinal areas. Penis is circumcised and appears normal. Scrotal ultrasonography with Doppler ultrasound demonstrates decreased blood flow to the testis. Question What is the most likely diagnosis? Answer Choices 1 Acute idiopathic scrotal edema 2 Epididymo-orchitis 3 Testicular torsion 4 Torsion of testicular appendage 5 Varicocele

testicular torsion Explanation Testicular torsion in the adolescent boy is a urologic emergency, the most common cause of acute scrotal swelling and pain, and the most common cause of testicular loss. Torsion occurs in 1:4000 and occurs most commonly on the left side in the United States. The cause is a congenital anomaly that occurs in approximately 12% of boys/men, in which the tunica vaginalis is attached too high, allowing the testicle to rotate freely on the spermatic cord and vascular pedicle in the tunica vaginalis. Approximately 40% of boys/men have the anomaly bilaterally. Testicular torsion usually occurs between 12 and 18 years of age with the peak age of 14. It may occur up to 30 years of age, and it is found in infants and occasionally neonates at the time of birth. Up to 50% of patients may have had prior episodes of mild intermittent testicular pain that has resolved spontaneously, due to intermittent torsion and spontaneous derotation. Associated symptoms may include nausea and vomiting (20%), fever (16%), abdominal pain (20 - 30%), and urinary frequency (4%). Physical examination may demonstrate a horizontal position of the testis, and it may be elevated compared to the uninvolved side. The cremasteric reflex is usually absent, but its presence does not rule out testicular torsion. Elevation of the scrotum does not relieve the pain. The diagnosis is a clinical diagnosis. Because it consumes precious time, ultrasound examination of the testis with color flow Doppler should only be ordered when the diagnosis is uncertain and can determine if there is blood flow to the testis. The studies are 86% sensitive and 100% specific in making the diagnosis if the only criterion is decreased blood flow. Radionuclide scans are 90 - 100% accurate in identifying decreased blood flow. Rapid diagnosis is critical; if surgical intervention is provided within 6 hours of onset, the salvage rate for the testis is 80 - 100%; after 6 hours, the salvage rate is approximately 0%. Acute idiopathic scrotal edema is uncommon, but presents acutely with the average age of presentation 6 years. 90% of patients have a unilateral presentation. The scrotal skin is red and tender, but the testis appears to be normal. The redness tends to extend off the scrotum onto the perineum or onto the penis. This tends to resolve spontaneously in 48-72 hours and leaves no sequelae. Doppler ultrasound, if done, demonstrates good blood flow to the testis with peritesticular edema and fluid in the scrotal wall. Laboratory examination is normal except for occasional eosinophilia. Acute epididymitis and/or orchitis is not a common pediatric diagnosis. It was first described in 1956. The onset tends to be more gradual, generally over a few days, with fever and dysuria. Elevation of the scrotum may reduce discomfort. The cause may be viral, such as adenovirus, mumps, or Epstein-Barr virus, or bacterial. Bacterial infection is often associated with structural changes in the urinary tract. Urinalysis and urine culture may be helpful in establishing the diagnosis. Typical treatment is with rest, analgesia, and antibiotics if there is concern about a bacterial etiology. If a bacterial cause is identified, urinary tract imaging should be performed. There have been rare reports of acute epididymitis progressing to testicular infarction. Torsion of the appendix testis may present similarly to testicular torsion. Tenderness is usually localized to the upper portion of the testis and, typically, a blue dot is seen on the scrotal skin resulting from the venous congestion in the appendix testis. This is a self-limited condition and does not require surgical intervention. There are 5 appendages to the testis, all of which serve no function. If one twists or infarcts, symptoms result. Pain is less intense than with testicular torsion, and the cremasteric reflex is usually present. Varicocele occurs in 10 - 15% of males, 16% of adolescents, and 20 - 40% of men evaluated for infertility. First described in adolescents in 1885, the most common age of presentation is adolescence and early adulthood. They are caused by incompetent or absent valves of the spermatic veins, resulting in dilatation of the veins of the pampiniform plexus. Rarely are they caused by compression of the renal vein by a tumor, an aberrant renal artery, an obstructed renal vein. Doppler ultrasonography can demonstrate retrograde blood flow. They are most common on the left side, are usually asymptomatic, but may present with vague scrotal discomfort and swelling. Of those with symptoms, 2% have intratesticular Varicocele and these are more common on the right side. The typical physical finding is the bag of worms within the scrotal sac. They may be missed on physical examination if the supine position, so the patient should be examined in a standing position. Patients should be referred to urologists for further evaluation and to discuss options for treatment which sometimes, but not always, requires surgery.

A 53-year-old man is taking a proton pump inhibitor for GERD symptoms, a beta blocker and a thiazide diuretic for hypertension, an SSRI for depression, and an over-the-counter NSAID as needed for aches and pains. He has developed gynecomastia and laboratory studies reveal an elevated prolactin level. If his hyperprolactinemai is due to one of his medications, which is the most likely cause? A the beta blocker B the NSAID C the proton pump inhibitor D the SSRI E the thiazide diuretic

the SSRI Many medications cause hyperprolactinemia, including SSRIs, tricyclic antidepressants, and antipsychotics. Hydralazine and methyldopa, but not beta blockers (A), may also raise prolactin levels; likewise opioids, but not NSAIDS (B). Cimetidine and ranitidine, but not proton pump inhibitors (C) are included among possible pharmaceutical causes. Thiazide diuretics (E) are not know to raise prolactin levels.

A 67-year-old man presents with a 3-week history of increasing shortness of breath; it occurs even while he is at rest. The patient was diagnosed with congestive heart failure in the past year, and he has been well controlled on oral medication. He has no history of tobacco use. He has gained 10 pounds since his last exam 2 months prior to presentation. On physical exam, there are diminished breath sounds and decreased tactile fremitus bilaterally at the base of the lungs. Dullness to percussion is also noted in the same area. He has 3+ bilateral pitting lower extremity edema. Question Based on the patient's physical exam and history, what is the most likely diagnosis? Answer Choices 1 Lung malignancy 2 Tuberculosis 3 Empyema 4 Spontaneous pneumothorax 5 Pleural effusion

the correct answer is pleural effusion. This patient has a history of congestive heart failure, which is one of the most common causes of transudative pleural effusions. Pleural effusions in the setting of heart failure are usually right-sided or bilateral; in this patient's case, physical exam findings point to bilateral pleural effusion. Dyspnea is a common symptom seen in patients with a pleural effusion. Based on his pitting edema on exam, it is clear that this patient has uncontrolled heart failure. The physical exam findings are also common with pleural effusion.

What tissue is lined with simple cuboidal epithelium? Answer Choices 1 Small intestine 2 Trachea 3 Lung alveoli 4 The distal convoluted renal tubules 5 The renal corpuscle

the distal convoluted renal tubules The correct response is the distal convoluted renal tubules. The small intestine is lined with simple columnar epithelium. The trachea is lined with pseudostratified ciliated epithelium. Both lung alveoli and the renal corpuscle (Bowman's capsule) are lined with simple squamous epithelium.

A 67-year-old man with an 80-pack/year smoking history and chronic bronchitis presents 30 minutes after suffering what was described as a "seizure" by his wife. According to his wife, this was his first seizure. During the seizure episode, the patient did not lose consciousness, but was observed to have developed an "involuntary jerking in his right arm and leg" that lasted about 5 minutes, with right arm weakness immediately following this involuntary activity. On physical exam, the patient appears drowsy, confused, and answers questions slowly. His right arm has objective weakness rated as 4/5; the remainder of the neurological exam is normal. Question Ico-delete Highlights What aspect of this patient's presentation most closely identifies this event as a simple focal seizure? Answer Choices 1 Involuntary extremity jerking 2 Right arm weakness is noted 3 There was no impairment of consciousness 4 There is postictal confusion 5 First seizure as an adult

there was no impairment of consciousness

A 50-year-old man presents with a 3-week history of a burning sensation in his feet. He also reports that he has difficulty rising from a sitting position. He is homeless and admits to drinking a lot of alcohol everyday. On examination, you note a decrease in the vibratory sensation in the toes, and the ankle jerk reflex is absent. Question What is the most likely diagnosis? Answer Choices 1 Thiamine deficiency 2 Riboflavin deficiency 3 Vitamin K deficiency 4 Vitamin D deficiency 5 Iron deficiency

thiamine def Vitamin B1 (or thiamine) deficiency results in beriberi, which is characterized by: A bilateral symmetric peripheral neuropathy beginning in the legs. Patients may present with burning sensation in the feet and difficulty rising from a squatting position. On examination, there may be decreased vibratory and position sensation in the toes. Ankle jerk and knee jerk reflexes may be absent. Wernicke-Korsakoff syndrome, which is comprised of nystagmus, ophthalmoplegia, ataxia, memory loss, and confabulation. Congestive heart failure with tachycardia, peripheral edema, and cardiomegaly. Causes include inadequate dietary intake, especially from a diet based on polished rice and alcoholism. Dietary sources rich in vitamin B1 include yeast, legumes, liver, whole grains, and enriched cereal products. In cases of riboflavin (or Vitamin B2) deficiency, patients present with angular stomatitis and cheilosis. On examination, they are pale, have atrophic glossitis, and the tongue may appear magenta. In cases of vitamin K deficiency, patients present with bleeding tendencies, which include epistaxis, menorrhagia, and hematuria. The prothrombin time (PT) and the activated partial thromboplastin time (aPTT) are usually prolonged. In cases of vitamin D deficiency, children can present with inability to walk unsupported due to muscle weakness and lower limb skeletal deformities, such as genu varum and genu valgum. They may also have prominent costochondral junctions. In cases of iron deficiency, patients can present with feeling weak, dizzy, and tired; they may experience syncope. On examination, they have pale conjunctivae and koilonychia.

A 55-year-old man is found on routine X-ray to have a growing lesion in the right middle lung field. He is a heavy smoker. The physical examination is within normal limits. Tuberculin test is negative. What is the management of choice? Answer Choices 1 Thoracotomy 2 Antibiotics 3 Reassurance 4 Observation 5 Chemotherapy

thoracotomy Because of the high percentage of carcinoma in heavy smokers, the excision of a growing lesion is indicated. The small mass in the lung is often discovered on X-ray examination and without any pulmonary symptoms. The patient's history of smoking suggests that the carcinoma is most likely a presenting complaint; as a result, lobectomy or excision of the growing lesions is indicated.

A 28-year-old woman presents with nervousness and palpitations associated with heat intolerance. On examination, there is no evidence of thyromegaly, but there is a palpable nodule that is "hot" on a thyroid scan. The TSH was low and T3 and T4 were both elevated. Which of the following is the recommended treatment for this patient? A Propylthiouracil (PTU) B Thyroid lobectomy C Total thyroidectomy D Radioiodine ablation

thyroid lobectomy In Graves' disease, the thyroid is diffusely enlarged in contrast to a toxic adenoma in which the thyroid is normal sized but with a palpable nodule. Surgery is the treatment of choice for a toxic adenoma. Surgical treatment of a toxic adenoma is a thyroid lobectomy and isthmusectomy. A subtotal or total thyroidectomy is indicated for toxic multinodular goiters or Plummer disease. Thionamides and radioiodine ablation are not effective therapies for toxic adenomas. (Coe, 2006, pp. 404-406) Coe NPW. Surgical endocrinology: thyroid gland.

Following a total thyroidectomy for papillary carcinoma, a 72-year-old man develops a heart rate of 140 and a temperature of 104.8F. He vomits almost continuously and has severe diarrhea. He is disoriented and mildly combative. Electrocardiography demonstrates sinus tachycardia. What is the most likely diagnosis? A pulmonary embolism B pneumonia C sepsis D thyroid storm E wound infection

thyroid storm Although rare, thyroid storm or crisis can occur following thyroid surgery, administration of radioactive iodine, or a stressful illness. It is characterized by extreme tachycardia, vomiting, diarrhea, dehydration, delirium, and high fever. Pulmonary embolism (A) may include tachycardia, along with chest pain and shortness of breath, but is unlikely to include GI symptoms. Pneumonia (B) and sepsis (C) are certainly possible, although the extreme nature of her findings more strongly suggest thyrotoxicosis. Wound infection (E) would more likely to present with localized tenderness and a less marked fever.

Excessive doses of some vitamins can cause toxicity. For a certain vitamin, early manifestations of chronic toxicity include dermatological findings like dry scaly skin and hair loss. Anorexia, vomiting, and hyperostosis may also be seen. What hypervitaminosis can cause these symptoms? Answer Choices 1 Hypervitaminosis A 2 Hypervitaminosis B6 3 Hypervitaminosis C 4 Hypervitaminosis E 5 Hypervitaminosis B3

too much vit A Very large doses of the fat-soluble vitamins are definitely toxic. Acute vitamin A intoxication was first described by Arctic explorers, who developed headache, diarrhea, and dizziness after eating polar bear liver. The liver of this animal is particularly rich in vitamin A. Nausea, vomiting, papilledema, and lethargy may be seen in acute toxicity. Chronic toxicity can occur when a person consumes 50,000 units per day for 3 months or more. Mouth sores may also be seen as an early manifestation along with those mentioned in the question. Hypercalcemia, increased intracranial pressure with papilledema, and decreased cognition may be seen in more serious cases. Hepatomegaly leading to cirrhosis may occasionally occur. Excessive vitamin A has also recently been related to increased risk of hip fracture. Elevated vitamin A in the blood is the only clue to the diagnosis. Elimination of vitamin A from the diet usually results in rapid recovery.

A 35-year-old man presents for medical care, but he is too embarrassed to tell the nurse his chief symptom. Eventually, he admits to severe, intense itching around his anus; it has been worsening the last several weeks. He further states that he has noticed increasingly severe and tearing pain in the anal area with each bowel movement. He ranks this pain as a 10/10 on a pain scale. This intense pain makes him try to avoid having bowel movements regularly. He admits to 1 episode of a small amount of bright red blood on the toilet paper as well as on the stool itself. Reviewing documentation on this patient reveals that this is the 4th similar episode in the last 14 months. The patient denies fever and diarrhea; according to him, he has never been diagnosed with inflammatory bowel disease. Question Considering the most likely diagnosis, what pharmaceutical intervention should be initiated to treat and heal this pathology? Answer Choices 1 Topical lidocaine 5% 2 Topical prilocaine 2.5% 3 Topical nitroglycerin 0.2 - 0.4% 4 Topical bacitracin ointment 5 Topical hydrocortisone cream 1%

topical ntg 0.2%-0.4% the correct response is topical nitroglycerin 0.2 - 0.4%. The patient is likely suffering from an anal fissure. Anal fissures often affect infants as well as middle-aged individuals. The majority of fissures are considered primary, and they are caused by local trauma, such as passage of hard stool, prolonged diarrhea, vaginal delivery, or anal sex. They are described linear or rocket shaped ulcers that are usually < 5 mm in length. Common presentation of anal fissures include a tearing pain that accompany bowel movements; there is also bright red rectal bleeding that is limited to a small amount noted on the toilet paper or surface of the stool. The patient described all of these components. Patients will also note perianal pruritus or irritation, all of which the patient admits to experiencing. Because this patient has had a several incidences very similar to the one he is currently presenting with, this episode should be categorized as a chronic anal fissure. Once a fissure is branded as such, treatments considered will include topical 0.2 - 0.4% nitroglycerin, topical 2% diltiazem ointment, or even injection of botulinum toxin directly into the internal anal sphincter. All of these treatments listed result in healing a chronic fissure 50 - 80% of the time; 40% of cases can recur after treatment. Topical lidocaine 5% and lidocaine plus prilocaine 2.5% are topical anesthetics that are indicated fortemporary relief of what the patient is experiencing; however, they will not promote healing of the lesion. Topical bacitracin is not indicated for symptom relief or for healing of anal fissures. Topical hydrocortisone cream 1% may decrease inflammation of the lesion; however, it will not promote overall healing.

A 19-year-old woman presents with worsening headaches. She reports a multi-year history of episodic, throbbing headaches. They have intensified, and she now misses classes and work periodically due to her headaches. Recently, the headaches occur about 4-6 times per month, up from 1-2 per month when she first started experiencing headaches. Her headaches last 2-3 days duration and are accompanied by nausea, vomiting, and light sensitivity. After the headache resolves, she denies any residual symptoms. She denies neurologic symptoms, such as vision or taste changes, gait disturbances, and memory loss. She has tried multiple over-the-counter pain medications without relief. Her mother and maternal aunt experienced similar headaches. She reports some increased stressors and less sleep since starting college. Her past medical history is unremarkable, with no known medical conditions (except for the headaches), no surgeries, no chronic medications, and no drug allergies. She has never been sexually active and reports regular menses. She denies the use of tobacco, alcohol and drugs. On physical exam, the patient appears comfortable and reports no headache at this time. Her entire exam, including neurological, is normal. Question In addition to treatment options for acute headache, this patient should be educated about which of the following daily medications to prevent headache episodes? Answer Choices 1 Cetirizine 2 Clopidogrel 3 Hydrocodone 4 Sumatriptan 5 Topiramate

topiramate This patient is suffering from migraine headaches, common headaches classically described as throbbing in nature and accompanied by nausea (and possibly vomiting), photophobia (sensitivity to light), phonophobia (sensitivity to sounds), and possibly visual changes and aura. Migraines are more common in women than men, but can occur in men. They are usually a chronic condition, beginning in adolescence or early adult years. Often a family history is present. Stress, sleep deprivation, and various foods can be among many triggers for migraines. With a classic history and no alarm features of a more serious condition, no testing would be necessary. Treatment approach includes offering treatment for acute headache and, with frequent migraine such as this patient's, prophylactic treatments, such as topiramate, propranolol, amitriptyline, valproic acid, and botulinum toxin type A. Topiramate augments GABA activity and has primarily been used for seizure disorders. It is approved and effective for prevention of migraine headaches.

A 30-year-old G1P0 woman who is 15 weeks pregnant undergoes "triple screening." The maternal alphafetoprotein (AFP), human chorionic gonadotropin (hCG), and unconjugated estriol (uE3) levels are all lower than normal. This suggests which of the following possible problems with the fetus? A cystic fibrosis B Down syndrome C homocystinuria D G6PD deficiency E trisomy 18

trisomy 18 The triple screen detects possible Down syndrome and trisomy 18. When the fetus has Down syndrome (B), the AFP and uE3 are low and the hCG is high, while trisomy 18 is suggested by low values in all three. A positive screen must be followed up by fetal karyotyping. Cystic fibrosis (A), homocystinuria (C), and G6PD deficiency (D) are not detected by the triple screen.

A 45-year-old man presents with significant weight loss (10 pounds in 4 months), cough with hemoptysis, and pleuritic chest pain. The chest X-ray shows ill-defined opacities in both the lungs; the opacities have a reticulonodular pattern. A transbronchial biopsy is performed, and it microscopically shows a few epithelioid cells with necrotic debris. What is the most likely diagnosis? Answer Choices 1 Aspergillosis 2 Squamous cell carcinoma 3 Pneumocystis carinii pneumonia 4 Oat cell carcinoma 5 Tuberculosis

tuberculosis The epithelioid cells suggest that it is a granuloma. The hemoptysis suggests that the granuloma has eroded enough parenchyma and involved a bronchus. A granulomatous infection along with hemoptysis is typical for tuberculosis. The history of significant weight loss and hemoptysis is also typical of tuberculosis, but it can be seen in other cases also. Aspergillosis is more likely to produce a fungus ball, which is defined on an X-ray. The inflammatory reaction would vary from acute to mixed to granulomatous. Squamous cell carcinoma is usually a central mass lesion. It is unlikely to produce epithelioid cells, which are seen in the biopsy of this patient. Pneumocystis carinii pneumonia has a granulomatous pattern in rare cases, but the granulomas are small and unlikely to produce hemoptysis. Oat cell (or small cell) carcinoma is centrally located and is unlikely to be found with epithelioid cells.

A 25-year-old woman presents with severe abdominal pain for the last few hours. The patient admits to pelvic pain and vagina discharge for over 1 month, but she refused to make an appointment with her provider to be evaluated. She last had unprotected intercourse with an ex-boyfriend 3 months ago. Her examination reveals a fever of 103oF and abdominally rigidity, with light palpation that is worst in the left lower quadrant. She does not tolerate a speculum exam. A pelvic ultrasound reveals a 4 x 5 cm left adnexal mass. Urine hCG is negative, WBC is elevated, with a left shift. What is the most likely diagnosis? Answer Choices 1 Appendicitis 2 Tubo-ovarian abscess 3 Ovarian cancer 4 Ectopic pregnancy 5 Diverticulitis

tubo-ovarian abscess Explanation This patient present with an acute abdomen that was preceded by pelvic pain and high-risk intercourse. Her history, physical examination, and diagnostic findings are all consistent with a tubo-ovarian abscess. In cases of appendicitis and diverticulitis, you would not expect to find an adnexal mass; the age and presentation of the patient are not consistent with ovarian cancer, and her negative hCG rules out pregnancy, including ectopic pregnancy.

Twins are at a higher risk for growth problems and prematurity, although uncommon defects unique to twins occur. Which of the following is the most common developmental defect in a monochorionic twin pregnancy? Answer Choices 1 Conjoined twins 2 Acardiac twins 3 Twin-twin transfusion 4 Twin embolization syndrome 5 Monoamniotic, monochorionic twins

twin-to-twin transfusion Explanation Twin-twin transfusion syndrome occurs in approximately 10-20% of monochorionic twins. Twin-twin transfusion syndrome (TTTS) is a condition that affects monochorionic twin pregnancies, in which blood is shunted from one twin to the co-twin through vascular anastomoses in the placenta. The donor twin becomes progressively anemic, hypotensive, and hypovolemic with resulting oligohydramnios and lagging growth. The recipient twin becomes polycythemic, hypertensive, and hypervolemic with resulting polyhydramnios and normal growth. The prognosis is fatal if left untreated, mortality rates approaching 70-90% in such cases. The sonographic findings suggestive of TTTS includes monochorionicity, discrepancy in amniotic fluid in the amniotic sacs with polyhydramnios of one twin (largest vertical pocket greater than 8 cm) and oligohydramnios of the other (largest vertical pocket less than 2 cm), discrepancy in size of the umbilical cords, cardiac dysfunction in the polyhydramniotic twin, and significant growth discordance (often > 20%). An acardiac parabolic twin (parasitic twin that fails to develop a head, arms and heart) occurs in approximately 1:35,000 pregnancies. Conjoined twins occur in 1:50,000 pregnancies. Monoamniotic, monochorionic twining is when the twins share one sac with both the amnion and chorion being a single sac. Twin embolization syndrome (TES) is a complication associated with monozygotic twins, following in utero demise of the co-twin.

A 42-year-old woman has a developed a solid and quite firm thyroid mass that is approximately 1 cm diameter by palpation. She does not have any hoarseness, difficulty breathing or swallowing, or symptoms of thyroid disease. She has never been exposed to radiation and has no history of cancer, nor does she have a family history of thyroid or other cancers. A thyroid panel is within normal limits. What is the next step in the care of this patient? A fine-needle aspiration biopsy of the nodule B observation to see if the nodule increases in size C radionuclide thyroid scan D resection of the nodule E ultrasonography of the neck

u.s of the neck

An 8-year-old child presents with acute fever. There is redness of the skin in the neck area. The thyroid gland is extremely tender to palpation. What diagnostic test could confirm the most likely diagnosis? Answer Choices 1 Serum T4 concentration 2 Serum TSH measurement 3 T4 and TSH measurements 4 24 hour uptake of radioactive iodine 5 Ultrasound of neck

ultrasound Explanation The symptoms are highly suggestive of acute suppurative thyroiditis. In that case, serum T4, TSH, and 24-hour uptake of radioactive iodine are normal. Ultrasound of the neck is a useful and non-invasive way to confirm the diagnosis. If an abscess is identified, ultrasound-guided needle aspiration can provide a sample for identification of the bacterial organism responsible for the infection. The most common organisms are Staphylococcus aureus and Streptococcus species.

A 47-year-old Hispanic woman presents with a 2-month history of persistent abdominal pains. She becomes uncomfortable after eating, especially after consuming fats, eggs, chocolate, fried foods, fatty foods, and rich desserts. Her pains are primarily located in her right upper abdominal quadrant and often radiate to her right shoulder blade. Physical examination is essentially unremarkable at this time. Her blood pressure is 122/82 mm Hg, she has a pulse of 72 per minute, and a temperature of 98.6°F with respirations 16 per minute. Her abdominal examination reveals no tenderness, no guard, no rebound, and normally active bowel sounds. Her liver and spleen are not palpable. She has no readily palpable abdominal masses. She is not jaundiced. She has no scleral icterus. Question What test should be given in order to confirm the probable diagnosis? Answer Choices 1 Oral cholecystogram 2 X-ray abdomen 3 Ultrasound abdomen 4 CT scan abdomen 5 Biliary scintigraphy 6 MRI abdomen

ultrasound of the abdomen Based on the patient's history, cholelithiasis is suspected. This condition is more common in females of Caucasian or Hispanic descent than in others. The incidence increases with age. Most (80%) of the stones in the US are the cholesterol type. Bilirubin, pigment, and calcium stones constitute the other 20%. Cholelithiasis may be asymptomatic (the presence of gallstones without symptoms), symptomatic (biliary colic), or complicated (e.g. cholecystitis, choledocholithiasis, cholangitis). Ultrasonography is a safe, reliable, and non-invasive test that can be performed at the bedside and is also safe in pregnant women. It is quite sensitive and specific for stones larger than 2 mm. Only about 10-30% gall stones are radio-opaque and hence abdominal X-ray may not be so useful. The oral cholecystogram is the X-ray taken after administering contrast medium to the patient. Although it can identify gall bladder pathology, it requires preparation and is not as convenient as the ultrasound. CT scan is not the first choice in suspected gallstones, although it may prove useful in intrahepatic stones. Biliary scintigraphy is of value in cystic duct obstruction. MRI is also an excellent choice but is expensive, requires sophisticated equipment, and is not used as the first choice investigation in most centers. Symptomatic stones are treated with cholecystectomy. Asymptomatic stones may also require surgery under special circumstances such as large (>2 cm) stones, those with spinal cord injuries affecting the abdomen, and in calcified gall bladder.

Your patient returns to your office for a follow up for non-insulin-dependent diabetes mellitus (NIDDM). Her HgA1c in the office is 6.4%. She is concerned about developing kidney disease from her diabetes and requests that you test her for this. What initial screening test should you order that would provide clues to potential diabetic nephropathy allowing for treatment to slow the disease progression? A 24-hour urine for protein B serum BUN/CR C urine microscopic D urine microalbumin E serum protein

urine microalbumin The correct answer is (D). An easy office dipstick or laboratory test for urine microalbumin should be done initially and periodically on diabetic patients who are at risk for diabetic nephropathy. Treatment should be initiated if microalbuminuria is found to slow disease progression. A urine microscopic for renal casts may be helpful if the patient has symptoms of kidney disease, but is not an initial screening test. Serum BUN/CR and GFR are useful tests for patients with known diabetic nephropathy to indicate the stage of chronic renal failure but is not elevated early in the disease progression, before urine microalbumin. A 24-hour protein is not indicated in this case as an initial screening test.

Generally, patients who are malingering: A use illness to attain a goal B have avoidant personalities C follow prescribed treatment regimens D have a history that agrees with their physical symptoms

use illness to attain a goal A Patients who are malingerers do not want to improve until their goal is met. Goals may be financial, occupational, or legal. These patients will act differently when they think they are not being observed. They may fake their symptoms in order to be admitted to a hospital or to obtain drugs. These patients have an antisocial personality disorder.

A 53-year-old man presents with increased difficulty swallowing and occasional regurgitation of his meals. His symptoms have been occurring with greater frequency and severity over the last 4 months. He also gets some shortness of breath, but attributes that to his weight and lack of physical activity. His past medical history is remarkable for chronic heartburn, which he treats intermittently with over the counter antacids. He takes no regular medications, and he has no allergies. He has not had any surgeries. He is smoker, but he denies use of alcohol and drugs. He works as a building inspector, and he lives with his wife and children. The patient is obese, but the rest of his physical exam is normal. Blood tests, electrocardiogram, and chest X-ray are done in the clinic; they are normal. He is referred for endoscopy, and esophageal biopsy shows specialized intestinal metaplastic cells (of columnar epithelium). Question What recommendation would best prevent complications from this patient's current condition? Answer Choices 1 Avoidance of antacids 2 Referral for fundoplication 3 Regular exercise 4 Use of proton pump inhibitors 5 Weight loss

use of ppi This patient is presenting with Barrett's esophagus, a type of chronic esophagitis in which the normal squamous epithelium is replaced with columnar epithelium. Barrett's esophagus is a complication of chronic gastroesophageal reflux disease (GERD) and can develop into esophageal adenocarcinoma. Use of proton pump inhibitors (PPIs) reduces the risk of cancer. In order to monitor for the development of cancer, a routine endoscopy should be periodically performed in patients with known Barrett's esophagus. While the antacids are not necessarily adequate to control the GERD and damage to this patient's esophagus, avoidance of antacids will have no role in prevent subsequent Barrett's esophagus complications, such as adenocarcinoma. While the acid reflux appears to be a major mechanism for damage to the esophageal tissue, anti-reflux surgeries do not appear to prevent complications from Barrett's esophagus and subsequent development of adenocarcinoma. This patient does not need referral for fundoplication (surgical 'wrapping' of the stomach around the esophagus). Regular exercise would be helpful for this patient in general terms for cardiovascular health and weight reduction. However, exercise does not play a significant role in reducing risks for Barrett's esophagus complications. Obesity is a risk factor for development of Barrett's esophagus. (Other risk factors include male gender, smoking, and especially chronic GERD.) Once Barrett's esophagus has developed, however, weight loss does not dramatically alter the risk of complications, such as adenocarcinoma. Weight loss should be recommended for any obese patient, but the PPIs are the best approach to reduce complications from his Barrett's esophagus.

A 20-year-old college football player presents with a chief complaint of a dull ache in his scrotum after prolonged standing on the sideline. It seems to get worse with vigorous activity and is relieved by lying down. Dilated veins in the left scrotum are observed on inspection, and both testicles are palpable and without masses. What is the most likely diagnosis? A varicocele B spermatocele C hydrocele D testicular mass

varicocele A A varicocele can be recognized by the presence of scrotal enlargement caused by dilation of the pampiniform venous plexus. Varicoceles present as a "bag of worms" in the spermatic cord and are more prominent when the patient stands. More than 80% of the time, varicoceles occur on the left side. Hydroceles and spermatoceles are caused by fluid collection and are usually asymptomatic. Testicular masses must always be included in the differential diagnosis of scrotal masses, as they generally present as painless.

A 10-year-old child is seen with his parents for a routine check up. During the review of symptoms, his parents mention that their son has been extremely thirsty and is going to the bathroom to urinate frequently. The patient agrees. The parents are concerned that their son has developed diabetes mellitus. The family history is negative for diabetes mellitus, but the mother has a history of familial hypothalamic diabetes insipidus. Screening blood work includes a CBC, hemoglobin A1c, and renal function tests, all of which are within the reference range. Which of the following serum analytes would you expect to be deficient? A Sodium B Glucose C Thyroxine D Prolactin E Vasopressin

vasopressin The correct choice is E, vasopressin. The reader must first understand that the patient's symptoms are classic for diabetes insipidus, with the increased thirst, frequency, and polyuria. These symptoms are also seen in diabetes mellitus, but this would be less likely since the patient doesn't have a family history of diabetes mellitus, and his hemoglobin A1c is not elevated. Choice A, sodium, should be within range, as long as the boy is able to drink when thirsty; otherwise, he would become hypernatremic. Choice B, glucose, is incorrect. Although the signs and symptoms also fit the pattern of diabetes mellitus, the glucose would most likely be elevated in this disorder, and not low. Choice C, thyroxine, is under control of TSH from the anterior pituitary gland, and choice D, prolactin, is secreted from the anterior pituitary gland. Vasopression is secreted from the posterior pituitary gland.

A 44-year-old man presents after an episode what is described as "passing out". He has a past medical history of hypertension, diabetes mellitus type II, and osteoarthritis. Earlier in the day, the patient visited his primary care physician for routine blood work. During his blood draw, the patient had an acute onset of syncope. The patient never had such an episode before. Vital signs upon presentation are as follows: BP 124/82 mmHg, P 88 beats/min, R14/min. EKG is within normal limits. Cardiovascular examination reveals a normal S1 and S2, with no rubs, murmurs, or gallops. The patient is being treated with atenolol and hydrochlorothiazide for his hypertension. His blood sugar is 96 mg/dl. Question What is the most likely cause of the patient's syncope? Answer Choices 1 Hypoglycemic episode 2 Arrhythmia 3 Neurogenic shock 4 Vasovagal episode 5 Orthostatic hypotension

vasovagal episode This patient is having syncope due to a vasovagal episode. This reaction is common in patients who are exposed to blood or are having their blood drawn. In fact, vasovagal episodes are the most common cause of syncope. These individuals typically present with loss of consciousness when exposed to a certain trigger. The trigger leads to an activation of the nucleus tract solitarius of the brainstem, which results in a surge of parasympathetic response, leading to the cardioinhibitory and vasodepressor effects. Although this patient has a history of diabetes mellitus, is not currently hypoglycemic. Hypoglycemia, if severe, can potentially lead to a loss of consciousness. However, diabetics who are hypoglycemic typically have a sympathetic response. Arrhythmia is not the correct answer choice. This patient does not have any cardiac history of arrhythmia. As the patient had the syncopal episode during the blood draw, and there were no ECG changes, this answer choice is unlikely. Orthostatic hypotension is a reasonable answer choice. However, this patient's acute syncopal episode in response to the blood draw makes a vasovagal reaction more likely. In addition, this patient has no history of dehydration or any other cause that would lead to this condition.

A 25-year-old asymptomatic, non-smoking male presents with a 9mm solitary pulmonary nodule, with dense central calcification on routine chest x-ray. The nodule's appearance is smooth and calcified with a well-defined edge. This same lesion was present on a work physical x-ray two years ago. Which of the following is the most appropriate management step for this patient? A Positron emission tomograph to rule out malignancy B Resection of the pulmonary nodule C Thoracic magnetic resonance imaging (MRI) D Watchful waiting with serial imaging E Pulmonary function tests

watchful waiting with serial imaging Solitary pulmonary nodules are categorized based upon the probability of malignancy. This patient is under the age of 30, with previous documentation of the lesion and without evidence of growth or change, which decreases the probability of malignancy. In this case, watchful waiting is indicated. Risk factors, such as advanced age, smoking, lesion growth or appearance change, and history of prior malignancy increase malignancy probability, and indicate a need for more defined evaluation.

A 35-year-old man presents with paralysis and confusion. His wife states her husband had been working in the yard about 3 days prior to presentation and suddenly began to not feel well. She states that it has progressed to the point where he appears quite confused, is extremely agitated, and does not seem to be able to close his right eye completely or smile on the right side of his face. The patient has a rash on the right arm that does not seem to be pruritic. Physical examination of the patient reveals findings of altered mental status, a cranial nerve palsy of the facial nerve (cranial nerve VII), exaggerated deep tendon reflexes, and the presence of a maculopapular rash on the right forearm. Question What is the most likely source of this patient's condition? Answer Choices 1 Mumps 2 West Nile virus 3 St. Louis virus 4 Rabies 5 Varicella-zoster virus

west nile virus This patient has encephalitis secondary to exposure to the West Nile virus. Patients with encephalitis, regardless of the cause, will present with altered mental status that has a huge variability (from subtle deficits to 100% unresponsiveness). Meningeal irritation may also be seen if involvement of the meninges is present (e.g., photophobia and nuchal rigidity). Seizures will also be seen, as well has hemiparesis, cranial nerve palsies, exaggerated deep tendon, or even pathologic reflexes. Patients will appear confused, agitated, or obtunded. The most common cause of viral encephalitis in the United States is the West Nile virus. Many times, patients may be misdiagnosed as having Guillain-Barre syndrome because of the presence of the symptom of flaccid paralysis that is specific to this infection. Another key characteristic of this encephalitis is the presence of a nonpruritic maculopapular rash, estimated to be seen in up to 50% of patients with this type of infection. This patient most likely has encephalitis secondary to the West Nile virus. A patient who is unvaccinated and presents with parotitis and altered mental status will more likely have encephalitis secondary to mumps. If a patient has tremors of the eyelids, tongue, lips, and extremities, the most likely cause is the St. Louis virus. Findings such as hydrophobia, pharyngeal spasm, and hyperactivity would be seen in a patient with rabies-induced encephalitis. A patient who has encephalitis from the varicella-zoster virus will more commonly have the dermatomal pattern of grouped vesicles that is typically seen in a zoster infection.

A 70-year-old woman who was found barely responsive at home by her daughter is brought to the emergency department. Evaluation reveals that she is in a hyperglycemic hyperosmolar state with a severe fluid deficit. Treatment is initiated with vigorous saline rehydration and a continuous infusion of insulin. At what point should her glucose be added to her treatment? A when her condition becomes stable B when her urine output reaches 50 mL/hour C when her blood glucose reaches 250 mg/dL D if she develops hypokalemia E if she begins to spill ketones in her urine

when her blood glucose reaches 250 mg/dL In hyperglycemic hyperosmolar states, the serum glucose rapidly corrects with fluid administration alone. However, with vigorous rehydration, glucose may fall precipitously and lead to severe hypoglycemia. To avoid this, glucose should be added to water, half-normal, or normal saline as soon as the patient's blood glucose is less than or equal to 250mg/dL. She should continue to receive insulin IV until she is stabilized (A) when it can be switched to subcutaneous administration. The goal of fluid therapy in this patient is restoring her urine output to 50 mL per hour (B) or more. Because insulin drives potassium into the cells and can cause hypokalemia (D), potassium chloride should be given unless the patient has chronic kidney disease or oliguria. Persons in a hyperglycemic hyperosmolar state typically do not spill ketones (E) the way persons with diabetic ketoacidosis do.

A 30-year-old man presents with a 1-year history of watery diarrhea. He has lost 5 kg. He has joint pains of 1-month duration. His stool examination for occult blood is negative. Stool culture is negative. No ova and cyst are seen in the stool. Laboratory tests fail to reveal rheumatoid factor or anti-nuclear antibodies. Serum calcium, phosphorous and alkaline phosphatase is 9.2 mg/dl, 4 mg/dl and 110 U/L respectively. A biopsy taken from the upper intestine shows distended macrophages in the lamina propria, which is found to be positive with PAS stain. What is the most likely diagnosis in this case? Answer Choices 1 Celiac sprue 2 Tropical sprue 3 Whipple disease 4 Disaccharidase deficiency 5 Amebic dysentery

whipple disease Whipple disease is a rare condition that can involve any organ of the body but principally affects the intestine, central nervous system, and joints. It is caused by gram-positive actinomycetes called Tropheryma whippelii. Histologically, the small intestinal mucosa is laden with distended macrophages in the lamina propria. These macrophages contain periodic acid-Schiff positive granules and rod-shaped bacilli by electron microscopy. These patients usually present with a form of malabsorption, including diarrhea and weight loss, and generally respond to antibiotic therapy. Celiac sprue is a rare chronic disease in which there is a characteristic mucosal lesion of the small intestine in the form of blunting of the villi and an overall increase in plasma cells, lymphocytes, macrophages, and eosinophils in the lamina propria. Also known as the gluten-sensitive enteropathy, non-tropical sprue, and celiac disease, it is due to sensitivity to gluten, which is an alcohol soluble, water insoluble protein component called gliadin of wheat and other closely related grains. Detection of circulating anti-gliadin or anti-endomysial antibodies strongly favors the diagnosis. Both the symptoms and the mucosal histology improve on withdrawal of gluten from the diet. Tropical sprue is a celiac-like disease occurring in people living or visiting the tropics. Bacterial overgrowth by an enterotoxigenic organism is said to be the cause for this disease. Intestinal changes are similar to celiac sprue, but injury is seen at all levels of the small intestine in contrast to celiac sprue, where it is concentrated in the proximal small intestine. It is mainly treated by broad-spectrum antibiotics. The disaccharidases, of which the most important is lactase, are located in the apical cell membrane of the villous absorptive epithelial cells, the deficiency of which leads to the incomplete breakdown of the disaccharide lactose into its monosaccharides, glucose, and galactose, leading to diarrhea from the unabsorbed lactose. Bacterial fermentation of the unabsorbed sugars leads to increased hydrogen production, which is readily measured in the exhaled air by gas chromatography. There are both hereditary and acquired forms. Histologically in both, there is no abnormality of the mucosal cells of the bowel. Malabsorption is promptly corrected when exposure to milk and milk products is terminated. Amebic dysentery is caused by Entamoeba histolytica, a protozoan, and generally causes diarrhea with blood and mucus. A stool examination will show the cysts and trophozoites of Entamoeba histolytica, which is diagnostic.

A 24-year-old male has an eight-month history of loose thought associations, social withdrawal, auditory hallucinations, and deterioration in his personal appearance and hygiene. Upon examination, he is noted to have a flat affect, perceptual distortions, and behaves like he is detached from his own actions. An atypical antipsychotic medication is chosen for this patient. Which of the following medications is classified as an atypical antipsychotic? A Chlorpromazine (Thorazine) B Haloperidol (Haldol) C Loxapine (Loxitane) D Molindone (Moban) E Olanzapine (Zyprexa)

zyprexa Olanzapine is the only atypical antipsychotic listed in the given answers. The remainder of the medications listed are typical antipsychotic medications.

A 3-month-old term infant presents for a wellness exam, and his parents report worsening in his feeding habits and rapid breathing. On exam, there is a grade III/VI harsh heart murmur heard over the left subclavicular region that starts in early systole, peaks at S2, and decrescendos until the next S1. The infant is tachypneic and has bounding pulses of his upper and lower extremities. All other findings are within normal limits. A chest x-ray shows an enlarged left ventricle and atrium and increased pulmonary vasculature markings. Based on the above findings, what is the most likely diagnosis? Answer Choices: A Atrial Septal Defect B Congenital Pulmonic Valve Stenosis C Ventricular Septal Defect D Patent Ductus Arteriosus E Congenital Aortic Valve Stenosis

D

A 3-month-old baby girl presents after becoming cyanotic post-feeding, and then crying inconsolably. The foster mother reports this is the 2nd episode of this type. The 1st episode was last week after the baby made a bowel movement, but it resolved quickly. She does not know anything about the birth history of the baby, but knows the birth mother received no prenatal care. Upon cardiac exam a loud systolic ejection murmur is ascertained at the left upper sternal border. An EKG reveals normal sinus rhythm and the chest X-ray reveals a "boot shaped" cardiac shadow. What is the most likely diagnosis? A Atrial septal defect B Coarctation of the aorta C Pulmonary stenosis D Tetralogy of Fallot E Ventricular septal defect

D

A 50-year-old man presents with chest pain. The pain is substernal, described as sharp, worsens with inspiration, and improves with leaning forward. A pericardial friction rub is appreciated on auscultation. What EKG finding is expected with this patient's condition? A Diffuse ST elevation B S1Q3T3 pattern C Delta waves D U waves E Sine wave pattern

A

A 15-year-old girl is referred to a cardiologist's office for workup of hypertension. Her mother reports a normal pregnancy and birth. There is no family history of heart disease. On physical exam you note the following: BP 140/70 left and right upper extremities, 90/70 left and right lower extremities, HR 85/min, RR 20/min. Brachial and femoral pulses are incongruent. You note pulsations in the suprasternal notch. Cardiac auscultation reveals a III/VI systolic ejection murmur. What would you expect to see on chest radiography? A Right ventricular hypertrophy and large pulmonary arteries B Left ventricular hypertrophy and a notch in the aorta C Boot shaped heart with right ventricular prominence D Prominence of the aorta, pulmonary artery and left atrium E Increased pulmonic vasculature

B

A 20-year-old woman presents for counseling after being diagnosed as a carrier of Emery-Dreifuss muscular dystrophy. She manifests a mild form of the disease, with only contractures of the Achilles' heels and elbows. Both her brother and her father have been diagnosed with the disease. What test will help to change the course of the disease in this young woman? A Electrocardiography B Creatine kinase C Electromyography D Muscle biopsy E Antibodies to emerin

A

A 76-year-old man with a past medical history of hyperlipidemia and diabetes mellitus presents to the emergency room with a 2-hour history of acute, severe, "crushing" left precordial chest pain; it is associated with nausea, vomiting, diaphoresis, and altered mental status. His physical exam is notable for an ashen and cyanotic appearance, hypotension, rapid and weak peripheral pulsations, distant heart sounds, elevated jugular venous distension and pulmonary crackles. A stat bedside chest X-ray reveals the following image. A Serum lactate levels are expected to be elevated B A low brain natriuretic peptide (BNP) confirms cardiogenic shock C Coronary angiography is contraindicated D A normal electrocardiogram rules out the contributing infarction E Pulmonary capillary wedge pressure (PCWP) of less than 15 mm Hg is expected

A

A 77-year-old woman with a history of ASCVD (Atherosclerotic Cardiovascular Disease) and mild, compensated congestive heart failure for which she is receiving digoxin, presents with occasional "skipped heart beats." Cardiac auscultation and an initial EKG reveal no ectopic beats. The physician decides to monitor the patient's rhythm for 24 hours. The following EKG was obtained while the patient was sleeping. What does the monitored strip in this patient show? A Wenckebach phenomenon B Mobitz type II AV block C Complete heart block D Parasystole E Multifocal atrial tachycardia

A

An 8-year-old girl, who weighs 70 lbs and is 52 inches tall, has been diagnosed with stage I HTN. Physical examination and lab studies are normal. Which additional test is most likely to detect target-organ damage abnormalities in this child? A Echocardiogram B Insulin level C Plasma renin level D Polysomnography E Renovascular imaging

A

A 49-year-old woman presents with a history of gradual onset of reduced exercise tolerance while working out at the gym. She is afebrile and otherwise feels well. Cardiac exam reveals a III/VI diastolic rumbling murmur located at the apex; it is heard best in the left lateral position. What is the most likely diagnosis? A Tricuspid stenosis B Mitral stenosis C Mitral valve prolapse D Pulmonic stenosis E Aortic regurgitation

B

A 6-week-old baby boy presents at your pediatric practice office by his mother. She reports that for the past week the baby has not been feeding well. Furthermore, he breaks out into a cold sweat on his forehead while feeding. Upon further questioning she reports that after extensive periods of crying he becomes extremely breathless, irritable, and extremely pale. The mother reports a normal vaginal delivery and denies any problems with her son at the time of discharge from the hospital following his birth. She reports a family history of congestive heart failure. Vitals are as follows: Pulse 130-regular, Respiration 34-regular, Blood pressure R arm 96/62 L arm 92/54 and R leg 70/42 L leg 74/40. Cardiac exam reveals 4+ carotid pulses bilaterally, 2+ brachial pulses bilaterally and absent femoral pulses bilaterally. A loud harsh systolic ejection murmur is noted at the base of the heart. The EKG reveals normal sinus rhythm with left ventricular hypertrophy. What is the most likely diagnosis? A Atrial septal defect B Coarctation of the aorta C Pulmonary stenosis D Tetralogy of Fallot E Ventricular septal defect

B

A 61-year-old male presents with a recent history of increased fatigue with mildly increased exertional dyspnea. Patient denies any significant past medical history but states that he had some heart problems as a child, though he was never clear as to what was the problem. On cardiac examination, you hear an early diastolic, soft decrescendo murmur with a high pitch quality, especially when patient is sitting and leaning forward. No thrill is felt. Based on this patient's presentation, you expect the patient to have A Tricuspid stenosis B Aortic regurgitation C Mitral stenosis D Mitral valve prolapse E Pulmonic stenosis

B

An 82-year-old man with a past medical history of hypertension, dyslipidemia, type II diabetes, and chronic kidney disease is being evaluated for progressive exercise-induced fatigue and shortness of breath over the last year. He also admits to more recent chest pain and lightheadedness, both of which occur with ambulation. He denies cough, fever, chills, lower extremity edema, or abdominal complaints. The physical exam revealed a narrow pulse pressure following blood pressure assessment. His cardiac exam noted a laterally displaced point of maximal impulse, as well as a mid systolic ejection murmur that is low-pitched, rough, rasping in character, and loudest in the second right intercostal space. This murmur radiates to the bilateral carotid arteries. His peripheral vascular exam demonstrated a delayed peak of his radial pulsations. What diagnostic procedure would be best for this patient? A EKG B Echocardiography C Lateral chest X-ray D Cardiac catheterization E Immediate coronary angiography

B

During a workup for secondary causes of hypertension in a thin, 22-year-old woman, you discovered she has fibromuscular disease (FMD) of her renal arteries. Workup also revealed a normal urinalysis, normal BUN creatinine and a normal TSH. She takes no illicit drugs. What is the mechanism by which FMD causes hypertension? A Atherosclerotic vessels carry a reduced blood supply to the kidneys B Thickening of the renal artery causing decreased vessel diameter which eventually leads to renal artery stenosis C Decreased cardiac output and increased vascular tone lead to hypertension D Catecholamine excess from apnea causes vasoconstriction E Decreased effective circulating volume with increased renin, vasoconstriction and systemic hypertension

B

A 12-year-old presents with an injury of his left arm and leg. He states that he felt dizziness during the 2nd mile of the long distance run organized by the school. He fell and lost the consciousness for several seconds, but after that he felt "normal". His father has been diagnosed with Emery-Dreifuss muscular dystrophy type 1. On examination, you find a few superficial excoriations; there is also symmetric humero-peroneal weakness involving the biceps, triceps, and peroneal muscles. There is also atrophy and contractures of Achilles-heel, elbows, and posterior neck. After taking care of his injuries, what test should you order? A CK B LDH C EKG D EEG E CT

C

A 61-year-old man presents to the local A+E with a 1-hour history of chest pain. An ECG is obtained and found to have T wave inversion and wide Q waves in leads II, III, and AVF. What condition are these ECG findings are most consistent with? A Acute ischaemia without myocardial infarction B Acute lateral myocardial infarction C Acute inferior myocardial infarction D Acute posterior myocardial infarction E Acute anterior myocardial infarction

C

A 65-year-old male presents to the office with complaints of left upper extremity hemiparesis 2 weeks ago, lasting 2 hours. His past medical history is significant for hypertension, coronary artery disease, and he underwent a right coronary artery stent 2 years ago. He has smoked a pack of cigarettes a day for 35 years. His physical exam demonstrates a right arm BP-90/60 mm Hg and a left arm BP-130/70 mm Hg. He has a Rt. supraclavicular bruit. His workup has included a normal echocardiogram, head MRI, carotid duplex, and cardiac stress test. An angiogram was then performed. Refer to the image. The angiogram demonstrates A Vertebral artery stenosis B Aortic stenosis C Innominate artery occlusion D Left subclavian stenosis E Right subclavian stenosis

C

A 65-year-old man presents to the emergency department via ambulance with a fever and multiple other vital signs suggestive of septic shock; the source is yet to be determined. Placement of a peripherally inserted central catheter (PICC) line is ordered and completed, and the patient is being transferred to the intensive care unit. However, just before this happens, a physician assistant on the case tells his attending physician that he suspects the patient has cardiac tamponade due to the presence of Beck's triad. What does this triad consist of? A Distant heart sounds, hypertension, distended neck veins B Tachycardia, hypertension, distended neck veins C Distant heart sounds, hypotension, distended neck veins D Bradycardia, hypotension, bounding carotid pulses E Tachycardia, hypotension, bounding carotid pulses

C

A 63-year-old woman presents to the local A+E with a 1-hour history of left shoulder pain and nausea. She has a past medical history of coronary artery disease and had a stent placed 5 years ago. An ECG is obtained and found to have large R waves and ST segment depression in leads V1, V2, and V3. These ECG findings are most consistent with what condition? A Acute ischemia without myocardial infarction B Acute lateral myocardial infarction C Acute inferior myocardial infarction D Acute posterior myocardial infarction E Acute anterior myocardial infarction

D

A 65-year-old man presents with a 2-week history of progressive dyspnea, orthopnea, and pedal edema. His history is significant for heavy smoking (30 cigarettes/day for more than 40 years), a 20 pound weight loss over past 3 months, loss of appetite, and weakness. His physical examination reveals pulse - 130 per min., blood pressure - 105/90 mmHg, paradoxic pulse, distant heart sounds, nonpalpable cardiac impulse, slight dullness at both lung bases, and bilateral pedal edema. His chest X-ray shows enlarged cardiac silhouette and a large pericardial effusion; they are confirmed by echocardiography. Acid-fast staining of the pericardial fluid is negative for Acid-Fast Bacilli (AFB). CT scan reports are awaited. What is the most likely etiology of this pericardial effusion? A Pulmonary TB B Viral pericarditis C Bacterial pericarditis D Malignant pericardial effusion E Congestive heart failure

D

A 3-month-old male infant presents for a well-baby check. There have been no other changes since the last visit. Upon exam, the infant is acyanotic and well appearing. Cardiovascular exam reveals a grade III/VI high-pitched, harsh pansystolic murmur heard best at the left sternal border, 4th intercostal space (ICS). A systolic thrill is palpable over the same area where the murmur is heard best. A mild left ventricular heave is evident. No additional murmurs are heard. The remainder of the exam is unremarkable. What is the most likely diagnosis? A Atrial septal defect B Coarctation of the aorta C Aortic regurgitation D Ventricular septal defect E Patent ductus arteriosus

D

A 30-year-old woman with no significant past medical history presents to the emergency room with severe, retrosternal, and left precordial chest pain; it has occurred over the past 3 days. Pain is sharp in quality; it is referred to the neck, arms and left shoulder. It is pleuritic. It is aggravated by inspiration and coughing as well as lying in a supine position. Her review of systems is remarkable for a low grade fever, myalgias, and arthralgias. It is relieved by sitting up and leaning forward. Upon physical exam, her vital signs are normal, and her skin is warm and dry. Auscultation of the chest wall with the diaphragm at the left lower sternal border reveals a high-pitched, rasping, scratching, and grating sound, most audible at the end of expiration. A bedside electrocardiogram reveals the following imaging. What is an additional expected diagnostic finding in this patient? A Lobar consolidation on chest radiography B Positive urea breath test C Evolution of q-waves on electrocardiogram D Elevated ESR and CRP levels E Bloody pericardial fluid aspirate

D

A 4-year-old boy presents with clubbing of his fingers, cyanosis, and a heart murmur; he is easily fatigued. A chest X-ray reveals right ventricular hypertrophy. Cytogenetic analysis of a skin biopsy reveals a 46,XY karyotype. What is the most likely diagnosis? A Down syndrome B Patau syndrome C Edwards syndrome D Tetralogy of Fallot E Atrial septal defect

D

A 70-year-old Hispanic man with a past medical history of hyperlipidemia presents with recurrent chest pain that has been occurring over the past month. This pain is provoked with activity, especially of his upper torso and left arm. The pain is described as sharp and generally located to the left pectoral area. He denies any weight changes, fever, or chills. He also denies cough, wheezing, pleurisy, calf pain, wheezing, vomiting, diarrhea, changes in bowel habits, cigarette, drug or alcohol use, sick contacts, or travel. Physical exam reveals an elderly Hispanic man in no acute distress. Vital signs, skin, cardiopulmonary, abdominal, and peripheral vascular exams are found to be within normal limits. The presence of what additional historical or physical exam finding would suggest a non-ischemic etiology of this patient's chest pain? A Pain associated with anxiety, nausea, shortness of breath, and syncope B Pain is relieved upon administration of nitrate drugs C There is an associated history of diabetes mellitus and hypertension D Chest pain that is reproduced with palpation E Quality of pain described as a pressing, tightness, or heaviness

D

A 78-year-old man with known left-sided congestive heart failure presents with a complaint of cough, worsening dyspnea with exertion, and orthopnea. What is the most direct cause of his symptoms? A Tricuspid insufficiency B Left ventricular hypertrophy C Decreased peripheral vascular resistance D Increased pulmonary venous pressure E Mucus plugging

D

A 64-year-old woman presents with severe chest pain and dyspnea. She has a past medical history of diabetes mellitus, hypertension, and hyperlipidemia. She appears pale, apprehensive, and diaphoretic. She is in a confused state and is agitated; her pulse is weak. In addition, she has a narrow pulse pressure, tachypnea, a weak apical impulse, and significant jugular venous distention. Her lungs are remarkable for bilateral crackles. Her bedside electrocardiogram is notable for the following imaging. What is the most likely expected clinical manifestation in this case? A Hyperresonance to lung percussion B Polyuria C Poor skin turgor D Hypotension E Warm and dry extremities

D Hypotension-The vast majority of cases of cardiogenic shock in adults are due to acute myocardial ischemia. The clinical definition of cardiogenic shock is decreased cardiac output and evidence of tissue hypoxia in the presence of adequate intravascular volume. The diagnosis of cardiogenic shock can sometimes be made at the bedside by observing the presence of hypotension

A patient is presenting with substernal crushing chest pain and shortness of breath for just 15 minutes prior to arriving to the emergency department. An EKG is obtained and appears suspicious for an acute myocardial infarction. What would most likely be seen in this patient's EKG? A S-T segment depression B T-wave changes C Ventricular bigeminy D Q-wave elongation E Hyperacute T-wave

E- first sign of MI

What is expected to be true regarding the murmur in this patient? A It decreases while in an upright posture. B It becomes less intense with the Valsalva maneuver. C It decreases with squatting. D It increases with sustained handgrip. E It increases with lying down.

C

A 65-year-old man presents to the office with complaints of 6 months of bilateral buttock and thigh cramping pain that occurs after walking 20 feet and is completely and quickly relieved with resting. His past medical history included hypertension treated with atenolol, and he had a stroke 3 years ago. He also complains of impotence for approximately the same duration of time. What is the patient's physical exam likely to include? A Absent femoral, popliteal, and pedal pulses B Absent popliteal and pedal pulses, normal femoral pulses C Absent pedal pulses, normal femoral and popliteal pulses D Normal femoral, popliteal, and pedal pulses E Normal femoral, normal popliteal, and diminished pedal pulses

A

1) A 75-year-old African-American man presents with a 5-month history of gradually progressive dyspnea that is especially pronounced when climbing stairs. He also has been noticing that his ankles and lower legs have "gotten larger" over roughly the same time period, which no longer allows him to fit into his sneakers. He denies fever, chills, chest pain, palpitations, cough, pleurisy, calf pain, abdominal complaints, sick contacts, or travel. His psychosocial history is noteworthy for chronic alcohol use. His physical exam reveals bibasilar rales, JVD of 5cm, an S3 gallop, a holosystolic murmur at the apex that radiates to the left axilla, and 2+ pitting edema to the level of the mid-calves bilaterally. A bedside echocardiogram was remarkable for biventricular enlargement. A Tachycardia B Fever C Asymmetric upper extremity blood pressures D Warm, moist skin E Acanthosis nigricans

A

A 65-year-old man with a 10-year history of hypertension controlled with lisinopril comes for pre-operative evaluation before his arthroscopic knee surgery. He has never smoked, and has never had either a heart attack, congestive failure, arrhythmia, valvular problems, or abnormal lipids. His activity level is limited by knee pain; he does light to moderate housework and occasional swimming without dyspnea or chest pain. He is sexually active. His medications include ibuprofen for knee pain. His BodyMassInex is 26.5((9) blood pressure is 120/80 mmHg, and the remainder of his exam is normal. His last electrocardiogram (EKG), taken 5 years ago shortly after a panic attack, showed normal sinus rhythm. His most recent (fasting) serum creatinine was 1.6 mg/dl, with a blood urea nitrogen of 20 mg/dl. Past serum creatinines were in the 1.4-1.8 mg/dl range. He has no proteinuria. What further pre-operative cardiac evaluation, if any, is indicated? A No perioperative cardiac testing needed, consider ekg B Cardiac catheterization is indicated C Holter monitoring is indicated D Echocardiography is indicated E Stress-echocardiography is indicated

A

A 57-year-old man is found to have a grade 3 systolic heart murmur. He has an enlarged spleen and small petechial hemorrhages on the arms and legs. Past medical history reveals mitral valve regurgitation with a grade 1 murmur, and it is determined that the patient is currently a candidate for surgical therapy. The patient is admitted and blood cultures are obtained. A hemolytic anaerobic Gram-positive coccus is isolated. What is the most likely cause of this patient's endocarditis? A Streptococcus pneumoniae B Viridans streptococci C Group A streptococci D Pseudomonas aeruginosa E Peptococcus sp

B

A 36-year-old woman presents with chronic dyspnea that is worse while lying prone. The patient reports progressive worsening of the symptoms. On physical examination, a heart murmur is detected upon cardiac auscultation, heard best with the bell over the apex. The murmur is a non-radiating, low-pitched diastolic rumble. A loud S1 and opening snap can also be heard in addition to an apical thrill and decreased pulse pressure. An EKG is done and shows an arrhythmia. What is the patient's most likely underlying condition? A Aortic regurgitation B Pulmonic stenosis C Mitral stenosis D Hypertrophic subaortic stenosis E Mitral valve prolapse

C

A 55-year-old man presents for re-evaluation of his blood pressure; he has no significant past medical history. At his visit 3 weeks ago, his blood pressure was 145/90 mm Hg. He admits to somnolence, confusion, and nonspecific bilateral visual disturbances over the past month; he denies any eye pain, blindness, ocular discharge, or floaters. His blood pressure today is 185/110 mm Hg. His fundoscopic exam reveals the following image What is an additional expected manifestation in this case? A Scleral icterus B Rhinitis C Headache D Fruity breath E Erythema nodosum

C Headache-Hypertensive retinopathy may cause papilledema, intraretinal hemorrhages, nerve fiber layer infarcts (cotton-wool spots), arteriovenous (AV) nicking, banking or beading, or copper or silver wiring of the retinal vessels.

A 25-year-old African-American man presents with blood pressure issues. He states that he attended a work-sponsored health fair; he had his blood pressure taken, and the health fair representative strongly urged him to make an appointment to be seen by his primary care provider. The fair occurred 3 months ago. He has randomly checked his blood pressure at different times since the health fair in local grocery stores, and although he does not remember the numbers, he knows that "they were above normal". The patient also believes that both his mother and father are taking blood pressure medication, but he is not 100% sure. Vitals at this time reveal a BMI of 30 kg/m2 and a waist circumference of 41 inches; blood pressure in the left arm is 175/95 mm Hg, and in the right arm it is 172/99 mm Hg. Based upon the diagnosis at this time, what would most likely be seen during the fundoscopic examination of this patient? A Drusen bodies B Microaneurysms C Deep retinal hemorrhages D Cotton-wool patches E Macular star

D

A 5-day-old newborn was born 5 weeks prematurely and presents to her first pediatrician's appointment. She did not have any feeding or breathing issues, so mother and child had only a 2 day stay at hospital. During the cardiovascular examination, the pediatrician notes that the newborn has a distinct murmur with a rough machine-like quality that is maximal at the second intercostal space at the left sternal border. The murmur starts after S1 and passes through S2 into diastole. The pediatrician is suspicious that the patient has a patent ductus arteriosus, but would like to order imaging to aid in diagnosis. What diagnostic study is the first choice to provide a definitive diagnosis? A Electrocardiography B Chest X-ray C Cardiac catheterization D Chest CT scan E Echocardiography

E

41-year-old woman presents for follow-up regarding elevated blood pressure. This is her third visit to the office, and her blood pressure has been elevated on multiple readings at each visit. She has a history of eczema but is otherwise healthy. Labs reveal the following: WBC 14.5 k/uL Hgb 13.5 g/dL HCT 41% PLT 152 k/uL Na 135 mmol/L K 2.8 mmol/L Cl 99 mmol/L CO2 32 mmol/L BUN 10 mmol/L Cr 1.02 mmol/L What factor is most suggestive of secondary hypertension in this patient? A Age B Female gender C History of eczema D Leukocytosis E Hypokalemia

E

A 27-year-old woman presents with a 3-day history of "sharp", diffuse chest pain. She states the pain is worse with movement and deep breathing. On examination, it is noted that the patient prefers to sit upright and lean forward; she states, "I feel better in this position". Vital signs include a BP of 126/72 mm Hg; HR is 82, RR is 18, O2 sat is 96% RA, and temp is 101.3? F. On exam, you appreciate a friction rub. What set of diagnostics should you order? A CXR, EKG, ECHO, DDimer, and BHCG B CBC, BMP, BHCG, and EKG C CBC, BHCG, EKG, ECHO, and CXR D CXR, EKG, DDimer, and BHCG E CXR, CBC, BMP, BHCG, EKG, and ECHO

E

A 3-month-old male infant has been brought into the pediatric clinic for assessment. The newborn's mother states that her child is not gaining adequate weight despite a regular breast feeding schedule. She additionally has noted that the child appears to get "very tired and inactive" during and after breast feedings, and that she is able to feel copious amounts of sweat on the child's skin following feedings. She denies any known illness in her child and recalls a normal birth. The general survey reveals a weight and length in the fortieth percentile, tachycardia, and tachypnea. The cardiac exam is remarkable for a bounding and hyperdynamic precordium, a holosystolic harsh murmur that is audible over the lower LSB, and a loud second component of the second heart sound. What is the best next diagnostic step for this child? A Chest radiograph B Electrocardiogram C Cardiac catheterization D Magnetic resonance imaging E Echocardiogram

E

Which of the following complications is commonly associated with subarterial VSD? A Infective endocarditis B Pulmonary hypertension C Congestive cardiac failure D Cor pulmonale E Aortic insufficiency (AI)

E

A 64-year-old woman with a past medical history of hyperthyroidism presents with new-onset palpitations. The patient states that she has intermittent palpitations, and she is worried about heart disease. She is currently asymptomatic and reports no chest pain or shortness of breath. An EKG is obtained. Vital signs reveal a T 98.6° F, BP 134/88 mm Hg, P 119 beats/min R 12/min. The pulse is noted to be irregular. The EKG is shown below. What arrhythmia does this patient most likely have? A Atrial flutter B Atrial fibrillation C Supraventricular tachycardia D Ventricular tachycardia E Torsades de pointes

B

A 65-year-old African-American man presents in your office for a follow-up for hypertension. Three months earlier you prescribed furosemide. He checks his blood pressure daily and states that it is markedly lower since he has been on the medication. However, it feels like his heart is skipping a beat once in a while. What deficiency is most likely? A Sodium B Potassium C Vitamin C D Thiamin E Riboflavin

B

A 43-year-old man presents for a general physical exam. He states that he has no significant past medical history. On physical examination, you note that the patient has a 3/6 diastolic murmur; it is heard best in the right upper chest. Blood pressure is 152/62 mm/Hg. No other abnormalities are noted. What study is best in regard to getting an accurate diagnosis for this patient? A EKG B Coronary heart catheterization C Echocardiogram with color Doppler D Cardiac CT scan E Chest X-ray

C

A 25-year-old Asian-Indian woman presents with pain, weakness, and increased numbness in her right hand for the last 24 hours. The patient also states that, for the last month, she has not felt very well; she has experienced malaise, night sweats, decreased appetite, and arthralgias. She notes that, for the last week when she washed her hands with cold water, her right hand would turn blue. On clinical examination, the patient has fever, tachycardia, decreased pulse, and decreased blood pressure in the right arm; Babinski's sign is negative. Auscultation on the right supraclavicular space reveals a bruit. Laboratory findings are an elevated ESR and mild anemia. What is the most appropriate next step in the management? A Aortic arteriography B Echocardiogram C CT scan of the chest D Chest X-ray E VDRL

A

A 38-year-old woman with a past medical history of rheumatic fever and endocarditis presents with progressive dyspnea on exertion associated with palpitations and intermittent episodes of left-sided chest pain. Both symptoms resolve at rest. Her physical exam reveals resting tachycardia and a widened pulse pressure. The cardiac exam is notable for a decrescendo, diastolic, high-pitched murmur, which is loudest at the left sternal border and accentuated with the patient leaning forward in full expiration. Abrupt distention and quick collapse is observed upon palpation of the peripheral arterial pulses, and booming systolic and diastolic sounds are auscultated over the femoral arteries. What is the most important diagnostic test in the evaluation of this patient at this time? A Transthoracic echocardiography B Chest radiography C Cardiac computed tomography D Electrocardiography E Cardiac troponins

A

A 25-year-old man presents after experiencing pronounced shortness of breath while at the gym. You cannot obtain satisfying information from him during the interview and you think that he might be mentally challenged. On examination he appears short for his age and you notice webbed neck, dental malocclusion, antimongoloid slanting of the eyes, and hypogonadism. Auscultation reveals high-pitched systolic ejection murmur maximal in the second left interspace with radiation to the left shoulder and ejection click that decreases with inspiration. His second heart sound (P2) is delayed and soft. The impulse of right ventricle is increased, and you palpate a thrill at second left intercostal space. What will be your best next diagnostic step? A Mental retardation assessment B Echocardiography/Doppler C Cardiac catheterization D Cardiac MRI E Karyotyping

B

A 32-year-old man with no significant past medical history presents with a 2-month history of increased dyspnea upon exertion; the dyspnea becomes apparent after walking 10 city blocks. He denies associated symptoms, such as fever, chills, changes in weight, chest pain, abdominal pain, nausea, and vomiting. He also denies any history of cigarette smoking, occupational risk factors, sick contacts, and recent travel. His physical exam reveals normal vital signs and no distension of his jugular vein. There is, however, a prominent right ventricular impulse along the lower-left sternal border that is associated with a palpable pulmonary artery; also, there is a mid-systolic ejection murmur at the upper left sterna border that does not vary in intensity with respiration. There is a fixed split second heart sound. The remainder of his examination is normal. What is the most likely diagnosis? A Patent ductus arteriosus B Atrial septal defect C Ventricular septal defect D Mitral valve prolapse E Pulmonic stenosis

B

A 68-year-old man with a past medical history of diabetes mellitus type II, hypothyroidism, and hypderlipidemia presents with a 1-hour history of constant moderate to severe "squeezing, pressure and tight" left-sided chest pain. Additionally, he admits to shortness of breath and nausea. He notes that he has had a 1-week history of similar, recurrent chest pain of about 10 minutes duration that had been occurring following exposure to the cold weather and consumption of a meal. He denies chills, abdominal pain, diarrhea, cough, and pleurisy. What physical exam finding is an anticipated finding consistent with this patient's most likely diagnosis? A Grouped, vesicular lesions of the precordium B Jugular venous distension C Chest wall tenderness D Deviated trachea E Flushed, warm and dry skin

B

The patient is a 20-year-old man who presents to a medical clinic for a routine physical required before he enlists in the army. His family history is significant for 2 relatives who had sudden cardiac arrest in their late 20's or early 30's. He does not know any more details about these relatives or their medical history. His review of systems is negative with the exception of rare episodes of heart palpitations. What is the most likely finding on the patient's cardiac examination if he has hypertrophic cardiomyopathy? A soft heart sounds and an S3 gallop B brisk heart sounds and an S4 gallop C early diastolic pericardial knock D S4 gallop and no murmur E 3 component friction rub

B

A 75-year-old woman presents with a 5-day history of blue toes. Her past medical history is remarkable for hypertension treated with hydrochlorathiazide for 12 years. Her mother had died from an aneurysm within her chest. Her physical exam includes a BP-160/96 mm Hg pulse of 85/min, normal cardiac rate and rhythm, and bilateral cyanotic feet with normal dorsal pedis and posterior tibial pulses. She undergoes evaluation, which includes an unremarkable echocardiogram, EKG, lower extremity arterial duplex, and Computed Tomography as shown. What additional radiological test should be done prior to treatment? A Coronary catheterization B Magnetic Resonance Imaging C Computed tomographic angiography D Abdominal ultrasound E Transthoracic echocardiogram

C

You are called to the emergency department at 2 P.M. to see a 44-year-old male patient. He is a 3-pack-a-day, unfiltered cigarette smoker with crushing chest pains. He has a wide-complex, rapid, regular tachyarrhythmia at 160 beats per minute. When you reach his examination room, you note his monitor also reveals evidence of "P" waves at 75 beats per minute. What type of rhythm do his symptoms show? A Paroxysmal supraventricular tachycardia B Sinus tachycardia C Ventricular tachycardia D Ventricular fibrillation E Asystole

C

A 74-year-old man with a past medical history of hyperlipidemia, myocardial infarction, rheumatic heart disease, and hypertension presents for a routine evaluation. He states that he has no complaints and feels well. The physical exam reveals an elderly man in no apparent distress whose vital signs are within normal limits. The cardiac exam is noteworthy for a soft S1 to auscultation, a systolic thrill palpable at the cardiac apex, lateral displacement of the point of maximal impulse, and a holosystolic murmur of IV/VI intensity, which is loudest at the apex and radiates to the axilla. The remainder of the exam is normal. What EKG finding would be most likely associated with this patient's presentation? A Wide QRS complexes B Diffuse ST elevations C Prolonged PR intervals D Negative P-wave deflection in V1 E Large P-wave amplitude in II

D

A long-standing patient of yours is experiencing acutely worsening symptoms and signs of his dilated cardiomyopathy; symptoms include fatigue, dyspnea with mild exertion, paroxysmal nocturnal dyspnea, severe lower extremity edema, clubbing, an S3 gallop, and jugulovenous distention. Medications this patient is taking daily include a β-blocker, adult dose aspirin, and an ACE inhibitor. What clinical intervention would be the most logical next step in helping resolve his current symptoms? A Initiation of a calcium channel blocker B Initiation of an aldosterone antagonist C Initiation of a statin D Initiation of a diuretic E Initiation of an angiotensin II antagonist

D

A newborn child is routinely evaluated in the pediatrician's office 1 month following delivery. The mother reports that the patient is behaving normally and is feeding well. The physical exam is remarkable for a murmur, which is located at the 2nd left intercostal space. The murmur is continuous throughout cardiac systole and diastole, nonradiating, and of a "machinery" quality. There is additionally a widened pulse pressure. The skin and mucosa is without cyanosis, and there is no evidence of fluid retention. What is the most likely diagnosis? A Ventricular septal defect B Atrial septal defect C Transposition of the great arteries D Tetralogy of Fallot E Patent ductus arteriosus

E

A 45-year-old man presents with a 5-minute history of left-sided chest pain; he describes the pain as "tightening". He denies any trauma to the chest, but he reports he has just been told his son had died in an accident. On examination, he has an apical systolic murmur. A Musculoskeletal pain B Herpes zoster C Spontaneous pneumothorax D Angina E Pericarditis

D

A 48-year-old man is brought to the ER complaining of difficulty breathing, fatigue, and intermittent chest pain for the past month. On further questioning, he states that the breathing seems to worsen when lying down. On physical exam, you note elevated respiratory and heart rates and pale, sweaty skin. On auscultation, rales are noted as well as a 3rd heart sound. Which of the following is the most likely diagnosis? A Right Ventricular failure B Pulmonary Embolism C Mitral Valve Stenosis D Left Ventricular failure E Chronic Obstructive Pulmonary Disease (COPD)

D

An African-American neonate suffers from respiratory distress and cyanosis shortly after birth. On physical examination, you note the presence of IV/VI ejection-type systolic murmur that is heard at the mid and upper left sternal borders. Also present on auscultation is a right ventricular tap along the left sternal border with a systolic thrill at the upper and mid-left sternal borders. The S2 heart sound is single with an absent pulmonic component. Electrocardiogram reveals right axis deviation with mild right ventricular hypertrophy. Chest X-ray examination reveals: (1) enlarged right atrium (2) decreased pulmonary vascular markings (3) a "boot-shaped" heart. What defect is part of this patient's condition? A Aortic stenosis B Atrial septal defect C Transposition of the great vessels D Ventricular septal defect E Anomalous coronary artery

D

While doing rounds one morning, you come upon a 42-year-old man suspected of having an infective endocarditis and currently undergoing an extensive workup. Which of the following represents the most definitive diagnosis of Infective Endocarditis based on Modified Duke Criteria? A 1 positive blood culture with Staphylococcus aureus with Osler's nodes and Roth spots B 2 positive blood cultures with Streptococcus pneumoniae with cutaneous hemorrhages and glomerulonephritis C Evidence of endocardial vegetation on echocardiography with Osler's nodes D 2 positive blood cultures with Staphylococcus aureus and development of a new regurgitant murmur E Fever >100.4 degrees Fahrenheit (38 degrees Celsius) with evidence of endocardial vegetation on echocardiography and glomerulonephritis

D

A 25-year-old woman presents the ER after a syncopial episode. She had loss of consciousness 3 times over the past 12 months. Each event occurred during or just after physical exercise. On PE: BP 110/70 mm Hg, HR 75/min, normal S1/S2, and a III/VI systolic ejection murmur is heard best at the left sternal border that decreases with squatting. The EKG shows a normal sinus rhythm with diffuse increased QRS voltage. What is the most likely diagnosis? A Hypertrophic Cardiomyopathy B Mild Mitral Valve Insufficiency C Moderate Pulmonary Stenosis D Severe Aortic Stenosis E Moderate Mitral Valve Stenosis

A

A 5-day-old male infant has subtle, unusual facial features (i.e., a triangular face, hypertelorism, and down-slanting eyes). He also has a webbed neck and low-set ears. Suspecting a congenital disorder, you order a complete work-up, including a CBC, coagulation profile, cardiac evaluation, karyotyping, and mutation analysis. PTPN11 (protein-tyrosine phosphatase, nonreceptor-type 11) mutations are detected. Echocardiography detects a cardiac defect. What is most likely to be found on echocardiography? A Pulmonary stenosis B Aortic valve stenosis C Tricuspid stenosis D Aortic regurgitation E Tricuspid regurgitation

A

A 73-year-old woman presents with acute lower leg swelling and pain. Venous duplex ultrasound confirms the presence of deep vein thrombosis. She is admitted for anticoagulation with heparin. What test is the most sensitive to monitor the effects of heparin? A Activated partial thromboplastin time (PTT) B International normalized ratio (INR) C Bleeding time D Platelet function E Prothrombin time (PT)

A

A 2-month-old female infant presents for a well-child visit. Her mother states that she is concerned about the patient's lack of interest in feeding as well as rapid breathing spells. You acknowledge these concerns, and during the physical examination, you note severe tachypnea, bounding peripheral pulses, and a rough, machinery murmur that is auscultated best near the 2nd left intercostal space. What is the most likely diagnosis? A Atrioventricular septal defect B Patent ductus arteriosus C Coarctation of the aorta D Ventricular septal defect E Still murmur

B

A 56-year-old man presents with moderately severe substernal and left anterolateral chest pain. There is some exacerbation of pain on inspiration, and it has been increasing in severity over the last 36 hours. He works as a truck driver and has a history of heavy cigarette smoking, hypertension, and obesity. Over the past week, he has experienced swelling and discomfort in his right calf. Examination shows BP of 90/55 mm Hg, P of 122/min, RR of 40/min, and temp of 37.6° C. The patient is mildly agitated and confused. Systemic examination reveals tachycardia, soft systolic murmur, and questionable ventricular gallop. Lungs show dullness to percussion at left base, with scattered crackles and wheezes throughout. Abdominal and neurological exams are negative. The right calf is 0.5 cm larger than left, with some deep tenderness and a trace of ankle edema. Laboratory analysis reveals hemoglobin 16.4g/dL, Hct 51%, WBC 12,300 cells/µL, PaO2 52 mm Hg, PaCO2 38 mm Hg, and pH 7.35. Chest radiograph shows borderline cardiomegaly and a prominent aorta, scattered patchy infiltrates bilaterally, and a small left pleural effusion. What is the most accurate diagnostic modality for diagnosing this patient's condition? A Impedance plethysmography B Computerized tomographic angiography (CTA) C Spirometry D Myocardial scan E Cardiac ultrasound

B

A 6-month-old infant with Tetralogy of Fallot is admitted for planned surgical correction of her heart defect. The night before the surgery, you decide to obtain an arterial blood gas as a baseline assessment of her gas exchange. She is currently hemodynamically stable, with heart rate of 108/minute, respiratory rate of 32/minute, and pulse oximetry shows oxyhemoglobin saturation of 85% on room air. She has been feeding well. What result on the arterial blood gas is the most consistent with her underlying diagnosis? A Elevated pCO2 B Decreased pO2 C Elevation in pO2 with supplemental O2 D Elevation in pO2 with crying E Elevated pCO2 and pH

B

A 52-year-old male presents to the emergency department due to acute onset of fever, malaise, and general arthralgias. His social history is pertinent for chronic use of marijuana and heroin. He was also recently laid off at his job and has been on what he calls a "drug binge" for several days. The emergency room physician does a thorough examination and thinks he hears a heart murmur. However, the patient is a poor medical historian and is not sure whether this is new. The emergency room staff begin running a number of labs and diagnostic tests with the concern that he may have infective endocarditis. What additional physical examination findings, along with positive diagnostic results, would confirm their diagnosis? A Blood culture positive for Staphylococcus aureus and presence of a deep vein thrombosis B 2 separate blood cultures positive for Staphylococcus aureus and presence of fever over 38°C C 2 separate blood cultures positive for Staphylococcus aureus and a tricuspid valve mass seen on echocardiogram D Tricuspid valve mass seen on echocardiogram and fever over 38°C E Fever over 38°C and presence of deep vein thrombosis and conjunctival hemorrhages

C

A 63-year-old woman with a 3-day history of hypertension, hyperlipidemia, and myocardial infarction presents to the emergency room with shortness of breath at rest. She has found it difficult to walk short distances due to shortness of breath. Additionally, she complains of orthopnea, nocturnal dyspnea, and generalized abdominal discomfort. She denies cough, fever, chills, diaphoresis, anxiety, chest pain, pleurisy, cough, nausea, vomiting, diarrhea, rashes, lightheadedness, and syncope. She is well nourished and afebrile, but tachypnic and diaphoretic. There is a diminished first heart sound, S3 gallop, laterally displaced PMI, bibasilar rales and dullness to percussion, and expiratory wheezing noted. The abdominal exam reveals distension, with hepatomegaly in the right upper quadrant. There is 2+ pitting edema of the lower extremities to the level of the mid calf, and the extremities are cool. What additional finding is expected in this patient? A Increased capillary refill and warm extremities B Bradycardia C Elevated jugular venous distension D Increased muscle strength E Daytime polyuria

C

A 67-year-old man presents with progressive shortness of breath, fatigue, and orthopnea. The patient cannot walk more than a few steps without taking a short rest, and he says his condition has deteriorated in the past 3 days. His blood pressure is 120/60 mm Hg, pulse is 84 BPM, respirations are 28/minute and labored, and his body temperature is 97.5° F. Refer to the image. What is the most likely diagnosis? A Pneumonia B Pleuritis C Pulmonary edema D Left ventricular hypertrophy E Lung abscess

C

A 70-year-old man is in the ICU post-coronary artery bypass grafts. He is post-op day 9 and becomes both hypotensive and tachycardic. His blood pressure improves with 1 bolus of IV fluid. 30 minutes later, however, he becomes hypotensive again. His heart sounds are muffled and the chest tubes draining the pericardium have had decreased output over the past 3 hours. The patient is afebrile. What is the most likely etiology for this patient's hypotension? A Hypovolemia B Cardiomyopathy C Pericardial tamponade D Sepsis E Myocardial infarction

C

A 74-year-old man presents with a 1-hour history of constant moderate-to-severe "squeezing, pressure, and tight" left-sided chest pain, accompanied by nausea. He has a past medical history of diabetes mellitus type II, hypothyroidism, and hyperlipidemia. He also has a 1-week history of similar, recurrent chest pain of about 10 minutes duration; the pain occurs following exposure to cold weather and consumption of a meal. He denies fever, chills, abdominal pain, diarrhea, cough, pleurisy, and shortness of breath. What additional findings would be most consistent with his most likely diagnosis? A The chest pain is relieved by sitting forward B This pain will be relieved with antacids and belching C Movement of the chest, trunk, and arms are provocative factors D Vomiting, diaphoresis, and weakness are associated findings E There is localized tenderness to the chest wall

D

A preterm female infant born to a 32-year-old woman with no known past medical illnesses presents for the infant's 1-week follow-up. The mother reports that the patient is behaving normally and is feeding well. The physical exam is remarkable for a murmur, which is located at the 2nd left intercostal space. The murmur is continuous throughout cardiac systole and diastole, nonradiating, and of a "machinery" quality. Additionally, there is a widened pulse pressure. The skin and mucosa is without cyanosis, and there is no evidence of fluid retention. Based upon the physical exam finding, what is the most likely diagnosis? A Atrial septal defect B Ventricular septal defect C Tetralogy of Fallot D Patent ductus arteriosus E Pulmonary stenosis

D

A 37-year-old woman with a recent onset of atrial fibrillation presents with new symptoms of fatigue and dyspnea on exertion that has now progressed to dyspnea at rest. The patient also admits to associated orthopnea and peripheral edema. On cardiac exam, prominent right ventricular and pulmonary arterial pulsations are visible and palpable. The second heart sound is widely split on auscultation and does not vary with breathing (fixed split). A loud systolic ejection murmur is heard in the second and third interspaces parasternally. The patient's past medical history is significant for a mumur diagnosed during childhood. What diagnostic test will identify the most likely etiology of the patient's symptoms? A Chest X-ray B Pulmonary function tests C Electrocardiogram (ECG) D Holter monitor E Echocardiogram

E

A 45-year-old presents with a 15-minute history of left-sided chest pain, which he describes as "crushing". He denies any trauma to the chest; he was sitting at home reading when it began. On examination, there is an apical systolic murmur. What is the most probable cause of his chest pain? A Esophageal reflux B Pneumonia C Aortic dissection D Pulmonary embolism E Unstable angina

E

A 70-year-old woman with a history of hypertension, hyperlipidemia, and myocardial infarction presents with a 3-day history of shortness of breath at rest. She has found it difficult to walk short distances due to this shortness of breath. Additionally she complains of orthopnea and nocturnal dyspnea. She denies cough, fever, chills, nausea, abdominal pain, vomiting, diarrhea, rashes, and edema. Upon physical examination, the patient is short of breath and requires numerous pauses during conversation. She is tachycardic and diaphoretic, and her extremities are cool. There is a diminished first heart sound, S3 gallop, laterally displaced PMI, bibasilar rales and dullness to percussion, and expiratory wheezing noted. There is no JVD noted; however, 2+ pitting edema of the lower extremities to the level of the mid calf is evident. What diagnostic test result would be considered most useful in differentiating a cardiac from noncardiac cause of this patient's presentation? A Hyponatremia on a basic metabolic profile B Elevations of T3 and T4 on thyroid assay C Sinus arrhythmia and low voltage on EKG D Pulmonary congestion pattern on the chest x- ray E Elevations of B-type natriuretic peptide

E

A 40-year-old man presents with atrial flutter with 2:1 atrioventricular (AV) conduction, giving him a pulse of 150 per minute, which is perfectly regular. His blood pressure is 70/40 mmHg. He takes no medications regularly. You plan to provide him with urgent direct current cardioversion with conscious sedation. What would be an appropriate level of energy for cardioversion in order to restore sinus rhythm in this patient? A 10 Joules B 15 Joules C 50 Joules D 200 Joules E 300 Joules F 360 Joules

C

A 57-year-old woman presents with progressive dyspnea and lower extremity edema. She has noticed the symptoms occurring over the last 3 months and also notes increasing fatigue over the last year. Past medical history is significant for diabetes and hypertension. Past social history is significant for previous alcohol abuse; patient has been sober for approximately 5 years. On physical examination, an S3 gallop is heard. Electrocardiogram reveals ST-T changes. Echocardiogram reveals left ventricular dilatation with an ejection fraction of 35%. What is the most likely diagnosis in this patient? A Hypertrophic cardiomyopathy B Restrictive cardiomyopathy C Dilated cardiomyopathy D Inflammatory pericarditis E Constrictive pericarditis

C

A 77-year-old man presents with significant persistent hypotension that has been worsening for 1 hour. Further investigation of this patient reveals a significant increase in the patient's heart rate as well as substantial tachypnea. Urinary output is too minimal to measure, and upon quick physical examination, the patient's extremities are cool to touch. After beginning fluid resuscitation for shock protocol, the patient's symptoms appear to be worsening. Based on this scenario, what type of shock would this patient be categorized as having? Answer Choices: A Obstructive shock B Distributive shock C Cardiogenic shock D Septic shock E Hypotensive shock

C

Your patient is a 78-year-old woman who is an inpatient status post-colectomy for colon cancer. On post-operative day 3, her oral temperature is noted to be elevated to 100.6° F. Chest x-ray and urinalysis are both negative for signs of infection. An infectious disease consult is placed in order to better define the patient's new fever. What physical examination findings would support a diagnosis of superficial thrombophlebitis? A Abdominal tenderness in the right upper quadrant B Positive Homans Sign in left calf C Abdominal tenderness in the left lower quadrant D Erythema and tenderness along the vein with IV insertion E Erythema and tenderness along her incision site

D

A 19-year-old man is brought into the ER following a motor vehicle crash in which he, the driver, sustained blunt trauma to the anterior trunk from striking the steering wheel and dashboard. The patient is alert, short of breath, hypotensive, and complains of acute chest pain. On auscultation, muffled heart sounds are heard. What is your initial diagnosis? A Pericarditis B Acute Myocardial Infarction C Pulmonary Edema D Tension pneumothorax E Pericardial Tamponade

E

In order to test for orthostatic changes, blood pressure and pulse are measured with the patient first supine then standing. What are the criteria for a positive orthostatic change (going from supine to standing)? A Drop in systolic pressure of 10 mm Hg, increase in diastolic pressure of 10 mm Hg, increase in pulse of 10 beats per minute B Drop in systolic pressure of 20 mm Hg, drop in diastolic pressure of 10 mm Hg, drop in pulse of 10 beats per minute C Increase in systolic pressure of 20 mm Hg, increase in diastolic pressure of 10 mm Hg, drop in pulse of 10 beats per minute D Increase in systolic pressure of 10 mm Hg, increase in diastolic pressure of 10 mm Hg, increase in pulse of 20 beats per minute E Drop in systolic pressure 20 mm Hg, drop in diastolic pressure of 10 mm Hg, increase in pulse of 20 beats per minute

E

A 62-year-old male with a past medical history significant for a 15-year history of hypertension presents with a chief complaint of severe tearing chest pain radiating through to the back. Blood pressure is 180/110 mmHg, heart rate 120 bpm, and respiratory rate 34/min. Physical examination findings include neck negative for bruits/JVD, lungs clear to auscultation, heart regular rhythm, normal S1/S2 with an S4 present, and grade III/IV diastolic rumbling murmur noted with patient leaning forward. Radial pulses are 1+ on right and 3+ on left. EKG reveals a sinus tachycardia and evidence of left ventricular hypertrophy. You decide that the patient most likely has a thoracic aortic dissection. A STAT chest x-ray shows a widening of the mediastinum. You stabilize the patient by delivering a beta blocker intravenously. In order to confirm your diagnosis, you order A Chest CT B Left heart catheterization C Right heart catheterization D Radionuclide Ventriculography E Transthoracic echocardiogram

A

A 62-year-old woman with a long-standing history of hypertension presents with severe headache; it started this morning and is rapidly worsening. During the interview, she suddenly collapses. Your brief examination shows that she responds with extensor posturing on external stimuli. Her deep tendon reflexes are 3, and you elicit Babinski bilaterally. You also notice that her breathing has a peculiar pattern: deep inspiration with a pause at full inspiration, followed by a brief insufficient release and the end-inspiration pause. How do you best describe her respiratory pattern? A Cheyne-Stokes B Apneusis C Ataxic D Cluster E Central neurogenic hyperventilation

B


Set pelajaran terkait

Ch 5/6, Pt. 4/1, Intro to Advertising

View Set

AP Biology - Unit 1 Progress Check: MCQ

View Set

Exam 1 and 2 Questions COSC 4360

View Set

Boolean Operators / Search Operators

View Set